Вы находитесь на странице: 1из 995

Chapters Pages

1-66
1.1 Chemistry and its Scope (1), 1.2 Brief History of Chemistry (2), 1.3 Matter and Energy (2), 1.4 Elements and
Compounds (4),1.5 Mixtures (6), 1.6 Alloys (7),1.7 Physical and Chemical Changes (8),1.8 Laws of Chemical
Combination (9), 1.9 Dalton's Atomic Theory (11), 1.1 0 Atoms, Molecules and Formulae (12), 1.11 Atomic and
Molecular Mass (13), 1.12 Avogadro's Hypothesis (16), 1.13 Mole Concept (17), 1.14 Equivalent Masses or
Chemical Equivalents (20),1.15 Methods for the Determination of Atomic Mass (21), 1.16 Types of Formulae
(23), 1.17 Percentage Composition of a Compound (23), 1.18 Determination of Empirical and Molecular
Formulae (25), 1.19 Chemical Equation (27), 1.20 Measurement in Chemistry: Fundamental and Derived Units
(28), Miscellaneous Numerical Examples (32), Summary and Important Points to Remember (39), Questions (42),
. Answers (43), Practice Problems (43), Objective Questions (48), Assertion-Reason Type Questions (55), Answers
to Objective and Assertion-Reason Type Questions (56), Brain Storming Problems for lIT Aspirants (57), Answers
(61), Integer Answer Type Questions (62), Answers (62), Linked Comprehension Type Questions (63), Answers
(65), SelfAssessment (65),Answers (66).

67-149
2.1 Introduction (67),2.2 Cathode Rays-Discovery of Electron (67),2.3 Positive Rays-Discovery of Proton
(69),2.4 Rutherford Experiment-Discovery of Nucleus (69), 2.5 Moseley Experiment-Atomic Number (70),
2.6 Discovery of Neutron (71),2.7 Rutherford Model (71), 2.8 Electromagnetic Radiations (72), 2.9 Emission
Spectra-Hydrogen Spectrum (73),2.10 Quantum Theory ofRadiation (75), 2.11 Bohr's Atomic Model (78), 2.12
Sommerfeld's Extension of Bohr Theory (82), 2.13 Particle and Wave Nature of Electron (87),2.14 Heisenberg
Uncertainty Principle (88), 2.15 Wave Mechanical Model ofAtom (90),2.16 Quantum Numbers (92), 2.17 Pauli's
Exclusion Principle (96), 2.18 Aufbau Principle (97), 2.19 Hund's Rule of Maximum Multiplicity (Orbital
Diagrams) (98), Electronic Configuration of Elements (99); 2.20 Photoelectric Effect (101),2.21 Some other
Fundamental Particles (102), 2.22 Isotopes (102), 2.23 Theories of Nuclear Stability (103), 2.24 The Whole
Number Rule and Packing Fraction (104), 2.25 The Magic Numbers (104), Miscellaneous Numerical Examples
(106), Summary and Important Points to Remember (109), Questions (114), Answers (116), Practice Problems
(116), Objective Questions (123), Assertion-Reason Type Questions (136), Answers to Objective and Assertion-
Reason Type Questions (137), Brain Storming Problems for lIT Aspirants (138), Answers (142), Integer Answer
Type Questions (143), Answers (143), Linked Comprehension Type Questions (144), Answers (147), Self
Assessment (148),Answers (149).

150-211
3.1 Radioactivity (150),3.2 Characteristics of Radioactive Radiations (150), 3.3 History of the Discovery of
Radioactivity (151),3.4 Analysis of Radioactive Radiations (151), 3.5 Cause of Radioactivity (152), 3.6 Theory of
Radioactive Disintegration (153), 3.7 Group Displacement Law (155), 3.8 Radioactive Disintegration Series
(157), 3.9 Rate of Disintegration and Half Life Period (158), 3.10 Average Life (162), 3.l1 Radioactive
Equilibrium (162), 3.12 Units of Radioactivity (163), 3.13 Artificial Transmutation (165), 3.14 Artificial
Radioactivity (167), 3.15 Nuclear Fission (167), 3.16 Nuclear Fusion (170), 3.17 Synthetic Elements Including
Transactinides (171), 3.18 Applications of Radioactivity (172), Miscellaneous Numerical Examples (175),
(viii)

Summary and Important Points to Remember (179), Questions (183), Answers (184), Practice Problems (185),
Objective Questions (189), Assertion~Reason Type Questions (198), Answers to Objective and Assertion-Reason
Type Questions (199), Brain Storming Problems for lIT Aspirants (200), Answers (204), Integer Answer Type
Questions (205), Answers (205), Linked Comprehension Type Questions (206), Answers (209), SelfAssessment
(210),Answers(211).

212-315
4.1 Introduction (212), Section 1: Gaseous State-4.2 The Gaseous State (212), 4.3 Gas Laws (213), 4.4 Ideal
Gas Equation (215), 4.5 Dalton's Law of Partial Pressures (220), 4.6 Diffusion of Gases and Graham's Law of
Diffusion (221),4.7 Kinetic Theory of Gases (226), 4.8 Maxwell-Boltzmann Distribution of Molecular Speeds
(228),4.9 van der Waals' Equation (232), 4.10 Critical Phenomenon and Liquefaction of Gases (233), 4.10.1
Experimental Methods for Liquefaction of Gases (234), Section 2 : Liquid State- 4.11 The Liquid State (242),
Section 3: Solid State-4.12 The Solid State (247), 4.13 Forms of Solids (248), 4.14 Isotropy and Anisotropy
(248),4.15 Differences between Crystalline and Amorphous Solids (248), 4.16 Types of Symmetry in Crystals
(249), 4.17 Space Lattice and Unit Cell (249), 4.18 Crystal Systems (250), 4.19 Designation of Planes in
Crystals-Miller Indices (251), 4.20 Crystallography and X-Ray Diffraction (252), 4.21 Analysis of Cubic
Systems (254),4.22 Packing ofIdentical Solid Spheres (258), 4.23 Types of Crystals (261), 4.24 Imperfection in
Solids (265), 4.25 Magnetic Properties (270), Miscellaneous Numerical Examples (273), Summary and Important
Points to Remember (276), Questions (282), Answers (283), Practice Problems (283), Objective Questions (288),
Assertion-Reason Type Questions (300), Answers to Objective and Assertion-Reason Type Questions (302), Brain
Storming Problems for lIT Aspirants (303), Answers (308), Integer Answer Type Questions (309), Answers (309),
Linked Comprehension Type Questions (31 0), Answers (313), SelfAssessment (314), Answers (315).

316-390
5.1 Introduction (316), 5.2 Solvent and Solute (316), 5.3 Types of Solutions (316), 5.4 Methods of Expressing the
Concentration of a Solution (316), 5.5 Solutions of Gases in Liquids (Solubility of Gases) (322), 5.6 Solutions of
Liquids in Liquids (325), 5.7 Theory of Fractional Distillation (329), 5.8 Solutions of Solids in Liquids (330), 5.9
Colligative Properties of Dilute Solutions (332), 5.10 Lowering in the Vapour Pressure (332), 5.11 Elevation of
Boiling Point (Ebullioscopy) (336), 5.12 Depression of Freezing Point (Cryoscopy) (338), 5.13 Osmosis and
Osmotic Pressure (342), 5.14 van't Hoff Theory of Dilute Solutions (344), 5.15 Determination of Molecular
Masses (345), 5.16 Reverse Osmosis (347), 5.17 Abnormal Colligative Properties (347), Miscellaneous
Numerical Examples (352), Summary and Important Points to Remember (356), Questions (360), Answers (360),
Practice Problems (361), Objective Questions (365), Assertion-Reason Type Questions (378), Answers to
Objective and Assertion-Reason Type Questions (379), Brain Storming Problems for lIT Aspirants (380),
Answers (385), Linked Comprehension Type Questions (386), Answers (388), Self Assessment (389), Answers
(390).

391-419
6.1 Introduction (391), 6.2 Particle Size and Colloidal State (391), 6.3 Types of Colloidal Solutions (392), 6.4
Preparation of Colloidal Solutions (393),6.5 Purification of Colloidal Solutions (394),6.6 Properties of Colloidal
Solution (395),6.7 Emulsions (399),6.8 Classification of Colloids Based on the Type ofPartidt:s of Dispersed
Phase (399), 6.9 Gels (40 I), 6.10 Applications of Colloids (402), Summary and Important Points to Remember
(403), Questions (405), Answers (406), Objective Questions (406), Assertion-Reason Type Questions (413)
Answers to Objective and Assertion-Reason Type Questions (414), Brain Storming Problems for lIT Aspirants
(415), Answers (416), Linked Comprehension Type Questions (416), Answers (417), Self Assessment (418),
Answers (419).
(ix)

les AND
420-517
7.1 Introduction (420),7.2 Terms Used in Thermodynamics (420), 7.3 Internal Energy (424), 7.4 First Law of
Thermodynamics (425),7.5 Enthalpy (426), 7.6 Heat Capacity (426),7.7 Expansion of an Ideal Gas (427), 7.8
Graphical Representation of various Thermodynamic Processes and the Calculation of Work done by Graphical
Methods (429), 7.9 louie-Thomson Effect (430), 7.1 0 Thermochemistry (434),7.11 Heat of Reaction or Enthalpy
of Reaction (434), 7.12 Enthalpy of Formation or Heat of Formation (436), 7.13 Enthalpy of Combustion or Heat
ofCombustion (437), 7.14 Enthalpy of Solution or Heat of Solution (439), 7.15 Enthalpy ofNeutralisation or Heat
ofNeutralisation (439),7.16 Enthalpies ofPhysical Changes (Phase Changes) (441),7.17 Hess's Law (The Law of
Constant Heat Summation) (441), 7.18 Influence of Temperature on the Heat of Reaction or Kirchhoff's
Equation (444), 7.19 Bond Energy or Bond Enthalpies (444), 7.20 Determination of Lattice Energy (Born-Haber
Cycle) (450), 7.21 Experimental Determination of the Heat of Reaction (451), 7.22 Limitations of First Law of
Thermodynamics (452), 7.23 Spontaneous and Non-spontaneous Processes (453), 7.24 Entropy (456), 7.25
Entropy Change During Phase Transitions (458), 7.26 Second Law of Thermodynamics (460), 7.27 Gibbs Free
Energy, (G), Change in Free Energy and Spontaneity (461), 7.28 Standard Free Energy Change (466), 7.29
Relationship between Standard Free Energy Change (aG") and Equilibrium Constant (466), 7.30 Physical
Significance of Gibbs Free Energy Change (Free Energy and Useful Work) (469), 7.31 Absolute Entropies and
Third Law of Thermodynamics (469), 7.32 Conversion of Heat into Work-The Carnot Cycle (471),
Miscellaneous Numerical Examples (473), Summary and Important Points to Remember (477), Questions (481),
Answers (482), Practice Problems (482), Objective Questions (489), Assertion-Reason Type Questions (501),
Answers to Objective and Assertion-Reason Type Questions (503), Brain Storming Problems for lIT Aspirants
(504), Answers (509), Integer Answer Type Questions (510), Answers (510), Linked Comprehension Type
Questions (5II),Answers(515), SelfAssessment(515),Answers(517).

518-591
8.1 Introduction (518),8.2 Rate of Reaction (Average and Instantaneous Rate) (519), 8.3 Law of Mass Action
(Guldberg and Waage, 1864) (523), 8.4 Rate Constant (523),8.5 Collision Theory of Reaction Rate (Arrhenius
Theory of Reaction Rate) (524), 8.6 Molecularity of Reaction (529),8.7 Order of Reaction (530),8.8 Pseudo-
Order Reaction (531),8.9 Reaction Mechanism (535), 8.10 Reactions of Various Orders (536),8.11 Methods for
Determination of Order of a Reaction (540), Miscellaneous Numerical Examples (548), Summary and Important
Points to Remember (554), Questions (557), Answers (558), Practice Problems (558), Objective Questions (565),
Assertion-Reason Type Questions (579), Answers to Objective and Assertion-Reason Type Questions (580), Brain
Storming Problems for lIT Aspirants (581), Answers (586), Integer Answer Type Questions (586), Answers (586),
Linked Comprehension Type Questions (587), Answers (588), SelfAssessment (589), Answers (591).

592-655
9.1 Introduction (592), 9.2 State of Chemical Equilibrium (593), 9.3 The Law of Chemical Equilibrium
(Application of Law of Mass Action) (594). 9.4 Reaction Quotient or Mass Action Ratio (597), 9.5 Activation
Energies for Forward and Backward Reactions (599), 9.6 Standard Free Energy Change of a Reaction and its
Equilibrium Constant (600),9.7 Equilibrium Constant Expressions for Some Reactions (601), 9.8 Le Chatelier's
Principle (608), 9.9 Application of Le Chatelier's Principle to Physical Equilibria (610), 9.10 Calculation of
Degree of Dissociation from Density Measurements (611), Miscellaneous Numerical Examples (612), Summary
and Important Points to Remember (621), Questions (624), Answers (625), Practice Problems (625), Objective
Questions (629), Assertion-Reason Type Questions (643), Answers to Objective and Assertion-Reason Type
Questions (644), Brain Storming Problems for lIT Aspirants (645) , Answers (649), Integer Answer Type Questions
(650), Answers (650), Linked Comprehension Type Questions (651), Answers (653), Self Assessment (654),
Answers (655).
(x)

IONICEQUILI.BR1U M 656-731
10.1 Introduction (656), 10.2 Ostwald's Dilution Law (656), 10.3 Common Ion Effect (658), 10.4 Solubility
Product (660), 10.5 Acids and Bases (668), 10.6 Relative Strength of Acids and Bases (671), 10.7 Acid-Base
Neutralisation-Salts (673),10.8 Ionic Product of Water (674),10.9 Hydrogen Ion Concentration-pH Scale
(675),10.10 pH of Weak Acids and Bases (676)g 10.11 Buffer Solutions (679),10.12 Salt Hydrolysis (686), 10.13
Theory of Indicators (693), Miscellaneous Numerical Examples (695), Summary and Important Points to
Remember (700), Questions (705), Answers (706), Practice Problems (706), Objective Questions (711),
Assertion-Reason Type Questions (726), Answers to Objective an~Assertion-Reason Type Questions (727), Brain
Stormingl?roblems for lIT Aspirants (728), Answers (732), Integer Answer Type Questions (732), Answers (732),
Linked Comprehension Type Questions (733),Answers (735), SelfAssessment (736), Answers (737).

9~t~[)J.\'fIO,NJ.\'NDREDUCTIOi" '(Redox ReacUons~


_. __ -,. -. "U _ _ ,__ Uu _.:. _ - ,
738--173
11.1 Molecular and Ionic Equations (738), 11.2 Oxidation and Reduction (739), 11.3 Modem Concept of
Oxidation and Reduction (740), 11.4 Ion-Electron Method for Balancing Redox Reactions (741), 11.5 Oxidation
Number (Oxidation State) (743), 11.6 Special Examples of Oxidation State Determination (745), 11.7 Oxidation
Numbers (States) in Different Types of Elements (747), 11.8 Valency and Oxidation Number (748), 11.9 Balancing
Oxidation-Reduction Reactions by Oxidation Number Method (749), 11.10 Disproportionation and Oxidation-
Reduction (751),11.11 Autoxidation (752),11.12 Formal Charge (752),11.13 Stock Notation (753),11.14
Stoichiometry of Redox Reactions in Solutions (753), Summary and Important Points to Remember (754),
Questions (756), Answers (759), Objective Questions (760), Assertion-Reason Type Questions (766), Answers to
Objective and Assertion-Reason Type Questions (767), Brain Storming Problems for lIT Aspirants(768), Answers
(769), Integer Answer Type Questions (770), Answers (770), Linked Comprehension Type Questions (771),
Answers (772), SelfAssessment (772),A:nswers (773).

Elf;CT,ROC'H,EMISTRV
,<i.,.f-.o;1 '-~_:.;. :j'''; <. ; ,',
774-865
Section 1: Electrolytes and Electrolysis- 12.1 Introduction (774), 12.2 Preferential Discharge Theory (775),
12.3 Faraday's Laws of Electrolysis (776), 12.4 Applications of Electrolysis (778), Section 2: Conductance and
Conductors-12.5 Arrhenius Theory of Electrolytic Dissociation (784), 12.6 Factors Pertaining to Degree of
Ionisation (786), 12.7 Electrolytic Conductance (786), 12.8 Kohlrausch's Law (789), 12.9 Theory of Weak
Electrolytes (789), Section 3 : Electrochemical Cell-12.l0 Electrochemical Cell (792), 12.11 Daniell Cell
(792), 12.12 Salt Bridge and Its Significance (793), 12.13 Representation of an Electrochemical Cell (Galvanic
Cell) (794), 12.14 Electrode Potential (794), 12.15 Standard Electrode Potential (795), 12.16 Reference Electrode
(Standard Hydrogen Electrode, SHE or NHE) (795), 12.17 Measurement of Electrode Potential (795), 12.18 EMF
of a Galvanic Cell (797), 12.19 Reversible and Irreversible Cells (797), 12.20 Some other Reference Electrodes
(797), 12.21 Prediction for Occurrence of a Redox Reaction (798), 12.22 Electrode and Cell Potentials-Nemst
Equation (798), 12.23 Electrochemical Series (803), 12.24 Primary Voltaic Cell (The Dry Cell) (808), 12.25
Secondary Voltaic Cell (Lead Storage Battery) (809), 12.26 Fuel Cell (809), 12.27 Concentration Cells (809),
12.28 Commercial Production of Chemicals (811), Miscellaneous Numerical Examples (816), Summary and
Important Points to Remember (824), Questions (829), Answers (830), Practice Problems (831), Objective
Questions (839), Assertion-Reaso)}. Type Questions (852), Answers to Objective and Assertion-Reason Type
Questions (853), Brain Storming Problems for lIT Aspirants (854), Answers (858), Integer Answer Type Questions
(859), Answers (859), Linked Comprehension Type Questions (860), Answers (863), Self Assessment (863),
Answers (865).

866-899
Adsorption : 13.1 Introduction (866), 13.2 Distinction between Adsorption and Absorption (866), 13.3
Mechanism of Adsorption (867), 13.4 Types of Adsorption (Adsorption of Gases) (867), 13.5 Adsorption
Isotherms (868), 13.6 Adsorption from Solution Phase (870), 13.7 Adsorption Isobars and Isostere (870),
(xi)

13.8 Applications of Adsorption (871), Catalysis : 13.9 Introduction (872), 13.10 Homogeneous and
Heterogeneous Catalysis (872), 13.11 Types of Catalysis (873), 13.12 Characteristics of Catalysis (874), 13.13
Theories of Catalysis (876), 13.14 Acid-Base Catalysis (878),13.15 Enzyme Catalysis (879),13.16 Catalysts in
Industry (882), 13.17 Zeolites (883), 13.18Automobile Catalytic Converter (883), Summary and Important Points
to Rqmember (884), Questions (886), Answers (886), Practice Problems (887), Objective Questions (888),
Assehion-Reason Type Questions (893 ),Answers to Objective and Assertion-Reason Type Questions (894), Brain
Storming Problems for lIT Aspirants (895), Answers (897), Linked Comprehension Type Questions (897),
Answers (897), SelfAssessment (898), Answers (899).

~()LUMETRICANALYSIS 900-945
14.1 Important Termed used in Volumetric Analysis (900), 14.2 Concentration Representation of Solution (900),
14.3 Classification of Reactions Involved in Volumetric Analysis (901),14.4 Calculation of Equivalent Mass of
Different Substances (902),14.5 Acid-Base Titrations (907),14.6 Titration of Mixture of NaOH, Na2C03 and
NaHC0 3 by Strong Acid Like HCl (908),14.7 Redox Titrations (916),14.8 Iodometric and Iodimetric Titrations
(916), Questions (926), Answers (931), Objective Questions (932), Assertion-Reason Type Questions (937),
Answers to Objective and Assertion-Reason Type Questions (937), Brain Storming Problems for lIT Aspirants
(938), Answers (941), Linked Comprehension Type Questions (942), Answers (944), Self Assessment (944),
Answers (945).

_,,~!()~'qH10METRY. (Chemical Formulae and Equations) 946-967


Stoichiometry : Quantitative Relations in Chemical Reactions (946), Miscellaneous Numerical Examples (954),
Questions (957), Answers (958), Objective Questions (959), Answers to Objective Questions (962), Brain Storming
Problems for lIT Aspirants (963), Answers (965), Linked Comprehension Type Questions (966), Answers (966), Self
Assessment (967),Answers (967).

SPONTANEOUS ASSESSMENT SECTION


iil liT ENTRANCE TEST PAPER: MODULE-1 968-971
iil liT ENTRANCE TEST PAPER: MODULE-2 972-975
iil liT ENTRANCE TEST PAPERS
• Test Series I 976
Answers with Hints for Selected Questions 981
• Test Series II (Graphical Aptitude) 983
Answers 989
• Test Series HI (Test of Matching Aptitude) 990
Answers 993
• Test Series IV (Test of Reasoning Aptitude) 994
Answers 994
iil LOG TABLE 995
iil ANTILOG TABLE 996
1
BASIC PRINCIPLES'

1..1' CHEMISTRY AND ITS SCOPE (2) Inorganic chemistry: It deals with the study of all
known elements and their compounds except organic
Chemistry is a branch of physical science which deals with the compounds. It is concerned with the materials obtained from
study ofmatter, its physical and chemicalproperties, its chemical minerals, air, sea and soil.
composition, the physical and chemical changes which it (3) Physical chemistry: It is concerned with the physical
undergoes and the energy changes that accompany these properties and constitution of matter, the laws of chemical
processes. combination and theories go.verning reactions. The effect of
All objects in this universe are composed of matter. Most of temperature, pressure, light, concentration, etc., on reactions
these objects are visible (solids and liquids) but some are come under the scope of physical chemistry.
invisible. Chemistry is termed as a material science because it is (4) Analytical chemistry: It deals with various methods of
concerned with all material substances such as air, water, rocks, analysis of chemical substances both qualitative and quantitative.
minerals, plants, animals including man, the earth on which we It includes chemical and physical methods of analysis.
all live, and other planets. According to one of the famous A number of specialised branches have been introduced as to
scientists of twentieth century, Linus Pauling, Chemistry is the
cope with the extraordinary expansion in the subject of
science of substances, their properties, their structure and their
chemistry. Some of the specialised branches are:
transformations.
(i) Biochemistry: It comprises the studies of the substances
Chemistry is a very interesting subject which touches almost
related to living organisms and life processes.
every aspect of our lives, our culture and our environment. It has
(ii) Medicinal chemistry: It deals with the application of
changed our civilization to a great extent. The present day
chemical substances for the prevention and cure of various
chemistry .has provided man with more comforts for a healthier
diseases in living 'beings.
and happier life. A large number ormaterials which we use these
days were unknown at the turn of the century. A few decades (iii) Soil and agriculture chemistry: It deals with the:
back, our clothes and footwears were exclusively of natural analysis and treatment of soils so as to increase its fertility for the'
origin such as vegetable fibres, wool, hair, skin of animals, etc., better yields of crops. It is concerned with the chemicals used as
but, now the synthetic fibres produced in chemical factories have fertilizers, insecticides, germicides, herbicides, etc.
largely replaced them. Modem chemistry has given man new (iv) Geochemistry: It includes the study of natural
plastics, fuels, metal alloys, fertilizers, building materials, drugs, substances like ores and minerals, coal, petroleum, etc.
energy sources, etc. (v) Industrial chemistry: It deals -with the study of
During the last few decades, the expansion of chemistry has chemical processes for the production of useful chemicals on a
been tremendous. The field has become wide and complex. For large scale at relatively low costs.
convenience and better understanding of the subject, it has been (vi) Nuclear chemistry: It is the most recent branch. It
divided into various branches. The four main branches of includes the study of nuclear reactions, the' production of
chemistry are: radioactive isotopes and their applications in various fields.
(1) Organic chemistry; (2) Inorganic chemistry; (vii) Structural chemistry: It deals with various
(3) Physical chemistry; (4) Analytical chemistry. techniques used for elucidation of the structure of chemical
(1) Organic chemistry: It is concerned with the study of substances. It is concerned with the properties of substances in
compounds of carbon except carbonates, bicarbonates, cyanides, terms of their structure.
isocyanides, carbides and oxides of carbon. It is actually the (viii) Polymer chemistry: It includes the study of chemical
study of hydrocarbons and their derivatives. substances of very high molecular masses of the order of 100,000
or greater, called polymers-natural or artificial. This branch is
2 G.R.B. PHYSICAL CHEMISTRy.fDR COMPETITIONS

gaining popularity as the use of plastics, rubber, synthetic fibres, Pri~~tley and Carl Scheele. Priestley was a very conservative
silicones, etc., is on the increase thes~ days. scientist. Even after his discovery of oxygen, he still believed in
(ix) Limnochemistry: It deals with the study of chemistry phlogiston theory.
involved in the river water or water reservoirs. (v) Modern period: Lavoisier (1743-1793), a French
(x) Phytochemistry: It includes the study of chemistry of plants. chemist, is regarded as the father of modern chemistry. He
Thus, it can be said iliat theteisno othel'bfaQ,c1(ofscience presented the exact explanation of combustion by proposing that
~4ichis so wide in its<scope as cheniistty;" . " . oXY8¢I1 is necessary for combustion. This concept was largely
responsible for the overthrow of the phlogiston theory. Among
t~2 aRIEFHISTORY OF CHEMISTRY his other contributions, he showed that water is composed of
hydrogen and oxygen, proposed the theory of indestructibility of
It is difficult to specify the date when science of chemistry came fl\atter, presented a clear defmition of an element and proposed a
into existence; however, its 'groWtli must have gone side by side system of chemical nomenclature.
with the growth of civilization. Broadly, the history of chemistry Another major step towards modem chemistry was taken in
can be studied under five periodS of its development. the first decade of the nineteenth century when the English
(i) Ancient period up to 350 A.D.: In ancient times, many chemist, John Dalton, postulate~ that all elements are made up
chemical operations such as souring of milk, conversion of sweet of atoms. He pictured atoms as tiny, indestructible units that
juices into wines, the conversion of wines into vinegar, etc., were could combine to form compound atoms or molecules. Dalton
known: Around3000,RC'" techniques of making glass, pottery, proposed that each element has its own kind of atom,s and the
pigments, dyes, perfuines and extraction of metals especially atoms of different elements differ in essentially nothing but their
gold* and silver were known in China, India, Egypt and Greece. masses. He determined the relative fl\asses of atoms of many
The beginning of chemistry as a science could probably be set elements. Thus, a new era had begun. The other important
about 400 B.c., when the theory was proposed that everything is chemists of this period are:
cOlllPosed of four elements: earth, air, fire and water. The first (a) Richter-Law of Reciprocal Proportions (1794)
.. bQOkof chemistry was written in Egypt around 300 A.D. The
(b) Proust-Law of Definite Proportions (1799)
terril chemistry meant the Egyptian art.
(c) Gay-Lussac-Law of Combining Volumes of Gases
(ii) The aicli"emicai period (350-1500): During this (1808)
period, scientists called alchemists tried to discover two things:
(d) Avogadro-Avogadro Hypothesis (1811)
an elixir of life which could make m:an eternally young and a
(e) Berzelius-Introduced the Modem Symbols for
philospher's stone which could transmute base metals like zinc,
Elements (1813)
copper, iron, etc., into gold. The alchemists failed in their efforts
because no philospher's stone and elixir of life actually existed (f) Faraday-Laws of Electrolysis (1833)
but we are indebted to them for designing new types ofapparatus (g) Thomas Graham""':"'Law of Gaseous Diffusion (1861)
and for discovering new chemical operations such as distillation, (h) Mendeleev-Periodic Law and Periodic Table (1869)
sublimation, e~traction of gold by amalgamation process and (i) Arrhenius-TheoryofIonization (1887)
preparation of caustic alkalies from ashes of plants., (j) Henry Becquerel-Discovery of Radioactivity (1896)
(iii) tlatro chemistry period (i500~1650): . During this (k) Madam Curie-Discovered Radium and Polonium
era, chemists paid their attention towards medico-chemical (1898)
problems. They believed tllat the primary object Of c4emistry was The twentieth century is' regarded as an active era of
to prepare medicines and not to make gold from base metals. chemistry. During this period, chemistry has fl\ade many
DUring this period, the study of gases was begun and quantitative contributions to human knowledge and civilization. Now, we live
experiments were undertaken for the first time. Robert Boyle in a world of synthetic fl\aterials. Chemistry of today is actually
(1627-1691) found that when it metal is heated in air, the mass· helping in solving major problems of our present day civilization
increases. He also established the relationship between volume such as population explosion, food and diseases, depletion of
and pressure of a gas. In 1661, Boyle wrote. the book 'The sources of energy, depletion of natural sources and environmental
Skeptical Chyrnist' in which he criticised' the hasic ideas of pollution ..
alchemy.
(iv) The phlogiston period (1650-1774): The phlogiston 1::~, MATTER AND ENERGY
theory was proposed by Ernst Stahl (1660-1734). Phlogiston
Besides life, matter a~denergy are regarded the two fundamental
was described as a substance in a combustible fl\aterial which is
entities with which whole of the universe is composed of. Matter
given off when the material bun:J.s. This theory persisted for about,
is anything that has mass and occupies space. All bodies in the
100 years arid was a centre of much controversy. During the end
universe conform to this definition. Mass is the quantity of matter
of the eighteenth century, much work was done with gases,
in a particular sample of matter. Mass ofa body is constant and
especially by Joseph Black, Henry Cavendish, Josepth
does not change regardless of where it is measured. The mass of a
-",. ~ . '

*Gold was probably the ftrst metal to be used because it occurred as a free metal in the earth. :
t Iatro is a Greek word m~aning a physician.
BASIC PRINCIPLES 3

body would be the same on the moon as it is on the earth. Our structure and are very near to one another. This form of matter is
senses of sight and touch usually tell us that an object occupies associated with minimum amount of energy.
space except in the case of colourless, odourless and tasteless in liquid state, substances have no definite shape but possess a
gases where some other evidence is required to satisfy the fixed volume. There is slight effect of pressure and temperature
definition of matter. on their volumes. They have the ~perty of flowing. The
The term weight should not be used in place of mass as it has a particles are nearer to one another than in a gas and this form of
different meaning. The term weight refers to the force with which matter is associated with energy more than solids.
an object is attracted towards earth. An object resting on earth In a gaseous state, substances have no definite shape and
experiences a force called its weight, W, that is equal to its mass volume. Gases fill completely any vessel in which they are
m, mUltiplied by the acceleration due to gravity g, thatis, confined and thus occupy the whole space available to them.
W=mg There is a large effect of pressur-eand temperature on their
The weight of an object thus depends on the value of' g' which volumes. The particles are far apart from one another and move
varies from place to place. However, the mass of an object is with very high speeds in all possible directions. This form of
determined by comparing the weights of two objects, one of matter is associated with maximum amount of energy.
known mass, the other of unknown mass in the same location on Sublimation
earth as both experience the same gravitational acceleration.
Solidification
Matter is indestructible, i, e., it can -neither be -created, nor
destroyed, but it can change its form; thus, the total quantity of Solid Liquid , Gas
matter of the universe is constant. ' .~~
Energy is defined as the capacity of doing work. Freezing Condensation
Anything which has the capacity to push the matter from one place
to another possesses energy. There are various forms of energy + energy + energy
such as heat, light, etc. Energy is neither created, nor destroyed, but Solid '~ Liquid ~ Gas
-energy . ,- energy
can only be transformed from one form of energy to another.
AbsorPtion 'of energy
The world became aware of the fact that matter can be
converted into energy with the discovery of nuclear reactions, Release of energy
especially nuclear fission and nuclear fusion. The relationship Depending on temperature and pressure, a substance can exist
between mass and energy was given by Einstein. The. famous in anyone of the three forms of matter.
relation is: ... ' (ii) Chemical classification: Matter exists in nature in the
2
E =mc form of chemical substances. A pure substa~ce is defined as a
where, E = energy, m = mass and c = velocity of light. variety of matter, all samples of which have same composition
and properties. Pure substances are divided into elements and
On account of this equation, the 'above two 'laws are
compounds. Most of the materials found in nature are in the. form
amalgamated into a single statement:
of mixtures consisting of tWo or more substances. There are two
"The total amount of matter and energy available in the types of mixtures-Homogeneous and Heterogeneous. Both
universe is fIXed." types of mixtures can be separated into their components (pure
Example 1. Calcula(e the amount of energy released in substances) by mechaniCal and physical methods. Th~
ergs, calories and in joules when 0.001 kg ofmass disappears. classification can be summarized in the following way:
[Given, Velocity of light = 3 X 108 ms-1 ]
Matter
Solution: According to Einstein equation, E lne
2
I '
m=0.00Ikg=lxlO- kg;c=3xI0 ms-
E
3 8

(1 x 10-3 )(3 X 108 )2 = 9x 1013 J


1
I
Physical classification Chemical classification
7

9xlO13 J
lJ = 10 erg = 0.24 cal
9xlO 13 xl0 7 erg=9x10 13 x 0.24 cal
1 1 . r Physical Pure,
1
Solids Liquids Gases Mixtures
= 9 x 1020 erg me.~ods substances
2.16 x 1013 cal
I
ClaSSification of Matter I
Homogeneous
1'
Heterogeneous
(i) Physical classification:, Matter can exist in anyone of
three forms, (a) solid, (b) liquid and (c) gas: Chemical
In the solid state, substances are rigid. They have a definite Compounds Elements
methods
shape and fixed volume. There is negligible effect of changes in
pressure and temperature on their volumes. The individual
particles that make up a solid occupy definite positions in the Inorganic Organic
4 G.R.B. PHYSICAL CHEMISTRY FOR COMPETITIONS

Properties of Matter: Properties are the characteristic (iii) or from names of the elements in other languages such as
qualities with the help of which different kinds of matter can be Latin, German, etc.
commonly recognised. In chemistry, substances are distinguished Na-Sodium (Latin name Natrium)
by two types of properties, viz (i) Chemical properties and (ii) Cu-Copper (Latin name Cuprum)
Physical properties.
Fe---Iron (Latin name Ferrum)
The chemical properties of substances are those in which they
undergo change in composition either alone or by interactions with Ag-Silver (Latin name Argentum)
other substances, i. e., to form new substances having different Pb-Lead (Latin name Plumbum)
compositions from the substances which undergo change. Au-Gold (Latin name Aurum)
The properties of substances which are observed in absence of K-Potassium (Latin name Kalium)
any change in composition under specific physical s~ate are
Hg-Mercury (Latin name Hydragyrum)
termed physical properties. Colour, density, melting point,
boiling point, hardness, refractive index, thermal conductivity, W-Tungsten (German name Wolfram)
electrical conductivity, malleability, ductility, etc., are some Out of 117 elements known, 88 have been isolated from
examples of the physical properties. The properties of matter can natural sources and the remaining have been prepared by
be further classified into two: (i) Intensive properties and (ii) artificial means. The man made elements are:
Extensive properties. The intensive properties are those which
S.No. Name Symbol S.No. Name Symbol
do not depend upon the quantity of matter, e.g. , colour, density,
melting point, boiling point, refractive index, etc. These 1. Neptunium Np 16. Hassium or Hs or
properties are same irrespective of the quantity of the substance. Unniloctium Uno
Chemical properties are also intensive properties. The extensive 2. 17. Meitnerium or
Plutonium Pu Mtor
properties of matter depend on the quantity of matter. Volume, Unnilennium Une
mass, weight, energy, etc., are the extensive properties.
3. Americium Am 18. Ununnilium Uun
:1;4 ELEMENTS AND COMPOUNDS 4. Curium Cm 19. Unununium Uuu
Elements are pure substances that cannot be decomposed into 5. Berkelium Bk 20. Ununbium Uub
simpler substances by chemical changes. The smallest particles 6. 21.
Californium Cf Ununtrium Uut
of an element possess the same properties as the bigger particles.
An element can also be defined as a pure substance which 7. Einsteinium Es 22. Ununquadium Uuq
consists of only one type of atoms. Due to discovery of isotopes, 8. Fermium Fm 23. Ununpentium Uup
this definition does not seem to be correct. The modem definition
9. Mendelevium Md 24. Ununhexium Uuh
of an element is that it is a simple individual which has a definite
atomic number (see atomic structure) and has a defmite position in 10. Nobelium No 25. Ununoctium Uuo
the periodic table. It carmot be decomposed in a chemical change. 11. Lawrencium Lr 26. Technetium Tc
In chemistry, the elements are the chemical alphabet and
compounds are the words, i. e. , combinations of elements. 12. Kurchatovium Ku 27 Promethium Pm
There are presently 117 different elements known. Every 13. Hahnium Ha 1 28.• Astatine At
element has been given a definite name and for convenience a 14. Seaborgium or Sg or \29. Francium Fr
nick name which in· chemical language is called a symbol. Unnilhexium Unh
Symbol is a small abbreviation to represent a full and lengthy
name of the element. Symbols have been derived: 15. Nielsbohrium or Bhor
Uns
(i) either by taking the first letter of the name of the element Unnilseptium I
which is capitalized: The elements from S. No. 1 to 25 are called transuranic
O-Oxygen N-Nitrogen F- Fluorine elements. The credit for the discovery of most of the transuranic
C-Carbon H-Hydrogen U- Uranium elements goes to the scientist G.T. Seaborg. The first artificially
P-Phosphorus S-Sulphur 1- Iodine produced element was technetium. It was synthesised in 1937 by
scientists at the University of California at Berkley.
(ii) or by taking the first letter and one more letter from the Most of the earth's crust is made up of a small number of
name of the element. The first letter is always capitalized. elements. Only ten of the naturally occurring elements make up
Ca-Calcium Ni-Nickel AI-Aluminium 99% mass of the earth's crust, oceans and atmosphere. The
Mg-Magnesium Co-Cobalt Bi-Bismuth following table shows the abundance of highly abundant
CI-Chlorine Br-Bromine Sa-Barium elements in nature:

Note: Among the naturally occurring elements, lH is lightest and 2~~ U is the heaviest atom.

l
BASIC PRINCIPLES I' 5

Abundance of Elements (Earth's Crust, Oceans and generally (i) brittle, (ii) non-lustrous, (iii) having low melting
Atmosphere) and boiling points, (iv) non-conductors of heat, (v) capable of
Oxygen 49.5% Chlorine 0.19% forming acidic oxides or neutral oxides, (vi) not capable of
evolving hydrogen from acids, and (vii) capable of forming
Silicon 25.7% Phosphorus 0.12%
volatile hydrides.
Aluminium 7.5% Manganese 0.09% There are some elements which do not fit completely into
Iron 4.7% Carbon 0.08% either the metal or non-metal class. Elements which have some
Calcium 3.4% N'$. Sulpur 0.06% '$. properties of both metals and non-metals are called semi:metals
t'-
Sodium 2.6% 0\ 0\ Barium 0.04% d or metalloids. The semi-metals are silicon, germanium, arsenic,
antimony and tellurium. .
Potassium 2.4% Chromium 0.033%
The above classification of elements is a rough one as certain
Magnesium 1.9% Nitrogen 0.030%
metals. like lithium, sodium, potassium possess low density;
Hydrogen 0.87% Fluorine 0.027% certain non-metals like hydrogen and graphite (a form of carbon)
Titanium 0.58% Zirconium 0.023% are good conductors of electricity. Metals rarely combine with
one another while non-metals combine with one another to form
If the entire universe is considered, then 90% of matter is
compounds. Metals and non-metals commonly combine with .
hydrogen. Helium is the second most abundant element
each other to form compounds.
amounting to 9% and the remaining elements make up only 1%
of the universe with oxygen, neon, carbon and nitrogen next in
Compounds
order of decreasing abundance.
The commercial use of an element depends not only upon its Compounds are also pure substances that are composed of
abundance but also upon its accessibility. Some of the common two or more different elements in a fixed proportion by mass.
elements such as copper, zinc, tin and lead are not abundant but Compounds containing more than four elements are rare. The
are found in nature in rich deposits from which these can be properties of a compound are altogether different from the
easily extracted. On the other hand, the elements such as titanium properties of the elements from which it has been constituted.
and zirconium which are found in abundance in nature are not The compound water has a definite composition, i. e., 11.2%
widely used because their ores are not rich and their extraction is hydrogen and 88.8% oxygen, Thus, the two are present in the
difficult and expensive. ratio of 1 : 8 by mass. The properties of water are totally different
from the properties of hydrogen and oxygen both. Hydrogen and
Metals, Non-metals and Metalloids oxygen are in gaseous state while water is in liquid state under
ordinary atmospheric conditions. Oxygen supports combustion
All the elements may be classified into two groups, metals while hydrogen is combustible but water is normally used for
and non-metals. The division is based on both physical and extinguishing fire. Component elements in compounds can be
chemical properties. separated only by chemical means and not by physical methods.
Metals are regarded as those elements which possess the Compounds are classified into two types:
following properties:
(i) Organic compounds: The' compounds obtained from
(i) They are generally solids at ordinary conditions. Mercury living sources are termed organic compounds. The term organic
is an exception which is in liquid state. is now applied to hydrocarbons (compounds of carbon and
(ii) They are lustrous in nature. hydrogen) and their derivatives.
(iii) They possess high density. (ii) Inorganic compounds: .! The compounds obtained from
(iv) They are good conductors of electricity and heat. non-living sources such as roc~ and minerals are termed
(v) They are malleable and ductile. inorganic compounds. The compounds of all elements except
(vi) They possess generally high melting and boiling points. hydrocarbons and their derivatives are included in this category.
(vii) They react with mineral acids liberating hydrogen. The number of organic compounds is very large in comparison to
(viii) They form basic oxides. inorganic compounds.
(ix) They form non-volatile hydrides if combine with Some Specific Properties of Substances: Some specific
hydrogen. properties of substances are given below:
(i) Deliquescence: The property of certain compounds of
(x) They have molecules usually mono-atomic in the vapour
taking up the moisture present in atmosphere and becoming wet
state. when exposed, is knpwn as deliquescence. These compounds are'
Sodium, calcium, aluminium, copper, silver, zinc, iron, nickel, known as deliquescent. Sodium hydroxlde, potassium hydroxide,
gold, mercury, etc., are the examples of metals. anhydrous calcium chloride, anhydro;us magnesium chloride,
The non-metals do not show the above properties. Six of the anhydrous ferric chloride, etc., are the hamples of deliquescent
non-metals, carbon, boron, phosphorus, sulphur, selenium and compounds. Sodium chloride is noti deliquescent but when
iodine, are solids. Bromine is the only liquid non-metal at room common salt is placed in atmosphere it becomes wet due to
temperature and normal pressure. The remaining non-metals; presence of an impurity of magnesium chloride.
nitrogen, oxygen, fluorine, chlorine, hydrogen, helium, argon, (ii) Hygroscopicity: Certain compounds combine with the
neon, krypton, xenon and radon are gases. Non-metals are moisture of atmosphere and are converted into hydroxides or
6 G.R.B. PHYSICAL CHEMISTRY FOR COMPETITIONS

hydrates. Such substances are called hygroscopic. Anhydrous microscppic .~evel. Tlle, c~mponeIits cm,m,ot, ~e s~en"by pak~ ~~
copper sulphate, quick lime (CaO), anhydrous sodium carbonate, a
or with the' heIp of microscope. The mixture is, uniform,
etc., are of hygroscopic nature. throughout having a single phase*. The homogeneous ri:riXttire is
(iii). Emorescence: The property of some crystalline isotropic in nature, i. e" every portion of it has the same
substances of losing their water of crystallisation on exposure composition and properties. '
, and becoming powdery on the surface is called efflorescence. Alloys such as brass, steel, 22-carat gold; solutions such as
Such salts are known as efflorescent. The examples are: common salt dissolved in water, sugar dissolved in water, iodine
Ferrous sulphate (F eSO4' 7H2 0), sodium carbonate dissolved in carbon tetrachloride, benzene in toluene, methyl
(Na2C03·lOH20), sodium sulphate (Na2 S04,lOH20) , potash alcohol in water; gasoline (a mixture of hydrocarbons), air, etc.,
alum [K 2S04 'Al2 (S04)3 ·24H 2 0] , etc: are some of the examples of homogeneous mixtures.
(iv) Malleability: This property is shown by metals. When A heterogeneous mixture is not uniform. It can have two or
the solid is beaten and does not break but is converted into a thin more phases. The components can be seen by naked eye or with
sheet, it is said to possess the property of malleability. Copper, the help of a microscope. It has anisotropic properties, i.e.,
gold, silver, aluminium, lead, etc"can be easily hammered into properties are not uniform throughout the mixture. Soil, a
sheets. Gold is the most malleable metal. mixture of sulphur and sand, a mixture of iron filings and sand,
(v) Ductility: The property of a metal to be drawn into smoke, etc., are the examples of heterogeneous mixtures.
wires is termed ductility, Copper, silver, gold, aluminium, iron, The components of a mixture differ in many of their physical
et<;:., are ductile in nature. Platinum is the most ductile metal. and chemical properties. The advantage of this difference is taken
(vi) Elasticity: When the stress is small" the ;solid:, in the separation of a mixture. The method of separation
completely regains its original shape, size or volume after the employed should not bring about the destruction of anyone of the
deforming force is removed. The solid is then said to be elastic. components. Some preliminary techniques based on physical
Steel, glass, ivory, etc., are elastic bodies. properties are described here in brief.
(vii) Plasticity: When stress is increased on a metal, a limit , (i) Filtration: This method is useful when one of the
is reached beyond which, if the stress is removed, the solid does components is an insoluble solid in a solvent. The insoluble solid
not come back to its original shape Or size. It acquires a permanent is obtained by filtration of the suspension through filter-paper.
deformation. Such materials can be given any shape without any For example, common salt containing sand is separated by
difficulty. filtration. The mixture is mixed with water. It is shaken so as to
(viii) Brittleness: The solid materials which, break into dissolve common salt. The sand remains insoluble. The
small pieces on hammering are called brittle. The solids of suspension is put to filtration. ,The sand cplleqts ,on the,
non-metals are generally brittle in nature. filter-paper. It is taken in a baSin and 'dried hy heating. The filtrate .
(ix) Hardness: A material is said to be harder than the other is taken in evaporating dish and heated till whole of the water is
if it can scratch it. The hardness is measured on Mho's scale. For evaporated. Solid common salt is thus obtained in the dish.
this purpose, ten minerals have been selected which have been Sugar containing charcoal, potassium nitrate containing saw
assigned hardness from 1 to 10. dust or mixtures having insoluble components can be separated
Hardness Mineral by filtration.
Hardness Mineral
(b) Sublimation: It is a process in which a solid'substance is
Talc 6 Orthoclase directly converted into its vapours by ~lication of heat and vapour is
2 Gypsum 7 Quartz reconverted into solid by subsequent cooling. The method is used
Calcite Topaz when one of the components undergoes sublimation and other
3 8
components are not decomposed by heating. For example, naphtha-
4 Fluorite 9 Corundum lene can be separated from common salt by sublimation. Similarly, a
5 Apatite 10 Diamond mixture of ammomwn chloride and potassiwn chloride can be
separated by sublimation as ammoniwn chloride sublimes on heating.
On Mho's scale, hardness of diamond is maximum and that of
(iii) Distillation: It is a process of converting a liquid into
talc is ri1inimum. If a material can scratch topaz but cannot
its vapour by heating and then condensing the vapours again into
scratch corundum it possesses hardness equal to 8.
the same liquid by cooling. Thus, distillati()n involves
vaporisation and condensation both.
1'.5 MIXTURES
Distillation =Vaporisation + Condensation
A mixture is a material containing two or more substances either This method is employed to separate liquids which have
/ elements or compounds or both in any proportion. Substances different boiling points or a liquid from non-volatile solid or
which form a mixture are milled components. Components are solids either in solution or suspension. The mixture of copper
present in the,mixtQre without loss of their identity. There are two sulphate and water or mixture of water (b.p. 100°C) and methyl
types of mixtures-:-homogeneous, and heterogeneous. In a alcohol (b.p. 4SOC) can be separated by this method.
homogeneous mixture, the components are mixed uniformly to
* Phase is defiried. as part ora system which hasunifonn properties and composition. A solution or mixture of sugar and water is a one phase system.
Every drop of the solution has same properties and sartte cdmpOsition.
BASIC PRINCIPLES . I
(Iv) Magnetic separati(ln: If one of the components of a Mixture
.--_ _ _ _...II_A_m....,agnet is moved in the mixture
mixture has magnetic properties, it can be separated by using Ii
magnet. Iron is separated from a mixture of iron filings and
Iron filings attracted Residue
sulphur by moving a magnet through the mixture. by magnet . 1Heated
(v) Solvent extraction: This method is based on the
preferential solubility of one of the components ofthe rnixture in
a particular solvent (usually a low-boiling organic solvent) which Vapours Residue
1
forms a distinctly separate layer with the other liquid if present in (Common salt + sand)
the mixture. For example, iodine present in water can be .
1Cooled
Shaken with
recovered with the help· of ether or carbon disulphide. For this Naphthalene water and filtered
method, a separating funilel is utilized. The aqueous solution of
iodine is taken in separating funnel to which ether is added. The

I
Filtrate Residue
funnel is shaken. Two layers are formed. The upper layer which
is dark brown consists of ether and iodine and the colourless Ewpo""",,
lower layer consists of only water. The lower layer is taken out.
to dryness .IDriOO ,.

The coloured layer is then poured out and ether is removed Common salt Sand
cautiously by distillation when iodine is left behind.
Two immiscible liquids such as water and oil can also· be (c) (i) Ammonium chloride sublimes on heating,
separated by the use of a separating funnel. (ii) Potassium chloride is soluble in water,
(iii) Powder~d glass is insoluble in water.
Example 2. How will you separate the following mixtures?
(a) Sulphur, potassium nitrate and charcoal, 1.6· ALLOYS
(b) Sand, common salt, iron filings and naphthalene, When two or more elements are melted together and resulting
liquid is allowed to solidify, the product so obtained'is called an '
(c) Powdered glass, ammonium chloride and potassium alloy ifit possesses metallic properties. An liiloymay consist of a
chloride. mixture of a metal with another metal, a metal with a non-metal·
Solution: or a metal with both metal and non-metal.
(a) (i) Sulphur is soluble in carbon disulphide, Alloys are prepared because they have properties more
(ii) Potassium nitrate is soluble in water, suitable,for certain applications than do the simple metals. Alloys
(iii) Charcoal is insoluble in carbon disulphide as are used because they are harder and stronger, have desirable
casting properties, special physical properties such as magnetic "
well as in water.
properties and resistance to corrosion in certain environments.
Mixture Meltingipoint of an alloy is normally lower than the melting point .
Stirred with CSz and filtered of eith¢r of the pure components. Thcnt}al and' .. 'electrical. - .
r------------L------~ ~ conduct~vities are n~n:nally reduced in alloys'. ... .. .... . c- -.' ...

I
Filtrate (soln. of sulphur is CS0 Residue
An fllloy contammg one component mercury IS called .
amalgam. Most of the metals form amalgams. Iron, platininn,
tungsten, etc., are few metals which do not form amalgams.
1Evaporated Alloys are mainly classified into two distinct types, namely
ferrous 'and non-ferrous. Ferrous alloys always contain iron,
Sulphur carbon and one or two of the other elem~nts such as manganese,
Filtrate Residue nickel, chromium, copper, vanadium, molybdenum, tungsten,
etc. When the percentage of carbon in the alloy is below 0.1, the
1Dried·
I Evaporated to
dryness

Potassium nitrate Charcoal


alloy is termed the iron alloy and if it is above 0.1, the alloys are
called steels. When iron is not present in the alloy, it is termed a
non~ferrous alloy. Some ofthe important alloys have been listed
below:· .
Alloy·. CompoSition Main uses'
(b) (i) Iron filings are separated by. a magnet,
(ii) Naphthalene sublimes on heating, 1. Brass Cu 60-;-80%, Utensils, COIJlOellser
(iii) Sand is insoluble in water. Zn 2()..-40% electrical goods, cartridge
shell .
2. Bronze Cli 75:-90%, Coins, statues, utensils.
Sn 10-25%
8 G.R.B. PHYSICAL CHEMISTRY FOR COMPETITIONS

3. Gennan Cu 56%, Zn 24%, Utensils, resistance coils (i) Combination: Two or more substances react to form one
silver Ni20% product. When a compound is obtained by the direct reaction
4. Gun metal Cu87%, Machine parts, guns between elements, it is termed direct union or synthesis.
Sn 10%,Zn3%
2HCI C+O z
S. Rolled gold Cu 95%,Al5% Artificial j ewellery
6. Magnalium AI94%,Mg6% Balance beams, light 2Mg + O 2 = 2MgO
instruments
7. Electron Mg95%, Zn 5% Construction of aircraft (ii) Decomposition: When a compound is broken down
into two or more simple constituents, the change is called
8. Duralumin Al 95%, Cu 4%, Making aeroplanes
decomposition. Often heat is utilised for the decomposition. Such
Mn 0.5%, Mg 0.5%
decomposition is termed thermal decomposition.
9. Type metal Pb 82%, Sb 15%, Making printing types
Sn3% 2HgO = 2Hg + O 2 ; 2KCI03 = 2KCI + 30 2
10. Solder Pb 50-70%, Soldering CaC0 3 = CaO+ CO2; 2NaHC0 3 = Na 2C0 3 + H 20+ CO2
Sn 30-50% (iii) Substitution: When one element enters .into a
11. Britannia Sn 93%, Tableware compound by the replacement of the other element, the change is
Sb5%,Cu2% termed substitution.
12. Wood's metal Bi 50%, Pb 25%, Electric fuses and other
CuS04 + Zn Cu + ZnS0 4 ; C 2H 6 + Cl 2 = C 2H s CI + HCI
Sn 12.5%, Cd 12.5% safety devices
13. Nichrome Ni 60%, Cr 15%, Electrical resistances 2KI + Cl 2 = 2KCI + 12 Zn + 2HCI = ZnCl 2 + H2
Fe 25% (iv) Addition: Something is added to a chemical substance
14. Constantan Ni 40%, Cu 60% Electrical resistances without elimination.
15. Monel metal Ni 70%, Cu 30% Chemical plants KI+I2 = KI3
16. lnvar Ni 35%, Steel 65% Surveying instruments,
C2H 4 + HBr = C2HsBr
pendulums, chronometers
OH
17. Stainless Fe 89.4%, Cr 10%, Utensils, ornamental
steel Mn 0.35%, C 0.25% pieces. CH3CHO+ HCN = CH3CH<
CN
1.7 PHYSICAL AND CHEMICAL CHANGES (v) Internal rearrangement: When nothing is added or
nothing is eliminated from a chemical substance but due to
Matter undergoes two types of changes; physical and chemical. A
rearrangement of the various atoms present in a molecule, a new
physical change is one in which a substance changes its physical
compound comes into existence. When ammonium cyanate is
state but keeps its chemical identity. In physical change, a new
heated, a new substance urea is formed.
substance does not come into existence. Mass remains the same.
Physical properties are altered. This is a temporary change. For NH 4 CNO= NH2CONH 2
example, water shows all of its chemical properties whether it is
The chemical change is termed isomerisation, when one
in the form of ice or water or steam. Ice melts to form water and
isomer is converted into another.
water can be converted again into ice by placing it in a freezer.
CH 3
When 10 g of ice melts, 10 g of water is obtained. Melting,
- evaporation, condensation, frc()zing, sublimation, distillation,
CH 2 -CH 2-CH3 AlU3iHCI) CH -bH-CH
passing of electric current through metallic conductor, making of 3 3
n-Butane 300"C 2-Methyl propane
magnet from an iron piece, are some examples of physical
changes. (vi) Polymerization: Two or more molecules of a
In a chemical change, a new substance or substances come substance combine to form a giant molecule,
into existence. The starting materials called reactants, are used up 3C2 H 2 - C6 H 6
and new substances called products, are formed. The Acetylene Benzene
composition of the new substances is different from that of the nHCHO (HCHO)n
starting materials. It is a permanent change as it is not easy to Fonnaldehyde Para fonnaldehyde
obtain the starting materials again from the products.
(vii) Double decomposition: An exchange of partners
Energy is always released or absorbed when chemical or occurs between two compounds.
physical changes occur. Energy is required to melt ice and energy I I
is required to boil water. Conversely, the condensation of steam AB+CD= AD+CB
to form liquid water always liberates energy, as does the freezing L.....J
. of liquid water to form ice. Chemical changes either release I I
energy (exothermic) or adsorb energy (endothermic). BaCI 2 + Na 2 S04
L-.....J
= BaS04 + 2NaCI
Chemical changes are of various types. The important ones
are: I I
KOH + HCl = KCl + H 20
L--J
BASIC PRINCIPLES 9

The reaction is also termed neutralisation, i. e., a reaction For example, carbon dioxide can be obtained by using anyone
between an acid and a base to form a salt and water molecule. of the following methods: '
I I
(a) by heating calcium carbonate,
FeCl 3 + 3HOH Fe(OHh + 3HCl
L..::....----l (b) by heating sodium bicarbonate,
Water
(c) by burning carbon in oxygen,
Reaction of above type is termed hydrolysis. (d) by reacting calcium carbonate with hydrochloric acid.
Whatever sample of carbon dioxide is taken, it is observed
1.8 LAWS OF CHEMICAL COMBINATION that carbon and oxygen are always combined in the ratio of
12 : 32 or 3 : 8.
In order to understand the composition of various compounds, it The converse of this law that when same elements combine in
is necessary to have a theory which accounts for both qualitative the same proportion, the same compound will be formed, is not
and quantitative observations during chemical changes. always true. For example, carbon, hydrogen and oxygen when
Observations of chemical reactions were most significant in the combine in the ratio of 12 : 3 : 8 may form either ethyl alcohol
development of a satisfactory theory of the nature of matter. (C2 H 5 0H) or dimethyl ether (CH 3 0CH 3 ) under different
These observations of chemical reactions are summarised in experimental conditions.
certain statements known as laws of chemical combination.
(iii) Law of multiple proportions: This law was put
(i) Law of conservation of mass: The law was fIrst stated forward by Dalton in 1808. According to this law, if two
by Lavoisier in 1774. It is also known as the law of elements combine to form more than one compound, then the
indestructibility of matter. According to this law, in all chemical different masses of one element which combine with a fIXed
changes, the total mass Qf a system remains constant or in a mass of the other element, bear a simple ratio to one another.
chemical change, mass is neither created nor destroyed. This ..Hydrogen and oxygen combine to form two compounds H 2 0
law ~as tested by Landolt. All chemical reactions follow this (water) and H 2 0 2 (hydrogen peroxide).
law. Thus, this law is the basis of all quantitative work in
In water, Hydrogen 2 parts Oxygen 16 parts
chemistry.
In hydrogen peroxide, Hydrogen 2 parts Oxygen 32 parts
Example: 1.70 g ofsilver nitrate dissolved in 100 g ofwater
is taken. 0.585 gofsodium chloride dissolved in 100g ofwater is The masses of oxygen which combine with same mass of
hydrogen in these two compounds bear a simple ratio 1 : 2.
added to it and chemical reaction occurs. 1.435 g of silver
chloride and 0.85 g of sodium nitrate are formed. Nitrogen forms fIve stable oxides.
Np Nitrogen 28 parts Oxygen 16 parts
Solution: Total masses before chemical change
Nitrogen 28 parts Oxygen 32 parts
Mass of AgN0 3 + Mass ofNaCI + Mass of water
Nitrogen 28 parts Oxygen 48 parts
=1.70g + 0.585g + 200.0g Nitrogen 28 parts Oxygen 64 parts
202.285 g N20 s Nitrogen 28 parts Oxygen 80 parts

Total masses after the chemical reaction, The masses of oxygen which combine with same mass of
nitrogen in the five compounds bear a ratio 16 : 32 : 48 : 64 : 80
= Mass of AgCl + Mass of NaN0 3 + Mass of water
or I : 2 : 3 : 4 : 5.
1.435 g + 0.85 g_+ 200.0 g (iv) Law of reciprocal proportions: This law was given
= 202.285g· by Richter in 1794. The law states that when definite mass of all
element A combines with two other elements Band C to form
Thus, in this chemical change,
two compounds and if Band C also combine to form a
Total masses of reactants = 'total masses of products compound, their combining masses are in same proportion or
bear a simple ratio to the masses of Band C which combine
This relationship holds good when reactants are completely
with a constant mass of A.
converted into products.

~(~)-----~-----
In case, the reacting materials are not completely consumed,
the relationship will be
Total masses of reactants = Total masses of products

r~ fR
+ Masses of unreacted reactants
(ii) Law of definite or constant proportions: This law
was presented by Proust in 1799 and may be stated as follows:
A chemical compound always contains the same element
combined together in fixed proportion by mass, i.e., a NaCI
chemical compound has a fIXed composition and it does not
For example, hydrogen combines with sodium and chlorine to
depend on the method of its preparation or the source from
form compounds NaH and HCI respectively.
which it has been obtained.
10 G.R.B. PHYSICAL CHEMISTRY FOR COMPETITIONS

In NaH, Sodium 23 parts Hydrogen one part . Example 4. In an experiment, 2.4 g of iron oxide on
In HCI, Chlorine 35.5 parts Hydrogen one part reduction with hydrogen yield 1.68 g of iron. In another
Sodium and chlorine also combine to fonn NaCI in which 23
experiment, 2.9 gO/iron oxide give 2.03 g of iron on reduction
with hydrogen. Show that the above data illustrate the law of
parts of sodium and 35.5 parts of chlorine are present. These are
constant proportions.
the same parts which combine with one part of hydrogen in NaH
and HCl respectively. Solution:
In the first experiment

/
(~)
~. ~o
. The~ssoflrorioxide = 2.4 g
The mass ofJron after reduction 1.68g

S(B)~~O(c)
The mass of oxygen = Mass of iron oxide - Mass of iron
(2.4 - 1.68) O.72g
Ratio of oxygen and iron = 0.72: 1.68 1: 2.33
. 802 .
• In the second experiment
Hydrogen combines with sulphur and oxygen, to fonn The mass of iron oxide = 2.9 g
compounds H 2 S and H 20 respectively. The mass of iron afteueduction 2.03 g
In H2 S, Hydrogen 2 parts SUlphur 32 parts The triass of oxygen (2.9.;.. 2.03) ~ 0.87 g
In H 20, . Hydfogen 2 parts Oxygen 16 parts
Ratio of oxygen and iron = 0.87: 2.03 =1: 2.33
Thus, according to this law, sulphur should combine with Thus, the data illustrate the law of constant proportions, as in
oxygen in the ratio of 32 : 16 or a simple multiple of it. Actually, both the experiments the ratio of oxygen and iron is the same.
both combine to fonn S02 in the ratio of 32 : 32 or 1 : 1.
The law of reciprocal proportions is a special case of a more EXample S. Carbon combines with hydrogen to form three
general law, the law of equivalent masses, which can be stated compounds A, B and C. The percentages ofhydrogen in A, Band
as under: Care 25, 14.3 and 7.7 respectively. Which law of chemical
"In all chemical reactions, substances always react in the . combination is illustrated?
ratio oftheir equivalent masses." Solution:
(v) Law of gaseous volumes: This law was enunciated by
Gay-Lussac in 1808. According to this law, gases react with Compound % of Hydrogen % of Carbon
each other in the simple ratio of their volumes and if the product
is also in gaseous state, the volume of the product also bears a A 25.0 (100 - 25.0) = 75.0
simple ratio with the volumes of gaseous reactants when all B 14.3 (100 -14.3) = 85.7
volumes are measured under similar conditions of temperature
and pressure. . C 7.7 (100 - 7.7) = 92.3

ratio 1: 1:2 In Compound A


H2 + Cl z = 2HCI
I vol I vol Zvol 25 parts of hydrogen combine with 75 parts of carbon
2H z + O2 = 2H zO ratio 2: 1: 2 I part of hydrogen combines with 75/25
2vol I vol 2vol = 3 parts of carbon
2CO+ Oz 2002 ratio 2: 1 : 2 In Compound B
2 vol I vol 2 vol
14.3 parts of hydrogen combine with 85.7 parts of carbon
N2 + 3H2 = 2NH3 ratio 1:3:2 1 part of hydrogen combines with 85.7/14.3
I vol 3vol 2 vol
= 6.0parts of carbon
In Compound C
: :::::I_SOME SOLVED EXAMPLES\I:::::: 7.7 parts of hydrogen combine with 92.3 parts of carbon
Ex~mple 3. What mass of sodium chloride would be 1 part of hydrogen combines with 92.3/7.7
decomposed by 9.8 g of sulphuric ~id, if 12 g of sodium = 12.0 parts of carbon
bisulphate and 2.75 g of hydrogen chloride were produced in a Th1,lS, the masses of carbon in three compounds A, B and C,
reaction assuming that the law ofconservation of mass, is true? which combine with a fixed mass of hydrogen are in the ratio of
Solution: NaCI + H 2S0 4 = NaHS0 4 + HCl 3 : 6 : 12 or 1 : 2 : 4. This is a simple ratio. Hence, the data
According to law of conservation of mass, illustrate the law of multiple proportions:
Total masses of reactants = Total masses of products Example 6. Two compounds each containing only tin and
oxygen had the following composition:
Let the mass ofNaCI decomposed be x g, so Mass % of tin Mass % of oxygen
x + 9.8 12.0+ 2.75 Compound A 78.77 21.23
= 14.75 Compound B 88.12 11.88
x= 4.95g Show how this data illustrate the law ofmultiple proportions?
BASIC PRINCIPLES 11

Solution: . Example 9. Carbon monoxide reacts oyith oxygen to form


In Compound A carbon dioxide according to the equation, 2CO + O 2 = 2COz.
21.23 parts of oxygen combine with 78.77parts of tin In an experiment, 400 mL of carbon monoxide and 180 mL of
oxygen were allowed to react, when 80% ofcarbon monoxide was
I part of oxygen combines with 78.77/21.23
transformed to carbon dioxide.
= 3.7 parts of tin All the volumes were measured under the same conditions of
In Compound B temperature and pressure. Find out ·the composition of the final .
11.88 parts of oxygen combine with 88.12 parts of tin mixture.
1 part of oxygen combines with 88.12111.88 Solution: 2CO + 02 = 2C0 2
2 vol I vol 2 vol
= 7.4 parts of tin
Thus, the mass of tin in compounds A and B which From the above equation, it is observed that volume of oxygen
combine with a fixed mass of oxygen are in the ratio 00.7: 7.4 required for the transformation of carbon monoxi& into carbon
or 1 : 2. This is a simple ratio. Hence, the data illustrate the law of dioxide is half the volume of carbon monoxide and the volume of
multiple proportions. carbon dioxide produced is same as that of carbon monoxide.
. Example 7. Illustrate the law of reciprocal proportions Volume of carbon monoxide transformed
from the following data: KCI contains 52.0% potassium, KJ = 80x 400 320mL
contains 23.(iO/o potassium and ICI contains 78.2% iodine. 100
Solution: In KCI: Potassium 52.0%, Hence, volume of oxygen required for transformation
Chlorine (100- 52)= 48% = ~ x 320= 160mL
2
In KI: Potassium 23.6%;
Volume of carbon dioxide produced
Iodine (l00 - 23.6) = 76.4%
= 320mL
23.6 parts of potassium combine with 76.4 ~arts of iodine
52.0 parts of potassium will combine with So, the composition of fmal mixture is
(76.4/23.6) x 52.0 168.3 parts of iodine. Carbon monoxide = (400 - 320)
The ratio of masses of chlorine and iodine which combines =80mL
with same mass ofpotassium = 48: 168.3 or I: 3.5 Carbon dioxide = 320 mL
In ICI: Iodine 78.2% and chlorine Oxygen = 180 - 160 = 20 mL
= (100-78.2) 21.8% Example 10. How much volume ofoxygen w(/l be required
for complete combustion of40mL of acetylene (C 2 H 2 ) and how
The ratio of chlorine and iodine in ICI = 21.8: 78.2 ::::: I: 3.5. much volume of carbon dioxide will be formed? All volumes are
Hence, the data illustrate the law of reciprocal proportions. measured at NTP.
Example 8. Zinc sulphate crystals contain 22.6% of zinc
and 43.9% of water. Assuming the law of constant proportions to
Solution: 2C 2 H2 + 5°2 - 4C0 2 +2H 2O
2 vol 5 vol 4 vol
be true, how much zinc should be used to produce 13.7 g ofzinc 40 mL
5 4
x40 mL x40 mL
sulphate and how much water will they contain? 2 2
40 mL 100 mL 80 mL
Solution: 100 g of zinc sulphate crystals are obtained from
So, for complete combustion of 40 mL of acetylene, 100 mL
= 22.6g zinc
of oxygen are required and 80 mL of carbon dioxide is formed.
I g of zinc sulphate crystals will be obtained from
=: 22.6/100g zinc 1'.91 DALTON'S ATOMIC THEORY
13.7 g of zinc sulphate crystals will be obtained from The concept that matter is composed of very small particles was
= 22.6 x 13.7 given by Indian and Greek philosophers. As early as 400 to 500
100 B.C. the Greek philosopher Democritus suggested that matter
= 3.0962gof zinc cannot be forever divided into smaller and smaller parts. The
ultimate particles were considered as indivisible. These particles
100 g of zinc sulphate crystals contain water were called atoms. The word atom has been derived from the
43.9g Greek word 'atomos' meaning 'indivisible'. These early ideas,
1g of zinc sulphate crystals contain water however, were not based on experiments but were mere
:::: 43.9f100g speculations. The existence of at'Oms was accepted by Boyle in
his bo'Ok 'The Sceptical Chymist' (1661) and by Newton in his
13.7 g of zinc sulphate crystals shall contain water b'Ooks ' Principia' and 'Opticks' (1704). The 'Old ideas were put 'On
43.9 3 a scientific scale by John Dalton in the years 1803 to 1808 in the
::::: 100 xl .7= 6.0143g
f'Orm of a theory known as Dalton's Atomic Theory which is a
12 G.R.B. PHYSICAL CHEMISTRY FOR COMPETITIONS

landmark in the history of chemistry. The main points of Dalton's Some elements exist in more complex molecular forms. The
atomic theory are: molecule of phosphorus consists of four phosphorus atoms and
(i) Elements consist of minute, indivisible, indestructible the molecule of sulphur consists of eight sulphur atoms. Such
particles called atoms. molecules having more than two atoms are said to be polyatomic.
(ii) Atoms of an element are identical to each other. They A representation of the molecule of an element involves use of a
have the same mass and size. subscript to the right of the elemental symbol. The diatomic
(iii) Atoms of different elements differ in properties and have molecule of chlorine is represented as C1 2 , whereas molecules of
different masses and sizes. phosphorus and sulphur are represented as P4 and S8'
(iv) Compounds are formed when atoms of different elements respectively.
combine with each other in simple numerical ratios such The molecule is the smallest possible unit of a compound
as one-to-one, one-to-two, two-to-three and so on. which shows the properties of the compound. The molecules of
(v) Atoms cannot be created, destroyed or transformed into all compounds contain two or more different types of atoms.
atoms of other elements. These differ from the molecules of elements which contain only
(vi) The relative numbers and kind of atoms are always the one type of atoms.
same in a given compound. Thus, it becomes clear that atoms are the components of
The theory convincingly explained the various laws molecules and the molecules are components of elements or
of chemical combination, but the theory has undergone a compounds.
complete shake up with the modem concept of structure of atom. The formula is a group of symbols of elements which
However, the Daltonian atom still retains its significance as the represents one molecule of a substance. The formula of a
unit participating in chemical reactions. The following are the substance represents its chemical composition. Water consists of
modified views regarding Dalton's atomic theory: molecules containing two hydrogen atoms and one oxygen atom
(i) The atom is no longer supposed to be indivisible. The which are represented as H 2 0. The subscript to the right of the
atom is not a simple particle but a complex one. symbol for hydrogen indicates the number of hydrogen atoms
(ii) Atoms of the element may not necessarily possess the contained in a molecule. No subscript follows the symbol for
same mass but possess the same atomic number and oxygen which means, by convention, that only one atom of
show similar chemical properties (Discovery of oxygen is contained in the molecule.
isotopes). The subscripts representing the number of atoms contained in
(iii) Atoms of the different elements may possess the same a molecule of a compound are in no way related to the number of
mass but they always have different atomic numbers and atoms present in the molecule of a free element. Although both
differ in chemical properties (Discovery of isobars). hydrogen and oxygen are composed of diatomic molecules, a
(iv) Atoms of one element can be transmuted into atoms of water molecule contains only one atom of oxygen and two atoms
other element. (Discovery of artificial transmutation). of hydrogen. The two hydrogen atoms present in H2 0 are not
molecular hydrogen but rather two hydrogen atoms that have
(v) In certain organic compounds, like proteins, starch,
chemically combined with an oxygen atom.
cellulose, etc., the ratio in which atoms of different
elements combine cannot be regarded as simple. There For a chemical formula to be correct, it must contain two
are a number of compounds which do not follow the law pieces of information: (i) it must indicate the elements in the
of constant proRortions. Such compounds are called make up of the compound, and (ii) it must indicate the combining
non-stoichiometric compounds. ratio of atoms of these elements in the particular compound. The
first information is provided by including in the formula correct
chemical symbols for all the elements in the compound. The
1.~Oi ATOMS, MOLECULES AND FORMULAE second piece of information is provided by subscripts, i.e.,
An atom is the smallest particle of an element. The atom of numbers written to the right slightly below the chemical symbols
hydrogen is the smallest and the lightest. Atoms take part in of the elements.
chemical combination and remain as indivisible. All atoms do not Nitric acid is a combination of hydrogen, nitrogen and oxygen
occur free in nature. Avogadro introduced the idea of another giving a base formula RNO. These elements combine in the ratio
kind of particles called the molecules. A molecule is the smallest I : 1 : 3. Therefore, the correct formula for nitric acid is RN0 3 •
particle of an element or compound that can have a stable Some compounds are composed of ions rather than of
and independent existence. A molecule of an element consists molecules. Ions differ from atoms and molecules by being
of one or more atoms of the same element. Certain elements are electrically charged particles of matter. The charges may
capable of existence as single atoms and their atoms can be be positive or negative I and generally vary in magnitude.
regarded as molecules. A molecule of an element that consists of The positively charged idns are called cations and negatively
one atom only is called monoatomic molecule as in the case of charged ions are called: anions. Simple cations· and anions
inert gases. Oxygen is not stable in atomic form but is stable in come into existence by IQss and acceptance of an electron or
molecular form. A molecule of oxygen is diatomic in nature, i. e., electrons by neutral atoms respectively. Ions that consist of
its molecule consists of two oxygen atoms. Hydrogen, nitrogen, several atoms held together by chemical bonds similar to those
fluorine, chlorine, bromine, iodine are also diatomic like oxygen. involved in the molecules are called polyatomic ions or complex
BASIC PRINCIPLES 13

ions. These complex ions differ from molecules in the sense that Common Name Chemical Name Chemical Formula
they bear a charge. Some of the common complex ions are:
Indian nitre Potassium nitrate KN0 3
N03" Nitrate PO~- Phosphate NH 4+ Ammonium
Limestone Calcium carbonate CaC0 3
so~- Sulphate CIO; Perchlorate PH4+ Phosphonium Lunar caustic Silver nitrate AgN03

SO~- Sulphite CO;- Carbonate MnO; Permanganate


Laughing gas Nitrous oxide Np
Litharge Lead monoxide PbO
When ions are present in a compound, the number of positive
Muratic acid Hydrochloric acid HCI
charges on a cation must balance with the negative charges on an
anion to produce electrically neutral matter. Since, the charge on Mohr's salt Ferrous ammonium FeS04(NH4)2S04
the anion may not always be equal to that on the cation, the sulphate ·6Hp
number of anions will not always be equal to the number of Milk of magnesia Magnesium hydroxide Mg(OH)2
cations. Microcosmic salt Sodium ammonium Na(NH 4 )HP04
Calcium nitrate consists of calcium and nitrate ions, Each hydrogen ortho-
calcium ion carries 2 units positive charge while each nitrate ion phosphate
carries I unit negative charge. Thus, to make net charge zero, two Marsh gas (Damp fire)Methane CH 4
nitrate ions will link with one calcium ion and the formula will be
Ca(N03 h, [Ca 2+ + 2NOi ]. Names and formulae of some Oleum Sulphuric acid H2~07
common chemical compounds are listed below: (Fuming)
Oxone Sodium peroxide Na 202
Common Name Chemical Name Chemical Formula
Plaster of Paris Calcium sulphate I
CaS0 4 ,-H2O
Alum Ammonium (NH4)2 S0 4' Al.2( S04)3 hemihydrate 2
aluminium sulphate '24H2O
Philosphers's wool Zinc oxide ZnO
Aspirin Acetyl salicylic acid C9H g04
Phosgene Carbonyl chloride COCl 2
Battery acid or Sulphuric acid H2SO 4
oil of vitriol Pearl ash Potassium carbonate K 2C0 3

Blue vitriol Copper sulphate CuS0 4,5HP Pyrene Carbon tetrachloride CCl 4

Baking soda Sodium bicarbonate NaHC0 3 Picric acid 2,4,6-Trinitrophenol C6H 2(OH)(N°2)3

Bleaching powder Calcium CaOCl 2 Quick lime Calcium oxide CaO


chlorohypochlorite Red lead (Minium) Lead tetroxide PbP4
BoraX Sodium tetraborate Na2BP7,I OH20 Sugar Sucrose C12H22011
Butter of tin Stannic chloride SnCI 4·5H2O Slaked lime (Milk of Calcium hydroxide Ca(OH)2
Caustic soda Sodium hydroxide NaOH lime)

Caustic potash Potassium hydroxide KOH Sid ammoniac Ammonium chloride NH4CI
Sugar oflead Lead acetate (CH 3COO)2Pb
Carbolic acid Phenol CJIsOH
Chile saltpetre Sonium nitrate NaN0 3 Sand Silicon dioxide Si02

Carborundum Silicon carbide SiC Table salt (Common Sodium chloride NaCI·
salt)
Corrosive sublimate Mercuric chloride HgCl2
TEL Tetra-ethyl lead Pb(C 2H s)4
Calomel Mercurous chloride Hg2Cl 2
Tear gas Chloropicrin CCl 3N02
Dry ice Carbon dioxide (solid) CO 2
Washing soda Sodium carbonate Na2C03,IOH20
Formalin Fonnaldehyde HCHO
(40% solution) Water glass Sodium silicate Na2Si03

Grain alcohol (Spirit) Ethyl alcohol . C2H sOH White vitriol Zinc sulphate ZnS°4 ,7H 2O

Green vitriol Ferrous sulphate FeS04,7H20


1~1~i! ATOMIC AND MOLECULAR MASS
Gypsum Calcium sulphate CaS04 ·2H20
One of the most important concepts derived from Dalton's atomic
Gammexane (BHC) Benzene hexachloride C6~C4;
theory is that of atomic mass, i. e" each element has a
llydrolith Calcium hydride CaH2 characteristic atomic mass. As atoms are very tiny particles, their
Hypo (Antichlor) Sodium thiosulphate Na2~03·5Hp
absolute masses are difficult to measure. However, it is possible
to determine the relative masses of different atoms if a small unit
14 G.R.B. PHYSICAL CHEMISTRY FOR COMPETITIONS

of mass is taken as a standard. For this purpose, mass of one atom Thus,
of hydrogen was assumed as unity and was accepted as standard. 23
Tbe atomic mass of an element can be defined as tbe number lamu' 1.9924 x 10- = 1.66 X 10- 24 g or 1.66 x 10- 27 kg
wbich indicates how many times tbe mass of one atom of the 12 .
element is heavier in comparison to the mass of one atom: of
hydrogen.
A =Atomic mass of an element
Mass of one atom of the element
A == Atomic mass of an element =----------------------
lamu
Mass of one atom of the element
The atomic masses of some elements on the basis of carbon-l 2
Mass of one atom of hydrogen are given below:
In 1858, oxygen atom was adopted as a standard on account of Hydrogen 1.008 amu Iron 55.847 amu
the following reasons: Oxygen 16.00 amu Sodiuml 22.989 amu
(i) It is much easier to obtain compounds of elements with
Chlorine 35.453 amu Zinc 65.38 amu
oxygen than with hydrogen as oxygen is more reactive than
hydrogen. Magnesium 24.305 amu Silver 107.868 amu
(ii) The atomic masses of most of the elements. become Copper 63.546 amu
approximately whole numbers but with hydrogen as standard the
atomic masses of most of the elements are fractional. The actual mass of an atom of an element
The mass of one atom of natural oxygen was taken to be 16.0. The atomic mass of an element in amu x 1.66 x 10-24 g
Thus, atomic mass of an ~lement So, the actual mass of hydrogen atom
1.008 x 1.66 x 10-24 == 1.6736 X 10-24 g
element
Similarly, the actual mass of oxygen atom
16 th part of the mass of one atom of oxygen
16 x 1.66 x 10-24 = 2.656 X 10-23 g
Mass of one atom of the element x 16 It is clear from the above list of atomic masses that atomic
Mass of one atom of oxygen masses of a number of elements are not nearly whole numbers.
Actually, the above values are average relative masses. Most of
By accepting oxygen as a standard, the atomic mass of the elements occur in nature as a mixture of isotopes.
hydr6gen comes as 1.008, sodium 22.991 and sulphur 32.. 066. (Isotopes-the atoms. of the same element having different
In 1961, the International Union of Chemists selected a new atomic masses). With very few exceptions, however, elements
unit for expressing the atomic masses. They accepted the stable have constant mixtures of isotopes. Chlorine is found in nature as
isotope of carbon (12 C) with mass number of 12 as the standard. a mixture containing two isotopes CI-35 (34.969 amu) and CI-37
(36.966amu).These are found in the ratio of75.53% (CI-35) and
Atomic mass of an element can be defined as the number
24.47% (CI-37). Therefore, the average relative mass of chlorine
which indicates bow many times the mass of one atom of the
is calculated as:
element is heavier in comparison to 112 th part of the mass of
(34.969 x 0.7553)+ (3'6.966 x 0.2447)= 35.46 amu
one atom of carbon-ll (12 C).
Based on the average mass, the atomic mass of chlorine is
A == Atomic mass of an element 35.46 or 35.5 amu but it is never possible to have-an atom having
a relative mass 35.5 amu. It can have relative mass of about 35.0
Mass of one atom of the element or 37.0 amu depending on the particular isotope. Thus, average
~ th part of the mass of one atom of carbon -12 relative mass of any naturally occurring sample of chlorine is
35.46 or 35.5 amu as it is a mixture of two isotopes present in
= Mass of one atom of the element x 12 definite proportion. The same reasoning applies to all other
Mass of 'One atom of carbon -12 elements.
[The quantity' A' was formerly known as atomic weight. The average atomic masses of various elements are
However, this term is no longer used as the word 'weight' means det~ed by multiplying the atomic mass of each is.otope by its
gravitational force.] fractional abundance and adding the values thus obtained. The
Atomic mass unit: The quantity .
~
12
mass of an atom of fractional abundance is determined by· dividing percentage
abundance by hundred.
carbon-12 (12 C) is known as the atotinc~aSS unit and is
~ " m·X a+nxb
abbreviated as amu. The actual mass of one atom of carbon-l 2 is Average ISOtOPIC mass = - - - - - -
. m+n
1.9924 x 10-23 g or 1.99.24 x 10-26 kg.
here, a, b are atomic masses of isotopes in the ratio m : n.
*The term Dalton is used for one atomic mass unit, 1 Dalton = 1 amu.
11. BASIC PRINCIPLES 15

..
Average IsotOPIC mass =-x X a+ X
b the mass of a molecule of a substance relative to the mass of an
100 100 atom of hydrogen as 1.008 or of oxygen taken as '16.00 or the
mass of one atom of carbon taken as 12. Molecular mass is a
here, x, y are percentage abundance of the two isotopes
number which indicates how many times one molecule of a
(y= 100-x).
substance is heavier in comparison to l~th of the mass of
Example 11. Boron has two isotopes boron-1O and boron-II
whose percentage abundances are 19.6% and 80.4% respectively. oxygen atom or l~ th of the mass of one atom of carbon-12.
What is the average atomic mass of boron?
M = Molecular mass
Solution: Mass of one molecule of the substance .
Contribution of boron-1O 10.0 X 0.196 = 1.96amu
12 th mass of one atom of carbon -12
Contribution of boron-II =ll.Ox 0.804 = 8.844amu
The mass of a molecule is equal to sum of the masses of the
Adding both =1.96+ 8.844 10.804amu atoms present in a molecule. One molecule of water consists of2
Thus, the average atomic mass of boron is 10.804 amu. atoms of hYQrogen and one atom of oxygen. Thus, molecular
. mass of water (2 x 1.008) + 16.06 18.016amu. One molecule
Example 12. Carbon occurs in nature as a mixture of of H 2 S04 (sulphuric acid) consists of2 atoms of hydrogen, one
carbon-12 and carbon-13. The average atomic mass ofcarbon is atom of sulphur and four atoms of oxygen. Thus, the molecular
12.011. What is the percentage abundance of carbon-12 in mass of sulphUric acid is
nature?
Solution: Let x be the percentage abundance of carbon-12;
=(2x 1.008)+ 32.00+ (4 x 16.00)
then (100 - x) will be the percentage abundance of carbon-I 3 . = 98.0160r 98.016amu

Therefore. 12x + 13(100- x) = 12.011 . Gram-molecular Mass or Gram Molecule


100 100' A quantity. of substance whose . mass. in grams is
or 12x+ 1300-13x= 1201.1 numerically equal to its molecular mass' is called gram-
or x= 98.9 molecular mass. In other words; molecular mass of a substance
expressed in grams is called gram-molecular mass ot gram
Abundance of carbon-12 is 98.9%. molecule. For exampl~, the molecular mass of chlorine is 71 and,
Gram-atomic Mass Or Gram Atum therefore, its gram-molecular mass or gram molecule is 71 g.
When numerical value of atomic mass of an element is Similarly, molecular mass of oxygen (02 ) is 32, i. e.,
expressed in grams, the value becomes gram-atomic mass or 2x 16= 32amu.
gram atom. The atomic mass of oxygen is 16 while gram-atomic Gram-molecular mass of oxygen 32 g
mass or gram atom of oxygen is 16 g. Similarly, the gram-atomic
masses of hydrogen, chlorine and nitrogen are 1.008 g, 35.5 g Molecular mass of nitric acid (HN0 3 ) is 63, i. e. ,
and 14.0 g respectively; Gram~atomic.mass orgram.atomof = 1+ 14 + 3 x 16 = 63 amu
every element consists of same number of atoms. This number Gram-molecular mass of nitric acid = 63 g
is called Avogadro's number. The value of Avogadro's number
is 6.02 x 1023 . . Gram-molecular mass should not be confused with the mass
Absolute mass of one oxygen atom of one molecule of the substance in grams. The mass of one
molecule of a substance is known as its actual mass. For
= 16amu =16x 1.66 x 10- g
24
example, the actual mass of one molecule of oxygen is equal to
Therefore, the mass of 6.02 x 10 23 atoms of oxygen will be . 32x 1.66 x 10-24 g, i~e., 5.32 x 10-23 g.
= 16x 1.66 x 10-24 x 6.02 X 1023 The number of gram molecules of a substance present in a
16g (gram-:atomic mass) given mass of a substance can be determined by the application
of following formula:
Thus, gram-atomic mass can be defined as the absolute
mass in grams of 6.02 x 10 23 atoms of any element. No. of gram molecules
Mass of a substance in grams
Number of gram atoms of any element can be calculated with
the help of the following formula: Molecular mass of the substance in grams
Mass of the element in grams . Molar mass in grams
No. of gram atoms Mass of smgle molecule - 23
Atomic mass of the element in grams 6.023 x 10
Molecular Mass = Molar mass in amu x 1.66x 10- 24 grams
Like an atom, a molecule of ~ substance is also a very small 'Example 13. Calculate the mass of 2.5 gram atoms of
particle possessing a mass of the order OflO-24 to 10-22 g. Similar oxygen.
to atomic mass, molecular mass is also expressed as a relative
Solution: We know that,
mass with respect to the mass of the standard substance which is Mass of the element in grams
an atom of hydrogen or an atom of oxygen or an atom of No. of gram atoms = - - - - - - - - - - - - -
carbon-12. The molecular mass of a substance may be defined as Atomic mass of the element in grams
16 G.R.B. PHYSICAL CHEMISTRY FOR COMPETITIONS

So, Mass of oxygen = 2.5 x 32 80.0g (ii) Molecule: The smallest particle of a substance (element or
compound) which has free or independent existence and possesses
Example 14. Calculate the gram atoms in 2.3 g ofsodium. all characteristic properties of the substance. A molecule of an
element is composed of like atoms while a molecule of a compound
Solution: No. of gram atoms 2.3 == 0.1 contains fixed number of atoms of two or more different elements. A
23
[Atomic mass of sodium 23 g] molecule may be broken down into its constituent atoms but the
atom is indivisible during a chemical change.
Example 15. Calculate the mass of 1.5 gram molecule of Avogadro after making the above differentiation, presented a
sulphuric acid. hypothesis known as Avogadro hypothesis which can be stated
Solution: Molecular mass of as follows:
H 2 S04 =2x 1+ 32+ 4 x 16= 98.0amu "Under similar conditions of temperature and pressure,
Gram-molecular mass of H 2 S04 =98.0g equal volumes of aD gases contain equal number of molecules."
Mass of 1.5 gram molecule of H 2 S04 =98.0x 1.5=147.0g Avogadro hypothesis explains successfully the formation of
hydrogen chloride.
Example 16. Calculate the actual mass ofone molecule of Hydrogen + Chlorine = Hydrogen chloride
carbon dioxide (C0 2 ),
I vol I vol 2 vol
Solution: Molecular mass of CO 2 = 44 amu nmolecules nmolecules 2n molecules
larrru 1.66xlO- 24 g I molecule I molecule 2 molecules
24 I 1
So, The actual mass of CO 2 == 44 x 1.66 x 10- '2 molecule '2 molecule I molecule
23 I atom I atom I molecule
7.304 X 10- g
(Both hydrogen and chlorine are diatomic in nature.)
Thus, the hypothesis explains that the molecules of reacting
1i.~_~~ AVOGADRO'S HYPOTHESIS gases break up into constituent atoms during chemical change
According to Dalton's atomic theory, elements react with each which then combine to form new molecules of the product or
other in the simple ratio of their atoms. Gay-Lussac proposed that products.
gases combine in simple ratio of their volumes. In an attempt to
correlate Dalton's atomic theory with Gay-Lussac law of gaseous Applications of Avogadro's hypothesis
volumes, Berzelius stated that under similar conditions of 0) Atomicity*: Atomicity means number of atoms present
temperature and pressure, equal volume of all gases contain in one molecule of an elementary gas. Hydrogen, oxygen,
the same number of atoms. This hypothesis was subsequently nitrogen, chlorine, etc., are diatomic in nature. Noble gases are
found to be incorrect as it failed to interpret the experimental monoatomic while ozone is triatomic in nature. Avogadro's·
results and contradicted the very basic assumption of Dalton's hypothesis helps in determining the atomicity of elements.
atomic theory, i. e., an atom is indivisible. For example, the (ii) Relationship between molecular mass and vapour
formation of hydrogen chloride from hydrogen and chlorine density: The vapour density of any gas is the ratio of the
could not be explained on the basis of Berzelius hypothesis. densities of the· gas and hydrogen under similar conditions of
Hydrogen + Chlorine = Hydrogen chloride
temperature and pressure.
I vol I vol 2 vol Density of gas
n atoms natoms 2n compound atoms Vapour Density (V.D.)
Density of hydrogen
I atom I atom 2 compound atoms
Jiatom Ji atom I compound atom Mass of a certain volume of the gas
Mass of same volume of hydrogen at
i. e., for the formation of I compound atom of hydrogen the same temp. and pressure
chloride, ~ atom of hydrogen and ~ atom of chlorine are needed.
In other words, each atom of hydrogen and chlorine has been If n molecules are present in the given volume of a gas and
divided which is against Dalton's atomic. theory. Thus, the hydrogen under similar conditions of temperature and pressure,
hypothesis ofBerzelius was discarded. Mass of n molecules of gas
The Italian scientist, Amedeo Avogadro, in 1811, solved the V.D.=---------------------
Mass of n molecules of hydrogen
above problem by proposing two types of particles from which
whole of the matter is composed.of. Mass of I molecule of gas
(i) Atom: The smallest particle of an element that can take
part in chemical change but generally cannot exist freely as such. Mass of I molecule of hydrogen

*Atomicity can be ascertained with the values of ratio of two specific heats of gases ( ~; )

1.66 (Monoatomic), = lAO (Diatomic), Cp 1.33 (Polyatomic)


Cv · .. Cv
BASIC PRIN.CIPLES 17
Molecular mass of gas apples or one gross books means 144 books or one gross oranges
Molecular mass of hydrogen means 144 oranges. In a similar way, for counting of atoms,
molecules, ions, etc., chemistS use the unit mole. The term mole
Mol. mass
was introduced by Ostwald in 1896. This is the Latin word
2 'moles' meaning heap or pile. A mole (mol) is defined as the
(since, moL mass of hydrogen = 2) number of atoms in 12.00 g of carbon-12. The number of atoms
Hence, 2x V.D.=Mol. mass in 12 g of carbon-12 has been found experimentally to be
6.02xlQ23, This number is also known as Avogadro's number
This fonnula can be used for the detennination of molecular
masses of volatile substances from vapour density. Vapour named in honour of Amedeo Avogadro (1776 - 1856). .
density is measured mainly by two methods: Thus, a mole contains 6.02 x 1023 units. These units can be
(a) Victor Meyer and (b) Duma's methods. atoms, molecules, ions, electrons or anything else.
. (iii) Gram-molecular volume: I g mole of any gas 1 mole of hydrogen atoms means 6.02 x 1023 hydrogen atoms.
occupies 22.4 litres or 22400 mL of volume at NTP or STP
1 mole of hydrogen molecules means 6.02 x 1023 hydrogen
conditions. 11<
The density of hydrogen at NTP is 0.00009 g mL- l . Thus, molecules.
0.00009 g of hydrogen will occupy volume at NTP 1 mL 1 mole of potassiiun ions means 6.02 x 1023 potassium ions.

1 g of hydrogen occupies volume at NTP = 1 mL 1 mole of electrons means 6.02 x 1023 electrons.
. 0:00009
The type of entity must be specified when the mole
Ig mole of hydrogen (2.0 16 g) occupies volume at NTP designation is used. A mole of oxygen atoms contains 6.02 x 10 23
i 2.016 = 22400 mL = 22.4 litre oxygen atoms. and a mole of oxygen molecules contains
0.00009 6.02 x 1023 oxygen molecules. Therefore, a mole of oxygen
According to Avogadro's hypothesis, equal volumes of molecules is equal to two moles of oxygen' atoms,
different gases contain same number of molecules under similar i. e. , 2 x 6.02 x 1023 oxygen atoms.
conditions of temperature and pressure. Thus, 22.4 litre or 22400 How much does one mole weigh? That depends on the nature
mL of any gas at NTP will contain one gram mole or its molecular of particles (units). The mass of one mole atoms of any element
mass in grams. is exactly equal to the atomic mass in grams (gram-atomic
Loschmidt number: Number of molecules in 1 em3 or 1 mL mass or gram atom) of that element.
of a gas at S.T.P. is known as Loschmidt number. For example, the atomic mass of aluminium is 27 amu. One
. 6.023x10 23 amu is equal to 1.6{i x 10-24 g. One mole of aluminium contains
LOSChmidt number = --.---
22400 6.02 x 1023 alJlIl1inium atoms.
= 2. 68x 1018 molecules mL-1 Mass of one atom aluminiUm = 27 x 1.66 x 10-24 g
(iv) Molecular formula: Avogadro's hypothesis helps in Mass of one mole aluminium =27 x 1.66 x 10-24 X 6.02 x 1023
[mding the molecular formulae of gases. Under similar =27g
conditions of temperature and pressure, 2 volumes of ozone after This is the atomic mass of aluminium in grams or it is one
decomposition give 3 volumes of oxygen. gram atomic mass or one gram atom of aluminium.
Deco~tion Similarly, the mass of 6.02 x 10 23 moleC"!Iles (1 mole) of a
Ozone ) Oxygen
2 vol 3 vol substance is equal to its molecular mass in grams c.r
gram-molecular mass Or gram molecule. For example,
2 molecules 3 molecules molecular mass of water is 18 amu. Thus, mass of one mole of
I molecu1e 3/2 molecules water will be 18 x 1.66 x 10-24 X 6.02 x 1023 , i. e., 18 g. This is
the molecular mass of water in grams or one gram-molecular
1 molecule 3 atoms
mass or one gram molecule.
Thus, the fonnula of ozone is ° 3, Mole concept is also applicable to ionic compounds which do
not contain molecules. In such cases, the fonnula of an ionic
1~t3 MOLE CONCEPT. compound represents the ratio between constituent ions. The
mass of 6.02 x 1023 fonnula units represents one mole of an ionic
For the counting of articles, the unit dozen or unit gross is compound.
commonly used irrespective of their nature. For example, one
dozen pencils means 12 pencils or one dozen apples means 12

... O°C or 273 K temperature and one atmosphere or 760 rom ofHg or 76 cm ofHg pressure are known as the standard conditions of temperature and
pressure (STP) or normal conditions oftemperature and pressure (NTP).
18 I G.R.B. PHYSICAL CHEMISTRY FOR COMPETITIONS

One mole ofBaC1 2 = 6.02 x 1023 BaCl 2 units Number of water mOlecules in one mole of water
23
= 208.2g BaCl 2 6.02 x 10
Molecular mass (fomrula mass) of BaCl 2
Number of molecules of water in 0.05 moles
:::: 6.02 x 1023 Ba 2+ ions + 2 x 6.02
X 10
23 CI- ions 0.05 x 6.02 x 1023
= 137.2+ 71.0 = 208.2g 3.01Ox 1022
One mole of a substance will have mass equal to formula As one molecule of water contains one oxygen atom,
mass of that substance expressed in grams. So, number of oxygen atoms in 3.010 x 1022 molecule of
It has been established by Avogadro's hypothesis that one water 3.010x 1022
gram-molecular mass of any gaseous substance occupies a
volume of 22.4 litres at NTP. One gram-molecular mass is Example 19. Calculate the mass of a single atom of
nothing but one mole of substance. Thus, one mole, sulphur and a single molecule ofcarbon dioxide.
i. e. , 6.02 x 1023 molecules of any gaseous substance occupies Solution:
22.4 litres as volume at NTP.
The following formulae satisfy the above discussion. Gram-atomic mass of sulphur = 32g
1 mole of a substance:::: 6.02 x 1023 particles ofthe substance Gram-atomic mass
Mass of one sulphur atom
Number of moles of a sub~tance 6.02 x 1023
Mass of substance in gram 32
::::---- 5.33 X 10-23 g
Mass of one mole of the substance in gram 23
6.02 x 10
No. of particles
Further, Number of moles = 23 Formula of carbon dioxide = CO 2
6.02 x 10
Molecular mass of CO 2 12+2xI6=44
Thus, Gram-molecular mass of CO 2 44 g
No. of particles = Mass of substance in gram Gram-molecular mass
6.02 x 10 23 Mass of one mole of the substance in gram Mass of one molecule of CO 2
6.02x
Mass of one atom of an element
Gram atom of an element = 4 4 = 7.308 x 10-23 g
6.02 x 1023
6.02 x 1023
Mass of one molecule of a substance Example 20. What is the mass of3.01 xl 022 molecules of
Gram-molecular mass of the substance ammonia?
=
6.02 x Solution: Gram-~olecular mass of ammonia 17 g
Number of molecules
:::: Vohune of gas in litres at NTP x 6.02 x 1023 Number of molecules in 17g (one mole)ofNH3 = 6.02 x 1023
22.4 Let the mass of 3.01x 1022 moieculesofNH3 be =xg

3.01 x 1022 x
,£ : :: ::: _SOME SOLVED eXAMPLES\ :::: : : ; So,
6.02x 1023 17
Example 17. A piece ofcopper weighs 0.635 g. How many 22
or x 17x 3.01x 10 =0.85g
atoms ofcopper does it contain? IeEE (Bihar) 19921
6.02x 1023
SolutiOn: Gram-atomic mass of copper:::: 63.5 g
Example21. From 200 mg of CO2 , 1021 molecules are
Number of moles in 0.635 g of copper 0.635 == 0.01
63.5 removed. How many moles ofCO2 are left?
23
Number of copper atoms in one mole = 6.02 x 10 Solution:
Number of copper atoms in 0.01 moles O.Olx 6.02 x 1023 Gram-molecular mass ofe02 :::: 44 g
6.02 x 1021
Example 18. How many molecules o/water-and oxygen Mass of 10 21 molecules of CO2 . 44 x 1021 = 0.073 g
6.02 x 1023
atoms are present in 0.9 g ofwater?
Mass of CO 2 left = (0.2 - 0.073) = 0.127 g
Solution: Gram-molecular mass of water :::: 18 g

Number of moles in 0.9g·of water = 0.9 = 0.05 Number of moles of CO 2 left 0.127 = 2.88 x 10- 3
18 44
BASIC PRINCIPLES I 19

23
::Example 22. How many molecules and atoms of oxygen 6.02 x 10
are present.in 5,6 lUres of oxygen (0 2 )at NTP?
x 60x 60x 24 x 365
Solution: We know that, 22.4 litres of oxygen at NTP
contain 6.021< 1023 molecules of oxygen, = 19,089x 109 years = 1.9089x 1010 years

So, 5,6litres of oxygen at NTP contain .if

= 5,6 x 6,02 x 1023 molecules


22.4
23
1. Ill> mgof a compound on vaporisation in Victor Meyer's
1.505 x 10 molecules apparatus displaces 44.8 mL of air measured at STP. The
molecularmass of the compound is: [CEE (Kerala) 20041
1 molecule of oxygen contains 2 atoms of oxygen (a) 116 (b) 232 (c) 58 (d) 44,8 (e) 46.4
[Ans. (c)]
So, 1.505 x 1023 molecules of oxygen contain [Hint: Molar mass of compound
2 x 1.505 X 1023 atoms = Mass of22400 mL vapour at STP
23
3.01 X 10 atoms = OJ 16 x 22400 = 58]
44.8
Example 23. How many electrons are present in 1.6 g of
2. A gas has a vapour density 11.2. The volume occupied by I g
methane?
of the gas at NTP is: (JCECE 2004)
Solution: Gram-molecular mass of methane, (a) I L (b) 11.2 L (c)22.4 L (d) 4 L
(CH 4 ) 12+ 4 = 16g [Ans. (a)]
Number of moles in 1,6 g of methane [Hint: Molar mass 2 x 11,2 = 22.4 g
:::: 1.6 OJ 22.4
Volume of I g compound at STP ;=. - - = I L]
16 22.4
3. 3 g of hydrocarbon on combustion with 11.2 g of oxygen
Nunlber of molecules of metharie in 0.1 mole
produce 8,8 g of CO2 and 5.4 g of H 20. The data illustrate
:::: 0,1 x 6,02 x 1023 the law of:
6,02 x 10 22 (a) conservation of mass (b) multiple proportions
(c) constant proportions (d) reciprocal proportions
One molecule of methane has = 6 + 4 = Ioelectrons [Ans. (a)]
[Hint: L Masses of reactants L Masses of products
22
So, 6,02 x 10 molecules of methane have (3 + 11.2) g (8,8 + 5.4) g
Hence, law of conservation of mass is verified.]
10 x 6,02 X 1022 electrons
4. The maximum number of molecules is present in:
23
6,02 x 10 electrons [CBSE (PMT) 2004; Manipal (Medical) 20071
(a) 15 L ofIt2 gas at STP (b) ~ L ofN2 gas at STP
Example 24. The electric charge on the electron is
(c) 0,5 g ofH2 gas (d) 10 g of 02 gas
1.602 x 10- 19 coulomb, How much '~harge is present on 0,1 mole
[Ans. (a)]
ofCu 2+ ions? [Hint:
Solution: Charge on one mole of ele«trons 15
Number of molecules in 15 L H2 = - x N = 0.669 N
:::: 6,02 x 1023 x 1.602 X 10-19 coulomb 22.4
:::: 96500 coulomb :::: 1 faraday Number of molecules in 5 L No =_5_ x N = 0.223·N
- 22.4
Charge on one mole of Cu 2+ ions
= 2 x 96500 coulomb 2 faraday Number of molecules in 0,5 g H2 = 0.5 xN =: 0,25 N
2
Charge on 0.1 mole of Cu 2+ ions
Number of molecules in 10 g 0, =: .!Q x N = 0.312 N]
=O.lx 2 0.2faraday - 32
ExalJlple 25. How many years it would take to spend one 5. Insulin contains 3.4% sulphur. Then, the minimum molecular
mass of the insulin is about:
Avogadro snumber ofrupees at a rate oflO lakh of rupees in one
second? (MLNR 1990)
(a) 940 amu (b) 9400 amu
(c) 3600 amu (d) 970 amu
Solution: Number of rupees spent in one second:::: 10 6
[Ans. (a)]
Nunlber of rupees spent in one year [Hint: .: 3.4 g sulphur is present in 100 g insulin
:::: 106 X 60x 60x 24 x 365
:. 32 g sulphur will be present in 100 x 32 g insulin = 940
Avogadro's number of rupees will be spent 3,4

:. Molar mass of insulin is about 940 amu]


20 G.R.B. PHYSICAL CHEMISTRY FOR COMPETITIONS

6. 25 g of MC1 4 contains 0.5 mol chlorine then its molecular active metals. A known mass of the active metal· is reacted with
mass is: (DPMT 2007) dilute mineral acid. Hydrogen gas thus evolved is measured
(a) 100g mol- 1 ,b) 200g mol- l under experimental conditions. The volume of hydrogen is then
reduced to NTP conditions. The mass of liberated hydrogen is
(c) 150g mol- l (d) 400g mol- l
determined using density of hydrogen (0.00009 at NTP).
[Ans. (b)]
[Hint: I mol of MCI 4 contaiils 4 mol of chlorine Equivalent IllIlSS Mass of element x 1.008
.,' 0.5 mol chlorine is present in 25 g of MCl 4 Mass of hydrogen

;;4 mol chlorine will be present in~ x 4, i.e.,200 g ofMCI 4.] Mass of element x 1.008
0.5
Volume in mL ot' hydrogen displaced atNTPx 0.00009

'1.'14 EQUIVALENT MASSES OR CHEMICAL Mass of element x 11200


EQUIVALENTS Volume in mL of hydrogen displaced at NTP

Equivalent mass of a substance (element or compound) is defmed (ii) Oxide formation method: A known mass of the
as the number of parts by mass of the substance which element is changed into oxide directly or indirectly. The mass of
combine or displace directly or indirectly 1.008 parts by mass oxide is noted.
of hydrogen or 8 parts by mass of oxygen or 35.5 parts by Mass of oxygen (Mass of oxide Mass of element)
mass of chlorine or 108 parts by mass of silver.
The equivalent mass is a pure number. When the equivalent Thus, the equivalent mass of the element
:-
mass of a substance is expressed in grams, it is called gram Mass of element x8
equivalent mass. For example, equivalent mass of sodium is 23, (Mass of oxide Mass of element)
hence, its gram equivalent mass is 23 g. • /
Mass of element x 8
The equivalent mass of a substance may ha~e different values
under different conditions. The -equivalent mass of an element Mass of oxygen
may vary with change of valency. For example, copper fornts two (iii) Chloride formation method: A known mass of the
oxides CUO and Cu 2 0. In CUO, 63.5 parts of copper combine element is changed into chloride directly or indirectly. The mass
with 16 parts of oxygen. Thus, equivalent mass of copper in this of the chloride is determined.
oxide is 6~5 31.75. In Cu 20' 2 x 63.5 parts ofcopper combine Mass of chlorine (Mass of chloride Mass of element)
with 16 parts of oxygen; thus, the equivalent mass of copper in Thus, the equivalent mass of the element
this oxide is:
Mass of element x 35.5
2 x 63.5 = 63.5
(Mass of chloride Mass of element)
2
Relation between atomic mass, equivalent mass and Mass of element x 35.5
valency: Suppose an element X combines with hydrogen to
. fom a compound, XH n , where n is the valency of the element X.
(iv) Metal to metal displacement method: A more active
n parts by mass of hydrogen combine with atomic mass of metal can displace less active metal from its salt's solution. For
elementX. example, when zinc is added to copper sulphate, copper is
1 part by m\lSS of hydrogen combines with precipitated. A known mass of active metal is added to the salt's
Atomic mass of element solution of less active metal. The precipitated metal after drying
n is accurately weighed. The masses of the displacing metal and the
.. Atomic mass of element. h . displaced metal bear the same ratio as their equivalent masses. If
B y ab ove defiImtlOn, IS t e eqUlva- El and E2 are the equivalent masses of two. elements and
n ml and m2 their respective masses, then,
lent mass of the element.
Atomic mass ml =~
Thus, Equivalent mass m2 E2
n
Knowing the equivalent mass of one metal, the equivalent
or Atomic mass Equivalent mass x Valency
mass of th.e other metal can be calculated.
Note: 'Detailed discussion'on equivalent masses of compounds (acids,
bases, salts, oxidising agents, reducing agents, etc.,) will be (v) Double decomposition method: This method is based
taken in chapter on volumetric analysis. on the following points:
The following methods are, used for the determination of (a) The mass of the compound reacted and the mass of
equivalent mass of elements. product formed are in the ratio of their equivalent masses.
(i) Hydrogen displacement method: This method is used (b) The equivalent mass of the compound (electrovalent) is
for those elements which can evolve hydrogen from acids, i.e., the sum of equivalent masses of its radicals.
BASIC PRINCIPLES 21

(c) The equivalent mass of a radical is equalto the fonnula (a) 4 (b) 2 (c) 3 (d) 1.5
mass of the radical divided by its charge. [ADS. (d)]

AB+CD ~AD+CB , of metal = 16 = ~


[Hint: Equivalent mass n
ppt.
Mass of AB Equivalent mass of AB Where x atomic mass of metal
nvalency of metal
Mass of AD Equivalent mass of AD Molecular fonnula of metal oxide M 20 n
Eq. mass of A + Eq. mass of B Mass of metal oxide := 2 (16n) + 16(n) = 1.5]
Mass of metal ' 2 (l6n)
Eq. mass of A + Eq. mass of D \
Knowing the equivalent masses of Band D, equivalent mass 1.15' ME"rHODS FOR THE DETERMINATION
of A can be calculated. OF ATOMIC MASS
(i) Dulong and Petit's Law: According to this law, the
product of atomic mass and specific heat of a solid element is
7. An unknown element fonns an oxide. What will be the approximately equal to 6.4. The product of atomic mass and
equivalent mass of the element if the oxygen content is 20% specific heat is called atomic heat. Thus,
by mass: [JEE (W8) 2008] Atomic mass x Specific heat = 6.4
(a) 16 (b) 32 (c) 8 (d) 64
[ADS. (b)] or . mass (approximate
AtOlTIlC . ) = - - .6.4
---
. Mass of element . Specific heat
[Hint: Eqwvalent mass of element := x8
Mass of oxygen In above fonnula, the specific heat must he in cal/g unit.
80 The equivalent mass of the element is determined
= x 8 =32]
20 experimentally and the valency, which is always a whole number,
8. A metal M of equivalent mass E fonns an oxide of molecular can be obtained by dividing approximate atomic mass with the
fonnulaMxOy- The atomic mass of the metal is given by the equivalent mass and changing the value so obtained to the nearest
correct equation: [PMT (Kerala) 2008] whole number. In this way, exact atomic mass can be determined
(a) 2E(y/ x) (b).xyE by multiplying equivalent mass with valency.
(c) EI y (d) y/ E Example 26. A chloride of an element contains 49.5%
E x chlorine. The specific heat of the element is 0.056. Calculate the
(e)-x
2, y eqUivalent mass, valency and atomic mass of the element.
[Ans. (a)] Solution: Mass of chlorine in the metal chloride = 49.5%
[Hint: Let atomic mass of metal Mis' a'. Mass of metal (100- 49:5) 50:5
Mass of metal = a x x
. Mass of metal
Mass of oxygen = 16 X Y Eqmvalent mass of the metal '" x 35.5
. Mass of element Mass of chlorine
Eqwvalent mass of element = x8
Mass of oxygen 50.5 x 35.5 36.21
E=~x8 49.5
16y According to Dulong and Petit's law,

a 2E(~)] ·
ApproXlmate . mass 0 fth e metaI = -----.",....---
atOlTIlC
.
6.4
. Specifjc heat
9. The percentage of an element Mis 53 in its oxide of mlllecular 6.4 114.3
fonnula M 203' Its atomic mass is about: 0.056
[PET (Kerala) 2oo8J
(a) 45 (b) 9 (c) 18 (d) 38 Valency = Approximate atomic mass _ 114.3 3.1 '" 3
(e) 21 Equivalent mass 36.21
[Ans. (e)]
., . Mass of element Hence, exact atomic mass = 36.21 x 3 = 108.63
[HIDt: Eqwvalent mass of element = '. x 8
. Mass of oxygen Example 27. On dissolving 2.0 go/metal in sulphuric
53 " acid, 4.5lg o/the metal sulphate wasformed. The specific heat of
=-x8:::9
47 the metal is 0.051 cal g-I. What is the valency of the metal and
Atomic mass = Equivalent mass x Valency exact atomic mass?
=9x3 =27 amu.] Solution: Equivalent mass of SO~- radical
10. The equivaLent weight of a metal is double than that of
oxygen. How many times is the weight of its' oxide greater Ionic mass = 96 = 48
than the weight of metal? Valency 2
22 G.R.B. PHYSICAL CHEMISTRY FOR COMPETITIONS

Mass of metal sulphate = 4.51 g Examples of isomorphous compounds are:


Mass of metal 2.0g (a) K 2 S04 and K 2Cr0 4 (potassium sulphate and potassium
Mass of sulphate radical (4.51 ~ 2.0) == 2.51 g chromate)
2.51 g of sulphate combine with 2.0 g of metal. °
(b) ZnS04 ·7H2 and FeS04·7H 20 (zinc sulphate and
ferrous sulphate)
So, 48 g of sulphate will combine with (c) KCI04 and KMn0 4 (potassium perchlorate and
potassium permanganate)
=2. x 48 = 38.24 g metal
2.51 (d) K2 S04·Al2 (S04 ll' 24H2 0and K2S04·Cr2(S04k24H20
(potash alum and chrome alum).
Equivalent mass of metal = 38.24
The following conclusions have been deduced from the
According to Dulong and Petit'slaw, phenomenon of isomorphism:
(i) Masses of two elements that combine with same mass of
Approximate atomic mass 6.4 =~ = 112.5 other elements in their respective compounds are in the ratio of
Specific heat 0.057
their atomic masses. .
Approximate atomic mass
Valency = . Mass of one element (A) that combines
Equivalent mass with a certain mass of other elements Atomic mass of A
Mass of other element (B) that combines .Atomic mass of B
= 112.5 = 2.9"" 3 with the same mass of other elements
38.24
(ii) The valencies of the elements forming isomorphous
Exact atomic mass 38.24 x 3 114.72
compounds are the same.
(ii) Canniizaro's method: Atomic mass of an element Example 28. Potassium chromate is isomorphous to
may be defined as the smallest mass of the element present in the potassium sulphate (K 2 S0 4 ) and is found to contain 26.78%
molecular mass of anyone of its compounds. For this purpose, chromium. Calculate the atomic mass ofchromium (K = 39.10).
the following steps are followed: Solution: Since, the formula of potassium sulphate is
(a) Molecular masses of a number of compounds in which the K 2S04, so the formula of potassium chromate should be
element is present are determined. K 2Cr04 as it is isomorphous to K 2S0 4.
(b) Each compound is analysed. Mass of the element is If the atomic mass of chromium is A, then
determined in the molecular mass of each compound. . formula mass of potassium chromate should be
(c) The lowest mass ofthe element is taken its ato~c mass. = 2x 39.1+ A + 64 =(142.2+ A)
The following table shows the application of this method:
% of chromium A x 100
Vapour· Molecular % of carbon Mass of carbon In (142.2+ A)
Compound .density mass = by mass In one molecular mass 100A
(V,D.) lV.D. compound of the compound . So, 26.78
(142.2+ A)
75.0 x 16
Methane 8 16 75.0 = 12 g
100 100A = 26.78 (142.2+ A)
80.0 x 30
Ethane 15 30 80.0 =24 g or A 26: 78 x 142.2 52.00
100 73.22
Carbon 42.9 x.28
14 28 42.9 =12 g (iv) Atomic mass fmm vapour density of a chloride: The
monoxide 100
following steps are involved in this method:
Carbon 27.3 x44
22 44 27.3 = 12g (a) Vapour density of the chloride of the element is
dioxide 100
determined.
81.8 x 44
Propane 22 44 81.8 =36 g (b) Equivalent mass of the element is determined.
100
Let the valency ofthe element bex. The formula of its chloride
Least mass of carbon is 12 g.
Thus, the atomic mass of carbon is 12.
.
will be MCI x'
Molecular mass = Atomic mass of 1M + 35.Sx
(iii) The law of isomorphism: Isomorphous substances
form crystals which have same shape and size and can grow in = A + 35.Sx
the saturated solution of each other. They have a property of Atomic mass Equivalent mass x Valency
forming mixed crystals. Isomorphous substances have same
composition, i. e., they have same number of atoms arranged A =Exx
similarly. Molecular masS = E x x + 35.Sx
BASIC PRINCIPLES 23
2 V.:O. x(E+ 35.5) Molecular formula mass
where, n
2V.D~ Empirical formula mass
x
E + 35.5 Molecular formula gives the following informations:
Knowing the value of valency, the atomic mass can be (i) Various elements present in the molecule.
determined. (ii) Number of atoms of various elements in the molecule.
Example 29. One gram ofa chloride was found to contain (iii) Mass ratio of the elements. present in the molecule.
0.835 g of chlorine. Its vapour density is 85. Calculate its The mass ratio of carbon and oxygen in CO 2 molecule is
molecular formula. 12 : 32 or 3 : 8.
Solution: Mass of metal chloride = Ig (iv) Molecular mass of the substance.
(v) The number written before the formula indicates the
Mass of chlorine = 0.835 g number of molecules, e.g., 2C0 2 means 2 molecules of
Mass of metal = (1- 0.835) = 0.165 g carbon dioxide.
. I f I 0.165x 35.5 (iii) Structural formula: It represents the way in which
E qillva ent mass 0 meta = - - - -
0.835 atoms of various elements present in the molecule are linked with
=7.01 one another. For example, ammonia is represented as:
2 V.D.
Va1ency 0 f the meta1= - - - H
E+ 35.5
2x 85 I
= N--H
7.01 + 35.5 I
4 H
Formula of the chloride = MC1 4
The formula indicates that three hydrogen atoms are linked to
Example 30. The oxide of an element contains 32.33 per one nitrogen atom by three single covalent bonds. '
cent of the element and the vapour density of its chloride is 79.
Calculate the atomic mass of the element.
1.17 PERCENTAGE COMPOSITION OF A
Solution: Mass of the element = 32.33 parts COMPOUND
Mass of oxygen (100- 32.33) = 67.67 parts Percentage composition of a compound is the relative mass of the
Equivalent mass of the element = 32.33 x 8 = 3.82 each of the constituent element in 100 parts of it. It is readily
67.67 calculated from the formula of the compound. Molecular mass of
2V.D. 2x 79 a compound is obtained from its formula by adding up the masses
VaIency 0 ftheeIement =4 of all the atoms of the constituent elements present in the
E + 35.5 3.82+ 35.5
molecule.
Hence, the atomic mass of the element = 3.82 x 4 Let the molecular mass of a compound be M and X be. the
mass of an element in the molecule.
= 15.28
Mass·of element
Percentage of element = x 100
1.16" TYPES OF FORMULAE M
X
As already stated in section 1.1 0, a formula is a group of symbols =-xI00
M
of the elements which represents one molecule of the substance.
Formula represents chemical composition of the substance. . Example 31. Calculate the percentage composition of
There are three kinds of formulae in the case of compounds. calcium nitrate.
(i) Empirical formula: .It represents the simplest relative Solution: The formula of calcium nitrate is Ca(N03 h.
whole number ratio of atoms of each element present in the
Thus, the formula mass or molecular mass
molecule of the substance. For example, CH is the empirical
formula of benzene in which ratio of the atoms of carbon and = At. mass of Ca + 2 x At. mass of N + 6 x At. mass of oxygen
hydrogen is 1 : 1. It also indicates that the ratio of carbon and 40 + 2 x 14 + 6 x 16
hydrogen is 12 : 1 by mass.
164
(ii) Molecular formula:· Molecular formula of a compound
is one which expresses as the actual number of atoIris of each % of Ca 40 x 100 = 24
element present in one molecule. C6H6 is the molecular formula 164
of benzene indicating that six carbon atoms and six hydrogen
%ofN= 28 x 100= 17
atoms are present in a molecule of benzene. Thus, 164
Molecular formula = n x Empirical formula %ofO 100-(24+17)=59
24 G.R.B. PHYSICAL CHEMISTRY FOR COMPETITIONS

Example 32. . Determine the percentage of water of


crystallisation, iron, sulphur and oxygen in pure ferrous sulphate
(FeS04·7H20 ). .
11. A gas mixture contains 50% helium and 50% methane by
Solution: The formula mass of ferrous sulphate volume. What is the percentage by mass of methane in the
mixture? IeEE (Kerala) 2004)
= At. mass of Fe -+ At. mass of S + 4 x At. mass ot:oxygen
(a) 19.97% (b) 20.05% (c) 50% (d) 75%
+7 x Mol. mass of J::i20
(e) 80.03%
= 56.0+ 32.0+ 4x 16.0+ 7x 18.0 [Ans. (e)]
[Hint: Molar and volume ratio will be same, i. e., 1 : L
= 278.0
:. Mass of 1 mole CH4 and He will be 16 and 4 g respectively.
So, % of water of crystallisation = 126 x 100 = 45.32 Mass of CH 4
278 Percentage by mass of CRt = x 100
Total mass
%of iron = 56 x 100=20.14 16
= x 100"" 80%]
278 20
12. The atomic composition of the entire universe is
%ofsulphur= 32 x 100=11.51
278 approximately iven in the table below:
64 of total no~ of atoms
%ofoxygen = - x ]00=23.02
278
H 93
(Oxygen present in water molecules is not taken into account.)
He 7
Example 33. It is found that 16.5 g of metal combine with
oxygen to form 35.60 g of metal oxide. Calculate the percentage Hydrogen atoms constitute what percentage of the universe
of metal and oxygen in the compound. by mass? .
Solution: (a) 77% (b) 23% (c) 37% (d) 73%
[Ans. (a)]
Mass of oxygen in oxide = (35.60- 16.50)= 19.10 g
[Hint: Mass of 93 'H' atoms = 93 amu
%of metal = 16.50 x 100= 46.3 Mass of7 'He' atoms = 28amu
35.60
93
. . 19.10 % Hydrogen by mass = x 100 = 77%]
%ofoxy.gen ~-- x tOO= 53.7 (93 + 28)
'., 35.60
13. Which pair of species has same percentage of carbon?
Exa~ple 34. Hydrogen. and o.xygim are combined in the
(a) CH 3COOH and C 6H l2 0 6
ratio 1: 16 . by mass in hydrogen peroxide. Calculate the
(b) CH 3 COOH and C 2 H 5 0H
percentagtt4Jf·hydrogen and oxygen in hydrogen peroxide.
(c) HCOOCH~ and Cl2H22011
Solution: 1i part<; of hydrogen peroxide contain hydrog~n
. = 1part . (d) C 6H I2 0 6 and (42H22011
[Ans. (a)]
100 parts of hydrogen peroxide contain hydrogen
1 [Hint: Percehtage of carbon in acetic acid = 24 x 100 = 40%
=~x tOO = 5.88 60
17
%of oxygen =(100- 5.88)=94.12 Percentage of carbon in CJil206 = I!:. x 100 = 40%]
180
Example 35. On analysis of an impure sample of sodium
14. Which of the following alkanes has 75% of carbon?
chloride, the percentage ofchlorine was found to be 45.5. What is
(a) Cz»t. (b) CH4 (c) C3Hg (d) C4 HlO
the percentage ofpure sodium chloride in the given sample? '.
[Ans. (b)]
Solution: The molecular mass of pure sodium chloride (NaCI)
= At. mass of Na + At. mass of chlorine [Hint: Percentage of carbon in methane:: 12 x 100 = 75%]
16
= (23 + 35.5) = 58.5
15. Which of the following two oxides of nitrogen have 30.5%
% of chlorine in pure NaCI nitrogen?
35.5 x 100= 60.6 (a) NO (b) NOz (c) N z0 4 (d) N 2 0 5
58.4 [Ans. (b) and (c)]
Thus, [Hint: Percentage of nitrogen in NO z = 14 x 100= 30.5%
% of purity of NaCI in the sample 46
:;:: 45.5 x tOO = 75 28
Percentage of nitrogen in N z0 4 - x 100 = 30.5%]
60.6 . 92
I
BASIC PRINCIPLES 25

1~1)'; DETERMINATION OF EMPIRICAL AND The empirical fonnula is C 2 H 6 0.


MOLECULAR FORMULAE Empirical fonnula mass (2 x 12) + (6 x 1) + 16 = 46
The. following steps are followed to detennine ~he empirical Example 38. A compound of carbon, hydrogen and
fonnula of the compound: nitrogen contains these elements in the ratio 9: 1 : 3.5. Calculate
(i) The percentage composition of the compound is the empirical formula. If its molecular mass is 108, what is the
detennined by quantitative analysis. molecular formula?
(ii) The percentage of each element is divided by its atomic Solution:
mass. It gives atomic ratio of the elements present in the
Element Atomic Relative number Simplest
compound. Element
ratio mass of atoms ratio
(iii) The atomic ratio of each element is the divided by the
minimum value of atomic ratio as to get the simplest ratio 0.75 = 3
Carbon 9 12 9 =0.75
of the atoms of elements present in the compound. 12 0.25
(iv) If the simplest ratio is fractional, then values of simplest
I I =4
ratio of each element is multiplied by a smallest integer to Hydrogen - =I
I 0.25
get a simplest whole number for each of the element. ' .
(y) To -get the empirical fonnula, symbols of various 3.5 == 0.25 0.25 = I
Nitrogen 3.5 14
elements present are written side by side with their 14 0.25
respective whole number ratio as a subscript to the lower
The empirical fonnula = C 3 H4N
right hand comer of the symbol.
The molecular fennula of a substance may be determined Empirical formula mass =(3x 12)+ (4x 1)+ 14;::: 54
from the empirical fonnula if the molecular mass of the substance n;::: Mol. mass ;::: 108 =2
is known. The molecular fonnula is always a simple multiple of Emp.mass 54
empirical fonnula and the value of simple mUltiple is obtained by
dividing molecular mass with empirical fonnula mass. Thus, molecular fonnula of the compound
= 2 x Empirical fonnula
Example 36. Calculate the empirical formula for a
compound that contains 26.6% potassium, 35.4% chromium and = 2x C3 H4 N= C6 HgN2
38.1% oxygen.· Example 39. A carbon compound containing only carbon
[Given K 39.1; Cr= 52; 0 = 16] and oxygen has an approximate molecular mass of 290. On
Solution: analysis, it is found to contain 50% by mass of each element.
Relative Simplest What is the molecular formula of the compo":nd?
Per- Atomic Simplest Solution:
Element number of whole number
centage mass. ratio
atoms ratio Simplest
I ti num ber SI mpIest whole number
Element AtomIc Reave
Potassium 26.6 39.1
26.6
=0.68 0.68 =1 Ix2=2 percentage mass of atoms ratio ratio
39.1 0.68

Chromium 35.4 52.0 35.4 == 0.68 0.68 =1 1x2=2 4.166 = 1.33


52 0.68 Carbon 50.0 12 4.166 4
3.125

Oxygen 38.1 16.0 38.1 = 2.38 2.38 = 3.5 3.5 x 2= 7


' 16 3.125 = 1
16 0.68 Oxygen 50.0 3.125 3
3.125
Therefore, empirical fonnula is K 2 Cr2 0 7• The empirical fonnula ;::: C4 0 3
Example 37. A compound contains 34.8% oxygen, 52.2% Empirical formula mass = (4 X 12)+ (3 x 16) = 96
carbon and 13.0% hydrogen. What is the empiricalformula mass Molecular mass 290
ofthe compound?
Solution: n = Mol. mass -290 = 3 apprmamate
. 1
y
Emp. mass· 96
Atomic Relative number Simplest
Element Percentage. Molecular fonnula = n x Empirical fonnula
mass of atoms ratio
3 x C4 0 3 = C12 0 9
34.8 = 2.175 2.175 = 1
Oxygen 34.8 16 Example 40. A compound on analysis, was found to
16 2.175
have the follOWing composition: (i) Sodium = 14.31%,
Carbon 52.2 12 52.2 =4.35 4.35 = 2 (ii) Sulphur ;::: 9.97%, (iii) Oxygen = 69.50%, (iv) Hydrogen
12 2.175 ;::: 6.22%. Calculate the· molecular formula of the compound
13.0 13.0 = 6 assuming that whole of hydrogen in the compound is present as
Hydrogen 13.0 13.0
2.175 water ofcrystallisation. Molecular mass ofthe compound is 322.
26 G.R.B. PHYSICAL CHEMISTRY FOR COMPETITIONS

Solution: 17. A compound has an empirical formula c;H4o. An


Atomic Relative number independent analysis gave a value of 132.16 for its molecular
Element Percentage Simplest ratio mass. What is the correct molecular formula?
mass of atoms
[CET (Kerala) 2004[
0.622 = 2 (a) C4H40S (b) ~oHI2
Sodium 14.31 23 0.622
0.311 (c) <;03 (d) C6H I2 0 3
0.311 (e) C4 HgOs
Sulphur 9.97 32 0.311 =1
0.311 [Ans. (d)]
6.22 [Hint: Molecular formula = (C2H40)n
Hydrogen 6.22 1 6.22 = 20
0.311 n= Molecular mass = 132.16 = 3
4.34 =14 Empirical formula mass 44
Oxygen 69.50 16 4.34
0.311
Molecular formula = (C2H40)3 = CJI1203]
The empirical formula = Na2SH20014
18. An organic compound containing C and H has 92.30%
Empirical formula mass (2 x 23)+ 32+ (20 x 1)+ (14 x 16) carbon. Its empirical formula is:
=322 (a) CH (b) CH3
Molecular mass = 322 (c) CH2 (d) CH4
Molecular formula = Na 2SH 20 014 [Ans. (a)]
Whole of the hydrogen is present in the form of water. Thus, [Hint: Percentage of carbon =-12 x 100 = 92.30%
10 water molecules are present in the molecule.· . 13
So, molecular formula = Na 2SO 4 ·lOH2 ° 'Or'

Relative
Atomic Simplest
fUUSTRATfONSOF,OB]ECTlviQUESY}'ONS Element Percentage
mass
number of
ratio
atoms

Carbon . ~2.30 12 7.69


Hydrogen 7.70 7.70

Empirical formula = CH]


19. Two oxides of a metal contain 50% and 40% of metal M
respectively. If the formula of first oxide is MO, the formula
of 2nd oxide will be:
(a)M02
(c)M2 0
[Ans. (b)]
Hint:
% x Molecular mass = 43.86 x 146 = 4
Compound 1 Compound 2
° 100 Atomic mass 100 16
M 0 M 0
Molecular formula C1;H IO0 4
50% 50% 40% 60%
Molecular mass= 12 x 6 + 10 xl + 16 x 4::: 146
50 g 50 g 40t 60g
'Or'
50 60
Relative 1g -::lg 1g I.5g
Atomic Simplest 50 40
Element Percentage number of
mass ratio 2
, atoms
Carbon 49.30 12 4.10 1.5x2::3
Formula: MO M 20 3]
20. Two elements X and Y have atomic mass 75 and 16
Hydrogen 6.84 6.84 2:5 x 2 = 5
respectively. They combine to give a compound having
Oxygen 43.86 16 2.74 I x 2:: 2 75.8% X. The formula ofthe compound is:
(a) XY (b) X 2 Y
The empirical formula C3Hs02
(C)X 2Y2 (d)X 2Y3
2X73
n::: ";2 [Ans. (d)]
73
[Hint: Molecular mass of X 213 2 x 75 + 3 x 16 = 198
Molecular formula:: 2 X C3Hs02 C1;H IO0 4 ]
BASIC PRINCIPLES 27
150
Percentage of' X = - x 100 = 75.80% Information Obtained from Chemical Equation
198
'Or' A balanced chemical equation provides the following
informations:
Relative
Atomic Simplest (i) What are the reactants and products involved in the
Element Percenta'ge number of
mass ratio
atoms chemical change?
X 75.80 75 1.01 Ix 2=2 (ii) The relative number of molecules of reactants and
y
products.
24.20 16 1.51 1.5 x 2 = 3
(iii) The relative number by parts of mass of reactants and
Formula = X 21'3] products.
21. The crystalline salt Na 2 S04 ·xH2 0 on heating loses 55.9% of (iv) Relative volumes of gaseous reactants and products.
its mass. The fonnula of crystalline salt is: For example, consider the following reaction:
(a) Na 2 S04 ·5H2 0 (b) Na 2 S04 ·7H2 0 CH 4(g)+ 20 2 (g)=C0 2 (g)+ 2H 20(g)
(c) Na 2 S04 ·2H2 0 (d) Na 2 S04 ·lOH2 0 This equation tells us that methane and oxygen are reactants
(e) Na 2 S04 ·6H2 0 [PMT (Kerala) 2007] and carbon dioxide and water are products~ One molecule of
JAns. (d)] methane reacts with two molecules of oxygen to produce one
[Hint: Molecular mass of Na 2S0 4 ·lOH20 molecule of coi' and two molecules of water or one mole of
=46+ 96+ 180= 322amu methane reacts with two moles of oxygen to produce one mole of
180 carbon dioxide and two moles of water or 16 g of methane reacts
% by mass ofHP = - x 100 = 55.9%]
. 322 with 64 g of oxygen to produce 44 g of CO 2 and 36 g of water.
This equation also tells that I vol. of methane reacts with 2 vol. of
oxygen to produce I vol. of CO 2 and 2 vol. of steam under
1.1.9; CHEMICAL EQUATION similar conditions of temperature and pressure.
A chemical equation is a symbolic representation of a chemical Limitations of Chemical Equation
change. A chemical equation fails to provide the following
The substances, in which the chemical change is brought, are informations:
called reactants and the substances which come into existence as (i) Actual concentration of the reactants taken and the actual
the result of chemical change are called products. The concentration of the products obtained.
relationship between reactants and products is represented in the (ii) Time taken for the completion of the chemical change.
form of a chemical equation. The symbols or formulae of the (iii) . Conditions applied for bringing the chemical change.
reactants are written on left hand side of equality (=) or ~ sign
(iv) Whether the reaction is reversible or irreversible.
and the symbols or formulae of products on right hand side. The
The following efforts have been made to make the chemical
symbols or formulae on both the sides are added by + sign. Such
equations more informative by introducing:
an equation is known as skeleton equation. The equation
(i) Experimental conditions: If a particular chemical
becomes balanced when total number of atoms of various
change occurs under certain temperature and pressure conditions,
elements are made equal on both the sides. Gases are always
these are mentioned above and below the (~ ) or ( = ) sign.
written in molecular form.
200 attn
KCl0 3 ~ KCl + 02 N2 + 3H 2 ~ 2NH3
450"C
This is the skeleton equation as it only represents reactant and
If the r.eaction occurs in presence of a catalyst, it is written
products involved in the chemical change but the following
.above the ( ~ ) or ( = ) sign.
equation is a balanced equation as the number of atoms of various
Pt
elements is equal on both sides. 2S0 2 + 02 ~ 2S03
2KCl0 3 = 2KCl + 302 (ii) Heat evolved or absorbed: Heat evolved or absorbed in
Reactant ~
a chemical change can be represented by adding or subtracting
The following notations are also used in chemical equations as the amount of heat on right hand side.
to provide more information about chemical change: N2 + 02 ~ 2NO- 43.2kcals
(i) Upper arrow (i) IS written immediately after the C + O 2 ~ CO 2 + 94.3 kcals
gaseous product. (iii) Reversible or irreversible nature: Reversible
(ii) Lower arrow (.!.) is written immediately after the reactions are shown by changing the sign of equality (=) or
insoluble substance (solid) which deposits from a solution. arrow (~ ) with sign of double arrow (, ').
(iii) Symbols, (s) for solid, (I) for liquid and (g) for gas are N 2(g)+ 3H2(g)~ 2NH 3(g)
also written to represent the physical state of the reactants
and products. Types of Chemical Equations
(iv) Symbol (aq.) is written for substances dissolved in water. Chemical equations are of two. types:
(v) Symbol (~) is written over an arrow or over an equality (i) Molecular equations
sign to represent heating. (ii) Ionic equations.
28 G.R.B. PHYSICAL CHEMISTRY FOR COMPETITIONS

Molecular equations are thoSe in which reactants and products Measures. This revised set of units is known as the
are represented in the form of molecules. International System of Units (abbreviated SI). Now the SI
BaCl 2 + Na 2 S0 4 BaS04 J, + 2NaCl units have been accepted by the scientists all over the world in all
2NaOH + H 2 S0 4 Na 2 S04 J, + 2H 2 0 branches of science, engineering and technology. '
Ionic equations are those in which reactants and products are The SI system have seven basic units. The various
written in ionic form. The molecular equation fundamental quantities that are expressed by these units along
BaCl 2 + Na2S04 BaS04 J, + 2NaCI with their symbols are tabulated below:
can be written in ionic form as:
Basic pbysical quantity Unit Symbol
Ba 2+ + 2CI- +2Na + + SO~- BaS04 J, + 2Na + + 2Cl-
Ba 2+ + SO~- = BaS04 J, Length Metre m
Note: Calculations based on chemical equations have been dealt in the Mass Kilogram kg
chapter 'Stoichiometry' in 'Inorganic Chemistry'.
Time Second s
1~20;' MEASUREMENT IN CHEMISTRY: Temperature Kelvin K
FUNDAMENTAL AN.D DERIVED UNITS Electric current Ampere amp orA
Chemistry is an experimental science. An experiment always Luminous intensity Candela cd
involves observation-of a· phenomenon under certain set of
conditions. The quantitative scientific observation generally Amount of substance Mole mol
requires the measurement of one or more physical quantities such
Sometimes, submultiples and multiples are used to reduce or
as mass, length, density, volume, pressure, temperature, etc.
enlarge the size of the different units. The names and symbols of
A physical quantity is expressed in terms of a number and a
sub-multiples and multiples are listed in the table given below.
unit. Without mentioning the unit, the number has no meaning. For
The name for the base unit for mass, the kilogram, already
example, the distance between two points is "four" has no meaning
contains a preftx. The names of other units of mass are obtained
unless a specific unit (inch, centimetre, metre, etc.,) is associated
by substituting other prefixes for prefix kilo. The names of no
with the number. The units of physical quantities depend on three
other base units contain prefixes.
basic units, i.e., units of mass, length and time. Since, these are
independent units and cannot be derived from any other units, they The use of SI system is slowly growing, however, older
are called fundamental units. It was soon realised that the three systems are still in use. Furthermore, the existence of older units
fundamental units cannot describe all the physical quantities such in scientific literature demands that one must be familiar with
as temperature, intensity of luminosity, electric current and the both old and new systems.'
amount of the .substance. Thus, seven units of measurement, Submultiples Multiples
namely mass, length, time, temperature, electric current, luminous
intensity and amount of substance are taken as basic units. All Prefix Symbol Sub-multiple Prefix Symbol Multiple
other units can be derived from them and are, therefore, called deci d 10- 1 deca da 10
derived units. The units of area, volume, force, work, density,
centi c 10-2 hecto h 102
velocity, energy, etc., are all derived units.
milli m 10-3 kilo k 103
SI Units of Measurement micro 10-6 mega M HI'
Jl
Various systems of units were in use prior to I91'l0. The 10-9
nano n giga G 109
common ones are the following:
pico P 10- 12 tera T 1012
(i) The English or FPS system: The system uses the foot,
the pound and the second for length, mass and time femto f 10- 15 peta P 1015

measurements respectively. It is not used now-a-days. atto a 10-18 exa E 10


18

(ii) MKS system: Here M stands for metre (a unit of zepto z 10-21 zeta Z 102 1
length), K for kilogram (a unit of mass) and S for second (a unit yocto 10-24 yotta Y 1024
of time). This is a decimal system. .
(iii) CGS system: Here the unit oflength is centimetre, the Greek Alphabets
unit of mass is gram and the unit of time is second. It is also a
Alpha A IX Nu N v
decimal system.
MKS system often known as metric system was very popular Beta B ~ Xi .:. ~
throughout the world, but the drawback with this system was that Gamma r y Omicron 0 '0
a number of different metric units for the same quantity were Delta 11 5 Pi n 11:
used in different parts of the world. In 1964, the National Bureau
of Standards adopted a slightly modified version of the metric
system, which had been officially recommended in 1960 by an
Epsilon
Zeta,
Eta
E
Z
H
,E Rho
Sigma
T\lu
P
L
p
cr
international body, General Conference of Weigbts and l'J 't 't
BASIC PRINCIPLES 29
Theta e e UpsiloJ:l. r \) frequently by chemists. Certain other units which are not a part of
Iota I t Phi <I> .<\1 SI units are still retained for a limited period of time. The term
Kappa Chi atmosphere (atm), the unit of pressure, falls into this category.
K 1( X X
Few of the old units along with conversion factors are given
Lambda A A Psi lJ' lJf below:
Mu M j.t Omega Q ro Length: The interatomic distances are reported in units of
Numerical Prefix. angstrom (A), nanometre (nm) or picometre (pm).
lA= 10-8 cm= 10- 10 m
Prefix VaHue Prefix Value
Inm=10-7 cm=10-9 m=WA
Herni (II2) Deca 10
Mono Undeca 11 Ipm=W- IO cm 10- 12 m 10-2 A
Sesqui 1":
2
Dodeca 12 Inm 103 pm
DiorBi 2 Trideca 13 Mass: The basic unit of mass is generally taken as gram (g).
Tri 3 Tetradeca 14 The gram is 10-3 kg.
Tetra 4 Pentadeca 15 I kilogram (kg) = 103 g
, Penta 5 Hexadeca 16
Hexa 6 Heptadeca 17 1 milligram (mg)= 10-3 g
Hepta 7 Oetadeca 18 1 microgram (Ilg) =10-6 g
Oeta 8 Nonadeca 19 While dealing with atoms arid molecules, the term atomic
Nona 9 Eicosa 20 mass unit (amu) is used. One amu is taken exactly as ..!.. of the
. 12
SI Units for Some Common Derived Quantities mass ~f an atom of the carbon isotope, C : I2

(a) Area = length x breadth 1amu= 1.6605 x W~24 g = 1.6605 X 10-27 kg


= m x m = m 2 [square metre] Volume: The units of volume are reported as cubic
.(b) Volume length x breadth x height centimetre (cm 3) and cubic decimetre (dm 3). Cubic decimetre is
termed litre while cubic centimetre is termed miUilitre.
= mx mX m=m 3 [cubic metre]
llitre (lit or L) (Wcm)3 = WOOcm 3 = 10-3 m 3
· mass kg k -3
(c) Denslty=---=-= gm lmillilitre(mL)=(lcm)3 =lcm3 (cc) = 10-6 m 3
volume m 3
So, . I litre = 1000 mL
(d) Speed = distanc~ covered = ~etre = m S-I
time bme Temperature:. The celsius temperature scale which is not
. change in velocity ms- . -2·
I a part of SI system, is ·employed in scientific studies. This scale is
(e) Acceleration = = - - = ms based on the assignment of O°C to the normal freezing point of
time taken s .
water and 100°C to the normal boiling point of water. The celsius
(f) Force = mass x acceleration scale was formerly called the centigrade scale. .
The unit of temperature in SI system is Kelvin. A degree on
=kg x ms-2
the kelvin scale has the same magnitude as the degree on the
kg ms-2 (Newton, abbreviated as N) . celsius scale but zero on the kelvin scale is equal to -273.15"C.
The temperature (0 K) is often referred to as absolute zero.
(g) Pressure =force per unit area
So, K =: (OC+ 273.15)
k -2
g ms =kg m- I
or °c (K - 273.15)
m2
(Pascal-Pa) There is another important temperature scale known as
fahrenheit scale. In this scale, the normal freezing point of water
(h) Energy = force x distance travelled
is 32"F and normal boiling point is 212"F. Thus, 100"C equals
=kgms-2 xm 180"F. Both the scales are related by the following equations:
kg m 2 s-2 ( joule-J) oC=~ x (OF - 32)[since, 100 parts on celsius scale
9
Some Old Units Stili in Use = 180 parts on fahrenheit scale]
The use of some of the old units is still permitted. The 'litre' ,
for example, which is defined as I cubic decimetre is used
30 G.R.B. PHYSICAL CHEMISTRY FOR COMPETITIONS

Pressure: There are three non-S1 units for pressure which Values of Some Useful Constants
are commonly used.
F\lndamental
(a) Atmosphere (atm) is defined as the pressure exerted by a Value In old units Value in SI units
constant
column of mercury of 760 mm or 76 cm height at O°C.
(b) Torr is defined as the pressure exerted by a I mm column 'Avogadro's
of mercury at O°C. number(N)
(c) Millimetre of mercury (mm Hg). Atomic mass . 1.6605 X 10-24 g 1.6605 X 10-27 g
These three units are related as: unit (amu)
Bohr radius (~) 0.52918A = 0.52918· 5.2918 X 10-11 m
latm=760torr 760mmHg 76cmHg 1.013 x 105 Pa
x 10-8 cm
Energy: Calorie has been used in the past as a unit of energy
Boltzmann 1.3807 X 10-23 JK- 1
measurement. The calorie was defmed as the. amount of heat constant (k)
required to raise the temperature of one gram of water from
14.5"C to 15.5°C. One calorie is defined as exactly equal to 4.184 Charge on e1ectron(-) 4.8029 x 10-10 esu (-) l.6021 X 10-19 coul
(e)
joules.
Ical 4.184 J or 11 = 0.2390 cal Charge to mass 1.7588 x 108 coul g-1
ratio el m of
1kcal = 1000 cal ==4.184 kJ electron
Conversion factors
I angstrom (A) = 10-8 cm=1O- IO m==IO-1 nm=10 2 pm
1 inch 2.54 cm.
39.37 inch = I metre
or

1 kg = 2.20 pounds (lb)


lcm =0.394 inch
1km=0.621mile
19 = 0.0353 ounce (0)
Electron rest
mass (me)
GaS constant
(R)
9.1091 x 10-28 g

1
0.0821lit atm deg- mol-

1.987
1

=2.0 cal deg-lmol- I


l
9.1091 X 10-31 kg

I
8.314 x 107 erg deg-1mor' 8.314 J K- mol-
.
1

..

1 pound (lb)=453.6g Molar volume 22.4 L mol- 1


1atomic IIm:ss unit (amu) == 1.6605 x 10- 24 g at NTP (Vm)
= 1.6605 x 1O- 27 kg Planck's constant 6.6252 x 10-27 erg sec 6.6252 X 10-34 J sec
(h)
= 1.492 x 10- 3 erg = 1.492 x 10- 10 J
Proton mass (mp) l.6726 x 10-24 g 1.6726 X 10-27 kg
3.564 X lOll cal =9.310 x 10 8 eV
Neutron mass 1.67495 x 10-24 g 1.67495 X 10-27 kg
=931.48 MeV (mn )
1atmosphere (atm) = 760 torr = 760 mmHg = 76cmHg Rydberg constant lQ9678 cm- 1
= 1.01325 x 10 5 Pa (R,,)

1calorie (cal)=4.1840x 10 7 erg =4.184 J Velocity of light 2.9979 x 1010 cm seC l 2.9979 X 108 m sec-I.
(c) in vacuum or 186281 miles sec-I
2.613 x 1019 eV
Faraday (F) 9.6487 x 104 C I equiv.
lcoulomb (coul) =2.9979 x 109 esu or 96500 C/equiv.
, 1curie (Ci) = 3.7 X 1010 disintegrations sec -I 0.8988xl0 1O N m 2C-2
lelectron volt (eV) 1.6021 x 1O-12 erg = L6021 x 10- 19 J or9x 109 N m 2e 2

= 3.827 xl 0-20 cal Derived SI Units


= 23.06 kcal mol- 1
Quantity with Symbol Unit (SI) Symbol
lerg 10-7 J = 2.389 x lO-scal 6.242 x 1011 eV
Velocity (v) metre per sec m
1electrostatic unit (esu) = 3.33564 x 10- 10 coul
Area (A) square metre
1faraday (F):= 9.6487 X 104 coul Volume (V) cubic metre
1dyne (dyne) = lO-s N Density (P) kilogram m- 3
1 joule = 107 erg = 0.2390 cal Acceleration (a) metre per sec2
llitre=1OOOcc 1000rnL=ldm3 Energy (E) joule (J) kgm 2
10- 3 m 3 Force (F) • newton (N) kgm
BASIC PRINCIPLES 31

Power(W) watt0N) 243.4 has four significant figures.


Pressure (P) pascal (Pa) N m-2
. 24.123 has five significant figures.
(ii) A zero becomes significant figure if it appears between
Resistance (R) ohm(n) V A-I
two non-zero digits. For example,
Conduction (e) ohm-I, mho, siemens m-2 kg- I S3 A2 or n- I 5.03 has three significant figures.
Potential difference volt (V) kgm 2 S-3 A-I 5.604 has four significant figures.
4.004 has four significant figures.
Electrical charge coulomb (C) A-s (ampere-second)
(iii) Leading zeros or the zeros placed to the left of the
Frequency (v) hertz (Hz) cycle per sec number are never significant. For example,
Magnetic tesla (T) kg S-2 A-I =N A-I m- I 0.543 has three significant figures.
flux x density
0.045 has two significant figures.
Popular Units and their SI Equivalents 0.006 has one significant figure.
(iv) Trailing zeros or the zeros placed to the right of the
Physical quantity Unit with symbol .Equivalent in SI unit· number are significant. For example,
Mass 1 amu larnu = 1.6605 x 10-27 kg 433.0 has four significant figures.
19
433.00 has five significant figures.
Energy 1 electron volt (eV) 1.602 x 10- joule
343.000 has six significant figures.
Length I A 10- 10 m (10- 1 om) (v) In exponential notation, the numerical portion gives the
number of significant figures. For example,
Volume litre 10-3 m 3 =dm 3
1.32 x 10-2 has three significant figures.
Force dyne 10-5 N
1.32 x 10 4 has three significant figures.
Pressure I atmosphere 760 torr (760 mm Hg)
(vi) The non-significant figures in the measurements are
101325 pa or 105 pa rounded off.
1 bar 101325 pa or 105 pa (a) If the figure following the last number to be retained is
1 torr 133.322 N m-2 less than 5, all the unwanted figures are discarded and
the last number is left unchanged, e.g.,
Dipole moment debye,IO- 1S esu-cm 3.324 x 10-30 cm
5.6724 is 5.67 to three significant figures.
Magnetic flux density gauss (0) 10-4 T (b) If the figure following the last number to be retained
Area of nuclear I bam 10-28 m 2 is greater than 5, the last figure to be retained is
cross section increased by I unit and the unwanted figures are
discarded, e.g.,
Nuclear Diameter 1 fermi (1 femto) 10-15 m
8.6526 is 8.653 to four significant figures.
(c) If the figure following the last number to be retained
Significant Figures
is 5, the last figure is increased by 1 only in case it
There is always some degree of uncertainty in every scientific happens to be odd. In case of even number the last
measurement except in counting. The uncertainty in . figure remains unchanged.
measurement mainly depends upon two factors: 2.3524 is 2.4 to two significant figures. '
(i) Skill and accuracy of the observer,
7.4511 is 7.4 to two significant figures.
(ii) Limitation of the measuring scale.
To indicate the precision of a measurement, scientists use the Calculations Involving Significant Figures
term significant figures. The significant figures in a number are In most of the experiments, the observations of various
all certain digits plus one doubtful digit. The number of measurements are to be combined mathematically, i. e., added,
significant figures gives the information that except the digit at subtracted, multiplied or divided as to achieve the fmal result.
extreme right, all other digits are precise or reproducible. For Since, all the observations in measurements do not have the same
example, mass of an object is 11.24 g. This value indicates that precision, it is natural that the final result cannot be more prec~se
actual mass of the object lies between 11.23 g and 11.25 g. Thus, than the least precise measurement. The following two rules
one is sure of frrst three figures (1, 1 and 2) but the fourth figure should be followed to obtain th~ proper number of significant
is somewhat inexact. The total significant figures in this number figures in any calculation.
are four. Rule 1: The result of an addition or subtraction in the
The following rules are observed in counting the number of numbers having different precisions should be reported to the
significant figures in a given m;asured quantity: same number of decimal places as are present in the number
(i) All non-zero digits are significant. For example, having the least number of decimal places. The rule is illustrated
42.3 has three significant figures. by the following examples:
I
32 G.R.B. PHYSICAL CHEMISTRY FOR COMPETITIONS

(a) 33.3 ~(has only one decimal place) posses~ed by the least precise term used in the calculation.
3.11 Examples are:
0.313 (a) 142.06
Sum 36.723 f-, (answer should be reported x 0.23 ~(two significant figures)
to one decimal place) 32.6738 ~ (answer should nave two

Correct answer = 36.7 significant figures)


Correct answer = 33
(b) 3.1421 (b) 51.028
0.241 x 1.31 ~(three significant figures)
0.09 ~(has 2 decimal places) 66.84668
Sum 3.473.1 ~ (answer should be reported to Correct answer = 66.8
2 decimal places) 0.90 = 0.2112676
(c)
Correct answer 3.47 4.26
(c) 62.831 ~(has 3 decimal places) Correct answer 0.21
- 24.5492 Note: (i) Same procedure is followed if an expression involves
multiplication as well as division. .
Difference 38.2818 ~ (answer should be reported
(ii) The presence of exact numbers in an expression does not
to 3 decimal places after affect the number of significant figures in the answer.
rounding off) Examples are:
Correct answer = 38.282 (a) 3.24 x 0.0866 = 0.055643 (b) 4.28 x 0.146 x 3 =44.84784
5.046 0.0418
Rule 2: The answer to a multiplication or division is
rounded off to the same number of significant figures as is Correct answer = 0.0556 Correct answer == 44.8

MISCELLANEOUS NUMERICAL EXAMPLES4'/////


."""" ..
Example 1. 0.44 g of a hydrocarbon on complete Solution: ~ccording to law of conservation of mass,
combustion with oxygen gave 1.8 g water and 0.88 g carbon Mass of lime + Mass of carbon dioxide = Mass of calcium
dioxide. Show that these results are in accordance with the law of carbonate
. conservation of mass.
560 g + Mass of CO 2 = 1000 g
Solution: A hydrocarbon is a compound which consists of
carbon and hydrogen only. It undergoes combustion forming Mass of CO 2 = 1000 - 560 = 440 g
carbon dioxide and water as products. Molecular mass of CO2 = 12 + 32 = 44 g (l mole)
Formula of carbon dioxide = CO 2 ;
Molecular mass 12 + 32 44 g No. of moles in 440 g of CO2 = 440 = 10
. 44
Formula of water H 2 0;
Molecular mass 2+ 16 = 18g 1 mole of CO 2 occupies volume at NTP = 22.4 litre
12 10 moles of CO 2 will occupy volume at NTP
Mass of carbon in 0.88 g of CO2 x 0.88 = 0.24 g
44
= 22.4 x 10·= 224 litre
Mass of hydrogen in 1.8 g of H 2 0 = 2 x 1.8 = 0.20 g Example 3. 10 mL of hydrogen combine with 5 mL of
18
Total masses of carbon and hydrogen in the products oxygen to yield water. When 200 mL of hydrogen at NTP are
0.24 + 0.20 = 0.44 g passed over heated CuO, the CuO loses 0.144 g of its mass. Do
these results correspond to the law ofconstant proportions?
This is equal to the mass of hydrocarbon before combustion.
Thus, the results are in accordance with the law of Solution: 1st Case:
conservation of mass. 2
Mass of 10 mL hytlrogen at NTP = - - x 10= 0.00089 g
Example 2. Calcium carbonate decomposes completely, 22400
on heating, into lime (CaO) and carbon dioxide (C0 2 ), 1 kg of 32
calcium carbonate is completely decomposed by heat, when 560 g Mass of5 mL of oxygen at NTP = - - x 5:::: 0.00714 g
22400
of lime are obtained. How much quantity of carbon dioxide in
grams, moles and litres at NTP is produced in the process?
BASIC PRINCIPLES I 37

8.923 0.884 X 10- 16 cm 3


Mass of 22400 mLofHg vapouratNl})"" - - x 22400 .
,',,' 1000 ' .
Volume 0.884 x 10- 16
199:87 g' Mass of virus = = -----
Specific volume 0.75
Hence, molecular mass of Hg 199.87g
::: 1.178xlO- 16 g
.,. 6.4
(c) Approxunate atotnlc mass = - - - ,6.4 193.93 Molar maSs of vi~ = Mass of single virus x 6.023 x 1023 .
Sp. heat 0.033 g
= 1.178 X 10- 16 x 6.023 X 1023
Valency of Hg = 193.93:::: 2 (nearest whole number)
100 . = 7.095 xl0 7

So, accurate atomic mass Eq. mass x Valency Example 27. Weighing 31 04 carats (1 carat::: 200 mg), the
Cullinan diamond was the largest natural diamond ever found.
= 100 x 2= 200g How many carbon atoms were present in the stone?
.. Mol. mass 199.88_
A tOI11lClty ==--- 1 Solution: Mass of the stone
At. mass 200
= 3104 x 200= 620800mg::::: 620.8g
Hence, mercury molecules are monoatomic. Number of atoms of carbon
Example 23. How many grams of CaO are required to
Mass in gram x 6.023 x IOZ3
neutralise 852g ofP401O? (lIT 2005)
Gram-atomic mass
Solution: The reaction will be:
6CaO+ P4010 ~2Ca3(P04h 620.8 x 6.023 x 10 23 =3.12 X 1015
12
852 g P40 lO == 3 mol P40 lO Example 28. A cylinder of compressed gas contains
1 mole of P4010 neutralises 6 moles ofCaO. nitrogen and oxygen in the ratio 3: I by mole. If the cylinder is
known to contain 2.5 x 104 g ofoxygen, what is the total mass of
.. 3 moles OfP401O will neutralise 18 moles of.CaO.
the gas mixture?
Mass of CaO= 18;- 56=1008g
Solution: Number of moles of oxygen in the cylinder
. ' Example 24. If 1 grain is equal to 64.8 mg, how many\
Mass in 2.5 x 10 4
moles of aspirin (mol. wt: == 169) are present" in ,'tt 5 grain aspirin \
tablet? . \ Molecular mass in gram 32
Solution: Mass of aspirin in the tablet == 64.8 x 5 324 mg = 781.25
== 0.324g NumberofmoiesofN 2 3x781.25 2343.75
Number of moles = Mass = 0.324 Mass of nitrogen in the cylinder 2343.75 x 28
Molar mass 169
65625g
1.92 x 10-3
= 6.5625 x 104 g
Example 25. If the volume occupied in a crystal by a
Total mass of the gas in the cylinder
molecule of NaCI is 47 x 10-24 mL, calculate the volume of the
crystal weighing Ig. = 2.5 xl 04 + 6.5625 X 104 9.0625 X 104 g
Solution: Number of molecules of NaCI Example 29. Atmospheric air has 78% N z ;21% 02;
Mass x 6.023 x 1023 0.9 % Ar and 0.1 % CO 2 by volume. What is the molecular mass
Molar mass
_1_' x 6.023 x 1023 = 1.03x·1022
ofair in the atmosphere?
Solution:
.
Molecular mass of mixture
58.5
,L %ofeach M 1
- - - - x 0 ar mass
Volume of crystal = L03 x 1022 x 47 X 10-24 = 0.484 mL 100
,Example 26. A plant virus is found to consist of uniform 78 21 0.9 0.1
= x 28+-x 32+-x40+-,x44=28.964
cylindrical particles of 150 A in diameter and 5000 A long. The 100 100 100 100
specific volume ofthe virus is 0.75cm 3 /g. Ifthe virus is considered
Example 30. The famous toothpaste Forhans contains 0.76
to be a single particle, find its molecular mass. (lIT 1999)
g of sodium per gram of sodium monofluoroorthophosphate
Solntion: Volume of cylindrical virus = nrzl

3.14 x C~O x 10- 8


r x 500Qx 10-
8
(Na3P04F)in 100mL.
(a) How many fluorine atoms arepresent?
(b) How much fluorine in milligrams is present?
38 G.R.B.
-
PHYSICAL CHEMISTRY FOR COMPETITIONS

Solution:
Molar mass ofNa3P04F 3 x 23 + 31+ 16 x 4 + 19= 183
183 g Na3P04Fcontains 19 g fluorine
22. x gram of CaC03 was completely burnt in air. The mass of the
.. 0.76g Na3P04Fcontains 19 x 0.76g fluorine solid residue formed is 28 g. What is the value of' x' in gram?
183
(EAMCET 20(5)
:::: 0.0789 g= 78.9 mg fluorine (a) 44 (b) 200 (c) 150 (d) 50
Number of fluorine atoms [Ans. (d)]
_M_ass_in--=-_ _ x 6.023 xl 0 23 [Hint: CaC0 3(s) CaO(s) + CO 2 (g)
lOOg 56g
Gram-atomic mass
56 g residue 100 g CaC0 3
= 0.0789 x 6.023 x 1023
19 :. 28 g residue 50g CaC03]
= 2.5 x 1021 atoms 23. The mass of carbon anode consumed (giving only carbon
dioxide) in the production of270 kg of Al metal from bauxite
t~;txample31. An alloy of iron (54.7%), nickel (45%) and by Hall process is:
manganese (0.3%) has a density of 8.17 g/cm 3 . How many (a) 270 kg (b) 540 kg (c) 90 kg (d) 180 kg
iron atoms are there in a block of alloy measuring [Ans. (c)]
10cmx 20cmx 15 cm? [Hint: 3C + 2Al Z0 3 --,--74Al + 3C02
3x 12g 4X27=108g
Solution:
Volume of the block of alloy = lOx 20x 15cm3 .: 108 g Al is produced by consuming == 36 g carbon
:. 270 x 103 g AI will be produced by consuming
:::: 3000cm 3
,Mass of the block 3000 x 8.l7g ::::2451Og 36 x 270 x 103 g carbon
108
Mass of i,ron in the block
54.7 x 24510; 13406.97 g 90 x 103 g':" 90 kg carbon]
100
24. The equivalent mass of an element is 4. Its chloride has
Number of iron atoms in the block __M_a_ss__ x 6.023x 10 23
Atomic mass vapour density 59.25. Then the valency ofthe element is:
004 003 ~2 WI
13406.97 x 6.023 x 1023 [Ans. (b)]
56
[Hint: Molecular mass of MCI n == 59.25 x 2 = 118.5
:::: 1.442 X 1026
a + 35.5 x n= 118.5 ... (i) .
iiS'Example 32. An analysis of pyrex glass showed 12.9%
B 20 3 , 2.2% A12 0 3 , 3.8% Na20, 0.4% K 20 and remaining is Equivalent mass x n + 35.5 x n =118.5
Si0 2. What is the ratio ofsilicon to boron atoms in the glass ? 4n + 35.5n == 118.5 ... (ii)
'(BCECE 20(7)
n= 3]
Solution:
25. Sulphur trioxideis prepared by the fol1owing two reactions:
Percentage compositio~ ofB z 0 3 12.9%
S8(s) + 802 (g) ~ 8S02~g)
Percentage composition of
SiOz 100 - [12.9 + 2.2 + 3.8 + 0.4] 2S02(g) + 02(g ) ~ 2S03 (g)
How many grams of S03 are produced from 1 mole S8?
80.7%
(a) 1280 (b) 640
,. Mass . 12.9
NumberofmoiesofB 20 3 == =-=0.184 (c) 960 (d) 320
, Molar mass 70 [ADs. (b)]
Number of moles of boron atoms 2 x 0.184 [Hint: From the given reaction, it is clear that, I mole Sg will
give 8 moles of S03'
Number of moles ofSi0 2 Mass = 80.7 = 1.345 :. Mass of S03 formed will be = 80 x 8 = 640 g.]
Molar mass 60 26. Calculate the number of millilitresa.t STP of H2 S gas needed
Number of moles of silicon atoms 1.345 to precipitate cupric sulphide completely from 100 mL of a
Number of atoms of silicon NA x 1.345 :::: 7.3 solution containing 0.75 g ofCuCI 2 in 1 L.
(a) 21.4 (b) 14.2
Number of atoms of boron NA x 0.184 1
(c) 41.2 (d) 124
Where, NA = Avogadro's number [Ans. (d)]
BASIC PRINCIPLES 39
[Hint: CuCl 2 + H2S ~ CuS + 2HCI the values of x and yare:
(a) 40,40
Number of moles of H2S = Number of moles of CuCl 2
(c) 30,30
=~=~.00557 [Ans. (a)]
134.5
[Hint: In RHS, there are 40 hydrogen atoms, hence only
Volume of H 2S =0.00557 x 22400 =124.8 mL J option (a) will be suitable.]
27. In the reaction,
As 2 SS +xHN0 3 ~5H2S04 + yN0 2 +2H 3 As0 4
+ 12H2 0

SUMMARY AND IMPORTANT POINTS TO REMEMBER


~_. -fa' 111,- • ... ... .... 111,_ ,lilt
1. Chemistry: Branch of physical science which deals properties ofa compound are altogether different from the
with the properties, composition and changes of matter. It properties of elements from which it has been constituted.
has several branches. Main branches are (i) organic 12. Mixture: A material containing two or more substances
(ii) inorganic (iii) physical and (iv) analytical.It is wide in its (elements or compounds) in any proportion, in which
scope and touches- almost every aspect of our lives. components do not lose their identity. Homogeneous mixtMre
2. Matter: It is anything which has mass and occupies space. has a single phase while heterogeneous has more than one
Matter exists in three physical states (i) solid (ii) liquid and phase. Mixture can be separated into components by
(iii) gas. It is chemically classified _into (a) elements physical methods.
(b) compounds and (c) mixtures. 13. Alloy: A homogeneous mixture of two or more
3. Energy: The capacity of doing work. It is of various forms. elements-metal and metal, ni.etal and non-metal or
One form can be converted into another but cannot be created non-metal and non-metal. They have unique properties.
c or de~troyed. The total amount of matter and en~:rgy available 14. Physical change: A temporary: change.mllo change in
in the universe is constant. The relationship between mass and chemical composition and mass. Physical properties alter. It
energy is given by Einstein equation, E = me 2 (where, can be reversed easily.
E energy, m mass, e velocity of light). 15. Chemical change: A permanent change, new substance is
4. Intensive properties: Do not depend on the quantity of formed which possesses different composition and
matter, e.g., colour, density, melting point, boiling point, etc. properties. It cannot be reversed easily. Chemical changes
5. Extensive properties: Depend on the quantity of matter, are of various types. The important ones are decomposition,
e.g., volume, mass, weight, etc. synthesis, substitution, addition, internal rearrangement,
6. Substance: A variety of matter, all samples of which have polymerisation, double decomposition, etc.
the same composition and properties. Pure substances are 16. Law of conservation of mass: (Lavoisier-:-1774) In a
divided into (i) elements and (ii) compounds. chemical change, mass is neither created nor destroyed. In
7. Element: A substance which cannot be decomposed into chemical reactions: ._.
anything more simpler by ordinary physical or chemical Total masses of reactants == Total masses of products.
means. 117 elements are known. 88 elements have been 17. Law of constant proportfons: (Proust-1799) A chemical
isolated from natural sources and remaining 29 have been compound always contains the same element combined
prepared by artificial means. Every element is represented by together in fixed proportion by mass.
a symbol which is a small abbreviation of its full and lengthy 18. Law of multiple proportions: (Dalton-l 808) When two
name. Oxygen is the most abundant element. Silicon, elements combine to form two or more compounds,_ the
aluminium, iron are second, third and fourth most abundant - different masses of one element which combine with a fixed
elements. Elements are classified as (i) metals mass of the other element, bear a simpl~ ratio to one another.
(ii) non-metals and (iii) metalloids. 19. Law of reciprocal proportions: (Richter-1794) When
8. Metals: Generally solids (Hg-exception). They have two different elements combine with the same mass of a
properties such as lustre, hardness, malleable, ductile, good third element, the ratio in which they do so will be the same
conductors of heat ·and electricity. Copper, zmc, iron, or simple multiple ifboth directly combine with each other.
aluminium are metals. In all chemical reactions, substances react in the ratio of their
9. Non-metals: Usually non-lustrous, brittle and poor equivalent mass~~.
conductors of electricity. Oxygen, carbon, nitrogen, 20. Law of gaseous volumes: (Gay-Lussac-1808) Gases
chlorine, helium, etc., are non-metals. react with each other in simple ratio of their volumes and if
10. Metalloids: Possess mixed properties of :metals and product is also in gaseous -state, its volume also bears a
non-metals both (e.g., As, 8b, 8n). simple ratio with the volumes of gaseous reactants under
11. Compound: Pure substance composed of two or more similar conditions of temperature and pressure.
different elements in a fixed proportion of mass. The
40 G.R.B. PHYSICAL CHEMISTRY FOR COMPETITIONS

21. Dalton's atomic theory: Every element is composed of 31. Gram molar volume: The volume occupied by one
small indivisible, indestructible particles called atoms. gram-molecular mass of any gas at NTP (O°C or 273 K and
Atoms of the same element are identical but differ in one atm or 76 em ofHg as pressure). Its value is 22.4 litre.
properties, mass and size. of atoms of other elements. Atoms 32. Vapour density:
of differe1Jt elements combine in. simple ratio to form Density of a gas
compounds. The relative number. and kind of atoms are V.D.
Density of hydrogen
always the same in a given compound. Atoms cannot be
Mass of a certain volume of a gas
created or destroyed.
Mass of same volume of hydrogen
22. Atom: The smallest particle of an element that takes part
in a chemical reaction. under same temperature and pressure
23 •. Molecule: The smallest· particle of· art element or 2Y.D.= Molecular mass
compound that can have a stable existence. 33. Mole: Amole(mol) isdefmed~sthenutnber of atoms in
24. For:mula: Gronp of symbols of elements which represents P,9 g of carbon-12 .. The number Qfatoms..is 6,07X10 23 •
one molecule of a substance. It represents also the chemical This number is called Avogadro's number.
composition. Mass of substan~e in gram
1
N 0.0f moes=--------------------~-------
25. Atomic
.
mass: Atomic mass of an1 element is the·ratio. of·
. Mass of one mole of the substance in gram
mass of one atom of an element.to 12th part of the mass of No. of particles
carbon.,,12. . i'
6.02 X 1023
Atomic mass of an element Volume of gas in Htres at NTP
= Mass of one atom of the element X 12 = 22.4
Mass of one atom of carbon-12 Mass of one atom of an element
26. Atomic mass unit (amu): ~th mass of carbon-12. It is Gram atom o'f an element
= 6.02 x
equal to 1.66 x 10-24 g.
Atomic mass of an el<~ment Mass of one molecule of a substance
Gram-molecular mass of a substance
Mass of one atom of the element
lamu 6.02 x 1023
34. Equivalent mass: The number of parts by mass of the
The actual mass of an atom of element Atomic mass in
substance which combine Or displace directly or indirectly
amu x 1.66 x 10-24 g. 1.008 parts by mass of hydrogen or 8 parts by mass of
The atomic masses of elements are actually average relative oxygen or 35.5 parts by mass of chlorine or 108 parts by
masses because elements occur as mixture of isotopes. mass of silver.
27. Gram-atomic mass or Gram atom: Atomic mass The equivalent mass of an element may vary with change of
expressed in grams. It is the absolute· mass· in grams of valency.
6.02 x 1023 atoms of any element. Eq. mass of an element
Mass of element in Mass of eleme;nt x 1.008
N 0.0f gram atoms = .
. Atomic mass of the element in grams Mass of hydrogen
Mass of element x 11200
28. Molecular mass: It indicates how many times one =-------------------------------
Volume in mL of hydrogen displaced at NTP
molecule of a substance is heavier in comparison to ~th of
. mass of one atom of carbon-12. Mass of a molecule is equal
= Mass of element x 8
Mass of oxygen
to sum of masses of the atoms present in a molecule.
29. Gram-molecular mass· or Gram molecule: Molecular Mass of element x 35.5
mass expressed in gram. It is the absolute mass in gram of Mass of chlorine
6.02 x 1023 molecules of any substance. . . m· E
35. Metal to metal displacement: _1 =_1
No. of gram molecules ~ E2
Mass of a substance in 36. Double decomposition: AB + CD ~ AD + CB
ppl.
. Molecular mass of the substance in gram
Mass of AB massofB
30. Avogadro's hypothesis: Under similar conditions of Mass of AD
temperature and pressure, equal volumes of all gases contain
same number of molecules. . Atomic mass of an element
:: Eq. mass of 1he element x Valency
I
BASIC PRINCIPLES 41

37. Dulong and Petit's law: Molecular formula mass


n'=--------
Atomic mass (approximate) = ~A< ·J~;mpirical formula mass
. SpeCIfic heat >(iM Structimi1:'It represents the way in which atoms of
38. Cannizzaro's method: Atomic mass of an element is the various elements are linked with each other.
smallest mass of the element present in the molecular mass 42. Percentage of element:
of anyone of its compounds. Mass of element
Percentage of element = x 100
39. Law of isomorphism: Isomorphous compounds form Molecular mass .
crystals wnicnhave same size and shape and can grow in the 43. Chemical equation: It is a symbolic representation of a
saturated solution of each other. chemical change. The equation becomes balanced when total
Masses of two elements that combine with same mass of number of atoms of various elements are made equal on both
other elements in their respective compounds are in the ratio the sides of equation. Chemical equations are of two types
of their atomic masses. (i) molecular and (ii) ionic. Chemical equation is based on
40. Atomic mass from vapour density of a chloride: law of conservation of mass. '
2 V. D. of a volatile chloride 44. Unit: It is the primary standard chosen to measure any
V aIency 0 f an e1ement = - - - - - - - - - -
Eq. mass + 35.5 physical quantity.
41. Types offormulae:. The seven units of measurement, namely mass, length, time,
(i) Empirical: It represents the simplest relative whole temperature, electric current, luminous intensity and amount
number ratio ofatoms of each element present in the of substance are taken as basic units. All other units can be
molecule of a substance. derived from them and are, therefore, called derived units. SI
(il) Molecular: It represents the actual number of atoms of units are used these days in all branches of science.
each element present in one molecule of a substance. 45. Significant ,figure: It is the total number of certain digits .
plus one doubtfu,l digit. .
Molecular formula =n x Empirical formula.
42 G.R.B. PHYSICAL CHEMISTRY FOR COMPETITIONS

Questions
1. Match the following, choosing one item from Column-X and 2. Matrix Matching Problems:
the appropriate related item from Column-Y. (According to the new pattern of lIT Screening)
[A] [A] Match the Column-X and Column-Y:
Column-X Column-Y Column-X Column-Y
( a) Efflorescence (i) Homogeneous mixture (a) Vapour density (i) Unitless
(b) Malleability (ii) Heterogeneous mixture (b) Mole (ii) I mol electrons
(c) Alloy (ii9 Mole (c) 12 g carbon (iii) Collection of 6.023 x 1023
(d) "1 amu (iv) (l/12)th mass of atoms
carbon-l 2 1
(d) 96500 C . (iv) Molecular mass x -
2
(e) Sulphurandsand (v) Tendency to lose water of
crystallisation [B] Match the Column-X and Column-Y:
(f) Amount of substance (vi) Property ofmetalbeiilg .... -Column-X .Column-Y
hammered into thin sheets
(a) 1.6 g CH4 (i) 0.1 mol
[B]
(b) 1.7 gNH) (ii) 6.023 x 1023 electrons
Column-X Column-Y
(c) HCHO (iii) 40% carbon
. (a) Equal volumes of all (i) Dalton's atomic theory (d) C6H 120 6 (iv) Vapour density =15
gases contain equal
[C] Match the Column-X and Column-Y:
number of molecules at
NTP. Column-X Column-Y
(b) The atom is indestruc- (ii) Law of conservation of (a) 1 amu (i) Heaviest particle of atom
tible. mass (b) Proton (ii) 1.66 x 10-27 kg
(c) All pure samples of the (iii) Avogadro's law
same compound contain (c) Neutron (iii) 931.5 MeV
the same elements (d) a-particle (iv) Positively charged
combined in the same
[D] Match the Column-X with Column-Yfor the reaction:
proportion by mass.
A +B2 ~AB2
(4) Total mass before and (iv) Dulong and Petit's law
after the chemical Column-X Column-Y
reaction is same.
(a) 300 atoms of A + 200 (P) B2 is limiting reagent
(e) Atomic mass (v) Gay-Lussac's law molecules of B2
6.4
(b) 100 atoms of A + 100 (q) A is limiting reagent
Specific heat molecules of B2
(f) Gases react in simple (vi) Law of constant (c) 5 mol of A +2.5 mol (r) None of the reactant is in
ratio of their volumes. proportions ofB2 excess
[C] . (d) 2.5 mol of A + 5 mol (s) 200 molecules of A B2 will
ofB2 be formed
Column-X Column-Y
(a) Most abundant element (i) Platinum
(b) Most abundant metal (ii) Diamond
(c) Liquid at room temp. (iii) Aluminium
"
(d) Hardest substance (iv) Plutonium
(e) Most ductile metal (v) Mercury
(f) Transuranic element (vi) Oxygen
· BASIC PRINCIPLES 43

1. [A] (a-v); (b-vi); (c-i); (d-iv); (e-il); (f-iii) 2. [A] (a-i, iv); (b-iii); (c-iii); (d-ii)
[B] (a-iii); (b-i); (c-vi); (d-ii); (e-iv); (f-v) [B] (a-i, ii); (b-i, ii); (c-iii, iv); (d-iii)
[C] (a-vi); (b-iii); (c-v); (d-ii); (e-i); (f-iv). [C] (a-ii, iii); (b-ii, iv); (c-i); (d-iv)
[D] (a-p, s); (b-r); (c-p); (d-q).

1. The density of mercury is 13.6 glmL. Calculate the diameter [Ans. 9.41 x 1023 CH 4 molecules, 9.41 x IOz3 carbon atoms and
of an atom of mercury assuming that each atom of mercury is 37.64 X 1023 hydrogen atoms.]
occupying a cube of edge-length equal to the diameter of
mercury atom. [Hint: No. of moles of methane = 25.0
(Atomic mass of mercury 200) 16.0
[Ans. 2.9 x 10-8 cm] One molecule of methane contains one carbon atom and four
2. A metal M of atomic mass 54.94 has a density of 7.42 g/cc. hydrogen atoms.]
Calculate the apparent volume occupied by one atom of the 12. How much sugar (C1 zH 22 0 11 ) will be required if each person
metal.
[Ans. 1.23 x 10-23 cc] on the earth is given 100 molecules of sugar? The popUlation
o(the earth is 3 x 1010.
3. Find the charge of I g ion ofN 3- in coulomb.
[Ans. 2.894 x 105 coulomb] [Ans. 170.43 X 10- 11 g]
13. A mixture of hydrogen and oxygen contains 20% by mass of
4. Calculate the volume at NTP occupied by 6.25 g of nitrogen.
hydrogen. What is the total number of molecules present per
[Ans. 5.0 litre] gram of the mixture?
5. 10 mL of hydrogen contains 2 x 103 molecules of hydrogen at [Ans. 7.528 x 1022]
certain pressure and temperature. Calculate the }lumber of [Hint: In I gram of the mixture, 0.2 g of hydrogen and 0.8 g of
molecules of oxygen whose volume is 200 mL at the same
temperature and pressure. oxygen are present. Moles of Hz = 0.2 = 0.1, moles of oxygen
2
4
[Ans. 4 x 10 molecules]
= 0.8 = 0.025. Calculate the number of molecules of hydrogen
6. The masses of equal volumes of a gas and hydrogen are 25.6 g 32 .
and 0.8 g respectively under same conditions of temperature and oxygen and then add.]
and pressure. Find the molecular mass of the gas. 14•. How many electrons ate present in 18 mL of water?
(MLNR 1995)
[Hint: Y.D. of the gas 25.6 32.0
0.8 [Hint: 18 mL water = 18 g water:::: I mole water 6.02 x 1023
Molecular mass 2 V.D. 2 x 32.0= 64.0] molecules, each molecule consists 10 electrons (8 electrons per
oxygen atom, 2 electrons for two hydrogen atoms). Total
7. One litre of a gas at NTP weighs 1.97 g. Find the molecular electrons = 10 x 6.02 X 1023 = 6.02 X 1024]
mass of gas.
15. Sulphur molecule is known to be composed ofS atoms of the
[Ans. 44.128]
element. In a sample of 192 g of pure sulphur, calculate (i)
8. How many moles of water are present in one litre of water? number of g-atoms of sulphur; (il) number of atoms of
[Ans. 55.5 moles] sulphur; (iii) number of moles of sulphur; (iv) number of
9. Calculate the mass of 6.02 x 1021 molecules of nitrogen. molecules of sulphur.
[Ans. 0.28 gJ [Ans. g-atoms = 6; No. of atoms 6 x 6.02 x 1023 ; No. of
10. 1.5276 g ofCdCl 2 was found to contain 0.9367 g of cadmium. moles = 0.75; No. of molecules =4.52 x 1023]
Calculate the atomic mas~ of cadmium.
[Ans. 112.54] [Hint: The atomic mass of sulphur is 32.]

Mass of Cd x 35.5 16. The vapour density of a mixture containing N02 and N 20 4 is
[Hint: Equivalent mass of cadmium
Mass of CI 38.3 at 27°C. Calculate the moles of NO z in 100 g of the
mixture. (MLNR 1993)
= 0.9367 x 35.5= 56.27
0.5909 [Hint: Mol. mass of mixture 2 x 38.3 76.6
Atomic mass Equivalent mass x Valency]
11. Calculate how. many methane molecules and how many No. of moles in 100 g of mixture = 100
76.6
hydrogen and carbon atoms are there in 25.0 g of methane?
(MLNR 1990; Dhanbad 1992) Let a g ofN02 is present in mixture.
44 G.R.B. PHYSICAL CHEMISTRY FOR COMPETITIONS

Moles ofN0 2 + Moles ofN 20 4 Moles of mixture 23. Use the data given inthe following table to calculate the molar
a 100 a 100 mass of naturally occurring argon: . , ..
-+_._-=-- or a= 20.10 g
46 92 76.6 Isotope Isotopic molar mass Abundance
MoiesofN02 in mixture 20.10:= 0.437] 36Ar 35.96755 g mol- I 0.337%
46
38 Ar 37.96272 g mor l
0.063%
17. Calculate the number of oxygen atoms in 88 CO 2 , What g
would be the mass of CO having the same number of oxygen 4°Ar 39.9624 gmor l 99.6%
atoms? (BITS 1990)
[Ans. 39.947]
[Hint: 88 g CO 2 = 2 moles of CO 2 , One molecule consists of 2
24. Density of oxygen atNTP is 1.429 g/litre.' Calculate the
oxygen atoms. standard molar volume of the gas.
No. of oxygen atoms = 2 x 2 x 6.02 x 1023 = 24.08 x 1023 [Ans. 22.39 litre mol-I]
25. How many iron atoms are present in a stainless steel ball
CO molecule has one oxygen atom.
bearing having a radius of 0.254 cm? The stainless steel
Mass of CO containing 24.08 x 1023 oxygen atoms contains 85.6% Fe by weight and has density of7.75 glcm 3 .
__2_8---,;:;:- x 24.08 x 1023 112 g) [Ans. 4.91 x 1021 ]
26.. The nucleus of an atom. X is supposed to be a sphere with a
18. Density of water at room temperature is 1.0 g cm-3. How many radius of 5 x 10- 13 cm. Find the density of the matter in the
molecules are there in one drop of water if its volume is 0.1 atomic nucleus if the atomic weight of X is 19.
cm3? [Ans. 6.02 x 1013 glmL]
[Ans. 3.34 x 1021 molecules] 27. Calculate the number of atoms of each element present in
[Hint: Mass of one drop:= Vol. x d 0.1 xl = 0.1 g 122.5 g ofKC10 3 .
[Ans. Number of atoms of'K' = 1 x 6.023 X 1023
No. of moles = 0.1; No. of molecules 6.02 x 1023 x 0.1 ]
Number of atoms of 'Cl' = 1 x 6.023 X 1023
=0

18 . ·18
19. Naturally occurring bqron consists of two isotopes, whose Number of atoms of '0' 3 x 6.023 x 1023 ]
atomic masses are 10.01 and 11.01. The atomic mass of
28. In an experiment, 1.0 g CaC03 on heating evolved 224 mL of
natural boron is 10.81. Calculate the percentage of each
isotope in natural boron. (MLNR 1994) CO2 at NTP. What mass of Cao (calcium oxide) is formed?
[Ans. % of isotope with atomic mass 10.01 20; % of isotope [Ans. Mass ofCaO = 0.56 g]
with atomic mass 11.01 = 80] [Hint: Mass of 224 mL of CO 2 =44 x 224 0.44 g)
[Hint: Let x be the percentage of the isotope with atomic mass 22400
10.01. 29. What mass of potassium chlorate (KCI03 ) on heating gives
1.491 g of potassium chloride (KCl) and 0.6721itres of oxygen
IO.Olxx+l1.01(l00-x)~1O.81 or x=20]
atNTP?
100 100
[Ans, Mass ofKCI0 3 = 2.451 g]
20. Chlorine has isotopes 3sCI and 37 Cl. There are three 35CI
[Hint: Mass of 22.4 litre of oxygen at NTP = 32 g)
isotopes for every 37 CI isotope in a sample of chlorine.
30. A compound AB completely decomposes into A and B on
Calculate the atomic mass of chlorine.
heating. 50 g of AB, on strong heating, gave 40 g of A. How
[Ans. A = 3 x 35 + 37 x 1 == 35.5 ] much quantity of AB should· be decomposed by heating to
4 obtain 2.5 g of B? How much quantity of A will be produced
21. Natural hydrogen gas is a mixtUre of IH and 2H in the ratio of in the process?
5000 : 1. Calculate the atomic mass of the hydrogen: [Ans. 12.5 g AB is to be decomposed, 10.0 g of A will be
[Ans. 1.000199] produced.]
22. Chromium has tne following isotopic composition: [Hint: AB...-? A + B)
50g 4()g 109
. FractioDai
Mass number Isotopic mass 31. If 12.6 g of NaHC0 3 is added to 20.0 g ofHCI solution, the
abundance
residue solution is found to weigh 24.0 g. What is the mass and
50 49.9461 x volume of CO2 released at NTP in the reaction?
52 51.9405 0.8379 [Ans. 8.6 g CO 2 released. Volume at NTP = 22.4 x 8.6 = 4.378
53 52.9407 0.0950 44
litre]
54 I 53.9389 0.0236 32. (i) 5.06 g of pure cupric oxide (CUO), on complete reduction
Calculate the value of x. by heating in a current of hydrogen, gave 4.04 g of
[Ans. 0.0435] metallic copper.
BASIC PRINCIPLES 45

(ii) 1.3 g of pure metallic copper was completely dissolved 39. Phosphorus and chlorine form two compounds. The first
in nitric acid and the resultant solution was carefully contains 22.54% by mass of phosphorus and the second
dried and ignited. .1.63 g CuO was produced in the j4.88% of phosphorus. Show that these data are consistent'
process. Show that these results illustrate th~ law of with law of multiple proportions.
constant proportions. (Hint: The ratio of the masses of chlorine which combines with
(Ans. In both cases, the ratio of copper and oxygen is 1 : 0.25. a fixed mass of phosphorus in two compounds is 3 : 5 which is a
Hence, the law of constant proportions is illustrated.] simple whole number ratio. Thus, the data illustrate law of
33. Metal M and chlorine combine in different proportions to multiple proportions.]
form two compounds A and B. The mass ratio M : CI is 40. A and B are two hydrocarbons. A and B are heated separately
0.895: I in A and 1.791 : 1 in B. What law of chenrical in excess of oxygen when 0.028 g of A gave 44.8 mL CO2 and
combination is illustrated?' 0.044 g of B gave 67.2 mL CO2 at NTP. Show that the results
[Ans. Masses of metal which combine with 1 part of chlorine are in' agreement witlt law of multiple proportions.
are in the ratio of 1 : 2, which is a simple ratio. Hence, law of [Hint: Determine the masses of CO2 at NTP and then masses of
multiple proportions is illustrated.] .
carbon.
34. 2.8 g of calcium oxide (CaO) prepared by heating limestone . 44
were found to contain 0.8 g of oxygen. When one gram of (A) Mass of CO 2 = 22400 x 41.8 == 0.088 g,
oxygen was treated with calcium, 3.5 g of calcium oxide was
'Mass of carbon == 0.024 g, mass of hydrogen 0.004 g.
obtained. Show that the results illustrate the law of definite
44
proportions. (B) Mass of CO 2 == - .-x 67.2 == O.l32g,
22400
35. By means of the given analytical results show that law of
Mass of carbon = 0.036 g, mass of hydrogen == 0.008 g.
multiple proportions is true:
Thus, the masses of carbon combining with same mass of
Mercurous chloride Mercuric chloride hydrogen are in the ratio of 4 : 3 which is a simple ratio. Hence,
Mercury = '84:92 % Mercury = 73.80% law of multiple proportions is followed.]
41. Aluminium oxide contains 52.9% aluminium' and carbon
Chlorine 15.08% Chlorine = 2620% dioxide contains 27.27% carbon. Assuming the validity of the
[Ans. The masses of mercury which combine with I part of law of reciprocal proportions, calculate the percentage of
chlorine are in the 'ratio of 2 : I, which is a simple ratio. Hence, aluminium in aluminium carbide.
law of multiple proportions is illustrated.] [Hint: From the data, it is observed that the ratio of masses of
36. 1 g of a metal, having no variable valency, produces 1.67 g of aluminium and carbon in aluminium carbide should be 3 : 1 or its
its oxide when heated in air. Its carbonate contains 28.57% of simple mu1tiple. Hence, percentage of aluminium iIi aluminium
the metaL How much oxide will be obtained by heating 1 g of carbide = 2.4 x 100. = 75.0]
the carbonate?
[Ans. 0.477 g] 42. Two volumes of ammonia, on dissociation gave one volume of
nitrogen and three volumes of hydrogen. How much hydrogen
Mass of metal Mass of metal in 1g of carbonate
[Hint: will be obtained from dissociation of 40 mL ofNH) ?
Mass of oxygen x
[Ans. 60 mLl
i.e., x == 0.1914 g of oxygen 43, The following results were obtained by heating different
oxides oflead in a current of hydrogen:
Mass of oxide = 0.2857 + 0.1914 = 0.4771 g] . (a) 1.393 g oflitharge gave 1.29~ g oflead.
37. 0.36 g of Mg cbmbines with chlorine to produce 1.425 g of (b) 2.173 g of lead peroxide gave 1.882 g oflead.
magnesium chloride. 9.50 g of anothefsample of anhydrous (c) 1.721 g of red lead gave 1.552 g oflead.
magnesium chloride gave, on electrolysis 2.24 litre of chlorine Show that these results ar.e in accordance with the law of
at NTP. Show that these data agree with the law of constant multiple proportions .
.proportions. .(Ans. Masses of lead that combine with same mas~Mf oxygen
(Hint: Mass of 2.24 litre of chlorine at NIP == ~ x 2.24 are in the ratio of 4 : 2 : 3 which is a simple ratio. So, 'the results
22.4 are in accordance with the law of multiple proportions.}
== 7.1 g. In both cases, the ratio of masses ofMg and CI is 1 : 3. 44.. Calculate the number of g-moles of CaO that could be
Hence, law of constant proportions is followed.] obtained from 42.54 g of CaC03 and convert. the number of
g-moles to grams.
38. Carbon dioxide contains 27.27% carbon, carbon disulphide
contains 15.97% carbon and sulphur dioxide contains 50% .42.54
[No. of g-moles =: == 0.4254,
sulphur. Show that these figures illustrate the law of reciprocal . 100
proportions. Mass of CaO 0.4254 x 56 == 23.8 !II
[Hint: The masses of oxygen and sulphur which combine with 45. I g of a metal M which has specific heat of 0.06 combines
1 part of carbon are in the ratio of 2.667 : 5.25, i.e., 1 : 2. In with oxygen to form 1.08 g of oxide. What i~ the atomic mass
sulphur dioxide, the masses of sulphur and oxygen are in the ratio ofM? . '
of 1 :. 1 which is a simple multiple of first. Hence, law of
reciprocal proportions is illustrated.]
46 G.R.B. PHYSICAL CHEMISTRY FOR COMPETITIONS

. . . 6.4 51. If the equivalent mass of a metal (M) is x and the formula of its
[Hint: A pproxunate atOIll1C mass = = 106.6
0.06
oxide is M m On' then show that the atomic mass of Mis 2xn
m
Equivalent mass of M = I x8 100
[Hint: m atoms of M combine with n atoms of oxygen.
0.08
106.6 1 atom of M combines with!!.. atoms of oxygen.
Valency = - - "" 1 m
100
· 2n
Exact atomic mass = I 00 x I =100 ] Hence, Vaency
I =-
m
,-'
A compound .contains 28% of nitrogen and 72% metal by
mass. 3 atoms of the metal combine with 2 atoms of the Atomic mass = Equivalent mass x Valency
nitrogen. Find the atomic mass of the metal. 2n 2nx]
xx
[Hint: Valency of metal = 2 and valency of nitrogen 3 m m
52. Two oxides of metals A and B are isomorphoUS. The metal A
. I f0 ' . mass of metal 72
EqUlva ent mass nItrogen =-14 ; whose atomic mass is 5,2, forms a chloride whose vapour
. 3 14/3 28 density is 79. The oxide of the metal B contains 47.1%
Equivalent mass of metal = 12 '. oxygen. Calculate the atomic mass of B.
[Hint: Let the valency of A be x. The formula of chloride
Atomic mass of metal == 12 x 2 =24]
= ACl"
47. The chloride of a solid metallic element contains 57.89% by
2 V.D. A + x x 35.5 or x",3
mass of the element. The specific heat of the element is 0.0324
cal deg -1 g -I. Calculate the exact atomic mass of the element. As the two oxides are isomorphous, the valency of B is also 3.
-: Ient mass 0 f B =--
EqUiva 52.9 x 8 = 8.99 atomic mass of
[Hint: Equivalent mass of the element = 57:89 x 355:::: 48.8 -, 47.1'
, . 42.11
B = ~.99 x 3 26.97]
Approximate atomic mass 200 53. A mixture of 1.65 x 1021 molecules of X and 185x ler l
0.0324
molecules ofY weighs 0.6.88 g. If molecular mass ofY is 187,
200 what is the molecular mass of X ?
Val ency =-""4 21 21
.. 48.8 [Hint: A x 1.65 X 10 + 187 x 1.85 x 10 =0.688 A 4135]
23
Exact atomic mass = 48.8 x 4 195.2] 6.02 x 6.02 x 10 '

54. The equivalent mass of a metal is 29.73 and the vapour density
48. Two oxides ofametal contain 63.2% and 69.62% of the metal.
of its chloride is BOA. Find out the atomic mass of the metal.
The specific. heat of the metal is 0.117. What are the formulae
of the two oxides? [Ans. Atomic mass 118.92]
[Ans. M0 2 andM20 3]
2 x V.D. 2 x (BOA)
49. White vitriol (hydrated zinc sulphate) is isomorphous with [Hint: Valency
Eq. mass + 35.5 = (29.73 + 35.5) "" 4]
MgS04 ·7H2 0. White vitriol contains 22.95% zinc and 43.9%
of water of crystallisation. Find the atomic mass of zinc. 55. Calculate the percentage of aluminium, sulphate radical and
water in potash alum.
[Hint: The formula of white vitriol should be znS0 4· 7HP as it
[Ans. Al = 5.6<)010; SO!- 40.51%; Water = 45.57%]
is isomorphous to MgS0 4 ·7H20, i.e., 7 water molecules are
56. Carbohydrates are represented by the general formula
associated with one zinc atom. 7HP = 7 x.18 126. Mass of Zn
with which 126 parts of water by mass ate associated Cm (H2 0)". On heating, in absence of air, they decompose into
22.95 x I 26 = 65. 87. Atomic mass of. zinc.] steam (HP) and carbon. 3.1 g of a carbohydrate,\on complete
= decomposition by heating in absence of air, leave a residue of
.j. 43.~0 .,
1.24 g of carbon. If the molecular mass of the carbohydrate be
50. li\.. solid element. burns in oxygen without a~y change in
J 80, find its molecular formula.
volwne (of gas) under similar conditions 01 temperature and
pressure. If the vapour density of pure gaseous product is 32, [Hint: Determine % of carbon in carbohydrate. It is 40%. Water
what is the equivalent mass of the element? is 60%. Empirical formula == CHzO. Molecular formula
[Hint: One vol. of oxide contains 1 vol. of 02' = 6 x CHP CJI1206']
57. A gaseous hydrocarbon contains 85.7% carbon and 14.3%
One mole of oxide contains one mole of 02' hydrogen. 1 litre of the hydrocarbon weighs 1.26 g at NTP.
Mol. mass of oxide = A + 32 = 2 V.D. 64 Determine the molecular formula of th~ hydrocarbon.
[Ans. C2H4]
So, A = 32 58. Equal masses of oxygen, hydiogen and methane are taken in a
contairier under identical conditions. Find the ratio of their
32 parts of element combine with'32 parts of oxygen.
volumes.
Equivalent mass of element = 32 x 8 8]
[Ans. 1: 16: 2]
So,
32
BASIC PRINCIPLES 47
59. How many moles are there in 1 m 3 of any gas at NTP? [Hint: Formula of divalent hydrated metal sulphate will he
[Ans. 44.6 moles] MS0 4 ·xH20
60. A hydrated chloride of metal contains 18.26% metal and
Molecular mass of salt = 65.4 + 96 + 18x
32.42% chloride ion by mass. The specific heat of metal is
0.16. What is hydrated chloride? = (161.4 + 18x) .
[Ans. CaCI 2 ·6H20] 18x
% of water = x 100 = 43.85
61. 1.878 gof MBrx when heated in a stream of HCl gas was 161.4 + 18x
completely converted to chloride MCl x which weighed 1.0 g. On solving, x=7
The specific heat of metal is 0.14 cal g-l. Calculate .the
.'. Molecular formula of hydrated salt = MS0 4 •7H 20 ]
molecular masses of metal bromide and metal chloride.
70. A person with fever has a temperature of 102.SOF. What is the
[Ans. Mol. mass of metal bromide = 285.54;
temperature in degree celsius?
Mol. mass of metal chloride = 1522]
[Hint: Use C= 5(F- 32)]
62. An au~omobile antifreeze consists of 38.7% C; 9.7% H and . 9
remaining oxygen by weight. When 0.93 g of it are vaporised
71. An ornamental ring contains 275 carats of diamond. How
at 200°C and 1 atm pressure, 582 mL of vapour are formed.
many grams diamond does it have?
Find the molecular formula of the antifreeze.
[Hint: 1 carat = 200 mg
[Ans. C;Ift;02]
:. Mass of diamond = 275 x 200 x 10-3 g]
63. A mineral contained MgO = 31.88 %; Si02 = 63.37% and
H20 = 4.75%. Show that the simplest formula for the mineral
72. 1 volume of a gaseous compound consisting C, H, °
on
complete combustion in presence of 2.5 volume of O2 gives 2
is H2Mg3Si4012.
°
(H = 1; Mg= 24; Si = 28; = 16)
vol. of steam and 2 vol. of CO 2. What is the formula of the
compound if all measurements are made at NTP?
64. How m~ny moles of NH3 are there in 250 cm 3 of a 30%
[Ans. C2H40]
solution, the specific gravity of which is 0.90?
73. 60 mL of a mixture of nitrous oxide and nitric oxide was
[Ans. 3.97 moles]
exploded with excess of hydrogen. If 38 mL of N2 was
65. Haemoglobin contains 0.25% iron by mass. The molecular formed, calculate the volume of each gas in the mixture.
mass of haemoglobin is 89600. Calculate the number of iron
[Ans. NO = 44 mL and N20 = 16 mL]
atoms per molecule of haemoglobin.
[Atomic mass of Fe = 56] 74. For a precious stone, 'carat' is used for specifying its mass. If 1
[Ans. 4 atoms] carat = 3.168 grains (a unit of mass) and 1 gram = 15.4 grains,
find the total mass in kilogram of the ring that contains 0.5
66. A sample of potato-starch was ground to give a starch like
carat diamond and 7 gram gold.
molecule. The product analysed 0.086% phosphorus. If each
molecule is assumed to contain one atom of phosphorus, what [Ans. 7.1 x 10-3 kg]
is the average molecular mass of the material? 75. The density of a gaseous element is 5 times that of oxygen
[Ans. 36000 amu] under -similar conditions. If the molecule of the element is
triatomic, what will be its atomic mass?
67. Insulin contains 3.4% sulphur. Calculate minimum molecular
mass of the insulin. [Ans. 53.33]
[Ans. 941.176 amu] 76. Calculate the number of electrons, protons and neutrons in 1
mole of 18 0 2- ions.
[Hint: For minimum molecular mass, one molecule of insulin
must have atleast orie sulphur atom:] [Ans. Electrons = 10 x 6.023 X 1023
68. Calculate the number of carbon, hydrogen and oxygen atoms Protons = 8 x 6.023 X 1023
in 18 g of glucose. .
Neutrons = 8 x 6.023 x 1023 ]
[Ans. 3.61 x 1023 carbon atoms, 7:22 x 1023 hydrogen atoms,
3.61 x 1023 oxygen atoms] 77. 600 mL of a mixture of 02 and D3 weighs 1 gm at NTP.
69. Hydrated sulphate of a divalent metal of atomic weight 65.4 Calculate the volume of ozone in the mixture.
loses 43.85% of its weight on dehydration. Find the number of [Ans. 200 mL]
molecules of water of crystallisation in the formula of
hydrated salt. [JEE (West Bengal) 2005]
I .'

48 G.R.B. PHYSICAL CHEMISTRY FOR COMPETITIONS

..;p:OBJECTIVE QUESTIONS
S~t·1: Questions with single correct answer
15. One sample of air is found to have 0.03% carbon dioxide and
1. The father of modetn chemistry is: another sample 0.02%. This illustrates that:
(a) Priestley (b) Lavoisier (a) aids a compound
(c) Dalton (d) Mendeleev (b) air is an element
2. A pure substance can only be: (cr'air does not follow the law of constant proportions
(a) a compound (d) air is a mixture
(b) an element 16. Which one of the following is not a mixture?
(c) an element or a compound (a) Distilled water
(d) a heterogeneous mixture (b) Sugar dissolved in water
3. A pure substance which contains only one type of atoms is (c) Liquefied Petroleum Gas (LPG)
called: . (d) Gasoline
(a) an element (b) a compound 17. Which of the following is a charactelistic property of both
( c) a solid (d) a liquid mixtures and compounds?
4. Which one of the phrases would be incorrect to use? . (a) Their properties are same as those of their components
(a) A mole of an element (b) A mole ofacompound (b) Energy is released when they are formed
(c) Their masses me equal to the sum of the masses of theil'
(c) An atom of an element (d) An atom of a compound
components .
5. A symbol not only represents the name of the element but also
(d) They contain the components in fixed proportions
its:
. 18. Name the scientist who stated that matter can be converted
(a) atomic mass (b) atomic number
into energy:
(c) atomicity (d) atomic volume
(a) Boyle (b) Lavoisier (c) Avogadro (d) Einstein
6. The credit for the discovery of transuranic elements goes to:
19. Which one of the following is not an intensive property?
(a) Hahn (b) Rutherford
(a) Weight (b) Pensity
(c) Seaborg (d) Curie
(c) Refractive index (d) Melting point
7. The most abundant metal in'earth's crust is:
20. The metalloid among the following group of elements is:
(a) iron (b) magnesium
(CPMT 1993)
(c) calcium (d) aluminium
(a) P (b) As (c) Al (d) Po
8. The most abundant element in earth's crust is:
21. Which of the following ailoys does contain Cu andZn?
(a) hydrogen (b) oxygen (e) nitrogen (9) silicon
(lIT 1993)
9. Which one of the elements is not found in nature? (a) Bronze (b) Bras~
(a) Radium (b) Technetium . (c) Type metal (d) Rolled gold
(e) Polonium (d) Helium 22. Which metal is present in german silver?
Ill. Which one of the following is not a compound? (a) Copper (b) Iron (c) Silver (d) . Zinc
(a) Marble' (b) Ozone 23. Which of the following processes results in the formation of a
(c) Carborundum (d) Quicklime neW compound? .
11. Which one of the following is not an element? (a) Dissolving common salt in water
(a) Diamond (b) Ozone (c) Silica (d) Graphite (b) Heating water
12. The direct change from solid to gaseous state is referred to as: (c) Heating platinum rod
(a) dissociation (b) decomposition (d) Heating iron rod
(c) sublimation (d) deliquescence 24. Which one of the following is not a chemical change?
13. Sulphur burns in oxygen to form sulphur dioxide. The (a) Sublimation (b) Combustion
properties of sulphur dioxide are: (c) Electrolysis (d) Rusting
(a) totally different from sulphur and oxygen 25. The law of mUltiple proportions is illustrated by the pair of
(b) similar to'sulphur compounds:
, (c) similar to oxygen (a) sodium chloride and sodium bromide
(d) more similar to sulphur than oxygen (b) water and heavy water
14. Which one of the following statements is incorrect? (c) sulphur dioxide and sulphur trioxide
(a) All elements are homogeneous (d) magnesium hydroxide and magnesium oxide
,(b) Compounds always contain two or more different elements 26. In compound A, 1.0 g nitrogen combines with 0.57 g oxygen,
(c) A mixture is not always heterogeneous In compound B, 2.0 g nitrogen unite with 2.24 g oxygen and in
(d) Air is a heterogeneous mixture compound C, 3.0 g nitrogen combine with 5.11 g oxygen.
These results obey the law of:
I
BASIC PRINCIPLES 49

(a) multiple proportions (b) constant proportions 36. Hydn;,gen combines with .chlorine to iornl HCl. It also
(c) reciprucai proportions (d) none of these combines with sodium to form NaH. If sodium and chlorine
27., Which one is the best example oflaw of conservation of mass? also combine with each other, they will do so in the ratio of
(a) 6 g of carbon is heated in vacuum, there is no change in ·their masses as:
mass (a) 23: 35.5 (b) 35.5 : 23
(b) 6 g of carbon combines with 16 g of oxygen to form 22 g (c) 1 : 1 (d) 23: 1
of CO2 37. Zinc sulphate contains 22.65% Zn and 43.9% H20. If the law
(c) 6 g water is completely converted into steam Of constant proportions is true, then the maSs of zinc required
(d) A sample. of air is heated at constant pressure when its to give 40g crystal!) will be: . .
volume increases'but there is no change in mass (a) 90.6 g (b) 9.06 g (c) 0.906 g (d) 906 g
28. A chemical equation: is balanced according to the law of: 38. 3 g of a hydrocarbon on combustion in excess of oxygen .
(a) multiple proportions (b) constant proportions produces 8.8 g ofCO2 and 5.4 g ofH,20. The data illustrates
(c) reciprocal proportions (d). conservation of mass the law of:
29. S02 gas was prepared by (i) burning sulphur in oxygen, (a) conservation of mass (b)· mUltiple proportions
(ii)· reacting sodium sulphite with dilute H2S04 and (iii) (c) constant proportions. (d) reCiprocal proportions
heating copper with conc. H2 S04 , It was found that in each
case sulphur and oxygen combined in the ratio of 1 : 1. The [Hint: Mass of carbon in 8.8 g CO2 '= ~ x 8.8 2Ag;
44
data illustrates the law of:
(a) conservation of mass (b) multiple proportions Mass of hydrogen in 5.4 g Hp = 2 x 5.4 0.6 g.
(c) constant proportions (d) reciprgcal proportions 18
30. A sample of CaC03 has Ca = 40%, C d 12% and 0= 48%. If Total mass of(e + 1I) = 2.4+ 0.6= 3.0 g]
is
the law of constant proportions true, then the mass of Ca in 39. In the reaction, N2 + 3H2 ~ 2NH3, the .ratio of volllmes of
nitrogen, hydrogenimd ammonia is 1 : 3 : 2. These. figures
5 g of Cac03 from another source will be:
(a) 2.0 g (b) 0.2 g ( c ) 0.02 g (d) 20.0 g illustrate the law of:
31. Potassium combines with two isotopes of chlprine (a) constant proportions (b) Gay-Lussac
. eSCI and 37CI) respectively to form two samples of KCI. (c) multiple proportions (d) reciprocal proportions
40. Two volumes of anllllonia, on dissociation gave Qne volume of
Their formation follows the law of:
nitrogen and three volumes of hydrogen. How much hydrogen
(a) constant proportions (b) multiple proportions
will be obtained from the dissociation of 10 litre ofNH3?
(c) reciprocal proportions (d)" none of these
(a) 30 litre (b) 10 litre (c) 15 litre (d) 2Q litre
32. Different proportions of oxygen in the various oxides of
41. If6litre ofH2 and 5.6 litre ofCl 2 are mixed and exploded in
nitrogen, prove the law of:
an eudiometer, the volume of HCI formed is:
(a) reciprocal proportions (b) multiple proportions
(a) 6.0 litre (b) 5.6 litre (c) 11.2litre (d) 11.6litre
(c) constant proportions (d) conserv~tion of mass
42. The law of constant proportions was enunciated by:
33. One part of an element A combines with two p<!l'ts of B
(a) Dalton (b) Berthelot (c) Avogadro (a) Proust
(another element). Six parts of eleI'llelit C combine wjth four
43. An important postulate ofDalton:s atomic theory is:
parts of element B. If A and C combine together, the ratio of
(a) an atom contains electrons, protons and neutrons
their masses will be governed by~
(a) law of definite proportions (b) atom can neither be created nor destroyed nor divisible
(c) all the atoms. of an elemeht are not -identical
(b) law of multiple proportions
(d) all the elements are available in nature in the form of
(c) law of reciprocal proportions
atoms
(d) law of conservation of mass
44. The atomic masses of the elements are usually fractional because: .,:'.
34. H2S contains 5.88% hydrogen, H20 ~ontains 11.11 % hydrogen
(a) elements consist of impurities
. while S02 contains 50% sulphur. These figures jllustrate the
(b) these are mixtures of allotropes"
law of:
(e) these are mixtures ofisobars
(a) conservation of mass (b) constant proportions
(d) these are mixtureS of isotopes
(c) multiple proportions (d) reciprocal proportions
35. Number.of atoms in"4.25 g ofNH3 is: (AFMC 2010)
of
·45. The chemical formula a particular compound represents:
3 (a) the size of its molecule
(a) 6.023xl023 . (b) 4x6.023xl02
(b) the sQ.ape of its molecule
(c) l. 7 X 1024 (d) 4.5 x 6.023 x 1023 (c) the total number of atoms in a molecule
. w (d) the number of different types of atomS in a molecule
[Hint: Number of molecules ofNH 3 =- x 6.023 X 1023
. M . 46. Which one of the following properties of an element is not
= 4.25 x 6.023 x 1023 variable?
17 (a) Valency (b) Atomic mass
Number of atom = 4 x 4.25 x6.023xto2, (c) Equivalent mass (d) All of these
17
= 6.023 x 102']
50 GR B. PHYSICAL CH~MISrIW FOR COMPI::TITIONS

41. 1 amu is equal to: 57. 74.5 g of a metallic chloride contains 35.5 g of chlorine. The
(a) 1.00758 g (b) 0.000549 g equivalent mass of the metal is:
(c) l66xlO-24 g (d) 6.02 X 10-23 g (a) 19.5 (b) 35.5 (c) 39.0 . (d) 78.0
48. Which one of the following relationships is correct? 58. The product of atomic mass and specific heat of any element is
a constant, approximately 6.4. This is known as:
(a) At. mass = 6.4 x Sp. heat.
(a) Dalton's law (b) Avogadro's law
(b) At. mass x Sp. heat = 6.4
(c) Gay-Lussac law (d) Dulong Petit's law
(e) At. mass x 6.4 = Sp. heat
(d) At. mass x Sp. heat x 6.4 = 1
59. The molecular mass of chloride, MCl, is 74.5. The equivalent
mass of the metal M will be:
49. A I g of an element gives A2 g of its oxide. The equivalent mass
(a) 39.0 (b) 74.5
of the element is:
(c) 110.0 (d) 35.5
(a) A2 - Al X 8
[Hint: Mol. mass = At. mass + 35.5 = Eq. mass x valency +35.5,
Al
Valency of M from the formula Mel is 1]
(c) Al X 8 60. 1 g of hydrogen is found to combine with 80 g of bromine. 1 g
A2 - Al
. ".
50. Al g of an element gives A2 g of its chloride; the equivalent
of calcium combines with 4 g of bromine. The equivalent mass
of calcium is:
mass of the element is: (a) 10 (b) 20 (c) 40 (d) 80
(a) _A---,I~ x 35.5 (b) A2 x 35.5 61. 2.8 g of iron displaces 3.2 g of copper from a solution of
A2 - Al A2 - Al copper sulphate. If the equivalent mass of iron is 28, the
equivalent mass of copper will be:
(c) A2 - Al x 35.5 (d) A2 - Al x 35.5
(a) 16 (b) 32 (c) 48 (d) 64
Al A2
62. The specific heat of a metal of atomic mass 32 is likely to be:
51. Which one of the relationship is wrong? (a) 0.25 (b) 0.24 (c) 0.20 (d) 0.15' .
(a) 2 Y.D. = Mol. mass
63. The equivalent mass of an element is 4. Its chloride has a
(b) At. mass = Eq. mass x Valency
vapour density 59.25. The valency of the element will be:
6.4
(c) At. mass = - - - (a) 4 (b) 3 (c) 2 (d) 1
Sp. heat 64. The equivalent mass of iron in the reaction,
H I ' Mol. mass
(d) va ency = - - - - 3Fe + 4H20 = Fe 30 4 + 4H2 would be:
Eq. mass (a) 21 (b) 56 (c) 42 (d) 10
52. In ml g of a metal A displaces rnzg of another metal B from its 65. The specific heat ofa bivalent metal is 0.16. The approximate
salt solution and if their equivalent masses are EI and E2 equivalent mass of the metal will be:
respectively, then the equivalent mass of A can be expressed (a) 40 (b) 20 (c) 80 (d) 10
as: 66. A sample of pure calcium weighing 1.35g was quantitatively
converted to 1.88 g of pure calcium oxide. Atomic mass of
(a) EI = rnz x E2
ml calcium would be:
(a) 20 (b) 40 (c) 16 (d) 35S
(c) EI = ml ;2 rnz (d) EI = ~: x E2 67. A metal oxide is reduced by heating it in a stream of hydrogen.
It is found that after complete reduction, 3.15 g of the oxide
53. When the specific heat of a metallic element is 0.214 cal g -I , have yielded 1.05 g of the metal. We may conclude that:
the atomic mass will be closest to which one ofthe following? (MLNR 1991)
(a) 1 (b) 12 (c) 30 (d) 66' (a) atomic mass of the metal is 4
S<l. Approximate atomic mass of an element is 26.89. If its (b) atomic mass of the metal is 8
equivalent mass is 8.9, the exact atomic mass ofthe element is: (c) equivalent mass of the metal is 4
(a) 26.89 (b) 8.9 (c) 17.8 (d) 26.7 (d) equivalent mass of the metal is 8
55. When an element forms an oxide in which oxygen is 20% of 68. Compounds with identical crystal structure and analogous
the oxide by mass, the equivalent mass ofthe element will be: chemical formula are called:
(a) 32 (b) AO (c) 60 (d) 128 (a) isomers (b) isotones
56. 0.32 g of a metal gave on treatment with an acid 112 mL of (c) allotropes (d) isomorphous
,/ hydrogen at NIP. Calculate equivalent mass of the metal: 69. Which pair of the following substances is said to be
,. I[AMtJ'(Eilgg.. ) lOlO] isomorphous?
(~) 58 (b 32 (c) 11.2 (d) 24 (a) White vitriol and blue vitriol
[Hint.: Mass of metal that displaces 11200 mL hydrogen at STP (b) Epsom salt and Glauber's salt
wHl b-o it~ equivalent mass. (c) Blue vitriol and Glauber's salt
:. Equi"~Jent mass of metal = 0.32 x 11200 = 32] (d) White vitriol and epsom salt
112
BASIC PRINCIPLES 51

c mass of chlorine is 35.5. It has two isotopes of atomic (c) 18.1 >.< 1023 molecules of CO2
~5 and 37. The percentage o~}~!#lyier~sotope is: (d) 3 g-atoms of CO2
, (b) 15 (c) 20'" (d) 25 85. Which among the following is the heaviest?
c mass of boron is 10.81. It has two isotopes with 80% IPMT (Kerala) 20061
)% abundance respectively. The atomic mass of the (a) One mole of oxygen
~ having 80% abundance is 11.01. The atomic mass of (b) One molecule of sulphur trioxide
ler isotope is: ( c) 100 amu of uranium
.81 (b) 11.01 (c) 10m (d) 21.82 (d) 10 moles of hydrogen
)f chlorine combines with a metal giving III g of its (e) 44 g of carbon dioxide
Ie. The chloride is isomorphous with MgCl2 . 6H2 0. The 86. The largest number of molecules is in:
; mass of the metal is: • (a) 28 gofCO (b) 46 gofC2H sOl:;l
(b) 30 (e) 40 (d) 69 (c) 36 g ofHzO (d) 54 g ofN2 0 s .
lpour density of a volatile chloride of a metal is 59.5 and 87. Which of the following has the smallest number of molecules?
Lllvalent mass ofthe metal is 24. The atomic mass of the (a) 22.4 x 103 mL of CO2 gas
1t will be:
(b) 22 g of CO2 gas
(b) 48 (c) 24 (d) 12
(c) 11.2 litre of CO2 gas
xide of an element possesses the molecular formula,
(d) 0.1 mole of CO 2 gas
. If the equivalent mass of the metal is 9, the atomic
)f the metal will be: 88. Thetmmber of grams of H2 S04 present in 0.25 mole of H2 S04
(b) 18 (c) 9 (d) 4.5 is:
lolecular mass of a compound having formula MO and (a) 0.245 (b) 2.45 (c) 24.5 (d) 49.0
tlent mass 20 is: 89. Number of molecules in I litre of oxygen at NTP is:
, (b) 40 (c) 28 (d) 20 3 3
(al 6.02 x ItY (b) 6.02x 102
~sity of air is 0.001293 g mL- 1• Its vapour density is: 32 22.4
·3 (b) 14.3 (c) 1.43 (d) 0.143
Divide with the density of hydrogen, i.e., 0.00009 g (c) 32 x 22.4 (d) 32
22.4
90. 4-.6 x I (iI~'atoms of an element weigh 13.8 g. The atomic mass
,f a substance when vaporised occupy a volume of 5.6
ofthe. element is:
. NTP. The molecular mass of the substance will be:
(a) 190 (b) 180 (c) 34.4 (d) 10.4
(b) 2M (c) 3M (d) 4M
apour densities of two gases'are in the ratio of I : 3.
molecular masses are in the ratio.oJ:
.3 (p) I : 2 (c) 2: 3 < (d) 3: 1
ganic compound on analysis was found to contain (c) 3 x 6.02 X 1023 (d) 4 x 6.02 X 1023
Vo of sulphur. The molecular mass of the 'eompound, if its 92. The total number of protons in 109 of calcium carbonate is:
llie contains. two sulphur atoms, is: (CPMT1992)
10 (b) 2000 (a) 3.01 i5x 1024 (b) l5057 x loZ4
1000 (d) 200000 "
tomic mass of an element is p. If valency is 3, the (cl. 2:0478 x. 1024 (4) 4.0956 x loZ4
r density of the volatile chloride will be: '" - 93. 19.7 kg of gold was recovered from a smuggler. The atoms of
,.75 (b) 6.675 (c) 667.5 - (d) 81 gold recovered are: (Au = 197) .
msity of a gas 'A' is three times that of a gas 'B'. If the . (a) 100 (b) 6.02 x 1023
lliar mass of A is M, the molecular mass of B is:
(c) 6.02 X loZ4 (d) 6.02 x loZ5
(b) M/3 - (c).fjM (d) M
94 .. The molecular mass of CO2 is 44 amu and Avogadro's number -
.fj is 6.02 x I oZ3. Therefore, the mass of one molecule of cn, is:
r density of a volatile substance is 4 in comparison to (a) '7.31 x 10-23 (b) 3.65 x 10-23 .-
le (Cl4 1~ Its molecular mass will be:
(b) 2 »(c) 64 (d) 128 (c) lOI X 10-23 (d) 2.01 x 10-23
e the wrong statement: 95. Equal volumes of different gases at any definite temperature
nole means 6.02 x 1023 particles and pressure have:
olar mass is mass of one molecule (a) equal weights (b) equal masses
olar mass is mass of one mole of a substance (c) equal densities (d) equal nlunb(=r of !1]ok.,;
olar mass is molecular mass expressed in grams 96. A gaseous mixture contains oxygen and nitrc)fycil in the ,.,,!iu vf
lIe of CO2 contains: (MLNR 1990; CBSE 1993) I : 4 by mass. Therefore, the ratio of their l:!!miYcr
Vl "'-" 1 n23 ..............-"" ,....c r-" molecules is:
G.R. B. PHYSICAL CHEMISTRY FOR COMPETITIONS,

m will react with ethyl alcohol to give: 110. A compound of aluminium and chlorine is composed of9.0 g
,of hydrogen (b) one mole of oxygen Al for every 35.5 g of chlorine. The empirical formula of the
'ofNaOH, (d) 1/2 mole of hydrogen compound is:
an atom of carbon is: (a) AICI (b) AICl 3 (c) AICl 2 (d) AICl 4
(b) 1.99 x 10-23 g 111. The haemoglobin from red corpuscles of most marnmels
contain approxImately 0.33% of iron by mass. The molecular
(d) 1.99 x 1023 g
mass of haemoglobin is 67200. The number of iron atoms in
of moles of Hz in 0.224 litre of hydrogen gas at each molecule of haemoglobin is:
'(MLNR 1994) (a) 4 (b) 3 (c) 2 (d) 1
(b) 0.1 (c) 0.01 (d) 0.001 112. The percentage ofP20 s in diarnmonium hydrogen phosphate,
:Jf 1 g helium at NTPin litres is: [(NH4 )2 HP04 ] is: (CPMT 1992)
(b) 0.56 (c) 2.8 (d) 0.28 (a) 23.48 (b) 46.96
24
s to contain 1.0 x 10 particles, the mass of ope (c) 53.78 (d) 71.00
;en is: 113. The percentage of nitrogen in urea (NH2CONH2), is: '
(b) 5.32 g (a) 38.4 (b) 46.6 (c) 59.1 (d) 61.3
(d) 16:0 g 114. The chloride of a metal has the formula MCI 3 . The formula of
)f 1 litre capacity each are separately filled with
°
[e, 02 and 3 , At the same temperature and '
ratio of the number of atoms of th.ese gases
its phosphate is:
(a) M 2P04 ' (b) MP04 (c) M 3P0 4 (d) M(P04 )2
115. 10 g of hydrofluoric acid gas occupies 5.6 litre of volume at
ferent flasks would be:
NTP. If, the empirical formula of the gas is HF, then its
1 (b)2:1:2:3 molecular formula will be: (At., mass ofF = 19)
3 (d) 3 : 2: 2 : 1
(a) HF (b) H3F3
mperatirre and pressure, two flasks of equal
filled with H2 and'SOz separately. Particles which (c) H2F2 (d) H4F4
10
lUmber, in the two flaskes are: [Hint: Molecular mass - x 22.4 =40]
(b) electrons 5.6
:s (d) neutrons 116. Calcium pyrophosphate is represented by the formul.a
a mixture of 6.02x 1023 oxygen atoms and Ca ZP20 7 . The molecular formula of ferric pyrophosphate is:
ydrogen molecules at NTP is: (a) Fe 2P2 0 7 (b) FePZ0 7
(b) 33.6 litre (c) Fe(P20 7 )3 (d) Fe4(PZ07)3
(d) 22.4 litre 117. The percentage of available chlorine in a sample of bleaching
of molecules' present in a drop of water, if its ' powder, CaOClz ·2H2O, is:
)5mL, are: (a) 30 (b) 50 '(c) 43.5 (d) 59.9
,21
(b) 1.66 x102 2
n 8. SI unit of energy is:
~3 (d) 1.66x 1024 (a) kg m 2 S-2 (b) kg m- I S2
was 'found tocolltain nitrogen and oxygen in the (c) kg m 2 S-I (d) kg m 2 8 2
n 28 g and oxygen 80 g, The formula ()f the
119. One ~cro gram is equal to:
(a) 10:3 g , (b) 103 g (c) 106 g (d) 10-6 g
(b) N 2 0 3
(d) N;04 " 120. Significant figures in 0.00051 are:
t formula of a compound cOQ.tain"ing sb% of W5 ~3 002 ~4
At. mass = 10) and 50% of the element.y. (At. 121. ' The number of significant figures in 6.02 xloZ 3 is:
;: {(IIlI" 1994J
(a) 23 (b) 3 (c) 4 (d) 26
(b) X 2Y
(d) X 2Y 3 " , 122. Express 0:006006 into scientific notation in three significant
digits:
til X (At. mass 75) and Y (A!. mass = 16),
ivea compound having 75.8% of X. The f~rmula
(a) 6.01x io-
3 (b) 6.006 x 10-3
lund i s : ( M L N R 1991) (~) 6.00 x 10":3 (d) 6.0 x 10-3
(by X 2 Y 123. The proper value of significant figures in 38:0+ 0.003;5
(d) X 2Y 3
+ 0.00003 is:
a certaincornpound was found to contain iodine
(a) 38 (b) 38.0035
in the ratio of 254': 80. The formula of the
: (At. mass ofl = 127,0 = 16) (c) 38.00353' (d) 38.0
(b) 12 0 124. Which of the following is the correct unit for measuring
nuclear radii?
(d) 120s
(a) Micron " (b) Millimetre
BASIC PRINCIPLES 53
~r to prepare 1 litre normal solution of KMn0 4, how 137. The density of a liquid is 1:2 glmL. There are 35 drops in 2
~ams of KMn04 are required if the solution is to be mL. The number of molecules in one drop (m.lar mass of
I acid medium for oxidation? (PET (MP) 2002) liquid = 70) is:
8g (b) 31.6 g (c) 62 g (d) 790 g (a) (1.21 NA
.
(b)(~J·
35
2 NA
'
!U1 oxide of a metal is converted to chloride completely . 35)
rields 5 g chloride. The equivalent weight of metal is: L2 .
(c)--z NA (d) 1.2NA
(KCET2002) (35)
.25 (b) 3.325 (c) 12 (d) 20 13lt A sample of PCI 3 contains iA mole;' the substance. How
:rofatoms 558.5 g Fe (At. wt. ofFe= 55.85 g morl) is: many atoms are there in the sample? IeEE (KeraJa) 20.04/
(AlEEE 20.0.2.) (a) 4 (b) 5/ 4'
ice that in 60 g carbon (b) 6.023 x lcY 3 (c) 8.431 X 1023 (d) ,572 x IcY
lfthat of8 g He (d) 558.5 x 6.023 x 1023 (e) 2A09 x lcY4
efix 1018 is: [MEE (Kerala) 2062J . .
139. The eqUivalent weigh
. N OH J;Yi"O ~
. NaH PO + H is:
2 4 2
°
I of ,nosphoric acid H 3P0 4 in the

~a (b) exa (c) kilo (d) nano reaction, a + 4 lBHU (Pre.~ 20051
:ga (b) 49 (c) 25 (d) 9&
mce in density is the basis of: LMEE (Kerala) 20.02) (all? Only one hydrogen ofH 3P04 is replaced, i.e,its basicity
ra filtration (b) molecular sieving ='1
. 1· MoleCular mass
wity separation (d) molecular attraction EqUlva ent mass = 98 = 98]
)mic absorption Ie Ids Basicity I
' 1 , j
:ctlVe of the source, pure sample of water ai:;'en. This is 140. 5.6 g of an organic compound on burning with excess of
-0 mass of oxygen and 11.11 % mass of)\''Kerala) 20.0.21 oxygen gave 17.6 g of CO2 and 7.2 g H20. The organic
led by the law of:
",.anJtrcompos1't'IOn compound is: [PET (Kcrala) 200.61
[lservation of mass .W)constant volume (a) C6H6 (b) C4Hs (c) C3HS (d) CH3 COOH
lItiple proportion (e) CH3CHO
141. The decomposition of a certain mass of CaC03 gave 11.2 dm3
y-Lussac~~l~electron weigh one kilogram? . of CO 2 gas at STP. The mass of KOH required to completely
lanv r [IIT(S.creening) 20.021 neutralise the gas i s : · (KCET 2006)
3 x 1023 (b) 1 x 1031 (a) 56 g (b) 28 g (c) 42 g (d) 20 g
9.108 3 1
[Hint: 11.2 dm of CO 2 at STP - mole CO 2
:3 X 1054 I 2
(d) X 108
9.108 9.1 08 x 6.023 KOH + CO 2 ~ KHC0 3
he numbers: 161 cm; 0.161 cm; 0.0161 cm. The
of significant figure for three numbers is:
~ mole CO2 will be neutrali)'ed by ~ mole KOH, i.e, 28 g KOR]
[AFMC CPune) 200.2l 142. How many moles of magnesium phosphate, Mg 3 (P04 )2' will
,5 (b) 3,3,3 (c) 3,3,4 (d) 3,4,4 contain 0.25 mole of oxygen atoms? (AIEE.E 2006:)
me of the following laws directly explains the law of (a) 0.02 (b) 3.125 x 10-2
Ition of mass? (AFMC 20(2)
(c) 1.25 X 10-2 (d) 2.5 x 10-2
d's rule (b) Dalton's law
[Hint:
gadro's law (d) Berzelius hypothesis .: 8 mole oxygen atoms are present in 1 mole Mg3 (P04 )2
las maximum number of atoms?
[Ill' (S.creening) 20031 :.0.25 mole oxygen atoms will be present in.!. x 0.25 mole
· 8 ·
. C(12) (b) 56 g Fe(56) Mjffi (PO 4}z' i. e., 3.125 x 10-2 mole Mg3 (PO 4hJ
. AI(27) (d) 108 g Ag(l08) 143. An element, X has the following isotopic composition,
lch of sulphur is present in an organic compound, if 200 X: 90%
:>mpound gave 1.158 g ofBaS04 on analysis?
199 X :8%
[PET (Kerala) 20.651
(b) 15% (c) 20% (d) 25% 202X :2%
the weighted average atomic mass of the naturally- occurring
_of H2 and 20 mL of O2 react to form water, what is element 'X' is closest to : ICDSE (Mt'd.) 2007)
e end oftl:!e reaction? (AFMC 2005) (a) 201 amu (b) 202 amu (c) 199 amu (d) 200amu
G.R:B. PHYSICAL CHEMISTRY FOR COMPETITIONS

age atomic mass of 149. Common salt obtained from sea-water contains 95% NaCI by
8 x 199 ] + [ ~ x 202 ]
mass. The approximate number of molecules present in 109
x 200 ] + [ salt is: (DPMT 2009)
100 100
) amu "" 200 amu] (a) 1021 (b)1022
statement for 14 g CO is : (VMMC2007) ~I~ OOI~
is 2.24 L at NTP [Hint: Mass ofNaCI in 10 g salt:;:: 10 x ~ ::= 9.5 g
1 . 100
pOis to mole of CO
2 Number of molecules ofNael 9.5 x 6.023 x 1023
pondto same mole of CO and N2 58.5
22 23
3
'1" ....nds 3.01 x 102 molecules of CO = 9.78 x 10 "" 10 ]
r of hydi . 150. 109 hydrogen and 64 g oxygen were filled in a steel vessel
) h' h hll;en atoms present III 25.6 g of sucrose
w IC a\ molar mass of 342.3 g, is: and exploded. Amount of water produced in this reaction will
be: [CBSE (PMT) 20091
(VITEEE 2008)
. (a) 3 mol (b) 4 mol
(b\9.91x 1023
(c) 1 mol (d) 2 mol
(d) 44 ,- ..,,23
. 10 64
nber ot mOil;;" vr ~'-""Y"'<;e [Hmt : I1H = - :;:: 5 110 2 - = 2
Mass 4_.
2 2 32
:;:: -----,c.'-
Molar mass 342.3 2H2 + 02 ~ 2HzO
0.075 Case I : If H2 is completely consumed then:
101es of hydrogen atom = 0.075 x 22 nH 0 5 mol
2
toms of hydrogen 0.075 x 22 x 6.023 x 1023 ~''le II: If0 2 is completely consumed then
= 9:?x 1023 ] . htl~ 2
~-:-x2 4 mol
Ipied by one molecule ~f• • (-density = 1 glcm 3) Since, O 2 gives It.~
[CBSE-PMT (Pre.) 2008] hence, it is limiting an!9unt of product on complete consumption
3 cm
3 (b) 5.5 x 10- 23 cm3 equal to 4.] . "'er of moles of water formed will be
3 cm
3
(d) 6.023 x 10- 23 cm 3 151. An organic compound made ot '-.
nitrogen. What will be its molecularYmmd N contains 20%
18 one nitrogen atom in it? ijh£ontains only
iSS of one molecule = . 23 g
. 6.023 x 10 (a) 70 (b) 140 -"Il) 2009]
= 2.98 X 10- 23 g (c) 100 (d) 65
23
M 2.98x 10- [Hint: % N '" Mass of nitrogen x 100
>fie molecule = - - '- = cm 3
Density Molecular mass
"" 3 x 10- 23 cm 3 ] 20 =.!.± x 100
~cn contains as many atoms as in: (KCET 2008)
m=70]
lydrogen (b) 5 g of hydrogen
152. Given that the abundances of isotopes 54 Fe, 56Fe and 57Fe are
Iydrogen (d) I g of hydrogen
5%,90% and 5% respectively, the atomic mass of Fe is:
s consists oruniform cylindrical particles of 1:50A , (IlT 2009)
and 5000A long. The specific volume of virus is
lfthe virus is considered to bea single particle, its (a) 55.85 u (b) 55.95 u
lass is: (c) 55.75 u (d) 56.05 u
)7 g mol-1 (b) 7.90 x 107 g mor I [Hint:AtomicrnassofFe= 5 x54+ 90 x56+ 5 x57
)7 g mol-I (d) 9.70 x 107 g mol-I 100 100 100
55.95 amu]
lme of single virus = n?h 153. The number of atoms in 0.1 mol triatomic gas is:
= 3.14 x (75 x 1O- 8 i x (5000 x 10- 8 ) (N A = 6.02x 1023 mol-I) ICBSE (PMT) 2010]
8.836 x 10- 17
cm 3 (a) 6.026x 1022 (b) 1.806x 1023
. I .' Volmne 8.836 X 10- 17 cm 3 (c) 3.6x 1023 (d) 1.8x 1022
mg e ViruS =
, Specific volmne 0.75 Ig [Hint: No. of atoms = O.lx 3 x6.02x 1023
16
1.178xlO- g :;:: 1. 806 X 1023 ]
ss of virus 1.178 x 10- 16 x 6.023 X 10 23

7.09 X 107 g mol-I J


BASIC PRINCIP~cS 55

questions given below may have more 4. Select the numbers with same significant figures:
than one correct answers (a) 6.02 x 1023 (b) 0.25
(c) 6.60 x 10- 34 (d) 1.50
'the following relationships are wrong?
3 S, Which are isomorphic to each other?
l"" 0.1 har (b) 1 litre = 1 r.m (a) CuS04·5HP· (b) ZnS04·7HzO
0.239 cal (d) 1 eV;-:1.11 x 10--4 J (c) FeS04·7H20 (d) FeS04·8H20
If the following numbers have same significant 6. 11.2 L of a gas at STP weighs 14 g. The gas could be:
(a) N2 (b) CO
) (b) 0.60 ~c) 6.0 (d) 60 (c) NO z (d) NzO
fthe following }).ive the same mass? 7, 8 g O2 has same number of molecules as that in:
oole of 02 gas (a) 14 g CO (b) 7 g CO
nole of SC?2 gas (c) 11 g CO2 (d) 22 g COz
3 x 1022 molecules of S02 gas
4 x 1023 molecules of0 2 gas

Assertion-Reason TYPE QUESTIONS


e following questions, two st?Nments are given as 6. (A) Empirical formula of glucose is HCHO.
A) and 'Reason' (R). AIlswer the questions by (R) Molecular formula of glucose will also be equal to HCHO.
er codes given bell''' . 7, (A) The volume of 1 mole of an ideal gas at I bar pressure at
Jth (A) aJ'lrl \t<.) are correct and CR) is the correct 25°C is 24.78 litre.
ana~4'1 of (A). ' (R) 1 bar == 0.987 atm.
Ith (A) and (R) are correct but (R) is not the correct 8. (A) Atomic weight ': Specific heat (cal/mol) x 6.4
anation of (A). (R) The formula is valid for metals only.
,) is correct and (R) is wrong. 9. (A) Number of moles of Hz in 0.224 L of H2 is 0.0 I mol.
") is wrong but (R) is correct. (R) 22.4litres ofH2 at STP contains 6.023 x 1023 mol.
lth (A) and (R) are wrong. (AIIMS 1996)
O2 and 1 g'03 have equal number of atoms. 10. (A) The equivalent weight of an element is variable.
;s of 1 mole atom is equal to its gram-atomic mass. (R) The valency of an elemeftt is variable. (!\JlMS 1995)
our density of sulphur vapour relative to oxygen is 2 11. (A) The number of significant figures in 507000 is three.
luse sulphur atom is twice as heavy as that of oxygen (R) In 507000, all the zeros are significant.
n. 12. (A) Law of conservation of mass is invalid for nuclear fission,
lOur density depends upon the molecular state of the fusion and disintegration.
stance in vapour state. (R) The law proposes that mass is neither created nor
vogram is equal to 1 amu. destroyed in a reaction. .
'gram is reciprocal of Avogadro's number. 13. (A) Mas,S spectrometer is used for determination of atomic
ole H2S04 contains same mass of oxygen and sulphur. mass of isotopes.
.ole H2S04 represents 98 g mass. (R) Isotopes are the atoms of same element having same
nole oxygen and N2 have same volume at same atomic number but differeni mass numbers.
perature and pressure.
,ole gas at NTP occupies 22.4 litre volume at STP.
G.R.B. PHYSIC/\" CHEMISTRY FOR COMPETITIONS

e' QUESTIONS :: :
: 1
t

\
3. (a) . 4. (d) 5. (a) 6. (e) 7. (d) 8. (b)
1. (e) 12. (e) 13. (a) \4. (d) 15. (d) 16. (a) \
9. (a) 20. (b) 21. (b) 22. l\l) 23. (d) 24. (a) 1
7. (b) 18. (d) 29. (c) 30. (a) J
31. (d) 32. (b)
5. (a) 36. (a) 37. (b) 38. (a) 39. (b) 40. (e)
3. (b) 44. (d) 45. (d) 46. (ti) 47. (e) 48. (b) \
t
1. (d) 52. (b) . 53. (c) 54. (d) 55. (a) 56. (b) i

9. (a) 60. (b) 61. (b) 62. (e) 63. (b) 64. (a)
7. (e) 68. (d) 69. (d) 70. (d) 71. (e) 72. (e)
5. (a) 76. (b) 77. (d) 78. (a) 79. (d) 80. (a)
3. (b) 84, (a) 85. <e) 86. (e) 87. (d) 88. (e)
1. (d) 92. (a) 93. (d) 94. (a) 95. (d) 96. (e)
9, (e) 100. (a) 101. (a) l'6't,(b) 103. (c) 104. (d)
7. (b) 108. (d) 109. (d) 110. (b) 111..(a) 112. (e)
5. (e) 116. (d) 117. (e) 118. (a) ,"1.Q, (d) l~o.(e)
3. (d) 124. (d) 125. (b) 126. (a) 127. (a) 12K.:(b)
."
L (d) 132. (b) 133. (e) 134. (a) 135. (e) ,"ll.· (d)
). (d) 140. (b) 141. (b) 142. (b) 143.• (d) ·'144. (a)
7. (b) 148. (a) 149. (e) 150. (b) 151 •. (a) 152. (b)

J. (b, «) 4. (a, c, d) 5. (b, e) 6. (a, b) 7. (b, c)

I-REASON TYPE QUESTIONS j


(e) 4. (d) 5. (b) 6. (e) 7. (b) 8. (b)
(e) 12. (b) 13.' (a)
BASIC PRINCIPLES 57

BRAIN STORMING PROBLEMS /1/1/1


lOBJECTIVE QUESTIONS] for ( lIT ASPIRANTS I
ng questions co~tain single correct option: [Hint: Mass = 18g
the following table: Number of molecules ofHzO in 18 g mass = 6.023 x IO z3
npound Mass of the compound Number of electrons in 18 g water = 6.023 x.l OZ3 x 10
II. mass) r (in grams) taken, = 6.023 x 1024 4
CO2 (44) 4.4 .: Each molecule of water contains 10 electrons.]
NO? (46) 2.3 6. What is the empirical formula of vanadium oxide if 2.74 g of
HzOz (34) metal oxide contains 1.53 g of metal?
S02 (64) 1.6 (a) VZ0 3 (b) VO (c) V20 S (d) ~07
two compounds have least mass of oxygen? tni.... % of V = 1.53 x 100= 55,83
:ular masses of compounds are given in brackets.) ?.74
(EAMCET 2004)
.. %of ° =44.17
nd IV (b) I and III (c) I and II (d) III and IV Element % Atomic ratio Simplest ratio
I. Mass of oxygen present = 4.4 x 32 = 3,2 g 55.83", 1.1 l.l
44 V 55.83
52 1.1
[I. Mass of oxygen present = 2.3 x 32 = 1.6 g 44.17 =.2.76 2.76
44.17 2.5
46
II. Mass of oxygen present = 6.8 x 32 = 6.4 g
° 16 1.1
34 V:0=2:5

V. Mass of oxygen present = ~ x 32 = 0.8 g


Thus, empirical fomlUla = VzO s]
64 7. Number of moles of electrons in 4.2 g ofN 3 - ion (nitride ion)
I IV have leaSt mass of oxygen.] is:
osphateofacertain metal M is M 3 (P04 h. The correct (a) 3 (b) 2 (c) 1.5 (d) 4.2
a of metal sulphate would be: 8. The ratio of volumes occupied by 1 mole 02 and I mole CO2
(S04 h (b) MS04 under identical conditions of temperature and pressure is:
(S04 h (d) M 2 S04 (a)l:1 (b)l:2 (c)J.:3 (d)2:1
rcentage of Se in peroxidase enzyme is 0.5% by mass 9. The maximum amount of BaS0 4 that can be obtained on
~ mass of Se 78.4 amu). Then, the minimum mixing 0.5 mole BaCl z with I mole H 2S0 4 is:
lIar mass of enzyme which contains not more than one (a) 0.5 mol (b) 0.1 mol (c) 0.15 mol (d) 0.2 mol
n is: [Hint: H 2S0 4 + BaCl z ---? BaS04 + 2HCI
i8x 104 amu (b) 1.568 x 107 amu 0,5 mole BaCt z will react with 0.5 mole H 2S0 4 to give 0.5 mole
i8x 103 amu (d) 1.568 X 106 amu BaS04 ] ,

.: 0.5 g Se is present in 100 g enzyme. 10. Ifl021 molecules are removed from 100 mg CO 2 , then number
'II be present In
. -100' x 78.4
. of moles of CO 2 left are:
~ Se WI g enzyme
0.5 (a) 6. lOx. 10-4 (b) 2.8x 10-3 .
= 15680amu (c) 2.28x 10-3 (d) 1.36 x 10-2
= ~.568x 10 amu]
4
[Hint: Number of molecules in 100 mg CO 2
mber of moles of a gas in I m 3 of volume at NTP is: Mass x 6.023 x 1023
(b) 0.446 (c) 1.464 (d) 44.6 Molar mass
1m 3 1000 L
= 0.1 x 6.023 x10 23
1000 44
nber ofnnles =--= 44.6]
22,4 = 1.368 X 1021
:d number of electrons present in 18 mL water (density Molecules remaining = 1.368 x 1021 1021 0.368 X 1021
) is:
21
:3x 1023 (b) 6.023 x 1024 ..
N urnb er 0 f mo Ies remammg
0.368xl0
= ---~
3x 1.025 (d) 6.023 x 1021 6.023 x
'.
\
\

G.R.B: PHYSICAL CHEMISTRY FOR COMPETITIONS

on one gram ion of Al 3+ ion is: LHint: In the isotope I~C: Number of protons 6
1
: e coulomb (b) - x NA x e coulomb Number of neutrons 8
3
New atomic mass will be 2 x 6 + ~ x 8 16
ecoulomb (d) 3 x NA x e coulomb 2
. 16-14 .
3
: gram ion of A1 + means one mole ion of AI 3+. % Increase In mass = x 100 = 14.28% ]
14· .
mole A1 + 3
3 x e x ~ coulomb.]
17. The mass and charge of I mole electrons will be:
lass ofNzO as well as CO2 is 44 g mol-I. At 25°C (a) I kg; 96500 C (b) 0.55 mg; 96500 C
:essure, 1 L N 2 0 contains n molecules of gas. The (c) 1.55 mg; 96500 C (d) 5.5 mg; 96500 C
~02 molecules in 2 L under same conditions will
18. The simplest formula of the compound containing 50% X
(atomic mass 10 amu) and 50% Y (atomic mass 20 amu) is: .
(b) 2n (a)XYz (b)X 2Y (C)X 2Y3 (d)XY-
[Hint: Element <>4 ""mmle ratio Simplest ratio
. is dissolved in 1 L water. The number of ions of
in 1 mL of this solution will be: 50 5
X 50 =5 -=2
19 22 20 10 2.5
1 (b) l2 x 10 (c) l2 X 1020 (4) o.v2 X 10
nber of mol.,,, of !'laCI 50 = 2.5 2.5
Y 50
Mass = 5.85 = 0.1 20 2.5
Molar mass 58.5
Formula = X 2Y]
lUS (Na + + cr )in 1 L 19. Rest mass of 1 mole neutrons (mn ::: 1.675 X 10-27 kg) is:
(a) 1.8 X 10-3 kg (b) l.008 x 10-4 kg
= 2 x 0.1 x 6.023 x 1023
(c) 1.08 X 10-3 kg (d) 1.008 x 10-3 kg
= 12.046 X 1022
[Hint: .Mass of 1 mole neutrons
12.046 X 1022 1.675 x 10-27 X 6.023 X 1023
)TIS in 1 mL L2x 1020]
1000 = 1.008 X 3
10- kg]
de has the formula X 20 3 '} t can be reduced by 20. Loschmidt number is the number of:
give free metal and water. 0.1596 g of metal oxide (a) molecules present in I mL of a gas at STP
g of hydrogen for complete reduction. The atomic (b) molecules present in 1 gram mole of a gas at STP
al in amu is:
(c) atoms present in 1 mL of a gas at STP
(b) 155.8 (c) 5.58 (d) 55.8
(d) atoms present in I gram mole of a gas at STP
P3 + 3H z ----7 2X + 3HzO 21. Which of the following statements is incorrect?
ml 31001
48) g 6g . 108 -23
(a) One gram mole of sIlver equals - - x 10 g
i2 is required by 0.1596 g oxide 6.023
ill be required by 159.6 g oxide (b) One mole of CH4 and 17 g of NH3 at NTP occupy same
volume
2a + 48 = 159-./5
(c) One mole Ag weighs more than t~at o~~o moles o~Ca
a = 55.8
(d) One gram mole of CO 2 is 6.023 x loZ times heaVier than
tomic mass of metal M. ] one molecule of CO2 .
(PH3 ) decomposes to produce vapours of One atom of an element 'X' weighs 6.664 x 10-23 gm. The
and H2 gas. What will be the change in volume number of gram atoms in 40 kg of it is:
LL of phosphine is decomposed? (a) 10 (b) 100 (c) 10000 (d) 1000
(b) 500 mL The density of a liquid is 1.2 g/mL. There are 35 drops in 2
(d) 500 mL mL. The number of molecules in I drop is (molecular weight
PH 3(g) ----7 Pig) + 6H2(g) ofliquid 70):
4 mL
10 mL
----7

----7
1 mL
100
4
6 mL
~ x 100
4
(a) ~.~ NA (b) C5Y
l
NA

10 mL ----7 25 mL ISO mL (c)~N (d)1.2NA


(35i A
~ses by 75 mL.]
neutron is assumed to half of its original value, 2
[H Volume of one drop = - mL
t of proton is assumed to be twice of its original 35
ile atomic mass of ': C will be: 2x L2 1.2
Number of moles in one drop: "1"" '"7" = ,.",,2
BASIC PRINCIPLES 59

[Hint: Na2S04·nH20"
nolecules in one drop = L22 x NA ]
(35) Molar mass (l42+lSn)
ne of a liquid will contain 4 mole? Molar mass of Mass of water == 12.6 x (142 + ISn)
oand its density is 1.4 glmL: ' 26.S
(b) 1.6 L (c) 0.8 L (d) 4.8 x l(J23 L 126'
18n'" - ' x (142+ 18n)
t x L liquid contain 4 mole of it. 26.8
n 7]
Mass
.ber of moles 29. Consider the following data:
Molar mass
Element Atomic weight
4 = x x 1000 x 1.4 A 12.01
. 2S0
B 35.5
x = 4 x 2S0 '" O.S L]
1.4 x 1000 A and B combine to form a new substance X. If 4 moles of B
combine with 1 mole of A to give I mole of X, then the weight
ratio of Fe 2+ to Fe 3+ in a mixture of FeS04 and
of I mole of X is:
having equal number of sulphate ions in both
(a) 154 g (b) 74 g (c) 47.S g (d) 160 g
I ferric sulphates is:
30. How many moles ofNa + ions are in 20 mi of 0.4 MNa3P04?
(b) 3: 2
(a) 0.008 . (b) 0.024 (c) 0.05 (d) 0.20
(d) none of these
, MV 0.4 x 20
S04 ~ Fe2+ + SO~- [Hint: No. of moles ofNa 3P0 4 =
1000 1000
04)3 ~ 2Fe3+ + 3S0~- = 0.008
Number of moles ofNa+ '" 3 x Number of moles ofNa 3P04
mole SO~- ions are furnished by both FeS0 4 and
3 x 0.008 = 0.024]
31. The element whose one atom has mass ofl 0.86 x 10-26 kg is:
moles of Fez+ "'x
(a) boron (b) calcium (c) silver (d) zinc
moles ofFe3+ =2 x 32. An electric discharge is passed through a mixture containing
3 50 cc of 02 and 50 cc of Hz. The volume of the gases formed
2 (i) at room temperature, (ii) at 110°C will be:
: Fe3+ ::x:-x
3 (a) (i) 25 cc (ii) 50 cc (b) (i) 50 cc (ii) 75 cc
= 3: 2]
(c) (i) 25 cc (ii) 75 c c ( d ) (i) 75 cc (ii) 75 cc
[Hint: 2H 2 (g) + 02(g) ~ 2Hp
f electrons present in 3.6 rng of N~ are:
(b) 1.2 x Hro (c) 1.2 x 1022 (d)2x 10- 50 cc Hz will combine with 25 cc 02 to form 50 cc Hp
3
Q21
:. 02 left '" 25 cc
umber of electrons in one ion ofNH! 10
At room temperature, HP will be in liquid state but at llO°.C, it
'ions in 3.6 mg NH! will be gaseous. Thus, volume of gases at 25°C and 110°C will be
3 25 cc and 75 cc respectively.]
3.6x 10- x 6.023 X 1023 ", 1.2x 1020 33. The mass of carbon p(esent in 0.5 mole ofK 4[Fe(CN)6] is:
IS
(a) 1.8 g (b) 18 g (c) 3.6 g (d) 36 g
ber lh,lectrons in 3.6 mg NH~ = 1.2 x 1020 x J/J [Hint: 1 mole of~[Fe(CN~ ]contains 6 mole carbon, i.e., 72 g
=1.2xl02 11 carbon,]
:tion 4A + 2B + 3c -----7 A 4 B 2C 3 , what will be the 34. Caffeine has a molecular weight of 194. If it contains 28.9%
. moles of product formed, starting from one mole of by mass of nitrogen, number of atoms of nitrogen in one
Ie of Band 0,72 mole ofC? : molecule of caffeine is:
(b) 0.3 (c) (d) 2.32 0.14 004 ~6 ~2 W3
A + 2B +3C
001 0,6 mol 0,72 !lJl1
:.......-? A4 B2C 3 [Hint: Mass of nitrogen in 194 amu caffeine 2S.9 x 194
.100
~ent case, reactant 'C' wili be the limiting reactant = 56amu
will give least amount of product on being completely :. One molecule of caffeine will contain 4 atoms of nitrogen.]
35. Chlorine can be prepared by reacting HCI with Mn02' The
~ives I mol product, reaction is represented by the equation,
>,

'C' will give 0.24 mol of product.] Mn02(s) + 4HCI(aq.) ~ CI 2 (g)+ MnCI 2 (aq,) + 2H 20(l)
of Na 2 S04'nH2 0 contains 12.6 gm of water. The Assuming that the reaction goes to completion, what mass of
, is: conc. HCI solution (36% HCI by mass) is needed to produce 2.5 g
- - _....... _ -
60 G.R.B. PHYSICAL CHEMISTRY FOR COMPETITIONS

36. What is the mass per cent of oxygen in Al2(S04k18H20? x = Atomic mass of metal
The molar mass ofthis substance is 666.43 g/mol : .: 0.006 g Hz reduces 0.1596 g M 20 3
(a) 9.60 (b) 28.8 (c) 43.2 (d) 72
:. 6 g Hz will reduce 0.1596 x 6 gMz03 = 159.6Mz0 3
37. 0.25 g of an element 'M' reacts with excess fluorine to produce 0.006
0.547 g ofthe hexafluoride MF6 • What is the element? 2x + 48 = 159.6
(a) Cr (b) Mo (c) S e d ) Te 2x = 111.6
38. How many electrons are present in 2 x 10-3 moles of 1~02-? x=55.8].
(a) 1.2 x 102 1 (b)9.6x 1021 (c)1.2x 1022 (d) 1.9 x 1022 46. In a compound ofm~lecular formula AmBn:
39. Fluorine reacts with uranium to form UF6 . (a) number of equivalents of A, B 'and AmBn are same
U(s) + 3F2 (g} UF6 (g) (b) number ofmru..es of A, Band AmBn are same
How many fluorine molecul~s are required to produce 2 mg of (c) m x moles 01 A n x moles of B = (m + n) X moles of
UF6 from an excess of uranium? The molar mass ofUF6 is 352 AmBn
(d) n x moles of A = m x moles of B = (m + n) x moles of
g mol-I.
AmBn
(a) 3.4 x 101 8 (b) I X 1019 (c) 2 X 1019 , (d) 3.4 x 102 1 47. 4.4 g of CO 2 and 2.24 litre of H2 at STP are mixed in a
40. What is the formula of a substance with mass percentages of container. The total number of molecules present in the
°
35.79% for S, 62.92% for and 1.13% for H? container will be:
(a) H ZS03 (b) H 2 S04 (c) HiSz07 (d) H 2 S2 0 g (a) 6.02"2 x 1023 (b) 12044 x 1023
41. In 1811, Avogadro calculated the formula of camphor by (c) 6.023 X 1026 . (d) 6.023 x 10M
means of elemental chemical analysis and by measuring the
density of its vapour. Avogadro found the density to be 3.84 48. A partiillly dried clay mineral contains 8% water. The original
gIL when he made the. measurement at 210°C at 1 atrn sample contained 12% water and 45% silica. The % of silica in
pressure. Which of the following is the correct formula of the partially dried sample is nearly:
camphor? (a) ~O% (b) 49%
(a) CIO H140 (b) CIO HI6 0 (c) ClO H I6 0 2 (d) C IO H18 0 (c) 55% (d) 47%
(e) None ofthese [Hint: Initial stage: Clay Silica Water
[Hint: Pm = dRT 43% 45% 12%
Final stage: (92 - x) x 8%
m = dRT = 3.84 x 0.0821 x 483 = 152.27
P 1 Ratio of silica and clay will remain constant, before and after
drying.
:. C IOH IIJ will be the correct formula.]
45 x
42. A quantity of aluminium has a mass of 54 g. What is the mass
of same number of magnesium atoms? 43 92-x
(a) 12.1 g (b)24.3 g (c) 48.6 g (d) 97.2 g x == 47%]
43. When 1 L of CO2 is heated with graphite, the vQlume of the 49. Which ofthe following is isomorphous with MgS04 ·7H20?
gases collected is 1.5 L. Calculate the number of moles of CO (a) Green vitriol (b) Blue vitriol
produced at STP: . (c) Red vitriol (d) Vitriol of mass
1 28 1 (d) 14 50, In the reaction;
(a) 11.2 (b) 22.4 (c) 22.4 22.4 I '-7 2NO
CO + 2 O 2 -------7 COz; N2 + Ory
[Hint: cO2 (g)+ C(s) -------7 2CO(g)
I-x 2x 10 ~ of mixture containing ('~<oon monoxide and nitro~en
Total volume = 1 - x + 2X = 1 + x 1.5 requn-ed 7 mL oxygen to form CO~ and NO, on combustIOn.
x = 0.5 L The volume 9fN2 in the mixture will be:
.. Volume of CO = 2 x 0.5 = 1 L (a) 7/2 mL t'0) 1712 mL (c) 4 mL (d) 7 mL
1
Nwnber of moles of CO = - - ] [Hillt: CO(g) +.~ O,(g) ---t CO 2(g)
22.4 x ·2-
xl2
44. Which of the following has greatest number of atoms?
(a) 1 g of butane (C4 H IO ) (b) 1 gofhitrogen (Nz) y y
(c) 1 g of silver (Ag) (d) Ig of water (H2 0) x + y= 10 ... (i)
°
45. A metal oxide has the formuia M 2 3' It can be reduced by Hz
to give free metal and water. 0.1596 g of M 2 0 3 required 6 mg
X
-+ y=7 ... (ii)
2
OfI:I2 for complete reduction. The atomic mass of the metal is:
Solving eqs. (i) and (ii),
(a) 27.9 (b) 79.8 x=-6 . and y=4]
(c) 55.8. (d) 159.8 51. 1.44 g of titanium (Ti) reacted with excess of0 2 and produced
[Hint: M 20 3 + 3H z -------7 2M + 3HzO x. gm of a nonstoichiometric compound Ti 1.440 ! . The value 0 f
. ,
,; (2x + 48) g 6g x is:
I
BASIC PRINCIPLES 61

(b) 1.77 Following questions may bave more tban one correct options:
(d) None of these
1. 1l.2 L of a gas at STP weighs 14 g. The gas could be:
Ile reaction:
(a) NzO (b) N02 (c) N2 (d) CO
Ti +02 ~ Til.4401
2. In which of the following pairs do 1 g of each have an equal
101es of titanium = Number of moles ofTil.440] number of molecules? .
1.44 x
(a) N 20 and CO (b) N2 and C30 Z
48 48 x 1.44 + 16
(c) N2 and CO (d) N 20 imd CO2
x=1.77g] 3. 8 g of oxygen has the same number of molecules as in:
le of75% alcohol by mass (d = 0.8 g/cm 3 ) must be (a) 11 g CO2 (b) 22 g CO2 (c)7 g CO (d) 14 g CO
pare 150 cc of 30% alcohol by mass (d 0.9 4. Which of the following has three significant figures?
(a) 6.02 x 1023 (b) 0.25
(b) 56.25 mL
(c) 6.60 x 10-34 (d) 1.50
(d) 3356mL
V mL of alcohol was used .. 5. 1 mole of I~ N 3- ions contains:
~ x V x 0.8 ~ x 150 x 0.9 (a) 7 x 6.023 X 1023 electrons (b) 7 x 6.023 x 1023 protons
100 100 (c) 7 x 6.023 X 1023 neutrons (d) 14 x 6.023 x 1023 protons
V= 67.5 mL]
6. 1 g atom of nitrogen represents:
(a) 14 g nitrogen
(b) 11.2 litre ofN 2 at NTP
(e) 22.4 litre ofN2 at NTP
(d) 6.023 x 1023 molecules ofN 2

.'.
'fIeIZ4
'ect option
2. (b) 3. (a) 4. (d) 5. (b) 6. (e) 7. (a) 8. (a).
10. (a) 11. (d) 12. (b) 13 .. (c) 14.. (d) 15. (e) 16. (b)
18. (b) 19. (d) 20. (a) 21. (a) . 22. (d) 23. (c) 24. (c)
26. (a) 27. (c) 28. (d) ~9. (a) 30. (b) 31. (d) 32. (e)
34. (a) 35. (b) 36. (d) 31. (b) 38. (e)· 39. (b) 40. (e)
42. (c) 43. (c) 44. (a) 45. (c) ';;46. (a) 47. (1;» 48. (d)
50. (c). 51. (b) 52. (c)
'e than one correct options
2. (e, d) 3. (a,c) 4. (a, c, d) 5. (b,'c) 6. (a)
-.

G.R.B. PHYSICAL CHEMISTRY FOR COMPETITIONS

Integer Answer TYPE QUESTIONS


o each of the following questions is a 7. 5 g Hz is allowed to react with 14 g Nz for the following
teger, ranging from 0 to 9. If the correct reaction:
.estion numbers X, Y, Z and W (say) are N 2(g)+ 3H2(g)~ 2NH3(g)
respectively, then the correct darkening What mass ofH2 will be left unreacted at the end of reaction?
lliook like the given figure. [Hint :N2 is limiting reactant, thus 14 g N2 will give 17 g NH3'
and xg H2 remains unreacted.
Mass before reaction = Mass after reaction
(5+ 14) (17+x)
x= 2g]
'ous alkane (Cn H 2n + 2 ) is exploded with oxygen. The 8. Calculate the number of moles of water in 976 g BaCI 2 · 2H2o.
! of 02 used and CO2 formed are in the ratio of 7 : 4. 9. If Avogadro's number be 3.01x 1023 then the atomic mass of
e the value of n. carbon will be:
C"H 2n+ 2 + [n+ n; 'J 02 ~ nC02 + (n+ I) H20 10. How many moles of R will be produced when 8 mol of P and 5
mol of Q are allowed to react according to the equation:
(n+ I) 2P+Q~R
n+-- 7 11. The mass ofI x IOZz molecules of blue vitriol (CuS04 ·xH20)
__ 2_=_
n 4 is 4.144 g. The value of 'x' will be:
n = 2] 12. What will be the mass (in kg) of 7.298 x 106 mol electrons?
l1any atoms do a mercury vapour molecule consist of, if 13. Silver (Atomic weight 108 g mor-I) has a density of 10.5 ~
pour density of mercury vapour relative to air is 6.92? cm~3. The number of silver atolllS on a surface area ofI 0- 12 m
ic mass of mercury is 200). The average molar mass of can be expressed in scientific notation as y x I if. The value of
~9 g/mol. xis: (lIT 2010)
Vapour density ofHg vapour Molar mass ofHg [Hint: Mass of I cm 3 Ag = I x 10.5 g
Vapour density of air Molar mass of Air 10.5 23
Number of atoms = - x 6.023 x 10
6.92 m 108 .
29
Number of atoms in I cm == [~~ x 6.023 X 1023JII3
m == 200g/mol 108
mass is same as that of atomic mass hence mercury vapour
18 monoatomic mercury.) Number of atoms in 1 cm2=[to.5 x 6,023 x 1023 J2I3
lole of an element contains 4.2 x 'if"
electrons. What is
108
Number of atoms in IO-2m 2 or 10-8 cm 2
.mic number of the element?
romolecule of iron has molar mass 2800 amu, it contains = to.5 x 6.023 x 1023 J213 x 10-8 == 1.5 X 107 )
III by mass. The number of iron atom in one formula unit 108
macromolecule is: 14. A student performs a titration with different burettes and finds
: Number of iron atoms in one formula unit of compound titre values of25,2 mL, 25.25 mL and 25.0 mL. The number of
= % x Molecular mass .= ~ X 2800 = 4 ] significant figures in average titre value is: (lIT 2010)
100 Atomic mass 100 56 . A . I 25.2+ 25.25+ 25.0
[Hmt : verage titre va ue
~s of 'A' and 10 moles of 'D' are mixed and allowed to . 3
lccording to the equation : 75.45 == 75.4 == 25.1
A+3D ~2C 3 3
l1any mol~s of C are present when there are 4 moles of A (In addition, result is repOlted upto least place of decimal) )
container? ,
l1any water mol~cules will be there in 3 x 10-23 g sample
er? .

r4f#eIZ4
t

2. (1) 3. (7) 4. (4) 5. (4) 6. (1) 7. (2) 8.(8)


~ ___..~~ln (ti) 11 ('\) 1") fLl, 1'1 f7, 1.~ f'n
BASIC PRINCIPLES 63

"~. LINKED COMPREHENSION TYPE QUEsnONS.~-


1 Answer the following questions:
1. The isotopes of chlorine with mass number 35 and 37 exist in
y, 'mole' is an essential tool for the chemical the ratio of. .... Its average atomic mass is 35.5.
r is a basic Sf unit adopted by the 14th general (a)I:1 (b)2:1 (c)3:1 (d)3:2
Neights and measurements in 1971. A mole contains 2. Which of the following isotopes is/are used to decide the scale
ltary particles as the number of atoms present in 12g of atomic mass?
of a gas at STP occupies 22.4 litre volume. Molar (a) l~C (b) I!C (c) 1~0 (d) I~
'ds and liquids is not definite. Molar mass of a 3. Atomic mass of boron is 10.81. It has two isotopes namely
'0 called gram-atomic mass or gram molar mass. The I~Band ~B with their relative abundance of 801fo and 20%
go of mole is plenty, heap or the collection of large
respectively. The value of x is: .
(a) 10.05 (b) lO (c) 10.01 (d) 10.02
le of a substance contains 6.023 x 1023 elementary
4. The ratio of the mass of 12 C atom to that of an atomQf element
om or molecule. Atomic mass unit (amu) is the unit of X (whose atomicity is four) is I: 9. The moleculal mass of
g., atomic mass ofsingle carbon is 12amu. element X is:
lowing questions: (a) 480 g mol- I (b) 432 g mol-I
; of one amu is approximately: (c) 36 g mol- 1 (d) 84 g mol- 1
(b) 0.5 g 5. 1zCand 14C isotopes are found as 98% and 2% respectively in
(d) 3.2 x 10-24 g any sample. Then, the number of 14C atoms in 12 g of the
)f a gas at STP are found to have a mass of 22 g. The sample will be:
,r mass of the gas is: (a) 1.5 mole atoms (b) 1.032 x 1022 atom~
(b) 44. (c) 88 (d) 33 (c) 2.06x ]02 atoms
1
(d) 2 g atoms
; of one molecule of water is approximately:
(b) 0.5 g • Passage 3
lO-z4 g (d) 3.2 x 10-23 g Empirical fonnula is the simplest fonnula ofthe compound which
ly atoms are present in 49 g of H ZS04 ? gives the atomic ratio of various elements present in one molecule of
023 X 1023 (b) 5 x 6.023 X 1023 the compounl. However, the molecular formula of the compound
gives the number of atoms of various elements present in one
)23 X 1023 (d) 7 x 3.02 X 1023 ' molecule of the compound.
sat STP contains 3 x 1022 molecules. The number of Molecular fonnula = (Ernpiricaljormula) x n
; in x L ozone at STP will be: Molecular mass
22 (b)4 X 1023 (c) 6.02 X 1023 (d) 3 x 1024 n=----~--
Emptrical fonnuZa mass
Iro's number is 1 x 1023 mor- I then the mass of one A compound may have same empirical and molecular formulae.
xygen would be: Both these fonnulae are calculated by using percentage composition
of constituent elements.
nu (b) 16 x 6.02amu
Answer the following questions:
1 (d) 16 x 10-23 amu' 1. Two metallic oxides contain 27.6% and 30% oxygen
respectively. If the formula of first oxide is M 3 0 4 , that of
the Avogadro's number then number of valence
second will be:
.n4.8g of02- is:
(a)MO (b)M02 (c)M20s (d)M2 0 3
(b) 4.2 NA (c) 1.6 NA (d) 3.2 NA
2. Which of the following compounds have same empirical
formula? .
(a) Formaldehyde (b) Glucose
'he atoms of same element; they have same atomic (c) Sucrose (d) Acetic acid
r'ent'mass numbers. Isotopes have different number 3. Which of the following represents the formula ofa substance
feir nucleus. If an element exists in two isotopes which contains 50% oxygen?
asses 'a' and 'b' in the ratio m: n, then average (a) N 2 0 (b) CO 2
mXa+nxb (c) NO z (d) CH3 0H
be .
m+n 4. An oxide of iodine (I = 127) contains 25.4 g of iodine and 8 g
Jpes of same element have same position in the of oxygen. Its formula could be:
'he elements which have single isotope are called (a) IZ03 (b) 120
nents. Greater is the percentage c.0mposition of an (c) 1205 (d) 1207
G.R.s. PHYSICAL CHEMISTRY FOR COMPETITIONS

lrofluoric acid gas occupies 5.6 litres of volume at Molecular formula of metal chloride = MCt,
e empirical fonnula of the gas is HF, then its Molecular inass = [n x 9.02 + n x 35.5J:: 132
fonnula in the gaseous state will be: .. 11::3
(b) H2F2 (c) H3F3 (d) H4F4 :. Atomic mass of metal:: 3 x 9.02 = 27.06 J
, species having different percentage composition of 2. The vapour density of a mixture containing N02 and N2 0 4 is
38.3 at 27°C. The moles ofN0 2 in 100 moles of mixture are:
)OH and C 6H I2 0 6 (b) CH 3 COOH and CzHsOH (a) 33.48 (b) 53.52 (c) 38.3 (d) 76.6
CH 3 and HCOOH (d) CzHsOH and CH 30CH 3 3. At STP, 5.6 litre of a gas weighs 60 g. The vapour density of
°
.od ofNa, C and contains 0.0887 mo(Na, 0.132
2.65 x 1022 atoms of carbon. The empirical fonnula
gas is:
(a) 60 (b) 120 (c) 30 . (d) 240
lpound is: 4. Which of the following two substances have same vapour
(b) Na 3C 50 2 density? .
°
(d) Na O.0887 C 2.65 x 10 22 0.132 (a) Glucose (b) Fructose (c) Sucrose' (d) Starch
5. Let NH4HS(s) is heated in a closed vessel to decompose.
4
loids are extracted from the extracts of the plants
na. Marijuana owes its activity to tetrahydro The vapour density of the mixture will be:
rich contains 70% as many as carbon atoms as (a) equal to that ofNH4 HS
~ and 15 times as many hydrogen atoms as oxygen (b) lesser than thatofNH4 HS
m of tetrahydro cannabinol is 0.00318. (c) greater than that ofNH4 HS
lowing questions: ' (d) cannot be predicted
If mass of the compound is:
mu (b) 314 amu (c) 143 amu (d) 341 amu • Passage 6
:If fonnula of the compound is:
Precision refers to the closeness of a set of values obtained fol'
3002 (b) C21HI403 identical measurement of a quantity. Precision depends on the
460 (d) none of these limitations of measuring devices and the skills with which it is used.
of oxygen atoms in 1 mol of the' tetrahydro However, accuracy refers to the closeness ofa single measurement to
lol is: its true value.
(b)NA The digits ina properly recorded measurement are known as
3
~ = 6.023 X 102 significant figures. These are meaningful digits in a measured or
ge composition of carbon in the compound is: calculated quantity. The greater the number of significant figures in
;% (b) 70.85% (c) 80.25% (d) 59.64% a reported risult. smaller is the uncertainty and greater is the
precision. The zeros at the beginning are not counted. The zeros to
15 the right of a decimal point are counted. In the numbers that do not
contain a decimal point. "trailing" zeros may 01', may not be
~ity 'of a compound is defined as the ratio of mass of a . significant. The purpose ofzeros at the end of a number is to COfJvey
, of gas to the mass of the same volume ofhydrogen the correct range ofuncertainty. .,
tical conditions oftemperature and pressure. . Answer the following questbms:
Mass ofcertain volu~~o[lfas (22.4 L) at STP 1. If repeated measurements give values close to one another, the
Mass ofsame volume ofH 2 gas (22.4 L) at STP number is:
Mw (a) surely precise . (b) surely accurate
=-
2 (c) surely precise and accurate (d) all of these are correct
~lecular: mass of gas =Vapour density x 2 2. The number of significant figures in a measUred number
, is a unitless quantitj; it is unaffected by variation of contains how man;, uncertain {lUmber of digits?
!d pressure. . (a) Zero (b) 1
llowingquestions: . (c) 2 (d) Cannot be predicted
density· ofa metal chloride ;is 66. Its oxide contains 3. In the number,2.4560, there are .5 signifkruitdigits. Which one
tal. The atomic mass of the metal is: ' is the least significant !iigit?
(b) 54 ,; (c) 27.06 (d) 2.086 (a) 2 (b) 4 (c) 0 (d) 6
, Number of equivalents = ~umber of equivalents. 4. if we add 296.2 and 2.256, we get the answer as 298.456 g.
of metal of oxygen The number of significa~t figures in the result are:
53 47 006 ~5 W4 003
-:::::-
E 8 5. In which of the following numbers, all the zeros are not
significant?
E = 9.02
BASIC PRINCIPLES 65

[/I~
Passage 1. 1. (c) 2. (c) 3. (d) 4. (d) 5. (a) 6. (c) 7. (a)
Passage 2. 1. (c) 2. (a, c) 3. (b) 4. (b) 5. (b)
Passage 3. 1. (d) 2. (a, b) 3. (d) 4. (c) 5. (b) 6. (b, c) 7. (a)
Passage 4. 1. (b) 2. (a) . 3. (a) 4. (c)
Pl)ssage 5. 1. (c) 2. (c) 3. (b) 4. (a, b) 5. (b)
Passage 6. 1. (a) .2. (b) 3. (c) 4. (c) 5. (d)

¢ SELF ASS·ESSMENT r~
ASSIGNMENT NO.1

8. Which one ofthe following is not a unit oflength?


SECTION;'I
(a) Angstrom (b) Light-year
Straight Objective Type Questions (c) Micron (d) Radian
This section contains 10 multiple choice questions. Each 9. Unit of J pa -] is equivalent to: .
question has 4 cijoices (a), (b), (c) and (d), out of which only .(a)m3 (b)cm 3
one is correct.
(c)dm3 (d) no~e of these
1. Cartisone is a molecular substance containing 21 atoms of
10. The relative abundance of two isotopes of atomic masses 85
carbon per molecule. The mass percentage of carbon in
and 87 are 75% and 25% respectively. The average atomic
cartisone is 69.98%. What is the molecular mass of cartisone? .
mass of element is:
(a) 36004 (b) 176.5 (c) 287.6 (d) 312.8
(a) 86 (b) 40 (c) 85.5 (d) 75.5
2. Total number of atoms present in 25 mg of camphor, CIO H16 0
is: SECTION-II
(a) 2.57 x 1021 (b) 9.89x 1019
Multiple Answers Type Objective Questions
(c) 2.67 X 1021 (d) 6.02x lifO 11. Mass of one atom of oxygen is/are:
. ;;. 3. The oxide of a metal contains 60% of the metal. What will be (a) 16 amu (b) 32 amu
the percentage of bromine in the bromide of the metal, if the (c) 16 gm (d) 2.656 x 1O-23gm
valency of the metal is the same in both, the oxide and the
bromide? 12. Which of the following compounds have same percentage·
(a) 93% (b) 87% (c) 70% (d) 77% composition of carbon?
4. The radius of water molecule having density 19 mL- 1 is: (a) C6 H120 6 (b) CH3 COOH
(a) 1.925 A (b) 73046 A (c) 19.25 A (d) 7.346 A (c) HCOOCH3 (d) C12 H22 0 11
5. 3 g of an oxide of a metal is converted completely to 5 g 13. Which of the following is/are correct about I mole electrons?
chloride. Equivalent mass of metal is: (a)-6.023 x Hr
3
electrons (b) 5048 x 10- 7 kg
(a) 33.25 (b) 3.325 (c) 12 (d) 20 (c) 96500 coulomb charge (d) None of these
6. Quantitative analysis of a compound shows that it contains 14. In which of the following numbers, all zeros are sib'11ificant?
0.110 mole of 'C', 0.0:)5 mole of 'N' and 0.165 mole of '0'. (a) 5.0005 (b) 0.0030 (c) 3.0.000. (d) 0.5200
Its molecular mass is about 270. How many atoms of carbon 15. Which of the following are correct SI units?
are there in empirical and molecular formulae of the (a) Amount ofsubstance in mol L- 1
compound respectively?
(b) Pressure of gas in pascal
Empirical formula Molecular formula
(a) 1 3 (c) Density of a solid in kgm- 3

(b) \2 2 (d) Force in newton


(c) 2 .6
(d) 3 2
SECTION-III
7. Total number of electrons present in 11.2 L of NH} at STP is: Assertion-Reason Type Questions
(a) 6.02 x 1023 (b) 3.01 X 1023 This section ;cohtains 4 questions. Each question contains
Statement~1 (Assertion) and Statement-2 (Reason). Each
(c) 3.01 x 1024 (d) 5.1 x 1024 question has following 4 choices (a), (b), (c) and (d), out of
which only one is correct. .
66 G.R.B. , PHYSICAl CHEMISTRY FOR COMPETITIONS

(a) Statement-! is true; statement-2 is true; statement-2 is a 20. 'Match the Column-I with Colunm-I1: '
corre,t explanation for statement-I. Column-I' Column-II
(b) Statement-I is true; statement-2 is true; statement-2 is not (a) N2 (P) 40% carbon by mass
, a correct explanation for statement-I. (bY CO (q) Empirical formula CH20
(c) Statement-lis true; statement-2 is false. (c) C6H1Z Oq (r) Vapour density = 14
(d) Statement-l is false; staternent-2 is true. (d) CH3 COOH (s) 14NA (NA 6.023 X 1023 )
16. State"ment-I: Avogadro's number is a dimensionless quantity. electrons in a mole
Bet:ause 21. Match the Column-I with Colunm-II:
Statement-2: It is a number of atoms or molet:ules in one Column-I Column-II,
gram mole. (a)IL (P)10- 5 N
17. Statement-I: An element has variable equivalent mass.,
Because (b) I J ' (q)0.2389 cal
Statement-2: The valency of element is varia~le. (c) 9.9 x 106 ,erg (r) 10- 3 m3
18. Statement-I: Vapourdensity ofCH4 is halfo(:0i. ' (d) J Dyne ;. (s) 6.25 X l(j8 eV
" 0" " - \

Because ' 22. Match the Colunm-I with Column-II:


Statement-2: 1.6 g of CH4 contains same number of electrons Column-I 'Column-It
as 3.2 g of02 • " (a) I g mole ofOz(g) (P) mass, 32 g
19. Statement-I: Specific gravity is dimensionless quantity. (b) 0.5 mole ofSOz(g) (q) mass, 24 g
Because' ' (c) 1 gofHz(g) (r)volume, 11.2 LatSTP
Statement-2: Specific gravity is relative, density of a (d) 0.5 mole of 0 3 (g) (s) 1.5 x 6.023 x 1023 atoms
~ubstance, measured with respect to density of water at 4 DC.

SECTION-IV' "
Matrix-Matching Type Questions
This section contains 3 questions. Each question contains
statement given in two columns which have to be matched.
, Statements (a, b, c and d) in Column~I have to bematchedwith
statements (p, q; r and s) in Colunm-II. The answers to these
questions have to be appropriately bubbled as i1iustrated in the
following example&:, . '
If the correct matches are (a-p,s)(b-q,r); (c-p,q) and (d-s);
then the correctly bubbled 4 x 4 matrix should be as follows:

p q r s
a
G @ (0 e
b
®e0 0
c
G e (0 0
d
® @ (0
"
. I. (a) 2. (c) "
:J..(b) 4. (a) 5. (a) 6. (c) 7. (c) 8. (d)
.' 9. (a) 10. (c) n. (a, d) 12. (a"b, c) 13. (a, b, c) 14; (a, c) 15. (b, c, d) 16. (a)
17. (b) 18. (c) 19. (a) 20., (a-r, s) (b-r, s) (c-p, q) (d-p. q) 21. (a-r) (b-q, s) (c-q) (d-~)
22. (a-p) (b~p, r,s) (c-r){d-q, s)
ATOMIC·STRUCTURE

2.1 INTRODUCTION through gases at low pressure in a discharge tube. [A common


discharge tube consists of a hard glass cylindrical tube (about
The word atom is a Greek word meaning indivisible, i. e. , 'an 50 cm long) with two metal electrodes sealed on both the ends. It
ultimate particle which cannot be further subdivided. The idea is connected to a side tube through which it can be evacuated to
that all matter ultimately consists of extremely small particles any desired pressure with the help of a vacuum pump.] Air was
was conceived by ancient Indian and Greek philosophers. The almost completely removed from the discharge tube (pressure
old concept was put on firm footing by Jou Dalton in the form about 10--4 atmosphere). When a high voltage of the order of
of atomic theory which he developed in the years 1803-1808. 10,000 volts or more was impressed across the electrodes, some
This theory was a landmark in the history of chemistry. sort of invisible rays moved from the negative electrode to the
According to this theory, atom is the smallest indivisible part of positive electrode (Fig. 2.1). Since, the negative electrode is
matter which takes part in chemical reactions. Atom is neither referred to as cathode, these rays were called cathode rays.
created nor destroyed. Atoms of the same element are similar in
size, mass and characteristics; however, atoms of different Gas at low pressure
elements have different size, mass and characteristics .
. In 1833, Michael Faraday showed that there is a relationship
between matter· and electricity. This was the frrst major
breakthrough to suggest that atom was not a simple indivisible
particle of all matter but was made up of smaller particles. Vacuum pump Cathode rays
Discovery of electrons, protons and neutrons discarded the
indivisible nature of the atom proposed by John Dalton. Fig. 2.1 Production of cathode raya
The complexity of the atom was further revealed when the
following discoveries were made in subsequent years: Further investigations were made by W. Crookes, 1. Perrin, 1.1.
(i) Discovery of cathode rays. Thomson and others. Cathode rays' possess the following
(ii) Discovery of positive rays. properties:
(iii) Discovery of X-rays. ( . I They travel in straight lines a~ray from the cathode with
(iv) qiscovery of radioactivity. very high velocities ranging from 1{)9 lOll cm per second. A
I Discovery of isotopes and isobars. shadow of m~tal1ic object placed in the path is cast on the wall
(vi) Discovery of quarks and the new atomic model. opposite to the cathode.
During the past 100 years, scientists have made contributions . (L) They produce a green glow when strike the. glass wall
which helped in the development of modem theory of atomic beyond the anode. Light is emitted when they strike the zinc
. structure. The works of I"'. 1'IIonnoII and Ental Iludaerfonl sulphide screen.
actually laid the foundation of the modern picture of the atom. It I They produce heat energy when they collide with the
is now believed that the atom consists of several particles called matter. It shows that cathode rays possess kinetic energy which is
subatomic particles like electron, proton, neutron, positron, converted into heat energy when stopped by matter.
neutrino, meson, etc. Out of these particles, the electron, the (iv) They are deflected by the electric and magnetic fields.
proton and the neutron are called fundamental particles and are When· the rays are passed between two electrically charged
the building blocks of the atoms. plates; these are deflected towards the positively charged plate.
They discharge a positively charged gola leaf electroscope. It
2.2· CATHODE RAYS-DISCOVERY OF· shows that c1\thode rays carry negative charge.
ELECTRON (v) They possess kinetic energy. It is. shown by the
experiment that when a small p,in wheel is placed in their path,
The nature and existence of electron was established by
. the blades of the wheel are set in motion. Thus, the cathode rays
experiments on conduction of electricity through gases. In 1859,
consist of material particles which have mass and velocity•
...... I'IIIck« started the stUdy of conduction .of electricity
68 G,FtB, PHY$ICAL CHEMISTR ",' COMPETITIONS

These particles carrying negative charge were called negatrQns -,J.6622x 10-19 coulomb. Since,
by Thomson. ch~}~~: known, it was, thus, de . rge.
Thename 'negatron' was changed to 'electron' by Stoney. ~Illl of the electron: Th be
(vi) Cathode rays produce X-rays. When these rays fallon a calculated from the value of eI m
material having high atomic mass, new type of penetrating rays
of very small wavelength are emitted which, are calle4 X-rays. e
m -
(vii) These rays affect the photographlc plate. ' elm
(viii) These rays can penetrate through thin foils of solid
= 9.1096 x 10-
28 gor9.1096x
materials and cause ionisation in gases through which they pass.
(ix) The nature of the cathode rays is independent of: This is termed as the rest mass of the electron, i. e. , mass of the
(a) the nature of the cathode and electron when moving with low speed. The mass of a moving
'.. 'c:''J£ (brtheg~s·lh th~Odlscharge tube. electron may be calculated by applying the following formula:,
In 1897, J. J. Thomson determined the elm value
. I' rest mass of electron
M f
(charge/mass) of the electron by studying the deflections of "". 0 movmg e eclron = - ~1- (~)'
cathode rays in electric and magnetic fields. The value of e / m has
been found to be -1.7588 x 108 coulomb/g.
[The path of an electron in an electric field is parabolic, given
as: where v is the velocity of the electron and c is the velocity of light.
eE 2 When v becomes equal to c, mass of the moving electron becomes
y=--x infinity and when the 'velocity of the electron becomes greater
2
2mv than c, mass of the electron becomes imaginary.
where, y = deflection in the path of electron in y-direction
Mass of the el~trgn relative to that of hydrogen atom~
e = charge on electron
E intensity of applied electric field Mass of hydrogen atom = 1.008 amu
m = mass of electron
v velocity of electron
=1.008 x 1.66 x 10-24 g (since 1 amu =1.66 x 10-24 g)
x =distance between two parallel electric plates = 1.673 X 10-24 g
within which electron is moving. 24
The path of an electron in a magnetic field is circular with Mass of hydrogen atom 1.673 x 10-
radius 'r given as: Mass of the electron 9.1 096 x 10-28
mv
r= 1837
eB
, 1
where, m= mass of electron Thus, Mass of an electron =-- x mass of hydrogen atom
v = velocity of electron 1837
e = charge on electron 1.008
B.= intensity qf applied magnetic field 1837
The radius of the path is prQPortional to momentum.] = 0.000549 amu
By performing a series of experiments, Thomson proved that An electron can, thus, be defined as a subatomic particle
whatever be the gas taken in the discharge tube and whatever which carries charge -1.60 x 10-19 coulomb, i.e., one unit
be the material of the electrodes, the value of e / m,ts always
negative charge and has mass 9.1 x 10-28 g, i. e. , _l_th mass of
the same. Electrons are thus common universal constituents of , 1837
all atoms. the hydrogen atom (0.000549 amu).
J.J. Thomson gave following relation to calculate [Millikan's oil drop method is used to determine the charge on
c~arge/mass ratio: an electron by measuring the' terminal velocity of a charged
e E spherical oil drop which is made stationary between two
m= rB2 electrodes on which a very high potential is applied.
where the terms have USual significance given before 61c 11r
. = 1.7588x lOll Ckg- 1
Charge on an electron' q' = E
(VI + v2 )

Electrons are also produced by the action of ultraviolet light or , where, 11 = cQefficient of viscosity of the gas medium
X-rays on a metal and i)'0111 heated filaments. J)-particles emitted 'VI' v 2 termlnal velocities
by radioactive materials are also electrons. E = field strength
The first precise measurement of the charge on an electron V
was made by Robert ,A. Millikan in 1909 by oil drop r =radius of the oil drop = 11 911 J
V2(/ cr)g
experiment. The charge on, the electron was found to be
(f density of oil; cr = density of gas; g gravitational force)]
A roM1CSTRUC!LfFl~ 69

2~3 POSITIVE RAYS-DISCOVERY OF present on the electron. Hence, for a smaller value of e / m, it is
PROTON definite that positive particles possess high mass.
Ox) e / m value is dependent on the nature of the gas taken in
With the discovery of electrons, scientists started looking for
the discharge tube, i. e. ,positive particles are different in different
positively charged particles .which were naturally expected gases.
because matter is electrically neutral under ordinary conditions.
Accurate. measurements of the charge and the mass of the
The first experiment that led to the discovery of the positive
particles obtained in the discharge tube containing hydrogen, the
particle was conducted by Goldstein in 1886. He used a
lightest of all gases, were made by lJ. Thomson in 1906. These
perforated cathode in the modified cathode ray tube (Fig. 2.2). It 4
particles were found to have the e / m value as + 9.579 x 10
was observed that when a high potential difference was applied
between the t':lectrodes, not only cathode rays were produced but coulomb/g. This was the maximum value of elm observed for
also a new type of rays were produced simultaneously from any positive particle. It· was thus assumed that the positive
anode moving towards cathode and passed through the holes or particle given by hydrogen represents a fundamental particle of
canals of the cathode. These rays were termed canal rays since positive charge. This particle was named proton by Rutherford
these passed through the canals of the cathode.. These were also in 1911. Its charge was found to be equal in magnitude but
named anode rays as these originated from anode. When the opposite in sign to that of electron.
properties of these rays were studied by Thomson, he observed Thus, proton carries a charge + 1.602 x 10-19 coulomb,
that these rays consisted of positively charged particles and e.,
i. one unit positive charge.
named them as positive rays. The mass of the proton, thus, can be calculated.
19
e 1.602x 10-
Mass of the proton = =- ---- 4
elm 9.579x 10
Positive
rays = 1.672 X 10-24 g
+
or = 1.672 x 10-:27 kg

-Cathode . 1.672 X 10- 24


Mass 0 f th e proton In amu = = 1.0072 amu
1.66 x 10- 24
Fig. 2.2
A proton is defined as a subatomic particle which has a mass
The following characteristics of the positive rays were
recognised: nearly 1 amu and a charge of +1 unit (+ 1.602 x 10 -19 coulomb).
(i) The rays travel in straight lines and cast a shadow of the Protons are produced in a number of nuclear reactions. On the
object placed in their path. basis of such reactions, proton has been recognised as a
(it) Like cathode rays, these rays also rotate the wheel placed fundamental building unit of the atom.
in their path and also have heating effect. Thus, the rays possess
kinetic energy, i. e., mass particles are present. tll;l RUTHERFORD EXPERIMENT--
(iii) The rays produce flashes oflight on zinc sulphide screen . DISCOVERY OF NUCLEUS
. (iv) The rays are deflected by electric and magnetic fields in a After the discovery of electron and proton, the question arose
direction opposite to that of cathode rays. These rays are attracted how these charged particles are distributed in an atom. The
towards the negatively charged plate showing thereby that these answer was given by lJ. Thomson in the form Of first model of
rays carry positive charge. the atom.
(v) These rays can pass through thin metal foils. He proposed that the positive charge is spread over a sphere in
(vi) These rays can produce ionisation in gases. which the electrons are embedded to make the atom as a whole
(vii) These rays are capable of producing physical and neutr~1. This model was much like raisins in a pudding and is also
chemical changes. known as Thomson s plum pudding model. This model was
(viii) Positive particles in these rays have elm values much discarded as it was not consistent with the results of further
smaller than that of electron. This means either m is high or the investigations such as.scattering ofa-particles by thin metal foils.
value of charge i~ small in comparison to electron. Since, positive In 1911, Ernest Rutherford and his co-workers carried out a
particle is fOrnle~ by the loss of electron or electrons, the charge series of experiments using a-particles* (Fig. 2.3 and 2.4). A
on the positive particle must be an integral multiple of the charge beam of a-particles was directed against a thin foil of about
0.0004 cm thickness of gold, platinum, silver or copper
*The radiationS emitted by radioactive substances consist ofa-particles. These particles are positively charged. These particles are actually helium.
atoms from which electrons have been removed. Each a-particle consists of a mass equal to about 4 times that of hydrogen atom and carries a positive
charge of two units~ It is represented by the symbol a or ~He. .
He ~ He2 + + 2e
Helium atom a-particle Electron
a-particles are usually obtained from a natural isotope of polonium-214.
70 I G.R. B. , PHYSI9Al CHEMISTRY FOR COMPETITIONS

respectively. The foil was surrounded by a circular fluorescent


.: Electrostatic PE
zinc sulphide screen. Whenever an a-partiCle struck the screen, it 41tEo r
produced a flash of light. ' . .
1 4Ze 2
. The following observations were made: ro = - - - -
2
(i) Most of the a-particles (nearly 99%) went straight without 41tEo mv
2
suffering any deflection, 1 . . 2Ze
(ii) A few of them got deflected thi-oughsmall angles.
(iii) A very few (about one in 20,000) did not pass through the where, E K is the original kinetic energy of the a-particles.
foil ataB but suffered large deflections (more than 90°) or even
· came back in more or less the direction from which they have Here, .1 9 X 10 9 Nm2 C- 2 (MKS)

41tEo
· come, i. e. ,'a deflection of 1800 •
= 1(CGS)
Movable
screen The distance of closest approach is of the order oflO- 14 m. So,
the radius of the nucleus should be less than 10- 14 m.
Following conclusions were drawn from the above observations:

~r~L~~~\ ';:~;~E:
(i) Since, most of the a:"particles went straight through the
metal foil undeflected, it means that there must be very large
empty space within the atom or the atom is extraordinarily
hollow.

/l\j/":~~:
(ii) A few of the a-particles were deflected from their original
paths through moderate angles; it was concluded that whole of
RadioaCtive"
substance;'
(Polonium)
~</
I_._~
.
'\ " the positive charge is concentrated and the space occupied by this
positive charge is very small in the atom. When a-particles come
closer to this point, they suffer a force of repulsion and deviate
Lead plate from their paths.
Deflected The positively charged heavy mass which occupies only a
a-particles small volume in an atom is called nucleus. It is supposed to be
Fig. 2.3 present at the centre of the atom.
(iii) A very few of the
Largely deflected Slightly deflected
a7particles a-particles a-particles suffered strong ~
deflections or even returned on ~ t
their path 'indicating that the 9- "0
nucleus is rigid an.d a-particles /') I!?
recoil due to direct collision with 'O:Ill
the heavy positivelycbarged mass. .z ~
oal
The graph between angle of
scattering and the number of - Scattering angle
a-particles scattering in the
Fig. 2.4 (b)
corresponding direction is as
shown in Fig. 2.4 (b).
Information of Rutherford's scattering equation can be
memorised by the following relations:
Slightly deflected (a) Kinetic energy of a-particles:
a-particles N=KI/[(1I2)mv2]2
Fig. 2.4(8) (b) Scattering angle '8':
Consider an a-particle of mass' m' moving directly towards a N =K 2 /[sin 4 (8/2)]
nucleus with velocity 'v' at any given time. As this a-particle (c) Nuclear charge:
approaches the nucleus, its velocity and hence kinetic energy
N =K 3 (Ze)2
continues to decrease. At a certain distance ro from the nucleus,
the a-particle will stop and then start retracing its depicted path. Here, N = Number of a-particles striking the screen and
This distance is called the distance of closest approach. At this K 1 , K 2 and K 3 are the constants.
distance, the kinetic energy of the a-particle is transformed into
electrostatic potential energy. If Z be the .atomic number of the
;,~~fit MOSELEY EXPERIMENT-ATOMIC
· nucleus, then
./'-
1 2 1 x2e NUMBER
'-mv
2 Roentgen, in 1895, discovered that when high energy electrons in
a discharge tube collide with the anode, penetrating radiations are
produced which he named X-rays (Fig. 2.5).
ATOMIC. STRUCTURE .1 71

Cathode rays van den Broek (1913) pointed outthat the atomic number of
an element is equal to the to~l positive charge contained in the
nucleus of its atom. Rutherford was also having the same opinion
that the atomic number of an element repli.esents the number of
+ protons in the nucleus of its atom. Thus, .
Cathode· Anode Atomic number of the element
= Serial number of the element in periodic table
Detracted
X-rays ~,"--~"" = Charge on the nucleus of the atom of the element
, , '
= Number of protons present in the nucleus of the
'!I." ~\ ",
",,~\ / / L X-rays beam atom of the element
"" - = NU)11ber of extranuclear electrons present in tha
,
" ' atom of the element
2::6
Defracted unit
2.6 DISCOVERY OF NEUTRON
Fig. 2.5 The discovery of neutron was actually made about 20 years after
X-rays are electromagnetic radiations of very small the structure of atom was elucidated. by Rutherford. Atomic
wavelengths (0.1-20 A). X-rays are diffracted by diffraction masses of different atoms could not be explained if it was
gratings like ordinary light rays and X-ray spectra are, thus, accepted that atoms consisted only of protons and electrons .
. produced. Each such spectrum is a characteristic property of the Thus, Rutherford (1920) suggested that in an atom, there must be
element used as anode. present at least a third type of fundamental particles which should
Moseley (1912-1913), investigated the X-ray spectril of 38 be electrically neutral and possess mass nearly equal to that of
different elements, starting from aluminium and ending in proton. He proposed the name for such fundamental particle as
gold. He measured the frequency of principal lines of a particular neutron. In 1932, Cbadwlek bombarded beryllium with a
series (thea-lines in the Kseries) of the spe.ctra. It was observed stream Of a-particles. He observed that penetrating radiations
that the frequency of a particular spectral line gradually increased were produced which were not affected by electric and magnetic
with the increase of atomic mass of the element. But, it was soon fields. These radiations consisted of neutral particles, which were
realised that the frequency of the particular spectral line was called neutrons. The nuclear reaction can be shown as:
more precisely related with the serial number of the element in
the periodic table which he termed as atomic number (Z). He ~Be + iHe ~ I~C + ~n
presented the following relationship: B~ryllium a-particle .Carbon Neutron
~.

.JV = a(Z - b) The mass of the neutron was determined. It was


where, v = frequency of X-rays, Z = atomic number, a and bare 1.675 x 10-24 g, i. e:, nearly equal to the mass of proton.
constants. When the values of square root of the frequency were Thus, a neutron is a subatolllic particle which has a mass
plotted against atomic numbers of the elements producing 1.675 x 10-24 g, approximately 1 amu, or nearly equal to the
X-rays, a straight line was obtained (Fig. 2.6).
mass of proton or hydrogen atom and carrying no electrical
charge. The e / m value of a neutron is thus zero.

21;'~r RUTHERFORD MODEL


20
.On the basis of scattering experiments, Rutherford proposed a
model of the atom which is known as nuclear .atomic model.
According to this model:
. (0 An atom consists of a heavy positively charged nucleus
w here all the protons and neutrons are. present. Protons and
neutrons. are collectively referred to as nucleons. Almost whole
of the mass of the atom is contributed by these nucleons. The
magnitude of the positive charge on the nucleus is different for
different atoms.
(ii) The volume of the nucleus is very small and is only a
minute fraction of the total volume of the atom. Nucleus has a
20 30 40 diameter of the order of 10- 12 to 10- 13 cm and the atom has a
Atomic number (Z) diameter of the order of 10-8 cm.
8
Diameter of the atom = 10- = 105
Fig. 2.6 . D"iameter of thenl,lcleus 10- 13
72 G.R.B. PHYSICAL CHEMISTRY FOR COMPETITIONS

Thus, diameter (size) of an atom is 100,000 times the diameter (iii) In the solar syst~!!l, there is only one planet which
of the nucleus.* . revolves in any particular orbit, but in the nuclear atomic model
The radius of a nucleus is proportional to the cube root of the more than one electron may rotate in any particular orbit.
number of nucleons within it.
R RoAI13 em Drawbacks of Rutherford Model
(i) According to classical electromagnetic theory, when a
where, Ro = 1.33 X 10- cm; A = mass number; R ::::: Radius of
13
charged particle moves under the influence of attractive force, it
the nucleus. ' .
loses energy continuously in the form of electromagnetic
Rutherford and Marsden calculated the density of the
radiations. Thus, when the electron (a charged particle) moves in
hydrogen nucleus containing only one proton. an attractive field (created by protons present in the nucleus), it
d = Mass [A x 1.66 X 10-24 g] must emit radiations. As a result of this, the electron should lose
. VoJume energy at every tum and move closer and closer to the nucleus
.( following a spiral path (Fig. 2.7). The ultimate result will be that
it will fall into the nucleus, thereby making the atom unstable.
A x 1.66 X 10-24 Bohr made calculations and pointed out that an atom would
=--~~~------------~
4 .' 8
collapse in 10- seconds. Since, the Nucleus
x 3.14 x (1.33 x 10- 13 )3 X A
3 . atom is quite stable, it means the
=1.685 X 1014 gf cm 3 electrons do not fall into the nucleus,
thereby this model does not explain
(iii) There is an empty space around the nucleus called
the stability ofthe atom.
extranuclear part. In this part, electrons are present. The number
(ii) If the electrons lose energy
of electrons in an atom is always equal to number of protons
continuously, the observed spectrum
present in the nucleus. As the nucleus part of the atom is
should be continuous but the actual
responsible for the mass of the atom, ~he extranuclear part is
observed spectrum consists of well
responsible for its volume. The volume of an atom is about 1015
defined lines of definite frequencies.
times the volume of the nucleus. Electron
8 24 Hence, the loss of energy by the
Volume of the atom = (10- )3 = 10- = 1015 electrons is not continuous in an atom. Fig. 2.7
Volume of the nucleus (l0-13)3 10-39
(iv) Electrons revolve round the nucleus in closed orbits with 2.8 ELECTROMAGNETIC RADIATIONS
high speed. The centrifugal force acting on the revolving These are energy radiations which do not need any medium for
electrons is being counterbalanced by the force of attraction propagation, e.g., visible, ultraviolet, infrared, X-rays, y-rays,
between the electrons and the nucleus. etc. An electromagnetic radiation is generated by osci1lations of a
This model was similar to the solar system, the nucleus repre- charged body in a magnetic field or a magnet in an electrical
sentingthe sun and revolving electrons as planets. The electrons field. The frequency of a. wave is the frequency of oscillation of
are,.therefore, generally referred to as planetary electrons. the oscillating charged particle. These radiations or waves have
electrical and magnetic ·fields associated with them and travel at
Dissimilarities between Nuclear Atomic Model and
right angle to these fields. The following are thus the important
Solar System
characteristics of electromagnetic radiations:
(i) The sun and the planets are very big bodies and uncharged • All electromagnetic radiations travel with the velocity of
while the nucleus and electrons are very sinall objects and charged. light.
(ii) The revolution of the planets in the solar system is • These consist of electric and magnetic fields that oscillate
governed by gravitational forces, while the revolution of in directions perpendicular to each other and perpendicular
electrons around the nucleus is governed by electrostatic forces. to the direction in which the wave is trave1ling.
[
S.No. Name Wavelength (A) . Frequency (Hz) Source
14 1 5 9
l. Radio wave 3xlO -3x10 I x 10 .,.;Ix 10 Alternating current of high frequency
2. Micro wave ,3x101 -6xld' 1 x 109 - 5 X 1011 Klystron tube
3. Infrared (IR) 6x 10 6 -7600 5 x 10" - 3.95 X 1016 Incandescent objects
16 14
4. , Visible 7600-3800 3.95 x 10 -7.9 X 10 Electric bulbs, sun rays
5. I
Ultraviolet (UV) 3800-150 7.9 x 10 14
2 X 10 16
Sun rays, arc lamps with mercuryvapours
6. X-Rays 150-0.1 2x lOll> - 3 X 1019 Cathode rays striking metal plate
7. !y-RaYS O.H).OI 1
19
3 x 10 -3 X 10 20
Secondary effect of radioactive. decay
8. ~osmic rays· ! O.ot-Zero ~ ~ 102°-Infinity .-.
Outer space

"'The diameter of various atoms lies in the range of 0.74 to 4.70 A(1 A :; 10-8 cm),
ATOMIC STRUCTURE 73
"
A wave is always characterized by the following six (vi) Time period: Time taken by the wave for one complete
characteristics: cycle or vibration is called time period. It is denoted by T.
(i) Wavelength: The distance between two nearest crests or I
T==-
nearest troughs is called the wavelength. It is denoted by A
Unit: Second per cycle.
v
(lambda) and is measured in terms of centimetre (em), angstrom
(A), micrometre ().un) or nanometre (nm).
·2~9 EMISSION SPECTRA-HYDROGEN
SPECTRUM
Spectrum is the impression produced on a screen when radiations
of particular wavelengths are analysed through a prism or
• Energy
diffraction grating. It is broadly of two types:
(i) Emission spectra (ii) Absorption spectra.
Difference between Emission and Absorption Spectrum
lA =10-8 em =10-10 m Emission spectrum Absorption spectrum
1 f.,lm= 10-4 em = 1O-ti m 1. It gives bright lines (coloured) It gives dark lines on the brigllt
Inm 10-7 cm= 10-9 m on the dark background. background.
=
1 em 108 A ==10 4 f.,lm==107nm 2. Radiations from emitting It is observed when the white
light is passed through the
source are analysed by the
(ii) Frequency: It is defined as the number of waves which substance and the transmitted
spectroscope.
pass through a point in one second. It is denoted by the symbol radiations are analysed by the
v (nu) and is measured in terms of cycles (or waves) per second spectroscope.
(cps) or hertz (Hz). 3. It may be continuous (if source These are always disconti-
=
AV distance travelled in one second emits white light) and may be nuous.
== velocity == c discontinuous (if the source
c emits coloured light).
or V==-
A Emission spectra: It is obtained from the substances which
(iii) Velocity: It is defined as the distance covered in one second emit light on excitation, i. e. ,either by heating the substances on a
by the wave. It is denoted by the letter 'c'. All electromagnetic flame or by passing electric discharge through gases at low
waves travel with the same velocity, i. e. , 3 x 10 10 cm/sec. pressure or by passing electric current discharge through a thin
filament of a high melting point metal. Emission spectra are of
AV== 3x 1010
two types:
Thus, a wave of higher frequency has a· shorter wavelength (a) Continuous spectra and (b) Discontinuous spectra.
while a wave of lower frequency has a longer wavelength. (a) Continuous spectra: When white light is allowed to pass
(iv) Wave number: This is the reciprocal of wavelength, through a prism, it gets resolved into several colours (Fig. 2.8).
i. e. ,the number of wavelengths per centimetre. It is denoted by The spectrum is a rainbow of colours, i. e., violet merges into blue,
the symbol v (nu bar). blue into green, and so on. This is a continuous spectrum.
_ 1
V==- Ultraviolet
A
l I . Violet 4000 A
It is expressed in cm- or m- .
Indigo
(v) Amplitude: It is defined as the height of the crest or
Blue
tlepth of the trough of a wave. It is denoted by the letter 'a'. It
determines the intensity ofthe radiation. Green
The arrangement of various, types of electromagnetic radiations Yellow

~~~~~~~=======~" Orange
in the order of their increasing or decreasing wavelengths or
frequencies is known as electromagnetic spectrum. ' Red 6500 A
, Frequency
V 3x107 (cycle/sec) )3x10 21 Infrared
).. (cm) = 103 f-(_W_av_el_en_gt_h_ 10-11

'"
,.~
Continuous
"'~ '" !-<
Low energy
~5 ~ High energy Fig. 2.8 Continuous spectrum of white light
ll.l
..l
<,... 0 0
Low frequency i:\:", ~, ;;: High frequency (b) Discontinuous spectra: When gases or vapours of a
~
ll.l
Long wavelength 9~
'">- '">-
~ ~ ~ :i
...l
!!l Short wavelength chemical substance are heated in an electric arc or in a Bunsen
~~ Sl
~ ~ ;;:
'" ..l
;:::l
:><: 'I- a
flame, light is emitted. If ray of this light is passed through a
prism, a line spectrum is produced. This spectrum Consists of a
limited number of lines, each of which corresponds to a different
74 G.R.B. PHYSICAL CHEMISTRY FOR COMPETITIONS

wavelength of Ffgbt. The line spectrum of each element is unique. Red Blue-Green Indigo Violet
Hydrogen spectrum is an example of line emission spectrum or
atomic emission spectrum.
When an electric discharge is passed through hydrogen gas at
low pressure, a bluish light is emitted. When a ray of this light is 6563 4861.4340 4102
passed through a prism, a discontinuous line spectrum of several
. isolated sharp lines is obtained. The wavelengths of various lines Fig. 2.9 (a) Balmer series In the hydrogen spectrum

[~
show that these Jines lie in visible, ultraviolet and infrared
regions. All these lines observed in the hydrogen spectrum can be Lyman series: v 1 = RZ
A
2
12
- ..!..]
n:
claSsified into six series.
Speetral series . Discovered by Appearing in n2 =2, 3,4,5, ...
Lyman series Lyman' Ultraviolet region Obtained in emission as well as in absorption spectrum ratio
Balmer series . Balmer Visible region ofm lh to nth wavelength of Lyman series.

~: =(:::)' ·{::::i:=:}
Paschen. series Paschen Infraled region
Brackett'series Brackett Infrared region
Pfund series Pfund Infrared region

=IlZ 2 [..!.. - ~l
Humphrey series Humphrey Far infrared region
Ritz presented a mathematical formula to find the Balmer ser.ies: v =!
A 22 ni.
wavelengths of various hydrogen lines.
=3,4,5,6, .. ;
V=l=~=R[~-~)
A c nl n2'
. n2
Obtained only in emission spectrum.

Where, R is a universal constant, known as Rydberg constant, ItS


value is 109,678 cm-I, n l and n2 are integers (such that n2 > n l ).
Paschet series: ±
v = =RZ 2 [312 - n1i]
For a given spectral series, n 1 remains constant while n2 varies
n2 =4,5,6,7, ...
from line to line in the same series.
The value of nl =1, 2, 3, 4 and 5 for the Lyman, Balmer,
Paschen, Brackett and Pfund series respectively. n2 is greater
Brackett series: . ±
v == =RZ 2 [ 412 - ::]
than nl by at least 1. .
n2 == 5,6,7,8, ...
Values ofnl and n~ for various series
Spectral series Value ofnl Value ofnz Pfuud series: v= 1 = RZ 2 [~_..!..]
2
A S2n2 .
Lyman series 1 2,3,4,5, ...
Balmer series 2 3,4,5,6, ... n2 = 6; 7, 8, 9, ...
.Paschen series 3 4,5,6,7, ... Note: (i) Atoms give line spectra while molecules give band spectra .
Brackett series 4 5,6,7,8, ... (ii) Balmer, Paschen, Brackett, Pfund series are found in
Pfund series 5 6,7,8,9, ... . emission spectrum.

. Electronic transition Name of line Wave no. Wavelemrth and'colour


n2 = 3 ----? n, = 2 Ha (First line) . v = ..!. =R [ l... _l...2] =5R A = 6563 A (Red)
(M). (L)
. A i 3 . 36
"2 = 4 ----? nl = 2 H,J (Second line) 1] 3R A = 4861 A(Blue-Green)
(N) (L) V=±=R[;2 42 =16
"2 = 5 ----? n1 = 2 H., (Third line) 2lR
:-
A= 4340 A (Indigo).
(0) (L) V=±=R[;2 _ 1
100
"2 = 6 ----? nl= 2 Ha (Fourth fine) 1] 8R A =4102A (Violet)
(P) (L) V=±=R[;2 62 = 36
(Above four lines were viewed in Balmer series by naked eye.)

Absorption Spectrum: Suppose the radiations from a conti- number of wavelengths in the emergent light than in incident
nuous source like a hot body (sun light) containing the quanta of light is called absorption spectrum.
all wavelengths passes through a sample of hydrogen gas, then Let the radiations of wavelengths AI' A2' A3, A4 ,AS are
the wavelengths missing in the emergent light give dark lines.on passed through the sample of hydrogen gas such that AI and A4
the bright background. This type of spectrum that contains lesser are absorbed then the absorption spectrum may be represented as:
ATOMIC STRUCtURE 75

,Example 2. Calculate the frequency and wave number of


Al
• r-----'---~ Az radiation with wavelength 480 nm.
Az • Sample Solution: Given,
A3 of
• Hydrogen . A ::: 480 nrn.= 480 x 10-9 m
A4 • gas
1 - - - - - - - 1 AS c::: 3 X 108 ml sec '
AS •
c 3 X 108 ms-:I
Frequency, v -:::', ' = 6.25 x 1014 S-I
, A 480x 10-9 m
Fig. 2.9 (b) Absorption Spectrum = 6.25 x 1014 Hz
Example 3. Calculate the energy associated with photon
'2.10' QUANTUM THEORY OF RADIATION
of light having a wavelength 6000 A. [h = 6.624 x I 0-27 erg~s~c.]
The wave theory successfully explains many properties of , 'c
electromagnetic radiations such as reflection, refraction, diff- Solution: We know that, E = hv =h . -
A
raction, interference, polarisation, etc., but fails to explain some
h::: 6.624 X 10-27 erg-sec; c 3 x 1010 cml sec;
phenomena like black body radiation, photoelectric effect, etc. ",
In order to explain black body radiation and photoelectric A 6000A::: 6000 x 10-8 cm
effect, Max Planck in 190 I presented a new theory which is
=(6.624XIO-27)X(3XIOIO)=3312 10-12
known as quantum tbeory of radiation. According to this S0" E 5 • X erg.
theory, a hot body emits radiant energy not continuously but 6x 10-
discontinuously in the form of small packets of energy called Example 4. Which has a higher energy, a photon of violet
quantum (in plural quanta). The energy associated with each light with wavelength 4000 A or a ,photon of red light with
quantum of a given radiation is proportional to the frequency of wavelength 7000 A? [h = 6.62 x 10-:34 Js] ,
the emitted radiation. '
Eocv
Solution: We know that, E ::: hv = h . E
Or E = hv where, h is a constant known as Planck's constant. Its ,A
numerical value is 6.624 x 10-27 erg-sec. The energy emitted or Given, h 6.62 X 10-34 Js, c::: 3 X 108 ms- I
absorbed by a body can be either one quantum or any whole
number multiple of hv,i.e.,2hv,3hv,4hv, ... ,nhv quanta of For a photon of violet light,
energy. A = 4000 A = 4000 x 10-10 m
, 8
Thus, energy emitted or ,absorbed = nhv, where n can have
values I, 2, 3,4, .... Thus, the energy emitted or absorbed is E = 6.62 X 10-34 x 3 x 10 4.96 x 10- 19 J
4x 10-7
quantised.
In 1905; Einstein pointed out that light can be supposed to For a photon of red light,
consist of a stream of particles, called photons. The energy of A::: 7000 A ::: 7000 x 10-10 m
each photon of light depends on the frequency of the light, 8
i. e., E hv. Energy is also related according to Einstein, as E = 6.62x 10-34 x 3 X 10 ::: 2.83 X 10- 19 J
E = mc 2 where m is the mass of photon. Thus, it was pointed out , 7000 X 10- 10
that light has wave as well as particle characteristics (dual Hence, photon of violet light has higher energy than the
nature). photon of red light.

::: :::aW5oME SOLVED ExAMPLES\ a:::"::' " ExampleS. What is the ratio between the energies of two
radiations one with a wavelength of 6000 A and other with
" Example 1. How many protons, electrons and neutrons' 2000 A?
are present in 0.18 g ?~ P? ' ,
Solution: No. of protons in one atom ofP Solution: A) = 6000 A and A2 = 2000A
= No. of electrons in one atom ofP 15 c C
No. of neutrons in one atom of P= (A - Z)= (30-15)= 15 E1 h·-andEz =h·-
Al AZ
30 0.18 '
O.l8g ISP= ::: 0.006 mol E) h· c Az 2000 1
30 Ratio, -=-x-::::; =--=
E z Al h· C Al 6000 3
No. of atoms in 0.006 mol = 0.006 x 6.02 x IOz3
or E z = 3EI
No. of protons in 0:006 mol i~p = 15 x 0.006 x 6.02 x 1023
5.418>< 102z Example 6. Calculate the wavelength, wave number and
frequency of photon having an energy equal to three electron
So, No. of electrons = 5A18 xlOz2 volt. (h =6.62 x 10-27 erg- sec.)
and No. of neutrons 5.418 x lOz2
76 G.R.B. PHYSICAL CHEMISTRY FOR COMPETITIONS

Solution: We know that, Number of photons = 1.00 9 =2.01 X 10 18


4.965x10- 1
E=h·v
Example 10. Find the number of quanta of radiations of
v=E (leV = 1.602 x 10- 12 erg)
h frequency 4.67 x 1013 S~I, that must be absorbed in order to melt
12 5 gofice. rhe energy required to melt 19ofice is 333.1.
= 3x (1.602 x 10- )= 7.26x 1014 S-I = 7.26-x 1014 Hz
6.62x 10-27 Solution: Energy required to melt 5 g of ice
10
A=~= 3x10 14 =4.132x10-5 cm
= 5x 333= 1665J
V 7.26x10 Energy associated with one quantum
_ 1 1 4 1 = hv = (6.62 X 10-34 ) X (4.67 X 1013 )
V =- 2.42 x 10 cm-
A = 30.91 X 10-21 J
Example 7. Calculate the energy in kilocalorie per mol of. Number of quanta required to melt 5 g of ice
the photons of an electromagnetic radiation of wavelength 1665 = 53.8; 1021
76001. 30.91 X 10-21
= 538x 1022
Solution: A = 7600A = 7600 x 10-8 cm
.... Example 11. Calculate the wavelength ofthe spectral line,
c 3x10 lO cms-1
when the electron in the hydrogen atom undergoes a transition
c 3 x 1010 from the energy level 4 to energy level 2 ...
Frequency, v = - = = 3.947 X 1014 S-I
Solution: According to Rydberg equation,
8
A 7600 x 10-

Energy of one photon = hv = 6.62 X 10-27 x 3.947 X 10 14 ..!.=R[~2 __l )


A x . y2
= 2.61 X 10- 12 erg
R=109678cm- l ; x 2; y=4
Energy of one mole of photons = 2.61 x 10- 12 x 6.02 X 10 23
= 15.71x 1011 erg 1 = 109678[''!'
A 4
1
16
J
Energy of one mole of photons in kilocalorie
3
=109678x-
= 15.71 x 1011 [1 kcal = 4.185 x 1010 erg] 16
4.185 x 1010 On solving, A 486 nm
= 37.538 kcal per mol Example 12. A bulb emits light of wavelength A = 45001.
Example 8. Electromagnetic radiation of wavelength The bulb is rated as 150 watt and 8% of the energy is emitted as
242 nm is just sufficient to ionise the sodium atom. Calculate the light. How many photons are emitted by the bulb per second?
ionisation energy in kJ mol -I ,h = 6.6256 X 10-34 Js. (lIT 1992) (liT 1995)
Solution: Energy emitted per second by the bulb
Sol~t ': A= 242 nm = 242 x 10-9 m
=150x 8 J
c 3x 108 ms- I 100
8 hc 6.626 x 10-34 x 3x 108
E=hv=h.!: 6.6256 x 10-34 x 3x10 E nergy 0 fl p hoton:::: - = .
A 4500 x 10-1(1
A 242 x 10-9
19
= 4.42 x 10- joule
=' 0.082 X 10- 17 J = 0.082 X 10-20 kJ
Let n photons be evolved per second.
Energy per mole for ionisation = 0.082 x 10-20 x 6.02 X 1023
493.6 kJ mol-I nx4.42xl0- 19 150x~
100
Example 9. How many photons of light having a n = 27.2 x lOiS
wavelength 4000 1 are necessary to provide 1.00 J of energy?
Example 13. A near ultraviolet photon of 300 nm is
'-'0In11',n: Energy of one photon absorbed by a gas and then remitted as two photons. One photon
is red with wavelength of 760 nm. What would be the wave
hv=h·!:
A number of the second photon?
(6.62 X 10-34 )(3.0 X 108 ) Solution: .
Energy absorbed = Sum of energy of two quanta
4000 x hc ·hc hc
---- = + -----
= 4.965 X 10- 19 J 300 x 760 X 10-9 . AX 10-9
ATOMIC STRUCTURE 77
On solving, we get, AgCI(s) ---7 Ag(s) + Cl
V (wave number) :::: ..!. 2.02 X 10- m- 3 I Escape ofchlorine atoms is prevented by the rigid structure of
A the glass and the reaction therefore, reverses as soon as the light
Example 14. Calculate the wavelength of the radiation is removed. If 310 kJ / mol of energy is required to make the
which would cause the photodissociatio11. of chlorine molecule if reaction proceed, what wavelength of light is necessary?
the Cl-Cl bond energy is 243 kJ mol -I. Solution: Energy per mole Energy of one Einstein
Solution: Energy required to break one CI-CI bond i. e. , energy of one mole quanta
Bond energy per mole
Nhc
:::: A
Avogadro's number
6.023 X 10 23 X 6.626 X 10-34 X 3 X 108
243 kJ 243 x 10
3
J 310x 1000
A
23 23
6.023 X 10 6.023 X 10 . A 3.862 X 10-7 m =3862 X 10-
10
m 3862A
Let the wavelength of the photon to cause rupture of one
CI-Cl bond be A.
fUlimATfONS ·Of OSJ£ClIVE
We know that, 1. The frequency of the radiation having wave number 10m-I is:
34 8 23
A:::: hc :::: 6.6 X 10- X 3 X 10 X 6.023 X 10 (a) tOs- 1 (b) 3 x 107 S-1
E 243 X ( c) 3 X loll S-I (d) 3 X 109
7
:::: 4.90 X 10- m:::: 490 nm [ADs. (d)]
1
Example 15. How many moles of photon would contain [Hint:
sufficient energy to raise the temperature of225 g of water 21°C A
to 96°C? Specific heat ofwater is4.18J g-I K -I andfrequency v cv =- e
= 3 x 108 X 10 =- 3 X 109 S-l]
A
of light radiation used is 2.45 X 109 S -I. 2. The energy of a photon of radiation having wavelength
Solution: Energy associated with one mole of photons 300nmis:
::::No xhxv (a) 6.63 x 10-29 J (b) 6.63 X 10- 19 J
:::: 6.02 X 10 23 X 6.626 X 10-34 X 2.45 X 109 (c) 6.63 X 10-28 J (d) 6.63 x 10- 17 J
2 1
:::: 97.727 X 10- J mol- [Ans. (b).]
Energy required to raise the temperature of 225 g of water by he 6.626 X lO-34 x 3 x 108
75°C =mxsxt [Hint: E A 6.63 X 10-19 J]
300 X 10-9
= 225 X 4.18 X 75 = 70537.5 J
3. The maximum kinetic energy of the photoelectrons is found
Hence, number of moles of photons required
to be 6.63 x 10- 19 J, when the metal is irradiated with a
:::: mst 70537.5:::: 7.22 X 104 mol
radiation of frequency 2 x 1015 Hz. The threshold frequency
Nohv 97.727 x 10-2
of the metal is about:
Example 16. During photosynthesis, chlorophyll-a (a) 1x Hy5 8- 1 (b)2xI015 8- 1
absorbs light ofwavelength 440 nm and emits light ofwavelength
670 nm. What is the energy available for photosynthesis from the (c) 3x 1015 S-I (d) 15 X Id 5 S-I
absorption-emission ofa mole ofphotons? [Ans. (a)]
[Hint: KE =h(v - vo)
Solution: M:::: [NhC] [Nhc] KE
vo = v - -
A absorbed A· evolved h

::::NhC[ 1
Aabsorbed
- .
Aevolved
J = 2X 101~
6.63 X 10- =1 X 1015 s-I]
_
19

6.63 x
23
:::: 6.023 X 10 X 6.626 X 10-34 X 4. The number of photons of light having wavelength 100 nm
which can provide 1 J energy is nearly:
3 X 108 [ 1 _ 1
440 X 10-9 670 X (a) 107 photons (b) 5 xJOIS photons
:::: 0.1197[2.272 x 106 -1.492x 106 ] (c) 5 x 1017 photons (d) 5 x 107 photons
:::: 0.0933 X 106 J I mol:::: 93.3 kJ I mol [Ans. (c)]
[Hint: E= nhe
Example 17. Photochromic sunglasses, which darken
A
when exposed to light, contain a small amount of colourless
AgCI (s) embedded in the glass. When irradiated with Ught, EA
metallic silver atoms are produced and the glass darkens. he
78 G.R.B. PHYSICAL CHEMISTRY FOR COMPETITIONS

5. The atomic transition gives rise to the radiation of frequency (vi) The emission or absorption of energy in the form of
(104 MHz). The change in energy per mole of atoms taking radiation can only occur when an electron jumps from one
place would be: . stationary orbit to another.
(a) 3.99 x 10-6 J (b) 3.99 J f),E:::: E bigh - E low :::: hv
24 30
(c) 6.62 X 10- J (d) 6.62 x 10- J Energy is absorbed when the electron jumps from inner to outer
[Ans. (b)] orbit and is eniitted when it rhoves 'from outer to an inner orbit.
[Hint: E =Nhv .
= 6.023 x loB x 6.626 X 10-34 x 104 X 1(f
= 3.99J]
.2l!~;l~ BOHR'S ATOMIC MODE;L Electrons excited
by absorbing
To overcome the objections of Rutherford's model and to explain
energy
the hydrogen spectrum, Bohr proposed a quantum mechanical (Energy absorbed)
model of the atom. This model w~s based on the quantum theory
of radiation and the classical laws of physics. The important
postulates on which Bohr's model is based are. the following:
(i) The atom has a nucleus where all the protons and neutrons ::::::::;'::::::::::::::::l Energy radia1ed
are present. The size of the nucleus is very small. It is present at when electrons
the centre of the atom. fall back
(ii) Negatively charged electrons are revolving around the n=3 (Energy emitted)
(M) n=4
nucleus in the same way as the planets are revolving around the
(N) n=5
sun. The path of the electron is circular. The force of attraction (0) n=6
between the nucleus and the electron is equal to centrifugal force (P)
. of the moving electron.
Fig. 2.10
Force of attraction towards nudeus centl'ifugal force
(iii) Out of infinite number of possible circular orbits around When the electron moves from inner to outer orbit by
the nucleus, the electron can revolve only on those orbits whose absorbing defini~e amount of energy, the new state of the electron
is said to be excited state (Fig. 2.10).
angular momentur.n * is an integral multiple of !!...., i. e. ,
Using the above postulates, Bohr calculated the radii of.
2n
various stationary orbits, the energy associated with each orbit
mvr:::: n!!.... where m:::: mass of the electron, v:::: velocity of and explained the spectrum of hydrogen atom.
2n .
electron, r = radius of the orbit and n =1; 2, 3, ... number of the Radii of various orbits: Consider an electron of mass' m'
orbit. The angular momentum can have values such as, and charge 't! revolving around the nucleus of charge Ze
(Z =atomic number). Let 'v' be the tangential velocity of the
h , 2h , 3h ; etc., but it cannot have a fractional value. Thus, the revolving electron and ' r' the radius of the orbit (Fig. 2.11). The
21t 2n 21t ,
electrostatic force of attraction between the nucleus and electron
angular momentum is quantized. The specified or circular orbits
(quantized) are called atationary orbits. kZex e kZe 2
(applying Coulomb's law)
(iv) By the time, the electron remains in anyone of the
stationary orbits, it does not lose energy. Such a state is called
v
grou.nd or normal state.
In the ground state, potential energy of electron will be Centrifugal
minimum, hence it will be the most stable state. force
(v) Each stationary orbit is associated with a definite amount
of energy. The greater is the distance of the orbit from the
nucleus, more shall be the energy associated with.it These orbits
are also called energy levels and are numb~red as 1, 2, 3,4, .... or
K, L, M, N, ... from nucleus outwards.
i.e., E, <.E2 < E3 < E4 ...
Fig. 2.11
(E2 - E, » (Ei - E 2 » (E4 - E3 ) ....

*Angwar momentum = lro, where; I b moment of inertia and ro =angular velocity;


O)= !: , where, v =linear velocity and radius; and I ,;" m?-. So angular momentum = mr2 . v = mvr.
r r
ATOMIC STRUCTURE 79

where, k is a constant. It is equal to 1 ,Eo being absolute of


Energy an electron: Let the total energy of the electron
4TtEo be E. It is the sum of kinetic energy and potential energy.
E =kinetic energy + potential energy
permittivity ofmedi~. In SI units, the numerical value of_l_
4TtEo _ 1 2 kZe 2
--mv - - -
is equal to 9 x 10 9 Nm 2 I C 2 • 2 r
[Note: In CGS units, value of k is equal to 1.] Putting the value of mv 2 from eq. 0),
2 2 2
. As force of attraction = centrifugal force E = kZe _ kZe :::: _ kZe
2 2 2
kZe mv
So, --=--
r2 r
or v2 =kZe
rm
2r r 2r
Putting the value of r from eq. (iii),
Ze 2
2 _ 1 kZe 2 4Tt 2 mkZe 2 21t 2 Z 2k 2 me 4
v - -_.- ... (i) E::::---x· =- ...(iv)
4TtEo rm 2 n2h2 n2h2
According to one of the postulates, For hydrogen atom, Z = 1
h 2Tt 2 k 2 me 4
Angular momentum:::: mvr:::: n - So, E::::
2Tt
nh Putting the values ofTt, k, m, eand h,
or v=-- ... (ii)
21tmr E =_ 2x (3.14)2 x (9 X 109 )2 x (9.1X 10-31 )x (1.6x 10- 19 )4
Putting the value of <v' in eq. (i), n 2 x (6.625 x 10-34 )2 ' '
n 2h 2 kZe 2
-"...-:"-:-. = - - or 21.79 x 10- 19
4Tt 2 m2 r2 mr ----,--- J per atom
n 2 h2
or r=~:---.".. ... (iii)
4Tt 2 mkZe 2 E _RH (where, RH =2.I8x 10-18 J)
Greater is the value of < n', larger is the size of atom. On the n2
other hand, greater is the value of <Z', smaller is the size of the 13.6 V
- - 2 e per atom (1 J:::: 6.2419 X 1018 eV)
atom. Across a period from left to right, atomic number < Z' n
increases with constant value of < n' hence atomic radius
decreases towards right. On moving down the group, both < Z' and -- - - kcall mo1
313.6 (l eV = 23.06 kcallmol)
'n' increase but due to shielding, Z* (effective nuclear charge) n2
remains same. Hence, on moving downwards, atomic radius _ 1312kJ/moI
increases due to increase in' n'. . n2 '

n2h2
For hydrogen atom, Z = I; so r = 2 2 ·' . energy m
Kmetlc . n th she11 = 13.6>< Z 2 ev
:'\7
4Tt mke
Now putting the values of h, It, m, e and k,
"1 energy m
. n th sheII -27.2 2x Z
2 V

r= .•
n 2 X (6.625 X10-34 i Potentla
n
e

4 x (3.l4i x (9.1x 10-31 ) x (1.6x 10- 19 )2 x (9x 109 ) Substituting the values of n :::: 1, 2, 3,4, ... , etc., the energy of
electron in various energy shells in hydrogen atom can be
= 0.529x n 2 x 10-10 m:::: 0.S29 x n 2 A calculated.
8 2
= 0.529 X 10- X n cm Energy sbeII E (Joule per atom) E (eV per atom) E (kcal/moI) .
where h:::: 6.625 X 10-34 J-sec, Tt = 3.14 I' ,..21.79 X 10- 19 .!..13.6 - 313.6

m= 9.1x 10-31 kg, e= 1.6 x 10-19 coulomb 2 -5.44 x 10-19 -3.4 -78.4
3 -2.42 X 10-
19 -1.51 -34.84
k 9xl09 Nm2/C 2
4 -1.36 X lQ-19 0.85 -19.6
Thus, radius of 1st orbit
0.529x 10-8 x 12 = 0.529 x 10-8 cm= 0.529 x 1O-10 m
Radius of 2nd orbit
=0.529x 10-8 x 22= 2.11 X 10-8 cm= 2.l1x 10-10 m o o o
Radius of 3rd orbit (for hydrogen atom)
0.529x 10-8 x 32 4.76x 10-8 cm= 4.76x 10- 10 m
2 .
and so on. and E" =£1 X Z (for hydrogen like species)
r" :::: 11 x n 2 fo~ hydrogen atom
2 where, £1 :::: energy of hydrogen fIrst orbit.
and ... r" =0.529 x -n A (for hydrogen like species)
Z
80 G.R.B. PHYSICAL CHEMISTRY FOR COMPETITIONS

Since, n can have only integral values, it follows that total Interpretation of hydrogen spectrum: The only electron
energy of the electron is quantised. The negative sign indicates in the hydrogen atom resides under ordinary conditions on the
that the electron is under attraction towards nucleus, i. e., it is first orbit. When energy is supplied, the electron moves to higher
bound to the nucleus. The electron has minimum energy in the energy shells depending on the amount of energy absorbed.
first orbit and its energy increases as n increases, i. e. , it becomes When this electron returns to any of the lower energy shells, it
less negative. The electron can have a maximum energy value of emits energy. Lyman series is formed when the electron returns to
zero when n = 00. The zero energy means that the electron is no the lowest energy state while Balmer series is formed when the
longer bound to the nucleus, i. e. , it is not under attraction towards electron returns to second energy shell. Similarly, Paschen,
nucleus. Brackett and Pfund series are formed when electron returns to the
For hydrogen h'ke species such as He . 2+ , etc."
+ ,L· third, fourth and fifth energy shells from higher energy shells
En =Z2 xEn for hydrogen atom. respectively (Fig. 2.l2).
Velocity of an electron: We know that,
Centrifugal force on electron
== force of attraction between nucleus and electron
mv Ze
2 2
(in CGS units) ... (i)
r
The angular momentum of an electron is given as:
mvr= 2n
nhf .. ·(i0
From eqs. (i) and (ii), we have Balmer 1.0--1---1 Brackett

v(;~)= Ze 2
series series

v= ~ [2n;2)
v = ~ x 2.l88 X 108 cmf sec ... (iii)
n
2.188 x 108 Lymanserles
v cm! sec (For hydrogen, Z = 1)
n Fig. 2.12
VI = 2.188x 108 cmlsec Maximum number of lines produced when an electron jumps
1 . .. n(n --, 1)
v2 = x 2.l88x 108 cm!sec from n th level to ground level IS equal t o ' . For example,
2 2
in the case of n = 4; number of lines -produced is 6. (4 -t 3,4 -t 2,
I x 2.188x 108 cm!sec 4 -t 1,3 -t 2, 3 -t 1,2 -t n When an electron returns from n2 .to
3 nl state, the number oflines in the spectrum will be equal to.
Here, VI ,V2 and V3 are the velocities of electron in first, (nz - nl )(nz nl + 1) ,
second and third Bohr orbits in hydrogen. . 2
From equation (iii), If the electron comes back from energy level having energy E 2
VI _ 2 V3 1 to energy level having energy E[, then the difference may be
and - == - and so on.
V2 1 . VI 3 expressed in terms of energy of photon as:
Orbital frequency: Number of revolutions 'pet"'second by E2 - E[ =!lE == hv
an electron in a shell is called orbital frequency; it may be or the frequency of the emitted radiation is given by
calculated as, !lE
v -
Number of revolutions per second by an electron in a shell h
Vel~city 2)V l( Since, !lE can have only definite values depending on the
Circur:.ference = = 2nr h 7 definite energies ofE2 and E 1 , v will have only fixed values in an
atom,
= Z2 x 6.66 x rol5 or v
c !lE
n3 A h
where, E. = Energy offrrst shell. he
Time period of revolution of electron in nth orbit (Tn) : or A
!lE
3 .
Since, hand e are constants, !lE corresponds to definite
~ x 1.5 x 1O-[? sec energy; thus, each transition from one energy level to another will
Z2
produce a light of definite wavelength. This is actually observed
as a line in the spectrum of hydrogen atom.
ATOMIC STRUCTURE 81

Thus, the different spectral Hnes in the spectra of atoms


cOiTespond to ditferent transitIOns of electrons from higher
Afirst
=R[_1
n[
_ 1 '1
+ J (nl 1)2
energy levels to lower energy levels.
Similarly for second, third and fourth lines,
Derivation of Rydberg Equation
n2 nl + 2; n z = nl + 3 and n z n, + 4 respectively
Let an excited electron from n2 sheli come to the n[ shell with the
release of radiant energy. The wave' number v of the corres- .. Rydberg equation may be written as,
ponding spectral line may be calculated in the foIIowingmanner: v= 1 RZz,nt
[ I 1]
21t 2 mZ 2 e 4 21t 2 mZ2 e 4 A (nl +X)2
M=E z -E[ (-) 2 2 -(-) 2' 2 '
n2 h n[ h
• where, x number of line in the spectrum.
he 2
21t mZ e
2 4 [_1__ 1 e.g., x = 1,2,3,4, ... for first, second, third and fourth lines in the
h2 nt ' n~ spectrum respectively. '
he
(ii) Series limit or last line of a series: It is the line of
where, M = hv shortest wavelength or line of highest energy.
A For last line, nz 00
i = J.. =21t2mZ2e l'_I___1_) , 4 R
=
'A 2
eh 3 , n I2 n2 )~last
· R' ; R
L yman 1·
orv=RZ [_1n __n1) 2
2 2
Imlt= Balmer limit

I 2 R ;
Pasch"en, 1"Imll =? B raek ett '1Imlt
" = R
21t 2me 4
, 3- 42
where, R= Rydbergconstant= 109743 cm- I
ch 3
This value of R is in agreement' with ,experimentally
Pfund limit R,
52
H~mphrey limit = 6'~
determined value 109677,76 cm- 1 • Rydberg equation for (iii) Intensities of spectral 'lines: The intensities of spectral
hydrogen may be given as, lines in a particular series decrease with increase in the value of
, l
v 1 = R [ -\- -
A nl
-\-J
n2
n2' i. e., higher state.
e.g.,
, Lyman series (2 ---; 1) > (3 ---; 1) > (4 ---; 1) > (5 ---; 1)
("2 ~nl)
ModificatIon of Rydber,gEquation
Balmer series (3 4 2) > (4 ---; 2» (5 ~ 2) > (6---;2) .' .
According to the Rydberg ~quation:

wave~umber [n~2 n~]


2 4 DeGfeasing intensity of the spectral lines
21t mZ 2e
lontzationEnergy and 'EXCitation 'E.nergy
It can be considered that the electron and the nucleus revolve
around their common. centre of mass. Therefore" instead of .the, Excitation potential for nl' ---;,112
Electronic charge
mass of the electron, the.,reduced mass of the system was
introduced and the equatiOilbecomes: E
Ionization potential for n 1 ---; 00 == 111

v = 21t J.1Z 2e
.
2
1: 3
4
[_1 __1]
2
, Electronic charge
Cl nl n22 , , The energy required to remove an electron from the groul\d
. , state to form cation, i.~. , to take the electrop to infinity, is called
Reduced mass '11' can be calculated as, ionization energy.
1 1 1
-=-+- IE E"" - Eground
11 m M
where, mass of electron
m IE:::: 0- El (H)= 13.6eV atom-I
and mass of nucleus
M :::: 2.17 X 10- 18 Jatom- I
mM
.. J.1 Z2
m+M IE=- x 13.6eV
(i) First line of a series: It is called 'line of longest , n2
wavelength' or 'line of shortest energy'. Z2
I1 _I_,X
For first line, •
10'
'"
n 12 Z2
2
82 I G. R.B. PHYSICAL CHEMISTRY FOR COMPETITIONS

(IE)H X Z 2 radial velocity and the other perpendicular to the radius vector
(IEh = 2 called transverse or angular velocity. These two velocities give
.. n
rise to radial momentum and angular or azimuthal momentum.
If an electron is already present in the excited state, then the
Sommerfeld proposed that both the. momenta must be integral
energy required to remove that electron is called· separation
energy. multiples of!!...... [Fig. 2.13 (b)].
21t
E separation E ~ - E excited
h
The following points support Bohr theory: Radial momentum n -
r 21t
(i) The frequencies of the spectral lines calculated from . h
Bohr equation are in close agreement with the Azimuthal momentum =n<l>-
frequencies' observed experimentally in hydrogen 21t
spectrum. nr and n<l> are related to the main orbit 'n' as:
(ii) The value of Rydberg constant for hydrogen calculated n nr + n<l>'
from Bohr equation tallies with that determined n nr + n<l> Length of major axis
or
experimentally. . n<l> n<l> Length of minor axis
(iii) The emission and absorption spectra of hydrogen like (i) 'n<l>'. cannot be zero becau~e un,der this condition, the
species such as He + , Li 2+ and Be 3 + can be explained ellipse shall take the shape of a straIght hne.
with the help of Bohr theory. (ii) 'n<l>' cannot be more than' n'because minor axis is always
Limitations of Bohr Theory smaller than major axis.
" • 't
(iii) 'n<l>' can be equa~ to ' n'.. Under this ~onq.ition, the major
(i) It does not explain the spectra ofmulti-electron atoms.
axis becomes equal to mmor aXIS and the elhpse 41kes the shape
(Ii) When a high resolving power spectroscope is used, it is of a circle. Thus, n<l> can have all integral values up to ' n' but not
observed that a spectral line in the hydrogen spectrum is zero. When the values are less than' n',orbits are elliptical and
not a simple line but a collection of several lines which when. it becomes equal to 'n', the orbit is circular in nature.
are very close to one another. This is known as fine
For n == I, n4J can have only one value, i. e., 1. Therefore, the
spectrum. Bohr theory does not explain the fine spectra
first orbit is circular in nature.
of even the -hydrogen atom.
(tii) It does not explain t~e splitting of spectral lines into a
For n =2, n. can have tWo values 1 and 2, i. e., ~e ~econd
orbit has tWo sub-orbits, one is elliptical and the other IS CIrcular
group of finer lines under the influence of magnetic field in nature.
(Zeeman effect) and electric field (Stark effect).
For n == 3, n4J can have three va~ue.s I, 2 and 3, i..e., t~ird or~it
(iv) Bohr theory is not in agreement with Heisenberg'S has three sub-orbits, two are elhphcal and one IS CIrcular m
uncertainty principle. ' nature.
For n = 4, n4J can have four values I, 2, 3 and 4, i. e., fou~h
III SOMMERFELD'S EXTENSION OF orbit has four sub-orbits, three are elliptical and fourth one IS
BOHR THEORY circular in nature (Fig. 2.14).
To accouht for the fine spectrum of hydrogen atom, Sommerfeld,
in 1915, proposed that the moving electron might descri~e
elliptical orbits in addition to circular orbits and the pucleus IS
situated at one of the foci. During motion on a circle, only the
angle of revolution changes while the distance from the nucleus
remains the same but in elliptical motion both the angles of
revolution and the distance of the electron from the nucleus
change. The distance from the nucleus is termed as~adius v,ector
and the angle of revol\ltion is known as :azimutluda~gle. The
tangential velocity of the electron at a particular instant can be
resolved into two components: one along the radius vector called
V Radial velocity Fig. 2.14
. V (Tangential velocity)
Sommerfeld thus introduced the concept of subenergy shells.
In a main energy shell, the energies of subshells differ slightly
from one another. Hence, the jumping of an electron from one
r-..1----I---..!---1 Major axis
energy shell to another energy shell will involve slightly different
amount of energy as it will depend on subshell also. This explains
to some extent the fine spectrum,Qf hydrogen atom. However,
Minor axis Sommerfeld extension fails to explain the spectra of
(a) (b)
multielectron atoms.
Fig. 2.13
ATOMIC STRUCTURE 'I 83

2i.7xlO- 12
: :: ::::_SOME SOLVED ExAMPLES\ =:::: : .
---'--.,,---'-'- erg = 5.425 x 10-12 erg,
22
~~ii"E~ample 18. Calculate the wavelength arid energy of
radiation emitted for the electronic transition from infinite to and. E"", = 0
stationary state of hydrogen atom, '(Given, R 1.09678 M == Change in energy E"",...., 5.425 X 10- 12 erg ,
xl0 7 m-l,h=6.6256xlO~34J-sandc 2.9979xl0 8 m:s.,-I) Thus, energy required to remove an electron from 2nd orbit
, = 5.425 X 10- 12 erg
Solution: l
A
R[~
nf ·~l
ni According to quantum equation,
c

r~nn~:2, 1
M=h·
A
R or A ~
M
or A I 1 9.11x 10-8 m (h = 6.625 x 10-27 erg - sec; c 3 x 1010 cml sec)
R 1.09678 x 107
and M = 5.425 X 10- 12 erg
We know that,
A (6.625 X 10-27 ) X (3 X 1010 )
E := hv h. c == 6.6256 x 10-34 X 2.9979 x So,
A 9.11 X 10-8 5.425 X 10- 12
=3.7xl0-5 cm
2.17 X 10- 18 J
Thus, the longest wavelength of light that can cause this
Example 19. Calculate the velocity (cm /sec) of an transit40n is 3.7 x 10-5 cm. '
electron placed in the third orbit of the hydrogen atom. Also
calculate the number of revolutions per second that this electron Example 21. Calculate the shortest and longest
makes around the nucleus. wavelengths in hydrogen spectrum ofLyman series,
Solution: Radius of 3rd orbit Or
32 x 0.529 x 10-8 4.761x 10-8 cm Calculate the wavelengths of the first line and the series limit
We know that, for the Lyman series for hydrogen. (R H == 109678 cm- I )
nh nh Solution: For Lyman series, n l I
mvr or v = - -
2n 2nmr For shortest wavelength in Lyman series (i. e. ,series limit), the
3 x 6.624 x 10-27 energy difference in two states showing transition should be
2 x 3.14 x (9.108 x 10-28 )x (4.761 X 10-8 ) maximum, i. e. , n2 00.,

= 0.729 x 108 cmlsec So,


T ·ime taken &lor one revolutton
' -2nr
v
A=_l_, =9.117x 10-6 cm
Number of revolutions per second 109678
1 v
'2nr 2nr = 911.7A
For longest wavelength in Lyman series (i.e., first line), the
v
energy difference~"two states showing transitiqR should be
0.729 x 108 minimum, i. e. , n2 ±it< .~,
2 x 3.14 x 4.761 x
= 2.4 X 1014 revolutions I sec So,., ±= L;?-RH 2; J ! RH
, Example 20. The electron energy in hydrogen atom is
4 I 4
. bE
12
Cl or A - x - = - - - - 1215.7x 10-8 c~
given y = - 21.7xlO- a cu Iate the energy reqUlre
. d to
3 RH 3 x 109678
remove an electron completely fromn= 2 orbit. What is the = 1215.7 A
longest wavelength (in cm) of light that be used to cause this
transition?
,Example 22. Show that the Balmer series occurs between
3647 A and 65631. (R 1.0968 x 10 7 m- I )
21.7x 10-12 Solution: For Balmer series,
Solution: E=- , .' 2 erg
n
Electron energy in the 2nd orbit, i. e. , n 2,
1Rr~-
A L22 n2
1.
G.R.B. PHYSICAL CHEMISTRY FOR COMPETITIONS

where, n = 3,4,5, ... 00 ·l~~ample.2~.:. If the. ' energy differenge,be/ween two


. . . , ,.. . .' .• . I' . . . .
To obtain the limits for Balmer series n = 3 and n = 00 elegtronic state.~ ..is 46:12 kcal mor. " what wjll be. lhe:freQllency
respectively. of tile light emitted when the electrons drop from hi.~he,. tolower
36
Amax (n 3) - - - - - = - states? (Nh == 9.52 x 10-14 kcal sec mor l ,where, N is the
R[_l _~]
22
5R
3 2
Avogadro s number a~d h is the Planck s constant)
Solution: l::iE = 46.12 kcal mol- 1
36 According to Bohr theory, l::iE == Nhv
= m
5 x 1.0968x 107 l::iE 46.12
or v =- :: -----:-:-
= 6563A Nh 14
9.52 X 10-
1 4
Amin (n=oo)=-----
R[~
22
__l ]
00
2
R = 4.84 X 1014 cycle sec-I

Example 26. According' to Bohr theory, the electronic


energy of the hydrogen atom in the nth Bohr orbit is given by
-~---,,-m
19
1.0968 x 10 7 E = -:. 21.76x 10- J
= 3647A . n . n2

f?L:~_,~~ample 23. Light of wavelength 128181 is emitted when Calculate the longest wavelength of light that will be needed
the electron ofa hydrogen atom drops from 5th to 3rd orbit. Find to remove an electron from the 3rd orbit of the He + ion.
the wavelength ofthe photon emitted when the electron falls from (liT 1990)
3rd to 2nd orbit. Solution: The electronic energy of He + ion in the nth Bohr
St)lution~ We know that, . orbit

.!A = R [-\- - -.12]


nl n2
where, Z= 2
When, III ='3 and n2 = 5,
Thus, energy of He + in the 3i:d Bohr orbit
1 [1 1] 16R 19
12818=R 9- 25 = 9x25 = 21.76 x 10- x4J
9
or 12818 9x 25 ... (i)
16xR l::iE = E~ - E3

When, n l == 2 and n2 3, == 0- [_ 21.76~ ~0-19 x 4]


.! R [.! _~] = 5R
A 4 9 . 36 21.76 x 10- 19 x 4
A 36 ... (ii) 9
5R he 6.625 x 10-34 x 3 X 108 X 9
Dividing eqn. (ii) by eqn. (i),
We know that, A ::
l::iE 21.76 x x4
A 36 16R 64 = 2.055 x 10-7 m
-- -x--
12818 5R 9x 25 125
Example 27. Calculate the ratio l?f the velocity of light
and the velocity of electron in the first orbit of a hydrogen
A == ~ x 12818= 6562.8 A atom. (Given, h = 6.624 X 10-27 erg-sec; m = 9.108 X 10-28 g,
125
r= 0.529 x 10-8 cm)
Example 24. The ionisation energy o/hydrogen atom is
1l6eV. What will be the iOliisafion energyofHe+ and Li 2+ ions? h
V=--
~~ Ionisation energy:: - (energy of the 1st orbit) 21tmr
6.624 x 10-27
Energy of the 1st orbit of hydrogen = 13.6 yV
Energy of the 1st orbit of He+ = -13.6 x Z2 (Z for He+ = 2) 2 x 3.14 x 9.108 x 10-28 x 0.529 X 10-8
= -13.6x 4 eV= - 54.4 eV == 2.189 X 108 cml sec
So, Ionisation energy ofHe+ ;. -(-54.4) =54.4 eV
2
. Energy of 1st orbit of Li + = -13.6 x 9 (Z for Li 2:. =: 3)
137

== -122.4 eV
Ionisation energy of Li 2+ == -(-122.4) = 122.4 eV
ATOMIC STRUCTURE I· 85
Example 28. The wavelength of a certain line in Balmer 3R
series'is observed to be 4341 A. To what value of 'n' does this 4
correspond? (R H =109678cm- l ) For hydrogen atom,

Solution: .!. =RH


A
[~-1.:]
22 n 2
.!., =R [_1 I2
A nt n2, ]
1 1 1
So, 3R = R
4
[l.: -.1-]
nt ni
= - - --'-----:-"----
4 4341x x 109678 1 3.
or
0.04 n; ni 4

=25 i.e., nl = 1 and n 2 = 2


0.04
Example 31. Calculate the energy emitted when electrons
or n·= 5 of 1.0 g atom ofbydrogen undergo transition giving the spectral
Example 29. Estimate the difference in energy between line of lowest energy in the visible region of its atQmic spectrum.
the first and second Bohr orbit for hydrogen atom. At what (RH = 1.1 x 107 m-I;c= 3x 108 ms-I;h = 6.62x 10-34 J-s)
a
minimum atomic number would. transition from n = 2 to n = 1 (liT 1993)
energy level result in the emission of X-rays with Solution: The transition occurs like Balmer series as
A = 3.0 x 10-8 m ? Which hydrogen-like species does this atomic spectral line is observed in visible region.
number correspond to? (liT 199~) Thus, the line of lowest energy will be observed when
Solution: AE hv = -
h·c transition occurs from 3rd orbit to 2nd orbit, i. e., nl =2 and
A n 2 = 3.
and .!.= R
A
[_In~ __ 1]
ni ±=R[2; - 3~] :6 R
AE , [1-2-2"1]
R·h·c E=hv h·~=6.62xJO-34 x3xlO8 x2 x 1.1 x lO'
,nl n2 A 36
3 = 3.03 x 1O~19 J per atom
AE=h.c.
, 4R
Energy corresponding to ).0 g atom of hydrogen
{j.625 X lQ-34 x 3 X 108 x 1.09678 X 10 7 x 3
="-----~------------- 3.03 x ) O-I~ x Avogadro' Ii number
4
~ 3.03 x 10- 19 x 6023 X 1023 J
=1.635 X 10- 18 J
= 18.25 X 104 J
For hYdro:n~I~:;::[iesi __1_] '" Example 32. How many times does the electron go around
ihe first Bohr s orbit ofhydrogen in one second? .
, nt ni 'v
SoJutif}o: Nmrtber of revolutions per second ... (i)
1]
.!.=Z2R[_1 __
2nr
8
A nl
2' 2 2.188x 10 /
n2 v= ' em see
n
1 " = Z2 x 1.09678 X 107 x
3.0x 10-8 12
[..!. -~]
22 ,..',X 10~;,,'
v = 2',18.8 ':~,' / see
'" ,=; 2'' ."'I'S'8"-x '1'08-, em
- 1'--'
Z2= 4 z4 Il
Z
3 x 10-8 x 1.09678 X 10' x 3 r=-xO.529A
Z
or Z=2 12 ,
The species is He + . = - x 0.529x 10-8 cm
1
, "Example 30. What transition in the hydrogen spectrum
0.529 x 10-8 em
have the same wavelength as Balmer transition n = 4 to n = 2 of
He+ spectrum? (liT 1993)
· 2.l88xl08
SolutiQD: For He + ion, .'. N urnber 0 f revo IutlOUS per sec = - - - - - - - - - - , -
2 x 3.14 x 0.529 x
I =Z2R[_1 _~]
A, nf n~, = 6.59x 1015
86 G.R.B. PHYSICAL CHEMISTRY FOR COMPETITIONS

;;)<:> Example 33. Calculate the wavelength of radiations [Hint: . 1iHe+ ) = Z 211 (H)
emitted, produced in a line in Lyman series, when an electron 22 x E = 4E]
falls from fourth stationary state in hydrogen atom. 8. The number of spectral lines that are possible when electrons
(R H L! X 107 1
111- ) (liT 1995) in 7th shell in different hydrogen atoms return to the 2nd
shell is:
Solution: 1 - R [-I __1_'j\
A- n; ni (a) 12
" [Ans. (b)]
(b) 15 (c) 14 (d) lQ

Llx 10 7 (~, _ _
11 (fl:! - nl )(llz n t + 1)
, '4 ) e 2 [Hint: Number of spectral lines
2
969;6 x IO-10metre (7 - 2)(7 2 + 1) = 15] <
A= 969.6A 2
Example 34. What is the degeneracy of the level of the 9. The ratio of radii of first orbits ofH, He + and Li 2 + is:
(a)I:2:3 (b)6:3:2 (c)I:4:9 (d)9:4:1
. hydrogen atom that has the energy (- RH
, . 9
J, ? [Ans. (b)]
n2
Solution: E =_ RH =_ RH [Hint: r=~ x 0.529 A
n, 'n2 ,9 Z
n 3 rH : rHe + : rLi2+

, Thus, 1= 0 and m ::::: 0 (one 3s--orbital) 1: 1 .!


1=1 and m = 1,0, + I (three 3 p-orbitals) ; ,~ , 2 3
1=2 and m =-
2, -1, 0, + 1,+ 2 (five 3d-orbitals) 6 : 3 : 2]
Xhus, degeneracy is nine (1 + 3 + 5 , 9 states). 10. The energy of second orbit of hydrogen is equal to the energy
i>:;:i?U'r1:~" ..." . of:
~;~;'?Jl:xample 35. Calculate the angular frequency of an
(a) fourth orbit of He + (b) fourth orbit ofLi 2+
electron occupying the second Bohr orbit ofHe + ion.
(c) second orbit ofHe+ (d) second orbit ofLi 2+
. ' 2nZe 2
Solution: Velocity of electron (v) = - - ... (i) [Ans. (a)]
nh Z2
2 2 [Hint: E= -2 x 13.6eV
.
RadIUS . .m an orb'It (In) =
0 fHe +Ion n h 2 ... (ii) n
4n 2 mZe E2 -- - 4 ~,or 'H'.
13,6
Angular frequency or angular velocity (00) ,
z2
v '21tZe 2 4n 2 mZe 2 Sn 3 mZ 2 e 4 .E = - x 13.6 eV
-=--x =-- --
rn nh n 2h2 n 3h 3 13.6 Z2
28 10 -2 x 13.6
Given, n == 2,m"" 9.1 x 10- g, Z== 2,e= 4.Sx 10- esu 4 n
Z2 1
h =:6.626 x 10-27 erg -sec (Z =1, n =. 2)]
4

00=
8x (2;r '
x 22 x 9.1 X 10~28 x (4.Sx 10-10 )4
'r-'- - -
11. What is the energy in eV required to excite the electron from
n =' lto n = 2 state in hydrogen atom? (n = principal quantum
number) teET (J&K) 20061
(2)3 x (6.626x 10-27 )3 (a) 13.6 (b) 3.4 (c) 17 (d) 10.2 '
[Ans. (d)]
[Hint: b.E = E2 EI

, 6. If the speed of electron in first Bohr orbit of hydrogen be 'x',


( - 13.6 J-( - 1~~6 J
then speed of the electron in second orbit ofHe+ is:
(a)x 12 (b) 2x (c) x (d) 4x 13.6 ( 1- ~J ! x 13.6 = 10.2 eVl
[Ans. (c)] 12. An electron in an atom undergoes transition in such a way that
[Hint: v =VI X Z = x x 2 = x] its kinetic energy changes from x to x , the change' in
II n ,2 4
7. If first'ionisation energy of hydrogen is E, then the ionisation potential energy will be :
energy of He+ would be: 3' 3 3 3
(a) + -x (b) -x (c) +-x (d)-x
(a) E (b) 2E (c) 0.5E (d) 4E 2 8 4 4
[Ans. (d)] [Ans. (a)]
ATOMIC STRUCTURE 87

[Hint: PE = - 2KE value of mass (m) increases, i. e. • in the case of heavier particles,
the wavelength is too small to be measured. de Broglie equation .
:. PE will change from - 2x to _ 2x is applicable in the case of smaller particles like electron and has
4
no significance for larger particles.
Change in potential energy (- ~ ) - (-2x)
(A) de Broglie wavelength associated with charged
particles
=_::+2x=3x]
2 2 (i) For electron:

,#,~,i~} PARTICLE AND WAVE NATURE OF


ELECTRON (ii) For proton:
In 1924, de Broglie proposed that an electron, like light, behaves
both as a material particle and as a wave. This proposal gave birth
I\=-rvA'.
~ O.286

to a new theory known as wave mechanical theory of matter. (iii) For a-particles:
According to this theory, the electrons, protons and even atoms,' A=0.10I A
when in motion, possess wave properties.
de Broglie derived an expression for calculating the
rv
wavelength ofthe wave associated with the electron. where, V =accelerating potential oftQe~~particles.
Accoram15to Planck's equation,
'-. '., · · · e (B) de Broglie wavelength associated with
E=hv h·,- ... (i) uncharged particles
:A
The energy of a photon on the basis of Einstein's mass- energy (i) For neutrons:
relationship is h 6.62 X 10-34
A=-- =-======
E = me2 .,. (ii) .J2Em ~2 x 1.67x 10-27 x E
where, e is the velocity of the electron.
Equllting both the equations, we get . = 0.286 A
e
h-=me
2' JE (eV) .
A ...
(ii) For gas molecules:
A =!!-. = h h
me p A=---
mxvnns
Momentum of the moving electron is inversely proportional to
h
its wavelength.
Let kinetic energy of the particle of mass 'm' is E. .J3mkT
I 2 where, k = Boltzmann constant
E=-mv
2 Bohr theory versus de Broglie equation: One· of the
2Em=m 2v
2 postulates of Bohr theory is that angular momentum of an electron
42Em = mv = p(momentum) is an integral multiple of~. This postulate can be derived with the
2n >

A=!!..= h help of de Broglie concept of wave nature of electron.


P :J2Em Consider an electron moving in a circular orbit around
Davisson and Germer made the following modification in de nucleus. The wave train would be associated with the circular
Broglie equation: orbit as shown in Fig. 2.15. If the two ends ofan electron wave
Let a charged particle, sayan electron be accelerated with a meet to give a regular series of crests and troughs, the' electron
potential of V; then the kinetic energy may be given as: wave is said to be in phase, i. e. ,the circumference of Bohr orbit is
1 2 equal to whole number multiple of the wavelength ofthe electron
- mv =eV wave.
2
m2v 2 = 2eVm
mv .J2eVm = p
h
A=--
. .J2eVm
A= ~ for charged particles of charge q
,,2qVm
de Broglie waves are not radiated into space, i. e., they are
always associated with electron. The wavelength decreases if the Fig. 2.15
88 G.A.B. PHYSICAL CHEMISTRY FOR COMPETITIONS

So, 21tr= n/.. In the case of bigger particles (having considerable mass), the
/.. 21tr value of uncertainty product is negligible. If the position is
or ... (i)
n known quite accurately, i. e. , Ilx is very small, Av becomes large
from de Broglie equation, and vice-versa. Thus, uncertainty principle is important only' in '
/..= h, the case of smaller moving particles like electrons.
... (ii) • for other canonical conjugates of motion, tIie equation for
mv.'
IT 21tr Heisenberg uncertainty principle may ~e given as:
Thus, -'=---=-- momentum = mass x velocity
mv n
. velocity .
h ==massx xbme
mvr = n . - (v =;: velocity of electron time
, 21t
and r=;: radii of the orbit) == force' x time
momentum x distance == force x distance x time
i. e., . '\.ngu1ar momentum = n . - h . .. ( III
... )
==energy J< time
21t
Ap Ilx == AE At
This proves tlut the de Broglie and Bohr concepts are in .' ·h·
perfect agreement with each other.· ' . AE At Co - (for energy and time)
41t
Similarly, A Ij) AS Co.!!:.... (for angular motion)
iil~ll;. ",HEISENBERG UNCj:RTAINTY 4~
PRINCIPLE . On the basis of this principle, therefore, Bohr picture of an
electron in an atom, which gives a fixed position in a fixed orbit
Bohr theory considers an electron asa material particle. Its
and definite velocity to an electron, is no longer tenable. The best
position and momentum can be determined with accuracy. But,
we can think of in terms of probability of locating an electron
when an electron is considered in the form of wave as suggested
with a probable velocity in a given region of space at a given
by de Broglie. it is not possible to asc~rtain simultaneously the
time. The space or it. three dimensional region round the nucleus
exact position and velocity of the electron more precisely at a
where there is maximum probability of finding an electron of a
given instant since the wave is extending throughout a region of
specific energy is called an atolUic orbital.'
space. To locate the electren, radiation with extremely short
wavelength is required. Radiation that has short wavelength is
very energetic in nature. When it strikes the electron, the Impact
: : ::::a_SOME SOLVED EXAMPLES\a::::::
causes a change in the velocity of the electron. Thus, the attempt Example 36. Calculate the wavelength associated with an
to locate the electron' changes ultimately the momentum of the electron moving with a velocity of10 10 cm per sec.
, electron. Photons, with longer wavelengths are less energetic and 28
Solution,: Mass of the electron = 9.10 x 10- g
cause less effect on the momentum of the electron. Because of
Velocity of electron == 1010 cm per sec
larger wavelength, such photons are not able to locate the
position of an electron precisely. h == 6.62 X 10-27 erg- sec
In 1927, Werner Heisenberg presented a principle known as According to de Btoglie equation,
Heisenberg uncertainty principle which states: "It is ,. h 6.62 X 10-27
/ . . = - = - - 28
----
impossible to measure simultaneously the exact position and . mv 9.l0x 10- x 1010 .
exact momentum of a body as smaU as an electron."
== 7.72x 10- 10 cm
The uncertainty of measurement of position, Ax and the
uncertainty of momentum, IIp or llmv are related by Heisenberg's 0.0772 A
relationship as: Example 37. Calculate. the uncertainty in.the position ofa
1lx·1:!.p Co hi 41t partieie when the uncertainty in momentum is:
or Ilx . llmv Co h I 41t (a) Ix 10-:-3 gcmsec- 1 (b) zero.
where, IT is Planck's constant. Solution: (a) Given, ..
28
for an electron of mass m(9.l0x 10- g), the product of AP= lxlO-3 gcmsec- 1
uncertainty is quite large~ h 6.62 x 1.0-27 erg- sec
6.626 x 10-27 1t == 3.142
Ilx ·llv Co - - ' - - - - -
.41tm According to uncertainty principle,
. h
6.626x 1O-2~ A,;r: .I:!.p Co -
Co - - - - - - - - - 41t
4x 3.14x9JOx . '27
So, Ilx Co .!!:.... • _1_ Co 6.62 x 10- x _I_
'" 0.57 erg- sec per gram approximately
. 41t Ap ··4 x 3.142 10-3 .
Ilx' Av = uncertainty product
=
When Ilx = 0, Av 00 and vice-versa. ' == 0.527 x 10-24 em
(b) When the viuue of Ap 0, the value of Ilxwill be infinity.
I
. ATOMIC STRUCTURE 89

'>', J1x.a~ple 38. .Calculate the momentum ofa p(1rtide which, Example 43. The kinetic energy of an electron is
has a de Brogiie wavelength of2.5 x 10- 10 m. 4.55 x 10-25 J. Calculate the wavelength, (h = 6.6 x 10-34 J-sec,
(h = 6.6 x lQ~34 kgm 2 s -I ) mass of electron = 9.1 x 10-31 kg).

Solution: MOnlentuhl =:! (using de Broglie equation) Solution: KE = ~ mv 2 = 4.55 x 10-25


A 2
6.6 x 10-34
or ~ x 9.1 X 10-31 x v 2 = 4.55x 10-25
2.5 X 10- 10 2
= 2.64 x 10- 24
kg m se'c-\ 2 2 x 4.55 X 10-25
or V =-----::-:--
9.1 X 10-31
Example 39. What is the mass of a photon ofsodium light
with a wavelength of5890"A'? v = 103 ms- I
(h = 6.63 x 10-27 erg - se~, c = 3 X 1010 cml sec) Applying de Broglie equation,
A= ~ ~
34
h 6.6x 10- = 0.72 x 10-6 m
Solution: A=-
mc mv 9.1 x 10-31 x 10 3
I h
or m=- Example 44. The speeds of the Fiat and Fe;rari racing
Ac cars are recorded to ± 4.5 x 10-4 m sec -I. Assuming the track
6.63 X 10-27
So, m = -------::------,-:- distance to be known within ± 16 m, is the uncertainty principle
5890 X 10-8 x 3 X 10 10 violatedfor a 3500kg car?
·=3.752xl0-33 g Solution: llx ft!.v = 4.5 X 10-4 X 16
' .. Example 40. The uncertainty in position and velocity of a =7.2 X 10-3 m 2 sec-I. . .. (i)
particle are 10- 10 m and 5.27x 10-24 ms- I respectively.
34
h 6.626 x 10-
Calculate the mass of the particle. (h = 6.625 x 10-34 J - s) ... (ii)
41tm4 x 3.14 x 3500
Solution: According to Heisenberg'S uncertainty principle,
h = 1.507 x 10-38
llx·mft!.v=-
41t Since, llx ft!.v ::2: hi 41tm
h Hence, Heisenberg uncertainty principle is not violated.
or m=----
41t llx· ft!.v Example 45. Alveoli are tiny sacs in the lungs whose
6.625 X 10-34 average diameter is 5 x 10-5 m. Consider an oxygen molecule
4 x 3.143 X 10- 10 x 5.27x 10-24 (5.3 x 10-26 kg) trapped within a sac. Calculate uncertainty in
= 0.099 kg the velocity of oxygen molecule.
Example 41. Calculate the uncertainty in velocity of a Solution: Uncertainty in position llx = Diameter of Alveoli
cricket ball ofmass 150 g ifthe uncertainty in its position is ofthe = 5x 10- 10 m
order ~fl A (h =6.6 x 10-34 kg m 2 s -I ). . h
llxft!.v::2:--
h 41tm
Solution: ft!.x·mft!.v=-
41t 6.626 X 10-34
ft!.v::2:-------:-----~
ft!.v=~_h__ 26 10
4x 3.14x 5.3 x 10- x 5x 10-
41t llx· m ft!.v"" 1.99 ml sec
6.6xlO-34
.6f-OBJEcTIVt· QUESTIONS'
4 x3.143 X W- IO x 0.150
= 3.499 X 10-24 ms- I 13. If the kinetic energy of an electron is increased 4 times, the
wavelength of the de Broglie wave associated with it would
. Example 42. Find the number of waves made by a Bohr
become:
electron in one complete revolution in the 3rdorbit. (liT 1994)
(a) 4 times (b) 2 times
Solution: VeloCity of the electron in 3rd orbit = ~
. 21tmr .) 1 .
(c - tImes (d) J.. times
where, m = mass of electron and r = radius of 3rd orbit. 2 4
_Applying de Broglie equation,
[Ans. (c)]
A = ~ = !!... x 21tmr = 21tr
mv m' 3h 3 . h
[Hint: A = - - where, E = kinetic energy
.' . 21tr 21tr- .J2Em'
No. of Waves == - := ~ x 3 = 3
A 21tr
90 G.R.B. PHYSICAL CHEMISTRY FOR COMPETITIONS

When, the kinetic energy of electron becomes 4 times, the ~.~ WAVE MECHANICAL MODEL OF ATOM
de Broglie wavelength will become half]
The mass of photon having wavelength 1 nm is: The atomic model which is based on the particle and wave nature
14.
of the electron is known as wave mechanical model of the
(a) 2.21 x 10-35 kg (b) 2.21 x 10-33 g
atom. This was developed by Erwin Schrodinger in 1926. This
(c) 2.21 X 10-33 kg (d) 2.21 x 10-26 kg model describes the electron as a three-dimensional wave in the
[ADS. (c)] electronic. field of positively charged nucleus. SchrOdinger
derived an equation which describes wave motion of an electron.
[Hint: A= h
me The differential equation is:
h 6.626 x 10-34 d2W d 2 W d 2W 8n 2m
m - = ---,:-----;:- +--+ +--(E V)W=O.
Ae 1 x 10-9 X 3 X 108 dy2 h2 ;
= 2.21 X 10-33 kg] where, x, y and z are cartesian coordinates of the electron;'
15. The de Broglie wavelength of 1 mg grain of sand blown by.a m = mass of the electron; E = total energy of the electron;
20 ms- I wind is: . V = potential energy of the electron; h = Planck's constant and
(a) 3.3 x 10-29 m (b) 3.3 x 10-21 m W(psi) = wave function of the. electron.
Significance of W: The wave function may be regarded as
(~) 3.3 x 10-49 m (d) 3.3 x 10--42 m
the amplitude function expressed in tenns of coordinates
[Ans. (a)]
34
x, yandz. The wave function may have positive or negative
[Hint: A =.!!..- = 6.626 x 10- = 3.313 x 10-29 m] values depending upon the values of coordinates.
mv 1 X 10-6 X 20 The main aim ofSchrOdinger equation is to give a solution for
16. In an atom, an electron is moving with a speed of 600 m sec-I the probability approach. When the equation is solved, it is
with an accuracy of 0.005%. Certainty with which the observed that for some regions of space the value of Wis positive
position of the electron can be located is: and for other regions the value of W is negative. But the
(h = 6. 6x 10-34 kgm2 s-I ,mas~ofelectron= 9.1 x 10-31 kg) probability must be always positive and cannot be negative. It is,
(AIEEE 2009) thUs, proper to use W2 in favour ofW.
(a) 1. 52x 10-4 m (b) 5.1x 10-3 m Significance of W1. : W2 is a probability factor. It describes
(c) 1.92x 10-3 m (d) 3.84 x 10-3 m the probability of finding an electron within a small space. The
[Ans. (c)] space in which' there is maximum probability of finding an
electron is termed as orbital.
[Hint: ACcuracy in velocity =0.005% .
The solution of the wave equation is beyond the scope of this
Av = 600 x 0.005= 0.03 book. The important point ofthe solution of this equation is that it
100 provides a set of numbers, called quantum numbers, which
According to Heisenberg's uncertainty principle, describe energies ofthe electrons in atoms, information about the
h shapes and orientations of the most probable distribution of
AxmAv;::'- electrons around the nucleus. .
411:
Wave function \If can be plotted against distance Or' from
6.6 X 10-34
Ax = -------:;::;--~- nucleus as,
4x3.14x9.1x x 0.03
1.92xI0-3 m] '"
<.:
17. Velocity of de Broglie wave is given by:
'If
e2 h 2
(a) -
V
(b) ~
me
(c) me
h
(d) VA t
[Ans. (b)]
.h h Node
[Hint: A
mv p
h
p=- 'If
A
mv=
hv t
e
v hv ]
me

For hydrogen wave function, number of nodes can be


calculated as, #
ATOMIC STRUCTURE 91.

(i) Number of radialnodes = (n -/-1)


(ii) Number of angular nodes = I 2s 2 (~J312 (2~ :: )e-ZrI2QO
(iii) Total number of nodes (n I)
(iv) Number of nodaLplanes =I
If the node at r = is also considered then no. of nodes will be
2p 2 _1 (zr )312 (zr) e-Zrl2ao
Note:
'n' (not n 1).
00
.J3 2a o ao
Examples: (i) For Js..orbital n =1, I 0, it will have no where, Z atomic number, ao = radius of flTSt Bohr orbit of
hydrogen.
radial or angular node.
(ii) For 2s-orbital, n 2, I O,it will have only one radial Plot of Radial Wave Function 'R' :
node.
18 28
(iii) For3s-orbital, n = 3, 1= 0, it will have two radial nodes. 2p

1~
(iv) For 2p-orbital, n 2, 1=1, it will have no radial node
but it has only one angular node.
(v ) For 3p-orbital, n 3, 1= l, it will have one radial and one
angular node.
For s-orbitals: ----+ r ----+

For p-orbitals:
°
(n - l) radial nodes + angular' node (n - 1) total nodes.
r
Number of radial nodes = (n -/- 1).
At node, the value of'R 'changes from positive to negative.
(n '2)radial nodes + I angular node = (n I) total nodes.
Plot of Radial Probability Density 'R 2,:
For d-orbitals:
(n - 3) radial nodes + 2 angular nodes (n - I) total nodes. 28 2p
d 2 like all d-orbitals has two angular nodes; The difference is
z .
t\ 1s . .,.
thanhe angular nodes are cones in a d 2 orbital, not planes.
z R2~
Operator form Schrodinger Wave Equation
H'P=E'P (Operatorfonn) r ----+ r ----+ r ----+
~ [ - -h- V ' 2 +v~]
2
The plots of probability, i.e., R2 or '1'2 are more meaningful
whereH= Hamiltonian operator
.
2
81.t m than the plots of functions themselves. It can be seen that for both
Is and2s orbitats, the probability has a maximum value at r= 0,
=1'+ V i.e., in the nucleus. In case of 2s orbital, one more maximum in
Here, t = Kinetic energy operator the probability plot is observed.
V Potential· energy operator
Plot of Radial Probability Function (41tr2R2):
Complete wave function can be given as
In order to visualize the electron cloud within a spherical shell
'I' (r,9,(j)= R(r) e(9)«I>«(j) is placed at radii 'r' and 'r + dY from the nucleus. Thus radial
'---v-' ~
Radial' part Angular part ,probability function describes the total probability qffinding the
Dependence of the wave function on quantum number can be electron in a spherical shell of thickness 'dr' located at the
given as, ' distance r from the nucleus.
'Pn1m (r, 9, (j) Rnl (r) elm (9)«1> m«(j) R.P.F.= (Volume of spherical shell) x Probability density
The function R depend only on 1'; therefore they describe the , . = (41tr 2 dr) x R2
distribution of the electron as a function of r from the nucleus.
These functions depend upon two quantum numbers, n and I. The 18 2s 2p
two functions e and <l> taken together . give the angular
distribution of the electron. N

The radial part ofthe wave function for some orbitals may be
given as, 1
n
312 In the plot of radial probability against 'r', number peaks, i.e.,
o 2 ~
Is
( ) e-Zrlao
region of maximum probability n - I.
92 G.R'.B. PHYSiCAL CHEMISTRY FOR COMPETITIONS

[Hint: For Is orbital, probability of finding the electron at the


fUlJmATfONf OF:OIl)ECTIVEQ.UESTIONS nucleus is zero.]
22. The radial wave equation for hydrogen atom is :
18.
,\f = , Ir:; (~}312 [(x l)(f 8x + 12)] e:'" .t12
t
Radial
~ov4 ao
where;, x = 2r / ao; ao = radius of first Bohr orbit.
probability . The minimum and maximum position of radial nodes from
density: :
I . Y:
,
Distance from n~cleus -+
nucleus are :
ao
(c)-,a
a
(d)~,4ao
.
o
If the above radial probability curve indicates '2s' orbital, the 2 2'
distance between the peak points X, Y is : [Ans.(b)]
(a) 2.07A '(b) l.59A (c) 0.53A (d) 2.12A [Hint: At radial node, \f 0
[Ans. (a)] ... From given equation,
[Hint: X = 0.53A, Y = 2.6A , x-l=Oandx2 -8x+12=0
Y-X=,2.6-0.53 2.07Al x 1=0 =) x 1
19. Plots for 2s orbital are :
i.e. , ~ =1; r = i' (Minimum)

x 2_ 8x + 12 = 0
(x 6)(x - 2) =0
when x-2=0
x=2
r--+ 2r = 2, i.~.,r =~ (Middle value)
llo
X, Y and Z are respectively
(a)R,R 2 and4n?R2 (b) R2, Rand 41t?R2, when x-6=O
2 2 x ='6'
(c)4n?R ,R andR (d) R2, 4n?R2 and R
2r =6
[Ans. (b)]
llo
[Hint: Y will be definitely 'R' because value of R ,cannot be
r = 31lo (Maximum)]
negative, thus X will be R2 and Z will be 41t?R. Z represents
radial probability function; its value will be zero at origin]
2~16 QUANTUM NUMBERS
20. The wave function CI') of 2s is given by :

\f2s 2lrt(:J f

2 {2~ ~} e~rl2ao "


As we know, to search a particular person in this world, four
things are needed:
(i) The country to which the person belongs
At r =ro, radial node is formed. Thus for 2s, 'iJ in terms of ao (ii) The city in that country to which the person belongs
is : (iii) The street in that city where the person is residing
, (a) 'iJ = ao (b) 'iJ = 2ao (c) 'iJ = ao /2 (d) 'iJ = 4ao (iv) The house number
[Ans. (b)]
Similarly, four identification numbers are required to locate a
[Hint: When r = ro, 'liz. = 0, then from the given equation: particular electron in an atom. These identification numbers are
2- r =0 called quantum numbers. The four quantum numbers are
Go discussed below..
r= 21lo]
21. The wave func!ion for Is orbital of hydrogen atom is given Principal Quantum Number
by: It was given by Bohr; it is denoted by 'n'. It represents the
1t -rfao name, size and energy ofthe shell to which the electron belongs.
tTl
Tis -
....
.J2 e The value of 'n' lies between I to "". '
where, ao = Radius of first Bohr orbit n = 1,2,3,4, ... 00
r =Distance from the nucleus (probability of finding Value of n 1• 2 3 4 5 6 7
,the electron varies with respect to it) Designation of shell K, L M N 0 P Q
What will be the ratio of probabilities offindiilg the electron~
at the nucleus to first Bohr's orbit ao ? (i) Higher is the vahie of' n', greater is the distance·, of the
shell from the nucleus.
(a) e (b) e 2 (c)1/ e 2 (d) zero
[Ans.(d)] Ii <"2 < r3< r4 < rs < ...
ATOMIC STRUCTURE 93
n2 : s- subshell -t 2 electrons d-subsheH -t 10 electrons
r=-xO.529A
Z . p-subshell-t 6 electrons f-subshell -t 14 electrons
(it) Higher is the value of 'n', greater is the magnitude of g-subshell -t 18 electrons
energy. .
Magnetic Quantum Number
El <E2 <E3 <E4 <E5 ...
Z2 This quantum number is designated by the symbol 'm'. To
E x.21.69x 10-19 JI atom explain splitting of a~gle spectral line into a number of closely
spaced lines in the presence of magnetic field (Zeeman effect),
Z2 Linde proposed that eleotron producing a single line has several
= - -2 X 313.3 kcal per mol
n possible space orientations for th~ same angular momentum
Energy separation between two shells decreases on moving . vector in a magnetic field, i. e. , under tbe influence of magnetic
away from nucleus. field each subshell,is further sub-divided into orbitals. Magnetic
(E2 -Ed>(E3 -£2»(E4 -E 3 »(E5 -E4) quantum number describes the orientation or distribution of
electron cloud. For each value of 'I', the magnetic quantum
(iii) Maximum number of electrons in a shell* = 2n 2 number 'm' may assume all integral values from -1 to + I
(iv) Angular momentum can also be calculated using including zero, i. e. , total (21 + 1) values.
principal quantum number
nh
°
Thus, when I 0, m = (only one value)
when I = I, m = -1, 0, + I (three values)
mvr=-
2n i. e. , three orientations.
One orientation corresponds to one orbital. Three orientations
Azimuthal Quantum Number
(orbitals) are designated as Px' pyand pz.
It was given by Sommerfeld; It IS also called angular When I 2,m -.2,-1,0,+ 1,+2 (five values), i.e., five
quantum number, subsidbuy quantum number orseeondary orientations.
r;
quantum number. It is denoted by' its value lies between 0, I, The five orbitals are designated as:
2. ' .. (n -I),
dxy' dyz , dzx, dx 2 - Y 2 and d z 2'
It describes the spatial distribution of electron cloud and
CL'1gular momentum. It gives the name of the subshell associated' When J= 3, m 3, +
1,0, I, + 2, + 3 (seven values),i.e. ,
with the main shell seven orientations.
I", 0 s-subshell; / I p-subshell; Different values of 'm ' for a given value of'/' provide the total
I ~ 2 d-subshell; l == 3 f-subshell; number of ways in which a given s, p, d and f subshells in
presence of magnetic field can be arranged in space along
I 4 g-subshell. ..
s',P.' d, f and g are spectral terms and signify sharp. principal.
x, yand z axes or total number of orbitals into which a given
diffused. fUndamental and generalised respectively. subshell can be divided.
The energies of the various subshells in the same shell are in When I :: 0, m = 0, i. e. , one value implies that's' subshell has
the order of s < P < d < f < g (increasing order), Subsh~lls only one space orientation and hencYl> it can be arranged in space
having equal I values but with different n values have similar only in one way along x, y or z axes. Thus, 's' orbital has a
shapes but their sizes increase as the value of 'n' increases. symmetrical spherical shap~,and is usually represented as in Fig.
2s-subshell is greater in size than Is-subshelL Similarly 2.16.
2p, 3p,4p-subshells have similar shapes but their sizes increase In case' of Is-orbital, the electron cloud is maximum at the
in the order 2p < 3p < 4p. . nucleus and decreases with the distance. The electron density at ~
Orbital angular momentum of an electron is calculated using particular distance is uniform in all directions. The region of
the expression maximum electron density is called antinode. In case of
'2s'-orbital, the electron density is again maximum at the nucleus
I-tl ~/(l+1)~=~/(/+1)1i and decreases with increase in distance. The '2s'-orbital differs in
2n
detail from a Is-orbital. The electron in a '2s'-orbital is likely to be
h
here, 1'1 " ' - found in tWo regions, one near the nucleus and other in a
2n spherical shell about the nucleus. Electron density is zero in
The magnitude of magnetic moment I-tL may be given as: nodal region.
. I-tL ~/(l+ I) BM .
where, BM == Bohr Magneton

1 BM=~=9.2732x 10-14 J
41tmc
Maximum electrons present in a subshell = 2(21 + I)
1s 2s-orbital

*No energy' shell in atoms of known elements possesses mor" than 32 electrons.
94 I G.R.B. PHYSICAL CHEMISTRY FOR COMPETITIONS

z z

Fig. 2.16
When 1= 1, 'm' has three values -1,0, +1. It implies that 'p' dxy dyz d;x .dx2 d;
Orbital i
subshell of any energy shell has three space orientations, i. e. , m ±2 ±1 ±1 ±2 0
three orbitals. Each p-orbital has du-mb-bell shape. Each one is
disposed symmetrically along one of the three axes as shown in Nodal planes:
Fig. 2.16. p-orbitals have directional character. Orbital Nodal planes
Orbital P z Px Py d X)' xz,yz
m
Nodal plane xy
° ±l
yz
±1
zx
d·yz
dzx
XY,zx
xy,yz
When 1= 2,' m' has five values -2, ...:.1, 0, + 1, + 2. It implies d x2 :.. y2 x-y O,x+ y=O
that d-subshell of any energy shell has five orientations, i. e., five dz2 No nodal plane, it has
orbitals. All the five orbitals are not identical in shape. Four. of a ring around the lobe
the d-orbitals dX)'· ,dyz, d zx ,dx 2 - y 2 contain four lobes while
There are seven I-orbitals designated as I 2 2' I y(x 2 y2)'
fifth orbital d z2 consists of only two lobes. The lobes of d X)' • x(x - y)

orbital lie between x and y-axes. Similar is the case for I z(x 2 - y 2) ' Ixyz ,Iz 3, f yz 2 and 1._2'
x< Their shapes are complicated
d yz and d zx' Four lobes of d x2 _ 2 orbital are lying along x and y-
axes while the two lobes of d 2 ~rbital are lying along z-axis and ones.
z
contain a ring of negative charge surrouliQing the nucleus in Positive values of ml describes the orbital angular momentum
xy-plane (fig. 2.16). component in the direction of applied magnetic field while the
negative values of mlare for the components in opposite
direction to the applied magnetic field.
ATOMIC STRUCTURE 95
Degenerate Orbitals
Orbitals which are located at the same energy level on the energy
level diagram are called degenerate orbitals. Thus, electrons have
equal probability to occupy any of the degenerate orbitals.
Px' Py and pz--,------t 3-fold degenerate
d-orbitals --,------t 5-fold degenerate
f-orbitals --,------t 7-fold degenerate
Degeneracy of p-orbitals remains unaffected in presence of
external uniform magnetic field but degeneracy of d and
-3 f-orbitals is affected by external magnetic field.
Fig. 21.6 (a) Space quantization in magnetic field Spin Quantum Number
It is denoted by's' and it was given by Goldschmidt.
Characteristics of Orbitals
Spin quantum number represents the direction of electron spin
(i) All orbitals of the same shell in the absence of magnetic around its own axis_
field possess same energy, i. e. , they are degenerate. (i) For clockwis~ spin,s = + Ji (i arrow representation).
(ii) All orbitals of the same subshell differ in the direction of (ii) For anticlockwise spin, s = - Ji (J- arrow representation).
their space orientation. Spin electron produces angular momentum equal to I-l s given by
, h
(iii) Total number of orbitals in amain energy shell is equal to I-l s = ~s( s + 1) - , where, s = + Ji
n 2 (but not more than 16 in any of the main shells of the known 2n
elements). Total spin of an atom = n x Ji. (n = number of unpaired
n = 1 No. oJ orbitals = (1)2 = 1(ls) electrons)
n = 2 No. of orbitals = (2)2 = 4 (2<>, 2px, 2py, 2pz ) Spin magnetic moment (!-.I. s ) is given by

n = 3 No. of orbitals = (3)2 == 9 (3s, 3Px' 3p y' 3p z , 3d XY' I-ls =~s(s+ 1)~
2nmc
3d yz , 3d zx , 3dx2 _ y2 , 3d z2 )
Each orbital can accommodate two electrons with opposite
n=4 No. of orbitals = (4)2 =16 spin or spin paired; paired electrons cancel the magnetic moment
and develop mutual magnetic attraction as shown in the
The division of main shells into subshells and that of subshell
following Fig. 2.17.
into orbitals has been shown below:
N S
Note: Magnetic quantwn nwnber also represents quantized value of
.. . . MagnetiC field .....,.. .
z-component of angular momeritwn of the electron in an orbital
through the expression
/-
....,,~.- ....,
--, /
... ",--_ .....
--
,/./.................
.
.... "-- .... , .... ,,--~,
............... -- -- , \ /
.

Lz=mC:) /
['
I
. /":'

[
..... :\ I "'. . ','
II
\

I \
/
['
I
/"

[
, ~ I .,.. " "
II
I \
I I \. I I I \ 1
If El is the angle between z-axis and angular momentum vector, 1 1 1 1+1/2 -1121 1 1 1
\ 1 I I \ 1 I [
Lz=L cos El I I
II
,~ // " II,
[
'[
[ I I
II
" ...,"
[
'[
[

jI,
m (~) = ~l(l + I) ~ ." .... _- / , --" jI ~ .... -- / , --" ,t
cos El ..... _-,,., '---~ ' ..... _-,,., '---;/
2n 21t
or m =~l(l + I) cos El
Main shell Subs hells Orbitals S N
1st·shell ___ _
(K-shell) 'n = 1
Fig. 2.17
15 (/= 0) 15(m=0)

2nd-shell -------< "",,- 25 (/ = 0) 25(m=0) Electrons having same spin are called spin parallel and those
(L-shell) n=2 ...............
'---
2p(/=1)
..,,""~O)
-- ------
............ ,2py(m=±1)
having opposite spin are called spin paired.
Spin paired ~ II]]
2Px(m=± 1)
---------------
35(m= 0)
Spin parallel ~ [fJ]
3rd-shell Spin multipliCity: .
(M-shell)
Spin multiplicity = [2:U + 1]
where; s = spin quantum number
e.g., carbon ls2 2<>2 2p2

Normal state [!IJ [!IJ IL--i----'--I_i-----'-_---'


Excited state '[!IJ [IJ IL--i----JIL--i----JL--i----J
96 G.R.B. PHYSICAL CHEMISTRY FOR COMPETITIONS
. )

Spin multiplicity == 2[ ( +~) 5+(-~) J+ I . ~ PriDeipal Azimuthal Magnetie


Q.No. Q.No. Q.No.
Spin
Q.No.
No. of No. of
electrons electrons
On' om' Os' ona ona
'I'
subshell mainsheQ
2~it7 PAULI'S EXCLUSION PRINCIPLE
Each electron in an atom is designated by a set of four quantum .4 O(s) 0 +~, - Yz 2
numbers. In 1925, Pauli proposed that no two electrons in an. 1(P) 6
atom r;an have same values ofall the four quantum numbers. 2(d) 10
An orbital accommodates two electrons with opposite spin; :"'3 +Yz, -;j
these two electrons have same values of principal, azimuthal· and
magnetic quantum number but the fourth, i. e., spin quantum
-2 +Yz2' -Yz2
number will be different. -I +Yz,- Yz 32
This principle, can be illustrated by taking example of . 3 (f) 0 +Yz,-Yz 14
nitrogen. +1 + Yz2' - /2
1/

N7 1s2' 2s2 2i +2 +Yz,-Yz


+3 +Yz,-Yz
1s2 2s2 2p; 2p.:. 2p~
Conclusions:
[ill;@]; (i) The maximum capacity ofa main energy shell is equal to
Principal quantum number (n) 2 2 2 2 .2n 2 electrons.
.I (ii) The maximum capacity Of a subshell is equal to 2(2/ + 1) .
Azimuthal quantum number (l) 0 0
electrons.
Magnetic quantum number·(m) 0 0 +1 -I 0 Azimuthal Maximum eapadty of
Subenergy sbell .
Spin quantum number (s) +Yz - Yz;+ Yz - Yz; + Yz +Yz + Yz Q. 'I' eleetrons 2(U + 1)
.s 0 2(2 x 0 + 1)= 2
Out of seven electrons no two have same values of all four
quantum numbers. With the help of this principle, it is possible to p I 2(2 x I + I) =6
calculate the maximUm number of electrons which can be d 2 2(2 x 2 + l) = lO
accommodated on main energy shells and subshells. [ 3 2(2x3+ 1)=14

No. of No. of (iii) Number ofsubshells in a main energy shell is equal to the
Prindplll Aziinutfud Magn~1!' SpiD value ofn. .
eleetrons eleetrons
Q.No. Q. 1\19. Q. No. Q.No.
ona ODa . No. of subenergy
on'· .'1' 'mf '!I' Valueofn Desiguted as
.ubsbeR maiD .beR .beIII
O(s) 0 +Yz,-Yz 2 2 I I Is
2 2 2s,2p
2 O(s) 0 +Yz,-Yz 2 3 3 3s; 3p, 3d
1(P) -I 8· 4 4 4s, 4p, 4d, 4[
+)-;'-)1, \
0 +Yz,-Yz 6 (iv) Number of orbitals in a,main energy shell is equal to n 2 .
+1 +Yz,-Yz n No. of orbitalt
== I s
3 O(s) . 0 +Yz,-Yz 2
-I
2 (2i =4 s, Px; Py' p=
+)1,,-)1, )
+~"-Yz ..
3
1(P) 0 6
(v) . One orbital cannot have more than two electrons. If two
+1 '+Yz,-Yz electrons are present, their spins should be in opposite
-2 +Yz,-Yz 18 directions. .
-I +Yz,-Yz IUlI$IMfONS ·01 iOBJlCTN£QUUOOJt5
2(d) 0 .+ Yz, - Yz lO
23. The orbital angular momentum of an electron in a d-orbital is:
+1 +Yz,-Yz (DeE 2007)
+2 +Yz,-Yz (a).J6~ (b).fi h (c) h . (d) 2h
2n 2n 2n 2n
[Ans. (a)]
ATOMIC STRUCTURE 97

[Hint: Orbital angular momentwn =,//(1 + I) -


·~h ~L.LI AUFBAU PRINCIPLE
21C
~h h Aufbau is a German word meaning 'building up'. This gives us a
== ,/2(2 + 1)- "".J6 sequence in which various subshells are filled up depending on
21C 21C
(Here, 1 2, for d-orbitals)] the relative order of the energy of the subshells. The subshell
24. Which of the following sets of quantum numbers is correct for with minimum energy is filled up first and when this obtains
an electron in 4 f-orbital? maximum quota of electrons, then the next suhshell of higher
(Jamia Millia lslamia Engg. Ent. 2007) energy starts filling.
(a) n =4, 1= 3, m =+ 4, s = + ~ The sequen~e in which. the various subshells are filled is the.
(b)n=4,1 4,m -4,s=- ~ following: .:
(c) n::: 4, I = 3, m = + 1, s =+ ~.
1== 0 1= 1 1== 2 (= 3
(d) n =3, I = 2, m::; - 2, s::; + ~
[Ans. (c)]
[Hiot:For4f, n = 4,1 = 3, m::; - 2, 1, 0, + 1, + 2, + 3 n=1
s=- Xor+ Xl
n=2
25. Match the List-I with List~II and select the correct set from
the following sets given below: .
List-I· List-U
(A) The number of sub-energy levels in an (I) n2
n=4
energy level . .
(B) The number of orbitals in a sub-energy (2) 3d n=5
level
(C) The number of orbitals in an energy I~vel (3) 21 + I .n==6
(D) n:;; 3, I::; 2, m = 0 (4) n
n=7
[PET (Raj.) 2005)
Sets (A) (B) (C) (0)
(a) 4 3 1 2 n::8
(b) 3 I 2 4
(c) 1 2 3 4
(d) 3 4 I 2
[Ans. (a)]
[Hiot: Number of orbitals in a shell = n2 Fig.2.18 Order of filling of various subshells

Number of subshells in a .shell = n Is,2s,2p,3s,3p,48,3d, 4p,5s,4d, 5p,6s,4f,5d ,6p, 78, 5f,6d, 7p.
Number of orbitals in a subshell = (21 + 1) The sequence in which various subshells are filled up can also
n = 3, 1 = 2, m = orepresents 3d] be determined with the help of(n + I )value for a given subshell.
The subsheU with lowest (n + I ) value is filled up tirs~.WheD
26. Which of the following is not possible?
[BCECE (Medical) 2007) two or more subshells have same (n + I) value, the.subshell
(a) n = 2, I = I, m = 0 (b) n ::; 2, I::; 0, m::; -I with lowest value of 'n' is fdled up first.
(c) n = 3, I ::; 0, m = 0 (d) n 3, l:::i: I, m:::-I
[Aos. (b)] SubsheU If 1 (If + I)
[Hint: When I = 0, 'm' will also be equal to zero.] Is I 0 1
2s 2 0 2
27. What is the maximum number of electrons in an atom that can
have the quantum numbers n ::; 4, me + 1?
[PMT (Kerala) 2007)
2p
3s
2
3
.I
0
3
3
} Lowest value of n

(a) 4
[ADs. (e)l
(b) 15 (c) 3 (d) 1 (e) 6 3p
48
3
4
'~J'
," .
I
0
4
4
} Lowest value of n

[Hint: n = 4; I 0; me =0 3d 3 2 5
1= 1; me = 1,0, + 1 '"',
1=2; m" = - 2, -1, 0, + I, + 2
4p
58
4
5
1
0
5
5 J Lowest value of n

./ 3; me ::; - 3, - 2, -1, 0, + I, + 2, + 3
There are three orbitals having me = + I, thus maximum numbe~
of electrons in them will be 6.]
98 G.R.B. PHYSIC,AL G~.E;MIl:?TRYFOR COMPETITIONS

Subshell n· ::ud; p x , Py and p z • All the three have sarnt: energy. The electron can
be accomniodated on anyone of thP. 2p-orbitals.fu the case of
~a1u~"of n
4d 4 2
carbon, sixth electron is also accommodated on 2p subshell and
5p 5 : ) . Lowest
its electronic configuration is represented as 1s 2 2s 2 2p2 but three
6s 6 0 ·6
orbital diagrams can be expected ..

~.
4f 4 3
, iJ,ii' .
~
5d
6p
5
6
2
7 1.' Lowest value of n . (I) [He] -
.
- - - ' Electrons are present on two
2s .L 2p .J different orbitals with parallel spins.
7s 7 0 7
, 5f 5 3 8 (ii) [He]i J, 1. -±-- Electrons are.present on two
2s .L 2p.J d'Ifferent
. orb'Ita. Is WI'th opposite
' spms.
.
6d. 6 2 8 ) Low", "lu, of n ,
7 8 (iii) [He] i t i J, _ _ Both the electrons are present on
. The energy of electron in a hydrogen atom and other single L2p.J . one orb'ItaI Wit. h' OPP9SI~
. spms.
. .
electron species like He + , Li 2+ and Be 3+ is determined solely by Experiments show that (i) orbital diagram is correct while (ii)'
the principal quantum number. The energy of orbitals in and (iii) are not correct. This has given birth to a new rule known
hydrogen and hydrogen like species increases as follows: as Hund's rule of maximum multiplicity. It states that electrons
1s< 2s= 2p < 3s= 3p = 3d < 4s= 4p= 4d = 4/ < ... are distributed among the orbitals of a subshell in such..a way·
The complete electronic' configuration of all the known as to give the maximum nuniber of unpaired electrons witll
elements have been given in the table on next page. It is observed parallel spins. Thus, the orbitals available in a subsheH are flrst
that few of the elements possess slightly different electronic filled singly before they begin to pair. This means that pairing of
configurations than expected on the basis of Autbau Prlnelple. electrons occurs with the introduction of .second electron in
These elements have been marked with asterisk (*) sign. s-orbitals, the fourth electron in p-orbitals, sixth electron in
:?2~1:.·; HUND'S RULE OF MAXIMUM d-orbitals and eighth electron in / -orbitals. The orbital diagrams of
nitrogen, oxygen, fluorine and neon are as given below:
. MULTIPLICITY (Orbital Diagrams)
Nitrogen (7) [He] 1t 1 i i
There is one more method of representing the electronic. config-
,. uration which is usually called as orbital diagra.~. In this method,
the' electron is shown by an arrow: upward direction i (clockwise . Oxygen (8) [He] i J, iti"i
spin) and downward direction t (anti-clockwise spin). ~L2p.J
To indicate the distribution of electrons among the orbitals of Fluorine (9) [He] itiJ,iJ,i
an atom, arrows are placed over bars that symbolise orbitals. 2s

Hydrogen, for exam~le, is represented as f. The next element


Neon. (10) [He] iJ,iJ,i,J,lt
~L2[JT
. atomtc
With " number 2'IS· he I'lum. It 'IS represente d as i J, , I,, e" . The orbital diagrams of elements from atOinic number 21 to 30
. . . 1s can be represented on similar lines as below:
both the electrons are present on the same orbital1s and are paired
(spins are in opposite directions). The next two. elements are Li Sc [Ar] 3d 1 4s2 [Ar] I IJ,
an:d Be with three and four electron~, respectively. These are
. represented by orbital diagrams as: Ti [ArJ3d 2 4s 2 1 1 1'-l-
i J, i V [Ar] 3d 3 4s 2 1 1 1 it
Li
1s2s
i J, i t Cr [Ar] 3d 5 4S1 i i i i i 1
Be --
1s2s Mn [Ar] 3d 5 4s2 I 1 1 1 'I tJ,
>:
These can also be written as: -
, i Fe . [Ar] 3d 6 4i IJ, 1 i 1 1 'tJ,
Li [He]-
2s Co [At-] 3.d 7 4s 2 IJ, tJ, 1 i 1 tJ,
Be [He] i J,
2s Ni [Ar) 3d 8 4s 2 IJ, IJ, IJ, 1 1 iJ,
-
In Be, 2Y-orbital has been completed. The fifth electron in the 1 J," . H
Cu· [Ar] 3d 10 4s1 IJ, tJ, H t
case of boron enters the next available subshell whiCh is 2p. Thus,
the electronic configuration of boron is 1s 2 2s 2 2pl . In the orbital Zn [Ar) 3d 10 4s2 It H i1 H tJ, tJ,
diagram [He] i J, - ~- , the 2p subshell has three orbitals 4s
. 2s 2p
ATOMIC STRUCTURE 99
" . ELEC1lRONIC,C,ONFIGURAnONOFELEMENTS
!
, Element At,·No. ls 2s 2p; .3s 3p ,3d ~: 4d ,
"'4p 4f ·S.s5pStlSJ. - - -
H
I
I I !
He 2 2 (Is completed)
Li
Be
B
3
4
5
2
2
2
I 1
2
2 I
1 . (2scompleted)

C 6 2 2 2
N 7 2 2 3 i
i
0 8 2 .2 4
F 9 2 2 5
i i
Ne 10 2 2 6 (2 p completed)
!
Na U 2 2 6 I
Mg i 12 2
i
2 .6 2
i
. (38 completed)
Al 13 2 2 6 2 1
Si 14 2
P 15 2
2
2
6
6
2
2
2
3
I ,-

i I I
S 16 2 2 6 2 4
CI 17 2 2 6 2 5
AT i i
18 l 2 6 2 6 (3 P completed)
K 19 2 2 6 2 6 1
Ca 20· 2 2 i 6 2 6 2 (48 completed)
Sc 21 2 2 6 2 6 1 2
Ti 22 2 2 6 2 6 2 2
V 23 2 2 6 2 6 3 2
*Cr 24 2 2 6 2 6 5 1
Mn 25 2 2 6 2 6 5 2
Fe 26 2 2 6 2 6 6 2
Co 27 2 2 6 2 6 7 2
Ni 28 2 2 6 2 6 8 2
*Cu 29 2 2 6 2 6 10 1
Zn £10 2 2 6 2 6 10 2 :(3d completed)
Ga 31 2 2 6 2 6 10 2 1
Ge 32 2 2 6 2 6 10 2 2
.As 33 2 2 6 2 6 10 2 3
Se 34 2 2 6 2 6 10 2 4
Br 35 2 2 6 2 6 10 2 5
Kr 36 I 2 2. 6 2 6 10, 2 6 ! (4p completed)
Rb 37
i
2 2 6 2 6 10 2 6 I t
Sr 38 2 2 6 2' 6 10 2 6 2 (5scompleted)
y 39 2 2 6 2 6 10 2 6 1 2
Zr 40 2 2 6 .2 .6 10 2 6 2 2
*Nb 41 2 2 .6 2 6 10 2 (; 4 I
·*Mo 42 2 '2 6 2 6 10 :2 6 5 ' 1
Tc 43 2 2 6 2 6 10 . 2 6 5 2
*Ru 44 2 2 6 2 6 10 2 6 7 1
*Rh 45 2 2 6 '2 6 10 2 6 8 1
i !
·Pd i 46 2 2 .6 2 6 10 2 6 10
*Ag 47 2 2 6 2 .6 10 2 6 10 1
Cd 48 2 i
i 2 6 2 6 10 2 6 10 2 (4d completed)
In 49 2 2 6 2 6 10 2 6 10 2 1
Sn 50 2 2 6 2 6 10 2 6 10 2 2
Sb 51 2 .2 6 2 6 10 2 6 10 2 3
Te 52 .2 2 6 2 6 10 2 6 I 10 2 4
I 53 2 :.t. 2 2
I 6 6 10 2 6 10 2 5
I ,
,I I I
Xe 2 Cf
!
54 I ""'""~~
2 6 2 6
- 10 I
- 2 6 10 2 6 (5p completed)
100 G.R.B. PHYSICAL CHEMISTRY FOR COMPETITIONS

ELECTRONIC CONFIGURATION OF ELEMENTS

m_.tl·~N~ I I K
L M 46 4.P- 4d ,,4/ 56 5p 5d Sf I 6.¥ 6p16d16f 16
Cs 8 18 2 6 6

ffi-f1
I
Ba 8 18 2 6 6 2 (68 completed)
'"La 57 8 18 2 6 6 I 2
'"ee 58 2 8 18 2 6 6 I 2
Pr 59 2 8 18 2 6 10 3 2 6 2
Nd 60 2 8 18 2 6 10 4 2 6 2
'"Pm 61 2 8 18 2 6 10 5 2 6, ' 2
8m 62 2 8 18 2 6 10 6 2 6 2
Eu 63 2 8 18 2 6 10 7 2 6 2
'"Gd 64 2 ,8 18 2 6 10 7 2 6 I 2
Tb 65 2 8 18 2 6 10 9 2, 6 2
Dy 66 2 8 18 2 6 10 10 2 6 2
Ho ,67 2 11 18 2 6 , 10 '11 2 6 2
Er 68 2 8 18 2 6 10 12 2 6 2
Tm 69 2 8 18 2 6 10 ' 13 2 6 :2
Vb 70 2" 8 18 2 6, t6;' 14 2 6 2
Lu 71 2 8 18 2 ,.,6 10 14 2 6 I 2 (4/ completed)
Hf 72 2 8 18 32 6 2 . 2
Ta 73 2 8 18 32 6 3 2
W 74 2 8 /
'i8 32 6 4 2
Re 75 2 $ 18 32 2 6 5 2
Os 76 2 " 8 18 32 2 6 6 2
Ir 77 2 8 18 32 2 6 7 2
'"Pt 78 2.. 8 18 32 2 6 9 1
'"Au 79 2 8 18 32 2 6 , 10, J
Hg 80 2 8 18 32 2 6 10 2 (5d completed)
Tl 81 2 8 18 32 2 6 10 2 1"
Pb 82 2, 8 18 32 2 6 10 2 2
Bi 83 2 8 18 32 2 6 10 2 3
Po 84 2 8 18 32 2 6 10 2 4
At 85 2 8 18 32 2 6 10 2 5
Rn 86 2 8 18 32 2 6 10 2 6(6p completed)
Fr
Ra
'"Ac
·Th
·Pa
PH+
90
91
2
2
8
8
8
8
8
18
18
18
18
18
32
32
32
32
32
2
2

;
2
6
6
6
6
6
10
10
10
10
10
0
2
2

;
2
6

I
2
I
1
2 (7scompleted)
2
2
2
'"U 92 2 ~ 18 32 2 6 10 3 2 6 I 2
·Np 93 2 8 18 32 2 6 10 4 2 6 1 2
Pu 94 2 8 18 32 2 6 10 6 2 6 2
Am 95 2 8 18 32 2 6 10 7 2 6 2
·Cm 96 2 8 18 32 2 6 10 7 2 6 I 2
*Bk 97 2 8 18 32 2 6 10 8 2 6 I 2
Cf 98 2 8 18 32 2 6 10 10 2 6 2
Es 99 2 8 18 32 2, 6 10 11' 2 '6 2
Fm 100 2 8 18 32 2 6 10 12 2 6 2
Md 101 2 8 18 32 2 6 10 B ,2 6 2
No 102 2 8 18 32 2 6 10 14 2 6 2
*Lr 103 2 8 18 32 2 6 10 14 2 6 I 2 (5/ completed)
KuorRf 104 2 8 18 32 2 6 10 14 2 6 2 2
HaorDb. 105' 2 8 18 32 2 6 10 14 2 6 3 2
Sg 106 2 8 18 32 2 6 10 14 2 6 4 2 I!!
Dh 107 2 ,8 18 i,~·g
32 2 6 10 14 2 6 5 2 ,ggi
Hs 108 2 8 18 32 2 6 10 14 2 6 6 2 !Jtj
Mt 109 2 8 18 32 2 6 10 14 2 6 7 2 "8
*UunorDs 110 2 8 18 32 2 6 10 14 2 6 9 I
*Uuuor Rg III 2 8 18 32 2 6 10 14 2 6 10 1
Uub 112 2 8 18 32 2 6 10 14 2 6 10 2 (6d completed)
I
.\

ATOMIC STRUCTURE 101


I

All those atoms which consist 'of at least one of the orbitals
singly occupied behave as paramagnetic materials because these

:::=
::[II- I]
hi
o
.. A
are weakly attracted to a magnetic field, while aU those atoms in
which all the orbitals are .doubly occupied behave as . where, Vo and Ao are threshold frequency and threshold'
diamagnetic materials because they have no attraction for wavelength respectively.
magnetic field. However, these are slightly repelled by magnetic Stopping potential: The minimum potential at which the
field due to induction. plate photoelectric. current becomes zero is called stopping
Magnetic moment may be calculated as, potential.
IfVo is the stopping potential, then
11=~n(n+2)BM eVo = h(v- v o )
eh
I BM (Bohr Magneton) -- Laws of Photoelectric Effect
.•. 4nmc·
where, n =no. of unpaired electron (i) Rate of emission of photoelectrons from a metal surface
is directly proportional to the intensity of incident light.
Exceptions to Aufbau Principle (ii) The maximum kinetic energy of photoelectrons is
In some cases, it is seen that actual electronic arrangement is directly proportional to the frequency of incident
slightly different from arrangement given by aufbau principle. A radiation; moreover, it is independent of the intensity of
simple reason behind fhis is that half-filled and fuU:'filled light used.
subshells have got extra stability. (iii) There is no time lag between incidence of {ight and
Cr24 -----+ ls2,2!i2l,3i3l3d 4 ,4s2 (wrong) emission of photoelectrons.
2 2 s 1 (iv) For emission ofphotoeJectrons, the frequency ofincident
-----+. lsL,2s 2l,3s 3l3d ,4s (right)
light must be equal to or greater than the threshold
Cu 29 -----+ . ls2, 2s 22l, 3s 23l3d. 9,4s2 (wrong) frequency.
-----+ ls2, 2s22p6, 3s23l3dlO, 4S1 (right)

Similarly the following elements have slightly different


configurations than expected: 28. The maximum kinetic energy of photoelectrons ejected from
Nb 41 ~[Kr]4d4581
a metal, when it is irradiated with radiation of frequency
· 2 X 1014 s-I is 6.63 x 10- 201 The threshold frequency of the
M042~[Kr]4dsSsl metal is: (PMT (Kerala) 2008)
Ru 44 ~ [Kr]4d 7 58 1 (a)2x Id4 s- 1 (b)3x Id4 s- 1
Rh 4s ~[Kr]4d8 S~I (c) 2 X 10- 14 S-I (d).l X 10- 14 s-I
Pd 46 ~[Kr]4dlO58° (e) I x 101 4 s- I
[Ans.(e)]
Ag 47 ~[Kr]4dlo581
[Hint: Absorbed energy Threshold energy + Kinetic_energy
Pt78.~[Xe]4f 145d9 fu'l of photoelectrons
Au 79 ~[Xe]4f 14 Sd lO fu'l . hv=hvo+KE
hvo =hv -' KE
La S7 ~ [Kr] 4d lO 58 2 Sp6 5ti 1fu'2 6.626 x 10- 34 X Vo = 6.626 x 10-.34 X 2X 1014 - 6.63 x 10- 20
CeS8 ~[Kr] 4d lo 4f2 58 2 Sp6 Sd°fu'2 1.3252 X 10- 19 _ {i63 x 10- 20
Yo""
6.6~6 x 10- 34
Gd64 ~[Kr]4dlo 4f7 58 2Sp 6 Sd I fu'2
Vo =9.99 X 1013 = 1014 S-l ]
220 PHOTOELECTRIC EFFECT 29. 1fAo and Abe- the threshold wavelength and the wavelength
of inCident light, the velocity of photoelectrons ejected will
Emission Qf electrons from a metal surface when exposed to light be:
radiations'f,fappropriate wavelength is called photoelectric
effect. The emitted electrons are called photoelectrons. · (a) ~;;(AO - A) (b) ~~(AO ~ A)
Work function or threshold energy may be defined as the
minimum amount of energy required to eject electrons from a
metal surface.
(c) 2hC(AO - A) .
m A.A.o
2h(1 I)
(d) _ . .....,..--
m A A o
According to Einstein, [Ans.(c)]
Maximum kinetic energy of the ejected electron · [Blnt: AbsOrbed energy = Threshold energy + Kinetic energy
=absorbed energy - work function . . of photoelectrons
I 2 he he 1 2
-mv
2
=hv-hv o -=-+-mv
max A Ao· 2
102 G.R.B. PHYSICAL CHEMISTRY FOR COMPETITIONS .,.
\
, h
v = 2hc (1..0 A)] (Slope}z/(Slope)l -=e]
~ m AAo I.
Me. ,.
j • " \. 1
32. Ground state energy of H-atom is (- E) ),the velocity of
30. A radiation of wavelength A illuminates a· metal iJnd ejtlCts
photoelectrons of maximum kinetic energy of leV. Another photoelectrons emitted when photon of energy E z strikes
~, photoelectron~ of
stationary Liz+ ion in ground state will be:
radi!)tion of wavelength ejects
_ ~2(Ez ..:. E\ ) (b) _ 1'-2-(E-2-+-9E-1-) ,
maximum kinetic energy of 4 eV. What will be the work
()
av-
m
v-" ,m
function of metal?
(a) leV (b) 2eV (3) 0.5eV (d) 3eV (c) v =~2(E2: 9E1 ) : (d)v ~2(E2 ~3Ed
(Ans. (c)]
[Hint: Absorbed energy = Threshold energy + Kinetic energy
[Ans. (c)]
" 'ofphotodectrons [Hint: Threshold energy ofLi 2+ =9E) . .
c Absorbed energy = Threshold energy + Kinetic energy of
h- Eo+ leV ... (i) photoelectrons
,A
c I 2
3h-=Eo + 4eV ... (ii) E2 = 9E) + -Zmv .
A
3(Eo + leV) =Eo + 4eV
mv 2 =·2(E'z 9E1)
=0.5eV] I2
Eo . _. _v== V (E: :,!EI? 1
31. The ratio of siopes of maximum· kinetic 'energ)pversus
frequency and stopping potential (Vo) versus frequency, in
photoelectric effect gives: ' 2.21 SOME OTHER FUNDAMENTAL
(a) charge of electron (b) planck's constant PARTICLES
(c) work function (4) threshold frequency Besides protons, neutrons and electrons, many more elementary
[Ans. (a)] particles have been discovered. These particles are also called
[Hint: hv = hvo +ef/o Fundamental particles. Some of these particles are stable while
eVo=hv":'hvo the others are unstable. Out of stable particles, the electron, the·
h h proton, the antiproton and the positron are four mass particles
Vo =-v --Yo ... (i) ,
·e e while neutrino,photon and graviton are three energy particles.
(Slopelt=h / e Among these, unstable particles are neutron, meson anq
(KE)max =hv - hv 0 ... (ii)
v-particles. The main' chara~teristics of the particles are given in
(SI()pe)2 = h table 2.1 below.
Table 1.1
Particle Symbol Nature ebara' au x 10= 10 MIll (amu) DIteovered. by
Positron ,e+, leO, + + 4.8029 0.0005486 Anders()n (1932)
Neutrino v 0 0 <0.00002 Pauli
Antiproton _ p- -4.8029 1.00787 Chamberlain Sugri and Weigh land (1955)
Photon hv 0 0 0 Planck
Graviton G 0 0 0
Positive mu meson Il+ + + 4.8029 0.1152 Yukawa (1935)
, Negative mu meson .. Il - 4.8029 0.1152 Anderson (1937)
Positiye pi meson 1t+ + + 4.8029 0.1514
Negative pi meson
NeUtral pi meson
1t

1t
-
0
0
-4.8029
0
0.1514
0.1454
} Powell (1947)

2.22 ISOTOPES
Isotopes are the atoms of the same element having differen l on atomic mass. Isotopes were first identified in radioactive
atomic masses (see determination of isotopic mass). The term elements by Soddy. In 1919, Thomson established the existence
'isotope' was introduced by Soddy. This is a Greek word of isotopes in a non-radioactive element, neon. Until now, more
=
meaning same position (lsos same, topes = position), since all than 1000 isotopes .have been identified (natural as well as
the isotopes of an e1ement occupy the same position in the artifichd). Out of these about 320 occur in nature, approximately
periodic table. Isotopes of an element possess identical chemical 280 of these are stable and the remaining 40 are radioactive.
properties but differ slightly in physical properties which depend
I
ATOMIC STRUCTURE 103

Conclusions ',' same two particles (i. e., protons) are at least forty times greater
i . .. "
than the repulsive forces; Thus, two major forces exist in the
(i) Number of n~trons present in the nuclei of various
nucleus. These rrre electrostatic and nuclear. The nuclear forces
isotopes of an ~lement is alw~ys different. The number of '
are stronger and the range of these forces is extremely small. The
neutrons is detennined by applying the formula N =A - Z
forces which operate between nucleons are referred to as
where A is mas~ number and Z is atomic number.
exchange forces. In order to account for the stability of the
Hydrogen has tHree isotopes, ~ H, fH and H. i nucleus, a theory known as meson theory was put forward by
A (MIll ~umber) Z No. of neutrons Yukawa, in 1935. Yukawa pointed out that neutrons .and protons
o are held together by very rapid exchange of nuclear particles
called pi mesons. These mesons may be electrically neutral,
2
positive ornegative (designated as nO , n + and n - ) and possess a
'tH 3 2 mass 275 times the mass of an electron. Nuclear forces arise from
Oxygen has tliree isotopes, 16 0 , 170and 18 O. a constant exchange of mesons between nucleons with very high
velocity (practically the velocity of light).
A Z No. of neutrons Let a neutron be converted into a proton by the emission of a
1:0 16 8 8 negative meson. The emitted meson .is accepted by another
proton and converted into a neutron.
llo 17 8 9
--
"J8 . _'"_TT __ "
."
n A ~ p~ +n-
gO 18- 8 '10'
n- + p; ~nB
(ii) In a neutral atom, the number of protons and the number
Similarly, a proton after emitting a positive meson is
of electrons are aht'ays the same, i. e., the electronic converted into a neutron and vice-versa,
configuration of all the isotopes of an element is the
p~ ~nA +n+
same. Thus, all the isotopes of an element show the same
chemical properties. However, the rates of reactions may n+ +nB ~p;
be different for different isotopes of an element.
or simply
(iii) All the isotopes of an element occupy the same position
in the periodic table. n ~ n- + p
(iv) The isotopes of an element differ slightly in physical There may be two more types of exchange, i. e., between
properties. The compounds formed by these isotopes will neutron-neutron and proton-proton, involving neutral pi mesons.
also have different physical properties. p n. 11.0 nO
"'-no '\no orsimply p"--; p and n~n
Determination of Isotopic Mass p It" nit" .n nO
37
Chlorine has two isotopes 17 Cl 3S and 17 Cl ; these are found
in nature in 3 : 1 ratio or 75%: 25% respectively. Isotopic mass'
Mass Defect-Binding Energy
may be calculated as: ". . , . It is observed that the atomic mass of all nuclei (except
Isotopic mass of chlorine hydrogen) is different from the sum of the masses of protons and
neutrons. For example, the helium nucleus consists of2 protons
3s 7
%of
- -- . 0 f Cl35 + %of ct3
Cl - x mass X mass 0 f Cl 37
and 2 neutrons. The combined mass of 2 protons and 2 neutrons
100 100 should be
75 25 = 2 x 1.00758 + 2 x 1.00893
=- x 35 + x 37 =35.5 4.03302 amu
100 100
OR The actual observed mass of helium nuclei is 4.0028 amu. A
Isotopic mass of chlorine . difference of 0.0302 amu is observed between these two values,
This difference is termed as mass defed.
Ratio of Cl 35 x mass of Cl 3s + Ratio of Cl 37 x mass of Cl 37
Mass defect = Total mass of nucleons Observed atomic mass
Sum of ratio
This decrease in mass (i. e., mass defect) is converted into
= 3 x 35 + 1 x 37 = 35.5
energy according to Einstein equation E mc 2 • The energy
4
released when a nucleus is formed from protons and neutrons. is
2.23 THEORIES OF NUCLEAR STABILITY called the binding energy. This is the forfe which holds all the',
Since, a nucleus contains positively charged protons, there must nucleons together in the nucleus. Binding eI1ergy can be d~fined in
exist a strong repulsive force between them. It has been other ways also, i. e. , the energy required tohreak the nucleus· into
calculated that there exists an electrostatic repulsion of constituent protons and neutrons. Binding energy is measured in
approximately six tons between two protons situated at a nuclear ' MeV (Million Electron-Volts), i. e. , 1 aml.l 931 MeV.
distance but at the same time the forces which bind the nucleus Binding energy =Mass defect x 931 MeV
are very high. It has been found that. nuclear forces attracting the
,
104 ' G.R.B. PHYSICAL CHEMISTRY FOR COMPETITIONS

Bjnding energy can also be calculated in erg. This is ':t«i THE WHOLE NUMBER RULE AND
Mass defect (amu) x J .66 X 10- 24 x (3 X 1010 )2 erg PACKING FRACTION
6
(l MeV = 1.60 x 10 erg) Aston believed that mass number values (sum of protons and,
The binding energy increases with the increase in atomic neutrons) of isotopes should be whole numbers on the scale of
number of the element. This indicates that heavier nuclei should oxygen ( 16 0 = 16) but actually it was observed that these were
, be' more stable than lighter nuclei. But, it is not so because not integers. The difference in the atomic mass of an isotope and
heavier nuclei above atomic number 82 are unstable. It is thus mass number was expressed by Aston (1927) as packing
clear that total binding energy of a nucleus does not explain the"" " fraction bythe following expression:
stability of the IUlcleus.
The total binding energy of a nucleus when divided -by the P k' fra' Isotopic atomic mass'- Mass number 104
, ac 109 ctlOn = " x
number of nucleons gives the average or mean binding energy Mass number
per nucleon. The binding energy per, nucleon is actually the
measure of the stability of the nucleus. The greater the binding ' the pack~109 fra'
energy per nucleon, more stable is_the nucleus. Thus, ctlOn 0 f IH = l.OO78~ I x 104 78 and
I
Binding energy per nucleon
Total binding energy
the pacmgk ~ fra'ctlon
. '0,f 3sCI
,=
34.980 35.0 x 104 =-5.7. The
Total nwnber of nucleons 35.0
packing fraction of oxygen is zero.
When binding energy per nucleon of a number of nuclei is
It is clear that the value of packing fractlon varies from one---~~­
plotted agairlst the corresponding 'maSs number, a graph is
atom to other. This is sometinie positive or zero but more often
, obtained (Fig. 2.19) whose characteristics are as follows:
negative.
Region of A negative packing fraction means that atomic mass is less
greatest stability than nearest whole number and this suggests that some mass has

f:
':> 8
I---r----,--
been converted into energy when the particular isotope has been
constituted. This energy is r~sponsible for nuclear stability. All
those having negative values of packing fraction are stable
Q) nuclei.
67 Apositive packing fraction generally indicates instability of
ju 6 1-1'---+- the nucleus. However, this statement is not correct for lighter
nuclei.
E5 In general, lower the value of packing fraction, the gr~ter is
~ 4H---+- the stability of the nucleus. The lowest values of packing
>.
fractions are observed for transition elements or iron family
~3
c:: indicating thereby maximum stability of their nuclei.
~ 21i---+-
c::
'g ,~~ ,THE MAGIC NUMBERS
iii
It has been observed that atoms with an even number of nucleons
o 20 40 60 80 100 120 140 160180200220 240 in their nuclei are more plentiful than those with odd number.
, Mass number, A - This indicates that a nucleus made up of even number of nucleons
Fig. 2.19 is more stable than a nuclei which consists of odd number of
(i) Binding energy per nucleon increases from l.l to 8.0 nucleons. It has also been observed that a stable nuclei results
MeV from mass number 2to 20. when either the number of neutrons or that of protons is equal to
(ii) Binding energy per nucleon increases from 8 to 8.6 MeV one of the numbers 2, 8, 20, 50, 82, 126. These numbers are
from mass number 20 to 40. ' called magic numbers. It is thought that the magic numbers form
closed nuclear shells in the same way as the atomic numbers of
(iii) Binding energy per nucleon remains 8.6 -8.7 MeV from
mass number 40 to 90. Iron (56) has the maximum value inert gases form stable electronic configuration. In general,
of8.7 MeV per nucleon. elements that have nuclei with magic number of !:fotons as well
(iv) The value of binding energy per nucleon decreases from a.-, magic number of neutrons such as jHe, I~O, 20 Ca, 2~~Pbare
8.6 to 7.5 MeV from masS number 90 to 240. highly stable and found in abundance in nature.
(v) Points for helium, carbon; oxygen lie quite high in the A survey of stable nuclei found in nature shows the following
graph showing that these nuclei are highly stable. trend:
The binding energy per nucleon can be increased in two ways: Protons Even Even Odd Odd
(i) Either by breaking heavy nucleus to those of NeutronS Even Odd Even Odd
intermediate mass numbers (process of fission) or No. of stable nuclei 157 52 50 5
(ii) By fusing lighter nuclei to form heavier nuclei (process Only five stable odd-odd nuclides are known; these nuclides
of fusion). 2H' 6 · 10 14N' d 180 '
are I ',3 LI, sB, 7 an 73 Ta.

ATOMIC STRUCTURE 105
.
: : :::n_SOME SOLVED EXAMPLES\ I:::: : : Example 50. Write down the values of quantum numbers
of all the electrons present in the outermost orbit of argon
Example 46. The minimum energy required to overcome (At. No. 18).
the attractive forces between an electron and the suifaceof Ag Solution: The electronic configuration of argon is
metal is 5.52 x 10- 19 J. What will be the maximum kinetic energy 1s 2 , 2s 22p 6 , 3s 2 3p2x 3p2y 3p2z
of electrons ejected out from Ag which is being exposed to UV Values of quantum numbers are:
light ofA = 360A'? n / m s
Solution: Energy ofthe'photon absorbed 3i 3 0 0 +~.-~
h . c 6.625 X 10-27 X 3 X 1010 3p; 3 ±l +~.-~
=- =--------:::---
8
A 360 x 10- 3p.~ 3 ±1 +~,-~
= 5.52 X IO- It erg 3p; 3 0 +~,-~
= 5.52 x 10- 18 J Example 51. (a) An electron is in 5f-orbital. What
E(photon) work function + KE possible values ofquantum numbers n, I, m and s can it have?
KE=5.52xlO- 18 -'7.52xIO- 19 , (b) What designation is given to an orbital having
(i) n = 2, I =I and (ii) n = 3, 1= O? .
=47.68 X 10- 19
J
Solution: (a) For an electron in 5f-orbital, quantum numbers
"''''E,lample 41~ Let a light ()f wavelength' Aand intensity ']' are: .----,-~--~-

strikes a metal su1face to emit x electrons per second. Average n = 5; , 1= 3 ; m =- 3,- 2, - I, 0, + 1, + 2, + 3


energy ofeach electron is y' unit. What will happen to 'x' and y'
when (a) A is halved (b) intensity] is doubled? and s =elt'hel' + -I or - -I
Solution: (a) Rate of emission of electron is independent of 2 2
wavelength. Hence, 'x' will be unaffected.' (b)(i) 2p, (ii) 3s
Kinetic energy of photoelectron = Absorbed- Threshold ,.Iit. ~1i6J..altlple 52. Atomic number of sodium is II. Write down
energy energy 'the four quantum numbers of the electronhavillg highest energy.
Solution: The electronic configuration of sodium is:
he
Y"'J:-w o 1s2 ,2s2 2p6 ,3s 1

when, A is halved, average energy will increase but it will not 3s-e1ectron has the highest energy. Its quantum numbers are:
become double. 1 1
n=3, 1=0, m=O" s=+ ' 2 or--2
(b) Rate of emission of electron per second 'x' will become
double when intensity ] is doubled. Average energy of ejected Example 53. An element has 8 electrons in 4d-subshell.
electron, i. e., 'y' will be unaffected by increase in the intensity of Show the distribution of 8 electrons in the d-orbitals of the
light. . ' element with~n small rectangles.
'" Example 48. How many orbits, orbitals and electrons are Solution: 4d-subshell has five d-orbitals. These are first
there in an atom having atomic mass 24 and atomic number 12? occupied singly and then pairing occurs. The distribution can be
Solution: shown in the following manner:
Atomic number = No. of protons = No. of electrons = 12 4d
Electronic configuration 2, 8, 2 [iJ,!iJ,!H! i ! i J
No. of orbits = (K. Land M ) Example 54. How many elements would be in the third
No. of orbitals on which electrons are present period ofthe periodic table if the spin quantum number ms could
=(one 1s + one 2s + three 2p + OI~e 3s) I
have the value - - ,0 and + - ?
I
Example 49. A' neutral atom has 2K electrons, 8L 2 2
electrons and 6 M electrons. Predict from this: Solution:
(a) its atomic number, (b) total number of s-electrons. (c) total n=3,/=O,m=0
number ofp:electrons, (d) total number ofd-electrons.
Solution: (a) Total number of electrons ' . ms = -1/ 0 +1/

~
/2,,72

(2+ 8+ 6)= 16 / = I; m -I, 0, + I ms = -~, 0, +~


So, Atomic number 16 . m - -1/ 0 +1/
s - 72' , 72

Electronic configuration = tS'2 ,2s 2 2 p 6 , 3$23 p 4 72' 0, +1/


m,t -- -1/ 72 }
(b) Total number of s -electrons =(1s 2 + 2s 2 + 3s 2 ) = 6 for each value of
/ =2; m =~ 2, -.1, 0, + I, + 2 magnetic '
(c) Total number of p-electrons = (2 p 6 + 3p4)= 10 {
quantum no ..
(d) Total number of d -electrons 0
106 G.A. B. PHYSICAL ~HEMISTRY FOR COMPETITIONS
i . • c

Number of elements 3s (3e) .' =I6.~ 8 x (lOOQ5486:= lS.99S~ aniu


3p (ge) Mass of the nucleus of l~O
3d (15e)
:= mass of 8 protons + mass of 8 neutrons
.. 27 elements will be there in third period pfperiodic table.
Example 55. The binding energy of He is 28.57 Me V. i 8x 1.0,0,757+ 8x 1.0,0,893 16.l32amu
What shall be the binding energy per nucleon of this element? Mass defect =(16.132-15.9956) Q.1364amu
Solution: The nucleus of j He consists of 4 nucleons. Binding energy =0,.1364 x 931=127MeV
. . Total binding energy Example 57. There are four atoms which have mass
So, 13mdmg energy per nucleon := - - - . --=---=- numbers 9,10,11 and 12 respectively. Their binding energies are
No. of nucleons
54,70,,66 and 78 MeV respectively. Which one of the atoms is
= 28.57 = 7.14 MeV most stable?
4 Solution: Stability depends on the "alue of binding energy
per nucleon.
Example 56. . Calculate the binding energy of the oxygen
isotope 1:0. The mass of the isotope is 16.0, amu. (Given A B C D

e= Q.QQQ5486amu, p = 1.0,0,757 amu and n l.OO893amu.) Binding energy (MeV) 54 70 66 78


Solution: . The isotope. I~O contains 8 protons, No. of nucleons 9 10 11 12
8 neutrons and 8 electrons. Binding energy per nucleon (MeV) 6 7 6 6.5
Actual mass of the nucleus.of I: 0 Thus, B is most stable.
:= 16- mass of8 electrons

MISCELLANEOUS NUMERICAL EXAMPLES~'/L/L<i


'. ... ~ .
..... .

These examples will give the sharp edge to the aspirants for Solution:
lIT and various other entrance examinations. Coulombic force of attraction =Centrifugal force
Exa.mple 1. The SchrOdinger wave equation for hydrogen 1 Zexe mv 2
atom is
4nEo T=~
_ 1 (1 [2
'112s - - - - - -
)3/2 ro ] -rlao
-- e . where, v:= velocity of electron
4~n ao ao
ao = distance between electron and nucleus
where ao is Bohr radius. If the radial node in 2s be at· ro ' then
find r in terms ofao. (liT 2004) 1 Ze 2 _ 2
-----mv
Solution: Given, 4nEo ao .

_ 1 (1 [2
'112s - - - - - -
)3/2 ro ] e -rlao
-- KE
1 2 1 Ze 2
-mv = - - - ; r -
2 . ' ~nEQ·.Lao
4~n ao ao
PE=-2xKE .
'II~s = Qat node 1 I, Ze 2
-2x~x-=
4ntb· 2ao
Example 3. Hydrogen atoms are excited from ground
state. Its spectrum contains wqvelength 486 nm. Find, what
ro = 2a o transition does the line corresponds to. Also find from this
information what other wavelengths will be present in the
Example 2. Consider the hydrogen atom to be a proton
spectrum?
embedded in a cavity of radius ao (1!ohr radiUS) whose charge is
neutralized by the addition ofan electron to the cavity in vacuum Solution: Wavelength 486 nm, i. e., 4860, A indicates that
infinitely slowly. Estimate the average total energy ofan electron the spectrum is in visible region, i. e. , Balmer series.
in its ground state in a hydrogen atom as the work done in the
above neutralization process. Also, if the magnitude of average 1. = Rz.2 [_1 ....:_11
KE is half the magnitude of average potential energy, find the A nr ni
average potential energy. (lIT 1996)
ATOMIC STRUCTURE 107

_ _1_ _ g ::::: 109677.76 x 12 [~- ~l . [1 1J
=109678 - - -
32 4 2
4860x 10- 2 n2

On solving, we get A = 1.8 X 10-4 cm


ni == 16 Example 7. A negatively charged particle called
Negatron was discovered. In the Millikan oil-drop experiment, s
n2 == 4 the charges of th£ oil-drops in five experiments are reported as
Thus, transition is from, 4 ~ 2 12 x 10- 19 coulomb; 4.8 x 10- 19 coulomb; 6.4 x 10-19 coulomb;
Other transitions in the spectrum are 8 x 10-:-19 coulomb and 9.6 x 10- 19 coulomb. Calculate the charge
4~3~2 on the negatron.
Solution: In Millikan's oil-drop experiment; the charges on
1 . [1 IJ
-::::: 109677.76 x ex - 2 - - 2.
A 3 4
the oil-drops are integral multiples of the charge of the particle.
" Dividing the charges of droplets bytbe lowest charge:
3.2~ 10- 19 __ 1 4.8 X 10- 19
A == 1875 X 10-7 cm ro 12x 10- 19
00
12x 10-19
15
Example 4. lfuncertainties in the measurement of
position and momentum of an electron are equal; calculate 6.4 x 10- 19 ." 8x 10- 19
uncertainty in the measurement of velocity. .
(iii)
12x 10- 19
=2 (iv)
12x
=2.5
Solution: According to Heisenberg's uncertainty principle, 19
9.6x 10- ::::: 3"
. h (v)
Ax.~p~- 12x 10- 19
41t
All the values are not integral; they can be converted to

. .~ = V~
Given, Ax ::::: [""h : : : 0.726 X 1O-I7 integers on rriultiplyingby 2.
:.Charge of the negatron will be
~:::::m~V 19
3.2 x 10- =.1.6 X 10- 19 C
. ~ 17 2
or AV= P ::::: 0.726 x 10- =7.98 x 1012 ms- I '. . .
m 9.1 x "Example 8. When a certain metal was irradiated .with
light offrequency 12x 1016 Hz, the photoelectrons emitted had
Example 5. How much· energy will be released when a
sodium ion and a chloride ion, originally at infinite distance are twice the kinetic energy as did photoelectrons emitted when the "
brought together to a distan.ce of2. 761 (the shortest distance of same metal was irradiated with light offrequency 2.0x 1016 Hz.
approach in a sodium chloride c rys tal)? Assume that ions act as Calculate v 0 for the metal.
point charges, .each with a magnitude of 1.6 x 10- 19 C. Solution: Applying photoelectric equation,
Permittivity constant of the medium is 9x 109 Nm 2C -2.
KE hv-hvo
Solution: Energy released
" KE
9 19 or (v-vo)==
== _ K = _ 9 X 10 x (16 X 10- )2 = -8.35 X 10- 19 J h
r . 2.76 x 10- 10 Given,
. Example 6. The angular momentum of an electron in a
... (i)
Bohr orbit ofH-atom is 4.2178 X 10-34 kg m 2/sec. Calculate the
spectral line emitted when an electron fails from this level to the
next lower level. and ... (ii)
. "h
Solution: We know, mvr= n Dividing equation (i) by equation (ii),
21t
34 v 2 -vo == = 2KEI =2
4.2178x 10-34 = n x 6.626 x 10-
2x 114 VI - Vo KEI KEI

n=4 or v 2 -vo =2v 1 -2v o


16 16
or v O =2v I - V2 2(2.0xI0 ) (3.2 x 10 )

=8.0 X 1015 Hz
108 G.R.B. PHYSICAL CHEMISTRY FOR COMPETITIONS

Example 9. An electron moves in an electric field with a ... (iii)


kinetic energy of 2.5 eV. What is the associated de Broglie
wavelength?
Solution: Kinetic energy

::::~mv2 (v:::: :1.)


::::~m(:J2 Thus, AE3~1 =fj£3~2 + AE2~1
::::
I h
--
2
.. Example n. If an electron is moving with velocity
2mA2 500 ms- I, which is accurate up to 0.005% then calculate
2 uncertainty in its position. [h == 6.63 x 10- 341s, mass of electron
or 1.2 =! h
== 9.1 x 10- 31 kg] [AIPMT (Mains)2008)
2mxKE

A==
.
h
28
[m=9.108x 10- g
. h 6.626 X 10-27 erg ~sec
1 Solution: Uncertainty in velocity
A
uv= 600 x 0.005 3
::::.x 10-2
. .
ms -I
100 .
~2mx KE leV= 1.602 x 10- 12 erg According to Heisenberg's
.. ---- •. - - -- uncertainty
-
principle
--c- -- - .,---------
c- 7 -

6.626 x 10:"27 h
AtAv~-
4rcm
~2 x 9.108 x 10-28 x 2.5 x 1.602 x 10- 12
At ~ h
== 7.7 x 10-8 cm 4nmAv
c.· Example 10.· Consider the following two electronic
transition possibilities in a hydrogen atom as pictured below: 6.63 x 10- 34
~----------------------
·4x3.l4x9.1x

=tt
n=3
. n=2 == 1.9 x to- 3 m

. n== I Example 12. Applying Bohrs model when Hatom comes


from n == 4 to n == 2, calculate its wavelength. In this process,
(a) The electron drops from third Bohr orbit to second Bohr write whether energy is released or absorbed? Also write the
orbit followed with the next transition from. second to first Bohr range ofradiatian. RH =2.18 X 10- 18 1, h = 6.63 x 10- 34 Js.
orbit.
(AIPMT 2008)
(b) The electron drops from third Bohr orbit to first Bohr
orbit directly. Show that the sum of energies for the transitions
Solution: Energy is released in this process; and the radiation
n = 3 to n =2 and n :::: 2 to n =·1 is equal to the energy oftransition
will belong to visible region (Balmer series)
forn=3ton=1. .. E= hc == RHZ2 [-\-_ -\-] -
. A nl n2
Solution: Applying, AE Riel [_I __ 1 ]. 2
n 12 n. 22 . X== R H'Z [-\- _ -\-]

For n == 3to n:::: 2;


.1. hc nl ni
2 18
2.18xlO- xI [.1 I]
AE3
~
i =RH [~-~]
22 32
=RH X 5
36
... (i) =6.63XIO-34 X3XI0 8 4-16
18
. 2.18xlO':' xtz [3]
For n = 2 to n = I; == 6.63 x lO-j4 x 3 x-108 16
8
... (ii) A= 6.63 x 10- 34 X 3 X 10 x .16 = 4866 x 10- 10 m
3x2.l8xto-
For n == 3to n 1; = 4866 A
' .....,.
ATOMIC STRUCTURE 109

SUMMARY AND IMPORTANT POINTS TO REMEMBER


1"", I " , "11, My • ~ $JIIItI ... ' us ,. .. ~

1. Atom is the smallest indivisible particle of matter 8. Isotones: These are atoms of different elements having
(proposed by John Dalton in 1808). same number of neutrons in the nucleus, e.g.,
2. All atoms except hydrogen atom are composed of three
ItC: l~, 19O
fundamental particles, namely, electron, proton and neutron.
Hydrogen atom has one electron and one proton but n~ neutron.
9. Electromagnetic radiations are energy waves containing
(a) Electron: The nature and existence of electron was both electric and magnetic vector perpendicular to each other.
established by experiments on conduction of electricity through
(i) These are transverse waves.
gases, i.e., discovery of cathode rays. In 1897, J.J. Thomson
(ii) They do not need any medium for their propagation. They·
determined e/ m value (-1.7588 x 108 coulomb/g) and proved travel with the velocity of light. '
that whatever gas be taken in the discharge tube and whatever be
the material ofthe electrodes, the value of e/ mis always the same. (iii) v:::: C ,V =frequency, c =veiocity oflight,
A '
Electrons are, thus, common universal constituents of all atoms. A = wavelength
Electron is a' subatomic pru.:ticle which carries charge
V ='= .~1. T
I wave'Ilwiwer, .:.;,;I = tIme
. per1
' 'od.
-1.60 x 10-19 coulomb, i.e., one unit negative charge and has A," v
maSs 9.1xlO-28 g (or 9.1 x 10-31 kg), i.e., _l_th mass of (iv) According to Planck's quantum theory, the energy is
1837 emitted or absorbed in the fOlTIl of energy packets called quanta.
hydrogen atom (0.000549 amu). The name electron was given by Quantum of visible light is called photon.
Stoney. ' Energy of one quantum == hv
(b) Proton: The nature and existence of proton was
established by the discovery of positive rays (Goldstein 1886). ==h5..
A
Proton is a subatomic particle which carries +1.6x 10- 19
coulomb or one unit positive charge and has mass 1.672 x 10-24 g h = Planck's constant
27
(or 1.672 x 10- kg), i. e., 1.0072 amu. The el m was determined 6.626 x 10-34 J sec
by Thomson in 1906 and the value is +9.579 x 104 coulomb/g. It
"': 10. Hydrogen spectrum: Hydrogen spectrum is a line
was named as proton by Rutherford. spectrum. The lines lie in visible, ultraviolet and infrared regions~
(c) Neutron: It is a subatomic particle which carries no All the lines can be classified into five series. Ritz presented a
charge. Its mass is 1.675 x 10-24 g (1.675 X 10-27 kg) or 1.0086 mathematical formula to find the wavelengths of various lines,
amu. It is slightly heavier than proton. It was discovered by
Chadwick in 1932 by bombarding beryllium with a.-particles. -.!.=V=R[~-~l
A. n n l 2

where, R is Rydberg constant (R == 10 9678cm- I ).


The e/ rh value of neutron is zero.
HI H2
3. According to the Rlith~rfotd's model of atom, (i) it
consists of nucleus of very small size and high density Lyman series (UVregion) 1 2,3,4,5, ...
(ii) electrons revolve round the nucleus in a circular path. BaImer series (Visible region) 2 3,4,5,6, ...
15
Radius of nucleus = 10- m Paschen series 3 4,5,6,7, ...

4.
Density of nucleUs = 108 tonnes/cc
Atomic number (Z) =Number of protons in the nucleus
Brackett series
Pfund series
} (IRregion) 4
5
5,6,7,8, ...
6,7,8,9, ...

S. Mass number (A) =Number of protons + Number of Balmer series consists offour prominent lines Ha., IIp, Ry and
neutrons H3 having wavelength 6563 A, 4861 A, 4340 A and 4102 A
6. Isotopes: These are atoms of same element having respectively. . .
same atomic number but different mass numbers, e. g., Balmer equation is,
<l H, ;H, fH): mCI, r;Cl) -.!.=R[~ __
l2 ]
A 22 n
7. Isobars: These are atoms of different elements having
same mass number but different atomic numbers, e.g., where, H == 3,4,5,6, ...
The Rydberg formula is used to calculate the wavelength of
:~Ar, teK, ~Ca any line of the spectrum
I
110 G.R.B. PHYSICAL CHEMISTRY FOR COMPETITIONS

V::V Ke2 ' '-


-,.~. ' . mr
where, x :::;: number of lines in the spectrum; x = 00 for series limit where,K _1_ = 9x 109 Nm2/C2
or last fine. Let, transition of electrons takes place from ni to n l 4nEo
shell; then the number oflines can be calculated as:
v ~ x 2.188 X 108 emf sec
· (n2 nl )(n2 - nl + 1) n
Number 0 f . Imes:::;: ...:.....:=---...!...:...:..-=---'----'-
2 (vii) Potential energy of electrons in a particular shell:
2
11. Bohr's atomic model: It is based on Planck's PE:::;: -KZe =- 27.2 xZ 2 eV
quantum theory. Its main postulates are summarised as: r n2
(i) Electrons revolve round the nucleus in circular path of
(viii) Kinetic energy of electrons in a particular shell:
fixed energy called stationary states.
(ii) Angular momentum of electrons are quantised, i. e. , KE=~KZe2 =+13.6 Z2eV
2 r n2
mvr=n(:n) 2
Total Energy, TE =- -1 -
KZe
-
2 r
(iii) The energy as weU as angular momentum both are 1
quantised· for electrons. It means they can have only certain TE=-PE"
'2
. yalues of energy and angular momenta. TE -KE
12. Importantf(JrJllulatioQs .. obtained from Bohr's (ix) Number of revolutions per second by an electron in a
atomic model which are valid for single electron species like shell:' .
H, He, + L':1+
I , Be3+ ,et c..•
Velocity v
(i) EI <E2 <E3<E4 = CircUJl?ference =-=-
21tr . h
(ii) (E2 -Ed>(E3 E 2 »(E4 -E3 ) ...
where, Ei ,E2 ,E3 , ... are energies of cOrresponding shells. (~) Frequency of electroJ,ls in nth orbit:
2 2 v' ..
n h '=-'-
. (iii) r.:::;: --:c--.....,,-- ,2nr
n 4n2 K~2 mZ
:::;: 662 XlO15Z 2
'K =_1_ =9x 109 Nm 2fC;
4nEo n3 •
2 . (xi) Period of revolution of electrons in nth orbit (Tn)'
r =~Z x 0.529 A (where, r is the radius of Bohr orbit of
2nr 1.5xlO- 16 n 3
T.=-= . sec
electrons.) n Vn Z2
(iv) Energy of.electrons in a partiCular sheli can he calculated
as:

E=--
Z2 2n 2 mK.2'e4
2 (xii) Ionization energy =E~ - En'
n h2

E ='-~ x 21.79 x 10- 19 J/atom


=o-(-~: x 13.6eV}
2
n Z2
. Z2 =-x116eV
2
= - -2 x13.6eV n
n . Z2
Z2 :;:: - 2 x 21.79 X 10-'9 J / atom
=- 2 x l312 kJ/mol n
n . I Z2' 2
Z2 RE (xiii) .-l. =_1- X ~
I 2 Z2 2.
n 12
n2 . .

RE = - l3.6 eV (Rydbetg energy) I, and 12 are ionization energies of two elements 1 and 2.
2
(v) En :::;:E\ln 2 ;En =E1 X~forhydrOgen-likespecies.
n . (xiv) AE'{Energy of transition) := RE
. , . ..
[-i-.n, -~)n2
(vi) Velocity of electrons in a particular shell or orbit can be
calculated as: . . }{f::::-13.6 eV
ATOMIC STRUCTURE 111

RH = Rydberg constant = RE:::: 109677 cm~-J .


he he 1 2
--'-=-+- mv
he . A Ao 2
Defects of Bohr theory: (i) It fails to explain the spectra of Vo and Ao are called threshold frequency and threshold
multi-el~ctron atoms. (ii) It fails to explain fine spectrum of
wavelength respectively.
hydrogen. (iii) It does not provide an .explanation why angular
18. Quantum numbers: The set of four integers required
momentum should always be an integral multiple of hl2n. (iv) It
to define an electron completely in an atom are called quantum
does not explain splitting of spectral lines under the influence of
numbers. The first three have been derived from Schrodinger's
magnetic field (Zeeman effect) and electric,. field (Stark effect). wave equation. . . ..
13. Sommerfeld's extension: Sommetreld (1915) introduced
(i) Principal quantum number: It describes the name,
the. idea of elliptical orbits. Except first orbit which is only
size and energy of the shell to which the electron belongs.
circular, the other orbits are elliptical. The second orbit has One
eIliptical and one circular suborbit. The third orbit has two n = 1,2,3,4, ... represent K, L, M, N, ... shells respectively.
elliptical and one circular suborbit.
14. Dual nature: Light has dual character, i. e. ,it behaves Formulae for radius, energy and angular momentum of
electrons are given earlier.
s~metimes like particles and sometimes like waves. de Broglie
(1924) predicted that small particles such as electrons should (ii) Azimuthal quantum number: It is denoted by'/'o It
describes the shape of electron cloud and number of subshells in
show wave-like properties along with particle charact~r. The
a shell.
wavelength (A) associated -with a particle of-mass m and moving ----------

with velocity v is given by the relationship A = ...!!:.- ; where, h is 1= 0, 1,2,3, ... ,(n -1)
mv 1= 0 (s-subshell); 1= 1(p-subshell); I = 2 (d-subshell);
Planck's constant. 1=3 (f-subshell). .
The wave nature was cQnfl11lled by Davisson and Germer's Orbital angular momentum of electron
experiment.
Davisson and Germer gave some modified equations for = ~/(l + l)..!!...- = ~/(l + 1)/i
calculation of de Broglie wavelength: 2n

A= h ; where, E = kinetic energy of the particle. when I = 0, electrons revolve in a circular orbit and when I 0, *
·hEm the electrons revolve round the nucleus in an elliptical path.
(iii) Magnetic quantum number: It is denoted by 'm'. It
A= h ; where, q = charge ofthe particle accelerated by describes the orientations of the subshells. It can have values
~2qVm from -I to +1 including zero, i. e., total (21 + 1) values. Each valQe
corresponds to an orbital. s-subshell has one orbital, p-subshell
the potential of V volt.
has three orbitals (p x , P y and p z), d-subshell has five orbitals
15. Heisenberg uncertainty principle: It is imp~ssible to
. (d xy " d}'2' d zx , d> _ y2 and d z2) and f-subshell has seven
measure simultaneously,both the position and moment"umof any
microscopic particle with accuracy. Mathematically, orbitals. One orbital can .accommodate either one or two
h electrons but not more than two. s-orbital is spherically
llx I:l.p::: 4n; where, llx = uncertainty 10 position and
symmetrical and non-directionaL p-orbitals have dumb-bell
I:l.p = uncertainty in momentum. It introduces the concept of shape and are directional in nature. Four d-orbitals have double
probability of locating the electron in space around the nucleus. dumb-bell shape but· d z2 has a baby soother shape. The total
16. de Broglie concept as well as uncertainty principle have numl?er of orbitals present in a main energy level is 'n 2 '.
no significance in everyday life because they are significant for (iv) Spin quantum number (s): It describes the spin orthe
only microscopic systems. electron. It has values + II2 and '-'1/2. (+) signifies clockwise
17. When radiations of a certain minimum frequency (vo ), spinning and (-) signifies anticlockwise spinning.
called threshold frequency, strike the surface of a metal, electrons
called photoelectrons are ejected from the surface. The ininimum
Spin angular.momentum = ~s(s +. l)..!!...-
2n
energy required to eject the electrons from the metal surface is
called threshold energy or work function.
( where, s = ~J
Absorbed energy = Threshold energy + Kinetic energy of
photoelectrons Total spin of an atom or an ion = n x.!.; where, 'n' is the
E=Eo +KE 2
number of unpaired electrons.
1 2
hv=hvo +-mv Spin !Jlultiplicity of an atom = (2rs + 1)
2
I
112 G.R.B. PHYSICAL CHEMISTRY FOR COMPETITIONS

Singlet state (Nonnal) Singlet excited 22. Autbau principle: Aufbau is a Gennan word meaning
i.! ± building up. The electrons are filled in various orbitals in an order
of their increasing energies. An orbital of lowest energy is filled
Spin multiplicity 1 first. The sequence of orbitals in the order of their increasing
;: : n:s+ 1 Spin multiplicity 1 energy is:
]S, 2s, 2p,3s, 3p,4s, 3d, 4p,5s,4d, 5p,6Y, 4.f, 5d, 6p,7s, 5f,6d, ...
=2xO+l=1 The energy of the orbitals is governed by (n +/) rule.
Triplet excited state (i) Subshell with lower of (n + I) has lower energy, hence
1 filled fIrSt, e.g., .
i 3p(n + 1 = 4) will be filled before 3d(n + I 5).

Spin multiplicity (ii) When (n + I) values are same, then the subshell with
lower value of 'n' is filled fIrst, e. g. ,
2XG+~)+1=3 3p(n+I=4) will be filled before 4s(n+1 4.)
because 3p has lower value of n..
19. (i) Number of subshells in a shell =n 23. Hund's rule: No electron pairing takes place in the
orbitals in a subenergy shell until each orbital is occupied by one
(ii) Number of maximum orbitals in ashell =n 2
.. ~le.ctro.n~ ~it1!~p..!U:~!Ie.Lspi!1' Ex:a(;.t1YJ!1l1f:fill~ar~~~llYiJi.lle.~
(iii) Number ofmaxiInum orbitals in a subshell =21 + 1 orbitals make the atoms more stable, i. e., p ,p ,d , d ,.f
(iv) Maximum number of electrons in a shell ;:::: 2n 2
(v) MaximUm number of electrons in a subshell
and /4 configurations are most stable.
All those atoms which consist of at least one orbital singly
=
2(21+ 1)
occupied behave as paramagnetic while all those atoms in
(vi) Z-component of the angular momentum depends
which all the orbitals are doubly occupied are diamagnetic in
upon magnetic quantum number and is given as:
nature.
, h) . Magneticmoment;::::~n(n+2)BM .
Lz =ml21t n = number of unpaired electrons
(vii) Number ofradiaVspherical nodes in any orbital 24. Half-filled and fully-filled suhshells have extra stability
due to greater exchange' energy and spherical symmetry
=(n~/:"'l)
around the nucleus.
]s orbital has no node; 2s orbital has one spherical
25. It is only dz2 orbitals which do not have four lobes like
node; 2porbitalhas no spherical node; 3porbital has
other d-orbitals.
one spherical node.
26. The d-orbital whose lobes lie along the axes is d 2 2 •
(viii) Schrodinger wave equation does not give spin x -)'
quantum number. . 27. Wave mechanical model of atom: It was
(ix) A plane passing through the nucleus at which the SchrOdinger who developed a new model known as wave
probability of finding the electron is zero, is called mechanical model of atom by incorporating the conclusions of de
nodal plane. Broglie and Heisenberg uncertainty principles. He derived an
The number of nodal plane in an orbital 1 = ... equation, known as Schrodinger equation.
.\'-Orbitals have no nodal plane; p-orbitals have one d 2", d 2", d 2", 8x2m
nodal pJane, d-orbitals have two nodal planes and so - -2+ - - + - - + (E-V)", 0
on. dx dy2 dz2 .

20. Pauli's exclusion principle: No two electrons in an The solution of the equation provides data which enables us to
atom can have the same set of all the four quantum numbers, i. e., calculate the probability of finding an electron of specific energy.
an orbital canuot have more than 2 electrons because three It is pOssible to determine the regions of space around the nucleus
quantum numbers (principal, azimuthal and magnetic) at the where there is maximum probability of locating an electron of
most may be same but the fourth must be different, i. e., spins specific energy. This region of space is tenned orbital.
must be in opposite directions. It is possible to calculate the
'" is the amplitude of the wave at a point with coordinates
maximum number of electrons which can be acconunodated on a
x, yand z. 'E' is total energy called eigen value and V denotes the
main energy sheD or slibenergy shell on the basis of this potential energy of the electron;
principle.
",2 gives the probability offmding the electron at (x, yand z).
21. Electronic configuration: The arrangement of
electrons in various shells, subshells and orbitals in an atgm is Operator form of the equation can be given as:
tenned electronic configuration. It is wrltten in tenns of nl x HW = E",
where n indicates the order of shell, 1indicates the subshell and x
the number of electrons present in the subshell.
ATOMIC STRUCTURE 113.

if ;: [- 4
8n m
1
A? :.- V called Hamiltonian operator·
.
30. Plot of radial probability density 'R 1 ,:

=T+V
T= Kinetic energy operator t \. 1ISS. ." . . .

V = Potential energy operator R2~


28. Complete wave function can be given as.: r --II>
",(r,O,<I»= R(r) ; e(o) 11>(<1»
'--v--' '---v---"
Radial part Angular part

Dependence of the wave function on quantum number can be


given as:
'" nlm (r,O, <1»= R~, I (r) e1m(0) ~m (<I» ,1s 2s 2p
29. Graph of radial wave fUnction 'R': At node" the
value of' R' changes from positive to negative.

2s 2p

In the plot of radial probability against 'r', number peaks; i. e.,


. region of maximum probability n - l.
r--ll>·

Number of radial nodes = (1'1 -I-I),


~"~"."... ...",,,,-.-- --_._,.""'_......--------

114 I G.R. B. PHYSIC~L GHI=MISTRy' FOR COMPETITIONS

1. Match the following: (c) Paschen series (r) Absorptionspectrum


[A] ( d) Brackett series (s) Ultraviolet region
(i) Aufbau principle (a)' Line spectrum in visible region
(ii) de Broglie
" ' '

(b) Orientation of an eI(:ctron in an


[D] Match the List-lwith List~II:'
, 'orbital List~: List-ll
(iii) Angular momentum (c) Photon'
(a) K-shelL (P) Electrons in elliptical
, (iv) Hund's rule (d) A ,hl(,,!v) " orbit
: (v) Barmers~ries' ,:n . (i)'tiedrofu6 conflgm.~ii~n
(b) L-shell (q) Electrons in circular orbit
(vi) Planck's law (f) mvr
(c) Hydrogen atom, (r) Shell of lowes,! energy
[B] ",
(i) , Thomson (d) Boron atom in groun,d (s) B04(s atomic model
(a)' Exclusion principle
state
(ii) Pauli (b) Radioactivity
(iii) Becquerel' , ' , (c) "Atomiemodel [E] Match the ions of List-I with the properties ofList~I1:
(iv) ,S~ddy (d) Cathode rays Liit-I List-ll
(v) 'Bohr (e) Neutron
(a) Mn 2 + (p) Diamagnetic
(vi) Chadwick (f) Isotopes
[C] (b) V 2+ (q) Paramagnetic
(i) Cathode rays (a) Helium nuclei (c) Zn 2+ (r) Coloured compounds , .
(ii) Dumb-bell ,(b) Uncertainty principle (d) Ti4+ (s) Magnetic moment = 2.82 BM
(iii) Alpha particles (c) Electromagnetic radiation
[F] Match the List-I with List-II:
(iv) Moseley (d) p -orbital
(v) Heisenberg (e) Atomic number List.;.I List-ll
(vi) X~rays (f) Electrons (a) Mg2+ (P) Zero spin multiplicity
2. Matrix Matching Problems (For lIT Aspirants):
(b) Fe 2+ (q) Spin multiplicity = 3
[A] Match the Colurnn-I and Column-II:
(c) C0 3 + (r) Total spin = 0 ,
Column-I ColulIID-D
(d) Ca 2+ (s) Total spin :::: 2
(a) X-rays (P) Davisson and Germer
, experiment , [G] Match the prope.rties of List-I with the formulae in List-II:
(b)' Atomic number (q) Crystal structure List-I ' . List'-ll
determination , determination
(c) Dual nature of matter (r) Moseley's law
(a) Angular momentum of ' (P) JI(l+I)~
electron 2n
(d) Dual nature of radiation (s) Bragg's law (b) Orbital, angular (q) lro
[B] Match the Columil-I and Column-II: momentum
(c) Wavelength of matter (r) nhl2n
Column'" Co)uDID-ll wave
(a) Lyman series (P) Visible region (d) Quantised. value( s) (s) hlp
(b) Balmer series (q) 'UVregion [H] Match the orbitals of List-I with the nodal properties of
"'0·
(c) Pfund ,series (r) IR region List-II:. '
(d) Light emitted by, sodium (s) Line emission spectrum List-I·, List-n
lamp
(a) 2s (P) , Angular node = 1
[C] Match the List-I with List-II in hydrogen atom spectrum:
(b) Is //I" (q) Radial node 0
List-I List-II (c) 2p (r) Radial node = 1
t.:
(a) Lyman series (P) Visible region , (d) 3p (s) Angular node = 0
, (6) Balmer series (q) Infrared region
I
ATOMIC STRUCTURE
"r 115
[1] Match the dectronic transitions of List-I with spectral [M] Match the List-I with List-II:
properties o~ List-1I: ~:;:T)Hl?f' ~
, List-I List-ll
List-I List-ll
(a) Radius of nth orbital (P) Inversely pj-opOltional to Z
(a) n=6~n 3 (P) 10 lines in the spectrum
(b) Energy of nth shell (q) Integral mul!ip!e' of h/21t
(b) n=7~n=3 (q) Spectral lines in visible
region
(c) Angular momentum of (r) Proportional to n 2
electron
(c) n= 5.~,n=2 (r) 6 lines in the spectrum
(d) Velo<.;ity of electron in nth (s) Inversely proportional to
(d) n=6~n 2 (s) Spectral lines in infrared orbit 'n'
region
[N] Match the entries in Column-! with the correctly related
[1] Match the List-1 with List-II: ~·quantumni.1mbet(s) inColulnn-II:
List:.t List·ll Column.l
,"

(a) Radius of eleGtron orbit (P) Principal quantum (a) Orbital angular momentum (P) Principal quantum
number of the electron in a hydrogen- number
(b) Energy of electron (q)-Azimutbalquantum. like at.Qmi,G orl:>ita,l
number (b) A hydrogen-like one electron (q) Azimuthal quarttum
(c) Energy of subshell (r) Magnetic quantum wave function obeying Pauli number
number principle

(d) , Orientation of the atomic (s) Spin quantum number (c) Shape, size and ori,entation (r) Magnetic quantum
of hydrogen-like atomic , number
orbitals
orbitals
[K] M.atch the List-I with List-II: (d) Probability density of (s) Electron spin quantum
List-I List-ll electron at the nucleus in number
hydrogen-like atom
(a) ElectrolJ. cannot exist in (P) de Broglie wave
[0] Match the List-lWith List-II: . ,(lIT 2006)
the nucleus :~!:-.

(b) Microscopic particles in (q) Electromagnetic wave List-I List-ll


, motion are associated with' (a) Wave nature of radiation (1') Photoelectric'
(c) No medium is required for (r) Uncertainty principle effect
propagation (b) Photon nature of radiation , (q) Compton effect
(d) Concept of orbit was (s) Transverse wave (c) , I,nteraction of a photon with an (r) Diffraction
replaced by orbital electron, such that quantum energy
is slightly equal to or greater than
[L] According to Bohr theory: (lIT 2006)
the binding ,energy of electron, is,
En == Total energy more likely to result in:
K /I = Kinetic energy
VII = Potential energy l.:.d) Interaction of a photon with an (s) Interference
,
electron, such that photon energy is
r" Radius of nth orbit
much greater than the binding
Match the following:
energy of electron, is more likely to
Column-I 'Column-ll result in:

(a) VnlKn ?' (p) 0 [P] Match the Column-! with Column-II:
(b) lfradius of nth orbit oc Enx; x = ? (q) Column-I Column-ll
(c) Angular momentum in lowest orbital (r) - 2 (a) Orbital angular (P) ~s (s + l)h/2n
momentum of an electron
(d) 1 ocZY ; y=? (s) 1
r" (b) Angular momentwn of (q) ~n (n + 2) BM
electron
(c) Spin angular mQ,mentum (r) nh/2n
of electron
(d) Magnetic 'moment of atom (s) h/2n
116 G.R.B. PHYSICAL CHEMI,STRY FOR COMPETITIONS·

[Q] Match the Coh,unn-l with Column-II: [R] Match the Column-I with Column-II:
Column-l (;vlumD~lI .- Column-I Column-II
(a) Scintillation (P) , Wave nature (a) Radial function R (P) Principal quantum
(b) Photoelectric effect (q) P~\ltiCle nature' number 'n'
(c) Diffraction (r) Particle nature dominates (b) Angular function e (q) Azimuthal quantum
over wave nature number 'I'
(d) Prin~iple of electron " (s) Wave nature dominates (c) Angular function CP (r) Magnetic quanfum
microscope over particle nature number'm'
(d) Quantized angular (s) Spin quantum number's'
momentum

1. [A] (i-e); (ii-d); (iii-f); (iv-b); (v-a); (vi-'c) [I] (a-,r, s) (b-p, s) (c~, r) (d-p, q)
[B] (i-d); (ii-a); (iii-b); (iv-'---f); (v-c); (vi-e) [J] (a-p) (b-p) (c--p, r) (d-r)
[C] (i-f); (ii-d); (iii-:-:a); (iv-e); (v-,b);(vi-c) [Kl{a-rHb--p)(c~;s) (d-r)-
2. [A] (a-q, r, s) (I~r) (c-p) (d-dpes not match) [L] (a-r) (~)(c-p) (d-s)
[B] (a-q, s) (b-p, s) (c-r, s) (d-fP, s) [M] (a-r, p) (b-r) (c-q) (d-s)
[C] (a-r, s) (b-p) (c-q) (d-q)~ [N] (a-p) (b-s) (c-p, q, r) (d-p, q)
[D] (a~, r) (b-p, q) (c-s) (d-p, q) [0] (a-r, s) (b-p, q) (c-p) (d-q)
[E] (a-q, r) (~, r, s) (c~p) (d-p) [P] (a-s) (b-r) (c-p) (d-q)
[F] (a-p, r) (~, s) (c~, s) (d-p, r) [Q] (a-q) (b-r) (c-p) (d-p, s)
[G] (a~, r)~) (c-s) (d~, r)
[R] (a-p, q) (~, r) (c-r) (d~, s)
[H] (a-r, s) (~, s) (c~, p) (d-p, r)

RACTICEPROBLE • '
1. An atom of an element contains 13 electrons. Its nucleus has 14 4. Calculate the number of neutrons in 18 mL of water. (Density
neutrons. Find out its atomic number and approximate atomic of water = 1)
mass. An isotope has atomic mass 2 units higher. What will be [Ans. 48.16 x 1023 ]
the number of protons, neutron~ and electrons in the isotope?
[Hint: One molecule of water contains = 8 neutrons J
[Ans. At. No. =13, atomic mass =27; the isotope will have
same number of protons and electrons 13 but neutrons will be 5. Find (i) the total number of neutrons and (ii) the total mass of
14 + 2 = 16] neutrons in 7 mg of 14C (assuming that mass of neutron
2. From the following find out groups of isotopes, isobars and mass of hydrogen atom). '
isotones: [Ans.(i) 24.08 x 1020 and (ii) 4 mg]
16 0 391(, 14C 239U 14N 40C 238U 77G'
8 '19 6 ' 92 , 7 , 20 a, 92 '-32 e, 6. Calculate the wavelength of a photon in Angstroms having an
17As 18 0 76G 78S energy of 1 electron volt.
33 ' 8 '32 e, 34 e
[Ans. Isotopes--same at. no. but different at. masses. [Hint: 1 eV = i.602 x 10- 19 joule;
16 0 18 0' 239 238 ' 77Ge 76 Ge h 6.62 x 10-34 J-s, c='3 X 108 ms- I
8 ' 8 '92 U ',92 U , 32 '32
h· c . 7 3
Isobars-same atomic masses but different at. numbers A= - :::: 12.42 x 10- m = 12.42 x 10 A]
E
14C 14N_ 77Ge 771>c
6 ',7 '32 '33'~ 7. ' A photon of light with wavelength 6000 A has an energy E.
Calculate the wavelength of photon of a light which
Isotones-same number of neutrons,
corresponds to an energy equal to 2E.
I~O, l~C; :gK, jgca;T:As, nSe] [Ans. 3000 A]
3. An element has atomic' number 30. Its cation has 2 units 8. Calculate the energy in kilocalorie per mol of the photons of
positive charge. How many protons and electrons are present an electromagnetic radiation of wavel~ngth 5700 A.
in the cation? [Ans. 56.3 kca1 per mol]
[Ans. Protons = 30, Electrons =28]
ATOMIC STRUCTURE I 117

9. Light of what frequency ·and wavelength is needed to ionise 6.626 X 10-34


sodium atom. The ionisation potential of sodium is
[Hint: A=----=j=======~========~
.J2Em
8.2 x 10-19 I.
[Ans. v = 1.238 X 1015 Hz; A = 242 nm]
=2.2 x 10- 12 m]
19. Determine the number of revolutions made by an electron in
10. Determine the energy of 1 mole photons of radiations whose
one second in the 2nd Bohr orbit of H-atom.
frequency is 5 x 1010 S-I. (h == 6.62 x 10-34 1- s) 2nr
[Ans. n= ]
. [Ans. 19.91] v
2
11. Find e / m for He + ion and compare with that for electron, 20. What is the speed of an electron whose de Broglie wavelength
[Ans. 4.87 x 107 coulomb kg-I] is 0.1 nm? By what potential difference, must have such an
electron accelerated from an initial speed zero?
12. A ball of mass 100 g is moving with a velocity ofl 00 m sec -1.
[Ans. 7.28 x 106 m/sec; 150 V]
Find its wavelength.
21. A green ball weighs 75 g; it is travelling towards observer at a
~ = 6.626 x 10-
34
[Hint: A == 6.626 x 10-35 m] speed of 400 crn/sec. The ball emits light of wavelength
mv 0.1 x 100
5 x 10-5 cm. Assuming that the error in the position of ball is
13. Calculate the wavelength of radiation and energy per mol
the same as wavelength pfitself, calculate error in the
necessary to ionize a hydrogen atom in the ground state.
momentum of the green ball.
[Ans. A == 9.12 X 10-8 m; 1313 kJ 1 mol] h
<[Hint:I:!x·t1p?-<-<
14. Bond energy of F2 is 150 kJ mol-I. Calculate the minimum 4n
frequency of photon to break this bond. h
t1p?:.--
[Ans. 3.759 x 10 14 8- 1] 41t I:!x
15. If an Einstein (E ) is the total energy absorbed by J mole of a t1 > h
.p - 4nA
substance and each molecule absorbs one quantum ofenergy,
then calculate the value of'E' in terms of A in cm. 6.626 x 10-27 23
t1p "" '" 1. 055x 10- ]
8 4 x 3.14 x 5 x
1.198 x 10 . 1-1 ]
[A ns. A < erg mo
22. What is the relationship between the e V and the wavelength in
16. How many chlorine atoms can you ionize hi. the process? metre of the energetically equivalent photons?
Cl ~ Cl+ + e by the energy liberated from the following [Ans. A == 12.4237 X 10-7 metre]
process: 23. What is the velocity ofanelectron(m= 9.1 Ix 10-31 kg) in the
CI + e ~ cr for 6 x 1023 atoms innermost orbit of the hydrogen atom?
given that electron affinity of chlorine is 3.61 eV and (Bohr radius 0.529 x 10- 10 m)
ionization energy of chlorine is 17.422 e V.
. [Ans. 2.187 x 106 m/sec]
[Ans. L24 x 1023 atoms]
24. In a hydrogen atom, an electron jumps from the thud orbit to
17. Find the velocity (ms-'I) of electron in first Bohr orbit of the first orbit. Find out the frequency and wavelength of the·
radius ao. Also find the de Broglie wavetength (in 'm'). Find spectral line. (RH L09678 x 107 m:- I )
the orbital angular momentum of 2 p orbital of hydrogen atom
[Ans. 2.925 x 1015 Hz, 1025.6A]
in units of hI 2Ti.
2.188 x 10 6 . _I
. 25. The energy of the electron in the second and third Bohr orbits
[Hint: v m sec . of hydrogen atom is -5.42 x 10- 12 erg and -2.41 x 10- 12 erg
. n
respectively~ Calculate the wavelength of the emitted radiation
2188 x 10 6
v= 2188 X 10 6 m sec-I when the electron drops from third to second orbit.
1
[Ans. 6.6 x 103 A]
h 6.626 X 10-34
A=-
mv 9.1 X 10-31 x 2.188 X 106
26. Calcuhi.te the wavelength m
angstroms of the photon that is
emitted when an electron in Bohr orbit n ;, 2 rCtums to the orbit
;:: 3.3 x lO- lo m 11 1 in the hydrogen atom. The ionisation potential of the
ground state oi"hydrogen atom is 2.17 x 10- 11 erg per atom.
Orbital angular momentum = ~ l(l + 1) h <

. . 2n [Hint: Energy of the electron in the I st orbit =- (ionisation


~1(1 + 1) h (": I = 1 for 2p) 11
potential), t1E = (31 4) x 217 x 10- erg per atom]
21t
[ADS. A 1220 A]
=..fi h ]
21t 27. Calculate the wave number for the shortest wavelength
18. The energy of an a-particle is 6.8 x 10-18 J. What will be the transition in BalrDer series of atomic hydrogen. (liT 1996)
wavelength associated with it? ICBSE-PMT (Mains) 2005) [ADS. 27419.25 cm- I ]
1181 G.R.B. PHYSICAL CHEMISTRY FOR COMPETITIONS

28. The wavelength of the first member of the Balmer series of 33. The velocity of an electron in certain Bohr orbit of H-atom
hydrogen is 6563 x IO-IO m. Calculate the wavelength of its bears the ratio I : 275 to the velocity of light. (a) What is the'
second member. quantum number 'n' of the orbit? (b) Calculate the wave
number of the radiation when the electron jumps from (n + I)
[Hint: :1 [;2 -),
== RH and :2 RH [;2 12 J
4 state to ground state.
[Ans. v 9.75 x 104 em-I]
5 16 20
==-x v I 3 X 1010 8
1..1 36 3 27 [Hint: (a) - = orv == == 1.09 X 10 em
c 275 275
nh nh
1..2 == 20 x 6563 X 10- 10,:" 4861 X 10.- 10 m] . v == - - == ------_=_~
27 21tmr 21tm X 0.529 x I
29. According to Bohr theory, the electronic energy of hydrogen h
or 11= -------:~-
atom in the nth Bohr orbit is given by, 21tm X 0.529 x XV
2176 x 10- 19 6.625 X 10-27
Ell == J
2 X 3.14 x 9.1 x
Calculate theiongest wavelength of light that will be needed
=2
to remove an electron from the 2nd orbit of Li 2 + ion. .
(b) Thus, II + 1 = 2 + 1 3. The electron jumps from 3rd orbit to
[Ans. 4.059 x 10-8 m] 1st orbit.]
30. Calculate the frequency, energy.. and .wavelengili<of the
radiation corresponding to spectral. line of lowest frequency in
34. Find out the wavelength of the next line in the series having
Lyman series in the spectra of hydrogen atom. Also c'alculate lines of spectrum of H-atom of wavelengths 6565 A, 4863 A,
4342 A and 4103 A.
. the energy of the corresponding line in the spectra of Li 2+.
(lIT 1991) [Ans. 3972 A]
7
[Ans. I.. == 121'; X 10- m;v = 2.47 x lOIS cycle , [Hint: All these lines are in visible region and thus, belong to
Balmer series, Next line is, therefore, from 7th orbit.]
E 16.36 X 10- 19 j, ELi 2+ ~ EH 9 x i6.36 X 10- 19 J
147.27 X 10-19 J]
35. Which jump is responsible for the wave number of emitted
radiations equal to Q.7490 x 106 m- I in Lyman series of
31. Calculate the ratio of the veIocity oflight and the velocity of
electron in the 2nd orbit. of a hydrogen atom. (Given hydrogen spectrum? (R 1.09678 x 107 m- 1 )
h = 6.624 X 10-27 erg-sec;m . 9.108 x 10- 28 g; [ADs. 3]
r 2.11 X 10-8 cm) 36. Calculate the ionisation energy of the hydrogen atom. How
[Ans. 273.2] . much energy will be required to ionise 1 mole of hydrogen
32. What hydrogen-like ion has the wavelength difference atoms? Given, that the Rydberg constant is 10974 X 107 m- I .
betWeen the first lines of Baf~er and Lyman series equal to [Ans. IE per hydrogen atom = 2.182 x 10-1.8 J
59.3 nm (RH = 109678 em-I)? IE per mole = 1314 kJ mol-I]
[Hint: Wavelength df 1st line in Balmer series, 37. Calculate the ionisation energy of (a) one Li 2 + ion and (b) one
r =Z 2RH
I..B
[l.. _l..J
22 32
2. R Z2
36 H
mqJe ofLi2+ ion. (Given, R == 10974 X 10-7 m- I )
[Ans. (a) 19.638 x 10- 18 J (b) 1.118 X 104 kJ mol-I]
t.. _ 36 38. A series of lines in the spectrum of atomic hydrogen lies at
or
B - 5R Z 656.46 nm, 4i6.27 nm, 439.17 nm and 410.29 nm. What is the
. H
wavelength of the next line in this series? What is the
Wavelength of 1st line in Lyman series is, ionisation energy of the atom when it is in the lower state of
J....
I..L Z2R
B [~
12 l..l
22.
transition?
[Ans. I..nexl 397.15 nm; IE = 3.40 e':l
4 39. A certain line of the Lyman series of hydrogen and a certain
or AL == ----;:-2
. 3xRH Z line of the Balmer series of He + ion have nearly the same
wavelength. To what transition do they belong? Small
Differencel..B - I..L = 59.3 X 10-7 == ~ - ~ differences between their Rydberg constant may be neglected.
5RHZ 3RH Z
[Ans. Hydrogen Helium
1
RHZ2
[36
5" '3
4J 2~1
3 __ 1
4 __ 2
6 __ 2
88 4 __ 1 8 __ 2]
z 2 == 7 . 9.0
59.3 x 10- x 109678 x 15 40.. What element has a hydrogen-like spectrum whose lines have
.. , or Z='3 wavelengths four times shorter than those of atomic hydrogen?
[ADS. He+]
Hydrogenclike species is Li 2+.]
ATOMIC STRUCTURE 119

41. What lines of atomic hydrogen absorption spectrum fall 53. CalCulate the momentum of the particle which has de Broglie
within the wavelength ranges from 945 to 130 nm? . wavelength I A (10- 10 m)and h = 6;6 x 10-34 J- se<;.
[Ans. 97.3; 102.6; 121.6 nm] [Ans. 6.6 x 10-24 kgm sec-I]
42. The binding energy of an electron in the ground state of an
54. The uncertainty of a particle in momentum is 3.3 x 10-2
atom is eq:ualto 24.6 eV Find the energy required to remove
both the electrons from the atom. . ,kg ms-I. Calculate the uncertainty in its positiQ'Q.
[Ans. 79 eV] (h = 6.6 x 10-34 J- sec)
43. What is the ratio of the speeds of an electron in the first and [Ans. 3.1 x 10- 14 m]
second orbits of a hydrogen atom?
55. Calculate the product of uncertainties of displacement and
[Ans. 2; 1] .
velocity ofa moving electron having a mass 9,1 x 10-28 g.
44. Find out the number of waves made by a Bohr electron in one
complete revolution in its third orbit. (lIT 1994) [Ans. 5.77 x 10-5 m2 S-I]
[Ans. 3] [Hint: ~ ·Ilv ~ ~]
45. The wave number of first line in Balmer series of hydrogen is 4~ .
15200 cm -I. What is the wave number of first line in Balmer 56. (a) A transition metal cation x 3 + has magnetic morrient .,[35
series ofBe 3+? BM. What is the atomic number of x 3+?
[Ans. 2.43 x 105 cm- I ] (b) Select the coloured ion and the ion having maximum
46. Calculate the speed of an electron in thegroJln(istate of
hydrogen atom. What ·fraction of the speed of light is this
1111 1111 11 1 D
value? How long does it take for the electron to complete one magnetic moment (i) Fe +, (ii) Cu +, (iU) Sc 3+ and
2

revolution around the nucleus? How many times does the


electron travel around the nucleus in one second? 2
(b)Fe + ~ 11111 11 11 11 I·
[Ans. 2.186 x Hf ms-J ; 7.29 X 10-3 ]
47. An electron, in a hydrogen atom, in its ground state absorbs
1.5 times as much energy as the minimum required for its
escape (i.e~ 13.6 eV) from the atom. Calculate the value of A
for the emitted electron. (iy) Mn 2 +

[Ans. 4.69 A]
[Hint: (a) 26, 26Fe~ 3d 6 4s 2

48. The radius of the fourth orbit of hydrogen is 0.85 nm. Fe3i; ~ 3d 5 4sO
Calculate the velocity of an electron in this orbit
(me 9.1 x 10-31 kg).
Il =In(n + 2) =..J5X7 = 55
Both these ions will be coloured and magnetic moment of Fe2 +
[Ans. 5.44 x 105 m sec-I]
will be greater.]
49. A beam of electrons accelerated with 4.64 V was passed 57. A photon of wavelength 4000 A strikes a metal surface, the
through a tube having mercury vapours. As a result of work function of the metal being 2.13 eV Calculate (i) energy
absorption, electronic changes occurred with mercury atoms 9f the photon in eV; (ii) kinetic energy of. the emitted
and light was emitted. If the full energy of single electron was photoelectron and (iii) velocity of the photoelectron.
converted into light, what was the wave number of emitted
[Ans.E = 3.10 eV; KE= 0.97 eV; Velocity = 5.85 x 105 Ins-I]
light?
[Hint: I eV = 1602 X 10-19 J]
[Ans. : 3.75 x 104 em- I I
so. An electron jumps from an outer orbit to an inner orbit with
58. Calculate the ratio between the wavelengths of an electron and
the energy difference of 3.0 eV. What will be the wavelength a proton, if the proton is moving at half the velocity of the
of the line and in what region does the emission take. place? electron (mass of the proton = 167 x 10-27 kg; mass of the
[Ans. A 4140A; visible region] electron = 9.11 x 10-28 g).
[Hint: leV 1.6 X 10- 12 erg] [Ans. 9.2455 x 10-2 m]
h
51 •. The first ionisation energy of a certain atom took place with an [Hint: Applyde. Broglie equation, A
absorption of radiation of frequency 1.5 x 1018 cycle per
second. Calculate its ionisation energy in calorie per gram atom. Wavelength of electron
[Ans. 1.43 x 108 cal l
[Hint: 1 calorie 4.18 x 107 erg Wavelength of proton
Apply E = h x v x Avogadro's number]
52. Find the wavelength associated' with an electron which has 59. A moving electron has 2.8 x 10-25 J of kinetic energy.
mass 9.1 x 10-28 g and 'is moving with a velocity of lOS cm Calculate its wavelength.
sec-I. (Given h 6.625 x 10- 27
erg-sec) (Mass of electron = 9.1 x 10-3,1 kg)

[ Ans. A 7.28 x 10- ern] 5


[Ans. 9.2455 'X IO-7 m ]
J20 G.R.B. PHYSICAL CHEMISTRY FOR COMPETITIONS

66. An electron has mass 9.1 x 10-28 g and is moving with a


[Hint: v = ~2XKE
_.- - ':= 784.46 ms -;IA.= -h]
m . - mv velocity of 105 cm/sec. Calculate its kinetic energy and
60. Helium has ·mass number 4 and atomic number 2. Calculate wavelength when h = 6.626 X 10-27 erg-sec.
the nuclear binding energy per nucleon (mass of neutron [Ans. 4.55 x 10-8 erg; A =7.28 X 10-5 cm]
= 1.00893 amu and proton = 1.00814 amu, He = 4.0039 amu
67. Calculate the de Broglie wavelengths of an electron and a
and mass of electron is negligible).
proton having same kinetic energy of 100 eV
[Ans. 7.038 MeV]
[Ans. Ae = 123 pm; Ap = 2.86 pm]
61. Calculate the mass defect and binding energy per nucleon of
1~9v'hichhas a mass 15.99491 amu. 68. Work function of sodium is 2.5 eV Predict whether the
wavelength 6500 A is suitable for a photoelectron or not?
Mass of neutron = 1.008655 amu
[Ails. No ejection]
Mass of proton = 1.007277 amu
Mass of electron = 0.0005486 amu
69. Calculate the de Broglie wavelength associated with a helium
atom iri a helium gas sample at 27°C and 1 atm pressure.
1 amu = 931.5 MeV
[Ans. 7.3 x 10- 11 metre]
[Ans. 7.976 MeV/nucleon]
14
62. The circumference of the second Bohr orbit of electron in the 70. The threshold frequency for a certain metal is 3.3 x 10
hydrogen atom is 600 nm. Calculate the potential difference to cycle/sec. If incident light on the metal has a cut-off frequency
which the electron has to be subjected so that the electron 8.2 x 1014 cycle/sec, calculate the cut-off potential for the
stops. The electron had the de Broglie wavelength
photoeleetron;
corresponding to the circumferen~e. -
[Ans; 2 volt]
' N b· f . " Circumference
[Hint. urn er 0 waves n = - - - - - 71. .Can you locate the electron within 0.005 nm?
Wavelength
[Ans. No.]
nA= 2nr [Hint: Use uncertainty principle to determine uncertainty in
2A = 600 velocity.
A= 300nm h
Let stopping potential is Vo. ~v2:---
4nm&
eVo =-1 mv 2 . ... (i) On substitution, you will get,
1
_~v 2: 1.16x \07 ms- I
A=·~
mv Velocity of electron is therefore expeCted to be as high as velocity
h of light. We may say that the velocity of electron is uncertain
V=- ... (ii)
Am within 0.005 nm.]
From equations (i) and (ii), 72. The photoelectric cut~offvoltage in a certain experiment is 1.5
volt. - What is the maximum kinetic energy of the
.
eVo =.!.2 m (~)2
Am
photoelectrons emitted?
[Ans. 2.4 x 10-19 joule]
2 o
h 73. A proton is accelerated to one-tenth the velocity of light. If its
V. -
0- 2mA2e velocity can be measured with a precision of ± 1%, what must
(6.626 x 1O-34 i be its uncertainty in position?
2 x (9.1 x 10-31 ) x (300 x 10-9)2 x 1.6 X 10- 19 (h = 6.6 x 10-34 J-s; mass of proton = 1.66 x 10-27 kg)
[Ans. L05 x 10- 14 m]
= 1.675 X 10-5 V]
74. In a photoelectric effect experiment, irradiation ofa metal with
63. The velocity of an electron of mas~ 9.1 x 10-31 kg moving
light of frequency 5.2 x 1014 sec- I yields electrons with
round the nucleus in the Bohr orbit (diameter of the orbit is
maximum kinetic energy 1.3 x 10- 19 '1. Calculate the Vo of the
1.058 A) is 2.2 x 10-6 m sec-I. If momentum can be measured
metal.
within the accuracy of 1%, then calculate uncertainty in
position (&) of the electron. [Ans. 3.2 x 10 14 sec-I]
[Ans. 2.64 x 103 metre] 7S. Calculate the wavelength of a CO2 molecule moving with a
64. An electron wave has wavelengtIi 1 A. Calculate the potential velocity of 440 m sec-I.
with which the electron is accelerated. [Ans. 2.06 x 10- 11 metre]
[..ins. 0.0826 volt] 76. The predominant yellow line in the speCtrum of a sodium
65. Calc1)late the de Broglie wavelength .associated with an vapour lamp has a wavelength of 590 nm. What minimum
a~particle having an energy of 7.7 x 10- 13 J and a mass of accelerating potential is needed to excite this line in an
6.6-x" 10':'24 g. (h';" 6.6 x 10-34 J-s) - electron tube having sodium vapours?
13 [Ans. 2.11 volt]
[Alis. 6.56 x 10- cm]

ATOMIC STRUCTURE. 121

77. Find out the wavelength of a track star running a 100 metre I mv 2 :::K Zex e
dash in 10.1 sec, if its weight is 75 kg. 2 ro
[Ans. 8.92 x 10-37 m] For Rutherford experiment,
78. At what velocity ratio are the wavelengths of an electron and a
proton equal? . .! mv 2 5.5 MeV::: 5.5 x )(1' x 1.6 x 10-19 J::: 8.8 X 10-13 J
2
24
(me == 9.1 x 10-28 g and mp = 1.6725 X 10- g) 9
8.8 X 10- 13 = 9x10 x2x79x
[Ans. Ve = 1.8 X 103] 10
vp 15
10 == 4.136 X 10- m
79. Through what potential difference must an electron pass to
have a wavelength of500 A? 10 = 4.IHm]
[Ans. 6.03 x 10-:4 eV ] 84. Photoelectrons are liberated by ultraviolet light of wavelength
h 3000 A from a metallic surface for which the photoelectric
[Hint: Use A== ~] threshold is 4000 A. Calculate de Broglie wavelength of
,,2eV m
electrons emitted with maximum kinetic energy.
80. Calculate the velocity of an ~-particle which begins to reverse
[Ans. A == 12 X 10-9 m]
its direction at a distance on X 10- 14 m from a scattering gold
nucleus (Z == 79). [Hint:
[Ans. 2.346 x 107 m! sec] KE = Quantum energy - Threshold energy. ..
81. Two hydrog~n atoms collide head.on and end up with zero 6.626 X 10-34 X3 X 10& 6.626 X 10-34 x 3 X 108
kinetic energy. E~ch then emits a ph:oton with a wavelength 3000x 4000 x
121.6 nm. Which transition leads to this wavelength? How
fast were the hydrogei1 atoms travelling before the collision? == 6.626 X 10- 19 4.9695 X 10-19
(Given, RH == 1.097 X 107 m- I and"'H == 1.67 X 10-27 kg) == 16565 X 10-19 joule
IAns. nl I; 112 = 2; 4.43 X 10 4
m sec-I]
[Hint: Wavelength is in UV region; thus nl will be 1.
.! mv 2 1.6565 x 10- 19
2
7
-----;;,::: 1.097 X 10 x 12 X'(112
12L6 x
- :~)
"1.
m4v 2 = 2 x 16565 X 10-19 x 9.1 X 10-31
mv = 5.49 x 10-25
112 2 34
I 2 he A ::: ...!:. = 6.626 X 10- = 1.2 X 10-9 m]
.- mv :::- mv 5.49 x
2 A
34 8 8S. Show that de Broglie wavelength of electrons accelerated V
-I x.
167 X 10-27 XV
2
:::
6.626 X 10- x 3 X 10
------;c-~-
volt is very nearly given by:
2 1216 x
v 4.43 X 104 m sec-I] AOn A) Iv50)112
(
82. Show that the wavelength of electrons moving at a velocity
very small compared to that of light and with a kinetic energy
of V electron volt can be written as,
[Hint: A= b
v'l,eVm
~
.A=~X
12.268 10-8 cm 2 ]1/2
",A= _h_ x 1020 A
.JV [ 2eVm
Use the relation, A =
[Hint: h
.J2Em =[ (6.626 x 10- )2 x 10
34
JI/2
20
[150J1I2]
Here, h = Planck's constant 2 x 16 X 10- 19 x V x 9.1 X 10-31 V
m ::: 9.1 X 10-28 g (mass of e- )
86. A 1 MeV proton is sent against a gold leaf (Z = 79). Calculate
E Kinetic energy of electron
== V 12
, eV== V x 1.6 X 10- erg]
the distance of closest approach for head-on collision.
[Ans. 1.137 x 10- 13 m]
83. What is the distance of closest approach to the nucleus of an
a-particie whiCh undergoes scattering by 1800 in z,} .
Geiger-Marsden experiment? [Hint: d == . 2 ' Do hke Q.No. 83]
. 41tco(M mv )
[Ans. 10 = 4.13 fm]
87. What is the energy, momentum and wavelength of the photon
[Hint: For closest approach, emitted by a hydrogen atom when an electron makes a
transition from n == 2 to n I? Given that ionization potential
is 13.6 eV.

122 .1 G.R.B. PHYSICAL CHEMISTRY FOR COMPETITIONS

[Ans. 16.32 x lO-19 J, 5.44 X 10-27 kg m I sec, 1218 A]


[Hint: IE == Z2 x 2l.69 X lO-19 J
[Hint: E I ';' -13.6 eV
3
-13.6 V
11180 X 10 = Z2 x 21.69 X lO-19
E2 --e 6.023 X 12
4
3 Z'" 3] .
till= x13.6eV 90. Suppose 10- 17 J of light energy is needed by the interior of the
4
human eye to see an object. How many photons of green light
= 0.75 x 13.6 x 1.6 x lO-19 J = l.632 X lO-18 J
(A 550 nm) are needed to generate ihis minimum amount of
he l.632 x 10- 18
energy?
A [Ans. 28]
34 8 91. How many hydrogen atoms in the ground state are excited by
A=6.626XIO- X3XI0 =1218xlO- lo m=1218A means of monochromatic radiation of wavelength 970.6 A.
l.632 x How many different lines are possible in the resulting
Ao=!!. emission spectrum? Find the longest wavelength among these.
p [Ans. Six different lines, A 1215.6 A] 0=
34
-h -_ 6.626 X 10- . x 10- k.g-m I sec ]
_ 544 27
P - E _ he
A 1218 x . [Hint: En .1 - A
88. Calculate the orbital angular momentum of the following -2169 X 10- 19 21.69 x 10'::1-9 6.6.26x 10-34 x 3 x- j()s---
orbitals: - - - ;2 ; - - - + - - - -
n 1 970.6 X 10- 10
(a) 3p (b) 3d (c) 3s
[Ans. (a) Jili (b)..J6n (c) 0] n",,4 4
[Hint: (a) fil ~l(l + 1) h for 3p, I = I =Ji
2rc
n !
A
=RZ 2 (~ni ni1 J' '3 -+--.---f-r-..3

(b) III =..J6 n for 3d, I 2


(c) III = 0 for 3s, 1=0] 1 = 109677.77
A
x f II ~
1
_!J 4
2
1
89. A single electron system has ionization energy 11180 kJ mor 1• six different lines
Find the number of protons in the nucleus of the system. A ~ 1215.68A]
[Ans. Z == 3]
ATOMIC STRUCTURE I 123

--!?OBlECTlVEQUESTIONS
Set-1: Questions with single correct answer (a) electrons (b) protons (c) neutrons (d) nucleus
13. Rutherford's scattering experiment is related to the size ofthe:
1. The ratio of elm for a cathode ray: (a) nucleus (b) atom (c) electron (d) neutron
(a) varies with a gas in a discharge tube 14. When alpha particles are sent through a thin metal foil, most of
(b) is fixed them go straight through the foil because:
(c) varies with different electrodes (a) alpha particles are much heavier than electrons
(d) is maximum ifhydrogen is taken (b) alpha particles are positively charged
2. Which of the following statements is wrong about cathode . (c) most part of the atom is empty space
rays? (d) alpha particles move with very high velocity
(a) They trav~1 in straight lines towards cathode 15. The radius of an atomic nucleus is of th~ order of:
(b) They produce heating effect [PMT (MP) 1991]
(c) They carry negative charge (a) 10- 10 cm (b) 10- 13 cm
(d) They produce X-rays when strike with material having
(c) 10-15 em (d) 10-8 cm
high atomic masses
3. Cathode rays are: 16. Atomic size is of the order of:
(a) electromagnetic waves (b) stream of a-particles (a) 10-8 cm(b) lO-IO cm (c) 10- 13 cm(dj l(r'6 cm
(c) stream of electrons (d) radiations 17. Atoms may be regarded ·as comprising of protons, neutrons
4. Cathode rays have: and electrons. If the mass attributed by electrons was doubled
(a) mass only (b) charge only and that attributed by neutrons was halved, the atomic mass of
(c) no mass and no charge (d) mass and charge both 12C would be:
S. Which is the correct statement about.proton? (a) approximately the same (b) doubled
(a) It is a nucleus of deuterium (c) reduced approx. 25% (d) approx. halved
(b) It is an ionised hydrogen molecule is. Positive ions are fonned from neutral atoms by the loss of:
(c) It is an ionised hydrogen atom (a) neutrons (b) protons
(d) It is an a-particle (c) nuclear charge (d) electrons
6. Neutron was discovered by: 19. The nitrogen atom has 7 protons and 7 electrons. The nitride
(a) 1.1. Thomson (b) Chadwick ion will have:
(c) Rutherford (d) Priestley (a) 10 protons and 7 electrons
7. The discovery of neutron came very late because: (b) 7 protons and 10 electrons
(a) it is present in nucleus (c) 4 protons and 7 electrons
(b) it is a fundamental particle (d) 4 protons and 10. electrons
(c)· it does npt move 20. A light whose frequency is equal to 6 x 1014 Hz is incident on a
(d) it docs not carry any charge metal whose work function is 2eV (h = 6.63 x 10- 34 Js,
leV'" 1.6 x. 1O- 19 J). The maximum energy of electrons
S. The fundamental particles present in equal numbers in neutral
emitted will be: (VlTEEE 2008)
atoms (atomic number 71) are:
(a) 2.49 eV (b) 4.49 eV (c) 0.49 eV (d) 5.49 eV
(a) protons and electrons (b) neutrons and electrons
[Hint: Absorbed energy = Threshold energy
(c) protons and neutrons (d) protons and positrons
+ Kinetic energy of photoelectrons
9. The nucleus of the atom consists of:
Absorbed energy hv .
(a) protons and neutrons
(b) , protons and electrons
= 6.626 x·lO- 34 x 6 X 10 14
(c) neutrons and electrons = 3.9756 x 10- 19 J
(d) protons, neutrons and electrons 19
3.9756 X 10- =2.49 eV
10. The absolute value of charge on the electron was determined by: 1.6 x 10- 19
(a) l1. Thomson (b) RA. Millikan
2.49 2 eV + Kinetic energy of photoelectron
(c) Rutherford (d) Chadwick Kinetic energy of photoelectron = 0.49 eV]
11. Atomic number of an element represents: (CBSE 1990) • 21. The size ofthe nucleus is measured in:
(a) number of neutrons in the nucleus (a) amu (b) angstrom
(b) atomic mass of an element (c) em (d) fermi
(c) valency of an element
22. The highest value of el m of anode rays has been observed
. (d) number of protons in the nucleus when the discharge tube is filled with:
12. Rutherford's experiment on .scattering of a-particles showed (a) nitrogen (b) oxygen (c) hydrogen (d) helium
for the first time that the atom has: [CMC (Vell~re) 1991]
124 G.R.B. PHYSICAL CHEMISTRY FOR COMPETITIONS

23. The particle with 13 protons and 10 electrons is: . (c) Dempster's mass.spectrograph .
(a) Al atom (b) A1 3 + ion (d) all of the above
(c) nitrogen isotope (d) none ofthese 36. Mass spectrograph helps in the detection of isotopes because
24. Which of the following atoms contains the least number of they:
neutrons? (a) have different atomic masses
(a) 2~iu (b) Z:U (b) have same number of electrons
(c) have same atomic number
(c) 2~jNp
(d) have same atomic masses
25. The number of neutrons in dip~sitive zinc ion (Zn 2+ , with 37. Which of the following statements is incorrect?
mass number 70) is: (a) The charge on an electron and proton are equal and
(a) 34 (b) 36 . (c) 38 (d) 40 opposite
26. Which of the properties of the elements is a whole number? (b) Neutrons have no charge
(a) Atomic mass (b) Atomic number (c) Electrons and protons have the same mass
(c) Atomic radius (d) Atomic volume (d) The mass of a proton and a neutron are nearly the same
27. Increasing order (lowest first) for the values of e / m 38. The charge on positron is. equal to the charge on:
(charge/mass) for electron (e 1 proton (p), neutron (n) and (a) proton (b) electron
a-particle (a lis: (c) a-particle (d) neutron
(a) e, p, n,a (b) n, p, e,a 39. Discovery ofthe nucleus of an atom was due to the experiment
(c) n,p,a,e (d) n,u,p,e carried out by: .
28. The mass of neutron is ofthe order of: (a) Bohr (b) Rutherford
(a) 10-27 kg (b) 10-26 kg (c) Moseley (d) Thomson
(c) 10-25 kg (d) 10-24 kg 40. Isobars are the atoms of: (CBSE 1991)
29. The. atomS of various isotopes of a particular element differ (a) same elements having same atomic number
from each other in the number of: (b) same elements having same atomic mass
(a) electrons in the outer shell only (c) different elements having same atomic mass
.(b) protons in the nucleus (d) none ofthe above
(c) electrons in the inner shell only 41. Which of the following pairs represents isobars?
(d) neutrons in the nucleus (a) ~Reand iRe fi
(b) Mg and ~iMg
30. Isotopes of the same element have: (c) t~K and ~gCa (d) tgK and ?gK
(a) same number of neutrons 42. Na + ion is isoelectronic with: (CPMT 1990)
(b) same number of protons (a) U+ (b) Mg2+ (c) Ca 2+ (d) Ba 2+
(c) same atomic mass
43. The triad .of nuclei that is isotonic is:
(d) different chemical properties
31. Which of the following conditions is incorrect for a well (a) I:C, IjN, I~F (b) l~C, IjN, IgF
behaved wave function (\if)? [EAMCE;T (Engg.) 2010]
(a) \if must be finite (b) \if must be single valued
. (c) \if must be infinite (d) \if must be continuous 44. Sodium atoms and sodium ions:
32. Atomic mass of an element is not a whole number because: (a) are chemically similar
(a) it contains el~trons, protons and neutrons (b) both react vigorously with water
(b) . it contains isotopes (e) have same number of electrons
(c) it contains allotropes (d) have same number of protons .
(d) aU of the above fi g
45. In Cl and Cl, which of the following is false?
33. Nucleons are: (a) Both have 17 protons
(a) protons and neutrons (b) Both have 17 electrons
(b) neutrons and electrons (e) Both have 18 neutrons
(c) protons and electrons (d) Both show same chemical properties
(d) protons, neutrons and electrons 46. Which of the following is isoelectronie with neon?
34. Isotopes of an element have: . (a) 0 2 - (b) F + (c) Mg (d) Na
(a) different chemical and physical properties 47. Neutrino has:
(b) similar chemical and physical properties (a) charge + I, mass 1 (b) charge 0, mass 0
(c) similar chemical but different physical properties (c) charge - L mass 1 (d) charge 0, mass 1
(d) .similar physical and different chemical properties 48. Positronium is the name given to an atom-like combination
35. Isotopes are identified by: . formed between: (JIPMER 1991)
(a) a positron and a proton
(a) positive ray analysis
(b) a positron and a neutron
(b) Astons' mass spectrograph
I
ATOMIC STRUCTURE 125

(c) a positron and an a-particle 61. A 600 W mercury lamp emits monochromatic radiation of
(d) a positron and an electron wavelength 331.3 nm. How many photons are emitted from
49. An isotone of ~~Ge is: the lamp per second? [PET (Kerala) 2010]
(a) ~iGe. (b) j~As (c) I!Se (d) j!Se (h == 6.626 x 10-34 Is, velocity oflight 3x 108 ms-I)
50. Which of the following does not characterise X-I1lYs? (a) Ix 1019 (b) Ix 102°
1
(lIT 1992) (c)lxl02 (d) Ix 1023
(a) The radiations can ionise gases 2
(e) Ix 102
(b) It causes ZnS to fluorescence
[Hint: Power Energy
(c) Deflected by electric and magnetic fields Time
(d) Have wavelengths shorter than ultraviolet rays
600= nhc
51. X-rays are produced when a stream of electrons in an X-ray Axlsec
tube: 34 8
600 == n X 6.626 10- x 3 X 10
(a) hits the glass wall ofthe tube
33L3x
(b) strikes the metal target 21
n==lx10 ]
(c) passes through a strong magnetic field v
62. Out of X-rays, visible, ultraviolet, radiowaves, the largest
(d) none of the above
frequency is of:
52. Radius of a nucleus is proportional to:
(a) X-rays (b) visible
(a) A (b) A I/3 (c) A2 (d)A 2/3 (c) ultraviolet' (d) radiowaves
53. The nature of positive rays produced in a vacuum discharge 63. The wave nnmber 'which,corresponds to electromagnetic
tube depends upon: radiations of 600 nm is equal to:
(a) the nature of the gas filled (a) 1.6 x 104 cm- I (b) O.l6x 104 cm- 1
(b) nature ofthe material of cathode
(c) nature of the material of anode (c) 16 x 104 cm- I (d) 160 x 104 cm- i
(d) the potential applied across the electrodes 64. Line spectrum is characteristic of:
. 54. Electromagnetic radiation with maximum wavelength is: (a) molecules (b) atoms
(MLNR 1991) (c) radicals (d) none of these
(a) ultraviolet (b) radiowaves 65. Which one of the following is not the characteristic of
(c) X-rays (d) infrared Planck's quantum theory of radiation? (AIIMS 1991)
55. The ratio of energy of radiations of wavelengths 2000 A and (a) The energy is not absorbed or emitted in whole number
4000 A is: . (eDSE 1994) multiple of quantum' .
(a) 2 (b) 4 (c) 1/2 . (d) 1/ 4 (b) Radiation is associated with energy
56. The ratio of the diameter of the atom and the diameter of the (c) Radiation energy is not emitted or absorbed continuously
nucleus is: . but in the form of small packets called quanta
(a) 10 5
(b) ItY (cPO (d) 10- 1 (d) This magnitude of energy associated with a quantum is
proportional to the frequency
57. The ratio of the volume of the atom and the volume of the
nucleus is: 66. Which of the following among the visible colours has the
minimum wavelength? .
(a) 1010 (b) 1012 (c) 1015 (d) 102°
(a) Red (b) Blue
58. Which of the following statements is incorrect?
(a) The frequency of radiation is inversely proportional to its (c) Green (d) Violet
67. The spectrum of helium is expected to be similar to that of:
wavelength
(b) Energy of radiation increases with increase in frequency (a) H (b) Na (c)He+ , (d) Li+
(c) Energy of radiation decreases with increase in wavelength 68. According to classical theory. if an electron' is moving in a:
(d) The frequency of radiation is directly proportional to its.• circular orbit around the nucleus:
wavelength . (a) it will continue to do so for sometime
59. Visible light consists of rays with wavelengths in the (b) its orbit will continuously shrink
approximate range of: (c) its orbit will continuously enlarge
(a) 4000A to 7500 A (d) it will continue to do so for all the time
(b) 4 x 10-3 cm to 7.5 x 10-4 cm 69. Bohr advanced tlJe idea of:
(a) stationary electrons (b) stationary nucleus
(c) 4000 nm to 7500 nm
(c) stationary orbits (d) elliptical orbits
(d) 4 x 10-5 m to 7.5 x 10-6 m
70. On Bohr stationary orbits:
60. Which of the following statements concerning light js false? (a) electrons do not move
(a) It is a part of the electromagnetic spectrum (b) electrons move emitting radiations
(b) It travels with same velocity, i.e., 3 x 1(Yo cm/ s (c) energy ofthe electron remains constant
(c) It cannot be deflected by a magnet
(d) angular momentum ofthe electron is..!!...
(d) It consi,ts A photons of same energy . 2n
126 G.R.B. PHYSICAL CHEMISTRY FOR CoMPETITIONS

7.1. Energy ofEohr orbit: (DPMT 1991) 2nme2


(a) increases as we move away from the nucleus (d) E" =:
n 2h2
(b) decreases as we move away from the nucleus .
81. According to Bohr theOlY, the angular momentum for an
(c) remains the same as we move away from the nucleus
electron of 5th orbit is:
(d) none of the above
(a) 5h/n (b) 2.5h/n (c) 5n/ h (d) 25h/n
72. Which of the following statements does not form part of
82. The value of Bohr radius of hydrogen atom is: (CBSE 1.991)
Bohr's model ofthe hydrogen atom?
(a) 0.:529 x 10-7 cm (b) 0.529 x 10-8 cm
(a) Energy of the electron in the orbit is quantized
(b) The electron in the orbit nearest to the nucleus has the (c) 0.529 x 10-9 cm (d) 0.529 x 10- 10 cm
lowest energy 83. The energy of an electron in the nth Bohr orbit of hydrogen
(c) Electrons revolve in different orbit nucleus atom is: . (CBSE 1992)
(d) The position and velocity of the electron in the orbit (a) - 13.6 eV (b) 13.6 eV (c) 13.6 eV (d) _ 13.6 eV
cannot be determined simultaneously n4 n3 n2 . n
73. Which of the following statements does not form a pa11 of 84. Which ofthe following electron ·transitions in hydrogen atom
Bohr's model of hydrogen atom? (DCE 2005) will require largest amount ofenergy? . (MLNR 1992)
(a) Energy of the electrons in the orbit is quantised (a) from n = Ito n 2 (b) from n = 2 to n 3
(b) The electron in the orbit nearest to theJlUcleus has the- (c) from n 00 to n =: 1 (d) from n = 3 to n 5
lowest energy <". .
85. For a hydrogen atom, the energies-that an electron canhave~
(c) Electrons revolve in different orbits around the nucleus are given by the expression, E 13.58/n 2 eV, where n is an
(d) The position and velocity of the electrons in the orbit integer. The smallest amount of energy that a hydrogen atom
cannot be determined simultaneously in the ground state can absorb is:
74. The radius of the first orbit of H-atom is r. Then the radius of (a) 1.00 eV (b) 3.39 eV (c) 6.79 eV (d) 10.19 eV
the first orbit ofLi2+ will be: [AMU-PMT 2009] 86. The energy of hydrogen atom in its ground state is -13.6 e V.
r (b) r The energy of the level Corresponding to n == 5 is:
(a) - (c) 3r (d) 9r
9 3 (CBSE 1990)
2
[Hint: r= n xO.S29A] (a) - 0.54 eV (b) 5.40 eV (c) -0.85 eV (d) - 2.72eV
z 87. Ell = - 313.6/ n 2 kcallmol. If the value of E 34.84
75. The energy liberated when an excited electron returns to its .. kcallmol, to which value does 'n' correspond?
ground state can have: (a) 4 (b) 3 (c) 2 (d) I
(a) any value from zero to infinity 88. The ratio of the difference between 1st and 2nd Bch orbits
(b) only negative values energy to that between 2nd and 3rd orbits energy is:
(c) only specified positive values (a) 112 (b) 1/3 (c) 27/5 (d) 5127
(d) none of the abovc 89. Bohr's model can explain:
76. On the basis of Bohr's model, the radius of the 3rd~orbit is: (a) spectnim of hydrogen atom only
(a) equal to the radius of first orbit (b) spectrum of any atom or ion having one .electron only
(b) three times the radius of first orbit (c) spectrum of hydrogen molecule
( c) five times the radius of first orbit (d) solar spectrum
(d) nine times the radius of first orbit . 90. The energy difference between two electronic states is 43.56
77. The ratio of 2nd, 4th and 6th orbits of hydrogen atom is: kcal/mol. The frequency of light emitted when the electron
(a) 2 : 4 : 6 (b) I : 4 : 9 drops from higher orbit to lower orbit, is:
(Planck's constant = 9.52 x 10- 14, kcallmol)
(c) I : 4 . 6 (d) I : 2 : 3
78. Which point does not pertain to Bohr's model of atom? (a) 9.14 x I(j4 cycle/sec (b) 45.1 x 1014 cycle/sec
(a) Angular momentum is an integral multiple of h/ (2it) (e) 91.4 x 1014 cycle/sec (d) 4.57 x 1014 cyc1e/se~
(b) The path of the electron is circular 91. Which of the following transitions of an electron in hydrogen
(c) Force of attraction towards nucleus = centrifugal force of
atom emits radiation the lowest wavelength?
(d) The energy changes are taking place continuously [EAMCET (Engg.) 2010]
79. The distance between 3rd and 2nd orbits in the hydrogen atom (a) n2 == to nl 2
00 (b) n2 4ton1 = 3
is: (c) n2 = 2 to nl "" 1 (d) n2 5 to n l == 3
(a) 2.646 x 10-8 cm (b) 2.116 x 10-8 cm 92. The wavelength of a spectral line for an electronic transition is
(c) 1.058 x 10-8 cm . (d) 0.529 x 10-8 cm inversely related to:
(a) number of electrons undergoing transition
80. The correct expression derived for the energy of an electron in
(b) the nuclear charge of the atom
the nth energy level in hydrogen atom is:
( c) the velocity of an electron undergoing transition
2n 2 me4 2n 2 me4
(a) En (b) En = (d) the difference in the energy levels involved 111 the
2h2
---n 2
, nh transition
I
ATOMIC STRUCTURE 127

93. The ionisation energy of the electron in the ls-orbital of the (a) 15,200cm- I (b) 60,800cm- 1
hydrogen atom is 13.6 eY. The energy of the electron after
(c) 76,000 cm- 1 (d) 1,36,800cm-:- 1
promotion to 2s-orbital is: IISC (Bihar) 1993)
(a) -3.4eV (b) -13.6eV 103. "The position and the velocity of a small particle like electron
(c) -27.2eV (d) O.OeV cannot be simultaneously determined." This statement is:
94 .. Which electronic level would allow the hydrogen atom to (a) Heisenberg uncertainty principle
absorb a photon but not to emit it? (b) Pauli's exclusion principle
(a) Is (b) 2s (c) 3s (d) 4s (c) aufbau's principle
95. The spectral lines corresponding to the radiation emitted by an (d) de Broglie's wave nature ofthe electron
electron jumping from 6th, 5th' and 4th orbits to second orbit 104. de Broglie equation describes therelationship of wavelength
belong to: associated with the motion of an electron and its:
(a) Lyman series (b) Balmer series (a) mass (b) energy (c) momentum (d) charge
(c) Paschen series (d) Pfund series 105. If the magnetic quantum number of a given atom is
96. The spectral lines corresponding to the radiation emitted by an represented by then what will be its principal quantum
electron jumping from higher orbits to first-Orbit belong to: number? [BHU (Pre.) 2005)
(a) Paschen series (b) Balmerseries (a) 2 (b) 3 (c) 4 (d) 5
(c) Lyman series (d) None of these J06. Which of the following relates to photons both as wave motion
and as a stream of particles? (IlT 1992)
97. In a hydrogen atom, the transition takes plac.efrOll1n 3 to
n = 2. If Rydberg constant is 1.097 x 107 m-I, the wavelength (a) Interference (b) Diffraction
of the emitted radiation is: (c) E == hv (d) E = me2
(a) 6564A (b) 6064 A 107. If uncertainty in the positiOil of an electron is zero, the
(c) 6664A (d) 5664 A uncertainty in its momentum would be:

[Hint: Apply.!. = R
A
[~
x j} Ca) zero,
(c) > h/(4n)
(b) < hI(4'n)
(d) infinite
108. Which one of the following explains light both as a stream of
98. The speed ofthe electron in the 1st orbit of the hydrogen atom
particles and as wave motion?
in the ground state is (e is the velocity oflight):
(a) Diffraction (b) A = h/ p
e e e e
(a) 1.37 (b) l370 (c) 13.7 (d) 137 (c) Interference (d) Photoelectric effect
109. A body of mass x kg is moving with velocity oflOO msec- I . Its
[Hint: Velocity of electron in the 1st orbit, v = h/(2nmr)
de Broglie wavelength is 6. 62 X 10-35 m. Hence x is:
, = 2.189 X 108 em/sec., velocity of light, c = 3 X 10 10 ern/sec. (h == 6.62x 10-34 J sec) [CET (Karnataka) 2009)
Ratio c/v l37] (a) 0.25 kg (b) 0.15 kg
99. , Find the value of wave number v in terms of Rydberg's (c) 0.2 kg (d) 0.1 kg
constant, when transition of electron takes place between two 110. A 200 g cricket ball is thrown with a'speed of 3.0 x 103 cm
levels ofHe+ ion whose sum is 4 and difference is 2. sec-I. What will be its de Broglie's wavelength?
(a) 8R (b) 32R (II =6.6 X 10-27 g cm2 sec-I ) [CET (Gujftrat) 2008)
9 9 (a) 1.1 x 10-32 cm (b) 2.2 x 10- 32 cm
(c) 3R (d) None of these (c) 0.55 x 10-32 cm (d) Il.Ox 10-32 cm
4
Ill. The electronic configuration of a dipositive ion M 2+ is 2, 8, 14
[Hint: nl +~ 4; n2 -nj 2 and its atomic mass is 56. The number of neutrons in 'the
V RZ2[~_~J nucleus would be:
nl ni ( (a) 30 (b) 32 (c) 34 (d) 42
112. An element with atomic number 20 will be placed in which
=R X 22 [~- ~J
2
32R] period of the periodic table?
1 3 9
(a) 5th (b) 4th (c) 3rd (d) 2nd
100. With the increasing principal quantum number, the energy
113. The frequency of radiation emitted when the ekc~on falls
difference between adjacent energy levels in hydrogen atom:
from n 4 to n 1 in a hydrogen atom will be (Given
(a) increases (b) decreases
ionisation energy of H=2.18x 10- 18 Jatom- 1 and
(c) is the same (d) none of these h = 6.626 X 10-34 Js): [Manipa, (Me d.) 2007)
101. An electron in an atom: rCEET (Bihar) 1992] (a) 1.54 x 1015 S-I (b) 1.03 x 1015 8- 1
(a) moves randomly around the nucleus
(b) has fixed space around the nucleus
(c) 3.08 X 101 5 Cd) 2 x 1015 s-i
(c) is stationary in various energy levels 114. In a multi-electron atom, which of the following, orbitals
(d) moves around its nucleus in definite energy levels described by the three quantum numbers will have the same
102. The wave number of first line of Balmer series of hydrogen is energy in the absence of magnetic and electric fields?
15200 cm- I • The wave number ofthe first Balmer line ofLi 2+ (AIEEE2005)
ion is: [liT !}~f:r:'i'''~';,,~~ 1992] (i) 17 = I, I=: 0, m 0 (ii) n == 2, I 0, m = 0
I
128 I G.R.B. PHYSICAL CHEMISTRY FOR COMPETITIONS

(iii) n =: 2, I I, m =: I (iv) n =: 3, I=: 2, m =: I 127. Any p-orbital can accommodate up to: (MLNR 1990)
(v) n =: 3, I ::= 2, m =: 0 (a) 4 electrons
(a) (i) and (ii) (b) (ii) and (iii) (b) 2 electrons with parallel spins
(c) (iii) and (iv) (d) (iv) and (v) (c) 6 electrons
115. Which of the ions is not having the configuration ofNe? (d) 2 electrons with opposite spins
(a) Cl- (b) F - (c) Na + (d) Mg2+ 128. How'many electrons can fit into the orbitals that comprise the
116. Which ofthe following has the maximum number of unpaired 3rd quantum shell n =: 3?
d-electrons? (KeET 2008) (a) 2 (b) 8 . (c) 18 (d) 32
(a) Ni 3+ (b)Cu+ (c)Zn2+ (d) Fe2 + 129. The total number of orbitals in a principal shell is:
117." Which of the following expressions gives the de Broglie (a) n (b) n2 (c) 2n 2 (d) 3n 2
relationship? IJEE (WB) 2008) 130. Two electrons in K-shell will differ in:
h h (a) principal quantum number
(a) p =: - (b) A
mv mv (b) spin quantum number
(c) A =: h
h (c) azimuthal quantum number .
(d) Am=
mp p (d) magnetic quantum number .
118. The principal quantum number of an atom is related to the: 131. Which one of the following orbitals has the shape of a
(MLNR 1990) baby-boother ?
(a) size of the orbital (a) dxy (b) d x 2 - y 2 (c) d z 2 (d) Py
(b) orbitafangular momentum
132. Which one of the following represents an impossible
(c) spin angular momentum
arrangement? (AIEEE 2009)
(d) orien.tation ofthe orbital in space n I m s n I m s
119. The magnetic quantum is a number related to:
(a) 3 2 - 2 112 (b) 4 0 o 1/2
(a) size (b) shape
(c) 3 2 . -3 112 (d) 5 3 o 112
(c) orientation (d) spin
120. The principal quantuIn number represents: (CPMT 1991)
(a) shape of an orbital
(b) nU!l).ber of e~ectrons in an orbit
(c) distance of electron from nucleus
(d) numbei: of orbitals in an orbit
121. The quantum number not obtained from the Schrodinger's
wave equation is: (lIT 1990)
(a) n (b) 1 (c) m (d) s
122. In a given atom, no two electrons can have the same values for
all the four quantum nU!l).bers. This is called: (CPMT 1990)
(a) Hund's rule (b) Pauli's exclusion principle
(c) Uncertainty principle (d) aufbau principle
123. The atomic orbital is:
(a) the circular path of the electron
(b) elliptical shaped orbit .
(c) three-dimensional field around nucleus
(c) 21 + I (d) 41 + 2
. (d) the region in which there is maximum probability· of
finding an electron [Hint: Number of electrons with same spin
·124. If the ionization energy for hydrogen atom is 13.6 eV, then the 1 x Total no. of electrons
ionization energy for He+ ion should be: 2
1 .
I PMT (Haryana) 2004] =: - x 2 (21 + 1) =: (21 + 1)]
(a) 13.6 eV (b) 6.8 eV 2
(c) 54.4 eV (d) 72.2 eV 136.. Which of the following represents the correct set of four
125. Principal,. azimuthal and magnetic quantum numbers are quantum numbers of a 4d-e1ectron? (MLNR 1992)
respectively related to: (a) 4,3,2, + 112 (b) 4,2, 1,0
(a) si?e,shape and orientation (c) 4,3, - 2, + 112 (d) 4,2, i, 1/2
.(b) shape, size and orientation 137. Values of magnetic orbital quantum number for an electron of
(c) size, orientation and shape M-shell can be: (PET (Raj.) 2008]
(d) none of the above (a) 0, 1,2 (b)-2,-1,0,+1,+2
126. Energy of e1ectron·in the H-atom is determined by : (c) 0, 1,2,3 (d) 1,0,+1
(a) only n (b) bothn and I 138. Correct set of four quantum numbers for the outermost
(c) n, I and m (d) all the four quantum numbers . electron of rubidium (Z = 37) is:
ATOMIC STRUCTURE 129

(a) 5,0,0,1/2 (b) 5, 1,0, 112 149• The energy ofaa~lectron of 2p y' orbital is:
.(c) 5,1, 1.112 (d) 6,0,0,112 (a) greater than 2px orbital
139. Which one of the following subshells is spherical in shape? (b) less than 2pz orbital
(a) 4s (b) 4p (c) 4d (d) 4f (c) equalto 2sorbitil
140. In hydrogen atom, the electron is at a distance of 4.768 A from (d) same as that of2px and 2pz orbitals
the nucleus. The angular momentum of the electron is : 150• The two electrons occupying the same orbital are
. [EAMCET (Med.) 2010] distinguished by: .
(a)· 3h . (b) ~ (a) principal quantum.number
211: 211: (b) azimuthal quantum number
h (d) 3h (c) magnetic quantum number
(c) -
11: 11: (d) spin quantum number
n2 151. The maximum number of electrons in a subshell is given by
[Hint: r = - x 0.529 A
z the expression: (AIEEE 2009)
n2 (a) 41 + 2 (b) 4/- 2
4.768 = x 0.529
1 (c) ()./ + I (d) 2n2
n=3 152. The electronic configuration of an atom/ion can be defined by
nh __ 3h] which of the following1
:. Angular m()mentum (mvr)
211: 211: (a) A1.dbau PriI1ciple
141. Total number of m values for n 4 is: (b) Pauli's exclusion principle
(a) 8 (b) 16 (c) 12 (d) 20 (c) Hund's rule of maximum mUltiplicity
142. What is the total number of orbitals in the shell to which the (d) All of the above
g-subshell first arise?
153. An electron has a spin quantum number + 112 and a magnetic
(a) 9 (b) 16 (c) 25 . (d)36 quantum number 1. It cannot be present in:
[Hint: For g-subshell, / = 4 (a) d-orbital (b).forbital (c) s-orbital (d) p-orbital
:. It will arise in 5th shell. 154. The value of azimuthal quantum number for electrons present
Total number of orbitals in 5th shell == n2 = 25] in 4 p-orbitals is:
143. In Bohr's model, if the atomic radius of the first orbit 1j, then (a) 1
radius offourth orbit will be : [BHU (Screening) 2010] (b) 2

(a) 4'1 (b) 6'1 (c) 161j(d) !L


°
(c) any value between and 3 except I
16 (d) zero
144. Which of the following statements is not correct for an 155. For the energy levels in an atom which one of the following
electron that has quantum numbers n = 4 and m = 2? statements is correct?
(MLNR 1993) (a) The 4s sub-energy level is at a higher energy than the 3d
, (a) The electron may have the q. no. I = + 1/2 sub-energy level
(b) The electron may have the q. no. I = 2 (b) The M -energy level can have maximum of 32 electrons
(c) The electron may have the q. no. 1= 3 (c) The second principal energy level can have four orbitals
(d) The electron may have the q. no. I = 0, I, 2, 3 and contain a maximum of 8 electrons
145. The angular momentum of an electron depends on: (d) The 5th main energy level can have maximum of 50
(a) principal quantum number electrons
(b) azimuthal quantum nwnber 156. A new electron enters the orbital when:
( c) magnetic quantum number (a) (n + /)is minimwn (b) {n + J)is maximum
(d) all of the above (c) (n+m)isminimum (d) (n+m)ismaximum
146. The correct set of quantum numbers for the unpaired electron 157. For a given value of n{principal quantum number), the energy
of a chlorine atom is: (DPMT 2009) of different subshells can be arranged in the order of: .
1 I (a) !>d> p>s (b) s> p>d>!
(a) 2,0,0, + (b) 2, I, -1, + (c)!>p>d>s (d)s>!>p>d
2 2
I 1 158. After filling the 4d-orbitals, an electron will enter in:
(c) 3,1, I,±- (d) 3,0,O,±- (a) 4p (b) 4s (c) 5p (d) 4f
2 2
159. According to t\ufbau principle, the correct order of energy of
147. The magnetic quantum number for valency electron of sodium
atom is:
3d,4s and 4 p-orbitals is: [eEl' (J&K) 2006)
(a) 3 (b) 2 (c) I (d) zero (a) 4p<3d<4s (b) 4s<4p<3d
148. The shape of the orbital is given by: (c) 4s < 3d < 4 p (d) 3d < 4s < 4 p
[PET (Raj.) 2008J
(a) spin quantum number 160. Number of p-electrons in bromine atom is:
(b) magnetic quantum number [PMT (Haryana) 20041
.(c) azimuthal quantum number (a) 12 (b) 15
(d) principal quantum number (c) 7 (d) 17
130 G.R.B. PHYSICAL CHEMISTRY FOR COMPETITIONS

161. [Ar] 3dlO 4sl electronic configuration belongs to: . 169. The radial probability distribuuon curve obtained for an
[PET (MP) 20081 orbital wave function ('If) has 3 reaks and 2 radial nodes. The
(a) Ti (b) Tl (c) Cu (d) V valence electron of which one of the following metals does
162. How many unpaired electrons are ther~ in Ni2+ ? (Z 28) this wave function ('If ) correspond to ?
(a) Zero (b) 8 (c) 2 (d) 4 [EAMCET (Med.) 20101
163. The electronic configuration of chromium (Z 24) is: (a) Co (b) Li (c) K (d) Na
[PMT (MP) 1993; BHU (Pre.) 20051 [Hint: Na ll ls2, 2s22l, 3s 1
'----v----'
(a) [Ne] 3i3 p 6 3d 44i (b) [Ne] 3s2 3p 6 3d 54s1 Valence electron
Number of radial node =: n-l-I (n == 3)
(c) [Ne] 3i3p 6 3d l 4i (d) [Ne] 3i 3p64i4p4
3-0-1 = 2]
164. The number of d-electrons in Fe 2+ (At. No. 26) is not equal to 170. ~pton (At. No. 36) has the electronic configuration [A]
that of the: (MLNR 1993) 4s 3d 104 p 6. The 37th electron will go into which one of the
(a) p-electrons in Ne (At. No. 10) . following sub-levels?
(b) s-electrons in Mg (At. No. 12) (a) 4/ (b) 4d
(c) d-electrons in Fe atom (c) 3p (d) 5s
(d) p -electrons in Cl- ion (At. No. 17) 171. An ion which has 18 electrons in the outermost shell is:
165. If the electronic structure of oxygen atom is written as (CBSE 1990)
f--2p---+ (a) K + Q » Cu.+ (c) Cs+ (d) Th 4 +
172: Which ofthe following has non-spherical shell of electronJa
ls2, 2s2 !i.j, 1i.j, 1 . 1; it would violate: fISC (Bihar) 1993]
(lIT 1993)
(a) Hund's rule (a) He (b) B (c) Be (d) Li
173. Which one of the following sets of quantum numbers is not
(b) Pauli's exclusion principle
possible for an electron in the ground state of an atom with
(c) both Hund's and Pauli'sprincipJes
atomic number 19? [PET (Kerala) 2006;
(d) none of the above
CET (Karnataka) 2009]
166. The orbital diagram in which 'aufbau principle' is violated,is:
(a) n = 2, I 0, m == 0 (b) n = 2, I = 1, m 0
2s f - - 2 p---+
(c) n = 3, I =: I, m -1 (d) n 3,1=2,m=±2
(a) [ill I i.j,1 i I (e) n 4,1=0,m=0
174. Helium nucleus is composed of two protons and two neutrons.
(b) If the atomic mass is 4.00388, how much energy is released
when the nucleus is constituted?
(c) :1 i.j, i (Mass of proton == 1.00757, Mass of neutron == 1.00893)
(a) 283 MeV (b) 28.3 MeV
(d) :1 i.j, :1 (c) 2830 MeV (d) 2.83 MeV
175. Binding energy per nucleon of three nuclei A, B and Care
167. The manganese (Z = 25)has the outer configuration: 5.5, 8.5 and 7.5 respectively. Which one of the following
4s ( 3p ) nuclei is most stable?
(a) [illi i.j, II i il i l i II i !I (a) A (b) C
(c) B (d) Cannot be predicted
(b) [ill 176. The mass of jLi is 0.042 less than the mass of 3 protons and
4 neutrons. The binding energy per nucleon in j Li is:
(c) (ill :1 i .j, il j.j, :1 i .(BHU1992)
(a) 5:6 MeV (b) 56 MeV (c) 0.56 MeV (d) 560 MeV
(d) 0 II j.j, II i.j, II i i i 177. Meson was discovered by:
(a) Powell (b) Seaborg
168. Which of the following elements is represented by the (c) Anderson (d) Yukawa
electronic configuration? 178.. In most stable elements, the number of protons and neutrons
f--2p-c-? are:
(a) odd-odd (b) even-even
(c) odd-even (d) even-odd
179. Nuclear particles responsible for holding all nucleons together
are:
(a) electrons (b) neutrons
(c) positrons (d) mesons
(a) Nitrogen (b) Fluorine 180~ The introduction of a neutron into the nuclear composition of
(c) Oxygen (d) Neon an atom would lead to a change in: (MLNR 1995)
ATOMIC STRUCTURE 1.131
(a) its atomic inass (a) 3 (b) 4 (c) 2 (d) 1
(b) its atomic number 191. A certain negative ion has in its nucleus 18 neutrons,and
(c) the chemical nature of the atom 18 electrons in its extranuclear structure. What is the mass
(d) number of the electron also number of the most abundant isotope of X ?
181. Which of the following has highest orbital angular (a) 36 (b) 35.46 (c) 32 (d) 39
momentum? 193. Which of the following statements is not correct?
(a) 4s (b) 4p (c) 4d (d) 4f
(a) The shape of an atomic orbital depends on the azimuthal
181. Which of the following has maximum number of unpaired
quantum number .
electrons? (PMT (Raj.) 2004; BHU (Pre.) 2005)
(b) The orientation of an atomic orbital depends on the
(a) (b) Fe 2+ (c) C0 2+ (d) C0 3+ magnetic quantum number
183. An electron isnot deflected on passing through a certain (c) The en~rgy of an electron in an atomic orbital of
region,because: multielectron atom depends on the principal : quantum
(a) there is no magnetic field in that region number
(b) there is a magnetic field but velocity of the electron is (d) The number of degenerate atomic orbitals of one type
parallel to the direction of magnetic field depends on the values of azimuthal and magnetic qUa)1tum
(c) the electron is a chargeless particle numbers
(d) none ofthe aboye 194. Gases begin to conduct electricity at low pressure because:
184. In MiHik1m's oil drop experiment, we make~use of:
(CSSE 1994)
(a) Ohm's law (b) Ampere's law (a) at low pressures gases tum to plasma
(c) Stoke's law (d) Faraday's law .(b) colliding electrons can acquire higher kinetic energy due to
185. A strong argument for the particle nature of cathode rays is: increased mean free path leading to ionisation of atoms
(a) they can propagate in vacuum (c) atoms break up into electrons and protons
(b) they produce fluorescence (d) the electrons in atoms can move freely at low pressure
(c) they cast shadows
195. An electron of mass m and charge e, is accelerated from rest
(d) they are deflected by electric and magnetic fields through a potential difference V in vacuum. Its final speed will
186. .As the speed of the electrons increases, the measured value of be: (eBSE W94)
charge to mass ratio (in the relativistic units):
(a) increases (a) ~ (b) 2eVIm
(b) remains unchanged (c) J(eVI2m) (d) J(2eVlm)
(c) decreases .
(d) first increases and then decreases 196. The difference in angular momentum associated with .the
187. Which of the following are true for cathode rays? electron in the two successive orbits of hydrogen atom is:
(a) It travels along a straight line (a) hln (b) hl2n (c) hl2 (d) (n -1)hI2lt
(b) It emits X -rays when strikes a metal 197. Photoelectric effect can be explained by assuming that light:
(c) It is an electromagnetic wave (a) is a form of transverse waves
(d) It is not deflected by magnetic field (b) is a form of longitudinal waves
ISS. Three isotopes of an element have masl' numbers, M, (M + 1)
(c) can be polarised
and (M + 2). If the mean mass number is (M + 0.5) then
which of the following ratios may be accepted for (d) consists of quanta .
M, (M + 1), (M + 2)in that order? 198. The photoelectric effect supports quantum nature of Jight
(a) I : 1 : I (b) 4: 1 : 1
because:
(a) there is a minimum frequency of light below which no
(c) 3 : 2 : 1 (d) 2 : 1 : 1
photoelectrons are emitted .
189. The radii of two of the first four Bohr orbits of the hydrogen
(b) the maximum kinetic energy of p\1otoelectrons depends
atom are in the ratio I : 4. The energy difference between them
only on the frequency oflight and not on its intensity
maybe: . .. .
(a) either 12.09 eV or 3.4 eV (6) .either 2.55 eV or 10.2 eV (c) even whet! metal surface is faintly illuminated the
photoelectrons leave the surface immediately
(c) either 13.6 eV or 3.4 eV (d) either 3.4 eV or 0.85 eV
(d) electric charge of photoelectrons is quanti sed
190. Photoelectric emission is .observed from a surface. for
frequencies VI and v 2 ofthe incidentradiafion (VI> V2 ).Ifthe 199. The mass ofa proton at rest is: .(CB.SE 1991)
maximum kinetic energies of the photoelectrons in the two (a) zero (b) 1.67 x 10-:-35 kg
cases are in the ratio 1 : k then the threshold frequency V 0 is (c) one amu (d) 9 x 10-31 kg
given by:
200. Momentum of a photon of wavelength A is: (eBSE 1993)
(a) v 2 - VI (b) k VI ~V2 (c) _k--=.._..!.. (d) v 2 - VI 2
k-l k-l k k (a) hlA (b) zero (c) hAle (d) hAle
191. The number of waves made by a l30hr electron in an orbit of 201. When X-rays pass through air they:
maximum magnetic quantum number +2 is: . ( a) produce light track in the air
(b) ionise the gas
.....
132 G.R.B. PHYSlpAL C~EMISTRY .FOR COMPETITIONS

(c) produce fumes in the air h


(a) L= nh (b) L=1(1+1)-
(d) accelerate gas atoms. 2n 2n
202. X.rays:· (CPMT 1991) mg h
(a) are deflected in a magnetic field (c) L= (d) L=-
2n . 4n
(b) are deflected in an electric field
214. Ground state electronic configuration of nitrogen atom can be
(c) remain undeflected by both the fields
represented by: (lIT 1999)
(d) are deflected in both the fields
203. Find the frequency of light that corresponds to photons of
energy 5.0x 10-5 erg: (AIIMS 2010)
1. [ill DIJ 11 11 11 I 2. ill] [ill 11 1 ~ 11 1
(a) 7.5xlO-21 sec-1
(c) 7.5xl02 1 sec~l
(b) 7.5 x 10-21 sec
(d) 7.5 x 1&1 sec
3. DO l!J 11 I ~ I ~ 1 4. [!] [!] 1J 1J 1J 1
E 5xlO-s erg (a) 1 only (b) 1,2 only (c) 1,4 only (d) 2,3 only
[Hlnt:v=-= .
. h 6.63 X 10-27 erg sec 215. Which of the following statement{s) are correct?
1. Electronic configuration ofCr is [Ar] 3d 5481 (At. No. of
= 7.54 xl02 1 sec-I]
Cr=24)
204. The energy of an electron in the flISt Bohr orbit of H·atom is 2. The magnetic quantum number may have negative value
-:-13.6 eV. The possible energy value(s) of the excited state(s)
3. In silver atom, 23 electrons have a spin of one type and 24
for electr~ns in Bohr orbits of hydrogen is/are: (lIT 1998)
of the opposite type (At. No~()fA,g== 47L _.........__. ___ _
(a) ..:.3.4 tV (b) - 4.2 eV 4. The oxidation state. of nitrogen in HN3 is -3 (lIT 1998)
(c) -6.8 eV (d) +6.8 eV (a) 1,2,3 (b) 2,3:4 (c) 3,4 (d) 1,2,4
205. The electrons identified by quantum numbers n and I, (i) n == 4, 216. The electronic configuration of an element is
1= 1 Oi) n = 4, 1= 0 (iii) n = 3, I = 2 (iv) n = 3,1 = 1 can be ls22i 2p6, 3i 3p6 3d 5, 4}. This represents: (lIT 2000)
placed in order of increasing energy, from the lowest to highest . (a) excited state (b) ground state
as: (lIT 1999) (c) cationic state (d) anionic state
(a) (iv) < (ii) < (iii) < (i) (b) (ii) < (iv) < (i) < (iii) 217. The quantum numbers + 1 and ~ ~ for the electron spin
(c) (i) < (iii) < (ii) < (iv) (d) (iii) < (i) < (iv) < (ii) represents: 2 2 (liT 2000)
206. The wavelength of the radiation emitted when an electron falls (a) rotation of the electron in clockwise and anticlockwise
from Bohr orbit 4 to 2 in hydrogen atom is: (lIT 1999) directions respectively
(a) 243 nm (b) 972 nm (b) rotation of the electron in anticlockwise and clockwise
(c) 486 nm (d) 182 nm directions respectively
207. The energy of the electron in the flISt orbit of He+ is (c) magnetic moment of electron pointing up and down
- 871.6 x 10-20 J..The energy of the election in the first orbit respectively
of hydrogen would be: (lIT 1998) (d) two quantum mechanical spin states which have no
20 20 classical analogues
(a) -871.6x 10- J (b) - 435 X 10- J
(c) -217.9x 10-20 J (d) -108.9x 10-20 J 218. Rutherford's experiment, which established the nuclear model
of the atom, Osed a beam of: (lIT 2002)
208. The wavelength associated with a golf ball weighing 200 g and (a) j3.particles, which impinged on a metal foil and got
moving with a speed of 5 mIh is of the o~ of: (lIT 2000) absorbed
(a) 10-10 m (b) 10-20 m (c) 10-30 m (d) 10-40 m (b) y·rays, which impinged on a metal foil and ejected
209. Who modified Bohr theory by introducing elliptical orbits for electrons
electron path? (CBSE 1999) (c) helium atoms, which impinged on a metal foil and got
(a) Hund (b) Thomson scattered . . >

(c) Rutherford (d) Sommerfeld (d) helium nuclei, which impinged on a metal foil and got
210. The uncertainty in momentum of an electron is 1 x 10-5 kg scattered
ms -1. The uncertainty in its position will be: 219. How many moles of electrons weiflt one kilogram?
(h = 6.62 x 10-34 kg.m2 .s) (CBSE 1999; BHU 2010) (Mass of electron = 9.108 x 10- J kg, Avogadro's number
(a) 1.05 x 10-28 m (b) 1.05 x 10-26 m = 6.023 x 1023 ) (lIT 2002)
(c) 5.27x 10-30 m (d) 5.25 x 10-28 m (a) 6.023 x 1023 (b) _1_ X lerl
9.108
211. The Bohr orbit radius for the hydrogen atom (n = 1) is
approximately 0.530 A. The radius for the first excited state (c) 6.023 x 10S"4 (d) I x 108
9.108 9.108 x 6.023
(n = 2) orbits is: (CBSE 1998) 7
(a) 0.13 A (b) 1.06 A (c) 4.77 A (d) 2.12 A 220. If the electronic configuration of nitrogen had ls , it would
have energy lower than that of the normal ground state
212. The number of nodal planes in px.orbital is: (lIT 2000)
configuration ls22i2p 3 because the electrons would be
(a) one . (b) two (c) three (d) zero
closer to the nucleus. Yet ls7 is not observed because it
213. The angular momentum (L) of an electron in a Bohr orbit is violates: (liT 2002)
given as: (lIT 1997)
I

I 133
(a) Heisenberg uncertainty principle 230. In ground state, the radius of hydrogen atom is 0.53 A. The
(b) Hund's rule radius ofLi 2 + ion (Z =3) in the same state is: .
(c) Pauli's exclusion principle (PET (Raj.) 2007]
(d) Bohr postulates of stationary orbits (a) 0.17 A (b) 1.06 A (c) 0.53 A (d) 0.265 A .
221. The orbital angular momentum of an electron in 2s-orbital is: 231. How many d-electrons in Cu + (At. No. = 29) can have the spin
(lIT 1996; AIEEE 2003; PMT (MP) 2004) quantum number (- ~ f! (SCRA 2007)
I h (a) 3 (b) 7 (c) 5 (d) 9
(a)+-- (b) zero 232. Which of the following electronic configurations, an atom has
22'1t
the lowest ionisation enthalpy? [CBSE (Med.) 2007)
h
(c)- (d)Ji !!.... (a) li2s 2 2p 3 (b) li2s'22p6 3s1
2'1t 2'1t
(c) ti2s 2 2p 6 (d) li2s 2 2p s
222. Calculate the wavelength (in nanometre) associated with a
proton moving at I x I (fl m sec -I. 233. The measurement of the electron position is associated with an
uncertainty in momentum, which is equal to 1 x 10- 18 g cm s - I.
(mass of proton = 1. 67x 10-27 kg, h::::; 6.63 x 10-34 J sec)
The uncertainty in electron velocity is: (mass ofan electron is
(AIEEE 2009)
9 x 10- 28 g) ICBSE-PMT (Pre.) 2008]
(a) 0.032 nm (b) 0.40 nm
(a) Ix lOS ems- I (b) Ix lotI cms- I
(c) 2.5 nm (d) 14 nm
(c)lxI09 cms-' . (d)lxI0 6 cms- 1
-[1IiDt ~A=~,/~ 16~'~~~"~~:nj3~ 234. The ionization enthalpy of hydrogen atom is L312 x 1 0 ! J - - -
mol-I. The energy required to excite the electron in the atom
::::; 0.397x 10-9 m 0.4 nm] from n = 1 to n = 2 is : (AIEEE 2008)
223. The value of Planck's constant is 6.63 x 10-34 J-s. The (a) 9.84 x lOS J mol- I (b) 8.51 x lOS J mor l
velocity of light is 3 x I rf mI sec. Which value is closest to the (c) 6.56 x lOS J mol-I (d) 7.56 x lOS J mol- I
wavelength in nanometer of a quantum of light with frequency [Hint: El ::::; - I.312 X 106 J mol-'
of8x lotS sec-I? . (CBSE(PMT) 2003] 6
E _EI _ 1.312xlO J 1- 1 ...
18 2- 2 - - 4 mo
(a) 5 x 10- (b) 4 x 10' 2
(c) 3 X 107 (d) 2 x 10-25 t.E= (E2 - EI)::: 1.312 X 10 (l-~) 6

224. Which of the following statements in relation to the hydrogen


atom is correct? (AIEEE 2005) =~ x 1.312 X 106 =9.84 x 105 J mol-']
4
(a) 3s-orbital is lower in energy than 3p-orbital
235. The wavelengths of electron waves in two orbits is 3.: 5. The
(b) 3p-orbital is lower in energy than 3d~orbital
ratio of kinetic energy of electrons will be: (EAMCET 2009)
(c) 3s--and 3p -orbitals are oflower energy than 3d-orbital
(a) 25: 9 (b) 5 : 3
(d) 3s, 3p -and 3d-orbitals all have the same energy
(c) 9:25 (d) 3:5
225. The number of d-electrons in Ni (At. No. ::: 28) is equal to that
of the: (CPMT (UP) 2004] [Hint: We know, A = ~
'I2Em
(a) sand p-electrons in F-
~:::~
(b) p-electrons in Ar (At. No. = 18)
(c) d-electrons in Ni 2 + 1.2 vE;
(d) total number of electrons in N (At. No. = 1) ~ ~
226. The number of radial nodes of 3s- and 2p-orbitals are 5 VE,~
respectively: [lIT (Screening) 2005) .. E,:E2 =25:9]
(a) 2,0 (b) 0, 2 (c) 1,2 (d) 2, I 236. Electrons with a kinetic energy of 6. 023 x 104 l/mol are
227. Which of the following is not permissible? (DCE 2005) evolved from the surface of a metal, when it is exposed to
(a) n = 4, 1= 3, m = 0 (b) n = 4, I = 2, m = 1 radiation of wavelength of 600 nm. The minimum amount of
=
(c) n 4, I = 4, m 1 = (d) n = 4, I = 0, m 0 = energy required to remove an electron from the metal atom is :
228. According to Bohr theory, the angular momentum of electron d ,. . •• (E~.l~fCE1:20f)1!)
in 5th orbit is: (AIEEE 2006) (a)· 2.3125x 10:Ci9
l ~ (b) 3~.10-19}'i , - ' . ~ ~ , ...
h h h h
(a) 25- (b) 1- (c) 10- (d) 2.5- (c) 6.02x 10- 19 J (d) 6.62x 10-34 J
'It 'It 'It 'It
229. Which of the following sets of quantum numbers represents [Hint: Absorbed energy = Threshold energy + kinetic energy
the highest energy of an atom? (AIEEE 2007) of photoelectron
(a) n = 3, I = 0, m ::::; 0, S + ~ = he
-r=Eo+KE
(b) n = 3, I = I, m = I, s = + ~
8 4
(c) n = 3, I = 2, m = I, s = + ~ 6.62x 10-34 X 3 x 10 E. 6.023 x 10 JI
(d) n = 4, I::: 0, m = 0, s = + ~ 600 x 10-9 = 0 + 6.023 X atom 1023
134 G.R.B. PHYSICAL CHEMISTRY FOR COMPETITIONS

3.31x.1O- 19 ";'Eo+lxI0- 19 2. Which IS the correct relationship?


Eo = 231xlO- J] 19 (a) EI of H 112 E2 of He+= 1/3E30f U 2 +== 1I4E4 of Be 3+

237. : :::::['nl~~ili]:~alue n n:::;~:;::~:~ of


l
and
(b) El (H) == E2 (He + ) E3 (Li 2 +) ,E4(Be 3+ )
(c) EI(H) = 2E2 (He+) == 3E3(Li2+) == 4E4(Be 3+)
(d) No relation
nl n2

(a) nl I, n2 = 2, 3, 4 .. .
3. Which is correct for any kind of species?
(b) n 1 == 2, n2 =3,4,5 .. . (a) (E2 - EI ) > (E3 - E 2 ) > (E4 - E 3 )
(b) (E2 -E,)«E3 E 2 )«E4 E 3 )
(c) nl == 3,n2 4, S, 6 .. .
(d) nl == 4, n2 5,6,7... (c) (E2 -E,)=(E3 E 2 )=(E4 E 3 )
138. Ionization energy of He+is 19.6 x 10- 18 J atom-I. The energy (d) (E2 - El ) = 1/4(E3 - E2 ) 1/9(E4 - )
oftne first stationary state (n == I) of-Li 2+ is: (AIEEE 2010) 4. No. of visible lines when an electron returns from 5th orbit to
:.../ (a) -22x 10-15 J atom- l (b) 8:82 x 10-17 J atom-I ground state in H spectrum is:
(a) 5 (b) 4 (c) 3 (d) 10
(c) 4.4lxlO-16 J at9m-' (d) -4.41 x 10- 17 J atom-'
S. Quantum numbers I 2 and m 0 represent which orbital?
Z2
[Hint: =-1 (a) dxy (b) dxz _ y2 (c) d z2 (d) d zx
lLi&!- zi 6. If n and I are principal and azimuthal quantum numbers
respectively, then the expression fo,{ calculating the total-
lLi&!- . numbers of electrons in allY energy level is:
I 11 I n-1
9 2(21 + I)
f ,&!-= - x19.6x 10- 18 (a) L 2(21 + 1) (b) L
LI 4 1 0 1= 1
I=n+' I=n-I
= 44.1 X 10- 18
(c) L 2(21 + 1) (d) L 2(2/ + 1)
=4.41 X 10-'7 J atom- 1
1=0 1=0
E ,&!- = -4.41 x 10- 17 J atom-I] 7. Order of no. ofrevolutionlsec YI' Y2, Y3 and Y4 forI, II, III and
@LI

139. The energy required to break one mole ofCI-C1 bonds in Cl 2


IV orbits is:
is 242 kJ mor'. The longest wavelength of light capable of (a)Y'>Y2>Y3>Y4 (b)Y4>Y3>Y2>Y,
breaking single CI-Cl bond is: (AIEEE 2010) (c) Y1 > Y2 > Y4 > Y3 (d) Y2 > Y3 > Y4 > Y1
(c == 3 X 108 m sec-I, N A 6.023 x Hr3 mol-') 8. Consider the following statements:
(a) 700nrn (b)494nrn (c)594nm (d)640nm (A) Electron density in the xy-plane in 3d 2 Z orbital is zero
x - y
rHO t 242 (B) Electron density in the xy-plane in 3d 2 orbital is zero
l-_Ift : Bond energy of single bond = 23 z
6.023 x 10 (C) 2s-orbital has one nodal surface
=4.017xI0-22 kJ
(D) For 2pz-orbital yz is the nodal plane,
=4.017xlO- 19 J
Which are the correct statements?
E= he (a) (A) :-nd (C) (b) (B) and (t)
A
(c) Only (B) (d) (A), (B), (e) and (D)
6.626 x 10-34 x 3 X 108
4.017 X 10- 19 9. The first emission line in the H-atom spectrum in the Balmer
A series appears at:
A = 4.94 X 10-7 m = 494 run1
240. In Sommerfeld's modification of Bohr's theory, the trajectory (a) 5R cm- 1 (b) 3R cm- 1 (c) 7R cm- I (d) 9R m1
of an electron in a hydrogen atom is: (JEE (WB) 20101 36 4 144 400
(a) perfect ellipse
10. I BM is equal to:
(b) a closed ellipse like curve, narrower at the perihelion (a) ~4 (b) ~ (c) ihe ., (d) eke
position and flatter at the aphelion position • m1te 41tm ,4m 1tm
(c) a closed loop on spherical surface 11. Radial probability distribution curve
(d) a rosette is shown for s-orbital. The curve is:
(a) Is
Set-2: The questions given below may have more
(b) 2s
than one corTeCI answers
(c) 3s
I. Correct order of radius of the 1st orbit of H, He+, Li 2 + and (d) 4s
Be 3+ i s : ' Time-
12. dz 2 orbital has:
(a) H > He+ > Li 2+ > Be3 + (a) a lobe along z-axis and a ring along xy-plane
(b) Be3+ > Li 2+ > He+ > H (b) a lobe along z-axis and a lobe along xy-plane
(c) He+ > Be 3+ > Li 2+ > H (c) a lobe along z-axis and a ring along yz -plane
(d) a lobe and ring along z-axis
(d) He+ > H > Li 2+ > Be3 +
I
ATOMIC STRUCTURE 135

13. When a light of frequency vI is incident ona metal surface the 20. CQnsider the followip.g sets, of quantum numbers:
photoelectrons emitted have twice the kinetic energy as did the ',; "n ' '. ','/;T; :. '. , . • m s
photoelectron emitted when the same metal has irradiated with (A) 3 0 0 + Yz
light of frequency V2' What will be the value of threshold (B) 2 2 + Yz
frequency?
(C) 4 3":'2 - Yz
(a) Vo = VI v2 (b) Vo VI 2V2
(d) Vo = VI + v 2
(D) I 0 -1 - Yz
(c) Vo = 2v, - v 2
14.. Heisenberg's uncertainty principle is not valid for:
(E) 3 2 3 +Yz
Which of the following sets of quantum numbers is not
(a) moving electrons (b) motor car possible? [eBSE (Med.) 2007]
(c) stationary particles (d) all of these (a) (A), (B), (C) and (D) (b)(B), (D) and (E)
15. Consider these electronic configurations for neutral atoms; (c) (A) and (C) (d) (B), (C) and (D)
(i) li2s2 2p 6 3s1 (1i) li2s2 2p 6 4sl 21. For three different metals A, B, C photo-emission is observed
Which of the following statements is/are false? one by one. The graph of maximum kinetic energy versus
(a) Energy is required to change (i) to (il) frequency of incident radiation are sketched as :
(b) (i) represents 'Na' atom (BHU (Screening) 2010)

~~ KL~-."-.
(c) (i) and (ii) represent different elements
(d) More energy is required to remove one electron from (i) than
(ii).
16. For the energy levels in an atom which one of the following
statements is/are correct?
(a) There are seven principal electron energy levels v- v-
(b) The second principal energy level can have 4 subenergy (a) (b)
levels and contain a maximum of 8 electrons
(c) The M energy level can have a maximum of 32 electrons
(d) The 4s subenergy level is at a lower energy than the 3d
subenergy level
17. Which of the following statements are correct for an electron
that has n = 4 and m :::: -:- 2 ?
(a) The electron may be in ad-orbital v- v-
. (c) (d)
(b) The electron is in the fourth principal electronic shell
(c) The electron may be in a p-orbital 22. For which of the following species, the expression for the
(d) The electron must have the spin quantum number = + 1/2
18. The angular momentum of electron can have the value(s): energy of electron in the nth [En:::: 13.6 Z2 J
atom-I has

(a) h (b) h
the validity? [SHU (Mains) 2010)
2n n
(a) Tritium (b) Li 2 +
(c) 2h (d)~~
n 22n (c) Deuterium (d) He 2+
19. Which ofthe following statements is/are wrong?
(a) If the value of 1= 0, the electron disttibution is spherical
(b) The shape of the orbital is given by magnetic quantum no.
(c) Angular moment of Is, 2s, 3s electrons are equal
(d) In an atom, all electrons travel with the same velocity
136 G.R.B. PHYSICAL CHEMISTRY FOR COMPETITIONS

Assertion-Reason TYPE QUESTIONS

Set-1 S.t-2
The questions given below consist of an 'Assertioa' (A) and
the 'ReasGII' (R). Use the following keys for the appropriate The questions given below consist of two statements as 'Assertion'
answer: (A) and 'Reasoa" (R); while answering these choose anyone of
them:
(a) If both (A) and (R) are correct and (R) is the correct reason for
(a) If (A) and (R) are both correct and (R) is the correct reason for
(A).
(A).
(b) If both (A) and (R) are correct but (R) is not the correct
(b) If (A) and (R) are both correct but (R) is not the correct reason
explanation for (A).
for (A).
(c) If (A) is correct but (R) is incorrect.
(c) If (A) is true but (R) is false.
{d} If(A) is incorrect but (R) is correct.
(d) Ifboth (A) and (R) are false.
I. (A) F-atom has less electron affinity than cr atom. 11. (A) A special line will be seen for 2px - 2py transition.
(R) Additional electrons are repelled more effectively by 3 p (R) Energy is released in the form of wave of light when the
electrons in CI atom than by 2p electrons in F-atom. electron dropsfrom2px to -2py orbital. (AHMS 1996}--
2. (A) Nuclide rgAI is less stable than ~Ca. U. (A) Ionization potential of Be (At. No. = 4) is less than B (At.
No. =5).
(R) Nuclide having odd number of protons and neutrons are
generally unstable. . (lIT 1998) (R) The first electron released from Be is of p-orbital but that
from B is of s-orbital. (AIlMS 1997)
3. (A) The first IE of Be is greater than that of B.
13. (A) In Rutherford's gold foil experiment, very few a-particles
(R) 2p-Olbital is lower in energy than 2s.
are deflected back.
4. (A) The electronic configuration of nitrogen atom is
(R) Nucleus present inside the atom is heavy.
represented as:

If rt Iii
14. (A) Limiting line in the Balmer series has a wavelength of
DO
and not as:'
[J[] 364.4mm.
(R) Limiting line is obtained for a jump of electron from
n = 00.
[U 15. (A) Each electron in an atom has two spin quantum numbers.
(R) Spin quantum numbers are obtained by solving
(R) The electronic configuration of the ground state of an Schrodinger wave equation.
atom is the one which has the greatest mUltiplicity. 16. (A) There are two spherical nodes in 3s-orbital.
5. (A) The atomic radii of the elements of oxygen family are (R) There is no planar node in 3s-orbital.
smaller than the atomic radii of corresponding elements of 17. (A) In an atom, the velocity of electron in the higher orbits
the nitrogen family. keeps on decreasing.
(R) The members of oxygen family are all more (It) Velocity of electrons is inversely proportional to radius of
electronegative and thus have lower value of nuclear the orbit.
charge than those of the nitrogen family. II. (A) If the potential difference applied to an electron is made 4
6. (A) For n = 3, I may be 0, I and 2 and may be 0, ±l and 0, ±l times, the de Broglie wavelength associated is halved.
and±2. (R) On. making potential difference 4 times, velocity is
(R) For each value of n, there are 0 to (n - 1) possible values doubled and hence d is halved.
of I; for each value of I, there are 0 to ±l values of m. 19. (A) Angular momentum of Is, 2s, 3s, etc., all have spherical
7. (A) An orbital cannot have more than two electrons. shape.
(R) The Iwo electrons in an orbital create opposite magnetic (R) Is, 2s, 3s,etc., all have spherical shape.
field. 10. (A) The radial probability of Is electron first increases, till it is
I. (A) The configuration ofB-atom cannot be 1s2 2s2 . maximum at 53 A and then decreases to zero.
(R) Hund's rule demands that the configuration should disph~! (R) Bohr radius for the first orbit is 53 A.
maximum multiplicity. 11. (A) On increasing the intensity of incident radiation, the
9. (A) The ionization energy ofN is more than that ofO. number of photoelectrons ejected and their KE increases.
(R) Electronic configuration of N is more stable due to half- (R) Greater the intensity means greater the energy which in
filled 2p-orbitals. tum means greater the frequency of the radiation.
18. (A) p-orbital is dumb-bell shaped. ll. (A) A spectral line will be seen for a 2px 2py transition.
(R) Electron present in p.:orbital can have anyone of the three (R) Energy is released in the form of wave of light when the
values of magnetic quantum number, i.e., 0, +1 or-I. electron drops from 2px to 2py orbital. (VMMC 2007)
I

ATOMIC STRUCTURE I 137

• Set-1
1. (b) 2. (a) 3. (c) 4. (d) 5. (c) 6. (b) 7. (d) 8. (a)
9. (a) 10. (b) 11. (d) 12. (d) 13. (a) 14. (c) 15. (b) 16. (a)
17. (c) 18. (d) 19. (b) 20. (c) 21. (d) 22. (c) 23. (b) 24. (a)
25. (d) 26. (b) 27. (d) 28. (a) 29. (d) 30. (b) 31. (c) 32. (b)
·33. (a) 34. (c) 35. (d) 36. (a) 37. (c) 38. (a) 39. (b) 40. (c)
41. (c) 42. (b) 43. (d) 44. (d) 45. (c) 46. (a) 47. (b) 48. (d)
49. (d) SO. (c) 51. (b)· 52. (b) 53. (a) 54. (b) 55. (a) 56. (a)
57. (c) 58. (d) 59. (a) 60. (d) 61. (c) 62. (a) 63. (a) 64. (b)
65. (a) 66. (d) 67. (d) 68. (b) 69. (c) 70. (c) 71. (a) 72. (d)
73. (d) 74. (b) 75. (c) 76. (d) 77. (b) . 78. (d) 79. (a) 80. (d)
81. (b) 82. (b) 83. (c) 84. (a) 85. (b) 86. (a) 87. (b) 88. (c)
89. (b) ...94)~J4!L. !1.. _(at .. !2.Jd), 93. (a) 94. (a) 95. (b) 96. (c)
97. (a) 98. (d) 99. (b) 100. (b) 101. (d) 102. (d) 103. (a) 104. (c)
105. (c) 106. (c) 107. (d) 108. (b) 109. (d) 110. (a) 111. (a) ~ 112. (b)
113. (c) 114. (d) 115. (a) 116. (d) 117. (b) 118. (a) 119. (c) 120. (c)
121. (d) 122. (b) 123. (d) 124. (c) 125. (a) 126. (a) 127. (e) 128. (e)
129. (b) 130. (b) 131. (c) 132. (c) 133. (c) 134. (b) 135. (c) 136. (d)
137. (b) 138. (a) 139. (a) 140. (a) 141. (b) 142. (c) 143. (c) 144. (a)
145. (b) 146. (c) 147. (d) 148. (c) 149. (d) 150. (d) 151. (a) 152. (d)
153. (c) 154. (a) 155. (c) 156. (a) 157. (a) 158. (c) 159. (c) 160. (d)
161. (c) 162. (c) 163. (b) 164. (b) 165. (a) 166. (b) 167. (b) 168. (b)
169. (d) 170. (d) 171. (b) 172. (b) 173. (d) 174. (b) 175. (c) 176. (a)
177. (d) 178. (b) 179. (d) 180. (a) 181. (d) 182. (a) 183. (a, b) 184. (c)
185. (a) 186. (a) 187. (b) 188. (b) 189. (b) 190. (b) 191. (a) 192. (c)
193. (c) 194. (b) 195. (a) 196. (a) 197. (d) 198. (a) 199. (c) 200. (a)
201. (a) 202. (c) 203. (c) 204. (a) 2OS. (a) 206. (b) 207. (c) 208. (c)
209. (d) 210. (c) 211. (d) 212. (a) 213. (a) 214. (e) 215. (a) . 216. (b)
217. (d) 218. (d) 219. (d) 220. (c) 221. (b) 222. (b) 223. (b) 224. (d)
225. (c) 226. (a) 227. (c) 228. (d) 229. (c) 230. (a) 231. (c) 232. (b)
233. (c) 234. (a) 235. (a) 236. (a) 237. (c) 238. (d) 239. (b) 240. (c)
• Set-2
1. (a) 2. (b) 3. (a) 4. (c) 5. (c) 6. (d) 7. (a) 8. (a)
9. (a) 10. (a) 11. (a) 12. (a) 13. (e) 14. (b, c) 15. (c, d) 16. (a, d)
17. (b, c) 18. (a, b, c) 19. (c) 20. (b) 21. (d) 22. (a, b, c)

1. (c) 2. (a) 3. (c) 4. (a) 5. (c) 6. (a) 7. (b) 8. (c)


9. (c) 10. (a) n. (d) 12. (d) 13. (b) 14. (a) 15. (d) 16. (b)
17. (c) 18. (a) 19. (b) 20. (b) 21. (d) 22. (d)
138 G.R.B. PHYSICAL CHEMISTRY FOR COMPETITIONS

r"~~;~7"'.i;l:lmii~ __it·]O$'i~GJu;t·]:j·4$~ftI~~~~~
IOBJECTIVE QUESTIONS] for ( lIT ASPIRANTS' .~ .I
The following questions contain a single correct option:
4. Maximum value (n + I + m) for unpaired electrons in second
1. The configuration of Cr atom is 3d 54s1 hut not 3d 44s 2 due to excited state of chlorine l7Ci is:
reason RI and the configuration of Cu atom is 3d 10 4S1 hut not (a) 28 (b) 25
3d 94s 2 due to reason R 2 , RI and R2 are: (c) 20 (d) none of these
(a) R 1: The exchange energy of 3d 54s1 is greater than that of [Hint: Configuration in second excited state may be given as:
3d 44s 2 •
R 2 : The exchange energy of 3d 10 4s1 is greater than that
of3d 94s 2 •
(h) R1: 3d 54s1 and 3d 44s 2 have same exchange energy but
3d 54s1 is spherically symmetrical.
n 3 3 3 3
2 3]
2 . Total n + 1+ m = 25]
R 2 : 3d 104s1 is also spherically symmetricaL m -I o +1 +2 +1
( c) R 1 : 3d 5 4s 1 has greater exchange energy than 3d 4 4i.
5. Which of the following is correctly matched?
R 2: 3d 104s1 has spherical symmetry.
(a) Momentum ofH-atom when electrons
(d) R 1 : 3d 54s1 has greater energy than 3d 4 4i.
R 2: 3d 104s1 has greater energy than 3d 94i·. return from n := 2 to n I 3Rh
4
[Hint: 3d 54s I is correct because it has greater exchange (b) Momentum of photon Independent of wavelength
possibilities of unpaired electrons. of light
1-2, 1-3, 1-4, 1-5
11 I 111 11 11 I :~~~~~lf~es:
(10)
2-3,2-4,2-5
3-4,3-5
(c) elm ratio of anode rays Independent of gas in the
discharge tube
2 3 4 5 (d) Radius of nucleus (Mass no.)1/2
4-5
[Hint: 1 = R [~ ~J := 3R
3d I04s 1 is correct because 3d to-orbitals are spherically A 12 22 4
symmetrical. ] h
A=-
2. Which of the following graphs is incorrect? p
p h = h x 3R 3Rh]
A 4 4
t
v
6. In hydrogen spectrum, the third line from the red end
corresponds to which one of the following inter-orbit jumps of
I the electrons from Bohr orbit of hydrogen?
(a) 4 ~ I (b) 2 ~ 5 (c) 3 ~ 2 (d) 5 ~ 2
7. In which of the following pairs is the probability of finding the
electron in xy-plane zero for both orbitals?
(a) (b)
(a) 3d yz , 4dx 2 - y 2 (b) 2Pz, d z 2
(c) 4dzx',3pz (d) None of these
tv tv 8. In which of the following orbital diagrams are both Pauli's
exclusion principle and Hund's rule violated?
I I (a)@] I I II J, I I (b)
r----,---.-:----.
@] I t It J, II
z ----+ n--+
(c) (d) (C)@] IH Itt I J, I (d) @] IHI II I
3. Which among the following is correct of 5B in normal state?
9. The distance between 3rd and 2nd Bohr orbits of hydrogen
2s 2p atom is:
(a) 0.529 x 10-8 cm (b) 2.645 x 10-8 cm
(a) [ill I I I Against Hund's rule
(c) 2.1l6xI0-8 cm (d) 1.058xlO-8 cm
2
[Hint: 1j r2 = (3 - 22) x 0.529 X 10-8 cm]
. (b)[i] L......!--'--'--'---' Against aufbau principle
as well as Hund's rule 10. Which diagram represents the best appearance of the line
(c) [ill '---'--'---'-----" Violation ofPau!i's exclusion spectrum of atomic hydrogen in the visible region?
principle and not Hund's rule (PET (Kerala) 2007)
(d) [ill I'----!...--"---,
L-I
Against aufbau principle
I
ATOMIC STRUCTURE 139
Increasing wavelength 17. A photosensitive material would emit electrons if excited by
photons beyond a threshold. To overcome the threshold, one
(a) would increase: (VITEEE 2007)
(a) the voltage applied to the light source
(b) the intensity of light
(b)
(c) the wavelength of light
(d) the frequency oflight .
(c) 18. Which of the following electronic configurations have the
highest exchange energy?
3d 4s
(d)
1 I I (a) I iii i I IITJ
(b) ITJ
(e)
11111 II II III (c) i
11. The 'm' value for an electron in an atom is equal to the number
of m values for 1= I. The electron may be present in: (d) IT]
~a) 3dx2 _ y2 . (b) 5~(X2_y2) 19. Which of the following graphs correspond to one node?
(c) 4/3
x Iz
(d) noneofthese
[Hint: Total values of m (21 + I) = 3 for 1 = 1 t
'If
m 3 is for f-subshellorbitals.]
12. If m = magnetic quantum number, I = azimuthal quantum
number, then:
(a) m=I+2 (b) m 2P + 1 -lao -lao
m-I (a) (b)
(c) 1= - (d) 1= 2m+ 1
2
[Hint: Magnetic quantum number 'm' lies between (-I, 0, + I);
thus total possible values of 'm' wiH be (21 + I). t t
m -I 'If 'If
m 21+1, i.e.,. 1= 2 ]
-lao
13. What are the values of the orbital angular momentum of an
electron in the orbitals Is, 38, 3d and 2p?
(0) (d)
(a) 0,0, .J61i, J2 Ii· (b) I, I, J4 Ii, J2 n
(c) 0, 1,.J6 Ii, J3 Ii (d) 0,0,50 1i,.J61i 20. Angular distribution functions of all orbitals have:
(a) I nodal surfaces (b) (l-l)nodal surfaces
[Hint: Orbital angular momentum = ~l(l + 1) h = ~1(1 + 1)il] (c) (n + I) nodal surfaces (d) (n 1- I)nodal surfaces
21£
21. If uncertainty in position and momentum are equal then
14. After np-orbitals are filled, the next orbital filled will be:
uncertainty in velocity is: [CBSE.PMT (Pre.) 2008]
(a) (n + l)s (b) (n + 2)p (c) (n + l)d (d) (n + 2)s
IS. The ratio of (E2 - Ej ) to (E4 E3 ) for the hydrogen atom is (a) fI
V;:
(b) fh
V"2n
(c) _1
2m
fI
V;:·
(d).!
m
fI
V;:
approximately equal to:
(a) ·10 (b) 15 (c) 17 (d) 12 h
[Hint: tJ,x·Ap;;:-
[Hint: 41£

(-~)-(-n 1 1
"9 - 16 _ 7 4
----x
7 1
Ap~ H.: when tJ,x = IIp

108 '" 15 ]
(-±) 3 144 3 mAv;;: rT
-(-I) 4 f41t
16. Which ofthe following electronic configurations hal1 zero spin Av;;:_I_
2m
fE]
V;:
multiplicity?
22. The number of waves made by a Bohr electron in an orbit of
(a) It Ii maximum magnetic quantum number 3 is:
(a) 3 (b) 4 (c) 2 (d) 1
(c) I i I J, [Hint: m == 3, I = 3, n = 4
[Hint: Spin multiplicity =(2u + 1)] For, n = 4, number of waves will be 4.]
140 G.R.B. PHYSICAL CHEMISTRY FOR COMPETITIONS

23. The number of elliptical orbits excluding circular orbits in the h -.


mv9x=3-
N·shell ofan atom is: 21t
003 004 002 WI h = 61tx
[Hint: For, N·shelI, n = 4. This shell will have one Circular and mv
three elliptical orbits.] A.=6ru:]
24. From the electronic configuration of the given elements K, L, 30. How many times does light travel faster in vacuum than an
M and N, which one has the highest ionization potential? electron in Bohr first orbit of hydrogen atom?
(a) M = [Ne] 3i3p2 (b) L=[Ne] 3s 23p 3 (a) 13.7 times (b) 67 times (c) 137 times (d) 97 times
(c) K=[Ne]3i3/ (d) N=[Ar]3d 1O
,4i4p 3 [Hint: v = ~ x 2.188 X 108 cmlsec
n
[Hint: L has half·filled p·subshell and it is smaller than N,
hence, L wiII have the highest ionization potential.] VI = ~ x 2.188 X 108 cmlsec
I
25. Which of the following pairs of electrons is excluded from an
Velocityoflight 3xHYo
atom? ---~---'=--- = 2.188 = 137'
tImes
]
I 1 Velocity of electron x 108
(a) n = 2, 1= 0, m = 0, s + - and n = 2, 1= 0, m = 0, s = + .;...
2 2 31. A compound of vanadium has a magnetic moment of 1.73 BM.
I . The electronic configuration of vanadium ion in the
(b) n = 2, I = I, m = + I, s = +- compound is:
-------- - --- . 2 -r'- (a) [Ar]3d 2 (b) [Ar]3d 14s0 (c)[Ar]3d 3{d) [Ar]3d °4sl-
and n ::: 2, I = 1, m =- 1, s = + -
2 [Hint: Magnetic moment = In(n + 2) BM
1 I
(c) n = 1, I::: 0, m:::.0' s::: + 2 and n = 1, I = 0, m = 0's:::: - 173 = In(n + 2)
2
I
Jj::: In(n + 2)
(d) n =3, I =2, m - 2, s = + - n:::: I (number of unpaired electrons)
2
V 22 ~3d 24i
I
andn 31=Om=Os=+
, , , 2 V 3+ -t 3d '4i (has one unpaired electron)]
[Hint: Both 2s electrons have same spin, hence excluded from 32. The orbital angular momentum of an electron in p·orbital is:
the atom.] [PET (Kerala) '2006]
(d) .!.~
26. Given set of quantum numbers for a multielectron atom is: (b) _h_ h
(a) zero (c) -
n I m s .fi1t 21t 221t
2 0 0 + 112
2 0 0 -1/2 (e) ~
2-v 21t
What is the next higher allowed set of 'n' and'/' quantum 33. When a hydrogen atom emits a photon of energy 12.1 e V, the
numbers for this atom in the ground state? orbital angular momentum changes by:
(a) n = 2, I = 0 (b) n = 2, I = I (a) L05 x 10-34 J sec (b) 211 x 10-34 J sec
(c) n=3,1=0' (d) n=3,I=I
27. In how many elements does the last electron have the quantum (c) 3.16 x 10-34 J sec (d) 4.22 x 10-34 J sec
numbers of n 4 and I = I ? [Hint: Emission of photon of 12.1 eV corresponds to the
(a) 4 (b) 6 (c) 8 (d) 10 transition from n =- 3 to n = L
[Hint: n::: 4, I::: I represent 4p·subshell containing six :. Change in angular momentum
electrons. Thus, there will be six elements having 4pl to 4l h
=-(~ -nl)-
electronic configuration.]
21t
28. If there are three possible values (-112, 0, + 112) for the spin =(3 -l)~=!:
quantum, then electronic configuration ofK (19)will be: 21t 1t
6.626 X 10-34
(a) Is3 , 2s3 2p 9, 3s3 3pl (b) Is2 , U2 p 6, 3i3p 6, 4S1 =
3.14
(c) Ii, U2 p 9, 3i 3p4 (d) none of these
= 2.11 x 10-34 J sec]
29. lethe radius offirst Bohr orbit of hydrogen atom is 'x' then de
34. The total energy of the electron of H·atom in the second
Broglie wavelength of electron in 3rd orbit is nearly:
quantum state is - E2 • The total energy of the He + atom in the
(d) ~ third quantum state is:
(a) 2m; (b) 61tx (c) 9x
[Hint: r" =n 2rJ 3
(a) -(~)E2 (b) G)E2 (c) (~)E2 (d) -C:)E2
13::: 91j = 9x
h [Hint: Energy of electrons in n th state
mvr=n Z2
21t =- -2 X 13.6eV
n
ATOMIC STRUCTURE 141

E2(H) = - 13.6 eV (a) 1..= 1.23 (b) 1..= 1.23m (c) 1.23 nm
(d) 1..= 1.23
1 .r;;, Jh .JV V
E 3 (He+ ) -
- 13.6 x 4 eV 40. An excited electron ofH-atoms emits of photon of wavelength
-- -9-
A and returns in the ground state, the principal quantum
number of excited state is given by :

For negative value of E2 , E3 will also be negative.] (a) ~AR (AR - I) (b) ~ (A~~ I) .
35. What is the ratio of the Rydberg constant for helium to·
hydrogen atom? (c) I (d) ~(AR - I)
(a) 1/2 (b) lI4 (c) lI8 (d) 1/l6 ~AR (AR -I) AR

[Hint:
-2n 2mZ 2e4
R = --,,-----
ch 3 [Hint: ±=R[~~ - ~] =RU- ~J
RHe = 2 X 22 = 8
RH Ixe ~=~ A.RA.R-I ]
RH =.!.] 41. A dye a!>sorbs a photon of wavelength A and re-emits the same
RHe 8 energy into two photons of wavelength AI and 1..2 respectively.
36. If the kinetic energy ofa particle is doubled, de Broglie The wavelength A is related to AI and A2 as : _____ _
wavelength becomes: (a) 1..= 1..11..2 (b) 1..= AI + 1..2
(a) 2 times (b) 4 times (c) .J2 times (d) _1_ times (AI + A2 i 1..11..2
.J2 (c) 1..= 1..11..2 (d) A~ A~
[Hint: 1..= _h_ , where, E = Kinetic energy of the particle AI + A2 AI + 1..2
.J2Em 42. The radii of maximum probability for 3s, 3p and 3d-electrons
h h
AI = - - ; 1..2 = ---;::::==- are in the order :
.J2Em J2x2Em (a) (rmax) 3$ > (rmax) 3p > (rmax )3d
Al =..fi i.e. A =~] (b) (rmax) 3$ = (rmax) 3p =(rmax )3d
1..2 ' , 2 ..fi
(c) (rmax) 3d > (rmax) 3p > (rmax )3$
37. Imagine an atom made up of a proton and a hypothetical
particle of double the mass of the electron but having the same (d)(rmax) 3d> (rmax) 3$ > (rmax )3p
charge as the electron. Apply the Bohr:s atomic model and Following questions may have more than one correct options:
consider all possible transitions of this hypothetical particle to 1. Select the correct relations on the basis of Bohr theory:
the fIrst excited level. The largest wavelength photon that will
be emitted has wavelength A (given in terms of the Rydberg (a) velocity of electron oc.!. (b) frequency of revolution oc ~
n n
constant R for the hydrogen atom) equal to: 1
9 . 36 18 (c) radius of orbit oc n 2Z 4
(d) force on electron
(a) 5R (b) 5R (c) 5R (d) i n
oc

R
[Hint: Energy is related to mass: 2. To which of the following species, the Bohr theory is not
applicable?
En oem
The longest wavelength Amax photon will correspond to the (a) He (b) Li 2+ (c) He 2+ (d) H-atom
transition of particle from n = 3 to n = 2 3. The magnitude of spin angular momentum of an electron is
given by: .
A~x =2R(;2~ ;2) (a)S = ~s(s + 1) in (b) S =:s 2:
A
max
=!!]
5R J3 h "1 h
(c) S =-x- (d) S =±-x-
38. What is ratio of time periods (11. I T2 ) in second orbit of 2 2n . 2 2n
hydrogen atom to third orbit ofHe+ ion? [Hint: Spin angular momentum == Js(s + I) ~
(a) -.! (b) 32 (c) 27 (d) 27 2n
27
n3
27 32 8
S=~.!.22(.!. + I) ~
2n
=.J3
22n
x~ ]
[Hint: T 00-
Z2 4. Select the correct confIgurations among the following:
T n
.-1.=_1_
3 x Z2 23 X_
_2 = _ 22 =_]
32 (a) Cr (Z= 24):[Ar] 3d 5,4sl
T2 zf ni
x 12 X 33
27 (b) Cu (Z = 29): [Ar] 3d 10, 4s1
39. The de Broglie wavelength of an electron accelerated by an (c) Pd (Z= 46):[Kr] 4d 1O,5so
electric fIeld of V volt is given by :
(d) Pt (Z = 78): [Xe] 4d 10 4i
I

142 G.R.B. PHYSICAL CHEMISTRY FOR COMPETITIONS

5. Which among the following statements is/are correct? 11. The maximum kinetic energy of photoelectrons is directly
(a) lJI2 represents the atomic orbitals proportional to ... of the incident radiation. The missing word
can be:
(b) The number of peaks in radial distribution is (n - I)
(a) intensity (b) wavelength
(c) Radial probability density Pnl (r) 41t? R;/(r}
(c) wave number (d) frequency
(d) A node is a point in space where the wave function (lJI ) has 12. Rutherford's experiment established that:
zero amplitude
(a) inside the atom there is a heavy positive centre
6. Select the correct statement(s} among the following: (b) nucleus contains protons and neutrons
(i) Total number of orbitals in a shell with principal quantum
(c) most of the space in an atom is empty
number 'n' is n 2
(d) size of nucleus is very small
(ii) Total number of subs hells in the n th energy level is n
13. Which of the following orbital(s) lie in the xy-plane?
(iii) The maximum number of electrons in a subshell is given
by the expression (41 + 2)
(a) d x2 _ y2 (b) dX), (c) dxz (d) Fa
(iv) m = 1+ 2, where 1 and m are azimuthal and magnetic 14. In which ofthe following sets of orbitals, electrons have equal
. quantum numbers .orbital angular momentum?
(a) (i), (iii) and (iv) are correct (a)13and2s (b) 2sand2p (c) 2p and 3p(d) 3pand3d
(b) (0, (ii) and (iii) are correct 15. Which ofthe following orbitals have no spherical nodes?
(c) (ii), (iii) and (iv) are correct (a) 13 (b) 2s (c) 2p (d) 3p
(d) (i), (iiyand (iv) are correct Hi~ For a'snellofpriiiCipalquaniiirri Ilurnber'n~~4, tnere are:~
7. Which among the following are correct about angular (a) 16 orbitals (b) 4 subshells
momentum of electron? (c) 32 electrons (maximum) (d) 4 electrons with 1= 3
17. The isotopes contain the same number of:
(a) 2 n (b) 1.5!: (c) 2.5ft (d) 0.5!:
1t 1t (a) neutrons (b) protons
8. Which of the following is/are incorrect for Humphrey lines of (c) protons + neutrons (d) electrons
hydrogen spectrum? 18. Which of the following species has less number of protons
(a) n2 7 -7 n! = 2 (b) n 2 10 -7 n! = 6 than the number of neutrons?
(c) n2 5 -7 n! = 1 (d) n2 11-7 n! = 3 (a) !~C (b) 19F (c) nNa (d) fiMg
9. In the Bohr's model ofthe atom: 19. The angular part of the wave function depends on the quantum
(a) the radius of nth orbit is proportional to n 2 numbers are:
(b) the total energy ofthe electron in the n th orbit is inversely (a) n (b) I . (c) m (d) s #

proportional to 'n' 20. Which of the following species are expected to have spectrum
(c) the angular momentum of the electron is integral multiple ,similar" to hydrogen? '
of h/21t (a)He+ (b) He 2+ (c)'U 2 + (d)U+
(d) the magnitude of potential energy of an electron in an orbit 21. Which of the following statements is/are correct regarding a
is greater than kinetic energy hydrogen atom?
10. Which among the following series is obtained in both (a) Kinetic energy ofthe electron is maximum in the first orbit
absorption and emission spectrums?
(b) Potential energy of the electron is maximum in the first orbit
(a) Lyman series (b) Balmer series
(c) Radius of the second orbit is four times the radius of the
(c) Paschen series (d) Brackett series first orbit
(d) Various energy. levels are equally spaced

[;/I~ ..-:- - - - - - - - - - - - - - - - - - - - - 1
• Single ,correct option
1. (e) 2. (d) 3. (c) 4. (b) :5~ (a) 6. (d) 7. (d) 8. (d)
9. (b) 10. (c) 11. (b) 12. (e) 13. (a) 14. (a) 15. (b) 16. (c)
17. (d) 18. (b) 19. (b) 20. (a) 21. (c) 22. (b) 23. (a) 24. (b)
25. (a) 26. (b) 27.. (b) 28. (a) 29. (b) 30. (c) 31. (b) 32. (b)
33. (b) 34.. (e) 35. (e) 36. (d) 37. (e) 38. (b) 39. (e) 40. (b)
41. (e) 42. (a)

• One or more than one correct ,options '4.


1. (a, b, d) 2. (a, e) 3. (a, c) (a,b,e) 5. (a, b, c, d) 7. (a, b, d) 8. (a, c, d)
.6. (b)
9. (a, c, d) 10. (a) 11. (e, d) 12. (a, e, d) 13. (a, b) 14. (a, c) 15. (a, e) 16. (a, b, e)
17. (b, d) 18. (b, c) 19. (b, e) . .20. (a, c) 21. (a, c}
.. "
ATOMIC STRUCTURE 143

Integer Answer TYPE QUESTIONS


This section contains 10 questions. The answer to each 48.24 nm. From the same orbit, wavelength corresponding to a
of the questions is a single digit integer, ranging from transition to third orbit is 142.46 nrn. The value of n2 is :
oto 9. If the correct answers to question numbers X, Y, 9. The energy corresponding to one of the lines in the Paschen
Z and W (say) are 6, 0, 9 and 2 respectively, then the series for H-atom is 18.16 x 10-20 J. Find the quantum numbers
correct darkening of bubbles will look like the figure: for the transition which produce this line.

[Hint: AE= 2. 18XlO-18[~-J..J


nf ~
1. For Li 2+, when an electron falls from a ~gher orbit to nth
orbit, all the three types of lines, i.e., Lyman, Balmer and
2o
18.l6xlO- = 2. 18XlO-
18
[i- :2]
Paschen was found in the spectrum. Here, the value of' n' will On solving, n = 6 ]
be: 10. The angular momentum of electron in the shell in which the
2. The emission lines of hydrogen contains ten lines. The highest . g-subshell first appears is x x ~. The value of x will be :
orbit in which the electron is expected to be found is : 21t
[Hint: Number of lines == n(n -1) = 10 [Hint: I = 4 for g-subshell
.. ..... . .2--- Thus, the subshell will first appear in (n = 1+ 1<= 5) 5th shell.._ -
.• n=5] h
Angular momentum (mur) = n -
3. Total number of nodes present in 4d orbitals will be: 21t
4. Spin mUltiplicity of nitrogen in ground state will be : h
=5-
5. Orbital frequency of electron in nth orbit of hydrogen is twice .2n:
that of 2nd orbit. The value of n is : n= 5]
6. If kinetic energy of an electron is reduce by (1/9) then how
many times its de Broglie wavelength will increase.
7. If electrons in hydrogen sample return from 7th shell to 4th
shell then how many maximum number of lines can be
observed in the spectrum of hydrogen.
8. An electron in Li 2+ ion is in excited state (n2) . The
wavelength corresponding to a transition to second orbit is

[~. ~'
.... ~
....
I
1. (1) 2. (5) .3. (3) 4.. (4) 5. (1) 6. (3) 7. (6) 8. (5)
9~ (6) l(t (5)
I
144 I G.R.B. PHYSICAL CHEMISTRY FOR COMPETITIONS

• Passage 1 5. Which of the following is not correctly matched?


The observed wavelengths in the line spectrum of hydrogen atom (a) 6569 A (Red) (b) ~ - 4861 A (Blue)
were first expressed in terms ofa series by Johann Jakob Balmer, a (c) liy- 4340 A (Orange) (d) He - 4101 A (Violet)
Swiss teacher.
s
Balmer empirical formula is: • Passage 2
Aformula analogous to the Rydbergformula applies to the series
i=RH [; - nI2]n=3,4,5, ...
of spectral lines which· arise from transitions from higher energy
level to the lower enetID' level ofhydrogen atom.
R H = 109678 cm-\ is the Rydberg constant.
A muonic hydrogen atom is like a hydrogen atom in which the
Niels Bohr derived this expression theoretically in 1913. The electron is replac~d by a heavier particle, the 'muon '. The mass of
formula is generalised to anyone electron atom/ion. the muon is about 207 times the mass ofan electron, while the charge
Answer the following questions: remains same as that of the electron. Rydberg formula for hydrogen
1. Calculate the longest wavelength in A (l A 10- 10 m) in the atom is:
Balmer series of singly ionized helium He +. Select the correct 1
artswer.-Ignore the nuClear monon iIi your calCulation. A
(a) 2651 A (b) 1641.1 A
(c) 6569 A (d) 3249 A Answer the following questions:

[Hint: _1-
AHe+
=RHZ2 [J.. -J..J
22 3 2
1. Radius of first Bohr orbit of muonic hydrogen atom is:
(a) 0.259 A (b) 0.529 A
207 207
= 109678 x 4 [:6J (c) 0.529 x 207 A (d) 0.259 x 207 A
2. Energy of first Bohr orbit of muonic hydrogen atom is:
AHe+ =1641.1 A]
(a) - 13.6 eV (b) -13.6 x 207eV
2. How many lines in the spectrum will be observed when 207
electrons return from 7th shell to 2nd shell?
(c) + 13.6 eV (d) +13.6 x 207 eV
(a) 13 (b) 14 (c) 15 (d) 16 207
[Hint: Number of lines in the spectrum 3. Ionization energy of muonic hydrogen atom is:
= (1~ - nl )(~ - n) + I) (a) +13.6 eV (b) +13.6 x 207eV
2 207
_ (7-2)(7-2+ 1) IS
(c) - 13.6 ~V (d) -13.6 x 207eV
2 207
7- 4. Angular momentum of 'muon' in muonic hydrogen atom may

ll1~~~
be given as:
6-
S- 15 lines in the spectrum.] (a) h (b) ~
(c) h (d) h
'IE 2n 4'IE 61t
4-
5. Distance between first and third Bohr orbits of muonic
3- hydrogen atom will be:
2- 2
(a) 0.529 x 2A (b) 0.529 x 7A
3. The wavelength of first line of Balmer spectrum of hydrogen 207 . 207
will be: (c) 0.529 x 8A (d) 0.529 A
(a) 4340 A (b) 4101 A (c) 6569 A (d) 4861 A 207 207
[Hint: .!.A = RH [J.. -~J
n2
22 • Passage 3
for fIrst line n = 3,
I [ I I]
-=109678 - - -2
A. 22 3
Nuclei that have 2,8,20,28,50,82 and 126 neutrons or protons
are more abundant and more stable than other nuclei ofsimilar mass.
It is suggested that in the nuclear structure ofthe numbers 2, 8, 20,28,
A= 6569A] 50, 82 and 126, which have become known as magic, nu11ll}(!rs, the
4. In which region of electromagnetic spectrum does the Balmer nuclei possessing magic numbers are spherical and have zero
series lie? quadruple moment and hence they are highly stable. Nuclear shells
(a) UV (b) Visible are filled when there are 2, 8, 20, 28, 50, 82 and 126 neutrons or .
protons in a nucleus. In even-even nuclei all the neutrons and protons
(c) Infrared (d) Far infrared
are paired and cancel out spin and orbital angular momenta.
ATOMIC STRUCTURE 145

Answer the followhig questions regarding the stability of The only way to explain the results, Rutherford found,. was to picture
nucleus: an atom as being composed of a tiny nucleus in which its positive
1. Which of the following element(s) is/are stable though having charge and nearly all its mass are concentrated. Scattering of
odd number of neutrons and protons? a-particles is proportional to target thickness and is inversely
(a) ~Li (b) I~B (c) iRe (d) IjN . proportional to the fourth power of sin %' where, (-) is :'cattering
2. Stable nuclei having number of neutrons less than number of
angle. Distance ofclosest approach may be calculated as:
protons are:
2
(a) iH (b) ~He Z;Z2e
1:. =-.--
mill 4nEoK
3. Doubly magic nucleus is ...
(a) 2~iPb (b) 2~Pb (c) 2~~Pb (d) 2~;Bi where, K kinetic energy ofa-particles.
Answer the follOwing questions:
4. Which among the following has unstable nucleus?
1. Rutherford's a'particle scattering experiment led to the
(a) IjN (b) l~N (c) I~N (d) 1~0 . con~lusion that: '
5. Which of the following has zero spin and angular momentum? (a) mass and energy are related
(a) ~~Ca (b) iH (c) I~C (d) {jCI (b) mass ,and positive charge of an atom are concentrated in
the nucleus .
(c) neutrons are present in the nucleus
• Passage4
(d) atoms are electrically rieutrill·
The substances which contain species wi{h unpaired electrons in 2. From the a-particle scattering experiment, Rutherford
their orbitals behave as paramagnetic substances. Such. substances concluded that:
are weakly attracted by the magnetic field. The paramagnetism is (a) a-particles can approach within a distance of the order of
expressed in terms of magnetic moment. The magnetic moment is 10'-14 m of the nucleus
related to the number of unpaired electrons according to the
(b) the radius of the nucleus is less than 10- 14 ~
following relation:
(c) scattering follows Coulomb's law
Magnetic moment,!l ~n(n + 2) EM (d) the positively charged parts of the atom move with
extremely high velocities
where, n number of unpaired electrons. 3. Rutherford's scattering formula fails for very small scattering
BM stands for Bohr magneton, a unit of magnetic moment. angles because:
(a) the gold foil is very thin
IBM = eh = 9.27 x 10-24 Am2 orJT- 1
4nmc (b) the kinetic energy of a-particles is very high
(c) the full nuclear charge of the target atom is partially
Answer the following questions: screened by its electron
1. Which of the. following ions has the highest magnetic (d) there is strong repulsive force between thea-particles and
moment? nucleus of the target
(a) Fe 2+ '(b) Mn 2+ (c) Cr 3+ (d) V 3+ 4. Alpha particles that come closer to the nuclei:
2. Which of the following ions has magnetic moment equal to (a) are deflected more (b) are deflected less
that ofTi 3+: . .
(c) make more collision (d) are 'slowed down more
(a) Cu 2+ (b) Ni2+ (c) C0 2+ Cd) Fe 2+ 5. Which of the following quantities will be zero for alpha
3. An ion of a d-block element has magnetic moment 5.92 BM particles at the point of closest approach to the gold atom, in
Select the ion among the following: Rutherford's scattering of alpha particles?
(a) Zn 2+ (b) Sc 3+ (c) Mn 2+ (d) Cr 3+ " (a) Acceleration (b) Kinetic energy
4. In which ofthese options do both constituents of the pair have (c) Potential energy (d) Electrical energy
the same magnetic moment?
r
(a) Zn 2+ and Cu + (b) C0 2+ and Ni 2+ • Passage 6.
(c) Mn4+ andCo + 1 (d) Mg2+ and Sc+
The splitting ofspectral lines by a magnetic field is called the
5. Which of the following ions are diamagnetic? Zeeman effect 'after the Dutch physicist Pieter Zeeman. The Zeeman
(a) H~2+ (b) Sc 3+ (c) Mg2+ . (d) 0 2- effect is a vivid confirmation of space quantization. Magnetic
quantum number 'm' was introduced during the study of Zeeman
• Passage5 effect. 'm' can have the (21 + 1) values (-1,0, + I). Magnetic
quantum number represents the orientation of atomic orbitals in
At the suggestion of Ernest Rutherford, Hans Geiger and Ernest three-dimensional space. The normal Zeeman effect consists of the
Marsden bombarded a thin gold foil by a-particles from a polonium splitting ofa spectra/line offrequency Vo into three components, i.e.,
source. It was expected that a-particles would go right through the
e . e
roil with hardly any deflection. Although, most of the alpha particles --B; v 2 Vo +--B
indeed were not deviated by much, a few were scattered through very 4nm 4nm
large angles, Some were even scattered in the backward direction. Here, B is magnetic field.
1.46 .1 G.RB. PHYSICAL CHEMISTRY FOR COMPETITIONS

Answer the following questions: .' 3. Which of the following quantum numbers is not derived from
1. Which of the following statements is incorrect with reference SchrOdinger wave equation?
to the Zeeman effect? ., (a) Principal. (b) Azimuthal
(a) In a magnetic field, the energy of a particular atomic state (c) Magnetic (d) Spin
. depends on the values of'm' and 'n' . 4. In any subshell, the maximum number of electrons having
. (b) Zeeman effect is us~d to calculate the elm ratio for an same value of spin quantum number is:
electron
(c) Individual Spectral lines split into separate lines. The (a)~l(l+l) (b) 1+2 (c) 21 + I (d) 41 + 2
distance between them is independent ofthe magnitude of 5. The orbital angular momentum for a 2p-electron is:
.the magnetic field .
(d) Tile Zeeman effect involves splitting of a spectral line of
(a) J3 Ii (b) 16 Ii (c) zero (d) J2'!!'-'
2n
frequency Vo into tbree components
2. A d-subshelt· in an atom in the presence and absence of • Passage 8
magnetic field is: .
Dual nature ofmatter was proposed by de Broglie in 1923, it was
lea) five~fold degenerate, non-degenerate
experimentally verified by Davisson and Germer by diffraction
(b) seven-fuld degenerate, non-degenerate
experiment. Wave character of matter has significance only for
(c) five-fold degenerate, five.:fold degenerate microscopic particles. de Broglie wavelength or wavelength of
·'(d) non-degenerate, five-fold degenerate matter wave can be calculated using the following relation:
3~ Which among the·. following. is/are . correct about the
orientation of atomic- orbitals in space? A= h
(a) s-orbitals has single orientation mv
(b) d-subs4ell orbitals have three orientations along
where, 'm' and v' are the mtl$S and velocity of the particle.
x, yand z directions .
de Broglie hypoth~sis suggested that electron. waves were being
(c) f-subshell have seven orientations in their orbitals
diffracted by the target, much as X-rays are. diffracted by planes of
(d) None of the above·
atoms in the crystals.
t Zeeman effect explains splitting of spectral'lines in: Answer the following questions:
(a) magnetic field .(b) electric field
1. Planck's constant has same dimension as that of:
(c) both (a)' and (b) (d) none of these
(a) vvork (b) energy
i. Inpresence of magnetic field, d-suborbit is:
(c) power (d) angular momentum
(a) five-fold degenerate {b) three-fold degenerate
2. Wave nature of electrons is shown by:
(c) seven-fold degenerate (d) non-degenerate
(a) photoelectric effect (b) Compton effect
(c) diffraction experiment Cd) Stark effect
Passage 7 3. de 'Broglie equation is obtained by combination of which of
Spin angular . momentum of an electron has no analogue in the following theories?
classical mechanics. However, it turns out that the treatment ofspin (a) Planck's quantum theory
angular momentum is closely analogous to the treatment of orbital (b) Einstein's theory of mass-energy equivalence
angular momentum. (c) Theory of interference
Spin angular momentum ~ s(s + 1) Ii (d) Theory of diffraction'
4. Which among the following is not used to calculate the de
Orbital anE,ular momentum = ~'l(l + I) It
Broglie wavelength?
Total spin of an atom or ion is a multiple ofl.. .Spin mUltiplicity is
. . . 2
(a) A';'~ (b) A h
"v mV
a factor to confirm the electronic configuration of an atom of' ion. h . h
(c) A"" r;:;::;- (d) A"" - _ .
. Spin multiplicity =(n:s + 1) v2Em ~2qVm
-. . . $
5. The wavelength .of matter waves associated with a body of
Answer the following qurstiolls: mass 1000g moving with a velocity of 100 mlsec is:'
1. Total spinofMn 2+ (Z 25)ion will be: . (a) 6.62 x 1O-39 .cm (b) 6.62 x 10-36 ern .
. (c) 6..626 X 10- 36 m (d) 3.31 x 10-32 m
(aY ~ (b) 1 (c) 5
2 ·2 2 6. An electron microscope is used to probe the atomic
2. which of the following electrq'~ic configurations have fo~r arrangements to a resolution of 5 A. What should be the electric
spin multiplicity? potential to which the electrons need to .be accelerated ?
(VITEEE 2008)
(a) 2.5 V (b) 6 V
(c) 2.5 kV (d) 5 kV
I

. ATOMIC STRUCTURE 147

• Passage 9 . . • Passage10
Orbital is the region in an atom where th~ probability ()Jfinding:
the electron is maximum. It represents three-dimensional motion oj The hydrogen-like species Li2+ is in a spherically symmetric state. .
an electron around the nucleus. Orbitals do ,not specify a definite S I with one radial node. Upon absorbing light the ion undergoes·
path according to the uncertainty principle. An orbital is described transition to a state S 2' The state S 2 has one radial node an? its
withthe,help o/waveJunction 'If. Whfmever an electron is described energy is equal to the ground state energy oj the hydrogen atom.
by a wave jUnction., we say that an electron occupies that erbital. (liT 2010)
Since, many wave Junctions are possible Jor an electron, there are Answer the following question;:
many atomic orbitals in an atom. Orbitals have different shapes; 1. The state S I is :
except s-orbitals. all other orbitals have directional character. OOb ~~ ~~ ~~.
Number oj spherical nodes in an orbital is equal to (n 1- 1). [Hint: 2.s: iss~etrical having one radial node.]
Orqital angular momentum oj an electro'n is ~ I (l + I) 'Ii. 2. Energy of the. state S I in uilits of the hydrogen atom ground
Answer the following questions: state energy is: . '
1. Which of the following orbitals is not cylindrically (a) 0.75 (b) 1.50 (c) 2.25 Cd) 4.50 .
. 9 ~
symmetrical about z-axis? E .2+(2s) --x 13.6
(apd z2 (b)4pz (c) 6s (d) 3dyz ~~~ u =2~]
EH -13.6 "
2. The nodes present in 5p-orbital are:
(8,) one planar, five spherical (b) one planar, four spherical 3. The orbital angular momentUm quantum mimber of the state
(c) one planar, three spherical(d) four spherical S2 is:
3. When an atom is pla~ed in a magnetic field, the possible (a) 0 (b) I (c) 2 (d),3
number of orientations for ~n· orbital .of azimuthal quantum [Hint: Orbital angular momentuniquantum number .of 3p
number 3 is: subsheU, i.e., I == 1
(a) three (b) one (c) five (d) seven
4. Orbital angular momentum of J-electrons is:
(a).fi'li' (b)/31i (c)JJi'li (d)21i
5. Which of the following orbitals haslhave two nodal planes?
(a)dxy (b)d yz (c)dx2 _ y2 (d)Allofthese

[/I~
Passage 1. 1, (b) 2, (c) 3. (c) 4. (b) 5. (c)
Passage 2. 1. (b) 2. (b) 3. (b) 4. (b) 5. (c)
Passage 3. 1. (a, d) 2. (a, b) 3. (c) . 4. (c) 5. (a)
Passage 4. i. (b) .2. (a) "3. (c) 4. (a, c) S. (b, c, d)
Passage 5. 1. (b) 2. (a, b, c) 3: (c,.d) 4. (a) 5. (b)
Passage 6. 1. (b) 2. (d) 3. (a,c) 4. (a) S. (d)
Passage 7. 1. (c) 2. (a) 3. (d) 4. (c) 5. (d)
Passage 8. 1. (d) 2. (c) .3. (a, b)
• 4. (~) S. (c) 6. (b)
Passage 9. 1. (d) 2. (c) 3. (d) 4. (c) S. (d)
Passage 10. 1. (b) 2. (c) 3. (b)
148 I G.R.B. PHYSICAl- CHEMISTRY FOR COMPETITIONS

<@>.SELF ~SESSMENT <:>


ASSIGNMENT NO.2
SECTION-I 11. Which phenomenon best supports the theory that matter has a
. Straight OJJjective Type Questions wave nature? (VITEEE 2008)
This section contains II multiple choice questions. Each (a) Electron momentum (b) Electron diffraction'
.questionhas 4 choices (a), (b), (c) and (d), out of which only (c) Photon momentum (d) Photon diffraction
. one is correct.
. 1. Which. one of the following leads to third line of Balmer SECTION-II
spectrum from red end (For hydrogen atOm)? Multiple Answers Type Objective Questions
(a) 2 5 (b)5~2 (c)3----72 (d)4~1 12. Which of the following is/are correct?
2. The orbital angular momentum and angular momentum (classical (a) An electron in excited state cannot absorb a photon
. analogue) for the electron of 4s-orbital are respectively, equal to:
(b) Energy of electroris depends only on the principal
i;;:. h h 2h
(a) 'V 12 - and (b) zero and- quantum numbers
21t 21t 1t (c) Energy of electrons depends only on the principal
(c).J6h and 2h (d).J2 ~ and 3h quantum number for hydrogen atom
.1t 21t 21t (d) Difference in poteritial energy of two shens is equal to the-
3. A sample of hydrogen atom is excited to n 4 state. In the difference in kinetic energy of these shells
spectrum of emitted radiation, the number of lines in the 13. Which of the following statements is/are correct?
ultraviolet and visible regions are respectively:
(a) Energy of 4s, 4p, 4d and 4J are same for hydrogen
(a)3:2 (b)2:3 (c)I:3 (d) 3: I
(b) Angular momentum of electron fro
.. 4. Number of de Broglie waves made by.a Bohr electron in an
(c) For all values of 'n', the p-orbitals have the same shape
orbit of maximum magnetic quantum number + 2 is:
(d10rbital angular momentum = nh/21t
(a) I (b) 2 (c) 3 (d) 4
14. Which of the following orbitals are associated with angular
. 5. First line of Lyman series of hydrogen atom occurs at A x A.
nodes?
.The corresponding line of He + will occur at:
(a) f (b)d (c) p (d)s
(a)4x (b)3x (c)x/3 (d)x/4
15. The correct statement(s) among the following is/are:
6. Electronic transition in He + ion takes from n2 to nl shell such
(a) All d-orbitals except d _2 have two angular nodes
that;
2n2 + 3n] 18 • . ...0) (b) d x 2 - y-., , d z ,:tie
...
on the ~xes
2n2 - 3n] = 6 .... (ii) (c) The degeneracy of p -orbitals remains unaffected in the.
then what will be the total number of photons emitted when· presence of external magnetic field
electrons transit to n] shell? .(d) d-orbitals have 3-fold degeneracy
(a) 2 I (b) IS (c) 20 (d) 10
7 .. Which of the following sets of quantum numbers is not SECTION-III.
. possible for an electron? [PET (Raj.) 2008] Assertion-Reason Type Questions .Fi
(a)n :;: I, I = 0, ml == 0, ms = 1/2 This section contains 5 questions. Each question contains
. . (b) n .= 2, I :;: I, mJ 0, ms = - 1/2 Statement-l (Assertion) and Statement-2 (Reason). Each
'i (c) n == 1, l I, mz 0, ms = + 1/2 question has following 4 choices (a), (b), (c) and (d), out of
(d)n =2,1=I,mJ :;:O,m, +112 which only one is correct
8.' Th~ average life of an excited state of hydrogen atom is of the ,ja) Statement-I is true; statement-2 is true; statement-2 is a
order of .10-8 sec. The number of revolutions made by an' correct explanation for statement-I.
electron when it returns from n :;: 2 to n :;: 1is: (b) Statement-l is true; statement-2 is true; statement-2 is not
,(a)2.28x 106 (b) 22.8x 106 (c) 8.23 x 106 (d)2.82x 106, a correct explanation for statement-I.
9. ~e wave number of a particular spectral line in the atomic (c) Statement-l is true; statement-2 is false.
. spectrum of a hydrogen like species increases 9/4 times when (d) Statement-! is false; statement-2 is true .
deuterium nucleus is introduced into its nucleus, then which of 16. Sbltement-l: Kinetic energy of photoelectrons increases with
the following will be the initial hydrogen like species? increase in the frequency of incident radiation,
Because
(a) Li 2+ (b) Li+ (c) He+ (d) Be3+
Statement-2: The number of photoelectrons ejected increases
10. Energy of electron in the first Bohr orbit of H-atom is - 313.6 with increase in intensity of incident radiation.
kcal mol-I; then the energy in seconq Bohr orbit will be: 17. Statement-I: Photoelectric effect is easily pronounced by
(a) + 313.6 kcal morl. (b) 78.4 kcal mol-I caesium metal.
(c) - 34.84 kcal mol-I (d) -!2.5 kcal mol- 1 Because
Statement-2: Photoelectric effect is easily pronounced by the
metals having high ionization energy.
I
ATOMIC STRUCTURE :149

18. Statement~I:Electrons in K-shell revolve in circular orbit. nh


( c) Wavelength of matter wave (r)
. Because 2n
Statement-2: Principal quantum number 'n' is equal to I for (d) Quantised value(s) (~) hlp
the electrons in K-shelL 23. Match the Column-I with Column-II:
19. Statement-I: Orbit and orbital are synonymous. Column-I Column-II
Because
(a) Elctrons cannot exist (p) cde Broglie wave
Statement~2: Orbit is the path around the nucleus in which
in the nucleus
electron revolves. .
(b) ".Microscopic particles (q) Electromagnetic
20. Statement-I:· C 6 = 1s2 , 181 , 2p 3 .is the electronic
in motion are wave
configuration in first excited state.
associated with
Because
Statement-2: Maximum energy by an electron is possessed in (c) No medium is required (r) Uncertainty principle
its ground state. for propagation
(d) Concept of orbit was (s) Transverse wave
SECTION-IV replaced by orbital
Matrix-MatchingJype Questions
This section contains 3 questions. Eac~ question contains SECTION-V
statements given in two columns which have to be matched. Linked Comprehension Type Questions
Statements (a, b, c and d) in Column-I have to be matched with A chemist was performing an experiment to study the effect of~
statements (p, q, rand s) in Column~II. The answers to these varying voltage on the velocity and de Broglie wavelength of the
questions have to be appropriately bubbled as illustrated iil the electrons. In first experiment, the electron was accelerated
following examples: through a potential difference of I kV and in second experiment,
If the correct matches are (a-p,s); (b-q,r); (c-p,q) and (d-s); it was accelerated through a potential difference 0(2 kV.
then the correctly bubbled 4 x 4 matrix should be as follows:' The wavelength of de Broglie waves associated with electron
p q r s is given by:

aft®0 A=_h_
J2qVm

b®_00 where, V is the voltage through which an electron is


accelerated.

eft_CD0 Putting the values of h , m and q we get:


A = 12.3 A
.jV .
d®®00 .....
Answer the following questions:
21. Match the Column-I with Column-II: 24. The wavelength of electron will be:
Column-I Column-II (a) 1.4 times in first case than in second case
(a) 48 . (p) Circular orbit around nucleus (b) 1.4 times in second case than in first case
(q) Non-direction orbitals (c) doubJe in second case than in first case
(b) 4p
. . 2h (d) double in first case than in second case
(c) Is (r) Angular momentum = 25. In order to get half velocity of eJectrons in second case, the
n
applied potential will be:
(d) . 3d (s) Number of radial node = 0
(a) 0.25 kV (b) 2 kV (c) 0.5 kV (d) 0.75 kV
22. Match the properties of Column-I with the formulae in
26. .The velocity of electron will be:
Column-I!:
(a) same in both cases
Column-I Column-II
(b) 1.4 times in second experiment than in first ewetiment
(a) Angular momentum of electron . (p) Jl(l + 1) 2: (c) double in second experiment than in first experiment
'(d) four times in the second case than in first case
(b) Orbital angular momentum (q) /(0

1. (b) 2. (b) 3. (a) 4. (c) 5. (d) 6. (d) 7. (c) 8. (c)


9. (d) 10. (b) 11. (b) 12. (a, c,d) 13. (a, b, c) 14. (a, b, c) "15. (a, b, c) . 16. (b)
17. (c) 18. (b) 19. (d) 20. (c) 21: (a-p, q,r) (b-r) (c-l?,q) (d-s)
22.' (a-q,r) (b-p) (c-s) (d~q,r) 23. (a-r)
.,. (b-p) (c-q,s)
. (d-r) 24.. Ca) 25. (a) 26. (b)
RADIOACTIVITY AND
LEAR TRANSFORMATION

3.1 .RADIOACTIVITY This example clearly shows that the phenomenon of


radioactivity does not depend on the orbital electrons but depends
Radioactivity is a process in. which n.uclei of certain. elements only on the composition of nucleus.
un.dergo spontaneous disin.tegration without excitation by
In the universe, there are only 81 stable elements having one
an.yextern.al means. or more non-radioactive isotopes. No stable isotope exists for the
All heavy elements from bismuth through uranium and a few elements above .2~~ Bi. Thus, bismuth is the heaviest stable
of lighter elements have naturally occurring isotopes which nuclide. Two earlier elem~nts technetium and promethium exist
.possess the property of radioactivity. These isotopes have only as radioactive isotopes (see table at the bottom) .
. unstable nuclei and attain stability through the phenomenon of
radioactivity.· The activity results in the emission of a complex 3.2 CHARACTERISTICS OF RADIOACTIVE
type of powerful radiations. known as alpha, beta and gamma RADIATIONS
rays. All those substances which have the tenden.cy to emit
these radiations are termed radioactive materials. The The following are the main characteristics of radiations emitted
property of disintegration of a radioactive material is independent by radioactive materials: "
of temperature,· pressure and other external conditions. (i) Photographic effect: Radiations affect the
R~oactivity is a nuclear phenomen.on, i. e., the kind' of photographic plate in a similar manner to that oflight. The effect
intensity of the radiation emitted' by any radioactive substance is is even observed in dark. The portions of the photographic plate
abSolutely the same whether the element·is present as such or in where radiations fall, become blackened after treatment with a
ailyone of its compounds. 2~~Ra isotope is radioactive. When developer.
this isotope is dissolved in. sulphuric acid, it is converted into This property is used for the detecthn of radioactivity.
radiwn sulphate (RaSO<t). The property of radioactivity in radium (ii) Scintillations: When radiations fall on the zjnc
sUIphate·anci·free radiUm isotope is.thesame,no doubt that the sulphide (ZnS) screen, flashes of light are produced. This is
radin.rn ion. in radium sulphate bas different number of electrons known as scintillations. The number of particles emitted iit unit
than free neutral radium isotope. time can be counted by noting the scintillations produced in the
Radium atom (Ra). Radium ion (Ra 2+) apparatus having a zinc sulphide screen.
No. of protons 88 88. The apparatus is called spinthariscope.
No. of electrons 88 86 (iii) Emission of heat: Radioactive materials continuously
Atomic mass 226 226 emit energy in the form of kinetic energy. Heat energy is
H ) _. He
Li Be I- B C '. N
- ! 0 F Ne I
Na Mg -_. i Al Si P S Cl Ar I
K Ca Sc Ti i
.V Cr IMn Fe Co Ni I Cu Zn Ga Ge As Se Br Kr I
Rb Sr Y I Zr Nb Mo I Tc' Ru Rh Pd I Ag Cd In I
Sn Sb Te I ,.X@
Cs Ba I Lu Hf Ta I W I Re Os Ir Pt I Au Hg Tl Pb Bi Po At Rn
,
Ft. Ra Lr· Rf Db Sg .1 Bh lIs Mt Uun I Uuu Uub Uuq. Uuh
La Ce Pr Nd I Pm Sm Eu Gd I 1b Dy Ho Er Tm Yb
Ac Th Pa U INp' Pu Am Cm J. Bt Cf Es. Fm . 'Md ·No
Shaded elements are radioactive
I
RADIOACTIVITY AND NUCLEAR TRANSFORMATlbN 151

produced when the radiation particles collide with matter. elements. At present over forty such materials are known' to exist
Radium produces 134.7 cals of heat per gram per hour. in nature.
(iv) Physiological effect: Radioactive radiations have
serious physiological effects, which may be cumulative over a
period of time. Even a short exposure to intense source of
radiations is sufficient to cause painful inflammation. Gamma
rays are most effective. In general, abnormal cells are more
affected than the healthy cells. On account of this property Electroscope
radiations are used to destroy cancerous tumours. box
(v) Ionisation of gases: This is the most important effect
observed .in the case of radioactive radiations. Radiations
produce ionisation in the gases through which they are passed.
This effect is used for quantitative measurement of p+++++
radioactivity. The radiations cause a number of molecules of the Air

~ 1> r---:--+--
~
Ionisation
gas to lose electrons"'and pass into positive ions. The electrons chamber
immediately become attached to the neutral molecules, thus 1>
making them negative ions. The total ions of one sign are equal Radioactive
material'
to the total ions of the other type. The rate of production of
these ions is proportional to the intensity of radiation. The
extent, to which a definite .quantity of a gas is rendered a
conductor by a radioactive substance, is a measure of. the
radioactive power of a radioactive substance. The apparatus used
for this purpose is called electroscope (Fig. 3.1).
Fig. 3.1 Electroscope for measurement of radioactivity
Geiger-Muller counter is based on this effect. The ionisation
chamber consists of 90% argon and 10% ethyl alcohol vapour at 3.4 ANALYSIS OF RADIOACTIVE
10 mm pressure. Due to ionisation, a flow of current occurs, RADIATIONS
which is measured after amplification.
In 1904, Rutherford and his co-workers observed that when
radioactive radiations were subjected to a magnetic field or a
3.3 HISTORY OF THE DISCOVERY OF
strong electric field, these were split into three types, as shown in
RADIOACTIVITY Fig. 3.2. The rays which are attracted towards the negative plate
In 1895, Henri Becquerelwasstudying the effect of sunlight on are positively charged and are called alpha (a.) rays. The rays
various phosphorescent minerals, among them a uranium ore. which are deflected towards the positive plate are negatively
During a period of several cloudy days, he left the uranium charged and are called beta (P) rays. The third type of rays which
sample in a drawer along with some photographic paper wrapped are not deflected on any side but move straight are known as
in black paper. Much to his surprise, he discovered that the gamma (y) rays. The important properties of these .radiations are
photographic paper had been fogged by exposure to some tabulated on next page:
invisible radiation .1fom uranium. He called this mysterious
property of the ore 'radioactivity'. Radioactivity means 'Y·rays y-rays
ray-emitting activity. He further observed that the radioactive +
mineral emitted these mysterious radiations day after day and + a.-rays
month after month and the emission seemed to be endless. The +
emission was completely unaffected by physical and chemical +
conditions. A year later, in 1896, Marie Curie found that besides + !3-rays
uranium and its compounds, thorium was another element which +
possessed tbe property of radioactiviry. In 1898, Marie Curie. +
and her husband P. Curie observed that the. uranium ore +
'pitchblende', contained more activity than was expected from
the uranium which it contained. It must be obviously due to the . _____ Radioactive --__
presence of some other radioactive elements which were far more -- substances, ---
radioactive than uranium. Finally, they isolated two new
radioactive elements polonium andradium.
Almost in the same period, G. C. Schmidt reported that
thorium compounds possessed radioactivity. In 1901, A~
Fig. 3.2 . (a) Deflection of radioactive rays in electric field
Debieme and F.S. Giesel discovered another new radioactive
and (b) Emission of radioactive rays and their
element actinium in uranium minerals. Further systematic deflection in a magnetic field.
researches led to the. discovery of many more radioactive
··152. I G.R.B. PHYSICAL CHEMISTRY FOR COMPETITIONS
"

. ~rays :v-rays ~rays


...···Prop~rty a-rays Property .a-rays y-rays

1. Nature These consist of These consist of y-rays are simi- 5. Photographic a-particles affect !3-rays effect on y"rays have little
small. positively negatively char- lar to X-rays. effect . the photographic a photographic effect on photo-
charged particles ged particles These are neu- plate. plate is greater graphic plate.
which are merely which have the tral in nature .. than a-particles.
nuclei of helium s.ame elm value They have very Like cathode
atoms, each con- as the cathode small wave- rays, !3-rays pro-
sisting of 2 pro- rays. ~rays are lengths of the duce X-rays.
tons . and • 2 merely elect- order of 10- 10 to
6. Effect on a-particles p..particles have y-rays have very
neutrons. These rons. The ~rays 1O- l3 m..
are represented as are represented zinc sulphide produce. little effect on little effect on
screen luminosity on ZnS ZnS screen due ZnS screen.
iHe aL?flor _?e. screen due to high to low kinetic
2. Velocity The a-rays are The ~rays are They travel with kjnetic' energy. energy.
ejected with high much faster than the velocity of Note:
velocities ranging a-rays.' They light
0) The quantum energy ofy-rays emitted by a radioactive substance
from 14 x 109 to have generally can have unique and discrete values. '.
17 x I 09 crn!sec. different veloci- (ii) The energy of f;X.-particles emitted by a radioactive substance
. . ties sometimes
The v. eloclty of bin th (a-emitter) has unique value.
·· d approac. g e
a-rays d~pen s velocity of light (iii) The energy of p..particles emitted by a radioactive substance,
upon the .kmd of $-emitter) can have any value between zero and end point
nucleus from energy.
which they are.
. emitted. 3.5 CAUSE OF RADIOACTIVITY
3. Penetrating' a-particles have p..rays are more Due to high ve- Except in the case of ordinary hydrogen, all other nuclei contain
power small penetrating penetrating locity and both neutrons and protons. A look at the stable nuclei shows that
power i'lue to than a-particles. non-material the ratio nl p (neutrons/protons) in them i$ either equal to I or
relatively larger This is due to character, y-rays more than 1. The ratio is "" lin all the light-stable nuclei up to
size. They are. small size- and are 10 10 times'
stopped by a high'" velocity. more.' penetrat-
calcium (ig
Ca) and thereafter the ratio is greater than 1 and
piece of alu- These are stop- ing than a-rays.
increases up to 1.6 for heavy stable nuclei as shown in the
minium. foil of ped by a.1 cm following table: .
6.1 rom thick- thick sheet of Neutron-proton ratio in some stable nuclei
npss. aluminium.
. . l1C 14N 16 20 40 64
':::Zr l~Sn l::Nd ,~Hg
Isotope 6' 7 sO loNe 20 Ca 30Zn
. Cardboard Aluminium Lead
n 6 1 8 10 20 . 34 50 70 90 122

a---f-- p 6 7 8 10 20 30 40 50 60 80
. I 1.13 1.25 1.40 1.50 1.53
nip

y The variation of n versus p for some nuclei is shown in


Fig. 3.3.
The stable nuclei lie within the shaded area which is called the
region or zone of stability. All the nuclei falling outside this
Fig; 3.2 (c) Comparison of penetrating power zone are invariably radioactive and unstable in nature. Nuclei
of a, fl and y-rays that faU above the stability zone have an excess of neutrons
4. ;Ionising a-particles pro- Due to low y-rays produce wbile those lying below have more protons. Both of these cause
power duce intense ion- value of kinetic minimum ion- instability. These nuclei attain stability by making adjustment in
isation in gases. energy ionising isatton or no the nIp ratio.
Ionising power is power is less ionisation. Two cases'thus arise:
100 times greater than a-particles (i) nl p ratio is higher than required for stability. Such
than fl-rays and but 100 times nuclei have a tendency to emit t}-rays, i. e. , transfonning a neutron
10,000 times greater than
into proton.
greater than. . y-rays.
y-rays. This IS ~n ~ 1p + _~e (f3-particle)
due to high
kinetic energy. Thus, in f}--emission nip ratio decreases. For example, in the
change 00f I~C to liN, nl p ratio decreases from 1.33 to 1.
RADIOACTIVITY AND NUCLEAR TRANSFORMATION I 153
'!C--t !~N+ _~e 7 · 16
12 32
e.g., 3 LI,
6C' 8 0 '16S' etc.
Similarly, in the following examples, the nIp ratio decreases (ii) For the elements (mass number A;::;: 40), there is·
during /3-emission: :> preference for the number of neutrons· to be greater than the
I;
number of protons (n > Z \ e.g., B is stable but '~C is not.
There are two stable elements : H and ~ He, in which numbci- of
nip ratio 17/15 Hi/16
neutrons i~ less than that of protons.
~~Kr --t ~~Rb +_?e (iii) Beyond Bi (Z =83~ all isotopes are unstable and
nip ratio 51136 5IJI37 radioactive. Th~i1 elements do not have a strong nuclear
"superglue" toifold nucleons together. .
Unstable Illustration :
140 Unstable region
Stable
,..
Nuclide ~Ratio
nuclei p Nature of Emission
120

i
(J)
c
100
I, . 35 8
16.
19
16
= 1.2
l3-emission
358--t
16 . 17 35Cl+ °e
-I
e
s
Cl>
~O
I~p ~(~ <1) Positron emission
9 - - t .8 0 + +1°e
z 9 p. 17 p 17

15 60
Co Lies below stability belt, it has a heavy .
..c
E n 58 nucleus and it decays by K-electron
ISfAg =- 1.23 capture.
-2. 40 p 47
12iAg+ _?e--t I~Pd + hv
20
It is a neutron rich species. It undergoes
n 146
2~~U -=: =1~9 decay by a-emission.
p 92
o 20 40 60 80 100 iJrh + iHe
21~ U - - t . 239
.
Number of Protons ----'--"'"
.

Some other examples : ~ Cu (positron emission), I~~ Xe (0;


Fig. 3.3 VatjaUon of number of neutrons with number of
protcjns in:stable non-radioactive.nuclei '.
or /3-decay), 2:~N p (a-decay), n p (/3-decay), l;~ I (K-electron
capture).
(ii) nIp ratio is lower than requ~red
for stability. Such
nuclei can increase nIp ratio by adopting anyone of the 3.6 THEORY OF RADIOACTIVE
following three ways:
DISINTEGRATION
(a) By emISsIOn of an alpha particle (Natural
radioactivity): Rutherford and Soddy, in 1903, postulated that radioactivity is a
. 2~~ U --t 2~ciTh + iHe (a-particle) nuclear phenomenon and all the radioactive changes are taking
.. place in thf nucleus of the atom. They presented an interpretation ..
nip ratio 146/92 = 1..58 144/90 1.60 "rbf the radioactive processes and the origin of radiations in the
(b) By emission of a positron (Artificial emission): . fonn of a theory known as theory of radioactive disintegration.
O· the main points of the theory are:
13
7 N --t I~C + +l e (i) The atomic nuclei of the radioactive elements are unstable
nip ratio '6/7 7/6 and liable to disintegrate any moment.
(c) By K-electron capture: (ii) The disintegration is spontaneous, i. e. , constantly
194 Au + -J0 e - - t 194 breaking. The rate of breaking is not affected by external factors
79 78 Pt like temperature, pressure, chemical combination, etc.
nip ratio 115179 116/78 (iii) During disintegration, atoms of new elements called
Alpha emission is usually observed in natural radioactive daughter elements having different physical and chemical
isotopes while emission of positron or K-electron capture is properties than the parent element come into existence.
observed in artificial radioactive isotopes. The unstable nuclei (iv) During disintegration, either alpha or beta particles are
continue to emit alpha or beta particles until a stable nucleus emjtted from the nucleus.
comes into existence. Tht: disintegration process may proceed in one of the
Conclusion: (i) For the elements (mass number A::;; 40), following two ways:
nature prefers the number of protons and neutrons in the nucleus (a) a-particle emission: When an a-particle [iHe] is
to be sameor perhaps one more neutron than protons. emitted from the nucleus of an atom of the parent element, the
154 I' G.R.B. PHYSICAL CHEMISTRY FOR COMPETITIONS

nucleus of the new element, called daughter element, possesses Isoters No. of atoms'" same, CO 2 , N2 0
atomic mass' or atomic mass number less by four units and No. of electrons same,
nllclearcharg~ of.atomic number less by 2 units because physical properties same.
(£·particle has mass of 4 units and nuclear charge of two units. Isodiapheres I ~otopicexcessmass(N-P) 92 U235 , 90Th231
The daughter element' after a-emission is called 'an .- same.
isodiaphere of parent ~Ienient.
-IX .
214Pb
82 ~
214Bi
83 + °e
-I
(Lead) (Bismuth)
Parent element ~ Daugh~er element
234 T h 234
Atomic mass W W- 4 90 ~ 91 Pa + -I°e
Atomic number Z Z- 2 (Thorium) (Protoactinium)

For example, in'the followiJ:lg transformations, each a-particle, 2~j Bi ~


213 Po
84 + -1°e.
. emission is accompanied by decrease of atomic mass by 4 and of (Bismuth) (Polonium)
atomic number by 2. .
SpeCial clIse: If in a radioactive transformation, 1 alpha and
2~~Ra ~ 2ilRn
+ iHe 2 beta particles are emitted, the resulting nucleus possesses the
(Radium) (Radon).
same atomic number but atomic mass is less by 4 units. A
2J~U ~ 2JciTh + iHe radioactive transformation of this type always produces an
(Uranium) ,(ThQ!ium) isotope of the parent element.
2~jBi ~ 2~TI + iHe w -IX W-4 -13 W~4C -Jl W-4
(Bismuth) (Thallium) Z A ~ Z-2 B ~ Z-l ~ ZD
.~~ - 2Jl Po ~ 2ifpb + iHe
A and D are isotopes.
(Polonium) (Lead)
(v) Gamma rays are emitted due to secondary effects; After
(b) p-particle emission: jj-particle is merely an electron the emission of an alpha particle or a beta particle, the nucleus is
which has negligible mass. Whenever a beta particle is emitted left behind in excited state due to recoil. The excess of energy is
from the nucleus of a radioactive atom, the nucleus of the new released in the torm of gamma rays. Thus, y-rays arise from
element formed possesses the same atomic mass but nuclear energy rearrangements in the nucleus. As y-rays are short
charge or atomic number is increased by 1 unit over the parent waveltmgth electromagnetic radiations with no charge and no
element. Beta particle emission is due to the result of decay of mass, their emission from a radioactive element does not
neutron into proton and electron. produce a new element.
. I I 0
on ~ IP+ _Ie On passing through an absorbing material, the intensity of
The electron produced escapes as a beta particle leaving y-radiation decreases exponentially with the thickness traversed
and is given by :
proton in the nucleus.
-Jl I Ioe-/iX
Parent element ~ Daughter element where j.l = Absorption coefficient,
Atomic mass W W x = Thickness
Atomic number Z Z+I 10 = Initial intensity
For example, in the following transformations, beta particle
I == Transmitted intensity
emission results in increase of atomic number by one without any .
All radioactive nuclei have the same probability of
change in atomic mass, i. e. , daughter element is an' isobar of
disintegration. However, a radioactive nucleus may undergo
parent element. (See table 3.1) dec1.\y' next moment while some other may have to wait for
Table 3.1 Isotopes, Isobars, Isotones, Isomers, lsoters billiqns of years to decay one cannot predict, when a particular
and'lsodiapberes atom will decay.
(vi) Internal conversion: An excited nucleus, in some
Chancterittks' " cases, may return to its ground state by giving up its excitation
= =
Z at. ao..,A mau ao.., Eumple.s energy to one of the orbital electrons around it. The emitted
N.= neatrous" P = prGtoDl electron' has a kinetic energy equal to the lost nuclear excitation
Isotopes Z == same, A == different :H, iH, ~H, 23~2U, 239~U . energy minus the binding energy ofthe electron in the atom.
Kinetic energy of the ejected electron
Isobars - Z different, A == same 228; Ra, 2~~c, 22~th
= Available excitation energy
Isotones N == same, nucleons == . i~Ar, ~K
different, Z different ~ Binding energy of the ejected electron
Isomers N same, P =same, This process is called internal conversion and emitted electron
Z == same, A == same is called conversion electron.
Nuclear energy levels ==
(vii) Brems strahlung (German word meaning 'Breaking
different
Radiation'): Continuous- y-radiations emitted when jj-particles
are slowed down by interaction with atomic nucleus.
RADIOACTIVITY AND NUCLEAR' TRANSFORMATION 155
Note: Counting of the number of a and I}-particles in a radioactive The above examples follow group. displacement law rigidly in .
transformation: accordance with· the statement. Howev~r, there are a nUlnber of
Parent element -----l> Daughter element examples where confusion arises regarding the position of the
element in the periodic table if the above statement is followed
rigidly.
N b f ' 1
urn er 0 a-partIc es = Change in mass
4
number
' ti Mg is /3-radioactive, It belongs to· group 2 (IlA) of the
periodic table. On losing a beta particle, it is transfonIlt;d to
=M 1 M2 aluminium mAl) which belongs to group 13 (IlIA), i. e., II
4 places right to the parent element.
Let 'x' aand 'y' I}-partic\es be emitted,
ti Mg ---7 tjAl + _~ e
Atomic number of parent element 2x + y
= Atomic number of daughter element ,2~t Th is a member of actinide series. All the fourteen
members of the actinide series have been placed along with
actinium in the III B group, i. e., group 3. It emits a beta particle
3.7 .GROUP DISPLACEMENT LAW and is transformed to protactinium (2~i Pal which also belongs 'to
This law was presented by Fajan, Soddy and Russel in 1913 to actinide series, i. e. , group 3 of the periodic table, .
explain the changes which occur when an alpha particle or a beta 2;riTh ~ 2~Pa + _~e
particle is emitted from a radioactive element. According to this
law, ''when an a-particle is emitted, the daughter element has Hence, group displacement law should be applied with great
atomic number 2 units less than that of the parent element. It care especially in the case of elements of lanthanide series (57 to
is consequently displaced two places (groups) to the left in the 71), actinide series (89 to 103), VIII group (26 to 28; 44 to 46; 76
periodic table. When a ~particle is emitted, the daughter to 78), IA and IIA groups. It is always beneficial to keep in mind
element has an atomic number 1 nnit higher than that of the the setup and skeleton of the extended form ofperiodic table.
parent element. It is consequently displaced one place (group) .
to the right iii tbe periodic table." /.; .

Examples
(i) Polonium eJ1 Po) belongs to group 16 (VIA) of the
periodic table. On losing an alpha particle, it is transformed into
lead (2~g Pb) which belongs to group 14 (IVA), i. e., two places to ::::::I_SOME SOLVED EXAMPLES\.I:::: : :
the left ofthe parent element, polonium.
Example 1. Calculate the number of neutrons in the
2i1Po ---7 2~gPb
remaining atom after emission of an alpha particle from 2t~ U
16 14
(VIA) (IV A) atom.
Solution: On account of emission of an alpha particle, the
(ii) Bismuth eJi
Bi) belongs to group 15 (VA) of the periodic
atomic mass is decreased by 4 units and atomic number by 2
table. It emits all alpha particle resulting in the formation of units.
thallium which belongs to group 13 (IlIA), i. e. , two places to the So, . Atomic mass of daughter element 234
left of the parent element, bismuth.
Atomic number ,of daughter element 90
213B'! 209TI
81
83
Number of neutrons atomic mass ~ atomic number
15 13
(VA) (III A) = 234 90?= 144

(iii) Carbon e:C) belongs to group 14 (IV A) and emits a Example 2. Radioactive diSintegration of 2i~Ra takes

i3-particle forming nitrogen ejN) which belongs to group 15 place in the following manner into RaC,
-a -a -a -~
(V A), i, e" one place to the right of the parent element. Ra ---7 Rn ---7 RaA ---7 RaB ---7 RaC
I:C ---7 liN + _~e Determine mass number and atomic number of Rae.
'(IV A) (V A)
Solution: Parent element is 2~~Ra.
(iv) Phosphorus (?~ P) belongs to group 15 (VA) and emits a .Atomic mass = 226
i3-particle forming sulphur (?i
S) which belongs to group 16 Atomic number
._/
= 88
(VIA), i. e., one place right to the parent element. RaC is formed after the emission of 3 alpha particles: Mass of
f~ P ---7 iJ S + _~ e 3 alpha particles = 3 x 4 := 12'
(V A) (VIA) So, Atomic mass ofRaC (226 - 12) = 214
156 G.R.B. PHYSICAL CHEMISTRY FOR COMPETITIONS

With emission of one <x-particle, atomic number is decreased So, 4x=20


by 2 and with the emission of one I)-particle, atomic number is 20
increased by 1. or x=- 5
4
So, Atomic number ofRaC == 88- (3 x 2) + 1== 83 Alternative method: This can also be determined by the
Example 3. A radioactive element A disinte.grates in the application of the following equation:
following manner, No. of[3-particles emitted = 2 x No. of a-particles emitted
a -~ -~
'A ~B----?C----?D - ( Z Parent Z End product )
2 2xx-(90- 82)
which ones of the elements A, B, Cand D are isotopes and
which ones are isobars? or x 5
Solution: Let the mass number and atomic number of Example 7. How many moles of helium are produced
element A be M and Z, respectively. The followins changes shall when one mole of 2~~ U disintegrates into 2~~Pb?
occur during disintegration:
-a -p -~
Solution: Radioactive change is
fA ----? ~=iB~ ~=ic~ MZ4D 238 U ----? 206 Pb
92 82
A and D are isotopes as both have same value ofZ. Decrease in mass =(238 - 206) = 32
B, C and D are isobars as these have same values of atomic Let the number of a-particles emitted be x.
mass.
4x 32
Example 4. 2~t Th disintegrates to give 2~~Pb as the final
x=8
product. How many alpha and beta particles are emitted in this,
Thus, 8 moles of helium are produced when one mole of
process?
2~~ U disintegrates into 2~~Pb.
Solution: 2~6 Th -:-----t 2~~ Pb
Parent Endpr09uct Example 8. How' many 'a' and '[3'-particles will be
Decrease in mass = (234 - 206) 28 emitted when 90Th234 changes into 84P0218?
Mass of a-particle 4 Solution: The change is;
90 Th 234 ----? Po 218
So, Number of a-particles emitted = 28 = 7 Parent 84 Endproouct
4
No. ofl)-particlesemitted =2 x No. of a-particles-{At. N.o. Decrease in mass =(234- 218) =16 amu
of parent - At. No. of end product) Mass of 1a-particle =4 amu
= 2'x 7- (90- 82)= 6
Therefore, number of a-particles emitted = 16 = 4,
Example 5. The atomic mass of thorium is 232 and its 4
atomic number is 90. During the course of its radioactive Number ofl)-particles emitted
disintegration 6a and 41)-particles are emitted. What is the
2 x No. of a-particles emitted (At. No. of parent - At.
atomic mass and atomic number of the final atom?
No. of end product)
Solution: Decrease in mass due to emission of 6o:-particles
6x 4= 24. ;::: 2 x 4 - (90 84) = (8 - 6) = 2
So, Atomic mass of the product atom = (232 - 24) = 208 Hence, number of a-particles emitted = 4
and number ofl)-particles emitted = 2
No. ofl)-particles emitted = 2 x No. ofa-particles
Example 9. 2~~U isa natural an a-emitter. After
. ~
(ZThnrium ZFinal atom)
a-emission, the residual nucleus.u XI in turns emits a I)-particle
4 =2x 6- (90- ZFinalatorn)
to prod.uce another nucleus U X 2' Find out the atomic number and
or ", ZFinal,atom = 82 , mass number ofU X I and U x 2' Also if uranium belongs to IIIrd
Example 6, An atom has atomic mass.. 232 and atomic group to which groupU XI andU Xl belong.
number 90. During the course of disintegration, it emits Solution'• 238U
92
4He----? 234
- 2 90 UXI
21)-particles and few a-particles. The resultant atom has atomic
234 (0), 234U
mass 212 and 'atomic number 82 . How many a-particles are ,90 U Xl - _Ie ----? 91 X2
emitted during this process?
Both U X land U X 2 will belong to IIIrd group because both lie
Solution: 1]le decrease u.;.~tomic mass
in actinideseries. '
. = (232- 212)= 20
'" occurs due to emission of a-particles. Let x
Decrease in· mass
be the number of alpha particles emitted. 1. During the tr!lns{ormation of ~ X to ~ Y, thenu.mber of
Mass of' x' a-particles = 4x I)-particles emitted are: [PET (KeraJa) 2006, 081
RADIOACTIVITY AND NUCLEAR TRANSFORMATION 157

(a)~ a-b Z - 2 x 2 + Number of ~-particles =:: Z - 3


(b) d + - - +c
4 2 . Number of ~- particles 1]

(c)
.
d+(~1
2
c (d) 2c - d +a b
j
3.8 RADIOACTIVE DISINTEGRATION SERIES
[Arts. (c)]
Elements beyond bismuth are all radioactive in nature. Most of
[Hint: No. of a-particles =:: ( a 4 b) them have several radioactive isotopes. These radioactive
elements . disintegrate to give new elements which again
Zj - 2a+ ~ Z2 . disintegrate to form other elements and so on. The process
~= -Z[ + 2a continues till a non-radioactive end product is reached.
(a - b)
=d c+ 2 - - - "The whole chain of such elements starting from the parent
4 element (radioactive) to the end element (non-radioactive) is
(a - b) called a radioactive series or a family".
=d+----c]
2 All the naturally occurring radioactive elements above atomic
2. A radioactive nuclide emits y-rays due to: number 82 belong to one of the three radioactive series. These are
(a) K -electron capture known as: .
(b) nuclear transition from higher to lower energy state (i) Thorium series . (ii) Uranium series
(c) presence of greater number of neutrons than protons (iii) Actinium series
(d) presence of greater number of protons thari neutrons Uranium and thorium series have been named on the basis of
[Ans. (b)] long lived isotopes of 238 U and 232 Th. The parent element of
[Hint: After a, J3-emission, nucleus goes to excited state; actinium series is 235 U but originally it was thought to be an
when it returns to normal state, emission of y-radiations takes isotope of actinium, 227 Ac. The three series are also referred to 4n
. place.] (thorium), 4n + 2 (uranium) and 4n + 3 (actinium) series as when
3. In which of the following transformations, the ~-particles are the mass numbers of various members belonging to these series
emitted? when divioed by four, either there is no remainder (as in thorium
(a) Proton to neutron (b) Neutron to proton \series) or the remainder is 2 (as in uranium series) or 3 (as in
" actinium series). The end product in all the three series is an
(c) Proton to proton (d) Neutron to n~utron
,.isotope of lead which is stable and non-radioactive in nature. The
[Ans. (b)] •
[Hint: ~n ---7 :H
+ _~e + antineutrino] following table shows the main characteristics of three
fadioactiveseries: .
4. In the radioactive decay:
AX
Z
AY
- - 7 Z+I
A- 4
- - 7 Z-l
A - 4 *
z - - 7 z-i Z - Atomic.'-,'"
masses
Halflife.
the sequence of emission is: First. Last when No.ofa.- No. of (3-
of first
(a) a,~,y (b) ~,a,y (c) y,a,~ (d) ~,y,a Series mem- mem- divided partieles particles
member
[Ans. (b)] her her by 4, the emitted emitted
in years
remain-
[Hint: 1x -
_~e ---7 z+ ~y; z+ lAy - iRe ---71=iz; ~
der
A 4z _y ---7 A-4Z * ]
Z-l Z-l
Thorium 2~~Th 1.4 x 10 10 2~~Pb 0 6 4
(4n)
5. Which of the following elements is an isodiaphere of 2~i U?
'ta) 2~3Bi (b) 2~2Pb (c) 23.kTh (d) 2t\Pa Uranium 2~~U 4.51 x 10 9 2~~Pb 2 8 6
(4n + 2)
[ADS. (c)]
[Hint: Is.odiapheres are formed by a-emission. Actinium 2~~U 7.07 x 108 z~~Pb 3 7 4
niz
U - iHe ---7 239~Th (Isodiaphere)] (4n + 3)

6. A certain radioactive material iX starts emitting a and ~


(i)''Thorium series(4n series):
particles successively such that the end product is : _ ~ y, The -u -13 -13 -(1. -a; -(1.
232Th -t 228Ra -t 228 Ac -t 228Th -t 224Ra -'" 220Rn ->. 216 pO
number of a and ~ particles emitted are : (VITEEE 2(68) ~ U ~ ~ u ~ ~ ~ M
85 At~
(a) 4 and 3 respectively (b) 2 and 1 respectively 208
81 Tl . 216

~212B·7
(c) 3 and 4 respectively (d) 3 and 8 respectively -~ -0: -0: -[3
[Ans. (b)] 208
Pb /
[Hint: ---7 1x t.-;r 82 "'-
-~
/
~[3
83 1 "'-
-[3~ -0:
in mass number ! 2 212 Po 212
82 Pb
Number of a-particles = 84 .
44
Z 2 x Number of a-particles + Number of~-particles Z - 3
158 'I G.R.B. PHYSICAL CHEMISTRY FOR COMPETITIONS

(ii) Uranium s.eries [(4n +2) series}: series in nature whose isotopic masses carry a numerical
-a -I} -I} a -a -a relationship of 4n + 1 The most reasonable explanation for the
2t~U - t 2t<iTh -t 2tiPa -t 2tiu -t 2igTh -t 2~;Ra - t 2~lRn absence of this series in nature was that no member of this series
was sufficiently long lived to have survived over the years since
210Tl' " 214 Pb I the series might have been formed. Except the last member, all

'~~Pb(" "('~po~
other members of this series have been obtained by artificial
means. The name of this series is given on the long lived isotope
' ) 'klBi ( ' 82 , of neptunium (Hal( life' 2~~ Np 2.25 X 10 6 years). This family
-0 /-13 ' -~ ~' . differs from the other three naturally occurring series in the
, 2~! P02~~ At

.',
following respects:

'7~~TI~ (a) The last member of this series is an isotope of bismuth


(2~j Bi) and not an isotope of lead.,
'~ 2~~'Bi~'/2~~Pb (b) The only member ofthis series which is found in nature is
. -13 p' -IX
tlie last member.
210
, J 84 0 (c) The series does not contain gaseous emanation.
(iii) Actinium series [(4n + 3) series]: - a -Il -a a -J:l
2~~ Np - t 2~~ Pa - t 2~~U - t 2~~ Th - t 2~~Ra -t 2~Ac
2~~-Fr
-a' ':'J:l ' a / ~13, a 213 Po '
235U-t 23lTh _-", 231 Pa -t 227 Ac ~223Ra -t 219Rn, 84 '
92 90 -cr 91 ' 89 ~ / 88 ,86
-~
2~~Bi ('-2~~Pb~'
7 ~ -~ -a
/ . 2~~Bi ('- 2~~At ('- 2~~Fr
a!

13
207 Tl
81. '
-13 227
90
Th -IX

-13 2~ TI -IX

. 2~; Pb/ 22k\Bi ~ 'k\Pb ." '~Po In this series, seven alpha and four beta particles are emitted.
-"
~21l PO , -13 84
3.9 RATE OF DISINTEGRATION AND HALF
LIFE PERIOD
Only 18radioactive isotopes with atomic number 82 or less, The radioactive decay of the different radioactive substances
are found in nature. 14 C is the exception because it < is , differ widely. The rate of disintegration of a given substance
, continuously synthesized in our atmosphere. All these natural depends upon the nature of disintegrating substance and its
radioactive elements have half-life longer than I 09yrs (age of total amount. The law of radioactive disintegration may be
earth). Another 45 ,radioactive isotopes having atomic number defined' as "the quantity of ra(}ioactive substance which
greater than 82 are also found innatuni and fall in above three disappears in unit time is directly proportional to the
natural decay series. amount* of radioactive substance present or yet not decayed."
The quantity of the radioactive substance which disintegrates or
Similarities between Radioactive Series disappears in unit time is called rate of disintegration.
(i) In' all the series, there is an element of zero group with Tbe rate of disintegration decreases with time as the
atomic number 86. This element comes in the gaseous state and is amount of radioactive substance decreases with time. One of
called emanation. Different names are given to three isotopes. the most important characteristics of the radioactive
These are radon in uranium series, thoron in thorium series and disintegration is that a certain definite fraction of a radioactive
actinon in actinium series. sample undergoes disintegration in a definite period of time. This
(ii) In all,the series, the last product is an isotope of lead time period does not depend upon the initial amount of the
(atomic number 82), 206Pb in uranium series, 207 Pb in actinium radioactive substance.
series while 208 Pb in thorium series. Due to this reason, lead is For example, whatever be the amount (initial) of 131 I taken, it
found in nature as a mixture of these three isotopes. becomes half within 8 days. This has been shown in Fig. 3.4 (a).
(iii) In all the series, there are certain elements which Initial amount of 131 I The amount of 131 I after 8 days
disintegrate in a branching process by emitting either ex or 20 grams 10 grams
B-particles. The species thus formed are then disintegrated in 10 grams 5 grams
such a way as to give a common product. 5 grams 2.5 grams, etc.
Rutherford mtroduced a constant known as half life period. It
Neptunium series [(4n +1) series]: For many years
is defined "as the time during which half the amount of a
scientists speculated upon the failure to find a disintegration given sample of the radioactive substance disintegrates".
Amount of a radioactive substance can be taken in terms of number of atoms or moles or grams, milligrams, etc.
RADIOACTIVITY AND NUCLEAR TRANSFORMATION 159

Amount of Amount of radioactive


Time
radioactive substance decomposed
(1)
substance (N) N)
0
t~12
er
2.1 N o=. 2. No
o
~ No =[l~JNo
2t1l2 1
-No
4 er - No
2
. 3
-No
4 [1- ±JNo
[1- ~]No
~No=GrNo
3t ll2 7N
gO

4t112
cr
I· o= - No
-N
16 2/
IS
.16
[ 1 ] No .
No=I--
16

o
'---v--'
8 16 24 32 40. 48
nt1l2
. (Ir
- No [I nr]No
tl12 Days-- .\2
Amount of radioactive substance left after n half life periods
Fig. 3.4 (a)
. N=Gr No

i
'Qi
No and . total time T
where, n is a whole number.
:::
nx t 112
~:::.SOME SOLVED EXAMPLES\ ::::.:: :
U
:"
c:
"C
0) .
Example 10. The half life period of radium is 1.580 years.
. >- How do you interpret this statement?
«I
0
~ Solution: Whatever quantity of radium is taken, it shall
c:
:" become half after the expiry of 1580 years. The following table
'0 explains the statement:
....
0)
..0 Quantity of radium Quantity of radium
E at present after 1580 years
::l
Z
100 atoms' 50 atoms
tll2 2tl12 3tl12 4tl12 50 gram 25 gram
Time ----+ 5 mole 2.5 mole
Fig. 3.4 (b) Example 11. The radioactive isotope 137 Cs has a half life
period of30 years'fjStarting with I mg of J37 Cs, how much would
Every radioactive element is characterised by a definite constant remain after 120years?
value of half life period, Half life period of an element is also a Solution: At this time, we have 1.0 mg of 137 Cs; after 30
measure of its radioactivity, since shorter the half life period, .the years, we shall have Qne half of the original, or 0.50 mg; after 60
greater is the number of disintegrations and hence greater its .years, we shall have 0.25 mg; after 90 years, we shall have 0.125
radioactivity. Half life periods vary from billions of years fQr mg and, finally, after 120 years, we shall have 0.0625 mg.
some radioisotopes to a fraction of a second. After30 years 0.50 mg
Halflife period is represented as t 112' 60 years 0.25 mg
Let the initial amount of a radioactive substance be No. 90 years 0.125 mg
After onehalflife period (t1l21 it becomes = No 12 120 years 0.0625 mg
After two half life periods (2t 112)' it becomes =N 0/4 Alternative solution: Total time I 20 years
We know that, . total time = n x t 112
After three half life periods (3t 112)' it becomes = N 0/8 So, 120' nx30

and Arter n half life periods (nt 1l2 )', it shall become = Gr No Thus, the quantity of the isotope left after· .
n4

(1)4 No =(1\4
Thus, for the total disintegration of a radioactive four halflife periods =l'2 '2). xl
substance an infinite time will be required. 1
-= 0.0625mg
16
160 G.R.B. PHYSICAL OtiEMISTRY FOR COMPETITIONS

. " Example 12.' A radioactive element has half life period of Example 15. A radioisotope has tll2 5 years. After a
30 days. How much ~fit ~ill be left after 90 days? given amount decays for 15 years, what fraction o/the original
Solution: Total time = 90 days isotope remains?
Solution: Half life (t 112 ) == 5 years
Half life (t I12 ) 30 days
Time for decay (T )::= 15 years
We know that, total time =n x tll2 ,
We know that, T = n x tl12
So, 90= n x 30 So, 15 = n x 5
n==3 or n=3
Thus, quantity left after three half life periods, . Let the original amouI¢ be = No
, Let the amount left after three half life periods be N
1 3
::= (z) No [N 0 original amount] fraction N / No

,I I
== xNo==-No
8 8,
We know that, N= lz1)11 '
r
No

Example 13. The half life period of 2~~PO is 140 days, In N _(1)11 _((\3

how many days 19 of this isotope is reduced to 0.25 g?


or
No - \Z -lzj == 8

Solution: Original quantity of the isotope (N 0) I g Thus, after 15 years lth of the original amount remains.
, Final quantity of the isotope NO. 25 g 8
Exampl.e 16.' if in 3160 years, a radioactive substance
We know !hat, becomes onejourth of the original amount, find its half life
period.
N 1
1 == - (1)11 xl Solution: ---
So, No 4
4 ..2
1
or
I"
So,
4 UY'
2)

m
Time taken T == n X tl12
n=2
== 2 x 140
280 days
or

or
lz'Ifn=2
(~r
Example14. The half life period oj. 234 U is 2.5 X 105 We know that, total time' T' =n xt 112
years. In how much time is the quantity of an isotope redJlced to So, 3160' 2xt Jl2
25% of the original amount? 3160
or = 1580 years
tll2 =
Solution: Initial amount of this isotope No == 100 2
Final amount of the isotope N == 25 The halflife period of the radioactive substance is 1580 years.

We know that, N = (~r No


So, '
7. a
Hltlf life of radioactive sample is 2x years. What fraction of
25=Gf x 100 this sample will remain undecayed after x years?
(a) .!. (b) 1 (c) I, (d) 2
or
1~0 =Gr 2fi
[Ans. (b)]
Jj

or I P)11 [Hint: A=.


t
l_o )
'10 0 (N
'" N
4 \2
0.693 _ 2.303 1oglO - (N 0)
or P)2 =(ll"
\2 . 2;
---
tl/2 t N

or n==2
0.693
--
2.303 1
- - oglO -
(No)
2x x N
Time taken T nxt ll2 1
2 loglo 2 = log
(NNO)\
= 2x 2.5x 105 5
= 5x 10 years
RADIOACTIVITY AND NUCLEAR TRANSFORMATION 161

N I . Suppose the number .of atoms of a radioactive substance


-=-
No .J2 present at the start of observation, i. e. , when t = 0, is No and after.
time t; the number of atoms remaining unchanged is N. At this
Fraction undecayed = _1_ ]
instant a 'very small number of atoms dN disintegrate in a small
.J2
8. Half1ife of a radioactive element is 10 days. What percentage time dt; the rate of chang~ of A into B is given by - dN . The
of the element will remain undecayed after 100 days? dt
(a) 10% (b) 0.1% negative sign indicates that number of atoms decreases as time
increases. Since, rate of disintegration or change is proportional
(c) 0% (d) 99%
to the total number of atoms present at that time, the relation
[Ans. (b)]
becomes .
[Hint: In ten times ofhalflife 99.9%, the element undergoes
decay; then percentage of undecayed radioactive element will dN
--=A·N ... (i)
be 0.1%.] .;'. dt
9. Which among the fOllowVtg relations is correct? , A' is called the disintegration constant or decay constant.
(a) t1l2 = 2t3/4 . (b) t1l2 = 3t3/4 dN
(c) t3/4 = 2t1l2 (d) t 3/ 4 '= 3t 1l2 Evidently - - = A' dt ." (ii)
N
[Ans. (c)]
.
[Hmt: 0.693 2.303 1
- - = - - og -
(No) If dt = I second, A = - dN
N
... (iii)
. til 2 t . N
Thus, Amay be defmed as the fraction of the total number
. 0.693 =2.303 log 100 of atoms' which disintegrate per second at any time. This is
til 2 t3/4 75 constant for a given radioa~tive isotope.
. . t3/4 =
10. Select the correct statement:
2t 1l2 ]
Integrating ~q. (ii), - J ~ = AJdt
(a) Same amount will decay in every half life -log N = At + C .. : (iv)
(b) Amount decayed in first half life is maximum C is the integration constant.
(c) Amount decayed in first half life is minimum When t = 0, N = No
(d) Amount decayed in a half life depends on the nature of putting the values in eq. (iv),
element -logNo=C ... (v)
[Ans. (b)] Putting the value ofC in eq. (iv)
[Hi~t! :>Amount decayed in first half life is maximum. Half of
-log N = At - log No or log No - log N =At
the initial amount is decayed in first half life.]
11. The half life period of a radioactive mineral is 15 min. What
percent of radioactivity of that mineral will remain after 45
min? [UGET (Manipal Medical) 2006]
(a) 17.5% (b) 15% (c) 12.5% (d) 10%
[Ans. (c)]
r
N
[Hint: n = 45 = 3 = No. of half lives No
15

N=NoGr =Ioox(~r =12.5%] I


12. Half life of a radioactive element is 16 hrs. What time will it
take for 75% disintegration? (DeE 2006) --Time--
(a) 32 days (b) 32 hrs (c) 48 hrs (d) 16 hrs Fig. 3.5 Fraction of radiOisotope remaining versus time
fAns. (b)]
[Hint: 75% decay takes place in t3/4 (3/ 4th life)
. "-
Slope =- 2.303
t3/4 = 2t1l2 = 2 x 16 = 32 hrs]
Disintegration constant: A chemical reaction whose rate of r
reaction varies directly as the concentration of one molecular IOg( ~o)
species only, is termed it first order reaction. Radioactive
disintegration is similar to. such a chemical reaction· as one
radioactive species changes into other. This change can be I
represented by the equation:
A'~B
- - Time ---::::--
-
Fig. 3.6 Log of fraction remaining versus time
162 G.R.B. PHYSII:;AL CHEMISTRY FOR COMPETITIONS

log No At % decayed amount 100 ~ 36.79:;::: 63.21


or or 2.303 log 10 :;::: At
N N Time during which 63.21 % substance undergoes decay is
1 2.3.03 No called average life. ,
or 1\,=-- 1oglO- ... (vi)
t N Relation between rate of decay and mass of given element
This equation is called kinetic equation and is obeyed by first
order reactions.
Rate(-~)=AXN .

Relationship between !talC life period and radioactive" A x No. of atoms of element undergoing decay
disintegration constant. .
= 0.69-3 .x mass. x Avogadro's number
When t:;::: t 1/2 , N t1l2 atormc mass
2
Putting the values in eq. (vi) Parallel Path Decay
2.303 No 2.303 2
1
I\, 1oglo ~- = - ' - 1ogIO Let a radioactive element 'A' decays to 'B' and 'c' in two
tl/2 . No/2 tll2 parallel paths: '
The value oflog 10 2 is 0.3010,
·
S0,
1 0.693
1\,=-- or
t1l2
t1l2 =
. 0.693
A
A<B C
Thus, half life period of a given radioactive substance does not Decay constant of
depend on the initial amount of a radioactive substance but depends 'A' Decay constant of' B' + Decay constant of' C'
only on the disintegration constant of the radioactive element. AA AB + AC ' ." (i)
Here, AB = [fractional yield of B] x A A
3.10 AVERAGE,LIFE' AC :;::: [fractional yield of C ] x A A '
It is the sum of the periods of ~xistence of aU the atoms Maximum Yield of Daughter Element
divided by the total number' of atoms of the radioactive
Let a radioactive element' A' decays to daughter element' B' .
SUbstance.
' AI'" TotaI'~'
Ihe tIme 0 f aIIthe atoms A )B .
verage he = , ,
Total nun1ber of atoms AA and AB are decay constants of 'A' and 'B'. Maximum
fo=
t dN I activity time of daughter element can be calculated as:
==---
No A t
max
= 2.303
(AB AA)
10 [AS] i
glO AA
Thus, average iife of a radioactive element is the inverse of its
disintegration'or decay constant.
I t 1 '2 3.11 RADIOAC11VE EQUILIBRIUM
Average life = -'- = 1.44t1l2
A 0.693 Let us consider thafa radioactive element A disintegrates to give
The average life of a radioactive substance is 1.44 times of its B which is also radioactive and disintegrates into C.
half life period. . A ----t B ----t C
The element B is said to be in radioactive equilibrium with A if
Alternatively:

We know that, A~ 2.303 I oglO ( -N 0


~,--
t N
I
j
its rate of formation from A is equal to its rate of decay into C. If
A1 and A2 are the disintegration constants of A and B, N 1 and N z
are the number of atoms of each radioactive element present at

or .
At = loge l; J
iN
equilibrium, then we have
Rate of formation of B = Rate of decay of A = AIN 1
and Rate of decay of B :;::: AzN 2
e AI =NO
-
At radioactive equilibrium,
N AINI =A 2 N 2
N -At A2 11 Al Average life of A
, or --:::!e or =- - -
No N2 'AI 11 A2 Average life ofB ZB
i 1 N
.Let t ; then = e- 1 Thus, the number of atoms of A and B are in: the ratio of
A No e their average life peri9~s.
N I Nl A2 _0.6931 (t1l2 h
-- 0.3679
.I No 2.718 N2 ~ - 0.693'/ (t1l2)1

% remaining amount = N x 100= 36.79 !!L=


No , N2 (t1l2 h
RADIOACTIVITY AND NUCLEAR TRANSFORMATION 1163
When AA of parent element is less than AB of daughter produces flash of light when it is
struck by electromagnetic
element, but both are not very small then a transient equilibrium radiation like v-rays, for detection ofy-rays.Sodium iodide (NaI)
is reached, when and thallium iodide (Til) are used as phosphor. Rutherford first of
all· used zinc sulphide (ZnS) as phosphor in detection of
NA AB -AA a-particles.
in fact it is steady state. ::~:::a.SOME SoLVED EXAMPLES\ I::::: :
·3.12 UNITS OF RADIOACTIVITY .ExampleI7.· Theha/fiije period of radium is 1600years.
In radioactivity, the number of atoms which disintegrate in uriit Calculate the disintegration constant ofradium. Mention its unit.
time is .of real importance rather than the total amount of the. · D'IsmtegratlOn
. . constant 1\ ~ =-0.693
-
S.0Iuhon:
radioactive substance expressed by mass or number of atoms, . t1l2
i. e. , the activity of a radioactive substance is the rate of decay or Since, t 112 1600 years
number of disintegrations per second. 'A = 0.693
The unit of radioactivity called Curie (Ci) is defined as that So,
'1600
quantity of any radioactive substance which has a decay rate
or A = 4.33 X 10-4 year-I
of3.7 x 10111 disintegrations per second.
. Example 18. The disintegration constant of 238 U ;.
This unit is a large one and hence smaller units like milli-
curie (mCi) and microcurie (!.lCi) are used. 1.54 X 10-10 year-t. CalCulate the halflife periodof 238 U.

1 millicurie = 3.7 x 10 7 disintegrations per sec ·


S oI ution: H aIfl'''''od
be pen ,t1l2 =0.693
--
I microcurie 4
3.7 x 10 disintegrations per sec .
A
There is another unit Rutherford (Rd) which is also used these . Since, A = 1.54 X 10- 10 yw- I

days. It is defined as the amount of a radioactive substance 0.693


which undergoes 10 6 disintegrations per second. Smaller units
SQ, tl/2 = = 4·5. x 109 years
1.54x 10-10
like mill i-Rutherford and micro-Rutherford are also used.
Example 19. The halflifo period ofradon is 3.8days. After
1 milli-Rutherford·= 10 3 disintegrations per sec (dps) how many days will only one-twentieth of radon sample be left
over?
I micro-Rutherford =I disintegration per sec
The SI unit of radioactivity is proposed as Becquerel which .
S olutlOn: We know th at, 1\
1 0.693
=- 0:693
- =- - =.0182day -I
refers to one dps. tl/2 3.8
=
1 curie = 3.7 x 10 4 Rutherford 3.7 x 10 10 Becquerel
Let the initial amount of radon be No and the amount left after
I curie 37 GBq
t days be N which is equal to N o .·
Here, G stands for 109 , i. e. , giga 20
Gray (Gy) =1 kg tissue receiving 1 J of energy Applying the equa~ion,
Sievert (Sv) gray x quality number of radiation t 2.303 IoglO No 7i
Quality number of la-particle = 20
Quality number of l~particle = 1 2.303
1 No _ 2.303 I 20
Specific activity of a radionuclide is its activity per kilogram 0.182 oglO No 120 - 0.182 oglO
(or dm 3 ) of the radioactive material. = 16.54. days
. (In some cases, specific activity is taken as the activity pel: Example 20. A counter rate meter is used to measure
gram.) the activity of a radioactive sample. At a certain instant, the
Radiation counter: There. are two main radiation counters count rate was recorded as 475 counters per minute. Five minutes
in practice. later, the count rate recorded was 270 counts per minute.
1. Geiger-Muller counter: It is used to count charged Calculate the decay constant and halflife period ofthe sample.
particles, e. g. ,a and ~particles, emitted by a raru.oactive nucleus. Solution: Let No and N be the number of atoms of the
This counter is simply a metal tube filled with a gas like argon. radioactive substance present at the start and after 5 minutes
In order to count and detect neutrons, boron trifluoride (BF3) respectively.
is added along with a gas in the G.M. counter. Neutron strikes Rate of disintegration at the start =AN 0 = 475
105 B nuclei 10 produce a-particle, which is then detected and
and rate of disintegration after 5 minutes = AN= 270
counted in Geiger counter.
Div.iding both, AN 0 = 475
I~B + ~n~ ~Li + iHe AN 270
2. Scintillation counter: y-radiationsare detected by
or .176
Scintillation counter. A phosphor is used in this counter which N
164 G.R.B. PHYSIC&- CHEMISTRY FOR COMPETITIONS

2.303 log 10 N o
We know tha.t, A As one helium atom is obtained by disintegration of one atom
t N. of radioisotope, the total number of atoms of the radioactive
~""= 2.303 I"oglo, 1'7'6 =.
0113'
mmute -I
isotope which have disintegrated in one hour
5 = 3.01 x 10
20 or 0.0003 x 1024
. d = 0.693 =0.693
HaIf JI'f'epeno - - = 6 . 1 mmutes
. The number of atoms of the radioactive isotope left after one
A 0.113 'hour,
Example 21. You have O.lg atom of a radioactive isotope N = (6.023 X 1024 - 0.000301 X 1024 )
1 X (half life 5 days)~ How many atoms will decay during the
11th day? 6.0227 X 10 24
Solution: Amount of radioactive substance = 0.1 g atom 1 2.303 No
, Using, "" - - 1og-
So,· ' No 0.1 x Avogadro's number t N
0.1 x 6.02 xl 0 23 , 24
1 2.303 1 6.023 x 10
= 6.02 x 10 22
atoms "" - - og - - - - - - 0 : - : -
t 6.0227 X 10 24
6.02 x 1022
Number of atoms after 5 days = ' = 3.01 X 10
22
= 2.303 x 2.1632x 10-5 =4.982xlO-5 hr- I
2
t 112 = 0.693/ (4.982 x 10-5 x 24 x 365) 1.58 years'
, , ,3nIxIO n , 'n'
Number of atoms after 10 days = = 1.505 x 10 Example 23. Calculate the average life of a radioactive
2
substance whose halflife period is 1650 years.
, Let the number of atoms left after 11 days beN.
We know that, Solution: Average life = 1. 44 x t 112
2.303 No = 1.44 x 1650= 2376 years
t A 1oglO N
Example 24. 90 Sr shows ~activity and its half life period
0.693
Given, t 11, A , No = 6.02 x 1022 is 28 years. What is the activity of a sample containing 1 g of
5 90 Sr?
22
11 ,2.303 x5 10 6.02 x 10 Solution: Activity = No. of atoms disintegrating per second
So,
0.693 glo N
22
:::: A x total number of atoms
or log 10 6.02 x 10 11 x 0.693 = 0.6620 0.693
N 2.303 x 5 A=-------
28 x 365 x 24 x 60 x 60
n 23
6.02 x 10 = Antilog 0.6620 = 4.592 6 0?3 X 10
N
Total number of atoms in 1 g of 90Sr
,
=-' -----
90
So, N 6.02 X 10 22 1.3109 X 1022 . . 0.693 6.023 x 10 23
Actlvlty = x ----
4.592 28x 365x 24x 60x 60 90
Atoms decayed during 11 th day = 5.25 X 1012 disintegrations per second
, [1.5050 x 1022 - 1.3109 x 1022 ]
5.25x 1012
0.1941 x 1022 141.89 curie
21
3.7 x 1010
1.941 x 10
. Example 25. A chemist prepares 1.00 g ofpure IJ C. This
Example 22. 109 atoms of an a-active radioisotope are
disintegrating in a sealed container. In one hour, the helium gas , isotope has halflife of21 minutes, decaying by the equation:
c;Uected atSfP is 11.2 cm 3 . Calculate the half life of the radio UC~lIB+Oe
6 5 1
1 I
isotope. (a) What is the rate ofdisintegrationper second (dps) at start?
Solution: Amount of radioactive isotope = 109 atoms, '
c'b) What a~e the activity and specific activity of IJ C at start?
23
or No tOx 6.023 X 10 atoms
(c) How much of this isotope ( 1~ C) is left after 24 hours of its
6.oi3 x 10 24
atoms
prep~ration ?
3
22400 'cm of helium contains == 6.023 x lOr atoms
6 02~ x 10 23
Solution: (a) Applying, - a; = ANO '
11.2 cm 3 of helium will contain = _._-1....._ _ x 11.2 atoms
22400 0.693 1x 6.02 X 10 23
x------
= 3.01 x 1020 atoms 21 x 60 11

= 3x 1019 dps
I
RADIOACTIVITY AND NUCLEAR TRANSFORMATION 165
,

. 3 X 1019 10 .. = -
ActiVity 0.693
- x -W- x. 6023 X 1023
(b) Activity = . 10 (1 curie = 3.7x 10 dps)
tll2 ·At. wt.
3.7 x 10
= 8.108x lOs curie 3.7x 10 10 = 0.693 x _1_. x 6.023 x 1023
{112 226
Sp. activity = 3 x 1019 X 10 3 = 3 X 10 22 dis/ kg s It will give the half life in seconds.]
=8.108xW II curie 17. Assuming that 226 Ra (til 2 == 1.6 X 103 yrs) is in secular
equilibrium with 238 U (t1l2 == 4.5 X 109 yrs) in a certain
.
(c) Applymg,N=No ( -1 •
n J
. t
. [ n= - = 24 x 60 = 68.S7 1 mineral, how many grams of radium will be present in for
. 2 . tll2 21 J every gram of 238 U in this mineral?
. N = 1x GJ 6S.57 = 2.29 X 10-21 ~ (a) 3.Tx 10-7
(c) 3.4 X 10- 7
(b) 3.4 x 107
(d) 3.7 x 107
rUUSTRATfdNf Of OBlECflVE-QU£STiONS [Ans. (c)]
NI 22~a _ tll2 226Ra
13. The time of decay for the nuclear reaction is given by [Hint:
N 2 238U - t 112 238 U
t = 5t1l2 . The relation between mean life 1: and time of decay
w/2261.6xHi3
't' is given by: ~ = 3.4 x 10-7 g]
. 1 1/ 238 4.5 x 109 '
(a) 21: In 2 (b) 51: In 2 (c) 21:4 In 2 (d) 4 In 2
1: 18. A certain radioactive isotope decay has a-emission,
AIX~ A I -4 y
[Ans. (b)] ZI .. ZI-2
[Hint: t = 5t ll2
half life of X is 10' days. If 1 mol of X is taken initially in a
In 2
t=5x- sealed container, then what volume of helium will be
A collected at STP after 20 days?
t=51:ln2r (a) 22.4 L (b) 11.2 L (c) 16.8 L (d) 33.6 L
14. The activity of a sample of radioactive element 100 A is 6.02 [Ans. (c)]
curie. Its decay constant is 3.7 x 104 S-I. The initial mass of [Hint: After 20 days 0.75 mol helium. will be formed.
the sample will be: :. Volume of helium at STP = 0.75 x 22.4 .
(a) 10- 14 g (b) 10-6 g (c) 10- 15 g . (d) 10-3 g ==16.8L]
[Ans. (c)]
[Hint: Activity = A x_w_ x 6.023 X 1023 3.13 ARTIFICIAL TRANSMUTATION
At. wt.
Transmutation is defined as the conversion of one element
6.02 x 3.7 x 1010 = 3.7 X 104 x ~ x 6.023 X 1023 into another or one type of atom into another. When this
100
15 conversion is achieved by artificial means, it is termed ··as
W=IO- g]
artificial transmutation. .
15. A fr<;:shly prepared radio medicine has half life 2 hours. Its The conversion of elements into one. another has been -the
activity is 64 times the permissible safe value. The minimum dream of the human race for many centuries. In the middle ages,
time after which it would be possible to treat the patients with it was popular under the name of ' Alchemy'. Alchemists were
the medicine is:
unsuccessful in this attempt as they were having .very little
(a) 3hrs (b) 9 hrs (c) 24 hrs (d) 12 hrs knowledge about the structure of atom. With the background of
[Ans. (d)] the clear picture of the structure of the atom, modem scientists
Hint: N=No(~r have realised that to convert one element into another, the
nucleus should be attacked and altered.
The first indication that a stable nucleus could be disrupted
was given by Rutherford in 1.919: He observed that when
nitrogen was bombarded with high speed a.-particles from 214 Po,
protons were emitted. Thus, nitrogen was changed into an isotope
n = 6 half lives of oxygen .

.. time=2x6=12hrs]
16. . .One gram of 226 Ra has an activity of nearly 1 Ci. The half life
Later on, Rutherford .and Chadwick showed that many other
of 226 Ra is: -
elements from boron to potassium with the excepticlr of carbon
(a) 1582 yrs (b) 12.5 hrs and oxygen - could be transmuted by bombardment with
(c) 140 days (d) 4.5 x 109 yrs a-particles. However with heavier elements, there was only
fAns. (a)] scattering of a-particles as these suffered a force of repulsion. It
fRint: Use the following relation for calculation of activity: was, thus, concluded that to bring transmutation in heavier
166 'I G.R.B. PHYSICALCHEMISTRY FOR COMPETITIONS

elements, the projectiles must have higher, energies than , 3. Often accompanied by release or Accompanied by release or ab- .
a-particles obtained from natural sources. It was suggested by .absorption of tremendous sorption of, relatively small
.Gatnow in 1928 that a proton <l H) would be a much more amount of energy. amount of energy.
effective projectile than an a-particle, but it was not available as a 4.' Rate of reaction is independent Rate of reaction is influenced by
high speed particle. , of external factors such' as tem- external factors.
The charged particles, like alpha particles, protons, deuterorts perature, pressure and catalyst.
can be made much more effective projectiles if they have high
Example 26. Calculate the energy in the reaction
velocity. Out of all the instruments which have been devised for
accelerating projectiles, the one. which has attracted the widest 2 : H + 2 ~n ~ iHe
interest is the cyclotron ofE.O. Lawrence. The projectile can be
, . accelerated to the speed of25,000 miles per second. Given, H 1.00813 amu, n 1.00897 amu and
The discovery of neutron by Chadwick, in 1932, added He 4.00388amu
another projectile for transmutation. The neutron being Solution: Loss of mass in the given nuclear reaction
electrically, neutral can penetrate easily into the atomic nucleus. ::: 2(1.00813+ 1.00897)- 4.00388
Although neutrons; are the most effective and versatile of
projectiles, yet they suffer the objection that they inust be = 0.03032 amu
produced, by transmutation at the time (If use. High speed E'lergy released::: 0.03032x 931 = 28.3 MeV
neutrons are obtained when berylliiim-9 is bombarded with
, a-particles, Types of Nuclear Reactions
~ Be + iHe ---? I~C + ~n (a) Projectile capture reactions: The bombarding particle
is absorbed with or without the emission ofy-radiations.
and slow neutrons are obtained by bombarding lithium-7 with 2~U + ~n ---? 2~iu + y
protons. tI Al + ~n ---? r~Al + y
~Li+'lH ---? lBe+ ~n (b) Particle-particle reactions: Majority of nuclear
In general, for the transmutation of lighter elements, charged reactions come under this category. In addition to the product
particles like alpha particles, protons, deuterons are used while nucleu&;.an elementary particle is also emitted.
for heavier elements, neutrons are used. 23N
II a + IIH,---? 12 23M'g + on
I

Nuclear Reactions 23Na + 12H


11
---? 24Na'+ IH
11 I
The reactions in whicb nuclei of atoms interact witb other
nuclei or elementary partiCiessucb as alpba particle, proton, . tfNa + iHe"'~ t~Mg + :H
, deuteron, neutron,etc., resulting in ttie formation of a new
nucleus and one or more elementary_ particles are called 147 N + 0I n ---? 14c
6
+ II H
nuclear reactions. Nuclear reactio~ are expre&sed in the same
fashion as chemical reactions, i. e. ,react~ts on left hand side and (c) Spallation reactions: High speed projectiles with
the products on right hand side of the sign of (= ) or (~ ). In all energies approximately 40 MeV may chip fragmentS from a
heavy nucleus, leaving a smaller nucleus.
nuclear reactions, tbe total number of protons and neutrons
are conserved as in cbemical reactions, the number of atoms ~~Cu + iHe +400 MeV -----4 iicI+14 :H+16 bn
of eacb element are conserved. The s~bols ~ n, : H, i He, H, t ~~As+ tH ---?~~Mn +9 IH+12 ~n
:-? e, +~ e and yare used to represent neutron, proton, a-particle, . 2t~ U + iHe ---? 18.74 W + 20 :H+.35 ~n
deuteron, electron, po&itron, v-rays respectively. A short hand
notation is often used for the representation of nuclear reactions. (d) Fission reactions: A reaction in which a heavy nucleus
As for eXlPllple, the nuclear reaction is broken down into two or more medium heavy fragments. The
, 14N
7,
+24He '---? 178 0+ IHI
process is usually accompanied. with emission of neutrons and
large amount of energy.
is represented as IjN(a, p) l~o. Some of the characteristics that
235U ,141
92 + Ion -----4 92 3' I M V
56 Ba + ?6 Kr + on + 200 e
differentiate between nuclear reactions and ordinary chemical
reactions are summarised below: (e) Fusion reactions: Light nuclei fuse together to
reproduce comparatively heavier nuclei.

1. Elements may be converted from No new element can be pro-


tH+ fH ---? iHe+ ~n+17.6MeV
one to another. . duced;
A fusion reaction is the source of tremendous amount of
2. Particles within the nucleus are Only outermost electrons partici- energy.
involved. pate. P.air production : Pair production is the most striking
example of mass-energy .equivalence.
I
RADIOACTIVITY AND NUCLEAR TRANSFORMATION 167

We can write pair production symbolically as : 3.15 NUCLEAR FISSION


Photon + Photon ~ Particle + Antiparticle . "The process of artificial transmutation in which heavy
Particle + Antiparticle ~ Photon + phQton nucleus is broken down into two lighter nuclei of nearly
A particle and' antiparticle can collide and annihilate each comparable masses with release of large amount of energy is
other, producing two high-energy gamma ray photons. Pair termed nuclear fission." The word fission is derived from its
production must obey the law of conservation of energy and resemblance to the biological process called fission in which a
momentum. living cell breaks up into two cells of roughly same size.
The follo~ing are the important contributions of artificial After the discovery of neutron, Fermi, in 1934, made an
transmutation: attempt to synthesise transuranic elements from uranium by
(i) Discovery of neutron bombarding with neutrons. This experiment was repeated in
(ii) Artificial radioactivity Germany by Hahn and Strassmann. In one ofthe chemical tests,
(iii) Nuclear fission they found that one of the products was an isotope of barium
(iv) Nuclear fusion along with the formation of an isotope ofthe element with atomic'
number 93 (neptunium). In 1939, they proposed that uranium
3.14 ARTIFICIAL.RADIOACTIVITY after capturing neutron undergoes two types of reactions-one
with 238 U isotope and the other with 235 U isotope.
In 1934, Irene Curie and F. Joliot observed that when boron and
alUIl)inium were bombarded by Cl-particles, neutrons, protons and (a) 2~~Uis converted into 2JjNpand 2J~Pu.
positrons were emitted. When bombardffient was stopped, the'
emission of protons and neutrons ceased but that of positrons did 238 U .:_ 1 , 239 U -13 ,239Np -13 ,239 pu
not. The emission of positrons continued with ti:Q1e but decreased 92 'on ~ 92 ~ 93 ~ 94
(Plutonium)
exponentially in a manner similar to natural radioactivity. Curie
and Joliot explained this observation by saying that during (b) 2~i U captures slow neutron and splits up into fragments.
bombardment, a metastable isotope is formed which behaves as a
radioactive element. This process was termed as artificial 2Ji U + ~n ~ 2J~U ~ I~Ba +~~Kr + 2 ~n
radioactivity•.
"The process in which a stable isotope is converted into a It has been observed that. during fission of "gi
U not only
radioactive element by artificial transmutation is called isotopes of Ba and K.r are formed but isotopes of v2.riollS other
artificial radioactivity." elements come into existence. These isotopes fall under two
When ~i Al is bombarded by a-particles, radioactive isotope groups. First type-isotopes having atomic masses from 80 to
~~ Pis formed. 110 and atomic numbers from 35 to 43 and second
type-isotopes having atomic masses from 120 to 150 and
~gSi + ~ H(95% of total conversion) atomic numbers 51 to 57. It is believed that only ~wo isotopes are
first formed as primary fission products which then give rise to
C ~~P + ~n (5% of total conversion) secondary products by successive disintegration.
I 30S' 0
~ 14 1+ +Ie 235 I ·~ 141
92 U + on 56 Ba + 92 I
36K.r + 3 on 1
Positron ~ 1~~Xe + ~~Sr + 2 ~n Primary products
In a similar manner, the artificial radioactivity was observed ~ 140Xe+ 95Sr+ In
54 38 0
when I~ B was bombarded by a-particles.
140 Xe ~ 140 Cs ~ 140 Ba ~ 140 La ~ 140 Ce
I~B+ iHeC I~C+)H 54 55 56 57 58
Secondary products
13N*+1
7 . on
. During fission, there is always loss of mass which is converted
. L I~~+ +~e. into energy according' to Einstein' equation E = me 2 . There is a
The following. are some of the nuclear reactions in which loss of about 0.215 amu mass during one fission. Thus, energy
radioactive isotopes are formed. released in one fission is equal to 0.215 x 93 L i. e., 200 MeV.
Chain reaction: Whatever are the primary products of
nNa + ~H ~ ~iNa* +)H [~iNa-l3radioactive]
fission of uranium, it is certain that neutrons are always set free.
2~~U + ~n ~ 2~iu* +y [2~iU-l3radioactive] If the conditions are so arranged that each of these neutrons can,
in turn, bring about the fission, the number of neutrons will
I~C+:H ~ I~N*+y[I~N-positronradiciactive]
increase at a continuously accelerating rate until wh;)le of the
25
12 Mg + 4He
2 ~
. 28 Al * + 11 H
13 [2138Al- ]J
A radioactive] material is exhausted. Such type of reaCtion is t:alled. chain
reaction. Ittakes very ~mall time and is uncontrolled. It ends in u.

* Half1ife period of i~p is 3.2 minutes.


168 G.R.B. PHYSICA.L CHEMISTRY FOR COMPETITIONS

terrible explosion due to release of enonnous amount of energy. percentage of 235 U in the sample is increased. The chain reaction
The chain reaction is shown in Fig. 3.7. , can be carried out under two conditions: (a) uncontrolled' (atom,
bomb) and (b) controlled (nuclear reactors).
Nuclear fuels: Nuclear fuels are of two types:
(i) Fissile materials: These, on bombardment with slow
neutrons, directly produce a chain reaction leading to release of
energy. Three fissile materials are in use at present. These are
f-----n 235 U, 239 Pu and 233 U. 235 U is obtained from natural sources
while 239 Pu and 233 U are obtained by artificial transmutation.
(ii) Fertile materials: A fertile material is one which by
itself is non-fissile in nature, can be converted into a fissile
material by reaction with neutrons. 238 U and 232 Th are fertile
materials. 238 U is converted into 239 Pu by the following nuclear
reaction:
238 U + In - - t 239U*~ 239Np*~ 239Pu
92 0 92 (23 min) 93 (2.3 days) 94

Similarly, 232 Th is converted into- ~33 U.


232Th + In - - t 233Th * ~ 233 Pa * 233U
90 0 9 0 , (23 min) 91 (21 days) 92

.......,....,.",-energy Applications of nuclear fission: Three practical


applications of nuclear fission are:
(a) Atomic bomb, (b) Nuclear reactor and (c) Power plant.

Fissile material Rssile material

Fig.3.7 Chain reaction in uranium-235

Before
The chain reaction is seif-propagating if theva.lue of explosion
multiplication factor is more than I.
Multiplication factor, ~
No. of neutrons produced in one step
K;::-------------
No. of neutrons produced in preceding step
At the time
The value of K is 2.5 for 235 U and 0.5 for 238 U. This shows of explosion
that if all other factors ate ignored, natural uranium which is a,
mixture of three isotopes CZ38 U = 99.29%, 235 U =0.7%,
234 U = 0.0006%) is not suitable for a chain reaction. The (a)
following two factors hinder the self-propagation of a chain Subcritical
reaction: 235U
Subcritical
(i) Leakage of neutrons from the system.
(ii) Presence of non-fissionable material.
"lithe system IS such that loss of neutrons is more than theIr ·-1- l
' 235 U'

I' .J...--" SUDcritioal


production, it is a subcritical stage. When the loss of neutrons is
equal to their production, it is said to be'the critical stage and
when loss of neutrons is less than their production, it is the
over-critical stage. Over-critical stage is required for
self-propagation of chain reaction. The leakage of neutrons from
00 "
~/-'
(b)
~I 235U

TNT explosive

(0)

the system can be reduced by suitable choice of size and shape of Fig. 3.8 (a), (b) a'nd (c) various designs used in the assembly
the fissionable material. The second source of loss of neutrons is of atom bomb. (The atom bomb is made in two or more
due to absorption of neutrons by non-fissionable material. It may pieces of the fissile material each smaller than the critical
be reduced by careful purification of natural uranium, i. e.,natural size. The moment these pieces are forced together, the
uranium is submitted to the process of enrichment by which the bomb explodes with terrific violence).

RADIOACTIVITY AND NUCLEAR TRANSFORMATION 169

(a) Atomic bomb: It is based on uncontrolled chain


reaction. The shape and size of the fissionable material is so
adjusted at the time of explosion that it reaches the over-critical
sta~e. In the atom bomb, a few pounds of fissionable material
e 3 U or 239,pU) is taken in the form of a number of separate
pieces; each piece is in subcritical stage (surface area is very
large, i. e., loss of neutrons is high.) At the time of explosion,
these pieces are driven together rapidly by using explosives like
TNT (trinitro toluene) lying behind each of 235U pieces as to
make one large piece of fissionable material. At this instant, the
over-critical stage is achieved and a fast chain reaction is set- up.
This results in a violent explosion with the release of tremendous I
amount of enefg'Y. Fig. 3.8 shows some of the designs of atomic·
bomb.
On account of explosion, the fragments fly apart with
tremendous speeds. These collide with each other and kinetic
S.&B +
J)-particle
+
13-particle
energy is changed to heat energy. The amount of energy liberated
in an atomic. explosion is of the order of the detonation of about Fig. 3.9 Nuclear fission ii" '111C!$ar reactor using enriched
20,000 or 30,000 tons of TNT raising the temperature to about ... uranium
107o C. Air expands suddenly and a shock w~ve of great next most efficient one is heavy water (D 20) but this is so
destructive impulse travels across. The explosion also produces a expensive that it has been used only in research reactors.
violent and intense blast of highly penetrating y-rays which are (iii) Control rods: . Boron or cadmium steel rods are used as
exceedingly dangerous. The radioactive dust (fallout) scatters control rods. These rods absorb neutrons and thereby control the
over wide areas causing contamination. rate of fission, e.g., .
The first atomic bOmb dropped over Hiroshima city during the
second World War in 1945 utilised 235 U and the second atomic 5 B 10 + 0 n 1 ~ 3Li·7 + 2 He 4
bomb dropped on Nagasaki made use of 239 Pu. India exploded
(iv) Cooling system: Liquid alloy of sodium and potassium
their first atomic bomb at Pokhran in Rajasthan in May 1974,
is used as coolant; it takes away the heat to the exchanger. Heavy
and used 239 Pu as the fissionable material.
water, polyphenyls and carbon dioxide have also been used as
Nuclear Power and India (Recent Developments) coolants.
Indian scientists recently repeated the history of II th May (v) Shielding: The reactor is enclosed in a steel
1974. Our great scientists successfully conducted five containment vessel, which is housed in a thick-walled concrete
underground nuclear tests at Pokhran range in Rajasthan, 24 building. Operating people are protected by a shield of
years after the nation had conducted the first such test. Three tests compressed wood fibres.
were conducted at 3.45 p.m. on 11th May 1998 and the two tests Nuclear reactors are used:
were made later on 13th May. These tests were up to the mark and L To produce 239 pu and 233 U : It is predicted that our
as per our expectations. limited supply of 235 U will last only another 50 years. However,
(b) Nuclear reactor or atomic reactor or atomic non-fissionable 238U and 232Th are plentiful and can be
pile: The reactor is the furnace of the atomic age, the place • converted into 239pu and 233 U . This conversion can be done in
where fissionable material is burnt for useful purposes. It is special type of reactors called breeder reactors. These reactors
essentially an instrument designed to allow a nuclear chain to not only produce large quantities of heat from fissiou but also
develop, under control. All the neutrons produced are not generate more fuel than they use because neutrons are absorbed
allowed to carry out the chain reaction. A fission reactor has five in a thorium or uranium blanket to form 233U and 23~PU. This
main components: (i) fuel, (ii) moderator, (iii) control rods, type of reactor requires the use of fast neutrons; no moderator is
(iv) cooling system and (v) shielding. needed, but control is more difficult. Heat must be transferred
(i) Fuel: Either imrlched uramum or natural uranium is veryeff'rciently because 239Pu melts at a relatively low
u:sually used as fueL Heterogeneous reactors employ the fuel in temperature of 640°C. The process in which non fissile Nuclei
the form of rods, plates or hollow cylinders. Homogeneous 2~~ U and 2~t Th are converted to fissile nuclei in breeder reactors
reactors employ solution of the fuel prepared in the moderator.
is given below:
(ii)- Moderator: The most efficient fission reactions occur.
with slow neutrons. Thus, the fast neutrons ejected during fission
must be slowed down by collisions with atoms of comparable
2~~U+ ~n ~ 2~U~ 2~~Np
mass that do not absorb them. Such materials are called
. moderators. The most commonly used moderators are ordinary 232 Th
90
+'1 n ~
0
232 Th
90 2~Pa~ 2~U
water and graphite. The most efficient moderator is helium. The
170 G.R.B. PHYSICAL CHEMISTRY FOR COMPETITIONS

2. To produce a strong beam of neutrons: These neutrons called thermonuclear reactions. Fusion reactions are highly
are used lor making various isotopes which do not occur in .exothermic'in nature because loss of mass occurs when heavier
nature. For example; i~p and ~~Co are produced from the nuclei is formed from the two lighter nuclei. To initiate a fusion
following nuclear reactiops: reaction is difficult, but once it is started, its continuity is
maintained due to huge release of energy. Some examples of the
52 Te + In
130
0 --1 131 + °e
531 -I fusion reactions are given below: .
31 P , 32p tH+ tH--1 iHe+24.9MeV
15 + on --1 15
32S+ 'n 32p+ IH
iH+ ?H--1 iHe + 2 hn + 11.0 MeV
16 0 --1 15 I l H+ fH--1 iHe + 20.0 MeV
59Co+'n
27 0 --1 ~Co tH+ iH--> iHe+ hn + 17.8 MeV
The non-radioactive isotope is taken iil aluminium capsule jLi + ~H--1 2 iHe + 17.7 MeV
which is placed inside the aluminium balL The ball is rolled into Hydrogen bomb is based on fusion reactions. Energy released
the reactor'where it is bombarded by neutrons slowed down by is so enormous that it is about 1000 times that of anatomic bomb.
paraffin wax. The bombardment is continued for required period, In hydrogen bomb, a mixture of deuterium oxide (D 20) and
which varies from element to dement. tritium oxide (T2 0) is enclosed in a space surrounding an
(c) Power plant (to generate electricity): The heat produced . ordinary atomic bomb. The temperature produced by the .
is utilised in generating steam which runs the steam turbines. The explosion of theatomiCoomb iriitiafestherusionreactlon--
electric generator is connected to the turbirie. The electric power
is obtainedfrorri the generatpr. The atomic reactor when used for
f t
between Hand H releasing huge amount of energy. The first
hydrogen bomb was exploded in 1952. So far, it has not been
.production ofelectricity is termed power plant. possible to bring about fusion under controlled conditions.
The first nuclear reactor was assembled by Fermi and his It is believed that the high temperature of stars including the
co-workers at the University of Chicago in the United States of sun is due to fusion reactions. Bethe and Weizsaeker, in 1939,
America, in 1942. In India, the first nuclear reactor was put into proposed that a carbon-nitrogen cycle. is responsible for the
operation at Trombay (Mumbai), in 1956. production of solar energy in which hydrogen is converted into
helium. The cycle is:
I~C+ lH-->I~N+y
13 N --1 13 C + +10 e
7 6

I~C+ lH-->ljN+y
I~N+ ~H-->I~O+y
I50--> 15 N + De
8 7 +1
Condenser 15 N +
7
IH --1 126 C+ 4He+y
I. 2
1 4 0 .
or 4 IH--> 2He+2+le+24.7MeV
I~ C acts as a kind of nuclear catalyst.
E. Saltpeter, in 1953, proposed· a proton-proton chain
reaCtion:
IH IH
1 + 1 -->1
2H + +Ie+y
0

Fig. 3.10 Power plant : Application of nuclear fission for the


~H+ ~H--> ~He + y
.. production of electricity

3.16 NUCLEAR FUSION ~He+ lH--> iHe+ +~e+y


A nuclear reaction in which- two lighter nuclei are fused
together to form a heavier nuclei is called nuclear fusion. In
such a process, more stable-nuclei come into existence as binding
As a potential source of commercial electrical power; the
energy per nucleon increases (see sec. 2.23). A fusion reactian is
fusion process has several advantages over the fission reaction.
difficult to occur because positively charged nuclei repel each
(i) The quantity of energy liberated in the fusion is much greater
other. At very high temperatures of the order of 106 to 107 K, the
than in fission. (ii) The products of fusion are non-radioactive.
nuclei may have sufficient energy to overcome the repulsive
Fission produces many unstable radioactive products. Fission
forces and fuse. It is for this reason, fusion reactions are also
reactors, therefore, pose a waste-disposal problem.
I
RADIOACTIVITY AND NUCLEAR TRANSFORMATION \"171
98 "'2' 99 1
, .
Difference between Nuclear Fission and Nuclear Fusion 42Mo+ IH~ 43 Tc + on

92 U+ I2H~ 238Np+i
Nuclear Fusion '
0 n,
Nuclear Fission 238 93
(i) This process oCcurs' in This process occurs' in lighter
Elements 93 and 94 were produced using neutrons (obtained
heavy nuclei. nuclei.
during fission) instead of accelerated positive nuclei. Neutron
Oi) The heavy nucleus splits The lighter nuclei fuse together to capture by 238U followed by f)-emission gives isotopes with mass
into lighter nuclei of form a heavy nucleus. number 239.
comparable masses,
(iii) The binding energy per The binding energy per nucleon ~~U+ ~n~ 2~U~ 23~Np~ 23~pu
nucleon increases. increases.
2~Pu kana-emitter with half life of2.4 x 104 years.
(iv) This reaction occurs at This occurs at a very high'
, ordinary temperature, temperature. Americium is formed in a similar way.
(v) The energy liberated in one The energy liberated in one fusion is /- 239 P (n, y) 240 P (n, y) 241 Pu -II 241 Am
94 U ~ 94 U ~ 94' ~ 95
fission is about 200 MeV. about 24 MeV.
(vi) This can be controlled. This cannot be controlled. As Z increases, the efficiency of nuclear reactions with
neutron bombardment falls sharply. Instead, nuclides in the
(vii) Products of fission are Products of fusion are usually Z = 95 to 99 range are bombarded with beams of helium nuclei
usually unstable stable and non-radioactive in
accelerated in the cyclotron to form nuclides with atomt_~'_c,~__ ~~
radioactive in nature. nature.
numbers 96 to 10 1.
(viii) Percentage efficiency is Percentage efficiency is high.
4He~ 10 n
less. % efficiency 94 Pu +
239 2
242Cm+
96
% efficiency =~x 100=0.38
= 200 x 100= 0.09 5x 931 2~~Am + iHe ~ 2~~Bk + 2~n
236x 931 [rH + ~ -t iHe + ~n
+ 17.8 MeV} 2~~ Cm + iHe ~ 2~Cf + ~n
(ix) The links of fission The links of fusion reactions are,
reactions are neutrons. protons.
2~Cf+ iHe~ fgciFm+2~n

2~tEs+ iHe~ rgfMd+~n


3.17 SYNTHETIC ELEMENTS INCLUDING
TRANSACTINIOES Beyond element with Z I 0 ~ increasingly heavier nuclei are
used as projectiles. These projectiles are accelerated by linear
Elements 43 (technetium), 61 (promethium), 85 (astatine) and all rather'than circular accelerators. Examples of nuclear reactions of
elements with Z> 92 do not exist naturally on the earth, because this type are the following:
no isotopes of these elements are stable. The elements coming- 246 12 . 254 41
after uranium (Z 92) are named transuranic or transuranium 96 Cm + 6C~ 102 N0+ on
elements. The actinide series which starts with the element
thorium (Z = 90) is complete at the element lawrencium 2~~Cf + IJB ~ rgjLr+ 6~n
(Z =103} The elements with Z 104 -112 have been reported
recently and are transition (d-block-fourth series) elements. 2~~ U + liN ~ 2~Es+ 3~n
These are called transactinides or super heavy elements. After
238 160~ 250Fm+41 n
the discovery of nuclear reactions early in the twentieth century, 92 U+ 8 100 0
scientists between 1937 and 1945, set out to make the missing
The superheavy elements have been discovered by
elements, i. e., technetium, promethium and astatine and three
bombardment with medium weight nuclei. For example, the
members of the actinide series, neptunium (Z =93~ plutonium
elements with Z:::: 107 and Z =109 have been obtained by
(Z = 94) and americium (Z = 95} The missing elements and all
bombardment of 2~';Bi with accelerated ~Cr and ~~Fe
the elements above atomic number 92 are called synthetic
elements as ,these have been synthesised by artificial respectively.
transmutation, i. e., by nuclear reactions. The credit for the 209 • 54C 261 21
83 B1 + 24 r~ 107 U ns+ on
discovery of most of the transuranic elements goes to Seaborg.
Much less is known about synthetic elements as these are 2% Bi + ~~Fe ~ rMUne + ~n
radioactive and short-lived. This is also due to their limited
availability. The production of synthetic elements requires The elements with Z = 104, 105, I 06 and 108 have also been
binuclear reactions between two positive nuclei that must be reported by the applications of the following reactions:
fused together against the force of electrical repulsion. Nuclear
accelerators were used for this purpose. High energy deuterons 24/sCf+ I~C~ ~JJUnq +4~n
were used to increase the atomic number of target nuclei by one
249 ~ 260Unp + 4 1 n
unit. 98 Cf + 15N
7 105 0
172 G.R.B. PHYSICAL CHEMISTRY FOR'COMPETITIONS

249 Cf 18 263 . I .
+ 80 ----;I06Unh+40n
98
21. A nuclear.reaction of 2t~ U with a neutron produces ~~Kr and
208
82
Pb + 58F
26 e
265 I
----; 108 Uno + on two neutrons. Other element produced in this reaction is:
(a) l~iTe (b) l~CS .(c) l~~Ba . (d) l~Ba
The elements up to 100 (fermium) undergo radioactive decay (VITEEE 2007)
mainly by emitting a.-particles or ~-particles. The elements [Ans. (d)]
become increasingly unstable as the atomjc number increases and
nobelium has a half life of only t:hTee seconds. With these heavy 235 U
36Kr + 56Ba + 2 ~n]
.
[Hint: + Ion ---+ 90
92
144

elements, spontaneous nuclear fission becomes the most 22. The product P of the nuclear. reaction
important method of decay. 252Cf could become a valuable 235U I P + 36Kr
92 1·
92 + on ~ + 3(on)lS;
neutron source.
The IUPAC names for the elements Z> 100 have been (a) I~JSr (b) I~JLa (c) l~~Ba (d) I~JCS
given below: [JEE (WB) 2008J
Unu Uns [Ans. (c)]
10 1 Unnilunium 107 Unnilseptium
[Hint: Let symbol of element is 1£P.
102 Unnilbium Unb 108 Unniloctium Uno
92=Z+ 36
103 Unniltrium Unt 109 Unnilennium Une
Z=56
104 Unnilquadium Unq 110 Ununnilium Uun 235 + 1 M + 92 + 3
105 Unnilpentium Unp III Unuriunium . Uuu'--
M= 141
106 Unnilhexium Unh 112 Ununbium Uub Thus, the element P will be IfJ Sa]
Elements with an even number of protons in the nucleus are
usually more stable than their neighbours with odd atomic 3.18· APPLICATIONS OF RADIOACTIVITY
numbers, i. e., they are less likely to decay. Also nuclei with both
(8) Use ofy-rays: y-rays are used for dhiinfecting food grains
an even number of protons and an evyn number of neutrons are
and for preserving foodstuffs. Onions, potatoes, fruits and fish,
more likely to be stable. A nucleus is more stable than average if
etc., when irradiated with y-rays, can be preserved for long,
the numbers of neutrons or protons are 2, 8, 20, 28, 50, 82 or 126.
periods. High yielding disease resistant varieties of wheat, rice,
These are called 'magic numbers'and can be explained by the
groundnut, jute, etc., can be developed by the application of
~hell ~tructure of the nucleus. This theory also requires the
nuclear radiations. The y-radiations are used in the treatment of
mclUSIOn of numbers 114, 164 and 184 in the series of magic
cancer. The y-radiations emitted by cobalt-60 can burn cancerous
numbers. The stability is particularly high if number of Rrotons
cells. y-radiations are used to sterilize medical instruments like
and the number of neutrons are magic numbers. Thus, 2 8~ Pb is
syringes, blood transfusion sets, etc. These radiations make the
very stable with 82 protons and (208 82) 126 neutrons. This
rubber and plastics objects heat resistant.
suggests that nuc1idesas Uuq (Z = 114, A = 278), Uuq (Z = 114,
(b) The age of the earth: The age of the earth has been
A ~ 298) ~d Ubh (Z = 126, A 310) might be stable enough to
eXist. ConSiderable efforts are being made to produce elements esti.mat.ed by uranium dating technique. The uranium ore (rock)
which IS found in nature is associated with non-radioactive lead
114 and 126 but the present techniques have so far only
succeeded in producing unstable isotopes. The elements up to which is believed to be the end product of radioactive
disintegration of uranium. A sample of uranium rock is analysed
Z 112 have been reported so far.
. for 238 U and·)06 Pb contents. From this analysis, let the quantities
~'OF OBJECTIVE QUESTIONS in mole-be N = 238U mole, No 238U mole + 206 Pb mole.

19. The radioactive isotope ~~Co which is used in the treatment of . Applying disintegration equ~tion,
cancer can be made by (n, p) reaction. For this reaction, the if' 2)03 log 10 -;
target l1J.lcleus is:
(a) ~Ni~ (b) i~Co
238U + 206 Pb
(c) ~Ni (d) ~~Co 2.303 log 10 ---::----
IManipal (Med.) 2007j . U
[Ans. (c)] 206Pb]
== 2.303 log 10 1 + - -
[Hint: ~Ni + ~n ---+ ~~Co + :H] [ 238U

· The value of 't' can be calculated by putting the value of)"


20. 147 N IS attack ed by doubly ~harged. helium ion, it emits a · h' 0.693 .
whIC IS equal to - - .
proton and: tll2
(a) I~F (b) I~O (c) I~O (d) I~F
'So, 2.303 x [ 2 0 6 Pb]
iJEE (Orissa) 2007j t= logJ01+--
0.693 238U
[Ans. (b)l
Here 't' corresponds to the age of earth which has been found to
be 4.5 billion years.
RADIOACTIVITY AND NUCLEAR TRANSFORMATION 173

Example 27. A sample of uranium mineral was found to Solution:


"
We know that, t =
2.303 x rN )
log \ NO
contain 206 Pb and 238 U in the ratio of 0.008 : 1. Estimate the age 0.693
of the mineral. (Halflife ~f238 U is 4.51 X 10 9 years)
Given, N
Solution: 2 303t 1!2 log [206
We know that, t ==' 1+ ~Pb] 5
0.693 .U 2.303 x 5577 I 5
So, t= og
0.693
Given, tl/2 == 4.51 X 10 9 years
or t == 2.303 x 5577 x 0.6989= 12953 years
Ratio by mass of 206 Pb: 238U 0.008: 1 0.693
Example 29. A piece of wood was found to have 14 cP C
Ratio by moles of 206 Pb: 238 U = 0.008 : _1_ = 0.0092 ratio 0.6 times that in a living plant. Calculate the period when
206 238 the plant died. (Half life of 14 C. ~ 5760 years)

So, t == 2.303 x 4.51


9
x 10 log [1 + 0.0092] Solution: We know that, t =
2.303 x
log
(N '\I
_0
0.693 0.693 N )

0.693
9
2.303 x 4.51 x 10 x 0.00397
"
So, .t= 2.303 x 5760 1og -
0.693 " 0.6
(lJ
0.0412 x 109 0.05945 X 109 years = 2.303 x 5760 x 0.2201
0.693 0.693
Hence, age of the mineral is 5.945 x 107 years. 4213 years
(c) Radio carbon dating: By using the halflife period of (d) Potassium-Argon method: The decay of radioactive
14C, it is possible to determine the age of various objects. In potassium isotope to argon is widely used for dating rocks. The
living material the ratio of 14 C to 12 C remains relatively constant. geologists" are able to date entire rock samples in this way,
When a tissue in an animal or plant dies, 14 C decreases because because potassium-40 is abundant in micas, feldspars and
the intake and utilization of 14C do not occur. Therefore, in the hornblendes. Leakage of Argon is however problem if the rock
dead tissue the ratio of 14C to 12C would decrease, depending on has been exposed to temperature above 125" c."
the age of the tissue. The age of the dead tissue is determined in (e) Rubidium-Strontium method: This method of dating
the following way. A sample of dead tissue is burnt to carbon is used to date ancient igneous and metamorphic terrestrial rocks
dioxide and the carbon dioxide is analysed for the ratio of 14C to as well as lunar samples. It is based on disintegration by beta
12c. From this data, the age of the dead tissue can be determined. decay of 87 Rb to 87 Sr. This method is frequently used to check
Thus: potassium-argon dates, because the strontium daughter element is
~
fI.
2.303 IoglO l/-N o J " not diffused by mild heating like argon.
t N IUJ.IS'reATIDNS OF OBJECTIVE OUIESTltONS
0.693
- - Ioglo - \)
2.303 0 (N 23. A wooden artifact sample gave activity of 32 I)-particles per
tll2 of C 14 t N second while the freshly cut wood gave activity of 64
No =Ratio ofC 14 I C 12 in green plant or atmosphere I)-particles per second in G. M. counter. Calculate the age of
the wooden artifact (t1/2 of 14C 5760 yrs) :
N Ratio of C 14 I C 12 in wood (a) 11520 yrs (b) 5760 yrs
or (c) 2880 yrs (d) 1440 yrs
[Ans. (b)]
No Activity ~f green plant per unit mass
N == Activity of wood per unit mass
[Hint: 0.693 == 2.303 l~glO (No)
tl/2 . \N
Although, the met\1od is suitable to a variety of organic
materials, accuracy depends on the half life to be used, variations 0.693 == 2.303
5760
10glo. (64)
32
in levels of atmospheric carbon-14 and contamination. (The half
life raaio cathi)n was redefined from 5570 ± 30years to 5730 ± 40 tage 5760 yrs 1 . "
years by IUPA.C). The rapid disintegration of carbon-14 24. The analysis of a rock shows thatthe relative number of 206 Pb
generally limits the dating period to approximately 50,000 years. and 238 U atoms is Pb/U == 0.25. If tl/2 of 238 U is 4.5 X}09
Example 28. The m;,Junt of "I: C isotope in a piece of yrs, then the age of the rock will be: .
wood is found to be one-fifth of that present in a 'fresh piece !Jf (a) 2.303 (4.5 x 109 ) log (~)
" 14 0.693 4
wood. Calculate the age ofwood. (Halflife of" C 5577 years)
I
174 .1 G.R.B. PHYSICAL CHEMISTRY FOR COMPETITIONS

(b) 2.303 (4~5X 109 ) 10 (.!.\J [H' t·


In. ---m--- - 0)
0.693 _ 2.303 Iog (N
0.693 ,g 4 t l/2U tage N
(c) 2.303 (4.5x 109 ) log (4) 0.693
, 0.693 , 2.303
-.-loglO (4)
-
4.5 x 109 tage ' I
(d) 2.303 (4.5 x 109 ) 10 (~J
0.693 g 5 , t.ge = 9 x 10 9
years]
[Ans.(a)] (t) Use of radioisotopes (tracers): Tracers hav,? been used
Pb Pb in the following fields:
[Hint: 0.25;· :. 1 + 1.25
U 'U (i) In medicine: Radioisotopes are used to diagnose many
U + Pb =1.25 diseases. For example, arsenic-74 tracer is used to detect the
U presence of tumours; sodium-24 tracer is used to detect the
No
-=L25=-
5 presence of blood clots and iodine-131 tracer is used to study the
N 4 activity of the thyroid gland. It should be noted that the radioactive
0,693 _ 2.303
- - - - - 1o g - (No) isotopes used in medicine have very short half life periods.
til 2 t N 9Q Y: This isotope is used in the treatment of joint effusion

0.693 - _ 2.303 and arthritis.


- - 1og (5)
-
4.5 X 109 t.ge 4 59 Fe: Used in the detection of anaemia.

. 2.303· 9 (5) 32 P: This isotope is used in the treatment of polycythaemia,


tage= (4.5xlO ) log - ]
0.693 4 thrombocythaemia, skeletal metastasis, prostate SR
25. Assuming that about 200 MeV of energy is released per and breast SR.
fission of 23';2 U nuclei, then the mass of 235 U consumed per Nuclear Medicine Scan: It is an advanced nuclear
technology used in diagnosis of diseases. Magnetic Resonance
day in a fission reactor of 'power I megawatt will be Imaging (MRl), a diagnostic medical imaging technique utilizes
.approximately:
the principle of nuclear magnetic resonance. The first images
(a) 10-2 g (b) I g (c) 100 g (d) 1000 g using magnetic reso;nance were published in early 1970s, and
[Ans. (b)] medical applications have accelerated in the world during the
[Hint: I MW = I d' x 24 x 60 x 60 J decade of 1983 to 1993. MRI is now a most. versatile, powerful
. 24x6x6x108 and sensitive diagnostic imaging modality available. Its medical
Number of fissions = = 2.7 x 1021
200 x importance can be summarised briefly as having, the ability to
Mass of uranium = 2.7 x 1021 x 235 x 1.66 x 10-24 == 1.05 g] non-invasively generate thin· section, functional images of any
part of the body at any angle and direction in a relatively short
26. What is the binding energy of the hydrogen nucleus?
period of time. MRI a1so visualizes the heart with exquisite
(a) Zero (b) 13.6 eV
anatomical detail at any angle and direction.
(c) More than 13.6 eV (d) Infinite
[Ans. (a)] The principle of MRI is applicable in human body because we
are all filled with small biological magnets, the most abundant and
[Hint: Nucleus of hydrogen has only one proton; hence its
binding energy will be zero.] responsive of which is the nucleus of hydrogen atom, the proton.
27. Which of the following is not the inverse square law force? Computerized Axial Tomography: . Computerized Axial
(a). Electric force (b) Gravitational force ,Tomography (CT or CAT),non-invasive diagnostic technique
(c) Nuclear force uses a· type of X-ray device that provides a clear view of soft
(d) Magnetic force betWeen two poles internal organ tissues in'the body. CT is used to diagnose various
[Ans~ (c)] conditions, in particular cancer. A CT scan. is the' computer
[Hint: Nuclear forces are short range forces which do not analysis of a sharply limited, thin X-ray beam paSsed
obey inverse square law.] . circumferentially through an area of the body, producing a
28. Lead is the final product formed by a series of changes in cross-sectional image, or slice.
whicb the rate determining stage is the radioactive decay of The mod~rn CT scanner comprises five major parts. A
uranium-238. This radioactive decay is first order with balf . high-speed X-ray tube cooled by oil, air and water forms the
life of 4.5 x 10 9 years. What would be the age of a rock X-ray source. Its X-ray detector, normally a bank of about 1,000
sample originally leadfree, in which the molar proportion of solid state-crystal microprocessors coated with caesium iodide,
. uranium to lead is now I: 3? IPET (Kel'aia) 20061 receives the attenuated X-ray signal as it passes through the
(a) 1.5 x 109 years , (b) 2.25 X 10 9 years various tissues and bones of the patient being examined. The
(c) 4.5 x 109 years (d) 9 x 109 years signal is electronically converted to binary data, which is read by
9
the computer-the heart of the CT imaging system. The CT has a
(e) 13.5 X 10 years gantry, a framework that is mounted in such a way that it
[Ans. (d)] surrounds the patient ina vertical plane, and contains a rotating
. sub-frame onto which the' X-ray source and detectors are
RADIOACTIVITY AND NUCLEAR TRANSFORMATION 175

mounted. A patient table (or couch) is positioned perpendicular thickness of materials, to test the wear and tear inside a car engine
and axial to the gantry so that it is able to travel along that axis. and the effectiveness of various lubricants. Radioactive carbon
Topographic images are produced by using an X-ray source has been used as a tracer in studying mechanisms involved in
and a detector moving in a coupled way relative to the patient. In many reactions "of industrial importance such as alkylation,
CT a thin fan beam of radiation rotates in a circular or spiral polymerisation, catalytic synthesis, etc.
motion "around the patient. Thousands of projected X-ray signals (iv) Analytical studies: Several analytical procedures can
are reconstructed by computer algorithms to produce digital CT be used employing radioisotopes as tracers.
. images, displayed by a high-resolution monitor. In this way the 1. Adsorption and occlusion studies: A small amount of
whole body can· be imaged from head to toe. radioactive isotope is mixed with" an inactive substance and the
activity is studied before and after adsorption. Fall in activity
Radiation Dosage in the Radiotherapy of Cancer
gives the amount of substance adsorbed.
Radiations and the particles emitted by radioactive nuclei are 2. Solubility of sparingly soluble salts: The solubility of
harmful for living organisms. These radiations cause genetic lead sulphate in water may be estimated by mixing a known"
disorders by affecting DNA. amount of radioactive lead with ordinary lead. This is dissolved"
Effect of biological radiations can be measured in terms of the in nitric acid and precipitated as lead sulphate by adding
unit called RAD. sulphuric acid. Insoluble lead sulphate is filtered and the activity
RAD = Radiation absorbed dose of the water.is measured. From this, the amount of PbS0 4 still
I RAD = The radiation which deposits I x 10-2 J of present in water can be estimated.
energy per kilogram of tissue. 3. Ion-exchange technique: Ion exchange process __oL
In order to measure biological destruction by radiation, an separation is readily followed by measuring" activity of
successive fractions eluted from the column.
other unit REM was introduced.
4. Reaction mechanism: By labelling oxygen of the water,
REM = RAD x RBE
mechanism of ester hydrolysis has been studied.
RBE = Relative biological effectiveness
RBE for a-particle = 10 unit ·#0 * ~O
RBE for ~ and y radiation = 1 unit
R-C( +HOH ~ R-~ * +R'OH
OR' OH
RBE for neutron = 5 unit
(ii) In agriculture: The use of radioactive phosphorus 32 p " 5. Study of efficiency of analytical separations: The
in fertilizers has revealed how phosphorus is absorbed by plants. efficiency of analytical procedures may be measured by adding a
This s"tudy has led to an improvement in the preparation of known amount of radioisotope to the sample before analysis
fertilizers. 14C is used to study the kinetics of photosynthesis. begins. After the completion, the activity is again determin~d.
The comparison of activity tells about the efficiency of
(iii) In industry: Radioisotopes are used in industry to
separation.
detect the leakage in underground oil pipelines, gas pipelines and
water pipes. Radioactive isotopes are used to measure the

MISCELLANEOUS NUMERICAL EXAMPLES~"d//


Example 1. One mole of A present in a closed vessel Example 2. 131 1 has half life period 13.3 hour. After 79.8
undergoes decay as: hour, what fraction of 131 I will remain? leBSE' (PMT) 211051
zm A ~ m-8
Z - 4B
+ 2(4He)
2

What will be the volume of helium gas colz"ected at STP after 20


days (t1l2 of A = 10days)?
Solution: N =. N o Gr
Solution: We know that,

N = No ("2lJn where, N = remammg


.. mOI~ of A :0 =Gf 6~ "'\
Example 3. A sample of 14 CO 2 was to be mixed with.

N=l(~r 4
ordinary CO 2 for a biological tracer experiment. "Ii
order that
10 3 cm 3 of the diluted gas at NTP should have 104 dis/min, how
many !-lCi of radiocarbon-14 are needed to prepare 60 L of the"
." 1
N UllU:,;:,r 0"fd"
ecayed mo 1es = 1- -1 = -3 diluted gas?
4 4
Number ofrnoks of helium formed Solution: 10 cm 3 of the diluted gas at NTP

,; 2 x number of c;:>cll'''::d moles of A = 2 x


" - "4
2 =22 = 104 dis/~in =~ dps
60
Volume of helium at STP = 2x 22.4= 33.6 litre :.60 L (60,000 Z~3)ofthe dilute gas atNTP
2
176 G.R.B. PHYSICAL CHEMISTRY FOR COMPETITIONS

104 X 60,000 d Number of moles of 206 Pb


0.1
60x 10 ps 206
Thus, no. of jlCi of 14 CO 2 needed
Applying the relationship,
4 2.303 [ 206 Pb ]
10 x 60,000 (ljlCi 3.7x104 dps) --log 1 + - -
A 238U
60xlOx3.7xl04
::: 27.03 jlCi
2.303 " -0.1]
Example 4. A radioactive nuclide is produced at a, = - - x 4.5x 109~log 1+ 206
constant rate of 'a. 'per second. Its decay constant,is A. IfN 0 be 0.693 " . 1
the number of nuclei at time t::: O,theriwhat will be the:,
maximum number ofpossible nuclei? 8
r .238

, a. a. A = 7.098 x 10 years
(a) A (b) N 0 + (d) -+No
i (c) No a. , Example 8., Calculate the mass of C 14 (half life 5720
Solution: Maximum number of nuclei w.ill be present when years) atoms which:give 3.7 x 10 7 disintegrations per second.
Rate of decay::: Rate of formation Solution: Let the mass of 14C atoms be mg.
AN a.
a. , Number of atoms in mg of 14C = m x 6.02 x 1023
"or, N=- 14
A
A = 0.693 == 0.693 = 3.84 X 10- 12 sec-I
Example 5. The half life of 212 Pb is 10.6 hour. It under-
half life 5720 x 365 x 24 x 60 x 60
goes decay to its daughter (unstable) element 212 Bi of half life
dN '
60.5 minute: Calculate the time at which the daughter element We know that, - =A·N
will have maximum activity. dt
i. e. , Rate of disintegration A x no. of atoms
Solution: Apb 0.693 = 1.0896 x 10-3 min-1
10.6 x 60 3.7x 10 7 = 0.693 x m x 6.02 x 1023
5720x 365.::<44x'~Q.}( 60~.~4'"
12
0.693
60.5
11.45 x 10-3 min-I = 3.84 X 1O- X'l1fi (j:02 ~;r023>
i
'.
14
2.303 So, m= 2.24 x 10-4 g
t = - 1o g
ABi
-
max ABi - Apb Apb Example 9. Prove that time required for 99.9% decay of a
radioactive species is almost ten times its half life period.
2.303 1 11.45 x 10-3
, X og----~
2.303 No
(11.45 X 10-3 - 1.0896 X 10-3 ) 1.0896 X 10-3 Solution: ' We know that, t = --log -
A N
227.1 min No 100, N'; (100 - 99t9) =0.1
Example 6. A radioactive isotope is being produced at a · reqUIre
: ' delor 999
constant rate x. Halflife of the radioactive substance is y'. After So, T1me . 0/0/ decay, ,2.303 100
t =- - 1og-
sometime, the number of radioactive nuclei becomes constant,
A 0.1
the value of this constant is ..... . ::: 2.303 x 3
Solution': At the stage of radioactive equilibrium, A'
Rate of formation of nuclide = Rate of decay of nuclide ' . d _ 0.693
Half lifepeno --A-
x=AN
x xy
= Time required for 99.9% decay 2.303 x 3 A
(In 2)/ y In 2 So, ---x--
Ha1flife period A 0.693
• Example 7. 2~~ U by successive radioactive decay
changes to 2~~ Pb. A sample of uranium ore was analysed and
Example 10. Half life of a radioactive substance A is two
found to contain 1.0 g of 238 U a~d 0.1 g of 206 Pb. Assuming that
times the halflife ofanother radioactive substance B. Initially the
, all 206 Pb has ac~mulated due to decay of 238 U, find the age of
number of nuclei of A and Bare N A and' NB respectively. After
the ore (half life of 238 U 4.5 x 10 9 yrs). ' three half lives of 'A', number of nuclei of both become equal.
N
~ Solution: Number of moles of 238 U = I The ratio of~ will be:
238 NB
RADIOACTIVITY AND NUCLEAR TRANSFORMATION 177

1 .1
(a) - (c) - No. of atoms which have disintegrated = 0.75 x 2.68 x 10 18
2 3 = 2.01 X 10
18
Solution: We know that, the amount remaining after n half
lives can be calculated as: Example 14. A certain radioisotope ~ X (Half life =10
iY. Ifl g atom of i X is kept in sealed vessel,
Gr days) de~ays to;

r
N=No
how much helium will accumulate in 20days?

Remaining amount of A = NAG


Solution: , .~ X 1- i Y + iHe
Remaining amount of B = N B
r1)6
1-
In two half lives, 3 of the isotope : X has disintegrated, i. e.,
4
\2 2g atom of helium has been formed from 2 g atom of 1X.
NA (~r =N B Gf 4
Volume of 1 g atom of helium
4
22400 mL

So, Volume of 3 g atom of helium 3 x 22400 rnL


4 4

Exarfiple1l. 1.0 g of 19;9 Au (t 112 = 65 hours) decays by 16800mL
t).-emzSsion to produce mercury. Example 15. Binding energy per nucleon of Hand ilie t
(a) Write the nuclear reaction for the process. are 1.1 MeV and 7 MeV respectively. Calculate the amount of
(b) How much mercury will be present after 260 hours? energy relegsed in the follOWing process:
Solution', (a) + 10 e tH+ 1 i H e
79 Au
198 ----) 8Hg
198 0-
Solution: Amount of energy released
(b) No. of half lives in 260 hours = 260 = 4 = k Binding energy of products
65

1)4 1 k Binding energy of reactants


Amount of gold left after 4 half lives = -(2 =-16 g [4x7] [4 x l.1]

Amount of gold disintegrated = 1 _ ~ = 15 g = 23.6 MeV


16 16
Example 16. Calculate the energy assoCiated with the
15 following nuclear reaction:
Amount of mercury formed = - = 0.9375 g
16
?I Al + ;H fiMg + iHe
Example 12. Calculate the probability (P) of survival of a 26.9815 amu 20141 amu 24.9858amu 4.0026amu

radioactive nucleus for one mean life. Solution: Mass defect (26.9815+ 2.0141)
Solution: Probability for survival =..!!....- = e -.1.1 (24.9858 + 4.0026)
. No O.OOnamu

t =mean life =-.!..


Energy of the reaction noon x 931 MeV
A 6.70 MeV
Example 17. A radioactive isotope zA m (t1l2 = 10 days)
b b 'l'ty =e -AxIiA =-1
ProaII
e decays to give Z 6B m 12 stable atom alongwith a-particles. If
Example 13. 1 milligram radium has 2.68 x 1018 ato~. m g of 'A' are taken and kept in a sealed tube, how much 'He 'will
Its half life period is 1620 years. How many radium atoms will accumulate in 20days at STP?
disintegratetrom 1milligram ofpure radium in 3240years? Solution: zA
In
----) Z-6 B m 12 + 3[zHe 4 ]
Solution: No. ofhalflives in 3240 years = 3240 =2 m
1620 Mole of A
m
(~)
2
Amount of radium left after two half lives = 1x Number of half lives = 20/10 2
= 0.25 mg
Amount ofradium disintegrated = (1- 02~\~.· aR5 mg •
178 ·1 G.R.B. PHYSICAL CHEMISTRY .FOR COMPETITIONS

= 1Gf =i Solution:
'gh f d" .
Wel t 0 ra lOactlve potasslUm =
75000 x 0.012 . 0.35
x-
Decayed moles = 1-114 = 3/4 . 100 100
Moles of 'He' fonned 3 x 3/4 = 9/4 = 0.0315 g
9 .. 0.693 X Weight ber
Volume of , He' at STP 22.4 x A cttVlty = x A'
yogad '
ro s num
4 t1/2 Atomic weight
= 50.4litre Activity ;;:; . 0.693 x 0.0315 x 6.023 x 1023
Example 18. A sample ofpitchblende is found to contain 1.3 x x 365x 24x 60 40
50% uranium and 2.425% of lead. Of this .lead only 93% was' = 4.81 X 105 dpm
Pb 206 isotope. If the disintegration constant is 1.52 x 10- 10 yr- I ,
Example 21. The sun radiates energy at the rate of
how old could be the pitchblende deposits?
4 X 1026 Jsec- l . If the energy of fusion process is 27 MeV,
. 50
Solution: Moles ofU 238 = = 2.1 X 10-
3
calculate the amount of hydrogen that would be consumed per
100x 238 day for the given process.
Moles of Pb 206 ..", 2.425 x 93 = 0.109 x 10-3 4 IH ~ iHe+2 ~e.
100 100 x 206
- - -
- " ~'.

N = (x+ y)= 2.1x 10-3 +. 0.I09x 10-3 = 2.209 x 10-3


o Solution: 27 MeV = 27x 106 x 1.6 x 10-:19

N X 2.1 X 10-3 = 43.2 X 10- 13 J


. I~ '-v2.303
- --
1og (N 0 J
~
Energy radiated by the sun per day
t = 4 x 10 26 x 3600 x 24 J day-I
3
1.52 X 10- 10 = 2.303 log 2.209 xl 0- 34.56x 1030 Jday-l
3
t 2,1 X 10-
43.2 x 10- 13 J of energy is obtained from
t = 3.3 X 108 years 4 arnu of H
. Exan . 19. Oll7nalysis, a sample of uranium are was == 4 x 1.66 X 10-24 g of H
found to contain 0.277 g of 82 Pb 206 and 1. 667 g of 92 u 238. The
54.56 x 1030 J of energy is obtained from
half life period of U 238 is 4.51 X 109 yrs. If all the lead were
24
assume.,.d to have come from decay of 92 U 238, what is the age of . 4 x 1.66 X 10- x 34.56 x 1030
the earth? 43.2x

238 1.667 = 5.31x 1019 g


Solution: . M 0 1es 0 fU =--
238 ", Example 22. A radioactive isotope X with half life of
. 'M I . f Pb 206 1.37 x 109 years decays to Y, which is stable. A sample of rock
oeso. =0.277
206 from moon was found to contain both the elements X and Y in the
No = 1.667 + 0.277 ratio 1: 7. What is the age of the rock?
238 206 Solution: We know that,
and N 1.667 0.693 _ 2.303
--- 1og - (No)
.238 tl/2 tage . N
1 t.= 2.303 log
lv'
No
N
1.667 0.277
0.693 . _ 2.303
1 (1
+ 7)
, 9 - - - oglo - -
1.37 x 10' t age 1
9 +--
2.303 x 4.51 x 10 1 238 2Q6 9
0.693 oglO 1.667 tage = 4.11 X 10 years
238 Example 23. A sample of radioactive substance shows an
9
L143 X 10 ye¥s intensity of2.3 millicurie at a time 't' and an intensity of 1.62
millicurie, 600 seconds later. What is the half life period of the
Example 20. 19K 40 consists of 0.012% potassium in
radioactive material?
nature, The human body contains 0.35% potassium by weight.
Calculate the total radioactivity resulting from 19 K 40 decay in a Solution:
75 kg human body. Halflife'of 19K 40 is qx 109 years.
RADIOACTIVITY AND NUCLEAR TRANSFORMATION 179

0.693 :::; 2.303 log ( 2.3) Solution:


11/2 600 1.62
Activity:::; 0.693 x _w_ x 6.023 X 10 23
11/2 :::; 1187 seconds 11/2 At. wt.
Example 24. Wha1 mass of 226 Ra, whose tl/2 :::; 1620 yrs 3.7 x 107 :::; 0.693 x ~ x' 6.023 X 1023
will give the activity ofl millicurie? 1620 x 365 x 24 x 3600 226
so w:::;I0- 3 g

SUMMARY AND IMPORTANT POINTS TO REMEMBER


~_ . . . . up, •• -.; ~.. . .•• up , _•• 'tIt
1. Radius of nucleus is calculated as: (xi) Potassium uranyl sulphate K(U0 2)(S04h was the
R :::; Ro -A 1/3, first compound found to be radioactive.
where, Ro :::; 1.1 x 10-15 m, A :::; Mass number of nucleus i
(xii) Tritium H is the lightest radioactive element.
5. Units of rate of decay:
Area of cross-section of a nucleus is expressed in barns
1 curie (Ci) :::; 3.7 x 1010 dis sec- I
(1 barn:::; 10-24 cm 2).
1 millicurie (mCi):::; 3.7 x 107 dis sec- I ,~ .\
. 3xMass
2. Nucleus denSIty p :::; - - - 1 microcurie ijJ.Ci) :::; 3.7 x 104 dis sec -I
41tRg
1 rutherf~rd (Rd) ~ 106 dis sec- I
Density of all nuclei is constant, nuclear density is very large
(= 1017 kg/m 3 ) compared to atomic density (= 103 kg/m 3 ). 1 millicw;ie (mCi) :::; 37rutherford
1 becquerel (Bq) :::; 1dis sec -I
3. 1 amu :::; 1.66 x 10-27 kg
. In terms of energy, 1 amu =931.5 MeV 6. Kinetic equation of radioactive decay:
4. Rate of radioactive decay is given as: N -N -')../. - oe (Exponential form)
Rate:::; A. x Mass x 6.023 x 1023 1 _ 2.303 1oglO. - (No)
Atomic mass 11.--.- (logarithmic form)
1 N
~- 0.693 x ~ x 6.023 x 1023
.1112 At. wt. A, :::; Decay constant
(i) Radioactivity is the phenomenon of spontaneous N <> :::; Initial amount of radioactive element
emission of certain radiations. It was discovered by N :::; Amount remaining after time' t'
Henri Becquerel in 1895. 'fi 0.693
7. H aIflI e tl/2 :::; -A,-
(ii) Marie Curie and her husband Piere Curie isolated
tWo radioactive elements polonium and radium.
Radium is 2 million times more reactive than Average life 't :::; l : :; ~
uranium, it is the most radioactive element. A, 0.693
(iii) Radium and polonium were isolated from 't :::; 1.44 x 11/2
pitchblende (U 30 g ). 8. Amount re';TIaining after 'n' halflives can be calculated as:
(iv) Francium is a liquid radioactive element in natural
state.
(v) Radon is a gaseous'radioactive element in natural state.
(vi) 2t~ U is ~he heaviest known natural element and it is Total time
n:::;
radioactive. Halflift<
(vii) a-particles evolved from radioactive elements
(i) A radioactive element undergoes 50% decay in one
possess energy up to about 10 MeV. They can
half life.
penetrate an aluminium sheet of 0.02 cm thickness. '
(ii) The time in which 63.2% radioactive element •
(viii) ~rays can 12enetrate an aluminium sheet up to 0.2 cm
undergoes decay is called average life't.
thickness.
(iii) The radio~ctive' element undergoes 99.9% decay in
(ix) y-rays are high energy electromagnetic radiations of
10 times ofhalflife.
. short wavelength of the _order of 10 pm. These are
(iv) An element undergoes 75% decay in twice of the
highly penetrating rays; they can penetrate up to 100
half life.
... cm thick aluminium sheet. -
(v) Total life span of a radioactive element is infinite.
(x) After y-decay, the daughter nuclide is the nuclear
isomer of parent nuclide which differs in half-life.

", "'.
180 GH.B. PHYSICAL CHEMISTRY FOR COMPETITIONS

9. Some radioactive elements undergo a and ~-decay in parallel IH ----t ~n + +~ e + neutrino + energy
path. Positron emission and K -electron capture are similar

C.
223Fr+4He , 'because both processes lower the number of proton by one
87 2
KJ unit.
227 Ac
89 .
18.
K,
- 227
90
Th + _I0 e
N umber of a-particles' x' = --''---:::'' ... (i)
4
Overall decay constant K = K 1 + K 2
Number of~-particles can be calculated using the following
K
Fractional yield of Fr = _,_I relation: '
K
Z, - 2x + Y Z2 ... (ii)
Fractional yield of Th 19. There are three natural and one artificial decay series:
K Uranium series (4n + 2)
10. At equilibrium, 238 U ) 2~~Pb
92 . (ga, 6jl)
A----tB~C ... Thorium series (~n)
AA
Amountof'A'
----'---=-=
232
90
Th
(6a, 4/1)
) 208
.82
Pb
Amount of' B' As tl12A Actinium series (4n + 3)
235 U 207
11. If an element undergo~s simultaneous a and ~-decay, then 92 82 Pb

't a'tl)
Artificial series is also called neptunium series:
,A =Aa + AI); 't =- - - (4n+ I)
't a + 'tf\ 237 Np _ _ _~ 2~Bi
93 (7a,
12. a-particles and y-rays have line spectra, but ~-particles have
a continuous spectrum. 20. If both parent and daughter elements belong to actinide
series (89-103) then they will belong to same group, i. e. ,
13. Geiger-Muller counter is used for detecting a, and
third group.
~-particles, cloud chamber is used for detecting radioactive
238 U 4He ----t 234 Th
radiations and for determining their paths, range and 92 2 90
3rdgroup 3rdgroup
energy. In scintillation counter, the particles of radiations a<f\<y
are detected by the flashes of light produced in the 21. ----7) Increasing penetrating power
scintillator. a>f\>y
14. In every nuclear reaction representing transformation of one ----7) Decreasing ionising power
a>f\>y
nucleus to other, the conservation' of charge number, ----7) Decreasing luminosity on ZnS screen
nucleons, energy c.nd linear momentum is followed. '
22. Emission of one 'a' and two 'W particles form an isotope of
15. a-emission takes place when nl p ratio is lower than required
the parent element :
for nuclear stability.
f'A- iHe-2 °e]----t M ;A
, 4 :H----t iHe + 2[+~ e] + energy
a-particle emission shifts the daughter element two positions 23. There are only 81 stable elements having one or more
left in the periodic table. non-radioactive isotopes. ,
24. No stable isotope exists for the elements above 2~~ Bi. Thus,
zMA 42He ----t zM_ 24B •
bismuth is the heaviest stable element.
(Here, A and Bare isodlapheres to each other.) 25. Two elements earlier than bismuth (Tc and Pm) are
16. ~-emission takes place when nIp ratio is higher than the radioactive.
required value for nuclear stability.
26. Isotop~ Use
~ n ~ : H + _~ e + antineutrino + energy 60 Co Cancerous tumour detection and treatment.
, Emission of I1-particles increases the atomic number by one
Detection and treatment of thyroid
hence, the daughter element occupies one position right to disorders.
the parent element. .
59 Fe Anaemia.
f'A L~e)----tz+~B
;/ 32 P Leucaemia and agriculture research.
Here, A andi B are isobars; thus ~-emission is isobaric
24Na 'Location of blood clots and circulatory
transformation. disorders.
17. In artificial radioactive elements, positrons are evolved 74 As Detection of presence of tumours.
'when nIp ratio is lower than the required value for nuclear
90 Y Treatment of joint effusion and arthritis.
stability.
RADIOACTIVITY AND NUCLEAR TRANSFORMATION 181

18 0Used in the study of mechanism of The nucleus bombarded is called target; the particles used
photosynthesis. for bombarding are called projectiles and the particles
Radioactive Brain scan. emitted are called subsidiary particles.
technetium 32. Particle accelerator: Various particle accelerators are
Note: Radioactive isotopes of carbon, chlorine and nitrogen used to give projectiles like protons, deuterons, a-particles
ate also used in the study ofvarious reactions. and other cationic projectiles having sufficiently high kinetic
27. Radiocarbon dating: This method is used to detennine energy to overcome the electrostatic repulsions of the target
age of wood. nuclei. Commonly used particle accelerators are linear
accelerators, cyclotron and synchrotron. Synchrotron is used
1
F\,
2.303 IoglO
--
(N-oI.1 as proton accelerator.
t N .I
33. Reactions of nuclear transfonnation are represented as:
-0.693
---
14
_ 2.303 IoglO - 0 ) (N ~ Be + iHe ----? l~C + ~n or ~Be (an) I~C
tl/ 2 C tage· N
~rNa + :H----? ~~Mg·+ ~n or nNa(pn) ~~Mg
No = 14a 12C in freshly cut wood or in the atmosphere
~i Al + ~n ----? riNa + iHe or ~iAI (na) ?iNa
or activity of freshly cut wood.
34. Artificial radioactivity was first studied by Irene Curie. In
N 14 C ;12 C in the given sample of wood or activity
this process, a stable nucleus is converted to radioactive
of given sample of wood. isotope on bombardment ors1.litableparl~Cle:· R8:dioactive-
28. Uranium dating or t'ock dating: It is used to calculate the isotope produced undergoes artificial decay.
age of a sample of rock and mineral, i. e. , before how many . 27 Al + 4He ----? 30 P + I n
13 2 15 0
years it was separated from the fire ball of earth. . Stab Ie Unstab Ie
30 p 30S'1 +
2.303 I (N 0 \J
F \ , - - - oglO I -
1 _ 15 ----? 14 0
+1 e
(
t1l2 = 2 .55 mm
.)
t \. N .. 35. Nuclear fission is the process in which a heavy nucleus
0.693 2.303 I
- - oglo -
(N 0) breaks up into two smaller nuclei on bombardment with
neutrons. Energy is released in the process of fission along
tl/2 238 U t age N
with freshly prepared neutrons.

=(2~8 + 2;6)
2~U+ ~n~~~u~ :~:::::::~:
No
N W
238
where, W amount of uranium in the sample 144C 90 21on
w = amount of 206Pb in the sample . 55 S + 37 Rb +

29. The force which binds the nucleons together in the nucleus is Mass defect of the reaction is converted to huge amount of
called nuclear force. These forces are short range forces energy.
operating over very small distances (l fermi, 10- 15 rri). Am (Mass defect) = L Masses of reactants L Masses
Nuclear forces are 1021 times stronger than electrostatic of products
forces. Energy released = Amc 2 .
30. Hideki Yukawa of Japan discovered mesons in 1935. If mass defect is I amu then 931.5 MeV energy is released.
Protons and neutrons are held together by their fast mutual 36. Critical mass: It is the minimum mass of fissionable
exchange. material required that will lead to a self~sust<l.ining chain
IH+ _I01l:----?ln
1 0 fission reaction. For 2t~ U, the critical mass is between 1 to
1 0 IH 100 kg.
on + +111: ----? 1
37. The material which directly undergoes fission is tenned as
:H+ g1l:----?:H
1 0 . 1
fissile material such as 235 U, 239 pU and 233 U. The material
on+ 0!J,----? on which can be converted to fissile material is tenned fertile
31. Artificial nuclear transmutation: Conversion of one material such as 2t~ U and 2~t Th.
element to other by bombardment of a stable element with
high speed subatomic particles. The first artificial 38. Breeder reactors not only involve the fission of 2~~ Ubut also·
transmutation was achieved by Rutherford in 1915 when he converts fertile material into fissile material, e. g., 238 U is
bombarded Ij N with a-particles emitted by 2~~ Po. converted to 239 Pu:
IjN+ iHe----? I~O+:H _~e _~
2~~ U + ~n ----? 2~ U ----? 2~ Np ----? 2~ Pu
27A1
13 +1
on 2 7M
----? IH
12 g + 1
182 G.R.B. PHYSICAL CHEMISTRY FOR COMPETITIONS'

39. Nuclear fission is a chain reaction. If it is uncontrolled, 44. Energy ofa star (sun) is due to nuclear fusion; this energy is
explosion occurs as in the atom bomb. Two or more pieces of called stellar energy. .
. fissile material (235 l) or 239 Pu ) having subcritical mass are 45. Hydrogen bomb is much more powerful than atom bomb and
brought. together rapidly by . means of conventional there is no restriction of critical mass in this bomb.
explosion. The subcritical masses combine to be 46. Neutron activation analysis is a teclmique of fmding .the
supercritical and then chain fission starts,. releasing large trace amount of an ;~lement present with the other. The trace
amount of energy. element is activated by bombarding with neutrons. It is a
40. The controlled chain fission reaction takes place.in nuclear . non-destructive method, e. g. , traces of silver present in lead
. reactors., In the~ reactors the energy is used for peaceful paintings can be detected by neutron activation analysis.
purposes. The heat energy produced in the nuclear reactors 47. Spallation reactions: It is similar to fission but differ in
. can be used to generate electricity. A ~ctor consists of: the fact that they are brought about by high energy.
(i) enriched fuel 2~ U (2-3%). - bombarding particles or photons. A number of smaller
(ii) heavy water (D 2 0) or graphite moderator. It slows particles are released along with the product, e.g.,
down the speed of fast moving neutrons.
(iii) control rods made of boron and cadmium. These 2~~ U + ~He ----+6 }H+ 13 ~n + ~iRa
rods absorb some neutrons and thereby control the
rate of nuclear fission. 48•. The isotope ~ H has nIp = 0 and i H has nIp = 2 which is
(iv) liquid alloy of sodium and potassium is used as a maximum.
coolant. .49. Only l~ C haS zero packing fraction. Packing fraction is
41. Nuclear fusion is the process in which two nuclei of light maximum for hydrogen and minimum for iron.
atoms fuse to fonn heavy nuclei with the liberation large .
-amount of energy. - - - .. Isotopic mass':'" Mass number
Packing fraction = x 10 4
rH+ rH----+ ~He + 23x 108 kJ/mol - . Mass number

rH+ iH----+ ~He+ ~n + 17.2x 108 kJ/mol Elements with negative packing fraction are stable because
42. Fusion reactions are thennonuclear reactions which require sOme of their mass is converted to binding energy.
very high temperature (106 K or more). . 50.I! C is produced in upper atmosphere due to bombardment of
43. Hydrogen bomb involves nuclear fusion. cosmic ray neutrons on atmospheric nitrogen.
RADtOAqJWITY AND NUCLEAR TRANSFORMATION 183

Questions
1.. Match the List-I and List-II and pick the correct answer from (c) Charge number is (r) Nuclear fusion
the codes given below: [PMT (Kerala) 20061 conserved
List-I List-II (d) Mass of products formed (s) Nuclear fissiori
(AtomiclMolecular (Corresponding pairs) is less than the mass of
.~ . species) reactants
(A). fSotopes I. 2~~ Ra and 2~~Ac [E] Match the Column-I with Column-II: .

II
(BtlSobars
".,:'.-~~/
2. t~Ar and igK . Column-I Column-II
.~(y:) lsotones 3. ;Hand tH (a) a-rays (P) Radiations, undeviated
in electric field .
(D) Isoster-s. 4. ~~Uand z~Th
(b) Il-rays (q) Produced when
(E) lsodiapheres 5. COz and NzO electrons strike metal
. (a) A-2, B-1, C-4, D-5, E-3 surface
(b) A-2, B-5, C-l, 0--4,E-3 (c) y-rays (r) Highest deflection in
(c) A-3,~lX--':2, D-5, E-4 electromagnetic field~~.~
(d) A-5, B-4, C-l, D-2, E-3 (d) X-rays (s) Nucleus of helium
(e) A-5, B-3, C-l, D-2, E-4 [F] Match the Column-I with Column-II:
2. Matrix Matching Problems (For lIT aspirants): Column-I Column-ll
[A] Match the Column-I with
.,. Column-II: ,. (a) a-emission (P) Mass number changes
Column-I Column-II (b) Il-emission (q) Atomic number and mass
(a) StabUity of nucleus (P) Depends on mass number are affected
number (c) y-emission (r) Atomic number decreases
(b) Density of nucleus (q) Packing fraction (d) \3+ -emission (s) Atomic number increases
(c) Spin angular momentum (r) Binding energy per
of proton nucleon 3. Write th~ complete nuclear"reactions:
(d) Dimensionless quantity (s) Independent of mass (a) ~Be+ iHe ~ I~C + ............. .
number (b) fH ~ ~He + ............ ..
[B] Match the Column-I with Column-II: (c) IjN + iHe~ I~O+ ............. .
Column-I Column-II (d) 2~iu + ~n ~ ~~Sr + Xe + 3 bn
(a) 21 3rd life (p) 63.2% decay (e) 7L' I
3 I+on~ 2 42H
e+·............ .. "
(b) Average life (q) 75% decay
.(f) 2~~ U + .............. ~ 2~iu ~ 2~jNp + ............ ..
(c) 11 A (r) 2 x tl/2
(g) [jN + ~n ~ fH + ............ ..
(d) Ten times of halflife (8) 99.9% decay
(h) ~ Li + .............. ~ ~Be + y - radiations
[Cl Match the nuclear transformations of Column-I with the
particles emitted of Column-II: (i) rH+ .............. ~ iHe+ ~n
Column-I Column-II (j) 27Al
13 + onI ~ 24N II a + ............ ..
(a) 209 . 4H 21 [A
83B1 + 2 e ~ 85 t + ... (P) :H (k) rjAI + iHe~ + .............. IH
235 U
(I)" 92 + on ~ .............. + 137T
1 .
(b) :Be+ iHe~ I~C+ ... (q) iHe 52 e + 97Z
40 r (fIT 2005) .
(c) ~Mg(~n ... ) tiNa (r) I2H (m) ~~Se~ .............. +2}e (HT 2005)
4. Write the particles"emitted from each nuclide in the following
(d) f~Na (fH ...) r6Ne (s) ~n
reactions:
(1)] Match the Column-i with Column-II: (a) 231Th ~ 23l pa 282JAc
90 (i) 91
Column-I Column-II
(b) 217 At ~ 113Bi ~ 209 T1
(a) Binding energy per (P) \3-decay 85 (i) 83 (ii) 81
nucleon increases (c) 239 U ~ 239Np 2~gpu
92 (i) 93.
(b) Mass number is conserved (q) a-decay
(d) 30p~ 30Si
15 (i) 14
184 G.RB. PHYSICAL CHEMISTRY FOR COMPETITIONS

5. Find the atomic number and mass number of the last member 9. To which group of the periodic table does the last member of
in the following series: the following series belong?
-ct -ct -ct -f:\ -f:\
(a)
226Ra
188 Rn RaA RaE RaC (a) 2~ U 23~Np ~ 2~Pu
ct -'f:\ - fl - ct.•
III Group
(b) fA B----+C~D~E -fl -f:\
(b) 140 Cs ~ 140 Ba ~ 140 La
6. Complete the following: 55 56 57
- ct -f:\
I Group
(a) 238 X ~ Y ~ 91Z ct ct
214 13. -fl (e) 2~1Ra 2~~Rn ~ 2~PO
(b) A~B~84C
II Group
7. Write the equations forthe following transformations:
10. Name the process represented below:
(a) :gK (p, d) (b) 'iN(n, p) (e) ~rNa (a, p)
(a) 23i2 U + ~n 1~~Ba + ~Kr + 3 ~n + 200 MeV
(d) ~Be (a, n).
8. . To which radioactive families do the following nuclides
(b) fH + tH iHe + ~n + 17.6 MeV
belong? (c) ~~Cu + iHe + 400 MeV ~ rJCl + 14 iH + 16 ~n
222Rn, 228Ra, 207Pb, 209Bi, 233Pa. (d) I~B + iHe ~ 137N + ~n
~136c++re.

[A~
1. (c) A-3, B-1, C-2, D-5, E-4 4. (a) (i) J3 (ii) a (b) (i) a (ii) a (c) (i) J3 (ii) J3 (d) (i) +~ e
2. [A] (a-p, q, r);(b s);(c s):(d ql 5. (a) Atomic mass 214, Atomic number 83
[Bl (a - q, r); (b p); (c p); (d - :;) (b) Atomic mass M - 8, Atomic number == Z 2
[C] (a s);(b s);(c-p);(d-q) -ct -13
6. (a) 238 X· ~ 234 y ~ 234 Z
[D] (a-p, q, r, s);(b-p, q, r, s);(c-p,q, r, s);(d-p, q, r, s) 92 90 91
[E](a-s);(b-r);(c p);(d p,q) -f:\
[F] (a - p, r)(b - s)(e - q)(d q)
(b)2~i A 2~jB 2i1c
3. (a) ~Be +iHe ~ I~C + ~n; 7. (a) igK + iH~ ?gK+ ~H (b) IjN + ~n ~ l:C + iH
(b) ?H~ ~He+ _~e;
(e) [iNa + iHe ~ f~Mg + iH(d) ~Be + iHe --,) I~C + ~n
(c) )4N+4He~
7 2
17 0+ IR
8 l'
(d) 22iu + ~n ~ ~~Sr + 1~!Xe + 3 ~n; 8. 222Rn belongs to (4n + 2) family, i.e" uranium family.
228 Rabelongs to (4n) family, i.e., thorium family.
( e) 7L' I
3 l+on~ 2 4H o.
2 e+_le, 207 Pb belongs to (4n + 3) family, i.e., actinium family.
(f) 2g~U + ~n ~ 2~U ~ 2gjNp + je; 209 Bi belongs to (4n + 1) family, i. e., neptunium series.
233 Pa belongs to (4n + 1) family, i.e., neptunium series.
(g) liN + ~n ~ I~C + ~H;
9. (a) 2~Pubelongs to actinide series, hence it is present in III
(h) jLi + :H ~Be + y-radiations;
group.
(i) [H+ iH~ iHe + ~n;
(b) 14~8Ce belongs to lanthanide series, hence it is present in III
(j) fIAI + ~n ~ tiNa + iHe; group.
(k) gAl + iHe ~ f~Si + ~H; (c) 21~Po belongs to VI group.
(I) 2~U + ~n ~ 2 ~n + 1~~Te + ~6Zr; ;; 10. (a) Nuclear fission (b) nuclear fusion (c) spallation reaction
(m) ~~Se ~ ~~Kr + 2 _?e (d) artificial radioactivity.
RADIOACTIVITY AND NUCLEAR TRANSFORMATION 185

• RACTICE PROBLEMS.'
1. Half life of 24Na is 14.8 hours. In what period of time will a Rate =
0.693
x--x
W N
sample of this element lose 90% of its ~t:tivity? tl/2 At. wt.
[ADs. 49.17 hour] 3
6.4 x 107 = 0.693 x 28.207 x 10- X 6.023 X 1023
2. A ~-particle emitter has. a half life of 60.6 min. At any instant tll2 238
of time, a sample of this element registers 2408 counts per
tll2 = 7.729 X lOll sec 2.45 X 104 years
second. Calculate the counting rate after 1.5 hours.
[ADs. 860 counts per sec] (ii) Use, 0.693 = 2.303 loglO (N I 0

3. A radio-isotope l~Phas halflife of 15 days. Calculate the time tll2 t N )

in which the radioactivity of 1 mg quantity will fall to 10% of 0.693


2.45 x
= 2.303 10
5000
r100 )
glo \ N
the initial value.
[ADs. 49.85 days] N 86.7 mg]
4. Consider an a-particle just in contact with 2~~ U nucleus.
8. A radioactive isotope decays as:
Calculate the coulombic repulsion energy assuming that the
distance between them is equal to the sum of their radii. ~A. z -:B~~=~C
[ADs. 24.2 MeV) The half lives of A and B are 6 and 10 months respectively.
5. The activity of a certain sample of radioactive element 'A' Assuming that initially only A was present, will it be possible
decreases to 1/ f2
of its value in 4 days. What is its half life? to achieve the radioactive equilibrium for B? If so, what would
Assuming that, be the ratio of A and B at equilibrium? What would happen if
the half lives of A and B were 10 and 6 months respectively?
fA [HiDt: At equilibrium, ratio of amounts of A and B will be
what mass of the sample will be left over after 24 days if we N A _ tll2 A _ 6 06
start with one gram of' A'? Calculate this in terms of M. NB - tllzB -10 .
[ADS. tl/2 = 8 days; mass of sample left over (1- 3 ~ 5) g] Ifthe halflives of A and B are to and 6 months respectively, then
B will decay faster than 'A', hence equilibrium will not be
, 2.303 1 (No)
[HiDt: A -- oglO - I achieved.)
t N /
9. Lowest level of 14C activity for experimental detection is 0.03
0.693 = 2.303 loglo (..fi) N dis per min per gram. What is the maximum age of an object
tllZ 4 that can be determined by 14C method? The activity of 14C in
tll2 = 8 days the atmosphere is 15 dis per min per gram of 14C

(2:1)" for next part.]


. (tl!Z for 14C 5730yrs).
Use N = No
[Ans. 51379.28 yrs]
10. An a~alysis of a rock shows that relative number of 87 Sr and
6. The half life of 2t~ U is 4.5 x 10 years. Uranium emits an
9
87Rb atoms is 0.052, i.e.,(87 Sr /87 Rb = 0.052). Determine the
a-particle to give thorium. Calculate the time required to get age of the rock. Given that halflife period for~-decay ofRb to
the product which contains equal masses of thorium and Sr is 4.7 x 1010 years.
uranium.
[ADS. 3.43 x 109 years]
[ADS. 4.55 x 109 yrs]
, I 87Rb x
[HiDt: No = 238 + 234
N = I [Hint:
238 y 0.052
0.693 2.303, (No))
Use, - - - - oglO - I x
tl/2 t N }
x +y 1.052
7. 32 mg of pure 2~PU02 has an activity of 6.4 x 107 sec-I.
N 1
(i) What will be the halflife of 2~Pu in years? No 1.052
(ii) What amount ofPu0z will remain if 100 mg Pu02 is kept
A 2.303 J
for 5000 years? - t - oglo N
[ADS. (i) 2.45 x 104 years (ii) 86.7 mg]
0.693
[HiDt: (i) Mass of 238pU = 238 x 32 = 28.207 mg - Ioglo 1052
2.303
- .
4.7x t
270
t = 3.43 X 109 years]
,186 G.R~. PHYSICAL CHEMISTRY FOR COMPETITIONS.

n. Hydrolysis.of ester was studied by isotopic .Iabelling method. is 32 days. Calculate the time after which the laboratory will
Write down the structures of products 4 and B in the given be safe for use; .
Teaytion: (lIT 2000) [Ans. 192 days] .
o [Iifnt:, No =64N .
, II 18
0.693' 2.303 I No
CH3-C-O-:c-H+ HOH~ A +B oglO -N'
tl/2 t
o 0.693 ,;,,2.303 log 10 [ 64N ]
II 18
32. ,t N
[ADs. (A) CH3- C- OH; (B)CzH s - 0 - H]
12. Arrange the following species in decreasing order of chemical . ,', t = 192 days] c' ,

reactivity and radioactivity: 17. Radium lIas, a half life 1600 years and its daughter elemi;lnl ,
lH, tH, fH radon has a half life 3.82 days. In an enclosure,tbe volume of.'
'radon was found constant for a week. Explain and calculat~
[Ans.Reactivity : H > fH > tH . the ratio of the number of radium and radon nucIe.i..Will the
.. 3H 2H IH]
, Radi oactiVltyi",?j >1 . ratio be constant after 400 years?
13. The half life of 212Pb is 10.6 hours and that of its daughter [Ans. 1528):< 105 ]
element 212 Biis60.5 minutes. Afterhow.. much time will th~ ·}"t~ooi~t; ";~~;;~+~"N'i(Ra)-;,;t1l2(Rn)
daughter element have maximum activity? N 2(Rn) tdRa)
[Ans. 3.78 hours]
NI 1600 x 365
[Hint: A = '{).693 . Ad 0:693 = 0.01145 min- I Nz 3.82
P 10.6 x60 ' 60.5 '
= 1.528 x lOs]
= 0.QQI089 min- 1
18. Calculate the raciius and den~ity of 2:;;2 u.
2.303 I Ad
(A - A ) oglo T:
tmax
.d P P
[ADs. R = 6.8 X 10- 13 cm; d = 2.979 X 10 14 g I ecl
[Hint: R =Ro A 113 = 1.1 X 1O- IS (235)113 =6.788 X 1O- IS m
- 2.303 10 [0.01145 ]
- 0.01145- 0.001089, glO 0.001089 = 6.788 x 10-13 cm
24
222.275810 0.01145 d = WIV = 235 x 1.66 x 10- g I cc
. glo 0.001089
! x 1t x (6.788 x 10- l3
i
= 227.1 min 3
= 3.785 hours] = 2.979 X 10 14 glee]
14. Radioactive element is spread over a room, its half life is 30 19. 2~U decays with emission of a and (3-particles to form
days. Its activity is 50 times the permissible value. After how ultimately 2~iPb. How many a and (3-particles are emitted per
many days will it be safe?
[Ans. 169.30 days] atom of Pb produced?

[Hint: No 50N [Ans. 7 a and 4 131


20. The half life of radium is 1600 years. After how rnuch time,
0.693
- - IoglO (No)
2.303 -
t N ..!..th part of radium will remain undisintegrated in a sample?
16 .
[Ans. 6400 years]
0.693 = 2.303 loglo (50N\J
21. The halflife of polonium is 140 days. In whattimewilll5 g of
30 t N
polonium be disintegrated out of its initial mass of 16 g?
[Ans. 560 days]
t = 169.3 days]
15. Calculate the energy released in joules and MeV in the [Hint: Polonium left is ..!..th of the initial, i. e., 4 half lives.]
following nuclear reaction: 16
tH+ tH~ ~He + ~n 22. The activity of a radioactive isotope falls to 12.5% in 90 days.
Calculate the half life and decay constaut of the radioactive
Assume that the masses of tH, ~He and neutron respectively isotope.
are 2.0141, 3.0160 and 1.0087 amu. [Ans. 30 days, 0.0231 day-I]
[Ans. 5.223 x 10- 13 J; 3.260 MeV] 23. The radioactivity of an element was found to be one millicurie.
16. A radioactive element due to an ~cident in research What will be its radioactivity after 42 days if it has half life of
laboratory gets embedded in its floor and walls. The initial rate 14 days?
of decay is 64 times the safe limit. The half life of the element [Ans. 0.125 millicurie]
RADIOACTIVITY AND NUCLEAR TRANSFORMATION 187

24. There are 106 radioactive nuclei in a given radioactive .


35. Calculate the ratio of -
N
after an hour has passed for a
element.· Its half life is 20 seconds. How many nuclei will No
remain after 10 seconds? (Given,.J2 =1.41) radioactive material of half life 47.2 seconds.
[Ans. 7 x 1O~(approximately)]
[Ans. ~ = 1.12 x 10-23]
No .
[Hint: N=NO(.!.)" =Htx(1)1i2 asn= 10 =.!.=~]
. . 2 2· 20 2 1.41
[Hint: At = 2.30310g IO No
25. A radioactive element decays at such a rate that after 68 N
minutes only one-fourth of its original amount remains. 0.693 x 60 x 60 =10 No]
Calculate its decay constant and half life period. 47.2. 2.303 glO N
[Ans. A= 0.0204 min- t , tl/2 =34 min.] ~6. The activity of the hair of an Egyptian mummy is 7
26. One gram of a radioactive element decays·by J3-emission to disintegrations minute-I OfI4C. Find the age of the mummy.
0.125 in 200 hours. How much more time will elapse until Given, tl/2 of 14C is 5770 years and disintegration rate of fresh
only 0.10 g of it is left? sample of 14C is 14 disintegrations minute -I.
[Ans. 21.46 hours] [Ans. 5770 years]
27. A wooden article found in a cave has only 40% as much 14 C 37. On analysis a sample of 238U ore was found to contain 20.6 g
activity as a·ftesh piece J)fwood. How old is the article? oCZ~~Pb and 23.8 g of 21;u. The half life period of 238U is
(t l / 2 for 14C 5760 years)
4.50 X 109 years. If all the lead were assumed to have come
[Ans. 7<517 years]
from decay of 2~;U, what is the age of the ore? (lIT 1996)
28. A sample of carbon derived from one of dead sea scrolls is
found to be decaying at the rate of 12.0 disintegrations per [Ans.. 4.49 x 109 years]
minute per gram of carbon. Estimate the age of dead sea scrolls
When carbon from living plants disintegrates at the rate of 15.3 38. It is known that 1 g of 226Ra emits 11.6 x HP atoms ofa. per
disintegrations per minute per gram. (t l / 2 for 14C= 5760 years) . year. Given. the half life of 226Ra to be 1600 years, compute
[Ans. 2020 years] the value of Avogadro's number.
29. One).lg of a radioactive iodine contained in thyroxine is [Ans. 6.052 x ItY 3]
injected into the blood of a patient. How long will it take for [Hint: Rate = A x number of atoms in one gram
radioactivity to fall to 50%, 25% and 10% of the initial value?
(t1/ 2 for t~~I = 8.05 days) A x Avogadro's number ]
[Ans. 8.05 days, 16.1 days, 26.75 days] 226
30. 1 g radium is reduced by 2.1 mg in 5 years by alpha decay, 39. A uranium mineral contains 238 U imd 206Pb in the ratio of 4 : 1
calculate the half life period. by weight.. Calculate the age of the mineral tl/2 238U
9
= 4.5 X 10 years. Assume that all the lead present in the
[Ans. Halflife = 1672 years]
mineral is formed from disintegration of 238u.
[Hint: Mass of radium left after 5 years = (1.0 - O.OOil)g
[Ans~ l. 648 x 109 years]
=0.9979 g 40. In a sample of pitchblende, the atomic ratio of 206Pb : 238U is·
2.303 No 2.303 I 0.23 : I. Calculate the age of the mineral ifhalflife of uranium
Apply A = - - 10glO - =- - 10gIO - - ]
t N 5 0.9979· • is 4.5 x 109 years. Assume that all lead has originated from
31. The activity of a radioactive substance falls to 87.5% of the uranium.
initial value in 5 years. What is the half life of the element? [Ans. 1.34 x 109 years]
Calculate the time in which the activity will fall by 87.5%. 41. The ratio of the atoms of two elements A and B at radioactive
[Ans. Half life = 9.52 years, t = 28.58 years] equilibrium is 5.0 x lOS: I respectively. Calculate balflife of B
32. Starting with 1.0 g of a radioactive sample, 0.25 g of it is left ifhalf life of A is 245 days.
after 5 days. Calculate the amount which was left after one [Ans. 4.9 x 10-4 days]
day..
42. Calculate the energy released in MeV during the reaction ~Li
[Ans. 0.758 g]
33. A sample of wooden artifact is found to undergo 9 +~H -7 2[iHe] if the masses of ~Li, :H and iHe are 7.018,
disintegrations per minute per gram of carbon. What is the 1.008 and 4.004 amu respectively.
approximate age of the artifact? The half life of t:C is 5730 [Ans. 16.76 MeV]
years and radioactivity of wood recently cut is 15
disintegrations per minute per gram of carbon. 43. 1.0 g of 2~~ Ra is placed in a sealed vessel. How much helium
34. Xenon-127 has a halflife of36.4 days. How much of a sample will be collected in the vessel in 100 days? (tl/2 of radium =
of xenon that originally weighed 1.0 g remains after 20 days? 1600 years)
(Dhanl)ad 1992) [Ans. 2.12 x 10-6 g)
[Ans. 0.6835 g]
188 . G.R.B. PHYSICAL CHEMISTRY FOR COMPETITIONS

44. The half life period of l~~Ce is 13.11 days. It is a ~-particle 47. The disintegration rate of a certain radioactive sample at any
instant is 4750 dpm. Five minutes later, the rate becomes 2700
emitter and the average energy of the ~-particle emitted is
dpm. Calculate half life of sample.
0.442 MeV. What is the total energy emitted per second in
watts by to mg of l~JCe? [Ans. tliZ = 6.13 minute]

[Ans. 1.84 watt] 48. One of the hazards of nuclear explosion is the generation of
90Sr and its subsequent incorporation in bones. This nuclide
[Hint: Rate of disintegrations per sec A x No. of atoms has a half life of 28.1 years. Suppose one microgram was
23
0.693 x 6.023 x 10 x om absorbed by a new-born child, how much 90Sr will remain in
13.11 x 24 x 60 x 60 141 his bones after 20 years? (ffr'1995)
[Ails. 0.61 J.l.g]
Total [')-particles emitted = 2.61 x 1013
2.303 1 initial
Total energy emitted = 2.61 x 1013 x 0.442 1.1536 x 1013 MeV [Hint: - - og
A remaining
Energy in erg = (1.1536 x 1013)(1.6 x 10-6 ) o . 2.303 -I 20 ... I I . .
II. = 28.1 Y t = y, ImtJa = J.l.g, remammg =x J.l.g
13
. 1.1536 X 10 x 1.6 X 10-6 1 84 ]
Energy m watt = 7 . watt
10 20 = 2.303 x 28.1 log 1
0.693 x
45. A sample of ~gSr has an activity of 0.5 mq; What is its x 0.61j.Lg] .
specific activity? (t112 of ~gSr = 19.9 years) . 49. It has been estimated that the carbon-14 in the atmosphere is
12 1 responsible for producing 60 atoms of nitrogen-14 and 60
[Ans. 7.4 x 10 dis. 8- ]
electrons every hour for each gram of carbon. We can quote
[Hint: Rate of disintegrations A x No. of atoms this disintegration rate as 60 counts hour- 1 g-l. A sample of
So, No. of atoms sea shell found near a sea shore was found to have a count of 4
7 counts hour- I g-I. Estimate the age of the shell.
0.5 x 3.7X 10 x 19.9 x 365 x 24 x 60 x 60 14 .
(tIl2 for C = 5730 years).
0.693
[Ans. 21000 years (approximately)]
1.675 x 10 16
50. Upon irradiating. californium with neutrons, a scientist
16
Mass =90 x 1.675 X z310 = 2.50 X 10-6 discovered a new nuclide having mass number of 250 and half
6.023 X lO g life of 0.5 hours. Three hours after the irradiation, the
7 observed radioactivity due to the nuclide was 10 dis/min. How
Speel'fiIC actiVIty
.. = 0.5 x 3.7 x 610 .=.
74 X 1012 dIS
.
g -I many atoms of the nuclide were prepared initially?
2.5 x 10-
[Ans. 2.8 x 104]
46. Calculate the Q-value of the reaction;
~Li + ~n ---7 iHe + ~H
Given, ~Li 6.015126 amu, iHe = 4.002604 amu
fH 3.016049 amu, bn = L008665amu
[Ans. +4.7835 MeV]
RADIOACTIVITY AND NUCLEAR TRANSFORMATION 189

=~ OBJECTIVE QUESTioNS
Set-1 : Questions with single correct answer 12. The velocity of a-rays is approximately:
1. Natural radioactivity was discovered by: (a) equal to that of the velocity oflight
(a) Rutherford (b) Becquerel (b) ~th of the velocity of light
(c) Curie (d) Schmidt 10
2. Radioactivity is due to: (c) 10 times more than the velocity of light
(a) stable electronic configuration (d) uncomparable to the velocity oflight
(b) unstable electronic configuration 13. a-rays have ionisation power because they possess:
(c) stable nucleus (a) lesser kinetic energy
(d) unstable nucleus (b) higher kinetic energy
3. Radioactivity is essentially: (c) lesser penetration power
(a) a chemical activity (b) a physical property (d) higher penetration power
(c) a nuclear property . (d) a property of non-metals 14. The radiations from a naturally occurring radioactive
substance as seen after deflection by a magnetic field in one
4. Radioactivity is. generally found in:
direction are:
(a) light nuclei (b) stable nuclei
(a) definitely a-rays (b) definitely ~-rays
(c) heavy nuclei (d) nuclei ofintermeilim\;'rinx;y.r·
- '\e}-both a and ~-rays (d) eithera or ~-rays
5. The activity of radioisotope changes with:
15. Which of the following statements about radioactivity is
(a) temperature (b) pressure
wrong?
(c) chemical environment (d) none of these
(a) It involves outer electrons activity
6. The rays are given offby a radioactive element from:
(b) It is not affected by temperature or pressure
(a) nucleus (b) valence electrons
(c) It is an exothermic process
(c) all the orbits (d) outer orbit
(d) The radioactivity· of an element is not affected by any
7. The alpha particles are: other element compounded by it
(a) high energy electrons 16. The radioactivity of uranium minerals is usually more in
(b) positively charged hydrogen ions comparison to pure uranium, This is due to presence of, .. in
(c) high energy X-ray radiations the mineral.
(d) double positively charged helium nuclei (a) actinium (b) thorium
8. The emission of beta particles is from: (CSSE 1999) (c) radium (d) plutonium
(a) the valence shell of an atom 17. Radioactive disintegration differs from a chemical change in
(b) the inner shell of an atom being: (MLNR 1991)
(c) the nucleus due to the . nuclear conversion (a) an exothermic change
proton --'i' neutron + electron , (b) a spontaneous process
(d) the nucleus due to the nuclear conversion (c) a nuclear process
neutron --'i' proton + electron (d) an unimolecular first order reaction
9. Identify the nuclear reaction that differs from the rest: 18. The ionising power of a, ~ and y-rays is in the decreasing
(a) Positron emission (b) K-capture order:
(c) ~-decay (d) a-decay (a) a > ~ > y (b) ~ >a > y
(e) y-decay [PET{Kcrala) 20081 (c) y > a > ~ (d) ~ > y >a
[Hint: Only y-emission does not change the nip 19. Which of the following radiations have least effect on both the
.(NeutronlProton, ratio) of the parent element.] photographic plate and zinc sulphide screen?
10. Gamma rays are: (MLNR 1990) (a) a-rays (b) ~-rays
(a) high energy electrons (c) y-rays (d) All have equal effect
(b) low energy electrons 20. y-rays are emitted from a nucleus due to:
(c) high energy electromagnetic waves (a) high nip ratio
(d) high energy positrons (b) excess energy possessed by nucleus after emission of a or
1t . Radium:;; a radioactive substance. It dissolves in dilute H2 SO4 ~-particles
and form!> _ ,;,,'rnpound radium sulphate. The compound is: (c) fission reaction
(a) no longer radiuactive (d) fusion reaction
(b) half as radioactive .:.s [he radium content 2 i. If a radioactive substance is placed in vacuum at IOcr C, its
(c) as radioactive as the radium content rate of disintegration in comparison to one atmospheric
(d) twice as radioactive as the radium content pressure:
190 G.R.B. PHYSICAL CHEMISTRY FOR COMPETITIONS

(a) is not affected 5 210


3. 206 4H
84 Po -----7 82 Pb + 2 e
(b) increases . '" In above reaction, predictthe position of Po in the periodic
(c) decreases table when lead belongs to IVB group:
(d) increases when the product is gas (a) lIA (b) VIB (c) IVB (d) VB
22. Ina-decay, nlp·ratio: 36. When 2i~Ra emits an a-particle, the new element formed
(a) may increase or decrease belongs to:
(b) remains constant . (a) third group (b) zero group

(c) decreases (c) fourth group (d) second group
(d) increases 37. The radius of nucleus is: (VITEEE 2007)
23. In ~-decay, nip ratio: (a) proportional to its mass number
(a) remains unchanged (b) may increase or decrease (b) inversely proportional to its mass number
(c) increases (d) decreases (c) proportional to the cube root of its mass number
24. A device used for the measurement of radioactivity is: (d) not related to its mass number
(a) mass spectrometer (b) cyclotron 38. The last product of 4n series is:
(c) nuclear reactor (d) G.M. counter (a) 2~~Pb (b) 2~Pb (c) 2~iPb (d) 2i~Bi
25. Which of the following does not contain material particles? 39. 4n + 2 series is known as:
[CET (Pb.) 19911 (a) actinium series (b) thorium series
. "-.-
(a) a-rays (b) ~-rays (c) y-rays (d) Anode rays (c) uranium series (d) neptunium series
26. If by mistake some radioactive substance gets into human 40. A radioactive element A on disintegration gives two elements
body, then from the point of view of radiation damage, the Band C. If B is helium and C is the element of atomic number
most harmful will be one that emits: 90 and atomic mass 234, the element A is:
. (a) y-rays (b) neutrons (c) ~-rays (d) a-rays (a) 2~U (b) 2~:Ra (c) 2~Th (d) 2~tPa
27. Radioactive decay is a reaction of:
41. Group displacement law was given by:
(a) zero order (b) first order
(a) Becquerel (b) Rutherford
(c) second order (d) third order
(c) Mendeleeff (d) Soddy and Fajan
28. If nip ratio is high, the nucleus tends to stabilise by:
42. 234 U has 92 protons and 234 nucleons total in its nucleus. It
(a) the emission of a ~-particie . .decays by emitting an alpha particle. After the decay it
(b) neutron capture becomes: (VTTEEE 2008; DUMET 2010)
(c) losing a positron (a) 232U (b) 232 Pa (c) 230 Th (d) 23°Ra
(d) anyone of the above 43. Starting from radium, the radioactive disintegration process
29. Emission of~-particles by an atom of an element results in the terminates when the following is obtained:
formations of: (a) lead (b) radon (c) radium A (d) radiumB
(a) isobar . (b) isomer (c) isotope (d) isotone 44. The only, most stable nucleus formed by bombarding either
30. Which of the following process will cause the emission of ijAi by neutrons or NNa by deutrons is: [CET (J&K) 2007]
X-ray?
(a) ?~p (b) t2Si (c) ~iMg (d) 1~~Ba
(a) a-emission (b) ~-emission
(c) K-electron capture (d) y-emission 45. Quantity of radioactive material which undergoes 106
31. When a ~-particle is emitted by the atom of a radioactive disintewations per second is called:
element, the new species formed possesses: IPET (MP) 19901 (a) Becquerel (b) Rutherford
(a) same atomic mass and atomic number less by one unit (c) Curie (d) Faraday
(b) same atomic mass and atomic number less by two units 46. The number of a-particles emitted per second by I g of 226 Ra
(c) same atomic mass and atomic number higher· by one unit is 3.7. x 1010: The decay constant is:
(d) same atomic mass and atomic ntimber higher by two units (a) 1.39 x 10- 11 sec- I (b) 13.9 x 10-11 sec- I
32. Successive emission of an a-particle and two ~-particles by mi· (c) 139 x 10~1O sec- I (d) 13.9 x 10- 10 sec- I
atom ofa radioactive element results in the formation of its: No. of atoms disintegrating per second = A
1Ill' (Screening) 19931 [Hint:
. Total number sf atoms present
(a) isobar (b) isomer (c) isotone (d) isotope 10
3.7 x 1010 = 226 x 3.7 x 10 =A]
33. The isotope 2~iu decays in a number of steps to an isotope of or 23 23
2~iPb. The groups of particles emitted in this process will be: 6.02 x 10 6.02 X 10
226
(a) 4a, 7~ (b) 6a, 4~ (c) 7a, 4~ (d) lOa, 8~
47. The decay constant of 226Ra is 137 x 10- 11 sec-I. A sample of
34. The number of a and ~-particles emitted in the nuclear
226 Ra having an activity of 1.5 millicurie will
reaction 2~~ Th -----7 2~~Bi are: [MLNR 1992;
contain ...... atoms:
JEE (Orissa) 20101
(a) 4.05 x 1018 (b) 3.7 x 1017 (c) 2.05 x 1015 (d) 4.7x lOW
(a)8a,l~ (b)4a,7~ (c)~a,7~ (d)4a,l~
RADIOACTIVITY AND NUCLEAR TRANSFORMATION 191

[Hint: 1 millicurie = 3.7 x 107 disintegrations per sec (a) I g of the sample (b) 0.5 g ofthe sample
1.5 millicurie = 5.55 x 107 disintegrations per sec (c) ~.25 g of the sample' (d) 0.01 g of the sample
57. 14C has a half life of 5760 years. 100 mg of the sample
5.55 x 107 containing 14C is reduced to 25 mg in: [PET (Raj.) 2006J
- - - = A = 1.37 xlO- ll ]
No (a) l1S20 years (b) 2880 years
48. One curie of activity is equivalent to: (c) 1440 years (d) 17280 years
(a) 3.7 x 10'7 disintegrations per sec 58. If 3 I 4 quantity of radioactive substance disintegrates in 2
hours, its half life period will be: (BHU 2006)
(b) 3.7 x 1010 disintegrations per sec (a) IS minutes (b) 30 minutes
(c) 3.7 x 10' 4 disintegrations per sec (c) 60 minutes (d) 90 minutes
(d) 3.7 x 103 disintegrations per sec 59. Initial mass ofa radioactive element is 40 g. How many grams
49. A sample ofi2K contains invariably ~Ar. This is becausei2K
of it would be left after 24 years if its half life period is of 8
years?
has tendency to undergo: . [JEE (Orissa) 2006J
(a) a-decay (b) positronium decay (a) 2 (b) S . (c) 10 (d) 20
(c) ~-decay (d) y-decay 60 .. Half life of radium is 1580 years. It remains 1116 after the
years ....... : (VMMC 2007)
50. The value of disintegration constant of a radioactive isotope:
(a) 1S80yrs (b) 3160yrs (c) 4740yrs (d) 6320yrs
(a) decreases with increasing temperature .
61. If hal~life period of radium is 1600 years, its average life
(b) decreases with increasing pressure period will be: .. .. .. ... . ............ . .... -
(c) increases with increasing concentration
(a) 2304 years (b) 4608 years
(d) is independent of temperature, pressure and concentration
(c) 230.4 years (d) 23040 years
51. If the amount of a radioactive substance is increased three
62. A radioactive isotope having a half life of 3 days was received
. times, the number of atoms disintegrating per unit time would:
after 12 days. It was found that there were 3 g of the isotope in
(a) be double
the container. The initial mass ofthe isotope when packed was:
(b) not be change
(a) 48g (b) 36g (c) 24g (d) 12g
(c) be triple
63. Radioactivity of a radioactive element remains I I 10 of the
(d) be .!.rd of the original number of atoms original radioactivity after 2.303 seconds. The half life period
3 is:
52. The half life of a radioactive element depends upon: (a) 2.303 (b) 0.2303 (c) 0.693 (d) 0.0693
(a) the amount of the element ~ 2.303 I ' a
[Hint: 1\.=-- oglO--
(b) the temperature t (a-x)
(c) the pressure
2.303 1 0.693
(d) none of these or A = --loglO - - = 1, T =- - = 0.693]
2.303· 1110 A
53. The decay constant of a radioactive sample is A. The half life
and mean life of the sample are respectively: (MLNR 1990) 64. A freshly prepared radioactive source of half life period 2
(a) 1/A, In 2/A (b) In 2/A, 11 A hours emits radiations. of intensity which is 64 times the
(c) A In 2, II A (d) AI Pn2' 1/A permissible safe level. The minimum time after which it would
54. Average life of a radioactive substance is: be possible to work with this source is:
(a) 0.44 times ofhalflife (b) 2.44 times ofhalflife (a) 6 hours (b) 12 hours
(c) 1.44 times ofhalflife (d) 0.693 times ofhalflife (c) 24 hours (d) 48 hours
55. Radium has atomic mass 226 and half life of 1600 years. The 65. A radio isotope has a half life of 10 days. If today there is 125
number of disintegrations per second per gram are: g of it left, what was its mass 40 days earlier? .
. . . (EAMCET 1991)
(BHU 1990)
(a) 600 g (b) 1000 g (c) 1250 g . (d) 2000 g
(a) 4.8 x 10'0 (b) 3.7 x 108
66. The half life period of four isotopes is given below:
(c) ,9.2 x106 (d) 3.7 x 10'0 (i) 7.6 years (ii) 4000 years
[Hint:
. No. of disintegrations per sec =A (iii) 6000 yeal'S (iv) 3.2 x lOs years
Total no. of atoms in one gram ofRa Which of the above isotopes is m~st stable?
0.693 (a) (iv) (b) (iii) (c) '(ii) (d) (i)
1600 x 365 x 24 x 60 x 60 67. The first indication that a stable nucleus can be broken down
or Ne ')f disintegrations per sec was afforded by:
0.693 x 6.023 x 1023 (a) Rutherford (b) Madam Curie
= 1600 x 365 x 24 x 60 x 60 x 226 ] (c) Soddy (d) Schmidt
68. The .first stable isotope which was transmuted by artificial
56. A radioactive sample has a half life 1500 years. A sealed tube
means was:
containing I g of the sample will contain after 3000 years:
(MLNR 1994) (a) l~O (b) IjN (c) l~C (d)' ~Be
192 G.R.B. PHYSICAL CHEMISTRY FOR COMPETITIONS

69. The instability of a nucleus is due to: (AIIMS 1999) 79. A positron is emitted from ~fNa. The ratio of the atomic mass
(a) high, proton: electron ratio and atomic number of the resulting nuclide is: (lIT 2007)
(b) high, proton: neutron ratio (a) 22110 (b) 22/11 (c) 23110 (d) 23/12
(c) low, proton: electron ratio [Hint: : H ~ ~n + +~ e
(d) low, proton: neutron ratio Positron

70. When ;;Al is bombarded with a-particles, a radioactive On positron emission, proton is converted to neutron, therefore,
isotope of phosphorus i~Pwith the emission of ... is formed. atomic number decreases by one unit but atomic mass remains
[CET (Gujarat) 20061 constant.
(a) neutrons (b) protons n . 23]
: rat1o·=~
(c) positrons (d) electrons p 10
71. Nuclear reaction accompanied with emission of neutron(s) is: 80. Hydrogen bomb is based on the principle of: (AIEEE 2005)
[PMT (MP) 1991) (a) nuclear fission (b) natural radioactivity
(a) i;Al + iHe ~ ~~P + ~n (c) nuclear fusion (d) artificial radioactivity
81. In nuclear reactors, the speed of neutrons is slowed down by:
(b) I~e + iH ~ I~N
(a) heavy water (b) ordinary water
(c) ;~P~ ?SSi + re (c) zinc rods (d) molten caustic soda
82. Which of the following is not a fissile material?
(d) 2~~Am + iHe ~ 2:iBk + ?e
(a) 235U (b}.238 U
72. Which of the following transformations is not correct?
(c) 233U (d) 239Pu
( a) 75A 4H 78B
33 s + 2 e ~ 35 r + Ion
83. Which one of the following statements is wrong?
(b) ~Li + iH ~ ~Be+ ~n (a) An atom bomb is based on nuclear fission
(c) iiSc + ~n ~ i~ea + ~n (b) In atomic reactor, the chain reaction is carried out under
control
(d) 2~Bi + ;H ~ 2~~PO + ~n (c) Fission reactions are the sources of sun's energy
73. The reaction, 2~iu + ~'1z ~ 1~~Ba + ~~Kr + 3 ~n represents: (d) Hydrogen bomb is always associated with atomic bomb
(a) artificial radioactivity 84. The fuel in atomic pile is:
(b) nuclear fission (a) carbon (b) sodium
(c) nuclear fusion (c) petroleum (d) uranium
(d) none of these 85. Large energy released in atomic bomb explosion is mainly due
74. I~e in upper atmosphere is generated by the nuclear reaction: to:
WET (MP) 19931 (a) conversion of heavier to lighter atoms
(a) IjN + iH~ lie + +~e+ iH (b) products having lesser mass than initial substance
(c) release of neutrons
(b) IjN ~ lie + +~e (d) release of electrons
(c) IjN+~n~l~e+lH 80. One gram of mass is equal to:
(d) IjN+ lH ~ I~e + iHe . (a) 5 x 1010 erg (b) 9 x 1020 erg
75. In the transformation of 2~~ U to 2~i U, if one emission is an (c) 7 x 105 erg (d) 11 x I{p erg
a-particle, what should be the other emission(s)?
87. If the energy released by burning 1 g of carbon is 3 x lOll erg,
.' (AIEEE 2(06)
then the amount of energy released by converting I g of
(a) two B- (b) two B- and one B+ carbon completely to nuclear energy would be equivalent to
(c) oneB- andoney energy produced by burning ...... g of carbon.
[Hint: 2~ U ~ 2~iu + iHe + 2_~ e] (a) 106 (b) 108
(c) 9 X 1020 (d) 3 x 1010
76. The reaction, ; H + fH ~ ~He + ~n is called:
. (CPMT 1')90) 88. Liquid sodium is used in nuclear reactors. Its function is:
(a) fusion (b) fission (a) to collect the reaction products
(c) endothermic reaction (d) spontaneous reaction (b) to act as heat exchanger
77. When the nucleus of uranium is bombarded with neutrons, it (c) to absorb the neutrons in order to control the chain'
breaks up into two nuclei of nearly equal mass. This process is reaction
called: (d) to act as moderator to slow down the neutrons
(a) nuclear fission (b) nuclear fusion 89. A sample of rock from moon contains equal number of atoms
(c) phYSIcal change (d) artificial radioactivity of uranium and lead (t1/2 for U = 4.5 X 109 years). The age of
the rock would be: IlJGET Manipal (Medical) 20061
78. Which one of the following is an artificial fuel for nuclear 9
reactors? . (a) 9.0 x 10 years (b) 4.5 x 109 years
(a) 238 U (b) 239Pu (c) 235U (d) 232 T h (c) 13.5 x 109 years (d) 2.25 x 109 years
RADIOACTIVITY AND NUCLEAR TRANSFORMATION 193

102. The radioactive de<:ay of ~~ X by a beta emission produces an


. [Hint: 2.303 I [I No. of unstable nucleus which spontaneously emits a neutron: The
t =-}..- oglo + No. ofU atoms
final product is: (MLNR 1995)
_ 2.3()3 x 4.5 x 109 10 . (1 + 1)] (a) ~~X . (b) ~~Y
0.693 glo
(c) ~!Z (d) ~~W
90. In treatment of cancer, which of the following is used?
103. ij AI is a 'stable isotope. f~ AI is expected to disintegrate by:
(a) 1~~1 (h) f;P (c) ~Co (d) ;H (llT H96)
91. In nuclear reactor, chain reaction is controlled by introducing: (a) a-emission (b) l3-emission
(AIIMS 1991) (c) positron emission (d) proton emission
(~) cadmium rod (b) iron rod . 8B 8B 0 · .
104. The mass defect of the nucIear reaction 5 ~ 4 e + 1 e IS.
(c) platinum rod (d) graphite rod
(Jlr~lER 191)9)
92. Wooden artifact and freshly cut tree are 7.6 and 15.2 min -I g-I
(a) ~m = atomic mass of(~Be - ~B) .
of carbon (t1/2 =
5760 years) respectively. The age of the
artifact is:· . (b) ~ = atomic mass of(~Be - ~B) + mass of one electron
(a) 5760 years (b) 5760 x 15.2 years (c) ~m= atomic mass of(~Be ~ ~B) + mass of the positron
7.6
(d) ~m = atomic mass of(~Be - ~B) + mass of two electrons
(c) 5760 x 7.6 years (d) 5760 x (i 5.2 - 7.6) year
15.2 105. Which of the following is the man-made radioact~v·
2.303 I 15.2 2.303 x 5760 1 2] disintegration series?
[Hint: t =
}..
. og
. 7.6
or t =
0.693 og (a) Thorium series (b) Neptunium series
93. The isotope used for dating archaeological finding is: (c) Uranium series Cd) Actinium series
(a) ~H ,b) l~O (c) l:C (d) ~iu 106. The .density of nucleus is of the order of:
tti; 105 kg m-3 • (b) ufO kg m-3
94. Which one of the following statements is wrong? 17 3
(c) 10 kg m- (d) J(f5 kg m-3
(a) Neutron was discovered by Cnadwick
(b) Nuclear fission was discovered by Hahn and Strassmann 107. A radioactive isotope having a h3.If life of 3 day's was received
after 12 days. It was found that there were 3 !,!, of the isotope in
(c) Polonium was discovered by Madam Curie
the container. The initial weight of the isotope when packed .
(d) Nuclear fusi(>n was discovered by Fermi was:
95. Neutrons are more effective projectiles thlln protons because (a) 12 g (b) 24 g
they:
(c) 36 g (d) 48 g
( a) are attracted by nuclei (b)· are not repelled by nuclei
108. A radioactive substance is decaying withtlf2 =30 (jays. On
.(c) travel with high speed (d) none of these being separated into two fractions, one of the. fractions,
96. The source of enormous energy of sun is: illllnediately after separation, decays with tl/2 =2 days. The
(a) fusion of hydrogen to form helium other fraction, illllnediately after separation would show:
(b) fission ofuranil,1m (a) constant activity (b) increasing activity
.(c) fusion of deuterium and tritium
(c) decay with t1l2 = 30 days (d) decay with t1l2 =28 days
(d) fusion of tritium to form helium
109. A radioactive substance has a constant activity of 2000
97. In the neutron~induced fission of 2~ U; one of the products is disintegrations per minute. The material is separated into two
~ Rb. In this mode, another nuclide rutd two neutrons are also fra(:tions, one of which has an initial activity of 1000
produced. The other nuclide is: [PMT (HP) 20061 disintegrations per second while the other fraction decays with
(a) l~Xe (b) l~CO· (c) l~~CO (d) l~~:Xe t1l2 = 24 hours. To the total activity in both samples after 48
98. ~~X 3a 13 ~ Y. The element Yis: hours of separation is:
IJEE (Orissa) 20081 (a) 1500 (b) 1000 (c) 1250 (d) 2000
(a) 2~~Pb (b) 2gPb 110. A radioactive element X has an atomic number of 100. It
decays directly into an element Y which decays directly into
(c) 2~~Bi (d) 2~~Bi the element Z. In both processes a charged particle is emitted.
99. Which radioactive isotope is used to detect tumours? Which of the following statements would be true?
(a) 74 As (b) 24Na (c)' 131 1 (d) 6Oeo (a) Y has an atomic number of 102
100. Natural uranium consists of 235U: (b) Y has an atomic number of 101
(a) 99% (b) 50% (c) Zhas an atomic number of 100
(c) 10% (d) 0.7% (d) Zhas an atomic number of 99
101. In the nuclear reaction, I;N + ~He ~
X is:
:X
+ iH the nucleus
(MLNR 1995)
111. Three isotopes of an element have mass numbers M, (M + 1)
and (M + 2~ If the mean mass number is (M + - 0.5), then
(a) nitrogen of mass 16 (b) nitrogen of mass 17 which of the following ratios may be accepted for.M, (M + 1),
(c) oxygen of mass 16 (d) oxygen of mass 17 (M + 2) in that order?
(a) I : 1 : 1 (b) 4: I : I (c) 3: 2 : 1 (d) 2: I : I
I

194 G.R.B. PHYSICAL CHEMISTRY FOR COMPETITIONS

112. Enrichment of uranium is made by: , 124. Thiosulphate ion (S20~-) on acidification changes to S02
(a) distillation (b) diffusion along with precipitation of sulphur,
(c) evaporation (d) bleaching 35S32S0~- + 2H+ ~ H 20 + S02 + S
113. Let us consider emission of a-particle from uranium nucleus:
which is the correct statement?
92 U235 - 2He
4
~ 90 Th
231
(a) S35 is in sulphur (b) S35 is in S02
e 92 e o e= 90
(c) S35 is in both (d) 8 35 is in none
p= 92 P 2 p=90
125. A radioactive element decays as,
n = 143 n 2 n '"" 141
a decay {-2illP decay
Shortage of two electrons in t"!"iorium is <,iue to: X y ~---,-----) Z
(a) conversion of electron to positron 11/2 ; 2 days

(b) combination with positron to evolve energy which oithe following statements about this decay process is
(c) annihilation incolTect?
(d) absorption in'the nucleus (a) After two hours, less than 10% of the initial X is left
114. Artificial radioactive elements are present in: (b) Maximum amount of Y present at any time before 30 min
(a) s-block (b) p-block (c) d-block (d) j~block is less than 50% of the initial amount of X
115. Half life of 6C 14 , if its A. is 2.13 x 10-4 yrs, is: ' ','l':9) (c) Atomic number of X and Z are same
(a) 3.5 xl04 years (b) 3 x 103 years (d) The mass number of Y is greater than X .
126. Among the following nuclides, the highest tendency to aecay
(c) 2 x 102 years (d) -4 x 103 years
by 03+) emission is: '
116. The 60 Co isotope decays with a half life of 5.3 years. How , (a) 59CU (b) 63 Cu (c) 67 Cu (d) 68 CU
long would it take for 7 i 8 of a sample of 500 mg of 6OCo to
disintegrate? 127; Identify [A] and [B] in the following:
(a) 21.2 years (b) 15.9 years 2i~Ac ~[A] '~RnPEE (WB) '.tOW)
(c) 10.6 years (d) 5.3 years (a) Po, Rn Th, Po (c)Ra, Th (d) Th, Ra
117. Isotope of uranium used in atomic bomb is' [Hint: 2~~Ac 22JoTh 2;~Ral
!PET {MP} 20081 128. f)-particle is emitted in radioactivity by: (AIEEE
('a) 237 U
92 ' (d) 2~U (a) conversion of proton to neutron
118. Which among the following is wrong about isodiapheres? (b) from outennost orbit
(a) They have the same ditference of neutrons and protons or , (c) conversion of neutron to proton
same isotopic number
(d) l3-particle is not emitted
(b) Nuclide and its decay product after a-emission are
129. The nuclear reaction,
isodiapheres
(c) zA M -----7 z _ 2BM - <I + 2He 4 ~~Cu + iHe-~ gCI + 14 :H + 16lP
'A' and' B' are isodiapheres is referred to as: WET (MP) 20lf2i
(d) All are correct (a) spallation reaction (b) fusion reaction
119. At radioactive equilibrium,the ratio of two atoms A and B are (c? fission reaction (d) chain reaction
3.1 x 109 : I, Ifhalflife of 'A' is 2 x 1010 yrs, what is halflife of 130. 226Ra disintegrates at such a rate that after 3160 yrs only one
'B'? fourth of its original amount remains. Half life of 226Ra will
(a) 6.45 yrs (b) 4.65 yrs (c) 5.46 yrs (d) 5.64 yrs be: , WET (MP)
120. The decay constant for an a-decay ofTh 232 is 1. 58 x 10- 10 S,-1 . (a) 790 years (b) 3160years
How many a-decays occur from 1 g sample in 365 days?
(c) 1580 years (d) 6230 years
(a) 2.89x 1019 (b) 1.298 x 1019
131. 2~U nucleus absorbs a neutron and disintegrates into 13~Xe,
(c) 8.219x 1019 (d) None of these
j:Sr and 'x'. What will be the product x?
121. :What percentage of decay takes plac~ in the average tife of a ICBSE {P:\'!T)
substance?
(a) 3-neutrons (b) 2-neutrons
(a) 63.21% (b) 36.79% (c) 90% . (d) 99%
(c) a-particles (d) f)-particles
122. SI unit of radioactive decay is:
(a) curie
,.
(b) rutherford a
132. A radioisotope, tritium H) has half life of 12,3 years. If the
initial amount of tritium is 32 mg; how many milligrams of it
(c) becquerel i (d) all of these
would remain after 49.2 years? (P1VH')
123. The number of neutrons acc'ompanying the formation of 1~~ Xc , (a) 1 mg (b) 2 mg
and j~Sr from the absorPtion of a slow neutron by 2~ilI (c) 4 mg (d) 8 mg
followed by nucl...:ar fission is: I~)99', 133. The radio nuclide 2~6 Th undergoes two successive j3-decays
(a) 0 (b) 2 (c) 1 (d) ). followed by one a-decay. The atomic number and mass
number of the resulting radio nuclide are: (ARn~E 1~J!J3~
RADIOACTIVITY AND NUCLEAR TRANSFORMATION 195

(a) 92, 234 (b) 94, 230 (c) 90, 230 (d) 92, 230 (a) I g ofRa (b) I g ofRaS04
134. The half life of a radioactive isotope is three hours. If the (e) 1 g ofRaBr2 (d) I g of Ra(HP04 )
initial mass of isotope were 256 g, the mass of it remaining ,145. An artificial transmutation was carried out on liN by an 0.-
lUldecayed after 18 hours would be: (AIEEE 2003) particle which resulted in an unstable nuclide and a proton.
(a)4g (b)8g (c)12g (d)16g What is the ratio of the atomic mass to the atomic number of
135. Consider the following nuclear reactions: the unstable nuclide? (SCR~ 2009)

Z39~M ---7 ;N + 2 iHe; ;N ~ ~L + 213+ (a) 17


8
(b).!2
7
(c) 17
9
(d).!2
'8
The number of neutrons in the element Lis: (AIEEE 2004) [Hint: ijN + I~O + lH
(a) 142 (b) 144 (c) 140 (d) 146 Mass Number = 17]
136. A radioactive element gets spilled over the floor of a room. Its Atomic Number 8
half life period is 30 days. If initial rate is ten times the 146. If 004 curie be the activity of 1 gram of a radioactive sample
permissible value, after how many days will it be safe to enter whose atomic mass is 226, then what is the half-life period of
the room? (AIEEt 200i) the sample? (1 curie=: 3.7 x 1010 dis sec-I) (SCRA2009)
(a) 100 days (b) 1000 days (a) L2x lIyl sec (b) L8~ 10l sec
l

(c) 300 days (d) 10 days


(c) L2x 1010 sec (d) 1.8xHY o sec
137. A, p.\loton of hard gamma radiation knocks a proton out of
;iMg nucleus to form: ' (AIEEE 2005) (Hint: Rate of decay = (1693 x ~ x 6.023 x 1023
. (a) the isotope of parent nucleus tl/2 Aw
(b) the isobar of parent nucleus Q.4 x 3.7 X 1010 = 0.693 x 1 x 6.023 X 1023
(c) the nuclide of ~~Na tl/2 226
(d) the isobar of rrNa tllz = 1.2 X1011 sec]
138. The element 2~~ Th belongs to thorium series. Which of the 147. The half-life period of uranium is 4.5 billion years. After 9.0
following will act as the end product of the series? billion ybars, the number of moles of helium liberated from the
following nuclear reaction will be :
[BHU (Pre.) 20051
2~~ U ~ 2~Th + iHe
(a) 2~~Pb (b) 2~Bi (c) 2~Pb (d) 2~Pb
Initially there was 1 mole uranium. .[PET (MP) 20101
139. 2~~ U emits 8(l-p~icl~s and 6~-particles. The neutron/proton (a) 0.75 mol (b) LOmol (c) 11.2 mol , (d) 22.4 mol
ratio in the product nucleus is: (AI!MS 2005)
(a) 60/41 (b) 61/40 (c) 62/41' (d) 61142
Set-2: The questions given below may have more
than one correct answers
140. Calculate the mass loss in the following:
~H+iH~ iHe + ~n 1. Match the following nidioactive series:
Given the masses: ~H 2.014 amu, rH = 3.016 amu; (A) 4n (i) Uranium series
iHe 4.004 amu, hn
= 1.008amu. IPET (Kcrala) 20051 (B) 4n + 1 (il) Neptunium series
(a) 0.018 ama (b) 0.18 amu (C) 4n + 2 (iii) Actinium series
(c) 0.0018 :unu (d) 1.8 amu (D) 4n + 3 (iv) Thorium series
(e) 18 amu ABC D
141. A nuclide of an alkaline earth metal lUldergoes radioactive
(a) (i) (ii) (iii) (iv)
decay by emission of the a-particles in succession. The group
of the periodic table to which the resulting daughter element (b) (iv) (ii) (i) (iii)
would belong is: [CRSE (PMT) 20051 (c) (iii) (i) ,(iv) (ii)
(a) 4th group (b) 6th group (c) 14th group (d) 16th group (d) (ii) (iii) 0) (iv)
142. In the reaction;H + rH ~ iHe + tn,if the binding energies 2. Match the following reactions:
of ;H, fH and iHe are a, b and c (in MeV) respectively, then (A) 4Be9 + 2He4 ~ 6C2 +... (i) 2He4
energy (in MeV) released in this reaction is:
(B) 6C2 + ... ~ + 2He 4 SBIO (ii) onl
ICBSE-PMT (Physics) 2005]
I4
(a)a+b-c (b)c+a-b (C) 7 N + .. ~ 8017 + I HI (iii) I D2
(c) c -: a b (d) a'+ b + c (D) 20Ca40 + ... ~ 19K37 +2He4 (iv) IHI
143. Two radioactive elements X and Yhave halflives 6 min and 15
ABC D
min respectively. An experiment starts with 8 times as many
(a) (i) Oi) (iii) (iv)
atoms of X as Y How long it takes for the number of atoms of
Xleft to equal the number of atoms of Yleft? (b) (ii) (iii) (i) (iv)
fPET (Keraia) 20081 (c) (iv) (ii) (iii) (i)
(a) 6 min (b) 12 min (c) 48 min (d) 30 min (d) (iii). (ii) (i) (iv)
(e}24 min 3. A radioactive element is present in VIII group of the periodic
144. Which of the following has the highest value of radioactivity? table. If it emits one a-particle, the fl;;;W position of the nuclide
(DPMT 2009) will be:

196 ..G.R.B. PHYSICAL CHEMISTRY FOR COMPETITIONS

(a) VIB (b) VIII what is the value of n?


(c) VIIB (d) IS (a) 3 (b) 4
4. Which statement is true about n / p ratio? (c) 5 (d) 6
( a) It increases by t}-emission 14. 60Co has tt'12 :::: 5.3 years. The time taken for 7/8 of the original
(b) It increa~s by a~emission sample to disintegrate will be:
(c) It increases by v-emission (a) 4.6 yrs (b) 9.2 yrs
(d) None of the above (c) 10.6 yrs (d) 15.9 yrs
5. How many a and t\-particles should be eliminated so that an 15. Which of the following is/are correct?
isodiaphere is form~d? , (a) a-rays are more penetrating than (}-rays ,
(a) na,nt} (b) na, (n + 1)13 (b) a-rays have grea!er ionizing power than l3-rays
, (c) na ' (d) nt} (c) l3-particles are not present in the nucleus, yet they are
6. Match the following.: emitted from the'nucleus
Series Particles emitted (d) v-rays are not emitted simultaneously with a and p-rays
, (al Thorium (i) Sa, 513 16. Select the wrong statement:
(a) Nuclear isomers contairi the Same number of protons and
(b) Neptunium (ii) Sa, 613
neutrons
(c) Actinium (iii) oo.,4t} (b) The decay constant is independent of the amount ofth~
(d) Uranium (iv) 7a, 413 substance taken
'A B C D (c) One curie = 3.7 x 1010 dis/minute
(a) (iv) (iii) (ii) (i) , (d) Actinium series starts with U 238
(b) (ii) (i) (iv) (iii) 17. In a iIuclear reactor, heavy water is used to:
(c) (iii) (i) (iv) (ii) (a) provide high speed to neutrons
(d) (i) (ii) (iii) (iv) "(b) reduce the speed of neutrons
( c) capture neutrons produced by nuclear fission
. 7. Which of the following are used as control rods in a nuclear (d) transfer the heat from the nuclear reactor
reactor? 18. The correct starting material and product of different
(a) Cadmium rods ' (b) Graphite rods disintegration series are:
, (c) Steel rods (d) All of these (a) 232Th,208Pb (b) 235U, 2~Pb
S.,Which of the following 'notations shows the product
incorrectly? > ~ ••• ; " (c) 238 U ,201Pb (d) 231Np,209Bi
(a) 96Cm242(a,2n) 91Bk243 (b) s.!3lO (a,n) 1N13 19. Which of the following is/are not true?
(e) 1NI4(n, p) 6C4 (d) 14Si28(d,n) ISP29 (a) The most radioactive element present in pitchblende is
uranium
9. WhiCh is true about decay constant (A)?
(b) 32 P is used for the treatment of leukaemia
(~) Unit"is time-I
(c) CO2 present in the air contains 12 Conly
(b). Value of A is always less than 1
(c) A is independent oftemperature (d) Omission of y~rays changes the mass number but not
atomic number
(d) A is defined as the ratio of no. of atoms disintegrating per
unit time to the total no. of atoms present at that time
20. ,which of the following is/are correct? '
. 10. Which of the following iS,not correct? (EAMCET2006) (a) 1 Curie = 3.7 x 1010 dis
(a) Nuclei. of atoms participate in nuclear reactions (b) 1 Rutherford =106 dis
ig
(b) igca and Ar are isotones, (c) 1Becquerel = 1dis
(c) 1 amu of mass defect is approximately equal to 931.5 Me V (d) 1 Fermi = 103 dis
(d) U~ium (U238) series is known as (4n + 2) series
21." Match the List-land List-II and select the cQrrectanswer using
, 1~. Correct order of radioactivity is: ' the codes given below the lists:
(a) I HI > lH2i> IH3 (b) I H3 > IH2 > IHI LiSt-I
3 l
(c) IH, > 1He >- 1H2 (d) IH3 > lHl = ja2 Nuclear radar
~t
12. At radioactive equilibrium, the ratio between 2 atoms of
radioactive elements.A and B is 3 x 1<f : 1. If tl/2 of A is I(jO i. Moderator A. Uranium
yrs what is tll2 ofB? 2. Control rods B. Graphite
(a)30yrs . (b)3yrs .. 3. Fuel rods C. Boron
(c) 3.3 yrs (d) None of these I 4. Coolant D. Lead
13. In the sequence of the Jollowing nuclear reaction, E. Sodium'
-(1 -fl '-fl na,
X9~8 Y~Z~L~ 90M218 JPET (Kerala) 20051
I
. RADIOACTIVITY AND NUCLEAR TRANSFORMATION 197

Codes: (b) I-a; II-b; 1II-c, IV--d .


(a) I-B, 2-A, 3-C, 4-E (c) I-c; I1-d; III~b; IV-a
(b) I-B, 2-C, 3-A; 4-E (d) l-d; II-c; III-b; IV-a
(c) l-C, 2-B, 3-A, 4-E ;l6. Column-I Column-II '
(d) I-C,2-D, 4-B (I)I~C (a) Unstable and ~-emitter
(e) I-D, 2-C, 3-B? 4-A
(II) riNa (b) Stable
22. Match the List-I and List-II and select the correct answer using (III) I~N (c) Unstable, positron emitter
the codes given below the lists:
(1V)I~C (d) Unstable, a-emitt~
List-1 List-II
"
Isotope Characteristics Correct matching is/are:.
· 40 (a) I only (b) III only
A. 20Ca 1. UnstaO'te:~-eniitter
(c) II and IV (d) I and III
B 133 1
• 53
2. Unstable, ~-ernitter 27. Which of the following statements is/are con:ect? ,
C 121 I 3. Unstable, positron emitter 1. A nucleus in an excited state may give up its excitation
• 53
energy and retum to the ground state by emission of
D. 2~Th 4.< Stable
electroIilagnetic'V-radiation. '
Codes: A B C 'D 2. 'V-radiations are emitted as secondary effect of a and
(a) 21 3 4 f3~emission., .
(b) 3
'I 2 4 3. The nuclear isomers produced by 'V-ray bombardment have
(c) 4
3 '2 the same atomic and mass number but differ in their
life-times (whatever their ground state may be).
(d) 4
2 3
4. X-ray and 'V-ray are both electromagnetic.
23. Match the List~1 with List-II aiid select the correct answer (a) 1 and 2 (b)1,2and3 (c)2and3 (d)1,2,3and4
using the codes given below the lists:
28.. Which of the following statements is/are correct?
List-I List-II 1. When an ele~tron is emitted by an atom and its nucleus
Isotope Charaderisties gets de-excited as a result, the process is called internal
A. 32p 1. Location of tumour in brain conversion.
B. 24Na ·2. Location of blood clot and 2. Electron capture and positron emission are idenJicaL
circulatory disorders 3. Neutrons are emitted in the electron capture process.
C. 60Co, 3. Radiotherapy 4. Pair production is a process which involves the creation of
D. 131 1 4. Agriculture research positron-electron pair by a photon of energy 1.02 MeV.
Codes: A B C D. (a) 1 and 2 (b) 1,.,2 and 4
I . ( c) 2, 3. and 4 , (d) Ail.are wrong
(a) 4 2 3
29. A nuclide has mass number (A) ~d atomic number (Z).
(b) 4 3 2
During a radioactive process if:
(c) 4 2 .31 1. both A and Z decrease, the process is called a-decay.
(d) 3 2 4 2. A remains unchanged and Z decreases by one, the process
24. Consider the following nuclear reactions: is called ~+ or positron decay or K -electron capture.
1. I~N+ iHe~ 110+ lH 3. both A and Z remain unchanged, the proce<ss is called
2. ~Be+ :H~ ~Li + ~He . 'V~decay. •
4; both A· and Zincrease, the process is called nuclear
3. ~iMg +~He~'~JSi+ ~n isomerism.
4. I~B+ iHe~ I~N + ~n the correct answer 'is:
(a) 1, 2and 3 (b) 2, 3 and 4
Examples of induced radioactivity would include the
·reactions: (c)1,3and4 (d) 1,2and4
(a) 3 and 4 (b) I and 2 (c) 1,3 and 4 (d) 1,2,3 and 4 30. In the decay process:
a - a -~
25. Match the Column-I Radio-isotope with Column-II Medicinal A~B~C~l>
use and select correct matching:
I. A and.B are isobars
0JInnt..I C.......
2.. A and D are isotopes
(I) 60 Co (a) Leucaemia
3. C and D are isobars
(II) I3l I (b) Anaemia
4. A and Care isotones
(III) 59 Fe (c) Cancerous tumours The correct answer is:
(IV) 32 P (d) Disorders of thyroid gland (a) 1 and 2 (b) 2 and 3
(a) l-c; II--d; III-a; N-b, (c) 3 and 4 (d) I and 4
198 . G. R. B. PHYSICAL CHEMISTRY FOR COMPETITIONS

31. The nuclide X undergoes a-decay and another nuclide Y,J3-- (iii) 2iz U + ~n ---? 1~~Ba + j~Kr + 3 ~n
decay. Which of the following statements are correct?
I. The 13- -particles emitted by Y may have widely different (iv)' ~~As + fH---? ~~Mn + 9:H + 12 hn
speeds. (v) ~H + fH ---? ~He + ~n
2. The a-particles emitted by X may have widely different Answers:
speeds. A: Projectile capture
3. Thea-particles emitted by X will have almost same
B: Spallation
speed.
C: Fusion
4. The J3-particles emitted by Y will have the same speed.
D: Projectile capture and particle emission
(a) I and 3 are correct (b) 2 and 3 are correct
E: Fission
(c) :3 and 4 are correct (d) I and 4 are correct
Select the. correct answers according to the given codes:
32. Fill in the blank space with a suitable answer selected from the
Codes: .(i) (ii) (iii) (iv) (v)
list below. Write only the letter (A, B, C, ... ,etc) of the correct
answer in the blanks. (~ A DEB C
Answer {b) DCA E B
(c) ABC D E
(i) 12 C+ IH---? 13 N
61 7
(d) E D C B A
(ii) gAl + :H ---? riMg + iHe

AssertiQn-Reason TYPE QUESTIONS


The questions given below consist of two statements each rrinted as 9. (A) The reactions taking place in the sun are nuclear fusion
Assertion CA) and Reason (R). While answering these questions reactions.
you are required to choose anyone of the following fonr: (R) The main reason for nuclear fusion reactions in the sun is
Ca) If both CA) and CR) are correct and (R) is the correct that H2 is present in the sun's atmosphere so that hydrogen
explanation for (A). nuclei can fuse to form helium.
(b) Ifboth (A) and (R) are correct and (R) is not the corred 10. (A) In a radioactive disintegration, an electron is emitted by
explanation for (A). the nucleus.
(c) I f (A) is correct but (R) is incorrect. (R) Electrons are always present inside the nucleus.
(d) Ifboth (A) and .(R) are incorrect. 11. (A) In radioactive disintegrations, 2He 4 nuclei can come out
(e) If (A) is incomictbut(R)is conect. of the nucleus but lighter 2He3 can't.
1. (A) Mass numbers of most of the elements are fractional. (R) Binding energy of 2 He 3 is more than that of 2He 4 .
(R) Mass numbers are obtained by comparing with the mass 12. (A) Protons are better projectiles than neutrons.
number of carbon taken as 12. (R) The neutrons being neutral do not experience repulsion
2. (A) The activity of I g pure uranium-235 will be greater than from positively charged nucleus.
the same amount present in U 3 0 6 . 13. (A) Enrichment of U23S from a mixture containing more
(R) In the combined state, th.. activity of the radioactive abundant U238 is based on diffusion ofUF6 .
element decreases. (R) UF6 is a gaseous compound under ordinary conditions.
3. (A) a-rays have greater ionising power than 13. 14. (A) The nucleus emits J3-particles though it doesn't contain
(R) a-particles carry 2+, charge while J3-partic1es carry only 1- any electron in it.
charge. (R) The nucleus shows the transformation
4. (A) J3-partic1es have greater penetrating power than a-rays but on 1 -? P + ~ + anti-neutrino for J3-emission.
less than y-rays. 15~ (A) Any kind of exchange force helps the nucleus to be more
(R) J3-particles are lighter than a but heavier than y. destabilised.
5. (A) During J3-decay, a new element with atomic number (R) 1t-mesons are exchanged between nucleons incessantly.
greater than one is obtained. 16. (A) Nuclide 13 Al 30 is less stable than 20 Ca 40. (lIT
(R) Protons and neutrons keep on changing into one another (R) Nuclides having odd number of protons and neutrons are
through meson. generally unstable.
6. (A) The average life of a radioactive element is infinity. 17. (A) During J3-decay, a new element with atomic number
(R) As a radioactive element disintegrates, lJlore of it is greater than one is obtained.
formed in nature by itself. (R) Protons and neutrons keep on changing into one another
7. (A) Hydrogen bomb is more powerful than atomic bomb. with the help of meson.
(R) In hydrogen bomb, reaction is initiated. 18. (A) The position of an element in periodic table after emission
8. (A) The archaeological studies are based on the radioactive of one a and two J3-particles remains unchanged.
decay of carbon-14 isotope. (R) Emission of one a and two S-particles gives isotope of the
(R) The ratio of C-14 to C-12 in the animals or plants is the parent element which acquires same position in the
same as that in theatrnosphere. periodic table.
I
RADIOACTIVITY AND NUCLEAR TRANSFORMATION 199

19. (A) Nuclear isomers have same atomic number and same mass another inverse square force which is very powerful,i.e.,
number but with different radioactive properties. .. nuclear force.
(R) U(A) and U(Z) are nuclear isomers. 24. (A) K -shell electron capture is detected by analysing the
20. (A) The emission of a-particles results in the formation of wavelength of X-ray emitted.
isodiapheres of parent element. (R) The wavelength of the X-ray is characteristic of the
(R) Isodiapheres have same isotopic number. daughter element and not the parent element.
. Cl (l. -(3
25. (A) Half life ofa radioactive isotope is the time required to
21. (A) 2t~U (IlIB)~ A ~ B ~ C decrease its mass number by half.
(R) Element B will be of IIA group.
(R) Half life ,of radioactive isotopes is independent of initial
22. (A)f3-particles are deflected more than a-particles in a given
amount of the isotope.
'electric field. .
26. (A) In a nuclear fission process, the total mass of fragments is
(R) Charge on a-particles is larger than on J3-particles.
always greater than the mass of the original nucleus. .
23. (A) The nucleus of gold is stable even though there is a very
. strong coulombic repulsion among the protons. (R) Difference in the mass due to the fission of a heavy
(R) The inverse square coulomb force is exactly balanced by nucleus is converted into energy according to mass-energy
conversion.

• Set-1 2. (d) 4. .. (e)


1. (b) 3. (c) 5. (d) 6. (a) 7. {d) 8• (d)
9. (e) 10. (c) 11. (c) 12. (b) 13. (b) 14. (d) 15. (a) 16. (c)
17. (c) 18. 'Ca) 19. (e) 20. (b) 21. (a) 22. (d) 23. (d) 24. (d)
25. (c) 26. (a) '17. (1:1) 28. (a) 29. (a) 30. (e) 31. (c) 32. (d)
33. (c) 34. (d) 35. (b) 36. (b) 37 •. (c) 38. (a) 39. (c) 40. (a)
41. (d) 42. (c) 43. (a) 44; (d) 45. (b) 46. (a) 47. (a) 48. (b)
49. (b) 50. (d) 51. (c) 52. (d) 53. (b) 54. (c) 55. (d) 56; (c)
57. (a) 58 .. (c) 59. (b) 60. (d) 61. (a) 62. (a) 63. (e) 64. (b)
65. (d) 66. (a) 67. (a) 68. (b) 69. (b) ·70. (a) 71. (a) 72. (c)
73. (b) 74. (c) 75. (a) 76. (a) 77. (a) 78; (b) 79. (c) 80. (d)
81. (a) 82. (b) 83. (c) 84. (d) 8S. (b) 86. (b) 87. (d) 88. (b)
89. (b) 90. (c) 91. (a) 92. (a) 93. (e) 94. (d) 9S. (b) 96. (a)
97. (b) 98. (b) 99. (a) 100. (d) 101. (d) 102. (d) 103. (b) 10~.··(d)
lOS. (b) 106. (c) 107. Cd) 108. (b) 109. (d) 110. (b,d)· 111. (b) 112. (b)
113. (b,c) 114. (d) lIS. (c) 116. (b) 117. (d) 118. (d) 119. (a) 120" (h)
121. (a) 122. (c) 123. (d) 124. (a) 12S; {d) 126. (c) 127. (d) 128. (c)
129. (a) 130. (c) 131. (b) 132. (b) 133. (c) 134. (a) 135. (b) 136. (a)
137. (c) 138. (a) 139. (c) 140. (a) 141. (c) 142. (c) 143. (d) 144. (a)
14S. (a) 146. (a) 147. (a)

• Set-2
1. (b) 2. (b) 3. (a, b,c) 4. (b) 5. (c) 6. (c) 7. (a) S. (a)
9. (c) 10. (b) 11. (b) 12. (c) 13. (b) 14. (d) 15. (b, c, d) 16. (c, d)
17. (b, d) 18. (a, d) 19. Ca, d) 20. (a, b, c) 21. (b) 22. (d) 23. (c) .24. (d)
25. (c) 26. (d) 27. (d) 28. (b) 29. (a) 30. (b) 31. (a) 32. (a)

[~~Z~'~~~t.e~t~i
1. (d) 2. (d) 3. (b) 4. (a) 5. (b) 6. (c) 7. (b) 8. (a)
9. (c) 10. (c) 11. (c) 12. (d) 13. (a) 14. (a) 15. (d) 16. (a)
17. (b) 18. (a) 19. (a) 20. (c) 21. (b) 22. (a) 23. (c) 24. (b)
25. (d) 26. (e)
200 I' , GRS.' PHYSICAL CI-:IEMISTRyFOR COMPETITIONS

>S"f'f~tn~:flq. ." :)!ii:lii.);j ~" :(1 ~ «.i~! 14: ~.""~~~~;~1~'~


- IOBJECTIVE'QuEsTloNs ) for ( lIT ASPIRANTS ,
The following questions contain single correct option: 1 _/
1. The average life ofa W gmsample of 200 RaE is,Tseconds and 2 =e'
average energy of the (3-particles emitted isE MeV. At what I = In (2) sec]
rate in watts does the sample emit energy? '6. The rate of decay of a radioactive sample is given by RI at time
(a)' SWNoE x 10:: 16 , (b)S(fn 2)WN oE x 10-13 I) and R2 at a later time 12, The mean life of this radioactive
,T' ,T sample is: '
R I
(c) Sm;oE X 10-13 (d) None is correct (a)T= -Lx...1.
R2 II
~int: Rate ofl3-particlesemitted per second (c) T = 12 - I)
W I W In (R 2 / RI )
=:A.X--xNo=-x-:-xNo
At. w t . ' T 200 [Hint:, R2 =Roe-Al2
Energy evolved per second ' RI =~ e-
Atl

=! x ~ x No xE X 1.6 X 10- 19
x Hf Jsec~1 , On dividing the above equations, we get
T 200 ' ,
R2 =1(ll - /2)
'" 8WN oE X10-16 watt per second] RI
T '
2. In nuclear fission, 0.01% mass is converte9 into energy. The
energy released by the fission of 100 kg mass will be:
In (~~) =A(/I' - 12)
(a) 9 x 1(j5 J(b) 9xlOII kJ (c) 9 X 1017 J (d) 9 x lO13kJ
-1 = (I) 12l
---"-'---=--
, '001 A. In (R2 IR I )
lHint: l\m = 100 K-'-, =om kg
100 ' T = (II - 12) ]
E =l\md- =0.01 x (3 x 108 i J , In (R2 IR1)
=? X 10 14 J =9 X lOll kJ] 7. Isodiapheres are the atoms of two elements having same
3. The activitY ofa radioactive substance is Rl at time tl and 112 at values of:
,time t2 (> II ). Its decay constant is A.. Then: (a) p/ n (b)(p - n) (c)(n - p) (d) n x p
(a) Rill =R2/2 , (b)R 2 =Rl i·(12- 11) LHint: Isodiapheres are funned by a-decay
, , MA _ 4He~ M-4
Z-2 B
'

Z 2
(c) R2 =Rl ;(11 - 12) (d) RI R2 = constant
t2 - II (n - p)in 'fA =(M - Z) - Z =(M - 2Z)
M 4 '
'-R 1) e- At2 (n-p)in" Z:2B={(M-4)-(Z-2)} (Z-2)
lHint: ---1 =-'~-- R2 = R/,,(II - (2) ]
RI ~ e-~tl '
= {M - Z -2} - Z + 2
4. The age of a specimen 'I' is related to the daughter/parent rati!) =(M -2Z)
by the equation: :. (n - p) ofisodiapheres are same.]

(a)t=~ln(~) "
(b) 1= -.!. In
'A.
(1 + DP) 8. In a sample of radioactive material, what fraction of the initial
number of active nuclei will remain undisintegrated after half
of a half life of the sample?

,A
=-1 In ( 2 + -P (d) I D)' , (a).! (b) _1_ (c) , (d) Ii-I
4 21i Ii
5., A radioactive substance is being produced at a constant rate of lHint: 0.693 =2.303 log (N 0)
200 nuclei/seC. The decay constant ofthe substance is 1 treC -I. , 1112 'I N
After what time will the number of radioactive nuclei become
2.303 x log 2 _ 2.303 I (No),
100? Initially, there are no nuclei present. . - - - ,- oglo -
11/2 (tI/2/2) N'
(a) 1 sec (b) 2 sec , (crln (2) sec (d) In ~2).sec,
10gIO (2112) =log10 ( :0 )
".int: N =No eAl
100 =200e-)( I
(:J= ~]
RADIOACTIVITY AND NUCLEAR TRANSFORMATION 201

9. Let T be the mean life of a radioactive sample. 75% of the


[Hint: A';' 2.303 log (No)
. active nuclei present in the sample initially will decay in time: t . N
(:) 2T (b) ~ (loge 2)T (c) 4T (d) 2 (loge 2)T
= 2.303 I (100) ... (i)
10. ll8pO (t1/2 = 183 sec) decays' to 2iiPb (t1l2:; 161 sec) by. 1 og 80
Il-emission, while 2iiPb decays by H-emission. In how much
~ - 2.303 1og ( -100)
1\.--- ... (ii)
time the number of nuclei. of iiPb will reach to the 2 . N
maximum?
(a)182seC (b) 247.5 sec (c)308sec (d)194.8sec 2.303 1o (g100)
- 2.303
- -o 1 g
( 100)
-
1..1 0.693 = (3.786 x 10- J sec-I) I 80 2 N

(~r = 1!0
[Hint:
1..2 =0.693 =4.304 x 10- 3 sec-I
161
N =64]
2.303 1
tmax =;---:;- og ~ 16. A sample of radioactive material has mass 'm', decay constant
1\.1 - 1\.2 1\.2 A and molecular mass 'M.lf N A is Avogadro's number, the
2.303 I 3.786 x 10- 3 initial activity of the sample is: .
3.786 x 10- - 4.304 x 10- og 4.304 x 10- 3 (a) Am (b) A m (c) AmN A. (d) ~MeA
. M. M
== _ . 2 ..303 [_ 0.05569]
5.183 x 10- 4 17. A .-adioactive nucleus can decay by two different processes.
247.5 sec] The mean value period for the first process is Z\ .and that for
11. Fusion reaction takes place at high temperature because: the second process is Z2' The effective mean value period for
the two processes is:
(a) atoms are ionised at high temperature
(b) molecules break up at high temperature (a)Zi +Zz (b)Z\ +Zz (C)~Z\Z2 (d) ZI 2
Z

(c) nuclei break.upat high temperature 2 Zi+~


(d) kinetic energy is high enough to overcome repulsion 18. The radioactivity of a sample is RI at time Ii and R2 at time Ti.
. between nuclei If the halflife of specimen is T, the number of atoms that have
12. In the radioactive change, di~integrated in time (T2 Ii ) is proportional to:
Ap
.z
AQ
~Z+l
A-4R . A-4S
~Z-l ~Z-l
; (a) (R1Ii R 2Tz ) (Q) RI - R2
the radiations emitted in sequence are: (c) (RI R2 ) (d) (R z - R\)T
. T
(a)Il,f:\,y (b)r'l,Il,y (C)y,Il,r'l (d)r'l,y,1l
a
13. The half life of radioactive isotope is 3 hours. If the initial
LHiBt: Rate
RI =AxN 1
A x Number ofatoms of element yet not decayed.
mass of the isotope were 256 g, the mass of it remaining
undecayed after 18 hours would be: . R2 =AxN 2
(a) 12 g (b) 16 g (c) 4 g (d) 8 g Number of atoms decayed in time (T2 - TI )

LHiBt: N =No Gr n = number of half lives = ~ = 6 •


3
_ R2 _
- A A-
_ (R2 - RI ) _ T(R2 - Rd
A . 0.693
: .. )lumber of atoms decayed in time (T2 - TI ) 0<: T(R2 - RI )]
= 256GJ =4 g] 19. Halflife period of.lead is: .
(a) zero (b) infinite (c) 1590 years (d) 1590 days
14. hi an old rock, the mass ratio of 2~~ U to 2~~ Pb is found to be
20. A freshly prepared radioactive sample of half life 2 hours
S95: 103. The age of the rock is (Mean life of z~~ U is To): .
emits radiation of intensity which is 64 times the permissible
In 698 safe level. The minimum time after which it would be possible
(a) To In 1.2 (b) T. In 698 (c) T. In 1.2 (d)T. ~ to work safely with tl1is source is:
o 595 0 Iri 2 o In 2
(a) 6 hrs . (b) 12 hrs . (c) 24 hrs (d) 128 hrs
~ _ 2.303 Iog (No)
[Hint: 1\.---
t
-
N LHint:
. (l)n
N =No"2 ..
1· 1 In (595 + 103)
To t

t=Toln(698) ]
595 :0 =(~r .
~=Gr
. 595
15. 80% of the radioactive nuclei present in a sample are found to
remain ul.1decayed after one day. The percentage ofundecayed n=6
nuclei left after two days will be: t=2x6 12 hours]
(a) 64 (b) 20 (c) 46 (d) 80
202 G,R.B. PHYSICAL CHEMISTRY FOR COMPETITIONS

Select the wrong statement among the following:


(a) Antineutrino can be deteCted during ~-emission
(b) Neutrino was predicted to conserve the spin of a nuclear
reaction
(c) Synchrotron can accelerate neutrons
(d) Area of cross-section of nucleus is about 1 bam
(1 bam 10-24 cm2 )
[Hint: . Synchrotron can accekrate only charged particles, not
the neutral particles like neutron J
A radioactive atom' X' emits a I)-particle to produce an atom
'Y' which then emits an a-particle to give an atom' Z':
(1) The atomic number of' X' is less than that of' Z'
(2)-The atomic number of' Y' is less than that of' Z'
(3) The mass number of' X' is same as that of'Y'
(a) 1,2 and 3 are correct
(b) I and 2 are correct
(c) 2 and 3 are correct
(d) 3 isc!)rrect
Which one .of the following is an -exact example of artificial
radioactivity? .
[Hint: i~Ca f~Al (a) nNa + bn tiNa + y
n 20 .
=1 17 = 1.31 ~iNa + : if ----7 ~iMg + hn
P 20 13
(b) iHe+ 147N----7I~O+:H
All are stable according to nip rule but experimental
observations confirm that i~Al is radioactive with half life of . I~O +hn ----7 I~O + Y
3,7 sec]
(c) iHe + fIAI ?2p + hn
25. ~I Aiis a stable isotope. ~j AI is expected to disintegrate by: 30 30S' 0
15 P ----7 14 1+ +1 e
. (a) a-emission (b) ~-emission
(c) positron emission (d) proton emission (d) 2~~ Ac ----7 2~~ Th + ~
[Hint: Number of neutrons will be reduced by I3-decay. 22~ Th ----7 224 Ra + u.N
90 88
6n
----? ~H + _7e+
Antineutrino + Energy] 31. Consider the following decay series:
26•. For a radioactive element, a graph of log N against time has a A ----7 B ----7 C ----7 D
slope equal to: Where, A, Band C are radioactive elements with halflives of
A A 4.5 sec, . 15 days and I sec respectively and D is
(a) + 2.303A (b) + ~- (c) - 2.303 (d) - 2.303A
2.303 non-radioactive element. Starting with I mole of A, the
At . number of moles of A, B, C and D left after 30 days are:
[Hint: log No logN = (a) one mole of D and none of A, Bar C
2.303
(b) 31 4 mol of B, 114 mol of D and none of A or C
IogN ==l(~) t+ log No (c) 1/4 mol of B, 3/4 mol of D and none of A or C
2.303
(d) 1/2 mol of B, 114 molofC, 114 mol of D and none of A
Y=Mx+C
(e) 1I4 mol of each A , B, C and D
-'A
.. ;.. Slope (M) = 1 32 • Consider the following nuclear reactions:
2.303
238 M --------7 x N + 2 4He
92 y 2
27. Two elements P and Q have half lives of 10 and 15 minutes
:AN
y
---7 A L
B
+2 (JA +
respectively. Freshly prepared samples of each isotope
initially contain the same number of atoms as each other. After the number ofnentrtms present in the element 'L' is:
.
30 mmutes, the ratIO
, number of P atoms ,
WIll be:
WI~ . ~IM
number of Q atoms (c) 140 I "J:. (d) 146
(a) 0,5 (b) 2 (c) I (d) 3 33. If nl is the nutrlber of radio-atoms present at time 't " the'
[Hint: In 30 minutes, there will be 3 half lives of P and 2 half following expression will be a constant:
lives ofQ.
:. Number of P atoms will be 1/8 th and number of Q atoms will
be l/4th of original atoms. (a)
t
Then, Number of atoms ofP = 118 == 1 ,i. e., 0.5]
(c) din nl
~umber of atoms of Q 1/4 2
. dt
RADIOACTIVITY AND NUCLEAR TRANSFORMATION 203

[Hint:
X ----7 Y
tl12 := 6.93 X 109 yrs]
=A What is the age of the rock?
nt dt
(a) 2.079 X 1010 years (b) 1.33 x 109 years
~(lA) -A (constant) 1 (c) 1.94 x 1010 years (d) IOld years
dt
a-emission ~ -emission ~ -emission [Hint: No =1+ 7 = 8 (Initial moles ofX )
34. 2~j Bi )A ------7) B )C
N I . (Remaining moles of X)
p-emission u-.emission
We know;
---4)D )E

'E' is.an element of stable nucleus. What is the element 'E'? 0.693 := 2.303 10gr N Q.)
tllz t \N
(a)2~iTI (b) 2i;Pb (c) 2~3Hg (d) 25J: Au
0.693 == 2.303 log ~
35. A radioactive element decays. to one third of its initial amount 6.93 x t 1
in time' t'. What fraction of the element would be left after 0.5
t time? t 2.079 X 10 10 years]

(a'73
I 1
(c)-
3
(d)j1 38. If the relation between time of decay (t) and half life period
(t1l2) is (t 4 t il2 ); the relation between t and mean life (1)
1S:
[Hint:
..
A=--Iog
2.303· ·lfNO\
-I
. t N /
(a) In 2 (b) Tf4 In 2 (c) 4T In 2 (d) 2T In 2
T2
-N = 1.mtIme
. "
t Following questions may have more than one correct options:
No 3
1. Which of the following nuclei are doubly magic?
2.303
A=--loglO 3 ... (i) (a) iHe (b) I~O (c) 2~~Pb (d) 2~~ U
t
After 0.5 t time: 2. Which of the following make up an isotonic triad?
A = 2.303 10 .l( No) ... (ii) (a) 146c, 1680, 157N (b) ~~Ge, ~~As, nGa
.. O.St g N
40 40 40 (d) 233 U 232 Th, 239 p.
(c) i8 Ar, 19 K, 20 Ca 92 , 90 94 U
Equating (i) and (ii) we get
N 1 3. In the decay proceSs:
N =J3] -u -~ -f:l
o A~B~C~D .iViiJ
36. Two radioactive isotopes A and B of atomic mass X and Yare (a) A and Bare isodiapheres (b) A and D are isotopes·
mixed in equal amount by mass. After 20 days, their mass ratio (c) B, C and D are isobars (d) A and Care isotones .
is found to be 1 : 4. Half life of 'A' is 1 day. What will be the 4. A nuclide X undergoes a-decay and another nuclide Y
halflife of B ? undergoes ~-decay. Which of the following· statements are
(a) 1.11 day (b) 0.6237 day correct? .
X y (a) The tl-particles emitted by Y may have widely different
(c)O.ll-day (d) 1.11- day
Y X •speeds
[Hint: Let 1 g of both A and B are taken initially. WA and WB (b)The a-particles emitted by X may have widely different
are the amounts left after 20 days. speeds .
A = 2.303 10 _I (c) The a-particles emitted by X 'will have almost the same
A 20 gW speed
A
, 2.303 I 1 (d) The tl-particles emitted by Y will have the same speed
AE = - - og
20 WB 5. Which among the following nuclides is/are likely to be stable?
A - A = 2.303 10 WE (a) f~P (b) tiMg
(c) Il~In (d) Iljg Cd
A B 20 gW
A 6. Which among the following is/are fissile?
2.303 log 4 = 0.0693 (a) 2395ZU (b) 2~~U (c) 2~pu (d) 2~~pu
20
0.693 7. Select the correct statements among the following:
0.0693 (a) The decay of mass during miclear fusion and nuclear
fission are 0.1 % and 0.23 I % respectively
= 0.693 ~ 0.0693
0.6237 (b) Lesser is the half life, more dangerous is the radioactive
I
element
0.693
tl/2B = 1.11 day] (c) K-electron capture emitsy-rays
0.6237
37. A sample of rock from the moon was found to contain the (d) Nuclear forces are about 1021 times stronger than
elements X and Y in 1:7 ratio by mole. Element X is coulombic forces
radioactive, it decays to Y with half life of 6.93 x 109 years
204 G.R.B. PHYSICAL CHEMISTRY FOR COMPETITIONS

8. A radioactive element has atomic number' Z' and mass nU.mber 11 .. The correct starting material and end product of different
'A'. Select the correct statements among th~ following: disintegration series are:
(a) Both 'A' and '2' decrease ina-decay (a) 232Th,208Pb . (b) 235U,206Pb
(b) Both 'A' and 'Z' remain unchanged in y-decay (c) 238U,207Pb (d) 237 Np ,209 B i
(c) 'A' remains unchanged and'Z' decreases by one; the
12. Select the wrong statement(s):
processis called ~+ (positron) decay or K -electron capture
(a) Nuclear isomers contain the same number of protons and
(d) Both 'A' and 'Z' increase in the nuclear isomerism neutrons
9. When nucleus of an electrically. neutral atom undergoes a (b) The decay constant is independent of the amount of the
radioactive decay process, it will remain neutral after the substance taken
. decay if the process is:
(c) 1 curie 3.7 x l(YO dis
(a) an a-decay (b) a ~-decay
(c) a y-decay (d) a K -capture process (d) Actinium series starts with 238U
, 10. Which of the folloWing. is/are characteristics of nuclear 13. Which of the following are synthetic elements? .
forces? (a) Tc (b) Pu (c)Np tal U
(a)These forces operate within small distances of2 x 10-13 em 14. Which of the folloWing nuclides belong to actinhim (U23S)
series? " . . ' .
(b) These forces drop to zero rapidly at a distance greater than (a) 213 pO .(b) 21Spo (c) 222Rn (d) 207Pb
104 x loZ fermi
15. In a nuclear reactor, heayy water is used to:
(c) They follow inverse square 1aw (a) transfer the heat from the reactor
(d) They are stronger than electrostatic forces of attraction (b) provide high speed neutrons for the fission reaction
. (c) reduce the speed of fast moving neutrons
(d) increase the speed of neutrons

'~""
[ ..... ~, '.

• Single correct option


1. (a) 2. (b) 3. (c) 4. (c) 5. (b) " 6. (b) 7. (c) 8. (c)
9. (d) 10. (b) 11. (d) 12. (b) 13. (c) 14. (b) 15. (a) . 16; (c)
17. (d) 18. (d) 19. (b) 20. (b) 21. (c) 22. (c) 23. (a) 24. (d)
25. (b) 26. (c) 27. (a) 28. (c) 29. (d) 30. (c) 31. (d) 32. (b)
33. (c) 34. (b) . 35. (a) 36. (a) 37 . (a) 38. (e)

• One or more than one correct options


1. (a, b, c) 3. (a, b, c)
2. (a, b) 4. (a, c) 5. (b, d) 6. (a, c) 7. (a, b, d)· 8. (a, b, c)
·9. (c, d) 10. (a, b, d) 11. (a; d) 12. (a; b) 13. (a,b, c) ,14. (b, d), 15. (a, c)
RADIOACTIVITY AND NUCLEAR TRANSFORMATION 205

IntegerAnswer TYPE QUESTIONS


4. . ~ Be captures a K-electron into its nucleus. What will be the
This section contains 7 questions. The answer to each mass number of resulting nuclide?
of the questio~ is a single digit integer, ranging from [Hint : In K-electron capture, a proton of nucleus changes into
oto 9. lithe correct answers to question numbers X, Y, neutron. '
Z and W (say) are 6, 0, 9 and 2 respectively, then the lH+ _~e~Jn
correct darkening of bubbles will look like the given
figure: ~Be+ _~e~ ~Li
5. . 2~
Th disintegrates to 2~~ Pb. How many of ~-particles are
evolved?
1. The total number of a and ~ ~articles emitted in the nuclear 6. What mass in milligram of 226 Ra, whose (t1/2 ::: 1620 yr), will
reaction: be required to yield I millicurie of radiation?
2~u.~ 2~iPb is : (lIT 2009) '7. The number of neutrons emitted when 2~~U undergoes
2. The t) 12 of a radionuclide is 8 hours. Starting with 40 g of the controlled nuclear fission to 11iXe and ~Sr is: (lIT 2010)
isotope, the amount in gm remaining after one day will be: [Hint: 2;iu~ It!xe+ ~Sr+ 3[Jn]]

3. If~ quantity of a radioactive nuclide disintegrates in two

hours, its half-life (in hour) will be:

[.~~"---------------------II
1~ (8) 2. (5) 3. (1) 4.(7) 5.. (4) 6. (1) 7. (3)
..
206 G.R.B. PHYSICAL CHEMISTRY FOR COMPETITIONS

4. The analysis of a rock shows the relative number of 238U and


• Passage 1 206Pb atoms (Pb/U = 0.25). The age of rock will be:
There are four radioactive decay series called thorium (4n); (a) 2.303 x 4.5 x 109 log 1.25 (b) 2.303 x 4.5 x 109 log 0.25
uranium (4n + 2}' actinium (4n + 3)and neptunium (4n + l)series. 0.693 0.693
Neptunium series is artificial while other three series are natural.
(c) 2.303 x 4.5 x 109 log 4 (d) 2.303 9 x 0.693 log 4
End products ofeach radioactive. decay series have stable nuclei. All 0.693 4.5 x 10
natural decay series terminate at lead but neptunium or artificial
Nathan Thomson, one of the first inhabitants of Lord Howe
series terminates at bismuth.
Island, decided to plant some European deciduous trees in his
Answer the following questions:
garden. Unfortunately the exact timing of planting the seeds is not
1. The end product formed in the disintegration of 2~~Ra is: known. Over the years, pollen produced by the trees accumulated at
(a) 2~41 Tl (b) 2~~Pb (c) Z;~Rn (d) 2~~Bi the bottom of the lake near Nathan ~ house. Very small quantities of
2. Actinium series begins with an isotope of: radioactive 210 Pb (t1l2 = 22.3 years) were deposited at the same
(a) actinium (b)'radium (c) uranium (d) polonium time. Note that European decid1Jrw~ ~:'." '. "",;.ifnate in their first year
3. 2l&Rn is a member of actinium series. Another member of (Jfgrowth.
same series is: In 1995, a team of researchers sampled a sediment cOIefromjh£L_~__ _
(a)2~2U (b)2~;Ac (d) 2~PO bottom of the lake. The examination of sediment core found that:
(a) Pollen of trees first occurs at the depth of50 cm.
4. The end products of uranium and actinium series are
(b) The activity of21o Pb at the top of sediment core is
respectively:
(a) 206 pb, 207Pb (b) 206Pb, 208Pb
356 Bq / kg and at 50 cm depth 1.40 Bq / kg.
Answer the following questions: '
(c) 207 P b, 208Pb (d) 206Pb, 208Bi
5. In what year did Nathan Thomson plant the seeds?
5. The starting isotope and the end product isotope of actinium (a)1719±2 (b)1819±2 (c)1519±2 (d)1919±2
series are:
[Hint: 0.693 = 2.303 log (No)
(a) 2~~Ac and 2~~Pb (b) 2~ U and 2~~Pb til 2 t.ge N
(c) 2~~ U and 2~~ Pb (d) 2~~ U and 2~~ Pb 0.693 2.303
- - o1g (356)
-
22.3 t.ge \ 1.40
• Passage 2 tage = 176 yrs] ,
6. Which step in the decay scheme explains how 2JOPb ends up
Initially the earth was a fire-ball; slowly it has cooled to form' in rain water while its parent 238U is only present in earth's
earth crust and its different layers. At the beginning 239~U was crust?
present and no 2~~Pb was there. With the passage of time, uranium (a) 238 U 234U (b) 234 U _ 230Th

decayed to 2~~Pb, The decay process is: (c) 230 Th (d) 226 Ra __ 222Rn

238 U 2~~Pb; tll2 of 238U = 4.5 x 109 yrs


92 (xa, • Passage 3
Answer the following questions: In the atmosphere, carbon dioxide is found in two forms, i.e.,
1. x and y in above decay series are: 12eo2 and Heo2 . Plants absorb e02 during photosynthesis. In
(a) 6, 8 (b)8,6 (c) 8, 8 (d) 6, 6 presence of chlorophyll, plants synthesise glucose.
hv
2. A sample of rock from South America contains equal number
6eOz + 6HzO ~ C 6H 1z06 + (£)2 i
of atoms of 238U and 206Pb. The age of the rock.will be:
(a) 4.5 x 109 years (b) 9 x 109 years Half life 0 / 4 e IS 5760 years. The analysis ofwooden artifacts for
14 e and 12e gives usefUl infonnation for determination of its age:
(c) 13.5 x 109 years (d) 2.25 x 109 years
All living organisms; because of their constant exchange 0/eo2
.
[H mt: - - Iog (No)
0.693 =2.303 - with the surroundings have the same ratio of 14e to 12e, i.e.,
,t1l2 tage N 1.3 x 1O~12. When an organism dies, the 14e in 'it keeps on decaying
0.693 = 2.303 log (~) asfollows:
4.5 x tage 1 14e
6
~ 14N
7
+ ~l°e + Energy
9
tage 4.5 X 10 yrs. Thus, the ratio 14e /12 e decreases with the passage of time. We
No = 1+ 1 = 2, N = 1] can measure the proportion 0/4 e in the remains of a dead organism
, 3. Atomic mass of 238 U is 238.125 amu. Its packing fraction will and determine how long ago it died. The method of carbon dating
be: can be used to date anything made of organic matter, e.g., bone,
(a) 5.25 (b) 0.125 (c) 12.5 (d) 1.25
RADIOACTIVITY AND NUCLEAR TRANSFORMATION 207

skeleton, wood: etc. Using carbon dating, materials have been dated 3. Select the incorrect information about 232 Th :
to about 50,000 years with accuracy. (a) It belongs to third group of actinide series
Answer the following questions: (b) 232Th is fissile material
1. 14C exists in atmosphere due to: . (c) It is a fertile material
(a) conversion of 12C to 14C (d) It belongs to 4n series
(b) combustion of fossil fuel
(c) bombardment of atmospheric nitrogen by cosmic ray • Passage 5
neutrons
(d) none ofthe above Geiger-Nuttal proposed that the activity oj a nucleus is inversely
2. A wooden piece is 11520 yrs old. What is the fraction of 14C proportional to its half or average life. Thus, shorter the half life oj
activity left in the piece? an element, greater is its radio(J:ctivity, i.e., greater the number oj
(a)0.12 (b) 0.25 (c) 0.50 (d) 0.75 atoms d.isintegrating per second. Half life and average life are
related with each other.
[Hint: 0.693= 2.303 log OJ\ .(N
tll2 . tage N 0:693 _ 1: X 0 693 or
A - .
0.693 = 2.303 10 (N o'I
5760 11520 g~N / Answer the following questions:
1. The halflife periods of four isotopes are given:
~ =0.25] 1= 6.7 years; II = 8000 years; HI = 5760 years; IV =2,35~x-
No
105 years.
3. In the process of photosynthesis, O2 gas is released from:
Which of these is most stable?
(a) CO2 . (b) H 2 0
(c) both H 2 0 and CO 2 (d) mechanism is not confirmed
WI MIT ~rn ~N
2. Mark the incorrect relation:
4. A piece of wood from an archeological' source shows a 14 C
(a) No = NeAl (b) 't' = 1.44to.5
activity which is 60% of the activity found in fresh wood .. ~.
today. The age of archeological sample will be:
(a) 4246 yrs (b) 4624 yrs ( c) 4628 yrs (d) 6248 yrs (c) N = No ( nn
-) (d) tll2 A In 2
. \2
5. A sample of ancient wooden boat is found to undergo
3. Half life of a radioactive element is 10 years. What percentage
9 dpm g - 1 of 14 C. What is the approximate age of the boat? .
of it will decay in 100 years? .
The rate of disintegration of wood recently cut down is
(a) 0.1% (b) 100% (c) 99.9% (d) 10%
15 dpmg- 1 Of I4 C.
(a) 4246.5 yrs (b) 5384 yrs (c) 4628 yrs (d) 2684 yrs
• Passage 6
• Passage 4 It has been estimated that the total energy radiated by the sun is
3.8 x 1026 J per second. The source oj energy oj stars is a
The mineral monazite is a rich source oj thorium, available in
thermonuclear reaction called nuclear Jusion. Fusion reactions are
large quantity in Kerala. A typical monazite sample contains 9%
not controlled. It is presumed that the. energy oj stars is due to two
Th0 2 and 0.35% U 30 g • 20sPb and 206Pb are the stable end products processes called proton-proton cycle and carbon-nitrogen cycle.
in the radioactive decay series oJ232 Th and 238 U r.espectively. All the Fusion cannot rake place at ordinary temperature. Thus. hydrogen
lead in monazite is oj radiogenic origin. . bomb uses a small fission bomb, which on explosion causes the
The isotopic ratio oj 208 Pb Th was Jound to be 0.104. The temperature to rise very high, about 107 K. We have yet to see how a
. half lives oj Til and U are 1.41 x 1010 years and 4.47 x 109 years hydrogen bomb can be used Jor peaceJul life-sustaining purpose .
., Energy released in the process aJJusion is due to mass deject. It is
respectively.
. also called Q-value.
Answer·. the following questions:
1. The time elapsed since the formation of monazite sample will Q t:,.mc2 , t:.m = mass deject
be:. Answer the fono"!;:~.questions:
(a) 1.34 x 109. years (b) 2.01 x 109 years 1. The bindine; energy per micleon of tH and iHe are l.l MeV
(c) 1.41 x 1010 years (d) 4.47 x 109 years and 7 MeV respectively. If two deuteron nuclei react to form a .
'
[H mt: - - = 2.303 1og -No)
0.693. l' single helium nucleus, then the energy released is:
Ca) 13.9 MeV (b) 26.9 MeV (c) 23.6 MeV (d) '19.2 MeV
tl12 Th tage . N
2. Mass equivalent to the energy 931 MeV is:
'0.693 == 2.303 log (1.104) (a) 6.02 x 10-27 kg (b) 1662 x 10-27 kg
tage
(c) 16.66 x 10-27 kg (d) 16.02 x 10-27 kg
tage 2.01 X 109 years]
3. Fusion reaction takes place at about:
2. . Estimated isotopic ratio of 206 Pb /238 U in the monazite
.(a)3xI02 K (b)3x 103 K
sample will be:
(a) 0.166 (b) 0.266 (c) 0:366 (d) 0.466
• (c) 3 X 104 K (d) 3 x 106 K
208 . G.R.B. PHYSICAL CHEMISTRY FOR COMPETITIONS

4. A star has 1040 deuterons. It produces ~nergy via the process: • Passage 8
r H + rH~ iH+ lH
Radioactive decay follows first order kinetics. The disintegration
rH+ iH~ ~He+ ~n of radioactive elements does not depend on the temperature. Unlike
If the average power radiated by the star is lO16 W, then the chemical first order reactions, . the nuclear reactions are also
deuteron supply of the star is exhausted in a time of the order independent of catalyst. Mean life and half life of nuclear decay
1 0.693'
of: process are 't = A and t!l2 = A . There are. a number of
(a) 106 sec (b) 108 sec (c) l(yzsec (d) lO16 sec
radioactive elements in nature; their abundance is directly
S. In a nuclear reaction,
proportional to half life. Amount remaining after· n half lives of
rH+ rH~ ~He+ ~n radioactive elemeniscan be calculated using the re{ation:
if the masses ofrHand ~He are 2.014741 amu and 3.016977
amu'respeCtively, then the Q-value of the reaction is nearly:
(a) 0.00352 M~V (b) 3.27 MeV
(c) 0.82 MeV (d) 2.45 MeV Answer tbe f{dlowing questions:
1. Which is/are true aboutthe decay constant?
• Passage7 (a) Unit 00.. is time- I .
(b) A is independent of temperature
. Moderator is a material whichis used to slow down the neutrons (c) A depends on initial amount of element taken
produced during l1UclearfISsion. The neutrons from the source are of
(d) Adepends on the nature of radioactive element
high speed and energy. Heavy water or graphite moderators slow
2. Amount of radioactive element (activity) decreases with
down the speed of the neutrons. The energy offast moving neutrons
passage of time as: .
decreases from 2MeV to 0.02535 eV;- it corresponds to the velocity of
(a) linearly (b) exponentially
220 m sec -1. At this velocity, the neutrons are in thermal equilibrium
with the moderator. Such neutrons are called 'thermal neutrons '. (c) parabolically (d) all of these
Thermal neutrons cause forther fission reaction. The essential 3. Half life of 6OCO is 5.3 yrs, the time taken for 99.9% decay
characteristics of moderators are: will be:
(i) its molar mass must be low, (a) 0.53 yrs . (b) 53 yrs
(ii) it should not absorb neutrons, (c) 530 yrs (d) 5300 yrs
(iii) it should undergo elastic collisions with neutrons. 4. Rate of radioactive decay is:
Answer tbe following questions: . (a) independent of time
1. The moderator in a reactor: (b) independent of temperature
(a) absorbs neutrons (c) dependent on catalyst
(b) accelerates neutrons (d) dependent on the amount of element not yet decayed
(c) slows down neutrons S. Select the correct relations:' .
(d) absorbs thermal energy produced in the reactors 0.693 1
(a) tl/2 '= - - (b) 1: = ~
2.. A good moderator should: . A A

(a) not be a gas only (c) 1: = 1.44 X t1/2 (d)'t=~·


(b) not have appetite for neutrons only 0.693
( c) be light in mass number only
(d) be all the above three .• Pas,sage 9 .
3. Which of the following is not used as a moderator?
In the disintegration ofa radioactive element,. a and ~-particles
(a),Heavy water (b) Graphite are evolved from the nucleus. .
(c) Beryllium (d) Sodium
. 4. Moderator in the reactor yields: ~n ~ iH + _? e + Antineutrino + Energy
(a) fast moving neutrons ,· 4 0
. (b) thermal neutrons . 4 ,H ~ 2He+ 2 +Ie + Energy .
(c) magnetic neutrons
Then, emission oj these particles changes the nucfear
(d) el\!Ctric neutrons configuration and results into a daughter nuclide. Emission of an
S. Which among the following characters make graphite a good a-particle results into a daughter element having atomic number
moderator? lowered by 2 and mass number by 4; on the other hand, emiSsion ofa
(a) Cross-sectional area of graphite is very high ~-particle yields an element having atomic number raised by one.
(b) Graphite'is a good conductOl: of electricity Soddy and Fajan proposed that the daughter nuclide may occupy
(c) There is elastic collision between graphite and neutron different positions in the periodic table.
(d) Graphite has weak van der Waals' force between two layers
RADIOACTIVITY AND NUCLEAR TRANSFORMATION 209
."
Ans~er the following questions: 4. How many a and ~-particles should be emitted from a
1. Which of the following combinations give finally an isotope radioactive nuclide so that an isobar is fonned?
of the parent element? (a)la,l~ (b)la,2~ (c)2a.2~ (d)n~
(a)a,a,~ (b)a,y,a (c)a,~,~ (d)~,y,a 5.· Select the correct statements among the following:
2. A radioactive element belongs to IIIB group; it emits one 'a' (a) Emission of a ~-particle results into isobar of parent
and one ~-particle to fonn a daughter nuclide. The position of element
daughter nuclide will be in: • (b) Emission of a ~-particle results into isodiaphere of parent
(a) IIA (b) IA (c) lIB (d) IVB element
3. During ~-decay, the mass of atomic nucleus: (c) Emission of one a and "two ~-particles results into isotope
(a) decreases by 1 unit (b) increases by I unit of the parent element
(c) decreases by 2 units (d) remains unaffected (d) Emission ofy-radiations may yield nuclear isomer

[ri~
~1. 1. (b) 2. (c) 3. (a) 4. (a) 5. (b)
~2. 1. (b) 2. (a) 3. (a) 4. (a) 5. (b) 6. (d)
~3. 1. (c) 2. (b) 3. (b) 4. (a) 5. (a)
Pauage4. 1. (b) 2. (c) ·3. (b)
Patage5. . 1. (d) 2. (d) 3. (c)
P~6. 1. (c) 2. (b) 3. (d) 4. (c) 5. (b)
Pas. . 1., 1. (c, d) 2. (d) 3. (d) 4. (b) 5. (a, c)
Pauagt8.. 1. (a, d) 2. (b) 3. (b) 4. (b, d) 5. (a, b, c, d) " ."<

Pauage9. 1. (c) i} 2. (a) 3. (d) 4. (d) 5. (a, b, c, d)


I
210 G.R.B. PHYSICAL CHEMISTRY FOR COMPETITIONS

r;? SELF ASSESSMENT r~


ASSIGNMENT NO.·3

SECTION-I 9. The decay of mass during nuclear fission and fusion are:
(a) 0.1% and 0.231% (b) 0.231% and 0.1%
Straight Objective Type Questions (c) 0.4% and 0.2% (d) 0.3% and 0.3%
This section contains 10 multiple choice questions. Each [Hint: Greater mass is converted to energy in nuclear fusion as
question has 4 choices (a), (b), (c) and (d), out of which only compared to "that of fission.].
ope is.correct. . 10. On large 'scale, tritium is produced by which of the following
X
1. I~ quantity of a radioactive substance decays in 2 hrs, its nuclear reactions? (SCRA 2009)
half life would be: (a) ~Li ~11 -? iHe + ~T
(a) 1 hour (b) 45 minutes
(b) ~ D + ~D -? ~T + : H
(c) 30 minutes (d) 15 minutes
2. Radio carbon dating is done by estimating in a specimen: (c) liN + 611 -? I~C liT

(VITEEE 2007) (d) I j N + : H -? + Other fragments


(a) the amount of ordinaJY carbon still present
(b) the amount of radio carbon still present SECTION-II
(c) the ratio of amount of I~ C to I ~ C still present Multiple Answers Type Objective Questions
11. Which of the following will emit positron?
(d) the ratio of amount of I~ Cto Ii, C still present
(a) f~p (b) I;N (c) fH (d) I~C
3. Which of the following are correct with respect to the unit of
radioactivity?
(i) The SI unit of radioactivity is curie (Ci)
[Hint: ~~P l; = I ) and
l
! 11
p
I"
< I J these nuclei emit
~
(ii) ICi 3.7x 10- 10 dis S-I positron.
(iii) 1 Bq 3.7 X IP- IO Ci ~~P ~ ;~Si + +~e
(iv) The SI unit of radioactivity is becquerel (Bq) IjN ~ I~C + +~el
{v} I Ci = 3.7 x 1010 Bq 12 .. If!!.. ratio is less than I, the nuclide can:
(a) (i) and (iii)
p .
(b) (iv) and (v)
(c)(i) and (ii) (d) (ii) and (iv) (a) K-capture (b) emit positron
(e) (i) and (v) (c) emit p-particle (d) emit a-particle
. 4. A freshly cut tree and a wooden artifact have 30.4 and 15.~ 13. For radioactive decay:
countsg- I min-I ofCI4 ofhalflife of5700 years. The age of (a) tJ/4 == 2 tli2 (b) t7/X 3 tli2
the artifact in years would be: . 10
(c) t99% = 2 (d) == - 150%
(a) 2850 (b) 5700 (c) 570 (d) 6930 . 3
(e) 11400 14. Which of the following statements islare cOlTect?
5. The radioactive isotope of cerium-137 of weight 8g was (a) Nuclear fusion produces more energy than nuclear fission
collected on 1st Feb. 2006 and kept in a sealed tube. On 1st (b) Nuclear fuSion takes place at very high temperature (106 K)
July, 2006, it was found that only 0.25 g of it remained. The
(c) Nuclear fusion yields radioactive product
half life period of the isotope is: IPET (Kerala) 2007]
(d) Nuclear fusion inyolves chain reaction
(a) 37.5 days (b) 30 days
15. Decrease in atomic Mmber is observed during:
(c) 25 days (d) 50 days
. (a) a-emission (b) p-emission
,(e) 60 days
(e) positroh emissiop C (d) K-capture
6. : The number of a and p-particles emitted in the nuclear
reaction SECTION-III
2:;foTh ~ 2~~Biare: (DCE 2007)
Assertion-Reason Type Ques~iions
(a) 4a and If) (b) 3a and 7p
This section contains 4 questions. Each question contains
(c) 8a and If) (d) 4a and 7p Statement-l (Assertion) and Staternent-2 (Reason). Each
7. A cyclotron C!.lnnot accelerate: question has following 4 choices (a), (b), (c) and (d), out of
(a) protons (b) deutrons which only one is COlTect.
. (c) neutrons / (d) electrons . (a) Statement-l is true; statement-2 is hue; statement-2 is a
8. Isotope 1128 has no medicinal importance because: correct explanation for statement-I.
(a) it is non-radioactive (b) it is poisonous . (b) Statement" I is true;statement-2 is true; statement-2 is not
(c) it is radioactive (d) none of these a conect explanation for statement-I.
I
RADIOACTIVITY AND NUCLEAR TRANSFORMATION I 211

(c) Statement-l is true; statement-2 is false. p q r s

Ie 0) 010
(d) Statement-l is false; statement-2 is true.
a
16. Statement-I: ~-particles are emitted by nucleus.
Because
Statement-2: Following transformation takes place in ~-emission. b 10 e Oi0
, 1on~, IH
ee0 0
0
1 +_I e
17. Statement-I: Nuclide i&Ca is less stable than igca. c
Because
'Statement-2: Nuclides having even number of nucleons are
stable.
d
® 0) 0 01
18. Statement-I: Energy is released in the nuclear fusion of 21. Match the Column-I with Column-II:
hydrogen nuclei to form helium nuclei.
Column-I Column-II
Because
Statement-2: Binding energy per nucleon of helium is greater (a) ;D + 1T --7 iHe + ~n + Energy (p) {3-emission
than hydrogen. (b) ~Be+ iHe~ I~C+ ~n (q) Artificial transmutation '
19. Statement-I: 1~!Ba + e- ~ I~~CS + X-ray 24M
It is an example of K -electron capture.
( c ) 12, g + 4H
2 e~
27S'
14 1 +
1
on (r) Discovery of neutrons
, Because ( d)ln~lH+
o 1-1
0e (s) Hyqrogen bomb
Statement-2: Atomic number of daughter nuclide decreases 22. Match the Column-I with Column-II:
by one unit in K -electron capture.
'Column-I Column-II
[Hint: Nucleus may capture electron from K -shell and the
vacancy is filled by electrons from higher shells; X-ray is released (a)n p++ ... .. (p) Positron emission
in this process.] (b)p+ ~n+ ... .. (q) {3-emission
20. Statement-I: The plot of atomic number (v-axis) versus
(c) X-ray emission (r) K -electron capture'
number of neutrons (x-axis) for stable nuclei shows a
curvature towards x~axis from the line of 45° slope as atomic (d) 4 [\ H] ~ ..... + 2 ~+ + Energy ,(s) q-ernission
number is increased.
Because SEC1"ION..V
Statement-2: Proton-proton electrostatic repulsions begin to Linked Comprehension Type Questions
, overcome attractive forces involving protons and neutrons in Nucleus of an atom resembles with a drop ofliquid. Density of
heavier nuclides. (lIT 2008) nucleus is very high, i.e., 108 tonne I cc or 130 trillion tonnes
[Hint: m -3. This density is about a trillion times greater than that of
t
Number of
water. Density of nuclei of all elements are same, it is
independent of atomic number or atomic mass. However, the
protons radius of nucleus depends on the mass number. Surface
tension of nucleus is also very high, i.e., about 1.24 x 1018
I
-
Number of neutrons
In heavier nuclei, attractive forces between proton-neutron
,overcome proton-proton electrostatic repulsion.]
times, the surface tension of water.
Answer the following questions:
23. The radius of l~ C nucleus is:
(a) 5 x 10- 15 m ' (b) 1.4 x 10- 15 in
SEC1-ION-IV (c) 3.5 X 10- 15 m (d) 6 x 10- 15 m
Matrix-Matching Type Questions
[Hint: r IQ x A Ji3 where A =Mass number
This section contains 2 questions. Each question contains
statements given in two columns which have to be matched. ro = 1.4 x 10- 15 m]
Statements (a, b, c and d) in Column-I have to be matched with 24. Ratio of volume of atom and nucleus is:
statements (p, q. r and s) in Column-II. The answers to these (a) 108 : 1 (b) 1015 : 1 (c) 1013 : I (d)Hj2:1
questions have to be appropriately bubbled as illustrated in the
25. Radius of nucleus is directly proportional to:
following examples:
(a) A2 (b) A l13 (C)[A]3 (d) A
lfthe correct matches are (a-p, s), (b-q, r), (c-p, q) and (d-s),
then correct bubbled 4 x 4 matrix should be as follows:

[,4~ ,
I
1. (c) 2. (c) 3. (b) 4. (b) 5. (b) 6., (a) , 7. (c) 8. (a)
9. (a) 10. (b) 11. (a, b) 12. (a, b) 13. (a; b, c, d) 14. (a, b, c) 15. (a, c, d) 16. (a)
17. (d) 18. (a) 19. (b) 20. (c) 21. (a -s)(b -q, r)(c cq) (d - p)
22. (a -q)(b -p)(c-r)(d -s) 23. (c) 24. (b) 25. (c)

4
STATES OF MATTER

4.,.1- INTRODUCTION.
Matter is capable of existing in three physical states: solid, liquid
and gas. Matter is anything which has mass and occupies space.
Any substance can exist in either of three states depending on
temperature and pressure. Liquid and solid states are condensed
• •••
••••
••
••

•• •
Cool or

....~r_es_s_ur_;
increase

Heat or
bJ ~
Heat
~

••••
decrease
states as they have much higher densities. Both liquids and gases pressure
are termed as fluids as they have flowing ability. ••• Liquid Solid
The three states of a substance are interconvertible by Gas
variation of temperature and pressure. A liquid state is
intermediate between the gaseous state (complete molecular Fig. 4.1 Schematic representation of states of matter
randomness) and the solid state (orderly arrangement of Some common characteristics of three forms of matter are
molecules). summarised below:

S,No. GIlStl Liquids . Solids


1. No defmite shape; can fill unifonnly any vol- No definite shapoe; i.e., assume the shape of Have definite shape and definite volume.
ume available; have indefinite volume. the container; have definite volume.
2. Extremely disordered particles; much empty Disordered clusters of particles; quite close to Ordered arrangement of particles; distances
space; particles have random motion; each other; random motion; considerable between the particles are very small and
intennolecular attraction very small but high intennolecular attraction; kinetic energy is fixed. Intennolecular forces are high; vibra-
kinetic energies (particles are free to move in less (particles free to move but always remain tional motion only.
all directions). . in contact with each other).
3. Low density. Intennediate density. High density .
. 4. Compressible. Slightly compressible. Nearly incompressible.
5. Fluid, diffuse rapidly. •Fluid; diffuse through other liquids. Not fluid; diffuse very slowly only through
!
solids.

SECTION 1 : GASEOUS STATE


4.2 TH6 GASEOUS STATE (li) Gases expand without limit. A gas sample can occupy
completely and uniformly the volume of any container.
Ofthe three states of matter, the gaseous state is the simplest and
shows greatest uniformity in b'ehaviour. Gases have the following (iii) Gases exert pressure on the walls of the container
general characteristics: unifOlmly in all directions.
(i) Gases are highly compressible. These can be compressed (iv) Gases diffuse rapidly through each other to form a
into smaller volumes, i. e., increasing their densities by applying homogeneous mixture. Conversely, different gases in a mixture
increased pressure. like air do not separate on standing.
I
STATES OF MATIER 213

(v) The characteristics of gases are described fully in terms of I


V cc - (when temperature is kept constant)
four parameters (measurable properties): P
(a) the volume, V, of the gas The proportionality can be changed into an equality by
(b) its pressure, P introducing a constant, k, i. e. ,
(c) its temperature, T
(d) the amount ofthe gas (i.e. ,mass or number ofmofes). V:::: k or PV=k
P
(a) The volume of the container is the volume of the gas
Alternatively, Boyle's law can also be stated as follows:
sample. Volume is expressed in litre (L), millilitre (mL) or cubic
centimetre (cm 3 )or cubic metre (m 3 } Temperature remaining constant, the product of pressure
and volume of a given mass of a gas is constant.
I L :::: 1000 mL; 1 mL= lO-3 L The value of the constant depends upon the amount of a gas
lL=ldm 3; Idm 3 =lO3 cm 3 and the temperature.
Mathematically, it can be written as,
I m 3 :::: 10 3 dm 3 == 10 6 cm 3 = 10 6 mL = lO3 L
PI VI P2V2 = P3V3 :::: ••.
(b) The pressure ofthe gas is the force exerted by the gas per Boyle's law can be verified by anyone of the following three
unit area of the walls of the container. The pressure of gases is ways graphically (Fig. 4.2):
• measured by a device known as manometer. Two types of
manometers, open-end manometer and closed-end manometer, Temperature-constant - -
are commonly used to measure gaseous pressure. f Mass-constant
Pressure of one atmosphere (I atm) is defined as the pressure f
that can support a column of mercury of 76 cm height at ODC V V
(density of mercury:::: 13.5951 g cm-3 ) and at standard gravity
= 980:665 cm S-2. One atmosphere is also referred to as 760 torr. P- 1/P-
(A) (8)
1 atm = 76.0 cm of mercury = 760 mm of mercury
=760 torr Temperature-constant
Mass-constant
SI unit of pressure is pascal (Pa) 'which is defined as the
pressure exerted when a force of 1 newton acts on a 1 m 2 area. f
PV
I atm 101.325 x 10 3 Nm-2 101.325 kPa
An older unit of pressure is 'bar' which is equal to 105 Pa. P-
(e) The temperature of the gas is measured in centigrade (C)
degree (DC) or celsius degree with the help of thermometers.
Fig. 4.2
SI unit of temperature is Kelvin (K) or absolute degree.
The first curve shows the variation of volume ofa given 9lass
K DC+ 273 of gas with pressure at constant temperature. The shape of the .
curve is rectangular hyperbola. This curve is also called
(d) Mass of gas is expressed in gram or kilogram.
isotherm.
lkg:::: 10 3 g The second curve showing the relationship between volume
The mass of the gas is also expressed in number of moles. and reciprocal of pressure is a straight line. It confirms the
statement that at constant temperature, volume of a given mass of
Mass in grams m gas is inversely proportional to the pressure. The third curve
moles of gas (n) =-
Molar mass M shows a straight line parallel to pressure-axis', This confirms that
(vi) All gases obey certain laws called gas laws. the product of pressure and volume of a given mass of a gas at
constant temperature is constant.
Location of straight line and curve changes with temperature
4.3' GAS LAWS in the isotherm.
(i) Boyle's law: It relates the volume and the pressure of a
given mass of a gas at constant temperature.

~T'
The relationship between the volume and the pressure of a gas
was studied by Robert Boyle in 1662. He found that increasing t
p
the pressure at constant temperature on a sample of a gas causes
the volume of the gas to decrease proportionately, i. e., if the
pressure is doubled, the volume becomes half and so on. Boyle's 1N ---'-+-
law states that at constant temperature, the volume of a
Fig. 4.2(0)
sample of a gas varies inversely with the pressure.
., ~"
,~ .

214 I G.R.B. PHYSICAL CHEMISTRY FOR COMPETITIONS

called absolute zero which corresponds to -273°C. At absolute


zero or 273 0 C, all molecular motions would stop and the
volume of the gas would become zero. The gas would become a
liquid or solid. Thus; absolute zero is that temperature at
which no substance exists. in the gaseous state. The
temperature in absolute is always obtained by adding 273 to the
v-- temperature expressed in 0C.
Fig.4.2(E) K eC+ 273)
This new temperature scale may be used for deducing Charles'
1------T1 law.
1------T2
I------,--Ta (Ta> T2 > T1 ) By substituting T for 273 + t and To for 273 in Eq. (i),
pv xT
Vt ==
To·
P--.
or == Vo
Fig.4.2(F) r To
According to Boyle's law, PV:: constant at constant
temperature or V constant (if pressure is kept constant)
T
:.log P + log V == constant
2.0
log P ",,' log V + constant

jg
log P d. 1.0
Q)
E
::l
(5
log V - - >
Fig.4.2(G)
(ii) Charles'law: It relates the volume and temperature of a
: -200 -100 0 100 200 300 "C
given mass of a gas at constant pressure. o 73 173 273 373 473 573 K
I;xperiments have shown that when 273 mL sample of a gas at Temperature -
O°C is heated to 1°C, its volume increases by 1 mL, i. e., it
become~ 274 mL. At 10°C, the volume increases to 283 mL if the Fig. 4.3
pressure remains constant in both cases. Similarly, when 273 mL This is Charles' law. It can be stated as follows:
sample of gas at O°C is cooled to -1°C, its volume decreases to
The volume of a given amr,unt of a gas at constant
272 mL while at ":'lOoC.the volume decreases to 263 mL if the
pressure varies directly as its absolute temperature.
pressure remains constant.
Thus, all gases expand or contract by the same fraction of their V oc T (if pressure is kept constant)
volumes at O°C per degree change of temperature, i. e. , for each Charles' law can be verified experimentally by plotting the
degree change of temperature, the volume of a sample of a gas values of volumes of a given amount of a gas under respective
changes by the fraction I of its volume at O°e. absolute temperatures at constant pressure. The straight line
. 273 confirms the above statement. ..
Let the volume of a given amount of a gas be Va at O°e. The (iii) Pressure~temperature law: It relates the pressure and
temperature is increased by to C and the new volume becomes VI . a
absolute temperature of a given mass of gas at constant volunie.
Thus, v..t
.
== Va + Va x t == Va (I +
273
_I_J
273
Volume remaining constant, the pressure ofa given mass
of a gas increases or decreases by 1 of its pressure vt ooe
273
or V. == V. (273 +
t a 273
tJ . ... (i) per degree change of temperature:
Po XI
A new temperature scale was introduced known as Kelvin PI =Po + - -
273
scale or ahsolute scale named after the British physicist and
mathematician Lord Kelvin. The lower limit of the scale is or PI =Po (1+_273 t
_)
STATES OF MATTER 215

R =P x V Pressure x Volume
or P = P. (273 + t1 or
I 0 273) T Temp.erature
Force Force
or !l= Po P ressure = = ----,-
T To Area (Length) 2

or P ex; T (if volume is kept constant) Volume = (Length) 3

At constant volume, the pressure of a given amount of a Force


-- - -2x (Lengt h)'-
gas is directly proportional to its absolute temperature. (Length)
So, R
Temperature
4.4 IDEAL GAS EQUATION Force x Length
This equation is obtained by combining Boyle's and Charles' Temperature
laws. .
Work
V oc.~ ... (i) (at constant temperature and definite mass) Temperature
P
V DC T ... Oi) (at constant pressure and definite mass) Thus, the value of R should always be expressed in units of
work per degree kelvilJ per mole.
Combining eqs. (i) and (ii); we get
T Numerical Values of R
Vex; - (for definite mass)
P R= PV
PV
. T
constant ... (iii)
T One gram mole of a gas at one atmospheric pressure and O°C
From the above relation, we have (273 K) occupies a volume 22.4 litre.
PI Vt P2 V2 P I atmosphere, T= 273 K, V = 22.4 litre
--=-- ... (iv)
Tl T2 R = I x 22.4
So,
From eq. (iii), 273
PV =R = O.0821litre-atm K-:-t mol- t
T If pressure is taken in dyne/cm2 and volume in mL,
or PV = RT for 1 mole gas P = 76 x 13.67 x 981 dyne/cm 2, V = 22400 mL for I mole,
PV = nRT for n mole gas ... (v) T=273K
When mass of the gas is taken in grams, the value of 11 will be 76 x 13.67 x 981 x 22400
So, R
given by: 273
Mass of the gas in gram = 8.314 x 10 7 erg K- 1 mol- 1
n=
Mol. mass of the gas in gram . Since, Ijoule = 10 7 erg, so
Let wand M be the mass and molecular mass of a gas; then
w
R = 8.314 joule K· 1 mol- 1
11=
M Since, I calorie =4.1 84 x 10 7 erg, so
Hence, eq. (v) becomes
8.314 x 10 7
R
PV= w RT ... (vi) 4.184 X 10 7
M
or P
w RT
M V
.---
Mass
Volume
Density (d )j Note:
== 1.987"" 2 calorie K- 1 mol- 1
Although, j{ can be expressed in different units, but for
pressure-volume calculations, R must be taken in the same units
or P d RT ... (vii) of pressure and volume.
M
Eqs. (vi) and (vii)are modified forms of gas equation. The Barometric Distribution
above equations are strictly followed by ideal gases. In the case of ordinary gases, pressure in the container is
unaffected by the gravitational field. But in high molecular mass
Nature of Molar Gas Constant R
polymeric gases, pressure varies with height.
Gas equation for one gram mole,
Let Po be the pressure at ground level and P be the pressure at
PV=RT height 'h'; then
216 I G.RB. PHYSICAL CHEMISTRY FOR COMPETITIONS

2.30310g lo (~):;:: _{ M:h} Solution: VI =10 litre V2 :;:: ?


PI:;:: latm P2 = 2atm
Siinilarly for density and number of moles the equation may Applying Bo~rle's law,
be given as: PI VI =PZV2
2.303 log 10 ( .!!..-J == - {!!!gh} So,
PIVJ lxlO
V2 =--=--=5htre
.
\d o R P2 2

2.303 log 10
.
(
t.no
~J : : -{M hf
R
g 1 Example 2. A sample of a gas occupies 600 mL at 27°C
and 1 atm. What will be the volume at 127°C if the pressure is
kept constant?
These relations are valid under isothermal conditions for
Solution: VI = 600 mL TI == 27 + 273 = 300 K
density, pressure and number of moles.
If temperature is not constant then the relations of barometric T2 :;:: 127 + 273 = 400 K

distribution may be2~~v::I:: 1O(!..-) == _ Mgh Applying Charles' law,


VI2
-=-
1j
V
T2
Po RT VI
V2 :::::-xT2
2,30310gI0(~J:::: Mgh 1j
no RT 600 x 400
=----
Open vessel concept: In open vessel of a gas, pressure and 300
volume are always constant.
= 800mL
PV == ntR1j ... (1) Example 3. A gas cylinder containing. cooking gas can
PV== n2RT2 ... (2) withstand a pressure of14.9 atmosphere. The pressure gauge of
the cylinder indicates 12 atmosphere at 27°C. Due to a sudden
Here, nl and n2 are number of moles at temperatures TI and fire in the building, the temperature starts rising. At what
Tz· temperature will the cylinder explode?
Dividing eq. (1) by (2), we get Solution: Since, the gas is confined in a cylinder, its volume
nl 1j. n2 T2 will remain constant.
Initial conditions Final conditions
Dry and moist gas: If volume under moist condition is
given then volume of dry gas can be determined. PI = 12atm P2 14.9atm

Pmoist Vmoist == Pdry Vdry


1j ::::: 27 + 273 = 300 K T2 =?
Applying pressure-temperature law,
Pdry Pmoi~t - Aqueous tension or vapour pressure of water
PI =P2
· h 'd' Partial pressure of water in air
ReIative unn tty =--~-------- TI T2
Vapour pressure of water
P2 xTI
So,
Payload (Lifting Capaeity of Balloon) PI
When a balloon is filled with lighter gas like H2 and He
== 14.9 x 300 == 372.5 K
(lighter than air) then it rises up due to the difference in the
12
density of air and the gas. Payload or lifting capacity of balloon
may be calculated as: Temperature in ° C = (372.5 273) = 99,SD C

Mass of air filled] [MaSS of gas filled in the


Pay Load:;:: [ . h b II - balloon at same tempera -
1 Example 4. A 1000 mL sample of a gas at -73°C and 2
atmosphere is heated to 123 °C and the pressure is reduced to 0.5
In tea oon. ture and pressure atmosphere. What will be the final volume?
Solution:
[~~~o:J Initial conditions Final conditions

:: :::::ISOME SOLVED EXAMPLES\::::::: PI =2atm P2 =0.5 atm


VI = 1000mL V2 =?
Example 1. A sample of a ga.'i occupies 10 litre under a
TI =-73°C:::::(-73+273) T2 ::::: 123 0 C = (123 + 273)
pressure ofl atmosphere. What will be its volume if the pressure
is increased to 2 atmosphere? Assume that the temperature of the 200K =400K
gas sample does not change.
STATES OF MATIER 217

We know that, or P wxRT


V M

dx
RT
M
(~= density == d J
2x
So,
200 Given, d==8.0kgm-3 ; R=8.314JK- 1 mol-I;
2x 1000x400
or T = - 40 + 273 == 233 K
200 x 0.5
and M == 36.5 g mol-I 36.5 x 10-3 kg mol- I
== 8000mL
Substituting the values in the above equation,
Example 5. A sample of a gas occupies a volume of 512
. P 8.0x 8.314 x 233 424.58x 103 Pa
mL at20°C and 74 cm ofHg as pressure. What volume would this
36.5x 10-3
gas occupy at STP?
Solution: Example 8. A certain quantity of a gas occupies 100 mL
Initial conditions Final conditions (SIP) when collected over water at 15°C and 750 mm pressure. It
~ =74 em P2 =76cm occupies 91.9 mL in dry state at NTP Find the aqueous vapour
pressure at l5°e.
1\ e
=20 C=(20+273) 293K, T2 oec= 273K Solution: Let the aqueous vapourpressure be p rnm.
VI 512mL V2 =?
Initial conditions NIP conditions
We know that, PI (dry gas) (750- p)mm P 2 ==760rnm
TI
VI = loomL V2 = 91.9 mL
74 x 512
TI == 15 + 273 == 288 K T2 273 K
293 Applying gas equation,
So,
v. _ 74 x 512x 273 xloo
-C.--_.....:::-C _ _
760x91.9
2 - 293 x 76
288 273
464.5mL 750 p== 760 x 91.9x 288
or
Example 6. 3.7 g of a gas at 25°C occupied the same 100x 273
volume as 0.184 g of hydrogen at 17°C and at the same pressure.
736.8mm
What is the molecular mass of the gas?
Solution: For hydrogen, p = 750- 736.8
w O.l84g; T==17+273=290K; M==2 13.2mm
Example 9. A balloon of diameter 20 m weighs 100 kg.
We know that, PV w RT Calculate its payload if it is filled with helium· at 1.0 atm and
M
27°e. Density of air is 1.2 kg m -3.
0.184 x R x 290 ... (i) (R = 0.082 dm 3 . atm K -I mol -I ) (lIT 1994)
2
For unknown gas, Solution:
w =3.7 g; T 25 + 273 298 K; M=? Volume of balloon == 4 TCr 3 =4 x 22 x (10)3 4190.47m 3
3 3 7
PV = 3.7 x R x 298 ... (ii)
M Mass of the air displaced 4190.47 x 1.2 == 5028.56 kg
Equating both the equations, PV
No. of moles of helium in the balloon
3.7 x R x 298 0.184 x R x 290 RT
M 2 1x 4190.47 X 10 3
== 170344
or M 3.7x 298 x 2 = 41.33 0.082 x 300
0.184 x 290
Mass of helium == 4 x 170.344 x 10 3 g =681.376 kg
Example 7. What is the pressure ofHCl gas at -40°C if its
density is 8.0 kg m-3 ? (R 8.314 J K -I mol-I) Mass of filled balloon == 681.376 + 100 == 781.376 kg
Solution: Equation for ideal gas,
Payload =Mass of air displaced - Mass of filled balloon
PV=~RT
M = 5028.56~ 781.376 4247.184 kg
218 I G.R.B. PHYSICAL CHEMISTRY FOR COMPETITIONS

Example 10. The density of a gas is 0.259 gel at 400 K Example 13. The density of oxygen is 1.43 g atSTP
and 190 ton: Find its molar mass. Determine the dens tty of oxygen at 17°C and 800 ton:
Solution: 760 torr = 1 atm Solution: Applymg . the "100"ffiula d = -MP
RT
190
So, 190 torr =- atm MP1
760 At STP,
RT,
d [PI = 760 torr; 11 = 273 K; d l 1.43g
We know that, P=-RT
M
At given conditions, d, = MPz
d - RT2
or M=-RT
P [P2 = 800 torr; Tz 290 K;d z ?]
190 dz P? xTI-
Given, d = 0.259 gL- 1 ; P= atm; So, --
760 d1 T2 PI
R = 0.0821 L- atm K- I mol-I; T 400K T
or x~xdl
PI Tz
0.259 x 0.0821 x 400 x 760
M =--------- 34.02 g mol-I _- 800·x273
190 - x 1. 43'
760 290
Example 11. The density of phosphorus vapour at 310°C
=1.417gL- 1
and 775 torr is 2.64 g dm- 3 . What is the molecular formuia of
phosphorus? Example 14. A car tyre has a volume of 10 litre when
d d inflated. The tyre is inflated to a pressure of3 aIm at 17°C with
Solution: We know that, P RT or M=-RT . ail: Due to driving the. temperature of the lyre increases to 47°C.
M P
(a) What would be the pressure at this temperature? (b) How
775 many litres of air measured at 47°C and pressure C?fl atm should
Given, d = 2.64 g dm-3 ; P atm'
760 ' be let out to restore the l:yre to 3 atm at 47°C?
R = 0.0821 dm 3 atm K- 1 mol-I Solution: (a) At constant volume,

and T = 310 + 273 = 583 K P l = Pz


TI Tz
So, M = 2.64 x 0.0821 x 583 x 760 123.9 g mol-I
775 or 3 = P2
290' 320
Atomic mass of P = 31 g mol-I

. amoecue=--
I I 123.939974 or 3.31 atm
NO.ofPatomsm . '"
31
(b) Pressure to be decreased in tyre
Hence, molecular formula of phosphorus P4
3.31 3.0 0.31 atm
Example 12. What percentage of a sample of nitrogen Let the volume of the gas to be taken out at 1 atmospheric
must be allowed to escape ifits temperature, pressure and volume pressure be V. As the temperature remains constant, i. e., 47°C,
arechangedjrom no°e, 3.0atm and1.65L to 1 IO°C,0.7atm and Boyle's law can be applied to determine V.
1.0L respectively?
lxV O.3lxlO
Solution:' Applying the formula, m (mass of the gas)
PV x M ,under both th'
or v = 3.1 litre
-~- e con d"thons,
RT Example 15. Oxygen is present in a one litre flask at a
. 3.0 x 1.65 x 28 pressure of 7.6x 10- 10 mm of Hg. Calculate the number of
Mass of gas before escapmg= 3.42 g oxygen molecules in theflask at O°e.
0.0821 x 493
PV
. 0.7 x 1.0 x 28 . Solution: n= ... (i)
Mass of gas after escapmg = 0.62 g RT
0.0821 x383
10
Percentage of nitrogen allowed to escape 7.6x 10- 10- 12
P= = atm
760
._ Q62) x 100 81.87
3.42 V = lL, T = 273 K, R 0.0821 L atm K- 1 mol-I
STATES OF MATIER' 219

10- 12 xl the temperature is increased by looe at the same volume, the


From eq. (i) n pressure increases by 10%. Calculate the temperature t and
0.0821 X 273
volume V (Molecular mass of the gas = 120). . (lIT 1999)
23
Number of molecules = 11 X 6.023 x 10 Solution: As the volume is constant, pressure law can be
10- 12 applied, Le.,
- - - - x 6.023 X IO z3
0.0821 x 273
Pz Tz
2.68 x 1010
1 t + 273
Example 16. An open flask c.ontains air at 27°e. or
Calculate the temperature at which it should be heated so that
l.l t +- 283
113 rd of air measured at 27°C escapes out. or t = - 173 ° C = 100 K
Solution: Let the initial number of moles of air at 27°C Now, applying gas equation,
(300 K) be =n
Number of moles of air left when the air is heated to PV=nRT
n 2n 12 x 0.082 x 100
the temperature T K = n - - - or V ==!!- RT 0.82 litre
3 3 P 120x 1
At constant pressure and constant volume,
nl11 n 2 T2
2n .
or n x 300= xT
3 1. '2.5 L of a sample of a gas, at 27°C and 1 bar pressure is
compressed to a volume of 500 mL keepipg the temperature
or T= 450K = (450- 273)= 177° C
constant, the percentAge increase in the pressure is:
Example 17. A mixture of co and CO 2 is found to haw; a (a) 100% (b) 400% (c) 500%· (d) 80% '
density of 1.5 gL- 1 at 30°C and 730 torr. What is the composition [Ans. (b)] I
ofthe mixture? [Hint:P,V] = P2V2
Solution: Let the average molecular mass be M. , I
I x 2.5 =~ x
M dRT = 1.5 x 0.0821 x 303 x 760 = 38.85 2
P 730 ~ 5bar
Let x mole of CO and (I x)mole of CO 2 be present. , Increase in pressure
% Increase = x 100
xx28+(J x)x44 38.85 Initial pressure
x.:: 0.3218
mole % of CO ='32. I 8 and mole % of CO 2 = 67.82
=4 x 100 = 400%]
I ,
Example 18.Calculate the value of molar gas constant, 2. Which of the following gases will have density of 1.8 g L-' at
'R'in (i)ccatmK -I mor l (ii)torrccK -1 mol- 1 (iii)kPa litre 760 torr pressure and 27°C? .
K -I mol-I. (a) 02 . (b) CO2 (c) NH3 (d) S02
[Ans. (b)].
Solution: (i) When the pressure is expressed in atmosphere
and volume in cc. [Hint: Pm = dRT
P 1atm; V = 22400 cc dRT x 300 = 44
m=-'-,-
PV = 1 x 22400 82 . I cc atm K- I moI-I P
R
nT lx273 :. The gas wilJ be CO 2,]
(ii) When the pressure is expressed in torr and volume in cc. 3. 10 g C2~ is filled in a bulb of I litre capacity which can
P =760 torr; V 22400 cc withstand a maximum pressure of \0 atm. At what
temperature will the pressure of gas reach the bursting limit?
R 760-
- x 22400
- - = 62360 torr cc K' -I mo1-1 (a) 76°C (b) 361.4°C (c) 92.4°C (d) 120'''C
1 x 273 [Ans. (c)]
(iii) When fhe pressure is expressed-in k Pascal and volume in [Hint: T = PV 10 x I 365.4 K = 92.4° C]
litre. . nR (1O/30)xO.OS21
p.= 101.3 kPa; V = 22.4 litre 4. .The vapour of phosphine g.as at 27°C and 3 bar pressure has
R PV = 101.3 x 22.4 8.31 kPa litre K- 1 mol-I density:
nT Ix 273 (a) 4.09 g mL -1 (b) 4.14 g L~]
(c) 2.04 kg L -1 (d) 2.04 g L- 1
Example 19. The pressure exerted by 12 g of an ideal gas
at temperature tOC in a vessel of V litre is one atmosphere. When [Ans. (b)] .
220 G.R.B. PHYSICAL CHEMISTRY FOR COMPETITIONS

Pm 3 x 34 V 1/100
[Hint: d - 4.14 g L- I ]
• RT 0.0821 x 300 VB 11200
V
5. Coefficient of volume expansion of a gas is: VB =2']
I 2
(a) 273 (b) 273 9. A bubble of volume VI is at the bottom of a pond at 15°C and
1.5 atrn pressure. When it comes at the surface, it observes a
(c) 2~3 (d) 2~3
[Ans. (a)]
pressure of I atrn at25°C and has volume V2;give (V2) :
. ~
[Hint: According to Charles' law, (DCE 2006)

VI = Vo (1 + _t_)
273
(a) 15.5
[Ans. (d)]
(b) 0.155 (c) 155.0 . (d) 1.55

VI = Vo (1 + at)
[Hint: PI VI = P2V2
TI T2
a = _1_ Coefficient of volume expansion
273 VI PzTl
Here, Vo = Volume at O°C and VI =Volume at t DC] 1.5 x 288
---.1.5]
6. A gas has a vapour density 11.2. The volume occupied by I Ix 298
gram of the gas at STP will be:
(a) 1l.2 L (b) 22.4 L (c) I L (d) 10 L 4.5 DALTON'S LAW OF PARTIAL
[Ans. (c)] PRESSURES
Mass When different gases that do not react chemically with each other
[Hint: Number of moles = - - - -
Molar mass are enclosed in the same container, they intermix rapidly and
exert a definite pressure. Again, each of the gas in the mixture
Mass
exerts its own individual pressure if it is present only in the same
2 x Vapour density container at the same temperature. John Dalton, in 1801,
established the relation between the pressure of the mixture of
2 x 11.2 22.4 gases and individual pressures of the constituent gases. This
relationship is known as Dalton's law of partial pressures. If
Volume of gas = n x 22.4 L states that,
1 At a given temperature, the total pressure exerted by two or
= x 22.4 L = 1 L]
22.4 more non-reacting gases occupying a definite volume is equal to
the sum of'the partial pressures of the component gases.
7. When a gas is heated from 25°C to 50°C at constant pressure
• Mathematically,
of 1 bar, its volume:
(a) increases from Vto 2V P= PA +"PB + Pc + ...
(b) increases from Vto 1.5V when P is the total pressure and p A , P B , Pc, . .. are the partial
(c) increases from V to 1.084V pressures of the component gases J1, B, C, ... respectively. The
(d) increases from Vto 1.8V
pressure that a component gas of the gaseous mixture
[Ans. (c)] would exert if it were only present in the volum~ under
consideration at a given temperature, is the partial pressure of
[Hint: VI = V2 the component.
Tl T2
~=~ Derivation of Dalton's Law
298 323 Let n l and n2 be the no. of moles of two non-reacting gases
323 'A' and 'B' filled in a vessel of volume' V' at t~mperature T.
V2 = V X = 1.084 V ]
298 Total pressure in the vessel 'F'may be calculated as,
8. The molecular weights of two ideal gases A and Bare
PV=(n l +n2)RT ... (i)
respectively 100 and 200. One gram of A occupies V litre of
volume at STP. What is the volume (in litre) occupied by one Individual or partial pressure may be calculated as,
gram of Bat STP? (EAMCET 2006) P AV nlRT ... (ii)
V
(a) - (b) V (c) V 2 (d) 2V PBV = il2RT ... (iii)
2
[Ans. (a)] Adding eqs. (ii) and (iii), we get
[Hint: Under identical cenditions of temperature and pressure: (PA +PB)V=(nl +n2)RT ... (iv)
VA Comparing equations (i) and (iv), we get
VB nB P == PA + PB (Dalton'S expression)
STATES OF MATTER 221

Dividing eq. (ii) by (i), we get This ability of a gas to mix spontaneously and to form a
. homogeneous mixture is known as diffusion. It is due to this
-.....!.....-=X A
P nl + nl
phenomenon that we can readily detect the fragrance of a flower
or a perfume or bad smell of hydrogen sulphide, sulphur dioxide,
PA x A x P chlorine, etc., when these are evolved or used in any work in
where, x A = mole fraction of 'A'. laboratory or in industry.
Similarly, dividing eq. (iii) by (i), we get
Effusion
PB xP
=XB
It is a process in which a gas is allowed to escape under
i.e., Partial pressure of a component
pressure througb a fine orifice or a small aperture made in
Mole fraction x Total pressure
the wall of a closed container. The difference between diffusion
Relationship between Total Pressure and Individual and effusion is that in the former case, the gas spontaneously
Pressure (before Mixing) of the Constituent Gases at streams through a porous partition while in the case of effusion,
Constant Temperature the gas is forced out through a small hole by applying some
At constant temperature, let V; volume of a gas A at a pressure external pressure. However, both the processes are essentially the
PI be mixed with V1 volume of gas B at a pressure Pl' Both these same.
gases do not react chemically. . Instantaneous rate of effusion is directly proportional to the
instantaneous gas pressure and inversely proportional to the
Totalyolume = Vj + V2 square root of molar mass. '
Let the total pressure be P and partial pressures of A and B be dP KP
P A and P B respectively. Applying Boyle's law,
dt JM
P A (VI + V2 ) PI V; ... (i)
and PB (VI + V2 ) P2 Vl ... (ii) _r P2 dP =~ rt dt
Adding eqs. (i) and (ii),
JP1 P JM Jo
PA + PB In(;~ J J!M
or P PIV; + P2 V2 InP 1
Vl +V2
In the laboratory, it is often convenient to collect a gas over
water. When a gas is collected over water, the observed pressure Hence, partial pressure decreases exponentially with time as
of the gas is equal to the sum of the pressure of dry gas and the
pressure of water vapour. . ,
Therefore, the pressure of dry gas
= pressure of moist gas pressure of water vapour 1
=pressure of moist gas aqueous tension p

Amagat Law of Partial Volume


Total volume of a mixture of gases which does not react at t- t-
constant temperature and pressure is equal to sum of individual Fig. 4.4 Variation of partial pressure of gas with time,
vo.lumes (partial volumes) of constituent gases. in the process of effusion.
V = ~ Vi =VI + V2 + V3 + ... + VII
Graham's La~of Diffusion or Effusion
4.6 DIFFUSION'OF GASES AND' This law was presented by Thomas Graham in 1833. The
GRAHAM'S LAW OF DIFFUSION law states that "At constant pressure and temperature, the
rate of diffusion or effusion of a gas is inversely proportional
When two or more non-reacting gases are kept side by side, they to the square root of its density."
have the tendency to mix with one another spontaneously and
form a homogeneous mixture. This can occur also if two gases Rate of diffusion oc .Ja
have different densities. The heavier gas moves up and the lighter
one comes down even against the action of gravity. Such a If'i and r2 represent the rates of diffusion of two gases and d 1
phenomenon is due to the fact that gas particles are moving at and d 2 are their respective densities, then
random with very high velocities and there is much ,p;
intermolecular empty space in the volume of any gas. When two rz .~ d l
gases are brought together, they mix with each other in no time.
222 G.R.B. PHYSICAL CHEMISTRY FOR COMPETITIONS

This equation can be written as: lj (M z JI!2

,
density of gas II rz=lM I

r2
density of hydrogen
density of gasI
density of hydrogen
:::

~:e;:::, (~:i'~''': ':=::c:::::,:: ',: :;


about the preferential effusion of lighter species relative to the
2VDgasII Mol. mass gas II
heavier ones. This enrichment can be applied to
2VDgasI Mol. mass gas I
separate isotopes. For example, the isotope U Z35 , the active
Volume of gas diffused component in atomic weapons arid in nuclear power generation,
Rate of diffusion = ---~,----­ has a' natural abundance of only 0.7% while U 238 is far more
Time taken for diffusion
abundant. By reaction' of the isotopic mixture with fluorine,
i. e. , the volume which diffuses in unit time. gaseous UF6 is produced and can be enriched by passing it
(i) Comparison of times taken for the same volume of two through a porous barrier. The process of diffusion through the
gases: Let the times of dj.ffusion for the same volume of two, porous barrier is repeated many times to achieve sufficient
gases be t I and t 2 respectively; then enrichment.
- .V
Separation factor' f ' :::
I
.2..= {d;::: ~M2 nl I n2
r2
tl :::
~ vd; MI where, nl n z and n l ' n z ' are the concentration of two isotopes
before and after diffusion.
t2

or
:: =J¥ ~~: =
Theoretical separation factor for single step f'

Let enrichment of species' 1' is achieved after' n' times then


~
(ii) Comparison of the volumes of the gases that diffuse in I
same time: Let Vi and Vz be the volumes of two gases that (/')" f
diffuse in the same time t. Then
VI
nlog f' l:lg/(n;; I n:i )
=:

nl I nz
.2..= t =~= {d; = 1M2
rz Yz V2 Vd; VM I
n'~ z
log _2::: log rn'ln'J
_I_ _
t
M[ nil nz
(iii) Effect of pressure on rate of diffusion: When pressure
is not constant then rate of effusion may be taken proportional to
pressure.
n (M J
- Iog - z -1
- og - -n:i1
) (n{ I
2 Mj , nil n 2
1
roc-- rocP
.JM 210g ( nil n 2
l
J'

Combination of these equations gives: nj I n 2 ' 2 log (f)


P n '::: ---'--"----=..:.. ( . '\
r oc - - log (M 2 IM j )
.fii log I MZ1'
\MI'
or .2..=P ~M2 j
::::::I_SOME SOLVED EXAMPLES \ I::: :::
r2 P2 MI
Example 20. A 10 litre flask contains 0.2 mole of methane,
How to Determine Rate of DiffusiOn? 0.3 mole of hydrogen and 0.4 mole of nitrogen at 25°C. What is
the partial pressure of each. component and what is the pressure
(a) Rate of diffusion is equal to distance travelled by gas per
inside the flask?
unit tim~ through a tube of uniform cross-section.
(b) NumberofmoIes effusing per unit time is also called rate Solution: P = nRT
of diffusion. V
. 0.2 x 0.0821 x 298
,(c) Decrease in pressure of a cylinder per unit time is called PartIal pressure of methane::: 0.489 atm
rate of effusion of gas. 10
. 0;3 x 0.0821 x 298
Partial pressure of hydrogen ::: - - - - - - - 0.734 atm
Application of Graham's Law of Diffusion , 10
According to Graham's law of diffusion, the ratio of rate of . If' 0.4 x 0.0821 x 298
P artla pressure 0 mtrogen::: - - - - - - - 0.979 atm
diffusion of two gases may be given as: 10
STATES OF MATTER 223
Total pressure = (0.489 + 0.734 + 0.979) atm
2.202 atm
Alternative solution: Total number of moles
.. 761x 900x 273
288x 760
854.2mL
= 0.2 + 0.3 + 0.4 == 0.9
. Example 23. When 2 g of a gaseous substance A is
Let the total pressure be P. introduced into an initially evacuated flask at 25°C, the pressure
We know that, is found to be I-atmosphere. 3 g of another gaseous substance B
P = total number of moles. RT is then added to it at the same temperature and pressure. The
V final pressure is found to be 1.5 atmosphere. Calculate the ratio
V = 10 litre, R = 0.0821 L atm K- I , T == (25 + 273) 298 K of molecular masses ofA and B assuming ideal gas behaviour.
Solution: Let the molecular masses of A and B be M A and
P 0.9 x 0.0821x 298 2.20atm
10 . M B respectively.
Partial pressure of CH 4 Mole fraction of methane Pressure exerted by the gas B = (1.5 - 1.0) 0.5 atm. Volume
x total pressure and temperature are same in both the gases.
ForgasA: P=latm, W 2g, M=M A
0.2 x 2.20 == 0.489 atm
0.9 We know that, PV= w RT
Partial pressure of Hz = Mole fraction of H2 x total pressure M
03 .
- ' x 2.20 = 0.733 atm 2 2RT
0.9 IxV - · R T or MA ... (i)
MA V
Partial pressure of N2 = Mole fraction of N z x total pressure
For gas B: P 0.5 atm, w = 3 g, M MB
= 0.4 x 2.20 0.978 atm
0.9 3 3RT
0.5 x V == . RT or M B = ... (ii)
Example 21. r[200 mLof N 2 at 25°C and a pressure of MB O.5xV
250 mm are mixed with 350 mL ofO 2 at 25°C and a pressure of Dividing Eq.. 0) by Eq. (ii),
300 mm so that, the volume of resulting mixture is 300 mL, what
WGuld be thefinal pressure o/the mixture at 25°C? 2RT 0.5xV
=--x
Solution: In the case of nitrogen, volume increases, its MB V 3RT
pressure must decrease. Let the new pressure be P", 2 • 2 x 0:5 I
250x 200 . ----
P"'z 300 == 166.6mm (Applymg Boyle's law) 3 3
In the case of oxygen, volume decreases, its pressure must Thus, M A : M B = 1: 3 .
increase. Let the new pressure be P0 7 • Example 24. Find the. total pressure exerted by 1.6 g
metHane and 2.2 gC0 2 contained in afoul' litreflask at 27°e
POz 3003~:50== 350 mm - (Applying Boyle's law)
Solution: PV (n, + n2 )RT
Total pressure PN2 +P 02
= (166.6+ 350)= 516.6mm
Px 4 r\ 1.616 + 2.2) x 0.0821 x 300
Example 22. 1.22 g ofa gas measured over water at 15°C 44
and a pl'essure of 775 mill o[ mercury occupied 900 mL
Calculate the volume ofdlJ} gas at NTP Vapour pressure of water P = 0.9236 atm
at 15°C is 14mm. Example 25. 1500mLflask contains 400mg O 2 and 60mg
Solution: Pressure of dry gas Pressure of moist gas H 2 at 100 0e
- Aqueous tension (a) What is the total pressure in the flask?
= 775 14 (b) If the mixture is permitted to react toform water vapour at
·IOO°C, what wi/the left and what will be their partial pressures?
=761 mm
Initial conditions NTP conditions Solution: (a) No. of moles of0 2 400:::: 0.0125
1000 x32
VI 900mL
No. of moles of H2 60 =: 0.03
PI == 761mm 1000 x 2
TI = (273 + 15):::: 288 K T2 = 273 K
.\ fO 0.0125 x 0.0821x373 0255
PZV2 P artJa pressure 0 2 = . =. atm
Since, 1.5
TI T2 0.03 x 0.0821 x
Partial pressure of H2 0.612atm
1.5
So, V2 ::::
TI P2 Total pressure= 0.255 + 0.612::: 0.867 atm
224 G.R.B. PHYSICAL CHEMISTRY FOR COMPETITIONS

(b) . 2H2 + 02 ::::: 2H 20 .1..::::: MS02


Initial 0.03 0.0125 0 r2 M
After reaction 0.005 0 0:025
. 0.005 x 0.0821 x 373 Thus,
180/15 f64 .
Pamal pressure of H2 ::::: . , == 0.102 atm
1.5 120/20 V'M
2=~
Partial pressure of H 0 0.025 x 0.0821 x 373 == 0.51 atm
2
1.5
3
Example 26. 20 dm of S02 diffuse through a porous So, M==16
partition in 60 s. What volume of O2 will diffuse under similar Example 30. The reaction between gaseous NH 3 and HBr
conditions in 30s? (lIT 1996) produces a white solid NH4Br. Suppose a small quantity of
Solution: Rate of diffusion of S02 = 20 dm 3 S-l gaseous NH 3 and gaseous HBr are introduced simultaneously
60 into opposite ends of an open tube which is one metre long.
Rate of diffusion of 02 ::: ~ dm 3
Calculate the distance of white solidformedfrom the end which
S-l
. 30 was used to introduce NH 3'
According to Gralj.am's law of diffusion, Solution: Let the distance of white solid from
NH3 end xcm.
(V 130) ~Mso, fM The distance of white solid from HBr end (100 - x) cm.
(20/60) = M02 '132 Rates of diffusion shall be proportional to these distances.
V;:;:: 14.ldm3 x _ ~MHBr
r2 (lOO-x) MNH3
Example 27. 50 volume of hydrogen take 20 minute to
diffuse out ofa vessel. How long will 40 volume of oxygen take to Mol. mass of lfBr::::: 1 + 80::::: 81
diffuse out from the same vessel under the same conditions? Mol. mass of NH3 = 14 + 3 =17
Solution: Rate of diffusion for hydrogen, 'i ::::: 50
So,
x {8i
20
40
(lOO-x) V17
Rate of diffusion for oxygen, r2
or, --x-_=2.18
Mol. mass of hydrogen::::: 2 (lOO-x)
Mol. mass of oxygen::::: 32
So, x 100 x 2.18- 2.18x
So
50/20::::: {32 or, 3.l8x 100 x 2.18
40lt '12 100 x 2.18
So, x=----
or 50 x 4 3.18
20 40
::::: 68.55cm
t == 64 minute
Example 31. At 27°C, hydrogen is leaked through a tiny
Example 28. Calculate the relative rates of diffusion of hole into a vesselfor 20 minute. Another unknown gas at the same
235 UF6 and 238 UFfS in the gaseous state (At. mass of F ::::: 19). temperature and pressure as that of hydrogen leaked through the
Solution: Mol. mass 235LTF6 = 235 + 6 x 19 = 349 same hole for 20 minute. After the effusion of the gpses the
mixture exerts a pressure of6 atmosphere. The hydrogen content
Mol. mass 238 UF6 = 238 + 6 x 19 = 352 of the mixture is 0.7 mole. If the volume of the c014tainer is 3 litre,
what is the molecular mass of the unknown gas? (lIT 1992)
= ~M2 == ~352 ::::: 1.0043 Solution: Let PH2 and Pun be the partial pressures of
r2 MJ 349 hydrogen and unknown gas respectively and w be the number of
II : r2 :: 1.0043 : 1.0000 moles of unknown gas.
Example 29. 180 mL of a hydrocarbon diffUses through a PH2 = 0.7 x 0.0821 x 300
porous membrane in 15 minutes while 120 mL of S02 under 3
identical conditions diffuses in 20minutes. What is the molecular
mass of the hydrocarbon? Pun w x 0.0821 x 300
3
Solution:
Adding both,
II ::::: rate of diffusion of hydrocarbon = 180 mL min-1
15 PH2 + Pun = 6= (1/3) x 0.0821 x 300(0.7 + w)
w 0.0308 mole
r2 ::::: rate of diffusion of S02 = 120 mL min- 1
20
STATES OF MAnER 225
Applying law of diffusion, P N2 M XeF"
Solution:
0.7/20 rXeF,;
[!Vi or M = 1033 PXeF,; kINe
0.0308/20 V2 .. Moiar volume
Example 32. The ratio of velocities of diffusion of gases A
and B is 1: 4. If the ratio of their masses present in the mixture is 1~2 tN2 57
--
2: 3, calculate the ratio oftheir mole fractions. [dtiH1JitiilrJ iIl9ljfflO] rXeF" Molar volume 38
~MB ·1 . . t
XeFIl
Solution:
rB MA
57 0.8 1M XeF"
or.
38 l.6 V28
. 157\2 (1.6,\2
Let masses WA and WB of A and B be present respective.
M XeF. = - I x I - I I x 28
\38)
11 \0:8)
WA
= 252
Mole fraction of A = -:=_M--'.:A______-
W WB Xe+nF 252
- A- + - 131+nx19 252
MA MB
n 6
WB
Molecular formula = XeF6
MB
Mole fraction of B = -::=--"'-::c::::-- Example 35.- 100 cm 3 of NH 3 d(ffilses through a pill hole
WB
+-- in 32.5 second.. How much time ,vill 60 cc of N 2 take to cjifJilse
MA ME under the same· conditions?
Ratio WA x
WE MA Solution:
1>'11:1 3 =
.11MN2
ME 2.1 1 . IN2 VMNH3
or x--= x - = -
WE M A . 3 16 24 100/32.5 {28
Example 33. A space capsule is filled with neon gas at 60lt V17
1.00 atm and 290 K. The gas effuses through a pin hole into outer
100x t {28
space at such a rate that pressure drops by 0.30 torr per second.
(a) If the capsule is filled with ammonia at the same 60x 32.5 V17
temperature and pressure,. what would be rate ofpressure drop? t = 25 second
(b ){fthe capsule is filled with 30.0 mol % helium, 20.0 mol %
oxygen and 50.0 mol % nitrogen at a total pressure of 1.0 atm and
a temperature of290 K, what would be the corresponding rate of
pressure drop? 10. A gas with molecular fonnula CnH211 + 2 diffuses through a
Solution: (a) The rate of pressure drop is directly proportional porous plug at a rate 1I6th of the rate of diffusion of hydrogen
to rate of effusion. gas under similar conditions. The fonnula of the gas is:

f",.Ne =~MNH3 =p7.0 0,92


(a)C zI-4
[Ans. (c)]
(b)CIOH22 (c) (d)C 6 H14
r:'lH) M Ne 20
rN . 0.30 . [Hint: ~ ,1r:M;
= __
- .e_ 0.326 torr 1 second Ii ~Ml
0.92 Q.92
(b) The average molecular mass of the gaseous mixture is --.:L =
Iii 6
0.3 x 4 + 0.2 x 32 + 0.5 x 28 = 21.6
. rw M2 == 72
Rate of drop of pressure = .1-- x 0.30 :. The fonnl\la o~ gas will be C,H 12 .]
.. V21.6
11. Under similar conditions which of the following gases will
= 0.29 torr 1 second diffuse four times as quickly as oxygen?
Example 34. One mole of nitrogen gas at 0.8 atm takes 38 . (a). He ' . (b) Hz (c) (d)
second to diffuse through a pin hole whereas one mole of an [Ans. (b)]
unknown compound of xenon with fluorine at 1.6 atm takes 57

~
[Hint: - ---
o,
second to diffuse through the same hole. Calculate the molecular
formula of the compound. (1I(lll~'19V9)
/02 -, IV! gas
I
226 I G.R.B.' PHYSICAL CHEMISTRY FOR COMPETITIONS

~
2 32
4= -.- or 16
Mg~s' lV/gas

or

, 12. The Jates of diffusion of hydrogen and deuterium are in the


ratio:
(a)I:1 (b)J2:1 (c) 4. I (d) I: 4
[Ans. (b)] ..

[Hint:

13. The time taken for effusion of64 mL of.oxygen will be same
as the time taken for' the effusion of which of the following
ga5es under identical conditions?
(a) 64 mL of Hz (bj 100 ofN2 " Derivation of Kinetic Gas Equation
\c)64rl1L of COl '" (d) 45.24 mL ofSO/' On the basis oftne postul(ltes of kinetic theory of gases, it is
[4l.ns .. ' (d)] possible to derive the mathematical expression, commonly
,'!."
1';.
known as kinetic gas equation, i. e. ,
[aint: 102 =
12
'gas PV= - mnc
3
64 ~ . 64
V =~ 32 or V = 45.2~mLl where, P = pressure of the gas, V == volume of the gas, In mass
of a molecule, n number of molecules presbnt in the given
14. Which of the following pairs of gases will have identical rate amount of a gas andc root mean square speed.
of effusion under similar c;onditions? l11eroot me~m square speed (rms speed) may be define}! as the
(a) Diprotium and dideuterium square root qf the mean of squares ohhe individualspeed of all
(b) Carbon dioxide and ethane' the molecules.
(c) Dideuter;um and helium 2 '. 2. ·2, . 2.
CI + C2 +C 3 + .. , + CII
(d) Ethene and ethane . qns spe:ed=
[Ans.«c)] n
[Hint: Dideuteritttn and hC'liumhave same molar'mass,hence
ti1ey\ViH diffuse with.idcnticai rate :under identical conditions,]
15. Two gas bulbs A and B are connected by a tube having a
• 'fcm
stopc6~k. BjJlb A.'hasa volume of 100 mL and contains
hydrogen. After. opening the' gas from 11 to the evacuated
bulbB, the pressure falls down to 40%. Tile voh.l'me(mL) of
B must be: ' [PET (Khala) 2006]
(a) 75 (b) 150 (c) 125 ,(d)2QO .
Before
(e) 250 'collision Ux ·
[Ans. (b)]
A
. [Hi~t: P1V j (A) + P2Vi B) = PR (VI + "-,-+-'----E;-;--'-;-:-+,./
: After -Ux
; 190 x InQ.:+ 0 x Vz = 40(100 + V2 ) I collision'
.)----------
V2 = 250 - 100 = 150 mL 1 ./
:;/- .
J /:
/
4~7KINETIC 'tHEORY OF GASES
,1'" /
/
/

./

This theOlY was a ge~eralization f6raboutideal gases, It was


presented by Bernoulli in 1738 and developed in i 860 by
Clausius, Maxwell, Kroning and Boltzmann. Postulates of
kinetic theoiy of gases are: . Fig.4.5
I

STATESOFMATIER 227
~
Consider a certainmass of a gas enclosed in a; cubical vessel of P == 1 mnc-
side'l' cm. Letthe total number of gas mole<;:ules be 'n' ami mass 3 V
of each molecule be 'm'. Let c be the root mean square speed. 1 . ?
Speed can be resolvedinto three components,i. e.;Ux'~y andU z or PV=~ mnc·-
3 .
parallel to the edges of the con.tainer (i.e, ,parallel to thJ:ec. axes x,
yand z ). . . This equation is taIled: kinetic\~as equation; .
C
2
=U2x +U 2 +U 2 For one gram mole of the gas,
'"Y z
n N (Avogadro's number)
· Consider the movement of a single molecule between opposite
.~ 6.02 Xi0 23
faces A and Bparallel to x~axis. When the molecule strikes with
one wall of the container, it bounces back with the same speed m x N =M molecular mass of the gas.
and subsequently strikes the opposite waH. The above kinetic equation can pe written as: .
The momentum of the molecule before collision with · ...··prr 1 c ..
. . r' =- M2
face A =mUx ' 3·
The momentum of the molecule after collision =. mUx .2 3PV
or c =
The change in momentum in one collision =mUx ~ (~ill (;x ) M

=2mUx ' c= ~3PV


• . . t ; M.
After collision; the mol<~cule must move. a distance 21 cm along
x-axis before making another impact' on . this wall. Since the'
velocity 'Ux ' remains unchanged, the time taken to travel a
· 21 .
[if [d = density]

distance 21 cm -'-' second.


Ux =~3:T
So, the number of collisions by the gas molecule in one second
l~~ . . Thus, c= ~3PV . ~3RT = {3P
~2i' M. M "Vd
The value of R should be taken in erg K- 1 mol-I, e.g.,
2 R 8.314. X 10 7 erg K- 1 mol- 1
Change of momentum per second = 2mUx x =mU
_x_
7
21 I
c == 14 X 10 xT = 1.58 {T, x 104 cml sec
· 'The changeofmorhentum on both the opposite faces A and B
. 2mU 2
M " VAi .
along x-'axi.s per .second would be dO).lble, i. e. '7' ' Calculation of Kinetic Energy
Similarly, . 1 2 ..
PV=- mnc '
2mU 2 ' 3
Y
Change in mome~tumper second along )I-axis For one gram mole of the gas,
,1
PVoc=RTand n N
. d'h . d I . . . 2mU; .
an c ange III momentum per secon a ong z-axiS = - - .- ..
. . ". · · · , · . 1 '" ~3 mNc
'.
2
RT
Hence, total change of momentum per second on all faces will
be .
2 2
or
2 1 hI 2 ( I ' ?
-' mHC= RT 1- mNc-
3 2 \2
KEPermolj "
2mU
= _' . 2mU 2mU 2
x + __·_Y + _'._z
__
I I I 3. KE =RT
3
= 2m (U2 +U 2 + ) 3
I . x Y
or KE=-RT
2m 2
1 Average kinetiC energy per mol does riot depend Ofl then1!rure
of the gas but depends on.1y On temperature. Thus,when two
Change of mor:ientum per second Force
. , . . , Force
= gases are mixed at the same temperature, ,here will be no rise or
Pressure = ~, decrease in temperature unless both react chemically..
. . Area
. Average KEper mole
2m c 2 ': }..':e!age kinetic energy per molecule
Pressure created by one molecule N
. 33
. - =-kT·
Pressure created by n molecules 2 N2
k = Boltzmann constant
228 ·1 G6.B. PHYSICAL CHEMISTRY FOR COMPETITIONS

The ratio R / N is constant and is known as Boltzmann (i) The root mean square speed: The speed in kinetic gas'
constant. Its numerical vallie isl.~8 x lO-16 erg K'I molecule -I.
equation, PV = 2mnc 2
, is the hypothetical speed possessed by
3
4.8MAXWELL-BOLTZMA,NN DISTRIBU"nON all the gas molecules when the total kinetic energy is equally
OF MOLECULAR SP£EDS distributed amongst them. The total kinetic energy of the n
molecules of the gas is sum of the kinetic energies of the
The gas molecules are moving in all possible directions. They individual molecules.
collide with one another and also with the walls of the container. Total kinetic energy
As a result of collisions, the speed and, direction of the gas
121212 12
molecules are ever changing, i.e., all the molecules in a given = - mCI + - mC 2 + - mC3 + ... + - mc ... (i)
2 2 2 2 11
sample of gas do not have the same speed.
The distribution of gas molecules among different possible Let c be the veloCity possessed by each of the n molecules;
then,
speeds was studied by MaxweU and Boltzmann using the theory
1 7
of probability. The results are mathematically expressed as, total kinetic energy 11 X - mc- ... (ii)

dNu = 4nN [ --J .


M
2nRT
3/2

e
--
1I1u

2RT
2
2
u du Equating both the equations,
121? 1212
11 X - mc.
2.

12
- mCI + mc, + mC3 + ... +;- mc"
m J312 _111112
2 2 2 - 2 2
= 4nN [ - - e 2RT u 2 du
2nkT' or
n
. Here, N = Total number of molecules, M = molar mass of gas
. This expression gives the number of molecules (dNu) having or c
speed between u and (tl + du) at temperature (T).
A plot of fraction of molecules in the speed range (u + du ~ Thus, rms speed is defined as the square root of the mean of
1
- ' - - '. \
(dNU .
agamst speedu 'IS: the squares of the speed of all the molecules present in the given
N du) . sample of the gas. The value of c is determined by using the
following expressions:.

GOC c ~3PV ~3RT = [3P


'M M Vd
In SI units, the values of R, P, V, M and d used are given
below:
1 dNu (i) R 8.314JK- l mol-lorR 8.314kPadm 3 K- l moC I
N du, (ii) V of the gas always in litre
(iii) P of the gas in kilopascal (kPa). I atm = 101.3 kPa and
I torr =0.133 kPa .
(iv) M in kg mol- 1
I
I
I
I
I
(v) d in kg m- 3
:vrms
U---'---
(ii) Average speed: It is the arithmetic mean of the various
speed speeds of the molecules.
"
Fig.4.6·
Average speed
V mp most probable speed n.
vav = average speed . equaI to ~8RT
I t IS --.
v lIDS root mean square speed nM
Kinds of Molecular Speeds It is related to rms speed by the following relationship:
Molecular speeds are of three types: Average speed 0.9213 x rms speed
(i) The ITnS speed,· . . I rms speed = 1.085 x average speed
(ii) The average speed and I (iii) Most probable speed: This is defined as the speed
(iii) The most probable speed possessed by maximum numb ex of molecules of a gas at a given
temperature.
Note:Root mean square speed explained the non·existence of gases tn the atmosphere of moon. Root mean square speed of gases exceeds the
escapeve!ocity of moon and hence gases escape from atmosph'ere-ofmoon against the graVi!,ltional barrier of moon_
STATES OF MATTER 229

" (2Rf (iii) DistIibutign of molecular speed also depends upon the
It is equal to fAT' molecular mass of the gas. At similar temperature a heaviei' gas
molecule has a naITOW distribution of speed than those of lighter
This is related to rms speed by the following relationship: gas molecules.
Most probable speed = {.JI) rms = 0.816 rms
(iv) The fraction of molecules having speeds greater than
minimum goes on increasing with increase in speed. It reaches to
a maximum value and then begins to decrease.
or rms 1.224 most probable speed (v) As long as temperature of the gas is constant, the fraction
The three kinds of molecular speeds are related to each other having a particular speed remains the same inspite of the fact that
as: the molecules change their speeds due to collisions. '
Mostpr6bable speed: Average speed: rms speed The increase in the temperature of the gas increases the
molecular speed. As a result, the most probable speed increases
I2RT. V8RT . l3RT with the increase of temperature and the distribution curve shifts
VM·1tM·. M towards right. The general shape of the curve remains the same
[8' but the maxima of the curve becomes somewhat flat at a higher
= J2: V-;:-13 temperature, i.e., there is a wider distribution of molecular speeds
= 1 : 1.128: 1.224 and the fraction of the molecules having high speeds increases.
However, the fraction of molecules possessing mostprobab~__
rms speed at any temperature to C may be related to its value at
speeds decreases with increase in temperature. ' '
STP as:
/(273+t) ,
. ... (i)
Co ~ 273 '

... (ii)

From eqs. (i) and (li), we get


• (3P /'-:-(2""-7-:-3 t-:-)
-1--.
t:
o
c, ,V~ ~ 273 ~
lL
3P(273.+ t)
273d
Some Essential Points for Distribution
,of Molecular Speed Speed----
(i) The fraction of molecules with very low or very high Fig. 4.8
. speeds is very small. Maximum fraction of molecules have speed
near to the most probable speed v mp' ....
....
Example 36. Calculate the kinetic energy in joule .(){8.00 g
ofmethane at 27°e.
Q
U
Solution: Number of gram moles of methane

1 dNu
1 T == (27 + 273) ==300 K
16 2

Ndt
R 8.321 K-1 mol-I

Kinetic energy for one gram mole =,~ RT


2
3
- x 8.32x 3001
2
Thus,
Fig. 4.7 Distribution of molecular speeds of various
gases at a constant temperature , kinetic energy for.!. gram mole = 1 x ~ x 8.32 ~ 3cn == 1872 1
2 2 2
(0) Total area under t~e curve is a measure oftotal number of " Exampl~37. 'Calculate the average andtoia! kim:tic
,lfIolecu1es in the collection. Thus, area under the curve remains energy of 0.5 mole alan Ideal gc/s at a°e.
constant at different temperature. '
I
230 (! R. B. PHYSICAL CHEMISTRY FOR COMPETITIONS
!
Solution: Averilgekint1tie en\,:rgy per molecule ofthe gas
R=8.314kPadm 3 K- 1 mol- 1 ;T=:273K;.
3'R ",
=--T M = 0.002 kg mol- 1
, 2N
. Slibstituting'theival~~s,
3
2
c'= ,Ir~-x-.8-.3"':"1-4-X~2-7-3 = 1845.15 illS-I
V 0.002
;'5.65><10'"714 ergS:65xlO-2l 1'
Example4L Oxygeliat 1 atmosphere and O°C has a
Total kinetic energy of 0.5 mole;ofihe gas density ofl.4299 g rl. Fin¢. thums. speed: ofoxygen molecule.
'=~RFXO.5 ' Solution: P = 1atm 101.3 X 10 3 Pa
2
d= 1.4290g L- 1 '= L4290k~ m~3
.= ~x 8.314 X 10"x 273 x05'·
2 We know that,
= 1.702 X 1010 el'g::::; 1.702 kJ
"
r3P - - - - - =461.15 ms- 1
Example 38. , Calculate' the pressure exerted by 1023. gas c=Vd
molec:uleseachofmass 10-22 gina: contai'n('" o/volume 1 litre,
Example 42. At wHat temperature will hydrogen
the rms, speed is 105 ems-I.
'molecules httve the Same7()ot' Mean~8qrmre 'gpeed "ay.' ntt1"(;rgen
Solution: Using kinetic gas e(lulltioti, . molecules at nOC?
1 mnc 2
p Solution: ~ 3RT = ~ 3R x 300,
3 V MH MN
Given, V llitre = 1000 mL =1000 cffi3; n = 1023; m = 10-22 g T ., , 300
. andc 10 5 cms-: I ,. '.' or or T -.-x2=21.43K
. 28
Substituting the values in the above equation,
110-22 x 10 23 X Example 43. CaJt:u}q.te the rootmean square, average and
P - x 3.33 X 107 dyne em:-2 ' most probable speed ofoxygen at 27°C
3 1000
Solution: 27°C =)00 K
Example 39. Calculate the 1"oot mean square speed of an
oxygen molecule at 288 K in SI.units. 13RT
Root mean square speed == V M
" q'

Solution: c R 8.314 X 10 7 erg K- 1 moi- I ; M 32g moFI;. =r 30QK


Substituting the values,
R = 8.314kPa dm 3 mo)-l, T = 288 K ..--",-,:,,~-=--~-

and
Substituting the values in above equation = 48356 em! sec
= 483,56 m! see
c= 3 x 8314:>< 288 473.79 ms-1
0.032 Average speed = ~8RT
--
Example 40. Calculate the root mean square' speed of nM
hydrogen molecule at STP ,

Solution: c =' ~3:


P = 1atm:;:: J 01.3 kPa 445,42,m1sec
3
V=22.4 dm , J2XRXT
Mostprobable s,peed = ','
M 2 g mol- = 0.002 kgmor 1
I
M
,-:-----::---
Substituting the value~,
3x 101.3 x 22.4
c= 1844.91 ms",l ,
0.002 = 39483 em! see
"Alternative method: . = 394.83 in/ sec
c . Example 44. Calculate the average. kinetic energy per
mole afC02 gas at 27°C in different units.
I

STATES OF MAnER I 231

Solution: KE= ~ nRT


2
3 .. . . '.
= - xl X 1.987 x 300= 894. 15cal
2
3
KE nRT
2 T2 = 300K
. 2~1 x 8-.314><300 t. n°C]
2 18. The root lnean square speed of molecules of nitrogen gas is v
ata certl}in temperature. When the temperature is doubled,
=3741 j
the molecules dissociate into individual atoms. The new rms
3.741 xW IO erg ~peed:qfthe atomjs: . . '

Example 45. A gas bulb of 1 litre capacitv contains


(a)Jiv . (b) 2v . (c)v (d) 4v

2.0 X 1011 molecules of nitrogen exerting a;ressureof [Ans. (b)]

7.57 x 10 3 N 2 . )3RT
Calculate tIle roqt mean square speed and
fn- . [Hi~t: VI '" M
temperature of the gas molecules. If the rati() of most probable
speed to the rootn!ean squarespeed is 0.82, ca!culate)he 'most T2 2T, M 2 = M 12
probable speed of these mole'cules at this temperature. '
(liT t 993) '°2 =
'f h 2.0x 19. The translational kinetic energy of an ideal gas depends only
Solution: Amount 0 t e gas = ' " nlole on its: . , '
6.023 x lO~"
(a) pressure (b) force
Given, V I litre =1O~Jm3; P == 7.57 X10 3 Nm-2 , (c) temperatul'e (d) molar mas.s
R 8.314JK- 1 mol"']. [Ans. (c)]
, ..... . .·PV 3 . '
Applying PV = nRT or T = [Hint: KE 2 RT :. Kinetic energy depends on temperature,]
I1R
20. At what temperature is the rms speed of H2 moleci1les the
10-3 x 7.57x ]0 3
T= ?l = 274.2K same as that of oxygen molecules at 1327°C?
2.0x 10- x 8.314 (a)I73K (b) lOOK (c)400K(d)523K
[Ans. (b)] "
6.023 x I

rms speed = ~3MRT


,
/3 x. 8.314x274.2
. 28 X 10- 3
494.22rr~s-1 [Hint: lf~~112 P~~212 = l
fT.; =
I /1600
Most probable speed == 0.8 x rms
fi ~ 32
0.8 x 494.22 ms- I == 395,376 ms- l TJ 100 K]

~DS OF OBJECTIVE QUESTIONS 21. If the temperature of I mole bfa gas is increascd'by 50°C,
ealcuHite the change in kinetic energy of the sy.stem: ".
16. At what temperature is the root mean square speed of N2 gas (DCE 2006 )
(a). 62.32 J (b) 6.235 J (c) 623.5J (d) 6235.0 J
equal to that of propane gas at STP?
[Ans. (c)]
(a) 173.7°C (b) 173.7 K (c) 273 K (d) ~40°C
["'ns. (b)] [Hint: .E= l'RT
2 . .(Kinetic .energyofl
. '. mole
[Hint: v rms (N 2 ) == v rm S<C 3H g )
6.£=3 R(T + 50) ~ RT
2 .. · 2
pRT == ~3R 273 3 . 3 ....
28. 44 -R~50 -x8.3J4x50
2 2
T=173;7K] = 623.55 J]
17. At what temperature is the kinetic energy of a gas molecule 22. At same temperature, calculate the llltio ofiwerage velocity or
half of its value at 327°C? S02 toCH4 : . . (DeE 2006)

(a) 13.soC (b) 150°C (c) 27°C (d) -123°C (a) 2:3 (b) 3:4 (c)l:i (~,;6
IAns. (c)] [Ans. (c)]

[Hint: KE ~RT pRT


2 11:m
232. I G~.R.B. PHYSICAL CHEMISTRY FOR COMPETITIONS

b==[~nr3 JX4N
where'r == radius of gas molecule
N == Avogadro's number
(b) Pressure correction: A molecule in the interior of the
gas is attracted by other molecules on all sides. These forces,
4.9 . VAN DER WAALS' EaUATION
thus, are not effective,· as equal and opposite forces cancel each
The gas which conforms to all of the postulates of kinetic theory other. However, a gas molecule which is just going to strike the
or which obeys Boyle's law and Charles' law l~gii\ly for all values waH of the vessel experiences an inward pull due to unbalanced
of temperature and pressure or which strictly follows the general attractive forces. Therefore, it strikes the wall with less
gas equation (PV nRT) is called ideal or perfect gas. Actually momentum and the observed pressure will be less than t~e ideal
no gas is ideal or perfect in nature. Under ordinary conditions, pressure.
only those gases nearly behave as ideal or perfect which have Pideal Pobs + pI
very low boiling points such as nitrogen, hy?rogen, etc. where P' is the pressure correction.
At low pressure and moderately high temperature, the-real
gases approach ideal behaviour (see fig.). Pressure Correction Depends upon Two Factors'
(i) The attractive force exerted on a single molecule about to
Real] atT strike the wall is proportional to the number of molecules per unit
----- Ideal 3

,OM
volume in the bulk of the gas.
\\=.. :Id:e:alll gas
!
Real] at T2
---- Ideal
(ii) The number of molecules striking the wall which is also
t
p
PV l
Real Jat T1
proportional to the number of molecules per unit volume of the
gas. .
~--- Ideal
Both these factors are proportional to .the density of the gas.
L..-.._ _ _ _ _ (T3> T2 >T 1 )
~

Therefore, the attractive force is proportional to the square of the


V- Pressure (P) - - density of the gas.
.(al (b) p' = total attractive force
2
Fig. 4.9 (a) Ideal and real gas, (b) Ideal and real gas (re~1 gas
=d
is approaching ideal behaviour with rise in temperature) . oc_._
2
V
It is observed that deviations from gas Jaws are high under
high pressure and low temperature. van def Waals suggested that or P'=~
V2
these deviations are due to the following two faulty assumptions-
where' a' is a constant depending upon the nature of the gas and V
. in the kinetic theory of gases:
is the volume of 1 mole of the gas.' .
(i) Actual volume of the gas molecules is negligible as . . a
compared to the total volume of the gas. Thus, corrected pressure,. ~deal =Fobs
. +V 2
(ii) Intermolecular attractions are not present in gases.
van der Waals pointed out that in the case of real gases, Making both the corrections, the general gas equation
molecules do have a volume and also exert intermolecular PV RT may be written as:
attractions especiaHy when.the pressure is high and temperature
is low. He applied two corrections: r\ P + ~J
V.
(V - b).=; RT
(a) Volume correction: van der 'Waals assumed that
molecules of a real gas are rigid spherical particles which possess The equation is called van der Waals' equation. van der Waals'
a definite volume. Thus, the volume of a real gas, i. e. , volume equation for n moles of the gas is:
available for compression or movement is, therefore, actual ( na
2 J.
\ P + V 2 (V - nb) nRT
.
volume minus the volume occupied by gas molecules. If b is the I
effective volume of the molecules per mol of the gas, the ideal
volume for the gas equation is (V b) and not V, i.e., vander Waals' equation is obeyed by real gases over a wide
corrected volume 'Vi' = V - b for one mole of the gas range of temperature and pressure and henpe this equation is
called equation of state for the real gases. The constants 'a' and
and for n mole of the gas, 'Vi'. V - nb.
'b' are called van der Waals' constants and they are characteristic
b is termed the excluded volume which is constant and
of each gas.
characteristic for each gas. The excluded volume 'b' is actual,ly
four times the actual volume of the gas molecules.
STATES OF MATTER 233
Unit of van der Waals' Constants Let us consider an isotherm at 13.1 0c. At low pressure, carbon
a and b are expressed in terms ()f the units of P and v. dioxide· is entirely gaseous and is represented by the point (A ) in
the isotherm. On increasing pressure, volume decreases as shown
n 2a
Pressure correction P' -2- by the portion AX 9f the isotherm, approximately in accordance
V with Boyle's law. At X, deviations from Boyle's law begin to
P'V 2 l>ressure correction x 2
appear and the volume decreases rapidly as the gas is converted
a =:---2-· = into liquid, At point Y, carbon dioxide has been completely
n (Mole) liquefied. Between X and Y, pressure remains constant and both
Thus, if pressure <!nd volume are expressed in atmosphere the gas and liquid phases are in equilibrium. The pressure
and litre respectively, the units of constant 'a' will be corresponding to the horizontal portion XY of the isotherm is the
atmosphere iitre 2 mol '-2 . .vapour pressure of the liquid at the temperature of the isotherm.
The isotherm at 21.5°e shows a similar behaviour except that
SIunit, 'a'·=Nm 4 mol-2
liquefaction starts at higher pressure and the horizontal portion
'b' is the effective volume of the gas molecules in one gram 'MN is shorter. As the temperature is raised, the horizontal portion
mole of the gas. Thus, the unit of 'b' is the same as for the
volume, i. e., litre mor- l .
SI unit, 'b' m 3 mol- l
Boyle temperature: The temperature at which real gas
behaves like ideal gas and obeys the gas laws over a wide range
of pressure is called Boyle temperature' Tb '. At this temper~ture,
PV remains constant for an appreciable range of pressure
a 1
Tb =
Rb 2T; IN M\
~/_...........;_ _ _ _\....
\ _-13.1°C
where, T; inversion temperature /Y X\ A
a, b van der Waals' constants
. dPV . ·VolLime--
At Boyle temperature, - - "" owhen P approaches zero.
dP . Fig. 4.10 (a) Isotherms of carbon dioxide
showing critical re!;lion
4.10 CRITICAL PHENOMENON AND of the isotherm becomes smaller and smaller until at 31.1 °e at
LIQUEFACTION OF GASES which it reduces to a point P. Point P is called critical point; at
During the early, part of nineteenth century; a number of gases this point the boundary between liquid and gas phase< (meniscus)
such as carbon dioxide, sulphur dioxide, ammonia, etc. were disappears indicating that both the phases have identical
liquefied by subjecting the gas to low temperature and high characteristics. Above 31.1°e, there is no' indication of
pressure. On cooling, the kinetic energy of the gas molecules liquefaction. The isotherm at criticaltemperature 31.1 °e is called
decreases. The slow moving molecules come nearer to each other critical isotherm. The tangent at critical point P is horizontal, so
due to forces of attraction and, thus, aggregate and are converted that, : : at critical point wiil be zero. Thus, the point 'P' is also
into liquid. The increase of pressure can also bring the gas
molecules closer to each other and, thus, is helpful in converting called the point of inflection.
a gas into liquid. The effect of temperature is. rather more·· It may be ·concluded from this explanation that in the area to
important than that of pressure. The essential conditions for . the left of the dotted line b~low the critical isotherm, only liquid
liquefaction of gases were discovered by Andrews in 1869·as a carbon dioxide exists. To the right of the dotted line, only gaseous
result of his study of Pressure-Volume-Temperature (P-V-T) carbon dioxide exists. The horizontal portion, within the dotted
relationship for carbon dioxide. It was found that above a certain line shows tne equilibrium between gas and liquid phase.
temperature, it was impossible to liquefy a gas whatever the
pressure was applied. The temperature below which the gas The van der Waals' Equation and the Critical
can be liquefied by the application of pressure alone is called Constants
critical temperature (Tc ). The pressure required to liquefy a
The van der Waals' equation for 1 mole of a gas is given by:
gas at this temperature is called the critical pressure (Pc)' The
volume occupied by one mole of the substance at the critical
temperature and pressure is called critical volume (Ve ) . .fp + va2 (V b)=:- RX
The results of Andrews experiments are shown in the
following Figure. 4.1 O(a) in which the pressure is plotted against This equation may be written as,
volume at various temperatures for carbon dioxide. Each . PV+ a Pb ~=RT
pressure-volume plot is called isotherm. V V2
234 'I G.R.B. PHYSICALCHEMISTRyFOR COMPETITIONS

or PV 3+ aV Dividing equation (v) by (iv),w6 get, ,


Dividing above eqiiationby 'P', , Vc b
3
V 3., +aV/~'bV,2' i.e., Vc ' 3b' ... (vi)
p' p
Substituting the value ofVe from eq. (vi) in (v), we get
Arrangingindescendirigpbwers' of V, we get·
, y 3 '('RT1'
b'+_'IV
2' aV
,+-,- , - , .,. (i)
(3b)
,
3= db
Pc
, p ; ", " f "
... (vii)
,'.Equation (i}isacubic equationinT~and ther~f6re. for. any
value of P andT, it will have three values of V, all of which may Finally, substituting the values of Pc and Vc in eq.(iii),we get,
be' real ,or ,one mayoe real, and, the other two imaginary. When
8a-
Tc =-'-' ( ... )
... Vlll
pressurevtirsusvolume plots are constructed using equation (i)
27Rb
we get the curves as shown in Fig: 4JO(b)..
. We can expressth~ constants p, band R in terms ofCritical
It is evident from the Fig. 4.10 (b) that the curves at and above
constants as:
the critical temperature are similar to those in Fig. 4.10, (a).
However, below critical temperature, the horizontal portion b=
determining the coexistence of gas and liquid is replaced by a (~) 3
shaped curve ABC in Fig. 4.10(b). Thus, this curve predicts that
there are three values of V corresponding to the points A, Band C.
At the critical point'P', the three roots of van der Waals' equation
are not only real and positiyebut also identical and equal to the
critical value Vc' This condition may be expressed as, ... (ix)

Equation (ix) is called equation of critical state. Critical


compressibility factor of a gas may be calculated as,

, If ~)(3b)
= PYc = 2 7 b = 0.375
RT t
R(~)
, 27Rb, /

J:~ constants .(gases, ~ ;l'

Gas .P",(abn)
2.3
,VQlum.e.~
12.8, ' 65 33.2
Fig. 4.10 (b) Isotherms of carbon dioxide according to Ne' 26.9 41.7 44.4
van der'Waals~ equation, c

N2 33. 6 90.1 ' 126.1


V 02.- 50.3 74.4 154 .5
CO 2 ,72.7 95.0 304.2
or V Vc 0
H2 0 '218.0 55,6 647.3
(V ~Ve )3=0
NIl 3 112.0 72.0 4055
'" (ii) , CH 4 45.8 99.0 191.0
At critical point, equations (i) and (ii) must be identical. Com- C2 H6 48.2 139.0 305.5
paring and equating the ,coefficients of like powers of V gives: C2 H4 50.5 124.0 417.2
RTc ' (: ... ),
3Vc b +-- ... 1Il
, Pc
4.10.1 EXPERIMENTAL METHODS FOR
3V
, C
2
'
=!!...,
P ... (iv) LIQUEFACTION OF GASE¥"
e Discovery of critical' phenomenon by Andr~s in 1869 showed
V 3 = ab ... (v) that gases cannot ,be liquefied by the application of pressure
c Pc alone; they must first be cooled below.their critical temperatures
In above equations,T::::: Te , P Pe at critical state. and then subjected to adequate pressure to cause liquefaction.
STATES OF MATIER .' 235
Principles involved in liquefaction are: , temperature of oxygen (-1 1SoC) whkhis filled in the imler tube
(I) A gas must be at or belowitscritica:l temperature,Lower of C. Oxygen gets liquefied during the compressioIlstroke of
the temperature below tile critic.al. value, ea::;ier would be the PumP~.[>3 and iscgl1ectedin.th,e I>ew~r flask D.
liquefaction. .
(b) Cooling by Joule-Thomson Effect
. (2) The gas is ~ooled either by doiqg external workor by .( Linde's process fridiquefaction of air)
expanding against the internal forces of molecular attraction. Wheria cOITlpressed gas isalioWed toexpandirltoalOW
Low temperaturefor liquelaction of gases can be.achieved by pressure or vacuum under adiabatic conditions,. a lowering of
the following techniques: . > •. ••.•• • .'. '. ••• ..., ••
temperature is observed. This is. kHownas Joule-Thoms0!i effect.
(a) Cooling by rapid evaporation of avcilatile liquid. In the expansion; molecules oIthe gas moVe far apart from one
(b) Cooling byJoule~Th6mson effect. . •... artothei. Work is'done by the gas molecUles to· over~ome
(c)Ccioling bya:diabatiC expansioninv6Ivingmechaiiical iriterniolecularforces. Work' is done· at the cbst of the' kinetic
work; . . ..
energy of gasinolecufes. Consequ:entlya co~ling· effect is
(~)Cooiing by Rapid Evapor~tion of a Volatile ~iquid ... observed. For each gas, there is a characteristic temperature
above which a gas onexpan~ionsh9ws.!iheating effect while
This method was first employed by Pictet arid CailIetet.· An below it the gas;cQQlsoneJS:pansiort.This temperature is known
easily volatile liquid is rapidly evaporated to c001 andliquef)! a as inversion t~niper~tureofthe gas. This temperature is related
less volatile liquid. This is the principle underlying the cascade to van der Waals' constant.a and b by the expression:
processJFig; 4·11 (3,)J for liquef"l(~ti09Pfq2·
•. r ,::;;24
I Rb
where, Ti is the inversion temperature of the gas.
Gases like H:2 and He have low inversion temperatures (TiHl
. == 193 K and Tj He = 33 K) and they. show heating effects in
Joule-Thomson expansion under ol"dinarytemperature. If these
gases are cooled below their inversioll temperatures, they also
show cooling effects.
Cooling jacket

Liquid oxygen -- '@tEE!

Fig. 4.11 (a) Cascade process for the


liquefaction of oxygen

The apparatus consists ofilireecompressiol1 pumps PI' P2 and


P3 and three' cotnpression chambers A, Band C.The compression
pumps are joined in series. The pump PI "compresses methyl
chloride gas which is cooled by cold water crrcuiating in outer Fig. 4.11 (b) linde's process for liquefaction of air .
jacket of A. As the critical temperature of methyl chloride is
143°C, it is liquefied at roomtemperature~ The liquid methyl Joule-Thomson effect is the basis forliqtiefaction of air by
chloride passes through valve VI into the outer jacket of B which Linde's process. The apparatus is shown in Fig. 4.11 (b).
is Connected to the suctionside of Pl. As a resultof reduced Pure and dry air is compressed toa pressure of about 200 atm.
pressure, the ·liqUid. rri~thyl chlori.de· evaporates and the Any ,heat produced during .the cOlllpression; is .rellloved by
temperature inB reaches -90°C. . .. . passing; the gas through a cooling jacket. Compr~ssed air is
allowed to expand 'suddenly through a nozzle (N) tQ iii. large
. The inner tube of B is filled with ethylene gas which is cooled
. chamber, where it gets cooled and the pressure reduced to about 1
to ~90°C. As the critical temperature of ethylene is 1()OC, it gets
atrnosphere: The cooled air is ma,de to pass through the outer tube
liquefied and passes through valve Vi .into the outer jacket of C.
and is sent back to the compressor. The incoming air further cools
Ethylene liquid is allowed to evaporate· under reduced pressure
on expansion. The process is continued until air liquefies and is
with the help of pmnp P2 resulting in a sharp· fall in the
collected at the bottom.
236 I. GG.R.B.· PHYSICAL CHEMISTRY FOR COMPETITIONS

(c) C()oling by Adiabatic Expansion Involving The extent to which a real gas departs from ideal behaviour is .
Mechanical Work expressed in terms of compressibility factor.
When a gas· is allowed to expand adiabatically against a . Z = Molar volume of real PV",
pressure, it does some external work at the cost of its kinetic Molar volume of ideal gas (VIII) RT
energy, due to which its temperdtur~ falls. This principle is used
in Claude's process for liquefaction of air. The; apparatus is When Z > I, V", > 22.4 L at STP and when Z < 1, Vin < 22.4 L
shown in Fig. 4.11 (c). at STP.
Pure, ·and dry air is compressed to about 200 atmospheric . (i) Z 1for ideal gas. Deviation from the unity indicates extent
pressure and then passed through the tube T. Compressed air of imperfection or non-ideality.
divides itself into two parts at 'A' and a 'portion enters through 'C' (ii) Real. gases have Z"" 1 at low pressure and high
fitted with an airtight piston. There, it expands, pushing .the temperature. In this case, the real gas behaves like ideal gas.
piston outward and performs some external work. . (iii) Z> 1 shows that it is difficult to compress the gas as
compared to ideal gas. It is possible at high pressure. In this case,
'T' repulsive forces dominate.
D (iv) Z < I shows that the gas is easily compressible as
Compressure compared.to ideal gas. It is possible at intermediate pressure. In
this case, attractive forces are dominant.
t . Plots of Compressibility Factor against Pressure

Figure 4.12(b) indicates that an increase in temperature shows


decrease in deviation ideal behaviour, i. e. , Z approaches unity
with increase in temperature. . .

Z>1

t Ideal gas
Z
Fig. 4.11 (c) Claude's apparatus for liquefaction of air (I) At very low pressure, PV RT, i.e., Z "" 1
(ii) At low pressure, PV <RT, Z< 1
The temperature of the gas thus falls. Cooled air then enters Z<1 (iii) At high pressure, PV > RT, i.e.; Z > 1
the chamber at B and cools the incoming air. Thetemperature of
P--
air is also reduceq on expansion due to Joule-Thomson effect.
The process continues until the air liqt\efies. Fig.4.12(a) Variation of compressibility factor against
. ~
,~-""

Tendency of Liquefaction of a Gas


Easily liquefiable gases have greater intermolecular force
which js represented by high value of 'a'. Thus, greater will be
the value of 'a 'more will be tis liqueflab'i/ity. .
Easily liquefiable gases are:.
etc. Z --------------- Ideal gas
Decreasing value of constant' a'
(Decreasing tendency of liquefi~bility)

Above gases have higher value of 'a' as compared to the .


permanent gases O 2 , N 2, H 2 , He, Ne, etc.
P--
Incompressibility of a Gas
. Fig. 4.12(b) Variation of compressibility factor against
van der Waals' constant 'b' represents the excluded volume, pressure ~t Ctifferent temperature
i. e. , four times the volume occupied by the gas per mole. Value of .
'b' remains constant over a wide range of temperature and Note: (i) In case ofH 2, Z > 1ail73 K andZ increases with the pressure.
pressure which implies that gas molecules are incompressible .. When temperature is less than Tl3 K, H2 has Z < L
Compressibility factor: Mathematically, it may be defined as, (ii) For gases other than hydrogen; at 273 K there is decrease in
. 'Z' at initial· stage, with increase in pressure, value of 'Z'
Z PV approaches a minimum; with increase in pressure.
nRT
STATES OF MATTER I ·237

(iii) In case of easily liquefiable gas like cal> Z dips largely Ratio of Molar Heat Capacity
below ideal line in low pressure region, see Fig. 4.12 (a). It is denoted as y.and it represents atomicity of the gas.
Specific Heat of Gases 5
. C R
Amount of heat required to raise the temperature. ofl g of a For mono atomic ga,'I, (y) = ~ = 1.66
substance through laC is called its specific heat. It can be . Cv 3
measured at constant pressure and at constant volume. 2 R.
Molar specific heat, i. e. ,heat required to raise 1 gram mole of
a substance through laC is called heat capacity.
2R
For diatomic gas, (y) =.L 1.40
Heat Capacity at Constant Volume ~R
It is denoted by C v. It may the heat supplied to one mole of a 2
substance to increase the temperature through I °c at constant Cp
volume. .
F ..
or tnatoLllC gas, (y) = -
. Cv
= 4R
3R
= 1.33

We know from kinetic theory that,
1 2 1 2 3 Na'\lNne YAt"f¥ty
PV -mnc or -mnc =-RT
3 2 2 He 5 3.01 1.99, 1.661
Thus, kinetic energy E
3 RT N2 6.95 4.96 . 1.99 1.4 2
2
When temperature is raised through 1°C then the increase in O2 6.82 4.83 1.99 1.4 2
kinetic energy may be given as: CO 2 8.75 6.71 2.04 1.30 3
3 3 3
Increase in kinetic energy -R(T+l) -RT=-R H2S 8.62 6.53 2.09 . 1.32 3
2 2 2
Now, C v . ~ R since at constant volume, heat supplied to raise Example 46. Specific heat of a monoatomic· gas at
2 constant volume is 315 J K -I and at a constant pressure is
the temperature is used up to increase the kinetic energy only.
525 J kg -I K -I. Calculate the molar mass of the gas.
3
Cv = - R (for mOIioatomic gases only, since monoatomic
2 Solution: Cp M x 525 and Cv Mx315
gases undergo translational motion only] where, M is the molecular mass.
3 R + x [for di and polyatomic molecules; vibrational C p -C v R (R=8.314JK,1 marl)
2
Mx525 Mx315=8.314
and rotational motions also contribute to
the total kinetic energy] M (525 315) = 8.314
where factor' x' varies from gas to gas and its value is zero for M = 8.314 = 0.0396 kg mol- I 39.6 g mol-I
monoatomic gases. 2lO
Example 47. Calculate the pressure exerted by 16 g oj'
Heat Capacity at Constant Pressure methane in a 250 mL container at 300 K using van der Waals'
It is denoted by C p; it may be defined as heat supplied to 1 equation. What pressure will be predicted by ideal gas equation?
mole of a substance in order to increase the temperature through a= 2.253 atm L2 mol -2, b 0.0428 L mol -I ,
1°C at constant pressure.
R = 0.082ILatmK- 1 mol-I
Heat supplied at constant pressure will be used in:
16
(i) Increasing kinetic energy, which is equal to ~2 R . + x. For Solution: GIven, 16 g CH4 = 1mole
16
monoat~mic
. gas increase in kinetic energy is equal to 3 R when
2
Applying van d7 wa:l~: elquation,

temperature is raised through 1°C. t,P + V 2 (V - nb)= nRT


(ii) Doing work due to volume expansion.
2
We know that, PV = RT for 1 mole of gas ... (i) P= nRT n a
When temperature changes to (T + I), then (V -nb) V2
P(V + L\V) = R(T + I) ... (ii) Substituting n = 1,
Subtracting ~q. fi) from eq. (ii), we have R:= 0.0821 L atm K- 1 mol-I; T = 300 K; V = 0.250 L;
PL\V = R \v'~,k done due to volume expansion b:= 0.0428 L mol-I
Thus, C p = 3 R + R = .2 R for monoatomic gases
2 2
Cp C)' +R orCp -Cv R (general equation)
238 G. R. B. PHYSICAL CHEMiStRY FaFi.coMPETITIONS
-~.

P (~)r p+ ~
\
The ideal gas equation predicts that, [AnS'. (b)]
p nRT" 1 x 0.082 Ix 300 = 98.52 a~m
.[Hint: nb) == nRT
V 0.250
Example 48. Calculate the' temperature of the gas if it
obeys van der Waals' equation from the following data. Aflask of For n I, (p +a. \(.v .:.P...) = iRT
2 . '., 4V2) , '2 2·
..
.
2.5 litre contaihs 10 mole of a gasundJ;- 50'atrrl. Given
a = 5.46atm Zitre 2 mol ::2 and b=o.03Ilitremol- l .

Solution: Given that, 26. van derWaals' constants of two gases .Kandt are as given:
P = 50 atm; V 2.5litre, n =: 10; a = 5.46 atm litre mol-
2 2
; a (litre-atm mol-2 ) b (litre mol ~l)
Gas X '5.6 . , OJ/65
,:~;~J)iQJlli~rervql-l;
i
R :=O.08211itre- atJ;nK- mQEt",
ctasY ·5.1. 0,012
Applying van der Waals' equation, . What is correct about the two gases?-
(b) Te(X) = Te(Y)
[
p+ant
V
J(V -nb)= .nRT (a) Te(X) > TeeY)
(c) Ve(X» Vc(Y) (d) Vc(Y) > Vc(X)
[Ans.(q)1
an2 (Hint: Gas X. Gas l'
[p+ (V-nb)
Vc = 3b 3 x 0,065 = 0.195 3 x 0.012 0.036]
or T= ---.~~. _ _c ' - - _
27. Select the correct statements about van der Waals' constant
nR
'b',
(2.5-10x 0.031)
r\ So.;.5,46XIOO)'
. (2.5i .
I'. Itisexc1uded volume

.~ "if:".;";.':;

IlLIlSTM1rONS
,'r.j
.
:,.
. 366.4 K
10 X 0.0821
'~il':.··
OF 'OBJECTIVE QUESnQNS ,
~_.,·.:~<'.:n
2. Its unitis mol litre- 1
3. It depends on intermolecular force
4. Its value ddpends on molecular size'
(a) 2,3 {b).I, ~,4. . (c).2, 3, 4 (d) 3, 4
[Ans. (b)]
23 •. At high.temperature and low pressure, the van der Waals' ; 28" .. Gases X, Y,
equation is reduced tp:
(a)(p+ a V· RT (b}PV=RT

(c)P(V b) RT
The gas with the highest critical temperature is: .
• [Ans.. (b)] . " . [PEt(K~rala) 2006[
[Hint:,At high temperatur·eand lowpressure,V. in (a); P (bJ Q '(c) Y (d),Z
comparison'to'liarid( ;2) is negligible in comparison to P. (e) X
[Ans. (e)]
Hence, the equation reduces to PV ~. RT .]
[Hint! . Cntic~1 temperattu-e, Tc
24. The constant'a'in'vander Waals'eqilatiOl1 is maximum in: 27Rb
(a) He (b) H2 :. treater is the value of (al b~ more is the criti~~i temperature
of gas:
(c) 0 2 , ( d ) N H ' l
[Ans. (d)]. . For gas X, Te will be maximum.
[Hint: Intermolecular force ofNH3 is maximum, hence its van T = 8a
del' Waals' constant (a) will also be maximum.] c . 27Rb . ; , .....

25. The van der Waals' equation for 0.5 mol gas is: ~ for X = c= 240; for all: oth~rgases, (!!.)
is lesser.]
b 0.025 " ' . ' b, '.
(
(a) I P+
a I/V . 2RT
=- 29. At high pressure, van der Waals' equ3,tion becomes:
\ \ 2 2
(DPMT 2006)
.. a
(2Vb)= RT (a) PV =C.RT (b)PV=,R], +v
(c) PV ==:RT-!!:.. Cd) pi! =RT + Pb
(C)(p+ a (2V-4b)=RT V
[Ans.' (d)]
STATES OF MATIER. 239

~ nb)
2
[Hint: (p + a n (V nRT
Solution:

(p + ;2) (V ~ b) = RT forI mol gas


}60m
(p + ;2) '" P
Athigh pressure,

P(V ~b)=RT .
. PV =RT +Pb]
_}60m (76 + 38) cm

(Initial state) \Aft~r heating)


. : :: :::I.SOME SOLVED ExAMPLES\ I::::: :
'. Example 49. Galculatethe 'total pressure ina 10 litre At initial stage:
cylinder. which contains 0.4' g ofheliutn, L6g of oxygenan'd 1:4 g Pressure of gas = Pressure of Hg + Pressure of·
ofnitrogen at 27°C Also calculate thepartial pressureojhelium . atmospheric air
, gas in the cylinder. Assume ideal behaviour, of gases.
=16+.76= 152cm
Given that, R '0.082litre- atm K- I mOrl. (lIT 1997)
T 300K.
Solution:
V=Yl 12 where, VI is volume of9ylinder.
0.4
nHe = 4 ., At final stage after heating:
Pre~sure of gas = PressureofHg + Pressuieof
.1,4 128 =0.05 . .' .... .. atmospheric air
PHexV IlHe xRxT 38 + 76 =:: 114'cm
PHe X 10= 0.1 x 0.0821 x 300 T=?
4 , .
PHe 0.2463 atm
Applying gas equation; w(,(have.. .
P x V= (/lBe + n02 + n N2 )RT 152x 114x(3Vl/4)
P x 10 ± (0.1 + 0.05 + 0.05) x 0.0821 x 300 2x 300 T
T= 114 X 3 x2x 300
P = 0,4926atm
152
Example SO. All evacuated glass vessel weighssO,Og = 3375K
when empty; 148.0 g wh~n filled with a liquid of densityO,98
E~ample 52 •. A thin tubeof~niform cross"section is sealed
glmL -I and 50.5gwhenfilled with an i~ealgas.at 760mm Hgat at both ends. It lies horizontally, the middle 5 cm containing Hg
300K. Determine the molecular weight ofthegas. (lIT 1998) and the two equal ends containing air at the same pressure Po.
Solution: Mass ofliquid=148 -50= 98g When the tube is held at an angle 60° with the vertical, the
lengths of the air column above and below the mercury are 46
' 'd
. Volume o.fI lqUl mass = 98 1.0.0mL and 44.5 em respectively. Calculate pressure Po in cm ofHg. (The
temperature of the system is kepJat30°C).
Volume of vessel = Volume of liquid
Mass of gas 50.5 - 50 0.5 g
On applying ideal gas equation, we have,
. PV", w RT
m
760 100
-x 0.5 x 0,0821 x 300
760 1000 m
m 123
Example ~; 1. A vertical hol{ow cylinder ofheight 1.52 m is
'. fitted with a movable j.'iston ofnegligible mass and thickness. The
lower half of the cylinder contains an ideal gas and the upper ,- Scm -,
halfisfilledwith mercury. The cylinder is initially at 300K. When
the temperature is raised half of the mercury comes out of the
cylinder. Find the temperature assuming the thermal expansion Solution: .At horizontal position, let the length of air column
of mercUlY to be negligible. in tube be L em.
"

240 G.(jii(·.B. PHYSICAL CHEMISTRY. FOR COMPETITIONS

2£ + 5 46 + 5 + 44.5 cm
L= 45.25cm
When the tube is held at 60° with the vertical, the mercUry Also P2 12a = Pill a
column will slip down.
12 = Pill.
Py + 5eos 60° = P x
P2
5 '.
Py = 2.5cmHg ... (i) 76x 10 (c)
2 ---8--' =9.3 em
From end X, Po x 45.25=Px x 44.5 76+-'-
P _ 45.25 P .fi
.... (ii)
x - 44.5 0 Example 54. The stop cock connecting two bulbs of
volume 5 litre and 10 litre containing an ideal gas at 9 atm and 6
From end Y, Po x 45.25 = Py x 46
atm respectively, is opened. What is the final pressure in the two
. P 45.25 P ... (iii) bulbs if the temperature remains the same?
y
46 0
Solution: PI VI + PZV2 PR (VI + V2 )
Substituting the, values of Px and Py in equation (i) we get
9x 5+ 6x lO=PR (15)
Po = 75.4
PR = 7atm
Example 53. A lOcm column ofair is trapped by a column
of Hg, 8cm long, in a capillary tube horizontally fixed as shown where, PR resultant pressure after mixing.
below, at latm pressure. Calculate the length of air column when Example 55. At what temperature is the average velocity
the tube islzxed at the same temperature (a) vertically with open ofO2 molecule equal to the root mean square velocity at 27°C?
,end up (b) vertically with open end down (c) at 45° with the -:;B~Jl\D1jJit95)
horizontal with open end up.
Solution: .

~8RT =
1---10cm--->4 nM M
8RT 3R x 300
nM M
T = 353.57 K
(a)
t = 80.57° C
. Example 5~. The composition of the equilibrium mixture
for the equilibrium Cl 2 ~ 2 CI at 1400 K may be determined
(b) by the rate of diffosion of mixture through a pin hole. It is found
that at 1400 K, the mixture diffuses 1.16 times as fast as krypton
l
diffuses under the same conditions. Find ihe degree' of
dissociation ofC1Q. equilibrium. '(II(jIt1)<)~5)
or Pill a = P2 12 a Solution: Equilibrium of dissociation of Cl 2 may ~e :.
where, a area of cross section of tube represented as:
11 '/2 = length of air column 02(g) ~ 20(g)
P2 =76+8=84cm 1=0 a 0
teq a(l 0:) 2a 0:'
12 = Pill = 76x 10
P2 84 Total moles = a(l- a ) + 2aa = a(l + a )
MCI2
= 9.04 em M mix = =--
a(l + a) (1 + a )
Pilla P2 12 a
(b) P2 = 76 8 68cm
76x 10
68
11.17 cm
R" ~:~:
1.16= - - -
(c) When the tube is held at 45° with open end up, the weight
. of Hg is borne partially by the gas and partially by the Hg.
Vertical height of Hg is a measure of additional pressure on gas, a; a = 0.1374
i.e.,
Example 57. A gaseous mixture of helium and oxygen is 0.5= 100 x V
found to have a density of 0.518 g dm- 3 at 25°C and 720 torr. 0.0821 x 273
What is the per cent by mass of helium in this mixture? V 0.1l2litre
Solution: We know that, Pm::: dRT According to van der Waals' equation~

(P+;2 )(V-b)=RT for 1 mole


720 X m = 0.518 X 0.0821 X 298
760 "
m 13.37 g / mol
[ 1~ + a
(0.112)2
1
[0.112 0] = 0.0821 x 273
Let mole fraction of He in mixture be a.
On solving, we get a = 1.253 L2 mol-2 atm
xHe=a,xO z (I-a)
Average mw a X M He + (1- a )M O 2 ,,~"'Es:ainple61. The density afthe vapours ofa substance at 1 .'..
')0,: ....

atm pressure and 50() K is 0.36 kg m-3. The vapours effuse


13.37::: a X 4+ (1- a )32
through a small hole at a rate of 133 times faster than oxygen
a 0.666 under the same~onditions:
%by mass of He == 0.666 x
4
x 100 (a) Determi~e: (i) molecular weight, (ii) molar volume,
0.666 x 4 +- 0.334 x 32 (iii) compressibility factor (Z) of the vapours, (iv) Which forces
among gas molecules are dominating. the attractive or the
::: 19.95%
repulsive?
~tExample 58. Find the temperature at which 3 moles of (b) If the. vapours behave ideally at 1000 K, determine the
S02 will occupy a volume oflO litre at a pressure of15 atm. average translational kinetic energy ofa molecule. (liT 2002)
a == 6.71atm litre 2 mor2 ; b =0.0564 litre mol-I
Soluti~n: (a) (i) rvapour __ ~02
an2j r02 Mvapour
Solution:
(
P+-I(V
2
-nb)=nRT
V
~
)
1.33=
(
15 +6.71x 9) (10- 3 x 0.0564)=3 x 0.082x T VM vapour
100
M vapour
18.1
T 624K Molar mass
(ii) Molar volume = - - - -
Example 59. van der Waals' constant 'b' of Ar is Density
3.22 x 10-5 m 3 mol -I . Calculate the molecular diameter ofAr.
= 18.1 = 50.25 x 10-3 m- 3
Solutiom We know that,
0.36
b 4 x volume occupied by the molecules
101325 x 50.25 x 10-3
in one mole of a gas (iii) Compressibility factor; Z =:: PV
, RT 8.314 x 500
=:: 1.225
(iv) Z> 1shows that repulsive forces are dominant.
(b) Translational KE per molecule
or
3.22xlO-5 x3x7 JlI3 3 R
=:: x-xT
r::: ( 4 x 6.02 X 1023 x 4 x 22 2 N
::: 0.1472 X 10-9 m 3x 8.314 x 1000
Diameter == 2r= 2x 0.1472x 10-9 2 6.023 X 1023
== 0.2944 x 1O-~ m =:: 2.07 X 10-20 J
~ ,
;ft(,:'EtampJ.e 60. The compression factor (compressibility . ,·";¥tExample 62. The ratio of rate of diffusion ~f helium ,and
factor) for 1 mole of a van del' Waals' gas at O°C and 100 methane under identical conditions ofpressure and temperature
atmospheric pressure is found to be 0.5. Assuming that the volume will be: lIlT (8) 20051
of a gas molecule is negligible, calculate the van del' Waals' (a) 4 (b) 2 (c) 1 (d) 0.5
constant' a'. , (liT 2001)'
Solution: We know that,
Z=PV
Solution:
r~:e4 ~ '::4 =Hi =~
=::

RT r He 2rCH4
242 G;RB. PHYS!CALCHEMISTRyFoRGOMPETIT!ONS

SECTION 2 : LIQUID STATE


4.11 THE LIQUID STATE much less and their velocities are small, liquids diffuse slowly in
comparison to gases.
Liquid state is intermediate between gaseous and solid states. The
liquids possess fluidity like gases but incompressibility like (vi) Evaporation
solids.
The process of change of liquid into vapour state on standing
The behaviour of liquids explained above gives some
is termed evaporation. Evaporation may be explained in tenns of
characteristic properties to the liquids such as definite volume but
motion of molecules. At any given temperature, all the molecules
no definite shape, incompressibility, diffusion, fluidity
do not possess the same kinetic energy (average kinetic energy is,
(or viscosity), evaporation (or vapour pressure), surface tension,
however, same). Some molecules move slowly, some at
etc.
intermediate rates and some move very fast. A rapidly moving
Properties of liquids can be explained on the basis of kinetic molecule near the. surface of the liquid may possess sufficient
molecular theory which has the following postulates: kinetic energy to overcome the attraction of its neighbours and
(1) A liquid is made up of molecules; atoms in rare cases,. escape. Evaporation is accompanied by cooling as average
e.g., Hg (mercury). kinetic energy of remaining molecules decreases.
(ii) The molecules of the liquid are quite close together. Evaporation depends on the following factors:
(iii) The intermolecular forces of attraction in a liquid are {a) Nature of the liquid: The evaporation depends on the
quite large. strength of intermolecular forces (cohesive forces). The liquids
(iv)" The molecules of liquids are in a state of constant rapid having low intermolecular forces evaporate faster in comparison
motion. to the liquids having high intennolecular forces. For example,
(v) The average kinetic energy of the molecules of a liquid is ether evaporates more quickly than alcohol, and alcohol
directly proportional to their absolute temperature. evaporates more quickly than water, as the intermolecular forces
A liquid state is regarded as a condensed gas or a molten solid. in these liquids are in the order:
.Liquid state, however, has much more in common with solid state Ether < Alcohol < Water
than with gaseous state. The following general characteristics are Alcohol < Glycol < Glycerol
exhibited by liquids:
, .increasing extent of hydrogen bonding
(i) Shape
(b) Surfaee ar~a: Evaporation is a surface phenomenon.
Liquids have no shape of their own but assume the shape of Larger the surface area, grealer is the opportunity of the
the container in which they are kept. No doubt, liquids are mobile molecules to escape. Thus, rate of evaporation increases with
but they do not expand like gases as to fill up all the space offered increase of surface area.
to them but remain confined to the lower part of the container.
Rate of..evaporation ex:: Surface area
(ii) Volume (c) Temperature: Rate of evaporation increases with the
Liquids have definite volume as th~ molecules of a liquid are increase of temperature as the kinetic energy of the molecules
closely packed and the cohesive forces are strong. The molecules increases with the rise of temperature.
are not quite free to occupy any space. Rate of evaporation oc Temperature
(iii) Density (d) Flow of air current over the surface: Flow of air helps
As the molecules in liquids are closely packed, the de!lshies of the molecules to go away from the surface of liquid and,
liquids are much higher than in gaseous state. For example, therefore, incre~es the evaporation.
d~nsity of water at 100°C and 1 atmospheric pressure is 0.958 g (vii) Heat of vaporisation
niL-I while that of water vapour under similar conditions as . The quantity of heat required to evaporate a unit mass of a
calculated from ideal gas law
. .
(d MPi
RTJ
is 0.000588 g niL-I.
.
given liquid at constant temperature is known as heat of
vaporisation or heat of evaporation. The heat of vaporisation
depends on the strength of the intermolecular forces within the
'(hi) Compressibility liquid. The value of heat of vaporisation generally decreases with
The molecules in a liquid are ,held in such close contact by increase in temperature. It becomes zero at the critical
their mutual attractive forces (cohesive forces) that the volume of temperaUlre. When the vapour is condensed into a liquid, heat is
any liquid decreases very little with increased pressure. Thus, evolved. This is called heat of condensation. It is numerically
liquids are relatively incompressible compared to gases. equal to the heat of vaporisation at the same temperature.
(v) Diffusion '(vlii) Vapour pressure
When two miscible liquids are put together, there is slow When the space above the liquid is closed, the molecules
mixing as the molecules of one liquid move into the other liquid. cannot escape into open but strike the walls of the container,
As the space available for movement of molec~les in liquids is rebound and may strike the surface of the liquid, where they are
I
STATES OF MATTEff 243

of Hg) for water, alcohol and ether at different temperatures are


given in the following table:

Substanee
. ~. . . ·····.···~·········.r,'W(: .
Water 4.6 17.5 55.0 355.5 760.3
Ethyl alcohol. 12.2 43.9 135.3 812.6 1693.3
Diethvl ether I 185.3 442.2 921.1 2993.6 4859.4
It is observed that non-polar or less polar liquids such as
diethyl ether and ethyl alcohol possess fairly high vapour
pressures on account of weak intermolecular forces (cohesive
forces) whereas polar molecules such as water which have
Fig. 4.13 (a) Evaporation in a closed vessel stronger intermolecular forces possess relatively lower vapour
pressures.
trapped. The return of the molecules from the vapour state'to the
The simplest method for measuring vapour pressure is the
liquid state is known as condensation. As evaporation proceeds,
barometric method. It consists of two barometer tubes as shown
the number of molecules in the vapour state increases and, in
in Fig. 4.14. A small quantity of the liquid is introduced into one
turn, the rate of condensation increases.
of the tubes. It changes into vapour and rises into vacuum above
The rate of condensation soon becomes equal to the rate of the
evaporation, i. e., the vapour in the closed container is in
equilibrium with the liquid. Vapour
Evaporalion~
~
Liquid ~ Vapour
Condensation

Rate of evaporation Hot water


----------------------
t
Rate

Rate of condensation

Time-
Fig. 4.14 The barometric method for measurement of
Fig: 4.13 (b) vapour pressure

At equilibrium, the concentration of molecules in the vapour the mercury. Some more of the liquid is introduced and the
phase remains unchanged. The pressure exerted by the vapour in process is continued till a small amount of the liquid is visible on
equilibrium with liquid, at a given temperature, is called the 'the surface of mercury. The difference in the levels of mercury in
vapour pressure. Mathematically, it is given by the gas equation: the two tubes is noted and this measures the vapour pressure of
the liquid at atmospheric temperature. The measurement can also
p=!!... RT CRT be made at any desired temperature by surrounding the tubes with
V
a jacket maintained at that temperature.
where, C is the concentration of vapour, in moVlitre. The vapour pressure of a given liquid at two different
In closed vessel, temperatures can be compared with the help of
Rate of evaporation ex: e - EI RT (Boltzmann factor) Clausius-Clapeyron equation.
Thus, rate of evaporation remains constant at constant
temperature.
Since, the rate of evaporation increases and rate of
condensation decreases with increasing temperature, vapour
2.:R [~ - ;J
where, M/ is the latent heat of vaporisation and R is the molar gas
pressure of liquids always increases as temperature increases. At
constant.
any given temperature, the vapour pressures of different liquids
are different because their cohesive forces are different. Easily The actual form of Clausius-Clapeyron equation is:
vaporised liquids are called volatile liquids and they have -M/
_ _v_sP:....+A
10glO P
relatively high vapour pressures. Vapour pressure values (in mm 2.303RT
.,
Note: Vapour and gas: A gas can be liquefied only below critical temperature, by applying pressure. There is no characteristic critical temperature for
vapours.
I

244 1 j3.R.B... PHYSI(:..AL CHEMISTRY FOR COMPETITIONS

When AH\lap =Latent heat of vaporisation and •A' is constant.


A graph between log 10 P and ..!. will be a straight line with ,
. T ,,
I

negative slope. 10.75


s: .
f
I

:
, '

!
I
Sfope of the line I
I

-LiH\lap e I
I
I
:--
2.303R i -0.50
I
I

~ ,
I
I
I
:; I
I
I

! I
I
I
I
I
I
I
1fT ~ ,
I
I

Fig. 4.15 Variation of vapour pressure wiJh temperature. ,_ a 20 40 60 80 100


Temperature -
(Ix) Boiling point
Fig.4.16 Vapour pressure curves for diethyl etliet:, ethy
The temperature at which- the vapour pressure of the liquid alcohol and water' . .- .
.becomes equal to the atmospheric pressure is called the boiling
point of the.liquid. When a liquid is heated under a given applied (c) In vaporisation, the vapour molecules diffuse from the
pressure, bubbles of vapour begin to fonn below the surface of liquid into the atmosphere but in boiling, molecules escape with
the liquid. They rise to the surface and burst releasing the vapour sufficient pressure into the space over the surface of liquid
into the air. This process is called boiling. The nonnal boiling
(x) Freezing point "
point is the temPerature at which the vapour pressure of a liquid
is equal to exactly one standard atmospheric pressure (760 rom of When a liquid is cooled, the kinetic energy of the molecules
Hg). Fig. 4.16 shows that normal boiling points of diethyl ether, goes on decreasing. A stage comes when the intennolecular
ethyl alcohol and water are 34.6°C. 78.5°C and 100°C forces become stronger and the translational motion is redu(1ed to
respectively. minimum. At this stage, the fonnation of solid begins and the
The temperature of the boiling liquid remains constant until all liquid is seen to freeze out. The point (temperature) at which the
the liquid has been vaporised. Heat must be added to the boiling vapour pressure of solid and liquid forms of a substance become
liquid to maintain the temperature because in the boiling process, equal is tenned as freezing point.
the high energy molecules are lost by the liquid. The higher rate Normal fr-eezing point of a liquid is the temperature at which
at which heat is added to the boiling liquid, the faster it boils. its liquid and solid forms are in e'tuilibrium with each other under
. The boiling point of a liquid changes with the change in a pressure of one atmosphere. Th.freezing point of a liquid is the
external pressure. A liquid may boil at temperature higher thlUl same as the melting point of its solid fonn. At the freezing point
normal under external pressure greater than one atmosphere; the temperature of the solid-liquid system remains constant until
conversely, the boiling point of a liquid may be lowered than all the liquid is frozen. The amount"'Ofheat that must be removed
normal below one atmosphere. Thus, at high altitudes where the to freeze a unit mass of the liquid:at the freezing point is called
atmospheric pressure is less than 760 rom, water boils at the heat of fusion.
temperature below its nonnal boiling point of 100°C. Food cooks The freezing point of a liquid is affected by the change of
.more slowly at higher altitudes because the temperature of external pressure. With increased external pressure, the freezing
boiling water is lower than it would be nearer the sea level. The point of some liquids rises while of others falls.
tl;mperature of boiling water in a pressure cooker is higher than
the normal, thus trulking it possible to cook foods faster than in . (xi) Surface tension
open vessels. it is .the property of liquids caused by the intennolecular
If a liquid decomposes when heated, it ~an be made to boil at attractive forces. A molecule within the bulk of the liquid is
lower temperature by reducing the pressure. This is the principle attracted equally in all the directions by the neighbouring
of vacuum distillation. Unwanted water is removed from many
food pr~ducts by boiling it away under reduced pressure. molecules. The resultant force on anyone molecule in the centre
of the liquid is, therefore, zero. However, the molecules on the
Boiling and evaporation are sjinilar processes (conversion of
liquid into vapour) but differ in the following respects: surface of the liquid are attracted only inward and sideways. This
(a) Evaporation talres place spontaneously at all temperatures unbalanced molecular attraction pulls some of the molecules into
but boiling occurs only at a particular temperature at which the the bulk of the liquid, i. e. , are pulled inward and the surface area
vapour pressure is equal to the atmospheric pressure. is minimized. Surface tension is a measure of tbis inward force
(b) Evaporation is a surface phenomenon. It occurs ortly at on the surface of the Uquid. It acts downwards perpendicular
the surface of the liquid whereas boiling involves formation of to the plane of the slirface. The unit of surface tension is dyne
bubbles belQw,the surface of the liquid. cm- I or Newton metre -1 (N m -1). It is a scalar quantity. Surface
STATES OF MATTERl

tension is, thus, dermed as the force ading on unit length of a - In general,the liquids which wet the material show capillary rise.
line imagined to be drawn tangentially anywhere on the free Those liquids which do not wet the material of a capillary always
surface of a liquid at rest. It ads at right angles on both sides show depression, e. g. ,glycerine. honey, mercury, etc.
of the line and along the tangent to the liquid surface. Surface When a liquid rises in a column, its upper surface becomes
tension depends on the are!)., not on the length, like elastic concave and when the liquid goes down in a capillary, its upper
propel!!f;ies. _ surface becomes convex.
As the intennolecular forces of attraction decrease with the
rise of temperature, the surface tension of a liquid, thus,
decreases with increase in temperature: Similarly, addition of
chemicals to a liquid reduces its surface tension. For example,
addition of chemicals like soaps, detergents, alcohol, camphor,
etc., lowers the surface tension of water.

B (i) (Ii)

Fig. 4.17 (~) •~crt'Of shape and radius on capillary rise


. ." "" ,.;., .
(i) Height of liquid in the column of capillary is"independent
of shape of capillary if its radh,Js is same.
(ii) Thinner is the capillary, greater is the height ofliquid in the
column.
(e) Surface energy:- The work in erg required to be done to
increase or extend surface area by 1 sq. em is caned surface
Fig. 4.1.7 (a) Molecules at the surface are under unequai tension energy. The units of surface energy are, therefore, erg per sq. cm
Many common phenomena can be explained with the help of (or joule per sq. metre, i.e., J m-2 in SI system). Surface tension
-su;rface tension. Spme are described here: ., ofliquid is numerically fqual to the surface energy.
(a) Small droplets are spherical in shape: The surface Surface energy, -
tension acting on the SUrface of the liquid tries' to minimise the =
1 Work done to increase the surface area by one unit
surface area of a-given mass ora liquid. It is known that for a Work
given volume. a sphere has the _minimum . surface area. On =-----
Change in area
account of this.. drops of liquids a~uireIl
a spherical
. ~-
sbaPe.
(b) Insects can walk on the surface of. water: .Many = force x displacement in the direction of force
insects can walk on the surface of watet'without drowning. This area
is due to the existence of Surface tension. The surface tension
makes the water S'Ilrface to behave like an elastic membrane and Force
=---
prevents the insects from drowning. _Length
(c) Cleaning action of soap and detergents: Soap and deter-
The unit of surface tension is therefore dyne em-I (force per
gent solutions due to their 'lower surface tensions penetrate into
the fibre and surround the greasy substances and wash them away. unit length). ; (
(d) Capillary action:; The tendenCy of a liquid to rise into Measurement of surface Jension: _The surface tension of a
narrow pores and tiny openings is called capillary action. The liquid is' measured by the" droP count' metbOO using a
liquids rise in the capillary tubes due to the surface tension. The ~~~~ -

force of adhesion between the molecules of water and the Let YI and d t be the surface tension and density Of water and
molecules of glass of the capillary tube is greater than the force of 12 and d 2 be surface tension and density of the liquid whose
• cohesion between water molecules. Due to this, surface of water surface tension' is to be determi~_d.-Usjng a .stalagmometer the
in a glass capillary tube curves upwards in convex shape and then number of drops fonned by the same volume of water and liquid
the force of surface tensi9" of water pulls the water up into the is detennined.
tube. The water rises into a capillary tube to such a height that the 11. :ntdz
weight of resulting water column is just balanced by the force of 12 n Zd 1
surface tension. Using the above relation, the surface tension Of the liquid, 1 2,can
The rise of oil in a lamp wick: and flow of water from the roots be calculated.
of a tree to upper parts are also the examples of capillary action .
. N~te; Attlrecriu\':al temperatuie. surfa\':etensl0n oniqUid~ #(wbere the meniscus between the liquid and the v~ di~-s).
SurfaCe tension is scalar quantity. ;. y. - - '. . .~ _ •
I
G.R.B. PHYSICAL CHEMISTRY FOR COMPETITIONS

Sutface tension afthe liquid can also be measured by ~apillary one having lower velocity will experience a rel1miing effect due
rise method. ' to the internal friction between the two layers. This internal
friction or resistance is called viscosity.
-rcos9 Imagine a liquid to be made up of a large number of thin
cylindrical coaxial layers.
It has been found that force of friction '.f' between two
cylindrical layers each having area ' A' sq. cm separated by a
distance •x' cm and having a velocity difference v cm Isec i~ .
given by:

Fig. 4.18 Capillary rise v


J='I1A-
When liquid rises in a capillary, upward force due to surface x
tension is balanced by gravitational foi:ce~
2n..rlcQs(j ::::! mg ,
.•
, .wbere, m =~~s,.bf liquid in column
----...,..-----r--- Vern/sec
..........:':-T-.
'\.\
/'\\\ xcm
r ::i,ra.<n~:of capillary
h = capillary rise ;(rc+,H - I - - - - - - - - ' ! I > - - - t - - - - (V + v) em/sec
g =gravitational force , '
Mass of liquid in column (m}::r V x p ",,'\.\.""~:.*'----~---
2 )\"~,-,, .
=7.tr hxp
.'.'''~J/.
,

21t ry cos e =1&r2 hpg ,. j"

rhpg .!
Fig. 4.1.9
(p :;: density of liquid) Here,." is., a constant known as coefficient of viscosity. If x:;::; I
Y = 2cos e
cm, A = Icm 2 and v = lcm1sec., then /='11.
If angle of contact between glass and liquid is zero, i. e.,
cose 1, then ' ' Thus, coefficient of viscosity can be defined as the force per
unit area needed to maintain unit difference of velocity
rhpg
1=-2- between two consecutive parallel layers of the liquid which
are one centimetre apart.
Effect of temperature on surface tensi~ : Surface tension Coefficient of viscosity is expressed in dyne cm -'2, sec. It is
decreases with rise in. temperature because the intermolecular more commonly expressed as poise, centipoise (10-2 poise) and
force also decre8l)es with rise in temperature. The decrease in millipoise (I 0-3 poise), after the name of Poiseuille who derived
surface tension with rise in temperature continues till ctitical the formula and gave the method for its determination. 1 poise is
temperature at which the surface tension of liquid becomes zero, equal to a force oft dyne per unit area whlch maintains a velocity
i.e., menisCus between liquid and vapour disappears. difference of 1 cm per second between two parallel consecutive
According to EOtvos equation, layers of the liquid 1 em apart.
M)2/3 The reciprocal of the coefficient ofviscosity is called Fluidity
Y= (P . =K(tc-1) (4)) .
III :;: Fluidity =!
:.As t increases, y decreases linearly, when t :;: t c ' y ::: 0, here '11
t c is the critical temperature. Liquids with low viscosity are terme4 mobile and others with
(xii) Viscosity high viscosity are called viscous.
Viscosity depends on the following factors:
All liquids have a characteristic property of flow. Some
(a) Intermolecular forces: The liquids with high
liquids like water, alcohol, ether, etc., flow, quickly· while
intermolecular attractive forces offer greater resistance to the
glycerine, castor oil, molasses, etc., flow very slowly. The
flow of molecules and thus possess high viscosity:
property of tbe liquids wbicb determines tbeir resistance to
(b) Molecular mass: The flow of molecules is inversely
flow, is called viscosity.
proportional to its mass. Liquids' having high molecular mass
When a liquid flows through a pipe, all parts of it do not move possefs greater viscosity. . .
. .
at the same rate. The thin layer in immediate contact with the wall ~

(c) The structure and shape of molecules: Viscosity


'

of the pipe is almost stationary. The velocity of flow of each generally increases as' the branching in. the chain increases.
successive layer of liquid increases progressively as we proceed Symmetrica:l molecules have low viscosity,
inward towards the centre. At the centre, the flow becomes
fastest. Each layer ofliquid moving with greater velocity over the
I
"

STATES OF MATTER 247

(d) Tempertlture: Intermo lecular forces decrease with rise .,... (e) Pressure: the increase of pressure increases the
in temperature. Thus, viscosity decreases with the increase of intermolecular forces. Thus, the viscosity of a given liquid
temperature. This property is used to select the lubricant for !l increases with increase of pressure.
machine and engine. The variation of viscosity with temperature' (f) Density : Viscosity of a liquid increases with increase in
can be expressed by the fOllowing relationship: density whil6 that of gas decreases.
11:::: Ae-EaIRT ... (i) Measurement 4)f viscosity: Viscosity' is generally
determined by Ostwald's method. It involves the comparison of
where, A Pre-exponential factor (constant) time of flow of equal volumes of two liquids through the same
E(/ =Activation energy capillary using an apparatus called viscometer.
Taking logaritlm1 of equation (i) we get: 2= dXI
1 n", dw x (II'
log 10 n == log 10 A x- .. ·Oi)
2.303R T
where, 11 w , n == coefficients of viscosity of water and liquid
respectively; d:::: density of liquid; d w =density of water;
t == time of flow of liquid; t w :z:: time of flow of water.

-E Note: Viscosity of gases increases with increase in temperature.


Slope = 2.30; R Comparison between viscous force and solid friction
Intercept =10910 A Points of differences:

1. It is directly proportional to the It is independent of the area of


llT- area of layers in contact. the surfaces in contact.

Fig. 4.20 2. It is directly proportional to the It is independent of the relative


relative velocity between the two velocity between two solid sur-
This equation represents straight line with formula y = mx + C liquid layers. ,faces.
Equation (ii) may also be given as

log 10
n2 Ea
~ = 2.303R 1} - T2
[I I] Point of similarities:
(i) Both viscous force and solid friction come into action
where ever there is relative motion.
where n 2 is the coefficient of viscosity at temperature T2 and (it) Both these forces oppose the motion.
111 is the coefficient of viscosity at temperature TI . (iii) Both are due to molecular attraction.

SECTION 3 : SOLID STATE

'4.12 THE SOLID STATE


the container. This is due to close packing of molecules and
In solid state, the particles (molecules, ions or atoms) are closely strong short range intermolecular forces between them.
packed. These are held together by strong intermolecular (ii) High density and low compressibility: Solids have
attractive forces (cohesive forces) and cannot move at random. generally high density and IQw compressibility due to close
These are held at fixed positions and surrounded by packing of molecules which eliminates free space between
other particles. There is only one form of molecular motion in molecules.
solids, namely vibrational motion by virtue of which the particles
(iii) Very slow diffusion: The diffusion of solid is
move about fixed positions and cannot easily leave the solid
negligible or rather very slow as the particles have permanent
surface. The following general characteristics are exhibited by
positions from which they do not move easily.
solids:
(iv) Vapour pressure: The vapour pressure of solids is
(i) Qefinite shape and volume: 'Unlike gases and liquids,
generally much less than the vapour pressure of liquids at a
solids have definite shape and rigidity. This is due to the fact that
constituent particles do not possess enough energy to move about definite temperature. Some particles near the surface may have
to take-up different positions. Solids are characterised by their high energies (kinetic) as to move away and enter the vapour
definite volume which does not depend on the size and shape of state.
I
248' 'I G.RB. PHYSICAL CHEMISTRY FOR COMPETITIONS

, (v) Melting point: The temperature at which the solid and Crystalline solids are anisotropic. Magnitude of some of the
the liquid forms of a substance exist at equilibrium or both the physical properties of crystalline solids such as refractive index,
forms have same vapour pressure, is called the melting point. On coefficient of thermal expansion, electrical and thermal
supplying heat energy, the particles acquire sufficient energy and conductivities, etc., is different in different directions, within the
move away from their fixed positions in space. This results in the crystal. For example, in the crystal of silver iodide (AgI), the
formation ofliquid state. The solids have definite melting points coefficient of thermal expansion is positive in one direction and
, depending on the strength of binding energy. However, in some negative in the other direction.
solids (a~orphous solids)'" the melting point is not sharp. B D

4/13; FORMS OF SOLIDS


0
• • •
0 0
Solids are divided jnto two classes on the basis of haphazard and
regular aJ'!&lgement of the building constituents.
(i) Amorphous- solids: ,The term 'amorphous' has been 0
• •0
• 0
derived noma Greek word 'Omorphe' meaning shapeless. In
amorphous solids the arrangement of building constituents is not 0
• • • 0 0
regular but haphazard. Although these solids possess some of the
mechanical properties such as rigidity,incompressibility,
• • • 0 0 0
refractive index, etc., but do not have characteristic shapes or
A,
geometrical forms. Amorphous solids in many respects resemble
liquids which flow very slowly at room temperature and regarded Fig. 4.21 Anisotropic behaviour of crystals
as supercooled liquids in which the cohesive forces holding the
molecules together are so great that the material is rigid but there The phenomenon of anisotropy provides a strong evidence for
is no regularity of the structure. Glass, rubber, plastics, etc., are the presence of ordered molecular arrangement in crystals. This
some of the examples of amorphous solids. can be explained with the help of Fig. 4.21 in which a simple two
dimensional arrangement of two different kinds of atoms has
Amorphous solids do not have sharp melting points. For
been depicted. When a physical .property is measured along the
example, when glass is heated, it softens and then starts flowing
slanting line CD, it will be different from that measured in the
without undergoing any abrupt change from solid to liquid state.
direction of vertical line AB, as line CD contains alternate types
Thus, amorphous substances are not true solids but can be
of atoms while line AB contains one type of atom~_ only.
regarded as intermediate between liquids and solids. '
(ii) Crystalline solids: In crystalline solids, the building . 4~1S DIFFERENCES BETWEEN
constituents arrange themselves in regular manner throughout the
CRYSTALLINE AND AMORPHOUS,
entire three-diinensional network. The ordered arrangement of SOLIDS '
building constituents (molecules, atoms or ions) extends over a . ..... ...
large distance. Thus, crystalline solids have long range order. A CrystalllnftoUds AmoiJbous solids
crystalline solid con;ists of a large number of units, 'called
1. They have definite and regular They do not have any pattern of
crystals. A crystal is, defined as a solid figure which has a geometry due to definite and or- arrangement of atoms, ions or
definite geometrical shape, with Oat faces and sharp edges. , derly arrangement of atoms, ions molecules and, thus, do not have
A cryst.nine subst~nce bas a sharp melting point. i. e., it or molecules in three dimen- any definite geometrical shape.
changes abruptly into liquid state. Strictly speaking 'a solid state 'sional space.
refers to crystalline state' or 'only a crystalline substance c~ be 2. They have sharp melting points' Amorphous solids do not have
considered to be a true solid' . and change abruptly into liquidlt. . sharp melting points and do not
change abruptly into liquids.
~4~14,~ ISOTROPY AND ANIS()TROPY 3. Crystalline solids are Amorphous solids are isotropic.
>
The substances which show same properties in all directions are anisotropic. Some of their physi- Their physical properties are
said to be isotropic and the substances exhibiting directional cal properties are different in dif: same in all directions.
differences in properties are termed anisotropic. . ferent directions.
Amorphous solids like liquids and gases are said to be 4. These are considered as true sol- These are considered pseudo-
'isotropic as arrangement of building constituents is random and ids. solids or supercooled liquids.
disordered. Hence, all directions are, equal and therefore,' 5. Crystalline solids are rigid and Amorpbous solids are not very
properties are same in all the directions. their shape is not distorted by rigid. These can be distorted by
mild distorting forces; bending or compressing forces.

* Melting point of a solid depends on the structure of the solid. It is used for the identification Of solids whether it is crystalline or amorphous.
. STATES OF MATTER· 249

Crystaillne 'sollds Amorphous sollds axes of symmetry are called diad, triad, tetrad and hexad,
respectively, if the original appearance is repeated twice (after an
6. Crystals are bound by plane Amorphous solids do not have .. angle of 180°), thrice (after an angle of 120°), four times (after an
faces. Tqe angle between any well defined planes. angle of 90°) and six times (after an angle of 60°) in one rotation.
two faces is called intertilcial an- When an amorphous solid is bro-
These axes of symmetry are also called two-fold, three-fold.
gle. For a given crystalline solid, ken, the surfaces of the broken
it is a definite angle and remains pieces are generally not flat and
four-fold and six-fold. respectively.
always constant no matter how intersect at random angles. In general, if the same appearance of a crystal is repeated on
the faces develop. . through an ang1e 0 f -
rotatmg 3600
- .. around an"Imagmary aXIS,. t he
When a crystalline solid is ham- n .
mered, it breaks up into smaller axis is called an n -fold axis.
crystals of the same geometr>cal In all, there are 13 axes of symmetry possessed by a cubical
shape. .
crystal like NaCI as shown in Fig. 4.22 (c), (d) and (e).
7.. An important property of crys- Amorphous solids do not have (iv) Elements of symmetry: The total number of planes.
tals is their symmetry. There are: any symmetry. axes and centre of symnletry possessed by a crystal are tenned as
(i) plane of symmetry, (ii) axis of elements of symmetry. A cubic crystal possesses a total of 23
symmetry and (iii) centre of elements of symmetry.
symmetry.
• Planes of symmetry =(3 + 6) =9 [Fig. 4.2.2 (aland (b)]
4~16' TYPES OF SYMMETRY IN CRYSTALS Axes of symmetry (3 + 4 + 6) =13
(i) Centre of symmetry: It is such an imaginary point within [Fig. 4.22 (c), (d) and (e)]
the crystal that any;line drawn through it intersects the surface of
the crystal at equal distances in both directions. A crystal always
Centre of symmetry =1 [Fig. 4.22 (f)]
possesses only one centre of symmetry [Fig. 4.22 (f)]. Total number of symmetry elements 23
Oi) Plane of symmetry: It is an imaginary plane which
passes through the centre of a crystal and divides it into two equal 4.17. SPACE LATTICE AND UNIT CELL
portions such that one part is exactly the mirror image of the
All crystals are"polyhedra consisting of regularly repeating arrays
other.
of atoms, molecules or ions which are the structural units. A
A cubical crystal like NaCI possesses, in all, nine planes of crystal is a homogeneous portion of a solid substance made of
symmetry; three rectangular planes of symmetry and six diagonal
regular pattern of structural units bonded by plane surfaces
planes of symmetry. One plane of symmetry of each of the above
making definite angles with each other. The geometrical form
is shown in Fig. 4.22 (a) and (b).
consisting only of a regular array of points in space is called a
tV lattice or space lattice or it can be defined as an array of points
showing how molecules, atoms or ions are arranged in different

Rectangular plane Diagonal plane


iJ to I

Axis of four-fold
sites, in three-dimensional space. Fig. 4.23 shows a space lattice. A
space lattice' can be subdivided into a number of small cells
known as unit cells. It can be defined as the smallest repeating
unit in space lattice which, when repeated over and over again,
results in a crystal of the given substance or it is the smallest
of symmetry of symmetry symmetry (three) block or geometrical figure from which entire crystal can be built
(a) (b) (c) up by its translational repetition in three-dimensions. A unit cell

:,
tV
of a crystal possesses all the structural properties of the given
crystal For example, if a crystal is a cube, the unit cell must also
have its atoms, molecules or ions arranged so as to give a cube.

$ I,I,

8, .
,
JI----Jl~-.._-~/

,
I
Lattice
points

I
,

,
,"
I

Axis of three-fold Axis of two-fold Centre of


symmetry (four) . symmetry (six) symmetry (one)
(d) (e) (f)
Fig. 4.22 Various elements of symmetry in a cubic crystal
. . z
(iii) Axis of symmetry: It is an imaginary straight line z
about which, if the Cl}'stal is rotated, it will present the same
appearance more than once during the complete revolution. The Fig. 4.23 Space lattiee and unit cell
250 G.R.B. PHYSICAL CHEMISTRY FOR COMPETITIONS

' . ,MUIIIlnum symmetry


System,' , aies Angles Examples
elemeats
1. Cubic ",
An the three equal All right al1gles Nine planes, NaCl, KCI, Zn'S, diamclfld,.
II b =0, a =J) =1 =,90 0
thirteen axes alUms
2. Orthorhombic All unequal AU right angles Three planes, KN0 3, BaS04.K 2 S04, rhombic
a ¢ b '" c, a'" f):;: y = 90° three axes sulphur. '
3. Tetragonal TwoequaJ All right angles Five planes, TiO:!, PbW04,Sn02' NH.J Br
a::to b ¢ C, a = J) = y =: 90° five axes
4•. Monoclinic All unequal Two right angles Oue plane, Monoclinic slliphur, KClO 3,
. (!¢ l"f= C, 0: = Y= 90°, ~ ¢ 90° one axis CaSOq ·2H 2O,
Na2B40,·10H20
5. Triclinic All unequal" None right angles No planes, CuS04 ·5H20, H3BO "
a ¢ b ¢ c, a ¢ J) ¢ r¢ 90° no axis K 2Cr207 .'.
6. Hexagonal Two equal Angle between equal Seven planes, ZnO, CdS, HgS, SiC, Agl
a b ¢ c, a =J) =90°, Y = 120 0
edges 120° seven axes
7. Rhombohedral All thie~ -equal All equal but none Seven planes, NaNO), ICI, quartz (CaC03 )
= b :::: c, a=: J) :::: y ¢ 90° 90° sj:ven axes
<l
, .
Each unit cell has three vectors a, band C ,as shown in Fig. 4.23.
The di'stances a, b and c are the lengths ,of the edges of the unit
'Cell and angles ex ,13 and y aro the angles between three imaginary
axes OX,or and OZ, respectively.
FCir example: For a crystal system, a'" b c and
Il .,; ~ !::: Y :;<:,90° :. Simple or primifive Body-centred Face-centred
(a) .tetragOnal (b) hexag()~l
(a) Cubic space lattices
(c) rlIomb?hedral (d) monoclinic
(Ans. (c)]
[Hint:P'rom the table a = b == c fdr rhombohedral and cubic
I / / / / I I • /
.c •~ ,~ •b •
~ystem both but a;o::: ~'=y '.1:90° for rhomb~hedral only.] c c • c
tl ,u f\ ,u

4.18 CRYSTAL SYSTEMS "
;{;-y
, b
1/
" , /. l-y
Lt-v " b ~ ,u
\
b
/ i-v • /
On the basis' of geometri'Cal considerations, theoretically there Simple or Body-centrad 6nd-centrec;l Face-centred
c-an ae 32 different co!'nbinations of elements of symmetry of a primitive
erysta1. These are called 34 systems. Seme of 'the systemS' have, (b) OrtMorhombic space lattices
been grouped together. In all, seven types of basic or primitive
unit cells have beenrecogriised among crystals. These are cubic,
orthorhombic, tetragonal, monoclinic, triclinic, hexagortaland /
c
/. /
c.
/ /
c
/ /
c
• /
rhombohedral. ,These are shown in Fig. 4.24 and their
characteristics are summarised in the following table.
f\
,
,0:
'b ~, 'a
\

W-y
~ ,U
, \b ~ .
, -SX b
All crystals do not have simple lattices. Some are more
~-y / I>§-y bV / Yen /
complex. Bravais pointed out that there can be 14 different ways Simple Body-centred Simple End-centred
in which similar points can be arranged in a three-dimensional (c) Tetragonal and monoclinic space lattices
space. Thus, the total number of space lattices belonging to all the
seven crystal systems are 14.
The crystals belonging to cubic system have three kinds of
Bravais lattices. These are:
c
(i) Simple cubic lattice: There are points only at the corners
of each unit.
(U) Face-centred cubic lattice: There are points at the comers Triclinic Hexagonal Rhombohedral
as well as at the centre of each of the six faces of the cube. (d) Triclinic, hexagonal and rhombohedral space lattices

Fig. 4.24 Bravais lattices


STATES OF MATTER

3
(iii) Body-centred cubic lattice:There are points at the a xd
comers as well as in the body-centre of each cube. ',Number of mol per unit cell = M
The number of Bravais space lattices in a given system is =
where, M molar mass
listed in the following table and shQwn in Fig. 4.24. Number of moleCUles per unit cell= Number of mole
No. of . '
x Avogadro' s number
Crystal .. . 3
, space'.: Lattice type a 3 xd a· xdxN
system ". Z= xN=----
lattices ;

M M
1. Cubic 3 Simple, face-centred and body-centred. Calculation of number of constituertt units in
2. Orthorhombic 4 Rectangular and body-centred rectan- hexagonal unit cell
gular prism; rhombic and body-centred (i) Constituent units at each comer oCunt~Cell iSCOftllT10n"
rhombic prism. among six unit cells hence contribute 1/6th to each unit cell.
3. Tetragonal 2' Simple and body-centred tetragonal (ii) Constituent unit at edge is common to three unit cells
prism. hence contribute 1I3rd to each unit celL
4. Monoclinic 2 Monoclinic parallelopiped, monoclinic (iii) Constituent units present at the body centre is considered
face-centred parallelopiped. in single unit cell.
(iv) Constituent unit at face centre 'is common between two
5. Triclinic Triclinic parallelopiped. unit cells hence contribute 112 to each unit cell.
6. Hexagonal Hexagonal prism.
7. Rhombohedral Rhombohedron.
In various unit cells, there are three kinds of lattice points:
points located at the comers, points·in the face-centres and points
that lie entirely within the unit cell. In a crystal, atoms located at
the comer and face-centre ofa unit cell are shared by other cells
and only a portion of such an atom actually lies within a given
unit cell. .
(i) A point that lies at the comer of a unit cell is shared among
eight unit cells and, therefore, only one-eighth of each such point
lies withln the given unit cell.
Fig. 4.25 Hexagonal unit cell
(ii) A point along an edge is shared by four unit cells and only
one-fourth of it lies within anyone cell.
(iii) A face-centred point is shared by two unit Gells and qnly In thehexagonaJ unit cell (Fig 4.25), t2 comers, 2 face centres
one half of it is present in a given unit ceiL and 3 constiruent Mits wlthin the unit cells me Dccupied~ hence
(iv) A body-cel.itred point lies entirely within the unit cell and' effective number of con$tituent units.present in the unit cell may
contributes one complete oint to the cell. be calculated as
~--~---------------------- 1 occupied + 1 occupied Central
Contrlbutioa to one unit eeU' z 6 x comers 2 x",lacecIutres + constltuen.' f UlUts
.
118 1 1
Edge 1/4
=-x 12+-x2+3
6 2
Face-centre 112 6
Body-centre
Total number of constituent units per unit cell 4.19 DESIGNATION OF PLANES IN
=-1 x occuple. d comers + 1 x occuple. dedge-centres CRYSTALS-MILLER INDICES
8 4
Planes in crystals are described by a set· of integers (h, k and I)
+ 1 x occupied face-centres + occupied body-centre. . known as Miller indices. Miller indices of a plane are the
2
reciprocals of the fractional intercepts of that plalle on the various
Determination ofNumber of Constituent units per unit cell:
crystallographic axes. For calculating Miller indices, a reference
Let edge length of cube = a cm plane, known as parametral plane, is selected having intercepts
Density of substance = d g cm-3 a, band c along x, yand z-axes, respectively (Fig. 4.26). Then,
Volume of unit cell =a 3 cm 3 the intercepts of the unknown plane are given with respect to
Mass of unit cell volume x density = (a 3 X d) g a, band c of the parametral plane.
252 G.R.B. PHYSICAL CHEMISTRY FOR COMP.ETITIONS '

z In (a) intercepts are 00 00

So, Miller indices are o o


The plane is designated (100)
In (b) intercepts are 00

So, Miller indices are 1 o


The plane is designated (110)
1illP'=<------ Y
In (c) intercepts are 1
So,. Miller indices are
The plane is designated (III)
The distances between the parallel planes in crystals are
designated as d hkl . For different cubic lattices these iDterplanar
spacings are given by the general formula,
Fig.4.26 Parametral (intercepts a, b, calong x, yand'z-axes) a
d(hkl) = --;=====

Thus, the Miller indices are: ~h2 +e


+'/ 2
h= ____________a____________ where, a is the length of the cube side while h, k and 1 are the
intercept of the plane along x- axis Miller indices of the plane.
b The spacings of the three planes (100), (110) and (Ill) of
k = -----------------------
intercept of the plane along y- axis simple cubic lattice can be calculated.
I= ____________c____________ a
d(IOO) = =a
intercept of the plane along z-axis ~12+0+0
Consider the shaded plane ABD in Fig. 4.26. The intercepts of a a
the shaded plane along X:' Y and Z-axes are a12, band el2, d(llO) = ~ I 2 +1 2 +0 = ",L.
{;:.2
respectively. Thus,
a a a
h=-=2;
, al2 d(1I1) = ~ I2 +1 2 +1 2 = ",;j
{;:.3

k=%=~ The ratio is,


I I
1=~=2 d(lOO) :d(llO) :d(lll) = I: J2: J3 = 1:0.707: 0.577
c/2
The plane is, therefore, designed a~ (212) plane. Similarly, d(hkl) ratios for face-centred cubic and body-centred
Note: (i) If a plane is parallel to an axis, its intercept with that axis is cubic can be calculated. For face-centred cubic,
taken as infinite and the Miller indices will be zero.
. I 2 .
d(loo) : d(llO) : d(lll) = I: J2 : J3 = I: 0.707: 1.154
(ii) While defining Miller indices for orthogonal crystal, X; Yand
Z-axes are considered crystallographic axes.
Some of the important planes of cubic crystals are shown in
For body-centred cubic,
oFig.4.27.
I I I
d(loo) :d(lIo) :d(llI) =-: {;:.: r;:.
2 ",2 2",3

=1:J2: ~
= I : 1.414: 0.577
y y y

110 111 4.20 CRYSTALLOGRAPHY AND X-RAY


(b) x (c) DIFFRACTION
CrystallographY is the branch of science which deals with the
, Fig. 4.27 Miller indices of planes in cubic lattice,
geometry, properties and 'structure of crystals' and crystalline
substances. Geometric crystallography is concerned with the
/ external spatial arrangement of crystal planes and geometric
shapes of crystals.
STATESOF MAnER I. 253
Atoms, molecules or ions are too small to be seen with naked phase with one another only if the difference in the path length of
eye. The arrangement of particles in crystalline solids is the waves reflected from the successive planes is equal to an
determined indirectly by X-ray diffraction. In 1912, Von Laue integral number of. wavelengths. OL and OM are the
showed that any crystal could serve as a three-dimensional perpendiculars drawn to the incident and reflected beams. The
grating to the incident electromagnetic radiation with wavelength difference in path lengths of waves deflected from the first two
approximately equal to the internuclear separations (::::: 10-8 cm) planes is given by: .
of atoms in the crystal. Such radiation is in the X-ray region of LN + NM = n'A (n = 1, 2, 3, ... )
the electromagnetic spectrum.
Since, the triangles OLN and OMN are congruent, hence
The apparatus used is shown in Fig. 4.28. A monochromatic LN=NM.
X-ray beam is directed towards the surface of a slowly rotated
crystal so as to vary the angle of incidence e. At various angles, So, Path difference UN
strong beams of deflected X-rays strike the photographic plate. as LN = d sin e where, d is the distance between two planes.
The photographic plate after developing shows a central spot due
So, Path difference = 2d sin e
to primary beam and aset of symmetrically disposed spots due to
deflected X-rays. Different kinds of crystals produce different When two reflected waves reinforce each other (maximum
arrangements of spots. reflection), the path difference should be = n'A (where.
n 1,2,3, ... ). ..
So, for maximum reflection,
n'A = 2d sin e
This relation is called Bragg's equation. Distance between two
Photographic film successive planes d can be calculated from this equation. With
X-rays of definite wavelength, reflections at various angles will
be observed for a given set of planes separated by a distance d.
These reflections correspond to n 1,2,3 and so on and are
spoken of as first order, second order, third order and so on. With
each successive order, the angle a increases and the intensity of
the reflected beam weakens.
The values of e for the first order reflection from the three
faces of sodium chloride crystal were found to be
5.9 0 ,8.4 0 and 5.20 respectively. As n and a are same in each
case, the distance d between successive planes in three faces will
be in the ratio of
Fig. 4.28 ,X-ray diffraction of crystals - - = 9.61: 6.84: 11.04
In 1913, Bragg found that Laue photographs are more easily sin 5.9 0 sin 8,40 sin 5.20
interpreted by treating the crystals as a reflection' grating rather = 1 : 0.70 : 1.14
than a diffraction grating. Fig. 4.29 illustrates the way by which
the crystal spacings can be determined by use of X-rays of a 1 2
single wavelength, 'A. The rays strike the parallel planes of the
.=I:J2:J3
crystal at arigle a. Some of the rays are reflected from the upper
This ratio corresponds to spacings along the three planes of a
plane, sO'me from the second and some from the lower planes. A . face-centred cubic structure. Thus, the NaCl crystal has
strong reflected beam will result only if all the reflected rays are
face-centred cubit structure.
in phase. The waves reflected by different layer planes will be in
..... Example 63•. X-rays of wavelength 1.54i'Strike a crystal
and are observed to be d41ected at an angle i[22.5°. Assuming
that n =1, calculate the spacing between the planes ofatoms that
are responsible for this reflection. .
Solution: Applying Bragg's equation,
n'A= 2dsin e
Given, n = 1, 'A = 1.54 A, e = 22.5°.
Using relation I. n'A == 2d sin a,
d =,; 1.54 = 1.54 = 2.01 A
2sin 22.5 0 2 x 0.383

Fig. 4.29 X-ray reflection from crystals Example 64. The first order reflection ofa beam ofX-rays
0.!jwavelength l.54Afrom the (lOO)face ofa crystal of the simple
254 G.R.B. PHYSICAL CHEMISTRY FOR COMPETITIONS

cubic type occurs at an angie ofl1.29°. Calculate the length of AC 2 = AB2 +BC 2 = a 2 +a 2 = 2a 2
the unit cell. or AC J2a
Solu$n: Applying Bragg's equation, Now, in .!l ADC,
2dsin8::::nA AD 2 .= AC 2 +CD 2
Given,8 11.29°, n:::: 1, A = 1.54 A = 1.54 X 10-8 cm = (F2a)2 +a 2 ::: 3a 2

d = 1.54 X 10-
8 8
1.54 X 10- = 3.93 X 10-8 cm
or AD 13a
2 x sin 11.29° 2xO.1957 d 13 a
a 2
d bkl = =a 13
Jh 2 +k 2 +.1 2 d=2r=-a
2
a = 3.93 x 10- cm
8
=length of the unit cell or Atomic radius, r=
13 a
4
Example 65. When an electron in an excited state of Mo
atom falls from L to K-shell, an X-ray is emitted. These X-rays are (c) Face-centred cubic unit cell
diffracted at angle of7.75° by planes with a separation of2.64 A. The distance between the two
What is the difJerence in energy between K-shell and L-shell in nearest neighbours is represented
Mo, assumfngajirst order difJraction (sin 7.75° 0.1349)? by length AB or BC, i. e., half of the
Solution: According to Bragg's equation: face diagonal, .4 C.
AC
nA 2d sin 8 d=AB=BC=-
2 Fig. 4.32
Ix A = 2x 2.64 sin 7.75° = 2x 2.64 X 0.1349 Face-centred cubic un!t
In MCD, AC 2 CD 2 + AD2
A =0.712A cell
2
a 2 +a = 2a 2
Energy difference between K and L-shell of Mo AC=J2a
hc
34 8
6.626xI0- x3xI0 =2.791xlO- 15 J AC J2 I
or d:::-= a=-a
A O,712x 2 2 J2
4.21
.
ANALYSIS OF CUBIC SYSTEMS
d
1
2r=-a
"
J2
The following characteristics are reflected by cubic systems . rad'lUS, r =
or A tomlC I a
when analysed mathematically:
(i) Atomic Radius (ii) No •.of Atoms Per Unit Cell
It is defined as half the distance between nearest neighbouring
Different types of cubic unit cells are given in following figure:
atoms in a crystal. It is expressed in terms of length of the edge .,

® ~ a
'a' of the unit cell of the crystaL
(a) Simple cubic unit CQ:U : In a
simple cubic unit cell, atoms at the
comers touch each other along the edge.
Distance between the nearest
B._---"'a--..
®
I
I
I
I
I
I
II II II

8
I ' I , I

neighbours,
d=AB:::: a=2r
i.... :!.·
• . *
.....:1:
. .

, , ~,' .
(where r = radius of the atom) . Fig. 4.30 v

or Atomic radius, Simple cubic unit cell

rt]~
r a =G.5a
2
. (b) Body-centred cubic unit
cell : The distance between the
two nearest neighbours is repres-
ented by length AE or ED, i.e.,
1/8 atom 1 atom 1/2 atom
half of the body diagonal, AD.
Simple cubic Body centred cubic Face centred cubic
d AE=ED AD (a) (b) (c)
2 Fig. 4.31
In .!lABC, Fig. 4.33 (a) Simple or primitive cubic .lattice;
~Body·centred cubic unit cell
(b) Body-centred cubic latti~~ (c) FacErcentred cubic lattice
STATES OF MATTER I
=- - =-rr = .
3
(a) Simple cubic structure~ Since, each comer atom is r ' = -v
· f actIOn rra /6 052
Pac k mg
shared by eight surrounding cubes, therefore, it contributes for ~ V a3 6
8 i. e., 52% of the unit cell is occupied by atoms and 48% is empty.
of an atom.
(b) Face-centred cubic str~ture: Since, four atoms are
Thus, for simple cube = 8 x.~ = 1atom per unit cell present in a unit cell, their volume is
8
(b) Face-centred cubic structure: Since, the structure v =4 x (~rr,.3)
possesses 8 comer atoms and six at the centre of each face, the
contribution will be Putting the value of r = a~,
2,,2
= 8x~ + 6 x ~ (Face atom is shared by two cubes)
136rr(2~r ::n
8 2
a3
= 1+ 3 = 4 atoms per unit cell v= =
(c) Bociy-centred cubic structure: Since, the structure
Volume of unit cell, V = a 3
possesses 8 comer atoms and one body-centre, the contribution
3
will be = 8 x ~ + 1= 2 atoms per unit cell. Packing fraction =3-v2a
rr~ =. . r r;:;- =0.74
8 3
j""I/2
(iii) Coordination Number i. e., 74% of unit cell is occupied by atoms and 26% is empty.
It is define<t'as the number of nearest neighbours that an atom (c) Body-centred cubic structure: Since, two atoms are
has in a unit cell. It d'epe)1dsiUpon structure. present in aunit cell, their volume is
(a) Simple cubic strp,tture: Coordination number = 6
(b) Face-centred cubic structure: Coordination number =12 v = 2 x (~rrr3 )
(c) Body-centrei"cubic structure: Coordination number = 8
(iv) D~!Islty of Lattice Matter . the value ofr=-
Puttmg -/3 a, 'v' = 2x -4 rr x [-/3)3
- a
Knowing the unit cell dimensions, the theoretical density of a 4 3 4
crystal can be calculated as: -/3rr a 3

. ate
Density f h umt. ce 11 = - -
Mass
-- of -unit-cell
-- 8
Volume of the unit cell Volume of unit cell, V = a 3

Mass of the unit cell = number of atoms in the unit 3


cell x mass of each atom
.
Pac ki ng f ractlOn =-/3rr8a a =--/38 rr =068
3
.
Atomic mass M
Mass of each atom = = i. e. ,68% ofthe unit cell is occupied by atoms and 32% is empty.
Avogadro's number No Characteristics of cubic unit cells are surnmerised in following
. ZxM nxM tables:
So, density of a umt cell =- - - = 3 (For a cube)
NoxV Noxa
$lnip1e Body· Face·
where, Z is the number of atoms in a unit cell and V is the volume ce.t~~ centered
of unit cell.
Volume, conventional cell a3 a3 ~3
For a cube V = a 3 where, a is the edge length of the cubic unit
cell. Lattice points per cell 2 4

(vy' Packing Fraction or Density of Packing Volume, primitive cell a3 I


-a
3 I 3
-a
It is defined as the ratio of volumes occupied by atoms in unit 2 4
cell to the total volume of the unit cell. Lattice points per unit volume 11 a 3
21 a 3
41 a3
A

· fra' Volume occupied by atoms in unit cell v


Pack mg ctton = ------''----'-------- Number of nearest neighbors 6 8 12
Total volume of the unit cell V 1/2
Nearest-neighbor distance a 3 al2 al2112
(a) Simple cubic structure: Let a be the cube edge and ,. = 0.866a = 0.707a
the atomic radius. 12 6 6
Number of second neighbors
V =volume of the unit cell =a
3
zI/2 a a a
Second neighbor distance
Since, one atom is present in a unit cell, its
4
Volume , v = -3 rrr
3
(r=a/2)
Packing fraction 1
-7t ~7t.fj ~7t.,f2
6 6 6
= 0.524 = 0.680 = 0.740
=~rr(~r rr~3
256 001 • G.R.B. PHYSICAL CHEMISTRY FOR COMPETITION&

Hexagonal Unit Cell Volume of unit cell a 3 : (4.24 X 10-8 )3 cm 3


Let us consider hexagonal unit cell of height 'h'; its
constituent units are spherical having radius 'r'. ZxM 2x 23
So, Density - - - : --------------
No x V (6.023 X 10 23
)(4.24 X 10-8 )3
l. a =2r. 1
1.002g cm-3
';Example 68. The density of KCl is 1.9893 g cm-3 and the
length of a side of unit cell is 6.29082 A as determined by X-ray
°diffractio,n, Calculate the value ofAvogadro number. s
Solution: KCl has face-centred cubic structure,
i. e., Z=4
ZxM
Avogadro's number == - -
dxV
Fig. 4.34 Hexagonal unit cell
Number of constituent units in the hexagonal unit cell: 6. Given that, d: 1.9893; M = 74.5; V == (6.29082 X 10-8 )3 cm 3

Thus, occupied volume in the unit cell: 6x i nr 3 4 x 74.5


Avogadro's number: -------,-----::--:-
3
1.9893 x (6.29082 x 10-8
Volume of unit cell = Area of base x height
: 6 x Area of equilateral triangle x height =6.017 X 1023
-,,".
~
:.---
= 6x ..[32
a x4r - " ',:t~Ei:ample 69. Silver crystallises in a face-centred cubic
4 3 .unit cell. The density of Ag is 10.5 g cm -3. Calculate the edge
= 6x ..[3 (2r) 2 x4r -~ length ofthe unit cell.
4 3 Solution: For face-centred cubic unit, Z 4.
(Height of the unit cell can be calculated geometrically) "
We know that, V = Z x M
Percentage occupied space occupied volume xl 00 Noxd
volume of unit cell
6x 4 nr 3 = 4 x 108 6.83 x 10-23
(6.023 x 10 23 )x 10.5
= ..[3 3 ~ x 100= 74.06%
'0 6_(2r)2 x4r- : 68.3 x 10-24
4. 0 .3
Let a be the edge \eIOgth of the unit cell.
:::::::_SOME SOLVED EXAMPLES\I:':::·:; , So, V a3

Example 66. A compound formed by elements A and B


or a3 =68.3 x 10-24
crystallises in cubic structure where A atoms are at the cOrners of a (68.3 x 10- 24 )1/3 cm
a cube and B atoms are at the face-centre. What is the formula of : 4.09 X 10- cm8
the compound.?
Solution: An atom at the comer of the cube contributes X to =409 pm
the unit cell. Hence, number of atoms of A in the unit cell iJ~~:;Ex'inple 70. An element occurs in bcc structure with a
cell edge. of288pm. The density of metal is 7.2g cm- 3 . How
0

=8xX 1.
AIl: atom at the face of the cube contributes ~ to the unit cell. many aton:s does 208 g of the element contain?
Hence, number of atoms of B in the unit cell: 6 x ~ = 3. Solution: Volume of the unit cell =(288 x 10-10 )3
Thus, th~ formula is AB 3 •
Example 67. At room temperature, sodium crystallises' in
=23.9 X 10-24 cm 3
body-centred cubic lattice with a:= 4.24 A. Calculate the
the()retical density of sodium (At. mass of Na = 23.0). Volume of 208 g of the element: _20_8 =28.88 cm 3
7.2
Solution: A body-centred cubic unit cell contains 8 atoms at
the 8 comers and 1 in the centre. Number of unit cells in 28.88 cm3 :::: _ _2_8_.88---:C"":'
Hence" 23.9x
Total number of atoms in a unit cell = 8 x ~+1 2 = 12.08 X 1023 unit cells
8
Each bcc structure contains 2 atoms.
STATES OF MATTER
" 257
23
So, Total atoms in 12.08 x 10 unit cells Example 74. Tke density ofcrystalline sodium chloride is
23 2.165 g cm -3. What is the edge length of the wiit cell.
= 2 x 12.08 X 10
= 24.16x 10 23 What would be the dimensions of cube containing one mole of
NaCl? .
Example 71. Lithium forms body-centred cubic crystals.
Solution: We know that,
Calculate the atomic radius of lithium if the length of the side ofa
unit cell of lithium is 351pm. N/MJ
Solution: In body-centred cubic crystals, p= lNA
r
13 a where; p = density = 2.165 gcm-3
4 M molar mass = 58.5 '
. 13 '
=,- x 351pm 151.98pm NA Avogadro's number = 6.023 x 1023 .
4
N = number of formula unit per unit cell
.Example 72. Ammonium chloride crystallises in a
= 4 (for fcc)
body-centred cubic lattice with a unit distance of 387 pm.
Calculate (a) the distance between oppositely charged ions in the 3 N (M
' .' 4 [ 58.5 ]
lattice and (b ) the radius of the NH4+ ion if the radius ofCI-ion a = NA pl = 2.165 6:023 X,10 23 .
is 181pm.
1. 794 X 10-22
Solution: (a) In a body-centred cubic lattice, oppositely
charged ions touch each other along the cross-diagonal of the a :: 5.64 X 10-8 cm
cube. . I Molar mass 58.8
So, 2rc +2ra = 13a M oarvoume=
1
Density
::.--
2.165
or rc + ra = 13 a 1 3
2 Edge length (a) [58.8 J / = 3 cm

13 x 387 = ~35.15 pm
2.165-
2 . Example 75. The density of potassium bromide crystal is
2.75 g cm -3 and the length ofan edge ofa unit cell is 654 pm. The
(b) Given that, ra 181 pm
unit cell ofKBr is one ofthree types ofcubic unit cells. How many.
. rc = 335.15- 181.0= 154.15pm formula units of KBr are there iii a unit cell? Does the urdt cell
have a NaCl or CsCI structure? .
Example 73. The unit cell cube length for LiCI (NaCl
structure) is 5.14A. Assuming anion-anion contact, calculate the Solution: . We know that,
ionic radius for chloride ion. .. N [M 1
Solution: In a face-centred cubic lattice, anions touch each p= NA J
other along the face diagonal of the cube. . 3·'
N=pxa x NA
4rCl - =-F2a M
.J2 a x 6.023 X 1023 .
3.89=4
4
Number of mass points per unit cell = 4
= .J2 x 5.14 1.82A·
4 It is NaCI type crystal, i.e., fcc structure.
Alternative: Distance between U+ and.Cl- ion Example 76. A unit cell of sodium chloride has four
formula units: The edge length of unit cell is 0.564 nm. What is
5.14 = 2.57 A CI- the denSity of sodium chloride? (liT May 1997)

~~CI
2
ZM Ax 58.5
Thus, distance between two chloride ions Solution: p =- - = - - - ' - - - - - - - - -
a 3 N (5.64 X 10-8 )3 x 6.023 X 10 23
=J (2.57i + (2.57)2
=2.16gcm-3
3.63A . 2.57 A
Example 77. Chromium metal crystallises with a
Hence, body-centred cubic lattice. The length of the unit cell edge is
radius of Cl- 3.63 = 1.82A found to be 287 pm. Calculate the atomic radius. What would be
2 the density of chromium in g/cm 3 ? (ItT July 1997)
258 I G.RB. PHYSICAL CHEMISTRY FOR COMPETITIONS
1
'Solutiou: In body-centred cubic unit cell, In bec unit cell, Z=2
aJ3 4r ..
, N'
Number of unit cells = ~ = 0.5 N A]
" 2
where, a = edge length,' r =radius of atom '
33. Sodium metal exists in bec unit cell. The distance between
r=c aJ3 = J3 x '4.29 ::::: 1.8574 A nearest sodium ;atoms is 0.368 nm. The edge length of the,
4 4 unit cell is: '
(a) 0.368nm (b)0.184nm
Example 78. When heated above 916°C, iron changes its
crystal structure from body-centred cubic to cubic closed packed (c) 0.575 nm (d) 0.424nm
lAns. (d)]"
structure. ASsuming that the metallic radius of the atom does not ,
change, ealcJlate the ratio of density of the bcc crystal to that of [Hint: In bee unit cell, aJ3 == 4r
the cep crystal. 4r 2
Solution: In body-centred packing, the efficiency of ,a=-,-=-x2r
Pllcking is 67.92%. In the cubic closed packing, the packing
..[3 J3
2
efficiency is 74.02%. == J3 x 0.368= 0.425 nm]
Let d l be the density when packing efficiency is 74.02% and
d 2 is the density when packing efficiency is 74.02%. ' 34. Ifth~ distance between Na+ and CI ions in NaCI crystal is
265 pm, then edge length of the unit cell will be? '
di = 67.92~~~.918 (a) 265 pm (b) 530 pm (c)795pm (d) 132.5 pm ,
dl 74.02 [Ans. (b)l'
[Hint: In NaCl:
IIlUmA1fONS OF OB)ECl"IVE QI:IESIIONS Edge length = 2 x distance between Na+ and CI -, ions,
= 2 x 265 = 530 pm] ,
30. AD. element 'A' has face-centred cubi.:: structure w~th edge 35. The interionic, distance for caesium chloride crystal will be:
, length equal to 361 pm. The apparent radius of atom 'A' is: , ' IPl\IT (MP) 2007] ,
(a) 127.6 pm ' (b) 180.5 pm ' ' 2a
(c) 160.5 pm' (d) 64 pm (a) a (b)~ (c) a/3 (d) /3
, 2 2
[Ans. (a)] ,
TAns. (c)]
[Hint: For face-centred 1.!llit cell,
a.Ji = 4r 4.22 PACKING ,OF IDENTICAL SOLID
361 x 1.414 ,SPHERES
----==r
4 The constituent particles in the formation of crystals are either
r 127.6 pm] atoms, iens' or molecules. These particles' may' be of various
31. , The packing fraction of the element that crystallises in simple shapes and thus, the mode of packing ,of these particles' will
cubic arrangement i s : ' , change ac«ording 'to their shapes. The simplest way will be to
consider these particles as spheres of equal size. The packing of
W~ OO~ ~~ 00% , spheres is done in such away as to use the available space in the
roost economical mal1Qer.
[Ami. (b)]
[Hint: In simple unit cell a == 2r
Z=I
, Occupied volume
Packing fraction == ---.::..----
Total volume

± 'ltr
3 ±'ltr
3
=_3_=_3_'_=~)
i
I
a3 (2r)3 6
32. Arrangement (i) , Arrangement. (ii)
How' many unit c.ells are present in 39 g of potassium that
crystallises in body-centred cubic structure? Fig. 4.35 -Two common ways ofpacki~g spheres
(a)N A ,(b)]I[ A
, " 4 "
(c) 0.5 N A (d) 0.75 N A • of equal size
There are two common ways .in which spheres of equal size
,[Ans. (c)]
can be packed. This has been shown in Fig~ 4.3~. The
" ~ass arrangement (i) is more economical in comparison to
[Hint: Number of atoms == x NA
i\tomicmass arrangement (ii) as 60.4% volume is occupied in arrangement (i)
'and 52.4% volume in arrangement (ii), Arrangement (i)
= 39 xNA'=N A ,
39 • , represents a close packing of spheres.
I

STATES OF MAnER

In arrangement (i), the spheres are packed in such a manner , spheres and IS a9breviated as hcp or ABABAB,. .. This is shown
that their centres are at the comers of an equilateral triaag!e. Each in Fig. 4.38. Iii' the second way, spheres are placed on the
sphere is surrounded by six other similar spheres as shown in Fig. unoccupied hollows of the first l~yer, marked by 'crosses'. It is
4.36. This arrangement can be extended iQJmeedimensions by observed that spheres of the third layer do not come over th9seof
adjusting spheres on the top- of hollows or' voids of the first layer. This arrangement of close packing is referred to as
two-dimensional layer which is called the first layer or 'A' layer. , ABC: However, it is noted that spheres in the fourth l~er will
in
There are two types of hollows the first layer which have been correspond with those in the first layer. When such an
marked by dots (.) and cross (x). All the hollows are equivalent. arrangement is continued indefmitely, the system is found to
The spheres of the second layer may be put either on hollows possess cubic symmetry 'and called cubic close packing Of
marked by dots or by crosses. Half of the hollows remain spheres and is abbreviated as ccp or ABCABC . .. This is shown
unoccupied in the second layer (Fig. 4.37). The second layer is in Fig. 4.39. The system ABCABC ... shows that there is a sphere
marked as 'B' layer. The spheres have been placed on the hollows at the centre of each face of the unit cube an~ thus, this system is
marked by dots. ' also referred to as face-centred cubic or fcc. - ,

Three-fold
Three spheres C
at the corners Six,spheres
of afjl equilateral surrounding B
,triangle a spherl:'
A
C
B
A
, Fig. 4.36 qlose picking of spheres
(a) (b) (c)

Fig. 4.39 'ABC ABC A . ; . or cubic close packing (ccp) ot'


spheres
It is noted that in both the above systems

~
hcp or ccp, each sphere is surrounded by
twelve other spheres shown in Fig., ,4.40.
There is a third arrangement of packing of I
I
I
spheres which is known as body-centred, I
I
Rg. 4.37 Building of second layer (8" sho,wn shaded) cubic arrangement (bee). This arrangement '
covering hollows marked by dots ( • ); The , is obtained when the spheres in tp.e first
hollows ~arked by a cross (x) unoccupied layer are slightly opened up, i. e. ,none of the
I
To build up:the third lIiyer of spheres, there are two alternative spheres touches each other. In the second I,
I

ways. In the' first way the spheres are placed on the hollows of layer, the spheres are placed at the top of
second layer. it is observed that each sphere of th4'd layer lies
exactly above the spheres of first layer. When this arrangement is
continued indefinitely, the system obtained. is found to possess
hollows in the first layer. In the third layer,
spheres are placed exactly above the first
layer. Each sphere in this system of packing
€®'
Fig. 4.40
is in contact with eight spheres, four in the
hexagonal symmetry and is called hexagonal close packing of Coordination
lower layer and four in the upper layer. This
arrangement has been shown in Fig. 4.41. number in hcp.\,
GJoid axis and ccp structure -•.
_ Most of the metals belonging to s-block'
and d-block elements possess anyone of the following clo~
packing arrangements: 7

(i) Cubic closed packed, (ii) Hexagonal closed packed. and:.,


(iii) Boqy-centred cubic packed. '
E~amples:
Metals Structure
A
Mg, Zn, Mo, V, Cd hcp
Cu, Ag" Au, Ni, Pt ccp'or fee
Li, Na, K, Rb, Cs, Ba bee ",
Fig. 4.38 ABABAB ... or hexagonal close packing (hcp) 01
spheres
I

·260 I G.R.B. PHYSICALCHEMISTRY FOR COMPETITIONS

0-0-0-0 :. Number of octahedral voids is equal to the number of


constituent units at packing sites. Each octahedral void. is
surrounded by six constituent units.
OO()O ........ !'>
'" '" , / I
/~
\
\
,
,,
'" '"
/ I
I
\
\ ,
'2 I \

~."
f ~-, - ..- - -- ~ 4
/

0000 /

3 /
,,
,
K

\
\
::'>. .-t- /7'-.

I
I
/
O-Packing sites
• - Octahedral voids
(A) (B) ,
\
\
I
1/
/

~/
Fig. 4.41 (A) Body-centred cubic packing of spheres
(B) Body-centred cubic arrangement '" ....... 0' .......
"-
.......

Interstitial sites in close packed lattices: In a close packing Fig. 4.44


of spheres, there is always some empty space left. The empty space
is called hole or void or interstitial site. Common interstitial sites in Tetrabedralvoids : In cubic close packing, there are eight
closely packed spheres are tetrahedral and octahedral. tetrahedTal voids in each unit cell. Tetrahedral voids are located
Tetrahedral site: When one sphere is placed upon the three on body diagonals. Two tetrahedral voids are located OR each
other spheres which are touching each other, tetrahedral structure body diagonal; these are represented in the Fig. 4.45.
. results. The four spheres leave a small. space in between which is
called a tetrahedral site. The sit~ is much smaller than that of the
spheres. However, when the spheres are bigger in size, the
tetnlhedral site becomes larger: In hcp and ccp, each sphere is in
contact with three spheres above and three spheres below. Thus,
there are. two tetrahedral sites associated with each sphere. 0 - Packing sites
Octahedral site: This site is formed at the centre of six • - Tetrahedral voids
spheres, the centres of which lie at· the apices of a regular
OCl:3hedron. Each octahedral site is created by superimposing two
equilateral triangles with apices in opposite direction, drawn by
joinmg centres of three spheres in one plane and three spheres in
other plane. There is an octahedral site for each sphere. Thus, Fig. 4.45
octahedral sites' are half of tetrahedral sites. Tetrahedral and.
octaheQral voids are represented in Fig. 4.42 and 4.43. Number of constituent units at packing sites (corners and
1 1 . .
face-centres) 8x-+6x-: 4
8 2
Number of tetrahedral voids: 8
. Thus, number of tetrahedral voids is twice the number of
constitueq.t units present at packing sites,
Fig. 4.42 Tetrahedral voids

.I/ff.
In a multilayered closest-packed structure, there is a
tetrahedral void above and below each atom, hence there are
twice as many tetrahedral voids as those of closest-packed
constituent units.
Tetrahedral and octahedral 'voids can be located as
- - - - - Second closest packed layer
Fig. 4.43 Octahedral voids --~--Tetrahedral voids
-----Octahedral voids'
Location of Octahedral and Tetrahedral Voids in Cubic -----Tetrahedral voids
Close Packing - - - - - First closest packed layer
Octahedral void.s: In the Fig. 4.44 a unit cell of ccp or fcc
lattice is drawn .. Corners .and face-centres are the packing sites. In . fWJ51RATfONS OF OBJfCnVf QUESTIONS
the face-centred cubic unit cell; octahedral voids are located at
. the body~centre and all the' edge certres of cube. 36. In a compound, atoms of element Yformccp lattice and those
Number of constituent units at packing sites == 8x 1 +6x 1 == 4 of element x.o,ccupy 2/3rd tetrahedral voids. The formula of
8 2 . the compound will be: (AlEEI': 2110X)
. Number of octahedral voids (one at body-centre a~d twelve at . (a)X 3Y4 (b)X 4Y3' (c)X 2Y3 (d)X 2Y
[Ans. (b)]
edge-centres) == 1+..!. x 12: 4 I .
. " 4 [Hint: Let number of atoms of Yused in packing = n
I
STATES OF MATTER 261

Number of tetrahedral voids =2n . ions have coordination number six. In CsCI, each Cs + ion is
2 4 surrounded by eight Cl- ions and vice-versa. Thus, both the ions
Number of atoms of X = x 2n := - n
3 3 have coordination Dl:lmber eight. In the case of CaF2 , the
coordination number of Ca 2+ - is eight while that of F - ion, is
Y:X:=lI:!lI =3:4
3 four.
Fonnula of compound == X 4Y3] The coordination numbers commonly encountered in ionic
37. A solid compound contains X, Yand Z atoms in a cubic lattice crystals are 8 (body-centred cubic arrangement), 6 (octahedral,
with X atoms occupying the comers, Y atoms in the body arrangement) and 4 (tetrahedral arrangement). '
centred positions and Z atoms at the centre of the faces of the It is impossible for both anions and cations to have closed
unit cell. What is the empirical fonnula of the compound? packed structures but if one of the ions is much bigger than the
(a) XY2Z3 (b) XJZ3 (c) X 2Y2Z3 (d) X 8YZ6 other, it is common for the bigger ions alone to approach a closed
(e)XYZ packed structure and smaller ions to fit into holes in this stru~re.
[Ans. (b)] Iri ionic crystals, the coordination numbers as well as the
1 geometrical shapes of the crystals depend mainly on the relative
[Hint: Number of atoms of X in a unit cell x 8"" I sizes of the ions. The ratio of the radii of the positive and negative
8
ions is called radius ratio. .
Number of atoms of Y in a unit cell = I '
Radius of positive ion (cation). r +
Number of atoms of Z in a unit cell = .!. x 6 == 3 Radius ratio = =_c_
2 Radius of negative ion r _
a
Empirical fonnula of the compound == X'YZ3] ,
Common coordination numbers are 3, 4; 6 and 8.
38. In a crystalline solid, atoms of X fonn fcc packing and the
atoms of Yoccupy all octahedral voids. Ifall the atoms along The following table shows the radius ratio values, permitted
one body diagonal 'are removed then the simplest fonnula of coordination numbers and the shapes of ionic crystals:
the crystalline solid will be :
(a) XY (b) X 4Y3 (c) X sY4 (d) X 4YS Radius ratio . Permitted Arrangement of
coordination .anions round the Example
(r/ I r;;)
[Ans. (c)] number cations
[Hint: Number of atoms of Xin fcc packing (at comers and face 0.155- 0.225, 3 Plane Triangular B2 0 J
8 x .!. +6 x .!. = 4
centFes of cubic unit cell)
, 8 2 0.225- 0.414 4 Tetrahedral ZnS
Number of atoms of Yat octahedral voids = 4 0.414- 0.732 ·6 Octahedral NaCI
Along one body diagonal there are two X atoms and one Yatom. 0.732-1.000 8 Cubic (body-centred) CsCI
Number of effective atoms of X after removal
Types of Structure of Ionic S'olit:
I 15
4 2x-=- Crystal structure of ionic solids are described into following
8 4
, . types: .
Number of effective atoms of Y after removal = 4
, 1
Crystal structure I
X:Y=~:3 (Type)
Example
4
= 5:4 1. Rock salt NaCI, LiCl, KBr, Rbi, AgCI, AgBr, FeU, LiiV
.. Simplest fonnula = Xs14 ] 2. Cesium chloride CsCI, CaS, CsCN
3. Fluorite CaF2, U0 2, BaCi2 , HgF2
4.23 TYPES OF CRYSTALS 4. Antifluo'rite K20, K2S, Li 20, Na20, Na2S
Crystals are classified into four main types depending upon the 5. Nickel Arsenite NiAS, NiS, FeS, CoS
nature of forces that hold the constituent particles together in the
crystal lattice. . 6. Rutile Ti0 2 , Mn02 , sn02
1. Ionic crystals: The forces operating in ionic crystals are 7. Perovskite CaTiO:;, BaTiOJ • SrTiOJ
, /
, electrostatic. The lattice points in such cryslals are occupied by, 8. Zinc blende . ZnS, CuCI, <;::dS, HgS .
positively and negatively charged ions. Each ion is surrounded by (sphalerite)
the largest possible number of oppositely charged, ions .. This
number of oppositely charged ions' surrounding each ion' is 9. Wurtzite ZnS, ZnO, BeO, M~S, AgI
termed its co()rdination number. The coordination numbers of Packing of ions in ionic crystal (Formation of NaC!
positive or negative ions Diay be same or different: For example, crystal): The formation ofNaCI crystal can be explained in the
in .sodiu~ chloride, each Na + ion is surrounded by six Cl- ions· following manner. The two ions fonn one ion-pair of opposite
and each CI- ion is surrounded 'by six Na + ions. Thus, both the charges by the electrostatic f"rce of attmction. Each of the ions
~62 J G,RB. PHYSICAL. CHEMISTRY FOR COMPETITIONS

has a strong residual field around it and will naturally attract Here,4r_ =..fia and (2r_ +'2r+) a
!qlother ion-pair much. in the same way as two magnets attract Pacldng fraction (<Il) :
each other, i. e., a cluster is formed. Two clusters combine
1
3
together to give it unit cell. Finally, large number of un# cells .<I>=4x-n
4 - J
+r! ) =1t- [(r+ +1.
unite togetHer to form ttv"ee-dimemsional cubic crystal. . 3 a3 3J2 r_
The unit cell of sodium chloride has fom sodium ions and four
chlotide ions.

.e:i). (a) Number of sodium ions


.
12 (at the edge-centres) x-
.
+ 1(at body-centre) = 4
. 1
.4

(b) Numbe~ of chloride ions == 8. (at the comers) x ~ + 6


. 8
. 1
(A) (8) (at face-centres) x=-= 4 .
. 2
r . Fig.4.46 Filing. up of ion pairs in stages: (A) shows an ion-Plir,
. (Ii') two ion-pairs and. (Q) four ion-pairs of .adtum chloride Structure of .Some Other Ionic Compounds
An examination of NaCI crystal (Fig. 4.47) makes the In CsCl cry[s~al: th]e coordination num~er of each ion is 8 as
following points clear: . .
the radius ratio rCs is 0.933. It is termed as 8 : 8 arrangement.
(ij Each Na + ion is surrounded by six Cl- ions at the comers
. CI-
of a regular octahedron and similarly each Cl- ion is surrounded
by six Na + ioris. It is, therefore, termed as 6 : 6 arrangement. The The arrangement of the ions in this crystal is body-centred cubic
radius ratio [rN: 1 'C~ =0.95/1.81 == 0.524] suggests that coordi- type, i. e., the unit cell has one ion at the centre and oppositely
charged ions at the comers of the cube. This has been shown in
nation number of each ion is six. :.
Fig. 4.49. This type of structure is possessed by CsBr, CsI, TICI,
TlBr, etc.
I_ S.63A
2.81SA
I- ..-1
'0 Na+ion are
represented
by hollow circles
• CI-ion are
CI-ion represented
by filled cirCles
octahedrally .
surrounded by
six Na+ ions

Na+ ion Fig. 4.49 Structvr~ of esct·


octahedrally
surrounded by Here, Cs + ion is present at the body centre and chloride ions at
six CI-ions
eight comers. In their limiting case : .
Fig .. 4.47 Unit cell representation on NaCI structure 2r_ = anda (2r_ 2r+ ) /3a
+ =
(ii) In.the octahedral structure, Cl- ions may be regarded as a =edge length of unit cell
.' having a cubic closed packed (ccp) arrangement in which all Pacldng fraction (<I» :

~.1t. [(r+r_ .)3 + 1].


octahedral holes 'are fiUed byNa + ions.
(iii) This type of structure is possessed by most of the alkali
'3x8
.metal halides (KCl, NaI, R:bF, RbI), alkaline earth metal oxides
and AgF, AgCl, AgBr, NH 4 Cl, NH4 Br, etc. ZnS crystal has two types of structures:
(i) Zinc blende structure and
(ii) Wurtzite structure.
(i) Zinc blende structure: I~ has cubic close-packedJccp)
structure. The S2- ions are present at the comers of thecutle and
at the centre of each face. Zinc "ions occupy half of the tetrahedral
sites, Each zinc ion is surrounded by foilr sulphide ions whiCh are
disposed towards the comers of a regular tetrahedron. Similarly,
Fig. 4.48
STATES OF MATTER 263-
each S 2- ion is surrounded by four Zn 2+ ions. This structure is . (ii) Each Na + ion is surromld~d by four 0 2- ions and each
similar to diamond. . 2
0 ':' 'io~ is surrounded by eight Na + ions. Thus, Na' 20has 4·: 8
coordination structure.
Other examples with antifluorite structures are:
C1 2 0, Na2S, K 20, Li 2S, K2S,~tc .

. Structures of Oxides of Iron


Iron is known .to form three major oxides; which are FeO,
Fe203, Fe304' These oxides are interconvertible due to
oxidation and reduction. Non-metal excess 'lion-stoichiometry is
aJso known in these oxiaes. Crystal structut:es of these oxides are
very interesting.
1. Structure of F eO : This oxide has a structure like rock salt
Fig. 4.50 Zinc bleilde (NaCl). Oxid~ ions (0 2-) form a face-.centred cubic close
: packing and Fe 2+ions occupy all octahedral voids:n is a perfect
41:, =.['1a rock salt type structure with formulaFeO: This oxide is kilown to
Packing fraction (<I» : form non-metal excess non-stoichiometric compound with the
composition FeO.9S0 (Wustite). In. the non-stoichiometrjc
_ . 4 (r; + r!
- 4 x-n )-
_ -n- [(r+)3
- + 1'] compound some of the Fe 2+ ions present in octim"ooral VQids are
3 a3 ..·if2' r_ . replaced by Fe 3+ ions. Three Fe 2+ ions Will be wPi)ced by two .
. .

(ii) Wurtzite structure: It has hexagonal close-packed Fe 3+ ions to maintain electrical neutrality. :'~i]P": .'
. ' . ~~r'::J'~J ". .
(hcp) structure. S2- ions adopt hcp arrangement and Zn 2+ ions [Normal Spinel Structure.: Spinel is the coililrioli ruiIile ·of.
occupy half of the tetrahedral sites. . .. . the mineral, MgAl204' ne. crystal structlii'e of this mitieral is
.The above structures are termed 4 : 4 arrangement· 'possessed also called spinel structure. In this .type of structUre, both
by ZnO, AgI, SiC, etc. tetrahedral and octahedrill voids' are sinl'ultaneously occupied.
CaF2 crystal: The Ca 2+ ions are arranged in cCp General formula of the compound having spinel structure .is .
arrangement, i. e. , Ca 2+ ions are present at all comers and at the
°
AB 2 4 where' A' stands for a divalent cation apd' B' stands for a' : .
trivalent cation; It has a cubic .close packed arrangement of oxid~ ,
centre of each face of the cube. The fluoride ions occupy all the ions. Half of the octahedral voids are occupied b}'trjvalent.·
tetrahedral sites. This is 8 : 4 arrangement, i. e. , each ca
2+ ion is cations and one-eighth of the tetrahedral voids are occtipiedby
surrounded by 8 F - ions and ea~h F - ion by four Ca 2+ ions. divalent cations. ..... .
Ferrites are ·the spinel type compo~ds" having the general
formula AFei04where A stands for divalent catioh such as
./ + ../ ../ Zn?+. F~rrites are used for~ing powerfiil, permanent" and
• . •
."", /' ..,/
nOIi-rustingmagnets. Ferrites are used in telephones and memory
, ,
• ... • \ I
unitS of computef\'J . .
- i/
.. •
..,/ ./ 2. Structure oi1ce30 4 : Magnetite, Fe 30 4, is considered as a
./ ,
,, ,,,
./ -/; ,, . mixed oxid,e. .It contains mix~e of FeOand Fe 20 3. It has aD.
• , ' I,,
,~ • hiverse sp,inei sttucture. Oxide ions (0 2-) from face-centred
~
, /" -V'''/
~ ,,/
/" c~bic arr~geme~t. Dipositive ions (Fe2+) are present in
:,i octahedr~l v~i&' and tripositive ions (Fe 3+) ~eequally
Fig. 4.51 Fluorite structure-Structure of;calcium fluoride distributed among tetrahedral and octahedrai voids.. .
Number of 0 2- iolls in the'unit cell . = 4
Structure of Sodium Oxide.(~a20),.......Antifluorite Thus, Number oftetrahedral voi~ = 8
. Structure fl; . . . and Number of octlhedralJ\Toids = 4
Sodium oxide ·has antifluorite structure. The structure is . N~ber-ofFe2+ ions present in octahedral void = 1 .
similar to the structure of calcium fluoride with the :differenc.e . Number ofFe 3"t ions pr~sent ill tetrahedral void = 1
that the anions occupy .the positions of cations and vice-versa.
. N~beT of Fe 3+ ions present in octahedral void = 1
(i) .It has cubIc close packing arrangement of 0 2- ions, i.e., .
Thus, half~f1he octahedral voids ~e OCCUpied by Fe 2+ and
.these ions are present at the comers and !it the centres of all the
Fe 3+ ions and o'ne-eighth of the tetrahedTal voids are occupied by
six faces of the unIt. Sodium ions on the other hand, are present at· .
fe"3~ ions: . ,', . .
the sites ofall the tetrahedral voids ..
\ '-"

."
-!F., .

... ~
264. I G.R.B. PHYSICAL CHEMISTRY FOR COMPETITIONS

3. Structure of Fei)3 : If all the Fe 2+ ions are replaced by different elements. These atoms are held together by covalent
Fe + ions, then ~ every three Fe 2+ ions can be replaced by two
3 bonds. Covalent crystals are of two types:
Fe 3+ ions. to maintain electrical neutrality, the ratio between Fe (i) The covalent bonds extend in three~dimensions fonning a
°
and will now be 2 : 3, i.e., we get Fe203' giant interlocking structure. Examples are diamond, silicon
carbide (SiC), aluminium nitride, etc.
Properties of Ionic Crystals
In diamond, each carbon atom is Sp3 ~hybridized and thus,
(i) Physical state: Ionic crystals are crystalline solids' at
covalently bonded to four other carbon atoms present at the four
room temperature. They are never liquids or gases under ordinary
comers . of a regular tetrahedron. This leads to a rigid
. temperature and pressure as the ions do not have freedom of
three~dimensional network. The entire crystal is regarded as one
. movement.
large carbon molecule and is called 'a macromolecule. The
Since, high energy is required to separate canons and anions structure of diamond explains the properties such as high density,
from one another from their alloted positions on accoun( of very non~conducting nature, extreme hardness and high melting and
strong. electrostatic forces of attraction, the ionic crystals: are boiling point of the diamond.
quite hard, have low volatility and have high melting and
boiling points.
(ii) .Electrical conductivity: Ionic crystals are insulators,
f. e. , do not conduct electricity in solid state. The reason is that
ions are held firmly in fixed positions in the crystal lattice and
cannot move when electric field is applied. However, when
melte~ they become good conductors of electricity, i. e. , ions
are free to move in liquid medium.
(iii) Solubility: Ionic crystals are soluble in polar solvents
or. solvents having high values of dielectric constant. They are
insoluble in non~polar solvents. When they in non~polar
medium colloidal solutions are formed, e.g., kerosine oil.
The dissolution of an ionic crystal in· a polar depends on • Carbon atom
the relative magnitudes oflattice and . ' Fig. 4.52 Structure of diamond
MX (s) + Energy ----7 M + (g)
(Ionic crystal) (Lattic~ energy)' . (ii) The covalent bonds extend in two~dimensions forming a
. giant layer. These layers are then held together by van der Waals'
M+ (g)+ x (Solvent) ~ [M (Solvent)x t +BEriergy
(Solvated ion) , (Heat of hydration)
forces. Examples are· graphite, boron nitride (BN), cadmium
X - (g) + y (Solvent) ----7 [X (Solvent) y
, (Solvated ion)
r+ Energy
,.;,~(Heat of hydration)
iodide, etc.
, In graphite, each carbon is sp2 ~hybridized and is thus bonded
~ ,,'

The ionic compound will be soluble if tota~'~ydration energy to three carbon atoms in the same plane. The C-C distance is
is higher thart lattice energy. \j
1.42 A which is intermediate between single bond distance (1.54
. ' Ionic crystals are good cJ}nductors in moit~lJ state or when A) and double bond distance (l.33 A). The unit cell consists of a
dissolved in water.' • 'f:';> ' hexagon of six carbon atoms. The unit cells are interlocked
(iv) Brittleness: Ionic solids are highly 'brittle in nature. giving a layer or sheet structure. The distance' between two
When external force is applied, these are easily broken into adjacent layers is 3.35 A. The structure explains the properties of
pieces. Ionic solids are composed of parallel layers which contain graphite such as being. soft, good conductor of electricity, flaky
cations and anions. When ~xtemal force is' applied, one layer and slippery.
slides a' bit over the other aQd like ions come in tront of each
other. Due to repulsion between two layers the ionic solid breaks
down.
Ionic crystals have high ,density due to close packing.
(v) Isomorphism: IonIC crystals possess same crystalline
structure, i. e. , show isomorphism if the ions (cations and, anions)
have same. electronic configuration. NaF and MgO are
isomorphous compounds.
Na+ F- and Mg2+ '
2,8 2,8 2,8
2 .. Covalent crystals: In covalent crystals, the lattice points Fig. 4.53 Structure of. graphite
are occupied' by neutral atoms either of same element' or of

"
STATES OF MAnER J. 265

In general, the packing of atoms in covalent crystals is loose in Ionic Covalent' Metallic Molecular"
Property
comparison to ionic and metallic crystals. Thus, covalent crystals , crystals crystals crystals crystals
have open structure. •
1. Constituent Positive'and [Neutral Ipositive ion IMolecules
3. Metallic crystals: The metallic crystals have positive particles negative atoms in polar
metal ions as lattice points surrounded by a sea of mobile occupying, ions sea of or non-polar
electrons. Each electron beiongs to a number of positive ions and lattice electrons
each positive ion belongs to a number of electrons. The force that points
binds a metal ion to a number of electrons within its sphere of i. Binding Electrostatic Shared Electrostatic !Weak polar
influence is known as metallic bond. Metallic crystals mostly force attraction pairs of attractions forces or van
electron's between 'der Waals'
belong to cubic face-centred (fcc), cubic body-centred (bcc) and
'IPOSitive ions forces
hexagonal close-packed (hcp) systems. Metallic crystals exhibit and electrons
the following properties: 3. Hardness Hard and Very hard Hard or soft Very soft
(i) Good conductors: On account of the presence of brittle
mobile electrons, metals are good conductors of electricity. The 4. Melting and High Very high High or low Low
electrical conductivity decreases with increase of temperature. boiling
This is due to the fact that positive ions also begin to oscillate point~

with the increase of temperature. The oscillation of positive ions 5. Electrical Bad Bad Good Bad
hinders the movement of mobile electrons. Metals are also good conductivity conductors; conductors conductors conductors'
conductors of heat. Good with few
conductors exceptions
(ii) Lustre: When freshly cut or scratched, most of the in fused
metals have a shining surface. When light falls on freshly cut state or in
surface, the electrons get excited. The excited electrons give off solution
energy in the form of light when they return to their original 6. Examples Ice, solid
r""I'.
NaCI, !Diamond, Cu, Na, Fe
position and hence the metal looks bright. The metallic properties IKN03, CO 2
are explained on the basis that the metallic bond is CaF2 igmphite,
carborun-
non-directional and non-rigid. The result is that M 11+ ions can be Idum
easily moved from one lattice site to' another. The nearest
neighbours can be thus changed easily and new metal bonds can ,.4.24 IMPERFECTION IN SOLIDS
be formed readily. This explains why metals are malleable and
An ideal crystal is one which has the same unit cell containing
ductile.
the same lattice points across the whole crystal. Any departure
(iii) Physical characteristics: Most of the metals are quite from perfectly ordered arrangement ofconstituent particles in the
hard and tough. They are malleable and ductile. They have high crystals is called imperfection or defect;
tensile strength. Except alkali metals, they have high melting and Perfect of ideal crystals can exist only at absolute zero or zero
boiling points. They also possess elasticity. Kelvin. At absolute zero the crystalline substances have zero
entropy; i.e., have no randomness. Above absolute zero-, entropy
4. Molecular crystals: In molecular crystals, the lattice points
or randoumess increases, i.e., some deviations from perfectly
are occupied by molecules which' do not carry any charge. The
ordered arrangem~nt takes places.
forces responsible for binding these molecules are dipole-dipole
attractions and van der Waals' forces. Dipole-dipole forces are The, imperfection may be intrinsic or extrinsic. Crystalline
present in solids when polar molecules occupy the lattice points as defects can profoundly after the properties of a solid material.
in the case of water. van der Waals' forces are more general and Imperfections or defects not only aher the'properti.es but also give
occur in all kinds of molecular crystals. The binding energy in rise to new characteristics. Extrinsic defects due to presence of
molecular crystals is" therefore, weak. Molecular crystals show the certain impurities give rise to interesting changes in the
following general properties: properties. For example, gem stones are crystals containing
impurities that 'give them beautiful colours. The presence ofFe 3+
(i) They are generally soft and easily compressible.
ions makes sapphire blue and Cr 3+ .ions presence makes ruby red.
(ii) They have low melting or boiling points. (Gems are actually crystals of Al 2 0 3 which is colourless).
(iii) They are bad conductors of electricity as the electrons are There are mainly two types of imperfections.
localised in the bonds. 1. Electronic imperfections
(iv) They have low density. 2. Atomic imperfections.
The following table provides a comparative summary of the
properties of various types of crystals:
266 G.R.B. PHYSICAL CHEMISTRY FOR COMPETITIONS

1.Electtoniclmpe,~ections (A) Defects in stoichiometric crystals,


This type of imp~rfection arises due to irregular arrangement (B) Defects in non-stoichiometric crystals,
ofelectrons in the crystafStructure. AtO,K (absolute zero), the (C) Impurity defects
electrons in both ionic and covalent solids are present in fully
(A) Defects in stoichiometric crystals
occupied lowest energy orbitals or states and these electrons do
not move under the influence of applied electric field. However, The crystals in which cations and anions are exactly in the
when temperature is raised above 0 K, some electrons may same ratio as indicated by their molecular formulae are termed
occupy higher energy states. For example, in crystals of pliTe stoichiometric· crystals. £oints defects do not alter the
silicon or germanium,some electrons. are released from' the stoichiometry of the solid; these defects are calfd as intrinsic or
covalent bonds above 0 K. These free or mobile electrons thermodynamic defect. Two types of point defects are observed
become responsible for electrical conductance. The bonds from in these crystals :
which the electrons have been removed on heating become (i) Schottky defect, (ii) Frenkel defect.
electron deficient and these are referred to as holes. Like free (i) Schottky defect: This defect was first observed by the
eleCtrons, holes also conduct electric current. Holes in an electric German scientist Schottky in 1930.. In this defect equal number of
field move in a direction opposite to that in which electrons cations and anions (from AB type crystal) are missing from tl:teir
move. Both electrons and holes present in the solids give rise to normal lattice sites, i.e., equal number of cationic and anionic
electronic imperfection. Electrons and holes are denoted by the vacancies or holes will be developed. The crystal maintains
symbols' e' and 'h' and their concentrations are represented by electrical neutrality.
'n' and 'p'. In pure covaient solids like silicon and germanium, This defect is usually observed in strongly ionic compounds
the number of free electrons and holes will be equaL Electrons which have high coordination number and the sizes of two ions
and holes can be obtained preferentially according to the (cations and anions) are almost same.
requirement by adding appropriate impurities. Examples: NaCI, ~CI, KBr,esCl, AgBr etc.
Silicon and getmanium are the elements of group 14th; thus,
they have maximum covalency of four. A number of solids can be Cation Anion
obtained by the combination of the elements of groups 13 and 15 vacancy vacancy
or groups 12 and 16 to get avetage valency of four as in silicon
and germanium. Examples are : InSb; AlP; GaAs. In these '@t®-e~-®
materials, In, Al and Ga belong to group 13 and sh, P and As ~{)~ +~
belong to group 15 of the extended fonn. of the periodic table.
Other examples are : ZnS,CdS,CdSe, HgTe. In these materials, ®-€>-® @
Zn, Cd andHg belong to group 12 and S, Se and Te belong to
., group 16 of the periodic table~ These combinations are not purely
~-~~~~
covalent but possess an ionic character due, to difference in the @-®-®--®-®
electronegativities of the two elements. These combinations (a) I.deal crystal (b) Schottky defect
. show interesting electrical, magnetiG and optical properties
which have been utilised in the electronic _ industries for ,Fig. 4.54
photovoltaic cell,S, light emitting diodes, thermocouples and
semiconductorS, etc. In 1 cm3 ofNaCI there are 1022 ions and 106 Schottky pairs,
thus there is one Schottky defect per 1016 ions.
2. Atomic Imperfections or Point' D~fects. Con-sequences of Schottky defect .
Whim the deviations exist in the. regular or periodic arrange- . (a) Due to Schottky defect, the density of crystalline solid
ment ofthe constituent particles (atoms or ions) in the crystal, decreases. ' .
these defects are termed atomic impeifections or point defects. (b) Lattice energy and thereby stability of the crystalline solid
These defects are caused by the following points. decreases. .
(i) When 'Some o.f the.constituent particles are missirigfrom . (c) Entropy increases.
their normal positions, these unpccupied position are termed (ii) Frenkel defect : This defect was discovered and studied
holes or vacanCies. by a .Russian scientist, Frenkel, in 1926. This defect is c.aused if
. (ii) When. some of the constituent particles are missing from some of the ions (usually the cations) of the lattice occupy
their positions and these particles have shifted either to interstitial interstitial sites leaving a corresponding n.umber of normal
vacant sites or to positions which are meapt for other particles. ' lattice sites vacant. This defect creates vacancy defect at originaI
Types of point defects : Point defects are classIfied into the site -and interstitial defect at new site. Frenkel defect is also
fallOWing three types: . called d.slocation defect. .
STATES OF MAnER
, \ .

. 2. Density is lowered in this defect. Density is unaffected in it.


Cation
vacancy 3. Dielectric constant remains Dielectric constant increases.
. same.
s---®;-®.--@
@t6 I@'
1'1 I Cation occupying '4. It is shown by those ionic. com- It is shown by those ionic com-
~-- ' -~-~-,-~ interstitial site . pounds which have high coor-. poUnds which have low COordin-
dination num.ber, sizes of cations Iat.ion number and ,size of cations
@--@/-0-<0--®
... ./ ! I 'I . I ,
and anions are almost equal. is smaller than size of amons .

®--®~-®--®--® (8) Defects in nonastolchiometric solids


The compounds in which the number ofpositive and negative
Fig. 4.55 Frenkel defect in the crystal ions are not exactly in the ratio as indicated by their chemical
formulae are called non-stoichiometric compounds. Non~stoichi­
Main conditions for Frenkel defect are: . ometric solids are also kllown asB~rthoUide compounds.
, ~ Coordination number should be low. Examples :.
,,"C'.
. (i) Ferrous oxide FeO exists in non-stoichiometric form as
~ Size of anion should be bigger than cation:
Feo,9s0called wiistite.
, Examples: This defect is not foundjn)f1ikali metal halides
Oi) Vanadium oxide- has non-stoichiometric formula VOx'
because cations and anions have almost equal sizes apd cations
where x lies between 0.6 and 1.3.
cannot be accommodate in interstitial sites. This defect is mainly
found in transition metal halides like Agel, AgBr, AgI, ZnS, etc. (iii) ZnO exists as a non-stoichiometric compound having
Silver ion (Ag + ) being smaller.in size can be accommodated in excess zinc.
(iv) Stoichiometric compounds like NaCI, KCI, etc., can be
interstitial site. AgBr shows both Frenkel and Schottky defect.
non-stoichiometric under suitable conditions,
Consequences of Frenkel' defect
the defects which ,bring change in overall chemical
(a) There is no change in the density of the solid. composition ~re termed as non-stoichiometric defects. In such
(b) There is increase in the dielectric constant as, the like· crystals, there is excess of either positive particles (cations) or
charge ions come closer. ' . negative particles (anions). However, the crystal as a whole is
(c) The solids having Frenkel defect show conductivity and neutral in natur.e. Ifpositive particles ~re in excess, the charge is
diffusion in solid state due to presenctl ofvacant'lattice sites. balanced by the presence of extra electrons. In case the negative'
(d) Lattice energy and thereby stability of cry~tal1ine solids particles are in excess, the charge is balanced by acquiring higher
decreases. oxidation state by metal atoms.
(e) This defect influences the chemical properties of ionic 'Non-stoichiometric defects are of two types:
compound. 1. Metal excess defect, 2. Non-metaLexcess defect
(f),Elltropy of the solid increases. . 1. Metal exc~ss defect: hi this defect, metal ions or cations
Th'erniodynamic facts about point defects in stoichio- are ill excess. This defect may develop on account Qfthe
metric solids : Frenkel and Schottky. defects are also called as following tWo ways: _ .
intrinsic or thermodynamic defects. (a) Anion vacancies: A negative ion may be missing from its
NUmber of point defects increases with temperature. The ,lattice site leaving behind avacancypr hole. This ,vacancy or hole
number of defects formed per ~it volume (cm 3 )' n' is given by, is occupied by an extra electron to maintain electrical neutrality.
, n==Ne-WI2RT , .This defect is somewhat similar to Schottky difect but differs in
having only one hole and not a pair as iii the~tatter case.
=
where, N number of sites per' unit volume (cm ) which 3

could be left vacant


. W == Work necessary to create a defect
.T =Absolute temperature .
®--®'~<O-®--@ ,
Electrical conductance of crystals increases' due to point
defects. When potential difference is applied to the solid then
cb--~-~-$--~ ,
ions may move from the lattice sites to occupy the holes or ®--@-~-®--®
yacancies. Such movement' is responsible for electrical
conductance, ®-~®-~-®--®
Difference between Schottky and Frenkel defect
'Fig~ 4,56 Metal excess defect due,.to-
Schottky defeCt ' Frenkel defect anion vacancy -
• .1 .

I,. Equal number of cationic and 'Some ions are displaced from,
anionic vacancies 3re present in norinallattice sites to the intersti- Anion vacancies in alk;ali metai halides are .produced by
thisdefect. . , t i a l sites. ..<' ' , 'heating the metal' halides, in the atmosphere of alkali metal
268 G.R.B. PHYSICAL CHEMISTRY FOR COMPETITIONS

vapours. Metal atoms get adsorbed on the surface of the metal . of the host ions, vacancies are created. For example, if molten
halides. Halide ions move towards the surface and combine with NaCl containing a little SrCI 2 or CaCI 2 as impurity is allowed to
inetal ions formed by the adsorbed atoms on account of their cool, some lattice sites will be occupied either by Ca 2+ ions or
ionisation. The electrons released in the process diffuse into the Sr 2+ IOns
. . Iace 0 f N a + Ions.
mp . For every C a 2+ Ion
. or Sr 2+ lon,
.

crystal and. occupy anion vacancies. The electrons trapped in two Na + ions are removed in the crystal lattice in order to
anion vacancies are referred to as F-centres. 'F' stands 'for maintain electrical neutrality. Thus, one of these lattice sites is
FarbeR, a 'German word, meaning colour. Greater is the number occupied by Ca 2+ or Sr 2+ ion and the other site remains vacant.
F-centres, more is the intensity of colour. For example, non-stoi- Cationic vacancies developed due to introduction of impurity
chiometric sodium chloride with excess sodium is yellow, non- cause higher electrical conductivity of ionic solids. Addition' of
stoichiometric; potassium chloride with excess of potassium is about 0.1 % SrCI 2 to NaCI increases the electrical conductivity
violet and non-stoichiometric; lithium chloride with excess of by 10000 times. Solid solution of CdCI 2 and AgCl is another
lithium is pink. example of this type of impurity defect.
(b) Excess cations occupying interstitial positions: Metal
excess defect may also be developed by the presence of extra Electric Properties of Solids
cation in the interstitial site. Electrical neutrality is achieved by On the basis of electrical conductivity, solids are divided into
an electron presence in another interstitial site. This defect is three categories:
similar to Frenkel defect. For example, when ZnO is heated, it (i) Metals (ii) Insulators (iii) Semiconductors
loses oxygen and turns yellow. Metals have conductivity 10 8 ohm-I cm -I and insulators
ZnO~Zn2+ +(1/2)02 +2e have10- 12 ohm-I cm- I .

@--®-@-@ Metals: Metals have free electrons and fixed kernels at


I I A+ I I their lattice sites:
$--~-~-~
I e I I I
M~MI!++ ne
kernel ... free electrons
@--0--@--®
Free electrons make the metal good conductor of electricity and
®--®-~-® heat. Conductivity of metals is nearly independent of impurity of
metals. Resistance ratio may be taken to check the impurity level
Fig. 4.57 Metal excess defect due to the presence of
of metals, i. e. ,p 300K / P 4. 2 K • Conductivity of metals depends on
extra cation in the interstitial position
. number of valence electrons, i. e., number of free electrons
The Zn 2+ ions thus formed occupy interstitial sites and the produced by the metal.
electrons occupy the neighbouring interstitial sites. The crystals
having metal excess defects contain few free electrons and, thus, Nature of Metallic Bond
such solids act as semi-conductors. A metal is regarded as group of positively charged metal ions
2. Non-metal excess or metal deficiency defect : These packed as closely as possible in a regular geometrical fashion and
contain lesser number of positive particles than negative immersed in a sea or pool of mobile electrons. The attraction
particles. These defects arise due to cation vacancies or anions force that binds the metal ions and mobile electrons is called
occupying interstitial sites. the extra negative charge is balanced metallic bonding.
by oxidation of some of the nearby metal ions. Thus, this defect 1. Conductivity: When potential difference is applied
occurs when metal shows variable valency, i.e., especially in the electrons (mobile) conduct electric current. It has been observed
case of transition metals. Examples are : FeO, FeS, NiO, etc. that the conductivity of most of the metals decreases with
Crystals with metal deficient defects are semiconductors of increase in temperature. At high temperature, positive metal ions
p"type. start vibration and create resistance to the flow of electrons; thus
resistance of metal increases. .
(C) Impurity defects in i0r:tic crystalline solids
Good thermal conductivity of metals can also be explained on,
~ A defect in art ionic solid can be introduced by adding foreign
the basis of mobile electrons. . .
ions. If the impurity ions have a different oxidation sfate than that
~ 2. Opaqueness and lustre: Mobile electrons absorb
quantum energy of visible light and become excited; when .
@-@-e-@-@
I I t I I
excited electrons return to normal state', visible light is evolved.
Since, the light is fully absorbed by metal hence it is opaque.
@-9--0-0--@ ~. Elasticity: On applying mechanical stress, temporary
deformation of the metal crystal takes place and the kernels with
@-eO-e-9 their electron cloud return to their original position as soon as
mechanical stress is removed.
c0-€?-®-~--® 4. Microproperties of metals: Some other properties of
metals are sUmllarised ahead:
Fig. 4~58 Impurity defect
STATES OF MATTER 269

(a) Metals possess high tensile stress. On heating, some covalent bonds are broken to give free
(b) Metals are usually hard and solid. eleetrons. These free electrons make it semiconductor.
(c) Some metals like alkali and alkaline earth· metals show Width of forbidden zone in semiconductors is very low; thus
photoelectric effect, i. e. , ejection of electron from metal surface on heating, electrons are easily tossed to conduction band from
when exposed to light of appropriate energy. valence band and conduct electric current.
(d) Metals have low ionization energy. (ii) Extrinsic semiconductors or Impurity
(e) Strength of metallic bond depends on the following semiconductors: These are formed by dopping impurity of
factors: lower or higher group.
Strength increases with: n-type Semiconductor
(i) increase in number of valence electrons, Dopping of higher group impurity forms· n-type
(ii) increase in charge on the nucleus, semiconductor, e.g., when 'As' is dopped to 'Ge', an extra
(iii) decrease in size ofpo~itive metal iO!l' valence electron of arsenic makes the mixture n-type
Valence band: Group of electrons containing valence semiconductor. Current carriers are negatively charged, hence
electrons. called n-type.
Conduction band: The free electrons for conduction
constitute conduction band. Extra valence
electron forming
Band structure of metals may be given as shown in Fig. 4.59. n-type semiconductor

Conduction band
Valence band
n-type Semiconductors
Conduction band
Valence band

Fig. 4.59 ~-- Conduction band


Donor level
There is no energy gap between valence and conduction band,
Valence band
i. e. , width of forbidden zone is zero. Thus, as soon as potential
difference is applied electrons jump to conduction band from Fig. 4.61 n-type SellJiconductor
valence band and conduct eiectric current. All valence electrons
are free to conduct electric current. When temperature is raised
conductance of metals decreases because at high temperature Impurity dopping creates donor level just below the
conduction band.
kernels vibrate and create obstruction in the flow of electron ..
On applying potential difference, electrons jump from donor
Semiconductors: Conductivity of semiconductors js· inter-
level to the conduction band and conduct electric current.
mediate between metals and insulators. These are of two types:
Free electron on heating
p-type Semiconductor
Impurity of lower group creates acceptor level just above the
. valence band. When·· electrons jump from valence band to
acceptor level, a p-hole (positively charged hole) is created in
valence band. On applying potential difference, these p-holes
conduct electric current.

, ./ , ./
, /Ge, /Ge, ./
Ge /Ge G-"e~_,c.-- p-hole
~ 'Ge ' Ga04---"Ge (electron
Ge/ 'Ge..... 'Ge/ deficient
/ 'Ge/ 'Ga./ 'Ge..... band)
Conduction band
-+-- Forbidden zone· / ' G e..... " . / "
Valence band p-type Semiconductors

Fig. 4.60
~."'~~~~~ Conduction band
(i) Intrinsic semiconductors:
These are insulators at room Acceptor le.vel
temperature and become semiconductors when temperature is Valence band
raised.
Fig. 4.62 p-type Semiconductor
G.A.B. PHYSICAL CHEMISTRY FOR COMPETITIONS
. ",r.r
Insulators: Such substances do not conduct el~ctric current; (ii) Diamagnetic Solids

"'z='
their conductivity is very low. Width of forbidden zone between The solids, which are weakly repelled by external magnetic
valence and conduction band i&very big; hence electrons are not field, are called diamagnetie solids and the property thus
tossed from valence ' . . , e~hibited is called diamagnetism.
band to conduction Conduction band ." "In diamagnetic solids, all electrons have paired spins, i.e., only
ban.d. On heat{ng,,>~~\ .• ,.I'\' Forbidden zone ,fully filled orbitals are present. Examples are: NaCI, Ti0 2,
they decompose but .. Valence blimd Zr0 2, KCI etc. Some diamagnetic solids like Ti0 2 show para
do not' give free.···,;·· .... . ' < .... . .
magnetism due to presence of slight non-stoichiometry:
conducting " . . Fig. 4.6~ .'
electrons. . (Iii) Ferr0n:tagnetic'Sollds
Examples: Classiflcation of transition metal oxides The solids which are strongly attracted by external magnetic
field and do not lose their magnetism when the external field is
Metallic semiconductors InsulatorS removed, are called ferromagnetic solids. The property thus,
vo , FeO, V2 0 S MnO exhibited is termed ferromagnetism.
TiO ,Ferromagnetic substances can be permanently magnetised.
. Fe203,Mn203
C0 3 0 4 , Mn 30 4 Only three elements, iron (Fe), cobalt (Co) and nickel (Ni) show
ferromagnetism at room temperature. Some other examples are:
.CuO EuO, Cr02, etc. Cr02 is the oxide used to make magnetic tapes
CU20 for use in cassette recorders.
TiO and NbO become superconductors at low temperature, Ferromagnetism arises due' to spontaneous alignment of
about 2 K. magnetic moments due to unpaired electrons in' the same
direction as shown below (Fig. 4.65).
4;25 MAGNETIC PROPERTIES
t Parallel spin
I 11III 111 '1111 1t I
(I) ParamagnetiC Solids
(spin in the .
The presence of one or more unpaired electrons In an atom; . same direction)
molecule or ion gives rise to paramagnetism. Such materials are
weakly attracted in the field.· When they are placed in it magnetic Fig. 4.65 Ferromagnetism
field they become parallel to the field. Domain significance of
unpaired electrons are given below: Mechanism of magnetisation of ferroml!gnetic substances:
Unpaired electron in one atom interacts strongly with unpaired
electron of neighbourhood atom, thus they align themselves
'9i' .1> ~ rt spontaneously in a common direction in a small volume of solid
called domains. Magnetic moments of all atoms in a domain are /

..,~
parallel to each other hence the do'main possesses a net magnetic
-e.. J25 jO dipole moment. In absence of external magnetic field, the
,
direction of magnetic moments in different, domains' .are
'&.. ,f?! -6+ )0 randomly oriented in different direction. When external magnetic
field is applied then the domains in which-the magnetic moments
'(In absenoe of magnetic field) (In presence of magnetic field) are parallel to external field, expandat the expense of remaining
domains and ultimately all the magnetic moments align
Fig. 4.64
themselves in the direction of external magn~tic field.
,Resultant magnetic. field for n-unpaired' electrons may be
calculated as:

-- - --'--
External magnetic field .
f.!M = In(n+ 2)
The magnetic field produced by unpaired electrons is due to---

--
(a) their spin and (b }their orbital motion.
. Magnetic moment is measured in Bohr Magnetqn (BM)
, ~ , ~.
BM=-~: . .' ,
.. 4n.)iicF ' . Fig. 4.66 Magnetisation of ferromagnetic substance
f.!s = 1.73 8M (one unpaired electron)
f.!s ~.'2.83 BM (two unpaired electrons) (iv) Antiferromagnetic Solids
.. f.!s. = 3.87 8M (three unpaired electrons)
f.!s = 4.90BM (four unpaired electrons)
The solids which are expected to show paramagnetism or
ferromagnetism on the basis of unpaired electrons, but act~ally
"~.

5.92,6.93,7.94 values for five, six and seven unpaired


electrons respectively.
STATES OF MATIER 271
. .

have zero net magnetic moment are called' antiferromagnetic


s,plids. i i tIi. ' iii
. 1 '. tJ',.+t
t i tI'i' i tI.tt.: Parallel spin. .
AntiparaUel spin,
. .' .",' "

. Fig. 4.68 Ferrimagnetism .. ~

Fig. 4.67 Antiferromagnetism .


. Exa~ple~are: Fe304 arid ferrites of the fo~ul~M2+'Fe204
where M = Zn., Cu, Mg, etc. .. . '::~.~L;b '
. ' . Magnetic moments are.ali~ed in a compensatory way so as to , Influence of temperafore on magnetic character oiiim~s ~
give zero n~t magnetic moment. Antiferromagnetism is due to the An magnetically ordered solids. (ferromagnetic, antiferro-
presence of equal number of· magnetic moments in oQPosite rnagnetic and ferrimagnetic) change into paraml;tgnetic state on
directions as shoWn above (Fig. 4.67). The importatO exmnples increasing temperature. This is due to .randomisation of their
are: MnO,.Mn0 2 , Mii 2 0 3, NiO, CoO, etc. spins. The following examples explain this point : ,
. (i) Ferrimagneticsolid, 'Pe304' becomes par~agneticwhel'l
(v) Ferrimagnetic Solids
heated to 850 K. , .
The s6lids which are expected to show large magnetism due to (ii) Antiferromagnetic solid~ ~03' becomes 'par~~etic
the presence of unpaired electrons hut in fact have small ,net , when heated to 750 K. ' "'
magnetic moment are calledferrimagnetic solids. . It should be noted 'that each' ferromagnetic solid has a
Ferrimagnetism arises due to alignment ofmagTIetic moments charaqteristic temperature above which itbecomes paramagnetic.
in parallel and antiparallel directio!ls inuneq~l numbers , The 'characteristic temperatUre' is called Curie· point for Curie
resulting in some net magi,letic moment. This alignment has been temperature.
shown ahead (Fig. 4.68). . " -....

Table 4:1 Magnetic Properties of Solid at a Glance

Properties Information Magnetic alignment I Example Appibtion

1. Diamagnetic Repelled weakly in magnetic field. Such 'solids' have Benzene,·NaCi, ..... Insulators ,'/'
only paired electrons. ' .1l1l1l1l1l Ti0 2.,V20 S' etc.,

2. Paramagnetic Have unpaired electrons; weakly attracted in. magnetic


, field. They cannot be permanently magnetised. '/-?l'\r?
" ° VO.
2, CuO, TiO· Electronic devices

3. Ferromagnetic Also, have unpaired electrons, Strongly attracted 'in ii,iiii i F~,Ni, Co, cr02 Cr02 is us~ in au-
magnetic field. Such solids can be permanently magnet- dio, video tapes,
ised, On heating to a temperature called Curie Point,
these solids change to paramagnetic solid.
, '

4. Antiferromagnetic In these !lolids, unpaired electrons align themselves in Cr2C3' CoO, COP4' Used in the instru-
such away that resultant magnetic moment is zero. F~O:i' MnO, Mn02 ments of magnetic
susceptibility .
measurement

5. Ferrimagnetic Unpaired electrons align themselves in such a way ~hat i J. J. i J. J. i J. J. . F~04' ferrites
there is a net magnetic mom\:lnt. , OR·
iiJ.J.J.itJ.J.J.ii .
and so on
I
.272 I' G.R.B. PHYSICAL CHEMISTRY FOR COMPETITIONS

Table 4.2 Dielectric Properties of Solid at a Glance

Properties Information Dipolar property Example. I Application

1. Piezoelectricity When a crystal of dielectrics is subjected to Development of charge in i Quartz and Used in mechanical
mechanical stress, then small magnitude cur- some part ofcrystal. IRochelle salt electric transducer,
rent is produced. It ,is called direct e.g., in record player,
Piezoelectric effect. transmission of direct
'signals, sounding of
. sea depths. '
2. Anti piezoelectricity In some solids, electric field develops me- Crystal suffers elastic defor~
chanical effect. mation in an electric field
3. Ferroelectricity "1 Plez,oelectriccrystals having permanent di- iii iii Ba TiO}, KH 2P0 4 Electromagnetic
. poles are said to possess ferroelectricity. Rochelle salt appliances.
, 'r, '
4. Anti ferroelectricity I' Pieioelectric crystals with zero dipole ~e PbZr0 3
said to possess anti ferroelectricity. Lead zirconate
" I
5. Pyroelectricity . Some polar crystals produce electric impulse Crystals of tartaric Used in fire alarms,
on heating. acid , and thermostat

, Superconductors
Copper and most other metals are good conductors of thereafter even higher values of Tc for other copper containing
electricity. But even the best conductors present some resistance oxides: 90 K for YBa 2 Cu 3 0 7, 125K for TI2Ca2Ba2Cu301O
to the flow of electricity and a part of electrical energy is and 133 Kfor HgCa2Ba2Cu308+x.[YBa2Cu307 is also called
converted into heat energy. Thus, generators, turbines, motors of 1-2-3 compound, i.e., I yttrium, 2 bariums and 3 coppers].
all kinds" transmission lines and other electrical equipment Room temperature 'Te' superconductors are yet to be
always operate at less than I 00% efficiency. .discovered. The great advantage of these materials that offer no
A superconductor is a material that loses all electrical electrical resistance is the elimination of heating effects as
resistance below a characteristic temperature called the superc 'electricity passes through. The energy saving and efficiencies
conducting transition temperature, ~.e., a superconductor that would be realised are substantial. Superconducting coils of
presents no resistance to the flow of electricity. wire could store electricity (in principle) forever, since no energy
Electrons move freely through a superconductor without any is lost. Thus, the energy can be tapped as needed. One of the most
resistance or friction. dramatic properties of a superconductor is its ability to levitate a
This phenomenon was discovered, in 1911, by the Dutch magnet. When a superconductor is cooled below its Tc and a
scientist Heike Kamerlingh Onnes, who found that mercury magnet is lowered towards it, the superconductor and magnet
abruptly loses its electrical resistance when it is, co'oled with repel each other and the magnet hovers above the superconductor
liquid helium to 4.2 K. Below' its critical t~mpeniture (Tc), a ' as though suspended in midair. The potential application of this
superconductor becomes a perfect conductor and an electric effect will be in the high speed, magnetically levitated trains. '
current, once started, flows indefinitely without loss of energy. Some applications of superconductors already exist. For
Most metals "become superconductors at very low temperatures example, powerful superconducting magnets are essential
"(generally around 2-5 K), but this temperature is far too cold for components in the magnetic resonance imaging (MRI)
any commercial applications. instruments used in medical diagnosis. Superconductors are also
Since, 1911, scientists have been searching for materials that used to make the magnets that bend the path of the charged
s\fperconduct or at, higher temperatures and more than 6000 particles in high energy particle accelerators. All the present
superconductors are now known. Niobium alloys are particularly applications, however, use' conventional :;mperconductors
good superconductors and in 1973, a niobium alloy, Nb 3Ge, was (Tc :5 20 K). Of course, the search goes on for materials with
found to show superconductivity at 23.2K. The situation changed higher values of Tc' For applications such as long distance
dramatically in 1986 when MiiDer and Bednorz reported a Tc of electric power transmission, the goal is a material that super-
conducts at room temperature. '
35K for the non-stoichiometric barium lanthanum copper oxide,
Bi:h La 2 - xCu0 4 , where x has a value of about 0.1. Soon
m··~

Exiunple 1. Thelt' is a collection of crystalline substances 8 3+ ions'" 1x 80"" 40


ilt a llexagOllal closed packing. (f the density of matte" fs 2
2,6g I ('nt~, what would be tlte average dTity of matler in A 2+ .. n 3+ : 0 2- 20:40: 80
collection'! WI,at fi'action of space is actlla/~v uhVccupied ?
SoJIftioli~
In hexagonal closed packing,' the packing iii "" 1: 2: 4 •
etlkiel1cy is 74.05%. . :. Formula is A1J2 0 4 ,
,', Density of matter = Packtns fraction x Toml density . ,·.E".mphi 4. At t425"C F'e crystallises in ct body...:etitlt'd
'\ (;ubic lattice whose edge length is 2.93A. AYSllmirrgthe atoms to
- ---x",.
74.05 '" 6-
-.t 93.-
g~
3 .be packed spheres, calculate .'
. .too (n) the radius bfthe spnehtS.
%empty space "" lOO~ 14.05 =25.95% . (b) the diatance bfttween centlt'fJ ofneig~bmtringspheres,
. (c) the ftumb!!r of atott/!! oJFe per un It tatticft and
." ·&xample 2. The unit celllrmgth of NaCl {s obserVed to be
O,St527ItHt by X-ray d(lfhlCtiOfI studies; the Hteasu~d del1sify of· (d) the total volume ocC'UpteJby an ofFe. atom
NaCI is ~.164 gcm-:l. COfrelale the difference of ob.,erved tJ11d Solution: (ar a/3 == 41' whm. q edge letqStb
calcltldhtd dlmsily and calculaft! % dfmissiHg Na + and cgl~ fons. . a.[j 2.93 x 13 .
Solution: We know that, . r=: _.-'. "" - , ... -- tiogA
3
·4· 4
. u dxN
Z~~='--~-
M .
.
(b) Oistattc~ betwMi t~ ~ €If neighbwting spheri:s
~ .
. : ! : : Zr:z: 2 x 1.2(;8.:: 2.531 A
where, z:= number of constituent unit!! per unit t~U
(c) N~, of Irtomst per Mit i:ell "" J x y& . . . 1:5: :2 .
"" 4 in fcc
a e edge let\gth '" 0.5627 x .10= 1 em (d) Volu~ ~ed by an irtom ~ iron %')'1;rJ ,
d =detlsity
N "" Avogadro's numbt!t
··En.,,, 5. In foct~eottf1!d cUMe (fCc) clj,atat laitice,
ttJgft length of the unit cell ill 4UO pm. Fmd t!tll diameter oj't!te
M IE \1\t)laf mtlliS grl!tJtegj !lph~fl which can be fined into the intllr!ltttial void
(0's621 x 10=1 )~ x d~fi023 x 1023 ;;;4 without dllliortimt of I m / k e : ' (lIT 2005)
Solution: 1ft ftt wit ~'etlt we ~ ~
58.S
aJ2~4t
d". 2.180$8 I ~m3
aJi 4OOx.[i
Observed dtmsity 2.1.M g I ~l1ij which Islei'll than ~81llubned
!!§
r=="'~::l4L4pm
4 4 •
density because some places lire mhlslnl, .
Actul\l constttucnt Ulitts per unit ~ellllan b~ Illil~ul\1t~d Il!~; 'Of o~ldutdnd yoW;
'l(rtR);tf;g
z "" (0.5627 ~ !O=i )3 X2.IM )(6.0;13 x 10~j .. 3,~69 lRi;;,tJ 1.r
58.5
iN ;;;; d = Zr:!§ 400= 2 x 14t.4=::: JJ1.16J'm
Missing units "" 4 = 3.969!!!! 0.031
Dial:flltt~f tlf ~f~Wi9t ~_~ 1.1.,1~ph.
% mlssing!§ 0.031 x lOO§; O'.?7!% . ·"IAmpl. 6, . fit!! dia1tJnf:1/ hfttW&'e1# P_Q: ~1O(fJJ/!/J$ ~l!I!rdJ
ttJ tlit! fUN! t)/llitl uttlllfltll '!f. ~odtum d/lliJrldft 0$ m JP1!IIIL. 1'IIw.
txamplt! 3. Itt d cubic latttt;'~, tlt~ dost!t1 pttcktld i1tl'tt('tU~ 3HftdleM angle tJ/ ffJj/;!tJittlfi OO§{ltlied #11 fhft X-fIiIj ~flijM
ofmlxed.oxlde~ ofthe IdUice b ,.."J~ up o/t}xfd~ im,a ; mw ~jgltth jJdttt1r1i IN ~:~7t1: t:t1lfulatlt Iltd 'wmlttlmtPh of the mdJow
of the tetre/hearnl voids dJ1l occupied hy divalimr ltllta (A;!~ ) 1'dt/latitJlt:·' " .
wit Ue olle ha(f af the octahedral voids ttl\' mxupi;ffl by trtVtilfilllt Solution: U!jili~ ~f§g~;!1 &~UjUOO~
IOH~ (B 3+ ). What Is the fimmtla oJ thf! oxides ? i
. 1i'A U§iH 9
Solution~ tet therli! be 1ft) Oi= In the Cty9tAI. 1~ 1. ~ ~ ~ It§i j(: Hr!Z §in § . 91
..
• Octahedral voids"" HO . 'A ~J§fjj(: Ht.!l mil!! §8'«';pm
Tetrahedral void!! 160 .. ~ filIJfm_I(', c§t:1 1m!! fHtbitf I/tI'Utfut'rt ofl(ma in which Cs+
A2+ 101111 a
.8
1X 150 20 if:lH (t1 jJltit1e/U Iii fhtt !JfJt!,V=N?lUPfff of tltu cube, If density is 3.99 g
tilr'":l :' .
"'J • 274 G.R.B. PHYSICAL CHEMISTRY FOR COMPET!TIONS
"I
,
, i
Ii
,/ . '

(a) calculate the length of the edge ofa unit cell. Example 10. A solid AB has the NaCI structure. Jfradius
(b) What is the distance between Cs + and CI - ions? of the cation A+ id20pm; calculate the maximum value of the
(c) Whatis the radius ofCs+ ion if the radius ofCrion is radilff of the anionB"". .:" ,
180pm?
Solution' We know that,
.
Solution: Since, NaCl has octahedral structure,
+
Thelin:riting ratio r~ = 0.414
(a) Z rB
M
Z 1, lCs+ + lCI- ::: ICsCl or
- =--=--=290pm
rB
rl 120
0.414 0.414
M 168.5
3 23 Example 11. You are given marbles of diameter 10 mm.
1= a x3.99x 6.023 x 10
They are to be placed such that their centres are lying in a square
168.5 bond by four lines each' of length 40 mm. What will be the
a = 4.123 X 10-8 <;m= 412 pm arrangement of marbles in a plane, so that, maximum number of
marbles elln be placed inside' the area. Sketch the diagram aMf
. (b) Distance between Cs + and Cl- ions derive expression for the number of marbles per unit area.
(lIT 2003)
aJ3 412 x.J3 = 356.8 pm
Solution:" In order to accomodate maximum number of
2
sph~:res, there should be hcp (hexagonal closed packing).
(c) res+ + '(r = 356.8
Area of square having spherical marbles 16cmz
res+ + 180= 356.8
Maximum number of spheres = 14 (full) + 8 (half)
res+ 176.8pm
Number of spheres = 18
E:x;ampie 8. If NaCl is doped with 10-3 mol per cent of
SrCl2 ' what is the concentration ofcation vacancy ? Per unit area (per cm 2) 18 1.125
16
Solution:
"
Na+ ' Cl- Nil+ cr Length PQ of square = 4 cm

Cl- Cl- Na+ Length·PR 5+4x5J3'=40mm=4cm


..
Sr 2+ cr Na+ Cl- 4cm

I Cl- Na+
Number of cationic vacancies per mol
10- x 6~023 x 10
3 23
Cl- Na+

6.023 x 1018 vacancies per m.ol


pl--40 mm--lQ
A B
A:rtt
\V
10 mm 10.lm

100
, ,',' Example 9. A metal crystallises into two cubic phas<;s,
c
face-centred cubic (fcc) and body-centred cubic (bce) whose
unit lengths are 3.5 and 3.0 A respectively. Calculate the ratio of
densities offcc and bcc. (IfF 1999) CD = 10 sin 60°:: 10.J3 sJ3
2
x At. mass
'Solution; Density offcc Example 12. Calc~late the density of diamond from .the
Av.no.xV1 '
fact that it has face-centred cubic structure with two atoms per
. . b Zz x At. mass, lattice point and a unit cell ofedge length 3.569 A.
and d ensIty 10 cc = -=-----
, ,'Av.no.xVz SoiuticJ1: Z::;: 8,M = 12

, 1'3 xp xN
Z::::--'---
M
For fcc Zl 4; VI = a 3 = (3.5 x 10-8 )3 '. ZM 8xl2
Densltyp = --'=,-:-.----'---:::--:----'---:;:::-
For bec . Z2 = 2; V2 = 0 3
= (3.0 X' W: 8 )3 1 N (3.569x 10- )3 x 6.023 X 1023
3 8

d fix
8
4 x (3.0 X 10- )3 = 1.259 :: 3.506g cm-3
d bee 2x (3.5 X 10-8 )3 , ,
i
I
STATES OF MA'fTER I· 275
Example 13. .An' element crystallises into a structure X 0.53 x 6.023 x 1023 .,
=~------~----------'-r---
which may he described by a cubic unit cell having one atom at 6.94
each corner of the cube and two atoms on one of its diagonals. If
=2
. the volume of this unit cell is 24 x 10-24 cm 3 and density of
'Example 17. Cu metalcry;tallises in face centred cubic
element is 7.2 g / cm 3 . Calculate the number of atoms present in.
lattice with cell edge. a 361.6 pm. What is the density of Cu
200 g of the element.
crystql? (Atomic mass ofcopper 63.5 amu, NA = 6.023 X 1023 )
Solution: Number of atoms in a unit cell (Z) 1 + 2= 3
reBSE-PM; (Mains) 20081
. l3 xp xN
Z~ t Solution: We know
M , 3 .
3 Z=a xdxNA
1 xp xN
M M
Z
d:::: ZM
24 x 10-24 x 7.2 x 6.023 x 10 23 ... (i)
34.69 a, 3NA
·3
__Mass
_ _ x 6.023 x 1023 Z:::: 4 for fcc unit cell; M :::: 63:5 g mor I
Number of atoms
Molar mass
tv.=361.6 x 10:- 10 cm; Nfl =6.023 x 1023
= 200 x 6.023 X 1023 ::: 3.47 x 1024
34.69 Putting these values in (i), we get
;'~:~~~Ejllmple 14. Analysis shows that nickei o~de has formula d= 4 x 63:5". = 8.94 g Co;-3 "
Nio,9S01' ,Whatfraction of nickel exist as Ni 2+ artil Ni 3+ ions? (36L6x 10- 10 )3 x 6.023 X 10 23 ' '
, .
Solution: Let number of Ni 3+ ions = x ';:::' 'Elample 18. An LPG cylinder' weighs 14.8' kg when
empty. When full, it weighs 29 kg and shows a pressure of 2.5
NumberofNi 2+ ions = (0.98 x) atm. In the course of use at 27°C, the weight of the full cylinder
reduced to 23.2 kg. Find out the volume of n.;,butane in cubic .'
Since, molecule is neutral, its total charge is zero.
metres used up at 27~C and1 atm. .
(+3 x x) + (0.98 - x)x2- 2:::: 0 (MoleculaFmass ofn butane == 58] ,ICBSE-pl\fr (!\1ains) 20081
Solution : Mass ofn-butane used = 29"': 23.2 == 5.8 kg
x =0.04, i.e., Ne+ = 0.04; Ni 2+ =0.94
Num·b er 'or moles of butane
' used 5800
- - ;:100
Example 15. . Calcium crystallises in face-centred cubi; . 5.8 '-.
unit cell with a = 0.556nm. Calculate denSIty if ' V = nRT ,lOOx 0.0821 300
(i) it contained OJ % Frenkel defect. P 1
(it) it contained 0.1 % Schottky defect. . " ~. ' == 2463 litre
Solution: (i) Frenkel defect does not alters the d~ity of
solid. '=2.463 m 3
4Ox4 Example 19. r and r _ represents radius of Na + and
+ "
Na ct. .'
cr ions respectively. If 'n' is the number ofNaClunits per unit
cell then give the equation you will use to obtain molar volume.
:::: 1.5455 g / cm 3 , [CBSE-PMT (Mains) 2009J
(ii) Schottky defect lowers the density of solid Solution: "Edge length of unit cell 'a':::: 2(r + + r - )
Na CI

Z 4 _ 4 x 0.1 =- 3.996 ,Volumeofunit~ell a3 8(r


Na
+ +rCI _)3
Volume occupied by on~Q1e ofNaCI
100
.
d = 40x 3.996
8(r +' + r - )3
(0.556 x 10-7 )3 x 6.023 X 1023 :::: Na CI xNA
3 n
= l.5440g / cm 23
where N A 6023 xl 0
.' Example 16. Density of lithium atom is Q.53glcm 3. The
e4ge lengthofLi is3.5A. Find out the number oflithium atoms in . Example 20. Two gases A and B ofmass 706 g and 167.5 g
a unit cell. Atomic mass oflithium is 6:94. (M.;; of A 32, and M~ ofB 20) are mixed: Then total pressure
of mixture is 25 bar. Calculate partial pressure ofA aM B. . ,
jCBSE-PMT (:'~ahis) 20051
iCBSE·PMT (Mains) 20091
3
Solution: Z= a xp x'N A Solution: Number of moles A, n A 70.6 ='2.:2.. .
M 32
I
276 G.R.B. PHYSICAL CHEMISTRY FOR COMPETITIONS

Numberof mo IesO·f· B ,178. =167.5


- - == 8'"
. .)75 = 2.2 x25== 5.20bar
20 2.2+8.375
Partial pressure of gas A., P.4 PB ::: 25-5.20= 19.8bar

. SUMMARY AND IMPORTANT POINTS TO REMEMBER


........, •• , . . . . . . . "'(c "'.... ............ ..... ... ''JI\tf.nu ......
1. Matter: Matter is aitY.thing which has mass and PV
(iv) Gas equatiom.~, -::: constant
occupies space. Matter exists in three states, viz., solid, liquid and ·T
gas. Any substance can exist in either of the three states
For one mole ofan ideal gas, PV:::: RT
depending on temperature and pressure. Liquid and solid states
are condenSed" states as they bave mucb bigber densities. Both For n moles of an ideal gas, PV =nRT
liquids and' gaSes are termed as fluids as they .have flowing For wg of the ga~ (mol. mass M)~
ability:
: .. 2; Gaseous state: It is the simplest state and shows PV=~RT
greatest uniformity in behaviour. The gases show almost the M
same behaviour irrespective of tbemical nature, colour or odour. wRT RT
or P::::--=dx-,. (d = density)
·The molecules in a gas possess random motion in aU directions. A V M 'M
.... gas baS neilher a definite shape nor a definite volume. Gases are
characterised by low density, high degree of conlpressibility and R is a universal gas constant. It is expressed in the units of work
expandability. Gases exert uniform pressure on the walls of the per degree kelvin per mol. Value of R in different units is
· container; They diffuse" rapidly and' intermix ,to torm R = 0.0821 litre -atm K -I mol-I
homogeneous mixtures. AU gases obey. certain laws called gas
laws. . • ...... ::::. 82.1 mL-atmK- 1 mor l
1. Gas laws: Gas laws are based on experiments. .= 8.314 X 107 erg K-1 mol- 1
. '(i) Boyle's" law: It states tha~th~ volum~ o(tbe given ...
amourit of a gas is inversely propor1:ional to its pressure at a = 8.314 JK- I mol- 1
constant temperature..
1. . . . ' = 1.987= 2eal K- 1 mol-1
- or PV ;:::: constant or l~VI == P V = P V = ...
V 0:::. 'p. .. 2 2 3 3
(v) .Avogadro's law: Equal volumes of all gases contain
......
. equal number of molecules under similar conditions of
or (at constant temperature) temPerature and pressure. 'kllume occupied by one mole of any
gas at NTP (O°C and 1 attn) is termed molar volunie. It is 22.4
~here:d 1 andd 2 are the densities of a gas at pressures PI and P2 • litre ot 22400 mL.·, . . . .
· The curves' between: P and V at .constant· temperature are {Nole :At 25"Cand I bar, one mol~ofa gas has a volume of 25 litre.]
hyperbolas and called isotherms. .. . ..' . (vi) Dalton's law of partial pressures: . The total pressure
(ii) Cbarles' law: At constant pressure, the volume of a . 'exerted by a mixture of gases which do not react chemically is
given amount of a gas is d,irectly proportional to its absolute equal to the sum of their individual pressures (partial pressures)
tempmture. . " . under similar conditions of temperature,
. .·V . Vi' V
VeeT or -=constant.or - : : : - 2= - : :3 : : ••.
V ,· . P =PI + Pz + P3 + ...
T . TI T2.·T3 ,Partial pre~ur.eof a gas "" Molar fraction
Absolute temperature scale or Kelvin scale starts with -27-:f'C . x Total pressure of the gas
· as zero. .(vii) Gr~balIi's law Ofdiffusion: i,.~e property Of gases t<;l
Absolute temp~rature (in K) :::: 273 + tOe' '. mix with'eachother to formhomogen~ous mixture irrespective of
The absolute zero is that temperature where no'gas can exist. .gravity is called diffusion. E.ffusion is a special case of diffusion
::,'The p}ots of ~~lume against tempe~atuTe (at constant pressure)' in which gas passes through a small aperture.. .
· are termed aSlsobars.'

I .... . . .' .

, . ' ' o
trnder similar conditions of temperature and pressure, the
. (iii) Pressure law or Gay-Lussac larv: At. cpnstant relative rates.of diffusion of gases are inversely proportional to
volume, the pressure of a given amount of a gas'varies directly as . the square root of their densities or Vapour densities or molecular
its absolute temperature. ' .' masses. This is'Graham's law of diffusion.

~ =J¥= ~~~~ =):~


. , p'". ' . p. . PI P, P3
. r:x;Tor -=constant or -::::~=-="
. ' ..r . . Ii T2 T3 ..
.
.
STATES OF MATTER 277

Ii ~ 1M2 '. (c) Most probable speed: It is the speed possessed by


r2 == P2 VMl maj()rity of the gas molecules. It is equal to
(2RT . ..
. 4. Kinetic theory of gases: The theory with the help of -VMO~(U8164x rms)
which behaviour of gases can be interpreted mathematically is
termed kinetic theory of gases. The main postulates of the theory .Most probable speed.iAverage speed .: nos speed •
are: : : I : 1.128: 1224
(a) All gases consist of very large number of small spherical 6. van der Waals' equation: It is au improVed form of
elastic particles called molecules. Tht!Se are in constant rapid
motion in all directions. .
ideal gaS' equation. It has ~ been derived after applying two
cOrrections due to two· \\Tong assumptions. in kinetic theory of
(b) There is a lot of empty space between the molecules: The gases. Two corrections are:
actual volwne of gas molecules is negligible in
comparison to
(i) volume correction, Yi = W - b ) and
total volume. of the gas. .
(c) The pressure exerted by the gas molecUles is due to its a
(ii) pressure correction, P; P"bs. +-.., .
rec~.rded by molecules against the walls of containing vessel. The V-
coHision~ between gas molecules are perfectly elastic, i.e., there
is no loss of energy during collisions.
(d) There is no effective force of attraction or repulsion
.The equatioo is
...
(p + ~)(v
y2
-b) =RT forone'mole.. .
between gas molecules. There· is no effect of gravity on the
motion of gas molecules. . n2a) (V - nb)= nRT for'
;
(e) The average kinetic energy of gas molecules is directly
and
(p + ~2 n'moles~

proportional to the absolute tempera~ oftbegas:


On the basis of above postulates. the following equ3bon, a and b are constants. Units of a are atm litl mor
l or Pa m 6

known as kinetic gas equation, bas been derived. mor: ~d 'units of b are litre Of" dm or m • This equation is
2 3 3

applicable tn real gases.


I .,
PY= mnc- Real gases: These do not obey various gas laws strictly
·3
under aD conditions oftemperatbre.and pressure. Deviations are
where, P = pressure; Y == volume; m== mass of gas" molecules; large under high pfessUfl!" and low tempenlture. . .
n :: number of gas molecules and c :: root mean square speed of Ideal gases: These obey laws strictly under all CQRditions..
the gas molecl.des.. Actually no gaS is ideal. .. .• .
(a) A~ very low pressure01" at Vel}" high temperature. Y is
Also PY=RT=.!.mnc2 =3.; mnc2 :;:;3 KE,. . a . "
3 3 "2 3 large, so y 2 and b can be negleeted.. The equation becomes
,. 3 3
or K.E - RT :: - py PY == RT. i.e.• the gas behaves like ideal gas.
2 2 . ·0 .
(b) At moderate pressure. Y is l\Ot so large and ~ cannot lie. •
K.E of single molecule = ~ RT .. .,' . y ..... .
2N neglected However. b can be neglected."so that we have.
3
=-IcT
2
(p+ ya )V=RT
l
'.
where, k == Boltzmann constant.. ·0
PY=RT--
5.
Molecular.speeds: There are three kinds of speed. Y
(a) Root mean square speed: IJ is the square root of the (c)· At bigh pressure, Y is so small that bcanoot be neglected
mean of the squares of the speeds ofail the molecules present in a but a / Y 2 can be neglected.. Hence. th4: equation becomes·
gas. ~ P(Y - b)=R~ .
2 ~? 2
1.'1 +Ci +1.'3 + •.• +1.',. PY=RT+Pb
n (d) In case of Hz and He. the· intenno~ forces of""

~~3:==~3~T f5 attraction are negbgtole. i.e... a is vtly smaD,. so th3t. ~. can be


. Y.
(b) Average speed: It is the arithmetic mean of speeds of neglected. The equation becomes
the molecules present in the gas. P(Y-b)=RT
.. PV=RT+Pb

=0.9213 x nus
218 GRB. PHYSICAL CHEMISTRY FOR COMPETITIONS

',7. ' Compressibility factor: The extent of d~viation ofa a - -'1_ (for,all gases)
real gas from 'ideal bebaVi~ur is expresse~in terms of v ~ 273
compress~bility factor, Z.
At constant pressure, increase involum~ of a gas, per degree
PV', .". rise of temperature percc of gas at O"C is called coefficient of
Z -'-; Z is unitless quantity.
" nRT 'l
j,:;. volume expansion., Similarly,.the pressure, coefficient' a p is
defined as, ' .
(i) Forideal gas Z =I ..
(ii) For real gas Z:;t 1. a p == PI -:- Po =-,1-' (l:'1.or aII'
gases)
When Z < 1, there is negative deviation from i4ealo'ehaviour . P0 x t 273 , . {7-

and the gas is' more compressible than itteaLgaS. (ii) CO, Nz., C 2 H4 and B2H.6 hllVe same moIir ma~s (28) and
, .When Z> 1, there' is positive' deviation ~~ideal behaviour shoW sarr,e rate of diffusion under id(lIltical cOfiditions. , . .
imd the gas is less compressible than ideal gast!· .' (iii), Atmolysis is the process 10 separate two gases on ·~he
, 8.· Critical ph~n9menon:, (i) Critical temperature is that basis, of difference in their deIl$ities and rate of diffusion...
temperature' abovi which,tbegas caDnQtbe liquefied with the _, (i~) Average molecular mass of air is 29. If a gas has
help o(pressure (ii) 'Critical pressure is the minimum pressure molecular mass less than air, it is lighter than air and if it has
required to liquefy the gas at critical temperature (iii) Critical moleCular mass greater than air, it is heavier than air, e.g., Cl 2
. ,volume is the volume ,occupied by one, mole of the gas at its (71), CO 2 (44) and S02 (64) are heavier than air, while H2 (2),
critical temperatu~ and critical pressure. , ije (4), NH3 (17) and H 20 (18) are lighter than air.
. , '. a, Sa . ' 11. Transport pheno~ena of gases:
, Vc =3b: PI;=;:; 27b2 T: = 27bR """.~ (i) Collision diameter (0')= Distance of closest approach
" Boyle's temperature: It is the temperature abovCII which ~he '. when two molecules are under collision. .
gas behaves like.. an ,ideal gas. In ~ of van der.Waals·
. .:'. .
':'~.:
, constants, ,:,1 ":r~·: ~·:l"·.':~
. . .' a
. Boyle's temperature(TB ) == -
.' . '. '. ' . Rb
I I,
. . .,' TB > T c , • t4-- 0' ----JIt1 '/

Inversiou temper-ature: Ii: is the temperature to which the


. ' .gas li!hould be cooled so that expansion is accompanied by further (it) Collision number:. It is the number of collisions of a
cooling.·.'·.".',· ' . m()lecule with other molecules in one second. .
9~ Molar "heat capacity: It is the quantity of heat N == .J'2nvav O' 2 N
, require~ ,to raise, the temperature of ~ne fuole ,of a substance .c V
. through JOC., . ' . . . , " , ) where, vav == Average velocity of molecules
Mo.tar1ooatcapacity = SpeCific heat capacity x Molar mass
(i) l\1olarneat capacity at constant pressure (C p ): IUs N == Number of molecules
'theq~tityofbeatrequiredto raise the temperatUre of 1 mole of V == Vf,llumeof triolecules
gas through 1°C Under Constant pressure. It is equal to Yz R. . (iii) Collisioll frequency (Z): It is the number of Jnolecula'
, ..'...•. (ii)Molar heat· capacity at constaut volume (C v): 'It is . collisions takitlg ptai;eper second per unit volum~.ofth~gas.
the quantity of heat required to raise the temperature of 1 mole of
gasilirough 1°C unde.r constant yohimeL It is equal to ~ R. , Z= ~1tVava2 (.N)2"
Cp -C v =R .:' .' "1/2 V ' fI

(iv) Mean fre~ path')", :' It is the ave.rage:qist;ancetravelled


"• "~DrmQ~oatomic~es,CP
C '
=1'=1.66
;
between two suecessivecollisions. ' '. ."
' • .1 . '" ','
v
')",1

For diatomic ga~es. . C p = ~ R == lAO


CvYzR .J'21ro
2
(~)
. 'C 8R>4 . ')", /IX T (at constant pressure) .
and fOJ: triatonnc gases, .2.. =- =- == 1.33
C v , 6R 3·· 1
')", /IX - (at constant temperature)
p.
(Cp')~=:nl X(Cp)l+n2~Cph
" . , Rl +n 2 A ex: 1.a (at constant temperature and pressure)·
~ -;Vo
10/'{i) Volume coefficient; a v -.-'-'-. '12. 'Liquid state: Liquid state is conden~ed gas or molten •
Vo xt' solid. ··Liquids have definite volume but no definite shape.'
I
STATES Or.: MATTER 279
Liquids diffuse slowly and ar~ almost incompressible. Some Or
important properties of liquids are: The force per unitarearequi(ed to mamtainunit difference of
It is the spontaneous change in which a ... velocity between two consecutive .parallellayers of the liquid
liquid changes into vapours at the surface ofliquid. It increases which are one centimetre apart. It-isexpressed in dyne cm-2 S-I
with increase, of surface area, increase in temperature and or poise. .
decrease in inteimolecularforces. It causes cooling. The reciproct!l of coefficiept of viscosity is called tluidlty.
,!"t'>:3s,m); It is the pressure exerted by the Liquids With hlw viscosity are'termed m6b:i).e while others with
vapours of a liquid 4n equilibrium with the' liquid atJl, ''''high viscosity are called viscous. Viscosity is high in liquids with
given temperature. It increases with increase -of temperature~ :. high intermolecular forces, high molecular masses and having
Liquids with low intermolecular forces have high vapour branching in the chain of the molecule. Viscosity decreases with
pressure. . . . . increase in temperature.
It is the temperature at whicli the vapour (vi) Reynold's nUIJlDer: The nature of flow of a liquid,
pressure of a liquid becomes' equal to atmospheric pressure. a
through tube depends on the value ofReynold~s numberN R'.
Temperature of the liquid remains constant after it etarts bolling 2rvp
as the energy supplied increases the potential energy but if!> NR --
11
kinetic energy remains constant. Boiling point changes with the \where, r;:: Radius of the tube
pressure of surr07di(n;:.) , I:lH JI '( 1' I) v = Average velocity of the fluid
p = Density of liquid
<~ . ' og PI = 2.303R 1) - T2
11 ;:: Coefficient of visCosity
where, at;l..v = latent heat of vaporisation; P2 and PI are vapour Case 1: When N R > 4000, the now is called turbulent flow.
pressures at T2 and 1)., " '
Case II: WhenNR < 2100, the flow is called larftinarflow.
0m'~a"i; ten:§llon: It is the force in dyne acting on the
surface at right angles to any line of unit length. It decreases 13. The solid state: The particles are closely packed and
generally with increase in temperature. It is expressed in J m -2 or are hFld toge$er by streng intermolecular forces. The· particles
Nm-Iordynecm- I . cannot move at random. They have only vibrational motion.
Solids' have definite· sWipe and definite volume. They possess
(a) Surface tension is molecular phenomenon of liquid c.highdensity"and low compressibility. :Solids are divided into two
involving the force of cohesion among the liquid molecules. -, classes~ (i) amorphous solids' and (ii) crystalline solids. In
(b) Maximum potential energy of liquid molecules is on the amorphous solids, the arrangement of building constitUents IS not:
surface. . regular but hap~d. Their mcl.ting points are not sharp¢ These
(c) Surface tension is numerically equalto the surface energy, are isotropic in nature, , .' , ,
The work required to increase or extend the surface area by 1 sq. In crystalline solidS, bUilding'constituents atTange themselves
em is called surface energy. in regular manner throughQut' the entire three'-dinlensional
(d) Surface tension is a scalar quantity. network. Crystalline solids consisto{large n1lJ!lber of units called
(e) Sparingly soluble solutes like soap and detergent decrease crystals. A crystal is defined as a solid figure which has definite
the surface tension of the liquid; However, fairly soluble solutes geometrical shape with flat faces and sharP edges. It has a sharp
like sugar and salt increase the surface tension of the liquid. melting point and anisotropic in nature: . '
(f) 'Liquid drops are spherical because liquid surface has a 14. Typ.es pf symmetry in crystals: The total nUmber of
tendency to acquire minimum surface area. planes; axes and c~tres of symmetry possessed by a crystal are
(g) Greater is the intermolecular. force, D?ore is the surface termed as elements of symmetry. '
tension. '(i) Centre of symmeky: It is an imaginary point wi¢rin the
e.g., Soap in water < Water < Mercury crystal such that any line drawn through'it intersects the surface
H 2 0> C2HS OH > CH 3 OCH 3 of the crystal at equal distances in bQth directions.· A crystal
_alw,,"ys possess only one centre of syn{metry.
CHiOH
CH2 0H (ii) Plane of symmetry: It is.an imaginary plane which
1
CHOH >1 passes through the centre of a 'Crystal and-divides it into two equal
CH 2 0H portions such that one part 'is exactly the mirrdr image.of·the
1
CH2 0H Glycol other.
Glycerol , . (iii) Axis of symmetry: It is an imaginary straight line
It is' the resistance in a liquid to flow. The about which, if the crystal is rotated, it will present- the same
force in· newton per square metre: required to maintain a appearance more than once during the complete revolution. '
difference of velocity of one metre per ~OJld between two A cubic crysta'fpossesses a total of 23 elements of symmetry. '
parallel layers of a liquid ata distance of one metre from each 15. Space- hittice and un}t cell: The, ge~metricai form
other i~ called coefficient of viscosity. It is expressed in g m:-I s -I . consisting of a regular array of points in space is called a space
"

G.~,a. PHYSIG~I,.: QHEMl.$TR:YFQRCQM~~TlTlQNS

lattice or an .lU'rny of ~int~ wowing how molecules.. atoms Of Bra~ presented a re!atiooship Mtwoen the wavelen!lth of the
ions are lUTt\ngediu dltl'er~ut: sites, in tbfee..dimensiOMI space is . Xwrays and the distince between the planes,
called ~. lattice,
~""2dsina
.T1w mWlt>it fOpoati~ unit in a SpaQe lattic~· whiQh wlwn
oopoated over and over again f(I&ults in a crystal of tlw given Where, 11 is an inwger such as. l.~, 3... ,' A the wavdength, €I is is
su\:la~ce iii termed· ~t geU. A unit cell poss~ all the the distance between ~ting pJaues. of particles<md e the angle
stt\lctural propertieso~ t~given ct'Y~t The edges of the unit of deflection Of glancing aJlgle.
cell are desijMted as "a. band c, tl.OO the angl~ between thfee 18. eh;\r_ct.th:s;Qfcqbic·systernll:.
ll\U\ginary axes as a., tl mld YNS~tively. . (i), Number of. atorns~ per unit. cell: (a) Shnplc c.ubjc
16~ Cry$(lll sys~,rn$: . ~ven types of hasic or primitive • stTuc~ne atom PQr unit cell, (b) Fact}-centred cubic
unit cells have been fec~ised among crystals. ~ aN: atmcture-4 atoms per unit cell and (c) Body·oontred cubic
L Cubw . a:,;;; be :. c (t"" a,;
y 9(f NaCl. KCl. ZnS
structUre:-2 atoms per un\t cell.
2. onllofhomhic 4::#:b~c Q,.::;,~ 'f "" 9()? .KNO:!. BaSQ4 (it) DtI)sity oflaJticematter "" If XM3.
Noxa
3. 'r~.wg®al . (l ",,/1 '" cd:;;;. fl '" y '" 9tf NH~Qr, SnO:f
4. MOOOQlinic. a 'Ii b :;II. C l':( "" 'I:;; 9W • MooQQlimc sulphur, where; n is the number of atoms in wlit cell, M is the atomic mass
I.\.~ go:' caSO,* • 2H~O . Of molecular mass.. Nois Avogadro's number '\}ld a edge length
Qfthe cullic unit celt '. .
ail; b~r. Q.~*Y;; ~ CuSO<t·SH2O, .
• .~ a3aO~ .. (ill) . At011\icradi.u$~ .
Q ",1J, ~ C Q:;;; f:\ =i; ~; lnO. Agl. SiC • .(a) Simpk cubic ool~RadiusQfatom a
. ,,;;;;IN . 2
1. Rhom~ «""H,;c «:;;;f:\:;;;y.9tJ' NaN~.CaC03 (b) Fa\»Q(lnttOO cubic ceU'-""R~dius ofatom == ~
AU Q~ 00 not have simple la.ttiQt)S. Theft) can he 14 212
diffeNllt ways in wh~ similtw ptlints can be ammgedin a th.fee
Qi~ s~, T~~tals ~longi"g toa oubk syst~n haw (c) .&d)H~entfed cubic .U-Radiusoi;tom::
. .J3
4 a
thNe kinds of l""~: . . '"
(i) Sb.nple cubi~ Ja~c~i' Thew aN points only I.\t tb~ . I? Close.. packing, of constitueAts; It refers to the-
QQ.QWfS. Qf ~tlCh unit: . . arrangement in which constituen~upy maximum available
(ii) ,Fa~e~en~f't;~' ~~a,.i~ lattice:.. Th~w aN po~nts at the spape. The two common types of cl~ packing are:
,(a) AB AB ." arnmgement This· gives hexagonal close
oo.n\(IfS ai!w(\ll as at the centre of ~aQh Qfthe ~x faces of the cube,
packing. - .
(~1:l\WJy~c~p..ed.cp~i,-;Jant~~L 'Thew aN points at ~
eQrnt)fS. as w~U ~ in Ui~ centw .of each cv"be, (be) ABC ABC '" arnmgement. This gives CUbical close-
~king, •
., . Ty~ \\f~tti~ pOO.l! C~tiQn to ~ wU( ~ll
In ~ packing of constituents. some interstitial si~s aoo
c~ ~ left T~ ~tes aN te~ vtilds. ~ interstitial s.i~ between
E~ . K
fQur sphews is tetrah~ and betwee-n six spheres is ootah~.
. Fac~.wmrt< ~ T~ ~e QCQupied by hard spheres in hop; cep (fcc) is 14%;
800y~tr\i ') in bQc it is 68o/~ in simple cubic it is 52% and in diamon<l it is.
34O/~ thus. only fcc and hcpaN cl~ packed stl'\lC~s'"
•. "~\\~ Qf alj~i metals ~~Q~ ~ of·Cs..... ~wn . (i)· hQp is ~1 in lW. M~ Ca. Cr, Mo. V anct.2n.
tw~ QX~. _ $\\lph~ of ~l~ wtb··:tMWs
(-i) QQP is p~nt in Fe. Cu. Ag. A\L, ~. At and N\,
~~~ W Mdhalm· Q(.~lver ~~~ Ail ba'le ~ (m) bQQ ~nt in alkali metals, .is
SAlt (NiCl ~) st~~,
0'<') All ~ g~ ha"e C\lp structure except belium which
.• CsC•• Csllr. Cst, TiCl. T~ TU haw esC. ~ ~t\tN. has. hQP stf\\Qture. . .
• CUCl. C\\Bt. CuI, A3l. ~ ~w ~nQ. hl_ stf\\9tuw . If <»' a.phews. aN PNSJmt tn packtng. th~ the numbers of
(~AS~), •
te~l and ootahedral voids aoo 2n and n n:~tiv~ly. '
• StCl~, Stf'~. ·&Cl~, ~~, CdF l ' agF ~ ht\~~ tluoril~ 2(); TJt.~,:.coOJ:'d!n.atioJl! n.~~~r:. TOO n~aws.t neighbours.
(C~:a)~-' . . w_th whi~h a gi"en sphere is in cootact i&~led coordination
• N(\~Q, K~O,~~s,. K~Shave anmluoo~(Li~O~) ~,It ~nds "pQn struc~.
s~tuw.
(a) ~ «uhWSlruQ~: ~iMtioo numb« :: 6
17.', ~~ag(~ Ja~:, W~ x..~s ar~ In!.1\d~. on a ~~w (h) ~tNd ~ stNc~; c~natkm n\lIDbel' "" 12
~c, th~~ ~~t\t~iuto ~ Q~~atl(\ ~~ ~~l~ .:tn·
(c) ~~entredcu~ struc~; coordiMtioo numbfit "" 8
di~ut~ Ftom.~h oft~ p,l~ x~ aNQ~&Qt~,
STATES OF MATIER .1 281

For iook solids, th~ {aUo of the: radi~ C)f cation \0 that Qf throughout the whole of the crystal. At absorutelet'O, the crystals
anion is caUe\! radiuS" mHo, The: ta.dius ta.uo for a givcm are: ide:al put with rise in tempemtufe. 'there is a chance of
coordination n~mber is fixed, distortion, .T~ def~t may ~. at a point along a line or over a
For radius ratio (0,155- (U~) wordination nl.lmller = 3 surface. .
(i) SchQttky defect: This def~t is cau.sed if some of the
For radius ratio (6,225- 0,414) wordination num~r= 4., •
lattice points ~ woccupied, The Points which ~ unoccupied
For radius ratio (0,414- 0,132) coordination number = 6 ~ called vacancies or holes. The number of missing positive and
For radius ratio ({},732-1.0) cootdination numoor = 8
.
21. lIfypes of crystals: ~ing upon the: Ra~ of
. . . negative ions is the same and, thus. the crystal is neutral. Such
defect is conunon in ionic compounds with high coordination
forces that hold the constituent pa.rticl~s. together. in the: crystal number where the ctations and anions are: of similar size, e.g"
.lattice, crystals ~ classlfie\!\nto four types: • . Nap, KCl. KDr, etc. ' •
.. (i) Ionic crystals: The fQrc.es are electrostatic,~ la~ (fi) Frenkel~: This defectt is cau.sed when some of the
points. are occupied by pos"iv~l~d. ne:gati'l~ly charged looS, ~ ions Qf ~ latti~' ~upy interstitial sites leaving lattice sites
These crystals <U~ quite hard, haw low volatility and have: hig.h vacant ~his defoot appears generaJly in ionic crystals in which
melting and boiling points.. These are ins\da\OfS. hl.lt bewme- good anion is much latger in sae than th.. cation, e,g.. Agar, ZnS.
contructors in molten state, Th~se a~ solubl~ in polat solvtnts .. In ~hottk.y defoot, density decteases while in Frenkel defect it
and highly brittle in natu~, NaCl. KCl. '&C1 2 , etc" belong to this remains the same, In both the defects. electrical conductivity is
class. • observed to SQOl.e: extent The ovetall ~hero.ical cOmposition of
. Oi) Coyalent crystals:', The wnstlwenl ~rtkl~. are the the substance dOes. not change, Howe., there ~ other defects
atoms of same or different elements. whi~ ~ hdd by strong . in which there is change in ovet'all chemical combination, Such
covalent bonds, These SQlids ~. ye:ry hard and have: very high defects ~ ~ aq.s~tfk defects. In such crystals,
melting points., Diamond and catOOnmdum ~lQ~ to this. class.. there i~~ of oitbel' pooitive pmicles or negative particles,
(iii). Metallic crystals: . Th~ m~alli~ cfyst~s. have positiv, Howev•• the ~s~ is. neutn\l in natwo,'
metal ions. as. )attic~ points sult'Qunded by a sea of mobile Positive particle excess defects: (i). A. negative ion may 6e
electroni. Each electron belongs to a number ofpos.itive ions and missing ooro. the lattie» site which is ~Up\~ by an extra
each positive ion belongs \0 a number Qf e:l~trons, The f~e that ~lecttron. (ii) An ~itta'l)()sitive particle i& pment in the interstitial
binds. a metal ion to a number of ~lectrons is known as m~taUlc position, T~ maintain neutrality, an extra el~tron also occupies a
bond, Met~lliQ Qryst~ls' mOstly' belong. to fQC. bec I;Uld he-p pla.~ in the intetstitial sJ)aCf. Such crystals are u.sually col~
systems, These are good conductQl'S, possess lustre: and aw hard, . and semiconduQtors..
tough. malleahle and ductile in na~, Positive particle deficiency defect! One of the positiv~
(iv) , Molecular crystals: The constitu~nt witsofm()l~ular­ ~les may ~ missing. from its lattice site, The extra negative
crystals are molecul~s, Th~ forces holdin~ the molecu.les ~ charg~' may be balan~ if a nearby positive particle aQqwres
weak. van der Waals' fQrces, MQloo\lh.\r s~ds. ~ soft and ~~tra :pQsitive ch~, ~h ~stals are alSQ semtcxmductors.,
vaporise easily. These: ~ bad conductors. Qf elootriQity and J3x.ample-s. Me: FeO, FeR NlO, etc,
.p%sess.low den~ty,Examples aN ice. solid. CO~. etQ, C~in ®f~ts in crystals arls.e du.e to Pfe~~ of chemical
/. 22. Ddects. in crystals: A~ ideal crystal is the on~ which impurities. Ih~1l' are bQ.wn as impuritydll'~ts, .
has the same wit cell1loo.ntaining the same lattice points
282 G.R.B .. PHYSIC,4L CHE~JIISTRV FOF! :;::O~iiPET!TIOj\~S

-10 Ma~'l"TIx
Mat,-;h Type QU!':3tlolD§: [D] Match the Column-I with Column-II:
[A]Match the crystalline solids in Colun'm-I with the Column-I Column-II
coordination number of their ions in Column-II:
(a) Boyle's temperature (P) a/Rb
Column-I Column-II
(a) CsCl (P) Coordination number of cation = 8 (b) ~ (Inversion temperature) (q) 8a/27Rb
(b) CaF2 (q) Coordination number of anion = 4 2

(c) ZnS (r) Coordination number of cation = 4 (c) Critical temperature (r) The gas cannot be
(d) Na 2 0 (s) Coordination number of anion = 8 liquefied above this
temperature, on
[B] The diagrams below depict the different processes for a applying pressure
given amount of an ideal gas_ Match the Column-I and
Column-II: (d) Critical pressure (s) a/27b 2

l~ l.~
[E] Match the Colunln~I with Column-II:
.column-I Column-II
(a) Real gas at high. (P) PV=RT+Pb
v- 11N -,-+- pressure
(i) (ii)
(b) Force of attraction (q) PV= nRT
,among gas molecules
is negligible
(c) At high temperature (r) 2=1
and low pressure
T- T-
(iii) (iv)
(d) Real gas at N.T.P. (s) (p+ ;22}V - nb)= nRT
i -7 initial state,
f -7 final state [F] Match the Column-I with Column-II:
Column-I Column-II
Column-I Column-II
(a) Fig. (i) : System proceeding (P) Temperature will'
from initial state to final state remain constant 3
(a) Internal energy of gas (P) 2RT
(b) Fig. (ii) : System from initial (q) Pressure will
state to final state 'decrease (b) Translational kinetic
(q) ~RT
(c) Fig. (iii) : System proceeding (r) Volume will be energy of ~as molecules 2
from initial state to fmal state' constant (c) The temperature at which (r) ....:J.73°C
(d) Fig. (iv) : System proceeding (s) Temperature may , ~here ill no molecular
from initial state to final state mcrease or motion
decrease or may
(d) The lowest possible (s) 3.716 kJ at 298K
first increase and
temperattire at which gas
then decrease
molecules have no heat
[C] Match Column-I with Column-II for ideal gases:
[G] Match the Cp'IUmJ,,-I with Col\lmn·II:
Column-I Column-II
, Column-I (A)lumn-II
( a) If temperature of given (P) Average speed of gas
gas is increased will increase (a) NaCl (PJ Schottkydefimt
(b) If the pressure of a given (q) Root mean square (b) ZnS (q) Frenkel defect
, gas is increased at speed of gas molecules,
(c) AgBr'L (r) 'Develops yellow colour on heating
'constant temperature will increase
due to F-centre
(c) If the den~\ty of a given (r) Most probable speed of
(d) KCl (s) Develops blue/violet colour.on
'gas is lowered at ' gas molecules will
heating due to F-centre
constant temperature increase
"

(d) If the volume of a given (s) Speed of gas molecules


'gas is increasc;d at will not change '
constant .fOmpeJ;,ature ,
'\"" '
STATES OF MATIER 283
[H] Match the crystalline. solids in Column-I with packing Column-I Column-II
arrangement in Column-II: (Crystal System) (Bravais lattice)
Column-I Column-II (a) Cubic. (P) Primitive only
(a) CaFz (P) Cations are in ccp arrangement (b) Hexagonal (q). Pri~tive, body centred
(b) NaCI (q) Anions are in tetrahedral voids . (c) Tetragonal (r) Primitive, face centred, body
(c) ZnS (r) Anions are in ccp arrangement centred .
(d) Na20 (s) Cations are in octahedral voids (d) Orthorhombic (s) Primitive, face centred, body
centred and end centred
[I] Match the crystal system in Column-I with unit cell
dimensions in Column~I1: . [K] Match the Column-I with Col.umn-II:
Column-I Column-II Column·I Column-II
(a) Cubic (P) a=b=c f3P
(b) Rhombohedral (q) ex == ~ y = 9a'
(a) Root mean square velocity (P)
Vd
(c) Orthorhombic (r) a:#b:#c '" (b) Average velocity (q) ~3:T
(d) Triclinic (s) ex:# () ;i!: y :# 9a' . oif~

~
[1] Match the crystal system in Column-I with Bravais lattice
in Column-II: (c) Most probable velocity (r)

(d) Velocity possessed by /2RT .


maximum fraction of
molecules
(s) v--;;-

1. [A] (a p, s); (b - p, q);(c - q, r);(d - r, s)


[B] (a- s); (b -p); (c-r); (d-q)
[GHa p, r); (b - q); (c p, q); (d - p, s)
[C] (a-p, q, r); (b-s); (c-p, q, r); (d s) [H](a p, q); (b r, s); (c q); (d - q)
[D] (a - p); (b ---' p); (c - q. r); (d - s)
[I](a p, q); (b p); (c q, r); (d - r, s)
[J] (a - r); (b p); (c q); (d - s)
[E](a p); (b-p); (c-q, r);{d-s)
[K](a p, q); (b. r); (c s); (d- s)
m~-~~~(b-~~~-~@ ~

-
1. A gas occupies a volume of 250 cm 3 at 745 torr and 25°c'
What atlditional pressure is required to reduce the gas volume
to 200 cm 3 at the same temperature?
PROBLEMS-
[AilS. 17.13]
[Hint: 11 == Pressure at the surface:; 1atm
== 76.0 x 13.6 x 981 dyne/cm2
[AiDS. 186.25 torr]. Pz == Pressure at a. depth of 50 m
2. A vessel of 120 cm3 contairis ~. certain mass of a gas at 20°C .= 76.0 x 13ix 981 + (50 x 100) x 1 x 981 dyne/cm 2
and 750 torr pressure. The gas was transferred to a vessel of == 981 [76.0 x 13.6 + 5000]
volnme 1'80 cm 3 • Calculate the pressure of the gas at the same = 981 x (6033.6) dyne/cm 2
temperature. .
[Ans. 500 torr] . Now apply Boyle's law; PI VI :; PzVz
3. A gaseous system has a volnme of 580 cm3 at a certain or == 76.0 x 13.6 x 981 = 0.1713
VI P2 ' 981 x 6033.6
pressure. If its pressure is increased by O.96Jiiltm, its volume
becomes. 100 cm3 . Determine the pressure of the system. % = 0.1713 x 100 = i7.13]
[Ans. 0.2 atm] 5. A sample of gas at room temperature is placed in an evacuated.
[Hint: p.;< 580 == (P + 0.96)100] bulb of v61ume 0.51 dm 3 and is found to exert a pressure of
4. A gas filled freely collapsible balloon is pushed from the 0.24 atm~ The bulb is connected to another evacuated bulb
surface level of a lake to a depth of 50 m. Approximately what whose volume is 0.63 dm3 • What is the new pressure'of the
per cent of its original v.,lnme will the balloon finally have, gas at room temperature?
assuming that the gas behaves ideally and temperature is same ~<\ns. 0.1074 atm]
/
at the surface and at 50 m depth?
284 G.R.B.' PHYSICAL CHEMIS1RY FOR COMPETITIONS

6. It is desired to increase the volume of 800 cm3 of a gas by 20% 15. Calculate the volume occupied by 5 g of acetylene gas at 50°C
keeping the pressure constant. To whattemperature should the and 740 rom of pressure. (liT 1991)
gas be heated, if the initial temperature is 22°C? IAns. 5.23 litre]
[Ans. 81Q.c] 16. Calculate the volume occupied by 7 g of nitrogen gas at 27°C
7. A chamber of constant volume contains hydrogen gas. When and 750 rom Hg pressure. (MLNR 1992)
the chamber is immersed in a bath of melting ice (O°C) the (Aos. 6.24 IiUe]
pressure ofthe gas is 800 tOtT. What pressure will be indicated 17. A spherical balloon of 2 I em diameter is to be filled up with
when the chamber is brought to 100CC? hydrogen at NTP from a cylinder containing the gas at 20 atm
[Ans. 1093 t o r r ] " and 27°C. The cylinder can hold 2.82 litre of water at NTP.
8. Calculate the volume occupied by 4.045 x I 0~3 molecules of Calculate the number of balloons that can be filled up....
(MLNR 1991)
oxygen at 27°C and having a pressure of700 tOtT.
[Ans. 17.97 dm).] [Aos. 10]
[Hiot: Volume of one balloon
' Nfl " 4,045 x IOn 0 672' P 700 atm ,
;: ~ nr3 ;: ~ x 22. x (21)).;: 4851 cm3
. lOt: o. 0 roo es n ::
[H '3:::';;; -
6.023 x 1O~ '760 ~.

3·372·
Now Use the gas equation, PY ::: uRT ]
;: 4.851 litre
9. Calculate the moles of hydrogen present in a 500 cm 3 sample
Let n balloons be filled; thus,. total volume of hydrogen used in
of hydrogen gas at a pressure of 760 rom ofHg and 27°C. filling balloons '.~
[Ans. 2.03 x 10-2 mol]
;: 4.85 I x n litre
(Hint: 500 cm3 :: 0.5 L, 760 rom ofHg = I atm. Now apply the.
Total volume of hydrogen in the cylinder at NTP,
. py)
gas equation, u '= RT Y =: 20 x 2.82 x 273,:::: 51.324 litre
300 x I
10. Calculate the volume occupied by' 4 mole of an ideal gas at
Actual volume ofH" 'to be transferred to balloons
2.S x Iff Nm- 2 pressure and 300 K temperature.
;: 51.324 - 2,82 (2.82 litre retained in the cylinder)
[Ans. 39.9 dro3] :::48.504 litre
[Hint: Y "" nRT , Given, n:: 4; R:::: 8314 NmK- 1 mol-I; . . 48 504
l' No. of balloons fined' n' '" - ' - == 10] .
, . 4.851
T = 300;1'= 2.5 x lOS Nm-2) i\
18•. An open vessel at 27°C is heated until three--fifth of the air in it
11. What is the volume occupied by 11 g of carbon dioxide at has been expelled. Assuming that the volume of the vessel
27"(; and 78.0 rom ofHS pressure? remains coostant. fmd the' temperature to which the vessel has
[Ans. tiliUe) been heated? (MLNR 1990).
(Hint: 780 rom of Hg = 780 atm; w '= 11 g; M "" 44 g mol-i. lAos. 477"C)
760 [Hint: Both volume and pressure are constant as the v:essel is
w RT open,
NowapplyY:: M .-;;-]
.'.e., nlBTI :;: n/tT2
12. A certain quantity ()f a gas occupies 200 mL when collected or "l""li
OVef watetat 15"C and 745 rom pressure. It occupies 182.6 mL "2 TI
in dry state at NTP. Firui the vapour pressure of water at 1soc.
IAns. Bmm] If Il, ;:. 1, I~;:
_. 1 -
3 2
-5
13. The density of a gas i~ found to be U)7 ~ L-I at 30°C and 2 T~
atmospheric pressure, What is its density at NTP?
-52 ::; -=-
300
or T? ;: 750 K 477' C ]
-
lAos. 1J49;L-I) 19. A gaseous mixture of He and 02 is found to have a density of
Dl~' D.. 0.518 g L-1 at 25"C and 720 torr, What is the per cent by mass
[Hiot: Apply F.I :: -:- R~"" P,. = -'" RT?
M - M -
ofheliwn in this mixture?
~;~ 1 [Ans. 19.88]
Dl', Dili (Hint: first detennioo the average molecular mass and then
14. At ~ top of a mountain." the ~m.eter reads O°C and the nUmbel- Qf ndes. ofheliwn and ox.ygen.
~ef ~ 700 mm Hg., At the bottom of the- mountain MoleS t)fhel~ = 0.665~ Moles. of oxygen = 0.335
the ~~ ~ lODe and \~ pressure 160 mm Hg. u_~ 4 x (t665
Com~ the ~ty of ~ ail' at the top with that at the 1,..,..".". per cent 0
f>..-*ti
no:1 wn => x 100
4 x 0.665 + 32 x 0335
oon~
[Ans. t.~ : I} ::: 19.831
[Hint: Density is in\'~tscl.}' ~ to "'~. 20. A sample ofoatural gas is 85,20/.. methane and 14.&0/0. ethane
by ~ What is the density of this mixture at IgoC and 748
dtQp- "" VI> '" T~I: ;: 303 x 100 '" un ] mmHg?'
~\O)ft\ VI 1; ~ 213 x 166 1 [ADS. 0.74gL- 1]
STATES OF MAnER 285
[Hint: Molecular mass of mixture, 29. At room temperature, ammorua gas at. one atmospheric
M =85.2 x 16 + 14.8 x 30 13.632 + 4.44 "" 18.07 pressure and hydrogen chloride at P atmospheric pressure are
100 100 allowed to effuse through identical pinholes from opposite
PM dRT ends of a glass tube of one metre length and uniform cross
d = PM =748 x 18.07 _ 0.74 L-I ] reaction. NH4 CI is first formed at a distance of 60 cm from the
RT 760 0.0821 x 291 g. end through which HCl gas is sent in. What is the value'of P?
[Ans. 2.198 atm]
21. 125 mL ofa gas A of pressure 500 mm is mixed with 200 mL r;;;- .
of another gas B at a pressure of 300 mm in a vessel of 150 mL [Hint:' rHel == I~ X PHC1 ]
capacity. What will be the total pressure of the resulting rNH3 VMHCI PNHl
mixture if the temperature is kept constant? 3<1. 'The rate of effusion of an unknown gas (X) through ~ pinhole
LAns. 816.67 mm] ...
is found to be 0.279 times the mte of effusion of hydrogen gas
22. Two vessels' whose volumes are in the ratio 2: 1 contain through the same pinhole, if both are at STP. What is the
nitrogen and oxygen at 800 mm and 680 mm pressures molecular mass ofthe unknown gas? [CEE (Bihar) 1991]
respectively, when they are connected together what will be
[Ans. 25.69]
the pressure of the resulting mixture? •
31. In a 2 m long narrow tube, HCl is allowed to diffuse in the tube.
[Ans. 760 mm]
from one end and NH3 from the other end. If diffusion is
23. A 10 litre flask at 298 K contains a gaseous mixture of CO· and started at the same time, predict at what poitit the white fumes
CO2 at a total pressure of2 atm. If 0,20 mole of CO is present, ofNH4 CI will form? .
find its partial pressure and also that of CO 2 ,
[Ans. NH4CI will form at 1.1886 m from the NH3 end of the
[Ans, Partial pressure of CO::: 0.49 atm; partial pressure of tube]
=
CO 2 1.51 aim] 32. The composition of air is approximately 80% N2 and 20%0 2
24. Calculate the total pressure in a mixture of 4 g of oxygen and
3 g of hydrogen confined in a total volume of one litre at o°c. by mass. Calculate density of air at 293' K and 76 cm Hg

[Ans. '25.18 atm]


[Hint: Determine total number of molts and then apply
PV =~otu,RT ]
(MLNR 199.1)
pressure ..
[Ans. 1.19 g L- 1]
.. .
[Hint: D,etermine first average molecular mass of air and then
apply D =
PxM
RT av ].
25. A gas cylinder contains 5-5% nitrogen, 20% oxygen and 25%
carbon dioxide by mass, at 760 mm pressure. Calculate the 33. Calculate the internal energy of one gram mole of nitrogen at
partial pressure of each gas. 150°C assuming it to be an ideal gas.
[ADs. 472.8 mm, 150.44 mm, 136.74 mm] .[A.s. 5.275 x 10 10 erg]
,26. The density of a mixture of N2 and 0. at NTP is J..3 gL-I. 34.' Calculate the kinetic energy of 5 mole of a gas at 27°C in erg
Calculate partial pressure ofO., and calorie.
tAns. 0.28 atm] [Aos. 1.8706 x Wi' erg, 4477, cal]
[liint: Let //, and 112 moles of N2 and O2 be. present in the 35. A glass tumbler containing 243 mL of air at 100 Jd>a and 20 0 e
mixture respectively. is turned upside down and immersed in a water bath to a depth
. . 2811 + 3211:1, opO.S metre. The air in the glass is compressed by the weight
... Average molecular mass of the mixture = 1 ... (I) of water above it. Calculate the vohlme of air in the glass
III +112
Average molecular mass from'general gas equation, assuming the temperature and the barometric pressure have
not changed.? '
M= D x RT 1.3. x 0.0821 x 273 =29.137 ... (ii) [ADs. 80.7 mL]
P I
[Hint: Hydrostatic pressure = h x d x g
So; ,,2~1l,"+ 32~ =29.137" or = 1.137
III + //2 Itt +112 -= 20.5 x 1 x 9.81
'= 201.105 kPa
or =0,28 =,mole fraction of oxygen Inside the water bath atmospheric pressure and hydrostatic
~+~ .
pressure acts; •
~artial pressure of 02 := 0.28 xl::: 0.28 atm]
F:otal =100 + 201.105 =3Q 1.105' kfa
27. At a definite pressure and temperature, 100 mL of hydrogen
diffused in 20 minute. How long will 40 mL of oxygen take to P1VI =P2V2
diffuse under similar conditions? 100 x 243 = 301.105 x V2
[ADs. 32 minute) V2 := 80.70 mL]
28. At a given temperature and pressure, 20 mL of air diffused 36. Calculate the rrns speed. of the molecules of ethane gas of
through a porous membrane in 15' second. Calculate the volume 1.5 litre at 750 mm of Hg pressure.
volume of carbon dioxide which will diffuse in 10 second if [ADS•. 1.225 x 104 cm/sec]
the vapour density of air is 14.48. . 37. The density of a gas at 1.5 atm is 1.52 g L- 1• CalCulate the nns
[Afts. 10.8 mL] speed of the molecules of the gas.
[Ans. 5.476 x 104 cm/sec]
i
28SJ G.R.B. PHYSICAL CHEMISTRY FOR COMPETITIONS

38. At what .temperature will the rms speed of hydrogen be the 48. A chamber contains mono atomic 'He' at STP, determine its
same as that of oxygen at 25°C? number density.
[Ans. 18.62 K or - 2~4.38° C] [Ans. 2.68 x 1025 m-3]
39 .. Calculate the 'average rms and most probable speed of S02
[Hint: Number density = .!...
molecules at 27°C. kT
[Ans. 3.15 x 104 cm/sec, 3.419 x 104 cm/sec, (where, k= Boltzmann constant)
2.792 x 104 cm/sec] PV =nRT
40. The. average speed at temperature 11 and most probable speed . R
=(nxN )-T
4
at T2 . of CO 2 gas is 9 x 10 cm/ sec. I;;alculate the values of . N
11 and T2 • =(n x N )kT
[ADs. 1682.5 K, 2143.4 K] P nxN ..
- = - - = number denSity
4L How many times would the average molecular speed of kT V . .
nitrogen increase as its temperature is raised from -73°C and .Number density = 1.01 X !05 = Z.<i8 X 1025 m-3 ]
127°C? 1.38 x 10 2 x 273
[Ans. 1.414 times] 49. Two perfect gases at absolute temperature 11 and T2 are mixed.
42. A 4 : 1 molar mixture of He and CH4 is contained in a vessel at . There is 1\0 loss of energy. Find the temperature of the mixture
20 bar pressure. Due to hole in tlie vessel the gas mixture leaks if masses of molecules are ~ and "Iz and the number of moles
out. What is the composition of the mixture effusing out . _0f'i:he gases are nl and n2 respectively.
~ (in 1994)

.
initially? __ nlTI + nzT2 ]
[Ans.8:1] [Ans. T
nl + nz
[Hint: Partial pressure ratio of He and CH4 is 16 : 4. .
[Hint: Let final temperature is T
For diffusion rHe _ ~MCH4 X PHe 333
- niRTI + - nzRT2 = - (n i + nz)RT
rCH 4 M He PCH 4 2 22.
nHe = {16 x 16 =8 T = niTI + nzT2 ]
ncH 4 V4 4 ni + nz
i. e. , mole ratio of diffusion for He and CH 4 is 8 : 1.] 50. van der Waals' constant' b' for oxygen is 32 cm3 /mo!. Assume
43. Pure 02 diffuses through an aperture in 224 second, whereas b is four times the· actual volume of a mole of "billiard-ball"
mixture of O 2 and another gas containing 80% 02 diffuses O 2 molecules and compute the diameter of an 02 molecule.
from the same in 234 second. What is the molecular mass of 51. The speed often particles in metre/sec are 0, 1,2,3,3,3,4,4,
gas?' 5 and 6. Find (a) average speed (b) the root mean square speed
[Ans. 46.6] (c) most probable speed. .
44. 56 g of nitrogen are confined to a 6 litre flask at 37°C . [Ans. cav = 3.1 m sec-I, Sm, = 12.5 m sec-I, '1np = 3 m sec-I]
. Calculate its 'pressure using van der Waals' equation for
52. Using van der Waals' equation, calculate the constant' a' when
nitrogen. a = 4.17 atm litre 2 mol- 2 and b = 0.037 litre mol-I.
~o moles of a gas confined in a four litre flask exerts a
[Ans. 8.1l75atm] pressure of 11 atm at a temperature of 300 K. The valUe of' b'
45. One mole of carbon dioxide was found to occupy a volume of is 0.05 litre mol-I. (II~ 1998)
1.32 litre at 48°C under a pressure of 16.4 atm. Calculate the [Ans. 6.46 atm litre 2 mor2]
pressure that would have been expected froni: .
53. (a) Calculate the pressure exerted by 5 moles of CO 2 in one
(i) the ideal gas equation,
(ii) van der Waals' equation. litre vessel at 47°C using van der Waals' equation. Also, reRort
(a = 3.59 atm litre 2 mor I; b = 4.27 X 10-3 litre mol-I the pressure of the gas if it behaves ideally in nature. Given
. that a = 3.592 atm litre 2 mol-2, b = 0.0427 litre mol-I.
and R = 0.0821litre-atm K- I mol-I)
(lIT 2002)
. [Ans-: (i) 19.96 atm (ii) 18;64 atm] (b) If volume occupied by CO 2 molecule is negligible, then
46. GfllCJflate'the compressibility factor for S02, if 1 mole of it calculate tqe pr~ssure exerted by one mol of CO2 ga,sat 273 K.
occupies 0.35 litre at 300 K and 50 atm pressure. Comment on . (Ill' 2000)
the result. .lAns. (a) 77.218 atm, 131.36 atm and (b) 0.9922 aim]
[Hint: Z= PV = 50 x 0.35 = 0.711 54. A compound alloy of gold and copper crystallises in a cubic
nRT 1 x 0.0821 x 300 lattice. in which gold atoms occupy corners of cubic unit cell
Since, Z < 1; the gas S02 will be more compressible than ideal and copper atoms occupy the centre of faces of cube. What is
gas.] formula of alloy compound?
4~i The average velocity of gas molecules is 400 m/ sec. Calculate [Ans. AuCu 3]
its rms velocity at the same temperature. (lIT 2003) 55. A compound formed by elements A and B crystallises in the
[Alls. 434 m/sec] cubic structure where A atoms are at the corn~rs of the cube
STATES OF MIlIHER 287

and Batoms at the centre of the cube. What is the formula of e,,', , A metallic element crystallises into a lattice containing a.
the compound? sequence of layers of AB AB AB ... Any packing of spheres
AB] leaves out voids in the lattice. What percentage by volume of
56 A fcc element (atomic mass == (0) has a cell edge of 400 pm. this lattice is empty space? (lIT 1996)
What is its density? [Ans. 26%]
[i\ reS·. 6.13 g Cm'-3] 6-5.. A lead(II) sulphide crystal has an NaCI structure. What is its
density? The edge length of the unit cell is' 500 pm.
Apply density = Z X M ,Z= 4 and V", (4 10-8 )3
NoxV
X
=
No 6.023 x J02 3 ; atomic mass: Pb 207.2, S = 32.
cm 3 (1 pm = 10-12 m:::; 10-10 em)] 12.708 gem-3]
The unit eell of metalli:c gold is face-centred cubic.
The face-centred unit cell of nickel has an edge length of
(a) How many atoms occupy the gold unit cell?
352.39 pm. The density of nickel is 8.9 g cm- 3 . Calculate the
(b) What is the mass of a gold unit cell?
value of Avogadro's number. The atomic mass of nickel is 58.7
(a) 4 atoms (b) 1.308 xl021 g]
and 1 pm is equal to 1O- 1O cm.
C. 7. Polonium crystallises in a simple cubic unit celL It has atomic
6.029 X 1023 ]
mass 269 and density 91.5 kg m - 3 • What is the edge length of
:;~, The unit cell of aluminium is a cube with an edge length of its unit cell?
405 pm. The density of aluminjum is 2.70 g em-3 . What type [AsS', .15 ..59 x 10-8 em]
of unit cell of aluminium is? The unit cell of nickel is a face-centred cube. The length of its
[i,.as. The unit cell is face-centred] side is 0.352 nm. Calculate the atomic radius ofnickeL
Apply the formula, density = Z x M and find the value O.I24nm]
NoxV . 69. Determine the simplest formula of an ionic compound in
ofZ.] . which cations arl; present at the comers and the anions occur at
59" A substance forms face-centred cubic crystals. Its density is the centre of each face.
1.984 g cm-3 and the edge length of the unit cell is 630 pm. AB3; A and B are cation and anion respectively.]
Calculate the molar mass of the substance. Sodium metal crystallises in a body-centred cubic lattice'with
[A[['~' 74.70 g mOrl] cell edge, a = 4.29 A. What is the radius of sodium atom?
5Q', Molybdenum forms body-centred cubic crystals whose (lIT .1994)
density is 10.3 g em-3. Calculate the edge length of the unit 1.8576 A]
celL The molar mass ofMo is 95.94 g mol-I. [HiEt: 4r .J3a]
[.~[,s. 313.9 pm] : J' • If three elements P, Q and R crystallise in a cubic type lattice
An element crystallises in a structure having a fcc unit cell of with P atoms at the corners, Q atoms at the cube centre and R
an edge 200 pm. Calculate its density if 200 g of this element atoms at the edges, then write the formula of the compound.
contains 24 x t 02 3 atoms. PQR3]
4.166 g cm- 3] The first order reflection of the beam of X-ray from a given
200
[Hbt: Molar mass: x 6.023 x 1023 = 5fr.19 g mor l .crystal occurs at S'15'. At what angle will be the third order
24 x reflection?
=a3 (200 x 10- 10)3
For fcc, Z = 4, V 15° 56 ']
Apply density = ~ ]
The figures given below show the location 'of atoms in three
'. NoxV crystallographic planes in a fcc lattice. Draw the unit cell for
61.. The element chromium exists in bcc lattice. with unit cell edge the corresponding structure and identify these planes in your
2.88 x 10- 10 m. The density of chromium is 7.2 x I(Y kg m-3 . diagram. (liT 2000)

~m~
How many atoms does 52 x 10-3 kg of ChrOinium contain?
6.04 x 1023 ] '.}

Q'i.J. A cubic solid is made up of two elements P and Q. Atoms Q


are present at the corners of the cube and atoms P at the
body-centre. What is the formula of the compound? What are
the coordination numbers of P and Q?
PQ, 8, 8]
288 I G.R.B, PHYSICAL CHEMISTRY FOR COMPETITIONS

-PYOBlECI!!e gUEmoNs ·
Set-1: Questions with single correct answer S6
[H int: liN '" -
I ~ 28 "" 2;
1. At constant temperawre, the product of pressure and volUlne·
ofa given amount ofa gas is Constant. This is: II-rl =xN~ X~()tal
(a) Gay-Lussaclaw (b) Charles' law l
= xIO=4atm~
(c) Boyle's law (d) Pressure law l+
2. A curve dra~ at constant temperature is called an isothenn. Po, :: lO-4 = 6atm]
This shows the relationship between: a
12. 273 n'll. of gas at stP was tlkel\ In ~7"C Ul\(l\SOO mm ~\lrt,
(a) P and.! (b) PVand V the final volume ()fthe gas \\IOllld be: . (CPMT 1992)
V tal 273 mt (b) 300 mt •
. i (c) 380 mt (d) S8~ mt •
(c) V and P (d)P and V
13. The density ot'the aas is equal to:. (CBSE 1991)
3. Charles' law is relJresented mathematically as:· (8) tIP (b) }JPIRT (e) Pj~T (d) MW
[P = PressUl'e, V::: Volul\te, r::: temperat\ll"e; R = Gas
(a)~ ". KV()1 . (b) l't "" KVo constant; It = nllmber()fmole, AI = mol~ularmass]
t
14. The density ()fa gas is 1.96,4 g dm-s at 273 K and 16 em Ha.
(c) ~.""Vo(I+2;3} (d) ~ =:Vo(l+ 2~3) the gas is: . .(KeET 2006)
(a) CH.. (b) C 21\ .(c) COl (d) Xc
4. Correct gas equation is: [UGET (Manipal Medical) 2006) 15. Compressed oxygen is sold at a pressure ()f 100 atMosphere In
(a) IV! ",,1; (b) ViT2 "" Vl 71 cylinder of 49 litre, the number ()f 1\\()les of o~b:\ the
8
P:<V2 T2 PI Pl cylinder is: ..
(a) 400 (b) 100 (e) 'lOG (d) :tOO
(c) ~1[ = P'J,T;2 (d) ViV, = p, P.
1[T2 I '). [Hint: One mole occupies a volume approi~mately l4.S Utre
Vi Vi
under ordinary ~tmospheric conditions,] . .•
5. In general gas equation, PV = nRT, V is the volume of:
16. If the pressure and absolute temperilwre of 2 litre of carbon
(a) n moles ofa gas (b) any amount ora gas
dioxide are doubled, the volume of carbon ,dioxide would
(c) one mole ofk gaS (d) one gram ofa gas become: • (eBSE 1991)
6~ In the equation of state of an ideal gas PI' = nRT, the value of (8) 7litre (b) S litre (c),4.lItre (d) 2 litre
universal gas constant would depend only on: (CPMT 1991) 17. One gram mole of a gas at NtP occupies 22.4 litr.e as "ol~me,
(a) the nature of the gas (b) the units of measurement This fact was derived from:'
(c) the pressure of the gas (d) the temperature of the gas (8) Dalton's theory (b) AvogadtQ's hyllt)thesis
7. The value of gas constant per degree per mol is approximately: (c) BerteHus hyllt)thesis (d) law of gaseous volumes *
WI~ 002~ ~3~ 004~ 18. 4.4 g ofCOz contains how many ntre of CO2 at STP?
8. Which one ofthe following is not the value of R? (AFMC 2004)
(a). 1.99 cal K- 1 mol-I (b) 0.0821Iitre:atln K- 1 mol- l (a) 2.4 litre (b) 2.24 litre (c) 44 litre (d) 22,4 litre
(c) 9.8 keal K- 1 mot' (d) 8.3 J K- 1 mol-I 19. Five gram each of the following gases at 8'7"C and 7S0 mm
9. One litre of a gas collected at NTP will occupy at 2 pressure are taken. Which of them will have the least volume1
. atmospheric .pressure and 27"C: • • , . ' , ( M L N R 1991)'
300' r ~ (b) 2 x 300 I'tre (a) HF (b) HC! (c) HBr (til Ht
(a) 2 x 273 ltre . , "2n 1 20. If molecular mass of02 and S02 are 32 and 64 respectively. tf
one litre of 02 at 15°C and 7S0 mm pressure contains N
273 r (d) 2 x 273 litre
(c ) -
2x -
300 ltre • 300 molecules\ the number-of molecules in two litre of S02 under
the same conditions of temperature and pressure will be:
10. 10 g of a gas at atmospheric pressure is cooled from 273°C to (MLNR 1991)
O°C keeping the volume constant; its pressure would become: (a) 2N (b) N (c} NIl (d) 4N
(a) -I atm I
(b) -.- atm (c) 2 atm (d) 273 .atm• n. Rate of diffusion ora sas is:
2' 273 (a) directly proportional to its density
11. 56 g of nitrogen and 96 g of oxygen are mi)(ed isothermally (b) directly proportional to Its molecular mass
and a total pressure of 10 atm, The partial,ressures of oxygen (c) directly propol'tional to the square of its molecular IbliS!!
and nitrogen (in atm) are respectively: IPET (KeraJa) 20101 (d) inversely proportional to the square root of its molecular millis
(a) 4, 6 (b) 5,5 (c) 2,8 (d) 8,2 22. The rate ofdiffusimt of hydrogen Is ·about: (CPMT 1991)
(e) 6,4 '(a) 0nG-halfthat of helium (b) 1.4 times that othellum
(c) twice that of heHum (d) four times that ofhellum
SrATESOF MATTER 289-
23. The rate of diffusion of methane at a given temperature is
(a) 1. (b) ~
twice that of gas X. The molecular mass of gas X is: 2 9
(lIT 1990; MLNR 1995) (c) I (d) 16
(a) 64.0 (b) 32.0 (c) 4.0 (d) 8.0 9 17
24. Some moles of 0z diffuse through a small opening in 18 35. A gaseous mixture of2 moles of A, 3 moles of B, 5 moles ofC
second. Same number of moles of an unknown gas diffuse and 10 moles of D is contained in a vessel. Assuming that
through the same opening in 45 second. Molecular mass of the gases are ideal and the partial pressure of C is 1.5 atm, the total
unknown gas is: pressure is:
(a) 3 atm (b) 6 attn (c) 9 atm (d) 15 atm
(a) 32 x (45)2 (b) 32 x (18)2
(45)z [Hint: 5 moles of C produce pressure 1.5 atm; therefore partial
(l8i
pressures of D =3 atm, B'; 1.5 x 3 =0.9 atm and
(c) (32i x 45 (d) (32)2 x ~ 5
18 45
A = 1.5 x 2 0.6 atm]
25. A bottle of dry ammonia and a bottle of dry hydrogen chloride 5
connected through a,long tube are opened simultaneously at 36. 50 mL of a gas A diffuse through a membrane in the same time
both the ends. The white ring first formed will be: as for the dif'fUSion of.40 mL of gas B under identical
(a) at the centre of the tube conditions of pressure and temperature. If the molecular mass
(b) near the ammonia bottle of A is 64; that of B would be: (CBSE 1992)
(c) near the HCI bottle (a) 100 (0) 250 (c) 200 (d) 80
(d) throughout the length of the tube 37. 3.2 g of oxygen (At. mass = 16) and 0.2 g of hydrogen (At.
. 26. 0.5 mole of each Hz, SOz and CH4 are kept in a container. A mass I) are placed in a 1.12 litre flask at O°C. The total
hole was made in the container. After 3 hours, the order of pressure of the gas mixture will be: (CBSE 1992)
partial pressures in the container will be : (a) I atm (b) 2 atm (c) 3 atm (d) 4 atm
ICET (Karnataka) 20091 38. Select the correct statement:
(a) Pso z > PCH 4 > PHz (b) PHz > Pso z > PCH4 In the gas equation;PV = nRT (CBSE 1992)
(c) PHz > PCH 4 > Pso 2 (d) Pso z > PH z > PCH 4 (a) n is the number of molecules of a gas
(b) n moles of the gas have volume V
27. The mass of 6.02 x 1023 molecules of CO is:
(c) V denotes volume of one mole
(a) 28 g (b) 14 g (c) 7.0 g (d) 56 g (d) P is the pressure ofthe gas when only one mole of gas is
28. 10 g of hydrofluoric acid occupy 5.6 litre·ofvolume at NTP. present '
The empirical formula ofthe gas is HF. The molecular formula 39. The density of a gas at 27°C and I atm is d. Pressure
of the gas will be (At. mass of fluorine 19): remaining constant, at which of the following temperatures
(a) H4F4 (b) HF will its density become 0.75d? (CBSE 1992)
(c) H3F3 (d) H2Fz (a) 20°C (b) 30°C (c) 400 K (d) 300 K
29. A.frre extinguisher contains 4.4 kg of CO2 , The volume of 40. If 4 g of oxygen diffuse through a very narrow hole, how much
CO2 delivered by this fire extinguisher at room temperature is: hydrogen would have diffused under identical conditions?
(a) 24.5 litre
(c) 10 x 24.5 litre
(b) lOOx 24.5 litre
(d) 1000 x 24.5 litre (a) 16 g (b) I g (c) i g (d) 64 g
30. The number of moles of Hz in 0.224 litre of hydrogen gas at
wl2
STP is: (MLNR ] 994) , [Hint:
(a) 1 (b) 0.1 (c) 0.01 (d) 0.001 4/32
31. A gas has a vapour density 11.2. The volume occupied by 1 g 41. A closed vessel contains equal number of hydrogen and
ofthe gas at NTP is: nitrogen molecules. The total pressure is 740 mm ofHg. IfNz
(a)IL (b)11.2L (c) 22.4L (d)4L moiecules are removed, the pressure would become/remain:
32. "The total pressure exerted by a number of non-reacting gases (a) double of740 mm ofHg
is equal to the sum of partial pressures of the gases under the (b) one-ninth of 740 mm ofHg
same conditions" is known as: (c) unchanged
(a) Boyle's law (b) Dalton's law (d) one-half of740 mm ofHg
(c) Avogadro's law (d) Charles' law 42. the pressure P exerted by a mixture of three gases having
33. Equal masses of methane and oxygen are mixed in an empty partial pres.sures Fi ,Pz and P3 is given b;..y·_,_ __
container at 25°C. The fraction of the total pressure exerted by
oxygen is: (a) P = p[ +P2 P3 (b) P = ~Fi + P2 + P3
(a)-
1 2
(c) - 1.
(d) x 273 (c) P Fi-PZ +P3 (d) P Fi +P2 +P3
3 3 3 298 43. Two sealed containers of same capacity at j:Qe same
34. Equal masses of methane and hydrogen are mixed in an empty temperature are filled with 44 g of hydrogen gas in one and 44
container at 25°C. The fraction of the total pressure exerted by g of CO2 in the other. If the pressure of CO2 is I atrn, what is
hydrogen is: the pressure in the hydrogen container?
(a) 1 atm (b) Zero atm (c) 22 atm (d) 44 atm

-
290 G.R.B. PHYSICAL CHEMISTRVFoR COMPETITIONS

44 .. Absolute zero is the temperature where all gases are expected (c) low temperature
to have: (d) high temperature
(a) different volumes (b) same volume 56. In a closed vessel, a gas is heated from 300 K to 600 K; the
(c) zero volume (d) none of~ese kinetic energy becomes/remains:
45. If a gas is heated at constant pressure, its density: (a) double (b) half . (c) same (d) four times
(a) will decrease (b) will increase 57. A mixture contains 56 g of nitrogen, 44 g of CO2 and 16 g of
(c) may increase or decrease (d) will·remain unchanged methane. The total pressure of the mixture is 720 mm Hg. The
46. Density of neon wilLbe highest at: partial pressure of methane is: (EAMCET 1991)
(a) SIP (b) O"C, 2 atm (a) 180 mm (b) 360 mm (c) 540 mm (d) 720 mm
(c) 273"C, 1 aJrn (d) 273°C, 2 atm 58.' The root mean square speed, of an ideal gas at 27°C is 0.3
, 47. Equal masses of ethane and hydrogen are mixed in an empty rnIsec. Its rms velocity at 927°C ,is: ,(EAMCET 1991)
container at 25"C. The fraction of the total pressure exerted by (a) 3.0 rnIsec (b) 2.4 rnIsec (c) 0.9 rnIsec (d) 0.6 rnIsec
hydrogen is: (lIT 199;J) 59. The rms speed at NTP of the species can be calculated from
(a) ] : 2 (b) I: 1 (c) 1 : 16 (d) 15: 16 the.expression: (EAMCET 1990)
48. Ac,cording to kinetic theory of gases:
(a) there are intemiolecular attractions
(a) f? (b) ~3: (c) ~3: (d) all of these
(b) molecules have considerable volume 60. At constant volume, for a fixed number of mole of a gas, the
(c) there is no intermolecular attraction pressure of the gas increases with rise oftemperature due to:
(d) speed of molecules decreases for each collision (a) increase in average molecular speed
49. Postulate of kinetic theory is: (b) increased rate of collisions amongst molecules
(a) atom is indivisible '(cl' increase in molecular attraction
(b) gases combine in simple ratio (d) decrease in mean free path
( c) there is no influence of gravity on the molec'llies of the gas 61. N.on-ideal gases approach ideal behaviour:
(d) none of the above '" (lIT 1999; KCET 2004)
50. Which of the following statements is not consistent with the (a) high temperature and high pressure
postulates of kinetic theory of gases? (b) high temperature and low pressure
(a) .Gases consist of large number of tiny particles (c) low temperature and high pressure
(b) .Particles are in const~t motion (d) low temperature and low pressure
(c) All the particles have same speed 62. The nltio of root mean square speed and average speed ofa gas
(d) Pressure is due to hits recorded by particles against the molecule, at a particular temperature, is:
walls of containing vessel (a) I: 1.086 (b) 1.086: I
51. A helium atom is two times heavier than a hydrogen molecule. (c) 2: 1.086 (d) 1.086: 2
At 298 K, the average kinetic energy of a helium atom is: 63. Most probable speed, average speed and rms speed are related
(a) two times that of a hydrogen molecule as:
(b) four times that of a hydrogen molecule (a) 1 : 1.224: 1.128 (b) 1.128: 1 : 1.224
(c) half that of a hydrogen molecule (c) 1 : 1.128 : 1.224 (d) 1.224: 1.128 : 1
(d) same as that of a hydrogen molecule 64. In a closed flask of 5 litre, 1.0 g of H2 is heated from
52 .. The kinetic theory of gases predicts that total kinetic energy of 300-600 K. Which statement is not correct? (CBSE 1991)
a gas depends 9n: (a) The rate of collision increases
(a) pressure of the gas (b) The energy of gaseous molecules increases
(b) temperature of the gas (c) The number of mole of the gas increases
(c) volume of the gas (d) Pressure of the gas increases
(d) pressure, temperature and volume of the gas 65. The root mean square speed is expressed as:
]112
~T
112 [
53. If a gas is allowed to expand at constant temperature then:
(a). [ %RT ] (b)
(a) number of molecules of the gas decreases
(b) the kinetic energy ~fthe gas molecules decreases
(c) the kinetic energy of the gas molecules increases (c) [3:] (d) [3:f2
(d) the kin~tic energy of the gas molecules remains the same
66. The rms speed ofhydrogenmoJecules at room temperature is
54. Gases deviate from ideal behaviour because molecules: 2400 m S-I. At room temperature the rms speed of oxygen
(a) are colourless (b) are spherical molecules would be:
(c) attract each other (d) have high speeds (a) 400 ms- l (b) 300 ms- l
55. Deviations from ideal behaviour will be more if the gas is
(c) 600 ms- I
(d) 1600 ms- I
subjected to:, .
(a) low temperature and high pressure
(b) high temperature and low pressure /

STATES OFMATIEFl

67. The molecules of which of the following gas have highest


speed?
°
78. The value of van der Waals' constant' a' for gases 2 , Nz, NH3
and CH4 are 1.360, 1.390, 4.170 and 2.253 Iitre 2-atm mol-2
(a) Hydrogen at - 50°C (b) Methane at 298 K
(c) Nitrogen at lOOO°C (d) Oxygen at O°C respectively. The gas which can be most easily liquefied is:
68, Which one of the following is an ideal gas? (a) 02 (b) N2 (c) NH3 (d) CH4
(a) Hydrogen (b) Nitrogen 79. According to kinetic theory of gases for a diatomic molecule:
(c) Carbon dioxide (d) None ofthese . (lIT 1991)
69. van der Waals' equation explains the behaviour of: (a) the pressure exerted by the gas is proportional to the mean
square speed of the molecules
( a) ideal gases • (b) real gases
(c) mixture of gases (d) diatomic gases (b) the pressure exerted by the gas is proportional to the root
mean square speed of the molecules
70. The critical temperature of a gas is that temperature:
(c) the root mean square speed is inversely pt:0portional to the
(a) above which it can no longer remain in the gaseous state
temperature
(b) above which it cannot be liquefied by pressure
(d) the mean translational KE of the molecule is' directly
(c) at which it solidifies proportional to the absolute temperature
(d) at which the volume of the gas becomes zero 80. A real gas obeying'van der Waals' equation will resemble an
71. The van der Waals' equation for a real gas is: ideal gas if the:
(a) (p + ;2] (V b) = nRT (a) constants a and b are small
(b) a is large and b is small

(b) r\ P + an22)
V
(V - b) = nRT
.
(c) a is small and b is large
(d) constants a and b are large
81. When the universal gas c~mstant (R) is divided by Avogadro's
(e) (P+;z](V+b)=nRT number (N), their ratiojs called:
(a) Planck's constant (b) Rydberg's constant
2 (c) Boltzmann's constant (d) van der Waals' constant
(d) P = nRT n a
(V - nb) - V2 82. The compr~s~ibility factor of a gas is defined as Z = PV. The
72. In van der Waals' equation of state for a non-ideal gas the term • RT
that accounts for intermolecular force is: . compressibility factor of an ideal gas is: [PMT (MP) 2004]

(a) (p + ;2] (b) (V - b) (e) RT (d) RtT


(a) 0 (b) I
83. An ideal gas is one which obeys:
(c) -1 (d) infinity

(a) gas laws (b) Boyle's law


73. The units of 'a' in van der Waals' equation
(c) Charles' law (d) Avogadro's law

are:
rP + anV J
\
2
2
(V - nb)= nRT
84. A mixture of three gases X (density 1.0), Y (density 0.2) andZ
(density OA) is enclosed in a vessel at constant temperature.
When the equilibrium is established, the gas/gases:
(a) atom litre 2 mor z (b) atom litre mol-2
(a) X will be at the top of the vessel
(e) atom litre mol- 1 (d) atom litre 2 mort (b) Y will be at the top of the vessel
74. If 1000 mLof gas A at 600 torr and 500 mL of gas Bat 800 torr (c) Z will be at the top of the vessel I

are placed in a 2 litre flask, the final pressure will be: (d) will mix hOmogeneously throughout the vessel
(a) 2000 torr (b) 1000 torr 85. 16 g of oxygen and 3 g of hydrogen are mixed and kept at 760
(c) 500 torr (d) 400 torr mm pressure at O°e. The total volume occupied by the mixture
75. Two samples of gases A and B are at the same temperature. will be nearly: [CMC (Vellore) 1991]
The molecules of A are travelling four times faster than the (a) 22Alitre (b) 33.6litre (c) 44800 mL (d) 4480 mL
molecules of B. The ratio of m A / mB of their masses will be': 86. Which of the following expressions does not represent Boyle's
(a) 16 (b) 4 (c) 114 (d) 1116 law?
2
[Hint. mA C 2 ] (a) PV constant (b) VOC 1
2 P
ms C1
76. The root mean square speed of a certain gas at 27°C is (c) V1T2 = V21] (d) FWi = P2V2
3 x lO4 em s-!. The temperature at which the velocity will be 87. When an ideal gas undergoes unrestricted expansion, no
6x 104 cms-! is: . cooling occurs because the molecules:
(a) ~4°C (b) 108°C (c) 1200 K (d). 600 K (a) exert no attractive forces on each other'
77 .. The temperature at which real gases obey the ideal gas laws (b) do work equal to loss o( kinetic energy
over a wide range of pressure is called: mPMT 2009) (c) collide without loss of energy
(a) critical temperature (b) Boyle temperature (d) are above the inversion temperature
(c) inversion temperature (d) reduced temperature 88. Which of the following mixture of gases at room temperature
does not follow Dalton's law of partial pressures?
292 G.R.B. PHYSICAL CHEMISTRY FOR COMPETITIONS

(a) NOz and 0z (b) NH3 and HCI (a) N molecules of 02 (b) 2N molecules of02
(c) CO and COz (d) SOz and 02 N N
(c) - molecules of 02 (d) - molecules of 02
89. The speed possessed by most of the gaseous molecules is: 2 4
(a) most probable speed (b) average speed 100. A balloon filled with N20 is pricked with a sharper po~t and
(c) root mean square speed (d) none of these plunged into a tank of CO2 under the same pressure and
90. In which of the foHowing pairs the gaseous species diffuse temperature. The balloon will:
through a porous plug along with the same rate of diffusion? (a) be enlarged (b) shrink
(EAMCET 1(90) (c) collapse completely (d) remain unchanged in size
(a) NO, CO (b) NO, COz 101. The three states of matter are solid, liquid and gas. Which of
(c) NH3 • PH 3 (d) NO, Czl\; the following statements is true about them?
91. Which of the following statements is correct? (a) Gases and liquids have viscosity as a common property
(a) 3PV = mnc z is the expression for real gases (b) The molecules in all the three states possess random
(b) At normal temperature and pressure most gases behave translational motion
nearly as ideal gases (c) Gases cannot be converted into solids without passing
(c) The molecules of real gases have both volume and mutual through the liquid phase
attraction (d) Solids and liquids have pressure as a common property
(d) Pressure depends on number of molecules and volume 102. Strong intermolecular forces exist in;
92. For one gram molecule of an ideal gas: (a) gases (b) liquids
PV 3 . PV (c) amorphous solids (d) crystalline solids
(a) =-cal. (b) -=2cal 103. Association of molecules in water is due to: .
T 2 T
PV PV (a) covalent bonding (b) hydrogen bonding
(c) -:::: 8.31cal (d) -:::: 0.0821cal (c) ionic bonding (d) van der Waals' forces'
T T
104. Which of the following statements is wrong?
93. When helium is allowed to expand into vacuum, heating effect
is observed. This is due to the fact that: . (a) Evaporation is a spontaneous process
(a) helium is an inert gas (b) Evaporation is a surface phenomenon
(b) heHum is a noble gas (c) Vapour pressure decreases with increase of temperature
(c) helium is an ideal gas (d) The vapour pressure of a solution is always less than the
(d) the inversion temperature of helium is very low vapour pressure of a pure solvent
105. Normal boiling point of a liquid is that temperature at which
94. At STP,. the order of mean square velocity of molecules of
H2, N2, O2 and HBr is: (CBSE 1991) vapour pressure oJ 1'be liquid is equal to:

(a) H2 > N2 > 02 > HBr °


(b) HBr> 2 > N2 > H2
(a) zero...
(c) 760 minofHg
(b) 380 mm ofHg
(d) 100 mmofHg
(c) HBr> H2 > 02 > N2 (d) N z >02 > H2 > HBr 106. Water boils at lower temperature on high attitudes because:
95: At constant temperature in a given mass of an ideal gas: (a) atmospheric pressure is low there
(eBSE 1991) (b) atmospheric pressure is high there
(a) the ratio of pressure and volume always remains constant (c) water is weakly hydrogen bonded there
(b) volume always remains constant (d) water in pure form is found there
(c) pressure always remains constant 107. When a student was given a viscometer, the liquid was sucked
(d) the product of pressure and volume always remains with difficulty; the liquid may be:
constant (a) benzene (b) toluene (c) water (d) glycerine
96. The non-compressible voiume of a gas is ... times the actual 108. Mark the statement which is correct:
volume of gas molecules:
(a) surface tension of a liquid increases with temperature
(a) 2 (b) 4 (c) 3 (d) 2.5
(b) addition of chemicals reduces the surface tension of a liquid
97. One mole .of an ideal monoatomic gas is mixed with 1 mole of (c) stalagmometer is used for measuring viscosity of the
an ideal diatomic gas. The molar specific heat of the,mixture at liquid
constant volume is:
(d) viscosity of the liquid does not depend on intermolecular
(a) 3 cal (b) 4 cal (c) 5 cal (d) 8 cal
forces
In an experiment during the analysis of a carbon compound, 109. With the increasing molecular mass of a liquid, the viscosity:
145 mL of H2 was co\lected at 760 mm Hg pressure and 27°C. .
(Jiwaji 1990)
The mass ofH2 is nearly:
(a) decreases (b) increases
(a) 10 mg (b) 6 g (c) 12 mg (d) 12 g
. w w (c) no effect (d) all are wrong
. [Hint: Apply PV ::: M RT; 1 x 0.145:::"2 x 0.0821 x 300] 110. The viscosity of which liquid is the maximum?
(a) water (b) glycol
99. The kinetic energy of N molecules of 02 is x joule at 123°C.
(c) acetone (d) ethanol
Another sample of O2 at 27°C has a kinetic energy of 2x joule.
The latter sample contains:

, ,,-.
I
STATES OF MATTER 293
111. The rise of a liquid in a capillary tube is due to: 120. A match box exhibits: (PET (MP) 19931
(a) viscosity (b) osmosis (a) cubic geometry (b) monoclinic geometry
(c) diffusion (d) surface tension (c) orthorhombic geometry (d) tetragonal geometry
112. With increase in temperature, the fluidity of liquids: 121. In the crystal of CsCI, the nearest neighbours of each Cs ion
(a) increa~s (b) decreases are: (PET (MP) 19931
(c) remains constant (d) may increase or decrease (a) six chloride ions (b) eight Cs Ions
113. If 111 and 112 are the coefficients of viscosity of two liquids, (c) six Cs ions .. (d) eight chloride ions
d l and d z their densities and tl and t z the flow times in 122. ,How many chloride ions are tberearound sodium ion in
Ostwald viscometer, then: sodium chloride crystal?
(a):!lL dl t 2 (b) .!!L=d2t z 003 008 W4 006
11z dzt l . 112 d 1tl 123. The number of atoms contained in one face-centred cubic unit
cell of monoatomic substance is:
(c) :!lL = dltl (d) :!lL = 1Pl\lT (Vellorc) 2006; PET (MP) 20071
'1z d zt 2 '12 d 1t2
(a) 1 (b) 2 (c) 4 (d) 3
114. Which of the following expressions regarding the unit of 124. The number of close neighbours in a body-centred cubic
coefficient of viscosity is not true? lattice of identical spheres is:
(a) dynecm-2 sec (b) dynecm2 sec-I
(a) 4 (b) 12 (c) 6 (d) 8
2
(c) Nm- sec =
(d) 1 poise 10- 1 Nm- 2 sec 125. The coordination number of each sphere in (hcc) arrangement
115. The boiling points of water, ethyl alcohol and diethyl ether are is:
100°C, 78.SoC and 34.6°C respectively. The intermolecular ' (a) 8 (b) 12 (c) 6 (d) 4
forces will be in the order of: 126. Bragg's law is given by the equation: (BHU 1990)
, (a) water> ethyl alcohol> diethyl ether (a) nA=28sin8 (b) nA=2dsin8
(b) ethyl alcohol> water> diethyl ether
(c) diethyl ether> et!,lyl alcohol> water (c) 2nA = d sin 8 (d) n 8 = ~ sin 8
2 2
(d) diethyl ether> water> ethyl alcohol 127. In Bragg's equation for diffraction of X-rays, 'n'represents:
116. The unit cell in a body centered cubic lattice is given in the (a) the number of mole (b) quantum number
figure. Each sphere has a radius, r and the cube has a side, a. (c) the order of reflection (d) Avogadro's number
What fraction of the total cube volume is empty.(T1FR 2010) 128. The coordination number for an atom in a primitive cubic unit
cell is:
(a) 6 (b) 8 (c) 10 (d) 12
129. The number of atoms per unit cell in a simple cubic,
face-centred cubic and body-centred cubic are . .
respectively:
(a) 1,4,2 (b) 4, 1,2
(c) 2,4, 1 (d) 4,8,2
130. In a solid lattice, the cation has left a lattice site and is located
at interstitial position, the lattice defect is: (VITEEE 2007)
(a) interstitial defect (b) vacarlcy defect
8 ? 4 ? (c) Frenkel defect (d) Schottky defect
(a) i--ft- (b) -3·ft-a3 131. Schottky defects occur mainly in ionic compounds where :
3 a3
(VITEEE 2008)
4 ?
(c) ~ (d) 2-3'ft a3 (a) positive and negative ions are of different size ,
(b) positive and negative ions are of same size
117. Which one is not the property of crystalline solid? (c) positive ions are small and negative ions are big
(a) . Isotropic (d) positive ions are big and negative ions are small
(b) Sharp melting point 132. What type of crystal defect is indicated in the diagram below?
(c) A definite and regular geometry Na+ Cl- Na+ cr Na+ Cl-
(d) High intermolecular forces Cl- Cl- Na+ Na+
118. The number of crystal systems known is: Na+ Cl- cr Na+ CI-
007 008 W6 004 Cl- Na+ Cl- Na+ Na+ (AIEEE 2004)
119. Tetragonal crystal system has the following unit cell
(a) Frenkel defect (b) Schottky defect
dimensions: (PMT (MP) 1993\
(c) Interstitial defect (d) Frenkel and Schottky defects
(a) a = b = c and a = ~ y = 90° = 133. In a closed packed array of N spheres, the number of
(b) a=b*canda=~=y=90°
tetrahedral holes is:
(c) a * b * cand a = ~ y 90° = = (a) NI2 (b) N
(d) a = b * c and a = ~ 900 , Y = 120°
(c) 4N (d) 2N
I
294 G.R.B. PHYSICAL CHEMISTRY FOR COMPETITIONS

134. In a closed packed array of N spheres, the octahedral holes are: (a) They are anisotropic
(a) NI2 (b) 2N (c) 4N (d) N (b) They are rigid and incompressible
135. AB crystallizes in a body centred cubic lattice with edge (c) They melt over a wide range of temperature
length ~ a' equal to 387 pm. The distance between two (d) There is no orderly arrangement of particles
oppositely charged ions in the lattice is: [CBSE (PMT) 20101 145. Which defect causes decrease in the density of a crystal?
(a) 335 pm (b) 250 pm (a) Frenkel (b) Schottky
(c) 200 pm (d) 300 pm (c) Interstitial (d) F -centre
[Hint: Distance between two oppositely charged ions r;
146. For tetrahedral coordination number, the radius ratio l(;+ I is:
(r+ + r-) = aJ3 = 387xJ3 (KCET 2008)
2 2 (a) 0.732- 1.0 (b) 0.225:'" 0.414
335.14 pm] (c) 0.414 - 0.732 (d) 0.155 0.225
136. Germanium or silicon becomes semiconductor due to: 147. What is the total number of ions present in one unit cell of
. (a) Schottky defect (b) chemical impurity . sodium chloride lattice? (EAMCET 2006)
(c) Frenkel defect (d) none of these (a) 2 (b) 6 (c) 12 (d) 8
137. A particular solid is very hard and has a high melting point. In 148. The formula for determination of density of cubic unit cell is:
solid state it is a non conductor and its melt is a conductor of a3 N 0 - 3 (b) Z x No -3
()
a --gcm ---gcm
electricity. Classify the solid: [CMC Vellore (Med.) 2008) ZxM M xa 3

(a) metallic (b) molecular


a3 x M _3 (d) Z x M -3
(c) network (d) ionic (C) ---gcm . 3 gcm
(e) amorphous
ZxNo Noxa
.. 138. Which one has the highest melting point? 149. The closest-packing sequence ABAB ..... represents:
(a) Ionic crystal (b) Molecular crystal (a) primitive cubic packing
(c) Covalent crys~l (d) Metallic crystal (b) body-centred cubic packing
139. For an ionic crystal of general formula AX and coordination (c) face-centred cubic packing
number 6, the value of radius ratio will be: [PMT (MP) 1993) (d) hexagonal packing
(a) greater than 0.73 (b) in between 0.73 and 0.41 150. The closest-packing sequence ABC ABC . .... represents:
(c) in between 0.41 and 0.22 (d) less than 0.22 (a) primitive cubic packing
140. The Ca 2+ and F - are located in CaF2 crystal respectively at (b) body-centred cubic packing
face-centred cubic lattice points and in: (AIIMS 2006) (c) face-centred cubic packing
(d) hexagonal packing
(a) tetrahedral voids (b) halfoftetrahedral voids
1St: The edge length of face centred cubic ccll of an ionic
(c) octahedral voids (d) half of octahedral voids
substance is 508 pm. If the radius of cation is 110 pm, the
141. In calcium fluoride structure, the coordination numbers of . radius of anion is: (AIEEE 2010)
calcium and fluoride ious are:
(a) 618 pm (b) 144 pm
(a) 8 and 4 (b) 6 and 8
(c) 288 pm (d) 398 pm
(c) 4 and 4 (d) 4 ;md 8
[Hint: Edge length =2 (r+ + r-)
142. The unit cell of a binary compound of A and B metals has a
ccp structure with A atoms occupying the comers and B atoms 508 = 2(110+ n
occupyi~g the centres of each faces of the cubic unit cell. If r- 144 pm]
during the crystallisation of this alloy, in the unit cell two A 152. Close packing is maximum in the crystal lattice of:
atoms are missed, the overall composition per unit Cell is: (a) face-centred cubic (b) body-centred cubic
• [C,ET (J&K) 2009) (c) simple-centred cubic (d) none 'of these
(a) AB6 (b) AB4 (c) ABs (d) A6B24 153. The ratio between the root mean square speed of H2 at 50 K
. I 3
[Hint: Number of atoms of A = 6 x - = - and that of 02 at 800 K is: (lIT 1996)
8 4 (a) 4 (b) 2 (c) I (d) 1/4
Number of atoms of B =6 x.!. =3 154. X mL of H2 gas effused through a hole in a container in 5
2 second. The time taken for the effusion of the same volume of
A:B=i: 3 =1:4 the gas specified below under identical conditions is:
4 (lIT 1996)
.. Composition of alloy:::: AB4 ] (a) 10 second: He (b) 20 second: O 2
143. In a solid lattice, the cation has left a lattice site and is located (c) 25 second: CO (d) 55 second: CO 2
at an interstitial position. The lattice defect is : 155. .The energy of a gas per litre is 300 joule. Its pressure will be:
[BHU (screening) 2008) (a),3xltfN/m 2 (b) 6xlifN/m2
(a) interstitial defect (b) vacancy defect (c) I05N/m 2 (d) 2xlOSN/m2
(c) Frenkel defect (d) Schottky defect 156. If the universal gas constant is 83 joule morl K-1 and the
144. Which of the following statements is incorrect about· Avogadro's number is 6 x I if3. The mean kinetic energy of the
amorphous solids? (KCET 2004) oxygen molecules at 327°C will be:
STATESOF MAHER 295

(a) 415xlO-23 joule (b) 2490 X 10-22 joule (a) 234.2 ml s (b) 342.2 ml s
(c) 1245 x 10-23 joule (d) 830 x 10-22 joule . (c) 432.2 ml s (d) 243.2 ml s
165. At a temperature T K, the pressure of 4 g argon in a bulb is P.
157. If increase in temperature and volume of an ideal gas is two
The bulb is put in a bath having a temperature higher by SO K
time~, then initial pressure of P changes to:
than the first one. 0.8 g of argon gas had to be removed to
[JEE (Orissa) 2006) maintain original pressure. The temperature T is equal to:
(a) 4P (b) 2P (c) P (d) 3P (lIT 1999)
158. The average kinetic energy of one molecule ofan ideal gas at (a) 510 K . (b) 200 K
27°C and I atm pressure is: (EAMCET 2009)
(c) 100 K (d) 73 K
(a) 900 cal K- I mol-I 166. ,At 2SoC and 730 mm pressure, 380 mL of dry oxygen was
(b) 6.21x 10-21 J rl molecule-I collected. lethe temperature is constant, what volume will the
(c) 336.7 J K- I molecule-I oxygen occupy at 760 mm pressure? (CBSE 1999)
(d) 3741.3 J K- I mol-I (a) 265 mL (b) 365 mL (c) 569 mL (d) 621mL
167. Which of the following solids shows electrical conduction?
[Hint: KE ~~T (CBSE 1999)
2N
(a) Graphite (b) Potassium chloride
=~x 8.314 x 300
2 6.023 x 1023 (c) Diamond (d) Sodium chloride
= 6.21x 10-21 J K- 1 molecule-I] 168. Schottky defect in solids is due to:
(PMT (Vcllorc) 2006; VITEEE 20061
159. The respective speeds of five molecules are 2, I.S, 1.6, 1.6 and
(a) a pair of cation and anion vacancies
1.2 km/sec. The most probable speed in km/sec will be: .
(b) occupation of interstitial site by a pair of cation and anion
(a) 2 (b) 1.58 (c) 1.6 (d) 1.31
(c) occupation of interstitial site by a cation
160. if one mole of a monoatomic gas ( y = ~) is mixed with one (d) occupation of interstitial site by an anion'
169. Copper crystallises in face-centred cubic lattice with a unit cell
mole of diatomic gas ( y ~). the value ~f y for the mixture is: . length of 361 pm. What is the radius of copper atom in pm?
[CBSE (PMT) 2009; AIEEE 2009)
(a) 1.40 (b) 1.50 (c) 1.53 (d) 3.07 (a) 108 (b) 128 (c) 157 (d) 181
161. The root mean square speed of hydrogen molecule of [Hint: a..fi = 4r
an ideal hydrogen gas kept in a gas chamber at (PC 361x..fi =4r
is 3180 metre/second. The pressure on the hydrogen gas-
r= 128 pm] ~
is: (density of hydrogen gas is 8.99 x 10-'2 kg 1m3 ,
J 70. The second OTder Bragg diffraction Qf X~rays with X = I A
1 atmosphere =-1.01 x lOS N 1m2) from a set of parallel planes in a metal oc<;!lfS at an angle of '
(a) 1.0 atmosphere (b) I.S atmosphere 60°. The distance between the scattering planes in the crystal
(c) 2.0 atmosphere (d) 3.0 atmosphere is: (CBSE 1998)
162. CH4 diffuses two times faster than a gas X. The number _of (a) 0.575 A (b) l.OOA
molecules present in 32 g of gas X is: (N is Avogadro number) (c) 2.00 A (d) 1.15 A
171. According to Graham's law at a given temperature, the ratio of
IEAMCET (Engg.) 2010)
the rates of diffusion rA I 'B of gases A and B is given ,?y: '
(a) N (b) N (c)N (d) N (JIT~998)

-a!
2 4 16 1/2 I12
(a) PA • ,MA (b)' MA . PA
[Hint: rCH 4
PB ( MB )
MB (PB J
'gas mCH4

2= rm; (C)PA:M B )
112
(d) MA .(PPB J
1/2
,
V16 PB ( MA MB A \
~=64 172. The root mean square~elocity of an ideal gas at constant
Number of molecules =__w_ _ x N pressure varies with density as: [IJT(Scrcening) 20(0)
molar mass
32 N
(a) 2 d (b) d (c) ..Jd (d) 1I..Jd
= 64 xN 2"] 173. Match the following:
_List~I List-II
163. At what temperature would the rillS speed of a gas molecule
(A) Inversion temperature (i)·a/ Rb
have twice its value at 100°C?
(B) Boyle's temperature (ii) 8a /27Rb
(a) 4192 K (b) 1492 K
(c) 9142 K (d) 24(}1 K (C) Critical temperature (iii) 2a / Rb
164. Find the rms speed of an argon molecule at 27°C (Molecular (a) A-(i), B-{ii), C-(iii) (b) A--(iii), B-{ii), C--{i)
weight of argon = 40 gm I mol): (c) A-(iii), B-{i), C-(ii) (d) A-(i), 8--(iii), C-(ii)
2961 G. A.B. PHYSICAL CHEMISTRY FOR COMPETITIONS

174. For three different gases values of van der Waals' constant 'a' and (d) Cannot· be predicted unless the volumes of the gases are
'b' are given. What is the correct order of liquefaction of gases? given
Gases a b 183. Which of the following exhibits the weakest intermolecular
X2 1.3 0.090 forces? (AIIMS 2000)
Yz 4.1 0.023 (a) NH) (b) HCI (c) He (d) H20
22 2.2 0.075 184. The compressibility of a gas is less than unity at STP.
Therefore: [lIT 2000; PMT (MP) 2004]
(a) XZ>Yl>Z:! (b) YZ >22 >X 2
(a) Vm > 22.4 litre (b) Vm < 22.41itre
. (c) Z:! > Y2 > X z (d) Xl> 22 > Y2
(c) Vm == 22.4 litre (d) Vm 44.8 litre
175. Match the following:
List-I List-II 185. The rms velocity of hydrogen is J7 times the rms velocity of
(A) Pc (i) 3b nitrogen. 1fT is the temperature of the gas: (liT 2000)
(B) II;, (ii) 8al27bR (a) T(H z ) T(N2 ) (b) T(H2 ) > T(N z )
(C) Tc (iii) a I 27b 2 (c) T(Hz)<T(Nz ) (d) T(H z )=J7T(Nz )

(a) A-{i), B-{ii), C-{iii) (b) A-{iii), B-{ii), C-{i) 186. The kinetic energy of any gas molecule at O°C is:
(a) zero (b) 3408 J
(c) A-{ii), B-{iii), C-{i) (d) A-{iii), B-{i), C-{ii)
176. By what factor does the average velocity of a gaseous (c) 2 cal (d) 5.66 x 10-21 I
molecule increases when the absolute temperature is doubled? 187. Densities of two gases are in the ratio 1 : 2 and their
(DUMET 2010) temperatures are in the ratio 2 : 1; then the ratio of their
(a) 1.4 (b) 2 (c) 2.8 (d) 4.0 respective pressures is: (BHU 2000)
177. The van der Waals' constant' a' for different gases have been (a) I : I (b) 1 : 2 (c) 2: I (d) 4: 1
given as : 188. Gas equation PV == nRT is obeyed by: (BHU 2000)
Gas a(atm L2 mol-2) (a) only isothermal process (b) only adiabatic process
02 1.36 (c) both (a) and (b) (d) none of these
N2 1.39 189. An ideal gas will have maximum density when: (CPMT 2000)
CH4 2.25 (a) P=0.5atm,T=600K (b) P=2atm,T 150K
NH3 4.17 (c) P = I atm, T = 300K (d) P = LOatm, T == 500K
The gas that can be most easily liquefied is : 190. The following graph illustrates: (JIPMER 2000)
[JEE (Orissa) ZOIO[
(a) 02 (b) N2 (d) NH3
178. Match the following:
List-I List-II
(A) Urms /[jav (i) 1.22
(B) Uav IU nip (ii) 1.13
(C) UrmslUmp (iii) 1.08
(a) A-{iii), B-{ii), C-{i) (b) A-{i), B-{ii), C-{iii) Temp. (O"C) -
(c) A-{iii), B-{i), C-{ii) (d) A-{ii), B-{iii), C-{i)
179. louie-Thomson coefficient is zero at: (a) Dalton's law (b) Charles'law
(a) critical temperature (b) inversion temperature (c) Boyle's law (d) Gay-Lussae's law
(c) absolute temperature (d) Boyle's temperature
191. 4.4 g of a gas at STP occupies a volume of2.24 L. The gas can
180. At a constant pressure, what should be the percentage increase
be: [CET (Haryaua) 2000)
in temperature for a 10% increase in volume?
(a) 02 (b) CO (c) NO z (d) CO2
(a) 10010 (b) 11% (c) 8.9%' (d) 12.5%
181. 5 g of unknown gas has pressure P at a temperature T K in a 192. At O°C and one atm pressure a gas occupies 100 cc. If the
vessel. On increasing the temperature by 50'C, I g of the gas pressure is increased to one and a half time and temperature is
was given out to maintain the pressure P. The original increased by one-third of absolute temperature, then final
temperature was: volume of the gas will be: (DCE 2000)
(a) 73 K (b) 100 K (a) 80 cc (b) 88.9 cc (e) 66.7 cc (d) 100 cc
(c) 200 K (d) none ofthese 193. Pressure of a mixture of 4 g of02 and 2 g ofH2 confined in a
182. Which of the following expressions correctly represents the bulb of 1 litre at O°C is: (AIIMS 2000)
relationship between the average molar kinetic energy, KE of (a) 25.215atm (b) 31.205atm
CO and N z molecules at the same temperature? (e) 45.215 atm (d) 15.210 atm
[CBSE (PMT) 2000) 194. Density ratio of O2 and H2 is 16: L The ratio of their rms
(a) KEco == KEN 2 velocities will be: (AHNIS 2000)
(b) KEco > KEN z (a) 4: I (b) I : 16 (c) 1 : 4 (d) 16: 1
- --
(c) KEco < KENz
STATES OF MATTER 297
195. The rate of diffusion of a gas having molecular weight just 203. A crystal lattice with alternate +ve and -ve ions has radius
double of nitrogen gas is 56 mL S-I, The rate of diffusion of ratio 0.524, its coordination number is:
nitrogen will be: (CPMT 2000) IPMT (Manipal) 2002]
(a) 79.19 mL S-1 (b) 112.0 m'L s-l (a) 4 (b) 3 (c) 6 (d) 12
(c) 56 mL S-1 (d) 90.0 mL S-1 204. The major binding force of diamond, silicon and quartz is:
196. The density of air is 0.00130 glmL. The vapour density of air [MEE (Kcrala) 2002]
will be: (DCE 2000) (a) electrostatic force (b) electrical attraction
(a) 0.00065 (b) 0.65 (c) 14.4816 (d) 14.56 (c) covalent bond force (d) non-covalent bond force
197. If 300 mL of a gas at 27°C is cooled to 7°C at constant (e) van der Waals' force
pressure, its final volume will be: (AIIMS 2000) 205. A compound is formed by elements A and B. This crystallizes
(a) 135 mL (b) 540 mL (c) 350 mL (d) 280 mL in the cubic structure when atoms A are at the corners of the
198. For an ideal gas, number of moles per litre in terms of its cube and atoms B are at the centre of the body. The simplest
pressure P, gas constant R and temperature Tis: formula of the compound is: (DPMT 2009)
(AIEEE 2002) (a) AB (b) AB2 (c) A2B (d) AB4
(a) PTfR (b) PRT (c) PfRT (d) RTfP 206 .. The coordination number of a metal crystallizing in a
199. The van der Waals' equation reduces itself to the ideal. gas hexagonal close packed (hcp) structure is: [PET (MP) 20041
equation at: [MEE (Kerala) 2001; CBSE (PMT) 20021 (a) 6 (b) 12 (c) 8 (d) 4
(a) high pressure and low temperature 207. The crystal system of a compound with unit cell dimensions
(b) low pressure and low temperature a = 0.387, b = 0.387, c = 0.504 nm and a = ~ = 900 and
(c) low pressure and high temperature
y = 1200 is: (AIIMS 2004)
(d) high pressure alone (a) cubic (b) hexagonal
200. Which of the following volume (V), temperature (T) plots (c) orthorhombic (d) rhombohedral
represents the behaviour of an ideal gas at one atmospheric 208. The maximum number of molecules is present in;
pressure? (lIT (Screening) 2002; PET (Kerala) 2007] [CBSE (PMn 2004]
(a) 15 L ofH2 gas at STP (b) 5 L ofN2 gas at STP
(c) 0.5 g ofH2 gas (d) 10 g of O2 gas

(22.4 L
at 273 K)
~36373 at
8L
K)
(22.4 L
at 27~ K)
~28373 at
6L
K)
209: The root mean square velocity of one mole of a monoatomic
gas having molar mass M is Urms ' The relation between
average kinetic energy (E) of the gas andUrms is:
t . [lIT (S) 2004)
Vfl) I • Vel)
T(K)- T(K)-
(a) (b) (b)Urms ~:!
(22.4 L (22.4 L
(d)U
rms
= ~ 3M
E
at 273 K) at 273 K) 210. Which of the following is not a property of liquid state?
t t (14.2 L
VeL) .........._ - _ Vel) "--_a_t3_7_3_K) IBHU (Pre) 20051
(a) Intermolecular force of attraction in a liquid is quite large
T(K)- T(K)-
(b) All liquids accompanied by cooling on evaporation
(c) (d) (c) Lower the boiling point ·of a liquid, greater is its vapour
pressure at room temperature
(22.4L (d) A liquid boils at higher temperature at the top of a
at 273K) mountain than at the sea level
Vel) 211. A certain sample of a gas has a volume of 0.2 litre measured at
1 atm pressure and O°C. At the same pressure but 273°C, its
volume will be: [BHU (Pre) 2005]
T(K)- (a) 0.4 L (b) 0.8 L (c) 27.8 L (d) 55,6 L
(e) 212. When electrons are trapped into the crystal in an'ion vacancy,
the defect is known as: [BHU (Pre) 20051
201. The following is a method to determine the surface tension of (a) Schottky defect (b) Frenkel defect
liquids: feET (J&K) 2007]
(c) stoichiometric defect (d) F-centres
Ca) single capillary method (b) refractometric method 213. In'the equation of state of an ideal gas PV = nRT, the valueof
(c) polarimetric method Cd) boiling point method the universal gas constant R would depend only on the:
202. The crystalline structure ofNaCI is: rCET (J&K) 20071 (KCET 2005)
(a) hexagonal close packing (b) face-centred cubic (a) nature of the gas (b) pressure of the gas
(c) square planar (d) body-centred cubic (c) units ofthe measurement (d) none of these
298 G.R.B. PHYSICAL CHEMISTRY FOR COMPETITIONS

214. If Z is the number of atoms in the unit cell that represents the (a) remain unchanged (b) be doubled
closest packing sequence ABC ABC ... , the number of (c) increase fourfold (d) be halved
tetrahedral voids in the unit cell is equal to: (AIIMS 2005) (e) be reduced to one-fourth
(a) Z (b) 2Z (c) ~ (d) Z [Hint: Use ~Vl = ~Vz ]
2 4 T, Tz
215. Equation of Boyle's law is: (I)PMT 2005) 223. To what temperature must a neon gas sample be heated to
(a) dP = _ dV (b) dP =+ dV double the pressure, if the initial volume of a gas at 7SOe is
P V .p V decreased by 15%? (PET.(Kerala) 20051
(c) d 2p = dV (d) d 2P = + d 2V (a) 319°e (b) 592°e (c) l28°e (d) 60 0 e
P dT P dt (e) 90 e
0

[Hint: . PV =constant [Hint: ~V! = ~V2


PdV + V dP=O T, Tz
dP =_ dV] PV 2Px 85V
P V 348 = Tz x 100
216. In a face-centred cubic unit cell, edge length is: T2 = 5916 K
(DPMT 2005) tz = 31&6°C]

(a) ~r (b) ~r (c) 2r (d) ~r 224. The surface tension of which of the following liquids is
maximum? ICBSE (Pre) 2005)
217. ~fvrms is 30R I12 at 27°e, then calculate the molar mass of the (a) e 2 Hs0H (b) CH 30H (c) H 2 0 (d) C6H6
gas in kilogram: (DPMT 2005) [Hint: (c) Due to hydrogen bonding, H 20 has the highest
(a) I (b) 2 (c) 4 (d) 0.001 intermolecular force, hence its surface tension is also
maximum.]
[Hint: v rrns =~3~T 225. An element(with atomic mass = 250 g) crystallises in a simple
cube. If the density of unit cell is 7.2 g cm -3 ,what is the radius
30Rl12 = ~3 x RmX 300 ofthe·element? [JEE (Orissa) 2006)
(a) 1.93 x 10-6 em (b) 1.93 x 10-8 em
m =O.OOlkg]
(c) 1.93 x 10-8 A (d) 1.93 x 10-8 m
218. The pressure exerted bx,.1 mole of methane in a 0.25 litre
container at 300 K using van der Waals' equation (given [Hint: Z=13xpXNA
=
, a 2.253 atm L2 mol-z, b = 0.0428 L mol-I) is: M
. (JEE (Orissa) 2005) 1= x 7.2 x 6.023 x 1023
[3
(a) 82.82 atm (b) 152.51 atm 250
(c) 190.52 atm (d) 70.52 atm I =3.86 X 10-8 em·
219. An ionic compound has a unit cell consisting of' A' ions at the 1= 2r (for simple cubic unit cell)
corners of a cube and 'B' ions on the centres of the faces ofthe
I 3.86 X 10-8
cube. The empirical formula for this compound would be: r =- = 1.93 X 10-8 em]
IAIEEE 2005; CET (Karnataka) 20091 2 2
226. Dominance of strong repulsive forces among the molecules of
(a) AB (b) AzB
the gas (Z = compressibility factor): (AIIMS 2006)
(c) AB3 (d) A3B
(a) depends on Z and indicated by Z '" 1
220. Which one of the following statements is not true about the
effect of an increase in temperature on the distribution of (b) depends on Z and indicated by Z > I
molecular speeds in a gas? (c) depends on Z and indicated by Z < I
(a) The most probable speed increases (d). is independent of Z
(b) The fraction of the molecules with most probable speed [Hint: When Z> I, the gas is less compressible than ideal
increases gas and the repulsive forces dominate.]
(c) -The distribution becomes broader 227. Total volume of atoms present in a face-centred cubic unit cell
(d) The area under the distribution curve remains the same as of a metal is (r is atomic radius): (AIEEE 2006)
under the lower temperature 20 3 24 12 16
(a) - 1tr (b) - 1t? ('_ (c) - 1t? . (d) - 1t?
221. A gas can be liquefied: (AFMC 2005) 3· 0 3 3 3
(a) above its critical temperature 228. If we know the ionic radius ratio in a crystal of ionic solid,
(b) at its critical temperature what can be known of the following? (CET (Gujarat) 20061
(c) below its critical temperature (a) Magnetic property
(d) at any temperature (b) Nature of chemical bond
222. If the absolute temperature of a gas is doubled and the pressure (c) Type of defect
is reduced to one-half, the volume ofthe gas will: (d) Geometrical shape of crystal
(PET (Kerala) 2005]
STATES OF MATTER

229. If a volume containing gas is compressed to half, how many I .J3


(a)-a:- a:-a
.J2
(b) la:.J3 a :.J2a
moles of gas remained in the vessel? (nCE 2006) 2 2 2
(a) Just double (b) Just half
1 .J3
(c)-a:-a:--a
1,
(d)
1 r: 1
a:",3a: r:a
(c) Same (d) More than double
230. The volume-temperature'graphs of a given mass of an ideal 2 4 2.J2 2 ",2
gas at constant pressures are shown below. What is the correct [Hint: Simple unit cell, r=al2
order of pressures? (EAMCET 2006; AIIMS 2008)
Body centred unit cell, r= aJ3
4
Face centred unit cell, r = a~}
, 2",2
239. The term that corrects for the attractive forces present in a real '
gas in the van der Waals' equation is: (lIT 2009)
z 2
OK 273K (a) nb (b) ~ (c) -an (d) -nb
T-- VZ
240. Which is not the correct statement for ionic solids in which
(a) f\ > P3 > Pz (b) f\ > Pz > P3 positive and negative ions are held by strong electrostatic
(c) Pz > P3 > f\ (d) Pz > f\ > P3 attractive forces? (DCE 2009)
, + '
231. IfNaCI is doped with 10-4 mol % of SrCll , the concentration
(a) The ratio r_ increases as coordination number increases
of cation vacancies will be (N A = 6.023 x 1023 ): .r
(CBSE (Medical) 2007)
(b) As the difference in size of ions increases coordination
(a) 6.02 x 1016 mol- I (b) 6.02 x 1017 mol- 1 number increases +
(d) 6.02 x 1015 mol- J (c) When coordination number is eight, the !:..- ratio lies
232. Coordination number ofNaCI crystal will be: between 0.225 0.414 ' r-
(PET (MP) 20071 (d) In ionic solid of the ty~e AX ~ZnS, Wurtzite) the
(a) 8:8 (b) 6:6 (c) 4:4 (d) 2:2 coordination number of Zn + and S - respectively are 4
233. The unit cell of Al (molar mass 27 g mo)-l) has an edge and 4
length of 405 pm. Its density is 2.7 g I cm3 . The cubic unit cell 241. The packing efficiency of the two-dimensional square unit cell
is: (PET (KeraJa) 20071 shown below is: (lIT 2010)
(a) face-centred . (b) body-centred (c) primitive
(d) edge-centred (e) simple
[Hint:z
3
a x d x NA = (405 X )0-10)3 x 2.7 x 6.023 x 1023 4
. AI 27
:. It is a face-centred cubic unit cell.]
234. The 8: 8 type packing is present in: (VITEEE 2007)
(a) MgFz (b) CsCI (c) KCI (d) NaCI I I
235. Equal masses of methane and oxygen are mixed in an empty I-L--t
container at 25"C. The fraction of the total pressure exerted by
oxygen is: (AIEEE 2007; EAMCET (Med.) 20101 (a) 39.27% (b) 68.02%
1 2 I 273 1 (c) 74.05% . (d) 78.54%
(a) "2 '3
(b) '3
(c) x 298 (d) '3 [Hint: a..fi =4 r
236. Percentage of free space in a body centred cubic unit cell is : a= 2.[2 r
[CBSE (PMT) 20081
(a) 34% (b) 28% (c) 30% (d) 32% Packi ng fra ct·Ion =Occupied area x 100
Total area
237. Which of the following statement is not correct?
2n?
[CBSE (PMT) 2008] -r::-z
(2",2r)
x 100= 78.5%]
(a) The number of carbon atoms in a unit cell of diamond is 4.
(b) The number of Bravis lattices in which a crystal can be
categorized is 14. Set-2: The questions given below may have more
(c) The fraction of the total volume occupied by the atoms in a than one correct answers
primitive cell is 0.48.
(d) Molecular solids are generally volatile. ]. If force of attraction between the molecules is negligible,
238. If a stands for the edge length of the cubic systems : simple van der Waals' equation (for one mole) will become:
cubic, body centred cubic and face centred cubic, then the ( a) PV = RT + Pb (b) P _ RT a
ratio of radii of the spheres in these systems will be
-V-b -VI
respectively: [CBSE (PMT) 2008] (c) PV = RT + a I V (d) PV=RT-aIV
300 G.R.B. PHYSICAL CHEMISTRY FOR COMPETITIONS

2. van der Waals' equation for: 8. According to Charles' law:


A. High pressure. (i) PV = RT + Pb 1
(a) Vex: - (b) (dVJ == K
B. Low pressure (ii) PV = RT - V
T dT p.

r! --;J
(2/

C. Force Qf attraction is
negligible
(iii) PV = RT + (2/ V (c) (dTJ
dV
=K
p
(d)
,T T p
=0

D. Volume of molecules is
. negligible
(2 '\
j
(iv) ( P + V 2 (V - b) = RT 9. In the following statements :
(A) ideal gases are liquefied only at very low temperatures
(a) A (iv), B (ii), C (i), D (iii) (B) ideal gases can not be liquefied
(b) A (i), B (ii), C (iii), D (iv) (C) ideal gas behaviour is observed by real gases at low
(c) A (iv), B (iii), C (ii), D (i) pressures
(D) ideal gases do not exist
(d) A (iv), B (ii), C (iii), D (i)
The correct statements are: (ISAT 2010)
3. van der Waals' constants for three different gases are given:
(a)A,B,CandD (b) A, BandC
Gases (2 b (c) B, C and D (d) C and D
X 3.0 0.025 10. Which of the following relationships is/are not true?
I2RT
Y 10.0 0.030 (a) Most probable velocity ~ M
Z 6.0 0.035
Which is correct? (b)PV = ~kT
(a) Maximum critical temperature-Y
(c) Compressibility factor Z == PV
(b) Most ideal behaviour--X nRT
(c) Maximum molecular volume-Z
(d) Average kinetic energy of gas =..!. kT
(d) All are correct 2
4. What is the ratio of mean speed of an 0 3 molecule to the rrns 11. According to kinetic theory of gasi?s: (UT 1991)
speed of an 02 molecule at the same T ? (a) the pressure exerted by a gas is proportional to mean
(a) (3n 17)1I2 (b)(16J 9n )112 square velocity ofthe molecules
(c) (3nf2 (d) (4n/9)112 (b) the pressure exerted by the gas is proportional to the root
mean square velocity of the molecules

s. ::T~i: rna; ~; represon::'( ~';), K


(c) the root mean square velocity is inversely proportional to
the temperature
V (d) the mean translational KE of the molecule is directly
proportional to the absolute temperature
(c) (dP
dV
J=- ~
V 2 (d) (dP
dV
J = vK2
T
12. A gas described byvan der Waals' equation: (lIT 2008)
(a) behaves similar to an ideal gas in the limit of large molar
volume
where, K constant.
(b) behaves similar to an ideal gas in the limit of large
6. If pressure of a gas is increasedby 1% when heated by 1°C, its
pressures
initial temperature must be (if volume remains constant):
(c) is characterised by van der Waals' constant that are
(a) 100 K (b) 100°C (c) 250 K (d) 250°C
dependent on identity of the gas but are independent ofthe
7. Which of the following are not the units of gas constant, R? temperature.
(a) dyne K- 1 mol- l (b) erg deg- J mor l (d) has the pressure tha~ is lower than the pressure exerted by
(c) cm3 K- l mol- l (d) kPa dm 3 K- 1 mol-I the same behaving ideally.

Assertion-Reason TYPE QUESTIONS


(d) If (A) is incorrect, but (R) is correct.
Set-1 1. (A) The heat absorbed during the isothermal expansion of an
The questions given below consist of an 'Assertion' (A) and the ideal gas against vacuum is zero.
'Reason' (R). Use the following keys to choose the appropriate (R) The volume occupied by the molecules of an ideal gas is
answer:
zero~ (liT 2000)
(a) If both (A) and (R) are correct, and (R) is the correct
2. (A) The pressure of a fixed amount of an ideal gas is
explanation of (A).
proportional to its temperature .
. (b) If both (A) and (R) are correct, but (R) is not the correct
(R) Frequency of collisions and their impact both increase in
explanation of (A).
proportion of the square root of temperature. (liT 2000)
(c) If (A) is correct, but (R) is incorrect.
STATES OF MATTER 1-361~
3. (A) Cp Cv R for an ideal gas. 16. (A) Under similar conditions of temperature and pressure, 02
(R) (dE)
dV T
0 for an ideal gas.
diffuses 1.4 times faster than S02'
(R) Density ofS02 is 1.4 times greater than that of02.
.17. (A) On compressing a gas to half the volume, the number of
4. (A) A lighter gas diffuses more rapidly than a heavier gas.
moles is halved.
(R) At a given temperature, the rate of diffusion of a gas is
(R) The number of moles present decreases with decrease in
inversely proportional to the square root of its density.
volume.
5. (A) On cooling, the brown colour of nitrogen dioxide
18. (A) The plot of volume (V) versus pressure (P) at constant
disappears.
temperature is a hyperbola in the first quadrant.
(R) On cooling, N02 undergoes dimerisation resulting in the
(R) V oc II P at constant temperature.
pairing of the odd electrons in N02.
19. (A) At constant temperature, if pressure on the gas is doubled,
6. (A) The value of the van der Waals' constant' a' is larger for
density is also doubled.
ammonia than for nitrogen.
(R) At constant temperature, molecular mass of a gas is
(R) Hydrogen bonding is present in ammonia.
directly proportional to the density and inversely
7. (A) The Joule-Thomson coefficient for the ideal gas is zero.
proportional to the pressure.
(R) There are no intermolecular attractive forces in an'ideal gas.
20. (A) IfH2 and Cl z enclosed separately in the same vessel exert
S. (A) At 27°C, the kinetic energy of 8 gram of methane is equal
pressures of 100 and 200 mm respectively, their mixture in
to the kinetic energy of 16 gram of oxygen.
the same vessel at the same temperature will exert a
(R) The total heat change in a reaction is the same whether the
chemical reaction takes place in one single step or in pressure of 300 mm.
several steps. . (EAMCET 2006) (R) Dalton's law of partial pressures states that total pressure
9. (A) A closed cylinder containing high pressure gas tends to . is the sum of partial pressures.
rise against gravity when the gas is allowed to escape 21. (A) Most probable velocity is the velocity possessed by
through an orifice at the bottom. maximum fraction of molecules at the same temperature.
(R) The velocity of escaping gas develops an upward thrust (R) On collision, more and more molecules acquire higher
proportional to the area of cross-section of the orifice. speed at the same temperature.
(SCRA2007) 22. (A) Compressibility factor (Z) for non-ideal gases is always
[Hint: Assertion is correct but reason is wrong as the velocity of greater than I.
escaping gas is inversely proportional to the area of cross- section (R) Non-ideal gases always exert higher pressure than
of orifice.] expected.
10. (A) Meniscus of a liquid disappears at the critical temperature. 23. (A) van der Waals' equation is applicable only to non-ideal
gases.
(R) Density of liquid and its gaseous phase become equal at
(R) Ideal gases obey the equation PV = nRT.
the critical temperature. [SHU (Screening) 200S)
24. (A) Helium shows only positive deviations from ideal
11. (A) Graphite is soft while diamond is hard.
behaviour.
(R) Graphite has three dimensional structure while diamond (R) Helium is an inert gas.
has planar. [SHU (Screening) 20081 25. (A) Gases are easily absorbed on the surface of metals,
12. (A) White tin is an example of tetragonal system. especially transition metals.
(R) For tetragonal system a b c and a.. p == y i' 90. (R) Transition metals have free valencies.
26. (A) S02 gas is easily liquefied while H2 is not.
Set-2 (R) S02 has low critical temperature while Hz has high critical
The questions given below consist of two statements each printed as temperature.
'Assertion' (A) and 'Reason' (R). While answering these questions 27. (A) Diffusion is used in the enrichmentofU235 .
you are required to choose anyone of the following four responses: (R) A lighter gas diffuses more rapidly than a heavier gas.
(a) If both (A) and (R) are true and (R) is the correct explanation 28. (A) Crystalline solids can cause X-rays to diffract.
for (A). (R) Interatomic distance in crystalline solids is of the order of
(b) If both (A) and (R) are true but (R) is not the correct 0.1 nm. (AIIMS 2004)
explanation for (A). 29. (A) The effusion rate of oxygen is smaller than that of
(c) If (A) is true but (R) is false. nitrogen.
(R) Molecular size of nitrogen is smaller than that of oxygen.
(4) lfboth (A) and (R) are false.
(AIIMS 2004)
13. (A) Sulphur dioxide and chlorine are both bleaching agents. 30. (A) The compressibility factor for hydrogen varies with
(R) Both are reducing agents. (AIIMS 1994) pressure with positive slope at all pressures.
14. (A) Nitrogen is unreactive at room temperature but becomes (R) Even at low pressures, the repulsive forces dominate in
reactive at elevated temperatures (on heating or in the hydrogen gas. (AIIMS 2005)
presence of catalysts). 31. (A) Graphite is an example oftetragonal crystal system.
(R) In nitrogen molecule, there is extensive delocalisation of (R) For a tetragonal system a b i' c, a.. =p == 90' and
electrons. (AIIMS 1996) y == 120'. (AIIMS 20(6)
15. (A) Noble gases can be liquefied. 32. (A) No compound has both Schottky and Frenkel defects.
(R) Attractive forces can exist between non-polar molecules. (R) Both defects change the density of the solid.
(AIIMS 1998) (AIIMS 2008)
302 I G.R.B. PHYSICAL CHEMISTRY FOR COMPETITIONS

[,4~ ." OBJECTIVE QUESTIONS


..
• Set-1
1. (c) 2. (d) 3. (d) 4. (a) 5. (a) 6. (b) 7. (b) 8. (c)
9. (a) 10. (a) 11. (a) 12. (c) 13. (b) 14. (c) 15. (d) 16. (d)
17. (b) 18. (b) 19. (d) 20. (a) 21. (d) 22. (b) 23. (a) 24. (a)
25. (c) 26. (a) 27. (a) 28. (d) 29. (b) 30. (c) 31. (a) 32. (b)
33. (a) 34. (b) 35. (b) 36. (a) 37. (d) 38. (bi. 39. (c) 40. (b)
41. (d) 42. (d) 43. (c) 4ft. (c) 45. (a) 46. (b) 47. (d) 48. (c)
49. (c) 50. (c) 51. (d) 52. (b) 53. (d) 54. (c) 55. (a) 56. (a)
57. (a) 58. (d) 59. (d) 60. (a) 61. (b) 62. (b) 63. (c) 64. (c)
65. (d) 66. (c) 67. (a) 68. (d), 69. (b) 70. (b) 71. (d) 72. (a)
73. (a) 74. (c) 75. (d) 76. (c) 77. (b) 78. (c) 79. (d) 80. (a)
81. (c) 82. (b) 83. (a) 84. (d) 85. (c) 86. (c) 87. (a) 88. (b)
89. (a) 90. (d) 91. (c) 92. (b) 93. (d) 94. (a) 95. (d) 96. (b).
97. (b) 98. (c) 99. (a) 100. (d) 101. (a) 102. (d) 103. (b) 104. (c)
105. (c) 106. (a) 107. (d) 108. (b) 109. (b) 110. (b) 111. (d) 112. (it)
113. (c) 114. (b) 115. (a) 116. (d) 117. (a) 118. (a) 119. (b) 120. (c)
121. (d) 122. (d) 123. (c) 124. (d) 125. (b) 126. (b) 127. (c) 128. (a)
129. (a) 130;' (c) 131. (b) 132. (a) 133. (d) 134. (d) 135. (a) 136. (b)
137. (d) 138. (c) 139. (b) 140. (a) 141. (a) 142. (b) 143. (c) 144. (a)
145. (b) 146. (c) 147. (c) 148. (d) 149. (d) 150. (c) 151. (b) 152. (a)
153. (c) 154. (b) 155. (d) 156. (c) 157. (c) 158. (b) 159. (d) 160. (b)
161. (d) 162. (b) 163. (b) 164.. (c) 165. (b) 166. (b) 167. (a) 168. (a)
- 169. (b) 170. (d) 171. (c) 172. (d) 173. (c) 174. (b) 175. (d) 176. (a)
177. (d) 178. (b) 179. (a) 180. (a) 181. (b) , 182. (a) 183. (c) 184. (b)
185. (c) 186. (d) 187. (a) 188. (c) 18:>. (b) 190. (b) 191. (d) 192. (b)
193. (a) 194. (c) 195. (a) 196. (d) 197., (d) 198. (c) 199. (c) 200. (c)
201. (a) 202. (b) 203. (c) 204. (c) 205. (a) 206. (b) 207. (b) 208. (a)
209. (b) 210. (d) 211. (a) 212. (d) 213. (c) 214. (b) 215. (a) 216. (a)
217. (d) 218. (a)· 219. (c) 220. (b) 221. (c) 222. (c) 223. (a) 224. (c)
225. (b) 226. (b) 227. (d) 228. (d) 229. (c) 230. (b) 231. (b) 232. (b)
233. (a) 234. (a) 235. (d) 236. (d) 237. (c) 238. (c) 239. (b) 240. (c)
241. (d)
• Set-2
1. (a) 2. (a) 3. (d) 4. (b) 5. (a) 6. (a) 7. (a, c) 8. (b, c, d)
9. (c) , 10. (d) 11. (b, d) 12. (a, c)

[J"'I~, : ASSERTION-REASON TYPE QUESTIONS

1. (b) 2. (c) 3. (b) 4. (a) 5. (a) 6. (a) 7. (a) 8. (b)


9. (c) 10. (a) 11. (c) 12. (c) 13. (c) 14. (c) 15. (a) 16. (c)
17. (d) 18. (a) 19. (c) 20. (d) 21. (c) 22. (d) 23. (b) 24. (b)
25. (a) 26. (c) 27. (b) 28. (c) 29. (c) 30. (a) 31. (d) 32. (d)
STATES o.~ MATTER 303

BRAIN STORMING PROBLEMS


lOBJECTIVE QUESTIONS) for ( lIT "SPIRANTS
, ,

The following questions have single correct option:


(a) n I V (b) nb
1. Which ofthe following graphs is not a straight line for an ideal
gas?
n2
1 I [Hint: Pressure correction = a - 2
(a) V -7 T (b) T -7 P (c) n - 7 - (d) n - 7 - V
T P
2. The quantity (PV I kBT ) represents the: n2
Pressure correction"" - , ]
(a) number of molecules in the gas V-
(b) mass of the gas 7. Which of the following conditions is favourable for
(c) number of moles of the gas liquefaction of gas?
(tl) translational energy of the gas (a) T> Tc; P > Pc (b) T < Tc; P > Pc
[Hint: PV = nRT (c) T=Tc'P=Pc (d) T=TC'P<Pc
PV 8. An ideal gas of certain mass is heated in a small vessel and
n=- then in a large vessel, such that their volume remains
RT
1 PV unchanged. The P - T curves are:
Number of molecules =N A - (a) parabolic with same curvature
RT
PV (b) parabolic with different curvatures
=(RI N A)T =PV- (c) linear with same slope
(d) linear with different slopes
where, kB =Boltzmann constant.]
[Hint:
3. I litre of N2 and 7IS litre of 02 at the same temperature and
pressure were mixed together. What is the relation between the
masses of the two gases in the mixture?
(a) MN z = 3Moz (b) MNz = SM oz
(c) MNz Moz (d) MNz = 16M oz
[Hint: PV =M RT T--
m
MN 2 RT
Px 1= __ Straight lines with different slopes will be obtained.]
... (i)
28 9. A spherical air bubble is rising from the depth of a lake when
P xl =Moz RT ... (ii)
pressure is P attn and temperature is T K. The percentage
8 32 increase in its radius when it comes to the surface of a lake will
Dividing eq. (i) by eq. (ii), we get be: (Assume temperature and pressure at the surface to be
M N1 ,= MOl] respectively 2T K and P 14.)
4. A box is divided into two equal compartments by a thin (a) 100% (b) 50% (c) 40% (d) 200%
partition and they are filled with gases X and Y respectively. ?,V pv. ...
[Hint: _1_,1 =....Ll; VI = InItial volume, V2 =final volume
The two compartments have a pressure of 250 torr each. The TI T2 '
pressure after removing the partition will be equal to:
(a) 125 torr (b) 500 torr (c) 250 torr (d) 750 torr
PVI !.. X V2
T 4 2T
5. The density of a gas A is twice that of a gas B at the same
V
temperature. The molecular mass of gas B is thrice that of A. V1 = -2
The ratio of the pressures acting on A and B will be: 8
(a)6:1 (b)7:S (c) 2:5 (d) 1:4 V2 = 8V1
[Hint: Pm = dRT
=8 x -4 nr3 =-4 n(8r)3
V2
PAm ' dART 3 3
- -A= - -
PBmB dBRT New radius =2r(:.lOO% radius will increase)]
I 10. It is easier to liquefy oxygen than hydrogen because:
x =2 (a) oxygen has a higher critical temperature and lower
PB 3
. inversion temperature than hydrogen
PA :PB = 6: 1]
6. In the corrections made to the ideal gas equation for real gases, (b) oxygen has a lower. critical temperature and higher
the reduction in pressure due to attractive forces is 'directly inversion temperature than hydrogen
proportional to:
304 G.R.B. PHYSICAL CHEMISTRY FOR COMPETITIONS

(c) oxygen has a higher ci:itical temperature and a higher [Hint: PV =nRT
inversion temperature than hydrogen
PV:::: W RT
(d) the critical temperature and inversion temperature of m
oxygen is very low W
U. 2 mole 'He' is mixed with 2 gm of Hz. The molar heat capacity Ro V:::: RT ... (i)
232
at constant pressure for the mixture is: w
PN V =-RT ... (ii)
(a) 17R (b) IlR (c)4R (d) 3R 2 28
6 6 2
P~ = 28
[Hint: C f\ for 'He' = 5 R; C P2 for H2 == 7 R PN2 32
· 2 2
nHe = 2; nH2 = 1
P~ =0.875PN2 ]
5 7 17. KCI crystallises in the same type oflattice as does NaCI.Given
nlC PI + nzC P2 2x-R+ Ix R
Cp 2 2 thatrNa + /'cr == 0.55 and rK+ /r,:T 0.74. Calculate the ratio
nl + nz 3
of the side of the unit cell for KCl to that ofNaCI
17R .
6
= ] [PET (Kcrala) 200S1
(a) 1.123 (b) 0.891 (c) 1.414 (d) 0.414
12. The van der.Waals' constant' a' for the gases 0z, N z, NH3 and
(e) 1.732
CH4 are1.36, 1.39,4.17 and 2.253 respectively, the gas which
can be most easily liquefied is: . [Hint: 0.74
. (a) 0z (b) N2 (c) NH3 (d) C~
[Hint: The van der Waals' constant' a' is directly proportional
to the intermolecular force; hence, the gas with greater value +I 1.55 ... {l)
of' a' can be most easily liquefied.] .
13. At what temperature will the molar kinetic energy of 0.3 mole r +
.1L + 1= 1.74 ... (2)
of 'He' be the same as that of 0.4 mole of argon at 400 K?
(a) 700 K (b) 500 K (cl 800 K (d) 400 K
'Cr
Dividing (2) by (I)
[Hint: KE 2 nRT 1.74
2 - =--"'---""-- x --"'-'---
n =1, for molar kinetic energy 1.55

(2 2
x Ix Rx T) Helium
(2 2
x 1x Rx 400) Argon
r
K
+ +r _
CI:::: 1.122]
rNa+ + 'el-
T=400K]
lS. 8: 8 coordination of CsCI is found to change into 6: 6 coor-
14. Let ~ and P be the saturated partial pressure and partial
dination:
pressure of water respectively. Then the relative humidity is
(a) on increasing the pressure
given by:
P (b) on increasing the temperature
. (a) Ps + P x 100 (b) - x 100 (c) on decreasing the pressure
Pa P,
(d) on decreasing the temperature
P
(c) ~ x 100 (d) (P + Pa ) xlOO 19. An alloy of Cu, Ag and Au is found to have copper constituting
P ccp lattice. If silver atoms occupy the edge-centres and gold is
15. Molar volume of a mono atomic gas at Po and To is 0.5 times its present at body-centre, the alloy has the formula:
molar volume at 10Po and at the same temperature. Ignoring (a) Cu 4 Ag 2Au (b) Cu 4Ag 4Au
the value of the van der Waals' constant 'a', the radius of the
(c) Cu 4Ag 3Au (d) CuAgAu
gas molecules is given by:
113 [Hint: In ccp an:angement, eu atoms occupy the corners and
(a) 0. 8RTo (b) Q. 6RTo face-centres.
[ )
Po Pr/f1t :. Number of Cu atoms = 8 x .! + 6 xl:::: 4
8 2

(c) l
(
3RTo
20Po N1t)
\113
I (d)
(
3RTo
20Po1t )
1/3
Number of silver atoms =Number .of edge - centres x!..4
=12x-41 = 3
where, N = Avogadro's number
16. A vessel is filled with a mixture of oxygen and nitrogen. At Number of gold atoms:::: I (at body -centre)
what ratio of partial pressures will the mass of gases be :. Formula of alloy =CU4Ag3Au]
identical? 20. The anions (A) form hexagonal closest packing and atoms
(M) occupy only two-third of octahedral voids in it; then the
(a) P(02) o.785P(N2 ) (b) P(02)= 8.75P(N2 )
general formula of the compound is:
(c) P(02) =114P(Nz ) (d) P(02) 0.875P(N2 )
(a) MA (b) A2 (c) M zA 3 (d) M3A2
I

STATES OF MATTER

21. In a face-centred cubic of A and B-atoms in which A atoms are 29. WhiCh of the following compounds represents an inverse 2: 3
at the comers of the unit cell and B-atoms at the face-centres, spinel structure')
one of the A-atoms is missing from one corner in unit cell. The (a) Felli [Fell Fe lll ]04 (b) Pb0 2
simplest formula of the compound is: (c) Al 2 0 3 Cd) Mn 3 0 4
(a) A7B3 (b) AB3 (c) A7B24 (d) A gB 2l 30. A solid solution of CdBr 2 in AgBr contains:
1 7 (a) Schottky defects
[Hint: Number of A atoms = - x 7 -
8 8 (b) Frenkel defects
1
Number of B atoms = 6 x - = 3 (c) Colour centres
2 (d) Frenkel as well as Schottky defects
A:B=7: 37 : 24 31. Which of the following cOlTectly represents the relation'
8 between capillary rise' h' and capillary radius 'r' ?
Molecular formula = A7B 24 ]

II~ 11\ .
22. Select the incorrect statement(s):
(a) Schottky defect is not shown by CsCI
(b) Frenkel defect is shown by ZnS
(c) hcp and ccp structures have different coordination hi
hi " -
numbers I ' f __
I
(d) at high pressure, the coordination number increases r_
23. Select the incorrect statement: (a) (b)
(a) In CsCI unit cell, r
Cs
+ + rCl - == J3
2
I
I
(b) In NaCI unit cell, rNa+ + rq_ 2
(c) In CsCI unit cell, 68% space is void
(d) In NaCl unit cell, 26% space is void
24. The ionic radii ofRb + and I - are 1.46 and 2.16 A respectively. r-- r -+
The most probable type of structure exhibited by it is: (e) (d)
(a) CsCI (b) NaCI (c) ZriS (d) CaF 2
25. Perovskite is a mineral with the formula CaTi0 3 • Which ofthe
[Hint: Capillary rise decreases with increase in the radius of
positive ions in the crystal is more likely to be packed in. the
tube.]
octahedral holes?
32. There is a depression in the surface of the liquid in a capillary
(a) Ti 4+ (b) Ti 2+ (c) Ca 2+ (d) O~
when:
26. Which of the following is incorrect ?[RHU (Sereening) 20(8) (a) the cohesive force is smaller than the adhesive force
(a) A real gas behaves like ideal gas over a wide range of (b) the cohesive force is greater than the adhesive force
pressure (- 100 atm) at Boyle point (c) the cohesive and adhesive forces are equal
(b) A real gas behaves like an ideal gas over a wide range of (d) none ofthe above is true
pressure (~ 100 atm) at critical temperature of the gas.
[Hint: Depression in the surface takes place When

aV I " ofor an ideal gas


(aU
(c) intermolecular attraction force of liquid called cohesive force
JT dominates the force of attraction between the liquid and the
(CJU\ a capillary called adhesive force.]
(d) -I = for a real gas obeying van der Waal's
av /T 33. Surface tension does not vary with:
l'

(a) temperature (b) vapour pressure


equation
(e) the size of surface (d) concentration
[ . , : A real gas cannot behave like ideal gas at critical
temperature. ] 34. Which amon~ the following has the least surface tension?
27. The number of atoms in 100 g of an fcc crystal with density (a) Benzene (b) Acetic acid
d = 10 g 1 cm 3 and cell edge as 200 pm is equal to: '. (c) Diethyl ether (d) Chlorobenzene
(a) 3 x 1025 (b) 5 x 1024 (c) 1 X 1025 (d) 2 x 1025 . 35. The.SI unit ,of the coefficient of viscosity is:
(a).Ns-1m- 1 (b) Nsm'2 (e) Ns'2 m -2(d)'Ns- 1 m- 2
28. In a ccp structure, the:
36. Compressibility factor for a gas under critical condition is:
(a) first and third layers are repeated
(a) 3/8 (b) 8/3 (c) I (d) lf4
(b) first and fourth layers are repeated
3
(c) second and fourth layers are repeated [Hint: F;.Vc = 8 RTc
(d) first, third and sixth layers are repeated
Z= ~1
RTc 8
306 G.R.B. PHYSICAL CHEMISTRY FOR COMPETITIONS

37. Critical temperature of H 2 0 is greater than that of O2 because X:Y:Z


H 20 molecules have: 15: 3: 6
(a) greater dipole moment 4
(b) V-shape 5:4:8
(e) lesser number of electrons .. Simplest formula will be X SY4 Zg.]
(d) it has only sigma bonds 42. Which of the following pattern has void fraction of 0.26 ?
38. For van der Waals' constants 'a' and 'b', the unit of the ratio!!. (a)AAAA... (b) ABABAB ...
b (c) ABCABCABC... (d) ABCCBAABC...
will be: 43. ,Two gas bulbs are connected by a thin tube. Calculate the
partial pressure of helium after the connective valve is opened
(a) atm litre- I (b) litre-atm mol- I
at a constant temperature of 27°C:
(c) litre-atm mol- 2 (d) litre mol- 1
2
an
[Hint: ~orrecljon =0 -;;:z . Unit of a = atm litre 2 mor2
1
Vcorreclion = nb Unit of b = litre mol-
2 2
. " f (a)
UUlt,O' - = atm litre mol- .
b litre mol-I
(a) 1 atm (b) 0.328 atm (c) 1.64 atm (d) 0.166 atm
= atm litre mor I ]
• PVO.63 x 1.2
39. Select the correct order of the following temperatures: [Hmt: nHe =. RT == 0.0821 ~ 300 = 0.03
Boyle temp. Critical temp. Inversion temp.
(A) (B) (C) = 2.8 x 3.4 = 0.39
1lt<e 0.0821 x 300
(a) A> C > B (b) B> A > C
0.03 0
(c) A > B > C (d) C> A > B xHe= :: = O. 714
n He + ~e 0.03 + 0.39
[Hint: To (Boyle temp.) =!!:....; Ti (Inversion temp.) = 2a Total pressure after connecting the bulbs will be :
. Rb Rb
ijV( + PZV2 == PR (VI + V2)
Tc (Critical temp.) ~ 0.63 x 1.2 + 2.8 x 3.4 PR (1.2 + 3.4)
27Rb
PR == 2.33 atm
.. Ti >To > Tel
40. The gas equation for a real gas is: PHe =xHe xPR =O.0714x2.33 O.166atml
P(V - b) =RT 44. Ammonia gas at 76 cm Hg pressure was connected to a
Here, the parameter 'b' is van der Waals' constant. The graph' manometer. After sparking in the flask, ammonia is partially
of pressure against temperature (isochore) will give straight dissociated as follows:
line of slope: 2NH3 (g) ;:=::! N z(g) + 3Hz(g )
the level in the mercury column of the manometer was found
(a) zero (b) (V~ b) (c) R/ P (d) negative
to show the difference of 18 cm. What is the partial pressure of
Hz (g ) at equilibrium?
41. A crystalline solid is made of X, Y and Z elements. Atoms of X
(a) 18 cm Hg (b) ,) cm Hg
form fcc packing; atoms of Y occupy octahedral voids while
(c) 27 cm Hg (d) 24 cm Hg.
atoms of Z occupy tetrahedral voids. What will be the simplest
formula of' solid if atoms along one body diagonal are' [Hint: 2NH3 (g) ~ N 2(g) + 3H2 (g)
. removed: 76 o o
(a)'X SY4Z g (b) XYZ (c) X gY4 Z S (d) X 2Y2 teq, 76-2x x 3x
[Hint: Number of atoms of X(at packing site, i.e., at corners 'Ital pressure after dissociation = 76' 2x + x + 3x = 76 + 2x
and face-centres) Increase in pressUre == 2x = 18
I 1 x 9cm.
=8x-+6x =4
8 2 Partial pressure ofH2 = 3x =27 cm]
Number of atoms of Y = 4 45. Two balloons A and B are taken at
Number of atoms of Z = 8 I 300K. Maximum capacity of balloon 1600 mL

a
Along one body diagonal there will be two X atoms, one Y A and balloon Bare 800 mL and 1800
atom and·two Z atoms are found and are removed . mL respectively. When the balloon
.' ' . I 15 system is heated; which one will
Number of atoms of X will be ::::: 4 - x 2 burst first?
, 8 4
Number;oi~t~ms of Y will be = 4 I ::::: 3 (a) Inner balloon (A)
(b) Outer balloon (B)
Number of'at6ms ofZ will be 8- 2=6
(c) Both balloons simultaneously
(d) Cannot be predicted
STATES OF MATTER 307

[Hint: For balloon A :


:: V2
Tl T2
700 800
t
pv
300 T2
T2 342.85 K v- p-
For balloon B : (c) (d)
:: V2
Tl T2 2. Which of the following equation(s) is/are correct On the basis
1600 1800 of ideal gas equation?
N
300 T2 (a) PV = RT (b) PV NkBT
NA
T2 337.5 K
d .
Thus, outer balloon B will burst first.] (c) PV - RT (d) PV = dRT
m
46.
where, N. =number of molecules, N A = Avogadro's number,
kB = Boltzmann constant
3. A gas can be easily liquefied:
( a) when its. inversion temperature equals the Boyle's temperatlu;:
(b) under.reversible adiabatic expansion
(c) under pressure when it is cooled below the critical temperature
Ga" (d) at low pressure and above critical temperature
[Hint: (i) Reversible adiabatic expansion lowers the
temperature hence facilitates the liquefaction of gas. (ii) A gas
can be liquefied below the critical temperature by increasing the
pressure.]
4. Which of the following quantities are same for all ideal gases
at the same temperature?
A gas jar of 10 litre volume filled with O2 at 300 K is (a) The kinetic energy of I mole
connected to glycerine manometer. The manometer shows 5 m (b) The kinetic energy of I g
difference in the level as shown in figure. What will be the (c) The number of molecules in I mole
. number of moles of O2 in the gas jar ? (d) The number of molecules in I g
(Give dglycerine = 2.72 g/mL; d mercury 13.6 g/mL} [Hint: (i) 1 mole of a gas always contains 6.023 x 1023 molecules.
(a) 0.64 mol (b) 0.4 mol (c) 0.94 mol (d) 0.36 mol (ii) Kinetic energy of ideal gases depends only on temperature.]
[Hint: (h x d x g)Hg (h x d X g)glycerine 5. Which of the following statements are correct?
(h X 13.6)Hg = (5 x 2.72~ycerine (a) Helium diffuses at a rate 8.65 time-t as much as CO does
%g =lm (b) Helium diffuses at a rate 2.65 times as fast as CO does
(c) Helium diffuses at a rate of 4 times faster than CO 2
(1 + 0.76) m
Pgas ==
= 1760mmHg (d) Helium diffuses at a rate 4 times as fast as S02 does
PV=nRT 6. The viscosity of a liquid molecule depends on:
1760 (a) the volume of the liquid'
x 10 = n x 0.0821 x 300 (b) the temperature of the liquid
760
(c) the surface area of the liquid
n =0.94 mol]
(d) the structure of the molecule
Following questions may have more than one correct options: 7. Viscosity is the property of:
1. Which of the following graphs represent Boyle's law? (a) liquids (b) gases (c) solids (d) all of these
8. Which of the following crystals have 6: 6 coordination?

ptt=
(a) NH41 (b) MgO (c) MnO (d) ZnS
9. Which of the following compounds represent a normal 2: 3
t
P
!t
spinel structure?
(a) rv~gll AI~II04 (b) COil (Colllh04
(c) Zn(TiZn) 0 4 (d) Ni(CO)4
10. Which type of crystals contain· only one Bravis lattice?
v- p- (a) Hexagonal (b) Triclinic
(a) (b) (c) Rhombohedral (d) Monoclinic
11. At which temperature, is a fenimagnetic solid con....erted to a
ferromagnetic solid?
308 G.R.S. PHYSICAL CHEMISTRY FOR COMPETITIONS

(a) 850 K (b) 300 K (c) 400 K (d) 600K Constant P


12. In NaCI structure, all the:
(a) octahedral sites are occupied i
V
(b) tetrahedral sites are unoccupied
(c) octahedral as well as tetrahedral sites are unoccupied
(d) octahedral as well as tetrahedral sites are occupied T--
13. The density of a certain solid AB (formula mass = 119) is 2.75
(e)
glcm 3 . The edge ·of the unit cell is 654 pm long. What is/are
true about the solid AB? 16. Molar mass ofNe atom is ten times ofH2 molecule. Which of
the following statement is/are true?
(a) It has bcc unit cell
(b) There are four constituents per unit cell (a) Both these gases have same Kinetic Energy at 27°C
(c) Unit cell constituted by anion is fcc (b) Hz molecule will travel 10 times faster than Ne atom at
(d) Structure is similar to ZnS same temperature
3 (c) Pressure of I mol Ne atom and 1 mole H2 molecules will
[Hint:. Z =a
xdxN be same
M
23 (d) Ten mole ofH2 and 1 mole ofNe will have same volume at
(654 X 10- 10 )3 x 2.75 x 6.023 x 10 '" 4] a temperature of 27°C
119 17. Select the correct conditions indicated below the following
14. The unit cell of a crystalline solid is bounded by f (faces), e' plots:
(edges) and c(interfacial angle). Which of the following
relations is correct? Isochores Isobars
(a) f + e = c + 2
(c) c + e = f + 2
(b) f + c = e + 2

~l/~v, I//p,
~,
(d) None of these
15. Which of the following diagrams correctly describes the (b)
behaviour of a fixed mass of an ideal gas? (T is measured in K) (aJ
[PMT (Kerala) 2007; eMS Vellore (Med.) 2008;
CET (Punjab) 20(8) -273°C 1°C
Temp(K) --II>
ConstantT Constant V (V1 > V2 > v:;) (P1 < P z < P3)

~
t t Isotherms isotherms

~ ~T'
p P

I
t ~T2
v- T-- (c) 11.,;

(a) (b)
Constant T Constant T Volume --+ - - P----'
i
PV
t
PV
(T1 < Tz < T 3 ) (T1 < Tz < T3)

p- V-
(c) (d)
. -
C~.~.(~I.}-~-----------I
Single correct option
1. td) 2. (a) 3. (e) 1. (c) 5. (a) 6. (d) 7. (b) 8. (d)
9. (a) 10. (c) 11. (a) 12. (c) 13. (d) 14. (b) 15. (c) 16. (d)
17. (a) 18. (b) 19. (c) 20. (c) 21. (c) 22. (a, c) 23. (c) 24. (b)
25. (a) 26. (c) 27. (b) 28. (b) 29. (a) 30. (d) 31. (b) 32. (b)
33. (c) 34. (e) 35. (b) 36. (a) 37. (a) 38. (b) 39. (d) 40. '(b)
41. (a) 42"/ (c) 43. (d) 44. (c) 45. (b) 46. (c)

• One or more than one correct options


1; (b, c) 2. (a, b) .3. (b, c) 4. (a, c) 5. (a, d) 6. (b, d) 7. (a, b) 8. (a, b)
9. (a, b) 10. (a, b, c) n. (a) 12. (a, b) 13. (b, c) 14. (b) 15. (e) 16. (a, b, c)
Ii" (a, c, d)
STATES OF MATTER

Integer Answer TYPE QUESTIONS·


9. 3.2 g of oxygen and 0.2 g of hydrogen are placed in 1.12litre
This section contains 14 questions. The answer to flask at O°c. The total pressure in atm of the gas .mixture will
each of the questions is a single digit integer, ranging be ......... ;
from 0 to 9. If the correct nnswers to question numbers 10. A mixture of non reacting gases exert a pressure of 5 atm. If
X, Y, Z and W (say) are 6, 0, 9 and 2 respectively, then one of the gases DCCUPY 40% volume of the mixture, what
the con-ect darkening of bubbles will look like the would be its partial pressure in atm?
given figure: 11. A gasis f.Qund to have the fon-nula (C3 0 2 )n' Its vapour density
is 34. The value ofn will be ............ . .
12. The weight of hydrogen gas obtained from42 g of CaH2 by
1. A substance AxBy crystallises in a face centred cubic lattice. treatment withwater is ............... gm.
Atoms' A' occupy each corner of unit cell and atoms of' B' 13. The stop cock containing two bulbs of volume 5 litre and 10
occupy centre of each face of the cube. Total number of atoms litre eontaining an ideal gas at 9 atm and 60 atm respeetively is.
of A and B in one unit cell will be ............. . opened. What is the final pressure in atin if the temperature
2. In KEr crystal structure, the number of second nearest remains the same?
neighbour ofK+ ions are ................. . 14. At 400 K, the root mean square. (rms) speed of gas X
3. Density oflithium at~m is 0.53 glcm3 ,The edge length ofLi is (molecular mass'" 40) is equal to the most probable speed of
3.5 A. The number of lithium atoms in a unit cell will be gas Y at60 K. The molecular mass of the gas Y is ........
........... (Atomic mass oflithium is 6.94) (lIT 2009)
4. A cubic solid is made up of two elements P and Q. Atoms of Q
are present at the comers of the cube and atoms of P. at the
body centre. The coordination number of P and Q will be
[Hint:lnt "'l~l
PR400
40
=PR60 m
5. If there are 3 moles of atoms present in the packing of pattern
ABC ABC ABC ...... The number of moles of tetrahedral voids
is equal to ............... .
50 = ~4 :30
6. The number of atoms in HCP unit cell is ............ . m 4]
7. How many effective Na + ions are present in rock salt NaCl? If
ions along one axis joining opposite faces are removed?
.8. How many moles of S02 will occupy a volume of 10 litre at a
pressure of 15 atm and temperature 624 K? (a 6.71 atm L2
mol-2 ; b 0.0564 litre moC l )

1. (4) 2. (12) 3. (2) 4. (8) 5. (6) 6. (6) 7. (3) 8•. (3)


9. (4) 10. (2) 11. (1) 12. (4) 13. (7) 14. (4)
310 G.R.B. ' PHYSICAL CHEMISTRY FOR COMPETITIONS

~•. LINKED COMPREHENSION TYPE QUESTIONS • •


• Passage 2
• Passage 1
The gas molecules randomly move in all directions and collide
The gases which strictly fol:ow the general equation with each other and with the wall of the container. It is difficult to
(PV nRT) are called ideal or perfect gases. Actually, there is no determine the speed of an individual molecule but it has become
gas which is perfect or ideal. A real gas is one which actually exists, possible to work out the distribution of molecules among different
whether it obeys gas laws strictly or not. Under ordinary conditions, molecular speeds. This is known as Maxwell Boltzmann distribution.
only those gases nearly behave as ideal or perfect which have very Consider the following graph about Maxwell's distribution of
low boiling points such as nitrogen, hydrogen, etC. The most easily speeds at two different temperatures 'Ii and T2 :
liquefiable· and highly soluble gases such as ammonia, carbon
dioxide, sulphur dioxide show large deviations.
A very convenient method of studying deviation of real gases from
ideal behaviour is through a compressibility jactor (Z). a 1---.----.----,..,._
Z PV
i S
nRT ~nn)
(i) Z 1, for ideal gases. Fraction of
tatal molecules
(ii) Z", 1. for real gases.•
~n.Jwer the following questions:
1. Consider the equation. Z =- PV- ;. which· of the following - Speed
nRT
statements is correct? Answer the following questions:
(a) When Z> 1,' real gases are easier to compress than the 1.. In the above graph the point 'P' refers to:
ideal gas (a) root mean square speed at 'Ii
(b) When Z = 1, real gases get compressed ea~ily (b) average speed at 'Ii
(c) When Z > 1, real gases are difficult to compress (c) most probable speed at 'Ii
(~ When Z =1, real gases are difficult to compress (d) highest possible speed at 'Ii
2. The compressibility of a gas is less than unity at STP, 2. The shaded area represents:
therefore: (a) number of molecules having speed between u j and u2
(a) Vm > 22.4 L (b) Vm < 22.4 L (b) number of molecules having speed less than the most
(c) Vm 22.4 L (d) Vm = 44.8 L probable speed
3. At low pressure, the van der Waals' equation is reduced to: (c) number of molecules having vrms at 'Ii
PV aP PVm bP (d) fraction of total molecules having average speed
(a) Z R; 1- RT (b) Z= RT 1+ RT 3. The·point Q refers to:

(c) PVm = RT ( d) Z PVm


RT
I ~
RT
(a) number of molecules with speed at P
(b) fraction of total molecules with speed at P
4. At Boyle's temperature, compressibility factor Z for a real gas (c) root mean square speed
is: (d) total kinetic energy of molecules at P
(a) Z = I (b) Z 0 4. Relation between 'Ii and T2 is:
(c) Z> I (d) Z < I (a) 1] T2 (b) 'Ii > T2
!Hint: At Boyle's temperature, the real gas behaves like an (c) 'Ii < T2 (d) cannot be predicted
ideal gas.] 5. Total area under the curve at 'Ii is:
5. The behaviour of a real gas is usually depicted by plotting (a) equal to that under curve at T2
compressibility factor Z versus pressure P at a constant (b) less than that under curve at Tz
temperature. At high temperature and pressure, Z is usually (c) greater than that under curve at T2
more than one. This fact can be explained by van der Waals' (d) can be greater or less than that under curve at T2 ,
equation when: depending on the nature of the gas
(a) the constant 'a' is negligible but not 'b' 6. Select the correct statement(s):
(b) the constant 'b' is negligible but not 'a' (a) Most probable speed increases with increase in
(c) both the constants 'a' and 'b' are negligible temperature
(d) both the constants 'a' and 'b' are not negligible (b) Fraction of total molecules with most probable velocity
6. The units of compressibility factor are: decreases with increase in temperature
(a) atm L -I (b) atm- I .(c) Area under the curve increases with increase in the temperature
(c) L -I (d) unitless (d) none of the above
I
STATES OF MAnER 311

7. 'The curve has which of the following characteristics? (a) zero refractive index (b) zero surface tension
(j)It has symmetrical distribution of molecules against (c) zero viscosity (d) zero critical temperature
molecular velocity, 7. Gas A can be liquefied at room temperature by applying
(ii)The area under the curve gives the total number of pressure but gas B cannot. This reflects:
molecules, (a) critical temperature of B is less than that of A
(iii)The maxima of the curve shifts towards right as the - (b) critical temperature of B is greater than that of A
tempe~ture is raised. ;< ~ (c) A and B have critical temperature greater than room
(iv) The area under the curve is independent of temperature, temperature
Select the correct statements from the codes given below: (d) critical temperatures of both are equal
(a) (i), (ii) (b) (i), (ii) and (iii) 8. The values of critical volumes of four gases A, B, C and Dare
(c) (ii), (iii) and (iv) (d) all are correct 0.025 L, 0.312 L, 0.245 Land 0.432 L respectively. The gas
with larger diameter will be:
(a) A (b)D
• Passage 3 (c) B (d) C
The essential conditions for liquefaction ofgases were discovered 4
[Hint: .Vc 3b 3x4Nx-nr'l]
by Andrews in 1869 as a result of his study of 3
pressure-volume-temperature relationship for CO2 , It was found that
above a certain temperature, it was impossible to liquefy a gas
• Passage4
whatever the pressure was applied. The temperature below which the Kinetic theory of gases is a generalization offered by Maxwell,
. gas can be liquefied by the application of pressure alone is called Boltzmann, Clausius, etc., to explain the behaviour of ideal gases.
critical temperature (Tc ). The pressure required to liquefy a gas at This theory assumes that ideal gas molecules neither attract nor
this temperature is called the critical pressure (Pc)' The volume repel each other. Average kinetic energy of gas molecules is directly
occupied by one m91e of the substance at the critical temperature proportional to the absolute temperature. A gas equation called
and pressure is called critical volume. Critical constants are related kinetic gas equation was derived on the basis of kinetic theory,
with van der Waals' constant as follows: PV I mnu 2
a 8a "3
Vc :::: 3b, Pc :::: - - 2 ' Tc
27b 27Rb Answer the following questiol"
Answer the following questiol'., 1. Gas Density
1. The relationship between Pc, Vc and Tc is: A 0,82 g L- 1
(a) PcVc RT (b) PcVc :::: 3RTc B 0.26gL- 1
3 3 O,51gL- 1
5
(c) PcVc = RTc (d) PcVc :::: 8 RTc C
Pick up the correct statement/statements:
2. Which of the following parameters is three times the van der 1. gas A will tend to lie at the bottom.
Waals' constant 'b'? 2. the number of atoms of various gases A, Band Care
(a) Critical volume (b) Critical temperature same.
(c) Vapour density (d) Critical pressure 3. the gases will diffuse to form homogeneous mixture,
3. The critical temperature of: 4. average kinetic energy of each gas is same,
(a) a substance means the temperature above which the (a) 2,3 (b) 1,4' (c) I (d) 3,4
substance is in vapour form "'. 2. Select the incorrect statement(s) about the real gases:
(b) a gas is the temperature below which it can be liquefied by I. The molecules attract each other.
application of pressure
2. They show deviation from Boyle's and Charles' law.
(c) water is lOO°C , 3. Volume of gas molecules are negligible.
(d) none of the above
4. The molecules have negligible mass,
4. The pressure required to liquefy a"1ns ~ -at the critical
(a) 2,3 (b) 1,4 (c) I (d) 3,4
temperature ts called:
3. The average kinetic energy per molecule of an ideal gas is
(a) reduced pressure (b) critical pressure
equal to:
(c) vapour pressure (d) atmospheric pressure
(a) 0,5 kJ (b) O.5RT
5. Gases A BCD (c) 1.5kT (d) 1.5R:F
Pc (atm) 2.2 14 35 45 4. Which of the following do not pertain to the postulates of
r;, (~) 5.1 33 127 140 kinetic theory of gases?
Which of the above gases cannot be liquefied at 100 K and 50 - (a) The gas molecules are perfectly elastic
atm? (b) Speed of gas molecules are ever changinJ
(a) D only (b) A only (c) Pressure exerted by the gas is due to the collision of
(c) A'andB (d) C andD molecules with the walls of the container
6. At critical point, the meniscus between liquid and vapour (d). Kinetic energy of a gas is given by the sum of 273 and
disappears due to: temperature in Celsius scale
314 G.R.B. PHYSICAL CHEMISTRY FOR COMPETITIONS

r'6>. SELF ASSESSMENT r~


ASSIGNMENT NO.4

SECTION-I 9. If the graph is plotted for 1 mol gas in such a way that PV is
plotted against P then intercept of the graph for real gas will be:
Straight Objective Type Questions
(a) RT + Pb + a (b) RT
This section contains 14 multiple choice questions. Each
(c) RT - Pb + a (d) RT + Pb + ab + a
question has 4 choices (a), (b), (c) and (d), out of which only
one is correct. 10. The ratio of Boyle's temperature and critical temperature for a
x gas is:
1. Boyle's temperature offour gases are given below:
, 1
(d) ~
8 (b) 27
Gases ABC D (a)- (c) -
g- TB . 120 K 25 K 500 K 410K 27 8 '2 1
'G which gas can be liquified most easily? 11. With which of the following elements silicon should be doped
,i so as to give p-type semiconductor? ICBSE (PMT) 20081
WA ~B ~C ~D
Ie 2. A gas is enclosed in Ii vessel of volume Vat temperature 1\ and (a) Selenium (b) Boron (c) Germanium (d) Arsenic
h pressure P; the vessel is connected to another vessel of volume 12. Total volume of atoms present-in a face centred cubic unit cell
'r of a metal is (r is atomic radius): p{PMER 20081
V / 2 by a tube and a stop cock. The second vessel is initially
'it
evacuated. If the stop cock is opened, the temperature of (a)16 n ? (b)20n? (c) 24 n ? (d)gnr3 .
second vessel becomes T2 • The first vessel is maintained at a 3 3 3 3
temperature 1\. What is the final pressure ~ in the apparatus? 13. How many effective Na + .and cr ions are present respectively
Ct
(a) 2PT2 (b)2PT2 (c) PT2 (d) 2PT2 in the rocksalt (NaCl). If ions along the axis joining opposite
2T2 +1\ T2+21\ 2T2 +1\ 1\+T2 faces are removed ?

.-'
3. The ratio of rate of diffusion of gases A and B is 1: 4. If the . (a)4,~ (b)~,~ (C)~,4 (d) 3,3
ratio of their masses present in the mixture is 2 : 3, what is the
ratio of their mole fraction? 14. Kinetic energy and pressure of a gas of unit mole are related as :
1 2 3', E
(a)-
1
(b)-
1
(c)- (d)~ (a)P=2E (b)P=-E (c)P=-E (d)P=-
8 12 16 24 3 2 2
4. D~nsity of a gas at STP is 2g / L while the expected density is
1.8 g / L assuming its ideal behavi~ur. Then: SEC1"ION-1i
(a) gas behaves ideally
(b) forces of attraction are dominant among gas molecules Multiple Answers Type Objective Questions
(c) forces of repulsion are dominant among gas molecules 15. Which of the following measure the deviation from ideal
, (d) none of the above behaviour of gas?
5. In a crystalline solid, anions B are arranged in ccp lattice and
(a) Collision diameter (b) Collision frequency
cations A occupy 50% of the octahedral voids and 50% of the (c) Compressibility factor (d) van der Waals' constant 'a'
tetrahedral voids. What is the formula of the solid? 16. In face-centred cubic unit cell:
(a) AB (b) A3B2 (c) A2B2 (d) A2B3 (a) face di~gonal·ofthe cube is a.fi or 4r
6. In a compound XY2 0 4 , oxide ions are arranged in ccp and (a = edge length, r = radius of constituent units)
cations X are present in octahedral voids. Cations Y, are (b) effective number of atoms in the unit cell is 4
equally distributed among octahedral and tetrahedral voids. ' (c) 8 tetrahedral voids per unit cell
The fraction of the octahedral voids occupied is: , (d) rank of the unit cell is 3
1 1 1 1 17. Which of the following mixtures of gases at room temperature
(a) "4 (b) "2 (c)(j (d)"S
follow Dalton's law of partial pressures?
7. An ideal gas: (a) NO and 02 (b) CO and CO2
(a) can be liquefied if its temperature is more than critical (c) NH3 and HCl (d) S02 and O 2
temperature 18. A real gas can be liquefied:
4r
(b) can be liquefied if its pressure is more than critical pressure (a) under adiabatic expansion
(c) cannot be liquefied at any pressure and temperature (b) above critical temperature
;ell (d) can be liquefied if its temperature is more than Boyle's (c) when cooled below critical temperature under applied pressure
temperature (d) at temperature lower than critical temperature and
8. A mineral is made of calcium, titanium and oxygen Ca 2+ ions pressure higher than critical pressure
located at comers, Ti 4 + ions at the body-centre and 0 2- ions at 19. Which of the following statements is (are) correct?
face-centres of the unit cell. The molecular formula of the (a) In body-centred cubic unit cell, the coordination number is
mineral is ....... 12
(a) CaTi 30 4 (b) CaTi03 (c) CaTi0 2 (d) CaTi 20 3 (b) The coordination number of each type of ion in CsCl is 8
STATE;S OF MAnER' 315

(c) A unit cell of an ionic crystal shares some of its ions with Statements (a, b, c and d) in Colurrm-I have to be matched with
neighbouring unit cell statements (p, q, rand s) in Colurrm-II. The answers to these
(d) If rNa+= 95 pm; rCl - = 181 pm; then edge length of unit questions have to be appropriately bubbled as illustrated in the
cell ofNaCI is 552 flm following examples:
If the correct matches are (a-p,s); (b-q,r); (c-p,q) and,(d-s);
SECTION-III then correct bubbled 4 x 4 matrix should be as follows:
Assertion-Reason Type Questions p q r s
This section contain'S 6 questions. Each question contains
Statement-l (Assertion) and Statement-2 (Reason).' Each
a®®0 ®
question has following 4 choices (a), (b), (c) and (d), out of
which only one is correct.
b ® ® 0 CD
c ® @ CD
(a) Statement-l is true; statement-2 is true; statement-2 is a
correct explanation for statement- L
0
(b) Statement-l is true; statement-2 is true; statement-2 is not d ® ® 0 0
a correct explanation for statement-I.
(c) Statement-l is true; statement-2 is false. 26. Match the Colurrm-I with Column-II:
(d) Statement-l is false; statement-2 is true. Column-I Column-II
20. Statement-I: The pressure inside the LPG cylinder remains (a) 'High temperature (P) Z oF I
constant even when it is in use at room temperature. ' a
(b) Extremely low pressure (q) Pb <-
Because V
Statement-2: Vapour pressure of any liquid is independent of (c) Very high pressure (r) Z::: 1
its amount; it, depends only on temperature. a
21. Statement-I: If a gas has compressibility factor (Z) greater (d) Low pressure (s) Pb >
V
than unity, then repulsive forces are dominant.
27. Match the Colunm-I with Colunm-II:
Because
Statement-2: Value of Z decreases with increase in pressure. Column-I Column-II
22. Statement-I: The value of Boyle's temperature for a real gas (a) Constituent units occupy only comers (P) 67.98%
. (Tn
IS a J. (b) Constituent units occupy comers as
. ' well as face-centres
(q) 26.17%
Rb Because
Statement-2: At Boyle's temperature, Tn, real gases behave (c) "'Constit:uent units occupy comers and (r) 74.01%
ideally over a long range of pressure. body~centre
23. Statement-l: CaC03 shows polymorphism.
(d)' Constituent units occupy comers and (s) 52.33%
Because
edge-centres
Statement-2: CaC03 exists in two forms called aragonite and
calcite. ' 28. Match the Colunm-I with Colunm-II: (UT 2007)
24. Statement-l: Lead zirconate is piezoelectric solid. Column-I
'"
Column-II
Because
Statement-2: Lead zirconate crystals have no dipole moment. (a) Simple cubic ana (P) Have these cell
25. Statement-l: Band gap in germanium is small.
face-centred cubic parameters. abc
Because, anda ~=y
Statement-2: The energy spread of each germanium atomic (b) Cubic and rhombohedral (q) Are two crystal systems
energy level is infinitesimally small. (liT 2007),; (crql;i.bf$'1wd tetragonal (r) Have only two
~ ,. -, ".':": ,:;,'
crystallography angles
SECTION-IV of 90°
Matrix-Matching Type Questions {dj~:Hexagonal and (s) Belong to same crystal
This section contains 3 questions. Each question contains monoclinic system
statement given in two colurrms which'have to be matched.

.
[J4~', -------!~/~.- - - - - - - - - I ..
I
1. (c) 2. (a) 3. (b) 4. (b) 5. ,(b) 6. (b) 7. (c) 8. (b)
9. (b) , 10. (b) 11. (b) 12. (a) 13. (d) 14. (b) 15. (c, d) 16. '(a, b, c)
17. (a, b, d) 18. (a, c, d) 19. (b, c, d) 20. (c) 21. (c) 22. (b) 23. (a) 24. (c;)
25. (c) 26. (a-p) (b-p) (c-p, s) (d-p, q) 27. (a-8) (b-r) (c-p) (d-q) 28. (a-p, s) (b-p, q) (c-q) (d-q, r)
5
SOLUTIONS
(General and Colligatlve Properties)

5.1 INTRODUCTION 5.3 TYPES OF SOLU"nONS


When two or more chemically non-reacting substances are AU the three states of matter (gas, liquid or solid) may behave
mixed, they form mixtures. A mixture may be heterogeneous or either as solvent or solute. Depending on the state of solute or
homogeneous. A heterogeneous mixture consists of distinct solvent, mainly there may be the following seven types of binary
phases and the observed properties are just th.e sum of the solutions: .
properties of individual phases. However, a homogeneous S.No. . Solvent
Solute Example
mixture consists of a single phase'which has properties that may
differ drastically from those ofthe individual components. 1. Gas Gas Air
A homogeneous mixture whose composition can·be varied 2. Gas Liquid Aerated water (C0 2 + H 2 O)
within certain limits is termed a true solution. 3. Gas Solid Hydrogen in palladium
The constituents of a solution cannot be separated by Liquid Liquid Alcohol in water, benzene in
4.
filtration, settling or centrifugal action. All solutions are toluene
characterised by (i) homogeneity, (ii) absence of settling and (iii)
the molecular or ionic state of sub-division of the components. 5. Liquid Solid Mercury in zinc amalgam
When the solution is composed of only two chemical 6. Solid Liquid Sugar in water, common salt
substances, it is termed a binary solution. Similarly, it is called in water
ternary and quaternary if it is composed of three and four Solid Solid Various
. components, respectively. thus; a solution may be regarded as
a single phase containing more than one component. The solution of liquid in' gas or .solid in gas is not possible
because the constituents cannot form a homogeneous mixture.
5.2 SOLVENT AND SOLUTE For a given solution, the amount of the solute dissolved in a
unit volume of solution (or a unit amount of solvent) is termed as
Every solution consists of a solvent and one or more solutes. the concentration of the solute. Solutions containing relatively
Solvent in a solution is its constituent substance which has the high concentration of solute are called concentrated solutions
same state of aggregation as that of the solution. Generally, the while those of relativelY low concentratioll of solute are termed
component present in greater amount than any or all the other as dilute solutions. '
components is called the solvent. For the solubility of solids in
In this chapter, we shall discuss only three types ·of solutions:
liquids, where the liquid is present in large excess over the solid,
(i) Gas in liquid.
there is no ambiguity in these terms, the solid being the solute and
the liquid being the solvent. However, if the solution is such that (ii) Liquid in liquid.
the state of aggregation of the solution is the same as that of a (iii) Solid in liquid.
component present in smaller amount, the latter is called the
solvent For example, in a syrup (liquid solution) containing 60%
5.4 METHODS OF EXPRESSING THE
sugar (a soiid) and 40% wl'lter (a liquid-same aggregation as CONCENTRATION OF A SOLUTION
solution), water is tenned as the solvent. In a solution of alcohol The concentration of a solution can be expressed in a number of
and water, the substance present in a larger proportion by mass is ways. The important methods are:
called the solvent. But if both the liquids have same masses in (i) Mass percentage or Per cent by mass: It is defined as
solution, it becomes difficult to differentiate between solvent and the amount of solute in gram present in 100 gram of the solution.
solute.
. SOLUTIONS 317

Mass percentage of solute Mass of solute x 100 (vi) Mole fraction: This method is used when the solution
Mass of solution is constituted by mixing two or more components. It is defined as
the ratio of number of moles of one component to the total
Mass of solute
= x 100 number of moles of the solution (i.e., all the components). Taking
Mass of solute + Mass of solvent three components A, Band C.
Mass of solute
=Volume of solution x Density of solution
xlOO Components A B C
Mass (in gram) WI w2
. Mass of solute . . Molecular mass m] m2
The ratio -_.... IS termed as mass fraction.
Mass of solution
No. of g moles
Thus, ml m2
Mass percentage of solute = Mass fraction x 100 Total number of g moles
ml m2 m3
10% solution of sugar means that 10 gram of sugar is present
in 100 gram of the solution, i.e., 10 gram of sugar has been Thus, Mole fraction of A =
dissolved in 90 gram of water. wJ
-+
(ii) Per cent by volume: It is defined as the volume of ml m2 m3
solute in mL present in 100 mL solution.
Volume of solute Mole fraction of B fB
Per cent of solute by volume = x 100
Volume of solution mJ m2
(iii) Per cent mass byvolume: It is defined as the mass of
solute present in 100 mLof solution. Mole fraction of C =
WI
Per cent of solute mass by volume -+
m1 m2 m3
Mass of solute
----xIOO The sum of mole fractions of a solution is equal to 1, i. e~
Volume of solution
fA + IB + .fc '~~'."
(iv) Strength or concentr~tion (Gram per litre): It is In a binary solution,
defined as the amount of the solute in gram present in one litre of
the solution. Mole fraction of solute + Mole fraction of solvent = I
Concentration of solution Let n moles of solute (B) andN moles of solvent (A) be present
Mass of solute in gram in Ii solution.
=
Volume of the solution in litres Mole fraction of solute _11_ ~XB
N+n
Mass of solute in gram
~ x 1000
Volume of the solution in mL Mole fraction of solvent ~= X A
N+n
Concentration in gram per litre is also tenned as strength of
the solution. Let W g of the solute be present in V litre of solution; Thus, X A +XB =I
then Mole fraction is independent of temperature of the solution.
(vii) Molality: It is defined as the number of moles of the
Strength or concentration of the solution ; gL- I solute present in 1 kg of the solvent. It is denoted by m
lNote: V is not the volume of the solvent. V is actually the final Number of moles of solute
volume after dissolving a definite quantity of solute in the Molality (m) = --~--------­
Number of kilo-grams of the solvent
solvent.]
(v) Paris per million (ppm): When the solute is present in Let WB gram of the solute of molecular m~s mB be present in.
trace quantities, it is convenient to express the concentration in WA gram of the ~01vent, then
parts per million (ppm).lt is defmed as the quantity of the solute
in gram present in 106 gram of the solution. Molality (m) = x 1000
mB xW A
Mass of solute 10
opm= x 6 Relation between mole fraction and molality:
• Mass of solution
n N
Atmospheric p0!lution in cities is also expressed in ppm by X B =--andX A =
. N+n N+n
volume. It refers to the voluT.e of the pollutant in 106 units of
volume. 10 ppm of S02 in air means 10 mL of S02 is present in Moles of solute
- - - - - - =: --"'---'-=-

106 mL of air. Moles of solvent mB X WA


318 G.R.B. PHYSICAL CHEMISTRY FOR COMPETITIONS

x 1000 WB X 1000 Molarity of mixing: Let there be three samples of solution


--''---- : :;: : :;: m
X A xm A mB XWA (containing same solvent and solute) with their molarity
M t ,M 2 ,M3 and volumes V!,V2 ,v3 respectively. These
or m solutions are mixed; molarity of mixed solution may be given as:
MIVt +M2V2 +M3 V3 :::;:: MR (VI +V2 +V3 )
Note: (i) Molality is the most convenient method to express the where, M R Resultant molarity
concentration because it involves the mass of liquids rather
than their volumes. It is also independent of the variation in VI + V2 + V3 : :;: Resultant volume after mixing
temperature. Note: Molarity is dependent on volume; therefore, it depends on
(ii) Molality and solubility are related by the following relation: temperature.
Solubility x 10 1M Molar solution, i. e, molarity is I
Molality :x: - - - - - - - - - - - O.5Mor MI2 Semimolar
Molecular mass of the solute
0.1 M or MilO Decimolar
Mass of solute in gram x 100] 0.01 M or Mil 00 Centimolar
[ Solubility 0.001 M or Mil 000 Millimolar
Mass of solvent in gram
Relationship between molality and molarity:
(viii) Molarity (Molar concentration): It is defined as the 1 p mB
-=----
number of moles of the solute per litre or per dm 3 of the solution, m M 1000
i.e.,
where, p density of solution
Number of moles of solute
Molarity (M) :::;:: - - - - - - - - - m= molality
• Number of Htres of solution
"M molarity
or Molarity x Number of litres of solution
mB :::;:: molar mass of solute
: :;: Number of moles of solute
Relation between molarity 'M' and mole fraction:
Let wB g ofthe solute of molecular mass mB be dissolved in V
Let M be the molarity of solution, and x A , X B be mole'
litre of solution.
fractions of solvent and solute, respectively.
Molarity of the solution:::;:: --=-- Suppose n A and nB moles of solvent and solute are mixed to
mB xV form solution.
or Molarity x mB Strength of the solution Mass of solution : :;: nAmA + nBmB ... (i)
V
where, m A and mB are molar masses of solvent and solute,
If V is taken in mL (cm 3 ), then respectively. '
Molarity of the solution:::;:: x 1000 Volume of solution:::;:: Mass : :;: (n Am A + n BmB )
mB xV Density d
The unit of molarity is mO.llitre -lor mol dm -3. Nuinber of moles of solute x _ _ _10,-0_0_ __
Molarity
Molarity (second method): Let d = density of solution in Volume of solution
glmL and let it contains x% solute by mass. 1000xd
M nB x
Then, mass of I litre solution (1000 x d ) g (nAmA +,nBmB)

Mass of solute in 1 litre : :;: ~ x (1000 x d ) g Dividing both numerator and denominator by (n A + nB ),
100
1000x d
:::;:: (xx d x lO)g M x------------
nB
Number of moles of solute in I litre --'-'-- x mA + x mB
nA+nB nA+nB
Massof solute in gram x x d x 10
xlOOOxd
Gram molecular mass of solute mB M
xAm A +xBmB
,where mB molecular mass of s,olute
(ix) Normality: It is defined as the number of gram
M:::;:: xxdx 10 equivalents of solute present per litre of solution. It is denoted by
mB 'N'.
Molarity of dilution: Number of gram equivalents of solute
Normality (N)
Before dilution After dilution Number of litres of the solution
MtVt M 2V2 or Normality x Number. of lit res of the solution
Number of gram equivalents ofthe solute
SOLUTIONS

Let wagram of the solute of equivalent mass E a be present in


V litre of the so lution, then,
: :: :::I_$OME SOLVED EXAMPLES\I::::: :
. walEa wa
Normabty = = -....=::....-- Example 1. Calculate the masses of cane sugar and water
V Ea xV required to prepare 250 g of25% cane suga~ solution.

Normality x Equivalent mass =W B Solution: Mass percentage of cane spgar 25


V We know that,
Strength of the solution in gIL
Mass of solute
Solutions are expressed as:
Mass percentage =Mass of solution
x 100
N N N N
1N 2 N - - - - - etc. Mass of cane sugar
, '2'10'100'1000' So, 25 --:------- X 100
250
Like molarity, normality is also dependent on volume,
2Sx 250
therefore, it depends on temperature. or Mass of cane sugar = = 62.5 g
100
Normality of solution may be represented as,
Mass of water (250 62.5) ::;; 187.5 g
1N Normal solution, i.e., normality is 1
Example 2. Calculate the molarity of the following
N solutions:
0.5 Nor Seminormal
2 (a) 4 g of caustic soda is dissolved in 200 mL of the solution.
N (b) 5.3 g of anhydrous sodium carbonate is dissolved in 100
0.1 Nor- Decinormal
10 mL of solution.
(c) 0.365 g ofpure HCI gas is dissolved in 50 mL ofsolution.
0.Q1 Nor N Centinormal
100 Solution: (a) Molecular mass ofNaOH 40

0.001 Nor N Millinormal No. of gram moles in 4 g of NaOH = =0.1


1000 , 40
Relationship between normality and molarity: We know
that, Volume of solution in litre = 200 0.2
1000
Molarity x Molecular mass Strength of the solution (gIL) · No. of moles of solute
MoIanty ::;; - - - - - - - - - - .2:.!. 0.5 M
. Volume of solution in litre 0.2
Similarly,
(b) Molecular mass of anhydrous Na2C03 = 106
Normality x Equivalent mass =Strength of the solution (gIL) No. of gram moles in 5.3 g of anhydrous Na2C03
Hence, 5.3 ::: 0.05
Molarity x Molecular mass Normality x Equivalent mass 106

Normality Volume of solution in litre = 100 =0.1


Molecular mass 1000
or ------=n
Molarity Equivalent mass No. of gram moles of solute
Molarity = - - - - - - - - - -
So, Normality =n x Molarity VQlume of solution in litre

[N9mlidity (second method): Let 'd' is the density of = 0.05 0.5M


solution in glmL and x is the percentage ofthe solute by mass. 0.1
Then, (c) Molecular mass of HCI =36.5
N= xxdxlO ] No. of gram mol~s in 0.365 g of pure HCI = 0.365
Equivalent mass of soitl;te 36.5
(x) Formality: It is the number of formula mass in gram Volume of solution in litre = 50
present per litre of solution. In case, formula mass is equal to 1000
molec1.!!~r mass, formality is equal to molarity. Like molarity and No. of gram moles of solute
normality, the lVrmality is also dependent on temperature. It is Molarity =- - - - - - - - - -
Volume of solution in litre
used for ionic COtupt:1mds in which there is no existence of a
molecule. Mole of ionic compounds is called formole and 0.365 1000
=--x 0.2M
molarity as formaHty. 36.5 50
320 G.R.B. PHYSICAL CHEMISTRY FOR COMPETITIONS

Example 3. The density of a solution containing 13% by · 0 fsolutlOn


V = vo Iume ' . I' 250
In ltre = - - 0.25
mass of sulphuric acid is 1.09 g/mL. Calculate the molarity and 1000
normality of the solution.
Molarity = 1
Solution: Volume of 100 g of the solution 10
= 100 100 mL Hence,
1 w
d 1.09 10 106 x 0. 25
100
----litre 106 x 0.25 = 2.65
1.09 x 1000 or w
10 g
1
---litre Example 6. 214.2 g of sugar syrup contains 34.2 g of
1.09xJO sugar. Calculate (i) molality of the solution and {ii) mole fraction
13 ofsugar in the syrup.
Number of moles of H 2 S04 in 100 g of the solution
98 Solution: (i) Mass of sugar=. 34.2 g
· No. of moles of H? S0 4 13 1.09 x 10 34.2 34.2
M o1anty = - = x--- No. of moles of sugar ----= 0.1
Volume of soln. in litre 98 Mol. mass 342

1.445M Mass of water (214.2 - 34.2)


[Note: In solving such numericals, the following fonnula can be 180 k
applied:
= 180 g 1000 g
% strength of soln. x density of soln. x 10
Molarity = ----~--------­ No. of moles of water 180 = 10
Mol. mass 18
Similarly, No. of moles of sugar = 0.1 x 1000
of soln. x density'of sola. x10 .; Molality =- ------
Mass of water in kg 180
" Nor~~ity
Eq. mass
0.555m
We know that,
(ii) Total number of moles = 10.0 + 0.1 = 10.1
Normality = Molarity x n

= 1.445 x 2 I n = Mol. mass = 98 = 2J ,"


,.~.
. No. of moles of sugar
Mole fraction of sugar = - - - - - - - -
l Eq. mass 49 Total number of moles
=2.89N = 0.1 = 0.0099
Example 4. Calculate the molarity of pure water 10.1
(d=lg/mL). Example 7. A solution contains 410.3 g ofH 2 SO4 per litre
Solution: Consider 1000 mL of water. of solution at 20°C. If the density is 1.243 g/mL. what will be its
Mass of 1000 mLofwater ~ 1000 x 1 = 1000 g molarity and molality?
1000 Solution: Mol. mass of H 2 S0 4 98
No. of moles of water = - - 55.5
18 410.3
No. of moles of water No. of moles of H 2S04 = - - = 4.186
Molarity 98
Volume in litre
· fH SO I' No.ofmolesofH? S 04
55.5 Mo1artty 0 2 4 so utlOn = -
55.5M Volume of soIn. in litre
. 4.186 = 4.186M
Example 5. Calculate the quantity of sodium carbonate 1
(anhydrous) required to prepare 250mL M solution.
10
Mass of 1 litre H2 SO 4 solution =1000 x 1.243 =1243 g
Solution: We know that, Mass of water = (1243 - 410.3) 832.7 g = ~: kg
w
Molarity No. of moles of H 2 S04
MxV Molality of solution
Mass of water in kg
where, w= mass ofNa2C03 in g,
4.186 x 1000
M = molecular mass ofNa2C03 in gram 106 832.7
= 5.027m
SOLUTIONS 321

Example 8. The density ofa 2.03 M sQlution ofacetic acid V&' No. of moles of S20~- ions::: No. of moles of Na2S203
(molecular mass 60) in water is 1.017 glmL. Calculate the .
molality of the solution. =3
Solution: Strength of the solution MolalityofS20~- ions: 3 x1OO0=3.86m
Molarity x mol. mass 776
== 2.03 x 60 121.8 gl L Example 10. A solution is prepared by dissolVing 5.64 g of
Density ofsolution = 1.017 gl mL glucose in 60 g of water. Calculate the following:
Mass of 1 litre of solution = 1000 mL x 1.017 gl mL (i) mass per cent ofeach ofglucose and water;
"" 1017 g (ii) molality of the solution,
895.2 (iii) mole fraction of each ofglucose and water.
Mass·of water ::=1017 -121.8 = 895.2 g = kg
1000 Solution: (i) Total mass of solution
Molality = 2.03 x 1000 ::: 2.267 m ::: 5.64 + 60.= 65.64 g
895.2
Mass per cent of glucose = 5.64 x 100 = 8.59%
, Example 9. The density of a 3 M sodium thiosulphate 65.64
solution (Na2S 203 ) is 1.25 glmL. Calculate (i) the percentage by
Mass per cent of water::: (100 - Mass per cent of glucose'
mass of sodium thiosulphate, (ii) the mole fraction of
sodium thiosulphate and (iii) molalities ofNa + and SzO;- ions. :;: (100 8.59)=91.41%

Solution: (i) Mass of 1000 mL ofNa zS 2 0 3 solution (ii) No. of mores of glucose = 5.64
180
1.25 x 1000= 1250 g . 60
Mass of water in kg = - -
Mass ofNa z Sz 0 3 in 1000 mL of3 Msolution . 1000
3 x Mol. mass of NazSz03 Mo11 '
a lty 5.64 x -
=-- 1000
- : 05'
. 22 m
180 60
= 3x 158= 474 g
Mass percentage ofNazS203 in solution (iii) No. of moles of glucose = 5.64 :;;;; 0.0313
180
474 x 100 37~92 . 60
No. of moles of water:;;;; - :;;;; 3.333
1250 18
Alternatively, M :::: x x d x 10 Molefraction of glucose; 0.0313 =.0.0093
mA 3.333 + 0.Q313
3 xX 1.25 x 10 Mole fraction of water = 3.333 :::: 0.9907
158 3.333 + 0.0313
x= 37.92 Example 11. What would be the molality of a solution
made by mixing equal volumes of 30.0% by mass of H 2 S04
(ii) No. of moles of Na 2S2 0 3 :;: 474 =3 (density 1.218g cm- 3 ) and 70% by mass of H 2 S0 4 (density
158
1.61Og cm-3 )?
Mass of water =(1250 - 474)::: 776 g
Solution: Let 100 mL of one solution be mixed with 100 mL
776
No. of moles of water =- : 43.1 of the other solution.
18 Mass of 100 mL of 30% H 2 S04 100 x 1.218 :::: 121.8 g
Mole fraction of Na2S203 = .3 . =_3_=0.065 Mass of H 2 S04 in 12L8 g of 30% H2 S0 4
43.1 +3 46.1
30
:::: - X 121.8 g 36.54 g
(iii) No. of moles of Na+ ions 100
2 x No. of moles of Na2S203
. . " Mass of water = (1.21.8 - 36.54) :::: 85.26 g
=2x3 6
MassofloomLof70%H2 S04 100x1.6l 161.0g
· fN + . No. of moles of Na + ions !\1ass of H2S04 in 161.0 g of 70% H 2 S04
Mo1a1lty 0 a Ions - . - - - - - - -
Mass of water: in kg 70
=- x 161.0 g = 112.7 g
6 100
=-xlOOO
776 Mass of water.::: (161.0 -112.7)
..
=48.30 g
= 7.73-m Total H 2 S04 in solution = 36.54 + 112.7: 149.24 g
322 , A.B. PHYSICAL CHEMISTRY FOR COMPETITIONS

149.24 Example 15. Reaction,


No. of moles of H 2 S04
98 2.Br- (aq) + Cl z (aq) ---? 2Cl(aq) + Br2 (aq)
Total mass of water in solution =(85.26 + 48.30) is used for commercial preparation of bromine from its salts.
= 133.56 g
133.56 kg Suppose we' have. 50 mL of a 0.06 M solution of NaBr. What
1000 volume of a 0.05 M solution of Clz is needed to react completely
with the Br-? '
· No. of moles of Hz SO 4 149.24 1000
II
MoaIty= =--x-- Solution: 2Br- (aq) + Clz(aq) ---? 2Cl-(aq) + Br2 (aq)
Mass of water in kg 98, 133.56
M1V1 (Br- ) = M 2Vz (C1 2 )
=11.4 m
Example 12. The moleft-action ofCH3 0H in an aqueous
nl nz
solution is 0. 02 and its density is 0. 994 g cm-3. Determine' its where, nl and n 2 are corresponding coefficients.
- 0.06 x 50 = 0.05 X Vz
molarity and molality.
Solution: Let x mole of CH3 0H and y mole of water be 2 1
present in solution.
Example 16. Calculate the molarity, molality and mole
Mole fraction of CH3 OH =_x_ = 0.02
, x+ y fraction of ethyl alcohol in a solution of total volume 95 mL
. y x 1 prepared by adding 50 mL ofethyl alcohol (density = 0.789 mel)
So, - = 49 or - = to 50 mL water (density = 1.00 g m/:,-l ).
,x y 49
. x - 1000 Solution:
Molahty = - - x 1000= - - = 1.13 m Vol. x density
18xy 18x49 No. of moles of ethyl alcohol = - - - - -
. Total mass Mol. mass
Volume of solutIOn = mL
density = 50x 0.789 = 0.8576
- 46
32x+18y
' I'ltre=------~
32x+ ll'tre
0.994 x 1000 994' Vol. x density 50 x I
No. of moles of water =------ =
Mol. mass 18
Molarity --x--x994
32x+ 18y =2.7777
\
994 · ' No. of moles 1000
__9_94_ _ = 1.0875 M MoIartty = x
32+ 18xyl x 32+18x49 VoL of sol. in mL
, ,Example 13. Calculate the concentration of NaOH 0.8576 x 1000 = 9.0~~ 1f
solution in g/mL, whic:i has the same normality as that of a 95
solution ofHCI of concentration 0.04 g/mL. .
Molality = No. of moles of .ute x 1000
Solution: N Ws x 1000 0.04 x 1000 = 1.095 Mass of solvent "ams
HC1
Es xV 36.5x 1
== 0.8576 xIOOO= 52m
NNaOH ""N Rcl 50
1.095 = ws x 1000 · ==
MoI e f ractIOn 0.8576 0.8576 0236'
== - - = .
40x I 0.8576+ 2.7777 3.6353

Ws 0.0438g/mL
5.5' SOLUTIONS- OF GASES IN LIQUIDS
"Example 14. How many Na+ ions are present in 50mL of , (Solubility of Gases)
a 0. 5 M solution ofNaCl?
, MV All gases are soluble in water as well as in other liquids to a
Solution: Number of moles ofNaCI = - - greater or lesser extent. Solutions of gases in water play an
1000
important role in our daily life. The common examples are
0.5 x 50 = 0.025 carbo rated beverages, i.e., soft drinks, household cleaners
1000 containing aqueous solutions of ammonia, formalin-an aqueous
NaCI---? Na + + Cl- solution of , formaldehyde, etc. The natural waters contain
Number of moles of Na + = Number of moles of NaCI dissolved; 02 which is vital for the existence of aquatic life in sea,
! 0.025 ' . rivers and lakes.
The solubility of a gas in a liqui4 is expressed in terms of
'Number of ions of Na + 0.025 x 6.023 X 10 23
absorption coefficient. It is defined as the volume of the gas in
1.505 X 10 z2 mL that can be dissolved by 1 mL of a liquid solvent at the
SOLUTIONS 323
temperature of the experiment at one atmospheric pressure. The (y == rnx) passing through origin. Fig. 5.2 shows the variation of
volume of the gas is measured at STP. Thus, if v is the volume of solubility of some gases against equilibrium pressure. The
the gas dissolved, reduced to STP, V is the volume of the solvent straight line graphs show the validity of Henry's law.
and P is pressure of the gas in atmospheres, then the absorption
coefficient, a, Is given by . Slope of the line
'tan 0': K
v
i tan 01 = K(for N2)
VP
The following factors affect the solubilities of gases in liquids: .
(i) Nature ofthe gas and solvent: Generally, the gases which
can be easily liquefied are more soluble in common solvents. For·
yN'
.' ,,~.. ~\ 61 ~
tan 02 : K(for 02)

example, CO 2 is more soluble in water than oxygen or hydrogen. I;""" I •

The gases which react with the solvent posses higher solubility.
Pressure of gas ~
For example, HCI and NH3 are highly soluble in water. overtiquid .
Oxygen, nitrogen and carbon dioxide are much more soluble
Fig. 5.2 Variation of solubility of a gas with pressure
in ethyl alcohol than in' water. at the same temperature and
pressure. If solubility of the gas is known at one particular pressure.,
(ii) Temperature: The solubility of most of the gases'in then it can be calculated at other pressures using the followin!t
liquids decreases with increase of temperature as the dissolution relation,
is an exothermic process. When water is heated in a beaker, ~==PI ... (ii)
bubbles of air are formed on the sides of the glass before the m2 P2
water boils. As the temperature rises, the dissolved air molecules
begin to ."boil out" of the solution long before the water itself . where 'mI' is the solubility of the gas at pressure 'PI' and 'm2' is
boils. the solubility of the gas at pressure 'P2 '.· .
When a mixture of two or more non-reacting gases is brought
t in contact with a solvent, each constituent gas dissolves in
~0.003 proportion to its own partial pressure. Henry's law can be applied
is to each individual gas independent of the presence of other gas. If
gO.002
we use the mole fraction oftne gas in the solution as a measure of
a its solubility, then the law can be stated as, "molefraction of the
il 0.001
:l
'0
gas in a solution· is proportional to the partial pressure' of the
rJ)
'. 20 40 60 80 100120 . gas."
Temperature CC) ~ xocp
.Fig. 5.1 Temperature dependence of O2 gas where, x ==·mole fraction of the gas in solution
solubility in water and p == partial pressure of the gas.
The reduced solubility of molecular oxygen in hot water has a
or x=K' p
direCt bearing on thermal pollution~ i.e., the heating of the
environinent--:usually waterways-to temperatures that are 1
or p= K,x==KHx ( KH == },) .. :(iii)·
harmful to its living inhibitants.
(iii) Pressure : The most important factor which influences . K H is called Henry's law constant. Different gases havt::
the solubility of a gas in a liquid is the pressure. The .effect of diff~rent' values of KHat a defmite temperature for a given
pressure on the solubility of the gas in the liquid is given by solvent.
Henry's law, which states that the solubility of a gas in a liquid is
directly proportional to the pressure of the gas over the solution TableS.l Values ofHenry~s Law Constant (K H ) for Some
. at a definite temperature. The solubility is taken as the mass of Common Gases in Water
the gas dissolved per unit volume of the liquid ..Thus, if m is the
Gas Temperature (K) XJI
mass of the gas dissolved per unit volume of the solvent and P is
the pressure of the gas in equilibrium with the solution, then He 293 144.97
mocp Hz 293 69.16

... (i) N2 293 76.48


or m==KP
N2 ,]03 88.84·
wh~re, K is the proportionality constant. .
°2 293 34.86
When P == I, m == K, i. e., the solubility of the gas at unit
393 46.82
pressure is equal to constant K. The value of K is different for
each gas at a particular temperature. The magnitude ofK depends From equation (iii), it can be inferred that high"-' is the value
uponthe nature of the gas, nature of solvent, temperature and the of Henry's law constant, the lower is the solubility <l
the gas in
units of pressure. Equation (i) represents equation of straight line the l i q u i d . · · . . \ .
I
324 G.R.B. PHYSICAL CHEMISTRY FOR COMPETITIONS

Limitations of Henry's Law 6.56x 10-2


Henry's law holds good if the following conditions are 5x P2
fulfilled:
P2 = 0.762 bar
(i) The pressure is not too high,
(ii) The temperature is not very low,
(iii) The gas does not chemically combine with the solvent.
The gas molecules do not either dissociate or associate in the 1. 0.5 MH2 S04 is diluted from 1 litre to 10 litre, normality of the
solvent. In case the dissolved gas reacts with the solvent, higher resulting solution is: (AFMC 20(5)
solubilities can result. The solubility of ammonia in water is (a) IN (b)O.IN (c) ION (d) liN
much higher than expected because of the following reaction: [ADs. (b)]
NH3 +H20=NH40H~NH~ + OW . [Hint: M\V1 M 2V2
Carbon dioxide also reacts with water. 0.5 xl M2 X 10
CO 2 +H20~H2C03 M2 =0.05
Another interesting example concerns the dissolution of x
N = M basicity of acid = 0.05 x 2 0,1 N]
oxygen in blood. Normally, oxygen gas is only sparingly soluble 2. Molar solution means 1 mole of solute present in:
in water. However, its solubility in blood is abnormally high tReECE (Medical) 20051
because. of the high content of haemoglobin (Hb) molecules, (a) 1000 g of solvent (b) 1 litre of solvent
which are eventually delivered to the tissues for use in (c) 1 litre of solution (d) 1000 g of solution
metabolism. [Ans. (c)]
Hb+40 2 ~Hb(02)4 3. The molarity ofa solution containing 50 g ofNaCI in500 g of
Due to these processes, Henry's law is not strictly followed. a solution and having a density of 0.936 g/cm 3 is:
(a) 1.5M (b) 1.6M (e) 1.8M (d) 1.2M
Applications of Henry's Law [Ans. (b)]
1. Soft drink bottles are sealed at high pressure, to increase the [Hint: M = x x d x 10 ... (i)
solubility of CO 2 in the soft drink. . mB
2. At high altitude, partial pressure of oxygen is low, it leads to
x = 50 x 100 = 10
low concentration of oxygen in blood of people living there. Low 500
cont1~n'tration of oxygen develops anoxia, i.e., unable to think
x 10 = 1.6M]
M = 10 x 0.936
and act properly. 58.5
3. When scuba divers go deep in the sea, solubility of 4. 20 rnL of 0.5 M HCI is mixed with 30 rnL of 0.3 M HCI, the
. atmospheric gases increases in blood. When the divers come up, molarity ofthe resulting solution is:
there is release of dissolved gases and it leads to the formation of (a) 0.8 M (b) 0.53 M (e) 0.38 M (d) 0.83 M
bubbles of nitrogen in our blood capillaries and hence there is [Ans. (c)]
painful sensation caU:::-d bends. To avoid bends; the tanks of
[Hint: M1V\ + M 2V2 =MR(Vj +
scuba divers are filled with 11.7% 56.2% N2 and 32.1%
oxyge.n. 0.5 x 20 + 0.3 x 30 = M R X 51
LNote : (i:) Gases are less soluble in aqueous solutions of electrolytes M R =0.38M
than in pure water. It is called salting out effect. where, M R =resultant molarity 0 mixture.]
. (ii) Non-electrolytes such as sugar if present in a solution also 5. How many moles and how man) .:>.ams ofNaCI are present in
reduce the solubility of gases in water.] 250 rnL of a 0.5 MNaCI solution?
(a) 0.125 mol; 7.32 g (b) 7.32 niol; 0.125 g
ExalDple 17. Calculate the concentration ofCP2 in a soft
drink that is bottled at partial pressure ofCO2 of 4 atm' over the i (c) 0.125 mol; 0.125 g . (d) 7.32 mol; 7.32 g
[Ans. (a)]
liquid bt 25° C. The Henry s Law constant for CO 2 in water at ','
MV 0.5 x 250 .. .
25 0 C is 3.1 x 10-2 mol/litre-atm. . . [Hint: Number of moles = - - :: = 0.125
Solution: According to Henry's Law: I' i 1/ /' 1000 1000

f
: i s =KP 3.1 x lOt x 4 = 0.12 mol litre-I p Mass of NaCI = 58.5 x 0.125 = 7.32 g]
:. 6. Density of a 2.05 M solution of acetic acid in water is 1.02
ExampIe;l8./ The pdrtial pressure of ethane.: q~?r a ':"; glmL. The molality of the solution is: (AJEEE 20(6)
saturated soltiiiofo containing 6.56 x 10-2 g of ethane t~~ [. b~r. If (a) 1.14 mol kg- I (b) 3.28 mol kg- l
the solution conlains 5 x 10-2 g 'of ethane, th{!n wha{sh~tt bifhe (e) 2.28 mol kg:-l (d) 0.44 mol kg-I
partial pressuJ pf the gas? . • ....,,_ ... [ADs. (e)]
Solution: ~ = PI . [Hint: m= ¥xlOOO 2.05x 1000
S2 P2 1000d MmB 1000x 1.02- 2.05 x 60
=.2.28 mol kg-I]
I
SOLUTIONS
'" 325
7. The hardness of water sample containing 0.002 mol of (0.5 0.1) x = 0.4- 0.2
magnesium sulphate dissolved in a litre of water is expressed OAx = 0.2
as: ; -i-'F'f -:<":',,-;~,-,) 2-0n:; x = 0.5L
(a) 20 ppm (b) 200 ppm (c) 2000 ppm (d) 120 ppm :. 0,5 L of A and 1.5 L of B should be mixed.]
(e) 240 ppm
,[Ans. (b)] ~~6 SOLUTIONS OF LIQUIDS IN LIQUIDS
6
[Hint: 10 g water contains (0.002 x 1000) mol MgS04 When one liquid dissolves in another, the molecules of the
1 mol MgS04 1 mol CaC0 3 solvent are caused to move apart so as to accommodate the solute
2 mol MgS04 ;E 2 mol CaC0 3, i.e.,2 x 100 g CaC03 molecules. Similarly,the solute molecules must also be separated
so that they can take their places in the mixture. In both these
. . Hardness of water = 200 ppm]
processes, energy is required. Finally, as the solute and solvent
8. The density (in g mL-1) of a 3.6 M sulphuric acid solution, molecules are brought together, energy is released because ofthe
i.e., 29% H2 S04 (molar mass == 98g mor l ) by mass will be: attractive forces between them. When solute and solvent,
('>"IEEE 20()7) molecules are strongly attracted to each other, more energy is
(a) 1.45 (b) 1.64 (c) 1.88 (d) 1.22 released in the final step. Three cases may arise under these
[Ans. (d)] circumstances. The overall dissolution process results either in
evolution of heat or absorption of heat, or energy released in the
[Hint: M = x x d x 10 final step is the same as the energy absorbed in the first two, i. e. ,
ms
net change is zero.
3.6 x 98 '" 1.22 mL-l] Examples:
29 x 10 ' g 1. Benzene and carbon NO evolution or absorption
9. I litre solution containing 490 g of sulphuric acid is diluted to tetrachloride: of heat.
10 litre with water. What is the normality of the resulting 2. Acetone and water: Evolution of heat.
solution? (~';crt \~I;U7) 3. Ethyl alcohol and water: Absorption of heat.
(a) 0.5 N (b) 1.0 N (c) 5.0 N (d) 10.0 N A liquid mayor may nof be soluble in another liquid.
[Ans. (b)] Depending upon the relative solubility of a liquid in another, the
[Hint: N : : ; - 'xL1000
__
490 x 1000 10 following three cases are possible:
Es xV 49 x 1000 Liquid-Liquid Systems .'
N/V I =N Z V2
__________~I --------------1
IOxl N 2 xlO 1. Liquids that are 2. Liquids that 3. Liquids that
N z = 1] completely - are partially are practically
10. 250 mL ofa Na 1 C0 3 solution contains 2.65 g ofNa 1C03 . 10 miscible. miscible. immiscible,
mLofthis solution is added toxmL of water to obtain 0.001 M Examples: Examples: Examp!Js:
Na 2 C03 solution. The value of x is .... Benzene and Ether and wa- Benzene
toluene; ter; Phenol and water;
(Molecular mass ofNa 1 C03 = 106 amu) [SHe (Mains) 20081
Ethyl alcohol and water; Carbon tetra-
(a) 1000 (b) 990 (c) 9990 (d) 90
and water; Nicotine and chloride and
[Ans. (b)] Carbon tetra- water. water; Ben"
[Hint: Molarity of solution. chloride and zene and
M
Ws x 1000 = 2.65 x 1000 0.1 benzene. alcohol.
ms X V 106 x 250
M/V1 =M2 V2 Miscible liquids form three types of solutions, which can be
0.1 x 10 =0.001 (10 + x) ideal or non-ideal solutions.
x= 990mL] • > Ideal' solution:. An ideal solution is' one in which the '
H.The yolumes ohwo-HCI s{)lutions A (O:5N ) and B (0.1 N) to molecules attract one another with equal force irrespective of
their nature. Thus, a solution composed of two components
be mixed for preparing 2 L of 0.2 HfiC! are : , _, ____ _
A andB will bean ideal one if- the forces between
EAMCET (IVIed) 2(}o8!
A and A, Band B should be the same. An ideal solution
(a) 0.5 L of A + 1.5 L of B
(b) 1.5 L of A + 0.5 L of B
possesses the follOWIng characteristics:
(c) 1 L ofA + I L of B (i) Volume change of mixing should be zero.
(d) 0.75 L of A + 1.25 L ofB AVmix 0; Vsolvent + Vsolute = Vsol ...1ion
[ADs: (a)] (ii) Heat change on mixing should be zero.
[Hint: Let x L of A and (2 x) L of B are mixed. MI mix = o(Heat is neither absorbednor evolved.)
M1V/ + M 2V2 =MR(V/ + V2 ) (iii) There should be no chemical reaction between solvent
0.5 x x + 0.1 (2 - x) 0.2 x 2 and solute.
326 (7, R. B. PHYSICAL CHEMISTRY FOR COMPETITIONS '

Solvent+ Solute ~ Solution (vi) Ideal solutions must obey Raou!t's law at all

1Non-i~l
H2 0 + NH3 ~NH40H concentratious.
H 20 + CO 2 ~H2C03 The following are some of the binary mixtures which show the
H 2 0 + CaO ~Ca(OHh properties of ideal solutions:
(a) Benzene and toluene,
(iv) Solute molecules should not dissociate in the ideal (b) Carbon tetrachloride and silicon tetrachloride,
solution. (c) n-Hexane and n-heptane,
(d) Ethylene dibromide and ethylene dichloride.
NaCI Aqueous medi"n; Na + + CI- ) Raoult's Law
Non-ideal
Aqueous mediwn
H 2S04 )' 2H + + SO~- According to this law, the partial pressure of any volatile
constituent of a solution at a constant temperature is equal to
the vapour pr~ssure of pure constituent multiplied by the
(v) Solute moleculeS should not associate in ideal solution.
mole fraction of that constituent in the solution.

2
'0'
" eOOH Dissolved
benzeneIn
. . •
(ASS0711on)
err.
~
0-.+0
O---HO
,y

+0 0-
"'-
I """OH---OVC( ) 'I Non-Ideal
b
.
~.
Let a mixture (solution) be prepared by mixing n A mole ofo
liquid A and nB mole of liquid B. Let P A and PB be the partial
pressures of two constituents A and B in solution and P ~ and P ~
the vapour pressures in pure state respectively....

Table S.2 Comparison between Ideal and Non-ideal Solutions

Non-ideal solutions
Ideal solutions
Positive deviation from Raoult's law Negative deviation from Raoult's law

I. Obey Raoult's, law at every range of Do not obey Raoult's.law. Do not obey Raoult's law.
concentration.

=
2. IlH mix 0; neither heat is evolved nor IlH mix > O. Endothermic dissolution; heat IlH mix < O. Exothermic dissolution; heat
absorbed during dissolution. - is absorbed. is evolved.

3. AVmix 0; total volume of solution is AVmix > O. Volume is increased after AVmix < O. Volume is decreased during
equal to sum of volumes of the dissolution. dissolution.
co~ponents.

·4. P = PA + PB P~-XA + p~ X B PA ). p~ X A; PB > pZ X B PA <P~XA; PB <p~ X B


i·e.,PA = P~XA;PB'= pZ X B :·PA + PB > p~ X A + pZ X B :·PA + PB < p~ X A + p~ X B

5. A - A, A B, B - B interactions A - B attractive force should be weaker A B attractive force should be greater


should be same, i.e., 'A' and 'B' arei(!enti- than A - A and B - B attractive forces. than A - A and B - B attractive forces.
cal in shape, size and character. 'A' and 'B' have different shape, size and 'A' and 'B' have different shape, size and
• character. character.

6. Escaping tendency of 'A' and 'B' should 'A' and 'B' escape easily showing higher Escaping tendency of both components
be same in pure liquids and in the.solu- vapour pressure than the expected value. 'A' and 'B' is lowered showing lower
tion. vapour pressure than expected ideally.

Examples: Examples: Examples:


dilute solutions; acetone + ethanol; acetone + aniline;
benzene + toluene; acetone + CS 2; acetone + chloroform;
, n-hexane + n-heptane; water + methanol; CHpH + CI'I 3COOH;
chlorobenzene + bromobenzene; water + ethanol; H 2 0+HN0 3 ;
ethyl bromide + ethyl iodide; . CCl 4 + toluene; yhloroform + diethyl ether;
n-butyl chloride + n-butyl bromide. CCl 4 + CHCI 3 ; water + HCI;
acetone + benzene; acetic acid + pyridine;
CCl 4 +CHPH; chloroform + benzene.
cyclohexane + ethanol.
SOLUTIONS I 327

Thus, according to Raoult's law, =XAPJ+XBP:


p A :::
n A +nB
P ~ = mole fraction of Ax P ~ = X A P J This law, in fact, is the major deciding factor, whether
a solution will be ideal or non-ideal. Ideal solutions obey Raoult's
law at every range of concentration. Non-ideal solutions do
and PB nB pg = mole fraction of B x p~ X B PBD
not obey Raoult's law. They show either positive or negative
nA + nB
deviation from Raoult's law. For comparison between ideal
If the total pressure be P, then
and non-ideal solutions a table has been given below. (Only
P PA + PB binary combinations of miscible liquids have been considered.) .

Graphical Representation of Ideal and Non-ideal Solutions


Ideal deviation . Positive deviation Negative deviation

~ ~
:::l
tJ)
<Jl
pg :::l
<Jl
<Jl
~ ~
0. 0-
....
:::l
....
:::l
....
:::l
0
0.
0
0.
o
0.
~ ~ ~

Mole fraction XA = 1 Mole fraction Xs= 1 XA = 1 Mole fraction


Xs=O XB= 1 Xs=O XA=O Xs 0

Fig. 5.3
Relation between Dalton's Law and Raoult's Law: pressure of the solution and the mole fraction of methanol in the
The composition of the vapour in equilibrium with the vapour.
solution can be calculated applying Dalton's law of partial Solution: Mol. mass of ethyl alcohol == C zHs OH = 46
pressures. Let the mole fractions of vapours A and B be 60
f A arid f B respectively. Let P A and P B be the partial pressures No. of moles of ethyl alcohol = - 1.304
46
of vapours A and B respectively and total pressure P.
Mol. mass of methyl alcohol = CH 3 0H = 32
PA=fAP ... (i) . 40
PB == fB P ... (ii) No. of moles of methyl alcohol = = 1.25
32
PA X A p~ ... (iii)
~ X A' , mole fraction of ethyl alcohol ::: 1.304 = 0.5107
PB =X B pg .. , (iv) .. . 1.304 + 1.25
Equating eqs. (i) and (iii),
fA P=X A p~ ,X B', mole fraction of methyl alcohol 1.25
or
X A p~ PA 1.304+ 1.25
fA = 0.4893
P P
Similarly, equating eqs. (ii) and (iv), Partial pressure of ethyl alcohol X A .p~ ::: 0.5107x 44.5
. . 0
- X B PB =PB 22.73 mmHg
f B- -
P P Partial pressure of methyl alcohol = X B .p~ 0.4893 x 88.7 .
Thus, in case of ideal solution the vapour phase is richer with = 43.40 mmHg
more volatile component, i.e., the one having relatively greater Total vapour pressure of solution = 22.73 + 43.40
vapour pressure. = 66.13mmHg
Example 19. The vapour pressures of ethanol and, Mole fraction of methyl alcohol in the vapour
methanol are 44.5 mm and 88.7 mm Hg respectively. An ideal Partial pressure of CH3 0H 43.40 :=c 0.6:":63
.solution is formed at the same temperature by mixing 60 g of
Total vapour pressUre' 66.13
ethanol with 40 g of methanol. Calculate the total vapour
328 G.R.B. PHYSICAL CHEMISTRY FOR COMPETITIONS

Example 20. Two liquids A and B form an ideal solution. 25 = 0.25' n - 35 0.3
At 300K, the vapour pressure of a solution containing 1mole ofA 100 ' B - 114

and 3mole ofB is 550mm ofHg. At the same temperature, if one


0.25 0.3
mort! mole of B is added to this solution, the vapour pressure of xA = ; Xo=----
the solution increases by 10 mm of Hg. Determine the vapour 0.25+ 0.30 0.25+ 0.30
pressure ofA and B in their pure states. =0.45 =0.55
Solution: Let the vapour pressure of pure A be ::: p~; and o 0
p::: PAxA + POxB
the vapour pressure of pure B be p~ .
105.2x 0.45+ 46.8x 0.55
Total vapour pressure of solution (1 mole A + 3 mole B)
::: 47.34+ 25.74= 73.08 kPa
::: X A • p~ +X 0 • p2 [X A 11> mole fraction of
A and X 0 is more fraction of B]
1, 0 3 0
550 - PA + - Po 12. Mole fraction ofthe component A in vapour phase is Xl and the
4 4
mQle fraction of component A in liquid mixture is x" then
or 2200= p~ + 3PZ ... (i)
(p~ vapour pressure of pure A; p~ vapour pres;ure of
Total vapour pressure of solution (1 mole A + 4 mole B) pure B), the total vapour pressure of liquid mixture is:
1 0 4 0
::: PA + PE (a) p~ (b) p~ (c) P~ (d) P~ x2
5 5 XI X2 x2 XI
/ 1 0 4 0
560=- PA +- Po [Ans. (a)]
5 5 [Hint: PA = P~ x2' vapour pressure of ,A'
or 2800 p~+4p~ ... (ii) Mole fraction of A in vapour J!!L
Solving; eqs. (i) and (ii), Plotal

p~ =:= 600mm ofHg::: vapour pressure of pure B xt


P~ X
= - -z
p~ 400 mm ofHg ::: vapour pressure of pure A P
o
. Example 21. An aqueous solution containing 28% by P PA X2]
XI
mass ofa liquid A (mol. mass 140) has a vapour pressure of160
mm at 37°C. Find the vapour pressure of the pure liquid A (The 13. Vapour pressure of pure A (p~ )::: 100mm Hg
vapour pressure ofwater at 37°C is 150mm} . Vapour pressure of pure B (p~) 150mm Hg
Solution: For two miscible liquids, 2 mole ofliquid A and 3 mole ofliquid B are mixed to form an
P tolal Mole fraction of Ax p~ + Mole fraction of B x p~ ide!,1 solution. The vapour pressure of solution will be:
(a) 135 mm (b) 130 mm (c) 140 mm (d) 145 mm
- 28 [Ans. (b)]
No. of moles of A ::: - 0.2
140 2 3
[Hint: XA "5' XB =="5
Liquid B is water. Its mass is (100 - 28), i. e., 72.
No. of moles of B 72 ::: 4:0 P ::: P~ X A + P~ xB
18 2 3
== 100 x - + 150 x - =40 + 90 == 130]
Total no. of moles ::: 0.2+ 4.0= 4.2 5 5
Given, P tolal = 160 mm 14. The vapour pressure ofa certain pure liquid A at 298 K is 40 m
bar. When a solution of B is prepared in A at the same
pZ = 150mm temperature, the vapour pressure is found to be 32 m bar. The
- 0.2
So, 160 --xP1 0
+ 4.0 x 150 mole fraction of A in the solution is:
4.2 4.2 (a)05_.. . (b) 0.2 (c) 0.1 (d) 0.8
p~::: 17.15X4.2=360.l5mm [Ans. (d)]
0.2 . [Hint: P = pOxA
32 = 40xxA or xA =0.8]
Example 22. Heptane and octane form ideal solution. At
15. 100 mL of liquid A and 25 mL of liquid B are mixed to form a
373K, the vapour pressures of the two liquids are 105.2/cPa and
solution of volume 125 mL. Then the solution is:
46.8/cPa respectively. What will be the vapour pressure, in bar,of .
--(a) ideal
a mixture of25g olnepfaneand 35 gO! Delane? ~-.
(b) non-ideal with positive deviation
Solution: (A) Heptane C7 H 16 mA = 100 (c) non-ideal wit!). negative deviation
(B) Octane C 8 H I8 mE 114 (d) cannot be predicted
[Ans. (a)]
[Hint: ~ Vmix == 0, hence the solution is idea\.]
SOLUTIONS 329

16. The vapour pressure of pure benzene at 88°C is 957 mm and


that of toluene at the same temperature is 379.5 mm. Calculate Vapour
'the composition of benzerie-toluene mixture boiling at 88°C: , T Liquid
(a) xbenzene 0.66; Xtoluene 0.34
(b) xbenzene = 0.34;. Xtoluene =; 0.66 -c:
iT: --- ----------
II

(c) x benzene = X toluene = 0.5 '0


a. Tx
I
---+------~--!-.--,---'~
(d) x benzene 0.75; Xtoluene =; 0.25 2? Tz ---1-------+--
._ I :
[Ans. (a)] =0 I I

[Hint: p =pgenzene xbenzene + P~luene Xtoluene III :


I
I
:
I
I
760 = 957· xbenzene + 379.5(1- xbenzene) A 100% Y x Y1 XIZj B 100%
Xbenzene = 0.66
B 0% Composition A 0% •
and xtoluene I - 0.66 0.34 ] •
17. At 2S'C, the total pressure of an ideal solution obtained by
mixing 3 mole of A and 2 mole of B, is 184 torr. What is the Fig. 5.4 Liquid-vapour equilibrium diagram
vapour pressure (in torr) of pure B at the same temperature of a solution of type-I
(Vapour pressure of pure A at 25°C is 200 torr.)?
Suppose a solution of composition x is heated. When the
(CET (J&K) 20061
(a) 180 (b) ,160 (c) 16 (d) 100 temperature Tx is reached, the boiling will start. At this
[Ans. (b)] temperature, the vapour coming off from x will have the
3 3 composition Xl • Since, Xl is richer in B than X, the composition of
[Hint: XA 0.6 the residual liquid will become richer in A. Let the new
nA + nB 3+ 2 5
, composition be y. This liquid cannot boil at temperature Tx but
x B = 0.4 will require higher temperature Ty . The vapour coming off at
P = P~XA + P~XB temperature Ty will also be richer in B as represented by YI'
\ 184 200 x 0.6 + p~ x 0.4 Hence, the composition of the residue will again be enriched in
, 1,.'
pZ x 0.4
184 = 120 + A. Thus, if the process is allowed to continue, the boiling point of
solution will go on increasing towards TA , the boiling point of
o 64
PE =- = 160 torr] pure liquid A. At the same time the solution becomes more and
0.4
more rich in A. If the process is continued for a sufficiently long
18. The mass of glucose that would be dissolved in 50 g of water
time, pure liquid A can be obtained.
in order to produce the same lowering of vapour pressure as is
produced by dissolving 1 g of urea in the same quantity of Now, if the initial vapours are condensed, the solution
water is: (KCET 2006) obtained wilt"have the composition XI and boils at temperature
(a) 1 g (b) 3 g (c) 6 g (d) 18 g T z . Evidently, the distillate is richer in B than before. If the
[Ans. (b)] process of condensation and redistilling is continued, the final
distillate will be pure B component.
[Hint: . (~P)glucose =(~P)urea
Thus, the two liquids forming a solution of type-I can be
(XB )glueose = (xB )urea separated by fractional distiUa~on.
i.e, ( nE) (nE)
nA glucose \. nA urea
Solution of type-II (Positive deviations from Raoult's
law): The boiling temperature-composition curves for the
WE 18 1 X 18 liquid and vapour phases have been shown in Fig. 5.5. The
-x two curves meet at a minimum point C where the liquid and vapour
50 180 50 x 60
phases have the same composition. The liquid mIxture at point C
WE = 3 g]
will boil at ,constant temperature T without undergoing any
5!7 THEORY OF FRACTIONAL DISTILLATION
The process of separation oCone liquid from 'a"nother liquid
(binary mixture) having different boiJing points by distililltionis
termed ,fr.actional distillation. The information, whether a
partlcul;r solution of two liquids can be separated by distillation
or not, is provided by the study of liquid-vapour equilibrium
diagrams at· constant pressure, say atmospheric pressure. The
separation is possible only when the vapour phase has a A 100% X X1 Y1 Y B 100%
composition different from that ofthe boiling liquid mixture. B 0% Composition A 0%
Liquid pairs of type-I (Ideal solutions): The boiling

temperature composition curves for liquid and vapour phases in Fig. 5.5 Boiling point-composition curves
the case of binary solutions of first type are represented in Fig. in bil'!ary solution of type-II
5.4.
330 G.R.B. PHYSICAL CHEMISTRY FOR COMPETITIONS

change in composition. snch a mixture which boils at constant It is never possible to separate a mixture completely into the
temperature and distils over completely at the same pure components A and B. It mainly gives a constant boiling
temperature without any change in composition, is called mixture (azeotropes) which can never be separated by
constant boiling mixture or azeotropic inixture. distillation. .
Consider the distillation of a mixture of composition x. , There are several liquid pairs which form maximum boiling
The vapour given off has the composition xl' The composition of point azeotrope. Some examples are tabulated below:
residual liquid will shift towards A. In the mean
% Composition Boiling point
time thecompositioRofthe distillate shifts towards C. Ultimately Mixture
of azeotrope (pressure .. 1 atm)
by repeated fractional distillation, the mixture of composition C ---.-~-----.~-----~---.~-'---

will be obtained as distillate and pure liquid A will be left as 1. Nitric acid-Water 68% Nitric acid 125.SoC
residue. It will never be possible to have pure B. 2. Acetic acid-Pyridine 65% Pyridine 139.0°C
When a mixture of composition y is distilled, th~pour given 3 .. Chlorofonn-Acetone 80% Chlorofonn 65.0°C
off has the coml?,osition Yl'j. e., the composition~ residual
liquid will Shifttowards R Ultimately the mixture of composition 4. Hydrogen chloride- 79.8% Water 108.6°e
Water
C will be obtained as distillate and pure liquid B will be left as
residue. It will nev~r be possible to have pure A.
There are several liquid pairs which form minnnum boiling 5.8 SOLUTIONS OF SOLIDS IN LIQUIDS
point azeotropes. Some examples are given in the table:
Solutions of this type are most common. In solutions of solids in
Table 5.3 Some Azeotropic,Mixtures liquids, the liquid is invariably referred to as a solvent and the
solid dissolved in it as the solute. If a solute is added in small'
% Composition of Boiling point
Mixture amounts at a time to a given amount of a solvent at a constant
azeotrope (pressure = 1 atm)
--..-- temperature, with vigorous stirring or the solvent after each
1. Water-Ethanol 95.97 Ethanol 78,15°C addition, a stage is reached when the added solute no more
2. Pyridine-Water 57.00 Pyridine 92.60°C disappears, i. e. ,goes into solution but remains undissolved. The
solution is then said to be saturated. A solution which remains in
3. . Ethanol-Benzene 32.40 Ethanol 67.80°C
contact with undissoived solute is termed as saturated. It can
4. Acetic acid-Toluene 28,00 Acetic acid 105.40°C also be defined as one which is in eqUilibrium with the excess
Solutions of type-UI (Negative deyiations from Raoult's of solid at a particular temperature.
law): The boillng temperature-composition curves for the The amount of solute dissolved in 100 g of a solvent to form a
liquid and vapour phases have been shown in Fig. 5.6. The curves saturated solution at a given temperature is termed the solubility
meet at point C. At this point, both liquid and vapour phases have of the solute in the given solvent at that temperature. Each
same composition, The constant-boiling mixture has maximum substance has a characteristic solubility in a given solvent at a
boiling point. de.finite temperature.
When a solid is added to the solvent, the particles from the
solid diffuse into it. The solute. and solvent molecules move
c
t constantly in the solution phase. Some of the particles of the
solute retJ,lm to the solid state due to collisions. Thus, two
e
'0
A
opposite processes operate simultaneously.
c.. k,.. :
Cl
.5 ! 9(qcy, (a) Dissolution: Particles of solute leaving the solid and
'5 ,, ,,
I ,
dissolving in the solvent.
co (b) Recrystallisation: Solute particles returning to the solid
'Y 'Y1
A 100% form. .
80% -
Composition 8 100%
AO% When these two processes move with same speed, an
equilibrium stag~ is reached,
Fig. 5.6 Boiling temperature-composition
. curves in binary solution of type-III Solute (solid) ~ Solute (dissolved)

Consider the distillation of a mixture of composition x. The Thus, a dynamic equilibrium exists in a saturated solution.
vapour coming off is richer in A as indicated by composition When a saturated solution prepared at a higher temperature is
Xl . The composition of the residual liquid shifts towards C. As the
cooled, it gives a solution which contains usually more of solute
distillation proceeds, the composition Of the distillate moves than required for the saturated solution at that temperature. Such
towards A and that of residue towards C. a solution is referred to as a supersaturated solution. It is
Similarly, a mixture of composition lying between Band C, usually unstable and changes to saturated solution when excess
say y, on distillation will give vapour richer in B as indicated by of solute comes out in solid state. '
composition Yl' The composition of residual liquid shifts The: following factors influence the solubility of a solute in a
towards C and di'stillate towards A on repeated distillation, liquid:
SOLUTIONS

(1) Nature of solute,


200 ~;o"'li
(2) Nature of solvent,
V
~Ii . /VI
(3) Temperature. 190 ~-'<:/ ~
180
i
I i
/"" I

f-~
1. Nature of solute: The solutes (solids) can be classified as II Vi
ionic and non-ionic solids. The ionic solids consist of positively 170
~V ~rr;
and negatively charged ions. It is the force of attraction between 160 r-
/ ./ V I i

the ions, L e., lattice energy which opposes the tendency of a 150
I /~ 1.0,
solute to dissolve. This force of attraction is different in different ~ 140
j/ ..:.-
f?1- t
V
~ 130
I ~l
ionic solids depending on the charges present on the ions and if I
C)
120 i d
distance between ions (ionic radii). The ionic solutes having high 0
~'"A / ' V
lattice energy like BaSO 4 are less soluble while those having less
0

Q;
110
100
~,. V L 'f:,.'O{ ____.....-
lattice energy have more solubility. The ions are solvated by the ,c. I. /1 / ....- ~
90
solvent molecules and in this process energy (known as hydration ~ ~f?/ J.:--
80
70 ~w~
energy) is released. When the hydration energy is high, the ionic :0
:::J --/ ,
~
~

~/1'
(5
s'olid is more soluble. CIl
60 V ' ...... ~a2so~ All
Many non-ionic substances dissolve in polar solvents due to
50
1<..,9.C =?
hydrogen bonding. Generally, if the solute and solvent have
__ ';c,rA. ~ .- ~ NaCI
/"
similar characteristics, i. e. , both are polar or both non-polar, the 40
30 1~~~2~O4- ,---

~ftf
solubility is high and if both are dissimilar, the solubility is found
low. 20 ,--,'"
Na~'e,oy
10
2. Nature of solvent: Ionic solids dissolve to a larger extent Ft-1
in a solvent having a high dielectric constant as compared to 10 20 30 40 50 60 70 80 ,90 100
solvents oflow dielectric constants. Dielectric constant of water Temperature in degree celsius
is 80 while that of methyl alcohol is 33.5. An ionic solid,
therefore, dissolves more readily in water than in methyl alcohol. Fig.5.7 Solubility curves of some compounds
Benzene has a very low dielectric constant of 2.3 and, hence, 1. Mass of empty dish w g
ionic solids do not dissolve in benzene. 2. Mass of dish + solution = WI g
For non-ionic solids, the guiding principle is 'like dissolves 3. So, Mass of solution (wI - W) g
like', Le., if the solvent is polar, it will dissolve the polar solutes 4. Mass of dish + residue ~ W2 g
and if it is non-polar, it will dissolve the non-polar solutes in it. So, Mass of residue = (w2 w) g x g
3. Temperature: The solubility of a solute in a given and Mass of solvent (Mass of solution
solvent varies appreciably with temperature. A few curves drawn - Mass of residue )
between solubility in water and temperature are given in Fig. 5.7. = (WI - W) - (W2 - W)
It is observed that the solubility of NaCI increases' very = (WI W2)= yg
slightly with an increase in temperature whereas those of
KN0 3 ' NaN0 3 , AgN03 and KI, etc., increase greatly. A Thus, the solubility of salt = x x 100 g per 100 g ~f solvent.
y
sharp break in a solubility curve indicates the formation of
a compound whose ~olubility is different from that of the Example 23: 50 g of a saturated aqueous solution of
substance from which it has been formed as in the case of potassium chloride at 30°C is evaporated to dryness. when 13.2 g'
N a 2 SO4 ·lOH2 O. It losses its water of crystallisation at ofdry KCf was obtained. Calculate the solubility ofKCf in water
32.3°C and is converted into anhydrous form. There are few at 30°C.
substances like calcium acetate, cerium sulphate, calcium Solution: ' Mass of water in solution (50-13.2) 36.8g
chromate, etc., which show a decrease in solubility with rise in
temperature. Solubility of KCl = Mass ofKCl x 100 13.2 x 100= 35.87
. Mass of water 36.8 g
Generally, solubility depends on heat of solution. If a
substance dissolves with absorption of heat, the, solubility Example 24. How much copper sulphate will be required.
increases with rise of temperature. On the other hand, if a to saturate 100 g of a dilute aqueous solution ofCuSO4 at 25° C
substance dissolves with evolution of heat, the solubility if 109 of the dilute solution leave on evaporation and dlying 1.2
decreases with rise of temperature. g of anhydrous CuSO4? The solubility ofCuSO4 in water at 25° C
Determination of solubilil); The solubility of salts is is 25.
generally determined by gravimetric method. First of all a Solution: 100 gof dilute solution of CUS04 contain
saturated solution is prepared. Some part of this saturated = 1.2 x 10= 12.0g CUS04
solution is weighed out in a porcelain dish. The solution is Mass of water present in dilute solution
evaporated slowly to dryness ona sand bath. The dish is cooled = (100-12) 88g
and weighed again. The observations are recorded as follows: To saturate 100 g of water, CuS0 4 required 25 g
332 G.R.B. PHYSICAL CHEMISTRY FOR COMPETITIONS

So, to saturate 88 g of water, CuSO 4 required ~x88 termed lowering in vapour pressure and the ratio [ Po ~P S ] is
100
=22g
termed relative lowering in vapour pressure.
Thus, the mass of CuSO 4 to be added to 100 g of dilute
solution to saturate it (2212)=IOg Raoult, in 1886, established a relationship between relative
lowering in vapour pressure and composition of the solution after
5.9 COLLIGATIVEPROPERTIES OF a series of experiments in various solvents. The relationship is
DILUTE SOLUTIONS known as Raoult's law. It states that the relative lowering in
A dilute solution is one in which the amount of the solute is vapour pressure oCa dilute solution is equal to mole fraction
very small in comparison to the amount of the solvent. The of the solute present in the solution.
dilute solutions show more or less ideal behaviour as the heat and If n moles of solute be dissolved in N moles of the solvent, the
volume changes, accompanying the mixing of solute and solvent, mole fraction of the solute will be _n_ .
are negligible for all practical purposes. Dilute solutions obey n+N
Raoult's law, . Po - Ps n
Dilute solutions containing non-volatile solute exhibit some Accordmg to Raoult's law, = -'--
special properties which depend only upon the number of solute Po n+N
particles present in the solution irrespective of their nature. These This is the mathematical expression for Raoult's law.
properties are termed as colJigative properties. The colligative [Modified form of Raoult's law: The above relationship
properties are: can be written as,
(i) Lowering in the vapour pressure, n+N N
(ii) Elevation inthe boiling point, -=-=--=--=1+-
Po - P s n n
(iii) Depression in the freezing point and
(iv) Osmotic pressure. Po _ 1 =N or --""-"-_ == N .
or
Colligative ·properties are the properties of dilute solutions, Po Ps n Po - p, II
that is why these are termed as colligative properties of dilute
solutions. These properties are related to one another. Thus, if po-ps=n WAX~
or
one is measured, the other can be calculated. The importance of Ps N rnA wB
these properties lies in the fact that they provide methods for the
This equation gives accurate results and is easy to apply.]
determination of molecular masses of dissolved solutes. The
results are excellent if the following three conditions are Derivation of Raoult's Law for a Dilute Solution
satisfied: When a non-volatile solute is dissolved in a volatile solvent, a
(i) The solution should be very dilute. fraction of the surface of solvent is blocked by solute molecules
(ii) The solute should be non-volatile. where no evaporation occurs. Thus, under similar conditions, the
(iii) The solute does not dissociate or associate in sQlution. , vapour pressure is decreased. The vapour pressure of the
solution, thus, depends, upon the number of molecules of the
5~10 LOWERING IN .THE VAPOUR solvent present on the surface of the solution. The number of
PRESSURE such molecules is proportional to mole fraction of the solvent.
So, the vapour pr~ssure of solution,
When a non-volatile solute is added ,to a solvent, the vapoUr
pressure is lowered due to the following reasons: 'Ps'oc~
(i) Percentage surface area occupied by the solvent decreases. n+N
Thus, the rate of evaporation and vapour pressure decreases. The " N
solute molecules occupy the surface, and so the per cent surface or Ps k · - -
,area occupied by the solvent decreases. -n.¥N·
(ii) According to Graham's law-G}.evaporation, (k is proportionalityfaetor)
. 1 For pure solvent, n=0
Rate 0 f evaporatIOn oc ~
"density N
When a non-volatile solute is dissolved in a liquid, its density
and hence, Po =k == k '" (ii)
O+N
increases. Thus, both rate of evaporation and vapour pressure are Putting the value of kin eq. (i)
lowered. N
If Po is the vapour pressure of pure solvent and Psis the Ps =Po--
n+N
vapour pressure' of the solution, the 'difference (Po - P s ) is

Note: Isopiestic solutions: Two solutions having same vapour pressure at same temperature are known as isopiestic solutions.
SOLUTIONS 333

N Loss in mass in the solution bulbs oc P s


or :::--
Po n+N Loss in mass in the solvent bulbs ex: ( Po - P s )
Total loss in both sets of bulbs oc [ P s + ( Po Ps)]
or 1-.!!..£==1-~ oc Po
Po n+N
Total loss in mass of both sets of bulbs is equal to gain in mass
or Po Ps ==_n_ ... (iii) of guard tubes .
Po n+N Po -: P s Loss in mass in solvent bulbs
Thus == ---~-----~-------
This is Raoult's equation. 'Po Total loss in mass in both sets of bulbs
If a solution is made by dissolving WB g of solute (molecular Loss in mass in solvent bulbs
mass mB) in wAg of the solvent (molecular mass mA)' the mole :::--------------------
Gain in mass of guard tubes
fraction of the solute will be
Further, we know from Raoult's law,
Po Ps == ____-=:..~_"'____
Po WA / mA + WB / mB
WA + WB
Loss in mass of solvent bulbs
mA mB
Gain in mass of guard tubes WA /mA + W B / mB
If the solution is very dilute, wB can be neglected in the The above relationship is used for calculation of molecular
mB
. masses of non-volatile solutes.
denominator as compared to WA • The eq. (iii), thus, becomes For very dilute solutions, the following relationship can be
mA applied:
PO-Ps==WBxmA ... (iv) Po P s Loss in mass of solvent bulbs == WB xmA
Po mB XWA Po . Gain in mass of guard tubes mB x WA
This relationship is useful in the determination of the
molecular mass of dissolved solute by measuring relative Vapour Pressure of Liquid and Temperature
lowering of vapour pressure. Liquid ~ Vapour Mf yap. > 0
Vapour pressure of liquid increases with temperature. Here,
Measurement of Relative Lowerjng in Vapour Mf is enthalpy of vaporisation. A 'graphical representation of
Pressure (Ostwald and Walker Method) vapour pressure versus temperature is given below:
The apparatus used is shown in Fig. 5.8. It consists of two sets
of bulbs. The first set of three bulbs is filled with solution to half
of their capacity and second set of another three bulbs is filled
with the pure solvent. Each set is separately weighed accurately.
Both sets are connected to each other and then with the accurately
weighed set of guard tubes filled with anhydrous calcium r
chloride or some other dehydrating agents like P20S' conc.
H 2S04 etc. The bulbs of solution and pure solvent are kept in a
thermostat maintained at a constant temperature.
Dry air
Temperature - - 1/T - -
Fig. 5.9 Variation of vapour pressure with temperature
If vapour pressure of a liquid is known at a temperature; it can
be calculated at another temperature using Clausius-Clapeyron
equ'ation:
S91ution Solvent Weighed CaCl2 tubes

Fig. 5.8 Ostwald and Walker method


P2 )
loglo ( PI
[I 1]
Mf yap.
== 2.303R TI - T2
A current of pure dry air is bubbled through the series of bulbs Here, PI == Vapour pressure at temperature ~
as she'.'''' in Fig. 5.8. The air gets saturated with the vapours in
P2 == Vapour pressure at temperature T2
each set of bu:bs. The air takes up an amount of vapours
proportional to th.., vc.;Jour pressure of the solution first and then : :::::a_SOME SOLVED EXAMPLES\r.:::::
it takes up more amount of vapours from the solvent which is
proportional to the difference in the vapour pressure of the Example 25. Calculate the vapour pressure lowering
solvent and the vapour pressure of solution, i. e. , Po Ps' The caused by addition of 50 g of sucrose (molecular mass 342) to
two sets of bulbs are weighed again. The guard tubes are also 500 g ofwater if the vapour pressureofpure water at 25°C is 23.8
weighed. mmHg.
334 G.R.B. PHYSICAL CHEMISTRY FOR COMPETITIONS

Solution: According to Raoult's law, Example 28. What mass of non-volatile' solute· (urea)
Po - p~ n needs to be dissolved in 100 g of water in order to decrease the
Po n+N vapour pressure of water by 25%. What will be the molality of
n solution? . (liT 1993)
or I1p=--· Po Solution: Using Raoult's law in the foHowing form,
n+N
50 '500 . Po - Ps wM
Given: n - = 0.146; N = = 27.78 and Po = 23.8
342 18 Ps Wm
Substituting the values in the above equation,
0.146 If Po = 100mm, then Ps 75mm
I1p = x 23.8 0.124 mm Hg
0.146+ 27.78 100 75 wX 18
Example 26. The vapour pressure of pure benzene at a ' 75 100x 60
certain temperature is 640mm Hg. A non-volatile solid weighing.
2.l75g is added to 39.0gofbenzene. The vapour pressure of the w=ll1.1
solution is 600 inm Hg. What is the molecular mass of the solid , w x 1000 111.1 x 1000
substance? (liT 1990; M~NR 1992) M01alty
. I =--- 18.52 m
mxW 60x 100
. Solution: According to Raou1t's law,
Po Ps =_n_ Example 29. A current of dry air was bubbled through a
Po n+N . bulb containing 26.66 g of an organic compound in 200 g of
water, then through a bulb at the same temperature, containing
Let m be the molecular mass of the solid substance.
water and finally through a tube containing anhydrous calcium
n=2.175· N 39 0.5 chloride. The loss of mass in bulb containing water was 0.087 g
m' 78
and gain in mass of the calcium chloride tube was 2.036 g.
(molecular mass of benzene 78) Calculate the molecular mass of the organic substance.
Po = 640 rom; Ps = 600 mm
Substituting the values in above equation,
Po Ps Loss in mass bulb
Solution:
2.175 Po Gain in mass of CaCl 2 tube
640 600 2.175 0.087
640 2.175+ O.5m 2.036
m
m 2.175 x 16-2.175 65.25 Let the molecular mass of the organic substance be m
According to Rao'ult's law,
0.5
Example 27. A solution containing 30 g of a non-volatile Po Ps
"'--"-~=---
wlm
solute in exactly 90g of water has a vapour pressure of21.85mm 'Po w W
-+
of Hg at 25°C. Further, 18 g of water is then added to the m M
solution; the new vapour pressure becomes 22.15 mm of Hg at 26.66
25°C. Calculate (a) molecular mass of the solute and (b) vapour 0.087 m =____
26.66 _
pressure of water at 25°C. (~llNR [<)90)
2.036 _26_.6_6 + _20_0 26.66+ 200 m
Solution:. Let the vapour pres~ure of water at 25°C be Po
m 18 18
and molecular mass of the solute be m
Usirig Raoult's law in the following form, m= 53.75
Po - Ps wM Example 30. The vapour pressure of a dilute aqueous
Ps Wm solution ofglucose is 750mm Hg at 373K. Calculate (i) molality,
(ii) mole fractio.n of the solute.
For solution (I), (Po - 21.85) 30x 18
21.85 90x m
... (i) solution: Po = Vapour pressure of water 373 K = 760 at
mmHg
' (II) (Po -22.15) 30Xl.8
. IutlOn
For so , = --- ." (ii) Using Raoult's law in the following fonn,
22.15 108 x m
Dividing eq. (i) by eq. (ii), Po Ps wM
(Po 21.85) 22.15 108 6 Ps Wm
--'-"--"----- x - - - - -
21.85 ( Po - 22.15) 90 5 760- 750
or ~x18
Po = 23.87 mm of Hg 750 Wm
Substituting the value of Po in eq. (i) w 10
or
m= 67.9 Wxm
SOLUTIONS 335-

'. w lOx 1000


Molahty==--xIOOO== ==0.74m 1.0 ~x1000
Wxm 750x 18 mxW
Ps == Mole fraction of solvent x Po; or ~==~ 0.001
mx W 1000
Mole fraction of solvent 750
760 Applying Raoult's law for dilute solution,
. r 750) Po - Ps ~xM
So, Mole fraction of solute = ll- - = 0.0132 (M 18)
760 Po mXW
Example·31. Calculate.the vapour pressure ofan aqueous 760- .
0.001 x 18 (Po == 760mm at 100°C)
solution which contains 5 mass per cent of urea. The vapour 760
pressure ofpure water is23.5mm Hg. Molar mass of urea is 60. or Ps == 760-760x O;OOlx 18
Solution: Mass of urea == 5 g == 760-13.68
Mass of water (100 - 5) = 95 g == 746.32mm
Example 34. The vapour pressure of pure benzene at
No. of moles of urea = ~ = 0.083 50°(' is 268 mm ofHg. How many moles ofnon-volatile solute per
60
mole of benzene are required to prepare a solution of benzene
No. of moles of water 95 = 5.278 . haVing a vapour. pressure 167 mm ofHg at 50°C?
18 Solution: Applying Raoult's law in the following form:
.Total number of moles = 5.278 + 0.083 Po - Ps == wM wi m
5.361 Ps Wm WIM
. f I 5.278
M o Ie fr action 0 so vent = - - = No. of moles of solute per mole of benzene
5.361
Ps == Mole fraction of solvent x Po or n (268 - 167) 0.6047"" 0.605 .
N 167 .,
== 5.278 x 23.5 == 23.14 mm
5.361 Alternative method: We know that, Ps = Mole fraction of
Example 32.109 ofglucose (molar mass 180) and 20 g of solvent x Po
sucrose (molar mass 342) are dissolved in 100 g of water. What or 167 == Mole fraction of solvent x 268
will be the vapour pressure of the resultant solution if the vapour Mole fraction of solvent 167 0.623
So,
pressure of water is 35 mit! Hg? , 268
Solution: Mass of glucose == 109 Mole fraction of solute 1- 0.623 = 0.377 .
No. of moles of glucose 0.0556
n Mole fraction of solute 0.377 0.605
Mass of SUcrose == 20 g N Mole fraction of solvent 0.623
No. oemoles of sucrose 0.0585
.Mass of water 100g
No. of moles of water = 5.556 19. The vapour pressure of water at 23°C is 19:8 mm. 0.1 mole
Total number of moles 5.556+ 0.0585 + 0.0556 glucose is dissolved in 178.2 g of water. What is the vapour
/
pressure (in mm) of the resultant solution?
=5.67
(a) 19 (b) 19.602 (c) 19.402 (d) 19.202
'. f . 5.556 [Ans. (b)]
M oI e fractIon 0 water == _.-
5.67
[Hint: nB= 0.1, nA ~ 178.2 = 9.9
Vapour ptessureof solution = Mole fraction of water x Po 18
5.556 x 35 == 34.3 mm Hg 9.9 =0.99
5.67· + 0.1
Ex..::!,!,1p 33. Calculate the vapour pressure ofan aqueous P poxA
solution of1.0mu?al plucose solution at 100°e. 19.8 x 0.99 = 19.602 mm]
Molality _w_ x 1000 20. What is the vapour pressure of the solution containing 34.2 g
Solution:
rro).: W of sucrose per 100 g of water at 2Y'C? The vapour pressure of
water at 2Y'C is 23.75 mm.
where, w == mass of solute in grams; (a) 20.3 mm (b) 23.10 mm
. W== mass of solvent in grams (c) unpredictable (d) 23.33 mm
336 G.R.B. PHYSICAL CHEMISTRY FOR COMPETITIONS

[Ans. (d)] vapour pressure of water at 25°C is 17.5 rnm, what is the
molecular mass of the solute? [EAMCET (Engg) 2008j
[Hint: nA = = 100 = 5.55 (a) 206 (b) 302 (c)318 (d) 276
rnA 18
[ADS. (c)]
n 34.2 0.1 [Hint:
B .342 100/18
(17.5 0.225)= 17.5 x----~
x = 5.55 = 5.55 0.982 100/18+ -
A
5.55 + 0.1 5.65
p= poxA 17.275 (100 +~) 100
23.75 x 0.982 23.33 rum] 17.5 18 mB 18
21. Lowering of vapour pressure due to a solute in I molal
aqueous solution at 1000e is: 0.987 ( 5.55 + !:) 5.55
(a) 13.44 mm Hg (b) 14.12 mm Hg 25
(c) 31.2 mm Hg (d) 35.2mmHg 5.55 + - = 5.628
mB
[Ans. (a)]
mB '" 318mm]
x 1000
[Hint: m XB = mole fraction of solute
(l-xB)m A
rnA molar mass of solvent
5~11 ELEVATION OF BOILING POINT
1= x 1000 (Ebullioscopy)
(I xB)xl8 The boiling point of a liquid is the temperature at which its
XB 0.0176 vapour pressure is equal to the atmospheric pressure. The
vapour pressure of a liquid is lowered when a non-volatile solute
xA = 1- 0.0176= 0.9824 is added to it. Hence, the temperature of the solution when its
P = poxA vapour pressure will be equal to atmospheric pressure will be
= 760 x 0.9824 = 746.62 higher than the temperature of the pure solvent. In other words,
the boiling point of the solvent is elevated by the addition of
tlp = Po P = 760 746.62
non-volatile solute. The difference in the boiling point of the
'" 13.4 ] solution and the boiling point of the pure solvent is termed
22. The mass of a non-volatile solute (molecular mass = 40) elevation of boiling point.
which should be dissolved in 114 g octane to reduce its vapour Elevation of boiling point,
pressure to 80% will be: (t1T) Boiling point of the solution - Boiling point of pure
(a)20g (b)30g (c) 109 (d)40g solvent
[ADS. (e)] This can be better understood by plotting a graph of vapour
[Hint: If Po 100, then P 80
. pressure against temperature for a pure solvent and two solutions
of different concentrations. The curves orthe solutions always lie
P= Po XA below the curve of th,e pure solvent. The line poe represents the
80 = 100 xXA atmospheric pressure. To', TI and T2 represent the boiling points
xA =0.80 of pure soJvent, solution I and solution II respectively. The
114/114 vapour pressure of pure solvent, solution I and solution II at'
1141114 + wB /40 temperature To are Po, p] and P2 respectively.
'0,8=---
1 + Ws /40

1 + Ws = 1 t
40 0.8
wB 10 g]
. 23. 3g urea is dissolved in 45g of water. The relative lowering of
vapour pressure is : IComed (Karnataka) 20081
(a) 0.05 (b) 0.04 (c) 0.02 (d) 0.01
[Ans. (c)]
3 To T1 T2
(HiDt: ns = 0.05; 45 = 2.5
60 18 Temperature :......-

Xs = 0.05 0.01 96 = 0,02 ]


Po 2.5+ 0.05 Fig. 5.10
24. When 25g of a non-volatile solute is dissolved in lOOg of
water, the vapour pressure is lowered by 0.225 mm. If the
I
SOLUTIONS 337

Assuming that the solutions are very dilute, these curves may K
Then, I1T=-==K b
, be approximately taken as straight lines near the boiling point. lOOO
Thus, AA CE and AABD are similar.
. AC AE K b is called molal elevation constant. II 'J:; defined as the
Therefore, --- elevation in boiling point produced when 1 mole of the solute
AB AD
T2 -To == is dissolved in 1000 g of the solvent.
or
T\ -To Po PI Thus, K 1000K b
I1T2 11P2
or Putting this value in eq. (i),
1111 I1Pj
or I1T ex: I1P I1T == 1000K b WA
... (iii)
From Raoult's law for dilute solution, mA WB.

Po - Ps _ WB xm A or I1T == Molality x K b
Po .,mB XWA

( P s == vapour pressure of solution)


WB xm A
(since
" mB
WB
x wA
x 1000 == molality I
or Po Ps == . Po
mB xWA
The elevation in boiling point of a solution o.
For the pure solvent, Po (its vapour pressure at the boiling non-electrolYte is propo!tional to its molality and equimolal
point) and mA (its molecular mass) are constant. Therefore, solutions of all the sUbs'tances in the same solvent will show
equal elevation in boiling points. These are known as Raou!t's.
Po - Ps ex: ''----'''---
mB xWA laws of elevation of boiling point.
Molal elevation constant is characteristic of a particular
or I1p ex: 11 T ex: - - - ' ' - - - - solvent and can be calculated from the thermodynamical
mB xW A
relationship.
RT2
or I1T K· ---"'-- ... (i) K - b
mB XWA b - 1000L
v
where K is a constant, called as elevation constant. where, R is molar gas constant, Tb is the boiling point of the
When, WB == 1, (one mole of solute) and WA == 1g, then solvent on kelvin scale and Lv the latent heat of vaporisation of
mB solvent in calories per gram. '
, ?
I1T==K For water K =- 2x(373t = 0.515
Thus, boiling point constant is equal to the elevation in boiling b lOOOx 540
point which would be theoretically produced when I mole of a
·The molal elevation constants for some common solvents are
non-volatile solute is dissolved in I g of the solvent.
given in the following table:
If WB == 1 and WA = 100 g,
mB

I1T=- K K l1IWII cIrtwiIdtiI C'tt" u-


Then, I
s.huIt 8fIt.{"q
lOO (K. . . . .-l)
K is called molecular elevation constant. It is defined as the
I Water 100.0 0.52
elevation in boiling point produ~ed whtn.l mole of the solute Acetone 56.0 1.70
is dissolved in 100 g of the solvent,·; Chloroform 61.2 3.67
Thus, K lOOK I. Carbon tetrachloride 76.8 5.02
B'enzene 80.0 2.70
Putting this value in eq. (i),
l'l::hyl alcohol 78.4 1.15
I1T== lOOK '.-~ ... (ii)
If K b' W A ,I1T. and W B are known, molecular mass of a
non~volatile solute can be determined.' The most convenient
WB
If - =1 and w A == 1000 g, method for the determination of I1T in the laboratory is the
mB t:nu{sberger method.
* This relation may also be written as:
/:iT 1000 xw
mXW
where, wand Ware masses of solute and solvent respectively and m = molecular mass of solute.
I
338 G.R.B. PHYSICAL CHEMISTRY FOR COMPETITIONS

5.12 DEPRESSION OF FREEZING POINT or


(Cryoscopy)
Freezing point of a substance is defined as the temperature at For the pure solvent, Po and mB are constants. Therefore,
which the vapour pressure of its liquid is equal to the vapour
pressure of the corresponding solid. Since, the addition of a Po - Ps oc_....!O-_
mB w A
non-volatile solute always lowers the vapour pressure of a
solvent, therefore, it will be in equilibrium with solid phase at a or Apoc--=--
lower pressure and hence at a lower temperature. The difference mB w A
between the freezing points of the pure solvent and its solution is WB
called depression of freezing point. or Ap ex; AT oc - - " - -
mB wA
Depression of freezing point
or AT K ._"'-- ... (i)
(AT) == Freezing point of the solvent mB wA
Freezing point of the solution where, K is a constant, called depression constant.
This can be better understood by plotting a graph of vapour
pressure against temperature for a pure solvent and two solutions, When, WB I (one mole of solute) and W A == I g
mB
solution I and solution II. CFB is a curve for a solid solvent. The
solvent, solution I and solution II vapour pressure curves meet AT=K
CFB curve at points B, F and C respectively. Thus, To , Tl and T2 Thus, depression constant is equal to the depression of the
are the freezing points of pure solvent, solution I and solution II freezing point which would be theoretically produced when one
respectively. The vapour pressures at temperatures To ,~ and T2 mole of a non-volatile solute is dissolved in I g of the solvent.
for solvent, solution I and solution II are, thus, Po, PI and P2
respectively. If wB = land Ws = 100g,
For very dilute solutions, the curves FD and CE are almost , mB

straight lines and CB is also nearly a straight line. The ABEC and
K
ABDF are similar. AT
100
EC BE
So,
DF BD =K'

.or To T2 = Po P2 K' is called molecular depression constant.


To -Tl Po -PI It is defined as the depressiun of freezing point produced
when 1 mole of the solute is dissolved in 100 g of the solvent.
AT2 AP2
or --=--
,Thus, K == lOOK'
or Putting this value in eq. (i),
AT == lOOK' X
... (ii)
,mB xW A

WB
If - == I and W A = 1000 g
mB
K
AT
1000
Kf
Kf is called molal depression constant.
T Z T1 To
It is defined as the depression of freezing point produced
Temperature -
when 1 mole of solute is dissolved in 1000 g of the solvent.
Fig. 5.11 Thus, K == 1000Kr
. From Raoult's law for dilute solurions, Putting this value in eq. (i),
Po - Ps wB. mA
Po mB w A AT == 1000Kf -=-- ... (iii)
mB wA
* This relation may alsQ'be written as:
' W O O Kr x W
AT
ImxW
where W and Ware the masses of solute and sol ver:t respectively and m =molecular mass of solute.
SOLUTIONS

or AT = molality x K J (iv) Solute does not form a solid solution with solvent in frozen state,
i.e., only solvent separates in solid state on freezing the solution.
KJ is characteristic of a particular solvent and can be (v) Equimolal quantities of different substances dissolved in the
calculated from the thermodynamical relationship . same quantity of solvent bring out the same depression in
0.002 2 freezing point of the solvent under identical conditions.

: : ::::: SOME SOLVED EXAMPLES :::::::


where, TJ is the freezing point of solvent in absolute scale and LJ Example 35. On dissolving 1O.8g glucose (m. wt. =' 180) in
the latent heat of fusion in calories per gram of the solvent. For 240 g ofwater, its boiling point increases by 0.13° C. Calculate the
water, molecular elevation consta~t of water.
K _ 0.002 x
Solution: AT = lOOK I x w
J - 80 WXm -
= 1.86K kg mol- l
Dr K' = ATx Wx m
The molal depression constants for some common solvents are . 100 x w
given in theJollowing table:
Given, AT 0.13" C, W = 240g,m 180and w= 1O.8g
Table S.S Molal Depression Constants of Some Solvents K' = O.13x 240 x 180 = 5.20
.
; . Molal elevati,n eonstant 100 x 10.8
Solvent F.pt; (OC) . . (Kkg~orl) Example 36. A solution of 2.5 g of a non-volatile solid in
Water 0.0 . 1.86 100g benzene is boiled at 0.42°C higher than the boiling point of
pure benzene. Calculate the molecular mass of the substance.
Ethyl alcohol -114.6 1.99
Molal elevation constant ofbenzene is 2.67 K kg mo/- J •
Chloroform -{)3.5 4.70
1000K b x w
Carbon tetrachloride -22.8 29.80 Solution: In == ----"---
WxAT
Benzene 5.5 5.12.
Camphor 179.0 39.70 Given, K b == 2.67, w = 2.5g, W 100g, tlT == 0.42
m 1000 x 2.67x 2.5 = 158.9
If K J' w A ,AT and wB are known, molecular mass of a 100 x 0.42
non-volatile solute can be determined. AT is measured by
The molecular mass Df substance is 158.9.
Beckmann's method in the laboratory.
The molecular mass of non-volatile and non-ionisable solute Example 37. The molal elevation constant for water is
can be calculated using following formula: 0.56 K kg mol -1. Calculate the boiling point of a solution made
K J wB by dissolVing 6.0 g ~f urea (NH 2CONH 2) in 200g of water.
mB = - X - . x 1000
AT wA Solution: AT = 1OOOK b x w
where, WBand W A are the masses of solute and solvent· mxW
respectively. , Given, Kb =0.56Kkg mDI- J , w =6.0g, W 200g, m= 60
Anti-freeze solutions: Water is used in radiators of cars an!!
other automobiles. In cold countries where the atmosi$.erie.. AT = 1000 x 0.56 x 6.0 0.280 C
temperature becomes less than zero degree, the water: in 'he " 200x 60
radiators would freeze. Anti-freeze solutions are useful under
Thus, the bDiling point Df sDlutiDn . b.pt. Df water + AT
these conditiDns when the vehicles are used in the region~ Df
sub-zero temperature so. that the water dDes nDt fr~eze in =-; (l00° C+ 0.28° C) 100.28° C
radiators. Such solutiDns are made by dissolving ethylene glYCDl Example 38. By dissolving 13.6 g of a substance in 20 g oj
in water. Freezing pDint can be lowered to. the desired' extent by water, the freezing point decreased by 3.T'C. Calculate the
varying the cDncentratiDn Df ethylene glycoL Glycerol can also molecular mass of the substance 'Molal depression constantfar
be used as anti-freeze. wf?tcY" 1.863 K kg mol -1)
Freezing mixture : It is a mixture of ice and commDn salt
1000KJ x w
(Nact). It is used in the making Df ice-cream and in the Solution: m=------'----
labDratDries to. create low temperatures. With the help' Df this WxAT
mixture, a temperature as IDW as-33°C (240 K) can be achieved.
Given, KJ = 1.863 K kg mD: 1
Note: Ebullioscopic and cryoscopic methods are effective when.
(i) Solutions are dilute. Solutions obey Raoult's law. w == I3.6g, W 20g, AT -=: 3.7° C
(ii) The solute is non-volatile.
(iii) There is no association or dissociation of solute molecules in the m= 1000x 1.863 x 13.6 == 342.39
solution. 20x 3.7
340 I G.R.B. PHYSICAL CHEMISTRY FOR COMPETITIONS

Example 39. On dissolving 0.25 g of a non-volatile A +85.29= 110.86


substance in 30 mL benzene (density 0.8 g / mL), its freezing
or A (110.86- 85.29) 25.57
point decreases by OAO°e. Calculate the molecular mass of
non-volatile substance (K I 5.12 K kg mol-I). Thus, the atomic masses of A and Bare 25.57 and 42.645
respectively.
Solution: Mass of benzene, W volume x density
Example 42. 1.355 g of a substance dissolved in 55 g of
=30xO.8=24g
CH 3 COOH produced a depression in the freezing point of
Given, K f 5.12 K kg mol-I, w = 0.25 g, !:"T == OAOoe. 0.618°e. Calculate the molecular weight of the substance
We know that, (Kf == 3.85). (CECE (Mains) Bihar 2005)
1000Kr x w x 1000
m Solution: !:"T =Kf x--"----
Wx!:,.T mB XWA

1000x 5.12x 0.25 where, wB = mass of solute, mB molar mass of solute, wA


133.33
24 x 0040 mass of solvent
Example 40. A solution ~f1.25 g of a certain non-volatile 0.618 = 3.85 x 1.355 x 1000 .
substance in 20 g of water freezes at 271.94 K. Calculate the mB x 55
molecular mass of the solute (K I == 1.86K kg mol-I). mB = 153047
Solution: Freezing point of solution 271 ;94 K Example 43. An aqueous solution of a non-volatile solute
boils at 100.17°e. At what temperature will the solution freeze?
Freezing point of water:= 273.0 K (Given: Kb = 0.512K kg mol- I and KI = 1.86K kg mol-I)
!:,.T== (273 271.94)= 1.06K Solution: We know that,
1000K r x w !:"Tb molality x K b
We know that, m== .
Wx!:,.T 0.17 molality x 0.512
Given, Kf == 1.86 K kg mOr'l, w 1.25 g, W 20 g and Molality of the solution = 0.17 m
. 0.512
!:"T 1.06K.
1000 x 1.86x1.25 Let depression in freezing point be !:"TI
m = = . - - - - - - 109.66
20x 1.06 !:"TI == molality x K I
Example 41. .7wo elements A and B form compounds = 0.11 x 1.86= 0.62°e
having molecular formulae AB2 and AB 4 . When dissolved in 0.512
20.0 g of benzene, 1.0 g of AB2 lowers the freezing point by Thus, the freezing point of the solution is
2.3 0c, ~vhereas 1.0 g of AB 4 lowers the freezing point by 1.3 0e.
0.00 - 0.62 = - 0.62°e
The molal depression constant for benzene in 1000 g is 5.1.
Calculate the atomic masses ofA and B. . . Example 44. 18 g ofglucose, C 6HI20 6' is dissolved in 1kg
Solution: We know that, of water in a saucepan. At what temperature will the water boil
1000KI xw (1.013 bar pressure)? K bforwater is 0.52 K kg mOrl.
m=---'--- x 1000
Wx!:,.T Solution: !:"T Kb X - - " - - - -
mB xW A
. 1000x5.1xl
Molecular mass of AB2 (from given data) = - - - - -
20x 2.3 !:"T = 0.52 x 18 x 1000
180x 1000
= 110.86
. 1000x5.lxl T- To =0.052
and Molecular mass of AB4 (from given data) =- - - - -
1.3 x 20 T 373 = 0.052
196.15 T= 373.052K
Further, AB4 = A + 4B == 196.15 ... (i) Example 4~" A solution of urea in water has boiling point
of 100.lS"'e. Calculate the freezing point of the same solution if
AB2 A + 2B = 110.86 ... (ii) K/and Kbforwaterare1.87KkgmoZ-1 andO.52K kg'moZ- 1
Subtracting eq. (ii) from (i), respectively..
2B = 85.29 Solution: !:"Tb (100.15 - 100) = 0.15°e
B 42.645
Putting the value of Bin eq. (ii), We know that, !:"Tb = molality x K b
,.
I

SOLUTIONS

Molality = tJ.Tb == 0.15 == 0.2884


Kb 0.52 fUUsmTfONS OF OBJECTIVE QUESTIONS
tJ.Tf molality x K f
== 0.2884 x 1.87 == 0.54° C 25. Calculate the molal depression constant of a solvent which has
freezing point 16.6°C and latent heat of fusion 180.75 J g-I:
Thus, the freezing point of the solution == - 0.54°C . IJEE (Orissa) 2005)
Example 46. In a cold climate, water gets frozen causing (a) 2.68 (b) 3.86 (c) 4.68 (d) 2.86
iamage to radiator of a car. Ethylene glycol is used as [Ans. (b)]
antijreezing agent. Calculate the amount of ethylene glycol to be
' K _ RTo2
added to 4 kg of water to prevent it from freezing at -6°C (Kf for [HlOt: I - WOOL To = 273 + 16.6 = 289.6 K
water 1.85 K kg mol -I ). (liT 1992)
Solution: Given, . 8.314 x (289.6i
1000 x 180.75
tJ.T = 6°C, W == 4 kg = 4000 g, m == 62, K f == 1.85
3.86]
mxWxtJ.T
w=---- 26. The elevation in boiling point for 13.44 g of CuCI 2 dissolved
1000 x Kf
in 1 kg of water as solvent will be (Kb 0.52 K kg mol-I;
62x 4000 x 6 == 804.32g molar mass ofCuCl z == 134.4 g/mo!): (lIT 2005)
1000x 1.85 (a) 0.05 (b) 0.10 (c) 0.16 (d) 0.20
[Ans. (c)]
Example 47. A solution containing 0.2563 g of [Hint: i = 3, assuming complete ionization ofCuC1 2
naphthalene (molecular mass == 128) in 50 g of carbon
tetrachlorMe yields a boiling point elevation ofO.201°C while a x 1000
solution ofO.6216g ofan unknown solute in the same mass ofthe
solvent gives a boiling point elevation of 0.647°C. Find the
:: 3 x 0.52 x 13.44x 1000 0.156"" 0.16]
molecular mass ofthe unknown solute. 134.4 x 1000
Solution: We know that,
27. A solution containing 7 g ofa solute (molar mass 210 gmol- I )
K _ tJ.Tb xWXm in 350 g of acetone raised the boiling point of acetone from
b - 1000Xw 56°C to 56.3°C. The value of ebullioscopic constant of
acetone in K kg mol- 1 is:
K :: 0.201x50x128 5.019 (a) 2.66 (b) 3.15 (c) 4.12 (d) 2.86
b 1000 x 0.2563 [Ans. (b)}
Kb is now used in the second part of the problem. x 1000
[Hint:
1000 x xw
m
tJ.Tb xW 0.3:: K x 7 x 1000
b 210x 350
1000x 5.019x 0.6216' 96.44
K =0.3x21Ox350=3.15Kk mol-I]
0.647 x 50 b .7 x 1000 g
Example 48. Calculate t~e molal depression constant of a' 28. The normal boiling point of toluene is I1 ,)7 (. and its
solvent which has freezing point 16.6'C and latent heat offusion boiling point elevation constant is 3.32 K kg moll. The
180.75J g-I. enthalpy of vaporization of toluene is nearly:
RT2 (a)17kJmol- 1 (b)2lkJrnol- 1
Solution: K = f (c) 51 kJ mol-I (d) 68 kJ mol-I
f 1000xLf [Ans. (b))
2
8.314JK- I mol-I,
3
R [Hint: K = RTo 3.32 = (8.314 x 10- ) x (383.7)2

b lOOOL' 1000 x L
Tf 16.6'C=273+16.6=289.6K.
L 0.368 kJ I g
Lj = 180.75 J g-l . Latent heat per mol-= 0.368 x molar mass of acetone
. 0.368 x 58 = 21.344 kJ :;tAl I]
Substituting. the values in the above equation,
29. An aqueous solution freezes at -O.l86°C (Kf = 1.86 K kg
K == 8.314 X (289.6)2 = 3.86 mol-I, Kb = 0.512 K kg mol-I). The elevation ofb. pt. ofthe
f 1000x 180.75 solution is:
342 G.R.B. PHYSICAL CHEMISTRY FOR COMPETITIONS

(b) 0.512 (c) 0.512 (d) 0.0512 Difference between Osmosis and Diffusion
(a) 0.186
1.86 The two processes, diffusion and osmosis, can be
[Ans. (d)) distinguished in terms of the following aspects:
[Hint: llT = K j x m (i) In diffusion, solute as well as solvent molecules flow in
0.186 = 1.86 x m opposite directions while in osmosis the flow of solvent
m 0.1
molecules occurs in one direction only.
(ii) For osmosis, a semipermeable membrane* is required
llT = Kb x m =0.512 x 0.1 = 0.0512) while for diffusion it is not required.
30. The amount of urea to be dissolved in 500 cc of water
( K r 1.86) to produce a depression :::'" :JJ 86°C in the Semipermeable membrane
freezing point is: (UGET (ManipaJ) 2006) "A membrane which allows the solvent molecules to pass
(a)9g (b)6g (c)3g (d)O.3g through it but prevents the passage ofsolute molecules through it
[Ans. (c)] is called a semipermeable membrane." . . .
[Hint: '-'AT =-- K j x wB X 1000 Examples of semipermeable membranes are too many.
mB x WA Parchment paper, membranes covering the animal and plant cells
x 1000 and many gelatinous inorganic substances such as calcium
0.186 1.86 x
60x 500 phosphate and copper ferrocyanide, etc., act as semipermeable
WB = 3 g] membranes. Animal and plant membranes are not all completely
semipermeable. The best semipermeable membrane used is
31. What should be the freezing point of aqueous solution
. prepared by deposition of copper ferrocyanide, CU2[Fe(CN)6]'
containing 17 g ofC 2 H sOH in 1000 g of water (Kj for water
in the pores of a porous pot.
J.86deg kg mol-.1)?
Working of semipermeable membrane : Sev~ral theories
(a) 0.69°C (b) 0.34°C have been put forward to explain thewv,~...~ng Vi semIllt:rmeable
(c) O.O°C (d) 0.34°C membrane. These are :
[Ans. (a)]
(a) The sieve theory: The theory was suggested by Traube.
[Hint: llT =Kr x x 1000 = 1.86 x 17 x 1000 = 0.69 According to this theory, a semipermeable membrane contains a
mB x WA 46 x 1000 large number of small pores which eet like a sieve. The pores
Freezing point of solution =0 - 0.69; 0.69° C] allow the smaller molecules of solvent to pass through it but does
.not allow the larger molecules of solute to do so. However, this
5.13 OSMOSIS AND OSMOTIC PRESSURE theory fails in those cases of semipermeability where the solute
molecules' are smaller than the solvent molecules.
Osmosis: When a semipermeable membrane is placed between (b) The solution theory : According to this theory, a
a solution and a solvent, it is observed that solvent molecules membrane is permeable to those substances which dissolve in it
enter the60lution arid the volume of solution increases. It is also and is impermeable to those which are insoluble in it. Thus, a
observed that if two solutions of unequal concentrations are layer of phenol acts as a semipermeable membrane when placed
separated by a semipermeable membrane, the solvent molecules
between a solution of Ca(N03 h. Phenol allows water to pass
from a solution of lower concentration move towards a solution through it because water is soluble in phenol. It is impermeable to
of higher concentration. This phenomenon was first observed by
Ca(N03 h because Ca(N03 h is insoluble in phenol.
Abbe Nollet (1748) and termed as Osmosis (Greek, osmos to
(c) Vapour pressure theory : According to this theory, a
push). Osmosis is defined as the spontaneous flow of solvent
solvent can pass through a semipermeable membrane because the
molecules through semipermeable membrane from a pure
vapour pressure of the pure solvent is higher than the vapour
solvent to a solution or from a dilute to a concentrated
pressure ofthe solution. This theory is widely accepted because it
solution.
explains the phenomenon of osmosis.
The phenomenon of osmosis can be demonstrated by the
Osmotic pressure: A porous pot is taken and a
following experiment:
semipermeable membrane of copper ferrocyanide is deposited in
Two eggs of same size are taken and their outer hardshell is its walls. It is fitted with a long glass tube with the help of a
removed by dissolving in dilute hydrochloric acid. One of the rubber stopper. It is filled with concentrated aqueous sugar
eggs is placed in distilled water and the other in saturated salt solution and placed in distilled water. Osmosis occurs and the
solution. level· of the solution in glass tube rises over a period of time.
After sufficient time, it is noticed that the egg placed in water After a few days, the level becomes stationary. At this
swells up and that placed in salt solution shrinks. In the first case, eqUilibrium state the hydrostatic pressure of the liquid column
water enters the concentrated egg fluid while in the second case, exactly balances the tendency which enables the water molecules
water comes out of the egg as salt solution is more concentrated to pass through semipermeable membrane.
than the egg fluid. ,
* Membranes which allow the passage of only solvent molecules through them are called semipermeable membranes. Egg membrane, goat's bladder
and cell membranes can serve as semiperineable membranes but these are not perfect for laboratory measurements. Artificial membranes of gelatinous
inorganic substances such as copper ferrocyanide are used these days as semipermeable membranes.
I
SOLUT.IONS I 343

Maximum level solution in order to stop the flow of the solvent into the
- .
solution through semipermeable membrane is equal to

: I
. Hydrostatic
pressure

Initial level
0smotic pressure.

Determination of Osmotic Pressure: Berkeley and


Hartley's Method
Various methods are employed for the measurement of
osmotic pressure but the best known method was suggested by
Berkeley and Hartley. The apparatus used is shown in Fig. 5.14.
Semipermeable A porcelain tube having copper ferrocyanide membrane in: its
membrane walls is enclosed in a metallic jacket. The porcelain tube is fitted
with a reservoir of pure solvent at one end and a capillary tube at
Sugar solution
the other end. In a metallic jacket, there is an arrangement for
applying external pressure which is measured with the help of

~_ ~_ ~_ ~_ ~_~_~_~_ ~__~_ ~_ ~_ t-- Water pressure gauge.

Fig. 5.12 Hydrostatic pressure determination


The hydrostatic pressure developed as a result of osmosis Stopcock
Capillary
is a measure of osmotic pressure of the solution. Osmotic tube
pressure is also defined as the hydrostatic pressure built up on
the solution wbichjust stops the osmosis.
Osmotic pressure := hydrostatic pressure
1t = hdg Semipermeable
membrane
where, h = increase in level in the tube of unit cross section,
=
d density of solution and g acceleration due to gravity. Steel vessel
Actually, this will not be an exact measure of osmotic pressure Fig. 5.14 Berkeley and Hartley's method
of the solution originally taken because sufficient dilution has
taken place with time. Procedure: The porcelain tube is filled with pure solvent
If osmosis takes place due to concentration gradient i.e., when and the metallic jacket with solution. The level in the capillary
two solutions of different concentration are separated by tube will tend to move down as the solvent flows towards
semipermeable membrane then, solution due to osmosis. External pressure is now applied onthe
,solution by the piston so that level in capillary remains stationary.
1t = hdg := AeRT
The reading of pressure gauge is recorded. This is the osmotic
Alternative definition: An apparatus as shown in Fig. 5.13
pressure of the solution.
consists of a water-tight chamber which is divided into two
halves by a semipermeable membrane and fitted with a Advantages: (i) It is a quick and accurate method.
water-tight piston and a flow indicator in separate compartments. (ii) The concentration of the solution does. not change
The compartment having a piston is filled with solution and the because flow of solvent is not permitted into solution; so the
other compartment is filled with a pure solvent. Water (solvent) results obtained by this method are reliable ..
tries to flow into the solution side. To check this tendency, a (iii) As the osmotic pressure is balanced by external pressure,
certain pressure shall have to be applied by the piston. This there is no strain on membrane and the danger of itr bursting is
external pressure.is thus a measure of osmotic pressure of the eliminated. So, this method can be used to measure high osmotic
solution. The external pressure which must be applied on tbe pressures also.
External
Comparison of osmotic pressures: de-Vries developed an
pressure approximate method for comparing the relative osmotic
Flow
~ indicator
pressures of aqueous solutions. A solution of low osmotic
pressure is termed hypotonic and solution of high osmotic
pressure isterrned hypertonic. The,protoplasmic layer, lining the
cell walls of plant cells is easily penetrated by water but is almost
impermeable to the substances dissolved in' the cellular fluid.
When a plant cell is placed in hypotonic solution, the water is
drawn in and the cell swells. If the cell is placed in a hypertonic
solution, water will diffuse out of the cell flu:,i and partial
collapse of the cell will take place when hypotonic solution is
separated from hypertonic solution using semipermeable
Fig. 5.13 membrane, then osmosis takes place from hypotonic to
344 G.R.B. PHYSICAL CHEMISTRY FOR COMPETITIONS

hypertonic because solvent concentration is greater in hypotonic 5.14 VAN'T HOFF THEORY OF DILUTE
solution. This phenomenon is known as plasmolysis. The change SOLUTIONS
in cell can be observed under a microscope. When the plant cell is.
van't Hoff realised that an analogy exists between gases and
placed in the solution of same osmotic pressure as that of the cell
solutions provided osmotic pressure of solutions is used in place
fluid, no change in the structure of the cell is observed. Such
of ordinary gas pressure. He showed that for dilute solutions of
solutions having same osmotic pressures are termed isotonic. By non-electrolytes the following laws hold good:
putting, therefore, cells of the same kind into solutions of
1. Boyle-van't Hoff law: The osmotic pressure (P or 1t)
different concentrations, it can be ascertained whether the
solution is hypertonic or hypotonic or isotonic. of a solution is directly proportional to its concentration (C )
when the temperature is kept constant. The concentration of the

<~..... " / ~~:I~~~e


solution containing one gram mole in V litre is equal to
f l,) ~(C=~).
cellUlose .. , .....)
cell wall ~.&_Plasma
Thus, Po<; C (when temperature is constant)

or poc_1
V
or PV constant or 1t V constant
(A) (8)
van't Hoff presumed that the osmotic pressure is due to the
Fig.5.15 Plasmolys,is' bombardment of solute molecules against the semipermeable
membrane as the gas pressure is due to hits recorded by gas
, Some biological explanation on the basis of osmosis are given molecules against the walls of its container.
below: . '
2. Pressure~Temperature law (Gay-Lussac-van't Hoff
(i) In animals, circulation of water to all parts of body takes
law): Concentration remaining same, the osmotic pressure of a
place due to osmosis. . .
dilute solution is directly proportional to its absolute temperature
(ii) Plant roots absorb water from soil due to osmosis. Concen- (T), i.e., .
tration of cell sap illside the root hair cells is higher than that of
water present in the soil. Water enters the root cells due to
endosmosis. P 1t
or - = constant or - := constant
(iii) Water absorbed by plant roots is circulated in the entire T T
. plant body and reaches to the top of a tall tree due to osmosis. Combining the two laws, i. e., when concentration and
(iv) Osmosis helps in plant growth and germination of s~eds. temperature both are changing, the osmotic pressure will be
(v) Red blood cells burst when placed in water; it is due to given by:
endosmosis. pocCT
(vi) Vari?us functions of plants are controlled by osmosis,
e.g., stretchmg of leaves and flowers, opening and closing of or p:= kCT
flowers.
(vii) Use of salt and sugar in pickles and jams acts as or
1
P=k·_·T
V
( ~ince, C == ~)
preser~atives. It prevents growth of bacteria and fungi by
osmOSIS. or PV STornV ST
(viii) Dead bodie3 swell under water due to endosmosis.
S is called molar solution constant.
(ix) When dried fruits and vegetables are placed in water, they
Here, V is the volume of solution containing one gram mole of
slowly swell and return to the original form. It is again due to
the solute. The value of S comes out to 0.082 litre atm K- 1 mol- 1
endosmosis of water into the fruits and vegetables.
'. Intravenous drip of saline water: Saline drip to the patients which is in agreement with the value of R, the molar gas constant.
IS also based on the principle of osmosis. In case, the solution contains n gram moles in V litre, the general
equation would become:
(i) A 0.91% solution of NaCI in water is isotonic to human
blood. Hence, in this solution red blood cells neither swell nor PV nST or1tV=nST
shrink. 3. Third law: Equimolecular solutions of different solutes
(ii) Aqueous solution of NaCI with concentration less than exert equal osmotic pressure under identical conditions of
0.91 % is hypotonic to blood. On placing red blood cells in this temperature. Such solutions which have the same osmotic
solution, endosmosis results into bursting ofRBCs. pressure are termed isotonic or iso-osmotic. When two isotonic
(iii) Aqueous solution ofNaCI with concentration more than solutions are separated by a semipermeable membrane, no flow
of solvent molecules is observed on either side.
0.91 % is hypertonic to blood. On placing red blood cells in it
exosmosis or plasmolysis results into shrinking of cells.
SOLUTIONS

The law is similar to Avogadro's hypothesis. It can be stated


or
as, "Equal volumes of dilute solutions of different solutes,
having the same temperature and osmotic pressure, contain
equal number of molecules". This is the condition for isotonic solutions.
If molecular mass of one solute is known, the molec,ular mass
For solution I, PV = n i ST of the other can be determined without using osmotic pressure
For solution II, values.
Thus, ni must be equal to n 2 when P, V and T are same. Osmotic pressure of mixture of two solutions:
The analogy of dilute solutions with gases is thus perfect. This Case I: Let two solutions of same substance having
led van't Hoff to suggest that a solute in dissolved state (Le., in different osmotic pressures 1t 1 and 1t2 are mixed. Osmotic
solution) behaves as a gas and the osmotic pressure of the pressure of the resultant solution can be calculated as, .
solution is equal to the pressure which the solute would exert 1t I VI + 1t 2V2 = 1tR (VI + V2 ) •
if it were a gas at the same temperature and occupying the where, VI and V2 are the volumes of two solutions and 1t R is the
same volume as that of the solution. This statement is known as resultant osmotic pressure.
van't Hoff theory of dilute solutions.
Case II: Let n l and n2 are the number of moles of two
different solutes present in VI and V2 volumes respectively.
5.15 DETERMINATION OF MOLECULAR Osmotic pressure of the mixture can be calculated as,
MASSES n l i l RT n2 i2RT
1t = 1t1 + 1t2 = + --='-=--
In the case of dilute solutions, it has been stated that the equation (~ + V2 ) (VI + V2 )
PV = nST holds good. Instead of one gram mole of the solute
present in V litre of solution, let wB gram of solute (mol. mass 1t= (nli l +n 2 i 2 ) RT
mB) be present in V' litre of solution; then , (VI + V2 )

n = WB and V V' Here, i l and i z are van't Hofffactor'for the two solutes.
mB
Thus, the equation PV = nST becomes: ······1
•••••• SoME SOLVED EXAMPLE51••••
•••···•.
PV ' = wB ·ST , Example 49. 200 em 3 ofan aqueous solution contains 1.26
me
g of a polymer. The osmotic pressure of such solution at 300 K is
wB xS xT found to be 2.57 x 19-3 bar., Calculate the molar mass of the
or me -
- --"PV-' - - polymer. .
Knowing the value of P experimentally, the value of mA ' i. e. , 1tV WB RT
Solution:
molecular mass of the solute can be determined. ma
Consider two solutions I and II having ni and n2 moles of the
,WB RT
solute in VI and V2 · litres of solution respectively. Let PI and P2 me=-x-
be their osmotic pressures at the same temperature (T). V 1t

From the equation PV nST, _1.26


- - x 0.083x300_61038
- gmo)-1
0.2 2.57 x
For solution I, l1VI = nlST
Example 50. A solution is prepared by dissolving 1.08 g oj
or PI =!!.L ST human serum albumin, a protein obtainedfrom blood plasma, in
~ 50cm 3 ofaqueous solution. The solution has an osmotic pressure
For solution IJ" P2V2 = n2 ST of5.85 mm Hg at 298K:
(a) What is molar mass of albumin?
or P2 =V ST (b) What is height of water column placed in solution?
2
d (H 2 0) 19 cm- 3
Ifboth solutions are isotonic, i. e., PI = P2 , obviously, Solution: (a) Molar mass of albumin can be calculated
!!.L ST =!!1. ST using following relation:
VI V2 xRT
me =--=-- .. :(i)
1tV
or !!.L =n2 Given, we 1.08g;R=0.0821litreatmK- I mol- 1
VI V2
5.85 50
Wilmi W2 1m2
T 298 K 1t = - atm' V = - - 0.05 litre
or --=--- .' 760 ' 1000
~ V2
346 G.R.B. PHYSICAL CHEMISTRY FOR COMPETITIONS

Substituting these values in eq. (i) 1· .


V = - x 0.0821 x 273 =22.4litre
m = 1.08 x 0.0821 x 298 ,,; 68655 I mol 1
B (5.85/760) x 0.05 g Example 55. Find the osmotic pressure of MI20 solution
(b) n=h·d·g
of urea at 27°C (S = 0.0821 lit atmK- 1 mol-I).
5.85 x 101325 = h x I ~ 10-3 x 9.8
Solution: Applying the equation PV = n· ST,
760
P= n .ST
h 7.958x104 m 7.958x 106 cm V
. Example 51. Calculate osmotic pressure of5%solution of or P= Molarity x S x T
cane sugar (su<:rose) at 15°e.
Given,
Solution: m= moL mass of sucrose (C I2 H 22 0 ll ) =342 . . 1
molanty == = 0.05, S = 0.0821 and T 27 + 273 := 300 K
w=5g, v = 100mL O.llitre . 20
S 0.082, T = (15 + 273) 288K Substituting values,
Applying the equation PV:= w ST, P = 0.05 x 0.0821 x 300 == 1.2315 atm
. m
5 1 . . Example 56. The osmotic pressure of a solution of an
P=-. x - x 0.082 x 288 organic substance containing 18g in one litre ofsolution at 293K
342 0.1 . is 2.414 x 105 Nm -2. Find the molecular mass of the substance if
= 3.453atm S == 8.3JK- 1 per mol.
Example 52. The solution containing 109 of an organic
compound per litre showed an osmotic pressure of 1.18 Solution: Applying the equation,
atmosphere at O°e. Calculate the molecular mass of the . PV:::"·ST
compound (S := 0.0821 litre atm per degree per mol ). m
Solution: Applying the equation, or m==~·ST
PV
m=~·ST
PV Gi~en, P=2.414x10 5 Nm-2,V 1.0 lit = Ix 10-3 m3,.
Given, w= 10 g, P = 1.18 atm, V = 1 litre, S = 0.0821 and S == 8.3 JK- 1 per mol, w = 18 g and T =293 K
T:= 273K. 18
10 m= . 5 x8,3x293 181.33
m=--xO.0821x273 189.94 2.414 x 10 x 1X 10-3
1.18 x 1
Example 57. A 5% solution of cane sugar is isotonic with
.. Example 53.The· osmotic pressure of a solution 0.877% solution o/urea. Calculate the molecular mass of urea if
containing 30 g of a substance in 1 litre solution at 20°C is 3.2 the molecular mass ofcane sugar is 342.
atmosphere. Calculate the value of S. The molecular mass of
Solution: Let the molecular mass of urea be m2'
solute is 228 . .
5
Solution: Applying the equation, . Molar concentration of sugar =--'--
. ml xVI 342 x 0.1
PV=:::"·ST
m 0.877
and Molar concentration of urea = w2
S=mxPxV m2 xV2 m2 x 0.1
or
wxT For isotonic solutions,
Given, m:= 228, P = 3.2 atm, V 1litre, w 30 g and ~=~
T=20+273 293K m1V1 m2V2
S = 228 x 3.2 xl 5 0.877
30x 293 342 x 0.1 m2 x 0.1

= 0.083 litre atm per degree per mol m2 =0.877 x 342 = 59.987
5
Example 54.What is the volume ofsolution containing Ig
mole of sugar that will give rise to an osmotic pressure of OF OBJECTIVE quESTIONS
1atmosphere at O°C?
.Solution.: Applying the equation PV == n· ST, 32. . Find out the osmotic pressure of 0.25 M aqueous solution of
urea at 27°C (R 0.082 litre atm K- 1 mol-I, R =.1.987 cal
V=n xSxT
P K-1mor I). ICET (Gujarat) 10081
(a) 6.15 atm (b) 0.615 atm (c) 0.0615 atm (d) 61.5 atm
Given,n = 1,P == latm, S = 0.0821 and T == 273K [Ans. (a)]
SOLUTIONS 34L

[Hint: Tt = CRT
37. The temperature at which 10% aqu~~us solution (~) of
= 0.25 x 0.0821 x 300
= 6.157 atm] glucose will exhibit the osmotic pressure of 16.4 atm, is :
3 l
33. Two solutions of glucose have osmotic pressures 1.5 and 2.5 (R 0.082dm atm K-1mol- ) IPMT (Kerala) 2008)
atm. I litre of first solution is mixed with 2 litre of second (a) 360°C (b) 180 K (c) 90 K (d) 300 K
solution. The osmotic pressure of the resultant solution will (e) 360 K
be: [Ans. (e)]
(a) 1.62 attn (b) 6.12 attn (c) 1.26 atm (d) 2.16 atm [Hint: TtV :=:nRT
[Ans. (d)]
TtV=W RT
[Hint: Tt IVl + Tt ZV2 Tt R (VI + V2) m

1.5 x 1+ 2.5 x 2:=: TtR X 3 r


100 \
dOOO
10
16.4 x -j:=: - x 0.082 x T
180
TtR : =6.5
: - = 2.1
6 atm ] T 360K]
3
34. 18 g glucose and 6 g urea are dissolved in 1 litre aqueous 5.16 REVERSE OSMOSIS
solution at 27°C. The osmotic pressure of the solution will
be: When a solution is separated from pure water by a
(a) 3.826 atm (b) 4.926 atm semipermeable membrane, water moves towards the solution on
(c) 2.92 atm (d) 9.42 atm account of osmosis. This process continues till osmotic pressure
[Ans. (b)] becomes equal to hydrostatic pressure or osmosis can be stopped
by applying external pressure equal to .osmotic pressure on
[Hint: TtV (nl + nz)RT solution. If external pressure greater than osmotic pressure is
applied, the flow of solvent molecules· can be made' to proceed
TtV =(W\ + W2)RT from solution towards pure solvent, i. e;, in reverse direction of
ml m2
the ordinary osmosis. This type of osmosis is termed reverse
osmosis. Reverse osmosIs is used for the desalination of sea
Tt X ,1 = (18
- + -6 ) x 0.0821 x 300
180 60 water for getting fresh drinking water.

Tt :=: 4.926 atm1


S~17 AB~ORMAL COLLIGATIVE
35. A solution containing 10 g per dm 3 of urea (m. w. = 60) is
isotonic with a 5% solution of a non-volatile solute. The
PROPERTIES
molecular mass of this non-volatile solute is: The colligative properties of solutions depend on the number of
I.CBSE (Medical) 2006) solute particles present in solution. Various relations derived for
(a) 250 g mol- l (b) 300 g mor l . colligative properties hold good in dil!lte solutions only when
(c) 350 g mor l (d) 200 g mor l there is no change in molecular state of solute. -In case, the total .
[Ans. (b)] number of particles of the solute changes in' solution, the
[Hint: Tt I (urea) Tt2 (unknown solute) colligative preperty shall also ch~mge accordingly. The number of
particles in solution may change in two ways:
C I (urea) =C 2 (unknown solute)
(i) By dis~ociation:' When the substance is an electrolyte.
WB x 1000J '= [WB X 1000] i. e. , the substance undergoes ionisation and number of particles
[ mB x V urea mB X V unknown solute increases in solution, the ions act as particles. The number of
particles, thus, increases on ionisation and the value of colligative
10 X 1000. 5 X 1000
property increases accordingly.
60 X 1000 mB X 100
(ii) By association: When the substance undergoes
mB =300 g m~rl ] association, i. e. , two or more molecules of the solute associate to
36. The osmotic pressure of a solution at O°C is 4 atm. What will form.a single giant particle, the munber of particles decreases and
be its osmotic pressure at 546 K under similar conditions? consequently the value of colligative property decreases. Thus,
W4~ ~2~ W8~ 001~ Nonnal value of colligative property
[Ans. (c)] oc No. of particles of solute taken
·~=CRTI Abnormal value of colligative property
[Hint:
Tt2 CRT2 oc No. of particles of solute after

'~=Tl ionisation or association


Tt2 T2 Abnormal colligative property
Thus, the ratio, , may have tb.e
4 273 Normal colligative property .
Tt2 546 value either more than 1 or less than 1. The ratio is termed van ~t
Tt2 = 8 atm] Hoff factor which is represented by , i '. Thus,
348 G.R.B. PHYSICAL CHEMISTRY FOR COMPETITIONS

. Table 5.6 Illustrations of van't HOff Factor "i' for Different Solutes
No. of particles In van't Hoff
.Example Abnormal
S. Solute type Ionisation or associadon the solution from factor'i' molecular mass
No. t mole solute
1. Non-electrolyte Urea, sucrose, glucose .- J 1 mnormal

2. ~inary electrolyte
ABtype
NaCl, KCl
CH3CO(JR, etc.
AB
I - a-
A+ + B-
a- a
2 1+ a 1+ a
mnormal

3. Ternary electrolyte CaC1 2, BaCl 2 AB2 ~ A2+ + 2B- 3 1 + 20. mnormal


AB2 type or A2B rI2S04' K 2 [PtC1 6 ] I - a- a- 2a- 1 + 20.
type A2B ~ 2A+ + B 2- 3 1+ 20. lnnormal
,
I a 2a- a· l:t- 20.

AB3 ~ A 3+ + 3B-.
I
4. puatemary . AlCI 3, K 3[Fe(CN h] . 4 1 + 30. . ~ormal
~lectrolyte AB3 or PeC1 3, K 3P0 4 . . I-a a 3a 1+30.
iA3B type A3B ~ 3A+ + B 3- . 4 1 + 30. mnormaJ
I -a- 3a a 1+ 30.

5. Association of Benzoic acid forming nA~An mnormal


1
~) a]
r
~mer in benzene
(1-~)o.]
olute I-a
a - [1-(1
n [ 1.
"
6. G,""," ol"trolyto mol, oholote ABn _ l ~ A+ (If-I) + (n -1) B- moormal
n [1 + (n -1)0.]
iABn _ 1 iving n ions in the 1- a a (n ~ I)a [1+ (n -1)0.]
·olution

= (£¥7 )obs.
(AP}nonnal
1+(~ l)a
Thus, van't Hoff factor' i ' = _-,-n~-,--_ < 1if n is 2 or more
1
(ATb )Obs.= (AT! )obs.
1- i
(ATb )normal (AT! )nonnal a=---
I lin
Actual no. of in solution
=-------''--------
No. of particles taken As, Colligative property oc I
moL mass
(i) Suppose one molecule of an electrolyte gives' n' ions on
Observed colligative property . Normal mol. mass
dissociation and' a' is its degree of ionisation. Obviously, So, ==
Number of ions produced na Normal colligative property Observed mol. mass
and Number of unionised molecules = I a Normal mol.
Thus, i=------.:...-
Total number of particles in solution 1- a + na Observed mol. mass
= 1+ (n -l)a In case of dissociation,
Thus,
. . l+(n l)a f
-Normal
- - -moL
- -mass
- = I·+ ( n-. 1) a> I
van't Hoff factor' i' = --'---'-- > Ii n is 2 or more Observed mol. mass
I
Observed mol. mass will always be less than normal mol. mass.
i-I
and a=-- In case of association,
n 1 . _ Normal mol. mass -_ I + ( --
1-
1 a< 1 I.)
(ii) Suppose 'n' molecules associate to form one giant Observed mol. mass n
molecule and' a' is the degree of association; then. Observed mol. mass will always be higher than normal mol. mass.
nA (A)"
(I-a) alll Relathm between osmotic pressure and vapour pressure

Total number of particles in solution 1- a + a / n Let an aqueous dilute solution filled in a capillary tube is
closed at one end by a semipermeable membrane. The tube is
=1+(~-IJa placed in pure solvent (water). Entire apparatus is closed by a
beUjar. At osmotic equilibrium, the belljar is saturated with water
. ;
SOLUTIONS 349

vapour. At equilibrium osmotic pressure (1t) becomes equal to Partial pressure of immiscible liquids
hydrostatic pressure. Let 'A' and 'B' be the two volatile and immiscible liquids; p A
1t=hxd ... (i) and p B be the partial pressures of 'A' and' B' respectively.
where 'h' is height in the column, 1t is osmotic pressure. . _PA .=_nA
Then,
Let, Po = Vapour pressure of pure solvent PB nB
p Vapour pressure of solution where, M A and M B are molar' concentrations of 'A' and 'B'
respectively.
_ WA ImA
PB WBlm E
......r----- Saturated space where, W A and WB are weights of' A' and' B', m A and mB are
with water vapour molecular weights of' A' and' B' respectively.
Example 58. Phenol associates in benzene to a certain
L~---+-- Solution e;tent for a dimer. A solution containing 20 x 10-3 kg ofphenol in
1.0 kg of benzene has its freezing point decreased by 0.69 K.
Pure solvent
Calculate the fraction of the phenol that has dimerised (Kf of
benzene is 5.12°K kg mol-I). .
. Semipermeable
membrane Solution: Observed mol. mass
1000xK f Xw
=---~--
WxllT
Fig. 5.16 3
x 5.12x 20x 10- := 148.4
Pressure at level 4 = Po; Pressure at level 0. = p. Pressure at Ix 0.69
4 will be greater than pressure at 0.. Then
Normal mol. mass of phenol (C 6 HsOH) =94
Po - P = hD ... (ii)
Normal mol. mass 94
where'D' is density of solvent vapour at pressure Po. So,
Observed moL mass 148.4
Dividing eq. (i) by eq. (ii), we have,
1t hd d
Po - p hD D
94 . a;
at a ftxed temperature diD is constant. 1--
.. 1t = constant ( Po - p) ... (iii) 148.4 2

i.e., 1t "" (Po p)or IIp ... (iv) or a; =0.733 or 73.3%


Thus, osmotic pressure is directly proportional to the lowering Example 59. The freezing point depression of 0.001 m
of vapour pressure. K AFe(CN)6] is 7.10 X 10-3 K. Determine the value ofx. Given,
. I
We know that, PoV nRT Kf = 1.86 K kg mor for water.
W
PoV=-RT
M Solution: IlxF ix Kf x m
i.e. , PoM W RT DRT
7.1 0 x 10-3 i x 1.86 x 0.001
V
RTD
Po=-- ... (v) i= 3.817
M
Dividing eq. (iii) by eq. (v), we get i-I
Po - P
- - - 1tX
D M
x--
n I
Po d RTD
1= 3.817 I
IIp M
-=1t (x+I)-1
Po dRT
x 2.817=3
At a ftxed temperature, 1t and M is constant.
dRT .. Molecular formula of the compound is K3 [Fe(CN)6 ].
Thus, !lp "" 1t, i. e., osmotic pressure is proportional to Example 60. A M solution ofpotassium ferrocyanide is
Po 10
relative lowering of vapour pressure. 46% dissociated at 18°C. Whatwill be its osmotic pressure?
350 G.R.B. PHYSICAL CHEMISTRY FOR COMPETITIONS

Solution: Normal osmotic pressure ~xS xT == 1000x 5.12x 0.2 == 113.78


mXV . 20x 0.45
Nonnal molecular mass of acetic acid 60
(when no dissociation has taken place)
2CH 3 COOH~(CH3COOH)2
w Before assoCiation I' 0
0.1,v ::; 1litre, S ::; 0.0821,T ::; 18 + 273 == 291 K After association (I-a) a/2
m
van ,t H0 ff f actor == -
Normal
- - - mol.
- - 'mass
--
Normal osmotic pressure = 0.1 x 0.0821 x 291 2.389 atm Obs. mol. mass
1
l-a+a/2
Potassium ferrocyanide is an electrolyte. It dissociates as: =
1
K 4 Fe(CN)6 ~4K++[Fe(CN)6]~ ~=l_a
(l a) 4a 'a
113.78 . 2
Total number of particles::; 1 a + 4a + a = 1 +4a or a == 0~945
,or 94.5% associated.
a =0.46; so, 1+ 4a = 1+ 4 x 0.46::: 2.84
Observed osmotic pressure . 2.84
-------=--- = - -
Normal osmotic pressure 38. OJ)()2 molar solution ofNael having degree of dissociation of
Observed osmotic pressure =2.84 x 2.389 90% at 27°C has osmotic pressure equal to:
(a) 0.94 bar . (b) 9.4 bar
::; 6.785atm (e) 0.094 bar (d) 9.4.x 10-4 bar
Example 61. A 0.5% aqueous solution of KCI was found [Ans. (c)]
to freeze at - 0.24" C. Calculate the van 't Hofffactor and degree i-I
[Hint: a=
of dissociation of the solute at this concentration ( Kf for water n-I
. = 1.86K kg marl).
. 1000 x xw 09=i-l' i 19
. 2 -1' .
Solution: Observed mol. mass ofKC1:::::' .
. t:.!.T>< W
'It = iCRT
Given, K f = 1.86K kg mol-I, w 0.5 g, W 100g, t:.!.T 0.24 =1.9 x 0.002 x 0.082 x 300
= 0.094 bar]
So ' Ob served mo.l 'm~<;',
' (f)KCI
: : ; 1000 x 1..86 x 0.5 ::; 38 .75
, , , '. 0.24 x 100 ' 39. A 0.2 molal solution of KCl freezes at - 0.68"c' If Kf for
Normal mol. mass of KCI = 39 + 35.5 74.5 H2 0 is 1.86, the degree of dissociation of KCl is:
(a) 75% (b) 83% (c) 65% (d) 92%
van" t H0 ff f actor = -
Normal
--- mol.
-mass
-- [Ans. (b)]
Observed mol. mass
[Hint: /)"T i x Kf x m
74.5
1.92 0.68 i x 1.86 x 0.2
38.75 i =1.83
KC1 ...---'! K + + CI - (a is the degree of ionisation) i 1
(I-a) Cl a a=--
'n 1
Total number of particles 1.,.. a + a + a ::; 1 + a
1.83 - 1 = 0.83
i= l+a 2-1
1.92==I+a Ionization = 83%]
So, a 1.92 - 1== 0.92 40. A certain substance' A' tetramerises in water to the extent of
i.e., 92% dissociated: 80%. A solution of 2.5 g of A in 100 g of water lowers the
freezing point by 0.3"c, The molar mass of A is:
EX!lmple 62. The freezing point of a solution containing
(a) 122 (b) 31 (c) 244 (d) 62
0.2 g of acetic acid in 20 g of benzene is lowered by 0.4S"'c. [Ans. (d)]
Calculate the degree ofassociation ofacetic acid in benzene (Kf
l-i
for benzene is 5.12K kg mol-I). (liT 1994) [Hint: a
Solution: Let the observed molecular mass of acetic acid be n
l-i
1000 x xw 0.8=--1; i=0.4
mobs. == - - - " ' - - - 1-
Wxt:.!.T 4
SOLUTIONS ·351

AT =iKf xm 4.92 == i x 0.1 x 0:0821 x 300


x 1000
0.3 = 0.4 x 1.86 x i 1.99
ma xWA i-I
(1.=--
0.3 0.4 x 1.86 x 2.5 x 1000 n-l
ma x 100
0.99
Ina = 62] 0.49
2
41. van't Hoff factor of Hg 2 Cl 2 in its aqueous solution will be
Percentage ionization = 49%]
(Hg 2 Cl 2 is 80% ionized in the solution):
(a) 1.6 (b) 2.6 _(c) 3.6 (d) 4.6 43. Which of the following solutions will exhibit highest boiling
[Ans. (b)] -point? (KCET Z006)
(a) 0.Ql MNa 2S04 (b) 0.Ql MKN03
[Hint: Hg 2 Cl z
(c) 0.015 Murea (d) 0.015 M glucose
IAns. (a)]
i-I
(1.= [Hint: - AT= i x kb x m
n-l
i x m of Na 2 S04 is highest, hence its boiling point will also be
0.8:::;: i-I highest.
3 1
Na 2 S04 i.x m == 3 x 0.01 = 0.03
i == 2.6]
KN0 3 i x m = 2 x 0.01 = 0.02
42. 0.1 M aqueous solution of MgCl 2 at 300 K is 4.92 atm. What
Urea i x mIx 0.015 =0.015
will be the percentage ionization of the salt? Glucose i x mix 0.015 0.015]
(a) 49% (b) 29% (c) 39% (d) 69%
[Ans. (a)]
[Hint: 1t = iCRT
352 G.R. B. PHYSICAL CHEMISTRY FOR COMPETITIONS
..

. MISCELLA.NEOI,IsNUMERlCAL ExAMPLES4" " ,


. Example 1. Calculate the freezing point of an aqueous Sol""tion:
solution of a non-electrolyte having an osmotic pressure of2.0 6.Ptheo. :: Lowering in vapour pressure when there is no
atmosphere at 300K . dissociation
(K = 1.86K kg mo[-l ,R = 0.821litre-atmK": 1 mol-I)
f
(lIT 1993) = Po x wM (given, Po = 760mrn, w =14 g,
Wm
Solution: We know that, W 200 g, M = 18, m = 164)
P CRT
760x 14 x 18:: 5.84 mm
or, C P _~_2_ _ mollit-! 200x 164
RT 0.0821 x 300
70
In dilute solution, the density of water can be taken as 1.0 g Degree of dissociation 0.7
cm-3 . 100

Hence, 2 mollie! = 2 mol kg- I Ca(N0 3 ) 2 . . - - Ca 2+ + 2N0 3


0.0821 x 300 0.0821 x 300 (n= 3)

Let 6.T be the depression in freezing point. 6.Pobs. = No. of particles after dissociation
6.T = Kf x molality 6.Ptheo. No. of particles when there is no dissociation
2
= 1.86 x =0.151K I+(n-l)a. xU7
0.0821 x 300 --'--;---'--- = 2.4
I
Freezing point =(27.3 0.151) =272.749 K or - 0.151° C
So, 6.Pobs. = 2.4 X 6.Ptheo. = 2.4 x 5.84
Example. 2. Calculate the amount ofNaCI which must be =·14.02mm
added to 100 g water so that freezing point is depressed by 2 K
For water, Kf 1.86K kg mol-I. Po Ps:: 6.Pobs. = 14.02
Solution: NaCI is a strong ele\,:trolyte. It is completely Ps Po 14.02= 760-14.02:: 745.98mrn
dissociated in solution. Example 4. Calculate the osmotic pressure of solution
Degree of dissociation, a. obtained by mixing 100 mL of 3.4 % solution of urea (molecular
mass = 60) and 100 mL ofl.6% solution ofcane sugar (molecular
NaCI + CI- • mass 342) at 20°C.
(n = 2)

No. of particles after dissociation 1 +' (n -1)a.


Solution: No. of moles of urea = 3.4 = 0.0567
60

No. of moles of cane sugar :: ~ = 0.0047


= 1+ (2 - 1) x 1=2
No. of particles after dissociation 342
,
6. Ttheo . No. of particles when there is no dissociation Total number~f moles (0.0567 + 0.0047) = 0.0614

--=2
2 Total volume of solution =(100 + 100) = .200 mL
6. Ttheo . = Q.2litre.····
'. .
or 6. Ttheo . = 1 PV = (Total numbitr ofmoles} x. S x T
Let w g of NaCI be dissolved in 100 g of water. 00614 ..... ,'" ....•.
1000 x xw P = -'-~ x 0.0821 x 293:: 7.385 atm
So, 6. Ttheo . =.---""---
0.2
Wxm
Example S. Calculat~ the normal boilingpoint ofa sample
or w 6.r;heo
. xWxm I x 100 x 58.5 \t .. ofsea water found to contain 3.5%ofNaCI and O. 13% ofMgCl2 by
1000x K f 1000x 1.86 mass. The normal boiling point of water is 100°C and K b (water)
0.51 K kg mol -I. Assume that both the salts are completely
=3.145g' ionised.
Example 3. The degree of dissociation ofCa(N03 h in a Solution: Mass ofNaCI = 3.5 g
dilute solution containing 14 g of the salt per 200 g of water at
100°C is 70%. If the vapour pressure of water is 760 mm, No. of moles of. NaCl = ~
calculate the vapour pressure ofsolution. (lIT 1991) 58.5
SotUTIONs"
-
Number of ions furnished by one molecule ofNaCI is 2; 100
Volume of solution:::: litre '
So, actual number of moles of particles furnished by 1.04 x 1000
sodium chloride:::: 2 x 3.5
58.5 n; = 2 x 3.5 x1.04x 1000x 0.0821 x 293 = 29.93 atm
58.5 100 -
Similarly, actual number of moles of particles furnished by
'Example 8. Molality of a solution in aqueous medium is
.
magnesIum chl'd
on e:::: 3 X -
0.13
-' 0.8 . Calculate its mole fraction and the percentage by mass of
95 solute if molar mass of solute is 60.
Solution: We know that,
Total number of moles of particles;::; (2 x 3.5 +3 x 0.13,)
58.5 95 ~ 'm x 1000
... (i)
(l-xB)xm A
== 0.1238
wl),ere, xB :::: mole fraction of solute'
Mass of water = (100- 3.5- 0.13)= 96.37 g ~~~Z kg mA = molar mass of solvent
0.8= xB x 1000
Molality :::: 0.1238 x 1000 = 1.2846 (l-xB)x 18
96.37 x B == 0.014
tlTb :::: Molality x Kb Let WB =x g, W A =100 g
=1.2846 x 0.51= 0.655 K w X 1000
mB= '
Hence, boiling point ofsea water:::: 373.655 K or lOQ.655"C. mB XWA
"Example 6. ' A solution of a non-volatile solute in water 0.8= xX 1000
has a boiling point of375.3K. Calculate the vapour pressure of 60x 100
water above this solution at 338 K. Given, Po (water) :::: 0.2467
x=4.8%
atm at 338K and K bfor water = 0.52.
Example 9. Calculate the boiling point of a solution
Solution: tlTb:::: (375.3 - 373.15):::: 2.15 K
containing 0.61 g of benzoic acid in 50 g of CS 2 (I) assuming
We know that, 84% dimerisation of the acid. The boiling point and K b ofCS 2
tlTb =: Molality X Kb are 46.2 0 C and 2.3 K kg mor l r.espectiveiy. (lIT 1997),
Solution: tlT :::: i x K b x Molality
2.15:::: Molality x 0.52
. K
==lX b xwx1000
---
Molality:::: 2.15 :::: 4.135 , mXW
0.52
i-i
i. e;, 4.135 moles of the solute present in 1000 g of water (55.5 a=---
1 lin
moles).
0.84::::~
Mole fraction of water :::: 55.5 :::: _5_5_.5_ 1 112
4.135+ 55.5 59.635 i= 0.58 '
Vapour pressure of water above solution tlT:::: 0.58 x 2.3 x 0.61 x 1000:::: 0.1334
:::: Mole fraction x Po 122x 50
T
To::= 0.1334
:::: ~ X 0.2467 =: 0.23 atm T :::: T~ + 0.1334::: 46.2 + 0.1334:::: 46.3334
59.635
Example 10. A very small amount of a non-volatile solute
,:'Example 7. Sea water is 3.5% by mass of a salt and has a
(that does not dissociate) is dissolved in 56.8cm 3 of benzene
density 1.04 g cm -3 at 293 K. Assuming the salt to be sodium
(density 0.889 g cm- 3 ).At room temperature, vapour pressure of
chloride. calculate the osmotiC pressure of sea water. Assume
complete ionisation of the salt. ' this solution is 98.8 mm Hg while that of benzene is 100 mm Hg.
Find the molality of the solution. If the freezing temperature of
Solution: Mass ofNaCI :::: 3.5 g this solution is 0.73 degree lower than. that ofbenzene, what is the
value of molal freezing point depression constant of benzene?
No. of moles:::: 3.5 , (DT199~
58.5 tlp ,
Solution: -=X A
Actual number 6f moles of particles of solute in solution
Po .
2x 3.5 100- 98.8
--- XA
58.5 100
3541 ~.R.B. PHYSICALCH6MISTRYFOR COMPETITIONS

X A =0.012 w= 56.66g
Molality = X A X 1000 = 0.OI2x ~OOO= 0.1557
(1- X A )mB' 0.988 x 78 V=~
d
AT = KI x Molality 50= 56.66
0.73 = KI x 0.1557 d
KI =4.688 < d =1.13 gl cm~
~if,:rtlilDp~ 11. The solution ofa non-volatile solute in water ~4::tx.lDple 14. A 1.2% solution of NaCI is isotonic with
freezes at - 0.30 0 C. The vapour pressure ofpure water at 298 K 7.2% solution ofglucose. Calculate the van ~ HoJJfactor ofNaCl.
is 23.51 mm Hg and KI for water is 1.86 degree/molal. (MLNR 1997)
Calculate the vapour pressure ofthis solution at 298 K . Solution: NaCl glucose
(liT 1998) = 7t(glucose)
Solution: AT = KI x Molality =C 2RT
0.3 = 1.86 x Molality
iCt = C2
Molality = 0.161 {12I:S.5J = 7.2~I80
' XA x 1000
. 'M0 lal tty = --"'---- i = 7.2x 58.5 = 1.95
(I-XA )xmB
1.2 x 180
. 0.161= X A x 1000 [l'J~ii3IDple IS. 1.4 g of acetone dissolved ,in 100 g of
(I-XA)I8 benzene gave a solution which freezes at 277.12 K. Pure benzene
X A =0:00289 freezes at 278.4 K. 2.8 g ofsolid (A) dissolved in 100 g ofbenzene
gave a solution which froze at 277.76 K. Calculate the molecular
tJ.p =XA mass of(A). (lIT 2000)
Po. Solution:. We .know that,
23.51- Ps = 0.00289 AT= K x wx 1000
23.51 f mxW
Ps =23.442mm when:, AT = Depression in freezing point
~Yf:.EiaiDpie 11. x g of a non-electrolytic compound (molar
mass = 200) is dissolved in 1.0 litre of0.05 M NaCI solution. The Kf = Molal depression constant of benzene
osmotic pressure of this solution is found to be 4.92 aim at. 27 0 C. w == Mass of solute
Calculate the value of' x' . Assume complete' dissociation ofNaCI
m = Molecular mass of solute'
and ideal behaviour of this solution. (liT 1998)
Solution: W = Mass of solvent
(i) For NaCI: 7t = iCRT = 2 x 0.05 x 0.0821 x 300 Case I: (278.4 _ 277.12)= K x 1.4 x 1000
=2.463 attn I 58x 100
(ii) For unknown compound: . 14
1.28= KI x 58 ... {I)
7t=CRT =-=- x 0.0821 x 300= 0.1 23 lx attn
200 Case n: (278.4- 277.76)= K x 2.8x 1000
f
Total osmotic pressure 7t = 7tJ + 7t2 m(A) x 100

4.92= 2.463 + O.l231x 28


O.64=KI - ... (2)
x= 19.959g m(A)

....
i~~"* p.le 13. The freezing point of a solution containing· Dividing eq. (l) by eq. (2), we get
50 cm 3 of ethylene glycol in 50 g of water is found to be -34° C.
m(A) =232
Assuming ideal behaviour, calculate the density of ethylene glycol
. (KI for water = 1.86K kg mol- t ). (lIT 1999)
ii.'~"iulDpIe16. To 500 cm 3 of water, 3.0x 10-3 kg ofacetic
"
acid is added. /f23% ofacetic acid is dissociated, what will be the
Solution: AT= K x wX 1000 depression in freezing point? Kf and denSity ofwater are 1.86 K
I mxW kg-I morl andO.997gcm-3 respectively. (lIT 2000)
Solution! Mass of solute = 3.0 x 10-3 kg =3.0 g
34 = 1.86 x wX 1000 .
. 62x 50 Molecular mass of solute = 60
SOLUTIONS 1355.
Mass of solvent = 500 x 0.997 = 498.5 g Solution: Molal elevation constant may be calculated as~
We know that, K _ RTo 2 (where, To =boiling point of pure solvent
b(()OO - 1000 Lv Lv = latent heat of vaporization
' . . ((l ) ::: -i-I
Degree 0 fd lssoclatlon .- . pergram
n-I RTo2 .. AHv
= Lv = - - )
0.23= i-I l000 AHv mB
2-1 mB (here; AHv =molar latent heat of .
vaporization .
or i = 1.23
mB = molar mass of solute)
AT =i x K f x molality
·
Smce, AC' AHv
L.W v = - -

=1.23 x 1.86 x .3 x 1000 =0.229 To


60x 498.5 here, AS v = entropy of vaporization
i.e., depression in freezing point = 0.229
By considering AS v as almost constant, K b oc To.
Example 17. Osmotic pressure ofa solution is l.3atm. The
density ofsolution is 1.1 glcm 3. Calculate osmotic rise. Kb(x)= 0.68; Kb(Y)= 0.53 and Kb(z)= 0.98
(l aim =76cm Hg, d Hg =13.6glcm 3
) ;£~ :Example 20. 1.22g C 6 H sCOOH is· added into two
solvents and data ofATb and K b are given as:
Solution: 1t == hdg
(a) In l00gCH3 COCH3 ; ATb = 0.17; Kb = 1.7 kg kelvin! mol
~,·,~,··"'t;SX16xtJ:6'xg == 11 xl;lx'g~ '(b}lnl00gbe1rzenei~'~Tb = (j]3;Kb='~:61g ·~IViiirmol··· .
"1.3 x 76X' 13.6 Find out the ":'ole'cular ~eight ojC6 HsCOOlj i",bQth. (:~es
h= . cm
1.1 and interpret the result. (lIT 2004)
• wBxlOOO
== 1221cm Solution: (a) AT =Kb X -.:---
mB xW A
·Example 18. (a) 109 ofa certain non-volatile solute were
0.17= 1.7x 1.22 x 1000·
dissolved in 100 g water at 20°C. The vapour pressure was
mB x 100
loweredfrom 17.3555mm to 17.2350mm; calculate the m.wt. of
the solute. mB =122
(b) The vapour pressure ofpure water at 25°C is 23.62 mm. (normal molecular mass of benzoic acid)
What will be the vapour pressure ofa solution ofl.5 g urea in 50g (b) AT= Kb xWB X 1000
o/water? (IIT 2001) mB xWA
0.13 = 2.6x 1.22 x 1000
Solution: (a) We know that, Po - P = wB X mA
mB X 100
p mB X wA
mB =244
17.3555-17.2350 lOx 18 (Abnormally doubl,? molecular mass of benzoic acid, it shows
17.2350 mB x 100 association of benzoic acid in benzene.)

mB == 258.5 ~t'fs;t.J....ple 21. How much C 2 H sOH should be added to


I litre H 20 so that it will not.freeze at - 2CfC?
(b) p= Po XA
50 K f =1.86°~C! m (BCECE 2006)
Solution: Mass of I litre water = 1000 g
p= Po nA ~ 23.62 x 18
nA+nB 50+1.5 AT=Kfx wB x 1000
18 60 . mB xW A

p = 23.41 mm Hg 20= 1.86 x WB x 1000


46x 1000
>"EUmple 19. Match the boiling point with K bfor x, Y and. WB = 494.6g
Z if molecular weight ofx, y and z are same:
b.pl. Kb Example 22. Depression in .freezing point of 0.1 molal
x 100 0.68 solution of HF is 0.201°C. Calculate percentage degree of
Y 27 0.53 dissociation ofHF. (Kf = 1.86 K kg mor 1) .
z 253 0.98 (liT 2003) [AlPMT (Mains) 2008)
356 I
solutiofi: We know Dividing eq. (i) by (ii)
I:!T = i x k f x m 0.465 .
--=l
0.201 = ix 1.86x 0.1 0.245
i 1.0806 1.897
The degree of dissociation HF may be calculated liS. i-I 1.897 - 1
We know, a=--
a i-I =1.0806-1' 0.0806 n-I 2- 1
n-I 2-1 . 0.897
Percentage dissociation = a x 100 0.0806 x 100 .. Percentage ionization = 0.897 x 100
. 8.06 =89.7
Example 23. There is K1 and sucrose solution with 0.1 M Example 24. 102% solution of glycerine .and 2% solution
COl .Jntration, if the osmotic pressure ofK1 and sucrose solution of glucose are isotonic. Molecular mass of glucose is 180 then
is 0.465 atm and 0.245 atm respectively. Thenfind the van i Hoff find out the molecular mass ofglycerine. ![Al1PMT,(M_ios):ZOO9J
factor ofK1 and its degree of dissociation.ilmm !(M~iDS} :Z'_l filoiutiofi : 'It glycerine == 'It glucose
soiution : For KI
C (glycerin~) =C (glucose)
= iCRT
'It
0.465 i x 0.1 x RT. ... (i) 120x 1000 2x 1000
For Sucrose mgIycerine xlOO 180x 100
'It = CRT mglycerine = 91. 8
0.245 = 0.1 x RT ... (ii)

1. SoiufioHi It is a homogeneous mixture .of two of more 4. Solutions of liquids In liquids! Miscib.1e liquids form
components· whose concentration can be varied within certain three types of solutions which may be ideal or non-ideal solutions.
limits. A solution containing only two components is termed a Ideal solution is that in which the attractive forces among the
. binary solution:. One component is called the solute while other solute and solvent molecules are of the same order as that of
as solvent. The component having the same physical state as the solute intramolecular and solvent intramolecular forces. The W
solution is called the solvent. In solutions, in which the two mix~g and I:!V mixing in such solutions is zero. Ideal solutions
components have the same physical state, the component present obey Raoult's law.
in larger proportion is termed the solvent and the other The solutions in which solute-solvent interactions are
componen:t is called solute. different from solute-solute and solvent-solvent interactions are
2. $()tubiUtyi A solution is said to· be saturated when it called non~ideal solutions. The non-ideal solutions do not obey
contains as much solute as it can dissolve at a particular Raoult's law for all concentrations.
temperature in presence of the undissolved solute. A super- W mix *" 0 and I:! V*"O in these solutions. There are two types
saturated solution contains more. quantity and an unsaturated of deviations from the ideal behaviour. .
solution contains less quantity of the solute than a saturated (i) If I:! Vmix > 0 and W mix > 0, then non-ideal solutions
solution. . . . show positive deviations, Such solutions form a constant boiling
The amount of the solute in grams which can dissolve at a mixture of defmite cOInposition (azeotropic mixture) having
particular temperature in 100 grams of the solvent when the . boiling point less than either of both the liquids. In these
solution is saturated is termed solubility of the solute. solutions . solvent-solvent and solute-solute interactions are
3. SolubUlty of a 11l8tH It liquid: Gases which can be stronger than solvent-solute interactions. Examples are:
.easily liquified are more soluble in common solvents. The gases H20+CH3 0H;H 2 0+ C2~OH; C6H6 + CH3 0H;
which form ions in water are highly soluble in water. The C 6H6 + CCI 4 ; (CH 3 hCO+ C 6H6; (CH 3 h CO+ C2~OH.
solu~ility of a gas generally decreases with rise in temperature (ii) If I:!Vmix < 0 and W mix < 0, then non-ideal solutions
and mcreases with increase in pressure. show negative deviations, In these solutions, solvent-solvent and
Mass of gas dissolved per unit volume of a solvent is directly solute-solute interactions are weaker than solvent-solute
proportional to the pressure of the gaS at a given temperature. interactions. Such solutions also form azeotropic mixture having
This is Henry's law. boiling point higher than either of the two solvents. Examples
m KP (at constant temperature)
where, m is the mass of gas dissolved per unit volume of solvent,
P is pressure of the gas in equilibrium with the solution and K is
proportionality constant.
357

CC'li!ame ~ .JIldfiditinn .'


•"1i1mDmtla --" -

}, Mass percentage %(WIW) Mass of solute Number of parts by mass of solute per hun- No effect
x 100
Mass of solution dred parts of the solution.
1.,. Gram per litre gIL Mass of solute in grams Amount of the solute in grams present in Changes' with change
Volume of solution in litres - one litre of solution. of temperature.

$, Parts per million ppm Mass of solute Number of parts by mass of solute per 1(f No effect
X 106
Mass of solution parts of solution.
4, Molarity M Number of moles of solute Number of moles of solute per litre of solu- Changes with challge
Number of litres of solution tion . of temperature. '

~. Molality m . Number' of moles of solute Number of moles of solute present in I kg No effect


Number of kilograms of the solvent of the solvent.
l), Normality N Number of gram equivalents of solute Number of gram equivalents of the solute Changes with change
<
Number of litres of solution present in one litre of the solution. of temperature.

1.. Mole fraction X ~ Ratio of number of moles of one component No effect


nA + nB . to the total number of moles of solution.

$. Formality F Number of formula mass Number of formula mass in grams present Changes with change
Number of litres of solution per litre of solution. of temperature.

-
(" ~"W§ mw~ The partial pressure of any volatile
constituent of a solution at a constant temperature is equal to the
pressure. (po ~ Ps J is called relative lowering in vapour
vapour pressure of pure constituent multiplied by mole fraction
pressure.
of that constituent in the solution.
The relativ~ lowering in vapour pressure of a solution
P A == mole fraction of A x p~ ~ X A X p~ containing non-volatile solute is equal to the mole fraction of the
In the case of binary solution of two volatile liquids, the total solute present in the solution. this is Raoult's law.
vapour pressure of such solution can be given as: Mathematically,
Total vapour pressure of solution == p~ x X A + p~ X X B Po - Ps X n
. Po == solute == n+N
== P A + PB
Mole fraction of component A in vapour phase, Let w g of the solute (mol. mass == m) be dissolved in W g of
i. e., YA ==~== p~ XA solvent (molecular mass == M).
P p~ X A + p~ XB Po-Ps .wlm
Po wlm+WIM
p~ XB
similarly, YB == --~--=-- If the solution is very dilute wi m in denominator can. be
p~ X A + p~ XB neglected.
7. etl~y~ "~~~} The properties of dilute Po - P s _ wi m_ wM !'l.p
solutions containing non-volatile solutes, which depend upon the Po - WI M - mW or Po m A wB
number of solute particles in solution are called colligative
properties. These properties do not depend upon the natUre of ' .. Eley!l;Cicm M Wi~~: The boiling point of a
solutes and solvents. The four colligative properties are: solution containing non-volatile solute is always higher than the
(i) Relative lowering in vapour pressure boiling point of a pure solvent. The increase in boiling point is
(ii). Elevation in the boiling point termed elevation and represented' as !'l.Tb'
(iii) Depression in the freezing point I1Tb oc Molality of the solution
(iv) Osmotic pressure.
or !'l.Tb == m x K b . (K b is a molal elevation constant)
All these properties are dependent on the concentration of
solutes in solution. lOOOxK b Xw IOOOxK b xW A
or !'l.Tb == or . !'l.Tb == -----"---.....:-::-
!S.• ~o/~ JMy~iJ.l t'qHf "~fg:: When' a Wxm wBxmA
non-volatile solute is dissolved' in a solvent, its vapour
pressure decreases. ( Po - P s )is known as lowering in vapour where, W == mass of solute; m ==. molecular mass of solute;
W == mass of solvent
358 G.R.B. PHYSICAL CHEMISTRY FOR COMPETITIONS

RT? . osmotic pressure istenned hypotonic and the solution having


K b is equal to - - 9 - . higher osmotic pressure is termed hypertonic solution. The two
lOOOL
solutions having equal osmotic pressures are termed as isotonic .
.' whet"~.W"1);:: boiling point of solvent in.absolute scale; Condition for isotonic solutions is'' . •' .
L =latent heat of vaporization
3-=~
The units ofKb areK kg mol~l. miVI mVZ 2
10. Depression in freezing point: The freezing point of a
A 0.91% solution of pure NaCI is isotonic with human blood
solution containing non-volatile solute is always less than the
cells (RBC). An NaCI solution with concentration less than
freezing point of a pure solvent. The decrease in freezing point is
0.91% is called hypotonic and RBC will swell and burst in the
tenned depression and represented as AT, .
solution. An NaCI solution with concentration greater than
AT, Oc:: molality of the solution 0.91 % is called hypertonic and RBC will shrink in this solution,
i. e. , will "undergo plasmolysis or crenation.
or AT, =m x K, (K, is a molal depressionconstant)
12. van't Hoff fador: When the' solute undergoes
. l~x~xw lOOOx~x~ dissociation or association in solution, the number of particles
or AT, = or AT, =---..::..--- in solution increases or decreases and thus, the colligative
Wxm WB xmA
property changes accordingly. In 1886, van't Hoff introduced a
where, W= mass of solute; m = molecular mass of solute; factor' i' known as van't Hoff factor to express the extent of
dissociation or association of the solute in solution.
W = mass of solvent
van't Hoff factor, i
. . RT? .
K, is equal to __0_. . Observed colligativeproperty
. lOOOL . =------~-~-~~~~~-
Theoretical or nonnal colligative proPerty
where, To = freezing point Of solvent in absolute scale;
No. of particles atter dissociation or association
L =latent heat of fusion. =------~---------~-------------
No. of particles when there is no dissociation or association
The units of Kf are K kg mol-I .. Normal molecular mass of the solute
=
n. Osmosis and osmotic pressure: When a solution is Observed molecular mass of the solute
separated from the pure solvent with the help of a semipenneable
membrane, the solvent molecules from' the pure solvent move From the value of' i' it is possible to detefmine the degree of
towards· the solution: The movement of solvent molecules from dissociation or association.
low . concentration to higher concentration through In case of dissociation 'i' is greater than I.
semipenneable membrane is tenned as osmosis .. . I+(n-l)a
I =--'---'--
A.s a result of osmosis, a p~sure is developed which is 1
tenned as osmotic pressure, i. e. , the hydrostatic pressure
where, n is the number of particles per molecule and (l is the
which develops oil account of osmosis is called osmotic pressure.
. degree of dissociation. ,
It is equal to h· d . g. It is also found that if an equal and opposite
force is applied on the solution, the osmosis stops. Thus, the i-I
or a=---
excess pressure that .must be applied on the solution as to prevent· (n -1)
osmosis is also the measure of osmotic pressure.
Similarly, in case of association~ , i' is less than I.
Osmotic pressure of a dilute solution is given by van't Hoff
equation, P = CRT, where C = concentration, R = solution
constant and T =absolute temperature.
1+(;-1) a
i=-------
C=!!.=~ 1
V mxV.
where, n is the number of molecules which are associated to fonn
where, w= mass of solute in grams one giant molecule and IX is the degree of association.
V = volume of solution in Htres i-I
or
m =molecular mass of solute

i.e., P=~.RT
mxV
Molecular masses of electrolytes observed by the methods
Best method for the determination of osmotic pressure is based on colligative properties are always leSs than nonnal
Berkeley and Hartley's method. molecular masses.
The tenn hypertonic and hypotonic are applied to compare the
osmotic pressures of two solutions. The solution having lower
SOLUTIONS

13. Calculation of osmotic pressure from lowering of . To


,t7~_ avoid the freezing of water in ~(lar rildi~tor, in the
vapour pressure: - su~~o weather, ethylene glycol is adde,ftOiowc:r the freezing
Po -P fUnB point of water.' .
--=-- 18. Freezing point is same as melting point. Hence, instead of
Po RdT
depression in freezing point, depression in melting point can be
where, Po =vapour pressure of pure solvent determined. For this purpose camphor is used as solvent because
P =vapour pressure of solution it has high molal depression constant, i. e. , 40 K m -I . .
7t = osmotic pressure
mB =molecular mass of solvent 19. Relationship between different colUgative properties:
d-= density of solution in gmV I (i) Osmotic pressure 7t and boiling point elevation I1Tb:

= dRT
R gas constant in attn mL I deg I mol, i. e. , 82.1 . 7t =I1Tb X - --
lOOOK
14. Variation of vapour pressure with temperature b
(Clausius-ClapeyrOli equation): (ii) Osmotic pressure 7t and freezing point depression I1T, :
log 10 (PPr
2
) =~ [..!.. - ..!.]
2.303R 7j T2 7t =I1T, x - dRT
--
lOOOK ,
where, ~ =vapour pressure at7j (iii) Elevation in boiling point and relative lowering of
P2 =vapour pressure at T2 vapour pressure:
fjJ[ = latent heat of vaporization per mole
15. Variation of vapour pressure of a liquid with external
I1Tb =tOOOK b X -l1P ; mA =molar mass ofsolvent
mA Po
pressure:
(iv) Depression in freezing point and relative lowering of
IOglO(P2)=
PI 2.303RT
(P2-~) vapour pressure:

where, PI =vapour pressure at external pressure PI


P2 =vapour pressure at external pressure P2
20. Hygroscopic and deliquescent substances both absorb
V, =molar volume of liquid. moisture from the air. The former do not change their state, e.g. ,
16. Molecular masses of polymers are best determined by glycerol, whereas the latter dissolve into it, e.g., NaOH. They
osmotic pressure method because other colligative properties absorb moisture because the vapour pressure of their saturated
give so low value that they cannot be measured accurately. solution is'iess thantbat of water vapours present in surroUndings
at that temperature.
The substances like Na 2COl ·1 OH 2o which lose their water or
crystallisation are called emorescent.

/
• .360 G,R.B. PHYSICAL CHEMISTRY FOR COMPETITIONS

Questions
1. Match the following:. [D] Match the solutions in Column-I with their osmotic
Set-I properties in Column-II:"
Set-II
Column-I Column-II
(A) 10 vol HzOz 1. Perhydrol
. (a) SI: 0.1 M glucose, (P) S 1 and S 2 are isotonic
(B) 20 vol H Z02 2.5.358N Sz: O. lM urea ;
(e) 30 vol H Z02 , 3.1.785N (b) SI: 0.1 M NaCI, (q) No migration of solvent
Sz-0:lMNazS04 ~ross the membrane
(D)100 vol HzOz 4.3.03 %
(c)SI:O.lMNaCI, (r) 8.1 is hyp.ertonic to Sz
The correct match is : . Sz:O.1MKCl
(a)A-4,B- 3,C-2,D-I (d) SI:O.l M CuS04 , (s) SI is hyPotonic to S2
(b)A-l,B-2,C-3,D-4 Sz: O.IM sucrose
(c}A - I, B-3, C - 2, D- 4 [Note; Assume that the electrolytes are Completely ionised.]
(d) A- 4, B-2, C-:- 3, D -I (CPMT (UP) lOO8] [E] Match the solutions in Column-I with their colligative
2. _Matrix Matchillg Problems (For lIT asp.irants): properties in Column-II:
[A] Match the solutes in Column-I with the van't Hoff factors Column-I Column-II
in Column-II: '
(P) Solution with highest
Column-I Column-II
boiling point
(a) K 4 [Fe(CN)6] (P}I+IX
(b) 0.1 M NaCI (q) Solution with van't Hoff
(b) Al Z ( S04)3 (q) Greater than 1
factor greater than 1
0.
II (c) 0.1 M glucose (r) Solution with lowest
(cl NHZ-C-NH2 .(r) (1 + 4a- ) osmotic pressure
(d) CaClz (s) I (d) 0.1 M CaClz (s) Solution with lowest
IX =Degree of ionhation. freezing point
[B] Match the s,olutionsin Column-I with their nature in .[Note: Assume that the electrolytes ate completely ionised.]
Column-II: . [FJ Match ..the concentration terms of List-I with their
Column-I Column-iI informations in List-II: .
(a) Benzene + toluene (p) Non-ideal solution List-I List-II
(b) .Ethanol + water (q) Ideiil solution (a) Molarity (P) Number of gram formula mass
(c) Benzene + chloroform (r) !llf mix > 0 of solute dissolved per litre of
solution
(d) Carbon tetrachloride ( s) !llf mixing = 0
+ chloroform (b) Molality (q) Number of moles of solute
[C] Match the s0lutions in Column-I with their nature in dissolved per kg of solvent
. Column-II: (c) F<lrmality (r) Depends on temperature
Column-I Column-II (d) Strength of solution ( s) Numberof moles of solute
(a) n-hexane + n heptane (P) Can be perfectly dissolved per litre of solution
separated by distillation
(b) Acetone + chloroform (q) Maximum boiling •
azeotrope
(c) Acetone + aniline (r) Cannot be perfectly
separated by distillation
(d) Ethanol + water (s) Nearly ideal

1. (a) A-4, B-3, C-2, D-1. [D] (a-p, q); (b--s); (c-p, q); (d-r)
1. [AJ (a-q, r); (b-q, r); (o-s); (d-p, q) [E] (a-p, q, s); (b-q); (c-r); (d-q)
[B] (a-q, s); (b--p, r); (c-p); (d-s) [F] (a-r, s); (b-q); (o-p, r); (d-r)
[C] (a-p, s); (b-q, r); (c-q, r); (d-r)
1. 15 g of methyl alcohol is present in 100 mL of solution. If the [JJjpj; After mIXlng number of moles of sugar present
density of solution is 0.96 g mL- l , calculate the mass
= 2 x 3 + 3 x 2.5 =13.5; Final molarity = 13.5 = 1.35 M ]
percentage of methyl alcohol in solution. . 10 .
[Aus. 15.625%] '4, If20.0 mLof 1.0 M calcium chloride and 60 mL of 0.2 M CaC1 2
2, A solution is prepared by dis~lving IS g of cane sugar in 60 g are mixed, what will be the molarity of the fmal solution?
water. Compute the mass per cent of each component of solution.
[J\.Ds. 0.40 M]
[4»s, Mass percentage of sugar 20%, Mass percentage of water
80%]
IS. Calculate the molarity of each of the ions in a solution when
3.0 litre of 4.0 MNaCI and 4.0 litre of2.0 MCoCI 2 are mixed
$. The density of the solutionofa salt X is 1.15 gmL- I .20 mLof . and diluted to 10 litre.
the solution when completely evaporated gave a residue of
[.I\.I),s. Molarity Na + = 1.2 M; Molarity C02+ = 0.8 M; Molarity
4.6 g of the salt. Calculate the mass percentage of the solute in
solution. CI- =2.8 M]
[ADS. 20%] [lfint: NaCI = Na+ + CI-
I rool. I roole 1 rool.
4. (a) 5.85 g ofNaCI is dissolved in 200 mL of water. What will 3x4roole 12roole 1211101.
be the molarity of this solution?
Co0 2 = Co 2 + + 2Cr .
L4JJs.. 0.5 M] I roole. I mole 2 roole
2 x 4 rook 8 roole 16 roole
(b) Calculate the molarity of the solution obtained by
dissolving 20.6 g NaBr in 500 mL of water. . Total CI- ions 28 mole]
[.us. 0.4 M] U;, What volume of 96% H2 S04 solution (density 1.83 g/mL) is
6.. The density of a solution containing 40% by mass of HCI is required to prepare 4 litre of 3.0 M H2 S04 solution?
1.2 g/mL. Calculate the molarity ofthe solution. [Ans. 670 rnL]
[_s.. 13.15M] '[llbl,,: First detennine the molanty of 96% H2S0 4 and then
6. Concentrated sulphuric acid has density of 1.9 g/mL and 99% dilute to get the desired solution.]
H2 S04 by mass. Calculate the molarity of the acid. 17.. Calculate the molarity of each ion in solution after 2.0 litre of
[.4...s. 19.19 M] 3.0 M AgN03 is mixed with 3.0 litre of 1.0 MBaC12 •
7. A mixture of alcohol and water contains 54% water by mass. LAos. Molarity ofBa 2+ 0.6M; Molarity of NO; = 1.2 M)
Calculate the mole fraction of alcohol in this solution.
L_. 0.25]
[mn.t: BaCI2 + 2AgN03 = 2AgCI J.. + Ba(N03)2
;8~ What amount of oxalic acid is' required to prepare 250 mL
Initial 3 mole 6 mole 6 mole Ag+ will react
Ba2+ Ag+
0.1 N solution (Given: molecular mass of oxalic acid = 126)? with 6 mole cr to fonn
[MIs. 1.575 g] 6moieCr 6 mole AgCl(s)
~. 7.45 g of potassium chloride is dissolved in 100 g of water. NO;,
What will be the molality of the solution? After 3 mole 6 mole
[Ms.' 1m] reaction Ba2+ NO;
t~. A solution is prepared by mixing 50 g sugar in 100 g of water
at 25 c C. Calculate the following: Total volume 3.0 + 2.0 =' 5 litre. ]
(i) Mass per cent of water. 18. The density of a 10.0% by mass of KCI solution in water is
(ii) Mole fraction of sugar. 1.06 g/mL. Calculate molarity, molality and mole fraction of
(iii) Molality of the solution. KCI in this solution.
[Aas. (i) 66.6 (ii) 0.0256 (iii) 1.46 m] [~s.. Molarity 1.42 M; Molality = 1.489 m; mole fraction of
U. A solution contains 25% water, 50%ethanoic acid and 25% KCI=0.026)
ethanol by mass. Compute the mole fraction of each 19,. Calculate the molality and mole fraction of the solute in an
component: aqueous solution containing 6 g of urea per 500 g of water
L.. Mole fraction of water = 0.5022; (MoL mass of mea =:: 60).
Mole fraction of ethanoic acid = 0.3013; [AIls. Molality =0.2 M; Mole fraction =0.00359]
Mole fraction of ethanol. 0.1965] 2.0.. Calculate the molality of I litre solution of 93% H2 804
12. Find the amount of 98% pure Na 2 C03· required to prepare (mass/volume). The density of the solution is 1.84 g/rnL.
5 litres of2 N solution. , <ML.NR 1993)
[Ans. 540.8 g impure Na2C03] [ .....s.. 10.428 mJ ,
13. Calculate the final molarity when 2.0 litre of 3.0 M sugar 21. Molarity of H2 S04 is 0.8 and its density is 1.06 g/cm 3. What.
solution and 3.0 litre of 2.5 M sugar solution are mixed and will be its concentration in tenns of molality and mole
then diluted to 10 litres with water. fraction. ICBS.E (Mains) 20OS}
[Aus. m = 0.814; xH2S04 :: 0.014 )
362 G.R.B. PHYSICAL CHEMISTRY FOR COMPE:TITIONS
, .
22. A 6.90 M solution of KOH in water contains 30% by mass of 33. Calculate the mass of a non-volatile solute of molecular mass
KOH. Calculate the density of the solution. 40, which should be dissolvCd in 114g of octane to reduce the
[Ans. 1.288 g mL-1 ] vapour pressure to 80%.
[Ans. 10 g]
23. How many kilograms of wet NaOH containing 10010 water are
34. to g ofa certain non-volatile Solute was dissolved in 100 g of
required to prepare 60 litre of a 0.5 M solution? .
water at 20° C. The vapour pressure was lowered from 17.3555
[Ans. 1.33 kg] to 17.235 mm. Calculate the molecular mass of the solute.
24. In an experiment, 18.0 g of mannitol was dissolved in 100 g of [Ans. 257.45]
water, The vapour pressure of solution at 20° C was 17.226 mm 35. 0.534 g of solute is dissolved in 15 g of water then freezing
of mercury. Calculate the molecular mass of mannitol. The point temperature changes from O°C to -1.57°C. Molal
vapour pressure of water at 20°Cis 17.535 mm afmercury. depression constant of water, K f = 1. 85 K kg morl . Find out:
[Ans. Molecular mass of mannitol =181.02]
(i) Molal concentration
25. The vapour pressure of water at 200Cis 17.53 mm. Calculate
(ii) Molecular mass of solute (AIPMT (Mains) 20091
the vapour pressure of a solution at 200 C containing 6 g of (Alis. 0) 0.848 (ii) 41.98 amu]
urea in 100 g of water (molecular mass of urea = 60). 36. Addition of 0.643 g of a compound to 50 mL of benzene
[Ans. Vapour pressure of solution = 17.22 nun] (density 0.879 g/mL) lowers. the freezing point from 5.51° C to
=
26. The vapour pressure of ether (molecular mass 74) is 442 mm 5.03° C. If K f for benzene is 5.12,' calculate the molecular
Hg at 293 K. If 3 g of a compound A are dissolved in 50 g of mass of the compound. (lIT 1992)
ether at this temperature, the vapour pressure falls to 426 mm [ADS. 156]
Hg. Calculate the molecular mass of A aSsuming that the 37. A solution made by dissolving 0.32 g of a new compound in
.' solution of A is very dilute. 25 g of water has freezing point 0.201°C. Calculate the
[Ans. m = 123] molecular mass of the new compound.
[Ans. 118.4]
27. 18.2 g of urea is dissolved in 100 g of water at 50° C. The 38. The boiling point of a solution of 0.1050 g of a substance in
lowering of vapour pr~ssure produced is 5 mm.Hg. Calculate' 15.84 g of ether was found to beO.1°C higher than that of pure
the molecular mass of urea. The vapour pressure of water at ether. What is the molecular mass of the substance (Molecular
500 C is 92 mm Hg. elevation constant of ether per 100 g is 21.6)? (MLNR 1991)
[ADs. 57.05] [Ans. 143.18]
39. Calculate the' freezing point and the .boiling point at
[Hint: Use Po - P. = _n_ ,as the solution is not very dilute]
Po n+N I atmosphere of' a solution containing 30 g cane sugar
(molecular mass 342) and ISO g water.
28. Calculate the vapour pressure of a solution at IOWC Given: Kb = 0.513 and Kf = 1.86.
containing 3 g of cane sugar in 33 g of water. (Ans. . Boiling point 100.3; Freezing point - 1.09°C]
[Ans. 756.36 nun ofRg] 40. When 36.0 g of a solute having the empirical formula CH20 is
[Hint: Apply Po - P. = wM as the solution is very dilute. dissolved in 1.20 kg of water, the solution freezes at - 0.93" C.
Po Wm What is the molecular formula of the compound?
Po =760mm ofHg atlOO°Cand m:= 342] I [Ans. C2R 40 2 ]
29. An aqueous solution containing 28% by mass or'a liquid A 41. In winter, the normal temperature in Kullu valley was found to
=
(molecular mass 140) has a vapourpressure ofl60 mm at be, - II ° C. Is a 28% (by mass) aqueous solution of ethylene
3T C. Find the vapour pressure of pure liquid A' (the vapour glycol suitable for a car radiator?
pressure of water at 37°Cis 150 tnm). Kf for water = 1.86 K kg mor I.
[Ans. PA = 358.3 nun] [Ans. Yes. The given solution freezes at - 11.66° C]
[Hint: Apply Ptotal := mole fraction of liquid A x PA + mole 42. Latent heat of fusion of water is 80 calJg and its normal
fraction of water x PW' Mass of water = 100 - 28 = 72 g] freezing point is O°C.Calculate molal depression constant for
30. At 25° C, the vapour pressure of pure benzene is' 100 torr, =
water. (R 1.987cal)
while that of pure ethyl,alcohol is 44 torr. Assuming ideal [Ans. 1.86 K kg mol-I]
behaviour, calculate the vapour pressure at 25° C of a solution 43. A solution containing 7.5 g of urea (molecular mass = 60) in
which contains 109 of each substance. I kg ·of water freezes at the same temperature as another
[ADS. 64.775 torr] solution containing 15 g of solute' A' in the same amount of
3] • Estimate the lowering of the vapour pressure due to solute in a water. Calculate molar mass of' A'.
] .0 molal aqueous solution at 100° C [ADs. 120]
[ADs. 14nunofRgorl4torr] 44. Calculate the osmotic pressure of a decinormal solution of
32. Find the molality of a solution containing a non-volatile solute NaCI which is ionised to 80% at 27° C.
if the vapour pressure is 2% below the vapour pressure of pure [Ans. 4.43 atm]
water. (Dhanbad 1990)
45. What is the volume of a solution containing 2 g mole of sugar
[Ans. 1.133 m]
that will give rise to an osmotic pressure of I atmosphere at
S1lP? .
[ADs. 44.81itre]
SOLUTIONS

46. If the osmotic pressure of 5 g per litre solution of a compound 55. What should be the osmotic pressure of a solution of urea in
at 27° Cis· 0.025 atm, calculate the molecular mass of the water at 300C which has boiling point 0.052 K higher than
compound. pure water? Assume molarity and molality to be the same. K b
[Ans. 4926] for water is 0.52 K kg moi-I .
47. The osmotic pressure of blood is 7.65 atmosphere at 37°C. [Ans. 2.487 atm]
How much glucose should be used per litre for an intravenous 56. 4.0 g of a substance A dissolved in 100 g H 20 depressed the
injection that is to have same osmotic pressure as blood? freezing point of water by 0.1° C while 4.0 g of another
[Ans. 54.18 g of glucose per litre] substance B depressed the freezing point by 0.2° C. Which
48. 10 g of cane· sugar (molecular mass = 342) in I x 10-3 m 3 of one has higher molecular mass and what is the relation?
solutioh produces an osmotic pressure of 6.68 x 104 Nm-:-2 at [Ans. M A =2MH ]
57. A solution containing 28 g of phosphorus in 315 g of CS2
273 K. Calculate the value of S in SI units.
(boiling point 46.30 C) boils at 47.980 C. The molal elevation·
[Hint: S == PV constant for CS 2 is 2.34 K kg mol-I. Calculate the molecular
nxT
mass of phosphorus and deduce its formula in CS 2 •
Given: V == 1 X 10-3 m 3; P == 6.68 x 104 Nm-2;
[Ans. 123.8, P4]
10
n==- T=273 58. A solution of two volatile liquids A and B obeys Raoult's law.
342'
3
At a ce.rtain· temperature it is. found that when the pressure
S = 6.68 x lit x 1 x 10- x 342 = 8.3684 J K- 1 mol-I] abQve the mixture in equilibrium is 402.5 mm of lig, the mole
10 x 213 fraction of A in the vapour is 0.35 and in the liquid it is 0.65.
49. A 250 mL water solution containing 48 g of sucrose What are the vapour pressures of two liquids at this
=
(molecularrnass 342) at 300 K is separated from pure water temperature?
by means of a semipermeable membrane. What pressure must [Hint: Let PAand PH be the partial pressures of vapours and p~
be applied on solution as to prevent osmosis? and p~ be the vapour pressures of two liquids respectively.
[Ans. 13.8 atm]
50. What would be the osmotic pressure of 0.05 M solution of
PA =0.65 PAoand· PH =0.35PH0
sucrose at 5° C? Find out the concentration of a solution of PA + PH =402.5 (given)
glucose which would be isotonic with this solution of sucrose. In vapour phase,
(Molecular mass of sucrose = 342, Molecular mass of ~ =0.35 or PA = 0.35 x 402.5 = 140.875 rom
glucose = 180) PA + PH
[Ans. 1.14 atm;.Concentration of glucose soln. = 9.0 gIL] So, p~ =216.72 rom and pZ = 747.5 rom]
51. Calculate the osmotic pressure of 0.5% solution of glucose
59. Calculate the amount oeice thatwill separate out on cooling a
(molecular mass 180) at 18° C . The value of solution constant
solution containing 50 g of ethylene glycol in 200 g water to
is 0.0821·litre-atm K- I mol-I.
9.3°C(Kf for water = 1.86K kg mol-I). . (liT 1995)
[Ans. 0.6628 atm]
52. At 10° C , the osmotic pressure of urea solution is 500 mm. The [Hint: AT =Kf Xm
solution is diluted and the temperature is raised to 2SO C, when =
9.3 1.86·x m
the osmotic pressure is found to be 105.3 mm. Determine or m=5
extent of dilution. Thus, solution of5.0 molality will have freezing point of - 9.3 0 C.
[Hint: Original solution ... (i) Let W g water be needed to prepare 5.0 molal solution containing
50
- mole of ethylene glycol.
After dil:ution ... (ii} 62 . .
So, 50 x 1000 =5.0
or 62xW

or W =50 x 1000 =161.3 g


or 62x 5
Hence, ice separating out = (200 -161.3) = 38.7 g)
i. e., the solution is diluted to the extent that V2 is nve times the
60. At 293.2 K, the density of a 60% aqueous solution of methanol
original volume (VI}]
is 0.8946 g/mL. Calculate volume of,l mole ofthe solution.
53. Two solutions containing respectively 0.75 g of urea
[Ans. 28.46 mL/mol]
(molecular mass 60) and 1.5 g compound X in 100 g of water
boil at the same temperature. Calculate the molecular mass of [Hint: Let volume of solution be 100 mL
X. ... W = 100 x 0.8946 = 89.46 g
[Ans. 120] 60
I1cH OH =- = 1.875
54. A 5% solution of glucose is isotonic with 1.1 % solution ofKCI J 32
at 300 C. Calculate the degree of ionisation ofKCl. - 29.46 _ 1 636
[Ans. 0.88 or 88%]
----.& - .
~20 -

Total moles =1.875 + 1.636 =3.511


-
364
100 LAJJ~, 634.3 mm]
Volume per mole 28.48 mL/mol]
3.511 [Rint$Use Clausius-Clapeyron equation.

hl.. 1.2 kg ethylene glycol [ ~H20H 1was added in a car radiator 10glO [
P2
fJ J
1=~ (.! J..J]
2.303R Ii T 2.
CH20H
99, Calculate the increase in vapour pressure of water per
containing 9 litre water. The freezing of water was just atmosphere rise in external pressure at 10 0 C. The vapour
prevented when car was runnirig in the Himalayan valley at pressure of water at 10° C and I atm is equal to 9.2 mm.
temperature 4°C. Sudden fuunderstorm in the valley
[Am, 0.1%]
lowered the temperature to 6°C. Calculate the amount of ice
separated.
[A~s. 3 kg ice]
[Hint; Use loglO(~J = 2.303RT (Pz fJ)
x 1000 VI ~ molar volume of liquid]
t:.T K f x A --? Solute; B --? Solvent
x we
mA An aqueous solution of H2 S04 has density 1.84 g/mL.
6 == 1.86 x 1200 x 1000 Solution contains 98% H2S04 by mass. Calculate:
62xwe , (i) molarity of solution
We == 6000 g (ii). overall molarity of solution
[Weight of ice =Total weight of H2 0
wt. of H2 0 at - 60 C (iii). molar volume
9000 6000 3000g == 3 kg] .(iv) specific volume
2 g mixture of glucose and sucrose is dissolved in 1 litre water (v) relative decrease in vapour presStlre with respect to
at 298 K to develop osmotic 'pressure of 0.207 atm. Calculate .water, assuming H2S04 as non-ele«trolyte this high at
percentage composition of glucose and sucrose by mole as concentration.
well as by mass. [~t) (i) . M == x x d x 10
[~. me
Percentage by mole: glucose 65.194, sucrose 34.806; == 98 x 1.84 x 10 = 18.4
Percentage by mass:, glucose 50, sucrose = 50.] 98
6,3. If benzoic acid (mol. mass 122) is associated into double (ii) Weight of solution (1 litre) =1000 x 1.84 1840 g
molecules when dissolved, in benzene and the osmotic We (weight of H2S04 ) = ~ x 1840 1803.2 g
pressure of a solution of 5 g of benzoic acid in 100 mL 100
benzene ijl 5.73 atm at 10°C, what is the percentage association wA (weight of water) 1840 1803.2 36.8 g
of benzoic acid? Total number of moles = WA + we = 36.8 + 1803.2
[A»s,. 80%] mA 18
me 98
[mat;: n = iCRT = i n RT == 2.044 + 18.4 == 20.444
V Overall molarity = 20.444
5.73 =i x 5/122 x 0.0821 x 283 (iii) Molar volume = Volume = 1000
0.1 . Total moles 20.444
i = 0.6017
=48.914 mL/mol
1 0.6017 '" 0.8
1 (iv) Specific volume _1_ = _1_ mL/g 0.543 mL/g
density 1.84
% association 80% ] ,
(v) Molality (m) = We x 1000 98 x 1000 500
~. 0.1 formal solution ofNaCI is found to be isotonic with 1.10% mB x wA 98 x 2
solution of urea. Calculate the apparent degree of ionization of
x 1000
NaCl. m==
(l-xB)m A
[AIls. 0.83]
[Hi.nt·: n NaC1 = n urea 500 = --",_x_I_OO_O_
1 {1- xe)xI8
=nx RT
V Xe == 0.9

i x 0.1 x RT _1_,1_ x RT n According to Raoulfs law: == XB 0.9]


(n == RT)
60 x 0.1 V Po
i == 1.83 ~., You are given two samples of HCI with molarity 0.341 and
0.143; volume of each sample being 1 litre. What will be the
a == i I 1.83 1 0.83]
n 1 2 1 maximum volume of 0.243 M HCI that can be obtained by
mixing the given samples in the following two conditions?
65. The vapour pressUre of water at 100° C is 760 min. What will (i) When water is added for dilution.
be the vapour pressure at 95° C ? The latent heat of water at (ii) When no water is added.
this temperature range is 548 callg. [~s.. (i) 2.02041itre; (ii) 1.98 litre]
I
365

[Hiiif~ (i) Letxlitreof0.341 MHCland 1 litre of 0.143 MHCI Use, Po. P 'ltm
be mixed to obtain 0.243 M HCL Po dRT
MIVj + M ZV2 ==MR(V1 + V2 ) 1- P 25.224 x 18
0.341xx+0.143xl 0.243 (i+x) 1 (1 x x 0.0821 x 373
(0.341- 0.243)x 0.243 - 0.143
P =0.958 atm]
0.098x == 0.1
71, Calculate the osmotic pressure at 25 ° C of a solution
x == 1.0204 litre
containing 1 g of glucose and I g of sucrose in I litre of
Total volume 1 + x == 2.0204 litre solution. . .
Since, vOlume is greater than 2 litre, hence water is to be added.
If it were not known that the solute was a mixture of glucose
(ii) Let x litre of 0.143 M HCI and I litre of 0.341 M HCI are
and sucrose, what would be the molecular weight of solute
mixed to obtain 0.243 MHCl.
corresponding to the calculated osmotic pressUre?
M1V1 + M 2V2 == MR(V1 + V2 ) [,1\118; 0.207 atm,236.384]
0.143 Xx+ 0.341 xl == 0.243(1 + x)
[Hlriti Use; 'ltV = (nl + nz)RT to calculate osmotic pressure
O.lx == 0.098 I I \ .
x == 0.98 litre 'It X 1 = + -j
180 342
x 0.0821 x 298
Total volume == 1 + x == 1.98 litre
Since, volume is less than two litres, no water is to be added.] == 0.207 atm
69. What is the vapour pressure of a solution of glucose which has Use; 'ltV == ~ RT to calcuJate molar mass of solute.
an osmotic pressure of 3 atmosphere at 20° C? The vapour
/ m
pressure of water at 20°C is 17.39 rnm. Consider the density 2
0.207 xl -:- x 0.0821 x 298
of solution equal to that of solvent. m
[Ans. 17.35 mm] m = 236.384]
[Hiiiti Use: l!.p == 'ltm ] 7i. 100 mL aqueous solution of glucose with osmotic pressure
Po' dRT 1.2 atm at 25°C is mixed with.300 mL aqueous solution of
urea at 2.4 atm at 25° C. Calculate the osmotic pressure of
70. The osmotic pressure. of an aqueous solution of a
mixture.
non-electrolyte is 18.8 atm at 15° C. What will be the vapour
pressure of this solution at 100° C (Density of water at [Anj, 1.68 atmj
100°C"" 19/cc)? [itiiiti Use: 'ltIVI + 'ltzVz ='ltR(VI + V2 )
[Ans. 0.958 atm]
[Hliitl Use, ~ == 'lt2 to calculate osmotic pressure at 100°C.
TI T2
1.2 x 100 + 2.4 X 300 :.':: 1.68 atmj'
18.8 =~
500 .
278 373
'lt2 25.224 atm

(a) amorphous (b) azeotropic mixture


Set..,: QuestlOn8wlth·slnglecorreOtaniwer ( c) supersaturated (d) ideal
1. A solution is defined as a: 4. Azeotropic mixtures are:
(a) homogeneous mixture of tWo or more substances (a) mixture of two solids
(b) heterogeneous mixture of two or more substances (b) those which boil at different temperatures
(c) homogeneous mixture of liquid and solid components' (c) those which can be fractionally distilled
only (d) constant boiling mixtures
(d) homogeneous mixture. consisting of water as one of the S, The solubility of a gas in a liquid is directly proportional to the
components pressure ofthe gas. This statement is:
2, Ideal solution i;; formed when its components: (a) Raoult's law (b) Henry's law
(a) have zero heat of mixing only (c) van't Hoff's law (d) None of these
(b) have zero volume change on mixing only 6, Which of the following is not correct for an ideal solution?
(c) have zero heat of mixing and ZterO volume change (a) Raoult's law is obeyed for entire concentration range and
(d) can be converted into gases temperatures
). Solutions which distil without change in' composition or (b) Mimix = 0
temperature are called: (c) l!.Vmix 0
(d) 8.S mix 0
... -------..---...-. --------------

366 G.RB .. PHYSICAL CHEMISTRY FOR COMPETITIONS

7. The solubility of a gas in liquid increases with: 18. 'Among the following, that does not form an ideal solution is:
(a) increase in temperature (a) C6~ and C 6HsCH3 (b) C 2HsQ and C 2 Hs0H
(b) reduction of gas pressure . (c) C 6HsCI and .C6HsBr (d) C 2 HsBr and C 2HsI
(c) decrease in temperature and increase of gas pressure 19. Which of the following solution pairs can be separated by
(d) amount of liquid taken fractional distillation?
8. Saturated solution ofNaCI on heating becomes: (a) Water-HN03 (b) Water-HCI
(a) supersaturated (b) unsaturated (c) Benzene-toluene (d) C 2Hs0H-water
(c) relnains saturated (d) none oftbese 20. When two liquids A and B are mixed, their boiling points
9. A supersaturated solution is a metastable solution whose bec9me greater than both of them. The mixture is:
concentration: (a) ideal solution
(a) is equal to the solubility of the substance in solvent (b) non-ideal solution with negative deviation from Raoult's law
(b) is less than the solubility (c) non-ideal solution with positive deviation from Raoult's law
(c) exceeds the solubility (d) normal solution .
(d) continuously changes 21. The azeotropic mixture of water (b. pt. 1000 C) and HCI
10. When a crystal of a solute is introduced into a supersaturated (b. pt. 85° C) boils at 108.5 0 C. When this mixture is distilled,
solution of the solute: . it is possible to obtain:
(a) the solute dissolves (a) pure Hel
(b) the solution becomes unsaturated (b) pure water
(c) the solution remains supersaturated (c) pure water as well as HCI
(d) . the excess of solute crystallises out . (d) neither HCI nor H20 in their pure states
11. An electrolyte dissolves in water if: 22. Pressure cooker reduces cooking time because:(CPMT 1990)
(a) lattice energy is less than hydration energy (a) the heat is more evenly distributed inside the cooker
(b) . lattice energy is greater than hydration energy (b) a large flame is used
. (c) lattice energy is equal to hydration energy (c) boiling point of water is elevated
(d) dissolution is endothermic (d) whole matter is converted into stearn
12. The solubility of a substance is defined as the amount of solute 23. A molal solution is one that contains one mole of a solute in:
in grams; (a) 1000 g of the solvent (b) one litre of the solution
(a) present in 100 g of the solvent
(c) one litre of the solvent (d) 22.4 litre of the solution
(b) present in 100 g of the solution
24. In which mode of expression, the concentration of the solution
(c) present in 100 mL of the solution remains independent of temperature?
(d) present in I litre ofthe solution.
(a) Normality (b) Molality (c) Molarity (d) Formality
13. Which of the following will form an ideal solution?
25. 8 g NaOH is dissolved in one litre of solution. Its molarity is:
(a) C 2 Hs0Hand water (b) HN03 and water
(a) 0.8 M (b) 0.4 M (c) 0.2 M (d) 0.1 M
'(c) CH0 3 and CH3COCH3 (d) C6~ and C 6HsCH3
14. Which of the following shows positive' deviation from 26. If 18 g of glucose is present in 1000 g of solv~nt, the solution
Raoult's law? . is said to be: '." ' . ..
(a) C6~ and C 6HsCH3 (b) C6~ and C0 4 t
·'i/?:]ilfImoliB.' 1(6to, Im9.,I~:(crq;~~Iilf :M>Q.I molal~·:-~:::·.
27.' Th~ molarity ofpuie w~ter is: .... [CMC (VeUore) 1991]
(c)CH0 3 and C 2Hs0H (d) CHQ3 and CH3COCH3 .
15. Which of the following shows negative deviation from (a) 100M (b) 55.6M (c) 50M (d) IBM
Raoult's law? 28. For preparing' 0; I' M solution of H2 S04 in one litre,we need
(a) CH0 3 and acetone (b) C6~ and C 2Hs0H H2S04 :
(c) C 6HsCH3 and C6~ (d) C6~ and CCl 4 (a) 9.8g (b) 4.9 g (c) 49.0 g (d) 0.98 g
16. An azeotropic mixture of two liquids boils at a lower 29. Mole fractiOJi ofC 3 Hs(OHh in a solution of36 g of water and
temperature than either of them when: 46 g of glycerine is: (CPMT 1991)
(a) it is saturated (a) 0.46 (b) 0.36 (c) 0.20 (d) 0.40
(b) it does not deviate from Raoult's law 30, H20 2 solution used for hair bleaching is sold as a solution of
(c) it shows negative deviation from Raoult's law
approximately 5.0 g H2 0 2 per 100 mL of the solution. The
(d) it shows positive deviation from Raoult's law
molecular mass ofH2 0 2 is 34. The molarity of this solution is
1j. A maxima or minima obtained in the temperature composition approximately:
curve of a mixture of two liquids indicates: (a) 0.15 M (b) 1.5 M (c) 3.0 M (d) 3.4 M
(a) an azeotropic mixture
31. The number of moles ofsolute per kg of a solvent is called its:
(b )an eutectic formation .
(a) molarity (b) normality
(c) that the liquids are immiscible with one another
(c) mole fraction (d) molality
(d) that the liquids are partially miscible at the maximum or
32. 1000 g aqueOU$ solution of caC03 contains 109 of calcium
minimum
carbonate. Concentration of the solution is:
. SOLUTIONS 367

(a) IOppm (b) lOOppm . 35


Octane B, nB = _
..- = 0.31
(c) lOOOppm (d) 10000 ppm 114
33. A solution of CaC12 is 0.5 molJ1itre; then the moles of chloride X = . nA == 0.25 =0.45
A
ion in 500 mL will be: n A + nB 0.25+ 0.31 .
(a) 0.5 (b) 0.25 (c) 1.0 (d) 0.75 xB =0.55
34. If 5.85 g of NaCI are dissolved in 90 g of water, the mole o 0
P = PAxA + PBxB
fraction of solute is: [CMC (Vellore) 1991J
= 105 x 0.45 + 45 x 0.55
(a) 0.0196· (b) 0.01 (c) 0.1 (d) 0.2
=72 kPa]
35. What is the normality of 1 MH3P04 solution? (AlIMS 1991)
45. The vapour pressure of water at room temperature is 30 mm of
(a) 0.5 N (b) 1.0 N . Hg. If the mole fraction of the water is 0.9, the vapour pressure
(c) 2.0 N (d) 3.0 N of the solution will be: .
36. The molarity of a 0.2 N Na2COi solution will ~e: (a) 30mmofHg (b) 24 mmofHg
(a) 0.05M (b) 0.2M (c) 21 mm ofHg . (d) 27mm ofHg
(c) 0.1 M (d) OAM 46. A mixture of ethyl alcohol and propyl alcohol has a vapour
37. How many grams of H2 S04 are present in 0.25 g mole of pressure of 290 mm at 300 K. The vapour pressure of propyl
H 2S04 ? . (CPMT 1990) alcohol is 200 mm. If the mole fraction.of ethyl alcohol is 0.6, .
its vapour pressure (in mm) at the same temperature will be:
(al 2.45 (b) 24.5 .(c) 0.25 (d) 0.245 . . (AlEEE 2007)
38. Colligative properties of the solution depend upon: (a) 360 (b) 350 "
(a) nature of the solution (b) nature of the solvent (c) 300 . (d) 700
(c) number of solute particles (d) number of moles of solvent 47. When a non-volatile solute is dissolved in a solvent, the relative
39. Which of the following is a colligative property? . lowering in vapour pressure is equal to:
(AFMC2009) [AFMC 2004; PMT (Himachal) 2006J
(a) Surface tension (b) Osmotic pressure (a) mole fraction of solvent
(c) Optical rotation (d) Viscosity (b) mole fraction of solute
40. Which is not a colligative property? (c) concentration qf solute in g L- 1
(a) Osmotic pressure
(d) concentration of solute in g per 100 mL
(b) Lowering in vapour pressure
48. The vapour pressure of a dilute aqueous solution of glucose is
(c) Depression in freezing point
750 mm of mercury at 373 K. The mole fraction of solute is:
(d) Refractive index .
I' I. 1 (d) 1
41. Colligative properties are applicable to: (a) 10 (b) 7.6 (c) 35 76
(a) ideal dilute solutions (b) concentrated solutions
(c) non-ideal solutions (d) all of these [Hint:. P - P. = Mole fraction of solute
p. ..
42. The vapour pressure of a s.olution having solid as solute and
. P = Vapour pressure ofwater at 373 Kis 760mm]
liquid as solvent is: . [PET (MP) 2004J
49. The vapour pressure of a pure liquid 'A' is 70 torr at 27°C. It
(a) directly proportiomi.l to mole fraction of the solvent
forms an ideal solution with another liquid B. The mole
(b) inversely proportional to mole fraction of the solvent
fraction of B is 0.2 and total pressure of the solution is 84 torr
(c) directly proportional to mole fraction of the solute at 27°C. The vapour pressure of pure liqqid B at 27°C is:
(d) inversely proportional to mole fraction of the solute , [CET (Karnataka) 2009J
43. If Po and P are the vapour pressures of a solvent and its (a) 14 torr (b) 56 torr (c) 140 to1f (d) 70 torr
solution respectively and N, and N 2 are the mole fractions of [Hint: .PA =(1 - 0.2) x 70 = 0.8 x 70 =56 torr .
the solvent and solute respectively, then correct relation is:
PB = 84 - 56= 28 torr
(a)' P=PoN2 (b) p=:PoN,
=
(c) Po pN,. (d) p=Po(N,IN2 )
28 =0.2 x p~ ,orp~ =140 torr ]
44. On mixing, heptane and octane form an ideal solution. At SO. The vapour pressure of water at room temperature is 23.8 mm
373 K, the vapour pressure of the two liquid components Hg. The vapour pressure of an aqueous solution of sucrose
(heptane and octane) are 105kPa and 45 kPa respectively with mole fraction 0.1 is equal to:
vapour pressure of the· solution obtained by mixing 25 g of (a) 23.9 mm Hg (b) 24.2 mm Hg
heptane and 35 g of octane will be : (molar mass of heptane (c) 2L42mmHg (d) 3L44mmHg
= =
lOOgmor' and of octane 114gmol- )
l
(AIEEE 2010) 51. The vapour pressUre of pure A is 10 torr and at the same
(a) 96.2 kPa . (b) 144.5 kPa temperature when 1 g ofB is dissolved in 20 g of A, its vapour
pressure is reduced to 9.0 torr. If the molecular mass of A is
(c) 72 kPa (d) 36.1 kPa
200 amu, then the molecular mass of B is:
25
[BiDt: Heptane A, nA = tOO =0.25. (a) 100amu(b) 90amu (c) 75amu . (d) 120amu
368

st. If the various tenns in the following expressions have usual (a) 0.66M (b) 0.32M (c) 0.066M (d) 0.033M
meanings, the van't Hoff factor 'i' cannot be calculated by 6J. 'If 3 g of glucose (molecular mass 180) is dissolved in 60 g of
which of the following expression? water at 15°C, then the osmotic pressure of this solution will
(a) nV 4i nRT be:
(b) I!.Tf = ixK f Xm (a) .0.34 attn (b) 0.65 attn (c) 6.57 attn (d) 5.57 attn
(c) I!.Tb=ixKbxm 64. The relationship between the values of osmotic pressure of
(d) P~lvent -Psolution
P~lvent
=
N +n
i[_n_J 0.1 M solutions of KN03 (PI) and CH3COOH (P2 ) is:
(a) ~ = P2 (b) P2 > PI

53, Semipenneable membrane is that which pennitsthe passage (c)~= (d)PI ,>:P2
1~+P2 PI +P2
of: . . (CPMT 1:990)
. (a) solute molecules only (is. An electrolyte' A gives 3 ions and B is a non-electrolyte. If .
0.1 M solution of B produces an osmotic pressure P, then
(b) solvent molecules only
0.05 M solution of A will produce an osmotic pressure,
(c) solvent and solute molecules both
assuming that the electrolyte is completely ionised: .
(d) neither solute nor solvent molecules
(a) UP' (b) P (c) o.5P (d) 0.75P
$4, Which inorganic precipitate acts as semipenneable
66. l' molat solution of a non-volatile and non-electrolyte
membrane?
compound will produce an osmotic pressure ....... at O°C.
(a) Calcium sulphate (b) Barium oxalate
(a) 1 attn (b) 44.8 attn (c) 10.0 atm (d) 22.4 atm
(c) Nickel phosphate (d) Copper ferrocyanide
61. If 0.1 M solution of glucose andO.1 Mureasolution are placed
55. During osmosis, flow of water through a sem!penn~b!e on two sides of a semipenneable membrane to equal heights,
membrane is: [CBSE(Medi'Cal) 2006] then it will be correct to say that: .' (CaSE 1992)
(a) from solution having higher concentration only (a) there will be no net movement across the membrane
(b) from both sides of semipenneable membrane with equal (b) glucose will flow towards urea solution
flow rates
(c) urea will flow towards glucose solution
.(c) from both sides of seri:ripenneable membrane with unequal
(d) water will flow from urea solution towards glucose solution
flow rates
68. Which solution will exert highest osmotic pressure?
(d) fr.om solution having lower concentration only
(a) I M glucose solution (b) 1 Murea solution
56, At constant temperature, the osmotic pressure of a solution is:
I (c) I M alum solution (d) 1 MNaCI solution
(a) directly proportional to the conc~ntration
69. Which is the correct relation between osmotic pressure of
(b) inversely proportional to the concentration
0.1 MNliCi solution and 0.1 MNa2S04 solution?
(c) directly proportional to the square of concentration
(a) The ,osmotic pressure of Na2S04 is less than NaCl
(d) directly proportional to the square root of concentration solution
51. Osmotic pressure observed when benzoic acid is dissQlved in (b) The osmotic pressure of Na2S04 is more than NaCI
benzene is less than that expected from theoretical solution
conside~tions. This is becau~e: . ,[MGlMS '(\\lardba) 2""8] (c) Both have same osmotic pressure
(a) benzoic acid is organic solute
(d) None ofthe above
(b) benzoic acid has higher molar mass than benzene
70. A 0.6% urea solution would be isotonic with:
(c) benzoic acid gets associated in benzene [CEt(I(arnataka) >2009]
(d) benzoic acid gets dissociated in benzene
(a) 0.1 M glucose solution (b) 0.1 MKCI solution
58. Blood'cells do not shrink in blood because blood is: (c) 0.6% glucose solution (d) 0.6% NaCl solution
fPMt '(MM) ,2007]
71. Which one of the following solutions will have highest
(a) hypertonic (b) isotonic
osmotic pressure? (Assume .that all the salts are equally
(c) equimolar (d) hypotonic dissociated. )
59. Isotonic solutions have the same: (a) 0.1 M Al 2(S04h
(a) density (b) molar concentration (b) OJ MBaCl 2
. (c) nonnality (d) strength
(c) 0.1 MNa2S.o4
60. The osmotic pressure of solution increases if: (CPMT 1991) (d) The solution obtained by mixing equal volumes of (b)
(a) temperature is decreased and (c)
(b) concentration is decreased 72. The following solutions have equal concentratioBS. Which one
.(c) number of solute particles is increased will show minimum osmotic pressure?
(d) volume is increased (a) BaCl2 (b) AgN03
61. The, osmotic. pressure of a 5% solution of cane sugar (c) Na2S04 (d) (~hP04
(molecular mass 342) at 15°C is: 73. A solution of a substance containing 1.05 g per 100 mL was
(a) 3.46 attn (b) 3.64 attn (cl 4.0 atm (d) 2.45 atm found to be isotonic with 3% glucose solution. The molecular
61. A solution has an osmotic pressure of 0.821 atm at 300 K. Its , mass of the substance is:
concentration would b e : ( I I T 1990) (a) 31.5 (b) 6.3 (c) 630 (d) 63
SOLUTIONS 369
74. The osmotic pressure of equimolar solutions of BaCI 2 , NaCI (c) boiling point of the solution becomes higher than that of
and glucose will be in the order:· solvent .
(a) glucose> NaCI > BaCl 2 (b) BaCl 2 > NaCI > glucose (d) all are correct .
(c) NaCl> BaCl 2 > glucose (d) NaCl > glucose> BaCl 2 86. The molal elevation constant is the ratio of the elevation in
boiling point to:
75. At 25°C, the highest osmotic pressure is exhibited by 0.1 M
solution of: (CBSE 1994) (a) molarity (b) molality
(c) mole fraction of solute (d) mole fraction of solvent
(a) CaCl2 (b) KCI (c) glucose (d) urea
87. In cold countries, ethylene glycol is added to water in the
76. The plant cell will shrink when placed in:
radiators of cars during winters. It results in:
(a) water (b) a hypotonic solution
(a) reducing viscosity (b) reducing specific heat
(c) a hypertonic solution . (d) an isotonic solution
(c) reducing freezing point (d) reducing boiling point
77. The osmotic pressure of it solution at 273 K is 2.5 atm. Its
88. The best colligative property used for the determination of
osmotic pressure at 546 K under similar conditions will be:
molecular masses of polymers IS:
(a) 0.5 atm (b) 1.0 atm (c) 2.5 atm (d) 5.0 atm (a) relative lowering in vapour pressure
78. Which one of the following pairs of solutions will be expected (b) osmotic pressure
to be isotonic under the same temperature? (c) elevation in boiling point
. [CET (Gujarat) 2006; AMU (Medical) 20091
(d) depression in freezing point
(a) 0.1 MureaandO.l MNaCI
89. Ebullioscopy is concerned with:
(b) 0.1 Murea and 0.2 MMgCl2
(a) osmotic pressure of a solution·
(c) 0.1 MNaCI and 0.1 MNa2S04
(b) elevation of boiling point of a solution
(d) O.l MCa(N03h and 0.1 MNa 2 S04 (c) depression in freezing point of a solution
79. An aqueous· solution of methanol in water has vapour
(d) relative lowering in vapour pressure of a solution
pressure:
90. Cryoscopy is concerned with:
(a) less than that of water (b) equal to that of water
(a) osmotic pressure ofa solution
(c) more than that of water (d) equal to that of methanol
(b) elevation of boiling point of a solul'~on
80. An aqueous solution is 1 molal in KI. Which change will cause
(c) depression in freezing point of a solution
the vapour pressure ofthe solution to increase? .
[CBSE (PMT) 20101 (d) relative lowering in vapour pressure of a solution
(a) Addition ofNaCl (b) Addition of Na2S04 91. Molecular mass of a non-volatile solute can be determined by:
(a) Cryoscopic method (b) Victor-Meyer's method
(c) Addition of 1 m~lal KI (d) Addition of water
(c) Graham's method (d) Duma's method
81. Osmotic pressure of a solution containing 0.1 mole of solute
92. Beckmann's thermometer measures:
per litre at 273 K is (in atm):
(a) boiling point of the solution
(a) O.l x 273 (b) O.l x 0.0821 x 273 (b) freezing point of the solution
0.0821 1
1 (d) O.l x 0.0821 (c) any temperature
(c) - x 0.0821 x 273 1 273 (d) elevation in boiling point or depression in freezing point
0.1
82. A solution of urea contains 8.6 g per litre. It is isotonic with a 93. The freezing point of 1% of lead nitrate solution in water will
5% solution of a non-volatile solute. The molecular mass of be: [UGET (ManipaJ) 2006)
the solute will be: (a) 2°C (b) 1°C (c) O°C (d) below O°C
(a) 349 (b) 34.9 (c) 3490 (d) 861 94. A solution of 1.25 g of non-electrolyte in 20 g of water freezes
83; The hard shell of an egg was dissvlved in HCL The egg was at 271.94 K. If K f is 1.86 Kkg mol- 1 then molecular mass of
then placed in a concentrated solution of NaC!. What will the solute will be:
happen? (a) 207.8 (b) 179.79 (c) 209.6 (d) 109.5
(a) The egg will shrink 95. The ratio of the value of colligative property for KCI solution
(b) The egg will swell to that of sugar solution at thc same concentration is nearly:
(c) The egg will become harder (a) 1 (b) 2 (c) 0.5 (d) 2.5
(d) There will be. hardly any visible change 96. The freezing point order of the solution of glucose is:
84. Blood has been found isotonic with: (a) 10%>3%>2%> 1% (b) 1%>2%>3%> 10%
(a) saturated solution ofNaCI (c) 1%>3%>10%>2% (d) 10%> 1%>3%>2%
(b) saturated solution ofKCI 97. Which one of the following solutions has the highest b.p.?
(c) saturated solution of 1 : I mixture ofN",CI and KCI [UGET (Manipal) 2006]
(d) normal sodium chloride solution (a) 0.1 M NaCl (b) 0.1 M urea
85. When a solute is dissolved in a solvent: (c) 0.1 M BaCI2 (d) 0.1 M glucose
(a) vapour pressure of the solvent is decreased 98. The freezing point of 1 molal NaCl solution assuming NaCl to
(b) freezing point of the solution becomes less than that of be 100% dissociated in water is:
solvent
370 G.R.B. PHYSICAL CHEMISTRY FOR COMPETITIONS

(a) 1. 86°C (b) 3.72°C (a) KCl (b) BaCI2 (c) K 2S04 (d) C 6HI2 0 6
(c) + 1.86°C (d) + 3.72°C 114. A solution containing 3.3 g of as:.:bstance,in 125 g of benzene
9"9. Which of the following 0.1 M aqueous solutions will have the (b.pt. 80°C) boils at 80.66°C. If K b for benzene is 3.28 K kg
lowest freezing point? . (MLNR 1990) mor l the molecular mass ofthe substance will be:
(a) K 2S04 (b) NaCI (c) Urea (d) Glucose (CP:MT 1992)
100. The freezing point of a 0.05 molal solution of a (a) 130.29 (b) 129.20 (c) 132.20 (d) 131.20
non-electrolyte in water is: (MLNR 1990) 115. The elevation in boiling point of a solution of 109 of a binary
(a) - 1.86°C (b) 0.93°C (c) 0.093°C (d) O.93°C elbctrolyte (molecular mass 100) in 100 g of water is ATb . The
101. The :freezing point of equimolal aqueous solution will be value of K b for water is:
highestfor: (lIT 1990) (a) A~b . (b) 10 (c) lOATb (d) ~~
(a) qH;;NH3Cl (b) Ca(N03)2
(c) La(N03 )3 (d) C 6HI2 0 6 (glucose) 116. The molal freezing point constant for water is 1.86 K kg mor I.
101. Elevation in boiling point was 0.52°C when (j g of a compound Therefore, the freezing point of 0.1 MNaCI solution in water is
X waS dissolved in 100 g Qf water. Molecular mass of the expected to be: (MLNR 1994)
compound X is: (K b for water is 0.52 per 1000 g of water.) (a) - 1.86°C (b) - 0.186°C(c) - 0.372°C (d) + 0.372°C
(a) 120 (b) 60 (c) 600 (d) .180 117. Equal volumes of M 120 urea solution and M 120 glucose
103. The molal freezing point constant for water is ·1.86. If 342 g of solution are . mixed. The mixture will have osmotic pressure:
cane sugar is dissolved in 1000 g of water, the solution will (a) equal to either of the solution
freeze at: . (b) less than either of the solution
(a) 1.86°C (b) - 1.86°C (c) 3.92°C (d) 3.92°C (c) higher than either of the solution
104. The molal elevation consta~t of water is 0.51 .. The boiling (d) zero
point of 0.1 molal aqueousNa.cI solutionis nearly:(IIT 1991) 118. Assuming the salts to be unionised in solution, which ofthe
(a) 100.05°C (b) 100.l°C (c) 100.2°e (d) 101.0°C following has highest osmotic pressure?
l~S. The molal elevation of an unknown solution is equal to the (a) I%CsCl (b) l%RbCl (c)l%KCl (d) l%NaCf
· molal elevation of 0.1 M solution of urea. The concentration of 119. Pure benzene freezes at 5.3°C. A solution of 0.223 g of phenyl
unknown solution is: acetic acid (C6Hs CH2 COOH)in 4.4 g of benzene (K f 5.12
(a) 1M (b)om M . (c) 0.1 M (d) none of these K kg morl) freezes at 4.47°C ..From the observation one can
l06. Benzoic acid dissolved in benZene will show a molecular mass conclude that: (AFMC 2010)
of: . , (a) phenyl acetic acid exists as such in benzene
(a) 122 (b) 61 (c) 244 (d) 366 (b) phenyl acetic acid undergoes partial ionization in benzene
107. If the observed andnonnal osmotic pressures of a KCI (c) phenyl acetic acid undergoes complete ionization in
solution are.5.85 and3.20atm, the degree of dissociation of benzene
KCI is: (d) phenyl acetic acid dimerizes in benzene
(a) I (b) 0.082 (c) 0.82· (d) '0.28 no. The movement of solvent molecules from higher
lOS. The van't Hoff factor ofa 0.005 M aqueous solution ofKCI is concentration to lower concentration through sernipenneable
1.95. The degree of ionisa~on of KCI is: membrane under pressure is tenned:
(a) 0.95 (b) 0,97 . (c) 0,94 (d) 0.96 (a) ;smosis (b) reverse osmosis
tb'~. If the observed and nonnal osmotic· pressures of 1% NaCI (c)' dialysis (d) diffusion
solution are 5.7 and 3.0 atm, the degree of dissociation of t21. During depression of freezing point experiment, an
· NaCI is: equilibrium is established between the molecules of:
(a) 0.9 (b) 1.0 (c) 0.57 (d) 0.3 (EAMCET 2009)
llO. Which one of the following salts would have the same value of (a) liquid solvent and solid solvent
the van't Hoff factor as that of K3Fe(CN)6? (CSSE 1994) (b) liquid solute and solid solvent
(a) AI 2(S04h (b) NaCI (c) liquid solute and solid solute
· (c) Na2S04 (d) Al(N03 )3 (d) liquid solvent and solid solute
HI·. Equimolal solutions of A and B show depression in freezing In. The molecular weight of benzoic acid in benzene as
point in the ratio of2 : I. A remains in nonnal state in solution. determined by depression in freezing point method
B will be in ... state in solution. corresponds to: (liT 1996)
"
(a) nonnal (b) associated (a) ionisation of benzoic acid
(c) hydrolysed (d) dissociated (b) dimerization of benzoic acid
1.'':2. The values of observed and calculated molecular· mass of (c) trimerization of benzoic acid
Ca(N03 )2 are 65.4 and 164 respectively. The degree of (d) solvation of benzoic acid
ionisation of the salt will be: 12"3. If 0.15 g of a solute, dissolved in 15 g of solvent, increases the
(a) 0.25 (b) 0.50 (c) 0.60 (d) 0.75 boiling point by U216°C over that of the pure solvent, the
Jftl~. Which of the following compounds corresponds to van't Hoff molecular mass of the substance is (Molal elevation constant
factor to be equal to 2 in dilute solution? . . for the solvent is 2.16): (CBSE 1999)
SOLUTIONS 371

(a) 1.01 (b) 10 100% polymerisation, the number of molecules of benzoic


(c) 10.1 (d) 100 acid in associated state is:
124. The vapour pressure of benzene at a certain temperature is 640 (a) 1 (b) 2
rom ofHg. Anon-volatile and nori-electrolytic solid, weighing (c) 3 (d) 4
2.175 g, is added to 39.08 g of benzene. The vapour pressure 131. For [CrC1 3 . xNH3]' elevation in boiling point of one molal
of the solution is 600 rom of Hg. What is the molecular mass solution is double of one molal solution of glucose; hence x is,
of the solid substance? (CBSE 1999) . if complex is 100% ionised:
(a) 49.50 (b) 59.6 (a) 4 (b) 5
(c) 69.5 (d) 79.8 (c) 6 . (d) 3
125. A 5% solution of cane sugar (mol. mass = 342) is isotonic with 132. If glycerol and methanol were sold at the same price in the
1% solution of a substance X. The molecular mass of X is: market, which would be cheaper for preparing an antifreeze
(CBSE 1998) solution for the radiator. of an automobile?
(a) 34.2 (b) 171.2 (a) Glycerol (b) Methanol
(c) 68.4 (d) 136.8 .i,-c) Both are equal (d) None of these
126. The vapour pressure of a solvent decreased by 10 rom of 133. Two liquids A and B have p}: pBO = 1: 3 at a certain
mercury when a noncvohitile solute was added to the solvent.
temperature. If the mole fraction ratio of XA : xB = 1: 3, the
The mole fraction of the solute in the solution is 0.2. What
mole fraction of A in vapour in equilibriutn with the solution
should be the mole fraction of the solvent, if the decrease in
at a given temperature is:
the vapour pressure is to be 20 rom of mercury? (CBSE 1998)
(a) 0.1 (b) 0.2
(a) 0.8 (b) 0.6
(c) 0.5 (d) 1.0
(c) 0.4 (d) 0.2
134. Relationship between osmotic pressure at 273 K when 1%
. lowering in vapour pressure
[Hint: Mole fractIOn of solute = ---=-~-~-­ glucose (1t I ), 1% urea (1t 2 ), 81% sucrose (1t3) are dissolved
vapour pressure of solvent in 1 litre of water:
Comparing under the two conditions, (a) 1t1 > 1t2 > 1t3 (b) 1t2 > 1t1 > 1t3
0.2 10 (c) 1t3 > 1t1 > 1t2 (d) 1t1 = 1t2 = 1t3
\
mole fraction of solute 20 135. Which of the following property indicates weak
or mole fraction of solute = 0.4 intermolecular forces of attraction in liquid?
mole fraction of solvent = (1 - 0.4) J,0.6] (a) High heat of vaporization (b) High vapour pressure
127. Which of the following 0.10 m aqueous sol~\tion will have the (c) High critical temperature (d) High boiling.point
lowest freezing point? -\ (CBSE 1997) 136. One mole of sugar is dissolved in two moles of water. The
(ao) A1 2 (S04h (b) KI vapour pressure of the solution relative to that of pure water is:
" (c) C 6H12 0 6 (d) C 12H 22 0 11 \ (a) 2/3 (b) 113
128. A binary liquid solution is prepared by mixing n-heptane and (c) 3/2 (d) 1/2
ethanol. Which one of the following statements is correct 137. The highest temperature at which vapour pressure of a liquid
regarding the behaviour of the solution? (AIEEE 2009) can be measured is:
(a) The solution formed is an ideal solution (a) critical temperature (b) inversion temperature
(b) The solution is non-ideal, showing positive deviation from (c) critical solu. temperature (d) b.pt. of liquid ,
Raoult's law 138. The expression relating molality (m) and mole fraction (Xi ) of
(c) The solution is non-ideal, showing negative deviation solute in a solution is:
from Raoult's law
mMI
(d) n-heptane shows positive deviation, while ethanol shows (a) x2 = ----'--
negative deviation from Raoult's law l+mMI
129. X is a non-volatile solute and Y is volatile solvent. The
following vapour pressures are obtained by dissolving X in Y. (c) x
2
= 1+ mMI (d) x = 1- mMI
2
Xlmol L- 1 Y/rom i-Ig mMI mMI
0.1 ~ 139. The diagram given below is a vapour pressure composition
0.25 P2 diagram for a binary solution of A and B.
0.01 P3

The correct order of vapour pressure is :


[EAMCET (Engg.) 20101
(a)l-i<P2 <P3 (b) P3 <P2 <PI
(c) P3 < ~ < P2 (d) P2 < ~ < P3
130. 12.2 g of benzoic acid (M = 122) in 100 g water has elevation
in boiling point of 0.27. Kb = 0.54 K kg mol-I. If there is
xs -
372 G.R.B. 'PHYSICAL CHEMISTRY FOR COMPETITIONS

In the solution, A - B interactions are: 149. The molal b.pt. constant forwatefls LI.513°Ckg mol-'. When
(a) similar to A - A and B - B interactions 0.1 mole of sugar is dissolved in 200 g of water, the solution
(b) greater than A - A and B - B interactions boils under a pressure of 1 atm at: (AlIMS 1991)
(c) smaller than A - A and B - B interactions (a) 100.513°C (b) 100.0513°C
(d) unpredictable (c) 100.256°C (d) 1Ol.025°C
140. The best and accurate method for determining 'osmotic 150. The correct expression relating molality (m), molarity (M),
pressure is: (AFMC 1995) density (d ) and molar mass (M 2) of a solute is:
(a) negative pressure method
(b) Berkeley and Hartely method (a)m- M (b)m- M
d +MM2 ' d -MM2
(c) Morse and Frazer method
, (d) Preffer's method (c) m = d + MM 2 (d) m = d - MM 2
141. If all the following four compounds were sold at the same
M M
price, which would be cheapest for preparing an antifreeze 151. The expression relating mole fraction of solute (x2 ) and
solution for a car radiator? (AMU 1997) molarity (M ) of the solution is:
(a) CH30H (b) C 2 H50H MM MM
(a) x 2 = , (b) X 2 = '
(c) C 2H4 (OH}z (d) C 3 H5(OHh M(M, -M 2 )+p . M(M, -M2)-P
142. What is the osmotic pressure of the solution obtained by _M(M,-M2)+P (d) _M(M,-M2 )-p
()
cx 2 - x2 - ,
mixing 300 cm3 of 2% (mass-volume) solution of urea with MM, MM,
300 cm3 of3.42% solution of sucrose of20°C? (where P is the density of solution and M, and M2 are the
(R = 0.082 L atm K-' mol-I) (SCRA 2009) molar masses of solvent and solute, respectively.)
(a) 5 atm (b) 5.2 atm (c) 2.6 atm (d) 4.5 atm 152. The boiling point of a solution of 0.11 g of a substance in 15 g
of ether was found to be O.I°C higher than that of pure ether.
[Hint: nV = [( : : tea + (:: tcroJRT The molecular weight of the substance will be (Kb = 2.16):
[PET (MP) 2002]
n x 600 = [~+ 3 3.42J 0.082 x 293
x (a) 148 (b) 158 (c) 168 (d) 178
1000 60 342 153. 2.5 litre of 1 MNaOH solution are mixed with another 3 litre
n x 0.6 = (0.1 + 0.03) x 0.082 x 293 of 0.5 M NaOHsolution. Then the molarity of the resulting
n = 5.2atm] solution is: [CBSE (PMT) 2002]
143. A solution of 1 molal concentration of a solute will have (a) 0~80M (b) l.OM (c) 0.73M (d) 0.50M
lllaximum boiling point elevation when the solvent is: 154. In a mixture, 'A' and 'B' components show the negative
[PMT (MP) 2000) deviations as: (AlEP2002)
(a) ethyl alcohol (b) acetone (a) ~Vmix > 0
(c) benzene . (d) chloroform (b) ~Vmix < 0
144. :Isotonic solutions have: (DPMT 2000) (c) A - B interaction is weaker than A - A and B - B
(a) same boiling point (b) same vapour pressure interactions
(c) same melting point (d) same osmotic pressure (d) none of the above reasons is correct
145. The depressions in freezing point for 1 M urea, 1 M glucose 155. A soluti,?n contains non-volatile solute of molecular mass M 2.
and 1 MNaCI are in the ratio: [CET (Haryana) 2000) Which of the following can be used to calculate the molecular
(a) 1 : 2 : 3 (b) 3 : 2 : 2 mass of solute in terms of osmotic pressure?
(c) 1: 1 : 2 (d) none of these [CBSE (PMT) 2002]
146.' An aqueous solution of 6.3 g oxalic acid dihydrate is made up
to 250 rnL. The volume of 0.1 NNaOH required to completely
(a) M2 = rnz VRT
1t
(b) M2 = (i JR:
,neutralize 10 rnL of this solution is: (lIT 2001)
(a) 40 rnL (b) 20 mL (c) 10 rnL (d) 4 rnL (c) M=(i JnRT
2
,(d) M2 = (rnzJ
- -1t
V RT
147. The vapour pressure of a solvent A is 0.80 atm. When a
non-volatile substance B is added to this solvent, its vapour (rnz = mass of solute; V = volume of solution; n = osmotic
pressure drops to 0.6 atm. The mole fraction of B in the pressure)
solution is: iPMT (MP) 2000) 156. 0.6 g of an organic compound when dissolved in 21.7 g water
(a) 0;25 (b) 0.50 (c) 0.75 (d) 0.90 freezes at 272.187 K. The molar mass of the organic
148. An aqueous solution of sucrosy, C12 H 22 011 , containing compound is close to:
(K f of water is 1.86 deglmolality; freezing point is 273 K)
34.2 gIL has an osmotic pressure of 2.38 atmospheres at 17° C.
For an aqueous solution of glucose, C 6 H12 0 6 , to be isotonic (SCRA2009)
with this solution, it would have: (AMU 1997) (a) 61 g mol-' (b) 63 g mol-'
(a) 34.2gJL (b) 17.1g/L (c) 65 g mol-' (d) 67 g mol-'
(c) 18.0 gIL J,d) 36~0 gIL of glucose
SOLUTIONS

157. Osmotic pressure of urea solution at 10°C is 500 mrri. Osmotic (d) The relative lowering of vapour pressure is directly
. pressure ofthe solution becomes 105.3 mm, when it is diluted proportional to the original pressure
and temperature raised to 25°C. The extent dilution is: 165. Calculate the molal depression constant of a solvent which has
[PMT (MP) 2004) freezing point 16.6°C and latent heat of fusion 180.75 J g -I.
(a) 8 times (b) 5 times (c) 4 times (d) 7 times [JEE(Ori!\1Sa)2005)
158. The average osmotic pressure of human blood is 7.8 bar at Ca) 2.68" (b) 3.86 (c) 4.68 (d) 2.86
37°C. What is the concentration of an aqueous NaCl solution 166. If for a sucrose solution, elevation in boiling point is O.loC
that could be used in blood stream? (AIIMS 2004) then what will be the boiling point of NaCI solution for same
(a) 0.15 mollL (b) 0.30 mollL molal concentration? (B~U (Pre.) 2005)
(c) 0.45 mollL (d) 0.60 mollL· (a) O.loC (b) 0.2°C (c) 0.08°C (d) O.OloC
159. Camphor is often used in molecular mass determination 167. Which has minimum osmotic pressure? . (DeE 2005)
because: ICBSE-PMT (Pre.) 2004) (a) 200 mL of2 MNaCI solution
(a) it is readily available (b) 200 mL of 1 M glucose solution
(b) it has very high cryoscopic constant (c) 200 mL of2 Murea solution
(c) it is volatile (d) All have same
(d) it is a solvent for organic substances 168. Solution A contains 7 gIL of MgCl 2 and solution B contains
160. A 0.004 M solution of Na2S04 is isotonic with a 0.010 M 7 gIL of NaCl. At room temperature, the osmotic pressure of:
solution of glucose at same temperature. The apparent degree (DeE 2005)
of dissociation ofNa2S04 is: [lIT (S) 2004) (a) solution A is greater than B
(a) 25% (b) 50% (c) 75% (d) 85% (b) both nave same osmotic pressure
161. Which ofthe following liquid pairs shows a positive deviation (c) solution B is greater than A
from Raoult's law? IAIEEE 2004; AMU (Med) 2007) (d) can't determine
(a) Water-hydrochloric acid (b) Benzene-methanol 169. The van't Hoff factor for BaCl2 at 0.01 M concentration is
(c) Water-nitric acid (d) Acetone-chloroform 1.98. The percentage dissociation of BaCl 2 at this
162. Which one of the following statements is false? concentration is: [rET (Kerala) 2005)
(AIEEE 2004) (a) 49 (b) 69 (c) 89 (d) 98
(~) Raoult's law states that vapour pressure of a component (e) 100
over a solution is proportional to mole fraction 170. Equimolar solutions in the same solvent have: (AlfEE 2005)
(b) The osmotic pressure (1t) of a solution is given by the (a) same boiling point but different freezing points
relation 1t = MRT where M is molarity of the solution (b) same freezing point but different boiling points
(c) The correct order of osmotic pressure for 0.01 M aqueous (c) same freezing and boiling points
solution of each component is: (d) different freezing and boiling points
BaCl2 > KCl > CH3COOH > Sucrose 171. A solution of urea (mol. mass 56 g mol-I) boils at 100.18°C at
(d) Two sucrose solutions of same molality prepared in the atmospheric pressure. If Kf and Kb for water are 1.86 and
different solvents will have the same freezing point 0.512 K kg mol- I respectively, the above solution will freeze
depression at: [CBS¥-fMT (Pre.) 2005]
163. A solution of sucrose (molar mass = 342 g mol-I) has been (a) 0.654°C (b) - 0.654°C -
prepared by dissolving 68.5 g of sucrose in 1000 g of water. (c) 6.54°C (d) - 6.54°C
The freezing point of the solution obtained will be: 172. If 0.5 g ofa solute (molar mass 100 g mol-I) in 25 g of solvent
(K f for water = 186 K kg mol-I) [CBSE (PMT) 2010) elevates the boiling point by 1 K, the molar boiling point
(a) -0.372°C (b) -D.520°C constant of the solvent is: IfMT (K~rala) 2006]
(a) 2 (b) 8 (c) 5 (d) 0.5
(c) +0.372°C (d) -D.570°C
(e) 10
(Hint: I1T =K f X wB X 1000 173. Osmotic pressure observed when benzoic acid is dissolved in
mBxwA
benzene, is less than that expected from theoretical
= 1.86 x 68.5 x 1000 considerations. Thisis because: [KCET 2006)
342 x 1000 (a) benzoic acid is an organic solute
=0.372 (b) benzoic acid has higher molar mass than benzene
Freezing point of solution = 0 - 0.372°C (c) benzoic acid gets associated in benzene
= -D.372°C] (d) benzoic acid gets dissociated in benzene
164. Which of the following is incorrect? [CET (J&K) 2005) 174. A 5% solution by mass of cane sugar in water has freezing
( a) Relative lowering of vapour pressure is independent of the point of271 K and freezing point of pure water is 273.15 K.
nature of the solute and the solvent The freezing point of a 5% solution (by mass) of glucose in
(b) The vapour pressure is a colligative property water is: (AIIMS 2006)
(c) Vapour pressure of a solution is lower than that of the (a) 271 K (b) 273.15 K (c) 269.07 K (d) 277.23 K
solvent
/
374 G.R.B. PHYSICAL CHEMISTRY FOR COMPETITIONS

[Hint: AT = K x WB xlOOO contains 20 mole per cent of A in the (A + B) binary mixture


f m B xWA in the liquid phase? (SCRA 2007)
2.15 = K x 5 x IOQO ' (a) 0.2 (b) 0.3
... (i) (for sucrose)
f 342 x 95' (c) 0.5 (d) 0.7
20
'" AT'~'k x 5 x 1000 ... (ii) (for glucose)
[Hint: XA = 100 = 0.2, ..
f 180 x 95
Dividing eq. (i) by eq. (ii), we get, P =PA + PB := P~A + PZXB
:= 60 x 0.2 + 15 x 0.8 := 12 + 12 =24'
AT :::4.18 K
i";'To - 4.18 Mole fraction of A in the vapour phase, i.e." YA may be calculated
as:
= 273.15 - 4.18:= 269.Q7 K]
PA YA XP
175. 18 gglucose (C 6 HIZ 0 6 ) is added to 178.2 g of water. The
12 =YA x 24
vapour pressure of water for this aqueous solution at lOOoe is: 12
YA:=~ =0.5]
-* ' (AIEEE 2006) 24
(a) 759 torr (b) 7;60 torr (c) 76 torr (d) 752.4torr 181., When 20 g ofnaphthoic acid(C lI H s0 2 ) isdiss~lved in 50 g of '
176. Depression in freezing paint is 6 K for NaCl solution. If K f
benzene (Kf = 1.72 kg inor1),a freezing point depression of
for water is 1.86 Klkgmol;amount of NaC! dissolved in I kg
water~s: ' (DCE 2006) 2K is observed. The van't Hofffactor (i) is: (lIT 2007)
(a) 3.42 (b) 1.62 (c) 3.24 ' (d) 1.71
(a) 0.5 (b) I,
(c) 2 (d) 3
177. Observe the following obs,ervations:
x 1000
n obs obServed colligative property [Hint: AT= i x Kf x
neal := theoretical colligative property assuming normal mB xWA

, behavioUr of solute 2 = ix 1.72 x 20 x 1000


van't Hoff factor (I) is given by: (CET (J&K) 2006l 172 x 50
(a)i =nabs x 1teal ' (b) i = nobs + neal i =0.5]
(c) i = nobs - neal (d) i:= nob;' 182. At 80°C, the vapour pressure of pure liquid A is 520 mm of Hg
, n~ and that of pure liquid B is 1000 mm of Hg. If a mixture of
178. A 525% solution of a substance is isotonic with a 1.5% solution A and B boils at 80?C and 1 atm pressure, the amount '
solution of urea (molar mass =
60 g mor l ) in the same of A in the mixture is: (I atm = 760 mm of Hg) (AIEEE 2008)
(a) 50 mol per cent (b) 52 mol per cent '
solvent. If the densities of both the solutions are assumed to be
(c) 34 mol per cent (d) 48 mol per cent
equal to 1 g cm-3, molar mass ofthe substance will be: 0 0 '
[Hint: p:= PA ~A + PB XB
(AIEEE 2007)
760 = 520 x x A + 1000 (1 xA)
(a) 210.0 g mor l (b) 90.0 g mor l
x A := 0.50, i.e., 50mol per cent.]
(c) 115.0 g mor l , (d) 105.0 g mor l
[Hint:, For isotonic solutions:
,183. ,The vapour pressure of water at 200C is 17.5 mm Hg. If 18 g
of glucose (C6H 1Z 0 6 lis added to 178.2 g of water at20°C;the
" ' i l
~nz ':11: =CRT
vapour pressure of resulting solution will be: (AlEEE 2008)
C 1 =C2
(a) 17.325 mmHg (b) 17.675 mmHg "
1.5/60 5.25/m
--:=-- (c)15.75 mmHg (d) 16.5 mmHg
V V
m = 210]
[Hint: P := lXA
"-- .'
= 17.5 x 178.2/ 18
179. Concentrated aqueous sulphuric acid 98% H2 S04 by mass has
a density of 1.80 g mL- l . Volume of acid required to make 178.2I1g+ 18
180
one litre ofO.lM H2S04 solution is': (CRSE (Med) 2007}
17.325 min Hg]'
(a) 16~65mL (b) 22.20mL
184. Which of the following can be measured by the Ostwald-
(c) 5.55 mL (d) 11.l0mL Walker dynamic, meplOd.? " ' (KCET 2008)
M x x d x 10 = 98 x 1.80 x 10 = 18 , (a) Vapour pressUre of the solvent
[Hint:
mB 98 , (b) Relative lowering of vapour pressure
M1V1 =,M2V2 (c) Lowering of vapoUr pressure
18 X VI == 0.1 x 1000 (d) All of the above
100 185. Cryoscopic constant is the depression in freezing, point
VI =- =5.55 mLJ produced by : ICorned (Kamataka)2008}
18 '
180. Iftheyapour pressures of pure A and pure B at 298 K are 60 (a) l%solution (b) I molar solution
,and 15 torr respectively, what would be the mole fraction of (c) 1 molal solution (d) I N solution
A in vapour phase (at this temperature) in a solution that
186. Which among the following gas will greatly deviate from (Assume density of solution to be I glee)
Henry's law in water? [PMT (Kerala) 2008] [Hint: 1t x13.6xgx76 90xgxl
(a) H2 (b) N2 1t = 0.087 atm
(c) CH4 (d) CO2 1t = CRT
(e) Ar 0.087 == C x 0.082 x 298
187. Vapour pressure increases with increase in : C 3.56 x 10-3 M
[JEE (Orissa) 20081 llT =Kfxm
(a) concentration of solution containing non-volatile solute = 30 x 3.56 X 10-3 ::: 0.1
(b) temperature upto boiling point :. Freezing point of solution will be: 250.3 0.1
(c) temperature upto triple point =250.2 K]
(d) altitude of the concerned place of boiling
188. When mercuric iodide is added to the aqueous solution of Set-2: The questions given below may have more than
potassium iodide: [BHU (Mains) 2008] one correct answers
(a) freezing point is raised
(b) freezing point is lowered 1. The given graph indicates:
(c) freezing point does not change
(d) boiling point is raised
[Hint: 2KI + HgI2 ~ K2[HgI4]
Number of particles decreases due to above reaction hence
freezing point is raised.]
189. The Henry's law constant for the solubility ofN2 gas in water
at 298 K is 1X 105 atm. The mole fraction of N2 in air is 0.8.
The number of moles of N2 frOIn dissolved in 10 moles of
water at 298 K and 5 atm pressure is: (lIT 2009)
(a) 4 x 10-4 (b) 4 x 5
10- XA= 1
(c) 5xlO-4 (d) 4xlO-6 Xs=O
[Hi~t: ~2 =xN2 XPtotal
(a) (+) deviation (b) (-) deviation
0.8 x 5 = 4 atm (c) no deviation (d) none of these
According to Henry's law: 2. Which is the con-ect statement?
PN2 =KH XXN2 (a) Minimum boiling azeotropic mixture boils at temperature
4 Ixl05 x nN2 lower than either of the two pure components
nN2 xnH20 (b) Maximum boiling azeotropic mixture boils at temperature
5 higher than either ofthe two pure components
nN 2 + lIH2o= I X 10
(c) Minimum boiting azeotropic mixture shows (+)ve deviation
~2 4 (d) Maximum boiling azeotropic mixture shows (-)ve deviation
+ 10 3. A certain non-volatile substance (non-electrolyte) contains
--=-- = 0.25 X 105
nN2 40% C, 6.7% H, 53.3% O. An aqueous solution containing 5%
by mass of the solute boils at I OOJ5° C. Molecular formula of
~2 =4xlO-4 ] . the compound is: .
190. A· 0.002 m. aqueous solution of an ionic compound (a) CH2 0 (b) C2~02
Co(NH3 >s(N02 )et freezes at -O.00732°C. Number of moles <,(c) C6~206 (d) C12H22 0 11
of ions which I mol of ionic compound produces on being 4; Difference between diffusion and osmosis is:
dissolved in water will be : (K f = - L 86"C/m) (a) a semipermeable membrane is required for osmosis while
[eBSE (Med) 20091 diffusion requires no semipermeable membrane
(a) 1 (b) 2 (c) 3 ... (d) 4 (b) in osmosis the movement of molecules is only in one
[Hint: llT == iXKfxm . direction whereas in diffusion movement is on both sides
(c) in osmosis only the solvent moves while in diffusion
0.00732 i x l. 86 x 0.002
solute and solvent both move
i::::; 1.967", 2] (d) rione of the above
191. A solution is separated from pure solvent by a semipermeable . 5. For anon-volatile solute:
membrane at 298 k. The difference in the height of the (a) vapour pressure of solute is zero
solution and the solvent is 0~9 m. If K f and freezing point of (b) vapour pressure of solution = vapour.pressure of pure
the solvent are 30 K kg morl and 250.3 K, respectively, the solvent
temperature at which the solution freezes is: (lSAT 2010) (c) vapour pressure of solution = vapour prfssure of solvent
(a) 250.10 K (b) 250.25K in solution
~2~WK 002~iliK (d) all of the above
376 G.R.B. PHYSICAL CHEMISTFlY FOR COMPETITIONS

6. In which of the following cases does osmosis take place if the 11. Which is a dimensionless quantity?
solutions are separated by a semipermeable membrane? (a) Mole fraction (b) Molality
(a) 0.1 MNaCI and 0.2 M glucose (c) % by wt. ofsolvent (d) % by wt. of so lution
(b) 0.1 M sucrose and 0.1 M fructose 12. Which ofthe following concentration terms is/are independent
(c) 0.05MK4[Fe(CN)6]andO.1MCa~lz of temperature? ' [PET (Kerala) 2006;
(d) 10-3 MCaCl z and 1.5 x 1O-3 MNaCI CET (Gujarat) 2006; DCE 2007]
(a) Molarity
7. Match the following:
(b) Molarity and mole fraction
Solute (equimolar) 11: (OP) ratio
(c) Mole fraction and molality
(i) Urea, glucose, fructose (A) 1: 0.8: I (d) Molality and normality
(ii) NaCI, MgClz, K 2S04 (B) I : 2: 3 (e) 'Only molality ,
(iii) AliS04h, Na3P04,K4Fe(CN6) (C) 1 : 1 : 1 13. In the depression of freezing point experiment, it is found that:
(lIT 1999)
(iv) Glucose, NaCI, CaCh (D) 2: 3: 3 (a) the vapour pressure of the solution is less than that of pure
(i) (ii) (iii) (iv) solvent,
(a) A B C D' (b) the vapour pressure of solution is more than that of pure
solvent
(b) B D C A (c) only solute molecules solidifY at the freezing point
(d) only solvent molecules solidifY at the freezing point
(c) D B A C 14. Which of the following is/are correct for a solution of a
particular concentration?
(d) C ,D A B (a) Molarity is always less than molality
(b) Formality is equivalent to molarity
(c) Mole fraction is equivalent to mass fraction
8. Match the following graph:
(d) Normality of H 2 S04 solution is double than its molarity
A 15. Effect of adding a non-volatile solute to a solvent is:
(a) to lower the vapour pressure
(b) to increase its freezing point

(c) to increase its boiling point
I (d) to decrease its osmotic pressure
16. Whichofthe following formls an ideal solution?
BP
(a) Ethyl bromide + ethyl iodide
(b) Ethyl alcohol + water
(c) Chloroform + benzene
Mole fraction (d) Benzene + toluene
17. Osmotic pressure of a solution is:
A (i)( +) deviation (a) directly proportional to the molar concentration of the
B (ii) Ideal solution
C (iii) (-) deviation (b) inversely proportional to the molecular weight of the
solute
(a) (A) (i), (B) (ii), (C) (iii)
(c) inversely proportional to the temperature
(b) (A) (iii), (B) (ii), (C) (i)
(d) directly proportional to the volume of the solution
(c) (AHii), (B) (iii), (C) (i)
18.' Which ofthe following is/are true?
(d) none of these
(a) For the same solution, elevation in boiling
9. Vapour pressure of methyl alcohol and ethyl alcohol solutions
point = depression in freezing point
is represented by P = 1l5xA + 140 where x A is the mole
pO (b) van't Hoff factor for a dilute solution of BaCl2 is 3
fraction of methyl alcohol. The value of lim -.!L is: (c) Elevation in boiling point is due to increase in vapour
XA .... O xB pressure
(a) 255 (b) 115 (d) Depression in freezing point is due to decrease in vapour
(c) 140 (d) 135 pressure
10. To 10 mL of I M BaC12 solution 5 mL of 0.5 M K 2 S04 1: 19. In the following question, more than one of the answers given
added, BaS04 is precipitated out. What will happen? may be correct. Select correct answer and mark it according to
(a) F.pt. is increased (b) B.pt. is increased the code: [BHU (Mains) 2008]
(c) F.pt. is lowered (d) B.pt. is lowered A solution containing components A and B exhibits positive
deviation from Raoult's law only when

to.
SOLUTIONS 3r7-··

(1) AVmixing = + ve (a) 3 and 4 (b) I and 4


(2) Mf mixing = - ve (c) 1 and 2 (d) 2 and 3
(3) A B attraction forces < A A and B B attraction 21. Consider the following :
forces At constant pressure, boiling point of a solution is greater than
(4) A - B attraction forces> A - A and B - B attraction the boiling point of its pure liquid solvent because
forces I. Solute is non~electrolyte
Codes: 2. Solute is involatile
(a) 1, 2 and 3 are correct. 3. Chemical potential of solvent in solution is less than the
(b) 1 and 2 are correct chemical potential of solvent in its pure state at constant
(c) 2 and 4 are (!orrect pressure
(d) I and 3 are correct Which of the above are correct? (SCRA 2009)
20. Consider the following statements in respect of an ideal (a) 1,2 and 3 (b) I and 2 only
solution: (c) I and 3 only (d) 2 and 3 only
1. Raoult's law is valid for an ideal solution over the whole 22. Which values can be obtained from the information
concentration range represented by the vapour pressure curve ofa liquid?
2. Enthalpy of mixing is zero, i.e., Mf mix "" 0 1. Normal' boiling point
3. Volume of mixing is not zero, i. e., A Vmix ::1= 0 2. Normal freezing point
4. The components of ideal.solution cannot be separated by 3. Enthalpy of vaporisation (DUMET 2010)
fractional dtstillation. (a) i only (b) 1 and 2 only
Which of the statements given above is/are correct? (e) 1 and 3 only (d) 1,2 and 3
. (SCRA 2009)
378 G.R.B. PHYSICAL CHEMISTRY FOR COMPETITIONS

Assertion-Reason TYPE QlTESTIONS


The questiOll.sgiven below consist ofM\fibt~~ents each printed as 10. (A) 0.1 M solution of glucose has same increment in freezing
'Assertion' (A) and 'Reason' (R). While answering these questions point than 0.1 M solution of urea.
you are required to choose anyone of the following four options: (R) K f for both has different values. (AlIMS 1997)
(a) If both (A) and (R) are correct and (R) is the correct 11. (A) Molarity of 0.02 N solution ofHN0 3 is 0.02 M.
explanation for (A) .. (R) Molarity and normality of a solution are never equal.
(b) If both (A) and (R) are correct but (R) is not the correct 12. (A) Larger the value of cryoscopic constant of the solvent,
explanation for (A). lesser will be the freezing point ofthe solution.
(c) If (A) is correct but (R) is incorrect. (R) Depression in the freezing point depends on the nature of
(d) If (A) is incorrect but (R) is correct. the solvent.
13. (A) 0.1 M solution of NaCl has greater osmotic pressure than
.1. ( A) The dissohition of gases in water IS always an endothermic 0.1 M solution of glucose at same temperature.
process.
(R) In solution, NaCI dissociates to produce more number of
(R) Dissolution of gases in water proceed with negative value particles. .
of l!.S.
14. (A) Henry's law and Raoult's law are not independent, i.e., one
2. (A) Water boiling at 10000C at 1 atmospheric pressure in a can be derived from the other.
.beaker is not at eqUilibrium.
(R) The partial pressure is directly proportional to the mole
(R) It refers to an open system. fraction of the concerned species for ideal solutions.
3. (A) A solution which contains one gram equivalent of solute 15. (A) The water pouch of instant cold pack for treating athletic
per litre of the solution is ~alled a normal solution. injuries breaks when squeezed and NH4 N03 dissolves
(R) A normal solution means a solution in which the solute thus lowering the temperature.
does not associate or dissociate. (R) Addition. of non-volatile solute into solvent results into
4. (A) The sum of mole fractions of all the components of a depression of freezing point of solvent. (AllMS 2006)
solution i!l unity: 16. (A) In a pressure cooker, the water is brought t6 boil. The
(R) Mole fraction is a temperature dependent quantity. cooker is then removed from the stove. Now on removing
5. (A) Iodine is more soluble in CCl 4 than in water; the lid of pressure cooker, the water starts boiling again.
(R) Non"polar solutes are more soluble in non-polar solvents. (R) The impurities in water bring down its boiling point.
6. (A) Vapour pressUre of O.5M sugar solution is more than [Hint: In pressure cooker, water boils above 100" C. When the
0.5 M KCI solution. lid of cooker is opened, pressure is lowered so that boiling point
(R) Lowering of vapour pressure is directly proportional to the decreases and water boils again.]
number of species present in the solution. 17. (A) An. increase in surface area increases the rate of
7. (A) Non-ideal solutions·form azeotropic mixture. evaporation.
(R) Boiling point of azeotropic mixture is higher than boiling (R) Stronger the inter-molecular attractive forces, faster is the
points of its both components. rate of evaporation at a given temperature.
8. (A) One molar solution is always more concentrated than one 18. (A).An ideal solution obeys Raoult's law.
molal solution. (R) In· an ideal solution, solute-solute as well as
(R) The amount of solvent in 1 M and 1 m aqueous solution is solvent-solvent
.
interactions
. are similar to solute-solvent
not equal. interaction.
9; (A) Camphor is used as solvent in· the determination of
molecular mass of naphthalene and anthracene etc.
(R) Camphor has high molal elevation constant.
SOLUTIONS 379--

['~~~:·'-·'·:?li~6£ii!?r~~:-,"· c~~m~~~,' I
• Set-1
1.(a) 2.(c) 3. (b) (d)4. (b)
5. 6. (d) 7. (c) 8. (b)
9. (c) 10. (d) 11. (a) 12. (a) 13. (d) 14. (c) 15. (a) 16. (d)
n (a) 18. (b) 19. (c) 20. (b) 21. (d) 22. (c) 23. (a) 24. (b)
25. (c) 26. (d) 27: (b) 28. (a) 29. (c) 30. (b) 31. (d) 32. (d)
33. (b) 34. '(a) 35. (d) 36. (c) 37. (b) 38. (c) 39. (b) 40. (d)
41. (a) 42. (a) 43. (b) 44. (c) 45. (d) 46. (b) 47. (b) 48. (d)
49. (c) 50. (c) 51. (b) 52. (a) 53. (b) 54. (d) 55. (d) 56. (a)
57. (c) 58. (b) 59. (b) 60. (c) 61. (a) 62. (d) 63. (c) 64. (d)
65. (a) 66. (d) 67. (a) 68. (c) 69. (b) 70. (c) 71. (a) 72. (b)
73. (d) 74. (b) 75. (a) 76. (c) 77.(d) ,78. (d) 79. (a) 80. (d)
81. (b) 82. (a) 83. (a) 84. (d) 85.(d) 86. (b) 87. (c) 88. (b)
89. (b) 90. (c) 91. (il) 92. (d) 93. (d) 94. (d) 95. (b) 96. (b)
"

97. (c) 98. (b) 99. (a) 100. (c) 101. (d) 102. (b) 103. (b) 104. (b)
105. (c) 106. (c) 107. (c) 108. (a) 109.- (a) 110~ (d) 111. (b) i12. (d)
113. (a) 114. (d) 115. (a) 116. (c) 117. (a) 118. (d) 119. (d) 120. (b)
121. (a) 122. (b) 123. (d) 124~ (c) 125. (cr' 126. (b) 127~ (a) 128. (b)
'. 129. (d)
(d)
130. (b)
138. (c)
131. (a)
(c)
132. (b)
140. (b)
133. (a)
(a)
134. (b)
142. (b)
135. (b)
143. (c)
136. (a)
(d)
'., 137.
145. (c) 146. (a)
139.
147. (a) 148. (c)
141.
149~ (c) 150. (b) 151. (a)
144.
152. (b)
153. (c) 154. (b) 155~ (b) 156. (b) 157. (b) 158. (b) 159. (b) 160. (c)
161. (b) 162. (d) 163. (a) 164. (d) 165. (b) 166. (b) 167. (b) 168. (a)
169. (a) 170. ,(d) 171. (b) 172. (c) 173. (c) 174. (c) 175. (d) 176. (b)
177. (d) 178. (a) 179. (c),
. 180. (c) 181. (a) 182. (a) 183. (a) 184. (b)
185. (c) 186. (d) 187. (b) 188. (a) 189. (a) 190. (b) 191. (c)

• Set-2'-, (a,b, c) (d) , '


1. (a) (a; b, c, d)
2. (c)
3. 4. (a, c)
5. (c)
6. 7. 8. (b)
9. (c) 10. (b, c) 11. (a, c, d) 12. (c) 13. (a, d) 14. (b, d) 15. (a; c) 16. (a, d)
17- (a, d) 18. (b, d) 19. (d) 20. (c) 21. (d) 22. (b)
f

:. . ~~)<~~~ :~-i:~';;'~~-~:;::/;;~-~;,:'"' :' _, ~:t, '. -t.'_.··''i1J;;;;:~:f$\::... "... " "i,:.~\t.<~~:·:~: ..~::~~~.'~.r: ·~.-~1::
[" -:;-,SSt:aDO".!f'Q$9N'6f:YPE QUESllGN$;· "
.-,- ."", " :--..'"" .,!:_tt-·.F,.: ..-~ "-;: "",":';'~~~, .. ~·!."1:'::..M-\"!.'i~\1.1:'-~~.'_ . :" ." ":,\,: . .i-.'

1. (d) (a)
2. (c)
3. ,4.(c) (a)
5. 6. (a) 7. (c) 8. (d)
',9. (a) 10. (c) 11. (c) 12. (a) 13. (a) 14. (b) 15. (a) 16. (c)
17. (c) 18. (a)
380 G. R.B. PHYSICAL CHEMISTRY FOR COMPETITIONS

I· lOBJECTIVE QUESTIONS llor ( IITASPlRANTS


The following questions contain single correct option:
Substituting M J = 5 and = x we can calculate x: Y.1
1. Three solutions ofHCI having normality 12 N, 6 Nand 2 N are M2 4 V2 Y
mixed to obtain a solution of 4 N normality. Which among the 5. You are given 500 mL. of 2 N HCI and 500 mL of 5 N HCl.
following volume ratio is correct for the above three What will be the maximum volume of 3 M HCl that you can
components? make from these two solutions?
(a)l:l:5 (b)I:2:6 (c)2:1:9 (d)I:2:4 (a) 250 mL (b) 750 mL (c) 500 mL (d) 1000 mL
[Hint: NjVj + N 2V2 + N3 V3 NR(V1 + Vz + V3 ) 6. The mole fraction of a given sample of 12 in C 6H6 is 0.2. The
. 12 x 1 + 6 x 2 + 2 x 6 N R (9) molality of 12 in C 6H6 is:
NR 41 (a) 0.32 (b).3.2 (c) 0.032 (d) 0.48
2. 6.8 g H 20 2 is dissolved in 224 mL solution. This solution will x B x 1000
[Hint: m '" - ' < - - - - m molality
be labelled as: . (1- xB)mA x B = mole fraction of solute
(a) 0.224 V (b) 20 V (d) 10 V 0.2 x 1000 rnA molar mass of solvent ]
x 1000
m=---
[Hint: M 0.8 x 78
mB xV 7. A IS-volume sample of an H 20 2 solution is equivalent to:
200
(a)5.30N (b)l.77N (c)2.68N (d)7.50N
224 [Hint: V = 5.6N
Volume strength V M x 11.2
15 = 5.611
200
= - x IL2= 10] N = 2.68]
224 8. If PA is the vapour pressure of a pure liquid A and the mole
3. A solution weighing a g has molality b. The molecular mass of fraction of A in the mixture of two liquids A and B is x, the
solute if the mass of solute is c g, be: will partial vapour pressure of A is:
c
(a)-x-- 1000 (b) b
- x1000
-- (a)(l-x)PA (b)xPA (c) x P (d)! x PA
A
b (a-c) a (a-b) (1 x) x
9. Difference between diffusion and osmosis is:
( cb) - x
1000
-- c x1000
(d) - -- (a) a semipermeable membrane is required for osmosis while
c (a-c) a (b-a) diffusion requires no semipermeable membrane
1000 . (b) in osmosis, movement of molecules is in one direction
[Hint: Molality,m x
mB WA whereas in diffusion, the movement is on both sides
b cxtOOO (c) in osmosis only the solvent moves while in diffusion
mB (a c) solute and solvent both move
(d) all ofthe above
c x 1000 ]
10.
b (a c)
4. Two solutions of H 2S04 of molarities x and yare mixed in the
ratio of PI mL: V2 mL to form a solution of molarity MI' If 119
mm
they are mixed in the ratio of V2 mL: Vi mL, they form a
. . . . x d MI 5
Vapour
pressure 37
1 Vapour
solutIOn of molanty M 2' GIven VI I V2 = - > 1 an - =- , pressure
. y ~ 4 of toluene mm
of benzene
then X : yis:
(a)2:1 (b)4:1 (c)1:2 (d) 3: 1
1
[Hint: Molarity of the mixture can be calculated as: 0.2 0.4 0.6 0.8
M1V1 + M 2V2 MR(V1 + V2) Mole fraction of benzene
where, M R =resultant solution Choose the correct option:
VI xx + V2 X Y M1(V1 + V2) ... (i) (a) 'A' represents vapour composition and 'B' the liquid
V2 x x + VI X Y M 2(V1 + V2) ... (ii) composition
Dividing eq. (i) by eq. (H), we get (b) 'A' as well as 'B' represent liquid composition
V1x + V2y (c) Both' A' and' B' represent vapour composition
V:r + V1y M2 (d) 'A' represents liquid composition and 'B' the vapour
composition
SOLUTIONS

11. Insulin (C 2 HlO OS)n is dissolved in a suitable solvent and the


osmotic pressure (n) of solutions of various concentrations
(glcc) C is measured at 20"C. The slope of the plot ofn against
'c' is found to be 4.65 x 10~3. The molecular weight of insulin
is:
(a) 4.8 x lOS (b) 9 x lOS. (c) 3 X lOs (d) 5.17 x 106
Mole fraction Mole fraction
[Hint: nV WB xRT (a) (b)
mB

n ~(WB RT 'j TIlSmix TIlSmix


V ,mB

n =:; C x RT x 1000 ... (i)


mB
where C = concentration in glee,
y=xxm ... (ii) Mole fraction Mole fraction
Comparing eqs. (i) and (ii), (c) (d)
RT
Slope =:; - x 1000
mg 16. A solute forms a pentamer when dissolved in a solvent. The
RT van't Hoff factor 'j' for the solute will be:
mB =--x1000
Slope (a) 0.2 (b) 0.8 (c) 0.5 (d) 0.6
0.0821 x 293 x 1000 17. What is the molarity of HCI in a solution prepared by
4.65 x dissolving 5.5 g HCI in 200 g ethanol if the density of the
solution is 0.79 g mL -I?
= 5.17 X ~06]
(a) 21 M (b) 0.93 M (c) 6 X 10-4 M(d) 1.7 M
12. Solubility of deliquescent substances in water is generally:
(e) 0.58 M
(a) high (b) low
[Hint: V W (mass) = 205.5 = 260.13 rnL
(c) moderate (d) cannot be predicted
d 0.79
[Hint: A deliquescent substance absorbs large amount of water, x 1000
hence solubility is very high.] M=
mB xV
13. Consider the following vapour pressure-composition graph,
SP is equal to: 5.5 x 1000 = 0.58 M]
36.5 x 260.13
. 18. Which statement about the composition of vapour over an
ideal I: I molar mixture of benzene and toluene is correct?

1p~
Assume the temperature is constant at 25°C.
Vapour Vapour pressure data (2S°C):
pressure.
Benzene 75 mmHg
Toluene 22 mm Hg
(a) The vapour will contail\ higher percentage of benzene
0.2 0.4 0.6 0.8 1 (b) The vapour will contai~ higher percentage of toluene
(a)PQ + RS (b)PQ+QR +RS ( c) The vapour will contain equal amount. of benzene and
toluene .
(c)SR +SQ (d)PQ+QR
(d) Not enough information is given to make a prediction
14. Y g ofa non-volatile organic substance of molecular mass M is
[Hint: A: Benzene B : Toluene '
dissolved in 250 g benzene. Molal elevation constant of
benzene is K b • Elevation in its boiling point is given by: P= PA + PB
o 0
(a) M (b) 4K by (c) Kby (d) Kby P = PA X A + PB XB
Kby M 4M M I I
=75 x + 22x-
2 2
[Hint: IlT = Kb x x 1000
. mB XWA
= 37.5 + II = 48.5
y x 1000 4Kb y Mole fraction of benzene in vapour, YA PA = 37.5 0~78'
=Kb X M x250 =-;;;t] P 48
Similarly, mole fraction oftoluene in vapour, YB 0.22
15. Which of the fonowing represents correctly the changes in
:. The vapour will contain higher percentage of benzene.]
thermodynamic properties during the formation of I mole of
an ideal binary solution?
382 G.R.B. PHYSICAL CHEMISTRY FOR COMPETITIONS

19. When 1.2 g of sulphur is melted With 15g of naphthalene, the (a) th~re is positive deviafion from Raoult's law
solution freezes at 77 .ZOC. What is the molar mass of tbis form (b) there is negative deviation from Raoult's law
ofsulphur? (c)there is no deviation from Raoult's law
Data fOf naphthalene: (d)molecular masses of A and B are also required for
Melting point (m.pt.) 80°C calculating the deviation
Freezing point depression constant K f 6.80 K m- I I 2
[Hint: xA =3' ,xe = 3
(a) ISO g mol-I (b) 190 g mol-I
P = P~ xA + P~ xe
(c) 260 g mol- 1 (d) 450 g mol-1
= 150X.!+ 240 x 2 = 50+ 160=21Omm
[Hint:tJ.T = K x we x 1000 3 3
. f me xWA
Pexp < PcaJculated
K x 1000 :. There is negative deviation from Raoult's law.]
me -fx
tJ.T wA 27. A compound MX 2 has observed and normal molar masses
= 6.S x 1.2 x 1000 '" 190] 65.6 and 164 respectively. Calculate the apparent degree of
2.8 15 ionization of MX 2: .
20. Which concentrations can be calculated if the mole fractiot} . (a) 75% (b) 85% (e) 65% (d) 25%
and density of an aqueous solution of HCI are known? NormalllX)iar mass
[Hint:
I. Molality 2. Molarity 3. Percentage by mass Observed llX)lar mass
(a) 1 only (b) 3 only = 164 =2.5
(c) 1 and 2 only (d) 1,2 and 3 . 65.6
21. The vapour pressure of a liquid in a closed container depends i 1
IX= , n =3 (number of ions)
on: . n-l
(I) temperature of liquid (2) quantity of liquid
= 2.5 -1 = 1.5 = 0.75
(3) surface area of the liquid 3 -1 2
. (a) I only (b) 2 only :. Percentage ionization of MX2 will be 75%.]
. (c) I and 3 only (d) 1,2 and 3 28. Compound PdCI4 · 6H20 is a hydrated complex; I molal
·22. A solution is prepared by dissolving a 2.5 g sample of an aqueous solution of it has freezing point 269.28 K. Assuming
unknown compound in 34 g of benzene (C6~) boils at 1.3 SoC ·100010 ionization of complex, calculate the molecular formula
higher than pure benzene. Which expression gives the molar of the complex (Kf for water =1.86 K kg mol-I):
mass of the unknown compound?
Compound :Kb (a) [Pd(H20)6]CI4 (b) [Pd(HzO)4Clz]CI2· 2HzO
1 (c) [Pd(H20hC13]CI·3H20 (d) [Pd(H20hCI4l4H20
C6~ 2.53°Cm-
[Hint: tJ.T = i xKf x m
(a) 2.53 x 2.50 34
(b) 1.38 x - x 2.50 (273 - 269.28) =: i x 1.86 x I
1.38 2.53
3.72 i x 1.86
2.53 1
(c) 2.5 X 103 X - X- (d) 2.50 x 103 x 1.38 x 2.53 i= 2
. 34 1.38 34 i I
IX=--
23. If liquids A and B form an ideal solution: n-l
(a) the free energy of mixing is zero 2-1
(b) the free energy as well as entropy of mixing are zero 1=--orn=2
n-l
(c) enthalpy of mixing is zero
Thus, the complex should give two ions in the solution, i.e., the
(d) the entropy of mixing is. zero
complex will be (Pd(HzOh03]CI·3H20] .
24. Solute A is· a ternary electrolyte and solute B is a 29. Inulin is dissolved in suitable solvent and the osmotic pressure
non-electrolyte. If O.IM solution of B produces an osmotic ',(n) of solutions of various concentrations (glcm 3 ) , C is
pressure of2P at temperature 3T K thenO.05M solution of A
measured at 27~C. The slope of plot ofn against C is found to
at T K will produce an osmoticpressirre of: ,
be4.1 x 10-3. The molecular mass of inulin is: .
(a) P (b) UP (c) 2P (d) lOP
25. Which of the following plots represents the behaviour of an (a) 6x 106 (b) 3 X 106 (c) 6x 103 (d)3 x l<t
ideal binary liquid solution? [Hint: 1tV nRT
=:

(a) Plot of ~otal VS YA (mole fraction of A in vapour) is linear 1tV. -RT


W

(b) Plotof ~otal vs Yn is linear m ,


(e) Plot ofl! Pfutal VS YA is linear 1t=(~lxl RT
(d) Plot ofl/ ~ota! vs Ye is non-linear V/ m
Total vapour pressure of mixture of 1 mol A (p~ = 150torr)
26.
1t=c(R:) ... (i)
and 2 mol B (p2 240 torr) is 200 torr. In this case:
y=MX ... (ii)

,
SOLUTIONS

XB X 1000
Slope ofthe line will be RT , m m=l
m (1- xB)mA
xB = mole fraction of solute
i.e. , RT =4.lx W- 3 x 1000 mA == molar mass of solvent
1 = --'''-----
m (1 xB)XI8
m = RT ~._0_82_1_X_I_00_0-.o-x_3_0_0
= ..
-4.-1-x--;; 4.1 x 10-3 XB == 0.0176, xA 0.9824
P= POXjf
= 6x nf] P = 760 x 0.9824
30. pH of a 0.1 M monobasic acid is measured to be 2. Its osmotic = 746.624
pressure at agiven temperature T K is: !J..p = Po - P 760 ...: 746.624
(a) O.lRT (b) O.1IRT • l.IRT
(c) (d) O.OIRT . '" 13.4 mm Hg]
[Hint: . HA H+ + A- 34. The temperature at which the molarity of pure water is equal
t=O C o 0 to its molality is:
C-Ca ea Ca (a) 273 K (b) 298K
[H+ Ca.; [H+j=;: lO-pH (c) 277 K (d) none of these
35. Isopiestic solutions have:
Ca 10-2
(a) same vapour pressure (b) same osmotic pressure
O.1a =10-2 (c) same freezing point (d) same hoiling point
a==O.1 36. Molarity and molality of a solution of caustic soda are
i-I i respectively 11.12 M and 94.12 m. The density t>fthe solution
a==--; 0.1=
n-I 2 is:
i = 1.1 (a) 0.556 g mL- 1 (b) 5.56 g mL -I
(c) 55.6 g mL- 1
1t "'" iCRT (d) none of these
= 1.1 x 0.1 x RT = 0.1 IRT ]
31. The amount of ice that will separate on cooling a solution [Hint: d =M l r
,m
+ molar mass of SOlute)
1000
containing 50 g of ethylene glycol in 200 g water at -9.3°C,
K
f
1.86Kkg mol-I: ., d= 11.12(-._1_+~)
94.12 1000
(a)38.7lg (b)38.71mg (c)42g (d) 42 mg
= 0.556 g mL-1 ]
WB x 1000
[Hint:!J..T K f x --"'--- 37. Which of the following solutions has osmotic pressure nearer
mB x wA
to that of equimolar solution of K4[Fe(CN)6] ?
9.3 = 1.86 x 50 x 1000 (a) Na2S04 (b) BaCl 2
62xwA
(c) Al 2 (S04h . (d) C12H22011
W A = 161.29 g (amount of water present at ~ 9.3°C) 38. Equal amounts of a solute are dissolved in equal amounts of
:. Amount of ice separated == 200 -161.29 two solvents A and B. The lowering of vapour pressure for the
== 38.71 g] solution A is twice the lowering of vapour pressure for the
32. 2 moles each of liquids A and B are dissolved to form an ideal solution B. If M A and M B are the molecular weights of
solution. What will be the mole fraction· of B in .the vapour solvents A and B respectively, then:
phase? (a)MA MB (b)MA==MBI2
.',
p~ = 120 torr; p~ = 80 torr. (C)MA 4MB (d)MA 2MB
(a) 1/4 (b) 112 (c) 3/5 . (d) 2/5 Following questions may hayemore than one correct options:
[Hint: P = PA +PH ]. Consider the following solutions:
I. I M sucrose II. I M KCl
P == p~ xA
+ P~ xB
2 . 2 III. 1 Mbenzoic acid in benzene.IV.l M(NH3hP04
=120x-+ 80x Which of the following is/are true?
4 4 (a)All solutions are isotonic
:d: 60 +40 = 100 torr
(b) III is hypotonic ofI, II and IV
YB == mole fraction of B in the vapour phase .
(c) I, II and ~II are hypertonic ofIV
40 2
(d) IV is hypertonic of I, II and III
Ptota! 100 == 5 ]
2. Osmotic pressure of the solution depends on:
.33. Lowering of vapour pressure in 1 molal aqueous solution at (a) nature of solute
100°C is: .
(b) nature of solvent
(a) 13.44 mmHg (b) 14.12 mm Hg (c) temperature
. (c)31.2mmHg (d) 35.2 mmHg
(d) molar concentration of solute
[Hint: Molality and mole fraction are related as follows:
384 G.R.B. PHYSICAL CHEMISTRY FOR COMPETITIONS

3. The following is a graph plotted between the vapour pressures 2. The osmotic pressure of solution of Y will be lower than
of two volatile liquids against their respective mole fractions. that of X
3. The freezing point of the solution of X will be fower than
pg . that ofY .
4. The relative lowering of vapour pressure of both the
solutions will be the same
Vapour
pressure
1PA
0 Vapour
pressure Select the correct answer from the codes given below:
1 (a) 1,2,3 (b) 2, 3,4 (c) 1,2,4 (d) 1,3,4
7. For a given value of degree of dissociation, which of the
following have correct van't Hoff factor?
(a) NaCI i 2+a
(b) Ca(N03h 1+ 2a
(c) K 4 [Fe(CN)6] i = 1+ 4a
Which of the following statements are correct?
(d)(NH3 hP04 i = 3+ a
(a) WhenxA = I and xB == 0, then p = p~
8. 1 mole benzene (P~'lZene = 42 rom) and 2 mole toluene
(b) WhenxB == I and xA 0, then p = p~ (P~luene 36 rom) will have:
(c) WhenxA = I and xB == 0, then p < p~ (a) total vapour pressure 38 rom
(d) WhenxB ==1 and xA==o,thenp>p~ (b) mole fraction of vapours of benzene above liquid mixture
4. Which of the following combinations are correct for a binary is 7119 .
solution, in which the solute as well as solvent are liquid? (c) positive deviation from Raoult's law
(a) C 61\ and C 6 H5CH3 ; MSol > 0; 8Vso1 0 (d) negative deviation from Raoult's law
[Hint: P p~ X A + p~ x B A ~ Benzene, B ~ Toluene
(b) CH3 -
°II
C CH3 and CHCI 3 ; MSol < 0; 8VSo1 < 0 = 42 x 1. + 36 x 2
3 3
(c) H 2 0 and HCI; MSol > 0; 8VSo1 < 0 114
(d) H 2 0 and C 2 H sOH; 8Hso1 > 0~8~SOI > 0 =-=38mm
3
5. A solution containing 0.1 g of a non-volatile organic substance Mole fraction of benzene in vapour == Pbenzene = 42/3
P (molecular mass tOO) in 100 gof benzene raises the boiling PIOta! 38
point of benzene by 0.2° C, while a solution containing 0.1 g of = 7/19]
another non-volatile substance Q in the same amount of 9. The decrease in freezing point of an aqueous solution of a
benzene raises the boiling point of benzene by 0.4° C. What is substance is 1.395 J( and that in the freezing point of a
the ratio of molecular masses of P and Q ? (SCRA 2007) benzene solution of the same substance is 1.28 K. The
(a) 1:2 (b) 2:1 (c) 1:4 (d) 4:1 substance:
Kb x ( WB X 1000 ) (a) dissociates in aqueous solution as well as in the benzene
(8T )p rn B x WA p solution
[Hint:
(8T )g K ( x 1000 I (b) forms complexes in the solution
(c) associates in the benzene solution
b X rnB x WA )g
(d) . dissociates in the aqueous solution and not in the benzene
1000 J solution
0.2 _ ( 100 x 100 p 10. The vapour pressure of water at T (K) is 20 rom Hg. The
0.4 - ( 0.1 x 1000 J. following solutions are prepared at T (K):
100 x rnB Q I. 6 g of urea (mol. wt. = 60) is dissolved in 178.2 g of water.
II. 0.01 mole of glucose is dissolved in 179.82 g of water.
1. = (rnB)Q III. 5.3 g ofNa2C03 (mol. wt. = 106) is dissolved in 179.1 g of
2
100 water.
=50
rnB Identify the correct order in which the vapour pressures of
• • (rnB)p : (rnB)Q = 100: 50 solutions increase: IEAMCET (Medical) 2006)
2: 1] (a) III, I, II (b) II, III, I (c) I, II, III (d) I, III, II
6. Consider 0.1 M solution of two solutes X and Y. The solute X [Hint:
behaves as uni-univalent electrolyte while the solute Y 6/60 6
dimerises in solution. Which of the following statements are I. XB = +-
nA + nB 178.2118 60
correct regarding these solutions?
=0.01
1.. The boiling point of solution of X will be bigher than.. that
~Y . tlp =xB 0.01
Po
SOLUTIONS 385
, O.Of II: 'Consider ·lowermg. '9{ "vapour pressure (Ap), elevation in
II Xg = = -;-;::;;::-=--
. nA + no 179.82 + 0.01 boiling point (!!.Tb ) and depression in freezing point (!!.Tf ) of
18 a solvent for the same molar concentration of each of the
= 0.001 following three solutes:
t:.p = 0.001 1. BaCl2 2. NaCl 3: MgCl2
Po Which of the following is/are the correct sequence?
. (SeRA 2009)
III. Xo '" ~ = 17 9. 1 06
5 311
0.005
nA + no __ . + 5.3 (a) !!.p:3<2<1 (b) !!.Tb:1>2>3
18 106 (c) !!.Tf : 3 <2 < I (d) None of these
Ap . [Hint: van't Hoff factOr of both BaC12 and MgClz are same, i,e.,
- = Uo
Po 3, hence none of the given sequence are correct.]
= 3 x 0.005
=O.oI5
.. Vapour pressures of solutions will increase in the following
sequence:
(lII) < (I) < (II).]

[?4~';
• Single correct option
1. (b) 2. (d) 3. (a) 4. (a) 5. (b) 6. (b) 7. (c) 8. (b)
9. (d) 10. (c) 11. (d) . 12. (a) 13. (c) 14. (b) 15. (c) 16. (a)
17. (e) 18. (b) 19. (b) 20. (d) 21. (a) 22. (c) 23. (c) 24. (a)
25. (c) 26. (b) 27. (a) 28. (c) 29. (a) 30. (b) 31. (a) 32. (d)
33. (a) 34. (c) 35. (a) 36. (a) 37. (c) 38. (b)

• One or more than one correct options


1. (b, c, d) 2. (c, d) 3. (a, b) 4. (b, d) 5. (b) 6. (a) 7. ,(b, c) 8. (a, b)
9. (c, d) 10. (a) 11. (d)
386 G.A. B. PHYSICAL CHEMISTRY FOR COMPETITIONS

i 1
• Passage 1 [Hint: ex, , n = 5 since, K 4[Fe(CN)6] gives 5
n-l
ions in the solution
-iA 1.24 M aqueous solution ofK1 has density ofl.15 g/cm 3.
0.6= i-I
Answer the following questions about this solution: 5-1
i =: 3.4]
1. Percentage composition of solute in the solution is:
(a) 17.89 (b) 27.89 (c) 37.89 (d) 47.89
5. A solution of benzoic acid dissolved in benzene such that it
undergoes molecular association and its molar mass
2. Molality of this solution will be:
approaches 244. Benzoic acid molecules will exist as:
(a) 2.61 (b) 1.31 (c) 4.12 (d) 3.12
(a) dimer (b) monomer
3. What is the freezing point of the solution if 'the KI is
(c) tetramer (d) trimer
completely dissociated in the solution?
6. -The molar mass of the solute sodium hydroxide obtained from
(a) - 4.87°C (b) 3.22°C
the measurement of the osmotic pressure of its aqueous
(c) - 1.22°C (d) None of these
solution at 27°C is 25 g mOrl. Therefore, its dissociation
4. Experimental freezing point of the .solution is 4.46°C. What percentage in this solution is: [eET (J&K) 2009J
% ofKI is dissociated?
(a) 75 (b) 60
(a) 82% (b) 90% (c) 83% (d) None
(c) 80 (d) 70
5. Normality of this solution is: NOIDml molar mass
WQ~ 001~ M2~ 003n [Hint: i= ... (i)
AbnomJal molar mass
i 1
• Passage 2 a ~­
n-l
The colligative properties of electrolytes require a slightly i =: 1+ ex, for binary electrolyte
different approach than the one used for the colligative properties of 40
non-electrolytes. The electrolytes dissociate into ions in a solution. It'
1+ a = 25; ex, 0.6
is the number of solute particles that determines the colligative % ionisation 60 ]
properties of a solution. The electrolyte solutions, therefore, show
abnormal colligative properties. To account for this effect we define • Passage3
a quantity called the van i Hoff factor, given by:
M.any chemical and biological processes depend on osmosis
Actual number of particles in solution after dissociation
which is, the selective passage ofsolvent molecules through a porous
Number of formula units initially dissolved in solution membrane from a dilute solution to a mo~e concentrated one. The
osmotic pressure 1t" depends on molar concentration of the solution
1 ( Jor non - electrolytes);
(1t CRT). If two solutions are of equal solute concentration and,
i > 1 ( for electrolytes, undergoing dissociation) hence, have the same osmotic pressu.re, they are said to be isotonic.
i < I ( for solutes, undergoing association). If two solutions are of unequal osmotic preSS/ires, the more
concentrated solution is said to be hypertonic and the more diluted
. Answer the following questions:
solution is described as hypotonic.
1. Benzoic acid undergoes dimerisation in benzene solution. The Osmosis is the major mechanism for transporting water upward
van't Hoff mctor 'i' is related to the degree of association 'a' in the plants. Transpiration in the leaves supports the transport
ofthe acid as: mechanism of water. The osmotic pressure of seawater is about 30
(a) i 1 a (b) i= 1 + a atm; . this is the pressure that must be applied to the seawater
. a (separated from pure water using a semipermeable membrane) to get
(c) i =: 1-- (d) i = 1 + a drinking water.
2 2
Answer tbe foUowing questions:
2. A substance trimerises when dissolved in a solvent A. The
van't Hoff factor' i' for the solution is: 1. A plant cell shripks when it is kept in:
(a) 1 (b) 113 . (a) hypotonic solution (b) hypertonic solution
(c)3 (d) unpredictable (c) isotonic solution (d) pure water
3. For a solution of a non-electrolyte in water, the van't Hoff 2. 4.5% solution of glucose would be isotonic with respect to
factor is: ' ....... solution of urea.
(a) always equal to 0 (b)::;; 1 (a) 4.5% (b) l3.5%
(c) always equal to 2 (d) > 1 but < 2 (c) 1.5% (d) 9%
3. Glucose solution to be injected into the bloodstream must have
4. 0.1 M K 4 [Fe(CN)6] is 60% ionized. What will be its van't same ..... as that of the bloodstream.
Hoff factor? (a) molarity (b) vapour pressure
(a) 104. (b) 2.4 (c) 3.4 (d) 4.4 (c) osmotic pressure (d) viscosity
SOLUTIONS 387
4. .Isotonic solutions have same:
.
CH20H
(a) density (b) molarity I
(c) molality (d) mole fraction (c) CHOH. (d) C 6HI2 0 6
(e) normality I
CH20H
5. Osmotic ryse of a solution depends on:
2. 124 g each of the two reagents glycol and glycerol are added
(a) density (b) temperature
in 5 kg water ot the radiators in the two cars. Which of the
(c) nature of solvent (d) none of these following statements is wrong?
(a) Both will act as antifreeze
• Passage4
(b) Glycol will be better
(c) Glycerol is better because its molar mass is greater than
glycol
(d) Glycol is more volatile than glycerol
3. 620 g glycol is added to 4 kg water in·the radiator of a car.
What amount of ice will separate out at - 6°C? K J = 1.86 K
kg mol-I:
A 8 (a) 800 g (b) 900 g (c) 600 g (d) 1000 g
4. If cost of glycerol, glycol and methanol are same, then the
sequence of economy to use these compounds as antifreeze
Compartments 'A' and 'B' have the following combinations of will be:
solutions: (a) glycerol> glycol> methanol
A B (b) methanol > glycol> glycerol .
1. 0,1 MKCI 0.2MKCI . (c) methanol = glycol = glycerol
2. 0.1% (m/V ) NaCI 10% (mIV) NaCI (d) methanol > glycol < glycerol
3. 18 giL glucose 34.2 giL sucrose 5. Which among the following is the most volatile and the best
4, 20% (mlV ) glucose 10% (mIV) glucq~e antifreeze?
Answer the following questions: (a) CH 30H (b) C 2H sOH (c) Glycol (d) Glycerol .
1. Indicate the solutions which is/are isotonic:
. (a) 1 only (b) 3 only (c) 4 only (d) 2 only • Passage 6
2. The solutions in which the compartment 'B' is hypertonic: The solutions which boil at constant temperature like a pure
(a) 1,2 . (b) 2,3 (c) 3, 4 (d) 1,4 liquid and possess same composition in liquid as well as vapour state
3. Indicate the solution(s) in which the compartment' A' will are called azeotropes. The components of azeotropescannot be
show the increase in volume: separated by fractional distillation. Only non-ideal solutions form
(a) 1 (b) 2 (c) 3 . (d) 4 azeotropes. Solutions with negative deviation form maximum boiling
4. Indicate the solution(s)in which the compartment' B' will azeotrope and the solutionswithpositive deviation form miriimum
show the increase in volume: boiling azeotrope. Boiling poin't ofan azeotrope is never equal t~~e
(a) 1,2,4 (b) 1,2 (c) 2, 3 (d)3,4 boiling points of any ofthe components ofthe azeotrope.
5. The solution in which there will be no change in the level of Answer the following questions: .
the solution in the compartments' A' and 'B' is: 1. The azeotropic solutions of two miscible liquids:
(a) 1 (b) 2 (c) 3 (d) 4 (a) can be separated by simple distillation
(b) may show positive or negative deviation from Raoulfs
'l~w .
• PassageS (c) are supersaturated
(d) behave like single pure component and boil at a fixed
The boiling point elevation and the freezing point depression of
temperature
solutions have a number of practical applications, Ethylene glycol
(CHPH ,CHPH) is used in automobile radiators as an antifreeze 2. Solutions which distill without any change in composition or
because it lowers the freezing point of the coolpnt. The' same temperature are called:
substance also helps to prevent the radiator coolant from boiling (a) saturated (b) supersaturated
away by elevating the boiling point. Ethylene glycol has low vapour (c) ideal (d) azeotrope
pressure, We can also use glycerol as antifreeze, For boiling point 3. The azeotropie mixture of water and HCI boils at 108Se.
.elevation to occur, the solute must be non-volatile, but no such This solution is:
restriction applies .to. freezing point depression, For example. (a) ideal
methanol (CH PH), a fairly volatile liquid that boils only at 65°C is (b) non-ideal with positive deviation
sometimes used as antifreeze in automobile radiators. (e) non-ideal with negative deviation
Answer the foUowing questions: (d) cannot be predicted
1. Which of the following is a better reagent for depression in
freezing point but not for elevation in boiling point?
388 G.RB. PHYSICAL CHEMISTRY FOR COMPI;TITIONS

4. 100 mL liquid A and 50 mL liquid B are mixed to form 138 mL Answer the following questiOlis:
solution: IUs: 1. The freezing point ofthe solution Mis:
(a) ideal solution (b) high boiling azeotrope (a) 268.7 K (b) 268.5 K (c) 234.2 K (d) 150.9 K
(c) low boiling azeotrope '(d) none of these 2. The vapour pressure of the solution Mis:
5. Which, among the following combinations is maximum (a) 39.3 mmHg (b) 36.0mmHg
boiling azeotrope? (c) 29,5 mm Hg (d) 28.8 mm Hg
(a) H2 0 + CH30H (b) CCl 4 + CHCI3 3. Water is 1idde<l{to,~<S9Iution Msuch that the mole fraction of
(c) (CH3hCO + C 2H50H (d) H2 0 + HN03 water in the solutIon b~omes' 0.9: The boiling point of this
solution is: .
• Passage 7 (a) 380.4 K (b) 316.2 K (~)375.5 K (d) 354.7 K
The properties such as boiling point, freezing point and vapour (lIT 2008)
pressure of a pure solvent change when solute molecules are added . [Hint:
to get homogeneous solution. These are called colligative properties. XB X 1000 0.1 x 1000
1. Molality m
Applications of colligative properties, are very useful in daY-lo-day (l-xB) mA 0.9 x 46
life. One of its examples is the use of ethylene' glj/Col and water = 2.415
mixture as anti-jreezing liquid in the radiator ofautomobiles.
t:.T=Kf Xm
A solution M is prepared by mixing ethanol and water.
= 2 x 2.415 = 4.83
The mole fraction of ethanol in the mixture is 0:9:'
Freezing point of solution 155.7 - 4.83 150.9 K
Given : Freezing point depression constant of' wilter
=
(Kf for water) 1.86K kgmor 1 •
, 2. P == pOXA'
= 40x 0.9 = 36mmHg
Freezing point depression constant of ethanol (Kf for ethanol) 3. When water becomes solvent, the molality of solution
1
= 2.0Kkgmor " will be:
Boiling point elevation' constant of water (K b for water) m = x 100 = 0.1 x 1000 = 6:172
= 0.52K kg mor I XA x mA 0.9 x 18
Boiling point elevation constant of ethanol (K b for ethanol) t:.T=Kb x m=0.52 x6.172 = 3.209
1.2K kgmor l Boiling point of solution 373 + 3.209
Standard freezing point of water = 273 K =376.2K]
Standardfreezing point ofethanol: 155.7 K
Standard boiling point of water = 373 K
Standard boiling point ofethanol 351.5 K ,
Vapour pressure ofpure water 32.8- mm Hg
Vapour pressure ofpure ethanol = 40 mm Hg
Molecular weight of water = 18 g mor I
Molecular weight of ethanol' = 46 g mor 1
In answering the following questions, consider the solutions to be
ideal dilute solutions and solutes, to be non-volatile and
non-dissociative.

[~~
Passage 1. 1. (a) 2. (b) 3. (a) 4. (c) , 5. (b)
Passage 2. 1. (c) 2. (b) 3. (b) 4. (c) 5. (a) 6. (b)
Passage 3. 1. (b) 2. (c) 3. (c) 4. (b) 5. (a, b)
Passage 4. 1. (b)') 2. (a) 3. (d) 4. (b) 5. (c)
Passage 5. 1. (a) 2. (c) 3. (b) 4. (b) 5. (a)
Passage 6. 1. (b, d) 2. (d) 3. (c) 4. (b) 5. (d)
. Passage 7. 1. (d) 2. (b) 3. (b)
SOLUTIONS 389--

¢ SELF ASSESSMENT ¢
ASSIGNMENT NO.5

10. A solution of 7.45 g KCI in 1000 mL shows osmotic pressure


SECTION-I
of 4.68 atm at 300K. Calculate the percentage dissociation of
Straight Objective Type Questions KCI:
This section contains 12 multiple choice questions. Each (a) 70% (b) 80% (c) 90% (d) 10%
question has 4 choices (a), (b), (c) and (d), out of which only
11. The mass of glucose that should be dissolved in 50g of water
one is correct.
in order to produce same lowering of vapour pressure as is
1. The relative lowering of vapour pressure of a dilute aqueous produced by dissolving I g of urea in the same quantity of
solution containing non-volatile solute is 0.0125. The molality . water is : [MGIMS (Wardha) 2008)
of solution is about: [PEl;',.(J~~t;ala) 2007]
(a)lg (b)3g (c)6g (d)18g
(a) 0.70 (b) 0.50 (c) 0.90 (d) 0.80
12. Consider the following aqueous solutions and assume 100%
(e) 0.60 ionisation in electrolytes:
2. If the elevation in boiling point of a solution of 10 gm of solute I. 0.1 m urea II. 0.04 m Al2 (S04)3
(mol. mass = lOO) in lOO g of water is t::.Tb, the ebullioscopic
III 0.05 m CaCI 2 IV. 0.005 m NaCI
constant of water is: [PET (Kerala) 2007]
The correct statement regarding the above solutions is ;
(a) 10 ,(b) 100Tb (c) t::. Tb (d) t::.Tb IBHU (Mains) 20081
10
(a) freezing point will be lowest for solution I
(e)lOTb
(b) freezing point will be highest for solution IV
3. Which one of the following. aqueous solutions will exhibit . (c) boiling point will be highest for solution IV
highest boiling point? . (DPMT 2009)
(d) Vapour pressure will be highest for solution II
. (a) 0.1 M KN03 . (b) 0.1 M Na3P04
(c) O.lM BaCl2 (d) 0.1 M K 2S04
4. A solution containing 10 g per dm3 of urea (molecular mass SECTION-II
=60 g mor 1) is isotonic with a 5% solution of a non-volatile Multiple Answers Type Objective Questions
solute. The molecular mass of this non-volatile solute is: 13. K2HgI4 is 50% ionised in aqueous solution. Which of the
(AIIMS 2007)
following are correct?
(a) 300 g moC I · (b) 350 g mol-I
(a) n :::: 7 . (b) n :::: 3 (e) i::= 2 (d) i = 4
(c)200 g mor l (d) 250 gmoC I 14. Henry's law is invalid for gases like, ............... ..
. .
5. The molarity of 5 molal aqueous solution of NaOH having (a) CO2 . (b) S02 (c) Hel (d) N2
density 1.2 g / cc is: . . 15. The azeotropic mixture of two liquids:
(a) 5M (b) 4.1 M (c) 6M (d) 8M (a) boils at constant temperature
6. The value of osmotic pressure does not depend on: (b) can be separated by simple distillation
(a) concentration of solution (c) is super saturated
(b) van'tHofffactor (d) deviates from Raoult's law
(c) temperature 16. Cryoscopic constant of a liquid depends on:
(d) structure of solute particles (a) the latent heat offusion of solvent
7. Henry's law constants for O 2 and N2 are:· (b) the freezing point of solvent
(c) the concentration of solution
"02 = 3.3 X 107 , kN 2 :::: 6.51 X lO7
(d) the melting point of solute
Calculate the ratio of i.e., the ratio of mole fractions of 17. Which of the following is/are influenced by the temperature?
. XN2 (a) Freezing point (b) Boiling point
O2 and N2 dissolved in water at 25" C from air: (c) Vapour pressure (d) Osmotic pressure
(a) 0.62 (b) 0.92 (c) 0.42 (d) 0.52
8. The molal lowering of vapour pressure of water at 100 0 Cis: SECTION~III, ..
(a) 13.68 mm (b) 46 mm Assertion-Reason Type Questions
(c)-65 mm (d) 13.68 cm This section contains 5 questions. Each question contains
9. USP ethanol in aqueous solution in containing 95% ethanol by Statement-I- (Assertion) and Statement-2 (Reason). Each
volume. At 20 0 C, pure ethanol has a density of 0.789 g/ mL question has following 4 choices (a), (b), (c) and (d), out of
and USP ethanol density 0.813 g/mL. What is the mass which only one is correct.
percentage of ethanol in USP ethanol? (a) Statement-l is true; statement-2 is true; statement-2 is a
(a) 46% (b) 90% (c) 86% (d) 92.3% correct explanation for statement-I.
390 G.R.B. PHVSICALCHEMISTRY FOR COMPETITIONS

(b) Statement-l is true; statement-2 is true; statement-2 is not Statements (a, b, c and d) in Column-I have to be matched with
a correct explanation for statement.:t. statements (p, q, rand s) in Column-II. The answers to these
(e) Statement-l is true; statement-2 is false. questions have to be appropriately bubbled as illustrated in the
(d) Statement-l is false; statement-2 is true. following examples:
18. Statement-I: Solubility of ionic compounds in water depends If the correct matches are (a-p,s), (b-q,r), (c-p,q) and (d-s),
on both the lattice energy and the hydration energy. then correct bubbled 4 x 4 matrix should be as follows:
Because p q r s
Statement-2: Ionic compounds dissolve in water when their
hydration energy exceeds the lattice energy. a
e®0 01
----
19. Statement-I: Isotonic solutions have same molar
concentration.
Because
b
0 0
®
Statement-2: Solutions which have same osmotic pressure at
same temperature are known as isotonic solutions.
c
e· 0 0
[Hint: It is not necessary that solutions at same molar
concentration have same osmotic pressure, eg.,
d
®®0 0
1t(O.1M NaCl) > 1t(0.1 M glucose) because NaCI is ionised] 23. Match the Column-I with Column II:
20. Statement-I: The vapourpressure of 0.1 M Hg 2Ci 2 is equal Column-I Column-II
to 0.1 M AlCI 3 solution. (a) CCl4 + CHCl 3 (P) Raoult's law
". Because (b) C 6 H6 + H20 (q) Nearly ideal solution
. Statement-2: 0.1 M AlCI 3 and 0.1 M Hg 2Cl 2 give different (c) PA oc x A (r) Negative deviation from RaouIt's law
numbers of ions in a solution. (d) PA < P~XA (s) Positive deviation from Raoult's law
21. Statement-I: Elevation in boiling point will be high if the 24. Match the Column-I with Column II:
molal elevation constant of solvent is high. Column-I Column-II
Because
(Species completely ionised) (van't Hoff factor)
Statement-2: Elevation in boiling point is a colligative
(a) Hg 2Cl 2 (P)5
property.
(b) K 3[Fe(CN)6] (q) 4
11. Statement-I: Ethylene glycol is used as antifreeze for the
radiator of a car. '. (c) Ca3(P04h (r) 1
Because (d) glucose (s)3
. Statement-2: Ethylene glycol dissolves in water with the help 25. Match the Column-I with Column II:
of hydrogen bond and it lowers the fre,ezing point. Column-I Column-II
(a) Mole fraction (P) mol kg- J
SECTION-IV (b) Molality (q) Depends on temperature
Matrix-.Matdiing 'iype Questions (c) Molarity (r) Unitless
. This section contains 3 questions. Each question contains (d) Normality (s) Indepen~ent of temperature
statements given in two columns which have to be matched.

1. (a) 1. (c) 3. (b) 4. (a) S. (a) 6. (d) 7. (d) 8. (a)


D. (d) 10. (c) 11. (b) 12. (b) 13. (b, c) 14. (a, b, c) 15. (a, d) 16. (a, b)
'liT. (c, d) IS. (a) 19. (d) lO. (d) 21. (b) U. (a) 23. (a-q) (b-s) (c-p, q) (d-r)
'!4. (a~s) (b-q) (c-p) (d-r) 25. (a-r,s) (b-p,s) (c-q) (d-q)
.
6~1" INTRODUCTION It was soon realised that the above classification was not
perfect since many crystalline substances can be converted into
The foundation ofcolloid chemistry was laid down by an English
colloidal form by suitable means. The colloidal form of sodium
scientist, Thomas.Graham, in 1861. The credit for the various
chloride, a crystalloid, can be obtained in benzene. Silver, copper,
advances in this field goes to eminent scientists ,like Tyndall,
gold, etc., which are completely insoluble in water, can be
Hardy, Zsigmondy, N.R. Dhar, S.S. Bhatnaga'r and others.
transformed into colloidal state by suitable methods. X-ray
Thomas Graham classified the soluble substances into two examination of certain colloids like glue and gelatin revealed that
categories depending upon the rate of their diffusion through these were crystalline in nature. Thus, the above classification
animal and vegetable membranes or parchment paper. He was discarded, i. e., the term colloid does not apply to a
observed that certain substances diffuse freely through the particular class of substances but is a state of matter like
membrane, wnereas others do not diffuse. The former type of solid, liquid and gas. Any substance can be brought into
substances on account of their crystalline nature like common colloidal state by suitable means.
salt, sugar, urea, etc., were named crystalloids while the second
type of substances were termed as colloids (Greek word, Kolla, 6.~ PARTICLE SIZE AND COLLOIDAL
meaning glue-like). All inorganic acids, bases and salts and
STA"rE
organic compounds such as sugar, urea, etc., were included in
crystalloids while substances such as starch, gelatin, gums, silicic The colloidal state depends on the particle size. It is regarded as
acid, etc., helonged to the colloidal group. intermediate state between true solution and suspenSion. In

1ilble 6.1 Comparison of Suspenslon~ Colloids and True Solutions


SoNG. Property ~ CoDokl 1Nt . . . . .
--------~~--------~-----
1. Particle size5
> 10- cm or 10 3
A or 100 111j..l 10- cm ;0 10- cm or 10 A t~ 10
7 5 3
A < 10-7 cmor lO Aor I mil
or I mil to 100 mil
2. Visibility Visible with naked eye Visible ~ith ultramicroscope Not visible with any of the optical
means
3. Separation
(a) with filter paper Possible Not possible Not possible
(b) with membranes Possible possible Not possible
4. Diffusion Does not diffuse Diffuses very slowly Diffuses rapidly
5. Settling Settles under gravity Does not settle but it may settle under Does not settle
centrifuge
6. Nature Heterogeneous Heterogeneous Homogeneous
7. Appearance Opaque Generally clear Clear
392 ·1 G.R.B. PHYSICAL CHEMISTRY FOB COMPETlTIONS_

true solutions, the size of the particles of solute is very small and, phase or the continuous phase. This phase forms the larger part
thus,' these cannot be detected by any optical mean$ and freely of the icolloidalsolution. Depending' upon the physical· state of
, diffuse through membranes while ~ suspensions the' size of the disperSed and dispersion phases, the types of colloidal ~~l~tions
particles is large enough arid, thu~, clln be seen even by naked eye given in table 6.2 are possible. . ;:,~;' .•
and do not pass through filter paper. A colloidal sol;ution of gas in gas is not possible as'gaks are
True solutions are homogeneous systems while suspensions completely mispi~le and always form.tru.e.solutions. . .
Lyophobic and Lyophilic COlloid~.·;:,;;\·
are heterogeneous. These ate two extreme systems and. the
colloidal state is an iritermediate between the two. If we start with
... , '.
particles of large size (suspension) and go on subdividing them Colloidal solutions in which the disperSed~phaSelias very little
till we reach the size of particles found in solution,there is affinity for the dispersion phase are termed as lyophobic (solvent
continuous change in the characteristics. During transition, the hating) colloids. These are also termed suspensoids. Colloidal
, mixture passes through an intermediate state which shares the solutions of metals which have negligible affinity for solvents are
characteristics of both. This has been shown in Fig. 6.1. examples of this type. Generally, lyophobic sols are less stable.
On evaporation of solvent, the residue cannot be easily
100mll- 1mll ...·.. transformed back into colloidal state by ordinary means.
Lyophobic colloids are, therefore, also called irreversible
colloids.
Suspension . Colloidal solution True solution
Table 6.2 .--_.
000000 000000000 0000000010
000000 oc>Oc>ooooo 000000000
. Dispersed
Dispersion. '
1dlA 10.11. pbase
S.No. pbase Name . EX3qlpleS ..
(Colloidal
(Solvent)
. Fig. 6.1 , partides)
A system is said to be in colloidal state if particles of one or 1. Gas Liquid Foam Froths of air
more components have tbe size range*. loA to 10 3 A. 2. . Gas Solid Solid foam Pumice stone,
However, it is not possible to draw it sharp line of demarcation , dri~ sea foam
between colloidal. state and suspension.. The properties of one 3; Liquid Gas Liquid Cloud,'mist
. gradually appear into those of the other. Roughly speaking, the aerosol.
colloidal state is a heterogeneous dispersion of solute particles 4. Liquid. Liquid Emulsion· Milk; cream
of size ranging between loAful0 3 A into a solvent. . . 5. Liquid Solid Gel Jellies,curd,cheese
Some properties of the three systemS (suspension colloids and 6. Solid Gas Aerosol Smoke,haze
true solutions) are summarized ititable 6:1. 7. Solid Liquid. Sol Goldsol, sulphursol
8. Solid Solid Solid-sol Ruby glass,minerals, .
TYPES OF COlLOrOAL SOLU"FlONS . gems
The colloidal solutions consist of two-phases, one of which is Colloidal solutions in which the dispersed. phase has··
dispersed in the other, .This is called dispersed. phase, the considerable affinity for the dispe(Sion phase, i.e., colloidal
internal phase or the discontinuons phase. The phase in which solution, can be prepared by bringing them together, are called
the dispersion is done is termed dispersion phase, the external lyophilic (solvent loving) colloids. These are also known as
, Table 6:.3· Comparison of Lyophobic 'and Lyophilic Sols
. '.
S.No. Property Lyophobic sols (SuspenSoid) LYoPIillic Sols (Emnlsold)
1. Preparation Carinot be easily prepared. Special methods Can easily be prepared by shaking or warming
.are required for l'reparation. the substance with solvent.
2. Stability Are less stable. Atemore··stable.
3. Reversibility Are irreversible. Are reversible.
4. Viscosity· Viscosity is nearly the same a~ that of solvent. Viscosity, is much higher than that of solvent.
5. Surface tension Surface tension is almost the same as that of Surface tension is usually low.
the solvent.
6. Hydration or solvation These are less solvated as the particles h;l.Ve less These are highly solvated as the particles have
affinity for solvent. great affinity for solvent.
7. Charge The particles carry a characteristic charge either The particles have little or no charge at all.
positive or negative.
8. Visibility '·Particles can be seen under microscope. Particles cannot be seen under microscope.
9. Coagulation Easily coagulated by addition of electrolytes. Cannot be easily coagulated.

'" 1 A (Angstrom unit) =10- 8 cm = 10- 7 mm ·0.1 mil (milliIriicron) •



. THE COLLOIDAL STATE

emulsoids. Gelatin, proteins, starch; etc., are the examples of this Suspension
type. Colloidal solutions of this type are mQre stable and .are also
known as reversible colloids since.the residue left on evaporation
can be readily transformed back into colloidal state· simply by
adding solvent. When the dispersioIl medium is water, the above
colloidal solutions rure termed as hydrophobic and hydrophilic. i
(For comparison, seehable 6.3) , '
. The. colloidalsolptio~s are al~o named ~ccording. tOi t~e "
disperslOn phase or :m~dlum used.\ The colloIdal solutions tn'
alcohol and benzene. are known as alco~ols and benzosols .
r~sp.ec~VelY. ~e COllOidalsolution~:Vhe.re mater i~ used as the
·dispersion medlUIn are called hydro~rls or\aquasols~ . Fig. 6.2
, :' 1 I; \ ." .1 .
Note: Colloidal solutions of solids in liqui~ ar~ abbrevi~ted as sols. colloidal graphite and gum arabic in lampblack colloidal solution
(Indian ink).
i I· !
·6.4 PREPARATION OF COLLOIDA~ 2. Electro-dispersion (Bredig'sarc method): This method is
SOLUTIONS :I \ suitable for the preparation of colloidal solutions of metals like
. gold, silver, platinum, etc. An arc is struck between the metal
1. Preparation of lyophilic sols: \rhe c?lloidal solutions of electrodes under the surface of water containing some stabilizing
lyophilic colloids like starch, glue, gelatin, !:etc., can be readily
prepared by dissolving these substances in wfiter either in cold or
on warming. Solutions of colloidal electrolyt~s such as soaps and ~----Metal rod
dye stuffs can also be prepared similarly.
2. Preparation of lyophobic sols: To get a substance in
colloidal form either the substance in bulk is broken down into 'm~gIBm~~tlce
t~ bath
Electric arc
fme particles of colloidal dimension (1. A to 103 A) or increasing
the size of molecular particles as to form larger aggregates. In
some cases, a third substance is" usually added to mcrease the ~~~~~~~~1~~~~~Lwater + KOH
stability of the sol. These substances are called stabilizers. Th~s,
there are two ways by which the lyophobic sols can be prepared: Fig. 6.3
(i) Dispersion methods: By splittmg coarse aggregates of agen~s such as a trace of KOH. Thewater is cooled by immersing
a substance into colloidal size. ' . the container in·a.n ice bath. The intense heat of the arc vaporises
'(ii) Condensation methods: By aggregating very smail . some of the metal which condenses under cold water.
particles (atoms, ions or moI~ules) into colloid;d size. Note: I. This method is not suitable when the dispersion medium
is all organic liquid as considerable charring occurs.
Table 6.4 . 2. !his method cOinprises.both dispersion and condensation.
S. No. . Dispersion methods·.1 Condensation ,methods 3. Ultrasonic dispersion: The s~und waves of high
1. Mechanical dispersion Exchange of solvents frequency are usually called ultrasonic waves. These waves can
2. Electro-dispersion Change of physical state be produced when quartz crystal discs are connected with a high
Chemical methods: frequency generator. The application of ultrasonic waves for the
,3. Ultrasonic dispersion
preparation of colloidal solutions was frrst introduced by Wood
4. Peptization (i) Double decomposition
and Loomis, in 1927. Various substances like oils, mercury,
(ii) Oxidation·
sulphur, sulphides and oxides of metals can be dispersed into
(iii) Reduction
(iv) Hydrolysis
colloidal state very easily with the help of ultrasonic waves.
4. Pepmation: The dispersion of a freshly precipitated
material into colloidal solution by the action of an electrolyte in
Dispersion Methods
solution is termed peptization. The electrolyte used is called a
1. MechaniCal dispersion: Solid material is fIrst :finely peptizing agent. .
ground by usual methods. It is then mixed with dispersion medium
. A few examples of sols obtained by peptization are:
which gives a .coarse suspension.· The suspension is now
(i) Freshly prepared ferric hydroxide on treatment with a
. introduced into the colloid mill. The simplest form of colloid mill
consists of two metal discs held at a small distance apart from one small amount of ferric chloride solution at once forms a dark
red~ish brown solution. Ferric chloride acts as a peptizing agent.
another and capable of revolving at a very high speed (about 7000
revolutions per minute) in opposite directions. The particles are (ii) Freshly prepared stannic oxide on.treatment with a small
ground down to colloidal size and are then dispersed in the liquid. amoUnt, of dilute hydrochloric acid forms a stable colloidal
A .stabilizer is often added to stabilize the colloidal solution. solution ofstanmc oxide.
Colloidal graphite (a lubricant) and printing ink are made by this (iii) Freshly precipitated silver chloride can be converted into a
method. Tannin is used as a stabilizer in the preparation of coiloidal solution by adding a small amount of hydrochloric acid~
394 G.R.B. PHYSICAL CHEMISTRY FOR COMPETITIONS

(iv) Cadmium sulphide can be peptized with the help of Organic reducing agents such as formaldehyde, phenyl
. hydrogen sulphide. hydrazine, tannic acid, etc., can also be used .
The process of peptization thus involves the adsorption of AgN0 3 + tannic acid Silver sol
suitable ions (supplied by the electrolyte added-particularly a
AuCl 3 + tannic acid ----t Gold sol
common ion) and electrically chargt¥l particles then split from
the precipitate as colloidal particles. . (d) HydrolYSiS: Colloidal solutions of some salts can be
prepared by hydrolysis. A colloidal solution of ferric hydroxide is
Condensation Methods obtained by boiling a dilute solution oHerric chloride.
1. By exchange of solvents: If a solution of sulphur or FeCl 3 + 3HzO----t Fe(OH) 3 +3HCI
phosphorus prepared in alcohol is poured into water, a colloidal Red sol
. solution of sulphur or phosphorus is obtained due to low The colloidal solution of silicic acid is also obtained by
solubility in water. Thus, there are a number of substances whose hydrolysis of dilute solution of sodium silicate with 4N
colloidal solutions can be prepared by taking a solution of the hydrochloric acid which is added drop by drop with constant
substance in one solvent and pouring it into another solvent in stirring.
which the substance is relatively less soluble.
2. By change of physical state: Colloidal solutions of 6.5 PURIFICATION OF COLLOIDAL
certain elements such as mercury and sulphur are obtained by . SOLU1"IONS
passing their vapour through cold water containing a stabilizer Colloidal solutions prepared by above methods generally contain
(an ammonium salt or a citrate). excessive amount of electrolytes and some other soluble
3. Chemical methods: The chemical methods involve impurities. The presence of traces of electrolyte is essential for
chemical reactions in a medium in which the dispersed phase is the stability of the colloidal solution but larger quantities
sparingly soluble. A condition of supersaturation is produced but coagulate it. It is, therefore, necessary to reduce the concentration
the actual precipitation is avoided. Some familiar reactions used of these soluble impurities to a requisite minimum. The process
are: used for reducing these impnrities to a requisite minimnm is
(a) Double decomposition: (i) Arsenious sulphide sol: A known as purificatiou of colloidal solution. The purification of
1% solution of arsenious oxide is prepared in hot water. The colloidal solution is carried out by the following methods:
solution is cooled, filtered and is then gradually added to water (i) Dialysis*: It is a process of removing a dissolved
saturated with hydrogen sulphide, whilst a stream of Hz S is being substance from a colloidai solution by means of diffusion
pass~d through the solution. This is continued till an intense through suitable membrane. Since, particles in true solution (ions
yellow-coloured solution is obtained. Excess of Hz S is removed or smaller molecules) CaJ;1 pass through animal membranes
by bubbling hydrogen through the solution. (bladder) or parchment paper or cellophane sheet but colloidal
AS 2 S 3 +3H zO particles do not, the above can be used for dialysis. The appaiatus
Yellow sol used for this purpose is called dialyser. A bag of suitable
(ii) Antimony sulphide sol: A 0.5% solution of potassium membrane containing the colloidal solution is suspended in a
antimonyl tartarate is added drop by drop to water saturated with vessel through which fresh water is continuously flown
HzS, whilst H 2 S is being passed through the solution. Orange (Fig. 6.4). The molecules and ions diffuse througQ the membrane
coloured solution of antimony sulphide is obtained. into the outer water and pure colloidal solution is left behind.
CH(OH)COOK' CH(OH)COOK
2 I +3H zS----t2 I Dialysing membrane
CH(OH)COO(SbO) CH(OH)COOH

+ Sb + 2H 2 0 Water + Crystalloid
2S3
Orange sol
CrystallOid
(b) Oxidation: A colloidal solution of sulphur is obtained
Sol particle
by passing H 2S into a solution of sulphur dioxide.
2H zS + S02 ----t 2H z0 + 3S
Sol Fig. 6.4
Sulphur sol can also be obtained when Hz S is bubbled through (ii) Electro-dialysis: Ordinarily, the process of dialysis is
an oxidising agent (bromine water or nitric acid). quite slow. It can be made faster by applying an electric field if
(c) Reduction: Colloidal solutions of metals like gold, the dissolved substance in the impure colloidal solution is only
silver, platinum, lead, etc., can be obtained when their salts electrolyte. The process is then named electro-dialysis. The
solutions are acted upon by reducing agents. colloidal solution is placed between two electrodes. while pure
water is taken in a compartment on each side. Electrodes are
2AuCl 3 + 3SnCl 2 ----t 3SnCl 4 + 2Au . fitted in the compartment as shown in Fig. 6.5. The ions present
Sol

'*B1ood i~ a colloidal solution. In case of kidney failure, blood cannot be purified. Under such condition, the blood is separated from dissolved toxic
impurities. by dialysis and re-introduced in the blood stream. "
I

THE COLLOIDAL STATE

in the colloidal solution migrate out to the oppositely charged higher viscosity and lower surface tension in comparison to the
electrodes. dispersion medium.
S. Colligative properties: Colloidal particles are bigger
aggregates. Thus, the number of particles in the colloidal solution
Dialysing membrane is comparatively small as compared to true solution and hence,
the values of colligative properties (osmotic pressure, lowering in .
vapour pressure, depression in freezing point and elevation in
boiling point) are of small order as compared to values shown by
true solutions at same concentrations.
6. Tyndall effect: If a heterogeneous solution placed in
dark is observed in the direction oflight, it appears clear and if it
is observed from a direction at right angles to the direction of
Fig. 6.5 light beam, it appears perfectly dark. Colloidal solutions show a
(iii) Ultrafiltration: Ultrafiltration is the process of mild to strong opalescence, when viewed at right angles to the
separating the colloidal particles from the solvent and soluble path of light, i. e. , the path of the beam is illuminated by a bluish
solutes present in the colloidal solution by especially prepared light. This effect was first observed by Faraday and later studied
filters, which are permeable to all substances except the colloidal in detail by Tyndall and is termed as Tyndall effect. The bright
particles. cone of the light is called Tyndall cone (Fig. 6.6). The Tyndall
CoJloidal particles can pass through ordinary filter paper effect is due to the fa~t that colloidal particles absorb light energy
because the pores are too large. However, the pores of filter paper and then scatter in all directions in space. This scattering oflight
can be reduced in size by impregnating with colloidion solution illuminates the path of beam in the colloidal dispersion.
and subsequently hardened by soaking in formaldehyde. The
usual colloidion is a 4% solution of nitro-cellulose in a mixture of Eye 14 Eye 14
alcohol and ether. An ultra-filter paper may be prepared by
soaking the filter paper in a colloidion solution and hardened by
formaldehyde and finally dryil'lg it. Thus, by using ultra-filter
paper, the colloidal particles are separated from rest of the
materials. Ultrafiltration is a slow process. To speed up the
Light {
source
::::->:-:-»>
ecce:,', ... ..
,
. .. . .
process, pressu~e or suction is used.
The colloidal particles left on the ultra-filter paper are then ~

,>:-:,:-:-:
stirred with fresh dispersion medium (solvent) to get a pure
True solution Colloidal solution
colloidal solution.
Fig. 6.6
;'11\ PROPER1"IES OF COLLOIDAL SOLUTION Tyndall effect can be observed during the projection of picture
in the cinema hall due to scattering of light by dust and smoke
1. Heterogeneous character: Colloidal solutions are
heterogeneous in character as they consist of two phases: particles present there. Tyndall effect is observed only when the
(a) dispersed phase and (b) dispersion medium. following two conditions are satisfied:
(i) The diameter ofthe dispersed particles is not much smaller
2. Visibility: It is not possible to see colloidal particles with
than the wavelength of the light used; and
naked eye or with the help of microscope. It is a well known fact
that no particle is visible if its diameter is less than half the (ii) The refractive indices of the dispersed phase and the
wavelength of the light used. The shortest wavelength of the' dispersion medium must differ greatly in' magnitude. This
visible light is about 4000 A or 400 n1J..t. Hence, no particle of
diameter less than 200 n1J..t can be seen. The size of colloidal
particles is less than 200 n1J..t .
14- Eye
Recently, three new techniques have been developed to
determine the size and shape ofthe colloidal particles. These are:
(i) Scanning electron microscope (SEM).
(ii) Transmission electron microscope (TEM). Light , Tyndali cone
source
(iii) Scanning transmission electron microscope (STEM).
3. Filtrability: Colloidal particles pass through an ordinary
filter paper. However, the particles do not pass through
parchment and other fine membranes.
4. Surface tension and viscosity: The surface tension and Colloidal solution
viscosity of lyophobic sols are not very different from those of
the dispersion medium. On the other hand, lyophilic sols show Fig. 6.7

.
396 G.R.B. PHYSICAL CHEMISTRY FOR COMPETITIONS

. condition is satisfied by lyophobic sols. The. lyophilic sols show positive or negative. A list of some common sols with the type of
little or no Tyndall effect as there is very small difference in the charge on their particles is given in table 6.5.
refractive indices of the dispersed phase and the dispersion
. TabJe 6.5
medium.
Some examples of Tyndall effects are: S.No. Positively charged Negatively charged
(i) Blue colourotsky and seawater
L Metallic· hydroxides, e.g., Metals, e.g., copper, silver,
(ii) Visibility of tails of comets Cr(OH)" AI(OHh and gold-sols.
(iii) Twinkling of stars. Fe(OHhsols.
Tyndall effect is used to distinguish between a. colloidal and· 2. Basic dyestuffs, e.g., methyl- Metallic sulphides. e.g.,AS 2 S3,
true solution. Zsigmondy, in 1903,.used Tyndall effect to setup ene blue sol. Sb 2Sj, CdS sols..
an apparatus known as ultramicroscope. An intense beam of
3. Proteins in acidic·medium. Acid dyestuffs, Ie.g., . eosin,
light is focussed on the colloidal solution contairied in a glass congo red-sols. \
vesseL The focus of the light is then observed with a microscope
at right angles to the beam. Individual colloidal particles appear
. 4. Oxides, e.g., Ti02 sol. Sols of starch, gpm; gelatin,
clay, charcoal, etcJ
as bright stars against a dark background. Ultramicroscope does
not render visible the actual colloidal particles but only the light The charge on the sol particles is due to otie or more of the
scattered by them. Thus, ultramicroscope does not provide any follOwing reasons: .
information about the size and shape 'of colloidal particles. (i) Due to the presence of acidic or basic groups: A protein
7. Colour: . The colour: of colloidal solution depends on the molecule has a carboxylic group and a basic· NH2 group. The
wavelength of light scattered by the dispersed· particles .. The particles of proteins in sols can either have positive charge or

<
wavelength of light further depends on· the size and nature oithe negative
. charge depending
. upon the pH of the medium.
.
particles. The colour of colloidal solution also changes with the . .... + . .
way the observer receives the light. For example, milk and water · . .././NH2 . . . .. NH3. . _ (aCl'd'lC
mixture appears .blue .when viewed.by the reflected light and CH2'-.:. + HCl:---1' CH2 " +CI d')
. . "COOH . . . . 'COOH. . me mm
looks red by the transmitted light: Finest gold sol is red in colour.. GlyCine
As size of the particles increases, it beco~espurple,Jhen blue
and fmally golden yellow. .. . . · .·/N'B2 ... ' ... ' NH2 , . '. .

8. Brownian movement: When colloidal solutions are. CH 2", + NaOR:--~ CH2 ( .. ..•. +Na++H20
viewed under a powerful ultramicroscope, the colloidal particles . COOH . .. "COO~;
Glycine Negillively charged:
app~ar to be in a state of continuous motion in zig-zagdirectiotis
. .
all over the field of view. This motion was fjrst observed by the (alkaline. medium)
IsoeleCtric point of a conoid: 'Incase ofdolloidalsolution ..

• prOteins, the nature of charge depends on the pH oithe solution


called . isoelectric point Above: this pH, the partides are
negatively charged and below this pH, they have positive charge.
At isoelectric point, colloidal particles exist in the form ofZwitter
ion hence they do not migrate under the influence of extema1
electric field.
Fig.6.a Examples: Colloidal sol Isoelectric pH
British botanist Robert Brown and hence is known as Brownian Haemoglobin 4.3-5.3
movement (Fig. 6.8). This motion is independent of the nature of Casein from human milk 4.1-4.7
the colloid but depends on the size· of the particles arid the Gelatin 4.7
viscosity of solution. Smaller the size and lesser the viscosity, (ii) Due to self-dissociation: When colloidal particles such
faster is the motion. The motion becomes. intense at high as soaps or detergents are dissolved in water, ionised molecules
temperature. The distance betwe.en two points decreases at · associate to form a micelle. The outer surface will be thus
elevated temperature. . charged depending on the charge of the ions from which it is
The Brownian movement has been explained to be due to the formed. Thus, sodium palmitate solution will have negative
unbalanced bombardment of the particles by the molecules of the ~harge on its sol particles.
dispersion medium. The Brownian movement has a stirring effect C I5 H 31 COONa :---1' C ls H 31 COO- + Na +
which does not permit the particles. to settle. and thus, is sodium palmitate
responsible for the stability of sols. (iii) Due to electron I;apture by sol particles: e.g. ,during
9. Charge on colloidal particles: Colloidal particles electro-dispersion of metak .
always carry an electric charge. This charge is of the same type
(iv) :Ouetopreferential adsorption of ions: This is the
on all the particles in a given colloidal solution and may be either
most accepted view. The sol particles acquire positive or negative
charge by preferential adsorption of +ve or -ve ions. When two
THE COLLOIDAL STATE 397
or more ions are present in the dbpersion medinm, ; 41tTJu
preferential adsorption of the ion common to the· colloidal D
particle usually takes place. This can be explained by taking the
where, TJ = Coefficient of viscosity
following examples:
u Velocity of colloidal particles
(a) If silver nitrate solution is added to potassium iodide
D = Dielectric constant of the medium
solution, the precipitated silver iodide will adsorb iodide ions
The presence of equal and similar charges on colloidal
from the dispersion medium and negatively charged colloidal
particles is largely responsible in providing stability to the
solution will result. However, when KI solution is added to
colloidal solution because the forces of repulsion exist between
AgN03 solution, positively charged sol will result due to
same charged particles which prevent them from coalescing or.
adsorption ofAg + ions from dispersion medium.
aggregating when they come closer to one another.
AgIlr AgII Ag + 10. Electrophoresis: When electric potential is applied
Negatively charged Positively charged
across two platinum electrodes dipping in a colloidal solution, the
(b) If FeCl 3 is added to excess of hot water, a pos~tively colloidal particles move towards one or the other electrode. This
charged sol of ferric hydroxide is formed due to adsorption of movement of colloidal particles under an applied electric fIeld is
Fe 3+ ions. . called electrophoresis. Positively charged particles move
Fe(0H)3 /Fe3 + towards the cathode while negatively charged particles move
Positively charged
towards anode. This can be demonstrated by the following
However, when ferric chloride is added to NaOH a negatively experiment:
charged sol is obtained with adsorption of OH - ions. The apparatus consists of a U-tube provided with a stopcock
Fe(OH)3 /OH- through which it is connected to a funnel-shaped reservoir
Sn02 is positively charged colloidal sol due to adsorption of (Fig. 6.9). A small amount of water is fIrst taken in the U-tube
Sn 4+ ions. and then a requisite quantity of colloidal solution is placed in t~e
Sn02 + 4H+ ----7 Sn 4+ +2H 2 0
Sn02 + Sn 4+ ----7 [Sn02 ]Sn 4+
Positive colloid
On the other hand in alkaline medium, Sn02 forms negatively
~harged colloidal sol due to adsorption of SnO~- ions formed. Water
Sn02 + 20W ----7 SnO~- + H2 ° (dispersion)
medium
Sn02 + SnO~- ----7 [Sn02]SnO~-
Negative colloid
Colloidal
Electrical doub~e The surface of a colloidal particle solution
acquires a positive or a negative charge by selective adsorption
of ions carrying positive or negative charge respectively as stated
above .. This layer attracts counter ions from the medium which
form a second layer.
AgIlr I K+ AgIlAg+ Ir
The combination of the two layers of opposite charges around ag..--~Stop cock
the colloidal particle is called Helmholtz electrical double layer.
According to modem views, the fIrst layer of ions is fIrmly held Fig. 6.9
and is termed fixed layer while the second layer is mobile which
reservoir.. The stopcock is slightly opened and the reservoir is
is termed diffused layer. Since, separation of charge is a seat of
gradually raised as to introduce the colloidal solution into the
potential, the charges of opposite signs on the fIxed and diffused
U-tube. The water is displaced upwards producing, a sharp
parts of the double layer results in a difference in potential
boundary line in each arm. The platinum electrodes are fItted in
between these layers. This potential difference between the fIxed
the water layer and a voltage of 50 to 200 volts is applied. The
charged layer and the diffused layer of opposite charge is called
movement of the particles can be observed towards one of the
the electrokinetic potential or zeta potential.
electrodes by seeing the position of boundary. When the colloidal
+ I particles are negatively charged, the boundary moves down in the
+ I cathodic arm and moves upwards in the anodic arm showing that
+ I - particles move towards anode. It is, thus, possible to ascertain the
Solid + I - charge on particles by noting the movement of boundary.
+ I When electrophoresis, i. e. ,movement of particles is prevented
+
+ I _ by some suitable means, it is observed that the dispersion
I medium begins to move in an electric fIeld. This phenomenon is
Fixed Diffused
termed electro-osmosis.
layer layer.
398 G.R.B. PHYSICAL CHEMISTRY FOR COMPETITIONS

Summary of Electrokinetic Phenomena [Fe(CN) 6 J4- > PO~- > SO~- > CI-
Motion Causing Potential:
The minimum concentration of an electrolyte in millimole
Sedimentation Potential Streaming Potential per litre required to canse precipitation of a sol in 2 hours is
caUed flocculation valne. The smaller the flocculating value, the
Medium rests but dispersed Medium moves but dispersed higher will be the coagulating power of an ion.
particles move. particles rest.
Flocculation value of an ion depends on its charge:
Potential Causing Motion: Flocculation value ex _1_ where Z = charge of ion '
(Z)6
Electrophoresis Electro-osmosls
Flocculation value of M±, M 2± , M 3±" M 4± ions lies in
Medium rests but dispersed Medium moves but dispersed
following ratio.
particles move. particles rest.
1 I 1 1
11. Coagulation or precipitation: The stability of the (ll : (2)6 : (3)6 : (4)6
lyophobic sols is due to the presence of charge on colloidal 1 : 0.0156 : 0.00137 : 0.00024
particles. If, some how, the charge is removed,the particles will 100 : 156 : 0.137 : 0.024
come nearer to each other and thus, aggregate or flocculate and
Coagulation of lyophilic sols: There·are two factors which
settle down under the force of gravity.
are responsible for the stability of lyophilic sols. These factors
The flocculation and settling down of the colloidal particles is
are the charge and solvation of the colloidal particles. When these
called coagulation or precipitation of the sot: '
two factors are removed, a lyophilic sol can be coagulated. This
The coagulation of the lyophobic sols can be carried out by is done (i) by adding electrolyte and (ii) by adding suitable
following methods: solvent. When solvents such as alcohol and acetone are added to
(i) By electrophoresis: . In electrophoresis, the colloidal hydr~philic sols, the dehydration of dispersed phase occurs.
particles move towards oppositely charged electrode. When these Under this condition, a small 'quantity of electrolyte can bring
come in contact with the electrode for long, these are discharged about coagulation.
and precipitated. ' 12. Protection of colloids: Lyophilic sols are more stable
(ii) By mixing two oppositely charged sols: When than lyophobic sols. This is due to the fact that lyophilic colloids
oppositely charged sols are mixed in almost equal proportions, are extensively solvated, i. e. ,colloidal particles are covered by a
their charges are neutralised. Both sols may be partially or sheath of the liquid in which they are dispersed.
completely precipitated as the mixing of ferric hydroxide (+ve Lyophilic colloids have a unique property of protecting
sol) and arsenious sulphide (-ve sol) bring them in precipitated lyophobic colloids. When a lyophilic sol is added to the lyop-
form. This type of coagulation is called mutual coagulation or hobic sol, the lyophilic particles form a layer around lyophobic
meteral coagulation. particles and thus, protect the latter· from electrolytes. Lyophilic
(iii) By boiling: When a sol is boiled, the adsorbed layer is colloids used for this purpose are called protective colloids.
disturbed due to increased collisions with the molecules of The lyophilic colloids differ.in their protective power. The
dispersion medium. This reduces the charge on the particles and protective power is measured in terms of Gold Number. This
ultimately they settle down to form a precipitate. number was. introduced by Zsigmondy and is defined as the
(iv) By persistent dialysis: On prolonged dialysis, the number of milligrams of a lyophilic colloid that will just
traces of the electrolyte present in the sol are removed almost prevent the precipitation of 10 mL of a gold solon the
completely and the colloids become unstable. addition of 1 mL of 10% sodium chloride solution.
(v) By addition of electrolytes: When excess of an The gold numbers of some of the common protective colloids
electrolyte is added, the colloidal particles are precipitated. The are listed below:
reason is that colloidal particles take up ions carrying charge
opposite to that present on themselves. This causes neutralisation Table 6.6
leading to their coagulation. The ion responsible for
S.No~ Proteetlve Colloid Gold Number
neutralisation of charge on the particles is called the flocculating
ion. A negative ion causes the precipitation of positively charged 1. Gelatin 0.005-0.01
sol and vice-versa. 2. Haemoglobin :~"··(·0.03
It has been observed that, generaUy, the greater the valency
3. Gum arabic 0.15
. of the flocculating ion added, the greater is its power to cause
precipitation. This is known as Hardy-Schulze rule. In the 4. Egg albumin 0.08-0.10
coagulation of a negative sol, the flocculatmg power ofNa + , 5.. Potato starch 25
Ba2+ andAl 3+ ions is in the order of: 6. Sodium oleate 0.4
A1 3+ > Ba 2+ > Na +
7. Gum tragacanth, 2
Siinilarly, in the coagulation of a positive sol, the flocculation
8. Starch 25-50
power of CI- , SO;- ,PO~- and [Fe(CN) 6 J4- is in the order of:
THE COLLOIDAL STATE 399

Higher is the gold number, lower will be the protective power. heavy metal salts of fatty acids, long chain alcohols, lampblack,
Thus, gelatin and starch have the maximum and minimum etc.
protective powers. The use of protective colloids is widespread. Emulsions c'an be diluted with any amount of the dispersion
Gelatin is added in the preparation of ice cream as protective medium. On the other hand, the dispersed liquid, when mixed,
agent to the colloidal particles of ice. Argyrol, used as eye drops, forms at once a separate layer. The droplets in emulsions are
is a silver sol protected by organic material: . often negatively charged and can be precipitated by electrolytes.
Some facts: They also show Brownian movement and Tyndall effect.
(i) Blood is a colloidal solution, it is not coagulated by the Emulsions can be broken into constituent liquids by heating,
electrolytes (NaCI, Ca 3 (P0 4 h etc.) present in blood itself freezing, centrifuging, etc.
because proteins present in blood act as protective colloid. Detection of emulsion: Following tests can be used to
(ii) Milk of human mother is better protected than cow's and detect the type of emulsion:
buffaloes milk.
(i) Dye test: An oil soluble dye is shaken up with the
(iii) The proteins which are not coagulated at their iso-electric emulsion and a drop is seen under microscope. If coloured drops
pH are ~etter protective colloid. are seen, the emulsion is water in oil type otherwise of oil in
(iv) Ageing : It is spontaneous destabilisation of colloidal water type.
solution. In this process, dispersed is separated from dispersion
(ii) Viscosity test: Water in oil type emulsion has greater
medium by itself, i.e., no artificial method is used.
viscosity than oil in water type emulsion.
Congo Rubin Number (iii) Electrical conductivity test: Electrical conductivity of
oil in water type emulsion is much greater than water in oil type
Ostwald proposed that it is the amount of protective colloid in emulsion. If electrolyte like NaCI is added to qil in water type
mg which prevents the colour change in 100 mL of 0.0 I % Congo emulsion, its conductivity greatly increases.
rubin dye solution to which 0.16 g equivalent of KCI are added
(iv) Spreading test: Water in oil type emulsion easily
when observed after 10-15 min.
spread on the surface of an oil but not on the surface of water.
13. CoIJigative properties: The value of colligative
properties are of very small order because they have very small (v) Dilution test: Oil in water type emulsion can easily be .
van't Hoff factor. diluted with water. On the other hand water in oil type emulsion
cannQt be diluted with water. For example, milk-which is oil in
nAs 2 S 3 - - 7 (As 2 S 3 )n .
water type emulsion can be diluted with water, but butter or milk
n:::: 1000
" 1 I cream which are water in oil type emulsion cannot be diluted with
van't Hoff factor i - - water.
1000
(vi) Coalescence: It is the phenomenll of disappearance of the
Colligative properties are directly proportional to the van't
boundary between two particles (generally droplets or bubbles)'
Hoff factor.
in contact. Coalescence leads to the reduction of the total surface
area. The flocculation of an emulsion, viz., the formation of·
6.7 EMULSIONS aggregates, may be followed by coalescence. In extensive
condition, the coalescence leads to the formation of a macrophase
These arc liquid-liquid colloidal systems, i. e. , the dispersion of and emulsion is said to break. Coalescence of solid particles is
finely divided droplets in another liquid. If a mixture of two called sintering.
immiscible or partially miscible liquids is shaken, a coarse
dispersion of one liquid in the other is obtained which is called
~.6~81 CLASSIFICATION OF COLLOIDS
emulsion. Generally, one of the two liquids is water. There are
two types of emulsions:
BASED ON THE TYPE Of PARTICLES
(i) Oil dispersed in water (OIW type), OF DISPERSED PHASE
(ii) Water dispersed in oil (W10 type). Depending upon the type of particles of dispersed phase, colloids
In the first system, water acts as dispersion medium. Examples are classified as:
of this type of emulsions are milk and vanishing cream. In milk, (i) Multimolecular, (ii) Macromolecular" (iii) Associated
liquid fat is dispersed in water. colloids.
In the second system, oil acts as dispersion medium. Conimon (i) Multimolecular colloids: On dissolution, a large number
examples of this type are butter and cream. of atoms or smaller molecules of a substance aggregate together
Emulsions of oil and water are unstable and sometimes they to form species having size in colloidal dimension (diameter less
separate into two layers on standing. For the stabilization of an than 1 nm). The species thus formed are called multimolecular
emulsion, a third component called emulsifying agent is usually colloids. For example: Gold sol, sulphur soL In these colloids, the
added. The emulsifying agents form an interfaciallilm between particles are held together by van der Waals' forces.
suspended particles and the medium. The principal agents for (ii) Macromolecular colloids: Macromolecular substances
OIW emulsions are proteins, gums, natural and synthetic soaps, dissolve in suitable solvent to form solutions inwhich the size of
etc. For W10 emulsions, the principal emulsifying agents are macromolecules lie in colloidal dimension. These are called
I
400 G.R.B. PHYSICAL CHEMISTRY FOR COMPETITIONS

I
. macromolecular colloids and they are quite stable. For example: Micellisation with cationic te~'als are also known, e.g., in
Starch, cellulose, proteins and enzymes, etc. cetyltrimethyl ammonium chloride.;\1t may be diagramatically
(iii) Associated. colloids: Substances whose molecules represented as : .!

aggregate spontaneously in a given solvent to form particles ·of


colloidal dimensions are called associated colloids. These R
. I
substances which when dissolved in a medium at low R-N+-R
concentrations behave as normal, strong electrolytes, but at R
I
higher concentrations they exhibit colloidal properties due to the N-l:....R
formation of aggregated particles (also called associated 'R
. colloids). The molecules of soaps and detergents are usually
smaller than the colloidal particles. However, in concentrated R R
I I
solutions, these molecules associate and form aggregates of R-N+ N"t-R
colloidal size. These aggregates of soaps or detergent molecules / I
R R
are called micelles. The formation of micelles takes place only
R
above a particular concentration called critical micelle I
concentration (CMC). For soaps, the CMC is 10-4 to R-N, N"':-R
I R I
10.- 3 mol L-I. These colloids have both lyophobic and lyophilic R R
R-N+-R
parts. Micelles may contain the aggregateof(30-100)molecules I
or more. Soaps and detergents are strong electrolytes and when R
dissolved in water they furnish ions. Fig. 6.11
CI7H35COONa~CI7H35COO- +Na+
Soap
(iv) p-Dodecyl benzene sulphonate;

C12H,,~03 Na'
Surfactants are those substances which are preferentially
adsorbed at the interfaces like air-water, oil-water and
solid-water interfaces. Thus, it is the surfactant which is
responsible for micellisation and emulsification. Surfactants are
divided into three categories:
(i) Cationic surfactants: Such substances on ionisation
give a cation having hydrophobic and hydrophilic group, e.g.,
cetyl pyridinium chloride,
Fig. 6.10
C16H33-CN +CC
The negative ions aggregate~~to form a micelle of colloidal
size. The negative ion has a long hydrocarbon chain and a polar Cetyltrimethyl ammonium chloride, C I6 H 33 (CH 3 hN+Cr
group (-COO-) at one end. In micelle formation, the long
Octadecyl ammonium chloride, C 1s H 37 NH;CI-
. hydrocarbon chain (tail) which is insoluble in water is directed
towards the centre while the soluble polar head is on the surface Ionisation of these compounds may be represented as,
Water
in contact with water (Fig. 6.10). The charge on the micelle is C1s H37 NHiCI- ~ C1s H 37 NHi
responsible for the stability of this system. The cleansing action (Hydrophobic end) (Hydrophilic end)
of soap is due to these micelles. The grease stain is absorbed into (ii) Anionic surfactants: Such substances give anion
. the interior of the micelle and gets detached from the fabric. The which act as surfactant, e.g.,
dust particles sticking to the stain are also removed. This action is
S,!dium Pahnitate C1sH31 COONa
similar to emulsification of grease.
Sodium Oleate C 17 H35 COONa
Some other examples of micelle systems are as follows:
and salts of sulphonic acid having molecular formula
(i) Sodium lauryl sulphate CH3 -(CHz)ll S04 Na+
C n HZn + 1 S03M where~ M+ = Na +, K+, NH~, etc.
(ii) Sodium oleate C 17 H 3S COO- Na +
Water
(iii) Cetyltrimethyl ammonium bromide; C1s H 31 COONa ~ C1s H31 - COO- + Na+
CH3 (CH 2 )15 N+ (CH3 )3 Br (Hydrophobic (Hydrophilic
end) end)
THE COLLOIDAL STATE 401

(iii) Non-ionogenic surfadants: These surfactants do not


CMC '. ill Molar electrolytical
ionise or dissociate in aqueous medium, but these molecules also t;j:\ conductance
have hydrophobic and hydrophilic end. ~ ® Os~otic pressure
High molecular mass alcohol adds to several molecules of "-
® Surface tension
ethylene oxide to form hydroxy surfactant ~ @ Turbidity
E.,
(!(I)

C n H2n+ I OH+xCH 2 -CHz ----+


(1\"-
c..:::l
fI)
\'-..."..----®
.... (1\

"'-I
Cn H Zn + I (OCH zCH 2 )x OH
0(1)

~E
:::lQ)
_c
c·-
(1\(1)
>.0
(I)
'------v----~' '--v---'
(Hydrophobic end) (Hydrophilic end) G)
II:

0.01 0.02
Kraft point: The temperature above which a surfactant Concentration of sodium dodecyl sulphate
forms micelle is called kraft point. (mol dm-3)

Some Important Features of Micelle Fig. 6.12


1. At critical micelle concentration (CMC), surfactants begins
,~~1 GELS
to form spherical aggregates made of 30 to 100 ions with their
hydrocarbon tails in the interior of the aggregate and their When a colloidal solution is coagulated, a precipitate is usually
charged terminals exposed to water on the out side (see Fig. formed which mayor may not be gelatinous. Under certain set of
6.12). . Conditions, it is possible to optain the dispersed phase as more or
less rigid mass enclosing within it all of the liquid. The product in
2. Increase in temperature usually increases CMC.
this form is known as a gel and the process is called gelation. The
3. CMC also depends on chain length, greater is the chain gel is, thus, a colloidal semi-solid system rich in liquid phase. Gel
length of hydrocarbon tail, smaller is the CMC. Increase in the usually consists of two components; one is soli<;l, such as gelatin,
hydrophobic part of the surfactant molecules, favours the silicic acid, sodium oleate, etc., and the other is a liquid such as
formation of micelles. water.
4. CMC of ionic micelles decreases on addition of simple Gels may be elastic or non-elastic according to their
electrolyte like NaCl. Screening action of added ions reduces the mechanical properties. Elastic gels can be prepared by dissolving
repulsion between charged groups at the surface of micelles. the substance in warm water and cooling it till sets. Examples are
5. Below the Kraft temperature, solubility of surfactant is not agar-agar and gelatin gels. Non-elastic gels are prepared by
enough to form micelles. Kraft temperature increases with appropriate chemical action. The silica gel is formed by action of
increase in the number of carbon atoms. HCI with sodium silicate solution.

Number of carbon atoms in sodium 10 12 14


alkyl sulphates
Kraft temperature °C 8 16 30
6. Micellisation is found to be spontaneous; it is confirmed by
r;tegative value of llG in following equation:
llG = Mf - TllS ... (1) ",-+--".. Solid particles
Mf ve and TllS = + ve for micellisation
Standard enthalpy change of miceUisation may be calculated

,....,......,
as,
-72!!!!=--;;;c--+--- Entrapped liquid
Ml" = ll~ + TllSo
= -2. 303RT z d(log CMe)
dT .
Fig. 6.13 I·

PhYSical Properties of Critical Micelle Concentration Gel may shrink on keeping by loosing some of the liqu~held
Physico-chemical properties of surfactant solution changes
sharply in the region of the C.M.C. Variation of physico-chemical
by them. This is known as syneresis or weepirtg geL Some gels of
liquify on shaking and reset on being allowed to stand.This~
properties of surfactant solution with concentration is reversible Sol-Gel transformation is known as thixotropy/ The
represented in following figure: softening behaviour of a marshy land under trading is du.e; to
thixotropic behaviour of bent0nite clay present in such soils.
402 G.R.B. PHYSICAL CHEMISTRY FOR COMPETITIONS

Elastic g!,)l can imbibe water when placed in it and undergo· precipitated. The particles thus, settle uown on the floor ofthe
swelling, non-elastic gels are incapable of doing so. This chamber. The precipitator is called cottrell precipitator.
phenomenon is known as imbibition or swelling.
Dust free gases
~. . APPLICATIONS OF COLLOIDS ... II ~.:..

Most of the substances, we come across in our daily life, are Chimney
colloids. The meal we eat, the clothes we wear, the wooden
furniture we use, the houses we live in, the newspapers we read,
are largely composed of colloids.
The applications of colloid chemistry are limitless. These can
be divided mainly into two classes:
.I. Natural applications
2. Technical applications
1, Natural Applications
(i) Blue colour of the sky: Colloidal particles scatter blue
light. Dust particles along with water suspended in air scatter Fig. 6.14.
blue light which reaches our eyes and the sky looks blue to us.
(ti) Purification of drinking water: The water obtained
(ii) Fog, mist and rain: When a large m~s of air,
from natural sources often contains bacteria and suspended
containing dust particles, is cooled below its dewpoint, the
impurities. Alum is added to such water so as to destroy the
moisture from the air condenses on the surfaces of these particles
bacteria as well as to coagulate the suspended impurities and .
forming fine droplets. These droplets being colloidal in nature
make water fit for drinking purposes.
continue to float in the air in the form of mist or fog.
(iii) Medicines: Most of the medicines in use at;,6colloidal
Clouds are aerosols having small droplets of water suspended
in nature. For example, argyrol is a silver sol used as an eye
in air. On account of condensation in the upper atmosphere; the
colloidal droplets of water grow bigger and bigger in size, till lotion. Colloidal antimony is used in curing kala-azar. Colloidal
gold is used for intramuscular injection. Milk of magnesia (an
they come down in the form of rain. Sometimes, the rainfall
emulsion) is used for stomach disorders. Colloidal medicines are
occurs when two oppositively charged clouds meet.
more effective because these are easily assimilated.
It is possible to cause artificial rain by throwing electrified
sand or spraying a sol carrying charge opposite to the one on (Iv) Tanning: Animal hides are colloidal in nature. When a
hide, which has positively charged particles, is soaked in tannin,
clouds from an aeroplane.
which contains negatively charged colloidal particles, mutual .
(iU)Food articles: . Milk, butter, halwa, ice creams, fruit
coagulation takes place, which results in the hardening ofleather.
. juices, etc., are all colloids in. one form or the other.
The process is termed as tanning. Chromium salts have also been
(Iv) Blood; Blood is a colloidal solution of an albuminoid·
used in place of tannin.
substance. The styptic action of alum and ferric chloride solution
(v) Cleansing action of soaps and detergents: Already
is due t? coagulation of blood forming a clot which stops further
described in section 6.8.
bleeding. .
(vi) Photographic plates and films: The photographic
(v) sons~ Fertile~ soils are colloidal in nature· in which
plates or films are prepared by coating an emulsion of the light
humus acts as a protlMive colloid. On account of colloidal
sensitive silver bromide in gelatin over glass plates or celluloid
nature, soils absorb moisture and nourishing materials.
films.
(vi) FfJrDlation ~f delta~ River water is colloidal solution
(vii) Rubber industry: Latex is a colloidal solution of
of clay. Seawater contains a number of electrolytes. When river
rubber particles which are negatively charged. Rubber is
water meets the seawater,. the. electrolytes present in seawater
obtained by coagulation of latex.
coagulate the ~olloidal solution of clay which gets deposited with
the fOrpJ.ation of delta. (viii) Industrial products: Paints, inks, synthetic plastics,
rubber, graphite lubricants, cement, etc., are all colloidal
2. Technical Applications solutions. Asphalt emulsion is used in road construction.
.(i) Electrical precipitation of smoke: Smoke is a colloidal (ix) Disinfectants: The disinfectants such as dettol and
·solution of solid particles such as carbon, arsenic compounds, lysol give emulsion of oil in water type when mixed with water.
dust, etc., in air. The smoke, before it comes out from the . (x) In ~etallurgy: Su.1phideores are concentrated by
chimney, is led through a chamber containing. plates having a froth-flotation process. In this process; pulverised ore is treated
charge opposite to that carried by smoke particles. The particles with emulsion of pine oil.
·on coming in contact with these plates lose their charge and get
.'
THE COLLOIDAL STATE 403

SUMMARY AND IMPORTANT POINTS TO REMEMBER


"\.. ...' .", •• '''tII!; . __ "\.....' 'bI,•••"tII!;
. 1. Crystalloids and colloids: Thomas Graham classified (U) Condensation methods: By aggregating very small
the soluble substances into two categories; crystalloids and particles into colloidal particles. These include (a) exchange of
colloids. Crystalloids are the substances which in solution diffuse solvents (b) change of physical state and (c) chemical methods
readily through .animal or vegetable membranes, e. g. ,urea, sugar, such as double decomposition, oxidation, reduction, hydrolysis,
salt and other crystalline compounds. Colloids (Greek word, etc.
Kolla, meaning glue-like) are the substances which in solution S. Purification of colloidal solutions: The process of
diffuse very slowly or do not diffuse through animal or vegetable reducing the impurities (electrolytes or soluble substances) to a
membranes, e.g., gelatin, glue, silicic acid, etc. It was soon requisite minimum in a colloidal solution is known as
realised that many of the crystalloids can be converted into puri,fication of colloidal solution.
oolloidalform by suitable means. Thus, colloid is a state of matter
Dialysis: It is a process of removing a dissolved substance
like solid, liquid or gas in which any substance can be brought by
from a colloidal solution by means of diffusioh through suitable
suitable means.
membrane. Animal membranes, parchment paper or ceUoph!ll1e
2.CoUoidal state: It depends upon the particle size. It is sheet can be used for this purpose. The electrolytes can be readily
regarded as an intermediate state between true solution and removed by electrodialysis.
suspension. Colloidal state is a heterogeneous system in which
solute particles of size ranging between 10 A to 103 A (l 0-7 to . 6. Properties of colloidal solutions: ColloidaI solutions
possess the following properties:
10-5 cm) are dispersed into a solvent (dispersion medium).
(i) Heterogeneous character: Consist of two phases.
3. Types of colloidal solutions: A colloidal solution
(Ii) Visibility: Particles are not visible to naked eye or with
consists two types, viz~ dispersed or internal or discontinuous
the help of microscope.
phase and the dispersion or external or continuous phase.
Depending on these phases eight types of colloidal solutions are (iii) Flltrability: Particles pass through ordinary filter
possible. The important ones are: paper but not through parchment al!d other fine membranes.
(i) Solid dispersed in liquid-It is called sol. (Iv) Surface tension and· viscosity: These are not very
different from those of dispersion medium in the case of
(ii) Liquid dispersed in liquid-It is called emulsion.
lyophobic sols. Lyophilic sols show higher viscosity and lower
(iii) Solid dispersed in gas-It is called aerosol. surface t e n s i o n . · .
(iv) Liquid dispersed in solid-It is called gel.
(v) Colligative properties: These properties are of small
Lyophobic: Colloidal solutions in which dispersed phase order as compared to true solutions at same concentrations. --
has very little affinity for the dispersion medium. These are less
(vi) Tyndall effect: It is the scatt(lring of light from the
stable and irreversible in nature. These are also called
surface of colloidal particles. A beam of light passed through a
suspensoids.
colloidal solution becomes visible as a bright streak. The
LyophiUc: Colloidal solutions in which dispersed phase has illuminated path is called Tyndall cone.
great affmity for dispersion medium. These are stable and
(vii) Brownian movement: It is a ceaseless zig-zag motion
reversible in nature. These are also called emulsoids.
of colloidal particles. Smaller the size and lesser the viscosity..
The .colloidal solutions are also named according to the
faster is the motion. This is due to. unbalanced bombardment of
dispersion phase or medium used. particles by the molecules of dispe11(lion medium. It is somewhat
Dispersion medium Name of colloidal solution responsible for stability of sols as the particles are not allowed to
Water Hydrosols settle.
Alcohol Alcosols (viii) Cbarge on colloidal partieles: Colloidal particles are
Benzene Benzosols electrically charged either carrying positive or negative charge.
Air Aerosols The origin of charge in most cases is due to preferential
4. Preparation of colloidal soluthms: To get a substance adsorption of ions on the surface. When an electric current is
in colloidal form either the substance is broken down into fine passed through the colloidal solution, the particles move towards
particles of colloidal dimension or increasing the s~e of a particular electrode. This is termed electrophoresis or
molecular particles as to form large aggregates. Thus, there are cataphoresis. When only the medium is allowed to migrate and
two ways by which lyophobic sols can be prepared. not the particles, the phenomenon is known as electroosmosis.
(i) Dispersion methods: By splitting coarse aggregates of (ix) -Coagulation: The phenomenon of change of colloidal
a substance into colloidal size. The dispersion methods include state to suspension state is known as coagulation or precipitation
( a) mechanical dispersion (b) electro-dispersion (c) ultrasonic or flocculation of colloidal solution. The coagulation is affected
dispersion and (d) peptization. Peptization is a process of either by boiling of colloidal solution or hy mutual action of
converting precipitates into colloidal state by adding small colloidal sols (by mixing oppositely charged colloidal solutions)
amount of suitable electrolyte. . or by electrophoresis or by the addition of electrolytes.
404 G.R.B. PHYSICAL CHEMISTRY FOR COMPETITIONS

The positive ion is effective in coagulating negative sol and Emulsifying agent: It is added to "H vII and water emulsion
vice-versa. The effectiveness of anion is governed by to increase its stability. It has polar and non-polar groups; the
Hardy-Schulze rule; The rule states that the precipitating power former attaches to water and the latter to oil and so binds them
of an ion depends upon its valency, i. e. ,higher the valency of the together in an emulsion. Soaps and detergents act as emulsifying
ion greater is the precipitating power of the ion. agents.
For negative sol, the power varies as M 3+ > M2+ > M+ and 8. Gel: It is a colloidal system in which liquid is dispersed
for positive sol the power varies as [Fe(CN)6 ]4- > SO~- > Cl-. in solid. Fruit-jellies, cheese, etc., are the examples of gels. Gels
may be elastic or non-elastic according to their mechanical
Flocculating value: It is the minimum concentration in properties.
millimoles per litre of an electrolyte required to cause
9. Applications of colloids: The applications can be
precipitation of a sol in 2 hours. The smaller is the flocculating
divided mainly into two classes:
power greater shall be the precipitating power of an ion.
(x) Protective action: Lyophilic sols are more stable than (i) Natural applications:
lyophobic sols. Thus, lyophilic colloids have the property of (a) Blue colour of the sky
protecting lyophobic sols. When a lyophilic sol is added to (b) fog, mist and rain
lyophobic sol, 'the lyophilic particles form a layer around
(c) Food articles
lyophobic particles and thus protect them from electrolytes. The
(d) Blood
protective power oflyophilic colloid is measured in terms of gold
number. It is defined as the number of milligrams of a lyophilic (e) Soils
colloid that will just prevent the precipitation of 10 mL of a gold (f) Formation of delta
solon the addition of 1 mL of 10% NaCI solution. Lower the gold (ii) Technical applications:
number, higher will be the protective power. Gelatin (Gold
Number 0.005,....().01) and starch (Gold Number 25-50) have the (a) Electrical precipitation of smoke
maximum and mitrimum protective powers. (b) Purification of drinking water
(xi) Coacervation: The loss of the stability ofa lyophilic sol, (c) Medicines
quite ofien,resuits in a separation of the system into two liquid (d) Tanning
phases. The separation of colloidal sol into two liquid phases is (e) Cleansing action of soaps and detergents
called coacervation. (f) Photographic plates and films
7. Emulsion: It is a colloidal solution ofa liquid in (g) Rubber industry
another immiscible liquid. Emulsions are usually of two types: (h) Paints, inks, plastics, lubricants, cement, etc.
(a) Oil in water-Examples: Milk, cream, face cream, etc.
(b) Water in oil-Examples: Bl,ltter, cold cream, etc.
THE COLLOIDAL STATE 405

1. Match the following:


[A] (c) Butter (r) Aerosol
(i) Gold number (a) Coagulation (d) Soap sud (s) Gas as dispersion medium
(ii) Lyophobic (b) An emulsion [B] Match the terms of List-I with those of List-II:
(iii) Butter (c) Gold sol List-I List-D
(iv) Hardy-Schulze rule (d) Gel
(a) Coagulation (P) Scattering of light
(v) Micelles (e) Purification of colloidal
solution (b) Peptization (q) Purification of colloidal
solution
(vi) Purple of cassius (f) Protective colloids
(c) Tyndall effect (r) Addition of electrolyte
(vii) Cheese (g) Solvent hating
(viii) Dialysis (h) Associated colloids (d) Dialysis (s) Precipitation of
colloidal solution
[B] [C] Match the Column-I with Column-II and Column-III:
(i) Brownian movement (a) Aerosol Column-I Column-D Column-Ill
(ii) Water loving colloids (b) Ultramicroscope (Conoidal (Dispersed (Dispersion
(iii) Liquid dispersed in gas (c) Irreversible solution) phase) medium)
(iv) Tyndall effect (d) Sewage disposal (a) Colloidion (P) Water (u) Ethanol
(v) Hydrophobic (e) Smoke precipitator (b) Fog (q) Cellulose (v) Oil
(vi) Coagulation (f) Hydrophilic (c) Bqtter (r) Fat (w) Air
(vii) Electrophoresis (g) Emulsifying agent (d) Milk (s) Water (x) Water
(viii) Soap (h) Robert Brown [0] Match the terms in Column-l with their explanations in
Column-II:
Application I
[q Property Statement
discoverer
Column-I Column';'D
(a) Cottrell precipitation (P) Purification of blood
(i) Tyndall effect (a) Due to unequal 1. Sewage disposal
(b) Electrophoresis (q) Precipitation of
bombardment by .
colloidal particles by
solvent
addition ofelectrolytes
molecules
(c) Hemidialysis (r) Removal of pollutants
(ii) Coagulation (b) Surface area is 2. Smoke from industrial waste
large precipitator gases
(iii) . Brownian (c) Movement of 3. Ultramicroscope (d) Coagulation (s) Movement of charged
movement colloidal colloidal particles
particles under towards oppositely
the influence of charged electrode
electric field [E] Match the terms in Column-I with those of Column-II:
Column-I Column-D
(iv) Adsorption (d) Due 4. Ice cream
to neutralisation (a) Emulsifier (P) Colloidal sol. of
of charge graphite
(v) Electrophoresis (e) Due to scattering 5. Colloidal (b) Colloidal electrolyte (q) Detergent
of light medicines (c) Oil dag (r) Cellophane
(vi) Stability (f) Addition of 6. Robert Brown (d) Xerogel (s) Dextriu
protective colloid [F] Match the Column-I with Column-II:
2. Matrix Matching Problems (For IJT Aspirants): Column-I Column-D
[A] Match the colloids in Column-I with the classifications (a) Milk (p) Aerosol
in Column-II: (b) Dust (q) Emulsion
Column-I Column-D (c) Cheese (r) Gel
(a) Rain cloud (P) Gel (d) Froth (s) Foam

(b) Smoke (q) Foam


406 G.R.B. PHYSICAl. CHEMISTRY FOR COMPETITIONS

3. The coagulation of 100 mL of a colloidal solution of gold is 4. For the coagulation of 100 mL of arsenious sulphide solution,
completely prevented by adding 0.25 g of starch to it before 5 mL of I MNaCI is required. What is the coagulating power
adding 10.mL of 10% NaCI solution. Find out the gold number of NaCl?
of starch. [Hint: Total volume after addition of 5 mL of I M NaCI
(Hint=.lOmL of 10% NaCI solution is added to 100 mL of solution 105 mL
solution of gold. Thus, 105 x molarity of NaCI in colloidal solution 5 xl
Thus, I mL of 10% NaCl solution has been added to 10 mL
solution of gold. Molarity of NaCI in colloidal solution = 2..
105
Since, 100 mL gold solution required = 0.25 g starch
= 0.25 x 103 mg starch Concentration in millimole = ~ x 1000
105
3
So, 10 mL gold solution required = 0.25 x 10 X 10 47.6]
100
25 mg starch
Thus, by definition, the gold number of starch is 25.]

1. [A] (i-~ (ii-g); (iii-b); (iv-a); (v-:-h); (vi~); (vii-d); 2. [A] (a-r, s) (b-r, s) (c-p) (d-q)
(viii-e) [B] (a-r, s) (b-r) (c-p) (d-q)
[B] (i-b); (ii---'-f); (iii-a); (iv-b);(v~); (vi-d); (vii-e); [C] (a-q-u) (b-p-w) (c-p-v) (d-r-x)
(viii-g) [D] (a-r, s) (b-s) (c-p) (d-q) .
[C] (i--e-3); (ii-d-l); (iii-a--6); (iv-b-5); [E] (a-q, s) (b-q) (c-'-p) (d-r)
(v~-2); (vi-f- 4). [F] (a-q) (b-p) (c~r) (d-s)

....!£:::OBlECTIVEQUESTIONS
Set-1: Questions with Single correct answer a
7. Which of the following forms colloidal solution in water?
1. Difference between crystalloid and colloid is of: (CP\1T 1990)
(a) particle size (b) chemical composition (a) NaCI (b) Glucose (c) Ba(N03h (d) Starch
(c) ionic character (d) solubility 8.. The number of phases present in colloidal solution is:
2. Substances whose solutions can readily diffuse through (a) 2 (b) 4 (c) 3 (d) 1
animal membranes are called: 9. The colloidal system of a solid dispersed in liquid medium is
(a) colloids (b) crystalloids called: [CET (J&K) 26071
(c) electrolytes (d) non-electrolytes
(a) aerosol (b) sol (c) gel (d) foam
10. When dispersed phase is liquid and dispersion medium is gas,
3. Suspensions are:
the colloidal system is called:
(a) visible to naked eye
(b) invisible through microscop~' (a) smoke (b) emulsion (c) cloud (d) gel
(c) not VIsible by any means 11. Water loving colloids are called as:
(d) invisible under electron microscope (a) hydrophilic (b) hydrophobic
4. The size of the colloidal particles is in between: (CPl\IT 1990) (c) lyophobic (d) irreversible
(a) 10-7 _10-9 em (b) 10-9 -10- 11 cm 12. An emulsion is a colloidal solution consisting of:
(e) 10-5 10-7 em (d) 10-2 _10-3 em (a) two solids (b) two liquids
(c) two gases (d) one solid and one liquid
5. The size of it colloidal particle is: JP1\fT (1\11') 19901
13. The colloidal solution of gelatin is known as:
(a) > 0.1 ~ (b) 1 IIlt to 0.1 ~ (a) solvent loving (b) reversible
(c) <O.lml-l (d) morethan30ooIIlt
(c) hydrophilic (d) all of these
6. Ifliquid,is dispersed in solid medium, then this is called:
14. Sol is a type of colloid in which: (Raj.) IOnSl
• (DCE 20(9)
(a) solid is dispersed in liquid
(a) sol (b) emulsion
(b) liquid is dispersed in solid
(c) liquid aerosol (d) gel
(c) gas is dispersed in liquid
(d) solid is dispersed in solid
THE COLLOIDAL STATE

15. Butter is a colloid formed when: (a) dialysis (b) peptization


(a) fat is dispersed in fat (c) ultrafiltration (d) electro-dispersion
(b) fat is dispersed in water 28. Which of the following substances gives a positively charged
(c) water is dispersed in fat sol? [PMT (MP) 19901
(d) suspension of casein in water (a) Gold (b) Arsenious sulphide
16. Milk is: [PMT (MP) 2007; e.G PET 2008J (c) Starch (d) Ferric hydroxide·
(a) fat dispersed in water (b) water dispersed in fat 29. The kinetic activity of colloidal particles in dispersion
(c) water dispersed in oil (d) fat dispersed in fat medium is known as:
17. Smoke is an example of: (CPMT 1991) (a) electro-osmosis (b) cataphoresis
(a) solid dispersed in solid (b) solid dispersed in ~as (c) Brownian movement (d) electrophoresis
(c) solid dispersed in liquid (d) gas dispersed in solid 30. Brownian movement was discovered by:
18. Fog is a colloidal solution of: (a) Robert Brown • (b) Zsigmondy
[PMT (MP) 1991; CET (J&K) 2005; DUMET .2009] (c) Hardy-Schulze (d) Graham
(a) gaseous particles dispersed in gas 31. The Brownian movement is due to:
(b) gaseous particles dispersed in liquids (a) temperature fluctuations within the liquid phase
(c) liquid dispersed in gas (b) attraction and repulsion between charges on colloidal
(d) solid dispersed in liquid particles .
19. Lyophobic colloids are: (c) impact of molecules of the dispersion medium on colloidal
(a) reversible (b) irreversible particles
(c) water loving (d) solvent loving (d) convection currents
20. Sulphur sol contains: 32. Tyndall phenomenon is shown by:
(a) discrete sulphur atoms (a) dilute solution (b) colloidal solution
(b) discrete sulphur molecules (c) suspension (d) true solution
(c) water dispersed in solid sulphur 33. The sky looks blue due to:
(d) large aggregates of sulphur molecules (a) dispersion effect (b) reflection
(c) transmission (d) scattering
21. Which ofthe following is not a colloidal system?
34. Tyndall effect in colloidal solution is due to: (MHT-CET 2007)
(a) Bread (b) Muddy water
(a) absorption of light
(c) Concrete (d) Sugar in water
(b) scattering oflight
22. Peptization denotes: (MHT-CET 2007)
(c) reflection oflight
(a) digestion of food
(d) presence of electrically charged particles
(b) hydrolysis of proteins
35. The migration of colloidal particles under the influence of an
(c) breaking and dispersion into colloidal state
electric field is known as:
(d) precipitation of solid from colloidal dimension
(a)electro-osrnosis (b) Br(}'wnian movement
23. Colloids are purified by: (CPMT 1990)
(c) cataphoresis (d) dialysis
(a) Brownian motion (b) precipitation
36. The stability of lyophilic colloids is due to: [PMT (MP) 1990]
(c) dialysis (d) filtration
(a) charge on their particles
24. The separation of colloidal particles from those of molecular
(b) large size of their particles
dimension is known as:
(c) smaller size of their particles
(a) dialysis (b) electrophoresis
(d) a layer of medium of dispersion on their particles
(c) peptization (d) pyrolysis
37. 'Greater the valency, the higher is the coagulating power of
25. Bredig's arc method is used for the preparation of colloidal
ion.' This rule was introduced by:
solution of:
(a) Hardy-Schulze (b) Graham
(a) metals like silver, gold, etc. (c) Kossel and Lewis (d) Faraday
(b) organic compounds 38. Bleeding is stopped by the application of ferric chloride. This
(c) two liquids is because:
(d) inorganic compounds (a) the blood starts flowing in opposite direction
26. Which of the following reactions is not used for the (b) the ferric chloride seals the blood vessel
p!:eparation of a colloidal solution? (c) the blood reacts and forms a sOlid which seals the blood
(a) 2AuCl 3 + 3SnC12 ----l> 3SnCl4 + 2Au vessel
(b) FeCl3 + 3H20 ----l> Fe(OHh + 3HCl (d) the blood is coagulated and thus, the blood vessel is sealed
(c) 2Mg + CO2 ----l> 2MgO + C 39. Which property of colloidal solution is used to determine the
(d) 2H2 S + S02 ----l> 2H20 + 3S nature of charge on the particles?
27. When freshly precipitated Fe(OHh is boiled with water in the (a) Sedimentation (b) Electrophoresis
presence of few drops of dil. HCI, a hydrated ferric oxide sol is . (c) Dialysis· (d) Ultrafiltration
obtained. This method is termed:
408 G.R.B. PHYSICAL CHEMISTRY FOFlCOMPETITIONS

40. When excess of electrolyte is added to a colloid it: 51. Which of the following is not a property of hydrophilic sols?
(CBSE 1990) (a) High concentration of dispersed phase can be easily
(a) coagulates (b) g~ diluted attained
(c) precipitates (d) does not change (b) Coagulation is reversible
41. The colloidal solutions of gold prepared by different methods (c) Viscosity and surface tension are nearly as that of water
have different colours. This is due to: (MLNR 1993) (d) The charge on the particles depends on the pH value of the
(a) difference in size of colloidal particles medium; it may be positive, negative or even zero
(b) different concentration of gold 52. Gelatin is added in manufacture of ice cream in order to:
(c) presence of different types of foreign particles (a) prevent formation ofa colloid
(d) the variable valency of gold (b) stabilise the colloid and prevent crystallisation
42. The capacity of an ion to coagulate a colloidal solution (c) cause the mixture to solidify easily
depends on: . (d) improve flavour
(a) its shape 53. Which one of the following will act as best protective colloid?
~ (b) the amount of its charge (a) Gelatin (Gold No. 0.005)
(c) the sign of the charge (b) Starch (Gold No. 25)
(d) both, the amount and the sign o( the charge (c) Gum arabic (Gold No. 0.15)
43. Lyophilic sols are more stable than lyophobic sols because:
. (AFMC 1998) (d) Egg albumin (Gold No. 0.08).
54. Gold number is a measure of the:
(a) the colloidal particles have positive charge
(a) protective action by a lyophilic colloid on lyophobic
(b) the colloidal particles have negative charge
colloid
(c) the colloidal particles are solvated
(b) protective action by a lyophobic colloid on lyophilic
(d) there are strong electrostatic repulsions colloid
44. Which of the following will have the highest coagulating (c) number of mg of gold in a standard red gold sol
power for As2S 3 colloid?
(d) none ofthe above .
. (a) PO~- . (b) A1 3 + (c) S~- (d) Na +
55. On addition of one mL solution of 10% NaCI to 10 niL gold
45. A negatively charged suspension of clay in water ·will need for . sol in presence of 0.025 g of starch, the ,coagulation is just
precipitation the minimum amount of: prevented. The gold number of starch is:
(a) aluminium chloride ".. (b) potassium sulphate (a) 25.0 . (b) 2.5 (c) 0.25 (d) 0.025
(c) sodium hydroxide (d) hydrochloric acid· 56. The stability oflyophobic sols is due to:
46. Which of the following electrolytes is least effective in (a) Brownian motion only
causing flocculation offerric hydroxide sol? (MLNR 1991) . (b,1 electric chjlrge'oniy
(a) K3Fe(CN)6 (b) K 2 Cr04 , (eY b~tb BrowniaIl ffiQtf<)n and electric charge
(c) KEr (d) K 2S04 (d) particle size
47. Fe3+ ions coagulate blood. This show blood contains colloidal 57. Which one of the fl8tJQwing colloidal solutions is positive sol.?
particles bearing: . .. (CPMT 1992)
(a) negative charge (a) Blood
(b) positive charge (b) Clay soil
(c) no charge (c) Smoke
(d) either positive or negative charge . (d) Gelatin in strongly acidic solution
48. Gold number is a measure of: (CPMT 1991) 58. All colloidal solutions show:
(a) stability of colloidal system (a) very high osmotic pressure
. (b) coagulating power of a colloid (b) high osmotic pressure
(c) size of colloidal particles (c) low osmotic pressure
(d) efficiency of the protective colloid (d) no osmotic pressure
49: Gold number is a measure of: [PET (MP) 2004) 59. Which of the following is associated colloid?
(a) the amount of gold present in the colloidal solution (a) Soap (b) Detergent
(b) the amount of gold required to break the colloid . (c) Both (d) None of these
(c) the amount of gold required to protect the colloid 60. Which of the following is an emulsifier?
(d) none ofthe above (a) Oil (b) Soap (c) Solvent (d) KCI
so. Which of the following methods is used for destruction ofa 61. An emulsifier is a substance which:
colloidal solution? (a) helps in the dispersion ofIiquid in liquid
(a) Condensation (b) stabilises the emulsion
(b) Dialysis ( c) coagulates the emulsion .
(c) Diffusion through animal membrane (d) purifies the emulsion
(d) Addition of an electrolyte
THE COLLOIDAL STATE

62. The gold numbers of some colloidal solutions are given (a) flocculation value (b) gold number
below: (c) protective value (d) coagulation number
Colloidal solution Gold number 76. Purple of cassius is:
A 0.01 (a) colloidal solution of silver
B 2.5 (b) colloidal solution of gold
C 20 (c) colloidal solution of platinum
The projective powers of these colloidal solutions follow the (d) oxy acids of gold
order: [CET (J&K) 2006; AIEEE 2008] 77. Which type of molecules form micelles?
(a) C> B > A (b) A > B > C (a) Polar molecules (b) Non-polar molecules
(c) A B C (d) B > A > C (c) Surfactant molecules (d) Any of these
63. Dialyser is a name given to: 78. The name aquadag is given to the colloidal solution of:
(a) lyophilic colloids (a) copper in water (b) platinum in water
(b) lyophobic colloids (c) gold in water (d) graphite in water
(c) to a membrane which can separate colloids from the solution 79'1 A li4ui~ is found to scatter a beam of light but leaves no
(d) none of the, above ! .. resi4udvhen passed through the filter paper. The liquid can be
64. Floccull"/-tion value is expressed in terms of: describ,d as: ! I· • (AIIMS 1993)
(a) millimole per litre (b) mol per litre (a) a suspension i : (b) oil'
(c) gram per litre (d) mol per millilitre (c) a colloidal sol. (d) true solution
65. Which of the following has minimum flocculation value? . 80. Point out the false statement: [CET (Punjab) 1991]
[PET (MP) 1990] (a) colloidal sols, are homogeneous
(a) Pb 2 + (b) Pb 4+ (c) Sr 2 + (d) Na+ (b) colloidal sols. carry +ve or -ve charge
66. The charge of As2 S3 sol is due to the absorbed: '(c) colloidal sols. show Tyndall effect
(a) It . (b) OIr (c) 0 2 - (d) S2- (d) the size of colloidal particles ranges between 10--2000 A
67. Which oftl;le following has maximum flocculation value? 81. The potential difference between the fixed charged layer and
(a} [Fe(CN)6t-' (b) CI- the diffused layer having opposite charge is called:
(a) colloidal potential (b) zeta potential
(c) SO~- (d) PO~-
(c) electrostatic potential (d) none of these
68. Above critical micelle c·oncentration, particles get: 82. An example ofmicelle is:
[AMU (Medical) 2006]
(a) As20 3 sol.
(a) associated (b) dissociated (b) ruby glass
(c) both (a) and (b) (d) none of these . (c) Na 2 C03 solution
69. Continuous phase contains dispersed phase throughout.
(d) sodium stearate concentrated solution
Example is: (VJTEEE 2008)
83. Surface tension oflyophilic sols. is: [PMT (MP) 1992]
(a) water in milk (b) fat in milk
(a) Ic.werthan H2 0 (b) more than H2 0
(c) water droplets in mist (d) oil in water.
'(c) equalto H2 0 (d) none of these
70. The movement of dispersion medium in an electric field when
the dispersed particles are prevented from moving is called:
is
84.' Which not shown by sols.? [PMT (MP) 1992]
(a) Adsorption (b) Tyndall effect
(a) cataphoresis. (b) electrophoresis
(c) Flocculation (d) Paramagnetism
(c) electro-osmosis (d) Brownian movement
85. An example of solid-solid system is:
71. To coagulate gelatin sol, which of the following is most
(a) smoke (b) coke
effective?
(c) synthetic gems (d) pumice stone
(a) NaCl (b) Na3P04 (c) AlCI 3 (d) Alcohol
72. The emulsifying agent in milk is: [EAMCET (Med.) 2010J 86. Detergent action of synthetic detergents is due to their:
( a) lactic. acid (b) fat
(a) interfacial area (b) high molecular weight
(c) ionisation (d) emulsifying properties
(c) lactose (d) casein
87. Blood contains:
73. Colloidal solutions of metals like Cu, Ag, Au and Pt are
generally prepared by using: (a) positively charged particles
(a) peptization (b) Bredig's arc method (b) negatively charged particles
(c) exchange of solvent (d) oxidation method (c) neutral particles
74. Silver iodide is used for producing artificial rain because AgI: (d) negatively as well as positively charged colloids
(a) has crystal structure similar to ice 88. Acolloidal solution always has at least: (CPMT 1993)
(b) is easy to spray at high altitudes (a) one-phase (b) more than two-phases
(c) is easy to synthesise (c) a true solution (d) two phases
(d) is soluble in water 89. Silica gel is commonly used as:
7S. The minimum concentration of an electrolyte required to (a) wetting agent (b) drying agent
cause coagulation of a sol. is called: (c) solvent (d) catalyst
410 G.R.B. PHYSiCAL CHEMISTRYFOR~COMPEtrTlONS

90. Which is not a colloidal solution of gas in liquid? (c) concentration of electrolyte added to destroy the micelles
(a) Froth (b) Foams with tiny bubbles (d) concentration of micelles at room temperature
(c) Mist (d) Whipped cream 102. Cod liver oil is: (CPMT 1997)
91. Fog is a colloidal solution of: (a) fat dispersed in water (b) water dispersed in fat
(MLNR 1995; PMT (MP) 1991] (c) water dispersed in oil (d) fat dispersed in fat
(a) liquid particles dispersed in gas 103. Which is not lyophilic colloid? (PMT (MP) 1998}
(b) gaseous particles dispersed in liquid (a) Milk (b) Gum (c) Fog (d) Blood
(c) solid particles dispersed in liquid 104. At the critical micelle concentration, the surfactant molecules:
(d) solid particles dispersed in gas (CBSE 1998)
92. Emulsions of polyvinyl acetate are used in: (a) decompose (b) dissociate
(a) polishes (b) latex paints (c) associate (d) become completely soluble
(c) fireworks (d) rayons lOS. Which of the following ions can cause coagulation of
93. When white light is passed through a colloidal solution proteins? (KCET2000)
containing fine suspended particles of gold, then the scattering - (a) Ag+ (b) Na + (c) Mg2+ (d) Ca z+
light seen in a direction different from that of incident light is:
106. In Brownian movement or motion the paths of the particles
(a) yellow coloured (b) blue coloured are: (KMEE 2000)
(c) green coloured (d) red coloured (a) linear (b) zig-zag
94. When a sulphur sol is evaporated, solid sulphur is left. On (c) uncertain (d) curved
mixing with water no colloidal sol is formed. The sulphur sol
107. Which is used for ending charge on colloidal solutions?
is:
(CBSE 2000)
(a) hydrophilic (b) hydrophobic
(a) Electrons (b) Electrolysis
(c) reversible (d) lyophilic
(c) Positively charged ions (d) Compounds
9S. Tails of comets are visible due to:
108. Cloud or fog is a colloidal system in which the dispersed phase
(a) Tyndall effect (b) reflection
and the dispersion medium are: (KCET 2000)
(c) Brownian movement (d) none of these
(a) gas, liquid (b) liquid, gas
96. Milk is an example of:
(c) liquid, liquid (d) solid, solid
[CPMT 1995; PET (MP) 2004; CET (J&K) 2004]
109. The electrolyte which has the least effect in the coagulation of
(a) fat dispersed in water Fe(OH)3 sol is: [KCET 2008}
(b) water dispersed in fat (a) potassium carbonate (b) sodium sulphate
(c) water dispersed in oil (c) potassium ferrocyanide (d) potassium iodide
(d) fat dispersed in fat 110. Blood may be purified by: [PMT (MP) 20001
97. Smog is an example of: (a) dialysis (b) electro-osmosis
(a) .ice dispersed in air (c) coagulation (d) filtration
(b) water dispersed in air 111. Gold number was given by:
(c) smoke dispersed in air (a) _Ostwald (b) Zsigmondy
(d) smoke and water dispersed in air (c) William and Chang (d) Langmuir
98. Peptization denotes: [ISM (Dhanbad) 19941 112. The diameter of colloidal particle ranges from: (KCET 2004)
(a) digestion offood
(a) 10-9 m to 10-6 m (b) 10-9 m to 10- 12 m
(b) hydrolysis of proteins
(c) breaking of dispersion into colloidal state (c) 103 m to 10-3 m (d) 1O-3 6
m to 10- m
(d) precipitation of a solid from colloidal state 113. The Tyndall effect is not observed in:
99. Whipped cream is an example of: (a) suspensions (b) emulsions
Dispersion Medium Dispersed Phase (c) colloidal solutions (d) true solutions
(a) Liquid Gas 114. Which one is example of Micelle's system?
(b) Liquid Liquid (a) Soap + water (b) Rubber + benzene
(c) Liquid Solid (c) Protein + water (d) None of these
(d) Gas . Liquid 115. Sedimentation potential is the reverse of:
100. Cottrell precipitator acts on which ofthe following principle? (a) electro-osmosis (b) electrophoresis
(a) Hardy-Schulze rule (c) electrokinetic potential (d) Dom potential
(b) Distribution law 116. Smoke has generally blue tinge. It is due to:
(c) Le Chatelier's principle (a) scattering (b) coagulation
(d) Neutralization of charge on the colloidal particles (c) Brownian motion (d) electrophoresis
101. CMC (critical micelle concentration) is: 117. On adding AgN03 solution to KI solution, a negatively
(a) concentration at which micell~s are destroyed charged colloidal sol will be formed in which ofthe following
(b) concentration at which micelle formation starts conditions?
THE COLLOIDAL STATE

,(a) 100 mL ofO.! M AgN0 3 +100 mL ofO.! M KI 130. Potassium stearate is obtained by the saponification of an oil
(b) 100 mLofO.l MAgN03 + 50 mLofO.2 M KI or a fat. It has formula CH3 - (CH2)16 - COo- K+. The
(c) 100 mLofO.2 M AgN03 + 100mLofO.l MKI molecule has a lyophobic end (CH 3- ) and a lypophilic end
(d) 100 mLofO.l M AgN03 + 100 mLofO.l5 M KI COo- K+. Potassium stearate is an example for:
118. Peptization ofSnOz by NaOH gives: [PET (Kerala) 2005]
(a) [SnOz]SnO;- :2Na+ (b) [SnOz ]Sn 4 + :02- (a) lyophobic colloid
(b) lyophilic colloid
(c) [Sn02]Na+ : OIr (d) [Sn02 ]Sn 4 + : OIr
(c) multimolecular colloid
119. Alum helps in purifying water by: (AlEEE 2002)
(d) macromolecular colloid
(a) forming Si complex with clay particles
(e) associated colloid or micelle
(b) sulphate part whlch combines with the dirt and remove it
(c) aluminium which coagulates the mud particles 131. Which one of the following forms micelles in aqueous
solution above certain concentration? [CBSE (PMT) 2005)
(d) making the mud water soluble
(a) Dodecyl trimethyl ammonium chloride
120. Surface tension of lyophilic sols is: [PMT (MP) 20021
(b) Glucose
(a) lower than that of H 20 (b) equal to that of H20 (c) Urea
(e) more than that of H20 (d) none of these (d) Pyridinium chloride
121. Arsenic sulphide is negative sol. The reagent with least 132. Muddy water can be purified through coagulation using:
precipitating power is: [PMT (ManipaJ) 2002J [CET (J&K) 2005]
(a) AICl 3 (b) NaCI (c) CaF2 (d) glucose (a) common salt (b) alums
122. Which one of the following is correctly matched? (c) sand (d) lime
[CEE (Tamil Nadu) 2002) 133. The disperse phase in colloidal iron(III) hydroxide and
(a) Emulsion-curd (b) Foam-mist colloidal gold is positively and negatively charged
(c) Aerosol-smoke (d) Solid sol-cake respectively. Which of the following is not correct?
(AIEEK 2005)
123. When HzS gas is passed through nitric acid, the product is:
[CEE (Kerala) 20021 (a) Magnesium chloride solution coagulates the gold sol more
readily than iron(III) hydroxide sol
(a) rhombic sulphur (b) prismatic sulphur
(b) Sodium sulphate solution causes coagulation in both sols
(c) amorphous sulphur (d) monoclinic sulphur
(c) Mixing of the sols has no effect
(e) plastic sulphur
(d) Coagulation in both sols can be brought about by
124. Tyndall effect is shown by: [CECE (Bihar) Pre 2004]
electrophoresis
(a) precipitate (b) sol
134. An emulsifier is a substance which: (AIEEE 2005)
(c) plasma (d) solution
(a) stabilises the emulsion
125. On addition of one mL of 10% NaCl solution to 10 mL gold
(b) homogenises the emulsion
sol in presence of 0.25 gm of starch, the coagulation is just
prevented, starch has gold number: fPMT (MP) 2004) ( c) coagulates the emulsion
(a) 0.25 (b) 0.025 (c) 2.5 (d) none of these (d) accelerates the dispersion ofliquid in liquid
126. Which of the following forms cationic micelles above certain 135. Gold number is associated with: [BHU (Mains) 2007;
concentration? [CBSE (PMT) 20041 AMU (Engg.) 2010J
(a) Sodium dodecyl sulphate (a) electrophoresis
(b) Sodium acetate (b) purple of cassius
(c) Urea (c) protective colloid
(d) Cetyltrimethyl ammonium bromide (d) amount of pure gold
127. The smog is essentially caused by the presence of: 136. Which one of the following is a false statement?
(AIEEE 2004) iPMT (Kerala) 2007)
(a) 02 and 0 3 (a) Cell fluid is an example of sol
(b) 02 and N2 (b) Butter is an example of gel
(c) oxides of sulphur and nitrogen ( c) Hair cream is an example of emulsion
(d) 0 3 and N z (d) Whipped cream is an example of foam
128. Which one of the following is most effective in causing the (e) Cheese is an example of emulsion
coagulation of an AS zS3 sol? (EAMCET 2009) 137. The presence, of electric charge on colloidal particles is
(a) KCI (b) AIC1 3 indicated by the property, called: ICET (J&K) 2007]
(a) dialysis (b) solubility .
(c) MgS04 (d) K3[Fe(CN)61
129. The fresh precipitate can be transformed in colloidal solution (c) electrophoresis (d) osmosis
by: JCET (J&K) 2004) 138. Which of the following properties are characteristic of
(a) peptization (b) coagulation lyophobic sols? (SCRA 2007)
(c) diffusion (d) none of these 1. Low viscosity, 2. High viscosity, 3. Reversibility and
4. Coagulation by electrolytes at low concentration
412 G.R.B. PHYSICAL CHEMISTRY FOR COMPETITIONS

Select the correct answer using the codes given below:


Set·2: The questions given below may have more
(a) 2, 3 and 4 (b) 2 and 3 only
than one correct answers
(c) I and 4 only (d) 1 and 3 only
139. In an electrical field, the particles of a colloidal system move 1. Lysione is not used as:
towards cathode. The coagulation of the same sol is studied (a) disinfectant (b) germ killer .
using K zS04(I), Na3P04(1I), K 4 [Fe(CN)6] (III) and NaCI (c) treating eye disease (d) anti-cancer drug
(IV). Their coagulating power should be : 2. Which ofthefollowing are macromolecular colloids?
(PMT(Kerala) 2008] (a) Starch (b) Soap (c) Detergent (d) Cellulose
(a) (I) > (II) > (III);" (IV) (b) (III) > (II) > (I) > (IV) 3. Multimolecular colloids are present in:
( c) (III) > (I) > (II) > (IV) (d) (IV) > (III) > (I) > (II) (a) sol of sulphur (b) sol of proteins
(e) (IV) > (I) > (II) > (III) (c) sol of gold (d) soap solution
140. Cetyl trimethyl ammonium chloride is which type of 4. Methods used for the preparation of colloidal solutions are:
detergent? (CET (Gujarat) 2008) (a) peptization (b) hydrolysis
(a) Cationic (b) Anionic (c) Biosoft (d) Non-ionic . (c) ultrasonic dispersion (d) coagulation
141. The effective ion used in clarification of water is: 5. Isoelectric point is the pH at which colloidal particles:
(KCET2008) (a) coagulate
(a) A1 3+ (b) Ca 2 + (c) SO!- (d) PO!- (b) become electrically neutral
142. The number of moles of lead nitrate needed to coagulate (c) can move toward either electrode
2 mole of colloidal [Ag l]r is : (PET (Kerala) 2008] (d) none ofthe above
6. Consider the following statements for micelles, which is/are
(a) 2 ' (b) I (c).! (d) 2 (e) 5 correct? .
232
[Hint: 2 [Ag I] r + Pb2 + ~ PbI 2 + 2AgI (a) At critical micelle concentration, several properties of
solution of surfactants such as molar conductivity, surface
Thus, one mole of Pb(N03h is required to coagulate 2 mole of
[AgI] 1-.] tension and osmotic pressure change
143. Among the electrolytes Na2S04, CaCl z, AI 2 (S04)3 and (b) Micelles from ionic surfactants can be formed only above
NH4Cl, the most effectiye coagulating agent for Sb 2 S3 sol is: a certain temperature called the Kraft temperature
. (lIT 2009) ( c) Micelle formation is exothermic
(a) Na 2S04 (b) CaCl 2 (d) Micelles are associated colloids
(c) Al 2 (S04)3 (d)NH.tCI 7. Which of the following are negative colloids?
[Hint: Sb 2S:J sol is negative colloid, therefore, Al 2 (S04)3 will be (a) Fe(OHh sol (b) As 2 S J sol
most effective coagulating agent. . (c) Blood. (d) Gold sol
Al:l+> Ca2+> N.+> N[r~
8. Which of the following are examples of aerosols?
Decreasing coagulating power for negatively )] (a) Wbipped cream (b) Cloud
charged colloidal particles ofSbzS 3 ' ..
(c) Fog (d) Soap lather
144. A micelle formed during the cleansing action of soap is : 9. Tyndall effect is applicable when:
(EAMCET (Engg.) 2010) (a) the diameter of the dispersed particles is not much smaller
(a) a discrete particle of soap than the wavelength of the light used .
(b) aggregated particles of soap and dirt (b) the diameter of the dispersed particles is much smaller
(c) a discrete particle of dust . than the wavelength of the light used ..
(d) an aggregated particle of dust and water (c) the refractive indices of the dispersed phase and the
145. The dispersed phase and dispersion me9ium in soap lather are dispersion medium must be same
respectively: ,(PET (Kerala) 2010) (d) the refractive indices of the dispersed phase and the
(a) gas and liquid (b) liquid and gas dispersion medium must differ greatly in magnitude
(c) solid and gas (d) solid and liquid
THE COLLOIDAL STATE 413

Assertion-Reason TYPE QUESTIONS


The questions given below consist of two statements each printed as 6. (A) Colloidal sol. of Fe(OHh formed by peptization carries
'Assertion' (A) and 'Reason' (R). While answering these questions positive charge.
you are required to choose anyone of the following four options: (R) During the formation of positively charged colloidal
(a) If both (A) and (R) are correct and (R) is the correct particles of Fe(OHh, the electrons are lost by the colloidal
explanation for (A). particles of Fe(OHh-
(b) Ifboth (A) and (R) are correct but (R) is not the correct 7. (A) Colloidal solutions are purified by dialysis.
explanation for (A). (R) In the process of dialysis, colloidal particles pass through
(c) If(A) is correct but (R) is incorrect. parchment paper.
(d) If (A) is incorrect but (R) is correct. S. (A) Fe 3+ can be used for coagulation of AS2S3 sol.
(R) Fe 3+ reacts with As2S3 to give Fe2S3. (AIIMS 2006)
1. (A) Colloidal silver iodide is prepared by adding silver nitrate 9. (A) Fat is digested in the intestine by emulsification.
in slight excess to potassium iodide solution. When (R) Bile salts stabilize the einulsion so formed.
subjected to an electric field, the colloidal particles
10. (A) NH3Cl and RCOONa are colloidal electrolyte.
migrate to the anode.
(R) The substances which behave as electrolyte below a
(R) Colloidal particles absorb ions and thus become
certain concentration limit, beyond this limit colloidal sol.
electrically charged.
is formed, are called colloidal electrolyte.
2. (A) Lyophilic colloids such as starch, gelatin, etc., act as
11. (A) Sulphide ores are concentrated by froth flotation process.
protective colloids.
(R) Pine oil forms emulsion in water.
(R) Protective power of lyophilic colloids is expressed in
terms of gold number. 12. (A) The conversion of fresh precipitate to colloidal state is
called peptization.
3. (A) True solutions do not exhibit Tyndall effect.
(R) It is caused by addition of common ions. (AlIMS 2007)
(R) In true solutions, size of solute particles is much smaller
than the wavelength oflight used. 13. (A) Surfactant molecules form micelles above the critical
micelle concentration (CMC).
4. (A) The micelle formed by sodium stearate in water has
-COO- groups at the surface. . (R) The conductance of solution of surfactant molecules
decrease's sharply at the (CMC).
(R) Surface tension of water is reduced by the addition of
14. (A) Soap and detergent are macro-molecular colloids.
stearate. (AIIMS 2003)
5. (A) Lyophilic sols. are more stable than lyophobic sols. (R) Soap and detergent are molecular of large size.
(R) Lyophilic sols. are highly hydrated in the solution. 15. (A) Gold sol is hydrophobic and multimolecular.
(R) Gold sol is prepared by Bredig's arc method.
414 I G.R.B. PHYSICAL CHEMISTRY FOR COMPETITIONS

• Set-1
1. (a) 2. (b) 3. (a) 4. (c) 5. (b) 6. (d) 7. (d) 8. (a)
9. (b) 10. (c) 11. (a) 12. (b) 13. (d) 14. (a) 15. (c) 16. (a)
17. (b) 18. (c) 19. (b) 20. (d) 21. (d) 22. (c) 23. (c) 24. (a)
25. (a) 26. (c) 27. (b) 28. (d) 29. (c) 30. (a) 31. (c) ·.32: (ti)
33. (d) 34. (b) 35. (c) 36. (d) 37. (a) 38. (d) 39. (b) 40. (a)
41. (a) 42. (d) 43. (c) 44. (b) 45. (a) 46. (c) 47. (a) 48. (d)
49. (d) 50. (d) 51. (c) 52. (b) 53. (a) 54. (a) 55. (a) 56. (c)
57. (d) 58. (c) 59. (c) 60. (b) 61. (b) 62. (b) 63. (c) 64. (a)
65. (b) 66. (d) 67. (b) 68. (a) 69. (a) 70. (c) 71. (d) .72. (d)
73. (b) 74. (a) 75. (a) 76. (b) 77. (c) 78. (d) 79. (c) 80. (a)
81. (b) 82. (d) 83. (a) 84. (d) 85. (c) 86. (d) 87. (b) 88. (d)
89. (b) 90. (c) 91. (a) 92; (b) 93. (b) 94. (b) 95. (a) 96. (a)
97. (d) 98. (c) 99.· (a) 100. (c) 101. (b) 102. (c) 103. (c) 104. (c)
105. (a) 106. (b) 107. (b) 108. (b) 109. (c) 110. (a) 111. (b) 112. (a)
113. (d) 114. (a) 115. (b) 116. (a) 117. (d) 118. (a) 119. (c) 120. (a)
121. (d) 122. (c) 123. (c) 124 •. (b) 125. (d) 126. (d) 127. (c) 128. (b)
129. (a) 130. (e) 131. (a) 132. (b) -133. (c) 134. (a) 135. (c) 136. (e)
137. (c) 138. (c) 139. (b) 140. (a) 141. (a) 142. (b) 143. (c) 144. (b)
145. (a)

• Set-2
\

1. (b, c, d) 2. (a, d) 3. (a, c) 4. (a, b, c) 5. (a, b, c) 6. (a, b, d) 7. (b, c, d) 8. (b, c)


9. (a, d)

1. (d) 2. (b) 3. (a) 4. (a) 5. (a) 6. (c) 7. (c) 8. (c)


9. (a) 10. (a) 11. (a) 12. (b) 13. (b) 14. (d) 15. (b)
THE COLLOIDAL STATE I 415

BRAIN STORMING PROBLEMS


IOBlEc:rIVEQUESTIONS ) for ( lIT ASPIRANTS'
The following questions contain single correct option:
5. The coagulation value in millimoles per litre of electrolytes
1.. Which ofthe following statements are correct? used for the coagulation of AS ZS 3 are as below:
1. On the application of an electric field, the particles of I. NaCI = 52 II. KCl 51
lyophobic sol may move in either direction or not move at III. BaCl z '" 0.69 IV. MgSO 4 0.22
alL The correct order of their flocculating power is:
2. Surface tension of lyophobic sols is similar to that of the (a) I >11 >111 >IV (b)I>II>III IV
dispersion medium. (c)IV>III>II>I (d) IV =111>11>1
3. Electro-osmosis is the movement of the particles of 6. Cotrell precipitator works on the principle of:
dispersion medium under the influence of an electric field. (a) distribution law
Select the correct answer using codes given below: (b) addition of electrolyte
Codes: (c) neutralisation of charge on colloids
(a) 1,2 and 3 (b) 1 and 3 (c) 2 and 3 (d) 1 and 2 (d) Le Chatelier's principle
2. Which of the following statements are correct? 7. The substances involved in micellization are:
1. The smaller the gold number of lyophobic colloid, the (a) polyphilic in nature (b) non-polar in nature
larger will be its protective power. (c) diphilic in nature (d) uniphilic in nature
2. Lyophilic sols, in contrast to lyophobic sols are easily 8. The swelling of 'gel' when placed in water is called:
coagulated on addition of small amounts of electrolytes.
(a) gelation (b) imbibition (c) thixotropy (d) syneresis
3. Ferric chloride solution is used to stop bleeding from a 9. A lyophilic sol is at its isoelectric point then it is:
fresh cut because it coagulates the blood.
(a) negatively charged (b) positively charged
4. The flocculation value of arsenious sulphide sol is
(c) not charged ( d) none of these
independent of the anion of the coagulating electrolyte.
10. Sedimentation potential is reverse of:
Codes:
(a) electro-osmosis (b) electrophoresis
(a) 1,2 and 3 (b) 1,3 and 4 (c) 2,3 and 4 (d) 1,2 and 4
(c) electrokinetic potential (d) streaming potential
3. Match the List-I with List-II and select the correct answer
11. The potential difference between the fixed charged layer and
using the codes given below the lists:
the diffused layer having opposite charge is called:
List-I List-II
(a) zeta potential· (b) colloidal potential
A. Coagulation 1. Scattering
(c) Dom potential (d) streaming potential
B. Lyophilization 2. Washing of precipitates 12. Silver iodide is used for producing artificial rains because
C. Peptization 3. Purification of colloids AgI:
(a) is easy to spray at high altitude
D. Tyndall effect 4. Electrolyte
(b) is insoluble in water
Codes: (c) is easy to synthesize
A B C D (d) has crystals similar to ice
(a) 4 2 1 13. .All colloidal solutions show:
>,
(b) 2 3 4 (a) very high osmptic pressure
(b) high osmotic pressure
(c) 2 4 (c) low osmotic pressure
(d) 4 3 (d) no osmotic pressure
14. Colloidion is a colloidal solution of:
4. The flocculating power of the given ions for the specified
(a) sucrose in water
colloidal sols will be such that:
(b) cellulose in water
Arsenic sulphide sol Ferric hydroxide sol
(c) cellulose nitrate in water
(a) [Fe(CN)6]4-> PO~->SO~- >cr Al 3+ > Ba2+ > Na+
(d) cellulose nitrate in ethyl alcohol
[Fe(CN) 6 r- > PO~- > 15. During micelle formation:
SO~- > Cl- (a) Ml = + ve, t:.S = + ve (b) Ml ve, AS = ve
(c) Ml = - ve, t:.S = + ve (d) Ml + ve, t:.S :::: - ve
cr > SO~- > PO~- >
16. Which of the following is not the .property of hydrophilic
Fe(CN)6]4- solutions?
(d)Cr>SO~- >PO~- > [Fe(CN)6]4- Na+ > Ba z+ > Al 3+ (a) High concentration of ~ersed phase can be easily
obtained
416 G.R.B. PHYSICAL CHEMISTRY FOR COMPETITIONS

(b) Coagulation is reversible C. Paints 3. Solids in solid


(c) Viscosity and surface tension are nearly same as that of D. Jellies 4. Liquids in liquid
water 5. Liquid in solid
(d) The charge of the particles depends on the pH of the (a) A-4, B~2, C-I, D-5 (b) A-I, B-5, C-3, D-2
medium and it mlly be positive, negative or zero (c) A-4, B-5, C-l, D-2 (d) A-I,B-2,C-3,D-5
17. The coagulation of 100 mL of colloidal solution of gold is 22. At CMC, the surfactant molecules undergo:
completely prevented by addition of 0.25 g of a substance" X" (a) association (b) aggregation
to it before addition of I mL of 10% NaCI solution. The gold (c) micelle formation (d) all of these
number of" X" is:
23. The blue colour ofttie water of the sea is due to:
(a) 0.25 (b) 25 . (c) 250 (d) 2.5
(a) reflection of blue light by salts present in water
[Hint: Number of milligrams of protective colloid added in
(b) scattering of blue light by soL particles
10 mL of colloidal gold to prevent its coagulation on addition of
(c) refraction of blue coloured light by.the impurities present
1 mL of 10% NaCI solution is called its gold number.
in seawater
:. Gold number of present colloid = 25}
(d) absorption of radiation of different colours except blue
18. Select the non-elastic gel out ofthe following:
light
(a) starch (b) agar-agar (c) silicic acid (d) gelatin
24. Statement: To stop bleeding from an injury ferric chloride
19. The colligative properties of a colloidal solution compared to
can be applied.
the solution of non-electrolyte of same concentration will be:
Which comment about the statement is justified?
(a) same (b) higher
(PMT (Kerala) 2008)
(c) lower (d) higher or lower
(a) It is not true, ferric chloride is a poison
20. I mole of Agl/Ag + sol. is coagulated by:
(b) It is true, Fe 3+ ions coagulate blood which is negatively
(a) 1 mole ofK! (b) 500 mL of 1M K ZS04
charged sol
(c) 300 mL of I MNa3P04 (d) I mole of AgI
(c) It is not true, Cl- ions form positively charged sol, profuse
21. Match the List-I (Colloidal dispersion) with List-II (Nature of bleeding takes place
the dispersion) and select the correct answer using the codes
(d) It is true, coagulation takes place because of formation of
given below the lists: . (SCRA 2007)
negatively charged sol with CI~ ions
List-I List-II
(e) It is not true, ferric chloride is ionic and gets into blood
(Colloidal dispersion) (Nature of dispersion)
stream
A. Milk I. Solid in liquid
B. Clouds 2. Liquid in gas

1. (c) 2. (a) 3. (a) 4. (b) S. (c) 6. (c) 7. (c) 8. (b)


9. (c) 10. (b) 11. (a) 12. (a) 13. (c) 14. (c)' 15. (ll-) . 16. (c)
17. (b) 18. (c) 19. (c) 20. (a) 21. (a) 22. (d) 23. (b) lAo (b)

• • LI~~~~:~~~fPJtt;HENSIONTYPE Q~ESTlONS·.B;
" ~ '~"':"_","",.;. ;' c', ., .• ,,".' . ': :~;, f' _ '. " , . ,

• Passage 1
2. What type of molecules form micelles?
There are certain substances which behave as normal, strong
electrolytes at low concentration but at higher concentration they (a) Non-polar molecules
behave as colloidal solutions due to the formation of aggregated (b) Polar molecules
particles. Such colloids are called associated colloids and the (c) Surfactant molecules
aggregated particles are called micelles. Soaps and detergents are (d) Salt of weak acid and weak base
the examples of associated colloids. The formation of micelles takes 3. Micelles are formed only: '
place above certain concentration called critical micellization (a) below the CMC and the Kraft temperature
concentration (CMe) and a characteristic temperature. (b) above the CMC and below the Kraft temperature
Answer the following questions: (c) above the CMC and above the Kraft temperature
. 1. Micellesare: (d) below the CMC and above the Kraft temperature
(a) emulsions cum gels 4. Above. CMC, the surfactant molecules undergQ:
(b) associated colloids (a) dissociation (b) aggregation
(c) adsorbed catalysts (c) micelle formation (d) all of these
(d) idealsolutions
THE COLLOIDAL STATE 417
5. Micelles are used in: . .•.~J)assag~3 .
"....
(a) detergents (b) magnetic separation
The process of dialysis finds application in the purification of
(c) petroleum recovery (d) all of these
blood by artificial kidney. In this method, impure blood is introduced
in the artificial kidney apparaltts,. where the wa51e material
• Passage2 (electrolyte) diffuses through the memhrane. The membrane used in
Emulsions are also the. colloidal solutions in which disperse the dialyser is different from the· membrane used in osmosis. '['hese
phase as well as dispersion medium a';'e liquids. It may be oil in membranes allo wthe movement of ions through them.
water or water in oil type. Bancroft proposed that the phase in which Blood is a negatively charged sol. The haemoglobin particles
the emulsifier is more soluble becomes the outer phase of the carry a positive charge. Blood is slightly alkaline ,(pH 7.36-7.42).
emulsion. Emulsifiers can be used to stabilize the emulsion. Soaps, Acidic salts like alum and FeCl3 decrease the pH olthe bloud and the
detergents, proteins and gum. etc., are used as emulsifiers. denaturation of globular proteins present in blood takes place. Due
Answer the following questions: to denaturation, these globular proteins become fibrous which are
1. Addition oflyophilic solution to the emulsion fonns: insoluble and stop bleeding. Blood is lyophobic in natllre.
(a) a protective film around the dispersed phase Answer the following questions:
(b) a protective film around the dispersion medium 1. To stop bieeding, FeCl 3 is applied locally because:
(c) an aerosol (a) FeCl 3 seals the blood vessels
(d) true solution (b) FeCl 3 changes the direction of blood flow
2. Which of the following examples is/are oil in water type (e) FeCl 3 reacts with blood to form a solid substance which
emulsion? seals the blood vessel .
(a) Ink (b) Detergent (d) FeCl 3 causes denaturation of proteins present in blood
(c) Soap (d) Milk 2. Which of the following colloidal solutions does not contain
3. Emulsions canbe destroyed by: negatively charged particles'!
(a) the addition of emulsifier which tends to form another (a) Fe(OHh (b) As2 S3 (c) Blood . (d) Gold sol
emulsion 3. The coagulating power of an electrolyte for blood decreases in
(b) electrophoresis with high potential the order:
(c) freezing (a) Na+, AI 3+, Ba2+ (b) PO~-, SO~-, Cl-
(d) all of the above (c) Ae+, Ba 2+, Na+ (d) a-, SO~-, PO!-
4. Which of the following is homogeneous? 4. Which of the following statements is/are not true?
(a) Milk (b) Paint (a) Blood is positively charged sol
(c) Shampoo (d) None of these (b) Soap solution contains ionic miceiles as the colloidal
5. Milk is an emulsion in which: particles
(a) milk fat is dispersed in water (c) Blood is purified by the process of dialysis
(b) a solid is dispersed in water (d) Ca 2+ and K+ cause coagulation of blood if added in
(c) a gas is dispersed in water excess
(d) lactose is dispersed in water 5. Which of the following isiare lyophobic colloids?
(a) Blood (b) Starch (c) Gelatin (d) Gold

Passage!. 1. (b) 2. (c) 3. (c) 4. (b, c) s. (a, c)


Passage 1. 1. (a) 2. (d) 3. (b, c) 4. (d) 5. (a)
Pauagel. 1. (d). 2. (a) 3. (c) 4. (a) S. (a, d)
418 G.R.B. PHYSICAL CHEMISTRY FOR COMPETITIONS

<i? SEtFASSESSMENT¢
ASSIGNMENT NO.6.

SECTION-I SECTION-II
Straight Objective 'fYpe Questions
This section contains 10 multiple choice questions. Each Multiple Answers Type Objective Questions
question has 4 choices (a), (b), (c) and (d), out of which only 11. Which ofthe following are lyophilic in nature?
one is correct. (a) Gum (b) Sulphur (c) Starch (d) Protein
1. Dbpersed phase and dispersion. medium in butter are 12. Surfactant molecules form micelles in aqueous solution,
respectively: [EAMCET (Medical) 2007] which:
(a) solid and liquid (b) liquid and liquid (a) tend to congregate due to their hydrophobic tails
(c) liquid and solid (d) solid and solid (b) are colloidal-sized cluster of molecules
2. Which one of the following acts as the best coagulating agent (c) Provide protection due to their hydrophobic head
for ferric hydroxide sol? IPMT (Kerala) 2007] (d) none of the above
(a) Magnesium chloride (b) Hydrochloric I:)cid 13. Crystalloid and colloid differ with respect to:
(c) Aluminium chloride (d) Potassium oxalate (a) Tyndall effect
(e)· Potassium ferricyanide (b) particle size
3. The colloidal sol found effective in the treatment of eye (c) diffusion through animal or vegetable membrane
disease is: (d) number of particles per unit volume of solution
. (a) colloidal sulphur (b) colloidal antimony 14. Select the correct statements among the following:
(c) colloidal silver (d) colloidal gold (a) Milk is emulsion offat in water
4. When alcohol is added to the saturated solution of calcium (b) An emulsifIer stabilizes the emulsion
acetate in water then we fIrst of all get sol which changes to (c) Emulsifier forms a thin film around the droplets of
gelatinous mass called solid alcohoL Solid alcohol is ........ . dispersed phase
(a) aerosol (b) gel (d) Milk is an emulsion of protein in water
(c) solid foam (d) solid sol 15. Which ofthe following statements are true?
5.' Colloidal solutions of gold prepared by different methods are (a) Flocculation value is inversely proportional to the
of different colours because of: coagulating power .
(a)· variable valency of gold (b) Colloidal silica is a protective colloid
(b) impurities produced by different methods (c) Alum is used for cleaning muddy water
(c) different diameters of colloidal gold particles (d) Gelatin is added in ice cream, ii acts as emulsifier
(d) different concentration of gold particles
6. Emulsions of polyvinyl acetate are used in: SECTION.;.III
(a) polishes (b) latex paints
(c) fire works (d) rayons Assertion-Reason Type Questions
7. The outcome. of internal liquid of gel on shaking is caned: This section contain~~:5 questions. Each question contains
(aj syneresis . (b) imbibition Statement-l (Assertion) and Statement-2 (Reason).· Each
question has following 4 choices (a), (b), (e) and (d), out of
(c) thixotropy (d) precipitation
which ?llly one is correct.
8. Which of the following shows the maximum hydrophobic
(a) ,Statement-l is true; statement-2 is true; statement-2 is a
behaviour?
. correct explanation for' statement-I.
(a) Adenine (b) Glucose
(b) Statement-l is true; statement-2 is true; statement-2 is not
(c) Stearic acid (d) Glycine a correct explanation for statement-I.
9. The solution of natural rubber in benzene is an example of: (c) Statement-l is true; statement-2 is false.
(a) lyophobic colloids (b) macromolecular colloid (d) Statement-l is false; statement-2 is true.
(c) multimolecular colloid (d) associated colloid 16. Statement-I: Micelles are formed by surfactant molecules
10. When NaCI solution is added to Fe(OHh sol then: above the critical micellar concentration (CMC) .
. (a) [Fe(OHh ]Fe3+. is formed Because
(bn~e(~H)~lCl- is foimed. Statement-2: The conductivity of solution having surfactant
molecules decreases sharply at the CMC. (lIT 2007)
(c) Fe(OHh is coagulated
[Hint: Micellisation takes place above certain minimum
(d) [Fe(OHh]Na+ is formed
concentration called critical micellar concentratiori (CMC). Each
micelle contains at least 100 molecules, therefore conductivity of
solution decreases sharply at the CMC.]
THE COLLOIDAL STATE 419·

17. Statement-I: Aqueous gold colloidal solution is red in colour. 19•. Statement-I: Colloidal solutions are electrically neutral..
Because Because
Statement-2: The colour arises due to scattering of light by Statement-2: Dispersed phase and dispersion medium are of
colloidal gold particles. same charge.
18. Statement-I: An emulsion becomes stable if soap is added to ;20. Statemeitt-l: Colloidal sol scatter the beam oflight while true
. it. solutions do not.
Because Because
Statement-2: Soap contains hydrophobic and hydrophilic Statement':'2; The particles in colloidal sol are bigger than
parts. those of true solutions.

,'.,..

[:).f~f~.~:.
'.
--------~-___II
. ;;.~ .. '

1. (b) 2. (e) 3. (c) 4. (b) 5. (c) 6. (b) 7. (c) is. (a)


9. (b) 10. (c) 11. (a, c, d) 12. (a, b, c) 13. (a, b,. c) 14. (a, b, c) 15. (a, c, d) 16. (b)
17. (b) 18. (a) 19. (c) 20. (a)
R· 7
:CiH'EMICAL,THERMOOYNAMICS
AN'O THERMOCHEMISTRY

INTRODUCTIO~ given set of conditions but does not tell anything about the rate at
which the given process may proceed. It is only concerned with
Thermodynamics is a Greek word. Its literal meaning is motion initial anel final states of the system. For example,
or flow (dynamics) of heat (thermos). However, the term is used thermodynamics predicts that the reaction between oxygen and
in a more general way. Thermodynamics is the branch of hydrogen is possible under ordinary conditions but does not tell
science which deals with all changes in energy or transfers of whether the reaction is fast or slow.
energy that accompany physical and chemical processes. It is
concerned with interconversion of various forms of energy. .7.2 TERMS USED IN THERMODYNAMICS
Chemical thermodynamics is the branch of
Before we study the laws of thermodynamics, it is necessary to
thermodynamics which deals with the study of processes in
define some of the basic terms which are frequently used in its
which chemi,cal energy is involved.
discussion:
, ,Or
(i) System, Surroundings and Boundary: A system is
Chemical thermodynamics is one of the branches, of defined as a specified part of the universe or specified portion
physical ~emistry which deals with transport of heat either of the matter which is under experimental investigation and
as a result of physical change or as a result of chemical the rest of the universe, Le., all other matter which can
change.
interact with the system, is surroundings.
Chemical thermodynamics is concerned with the following
Anything which separates system and surroundings is called
questions:
boundary. The boundary may be real or imaginary; it may
(i) When two or more substances are put together, will they
be rigid or non-rigid; it may be a conductor or a non-conductor of
react?
heat. The terms diathermic wall and adiabatic wall are used for
(ii) If they do react, what energy changes will be associated conductor and non-conductor of heat boundaries respectively.
with the reaction?
For example, a reaction is carried out in a beaker. The contents
(iii) If a reaction occurs, at what concentrations of the of the beaker constitute the system, beaker serves as boundary
reactants and their products will equilibrium be established? and the beaker, the air and anything else in the vicinity constitute
Thermodynamics is not concerned with the total energy of the the surroundings.
.body but only with energy changes taking place during the
(ii) Types of System: There are three types of system:
transformation. The study of thermodynamics is based on three
(a) Isolated system: This type of system has no interaction
generalisations derived from experimental results. These
with its surroundings. The boundary is sealed and insulated.
generalisations are known as first, second and third law of
Neither matter nor energy can be exchanged with surroundings.
thermodynamics based on human experience and there is no
, A substance contained in an ideal thermos flask is an example of
formal proof for them. Scientists are of the view that nothing
an isolated system.
contrary to these laws will ever occur.
(b) Oosed system: This type of system can exchan~e
Limitations of thermodynamics: (i) The laws of
energy in the form of heat, work or radiations but not matter WIth
thermodynamics apply only to the matter in bulk, i.e.,
its surroundings. The boundary between system arid
macroscopic system and not to individual atoms or molecules of
surroundings is sealed but not insulated. For example, liquid in
the macroscopic system. Thermodynamics does not deal with
contact with vapour in a sealed tube forms a closed system. Heat
internal structure of atoms and molecules. (ii) Thermodynamics
can be transferred through the walls of the tube to the
can only predict the feasibility or spontaneity of a process under a
CHEMICAL THERMODYNAMICS AND THERMOCHEMISTRY 421

surroundings but total amount of matter remains the same as Density Number of moles
vapours cannot escape. Temperature Gibbs free energy
(c) Open system: This type of system can exchange matter
Pressure Enthalpy
as well as energy with surroundings. The boundary is riot sealed
and not Prsulated. Sodium reacting with water in an open beaker Mole fraction Entropy
is an &'ample of open system as hydrogen escapes and heat ofthe Molar enthalpy Internal energy
reaction is transferred to the surroundings.
Molar entropy
On the basis of composition, there are two types of systems:
Refractive index
(1) Homogeneous system: A system is said to be
. homogeneous when it is completely uniform throughout. A Specific heat
homogeneous system is made of one-phase only. Examples are: a Viscosity
. pure single solid, liquid or gas, mixture of gases and a true
Surface tension
solution.
(2) Heterogeneous system: A system is said to be Dielectric constant
heterogeneous when it is not uniform throughout, i. e. , it consists When the total mass, temperature, volume, number of moles
two or more phases. Examples are: ice in contact with water, tw<1 and composition have definite values, the system is said to be in a
or more immiscible liquids, insoluble solid in contact with ·a definite state. When there is any change in anyone of these
liquid, a liquid in contact with vapour, etc. properties, it is said that the system has undergone a change of
(iii) Thermodynamic Properties: These are of two types: state. '
(a) Iutensive properties: The properties which do not. (iv) State Functions or State Variables: Fundamental
depend upon the quantity of matter present in the system or size of properties which determine the state of a system are referred to as
the system are called intensive properties. Pressure, temperature, state variables or state functions or thermodynamic
density, specific heat, surface tension, refractive index, viscosity, parameters. The change in the state properties depends only
melting point, boiling point, volume per mole, concentration, etc.,' upon the initial and final states of the system, but is independent
are the examples of intensive properties of the system. of the manner in which the change has been brought about. In
(b) Extensive properties: The properties whose magnitude other words, the state properties do not depend upon a path
depends upon the quantity of matter present in the system are followed.
called extensive properties. Following are the state variables that are commonly used to
Extensive property is an additive propertY of the system. For a describe the state of the thermodynamic system:
heterogenaous system consisting of several phases, the total
1. Pressure (P) 2. Temperature (T)
value of.. extensive property will be equal to the sum of
3. Volume (V) 4. Internal energy (E)
contributions from several phases.
5. Enthalpy (H) 6. Entropy (8)
The following are some salient features of these properties:
7. Free energy (G) 8. Number of moles (n)
(i) In a system having two or more substances, the extensive
property will depend not only on the independent variables but (v) Thermodynamic Processes: When the thermodynamic
also on the number of moles of different componerits present in it. system changes from one state to another, the operation is called a
(ii) If an extensive property is expressed per mole or per gram, process. The various types of the processes are:
it becomes intensive property. For example, mass and volume are (a) Isothermal process: The process is termed isothermal
extensive properties but density (mass per unit volume) and if temperature remains fixed, i. e. , operation is done at constant
specific volume (volume per unit mass) are intensive properties. temperature. This can be achieved by placing the system in a
(iii) The product, sum and ratio of intensive properties are also constant temperature bath, i. e., thermostat. Fer an isothermal
intensive properties. Let X and Ybe two intensive properties, then process dT = 0, i. e. ,heat is exchanged with the surroundings and
X aX ..
(X + Y) ; XY ; ; - are mtensIve propertIes.
. the system is not thermally isolated.
Y ay (b) Adiabatic process: If a process is carried out under
(iv) Let X and Ybe the two extensive properties, then such condition that no exchange of heat takes place between the
(a) (X + Y) will also be an extensive variable. system and surroundings, the process is termed adiabatic. The
system is thermally isolated, i. e., dQ = O. This can be done by
(b) X and -ax WI'11 b' . varia
e mtensIve . bl es.
Y ay keeping the system in an insulated container, i. e. , thermos flask.
In adiabatic process, the temperature of the system varies.
Extensive and intensive properties (c) Isobaric process: The process is known as isobaric in
which the pressure remains constant throughout the change, i. e. ,
IDtensive properties
dP=o.
Molarity Heat capacity (d) isochoric process: The process is termed as isochoric
Molality Mass in which volume remains constant throughout the change, i. e.,
dV=O.
Concentration Volume

i
/
422 G.R.B. PHYSICAL CHEMISTRY FOR COMPETITIONS

(e) Cyclic process: When a system undergoes a number·of Pext == intensity factor
different processes and finally returns to its initial state, it is A V capacity factor
termed cyclic process. For a cyclic process dE = 0 ani! dH = O. Work (w) is a path-dependent function, it is a manifestation of
(1) Reversible process: A process which occurs energy. Work done on a system increases the energy of the system
infinitesimally slowly, i. e., opposing force is infinitesimally and work done by the system decreases the energy of the system.
smaller than driving force and when infinitesimal increase in the Work done on the system, w + ve
opposing force can reverse the process, it is said to be reversible Work done by the system, w == - ve
process. In fact, a reversible process is considered to proceed Heat may be defined as the quantity of energy which flows
from initial state to final state tbrough an' infinite series of between a system and its surroundings on account oftemperature
infmitesimally small stages and at every stage it is virtually in difference. Heat always flows from high temperature to low
state of equilibrium. A reversible process is an ideal process and temperature.
canp.ot be realised in practice. Heat absorbed or evolved, ~Q = ms At
(g) Irreversible process: When the process .goes from where, m mass of substance, s:::: specific heat .
initial to final state in single step in fmite time and cannot be and At "'" temperature difference.
reversed" it is termed as irreversible process. I.n such a case (i) Heat flowing into the system, ~Qor MI + ve.
equilibrium state exists only at the initial and fmafstages of the
The process in this case is endothermic in which the
process. An irreversible process is spontaneous in nature. It is
temperature ofthe system is raised and that ofthe surroundings is
real and canbe perforined in practice. All natural procefises are
lowered.
irreversible in nature.
(ii) Heat flowingout of the system, AQor MI::::- ve.
ReverSibte process . Irreversible proeess The process in this case is exothermic in which the
temperature of the system is lowered and that ofthe surroundings
.1. It is an ideal process and takes It is. a spontaneous process and is raised.
infinite time. takes finite time.
Units of Heat and Work: The unit of heat is calorie (cal). It
2. The driving force is infinitesi- The driving force is' much is defined as the quantity of heat required to raise the temperature
mally greater than the opposing greater than the opposing force. of one gram of water by loe.
force. Since, heat and work are interrelated, S1 unit of heat is the
3. It is in equilibrium at all stages. Equilibrium exists in the initial joule(J).
and final stages only, I joule ~ 0.2390cal
4. Work obtained is maximum. ,Work obtained is not maximum. l.calorie = 4.184 J
5. It is difficult to realise in It can be performed in practice. 1 kcal =4.184 kJ
prllctice. 1 litre -atrn =1 0 1.3 J
Nature of Work and Heat 1.013 X 10 9 erg
Work is a mode, of energy transfer to or from a system with
= 24.206 cal
reference to the surruundings. If an object is displaced tbrough a
distance d x against a force ofF, then the amount of work done is . :.; ::::a_SoME SOLVED ExAMPLES\:::::::
defined as
Wo··Fxdx i,·Example 1. A gas expands by 0.5 litre against a constant
There are many types of work and all of them could be pressure ofone atmosphere. Calculate the work done in joule and
expressed as the product of two factors: calorie.
(i) an intensity factor, Solution: Work = - Pext X volurnechange
(ii) a capacity factor.
:::: -1 x 0.5 0.5 litre -atrn
Some ofthem are:
( a) Gravitational work = (mg ) X h :::: 0.5 x 101.328J = - 50.664 J
where, m = mass of body, g = acceleration due to gravity, 0.5Iitre-atrn -0.5 x 24.20cal =-12.l0cal
h = height moved. . ' . ,Example 2. One mole of an ideal gas is put through a
(b) Electrical work charge x potential series of changes as shown in the graph in which A, Band C
QXV mark the three stages of the system. At each stage the variables
where, Q = charge, V == potential gradient are shown in the graph ..
(c) Mechanical work = P ext (V2 VI) Pext AV (a) Calculate the pressure at three stages ofthe system.
where, Pext = external pressUre, AV increase or decrease in (b) Name the processes during the following changes:
volume. ' (i) A to B (ij) B to C (iii) C to A and (iv) overall change.
Work. associated with change in volume of a system against
external pressure is called mechanical work.
I
CHEMICAL THERMODYNAMICS AND THERMOCHEMISTRY ·1423
=-PxdV= 12x 10 5 x 5x 10-3
24.0 (L) ----------,-----[:7lAI B = 6000J
. . I
I Work done in the process B -7 C is zero as volume remains
. I
, . . I
I
constant.
I
. I
Work done in the process C -7 D (The process is contraction,
1,2.0
.

V
-

(Ll --------------.
I
I
c .
. I

i
I
I
hence work is done on the gas)
. =P x dV =2x 105 x5x 10-3
I I
I I
I I
I
I
I
I
= 1000J
I I
I
I
I
I
(ii) Work done in the. process D -7 A is zero as volume
remains constant.
300 K 600 K
T • Net work done in the wl;1ole cycle = - 6000 + 1000 5000 J
Solution: (a) At stage A: i. e. , net work is done by the gas.
;Example 4. Calculate tlie work done when 1.0 mole; of
V 24.0L;T=300K;n l;R 0.08211itre-atmK-1 mol-: 1
water at 373 K vaporizes against an atmospheric pressure ofl.O
Substituting these values in the ideal gas equation, . aimosphere. Assume ideal gas behaviour.
Solution: Thevolume occupied by wateris very small and
PV=nRT,
thus the volume change is equal to the volume occupied by one
P = 1 x 0.0821 x 300 = 1.026atm gram mole of water vapour.
24.0 V nRT l.Ox 0.0821 x 373 31.0 litre
At stage B: Volume remains the same but temperatUre P 1.0
changes from 300 K to 600 K. Thus, according to pressure law, W=-Pext xAV -(1.0)x(31.0)litre-atm
the pressure will be doubled at B with respect to A.
Pressure at B 2 x 1.026= 2.052 atm = - (31.0) x 101.3 J = - 3140.3 J
At stage C: Temperature is 300 K and volume is half that of Example 5. Identify different steps in the following cyclic
stage A. Thus, according to Boyle's law, the pressure at C will be process:
doubled with respect to A.
Pressure at C = 2 x 1.026= 2.052atm
Temperature At A ,B
(b) (i) During the change from A to B, volume remains
and F is 11 and at C,
constant,"'the process is isochoric.
(ii) During the change from B to C the pressure remains
constant, the process is isobaric. .
I
p
DandEisT2
Tl >T2
.

(iii) During the change from C to A, the temperature remains


constant, the process is isothermal.
(iv) Overall; the process is cyclic as it returns to initial state. v-
Example 3. The diagram shows a P-V graph of a Solution: (i) A -7 B (Temperature and pressure are
. thermodynamic behaviour of an ideal gas. Find out from this . constant).
graph (i) work done in the process A -7 B, B -7 C, C -7 D and :. It is isothermal and isobaric process.
D -7 A, (it) work done il'! the complete cycle
A -7B""-)C -7D -7 A. (ii) B -7 C: It is adiabatic expansion in which temperature
Solution: (i) Work done in the process A -7 B (the process falls from Tl to T2 .
is expansion, hence work is done by the gas) (iii) C -7 D (Temperature and volume are constant)
:. This process is isothermal and isochoric ..
12i A B (iv) D -7 E (Temperature and pressure are constant)
",10 :. It is isothermal and isobaric contraction. .
~ 8
(v) E -7 F (It is adiabatic compression inwhich temperature
~
.(1) 6
increases from T2 to T[ ) .
c: (vi) F -7 A (Temperature and volume are constant).
'":s
0 4 .:. It is isothermal and isochoric process.
Il. 2 D C
··Example 6. One mole of a monoatomic gas is subjected to
following cyclic process:
2 3 4 5 6
V (litre) -

/
424 G.R.B. PHYSICAL CHEMISTRY FOR COMPETITIONS

. T 1, 20 atm. T 20 t 201't contribute to 'the absolute value of internal energy for a given .

A~r
substance in a given state is impossible. But one thing is certain
am, 'ffi that the internal energy of a particular system is a definite
quantity at the given moment, irrespective of the manner by
which it has been obtained. Internal energy like temperature,
p
pressure, volume,' etc., is a state function, i.e., total of all
C possible kinds of energy of a system is called its internal
T1, 2 atm, 10 litre
energy*.
v- It is neither possible nor necessary to calculate the absolute
(a) Calculate TI and T2 . value of internal energy of a system. In thermodynamics, one is
(b) Calculate !lE, q and W in calories in each step of cyclic concerned only with energy change which occurs when the
ptocess. system moves from one state to another. Let dE be the difference
Solution: (a) At A : of energy of the initial state (E j ) and the final state (E f ), then
!lE=E f -
PV = nRT
!lE is positive if E f > E i and negative if E f < E j'
20 x 1= 1x 0.0821 X TI A system may transfer energy to or from the surroundings as
Ii 243.6K heat or work or both,
AtB: PV nRT Characteristics of Internal Energy
20x 10= I x 0.0821 X T2 (i) Internal energy of a system is an extensive property.
T2 = 2436.05 K (ii) Internal energy is a state property.
(b) Path AB: Isobaric process (!lE = 0, q = W ) (iii) The change in the internal energy does not depend on the
path by which the final state is reached.
W =P8V = 20x 9= 180litre~atm (iv) There is no change in internal energy in a cyclic process.
180x 101.3 cal
:=
Thermal Equilibrium and Zeroth Law of
4.185
Thermodynamics .
= 4356.9 cal (Work in compression is positive) When a hot body is kept in contact with a cold body, the cold
Path BC: Isochoric process body warms up and the hot body cools down. The internal energy
w=o of the hot body decreases and that of cold body increases. The
. 3 transfer of energy from the hot body to a cold body is a
qv =8U = n C v 8T = ] x - R x (2436 - 243.6) non-mechanical process. The energy that is transferred from one
2
body to the other, without any mechanical' work involved , is
:::: ~ X 2 x 2192.4= 6577.2 cal called heat.
2
Two bodies are said to be in thermal equilibrium if no transfer
It is cooling process: qv == 6577.2 cal of heat takes place when they are placed in contact.
Path CA: It i.~ i"athermal compression dE o The temperature concept can be stated precisely by the fact
that systems in therm(11 equilibri;!m with each other have the
q = W = 2.303 nRT log, V2 same temperature.
VI
Courses of thermodynamics usually deal with the three laws:
. 10 the first, second and third laws, which constitute the subject
q =W 2.303 x 1x 2 x log -- = 1122.02cal
I matter of thennodynamics. However, at present an ever
increasing use is made in thermodynamics of the law of thennal
INTERNAL ENERGY equilibrium formulated by R. Fowler in 1931, i.e., the Zeroth law
of the,'7Tlodynamics. This law was formulated after the first and
Every !';ystem having some quantity of matter is associated with a
definite amount of energy. This energy is known as internal the second laws had been enunciated.
energy. The exact value of this energy is not known as it includes This law states: If two systems are in thermal equilibrium with a
all types of energies of molecules constituting the given mass of third system, they are also in thermal equilibrium with each other. .
matter such as translational, vibrational, rotational, the kinetic Conversely, the law can be stated as follows:
and potential energy of the nuclei and -electrons within r!"e Ifthree or more systems are ill thermal contact with each other
individual molecules and the. manner in which the molecules 1\' by means ofdiathermal walls alld are all in thermal equilibrium
linked together, etc. The internal energy is denoted by E. together, then any two systems taken separately are in thermal
equilibrium with each other.
E E translational + E rotational + E vibrational + E bonding + E electronic + ,.,
Now let us consider three systems A, Band C a') shown in Fig.
,\ccurate measurements of some forms of energy which 7.1. It is an experimental fact that if system A is in thermal
*The word internal is often omitted and the word energy implies internal energy of a system.
CHEMICAL THERMODYNAMICS AND THERMOCHEMISTRY 425-
equilibrium with system C and system B is also in thermal 1he net energy change of a dosed system is equal to heat
equilibrium with system C, then A and B are in thermal ab~orbed plus the work done on the system.
equilibriUl\1 with each other. Or
The net energy change of a dosed system is equal to heat
absorbed minus the work done by the system.
Or
It is impossible to construct a mobile or perpetual machine
that can work without consumption of any fuel energy.
;':Jl:,,,ample 7. If 500 calorie of heat energy are added to a
system and the system does 350 calorie of work on the
surroundings, what is the energy change ofthe system?
Solution: Heat absorbed, q 500cal
Work done by the system, w 350cal
Applying the first law of thermodynamics,
Fig. 7.1 Systems in therma.1 equilibrium
M = q + w 500+ (-350)= 150calorie
[,lit, FIRST LAW OF THERMODYNAMICS Example 8. If 100 calorie of heat are added tothe same
system as in example 1 and a work of 50 calorie is done on the
This law has been stated in vario¥s forms but is merely the law of system, calculate the energy change of the system.
conservation of energy. It was given by Robert Mayer and
Helmholtz. Solution: Heat absorbed, q = 100 cal
(i) Energy cannot be created or destroyed but it can be Work done on the system, w + 50 cal
converted from one form to another. Applying the first law of thermodynamics,
(ii) The total energy of the universe is constant. I:lE = q + w = (100 + 50) = 150calorie
(iii) Whenever a quantity of one kind of energy disappears, an In the above two examples, the final state is same but the paths
exactly equivalent quantity of energy in some other form must adopted are different. Thus, the change in energy of the system
appear. depends on the initial and final states but does not depend on the
(iv) It is impossibie to construct a perpetual motion machine path by which the final state has reached, q and ware, therefore,
which could produce work without consuming energy. not state functions but M is a state function.
(v) The total energy of an isolated system remains constant
though it may change from one form to another. Some useful conclusions drawn from the first law:
When a system is changed from initial state to the final state, it !:lE =
q+w
undergoes a change in the internal energy from E i to E f' Thus, (i) When a system undergoes a change !:l E 0, i. e. , there is .
!:lE can be written as: no increase or decrease in the internal energy of the system. the
M'='Er ~-Ei first law of thermodynamics reduces to
The change in internal energy can be brought about in two o q+w
ways: m q w
(a) Either by allowing the heat to flow into the system (heat absorbed from surroundings work done by the system)
(absorption) or out of the system (evolution). m w=-q
(b) By doing work on the system or the work done by the (heat given to surroundings = work done on the system)
system.
(ii) If no work is done, w = 0 and the first law reduces to
Consider a system whose internal energy is El . If the system is
supplied q amount of heat, the internal energy of the system will M=q
become E1 + q. Now if work w is also done on the system, the i. e. , increase in internal energy of the system is equal to the heat
final internal energy becomes E 2 • Thus, absorbed by the system or decrease in internal energy of the.
system is equal to heat lost by the system.
E2 '='E1 +q+w
(iii) If there is no exchange of heat between the system and
or E1 = q+ w
surroundings, q = 0, the first law reduces to
or M q+w
This is the mathematical statement of the first law of M w
thermodynamics. In this statement, q is the heat absorbed and w is It shows that if wmk is done on the system, its internal energy
the work done on the system. will increase or if work is done by the system, its internal energy
In case q is the heat absorbed and w the work done by the will decrease. This occurs in an adiabatic process.
system, then the relationship becomes (iv) In case of gaseous system, if a gas expands against the
M=q+ (-w)""q-w constant external pressure, P, let the volume change be !:lV. The
The first law of thermodynamics may also be stated as: mechanical work done by the gas is equal to -P x !:lV.
426 G.R.B. PHYSICAl.: CHEMISTRY FOR COMPETmONS

Substituting this value in M = q + w, 7.6 HEAT CAPACITY


M=q-P/).V Heat capacity of a system is defined as the quantity of heat
When /).V 0, required to raise the temperature ofthe system by one degree. Let
M = q or qv a very small quantity of heat dq be given to a system and the
The symbol qv indicates the heat change at constant volume. temperature of the system rises by dT.
. Example 9. A gas contained in (l cylinder fitted with a dq
Thus, Heat capacity = -
frictionless piston expands against a constant pressure 1 dT
atmospherefrom'a volume of 4 litre to a volume of14litre. In The heat capacity of a system, particularly in a gaseous
dolng so, it absorbs 800 J thermal energy from surroundings. system, determined at constant volume, is different from that
Determine M for the process. determined at constant pressure.
Solution: Given, q = 800 J At constant volume, q=M
/).V (14 4);"'10 litre
w=-Px/).V lxlO 10 litre -atm . So, Heat capacity at constant volume, C v = . (aE)
ar v
But 0.082 litre -atm 1.987 cal
At constant pressure, q /).E + P/).V = f:.,.H •
So, w=_10X1.987 -242.3cal
0.082 .So, Heat capa~ity at constant pressure,
". C . == lfaR)
aT· p p
But 1calorie =4.184 J
So, w= -242.3x 4.184= -1013.7 J
For 1 mole pf a gas, heat capacities at constant volume and
Substituting the values in equation, constartt pressure are denoted by C v and Cp ,respectively. These
M == q + w = (800- 1013.7) 213.7 J are termed as molar heat capacities. Thus, for 1 mole of a gas,

7.5 ENTHALPY C v= (~;)v


Heat content of a system at constant pressure is called enthalpy
denoted by 'H ' .
From first law of thermodynamics;
and Cp=(aR)
aT p
Q=E +PV ... (i)
The difference between C p and C v is equal to the work done
Heat change at constant pressure can be given as by 1 mole of gas in expansion when heated through 1°C.
/).Q /).E + P:/)' V ... (ii)
Work done by the gas at constant pressure =P AV.
At constant pressure heat can be replaced by enthalpy. For 1 mole of gas PV RT.
f:.,.H M + P /). V ... (iii)
When temperature is raised by 1°C, the volume becomes
Constant pressures are common in chemistry as most of the V+AV;
reactions are carried out in open vessels.
So; P(V + AV)= R(T+ 1)
At constant volume, /).V = 0; thus equation(ii) can be written
as or P /).V R
.' /).Q M
Hence, Cp -C v =PAV=R
:.' AH == Heat change or heat of reaction (in chemical process)
at constant pressure Ratio of heat capacity ( y ~:) depends on atomicity of gas.
¢.E = Heat change or heat of reaction at constant volume.
(i) In case of solids and liquids participating 'in a reaction,
C
f:.,.HM (P /). V "" 0) Atomleity Cy Cp=Cy+R y=J..
Cy
(ii) Difference between f:.,.H and M is significant when gases
5/2R
are involved in a chemical reaction. Monoatomic He, Ne, ~R ~R Y -- 1.66
f:.,.H M + P /). V Aretc. 2 2 3/2R
!!Jl = AE + /).nRT matomic ~R 7/2R =1.40
Here, P/).V == MRT °N
2, 2 , H2 , Cl z . 2
!.R
2
=
Y 5/2R
./).n == Number of gaseous moles of products - Number of Tri and polyatomic 3R 4R 4R
y : - = 1.33
gaseous moles of reactants, CO 2 , NH3 , S03 3R
Using the above relation we can interrelate heats of reaction at N0 2 , CH4 etc.
constant pressure and at constant volume.
Let n l and n2 moles of two non-reacting gases A and Bare
mixed then heat capacity of the mixture may be calculated as.
n l (CV)I +n 2 (C v h
(CV)mixture
I
CHEMICAL THERMODYNAMICS AND THERMOCHEMISTRY 427

7~7 EXPANSION OF AN IDEAL GAS surroundings and the temperature remains constant throughout
the expansion; •
(i) Isothermal Expan'Sion The work done by the gas in each step of expansion can be
In an isothermal expansion, heat is allowed to flow into or ou}- given as,
of the system so that temperature remains constant throughout the . dw (P~X! -dP)dV=-Pext ·dV=-PdV
process of expansion. Since, for an ideal gas, the internal energy,
dP· dV, the product of two infinitesimal quantities, is neglected.
llE, depends only on temperature, it follows that at constant
The. total amount of work done by the isothermal reversible
temperature, the internal energy of the gas remains constant, i. e. ,
expansion of the ideal· gas from volume VI to volume V2 is,
llE is zero.
therefore,
llE=O
w=_fV2 PdV
According to first law of thermodymimics, VI
llE q+w For an ideal gas, p=nRT
Since, for isothermal process, llE 0, hence V
q=-w So, w -nRT fV2 dV
JVI V
This shows that in isothermal expansion, the work is done by V2 •
the system at the expense of heat absorbed. The magnitude of q or Integrating, W = - nRT log e - =- 2.303nRT log
w depends on the manner in which the process of expansion is VI VI
carried out, i. e. ,whether it is carried reversibly or irreversibly. At constant temperature, acc.ording to Boyle's law,
Calculation of A Hcan be done according to the following PIVI =P2 V2
equation:
or V2 =~
H=oE+PV VI P2
or AH = llE + A(PV)
So, W = - i.303nRT log
or AH = llE + A.(nRT ) P2
Since, for isothermal process, llE and AT are zero Isothermal compression work of an ideal gas may be derived
. respectively, hence, similarly and it has exactly the same value with positive sign.
AH 0 . V .
wcompression 2.303nRT log ..J... = 2.303nRT log
Work done in reversible isothermal expansion V2 PI
Consider an ideal gas enclosed in a cylinder fitted with a
weightless and frictionless piston. The cylinder is not insulated. Work done in irreversible isothermal expansion
The external pressure, Pex! , is equal to pressure of the gas, Pgas' Two types of irreversible isothermal expansions are observed,
Let it be P. i. e., (i) Free expansion and (ii) Intermediate expansion. In free
Pex! P gas P expansion, the external pressure is zero, i. e. , work done is zero
when gas expands in vacuum. In intermediate expansion, the
If the external pressure is decreased by. an infinitesimal'
external pressure is less than gas pressure. So, the work done
amount dP, the gas will expand by an infmitesimal volume, dV.
when volume changes from P1 to V2 is given by
As a result of expansion, the pressure of the gas within the 2
cylinder fans to Pgas - dP, i. e., it becomes again equal to the W = - f V Pext X dV = - Pex! (V2 -
VI
Vi )
external pressure and,· thus, the piston comes to rest Such a
Since, Pext is less than the pressure of the gas, the work done
process is repeate~ for a number of times, i. e. ,in each step the gas
during intermediate expansion is numerically less than the work
expands by a volume dV.
done during reversible isothermal expansion in which Pex! is
Since, the system is in thermal equilibrium with the
almost equal to Pgas'
surroundings, the infinitesimally small cooling produced due to
expansion is balanced by the absorption of heat from the Maximum work: The work done by the system always
depends upon the external pressure. The higher the value of Pext ,
the more work is done by the gas. As Pex! .cannot be more than
Pexl dp Pgas' otherwise compression will occur, thus the largest value of
Pex! can be equal to P gas . Under this condition when expansion
occurs, the maximum work is done by the gas Qn the surroundings.
}dV
(ii) Adiabatic Expansion
In ·adiabatic expansion, no heat is allowed to enter or leave the
Pgas
system, hence, q Q Whenthis value is substituted in first law of
thermodynamics, AE q + w, we get llE = w.
In expallsion, work is done by the system on the surroundings,
Fig. 7.2 hence, w is negative. Accordingly AE is also negative, i.e.,
428 G.R.B. PHYSICAL CHEMISTRY FOR COMPETITIONS

(~r
internal energy decreases and therefore, the temperature of the
system falls. In case of compression, M is positive, i.e., internal or I ... (x)
energy increases and therefore, the temperature of the system

(~r
rises.
The molar specific heat capacity at constant volume of an or I
P2V2
ideal gas is given by .

C v (~~)v
=
or
or
PI V; Y
PVY
P2V/ ... (xi)
... (xii)
constant
or dE=C v ·dT ... (i)
and for finite change M = C v AT ... (ii) Thus, knowing y, Vi ,V2 and initial temperature, TI , the final
temperature, Tz , can be readily evaluated.
So, w=M=C v AT ... (iii)
ModifYing the eq. (x),
The value of AT depends upon the process whether it is
reversible or irreversible.

Reversible adiabatic expansion


Let P be the external pressure and AV the increase in volume.
Thus, the work done by the system is
w= P AV ... (iv) or' ... (xiii)
If AT is the fall in temperature, then
Cv AT -P AV ... (v) Thus, knowing y, PI 'P2 and initial temperature, the final
For very small change in reversible process, temperature, T2 , can be evaluated.
RT From eq. (v),
CvdT PdV - - ' dV (for I mole of the gas)
V Workdone=C v ·AT=Cv,rz R (T 1:)
(y I) 2 - I
dT dV
C v ·-. =-R·- ... (vi). nR
T V For n moles = (T2 1j)
(y -I)
Integrating the above equation between temperatures 1j and
Tz when corresponding values are VI and Vz , Irreversible adiabatic expansion

CV IT2
T}
dT = - R
T
J V2
VI
dV
V
In free expansion, the external pressure is zero, i. e. , work done
is zero. Accordingly, AE which is equal to w is also zero. If Mis
zero, AT should be zero. Thus, in free expansion (adiabatically),
v V AT =0, M:= 0, w = 0 and AH = O.
C v loge - R log -"l:.. = R log -.l.
TI eVe V In intermediate expansion, the volume changes from VI to V2
1 2
against external pressure, Pc'!.t •
T R V R VI
or log -"l:.. log -"l:.. -log- ... (vii) .W = - P cxt (V2 - VI )
1j Cv VI Cv Vz

We know that, CpTCv=R xR


or Cp -l=~
Cv .Cv or w=C v (Tz -RPext T2 PI T1P2_J
I Td
R P1P2
or I (y 1)=-
Cv Intema}
Heat
Proeess Condition energy Work done
Putting the value of ~ in eq. (vii),
change,
change AU
Cv
Free (U) constant 0 o asP 0
T V, expansion (V) constant AU nCvAT OasAV 0
log -"l:.. = (y - 1) log -.l. Isochoric
TI V2
Y-I Isothermal (T) constant W 0 2.303 nRT

=log ( ~: ) ... (viii) 10g(~J


1= 2.303 nRT

or ... (ix) 10g(;J



CHEMICAL THERMODYNAMICS AND THERMOCHEMISTRY 429
-
Isobaric (P) constant nCp AT nCvfJ.T P (V2 - VI) 3. Let compression of gas takes place from volume Vi to VI in
=nR(T2 -TI ) the finite number of stages then the work done on the gas can be
calculated by summing up the work of all stages.
Adiabatic (q) constant 0 W C v (TI -T2) Work done on the gas Shaded area ofthe diagram.
nR
= --
y-l
(TI - T2)
1
---
y-l
(i>tVI - P2V2 )
Cyclic w 0 Area of
process cycle VI Vi
Volume -----..
;l~l; GRAPHICAL REPRESENTATION OF Fig. 7.5 Indicator diagram (P-V) plot in which the compression
VARIOUS THERMODYNAMIC took place in the finite number of steps with varying pressure
PROCESSES AND THE CALCULATION 4. Graphically, the work of expansion can also be determined.
OF WORK DONE BY GRAPHICAL In case, if a graph is plotted between P and V, then the area under
the curve gives the external work done by the gas.
METHODS
1. Graphically, different processes can be represented as follows.

i _--+---Isobaric process i
Isothermal process

Adiabatic process
Isochoric process V1 V2
Volume ----to-
Volume ----to- Fig. 7.6 Shaded area gives the work done by the gas when
Fig. 7.3 (a) pressure remains constant
w exp == - Pext (V2 - Vi )
Expansion
Compression. = - ve work

i
p

Volume ----to-
i
p
~ Volume ----to-
i
Fig. 7.3 (c) V, V2
Fig. 7.3 (b) Volume ----to-
2. Let a gas is compressed from Vi or VI by an external Fig.7.7 Shaded area represents the work done by the gas
pressure of Pext ' The work done on the gas can be calculated by when both pressure and volume vary
the shaded a:rea of the graph represented in following Fig. 7.4. 5. Work done by a gas undergoing cyclic process : It is
Work done on the gas, w== - Pext (VI - Vi) detennined with the help of following three graphs.
= + ve work, Since VJ < Vi
(~~~1T1) i PN1T,

i I!!
:::l
Cf)
~P2V2T~~
• • Cf)
NPN2T 2
. I!! ~ •.• .,----<Il--...,.
Cf)

I!! :. : m
:::l
Cf)
a.. : ~

a..
~ Volume ----+- Volume ----to- Volume ----to-
(a) (bj (c)
VI Vi Shaded area Shaded area Shaded area
Volume ----to- gives the work gives the work gives the net work
~~~~ ~oo~~ ~~~~
Fig. 7.4 Single step compression of ideal gas against
external pressure Pext Fig. 7.8 Complete cyClic process of a gas

..
430 G.R.B. 'PHYSICAL CHEMISTRY FOR COMPETITIONS

6. Work done by the gas in a closed path ABCA. 9. Ifthe state of a system changes in such a way that its volume
remains constant, the process is called isochoric. Following three
plots represent isochoric process.
ip2"'~',A
",
:!!
:::l
~ P ....
!1l
,

1 B'
'
C
• P
it---- i
V P
0: .: .:
. .
V1 V2 '
Volume ----II>
Fig. 7.9 The gas in closed cyclic process ABCA V ----II> T ----II>
(a) (bl
Work done = Area of shaded triangle Fig.7.12 Isochores of Ideal Gas
I' ,,
= -' x Length of base x Length of perpendicular
2 10. If the state of system changes in such a way that pressure is
I constant, the process is called isobaric. '
=- X (V2 VI )(P2 - PI )
2
7. Let a system of gas passes from initial state A to B in
following three ways. The work done by the gas will be
calculated by the shaded area, P V P

A,

~B
V ----II> T ----II> T ----..
(a) (b) (el
Fig.7.13 Isobars of Ideal Gas.

11. If the state of a system changes in such a way that


V ----II> V ----II>
temperature remains constant, the process is called isothermal.
(a) (e)

Fig. 7.10 Graphical proof that the work is not a state


function
8. Work done in Clockwise and anti~clockwise cyclic V
i
P
process:

i
P o io
, V----II>
P
T----II>
(b)

Fig.7.14 Isotherm of Ideal Gas

7.9· JOULE-THOMSON EFFECT


T~
(el

An ideal gas is defined as the gas in which intermolecular forces


(a)
of attraction are absent. If such a gas expands into vacuum
,Fig. 7.11 Representation of closed cyclic process in adiabatically, no cooling is produced in the process, i. e., there is
clockwise and anti-clockwise direction no change in the int.emal energy which only depends upon the
, temperature.
Shaded area represents the net work done in the, cyclic However, when a real gas is forced through a porous plug into
process. a region oflow pressure, it is found that due to expansii~m, the gas
C.ase I : If the ~yclic process is in clockwise direction then on the side.oflow pressure gets cooled. The greater the difference
work done will be negative because the net work will he done by in pressure on the two sides, the higher shall "be difference in
~~~m, " temperature.
Case II : If the cyclic process is in ,anti-clockwise direction The phenomenon of producing lowering of temperature
then w<,>rk done will be positive because the net work is done on' when a gas is made to expand adiabatically from a region of
the system. high pressure into a region of'low pressure, is known as
,t
Joule-Thomson effect.
CHEMICAL THERMODYNAMICS AND THERMOCHEMISTRY 431

The experimental set-up is shown in Fig. 7.15. It consists of a temperature. Most of the gases have inversion temperature near
thermally insulated tube fitted with a porous plug and two about room temperature and thus cool at room temperature when
weightless and frictionless pistons X and Y. Two sensitive expansion is done. However, the inversion temperature for H2 is
thermometers are fitted on both the sides of the porous plug to - 80° C. Thus, at room temperature hydrogen warms on
record temperature. expansion. In order to produce further cooling by Joule-Thomson
Porous plug effect in hydrogen, it is necessary that it should be first cooled to
- 80° Cby either cooling devices.
The value of inversion temperature has been calculated by
using van der Waals' equation, .
2a
T=
, Rb
1
1
I
,
I
I
I
I

,, Thus, inversion temperature depends· upon van der Waals'


I I
I
1
-------1 }----_. constants a and b of the gas.
I I

: : : : ::I_SOME SOLVED EXAMPLES\ a:::: ::.


.. Example 10. 1mole ofFe203 and 2 mole ofAl are mixed at
temperature 25°C and the reaction is completed to give:
Fe203 (s) + 2AI(s) ~ Al20 3 (s)+ 2Fe(I); Ml - 850kJ
x Y
Fig. 7.15 The liberated heat is retained within the products, whose
combined specific heat over a wide temperature range is about
Consider that a certain amount of the gas is passed through the o.8Jg- I K -I. The melting point ofiron is 1530"C. Show that the
porous plug by slow movement of piston X, i. e. , a volume Vt -I)f quantity of heat liberated is sufficient to raise the temperature of
the gas at pressure PI be forced through the plug on the left side the product to the melting point r'f iron in order to get it welded.
of the plug. The gas in the right hand chamber is allowed to
Solution: Mass of products
expand to volume V2 and pressure P2 by moving the piston Y. The
change in temperature is recorded from the thermometers.
=Mass of one mole Al 2 0 3 + Mass of two mole Fe
Most of the gases are found to undergo cooling on expansion =214g
through the porous plug. However, helium and hydrogen are q ms ~T = 214 x 0.8 x (1803 - 298)
exceptions as these get warmed. = 257656J= 257.656kJ
The gas is compressed on left hand side. Thus, work is done
Heat required is less than heat released, hence the temperature
on the gas. It is equal to PI VI . The work done on right hand side
can be easily raised to the required value. .
by the gas is equal to P2 V2 •
. Example 11. Calculate wand AEfor the conversion of0.5
Total work done by thegas PtVI P2V2
mole of water at 100° C to steam at 1 atm pressure. Heat of
As q 0, the work done by the gas lowers its internal energy vaporisation of water at 100° C is 40670 J mol -I.
and consequently temperature falls. .
AE PI VI - P2 V2 Solntion: Volume' of 0.5 mole of steam at 1 atm.pressure

E2 -EI =PtVI -P2V2 = nRT = 0.5 x 0.0821 x 373 = 15.3 L


P 1.0·
(E2 + P2 V2 ) - (EI + PI Ji! ) = 0
or Change in volume Vol. of steam- Vol. of water
H2 HI =0
or . Mf=O = 15:3 - negligible = 15)L ,
Thus, Joule-Thomson effect occurs at constant enthalpy. Work done by the system,
w Pext X yolume change
Joule-Thomson Coefficient
= 1x 15.3 15.31itre-atm
The number of degrees of temperature change produced per
atmospheric drop in pressure under constant enthalpy conditions -= 15.3x 10l.3J 1549.89J
when the gas. is allowed to expand through porous plug is called 'w' should be negative as the work has been done by the system
Joule-Thomson coefficient. It is denoted by It. on the surroundings.
dT w -1549.89J
It=dP Heat required to convert 0.5 mole of water at 100° Cto steam
When It has positive value, the gas cools on expansion. If it 0.5 x 40670 J 20335 J
has negative value, the gas warms on expansion. Every gas has a
According to first law of thermodynamics,
definite value of t~1T'perature when the .sign .changes from
negative to po:;iti':"; fhis temperature is termed inversion AE q + w 20335 1549.89= 18785.llJ
432 G.R.B. PHYSICAL CHEMISTRY FOR COMPETITIONS

~"lfi;''Example 12. Calculate the work done when 50 gof iron is "Ex8ltaple 15. 5 moles of an ideal gas expand isothermally
dissolved in HCl at 25° C in (t) a clo.',ed vessel and (iI) an open and reversiblyfinm a pressure oflOatm to 2atm at 300K. What is
beaker when the atmospheric pressure is I atm. the largest mass which can be lifted through a height ofl metre in
Solution: (i) When the reaction is carried in a closed vessel, this expansion?
the change in volume is zero. Hence, the work done by the system Solution: Work done by the system
will be zero. (ii) When iron dissolves in HCI, hydrogen is p,' P
produced. = - nRT log _I := - 2.303nRT log _-L
e p. p. .
Z 2
Fe + 2HCI ~ FeCI 2 + H2
56g 10
50 g
I mole
=: 2.303 x 5.x 8.314 x 300 log -:= 20.075 X 103 J
2. x 50 mole 2
56
Let M be the mass which can be lifted through a height of 1 m.
Vohnue of hydrogen produced at 25° C
\Vork done in lifting the mass
nRT 50 0.0821 x 298
=-- -x----- 21.84 L
P 56 1 =Mgh = M x 9.8x 11
This is equal to volume change when the reaction is carried in So, M x 9.8 = 20.075 x 10 3
open beaker.
Work done by the system = - P!3.V -l.Ox 21.84 M := 2048.469 kg
21.84 litre -atm = 2212.39 J ~~~EX'ample 16. Two moles of an ideal monoatomic gas at
Example 13. 5 mole of oxygen are heated at constant NTP are compressed adiabatically and reversibly to occupy a
volume from 10° C to 20° C. What will be
the change in volume of 4.48 dm 3 • Calculate the amount of work done, AE,
the internal energy ofgas? The molar heat ofoxygen at constant final temperature and pressure of the gas. C v for ideal gas
pressure, (!p 7.03 cal mol-I deg- I and R 8.3IJ mor l 12.45J K -I mol-I.
deg- I •
Solution: For an ideal gas, y = Cp 1.667
8.31 ca1 mo I-I deg -I Cv
Solution: R 8.311 mol -I deg- I
4.18 Initia!,volume, Vi 2 x 22.4 44.8dm 3
I
= 1.99 cal mol-I deg- Initial pressure, PI = 1atm
We know that, -Cv R Initial temperature, 1] = 273 K
or C v =Cp R = 7.03 1.99= 5.04 cal mol- 1 deg- I Final volume, V2 :::: 4.48dm 3
Heat absorbed by 5 mole of oxygen in heating from 10° C to Let th~ final pressure be P2 and temperature be T2 •
20"C Applying PI V; Y PzV/
5 x C v x!3.T = 5x 5.04 x 10= 252cal
Since, the gas is heated at constant volume, no external work or =: lI'V2)Y (4.48)1.667.
is done, . P2 VI 44.8
£~, w=O
So, change in internal energy will be equal to heat absorbed, . or (10)1.667
!lE q+w 252+ 0= 252 cal
Example 14. Calculate the amount of work done by 2 Pz (1 0i'667 (PI I given)
mole ofan ideal gas at 298 K in reversible isothermal expansion
log P2 = 1.667 log 10= 1.667
from 10 litre to 20 litre.
Solution: Amount of work, done in reversible isothermal Pz antilog 1.667:= 46.45 atm
expansion, 46.45 x 4.48 x 273
Final temperature pzV.2·7'1
1,

w= - 2.303nRT log • PI Vi 1 x 44.8


. Vi
1268K
Given, n = 2, R 8.314JK- I mol- I ;T=298K,V2 =20Land
Work done on the system = n'Cv ·!3.T
VI lOL.
Substituting the values in above equation~ :::: 2 x 12.45 x (1268 273)
2 x 12.45 x. 995:= 24775.51
w = - 2.303 x 2 x 8.314 x 298 log 20
. 10 From the first law of thermodynamics,
2.303 x 2 x 8.314 x 298 x 0.3010 = - 3434.9 J !lE = q+ w 0+ 24775.5:: 24775.51
i. e. , work is done by the system.
CHEMICAL THERMODYNAMICS AND THERMOCHEMISTRY .I 433

Example 17. A certain volume of dry air at NTP is te,mperature 290.0 K. Calculate the final temperature of water .
expanded reversibiy to four times its volume (a) isothermally (specific heat of water = 4.184 JK -I g-I).. (lIT 1993)
(b ) adiabatically. Calculate thefinal pressure and temperature in Solution: Work done P x dV = 3.0 x (5.0- 3.0)
each case, assuming ideal behaviour.
= 6.0 litre -atm = 6.0 x 10 1.3 J
(~; for air = 1.4) . =607.8J
Let IlT be the change in temperature.
Solution: Let VI be the initial volume of dry air at NTP.
Heat absorbed = m xs x IlT
(a) Isothermal expansion: During isothermal expansion,
the temperature remains the same throughout. Hence, final = to.Ox 18x 4.184 x IlT
temperature will be 273 K. . Given, P x dV = mX sX IlT
Since, IlT P x dV _ _ _ _60_7_.8_ _
or 0.807
=0.25atm mXs 1O.0x 18,Ox 4.184
Final temperature 290 + 0.807 = 290.807 K
(b) Adiabatic expansion:
:.> . Example 20. 109 ofargon gas is compressed isothermally
1', (V. J
---.!...=-.-l. y-I and reversibly at a temperature of 27° C from 10 litre to 5 Utre.
Calculate q, w, AE and AIl for this process.
T2 VI
R = 2 cal K -I mol -I , log 2 0.30. Atomic weight ofAr= 4U
14-1
273 = 4VI =40.4 (lIT 1997)
Tz (
VI )
'

Solution:
V.
w = - 2.303nRT log -.-l.
. Vi
. 273
T2 = - = 156.79K 10 5
4°·4 2.303 x - x 2 x 300 log - = 103.635cal
40 10

Final pressure: PI = (V2) ~ AE=O


P2 VI q=AE-w
1.4 q = w =-103.635 cal
~= 4VI =41.4 .• ·~;;:~ExamJile 21. Calculate· the maximum work done in
P2 ( VI )
expanding 16 g of oxygen at 300 K and occupying a volume of
I 5 dm 3 isothermally until the volume becomes 25 dm 3 •
P2 == 0.143atm
Solution: Reversible work is maximum work.
.'Example 18. Calculate q, w, AE and AIlfor the reversible
isothermal expansion ofone mole ofan ideal gas at 127° C from a w = - 2.303nRT log (~ )
volume oflOdm3 to 20dm 3 •
.. 16 . 25 3
Solution: Since, the process is isothermal, 2.303 x - x 8.314 x 300 log 2.01 x 10 joule
32 . . .5
AE=AIl=O
From first law of thermodynamics, ~'•. ·Exampl~22.. How much heat is required to change 109.
AE=q+w=O ice at 0° C "to steam at 100° C? Latent heat of fusion and
vaporization for H 2 o are 80 cal i g and 540 oall g. respectively.
q= w r
Specific heat of water is cuUg.
w = - 2.303nRT log Solution: Total heat absorbed
VI
= AIl fusion + AIl temp. rise +-Mf vap. ,
20
=- 2.303 x 1 x 8.314 x 400 log- == lOx 80+ lOx I x 100+ lex;S~O.=.7200cal
" ",'

10
S;:>!"lliJtalD.ple23. A swimmer coming out Of a pool is covetted
2.303 x 1 x ~.314 x 400x 0.3010 . 'witli afllm ofwater weigh in. ftbout 80g.1fow much heat must be
= - 2305.3 J (Work is done by the system) supplied to evaporate this water? .
q = - w == 2305.3 J (Heat is absorbed by the system) Soiution: q = mL
'i,'L~i~_a...ple 19.
A gas expands from a volume of3.0dm3 to = 80 x 40.79
3
5.0dm against a constant pressure of3.0atm. The work done 18
during expansion is used to heat 10.0 mole of water of = 181.28kJ
434 G.R.B. PHY,SICAL
. CHEMISTRY FOR
,
COMPETITIONS

'A\I\Q, THERMOCHE;MISTRY
Thermochemistry is !.t branch of physical c~emistry which is
1. 5 moles of an ideal gas at 27°C ex~ands isothermally and concerned with energy changes accomJpanying chemical
reversibly from a volume of 6 L to 60 L. The work done in kJ transformations. It is also termed as chemical energetics. It is
is: based on the frrst law of thermodynamics.
(a) -14.7 (b) -28.72 (c) +28.72 (d) ~56.72 Chemical reactions are accompanied by evolution or
[Ans. (b)] absorption of heat energy. When reactants combine together to
form new products, there is readjustment of energies. During a
[Hint: W == - 2.303nRT log chemical reaction, the chemical bonds between atoms in the
VI
reactant molecules are rearranged in the product molecules, i.e.,
/ =- 2.303 x 5 x 8.314 x 300 log 60
6
chemical bonds in the reactants are broken down and new
chemical bonds are formed in the products. Energy is needed to
= -28.72 kJ] break the bonds of reactants and energy. is released in. the
formation of new bonds of products.
2. . 10 moles of an ideal gas confmed to a volume of 10 L is
released into atmosphere at 300 K whe,re the pressure is I bar. Exothermic Reactions
The work done by the gas is: Heat is evolved in these chemical reactions. It is possible
(R = 0.083 L bar K -I mol-I) when the bond energy of reactants is less than the bond energy of
(a) 249 L bar (b) 259 L bar (c) 239 L bar (d) 220 L bar products.
[Ans. (e)] . . At constant pressure,
[Hint: Initial volume, VI =10 L !JJI (Hp HR ) -ve, i.e., Hp <HR
V (final) == nRT == 10 x 0.083 x 300 :: 249L At constant volume,
2
p 1 AE=(Ep ER ) ve, i.e., Ep <ER
W =P L\V =I x (249 10)
Endothermic Reactions
== 239 L bar] Heat is absorbed in these chemical reactions. It is possible
3. ·1 litre~atm work is approximately equal to: when the bond energy of reactants is greater than the bond energy
(a) 101.3 J (b) 8.314 J (c) 931 J ( d ) 19.2 J of products. .
[Ans. (a)] At consti,lnt pressure,
[Hint: 0.0821litre-atm == 8.314 J (values of gas constant)
!JJI =Hp - HR + ve, i.e., Hp >HR
:.llitre-atm = 10l.3 J]
4. A system absorb~ 20 kJ heat and also does 10 kJ of work. The. At constant volume,
net internal energy of the system: AE =E pER =+ ve,
(a) increases by lO kJ (b) decreases by 10 kJ Sign conventions:
(c) increases by 30 kJ (d) decreases by 30 kJ AQ AE !JJI
[Ans. (a)] (-) (- )/{-)
Exothermic
[Hint: L\U =)q + w =20 -10 = 10 kJ]
Endothermic (+) (+f (+)
5. One mole of a gas is heated at constant pressure to raise its
temperature by 1'lC. The work done in joules is: Exothermic and endothermic chemical equations can be
(a) 4.3 (b)-8.314 . represented as:
(c) ~16.62 (d) unpredictable NaOH(aq.) + HCI(aq.) ~ NaCI(aq.) + H 2 0(l)
[Ans. (b)] Exothermic ..' + 13.7 kcal
[Hint:' w = -nR L\T or w= -P L\V . { NaOH(aq.) + HCI(aq.) ~ NaCI(aq.) + H 0(l);
2

= -1 x 8.314 x 1 = _ P ( n~2 _n~l) !JJI = - 13.7 kcal

Endothermic {C(S.) + H20(g)~ CO(g) + H 2 (g)- 31.4 kcal


=-8.314J] C(s)+ H20(g)~ CO(g) + H 2 0(g);
6. In open system:
(a) there will be exchange of both matter and energy !JJI =+ 31.4 kcal
(b) there will be no exchange of matter and energy
(c) there will be exchange of energy only {i~4r HEAT OF REACTION OR ENTHALPY
(d) there will be exchange of matter only OF REACTION
[Ans. (a)] Heat of reaction is defmed as the amount of heat evolved or
[Hint: Open system has imaginary boundary, therefore, both absorbed when quantities of the substances indicated by the
energy and mass can be exchanged.] chemical, equation have completely reacted. The heat of reaction
CHEMICAL THERMODYNAMICS AND THERMOOt'I9.4ISTRY 435

(or enthalpy of reaction) is actually the difference between the . The difference between the two values is equal to the heat
enthalpies of the products and the reactants when the quantities absorbed when 12 g of diamond is converted into 12 g of
of the reactants indicated by the chemical equation have amorphous carbon. This is termed as heat of transition.
completely reacted. Mathematically,
C (diamond) ----'? C (arriorphous); Ml 3.3 kcal
Enthalpy of reaction (or heat of reaction)
(iii) Enthalpies of solution: Enthalpies of reaction differ
Mf mp -IRR
when in one case dry substances react and in another case when
For example, the equation the same substances react in solution. For example, in the
H2 (g) + CI 2(g) = 2HC1(g) + 44.0 kcal orMf = :.....44 kcal reaction between hydrogen sulphide and iodine, .
indicates that when 2 g of hydrogen ( 1 mole) completely reacts H 2 S(g)+ 12(g)----'?2Hl + S; I1l;I
, = -17.2kcal
with 71 g of chlorine (l mole) to form 73 g ofHCI (2 mole), the
H2 S(g ) + 12 (solution) ----'? 2Hl (solution) + S;
amount of heat evolved is 44.0 kcal or the enthalpy decreases by
44.0 kcal or the reactIng system loses 44.0 kcal of heat or the Mf = _. 21.93 kcal
enthalpy change of the reaction, Mf = - 44.0 kcal. (iv) Temperature: Heat of reaction or enthalpy of reaction
Consider the following reaction: also depends on the temperature at which the reaction is cat:rj.ed
out. This is due to variation in the heat capacity of the system
CZH 4 (g)+ 3°2 (g) = 2CO z + 2H zO(I);!ill = -335.Skcal
with temperature. Due to this reason, entlullpies of reaction are
The equation indicates that reaction has been carried between calculated and expressed at a standard temperature of 25° Cor
1 mole of C2H4 and 3 mole of oxygen at constant volume and 298 K. However,if the reaction is not carried out at 25° C, the
25° C. The heat evolved.is 335.S kcal or the internal energy of the . temperature at which the reaction is performed, is indicated.
system decreases by 335.S kcal. (v) Reaction carried out at constant pressure or constant
Consider another reaction: volume: When a chemical reaction occurs at constant volume,
Os) + H 2 0(g)=CO(g)+ H2 (g) - 31.4 kcal the heat change is called the enthalpy of reaction at constant
or Mf 31.4kcal volume. However, most of the reactions are carried out at
constant pressure; the enthalpy change is then termed as the
This reaction indicates that 1 mole of carbon (12 g) reacts with enthalpy of reaction at constant pressure; The difference in the
1 mole of steam (IS g) to form I mole of CO and I mole of values is negligible when solids and liquids are involved in a
hydrogen and 31.4 kcal of heat is absorbed. The enthalpy of the chemical change. But, in reactions which involve gases, the
. system increases by 31.4 kcal or the total enthalpy of the products difference in two valueS is considerable. For this purpose see
. is 31.4 kcal more than the enthalpy of reactants. section 7.5.
/
Factors which Influence.~he Heat of Reaction !ill+6.nRT=Mf
There are a number of factors which affect the magnitude of or qy + 6.nRT = qp
heat of reaction.
(0 Physical state of reactants and products: Heat energy qv == heat change at constant volume, .
is involved for changing the physical state of a chemical q p = heat change at constant pressure.
substance. For example, in the conversion of water into steam, 6.n = total number of moles of products - total number of moles
heat is absorbed and heat is evolved when steam is condensed. of reactants
Considering the following two reactions: Bcill,aDlple 24. The heat of combustion of ethylene at 18° (:
·1· . and at constant volume is -335.S kcal when water is obtained in
H 2 (g)+ -02 (g)=H 2 0(g); Mf = -57.Skcal liquid state. Calculate the heat of combustion at constant
· 2 ·
pressure and at I'so C.
H 2 (g)+.! 0z(g)= H 20(l); Mf = - 6S.32kcal Solution: The chemical equation for the combustion of
2 C 2 H 4 is
It is observed that there is difference in the value of Mf if C2 H 4 (g)+ 30.3(g)= 2C02 (g) + 2H 2 0(1);!ill= - 335.8 kcal
water is obtained in gaseous or liquid state. Mf value in second I mole 3 moles 2 moles
case is higher because heat is evolved when stearri condenses. No. of moles of reactants =(1 + 3) = 4
Hence, physical state always affects the heat of reaction. .
No. of moles of products = 2
(il) AUotropic forms of the element: Heat energy is also
involved when one allotropic form of an element is converted So, 6.n = (2 - 4) = - 2
into another. Thus, the value of MI depends on the allotropic Given,!ill = - 335.Skcal, 6.n 2, R 2x 10-3 kcal
form used in the reaction. For example, the value or'Mf is
and T (1S + 273) = 291 K
different when carbon in the form ofdiamond or in amorphous
form is used. Applying Mf = llE + 6.nRT
C (diamond) + 02 (g) ----'? CO 2(g ); Mf = '- 94.3 kcal =-335.S+ (-2)(2 x 10-.3 )(291)
C (amorphous) + 02 (g) ----'? CO 2 (g); Mf 97.6kcal == -336.964 kcal
436 G.R.B. PHYSICAL CHEMISTRY FOR COMPETITIONS

i1ftil"lPple 25. The enthalpy of formation of methane at The knowledge of standard heats _.. <onnation of various
constant pressure and 300 K is -75.83 kJ. What will be the heat of substances can be used to calculate the heats of reactions under
formation at constant volume? (R = 8.3 J K -I mol -1 ) standard conditions, i. e. ,~ .
Solution: The equation for the fonnation of methane is 'AHO for' [swnof thestan?ard] [swnof thesta~d]
. = heats of formation - heats of formation
C(s)+ 2H 2 (i) CH4 (g); AH", -75.83 kJ the reaction
2 moles 1 mole of products of reactants

an == (1- 2) -1 i. e. , ~ = 1: tJIf (products) - 1: tJIf (reactants)


Given,AH =-75.83kJ,R = 8.3 x 10-3 kJK-1 morl ,T= 30CK
The heats of fonnation of all the elements in their standard
Applying AH = M + anRT
states are arbitrarily assumed to be zero.
-75.83"" M + (-1)(8.3 X 10-3 )(300) .Intrinsic energy: It may be defined as:
So, M = - 75.83 + 2,49 Intrinsic energy Heat of formation
73.34 kJ "Additional amount of energy whicbone gram mole of the
compound possesses above tbat Of its constituent elements.'"
IIfI ENTHALPY OF FORMATION OR HEAT Let us consider the combustion of carbon.
, OF FORMATION ' C(s) + 02 (g) -----1 CO 2(g );AH - 94.38 kcal
The process in which a compound is fonned from its constituent When I mole CO 2 is formed, 94.38 kcal heat is evolved. It
elements in their standard state is called formation., means, energy content of CO2 is less than energy content of C( s )
,;i' The amount of beat absorbed or evolved when 1 mole of the and 02 (g) by 94.38 kcaL Thus, intrinsic energy of CO 2 is 94.38
substance is directly obtained from its constituent elements is kcal.
called heat of formation. If we assume that intrinsic energy of elements is zero, then
C (amorphous) + O2 (g) -----1 CO 2 (g); AH == - 97.6 kcall mol intrinsic energy of compound may be calculated in the form of
Enthalpy of formation ofCO2 is -97.6 kcal . heat of fonnation. Hence, intrinsic energy may not be considered
as internal energy of the compound.
H2 (g) + Cl 2(g) -----12HCl(g); AH = - 44 kcal
m~il~~lnp.~ 26. Calculate the enthalpy change for the
Thus, enthalpy offonnation ofHCI is -22 kcal per mot following reaction:
Standard enthalpy ,of formation: The enthalpy of
CH4 (g) +'202(g) -----1 CO2 (g) + 2H20( I)
formation depends upon the conditions of formation, i. e. ,
temperature, pressure and physical states (gas, solid or liquid) or given, enthalpies offormation ofCH4, CO2 and H 20 are -74.8k.J
allotropic state of the reactants. If all substances of the chemical mol-I, -3915 kJ moZ-1 and -'-286.2kJ mol ",1 respectively.
reaction are in their standard states (i. e. , at 25 ° Cor 298 K and 1 Solution:
atmospheric pressu,re), the heat of fonnation or enthalpy . of "6 0 \

formation is called standard heat of formation or standard AHo ':" AHf (products) - AHf (reactants)

f
enthalpy of formation. It is denoted by M (volume constant) =[AHf (C02) + 2AHf (H:P) ] [AHf (CH4) + 2AHf (02) ]
f
or!1H (pressure constant). ;:= [-393.5+ 2x(-286.2)] [-74.8+ 2x 0]
It is very difl1cult to determine absolute values of enthalpies of = - 393.5 - 572.4 + 74.8
substances. However, relative enthalpies, of substances can be = -891.1kJ
determined if the enthalpies of free elements at 25° C and I
~,~j:#.nple 27. The standard heats offormation at 298 Kfor
atmospheric pres.sure are taken arbitrarily as zero.
CCI4 (g),H 20(g),C02(g)andHC1(g) are -25.5, -57.8,-94.1
. Consider th~ reaction between carbon and oxygen at 25° Cto
form carbon dioxide under atmospheric pressure. and-22.1 kcal mol-l respectively. Calculate L1H~98 for the
reaction.
C(S)+02(g.)=C02(g); AH=AHf ==-393.5kJ
,
'/

AHf =AH=Hp -HR =-393.5kJ CC14 (g ) + 2H20(g) -----1 CO2(g) + 4HCI(g)


H R = 0, i. e., enthalpies of free elements are taken as zex:o. Solution:
S?, AHf = AH =lip =enthalpy of CO 2 = - AHo,
393.5 kJ AHO
= f (products) -
AUO
f
UJ.l (reactants)

Thus, ' otlli'f 393.5 kJ.

TI;le compounds which have positive enthalpies of formation


;:= [AHf(COil + 4AH f(HO)] - [AHf(CCl 4 ) + 2AH/(H20 )]

are called endothermic compounds and are less stable than the ;:= [-94.1 + 4 x (- 22.1)] [- 25.5 + 2x (- 57.8)]
reactants.' The compounds which have negative enthalpies of =-94.1-88.4+25.5+ 115.6
formation are known as, exothermic compounds and are more
stable than reactants. = -182.5+ 141.1 = 41.4 kcal
CHEMICAL THERMODYNAMICS AND,THERMOCHEMISTRY 1437
~~;f:~~\E~ample 28. The molar heat offormation ofNH4N03 (s) These substances are, therefore, termed as fuels. An adult
is -367.5kJ and those ofN zO(g )and H 20(1 ) are + 8 1.46 kJ and requires 2500 to 3000 kcal of energy per day. Since, the values of
-285.78 kJ respectively at 25°C and 1 atmospheric pressure. enthalpies of combustion of different food articles are known; it
Calculate the All and IlE for the reaction, becomes easy to calculate our daily requirements and thus select
the articles ~f food so as to secure a balanced diet.
NH4N0 3 (s) ----" N 20(g) + 21120(1 )
The grading of food articles and various fuels can be done on
Solution: AlI° All! (products) - All! (reactants) the basis of the values of enthalpies of combustion.
The energy released by the combustion of foods or fuels is
=[AlIf (N;P) + 2x AlIf (H;P}] - [All! (NH4N03)] usually compared in terms of their combustion energies per gram.
It is known as calorific value. Tbe amount of beat produced in
'" 81.46+ 2x (-285.78) (-367.5) calorie or joule wben one gram of a substance (food or fuel) is
= 81.46- 571.56+ 367.5 completely burnt or oxidised.
-122.56 kJ When methane bums, 890.3 kJ 11101-1 of energy is released.
We know that, All IlE + AnRT CH4 (g) + 202 (g ) ----" CO 2 (g) -+ 2H z0(1),
1 mole(16g)
or IlE All AnRT
AHrn4 ':"- 890.3 kJ
An = I; R = 8.314 X 10-3 kJmol- 1 K- 1 ; T = 298K
IlE =-122.56 - (l )(8.314 X 10-3 )(298) So, the calorific value of methane:::: - 890.3 :::: - 55.6 kJ/ g
16
:::: -122.56 - 2.477 Caiorific values of some important foodstuffs and fuels are
iven below:
=-125.037kJ
FUel " Calorftte, value (kJ/g) " ~FoOd Ciiloflfle value (kJ/iJ:::
"~if:3'~ ENTHALPY OF COMBUSTION OR Wood 17 Milk 3.1
HEAT OF COMBUSTION Charcoal 33 Egg 6.7
Enthalpy of combustii.)U is the amount of heat evolved or Kerosine 48 Rice 16.7
decrease in enthalpy when 1 mole of the substance (compound or Methane 55 Sugar 17.3
element) is completely oxidised. The enthalpy of combustion, LPG 55 Butter 30.4
i. e., All is always negative. For example, H dro en 150 Ghee 37.6
CH4 (g) +,20~ (g) ----" CO 2 (g) + 2H 2 0; Out of the fuels listed, hydrogen has the highest calorific
All = - 890.3 kJ value; H;owever, it is not used as domestic or industrial fuel due to
C2 Hs OH(I) + 30 z (g) ----" 2COz (g) + 3H2 0( I); some technical problems. Of the various constituents of our food,
fats and carbohydrates serve as the main sources of energy. The
All :::: - 1367.4 kJ
calorific value of proteins is quite low.
2Cz H6 (g) + 70 z (g) ----" 4COz (g ) + 6H2 0(l); (U) Entbalpies of formation: Enthalpies of formation of
All = -745.6kcal , various compounds, which are not directly obtained, can be
Since, 2 mole of CzH6 are involved, hence enthalpy of calculated from the data of enthalpies of combustions easily by
combustion of ethane the application of Hess's law.
:::: _ 745.6 _ 372.8 kcal Heat of reaction == 1: Heat of combustion of reactants
2 - 1: Heat of combustion of products
1 ,
C(s)+ 2 °2(g)----"CO(g);AH=-26:0kcal,

C(s) + 02 (g) ----" CO2 (g); All = - 94.3 kcal


The enthalpy of combusti'on of carbon is not -26.0 kcal as l:~_.npie 29. The heats ofcombustion ofCH4 and C4H1O
combustion is not complete because carbon monoxide can further are-890.3kJmor 1 and-2878.7kJmor1 respecti~ely. Which of
be oxidised to carbon dioxide. The enthalpy of combustion of the two has greater efficiency as foe/per gram? '
carbon is thus -94.3 kcal. Solution: Molar mass of methane = 16
The enthalpies of combustion have a number of applications.
Some of these are described below: . 890.3
Heat produced per gram of methane:::: - - - = - 55.64 kJ
(1) Calorific values o,f foods and fuels: Energy is needed 16
for the working of all
machines. Even human body is no Molar mass of butane = 58
e~ception. Coal, petroleum,' natural gas, etc., serve as the
, , 2878.7
principal sources of energy for man-made machines, the food Heat produced per gram of butane:::: - - - =- 49.63 kJ
which we eat serves as a source of energy to our body. These 58
Isubstances undergo oxidation or combustion and release energy. Thus, methane has greater fuel efficiency than butane.

/
438 G.R.B .. PHYSICAL CHEMISTRY FOR COMPETITIONS

ii!Jt;,*a...ple 30. In a Gobar gas plant, gobar gas is formed by AH = AHI (products) - ' AHI (reactants)
bacterial fermentation of animal refuse . .It mainly 'contains
methane and its heat of combustion is -809 kJ mol ~l according [AHI(A1ZO) +2AHI (Fe)]-[2AHI (A1) + AHJ(FcZ0 3)j
to follOwing equation: = (-399+ 2 X 0)- [2x 0+ (-199)]
CH4 +202 ~COz +2HzO; AH=-809kJmor l = - 399+ 199= -200kcal
How much gobar gas would have to be produced per day for a At. mass of aluminium = 27, Mol. mass of Fez 0 3 = 160
small village of 50 families, if it is assumed that each family
requires 20,000 kJ of energy per day? The methane content in Volume of reactants 160 + 2x27 =50.77cm3
gobar gas is 80% by mass. S.2 2.7
Solution: Energy consumption of 50 families per day
Fuel value per cm 3 = 200 = 3.92 kcal
50 x 20,000 kJ = Ix 106 kJ SO.77
. 809 kJ of energy is obtain~d by h.urning methane =16 g r~l{E~ample 33. When 2 mole ofC zH 6 are completely burnt,
I x 106 kJ of energy will be obtained by burning methane 3129 kJ of heat is liberated. Calculate the heat offormation' of
C ZH 6 • AHI forCO z and HzOare -39SkJ and -286kJ respec-
16 x 106 = 1.98 x 104 g tively.
809 Solution: The equation for the combustion of C z H6 is:
= 19.8 kg
2C z H 6 +70 z ~4C02 +6Hz O; AH=-3129kJ
Since, methane content in gobar gas is 80% by mass, hence,
the mass of gobar gas needed . AH AHI (products) - AHI (reactants)
100 =[4x AHI (C02) +6x AH(Hp)]-[2x tlHI (CZ H 6)
= - x 19.8= 24.7Skg
80,
+ 7 AHI (02)]
~jample31. The standard molar heats of formation of
ethane, carbon dioxide and liqUid water are -21.1, -94.1 and -3129=[4 X (-39S)+ 6x (-286)] [2x AHf{C2H o) + 7x 0]
. - 68.3 kcal respectively. Calculate the standard molar heat of or 2 x AHf(C2H6) = -167
combustion ofethane. .
Solution: The required chemical equation for combustion So, AH - 167 = - 83.5 kJ
I (C2H 6) 2 .
of ethane is
2C z H6 (g)+ 702 (g) 4COz (g) + 6H zO(I); tlHo =? ~;;E:Jiample 34. The standard heats oJformation ofCH4 (g ),
COz~g) and H 20(g) are -76.2, -398.8 and -241.6 kJ mol ~I
The equation involves 2 moles ofCzH 6 ; heat of combustion
AHo r(!spectively. Calculate the amount ofheat evolved by burning 1m 3
of ethane will be
2 ofmethane measured under normal conditions. (lIT 1990)
Solution: The required equation for the combustion of
AHo = ~(products) - ~(reactants) methane is:

=[4 x AHf P'J2) + 6AHf (HP)] - [2AHf (CZH6) + 7 tlH; (02)]


= [4 x (-94.1)+ 6x (- 68.3)] - [2 x (-21.1)+ 7x 0] AH = AH( (products) - AHI (reactants)

= - 376.4 - 409.8 + 42.2 = AHI(C02) + 2x AHI (H2 0 ) - AHf{CH 4) 2AHI (02J


= - 744.0 kcal
=-398.8- 2x 241.6- (-76.2)- 2x 0
AHO
- - :::: H eat 0 f' comb " 0 f eth ane = - -
ustlon 744.0
- = 372.0kcal 80S.8 kJ mol- 1
2 2
.~!~ple 32. An intimate mixture of ferric. oxide and Heat evolved by burning 22.4 litre (I mole) methane
aluminium is used as solid fuel in rockets. Calculate the foel 805.8 kJ. So, heat evolved by burning 1000 litre (1 n? )
value per cm 3 of the mixture. Heats o.fformation and densities methane
are as follows:
= ~ 80S.8 X 1000 35973.2 kJ
22.4
....~t.~ple 35. A gas mixture of 3.67 litre of ethylene and
Density of Fez03 = 5.2 g lcm 3; Density of Al 2.7 g I cm 3
methane on complete combustion at 25° C produces 6.11 litre of
CO z . Find out the heat evolved 011 burning 1 litre of the gas
Solution: The required equation is:
mixture. The heats of combustion of ethylene and methane are
1
2Al + Fez03 ~ A1 zC 3 + 2Fe; AH ? -1423and-891kJmol- at2SoC. (liT 1991)
· CHEMICAL THERMODYNAMICS AND THERMOCHEMISTRY

Solution: C 2H4 + 302 ---72C0 2 + 2H 20 equivalent of an acid is neutralised by 1 gram equivalent of a base
a litre 2a litre in dilute solution. Some examples are:
CH 4 + 202 ---7 CO 2 + 2H 20 Strong acid +,Strong base = Salt + Water; Mf = -13."7 kcal
~ .
(3.67 - a) litre (3.67 - a) litre HN0 3 (aq.) t ~ilOH(aq.) = NaN0 3 (aq.) + H 20(/);
Given, 2a + 3.67 - a = 6.11 Mf = -13.7 kcal
HC1(aq.) + NaOH(aq.) = NaCl(aq.) + H 2 0(/); .
a = 2.44 litre
Mf = -13.75 kcal
Volume of ethylene in mixture = 2.44 litre
1 1
- H 2S0 4(aq.) + NaOH(aq.)=- Na2S04(aq.) + H20(/);
Volume of methane in mixture = 1:23 litre 2 2 .
Mf = -13.7kcal
Volume of ethylene in I litre mixture = 2.44 = 0.6649.1itre
3.67 It is observed that heat of neutralisation of a strong acid
against a strong base is always nearly the same, i.e., 13.7 kcal or
Volume of methane in llitre mixture = 1.23 = 0.335i litre 57 kJ, no matter what acid or base is employed. This constant
3.67
value is explained with the help of theory of ionisation. Both acid
24.45 litre of a gas at 25° Ccorrespond to 1 mole. and base are present in aqueous solution in the form of ions and
Thus, heat evolved by burning 0.6649 litre of ethylene when mixed, the following reaction oCcurs:
= - 1423 x 0.6649 = - 38.69 kJ H+ +A- + B+ +OH- B+ +A- +H 0; Mf=-13.7kcal
24.5 2
Strong acid Strong base Salt
and heat evolved by burning 0.3351 litre of methane
Cancelling the ions which are common on both the sides,
=_ 891 x0.3351=-12.21kJ H+ +OH- =H 20; Mf=-13.7kcal
24.45
So, total heat evolved by burning 1 litre of mixture Thus, beat of neutralisation of a strong acid and a strong
base is merely the heat of formation of water from H + and
=-38.69- 12:21
OH- ions. This is the common reaction whenever a strong acid
=-50.90kJ and a strong base are mixed and that is why the heat of
neutralisation is same.
7.1.4 ENTHALPY OF SOLUTION OR HEAT However, when a strong acid and
a weak base or a weak acid
OF SOLUTION and a strong base or a weak acid and a weak base are mixed in
The amount of heat evolved or absorbed when 1 mole solute is equivalent amount, the heat evolved or change in enthalpy is less
dissolved in excess of solvent (about 200 mole) is called heat of than 13.7 kcal. This is shown in the following examples:
solution. HC1(aq.) + NH 4 0H(aq.)= NH 4 Cl(aq.) + H 2 0(l);
Strong Weak
Some examples, of heat of solution are: Mf = -12.3 kcal
H 2S04 (/) + aq. = H2S0 4(aq.); Mf = - 20.2 kcal
HCN(aq.) + NaOH(aq.) = NaCN(aq.) + H 20(l);
KC1(s) + aq. = KC1(aq.); Mf= 4.4 kcal Weak . Strong
Mf = -12.3 kcal
KOH(s) +aq. = KOH(aq.); Mf = -13.3 kcal
Heat of ideal solution is taken zero. CH 3 COOH(aq.) + NH 4 0H(aq.) =CH 3 COONH 4 (aq~) +H 20(l);
Weak Weak .
Generally, dissolution of substances in a solvent is a Mf= -11.9 kcal
disintegration process. This process needs energy. In such cases,
energy is absorbed, i. e., Mf is positive. But in some cases, The reason for the lower value is that part of the heat energy
besides the process of breaking or ionisation, there is hydrate evolved is utilised in the complete ionisation of a weak acid or a
formation. During hydration heat is evolved. The net result is that weak base or both. Hence, the net heat of neutralisation is less
heat is either evolved or absorbed. There are also cases in which than 13.7 kcal. The neutralisation of NH 40H withHCl can be
heat of separation of ions is just equal to the heat of hydration and explained in the following way:
there is very little heat effect as in the case of sodium chloride. NH 4 0H(aq;)=NH; (aq.) + oir (aq.); Mf =Q
The heat of solution of NaCl is very small as the heat of and NH; (aq.) + Cl- (aq.) + H+ (aq.) + OH- (aq.)=NH; (aq.) .
ionisation is nearly equalto the heat of hydration.
+ Cl- (aq.) + H20(l);
'1~~5 ENTHALPY OF NEUTRALISATION OR Mf =-13.7kcal
HEAT OF NEUTRALISATION or NH 4 0H(aq.) + H+ (aq.) + Cl- (aq.) =NH; (~q.)+ Cl- (aq.)

The heat of neutralisation (or enthalpy of neutralisation) is + H 2 °(l);


defined as the heat evolved or decrease in enthalpy when 1 gram Mf = -12.3 kcal
/
I
I
G.R.B. PHYSICAL CHEMIS"FR¥ FOR COMPETITIONS

So, Q-13.7::::-12.3 Since, heatis liberated, heat of neutralisation should be negative.


or Q:::: 13.7 - 12.3 1.4 kcal So, heat of neutralisation = - 13.68 kcal.
~l:E~lUple 37. Whenever an acid is neutralised by a base,
Hence, 1.4 kcal of heat energy is absorbed for ionisation of
the ·net reaction is
NH 4 0Hwhich get subtracted from 13.7 kcal. Thus, the observed
value of heat of neutralisation is not-13.7 kcal but -12.3 kcal. H+(aq.)+0I1(aq.)~H20(1); AJI=-57.lkJ
Determination·of Heat of Neutralisation Calculate the heat evolved for the following experiments:
The heat of neutralisation (I) 0.50 mole of HCI solution is neutralised by 0.50 mole of
of an acid or base can be, r----Thermometer NaOH solution.
easily detennined in the" (il) 0.50 mole of HN0 3 solution is mixed with 0.30 mole of
laboratory with the help of KOH solution.
polythene or polystyrene, (iiI) 100 mL of 0.2 M HCI is mixed with 100 mL of 0.3 M
bottle. Bottle is fitted with
rubber cork through which a
Polythene
bottle
.
NaOH solution .
(iv) 400 mL of 0.2 M HiS04 is mixed with 600 mL of 0.1 M

thennometer and ainstirrer


fitted as. shown are
the Fig. ijlllll~ Solution
KOH solution.
Solution: According to the reaction,
7.16. 100 mL each of acid and
H+ (aq.)+ OH (aq. )----7H20(1);AJI = -57.lkJ
alkali of equal normality are
taken in separate bottles. The when I mole of H+ ions 'and 1 mole of OH- ions are neutralised,
temperature of each solution 1 mole of water is fonned and 57.1 kJ of energy is released.
Fig. 7.16
is regularly recorded. When
constant temperature is attained, the alkali solution is added to the (i) 0.50 mole HCl == 0.50 mole H+ ions
acid solution. The mixture is quickly stirred and the maximum 0.50 mole NaOH 0.50 mole OH - ions
temperature attained is noted.
CalCulations: On mixing, 0.50 mole of water is fonned.
Heat evolved for the formation of 0.50 mole of water
Suppose, the initial temperature of acid and .base
= 57.1 x 0.5 = 28.55 kJ
Final temperature of soiution after mixing (ii) 0.50 mole HN0 3 == 0.50 mOle H + ions
Rise in temperature:::: T2 .:.. 11 0.30 moleKOH 0.30 mole OH - ions
For the purpose of simplicity, the heat capacity of the bottle i. e. , 0.30 mole of H + ions react with 0.30 mole of OH- ions to
may be neglected as it is very small in comparison to that of fonn 0.30 mole of water molecules.
solution. The specific heat capacity of the solution is assumed to
be the same as that of water. Heat evolved in the formation of 0.3 mole of water

Q = Heat change in. reaction = Mass of solution = 57.1 x 0.3 17.13 kJ

x sp. heat x (T2 -11 ) (iii) 100 mL of 0.2 M HCl will give
Q is the heat evolved by the neutralisation.
0.2 x 100J = 0.02 mole of H + ions
. ( 1000
Therefore, the enthalpy of neutralisation:::: x 1000 x .!
. 100 x
and 100 mL of 0.3 M NaOH will give
where, x =normality of the acid or base.
I:~riaple 36. 150 mL of 0.5 N nitric acid solution at 0.3 x 100) 0.03moleofOH- ions
25.35°C was mixed with 150 mL of 0.5 N sodium hydroxide 1000
solution at the same temperature. The final temperature was
recorded to be 28.77° C. Calculate the heat of neutralisation. of i.e., 0.02 mole ofH + ions react with 0.02 mole ofOH ions to'
nitric acid with sodium hydroxide. fonn 0.02 mole of water molecules.
Solution: Total mass of solution =150 + 150 300 g Heat evolved in the formation of 0.02 mole of water
Q= Total heat produced:: 300 x (28.77 25.35)cal ==0.02x 57.1= 1.142kJ
= 300 x 3;42 1026cal (iv) 400mLofO.2M H2 S04 will give

.
S?-
Heat of neutralisation ::::
150
x I 000 x I
0.5 x 0.2 x 4001 =O.l6moleof H + ions
1000 )
1026 I
== - - x 1000 x - ::: 13.68 kcal
150 0.5 and 600 mL of O. 1M KOH will give

CHEMICAL THERMODYNAMICS AND THERMOCHEMISTRY 441---"

x+ y
(~X6001
1 ... (i)
0.06moleofOH ions
1000 .) x x 13.7 + yx 12.7= 13.5
Solving eqs. (i) and (ii), we get
.... (ii)

i. e. ,0.06 mole of H + ions react with 0.06 mole of OH - ions to


form 0.06 mole of water molecules. x=0.8,y 0.2
Heat evolved in the formation of 0.06 mole of water x:y=4:1
0.06x 57.1 3.426kJ
3
7;1:~' ENTHALPIESOF PHYSICAL CHANGES
Example 38: 100 cm of 0.5 N HCI solution at 299.95 K (Phase Changes)
were mixed with 100 cm 3 0.5 N NaOH solution at 299.75 K in a
. thermos flask. The final temperature was found to be 302.65 K. Heat energy is involved whenever a physical state of a substance
Calculate the enthalpy ofneutralisation ofHCI. Water eqUivalent is changed. Some important enthalpies of physical changes are
ofthermos flask is 44 g. defined below:·
Solution: . The initial average temperature of the acid and the (i) Enthalpy of fusion: It is the enthalpy change in
base converting 1 mole of the substance from solid state to liquid state
at its melting point. The enthalpy of fusion equals latent heat of
299.95 + 299.75 = 299.85 K
fusion per gram multiplied by the molecular mass. The value of
2 enthalpy of fusion gives an idea about the strength of the
Rise in temperature (302.65 299.85) 2.80 K intermolecular forces operating in a solid. The values of
Heat evolved during l1eutralisation enthalpies of fusion for iomc solids are much more than
(100+ 100+ 44)x 4.184 x 2.8= 2858.5J molecwarsolids in which molecules are held together by weak
van der Waals' forces.
Enthalpy of neutralisation 2858.5 x 1000 x _.1_ Melting
100 0.50 H20(S)--~> H 29(l); AH = 1.44 kcal
57.17kJ Freezing
H 2 0(l) H 2 0(s); . AH(freezing) = -1.44 kcal
. Example 39. When a studen.t mixed 50 mLoflM HCI and
50 mL ofl M NaOH in a coffee cup calorimeter. the temperature (ii) Enthalpy of vaporization: It is the enthalpy change in
of the resultant solution increases from 2}o C to 27.5° C. . converting 1 mole of the substance' from liquid state to gaseous
Assuming that the calorimeter absorbs only a negligible quantity . state (or vapour state) at its boiling point.
of heat, that the total volume of solution is 100 mL, its density
I g mL- 1 and that its specific heat is 4.18 J / g. Calculate : H2 0(1) --~ H20{g); AH = 10.5 kcal
(a) the heat change during mixing, Condensation
(b) .the enthalpy change for the reaction, H 2 0{g) ---~) H 20(l); AH(condensation) = -105 kcal

HCI(aq.) + NaOH{aq. ) ~ NaCI(aq.) + H 20{aq. ) Enthalpy of vaporization of a liquid gives an idea about the
strength of intermolecular forces operating' between the
Solution: (a) Number of moles ofHCI and NaOH added
molecules Of the liquid. .
MV = I x 50 = 0.05 (iii) Enthalpy of sublimation: Sublimation is the process
1000 1000 in which a solid on heating is directly converted into vapour state.
Mass of mixture· = V x d 100x 1= 10~g It is the heat change in converting 1 mole of a solid directly into
Heat evolved, q ms AT = 100 x 4.18 x (27.5 - 210) its vapour at a given temperature below its melting point.
12(s)~ 12 (g); AH = 14.9kcal
= 100 x 4.18 x 6.5J = 2717 J =2.717 kJ
The enthalpy of sublimation of a solid is equal to the sum of
(b) The involved reaction is: enthalpy of fusion and enthalpy of vaporization.
HC1(aq.) + NaOH(aq.)~NaCl(aq.)+ H 2 0
AHsublimation AH fus + AH vap
AH Heat evolved per mol
-2.717 =-54.34kJ l<i11: HESS'S LAW (The Law of Constant
0.05 Heat Summation)
.LExample 40. The enthalpies of neutralisation of a strong This law was presented by Hess in 1840. According to this
acid HA and weaker acid HB by NaOH are -13.7 and -12.7 law, if a chemical reaction can be made to take place in a
kcal/eq. When one equivalent of NaOH is added to a mixture number of ways in one or in several steps, the total enthalpy
containing I eqUivalent ofHA and HB ; the enthalpy change was change (total heat change) is always the same, i. e., the total
-13.5kcal. In what ratio is the base distributed between HA and enthalpy~change is independent of intermediate steps involved
HB? .
in the change. The enthalpy change of a chemical reaction
So.lution: Let x equivalent ofHA and yequivalent ofHB are depends on the initial and final stages only. Let a substance A
taken in the mixture be changed in three steps to D with enthalpy change from A to
442 G.R.B. PHYSICAL. CHEMISTRY FOR COMPETITIONS

B Mfl calorie, from B to C Mf 2 calorie and from C to D Mf3 (ii) For the determination of enthalpies of extremely slow
calorie. Total enthalpy change from A toD will be equal to the reactions.
sum of enthalpies involved in various steps. (iii) For the determination of enthalpies of transformation of
Total enthalpy change one allotropic form into another.
Mf steps Mfl + Mf2 + Mf3 (iv) For the determination of bond energies.
Mfreaction =k Bond energies of reactants
Now if D is directly converted into A, let the enthalpy change
- k Bond energies of products
be Mfdirect. According to Hess's law Mf steps + Mf direct = 0, i. e. ,
(v) For the determination of resonance energy.
Mfsteps must be equal to Mf direct numerically but with opposite
sign. In case it is not so, say Mfsteps (which is negative) is more (vi) For the determination oflattice energy.
than Mfdirect (which is positive), then in one cycle, some energy Thermochemical calculations: For making thermochemical
will be created which is not possible on the basis of first law of calculations, the following points are kept in mind: .
thermodynamics. Thus, Mf steps must be equal to Mf direct (i) Write down the required thermochemical equation. For
numerically. example, if heat of formation of methane is to be
Hess's law can also be verified experimentally with the help of determined, write down the following equation:
following examples: C(s)+ 2H2 (g)= CH4(g); Mf=?
(i) Formation of carbon dioxide from carbon: (ii) Try to obtain the required equation from the given data.
First method: Carbon is directly converted into CO 2 (g). This can be done in two ways:
(a) By adding, subtracting and multiplying the various
C(S) + 02(g) = CO 2 (g); A H =-94.0kcal given thermochemic!l1 equations. Or
Second method: Carbon is first converted into CO(g) and (b) Heat of reaction:::: Total enthalpies of products - Total
then CO(g) into CO 2 (g ), i. e. ,conversion has been carried in two enthalpies of reactants
steps: Note: In case of calculation of heat of fonnation, the enthalpies of free
1 elements can be arbitrarily fixed as zero at 25 0 C and 1
C(s) + - 02 = CO(g); Mf = - 26.0 kcal
2 atmospheric pressure.
I \,.J!:~mple 41. Calculate the standard heat offormation of
CO(g) + - 02 = CO 2 (g ); Mf = - 68.0 kcal carbon disulphide (I). Given that the standard heats of
2 ,
combustion of carbon (s), sulphur (s)and carbon disulphide (I)
'Totalenthalpy change C(s) to CO 2 (g); . Mf:::: -94.0kcal are -393.3,293.7 and -1108.76 kJ mol -\ respectively.
(ii) Formation of ammonium chloride from ammonia and Solution: Required equation is
hydrochloric acid: C(s)+2S(s) CS 2 (1); Mfl =?
First method: Given,
NH3 (g) + HCI(g) = NH4 CI(g); "'Mf 42.2 kcal
NH4 Cl(g ) + aq. = NH4 CI( aq.); Mf "" + 4.0 kcal
C(s)+ °2 (g)---+ CO2(g) (Mf= -393.3 kJ) ...(i)
S(s) + 02(g)---+ S02(g) (Mf:::: - 293.72 kJ) ... (ii)
NH3 (g) + HCl(g) + aq. = NH4 CI(aq.); Mf :::: - 38.1 kcal
CS z(I) + 302 (g) CO 2(g)+ 2S02 (g) ...(iii)
Second method:
(Mf :::: -1108.76 kJ)
NH3 (g) + aq. = NH3 (aq.); Mf 8.4kcal
HCl(g) + aq. = HC1(aq.); =
Mf -17.3 kcal
First method: Multiply the eq. (ii) by 2.
NH3(aq·) + ijCl(aq. )=NH4 CI(aq.); Mf "" -12.3 kcal 2S(s) + 202 (g) 2S0 2 (g) ... (iv) (Mf:::: - 587.44 kJ)

NH3 (g)+ HC1(g) + aq.=NH 4 CI(aq.); Mf 38.0kcal Adding eqs. (i) and (iv) and subtracting eq. (iii),
[c(s) + 2S(s) + 302 (g) CS 2 (I) - 302 (g)
Conclusions
CO2 (g) + 2S02 (g)- CO 2 2S02 ]
(i) The heat of formation of compounds is independent of
C(s) + 2S(s) ---+ CS 2 (I)
the manner of its formation.' .
This is the required equation.
(ii) The heat of reaction is independent of the time consumed
in the process. . Thus, Mf( =-393.3 587.44+1108.76::::128.02kJ
. (iii) The heat of reaction depends on the sum of enth~pies of Standard heat of formation of CS 2 (l ) = 128.02 kJ
products minus sum of the enthalpies of reactants. Second method:
(iv) Thermochemical equations can be added, subtracted or C(s) + 02 (g) ---+ CO2 (g); (Mf:::: - 393.3 kJ) ... (i)
mUltiplied like algebraic equations. S(s) + 02 (g) ---+ S02 (g); (Mf:::: ~ 293.72 kJ) ... (ii)
Applications of Hess's Law CS 2 (I) + 302 (g) ---+ CO 2 (g) + 2S02 (g);
(Mf::; -1108.76 kJ) ... (iii)
(i) For the determination of enthalpies of formation of those
compounds which cannot be prepared directly from the From eqs. (i) and (ii),
elements easily using enthalpies of combustion of Enthalpy of CO 2 == - 393.3 kJ
compounds. Enthalpy of S02 "" - 293.72 kJ
I
CHEMICAL THERMODYNAMICS AND THERMOCHEMISTRY 443

Enthalpy of 02 = 0 (By convention) or C2 H z (g) + H2 (g)----7C 2H4 (g)


Mf of eq. (iii) = Enthalpies of products Mf == -68.3- 310.6- (-337.2)= -378.9+ 337.2=- 41.7kcal
- Enthalpies of reactants We know that,
-1108.76= 393.3 + 2 X (- 293.72) - MfcS].(l) Mf = AE + llnRT
MfCS2 (l) (1108.76- 980.74) = 128.02 kJ
or AE = Mf -llnRT
Enthalpy of CS 2 (I) = 128.02 kJ .
~;(>;"Example 42. Calculate the heat of formation of acetic lln (1 2)=-I,R=2xl0- 3 kcalmor1K- 1
acid from the following data: and T (25 + 273) = 298 K
CH3 COOH(I)+ 202 (g)----72C0 2 (g) + 2H20(l) ...(i)
Substituting the values in above equation,
(Mf = - 207.9 kcal)
C(s)+ O2 (g )----7C02 (g) (Mf = - 94.48 kcal) ...(ii) AE = - 41.7 - (-I)(2x 10-3 )(298)
1 =- 41.7+ 0.596=- 41.104 kcal
H 2 (g) + - O2(g ) ----7 H 20(1 ) (Mf == - 68.4 kcal) ... (iil)
2 :~~~Example 44. Determine the heat of transformation of
Solution: First method: The required equation is C(diamond) ~. C(graphite) from the following data:
2C(s) + 2H 2(g)+ 0z(g) "'CH 3 COOH(I); Mf =? C(diamond) + 02 (g) ----7 CO2 (g) ...(i)
This equation can be obtained by multiplying eq. (ii) by 2 and (Mf =- 94.5 kcal)
also eq. (iii) by 2 and adding both and finally subtracting eq. (i). ...(it)
C (gl'Ophite) + O 2 (g) ----7 CO2 (g)
[2C + 202 + 2H2 + 0z - CH3 COOH(I) - 202
(Mf = - 94.0 kcal)
----72C02 +2H 20-2C02 -2H20]
Solution: Subtracting eq. (ii) from (i), the required equation
MfCH,COOH(l) 2 x (-94.48)+ 2 x (- 68.4)- (- 207.9) is obtained.
-188.96- 136.8+ 207.9 Mf transformation = - 94.5 - (-94.0)
-325.76+ 207.9 -117.86 kcal = - 94.5 + 94.0'= 0.5 kcal
Second method: From eqs. (ii) and (iii) Example 45. Methanoi can be prepared synthetically by
Enthalpy of CO 2 = - 94.48 kcal heating carbon monoxide and hydrogen gases under pressure in
°
Enthalpy of H2 = - 68.4 kcal the presence of a catalyst. The reaction is
CO(g) + 2H2 (g)----7CH 30H(l).
Enthalpy of 02 = 0 (by convention)
Mf of eq. (i) = Enthalpies of products - Enthalpies of reactants Determine the enthalpy of this reaction by an appropriate
207.9 2x (-94.48)+ 2(-68.4)- MfCHj:OOH(I) combination of the following data:
1
= -188.96- 136.8+ 207.9
MfCHf:OOH(I) C(gl'Ophite) + - 02 (g) ----7 CO(g) ... (i)
=-325.76+ 207.9=-117.86 kcal 2
(Mf 110.5 kl mol-I)
~,~::Jh~~ple 43. Given the following standard heats of
reactions : C(graphite) + O2 (g) ----7 CO2 (g) ...(it)
(a) heat of formation of,. water = - 68.3 kcal, (b) heat of (Mf - 393.5 kl mol-I)
combustion ofC2 H 2 = -310.6kcal and (c) heat ofcombustion qf .1
ethylene 337.2kcal. Calculate the heat of the reaction for the H 2 (g ) + 2°2 (g) ----7 H zO(I ) ...(iii)
hydrogenation ofacetylene at constant volume and at 25° C.
(Mf =- 285.9 kl mol-I)
Solution: The required equation is
C 2 H z (g)+ H2 (g)----7 C Z H4 (g); Mf? CH 30H(I) + ~ 02 (g ) ----7 COz (g) + 2Hz0(l) ...(iv)
Given, 2
(Mf 726.6kl mo/- 1 )
(a) Hz(g)+ 1 0z (g)----7H z O(l) ...(i) (Mf 68.3kcal)
2 Solution: The required equation can be obtained in the
5 following manner:
(b) C z Hz (g) + 0z (g) ----7 2CO z (g) + Hz 0(1) ... (ii)
2 - eq. (i) + 2 eq. (iii) + eq. (ii) - eq. (iv) = CO 2(g) + 2H2 (g )
(Mf = - 310.6 kcal)
----7 CH30H(g)
(c) CZH4(g)+30z(g)----72C02(g)+2H20(l) ... (iii)
(Mf = - 337.2 kcal) So, Mf = 110.5- 571.8- 393.5+ 726.6=-128.2kJ mol- 1
The required equation can be achieved by adding eqs. (i) and (ii) r~::~~t:l!:iample 46. How much heat will be required to make
and subtracting (iii). 2 kg of calcium carbide (CaC 2 ) according to the following
C 2H2 (g)+ Hz (g)+ 30 z (g)- CZ H4 (g)- 30 z (g) reaction?
----72CO z + 2H z0(/) - 2C02 (g) - 2Hz 0(1) CaO(s) + 3C(s)~CaC2(s)+CO(g)
444 G.R.B. PHYSICAL CHEMISTRY FOR COMPETITIONS

The heats offormation ofCaO(s), CaC 2 (s) and CO(g) are bond. The energy required to. break the bond is termed bond
-151.6, -14.2 and -26.4 kcal respectively. dissociation energy. The more precise definition is:' .
The amount of energy required to break one mole of bond of a
Solution: Ml = L MI'f (products) - LMI'f (reactants)
. particular type between the atoms in the gaseous state, i.e., to
separate the atoms in the gaseous state under I atmospheric
=[Mtf (CaC2) + Mtf (CO) ] - [MfJ (CaO) + 3A H'f (C) ]
pressure and the specified temperature is called bond dissociation
=[-14.2- 26.4]- [-151.6+ 3x 0] energy. For example,
= -40.6+ 151.6= 111.0 kcal H-H(g)~2H(g); MI = + 433 kJmol- 1
For formation of 64 g of CaC2 111.0 kcal of heat is required. CI- Cl(g) ~ 2CI(g); MI = + 242.5 kJ mol- 1
So, heat required for making 2000 g of .
H - Cl(g) ~ H(g) + CI(g ); MI =+ 431kJ mol- 1
CaC2 111.0 x 2000 = 3468.75 kcal
64 I-I(g)~21(g); MI + 15.1kJmol- 1
H-I(g)~H(g)+ I(g); MI ~ + 299 kJ mol-'
"i.ij:lINFLUENCE OF TEMPERATURE ON
.. THE HEAT OF REACTION The bond dissociation energy of a diatomic molecule is also
called bond energy. However, the bond dissociation energy
OR
depends upon the nature of bond and also the molecule in which
KIRCHHOFF'S EQUATION the bond is p;resent. When a molecule of a compound 'contains
Let us consider a reaction occurring at constant pressure. Heat of more than one bond of the same kind, the average value of the
reaction at constant pressure may be given as: dissociation energies of a given bond is taken. This average
MI=Hp -HR bond dissociation energy required to break each bond in a
eompound is called bond energy.'"
Differentiating the above equation with respect to 'T' at
Consider the dissociation of water molecule which consists of
constant pressure, we get:

(d:t =(~; t (d;; t


two O--H bond.s. The dissociation occurs in two stages.
H20(g)~H(g)+ OH(g); MI = 497.8 kJ mol- 1
OH(g ) ~ H(g) + O(g); MI = 428.5 kJ.mol- 1
(Cp)p -(Cp)R =ACp .
or d(MI)= AC p dT The average of these tw() bonds dissociation energies gives the
value of bond energy of O-H.
Integrating above differential equation within proper limit, we
497.8+ 428.5 _I
get: Bond energy of 0 - H bond == == 463.15 kJ mol
2
T2 dMl = ACp rT2 dT
fT. JT. Sirni~arly, the bond energy ofN-H bond in NH3 is equal to

MlT2 MIT, ACp (T2 - T,) one-third of the energy of dissociation ofNH3 and those ofC-H
AN AN
bond in CH4 is equal to one-fourth of the energy of dissociation
T2 TI =AC p ofCH 4·
Tz - Tl · . 1664
Bond energy C-H=-- =416kJmol- 1
Above equation is Kirchhoff's equation. It is. used to 4
calculate heat of reaction at a temperatufe provided it is known at [CH4(g)~C(g)+ 4H(g); MI = 1664 kJrnol- 1 ]
another temperature.
Change in heat of reaction due, Ito per degree change in Application of Bond Energy
temperature is equivalent to heat capacity difference between 1. Heat of a reaction = L Bond energy of reactants .
reactant and product.' I
L Bond energy of products
Kirchhoff's equation at constant volume may be given as:
Note:· In case of atomic species, bond energy is replaced by heat of
AE -AE
T2 TI =AC v atomization.
. Tz -Tl 2. Determination of resonance energy: When a
compound shows resonance, there is considerable difference
'j;'~i. BOND ENERGY OR BOND ENTHALPIES between the heat of formation as calculated from bond energk~.::...,­
and that determined experimentally.
When a bond is formed between atoms, energy is released. Resonance energy. = Experimental or actual heat of formation
Obviously same amount of energy will be required to break the - Calculated heat of formation.
"'Bond energy isalsQcalled, theheatoffortnation of the bond from gaseous atoms constituting the bond with reverse sign.
H(g)+ Cl(g)~H~Cl(g); All =-431kJmoF 1
Bond energy ofH--:-Cl=.;.. (enthalpy offonnation) = - (- 431) =:; + 431 kJ mol-I
CHEMICAL THERMODYNAMICS AND THERMOCHEMISTRY 445

Solution:
,; :::::I_SOME SOLVED EXAMPLES\ I:::::: H

:Jji;Ex~mple 47. Calculate the Mf of the reaction, I


N=N + 3(H- H) ~ 2 N-H; MJ=?
H
I I
H
H-C-CI(g)~C(s)+ 2H(g) + 2C1(g)
I Mf = [Mf(N:=N) + 3 x Mf(H-H)] - [6Mf(N-H)]
Cl = 945.36+ 3x 435.0- 6x 389.0 =-83.64kJ
Bond energy for C-H bond and C-Cl bond are 415 kJ and Heat of formation of NH3 = Mf = - 83.64 = - 41.82 kJ mol-I
326kJ respectively. 2 2
Solution: Mf = Swnof bond energies of reactants "Example 51. Calculate the resonance ~nergy ofN 20 from
the following data:
- Swn of bond energies of products
=[2x (C-=-H) + 2x (C-CI)]- 0 Mfj of N 20 = 82kJ mol-I

(All the products are free atoms) Bond energy of N ==N, N =N, 0=0 and N =0 bonds is
946,418, 498and607 kJmol-1 respectively. (lIT 1991)
= 2x 415+ 2x 326
= 830+ 652= 1482kJ . Solution: N=N(g)+ ~ (O=O)~ N=N=O(g)
2 '
'3;~: :E:i~niple 48. Calculate the enthalpy of the following'
reaction: 1
Mf !(NfJ) = [Mf(N==N) + '2 Mf(O=4))] - [Mf(N=4)) + Mf(N=N)]
H2C=CH2(g)+H~(g)~CH3 -CH3(g)
1
The bond energies of C-H, C-C, C = C and H-H are 99, = (946+ - x 498) - (607 + 418)
2
83,147 and 104 kcal respectively.
= 946+ 249-607- 418
Solution: The reaction is:
H H H H = 1195 - 1025 = 170 kJ mol-I

I I I I Resonance energy = Observed heat of formation


C=C(g)+H-H(g)~H-C-C~H(g); Mf=? ~ Calculated heat of formation
I I I I = 82~ 170= 88 kJ mol-I
H H H H
{£t'F~~llmple 52. The enthalpies for the following reactions at
Mf = Sum of bond energies of reactants 25°C'are given:
- Swnof the bond energies of products 1
= [Mfc==c +4xMfC_H + MfH- H']
'2 H2 (g)+ '21 02(g)~OH(g); Mf= 10.06kcal'

- [Mfc-{; + 6x MfC- fI ] H2 (g)~ 2H(g); Mf= 104.18kcal


= (147+ 4x 99+ 104)- (83+ 6x 99)=-30kcal O2(g) ~ 20(g); Mf = 118.32 kcal
~-i'i{f~'~3mple 49. The bond dissociation energies of gaseous
Calculate the O-H bond energy in the OH group.
ii;,CI2 and HCI are 104, 58 and 103 kcal mol-I respectively. Solution: Required equation is
Calculate the enthalpy offormation ofHCI(g ). H(g)+ O(g)~O-H(g); AH=?
Solution: The required equation is Given:~ H2 (g) + ~ O2(g) ~ OH(g); Mf = + 10.06kcal
iI' ' 2 2
- H2(g)+- CI2(g)~ HCl(g); Mf=? 1
2 2 H(g) ~ '2 H2 (g); Mf = - 52.09 kcal
1 1 ,
Mf,= [- MfH- H + - Mfcl-{;l] - [MfH-{;I] 1
2 ' 2 O(g)~'2 °2(g); Mf= ~59.16kcal

=~ xl04+~ x 58-103=-22kcalinol- 1 Adding, H(g) + O(g) ~ OH(g); Mf = -101.l9kcal


2 2
~'¥;fEiainple53. The standard molar enthalpies offormation
I~~~i:x~ple 50. Calculate the enthq.lpy of formation of ofcyclohexane (l)and benzene (I) at 25°C are -156and + 49kJ
ammonia from the following bond energy data: ' , mol -I respectively. The standard enthalpy of hydrogenation of
(N - H) bond = 389 kJ mol -"I; (I:( ~ H) bond = 435 kJ cyclohexene (l ) at 25° C is -119 kJ mol -I. Use these data to
mo/- I and (N=:N)bond= 945.36kJ mol-I. estimate the magnitude of resonance energy of benzene.
(lIT 1996)
446 G.R.B. PHYSICAL CHEMISTRY FOR COMPETITIONS

Solution: MIl (ClHg) 2 x (- 94.0) + 4 x (- 68.0) + 530.0 == 24 kcal


Cyclohexene(l) + H2 (g) ---7 Cyclohexane(l); MI 119 kJ
Enthalpy of fo~tion of cyclohexane( I) = -156 kJ mol-I 2C(s) + 3H2(g)---7 C2H 6 (g); 'MI= 20.0
SO,enthalpy of fonnation of cyclohexane = - 156 - (-119) kJ .2C(g)---7'2C(s); MI 344.0
=-37kJmol- 1 6H(g ) ~ 3H2 (g ); MI -312.0
Since, MI cyclohexane is -156 kJ rnol- I ,we can say that for every Adding 2C(g) + 6H(g) ---7 C2H6 (g ); MI - 676 kcal
double bond the energy decreases by an amount +119kJ rnol- I
So, enthalpy of formation of 6C-B bonds and one C-C bond
and therefore for the introduction ofthree double bonds (present is':'" 676.0 kcal. .
in benzene ring) the energy required
3C(s) + 4H2(g)---7 C3 Hg(g); MI.= 24.0 •
= 3 x 1I9 kJ rnol- I 357kJ rnol- I
3C(g)---73C(s); MI 516.0
Hence, theoretical MIf for benzene = (357 156)kJ rnol- I
8H(g)---7 (g); MI = -416.0
201 kJ rnol- 1
Adding 3C(g) + 8H(g) ---7 C 3 Hg (g); MI 956.0 kcal
Resonanceenergy =Theoretical MIf Observed MI f So, enthalpy of formation of 8C-H and 2C-C bonds is
(201~49)kJrnol-1 =152kJrnol- 1
-956kcal.
:':"'Ex~mple 54~ Calculate heat ofcombustion of ethene: Let the bond energy of C-C be x and of C-H be y kcal.

H) <H C=C' +30=0---720=C=0+2H-0-H


In ethane
In propane
x + 6y == 676
2x+ 8y 956
H H
On solving, x = 82 and y = 99
from bond energy data: C =C, C 0====0, 0- H
Thus, bond energy of C- C 82 kcal and
BE kJ mol-I: 619 414 499 724 460
bond energy of C- H = 99 kcal
Solution: Example 56. Using the bond enthalpy data given below,
MI Smn of bond energies of reactants calculate the enthalpy offormation of acetone (g).
, - Smn of bond energies of products Bond energy C - H 4114 kJ mol -I;
::: [MI(c=(;) + 4MI(C_H) + 3 x MI(o=o) ] Bond energyC -C 347.0kJ moZ- I ;
-:- [4 x MI(c=o) + 4 x M/(O-H) ] Bond energy C =0:::: 728.0 kJ mol-I;
[619+4 x 414 + 3 x 499] [4 x 724 + 4 x 460] .
Bond energyO=O= 495.0kJ mol-I;
=.:... 964 kJ rnol- I
Bond energy H - H == 435.8 kJ mol -I;
l~~;,(~xample 55. Using the data (all values are in kilocalorie
per mole at 25° C ) given below, calculate the bond en,ergy of MIsubC(S) 718.4 kJ marl
C-C and C-H bonds.
Solution:
MI;ombuslion of elhane == 372.0 3C(g) + 6H(g) + O(g) ---7 CH3 COCH3 (g )
MI;ombuslion of propane 530.0 °
In acetone, six C-H bonds, one C= bond and two C--C
bonds are present. Energy released in the formation of these
MIO jOrC(graphite) ---7 C(g) = + 172.0 bonds is
Bond energy of H-H bond +104.0 :::: -6x 413.4- 728.0- 2 x 347.0= - 3902.4 kJ rnol- 1
f
MI of H 20(1 )=-68.0 The equation of the enthalpy offormation of acetone is
, I . ?
MIl of CO2 (g) = - 94.0 3C(graphite) + 3H2(g)+ '2 02 (g) ---7 CH 3COCH3 (g); MI:=.

Solution: This equation ,can be obtained from the following equations


7 by adding:
C2 H6 (g) + '2 02 ---72C0 2 (g ) + 3H2 0(l); MI 372.0 3C(g) + 6H(g) + O(g) ---7 CH3 COCH3 (g);
MI = - 3902.4 kJ rnol- 1
AH f (C2 H6) == 2x (-94.0)+ 3x (-68.0)+ 372.0= -20kcal
3C(s)---73C(g); MI = 2155.2kJ rnol- I
, 1
C3 Hg (g) + 502 - : 3C0 2(g) + 4H 20(l); MI == 530.0 3H2(g ) ---7 6H(g ); MI = 1307.4 kJ mol-
CHEMICAL THERMODYNAMICS AND THERMOCHEMISTRY 447
I
and - Oz(g)~O(g); ill = 247.5 kJ mol- l [Hint: I1Jlr:'ction = 211Jl;(HF) - 211Jl;(HCl)
2
-352.18 = 2 x (-268.3) - 2x
3C(s) + 3Hz (g)+..!. Oz(g)~ CH 3COCH 3 (g); 2x = 2(-268.3)+ 352.18
2
x = - 92.21 kJ moFl]
ill = -192.3 kJ mol- l
11. Given two processes: .
I
2
P4 (s) + 3Cl z (g) ~ 2PC1 3 (l); ill = - 635 kJ

7. MI; 298 K of methanol is given by the chemical equation: PC1 3 (l) + Cl z (g) ~ PCls (s); ill = -137 kJ
(AIIMS 2005) The value of Mlf of PC Is is:
I
(a) CH4(g) + - 0z(g) ------7 CH30H(g) (a) 454.5 kJ mol- l (b) - 454.5 kJ
2 . (c) -772 kJ mol- l (d) - 498 kJ mol- l
(b) C (graphite) + ..!. 0z (g ) + 2H2 (g ) -'-------7 CH30H(l) [Ans. (b)]
2 . 1 3 635
[Hmt: - P4(s) + - CI 2(g) ~ PCIs(l); I1Jl = - - kJ
(c) C (diamond) +..!. 02(g) + 2H2(g) ------7 CH30H(I) 4 2 2
2
PCI 3 (1) + CI 2(g) ~ PCIs(s); I1Jl = -137 kJ
(d) CO(g) + 2H2(g) ------7 CH30H(l)
[Ans. (b)]
[Hint: In the process of formation, the compound must be On adding,.!. P4(s) + ~ CI 2(g) ~ PCls(s); I1Jl = -454.5 kJ]
4 2
formed from constituent elements in their standard state.]
8. The standard molar heat of fonnation of ethane, CO 2 and H2 ° 12. The enthalpy of combustion at 25°C of H2, cyclohexane
(I ) are respectively -21.1, -94.1 and --68.3 kcal. The standard (C6H12 ) and cyclohexene(C6HIO) are -241, -3920 and
molar heat of combustion of ethane will be: -3800 kJ/mol respectively. The heat of hydrogenation of
(a) -372 kcal (b) -240 kcal cyclohexene is: [CBSE (Med.) 2006]
(c) 162 kcal (d) 183.5 kcal (a) -121 kJ/mol (b) +121 kJ/mol
[Ans. (a)] (c) -242 kJ/mol (d) +242 kJ/mol
, 7 [Ans. (a)]
[Hint:. C2~(g) + - 02(g) ~ 2C0 2(g) + 3H20(g)
2 [Hint: H2(g)+.!.02(g)~H20(1) (11Jl=-241kJ) ... (i)
2
I1H = 2I1Jlf (CO z) + 3I1H.f(HzO) - I1JlJ (CZ H6) 17
= 2(-94.1) + 3(-68.3) -
(-211) = - 372 kcal] C6HlO + "2 °2(g) ~ 6C0 2(g) + 5H 20(l) ... (ii)

9. When ethyne is passed through a red hot tube, then fonnation (11Jl = - 3800 kJ)
of benzene takes place: C6Hl2 + 90 2(g) ~ 6C0 2(g) + 6H 20(I) ... (iii)
l (11Jl = - 3920 kJ) .
ill; (C2H 2)(g) = 230 kJ mol-
Eq. (i) + eq. (ii) - eq. (iii) gives
l
ill; (QjH 6)(g) = 85 kJ mol- I1Jl = - 241- 3800 - (-3920) =-121 kJ
Calculate the standard heat of trimerisation of ethyne to for C6HlO + H2 ~ CJI12]
benzene. 13. Given that:
3CzH z (g) ~ C 6H6 (g) 2Fe(s) + ~ 0z (g) ~ Fe203 (s) (ill =-193.4 kJ) ... (i)
(a) 205 kJ morl (b) 605kJ mol- l 2
(c) - 605 kJ mol- l (d) -205 kJ mol- l Mg(s)+..!. 0z(g)~MgO(s)(ill=-140.2kJ) ... (ii)
[Ans. (c)] 2
[Hint: I1Jlreaction = 11Jl;(Q, H6) - 311Jl;(CzH z) What is MI of the reaction?
3Mg + Fe Z 0 3 ~ 3MgO+ 2Fe [JEE (Orissa) 2005)
= 85 - 3(230)
(a) ~227.2 kJ (b) -272.3 kJ
= - 605 kJ mol-I] (c) 227.2 kJ (d) 272.3 kJ
10. F2 (g ) + 2HCI(g ) ------7 2HF(g) + Cl 2(g ); [Ans. (a)]
[Hint: Subtracting equation 1st from 2nd multiplied by 3,
MfO = - 352.18 kJ 3
2Fe(s)+ "2 0z(g) ~ Fez0 3(s)
Mf; (HF) = - 268.3 kJ
3
(11Jl ; -193.4 kT) ... (i)

The heat of fonnation ofHCl will be: 3Mg(s) +"2 02(g) ~ 3MgO(s) (11Jl = -420.6kJ) ... (ii)
(a) -22 kJ mol- l (b) 88 kJ mol-I
Subtracting eq. (i) from (ii),
(c) -92.21 kJ mol- l (d) -183.8 kJ mol- l
[Ans. (c)] 3Mg(s) + Fez03 ~ 3MgO + 2Fe; I1Jl = -420.6 - (-193.4)
= -227.2 kJ]

448 'G.R.B. PHYSICAL CHEMISTRY FOR COMPETITIONS

14. Given that: 18. Calculate the enthalpy change when 50 mL of om M


. 2C(s) + 202(g) ~ 2C02(g) (Ml 787 kJ) ... (i} Ca(OH)2 teactswith 25 mLofO.Ol MHCI. Given that Mlo
1 neutralisation of a strong acid and a strong base is 140 kcal
H2(g}+-02(g}~H20(l} (Ml -286kJ) ... (ii} mol-I.
2 '.
(a) 14 kcal(b)35 cal (c) to cal (d)7.5 eal
~H2(g}+~. O~(g}~ 2C02 (g} + H20(1) ... (iii} [Ans. (b)]
(LVI = -1301kJ) 0.01 x 25
[Hint: Number of moles ofHCI
Heat offOlmation of acetylene is: [JEE (Orissa) 2005J 1000 1000
(a)-1802kJ (b)+1802kJ == 25 X 10-5
(c) -800 kJ (d) +228 kJ
HCl~H+ +Cl-
[Ans. (d)]
[Hint: Required equation is: n + == 25 X 10-5
H
2C(s) + H 2(g) ~ C2Hig)
Number of rooles of Ca(OH)2 == MV == 0.01 x 50 = 50 X 10-5
Eq. (i) + eq. (ii) eq. (iii) gives 1000 1000
tili (-787) + (-286) - (-1301) n' = 2 x 50 X 10-5 = 10-3
OH-
== + 228 kJ]
The enthalpy changes for two reactions are given by the In the process of neutralisation' 25 x 10-5 mole H+ will be
15.
equations: completely neutralised
3 .. tili = 140 x 25 x 10-5 keal = 0.035 kcal =35 cal]
2Cr(s)+ O (g)---7 Cr2 0 3 (s); LVI =-1130kJ
2 2 . 19. Equal volumes of I MHO and I M H2 S04 are neutralised by
I
C(s) + "2 O2 (g) ---7 CO(g ); m=-llOkJ . I M NaOH solution and x and y kJ/equivalent of heat are
liberated respectively. Which of the following relations is
What is the enthalpy change in kJ for the following reaction? correct?
3C(s) + Cr2 0 3 (s)---7 2Cr(s) + 3CO(g) (a)x=2y (b)x=3y (c)x 4y (d)x=Xy
[Ans. (d)]
. (a)-1460kJ(b)-800kJ (c)+800kJ (d)+1020kJ [Hint: Since. H2S04 gives 2 moles W while HCl gives 1
(e) +1460 kJ mole H+ from 1 mole after ionisation. Hence, H2S0 4 will
[Ans. (c)] release double amount of heat as compared to HC1.
3 . Y
[Hint: 3C(s) + - 02(g) ~ 3CO(g); Ml -330kJ i.e., y=2xorx=. ]
2 2
tili = + 1130 kJ 20. Which of the following acid will release maximum amount
of heat when completely neutralised by strong base NaOH?
On adding, 3C(s) + Cr203(S) ~ 2Cr(s) + 3CO(g); (a) 1 MHCI (b) 1 MHN0 3
tili == 800 kJ] . (c) I MHC]04 (d) 1 MH 2S0 4
[Ans. (d)]
16. The enthalpy change Ml for the neutralisation of I M HCI by [Hint: Ionisation of H2S0 4 gives double amount ofW ions as
caustic potash in dilute sQlutiQn at 298 K is: (DPMT 2005) compared to other acids.
(a) 68 kJ (b) 65 kJ
H2S0 4 ~ 2H~ + SO~- ]
(c) 57.3 kJ (d) 50 kJ
[Ans. (c)] 21. Detennine the heat of the following reaction:
(Hint: Since. both HCl and KOH are strong. 57.3 kJ heat will . FeO(s) + Fe203 (s) ---7 Fe 3 0 4 (3)
be released.] Given informations:
17. . Enthalpy ofneutra1isation of the reaction between CH 3COOH 2Fe(s) + 02(g)~ 2FeO(s); Mlo =-544 kJ
(aq. ) and NaOH( aq. ) is -13.2 keal eq -I and that of the reaction 4Fe(s)+302(g)~2Fe203(s); Mlo= 1648.4kJ
Fe304(s)~ 3Fe(s)+ 202(g);Mlo +'11l8.4 kJ
between H2 S04 (aq.) and KOH(aq.) is -13.7 keal eq-I. The
(a)-1074kJ (b)-22.2kJ (c)+249.8kJ (d)+2214.6kJ
enthalpy of dissociation of CH3COOH( aq.) is:· [Ans. (b)]
(a) -0.5 keal eq-I (b) +0.5 kcal eq-l [Hint: 2feO(s) ~ 2Fe(s) + 02(g); IlW.; + 544 kJ
(c)~26.9 kcal eq-I . (d) +13.45 kcaleq-l
[Ans. (b)] 2F~03(S) ~ 4Fe($) + 30 2(g); 8.W = + 1648.4 kJ
[Hint: Dissociation enthalpy of CH 3COOH == 13.7 -13.2 6Fe(s) + 40 2(g) ~ 2F~04(S); /ilie = ,::,1xII18.4 kJ
= 0.5 kcal eq -1. Thus; 0.5 kcal eq -I heat will be used to On adding,
dissociate CH 3COOH completely.]
2FeO(s) + 2F~03(S) ~ 2F~04(S);

:. FeO(s) + F~03(s) ~ F~04(S); tili° = - 222 kJ]


CHEMICAL THERMODYNAMICS AND THERMOCHEMISTRY 449

22. Calculate the enthalpy of fonnation of I1Hf for C2 HsOH 26. The bond energies of C= C and C-C at 298 K are 590 and
from tabulated data and its heat of combustion as represented 331 kJ mol-1 re~pectively. The enth~lpy of polymerisation
by the following equations: per mole of ethylene is:
H2 (g)+ ~02(g) H 20(g) ... (i) (a)-70kJ (b)-12kI (c)12kI (d)-68kJ
(MIo -2418kJ mol-I) [Ans; (b)]
cO 2 (g) ... (ii) [Hint: The polymerisation of ethene may be represented as
(l1Ho 393.5 kJrnol- l ) nCH2 CHz -f-CH z -- CH z +/1
3H2 0(g)+2C02 (g) ... (iii) one mole ofC C bond is decomposed and two molt;s ufC----C
(MIo = -1234.7 kJ) bonds are formed per mole of ethene.
(a) -2747.1 kJ mol-I (b) -277.7 kJ mol- 1 :. AH = 590 2 x 331 -72 kJ per mol of ethylene.]
(c) 277.7 kJ mol- 1 (d) 2747.1 kJ mol-I 27. For the reaction,
[ADS. (b)] 2H2(g)+02(g) 2H2 0(g); 11H=-571kJ
[HiDt: Required equation: bond energy of (H-H) 435 kJ; of (0==0) = 498 kJ; then
2C(s) + 3H2 (g) + Yz02(g) C2H sOH(l) calculate the average bond energy of (O-H) bond using the
above data. (DeE 2005)
2C(s) + 20 2(g) 2CO z(g); AHO = -2 x 393.5kJ (a) 484 kJ (b) - 484 kJ (c) 271 kJ (d) -271 kJ
3 [Ans. (a)]
3H2(g) +
2
02(g) ---dHz0(g); AHo
.
=- 3 x 241.8 kJ
[Hint: 2(H--H) + 0 =0 2(H-O-H)
3HP(l) + 2C02(g) ---7 C2H sOH(l) + 30 2 (g); I1H ~(BE)reaclants ~(BE)prndl1CtS
AHo ::: + 1234.7 kJ -571 == [2 x 435 + 498] 4 x (BE)o_H

On adding, (BE)o_H 2 x 435 +4498 + 571 "" 484 kJ]

2C(s) + 3H2 (g) + I 02(g) ---7 CzHsOH(I); 28. Use the bond energies in the table to estimate I1H tor this
2 reaction:
AHo = -277.7 kJ mol-I CH2= CH2+ CI 2 ---7 CtCH2 - tHCl 2
23. . Which of the following methods for calculation of heat of a Bond. C-C C=C C-Cl C-H O-Cl
reaction is not correct?
Bond energy 347 612 331 414 243
(a) I1H reaction ~ I1HJ products ~ 11H,. reactants
(kJ/mol)
(b) I1H reaction ~(BE)reacrants ~(BE)producls
(c) I1H reaction ~l1Hcomb. (reactants) ~l1Hcomb. (products) (a) I1H = - 684 kJ (b) I1H =-154 kJ
(d) I1H reaction ~solution (reactants) ~11Hsolution (products) (c) I1H =+ 89 kJ (d)11H +177kI
[ADS. (d)] [Ans. (b)]
24. Heat of combustion of diamond and graphite are -94.5 kcal [Hint:
and -94 kca1!mol respectively. What will be the heat of AHreaction ~(BE)rcactant' - ~(BE)prnducts
transition from diamond to graphite? [(BEk=c + 4(BEk_H + (BE)Cl_C1]
(a) 0.5 kcal (b) +0.5 kcal (c) 1 kcal (d)-1 kcal
- [4(BEk_H + 2(BE b_Cl + (BE)c_c]
[Ans. (b)]
[(BE)c=c + (BEkl_C1] - [2(BEk_cl + (BEk_cl
[Hint: AH AH ""mb. IdialOOnd) - AH comb. (graphite)]
[612+243] [2x331+347]=-154kJ]
25. For the reaction,
~9. Heat of tormation of 2 moles of NH] (g J is -90 kJ; bond
3N zO(g) + 2NH3 (g) ---7 4N2 (g ) + 3H2 0(g);
M-fO = -: 879.6 kJ energies of H-H and N-H bonds are 435. kJ and 390 kJ
mol-I respectively. The value of the bond energy of N == N
If 45.9 kJ rnol- I ;
will be:
I1Hj[ H 20(g)] -:24L8kJrnol- 1 (a) 412.5 kJ (b) - 945 kI (c) 412.5 kJ (d) 945 kJ mol-I
then I1Hj[N20(g)] will be: [Ans. (d)]
H
(a) +246 kJ (b) +82 kJ' I
(c) -82 kJ (d) -246 kJ [Hint: N N + 3(H--H)---72N--H ;
I .
AH == -90 kJ
[Ans; (b)] I
H
AHrcaction = ~(BE)reactants ~(BE)products
-90 [(BE)N=rN + 3(BE)H __ H]- [6(BE)N_H 1
-90 x + 3 x 435 - 6 x 390
+ 2 AH/ [NH 3 1} ] x 945 kJ mol- 1 ]
450 G.R.B. PHYSICAL CHEMISTRY FOR COMPETITIONS

30. If values of Mf; of ICI(g ), Cl(g) and I(g) are respectively H2(g)+ ~02(g)~H20(l) =-286kJ) ...(ii)
17.57,121.34, 106.96 J mol-I. The value ofI-C1 (bond CzH2(g)+ 2~02(g)~ 2C02 (g)+ Hz0(l) ... (iii)
energy) in J mol- 1 is: (Mf -131OkJ)
(a) 17.57 (b}210.73 (c) 35.l5 (d) 106.96 The heat offormation of acetylene is: (VITEEE 2007)
[Ans. (b)] (a) 1802 kJ mol- 1 (b) + 1802 kJ mol- 1
[Hint: The process will be: (c) + 237 kJ moI- 1 (d) - 800 kJ mol- 1
I(g) + Cl(g) ~ I-Cl(g); Mfo =17.57 J morl [Ans. (c)]
Mf = Heat of atomisation of I(g) and Cl(g) [Hint: Required equation is:
- Bond energy of I-CI bond 2C(s) + H 2 (g) ~ C~2(g)
17.57 = 12L34 + 106.96 - x It can be obtained by adding eqs. (i) and (ii) and then
x = 210.73 J mol-I] subtracting eq. (iii) from it.
31. Calculate the Mf in joules for: Heat of formation of acetylene
C (graphite) ~ C (diamond) = (-787) + (- 286) - (-1310)
from the following data: +237kJmol- l ]
C (graphite) + 02 (g ) ~ CO2 (g); MfO = - 393.5 kJ
. C(diamond) + 02(g)~ CO2(g); Mfo = - 395.4 kJ .7.~0 DETERMINATION OF LATTICE
[CET (J&K) 2006) ENERGY (Born-Haber Cycle)
(b) - ~8.9 x 10
3
(a) 1900
3 Lattice energy of an ionic compound is defined as the amount of
(c) 190000 (d) +788.9 x 10
energy released when one mole of the compound is formed by the
[Ans. (a)] interaction of constituent gaseous cations and gaseous anions.
[Hint: C (graphite) + 02(g) ~ CO 2 (g) ; MfO = - 393.5 kJ
A+(g)+B-(g)~A+B- + Energy
CO 2 (g) ~ C (diamond) + 02(g); MfO =+ 395.4 kJ I mole (Lattice energy)

It is represented by the symbol, U. It is given negative sign as


On adding, C (graphite) ~ C (diamond); Mf 0 = + 1900 J
the energy is always released. Since, it is difficult to find the
= + 19kJ]
lattice energy by direct experiment, it is generally calculated by
32. The enthalpy changes for the following processes are listed indirect method known as Born-Haber cycle which is based on
below: Hess's law. The cycle can be easily explained by taking the
CI 2 (g) ~ 2CI(g); Mf = 242.3 kJ mol-I example of the formation of sodium chloride (NaCl). The form-
12(g) ~ 21(g); Mf = 151.0kJ mol-I ation of sodium chloride can be schematically represented as:
ICI(g) ~ I(g)+ CI(g); Mf = 211.3 kJ mol-I
U
Mf = 62.76 kJ mol-I NaCI o f i ( - - - - - - Na+ +

~
Given that, the standard states for iodine and chlorine are
12 (s) and Cl 2(g ), the standard enthalpy of formation for ICI
(g) is :
(a) -14.6 kJ mol- 1 (b) -16.8 kJ morl
(AIEEE 2006)
lp EA
(c) +16.8kJmol- 1 (d) +244.8kJ mol- 1 1 Na(s) SUblimation) Na(g)
2 CI12(9) AHsub
[Ans. (c)]
[Hint: The reaction is:
1 1 . Dissociation
12(8) + Cl 2(g) ~ IC1(g) 1.....-----=~=-----CI(g)
2 2 ~D
1 1
tJ.f H I _ C1 =[ 2 tJ.H'2(S) -> 12(g) + 2 tJ.H 1_ 1
Fig. 7.17

+ ~ tJ.H C1 - Cl J-[tJ.H I_ CI ] The formation of sodium chloride can occur either by direct
combination of sodium(s) and chlorine(g ) or in various steps.
1 I 1
=[-x62.76+ x151+-x242.3]-[211.3] Let the heat of formation of sodium chloride by direct
2 2 2
combination be -Q.
= 16.73 kJ mol-I]
The various steps in the formation of NaCl are the following:
33. Given that: In each step, either energy is absorbed or released.
2C(s)+02(g)~2C02(g) (Mf=-787kJ) ... (i)
CHEMICAL THERMODYNAMICS AND THERMOCHEMISTRY 451

Steps involved Energy change Solution: Applying the equation


1
(a) Solid sodium changes into gaseous sodium. +AHsub -Q Misub +-D+IP-EA +U
Energy is absorbed. It is sublimation energy. 2
Na(s) + sublimation energy ~ Na(g) and substituting the respective values,
(b) Gaseous sodium atoms change into gaseous +IP - 410 10S.S + 1 x 242.7 + 493.7 - 36S.2 + U
sodium ions. In this step, energy equivalent 2
to ionisation potential is absorbed. U -765.65kJmol- l
Na(g) + IP~Na+ (g) + e

(c) Half mole of molecular chlorine dissociate I 7~21 EXPERIMENTAL DETERMINATION OF


+-D
into one mole of gaseous atomic chlorine. In 2 THE HEAT OF REACTION
this step, energy equivalent to one half of the
The heat evolved or absorbed in a chemical reaction is measured
dissociation energy is absorbed.
1 1 by carrying out the reac~on in an apparatus called calorimeter.
"2 CI 2 (g) + "2 D = CI(g) The principle of measurement is that heat given out is equal to
heat taken, i. e. , , .
(d) Gaseous chlorine atoms change into chloride -EA Q (W+m)xsx(T2 -Tl ),
ions by acceptance of electrons. In this pro-
where, Q is the heat of the reaction (given out), W is the water
cess, energy equivalent to electron affinity is
released. equivalent of the calorimeter and mis the mass of liquid in the
Cl(g) + e~ cr-(g) + EA calorimeter and s its specific heat, T2 is the final temperature and
T, the initial temperature of the system. Different types of
(e) Sodium and chlorine ions are held together u calorimeters are used but two of the common types are:
by electrostatic forces to form Na+ cr. The (i) Water calorimeter and
energy equivalent to lattice energy is re- (ii) Bomb calorimeter
leased.
(i) Water calorimeter
Total energies involved in the above five steps
It is a simple form of a calorimeter which can be conveniently
Mi sub + 1 D + IP - EA + U used in the laboratory. It is shown in Fig. 7.IS.
2
It consists of a large vessel A in which a calorimeter B .is held
Thus, according to Hess's law on corks. In between the calorimeter and the vessel, there is a
I packing of aninsulati~g material such as cotton wool. Inside tne
- Q Mi sub + 2. D + IP - EA + U
calorimeter there are holes through which a thermometer, a stirrer
Example 57. Calculate the lattice energy for the reaction, and the boiling tube containing reacting substances are fitted. A
Li + (g)+ CI- (g) -tLiCI(s) known amount of water is taken in the calorimeter. Known
from the following data:
Thermometer
Misub(LI) 160.67 kJ mol-I; ~ D(CI2 )= 122.17 kJ mol-' Stirrer .
. 2
l
IP(Li) = 520.07 kJ mol-'; EA(CI)= - 365.26kJ mol-
and Mif(LiCl)= -401.66kJ mol-i.
Solution: Applying the equation
Q Mi+~D+IP-EA +v
and substituting the respective values,
-401.66 160.67 + 122.17 + 520.07 - 365.26+U B
U 839.31kJmol-'
Example 58. When a mole of crystalline sodium chloride
is prepared, 410kJ ofheatis produced. The hea(ofsublimation of
sodium metal is ISO.Sk.l. The heat ofdissociation ofchlorine gas Fig. 7.18
into atoms is 242.7 k.l. The ionisation energy ofNa and electron
amounts of reacting substances are taken in the boiling tube. The
affinity of CI are 493.7 kJ and - 36S.2 kJ respectively. Calculate
heat evolved during the reaction will be absorbed by the water.
the lattice energy ofNaCl. .
The rise in temperature is recorded with the heip of thermometer.
The heat evolved is then calculated from the formula
(W + m) x s x (T2 - Tj )

-,.
452 G.R.B. PHYSICAL CHEMISTRY FOR COMPETITIONS

(ii) Bomb calorimeter i.e., AE = - 32QO.67 kJ nlUi


This is commonly used to find the heat of combustion of (ii) We know that, ll.H == AE + AnRT
organic substances. It consists of a sealed combustion chamber,
15
called a bomb. A weighed quantity of the substance" in a C6 Hs COOH(s) + - O 2 (g) ~ 7C02 (g) + 3H 2 0(l)
dish along with oxygen under about 20 atmospheric pressure 2
is placed in the bomb which is lowered in water contained An=7 7.5=-0.5;R=8.314xlO-3 kJK- 1 mol- I ;T=288K
in an insulated copper vessel. The vessel is fitted with a
stirrer and a sensitive thermometer. The arrangement is shown in Substituting the values in the above equation,
3
Fig. 7.19. . ll.H == - 3200.67 + 8.314 x 10- x (- 0.5) x 288

Ignition = - 3200.67 - 1.197 == -3201.867 kJ mol- 1


wires
Stirrer Example 60. A sample of 0.16 g CH 4 was subjected to
III----Thermometer
combustion at 27° C in a bomb calorimeter. The temperature of
the calorimeter system (including water) was found to rise by
0.5° C. Calculate the heat of combustion of methane at
(i) constant volume and (ii) constant pressure, The thermal
capacity of calorimeter system is 17.7 kJ K -I and
Heat insulating R == 8.314 J K -I mol-I, (liT 1998)
container Solution: (i) Heat of combustion at constant volume, AE
:::: Heat capacity of calorimeter system x rise in temperature
Mol. mass of compound
x-------~~--
Steel bomb Mass of compound
16
=17.7x 0.5x- = 885
0.16
i.e., AE:::: 885 kJ mol-1
(ii) CH 4 (g) + 20 2 (g) -? CO 2 (g) + 2H 2 0(l)
An=I-3::::-2,T 300K,R== 8.314 X 10-3 kJK- 1 mol- I
Fig. 7.19
ll.H AE + AnRT
The temperature of the water is noted and the substance is
ignited by an electric current. .After combustion, the rise in :::: - 885 + (-2) x 8.314 x 10-3 x 300
'temperature of the system is noted. The heat of combustion can :::: - 885 - 4.988 == -889.988 kJ mol-I
be calculated from the heat gained by water and calorimeter.
. Example 61. The heat ofcombustion of ethane gas is -368
Since, the reaction in a bomb calorimeter proceeds at constant
kcallmol. Assuming that 60% of heat is useful, how many m3 of
voluine, the heat of combustion measured is AE.
ethane measured at NTP must be burned to supply heat to convert
AE == (W + m)(T2 -1j )x s x M kcal 50 kg of water at 10° Cto steam at 100° C?
WI Solution: Heat required per gram of water
Where, M is the molecular mass of the substance and WI is the =(90+ 540)cal 630 cal
mass of substance taken. Total heat neede4 for 50 kg of water
ll.H can be calculated from the relation,
:::: 50x 103 x 630 cal
ll.H = AE + AnRT
As the efficiency is 60%, the actual amount of heat required
Example 59. 0.5 g of benzoic acid was subjected to 3
combustion in a bomb calorimeter at 15°C when the temperature :::: 50 x 10 x 630 x 100 == 52500 kcal
of the calorimeter system. (including water) was found to rise by 60
0.55°C. Calculate the heat of combustion of benzoic acid (0 at No. of mole of ethane required to produce 52500 kcal
constant volume and (if) at constant pressure. The thermal 52500 142.663 mole
capacity ofthe calorimeter including water was found to be 23.85 368
kJ. . Volume of 142.663 mole at NTP = 142.663 x 22.4
Solution: (i) Heat of combustion at constant volume, AE 3195.65 litre = 3.195 m 3
Heat capacity of calorimeter and its contents
.. . MoL mass of compound
x nse In temperature x . . 1~22' LIMITATIONS OF FIRST LAW OF
Mass of compound THERMODYNAMICS
23.85 x'0.55 x 122 3200.67 kJ The essence of first law of thermodynamics is that all physical
0.5 and chemical processes occur in such a way that the total energy
of the system and surroundings is constant. The law correlates the
CHEMICAL THERMODYNAMICS AND THERMOCHEMISTRY

various fonns of energy, i. e., their interconversion into one A process which has an urge 'or a natural tendency to occur
. another in exactly equivalent amounts. The law has stood the test either oj its own or after proper initiation under the given set oj
of time as no transfonnation has violated the principle of conditions is known as spontaneous process.
conservation of energy. However, the first law of Spontaneous process does not mean that it takes place
thennodynamics has a number of limitations. instantaneously. It simply implies that the process has an urge to
1. The law does not give any information about the take place and is practicaify feasible. The actual speed of the
direction in which flow of energy takes place. For process may vary from very low to extremely fast. The rusting of
example, if two systems A and B which are capable of iron is a slow spontaneous process while the neutralisation
exchanging heat are brought in contact with each other, reaction between an acid and an alkali is a fast spontaneous
the first law of thennodynamics will only tell us that one process. It is a matter of experience also that all natural processes
system loses energy and other system gains the same are spontaneous and are irreversible, i. e. , move in one direction
amount of energy. But the law fails to tell whether the only. The reverse process which can be tenned as non-
heat will flow from system A to B or from system B to A. spontaneous can be made to occur only by supplying external
In order to predict the direction of the flpw of heat, one energy. For example, water can be made to flow upward by the
more parameter, i. e., temperature is required. The heat use of some external agency or energy is always required to lift
actually flows from a system which has higher the ball from the ground. The natural (spontaneous) processes
temperature. The process continues till both the systems follow a non-equilibrium path and as such are irreversible but
attain the same temperature. The law fails to answer why these processes proceed up to establishment of equilibrium. At
heat energy does not flow from cold system to hot system equilibrium state, the process stops to occur any further. Heat
though the energy is conserved in this way also. flows from a hotter body to a colder body till the temperature of
2. The law does not explain why the chemical reactions do both bodies becomes equal; there is no further flow of heat and
not proceed to completion. we say that the system has attained equilibrium. Some of the
3. The law does not explain why natural spontaneous familiar examplesof spontaneous processes are listed below:
processes are irreversible. (a) Spontaneous processes where no initiation is required
4. The law does not contradict the existence of self-acting (i) Dissolution of sugar or salt in water
refrigerator. Sugar + water ~ Aqueous solution of sugar
5. The law does not contradict the existence of 100% Salt + water ~ Aqueous solution of salt
efficient engine. "
(ii) Evaporation of water from water reservoirs such as
6. The difference between spontaneous and ponds, lakes, rivers, sea, open vessels, etc.
non-spontaneous processes is insignificant in view of the
H20(l)~ H 20(g)
first law.
The answers to above limitations are provided by second law of (iii) Flow of heat from a hot body to a cold body.
thennodynamics. However, before we study this law, let us (iv) Mixing of different gases.
understand the terms spontaneous, entropy and free energy. (v) Flow of water down a hill.
(vi) Reaction between Hz (g ) and 12 (g )to fonn HI(g).
1~18:1 SPONTANEOUS AND H 2 (g)+ I2(g)~2HI(g)
NON-SPONTANEOUS PROCESSES
(vii) Reaction between nitric oxide gas and oxygen to fonn
One of the main objectives in studying thennodynamics, as far as nitrogen dioxide.
chemists are concerned, is to be able to predict whether or not a
reaction will occur when reactants are brought together under a 2NO(g) + 02 (g ) ~ 2NO z (g)
special set of conditions (for example, at a certain temperature, (viii) Melting of ice into water.
pressure and concentration). A reaction that occurs under the
H20(S)~H2°(l)
given set of conditions is called a spontaneous reaction. If a
reaction does not occur under specified conditions, it is said to be (ix) A piece of sodium metal reacts violently with water t()
non-spontaneous. fonn sodium hydroxide and hydrogen gas.
The term spontaneity means the feasibility of a process. In 2Na(s)+ 2H20(l)~2NaOH(aq.)+ H 2 (g)
nature, we observe many processes which occur of their own. For
example, water flows down the hill without the help of any (x) When a zinc rod is dipped in an aqueous solution of
external agency, heat flows from a conductor at high temperature copper sulphate, copper is precipitated.
to another at low temperature, electricity flows from high CuS04 (aq.) + Zn(s) ~ Zns0 4 (aq.) + Cu(s)
potential to low potential. There are processes which require
some initiation before they can proceed. But once initiated, they (b) Spontaneous processes where initiation is required
proceed by themselves. The burning of carbon, burning of fuels (i) Reaction between H2 and ° 2 : This reaction is initiated
and petrol, etc., require some initiation. These processes are by passing electric spark through the mixture.
Electric
tenned· as spontaneous on account of their feasibility, i. e. , these 2H2 (g) + 02 (g) ----+ 2H 20(/)
can occur without the help of external work. spark
454 G.R.B. PHYSICAL CHEMISTRY FOR COMPETITIONS

H2 (g) + ~ O 2(g ) ~ H2 0(I);


(ii) Coal bums in oxygen or air when ignited, Coal keeps on "Mf =- 286.2 kJ morl
burning on,?e initiated.
Ignition
C(s)+ 02(g)~COz(g) C(s)+ Oz(g)~C02(g); Mf = - 395 kJ morl
(iii) Methane burns with o~ygen when ignited. N2(g)+3Hz(g)~2NH3(g); Mf= 92.4kJmol- 1
CH4 (g) + 20 z.(g ) ~C02 (g) + 2H 20(l)
In exothermic reactions, heat is evolved from the system, i. e.,
(iy) A candle made up of wax burns only when ignited.
energy is" lowered. Thus, exothermic reactions occur
(v) Calcium carbonate evolves carbon dioxide upon heating.
" Heat" spontaneously on account of decrease in enthalpy of system
CaC03(s)~ C~0(S)+C02(g) (Mf=-ve).
A process which can neither occur by itself nor by initiation is Hence," it can be concluded that the negative value of Mf
called a non-spontaneous process. may be the criterion of spontaneity.
Or Limitations of the criterion for minimum energy
A process which has no natural tendency or urge to occur is
said to be a non-spontaneous process. (a) SpOiltaneous endothermic reactions or processes: . A
number of endothermic reactions and processes are known whIch
Few examples of non-spontaneous processes are:
are spontaneous, i. e. , when Mf is +ve. Some examples are given
(i) Flow of heat from a cold body to a hot body. below:
(ii) Dissolution of gold or silver in water. (i) Evaporation of water or melting of ice takes place by
(iii) Flow of water from ground to overhead tank. absorption of heat from surroun4ings, i. e., these processes are
(iv) Hydrolysis of sodium chloride. endothermic (t:Jl + ve).
(v) Decomposition of water into hydrogen and oxygen.
H20(s)~H20(1); Mf=+6.0kJrnOl- 1
(vi) Diffusion of gas from a low pressure to a high pressure.
It- is not always true that non-spontaneous processes do not H20(1)~H20(g); Mf +44kJmol- 1
occur at alL Many of the non-spontaneous processes or changes
(ii) The decomposition of CaC03 is non-spontaneous at room
can be made to take place when energy from some external temperature but becomes spontaneous when the temperature is
source is supplied continuously throughout the change. raised.
(However, the processes that occur spontaneously in one
direction cannot, under the same conditions, also take place CaC03(s)~CaO(s)+ CO 2 (g); Mf = + 178.3kJ mol- 1
spontaneously in the opposite direction). For example, the (iii) Compounds like NH4 CI, KCl, etc., dissolve in water by
decomposition of water into hydrogen and oxygen (non- absorption of heat from water. Temperature of the water
spontaneous process) can occur when electrical energy is decreases.
supplied to water. The process stops when the passage of
electrical energy is stopped. NH4 CI(s)+ aq. ~NH: (aq.)+ Cl-(aq.);
Drivi~g fone for a spontaneous process: After having Mf = + 15.lkJ mol- I
learnt about spontaneous processes, a very obvious question
arises in our minds that why some processes are spontaneous? (iv) Dinitrogen pentoxide (N z0 5 ) decomposes spontan~""
Obviously there must be some kind of driving force which is ously at room temperature into NO z and Oz, although the
i:esponsible for driving the process or a reaction in a particular reaction is highly endothermic.
direction. 2N205(g)~4N02(g)+02(g);
The force which is responsible for the spontaneity ofa process Mf = 219kJ mol- l
is called the driving fOl'Ce.
(v) The decomposition of HgO becomes spontaneous on
Let us now discuss the nature of the driving force. heating.
1. Tendency to acquire minimum energy: We know that,
lesser is the energy, greater is the stability. Thus, every system 2HgO(s) ~ Hg(l) + Oz (g);
tends to acquire minimum energy. For example: Mf = 9O.3kJ mol- 1
(i) Heat flows from high temperature to low temperature so (b) Occurrence of reversible reaetions: A large ni.unber of
that heat content of hot body becomes minimum. reactions are reversible in nature. In these reactions, both forward
(ii) Water flows down a hill or a slope to have minimum and backward reactions occur simultaneously in spite of the fuct
energy (potential). that one reaction is exothermic (Mf = ve) and other
(iii) A wound watch spring tends to unwind so that endothermic (Mf = + ve) .
mechanical energy of watch becomes minimum.
All the above processes are spontaneous because of a (i) H2 (g) + 12 (g ) ~ 2HI(g) ; Mf=-ve
tendency to acquire minimum energy. 2HI(g)~Hz(g).+ Iz(g); Mf=+ve
It has been observed that most of the spontaneous chemical
Hz (g)+ 12(g)~2HI(g)
reactions are exothermic. For example:
CHEMICAL THERMODYNAMICS AND THERMOCHEMISTRY 455

(ii) N2 (g) + 3H 2 (g) ~ 2NH3 (g); !JJ[= -ve spontaneously in a direction in which randomness qf the system
2NH3(g)~ N 2 (g)+ 3H 2 (g); !JJ[= +ve increases.
Another example of achieving more randonmess is the sugar
N 2 (g)+ 3H2(g)~2NH3(g) dissolving in water. Before· the solid sugar dissolves, the sugar
(c) Reactions having zero!JJ[: A number of reactions are molecules are organized in a crystal. As the molecules' dissolve, .
known in which neither energy is evolved nor absorbed, but these they become distributed randomly and uniformly throughout the
are spontaneous. For example, esterification of acetic acid is a liquid. The opposite process never occurs, i. e. , sugar cubes do riot
spontaneous process. !JJ[ ofthis reaction is zero. form from the solution.
CH 3COOH + C2HsOH~ CH 3 COOC2H s + H 20
Thus, the second factor which is responsible for the
spontaneity of a process is the tendency to acquire maximum
(d) Exothermic reactions fail to achieve completion: randomness.
Every exothennic reaction rarely proceeds to completion even On the basis of second factor we may also explain the
though !JJ[ remains negative throughout. In fact, all spontaneous spontaneity of endothermic processes.
reactions proceed in a direction until an equilibrium is attained. (i) Decomposition ofcalcium carbonate:
Since, some spontaneous reactions are exothermic and others
CaC0 3 (s) ~. CaO(s) + CO 2 (g); !JJ[ = +178.3 kJ mot~l
are endothermic, it is clear that enthalpy alone cannot account for ..
spontaneity. There must be some other factor responsible for This process is spontaneous because the gaseous CO 2 produced
spontaneity or feasibility of a process. is more random than solid calcium carbonate.
2. Tendency to acquire 'maximum randomness: There is (ii) Evaporation ofwater:
another natural tendency that must be taken into account to H20(/)~H20(g); !JJ[=+44kJmol- 1
predict the direction of spontaneity.
Nature tends to move spontaneously from a state oflower The gaseous state of a substance is more random than the liquid·
probability to one of higher probability, i.e., things tend to state. Thus, evaporation of water is spontaneous which proceeds
in the direction ofmore randomness.
change from organized to disorganized. To illustrate what this
statement means, we consider a spontaneous process of Similarly, fusion of ice is also spontaneous because the
intermixing of two inert gases for which !JJ[ is zero. Two process again proceeds in the direction of more random state, i. e. ,
different inert gases, let us say helium (He) and neon (Ne), are liquid state is more random in comparison to solid state.
originally contained in different glass bulbs, separated by a H20(s)~H20(l); !JJ[= +6.06kJ mol-I
stopcock as shown in Fig. 7.20(a). To make the system as closed
(iii) Dissolution ofNH 4Cl in water:
one, the entire system is perfectly insulated.
When the valve is opened, the intermixing of the two inert NH4CI(s) + aq. ~ NH; (aq.) + Cl- (aq.);
1
gases occurs due to diffusion into one another. As the gases are !JJ[ = + IS.lkJ mol-
inert, there is no chemical interaction between them, the heat
When solid NH4 Cl(s) is dissolved in water, its ions become free.
change during intermixing is negligible. The process of
Free ions move randomly in all directions. Thus, the solution is a
intermixing is a spontaneous process. If the proce~s is examined
more random state as compared to solid NH4 Cl. Thus, the
critically, it is observed that when the valve is opened, both the
process of dissolution will be spontaneous because randomness
gases are provided larger volume to occupy, i. e., each gas
increases on dissolution.
achieves its own most probable distribution, independent of the
presence of other gas. The final distribution is clearly much more (iv) Decomposition of merClfric oxide:
\

probable than the initial distribution. There is, however, another 2HgO(s)~2Hg(/)+ 9i(g); !JJ[ = +9Q3kJ mol-I

is more random than reactant due to presence of gaseous 2 ,


Limitations of the criterion for
°
Here again the process will be spontaneous because the product

maximum
randomness: Like energy factor, the randonmess factor has
-He oNe also certain limitations. For example, in the liquefaction of a gas
Fig. 7.20 (a) Fig. 7.20 (b) or in the solidification of a liquid, the randonmess of the particles
Inert gases before mixing Inert gases after mixing decreases but still these processes are spontaneous. Thus, like
useful way oflooking at this process. The system has gone from a energy factor, randomness alone cannot be the sole criterion for
highly ordered state (all the helium molecules on the left, all the the spontaneity of a process. .
neon molecules on the right) to a more disordered or random Overall tendency as driving force for a spontaneous
state. Mixed gases cannot be separated on their own. Thus, process
diffusion is a spontaneous process acquiring more randomness.
From the above discussion, it is apparent that the spontaneous
In general, nature tends to move spontaneously from more
processes occur because of the two tendencies:
ordered to more random states, or a process proceeds
(i) Tendency of a system to achieve a state of minimum
energy.
456 G.R.B. PHYSICAL CHEMISTRY FOR COMPETITIONS

(ii) Tendency of a system to achieve a state of maximum Mathematical Definition of Entropy


randomness. The entropy change of a system may be defined as the integral
The overall tendency of a process to be spontaneous depends of all the terms involving heat exchanged (q) divided by the
on the resultant of the above two factors. The resultant of the two absolute temperature (T) during each infinitesimally sman
tendencies or overall tendency for a process to occur is termed change of the process carried out reversibly at constant
, the driving force. temperature (isothermally).
, '.,,,ere, it should be noted that these tendencies are
inde'liendent of each other. Both may act in the same or in JdS =~J Sqrev
opposite directions in a process.
Case I:' When enthalpyfactor is absent then randomness qrev
or ... (i)
factor decides spontaneity of a process. T
Case II: When randomness factor is absent then enthalpy or
energy factor decides spontaneity of a process. Ifheat is absorbed, then AS is positive (increase in entropy). If
heat is evolved, AS is negative (decrease in entropy). The value
Case III: When both factors take place simultaneously then
of AS, like l1E and All, is a definite quantity and depends on the
,magnitude of the tendencies becomes important to decide

_1
initial and final states of the system. It is independent of the
spontaneity.
manner in which the change has been brought about, i. e. , whether
ENTROPY the change has been brought about reversibly or irreversibly.
Several factors influence the amount of entropy that a system
Why do systems tend to move spontaneously to a state of has in a particular state. In general, ,
maximum randomness or disorder? The answer is that a (i) The value of entropy depends on the mass of the system.
, disordered state is more probable than an ordered state because Hence, it is an extensive property.
the disordered state can be achieved in more ways. The following (ii) A liquid has a higher entropy than the solid from which it
example illustrates the point. Suppose that you shake a box is formed. In a solid, the atoms, molecules or ions are
containing 10 identical coins and then count the number of heads fixed in position; in the liquid, these particles are free to
o
(H) and tails (T). It is very unlikely that all the I coins will come move past one another, i. e., liquid structure is more
up heads; i. e., perfectly ordered; arrangement is much less random and the solid more ordered.
probable than the totally disordered state in which heads and tails (iii) A gas has a higher entropy than the liquid from which it is
come up randomly. The perfectly ordered state of 10 heads can be formed. When vaporization occurs, the particles acquire
achieved in only one Way and the totally disordered state can be greater freedom to move about.
achieved in 210 (l 024) ways, i. e. , the totally disordered state is
(iv) Increasing temperature of a substance increases its
2 10 times more probable than the perfectly ordered state. If the entropy. Raising the temperature increases the kinetic
box contained 1 mol of coins, the, perfectly ordered state would energy of the molecules (atoms or ions) and hence their
be only one, but the disordered states would be much higher freedom of motion. In the solid, the molecules vibrate
23 '
(2 NA = 2 6,.02 x 10 ). It is thus, concluded that a change which with a greater amplitude at higher temperatures. In a
brings about randomness is more likely to occur than one that liquid or a gas, they move about more rapidly.
brings about order. The extent of disorder or randomness is In other words, the more heat the system absorbs, the
expressed by a property known as entropy. more disordered it becomes. Furthermore, if heat is
absorbed at low temperature, it becomes more disordered
Entropy is a thermodynamic state quantity which ·jsa
than when the same amount of heat is added at higher
measure of randomness or disorder of the molecules of the
temperature.
system.
Entropy is represented by the symbol'S'. It is difficult to Units of Entropy and Entropy Change
define the actual entropy of a system. It is more convenient to
define the change of entropy during a change of state. The change Since, entropy change is expressed by a heat term divided by
in entropy from initial to final state of a system is represented by temperature, it is expressed in terms of calories per degree, i. e. ,
AS. The entropy is a state function and depends only on the initial cal K- 1 • In SI units, the entropy change is expressed in terms of
and final states of the system. joules per degree, i.e., J K- 1• Entropy is an extensive property,
i. e., it depends on the mass of the substance; hence units of
AS = S final - S initial entropy are expressed as cal deg -\ mol-I (cal K -1 mol-I) or
jouledeg- l mol-I (JK- I mol-I).
When S final > S initial' AS is positive.
For a chemical reaction, Spontaneity in Terms of Entropy Change
AS S (products) - S (reactants) In an isolated system, such as mixing of gases, there is no
exchange of energy or matter between . the system and
surroundings. However, the mixing of gases is accompanied by
randomness, i. e. , there is increase in entropy. Therefore, it can be
CHEMICAL THERMODYNAMICS AND THERMOCHEMISTRY 457
stated that for a spontaneous process in an isolated system, the q
change in entropy is positive, i.e., llS > o. ASsystem :::; +
T
However, if a system is not isolated, the entropy changes of On the other hand, surroundings lose the same amount of heat at
both the system and surroundings are to betaken into account the same temperature. The decrease in the entropy of the
because system and surroundings together constitute the isolated surroundings will
system. Thus, the total entropy change (AS total) is sum of the
q
change in entropy of the system (ASsystem) and the change in ASsurroundings :::; - T
entropy of the surroundings (ASsurroundings), i. e.,
AS total = ASsystem + ASsurroundiQgs ... (ii) Total change in entropy:::; entropy change in system + entropy
For a spontaneous process, AS total must be positive, i. e., of the process change in surroundings

AS total =AS system + AS surroundings > 0 ... (iii) AS total = ASsystem + ASsurroundings
AS total is also termed as AS universe q -~=O
During a spontaneous process, the entropy of the system goes T T
on increasing till the system attains the equilibrium state, i. e. ,
When the reversible process is adiabatic, there will be no heat
entropy of the system becomes maximum and, therefore, no more
exchange between system and surroundings, i. e., q = 0
increase in the entropy of the system is possible. The
mathematical condition for equilibrium is, ASsystem =0, ASsurroundings =0
AS =o(at equilibrium for an isolated system) '" (iv) AS total = ASsystem + ASsurroundings =0
If AS total is negative, the direct process is non-spontaneous Entropy Change in Irreversible Processes
whereas the reverse process is spontaneous. Consider a system at higher temperature 1j and its
This can be illustrated by considering the entropy changes in surroundings at lower temperature Tz. 'q' amount of heat goes
the conversion of water to ice at three different temperatures. The irreversibly from system'to surroundings.
relevent entropy changes for the system and surroundings and
total change are given in the following table: ASsystem =-~
T.
I
Temperature Msystem M surrounding MlOtal

°c K JK-1 moe l JK-! moe! JK-1 moe!


ASsurroundings
1 272 -21.85 + 21.93 +0.08
0 273 -21.99 + 21.99 0.00
+1 274 - 22.13 + 22.05 -0.08

At 272 K: AS tota! = ASsystem + ASsurroundings


=- 21.85+ 21.93:::; + 0.08JK-1 mol- 1
AS proo:ss > 0 at 272 K or AS process > 0
Thus, freezing of ice at 272 K, H20(1)~H20(s) will be Hence, entropy increases in an irreversible process.
spontaneous.
Entropy Change for Ideal Gases
At 273 K: AS total :::; ASsystem + ASsurroundings Change in entropy for an ideal gas under different conditions
:::; - 21.99 + 21.99 = 0 may be calculated as:
(i) When changes from initial state (1) to final state (2) :
AS total :::; 0
Thus, at this temperature, water and ice will be at equilibrium.
H2 0(l) ~ H 2 0(s); AS:::; Oat 273 K (at equilibrium)
AS = 2.303n Cv log (~ ) + 2.303nR log (i. )
At 274 K: AS total :::; ASsystem + ASsurroundings
(when T and V are variables)
:::; -22.13+ 22.05= -0.08JK-1 mo.-I
AS<O AS = 2.303n Cp log (~ ) + 2.303nR log (~: )
Thus,freezing ~f water will be non-spontaneous at 274 K.
HzO(I) ~ H 2 0(s) AS < o(non-spontaneous) (when T and P are variables)
Entropy Change in Reversible Process (ii). Entropy change for isothermal process:
Consider an isothermal reversible process. In this process, let
the system absorb q amount of heat from surroundings at AS =2.303nR log (Vz J
temperature T. The increase in the entropy of the system will be ' VI
458 G.R.B. PHYSICAL CHEMISTRY FOR COMPETITIONS

IlS = 2.303nR log [ ~: J but its value depends on mass of the substance present in
the system.
(ii) Entropy of a system is a state function. It depends on the
(iii) Entropy change for isobaric process (at constant state variables such as T,P,Vand n which govern the
pressure): . state of the system.

IlS = 2.303n Cp log (i. ) (iii) The change in entropy taking place in going from one
state to another state does not depend on the path
adopted. It actually depends on the final and initial states
IlS = 2.303n Cp log (~ ) of the system.
Change in entropy, M = S final - Sinitial

(iv) Entropy change for isochoric process (at constant (iv) The entropy change for a cyclic process is zero.
volume): (v) The entropy change in the equilibrium state is zero

IlS = 2.303nCv log (i. ) (IlS = 0).


(vi) For natural processes, entropy ofumverse is i:;cre'~sing.
>0 IlS universe

IlS = 2.303n Cv log ( ~) (vii) In a reversible process, M total or


therefore
IlS wllverse 0 and

(v) Entropy change in mixing of ideal gases: Let nl mole of


IlSsystem =- IlSsurroundings
gas A and n 2 mole of gas B are mixed; then total entropy change (viii) For adiabatic reversible process,
can be calculated as: IlSsystem = IlSsurroundings IlS total = 0
M = - 2.303R[nl log XI + n2 log x2] In a spontaneous (irreversible process),
XI ,X2 are mole fractions of gases A and B, IlS total or M universe > 0,
nl i. e. , in spontaneous processes, there is always increase in
i.e., XI = ;X2 =--=-- entropy of the universe.
n. + n 2 nl + n2

7.25· EN"rROPY CHANGE DURING PHASE


TRANSITIONS
Solid, liquid and gas are the three forms of a matter. The change
IlS / mol = - 2.303R[x1 log XI + x2 log X2 ] of a substance from one form to another is known as phase
Entropy change in adiabatic expansion will be zero, IlS = O. transformation. Such changes occur at defmite temperatures
and are accompanied by entropy change. During these
Physical Significance of Entropy at a Glance transformations either heat is absorbed or evolved, i. e., the
1. Entropy as unavailable energy: Entropy is unavailable entropy either increases or decreases accordingly.
energy of the system. The entropy change for these transformations is given by
Unavailable energy IlS trans
Entropy = -------'===- T
Temperature in K
where, q rev is the heat absorbed or evolved and T is the
2. Entropy and randomness: Entropy is a measure of temperature of transition. q rev is actually the molar enthalpy
disordemess or randomness in the system. Increase in entropy change of the substance.
means change from an ordered to less ordered (or disordered) (i) Entropy offusion: The entropy offusion is defined as
state. the change in entropy when one mole of a solid substance
3 •. Entropy and probability: Entropy may be defined as a changes into liquid form at the melting temperature.
function of probability of the thermodynamic state. Since, we The heat absorbed is equal to the latent heat of fusion
know that both the entropy and thermodynamic probability
increase simultaneously in a process, hence the state of IlS S S _ MIfusion
fusion = liquid - solid - T
equilibrium is the state of maximum probability. f

Characteristics of Entropy For example, when ice melts, the change in entropy is given by
The important characteristics of entropy are summarised M S S Mlfusion 6000 J mol-I
below: fusion (ice) = water - ice
Tf 213
(i) Entropy is an extensive property. It is difficult to
determine the absolute value of entropy of a substance = 21.98J K- 1 mol- l

CHEMICAL THERMODYNAMICS AND THERMOCHEMISTRY 459


(ii) Entropy of vaporisation: It is defined as the change in Solution: We know that,
entropy when one mole of the liquid substance changes into Mlvapour
vapours (gas) at its boiling point. !1S vapour = ------'-
T bp
Mlvapour
!1S vapour = S vapour - Sliquid =------'- Given, !1Svapour = 109.8 J K- I mol-I
T bp
Tbp = 78.5 + 273 = 351.5 K
Where, MI vapour is the latent heat of vaporisation and Tbp is the
boiling point. Substituting these values in above equation, we get
MI
For example, when water is converted into steam, the change 109.8 = vapour
in entropy is given by 351.5
Mlvapour 40626Jmol- 1 Mlvapour = 38594.7 J mol-I
!1Svapour(water) = Ssteam - Swater = T
bp 373 = 38.594 kJ inol- I
=108.9JK- 1 mol-I Example 65. Calculate the entropy change for the
following reversible process:
(iii) Entropy of sublimation: Sublimation involves the
direct conversion of a solid into its vapour. The entropy of a -Tin ~ P-Tin
I mol al I aim I mol al I aim
sublimation is defmed as the change in entropy when one mole of
a solid changes into vapour at a particular temperature. (Mllrans = 2090 J mor I )

!1Ssub = S vapour - Ssolid = i Ub


Solution:
M
trans
= 20_9_0
Mltrans _ _
T 286
Where, Mlsub = heat of sublimation at temperature T.
= 7.3076JK- I mol- I
::: :::I_SOME SOLVED EXAMPLES\ I:::::: i"" K:Example 66. At O°e, ice and water are in equilibrium and
,:E~ample 62. The enthalpy change, for the transition of
enthalpy change for the process H20(s)~H20(l) is
6 k.! mol -I. Calculate the entropy change for the conversion of
liquid water to steam, Mlvapour is 40.8 k.! mOrl at 373 K.
ice into liquid water.
Calculate entropy change for the process.
Solution: We know that for the process of fusion,
Solution: The transition under consideration is:
H20(s)~H20(/)
H 20(l) ~ H 2 0(g)
!1S ' = MI fusion
We know that, !1S == MI vapour fus,on T
f
vapour T
= 6x 1000 = 21.98iK- I mol- I
Given, Mlvapour = 40.8 kJ mol-I .273
= 40.8 x 1000 J mol-I Example 67. The enthalpy of vaporisation of liquid
diethyl ether (C 2H s hO is 26k.! morl at its boiling point 35°C.
T= 373K
Calculate !1SO far conversion of:
Thus, M = 40.8 x 1000 = 109.38 J K- I mol-I (a) liquid to vapour,
vapour 373
(b) vapour to liquid at 35°C.
',;'Example 63. What is entropy change for the conversion of MI vaporisation 26000
one gram ofice to water at 273K and one atmospheric pressure? Solution: (a) ~S vaporisation = T - 30s
( Mlfus;on = 6.025 k.! mol -I ) bp
= 84.42JK- I mol- I
Solution: Mlfusion = 6.025 x 1000 J mol-I
(b) !1Scondensation = - Mlvaporisation
= 6025 Jg- I = 334.72Jg-1 = - 84.42 J K- I mol- I
18
Example 68. Calculate entropy change when 10 moles of
!1S ' = MI fusion an ideal gas expands reversibly and isothermally from an initial
fus,on T
f volume oflOlitre to 100 litre at 300K.

= 334.72 = 1.226JK-1 g_1 Solution: !1S = 2.303nR log (~~ J


273
,"Example 64. Calculate the enthalpy of vaporisation per = 2.303 x 10 x 8.314 log (\O;J
mole for ethanol. Given, M = 109.8J K -I mol-I and boiling
point ofethanol is 78.5°C. = 191.24 J K- I
4601 G.R.B. PHYSICAL CHEMISTRY FOR COMPETITIONS

Example 69. Oxygen gas weighing 64 g is expanded from I [Hint: 65;otol = Msystem
+ Msurrounding
atm to 0.25 atm at 30°C. Calculate entropy change, assuming the =+ ve for spontaneous process]
gas to be ideal. 38. Melting point of a solid is x K and its latent heat of fusion is

Solution: n=~ 64=2 600 cal mol-I. The entropy change for fusion of 1 mol solid
mwt. 32 is 2 cal mol-I K- 1• The value ofxwill be:

;~)
(a) 100 K (b) 200 K (c) 300 K (d) 400 K
AS = 2.303nR log ( [Ans. (e)]
[Hint: MIfusion -
--r-- - M fusion
2.303 x 2x 8.314 log (_1_) mp
0.25
600 =2
= 23.053 J K-
I
T
Example 70. Calculate the change in entropy when I mole T = 300 K]
nitrogen gas expands isothermally and reverSibly from an initial 39. The entropy change involved in the conversion of 1 mole of
volume ofllitre to a final volume ofl 0 litre at 27° C. liquid water at 373 K to vapour at the same temperature will
be:
Solution: AS 2.303 nR log (~ ) Mivap = 2.257 kJ I g
(a) 0.119 kJ (b) 0.109 kJ

= 2.303 x I x 8.134 log C~) (c) 0.129 kJ (d) 0.120 kJ


. [PMT (MP) 2007]
[Ans. (b)]
= 19.12JK- I
2.257 x 18
[Hint: M = 0.109kJlg]
OF T 373

34. For a liquid, enthalpy of fusion is 1.435 kcal mol- I and molar 7L~~. SECOND LAW OF THERMODYNAMICS
entropy change is 5.26 cal mol- I K- I. The melting point of
The second law of thermodynamics tells us whether a given
the liquid is:
process can occur spontaneously and to what extent. It also helps
(a) O°C (b) 273°C
us to calculate the maximum fraction of heat that can be
(c) 173 K (d) 100°C converted into work in a given process.
[Ans. (a)]
The second law ofthermodynamics like first law is a postulate
[Hint: 6Sfusion = --""""'- and has not been derived from any prior concepts. It is stated in
Tmp various forms. However, all the statements of second law have
1.435 x 1000 the same meaning.
5.26 1. Clausius statement: "It is impossible to construct a
Tmp
machine that is able to convey heat by a cyclic process from a
Tmp = 1435 273 K, i.e,O°C] colder to a hotter body unless work is done on the machine by
5.26 some outside agency. n
35. Latent heat of vaporisation of water is 540 cal g-I at 100°C. It means that work can always be completely converted into
Calculate the entropy change when 1000 g water is converted heat but heat cannot be converted completely into work without
to steam at 100°C. leaving some permanent change in the system or surroundings.
(a) 1447 cal (b) 2447 cal (c) 3447 cal (d) 4447 cal For example, heat produced in heat engine is never fully utilized,
[Ans. (a)] as part of it is always lost to surroundings or in overcoming
friction, etc. Thus, it can be said that the complete conversion of
[Hint: Mvap = Mlvap 540 x 1000 = 1447 cal] heat into work is impossible without leaving some effect
T 373
elsewhere.
36. Enthalpy of fusion of water is 6.01 kJ mol-I. The entropy
change of 1 mole of ice at its melting point will be: Or
(a) 22 kJ mol- I (b) 109 kJ mol-I It is not possible to convert heat into work without
compensation.
(c) 44 kJ mol-I (d) 11 kJ mol-I
Or
[Ans. (a)]
All forms ofenergy can be converted into heat energy but heat
[Hint: 68.. = Mlfusion = 6.01 x 1000 22 kJ mor l ] cannot be converted into other forms of energy fully by any
fusion r mp 273
process.
37. For spontaneous process: The other similar statements are:
(a) AStotal = 0 (b) AS total > 0 2. Thomson statement: The heat of the coldest body
(c) AStotal < 0 (d) none ofthese among those participating in a cyclic process cannot serve as a
(Ans. (b)] source of work.
CHEMICAL THERMODYNAMICS AND THERMOCHEMISTRY 461
3. Kelvin-Planck statement: It is impossible by means of dq
inanimate material agency to derive mechanical work or effort dS>-
T
from any portion ofmatter by cooling it below the temperature of
the coldest ofthe surrounding objects. dS ~ dq
4. It is impossible to construct a heat engine ofl 00 % thermal T
efficiency. This is the mathematical statement of second law of
thermodynamics.
5. Lud~g Boltzmann statement: In 1886, Ludwig
Boltzmann gave a most useful statement of the second law of dq dE + P dV This is the mathematical statement of
thermodynamics. Nature tends to pass from a less probable to first law of thermodynamics
more probable state. Combining both the laws of a reversible process,
The connection between entropy and the spontaneity of a TdS =dE+PdV
reaction or a process is exWessed by the second law of
thermodynamics in a number of ways. 7.2' GIBBS FREE ENERGY, (G), CHANGE IN
6. All spontaneous processes or naturally occurring FREE ENERGY AND SPONTANEITY
processes are thermodynamically irreversible. Without the help
ofan external agency, a spontaneous process cannot be reversed. As discussed earlier, there are two thermOdynamic quantities that
For example, the mixing of non-reacting gases is a affect reaction spontaneity. One of these is enthalpy, H; the other
spontaneous process. But these cannot be separated from the is the entropy, S. The problem is to put these two quantities
mixture without tqe application of special methods. together in such a way as to arrive at a single function whose sign
Heat energy can flow from a hot body to a cold body of its will ddermine whether a reaction is spontaneous. This problem
own (spontaneously) but not from a cold body to a hot body was fITst solved more than a century ago by J. WiUard Gibbs,
unless the former is heated. who introduced a new quantity, now called the Gibbs free
energy and given the symbol, G. Gibbs showed that for a reaction
7. The entropy of the universe increases in a spontaneous
taking place at constant pressure and constant temperature, AG
" process and remains unchanged in an equilibrium process.
represents that portion of the total energy change that is available
Since, the universe is made up of the system and the
surroundings, the entropy change in the universe (t.!.S universe) for
If:
0. e. ,free) to do useful work. tor example, AG for a reaction is
- 300 kJ, it is possible to obtain 300 kJ of useful work from the
any process is the sum of the entropy changes in the system
reaction. Conversely, if AG is + 300 kJ, at least that much energy
( t.!.Ssystem) and in the surroundings ( ASsurroundinglJ. in the form of work must be supplied to make the reaction to take
Mathematically, the second law of thermodynamics can be
place. Gibbs free energy of a system is defined as:
expressed as:
'The thermodynamic quantity of the system, the decrease il>
A spontaneous process: whose value during a process is equal to useful work done by the
system. ,.
AS universe = t.!.S system + t.!.Ssurroundings > 0
An equilibrium process: Mathematically, it may be defined as:
t.!.S universe = t.!.SsYStem + ASsurroundings = 0 G=H - TS ... (i)
For a spontaneous process, the law says that t.!.S universe must be " where, H"" enthalpy; S entropy and T absolute temperature.
greater than zero, but it does not place a 'restriction on either
We know that, H = E + PV ... Oi)
ASsystem or ASsurroundings' Thus, it is possible for either t.!.Ssystem or
t.!.Ssurroundings to b...: negative, as long as the sum of these two From eqs. (i) and (ii),
quantities is greater than, zero. For an equilibrium process, G=E+PV -TS
t.!.S universe is zero, i. e. , t.!.Ssystem and t.!.Ssurroundings must be equal in Free energy change at constant temperature and pressure can be
magnitude but opposite in sign. given as:
Thus, the entropy of the universe is continuously increasing. AG AE + P AV T AS
The main ideas of the first and second law of thermodynamics AG=AN T AS .•• (iii)
may be summarised as: (Gibbs-Helmholtz equation)
First law states that the energy of the universe is constant Here, AN AE + P AV
whereas the second law states that the entropy of the universe is Gibbs-Helmholtz equation is used to discuss the driving force,
continuously increasing and tends to a maximum. i.e., the overall criterion of spontaneity.
First law deals with the conservation of energy whereas the
[Note: Since,' H' and'S' are extensive property hence 'G' will also be
second law tells the direCtion of flow of energy.
an extensive property. Moreover, Gibbs function 'G' is a state
For a reversible process, function.] .
dS= dq or dq=T{IS Free Energy Change and Spontaneity
T
For an irreversible process, Let us consider a system which is not isolated from its
surroundings. In this case, total entropy change can be calculated
as:
I
462 G.R.B. PHYSICAL CHEMISTRY FOR COMPETITIONS

A5'total ASsystem + .ASSWTOIUldings ... (i) On adding, 2Fe203 (s) + 6CO(g ) ~ 4Fe(s) + 6C02 (g);
Let us consider the process at constant temperature and pressure. AGO =-52.2kJ/mol
Let qp amount of heat be given by the system to the
sUlToundings. Thus, both reactions proceed simultaneously.
The free energy of a reaction is the chemical analogue of
( qp ) surroundings - Mlsystem potential energy of mechanical systems. A body moves in the
_ _ _ _-""- = - Mlsystem direction in which there is decrease in potential energy. Similarly,
ASsurroundings ... (ii) in chemical system, the substance moves in a direction in which
T T
there is decrease in free energy, i. e. , AG is negative.
From equations (i) and (ii), In a chemical reaction, AG = Gproducts - Greactants. Thus,
Mlsystem spontaneous changes occur with a decrease in free energy, i. e. ,
AS total = AS system - - - ' - - AG is negative.
T
To sum up, the criteria for spontaneity of a process in terms of
or AG is as follows: '
or -T AS total = Mlsystem - T ASsystem ... (iii) (i) If AG is negative, the process is spontaneous .
(ii) If AG is zero, the system is in equilibrium. The process
According to Gibbs-Helmholtz equation,
does not occur.
AG=MI-TAS (iii) If AG is positive, the process does not occur in the
.. AGsystem = Mlsystem - T ASsystem ... (iv) forward direction. It may occur in the backward
direction.
From equations (iii) and (iv),
Conditions for AG to be negative or the process to be
AGsystem = - T AS total spontaneous
We know that for spontaneous process, AS total > 0 We know that, AG MI-TAS
.. AG = - ve for spontaneous process. MI TM Conditions !J.G
Thus,Jor spontaneous process T AStotal should be positive or AG
should be negative. -ve +ve (any) -ve spontaneous
(favourable) (favourable)
Case I: Let entropy and energy, both factors are favourable
'A process, i. e., MI = - ve and T AS = + ve
C'" -ve -ve I tili I> (T llS) -ve spontaneous
(favourable) (unfavourable)
AG MI-TAS
= (-ve)- (+ve) =-ve
+ve +ve IT!J.S 1>ltiliI -ve spontaneous
(unfavourable) (favourable)
Thus, AG Vt:_ )/' spontaneous process.
Case II: Let both energy and entropy factors oppose a Role of Temperature on Spontaneity
process, i. e., MI = + ve, T AS = - ve. In Gibbs-Helmholtz equation:
:. From AG MI T AS AG=MI-TAS,
=+ve (-ve) +ve not only MI and AS but also temperature 'T' is a determining
factor for spontaneity, i. e., for AG to be -ve. Let us discuss
Thus, AG is positive for a non-spontaneous process.
exothermic and endotherrriic reactions to show the influence of
Case III: Let both tendencies be equal in magnitude but temperature.
opposite, i. e. ,
1. Exothermic processes: In exothermic reactions,
MI=+ve and T AS +ve and MI=T AS MI = - ve (favourable condition)
.. From AG MI T AS Case I: When T AS is positive, i.e., favourable, then flG
will be negative and the process will be spontaneous at all
=0
temperatures.
In this condition, the process is said to be at equilibrium. Case II: When T AS is negative, i.e.,unfavourable, then flG
Coupled Reactions will be negative when IMIl > IT AS ~ To reduce the magnitude
We know that the reactions which have AG = + ve are of T AS,the temperature should be low. Thus, exothermic
non-spontaneous. However. such reactions can be made reactions can be made favourable (when T AS =- ve) by
spontaneous when coupled with a reaction having very large lowering the temperature.
negative free energy of reaction, 2. Endothermic reactions; In endothermic reactions,
MI = + ve (unfavourable conditions).
e.g., 2Fe203(s) 4Fe(s) + 302 (g); AGO = + 1487kJ/moi
In case of endothermic reactions, AG will be negative when
6CO(g) + 30 2 (g) 6C0 2 (g ); AG ° = -1543.2 kJ/mol IAH I< IT AS f To increase the magnitude of T AS, temperature
CHEMICAL THERMODYNAMICS AND THERMOCHEMISTRY 463

(T) should be increased. Thus, endothermic reactions can be Calculate AG at 700 K when enthalpy and entropy changes are
made favourable by increasing the temperature. -1l3.0kJ marl and -145J K- l mor l respectively.
Thus, temperature also plays an important role in deciding the Solution: We know that, AG AlI- T AS
spontaneity of a reaction. A reaction which is non-spontaneous at Given, All -113 kJ111Ol- 1
low temperature becomes spontaneous at high temperature and
vice-versa. 113000 J 11101-1
AS = -145 J K~I 11101-1
Sign of MI, AG and AS and prediction of spontaneity
T=700K
Ml AS /1G Remarks Substituting these values in the above equation,
-ve +ve -ve i Spontaneous at all we get AG 113000-700x (-145)
temperatures
11500J111Ol-1
-ve. -ve -ve (at low temperature) Spontaneous
= -11.5 kJ 11101- 1
+ve (at high temperature) Non-spontaneous
+ve +ve +ve (at low temperature) Non-spontaneous Example 73. In the reaction A + + B ---7 A + B+, there
-ve (at high temperature) Spontaneous is no entropy change. If enthalpy change is 22 kJ ofA +, calculate
+ve -ve +ve Non-spontaneous at all AG for the reaction.
•temperatures Solution: For the given reaction,
MI All 22 kJ, AS =0
.. From, AG=AlI TAS
+ -
AG = 22- T x 0 22kJ11101- 1
+ Spontaneous at high tem- Spontaneous at all temper-
perature and non-sponta- atures Example 74. All and AS for the reaction
neous at low temperatures Br2 (I) + Cl2 (g) -'I 2BrCI(g) are 29.37 kJ and 104.0J K -I
- Non-spontaneous at all Non-spontaneous at high respectively. Above what temperature will this reaction become
temperatures temperatures and sponta- spontaneous?
neous at low temperatures Solution: According to Gibbs-Helmholtz equation,
AG =All - T AS
:::::::_SOME SOLVED EXAMPLES\ ::::: : : For spontaneous process, AG < 0
Example 71. MI and AS for the reaction, i. e., All T AS < 0
. 1
Ag 2 0(s)---72Ag(s)+-02 (g) TAS>AlI
2 All
are 30.56kJ marl and 66.0J K- I marl respectively. Calculate T>-
AS
the temperature at which free energy change for the reaction will
be zero. Predict whether the forward reaction will be favoured T> 29.37 x 1000
T > 282.4 K.
above or below this temperature. 104
Solution: We know that, .Example 75. Calculate the temperature at which liquid
AG = MI T AS water will be in equilibrium with water vapour.
At equilibrium, AG=O AlIvap = 40.73kJ marl and AS vap 0.109kJ mor l K- 1
so that 0== All T AS
All Solution: Given, All = 40.73 kJ 11101-1
or T=-
AS
AS == O. 109 kJ 11101-1 K- I
Given that, All 30.56 kJ 11101- 1
and AG=O
=30560 J mol-1
Applying AG=AlI TAS
AS = 66.0JK- 1 mol- l
0= 40.73 Tx 0.109
T = 30560 463 K
66 T = 40.73 = 373.6K
0.109
Above this temperature, AG will be negative and the process will
be spontaneous in forward direction. Example 76. Zinc reacts with dilute hydrochloric acid to
give hydrogen at 17° C. The enthalpj/ofthe reaction is -12.55 kJ
Example 72. For the reaction,
marl ofzinc and entropy change equals 5.0J K-,I marl for the
2NO(g) + O 2(g) ---7lN02 (g)
reaction. Calculate the free energy change and predict whether
the reaction is spontaneous or not.
464 G.R.B. PHYSICAL CHEMISTRY FOR COMPETITIONS

Solution: Given, Mf = -12.55 kJ mol- l Calculate K p at each temperature and predict the direction of
reaction at 300 K and 1200 K, when Peo = Peo2
!:J.S = 5.0JK- 1 mol- l
= PH 2 = PH20 = 1atmat initial state.
= 0.005 kJ K- l mol- l Solution: At 300 K: !:J.Go = Mfo - T !:J.S 0
T = 17 + 273 = 290 K = - 41.16 - 300 x (- 4:24 x 10-2 )
Applying !:J.G=Mf-T !:J.S
= -28.44 kJ
= -12.55 - 0.005 x 290
Since, !:J.G ° is negative hence reaction ,is spontaneous 10
= -12.55 -1.45 = -14.00 kJ mol- l forward direction,
Since, !:J.G is negative, the reaction will be spontaneous. !:J.Go = -2.303RT log Kp
Example 77. Mf and !:J.S for the system -28.44 = - 2.303 x 8.314 x 10-3 x 300 log Kp
H 20 (I) ~ H 20(g) at 1 atmospheric pressure are 40.63 kJ
mol- l and 108.8 J K -I mol- l respectively. CalcuLate the Kp = 8.93x 104
temperature at which the rates of forward and backward At 1200 K: !:J.Go = Mfo - T!:J.S °
reactions will be the same. Predict the sign offree energy jor this
transformation above this temperature. = - 32.93 - 1200(- 2.96 x 10-2 ) = + 2.59 kJ
Solution: Given, Mf = 40.63 kJ mol- l Positive value of !:J.G ° shows that the reaction is spontaneous
!:J.S = 108.8 J K- l mol- l = 0.1088 kJ K- l mol- l in backward direction
(when the system is in equilibrium) !:J.Go = - 2.303RT log Kp
!:J.G=O
..' ..
.~
Applying !:J.G=Ml-T !:J.S 2.59 = - 2.303 x 8.314 x 10-3 x 1200 log K p
0= 40.63 - T x 0.1088 Kp =0.77
T = 40.63 = 373.4 K .. Example 80. The standard Gibbs free energies for the
0.1088 reactions at 1773 K are given below:
The sign of !:J.G above 373 K, i.e., say 374 K, may be C(s) + 02 (g) -----t CO 2 (g); !:J.Go = - 380 kJ mol- l
..:alculated as follows: 2C(s) + 02 (g)~2CO(g); !:J.Go = - 500kJ mol- l
\gain applying !:J.G = Mf - T !:J.S
Discuss the possibility of reducing Al20 3 and PbO with
= 40.63 - 374 x 0.1088 carbon at this temperature,
= 40.63 - 40.69 = - 0.06 kJ 4AI + 302 (g)-----t 2AI20 3 (s); !:J.Go = -22500kJ mol- l
!:J.U will be negative; hence, the reaction will be spontaneous. 2Pb + 02 (g) -----t 2PbO(s); !:J.Go = -120 kJ mol- l
. Example 78. Fl.,' the reaction, Solution: Let us consider the reduction of Al 20 3 by carb?n:
SOCl 2 + Hp -----t S02 + 2HCI,
2Al 20 3 + 3C(s) -----t 4Al(s) + 3C0 2 (g);
the enthalry of reaction is 49.4 kJ and the entropy of reaction is
336 J K - . Calculate!:J.G at 300 K and predict the nature of the !:J.Go = - 380 x 3 + (22500) = + 21360 kJ
reaction. 2Al 20 3 + 6C(s)-----t4Al(s) + 6CO(g);
Solution: !:J.G = Mf - T !:J.S
!:J.Go = - 500 x 3 + 22500 = + 21000 kJ
= 49.4 - (300 x 336 x 10'-3 )
Positive values of !:J.G ° show that the reduction of Al2 0 3 is
=-51.4kJ not possible by any of the above methods.
Since, the free energy change is negative, the given reaction is Now, let us consider the reduction ofPbO.
spontaneous. 2PbO(s) + C -----t 2Pb + CO 2;
Example 79. The standard enthalpy and entropy changes !:J.GO = + 120+ (-380)= -2GOkJ
for the reaction in equilibriumfor the forward direction are given
below: 2PbO(s) + 2C-----t 2Pb + 2CO;
CO(g) + H20(g)~C02 (g)+ H2 (g) !:J.Go = + i20+ (-500)= -380kJ
l
Mt{OOK = -41.16kJ mor Negative value of!:J.G ° shows that the process is spontan(:olls
and PbO can be reduced by carbon.
!:J.S~OOK = -4.24 X 10-2 kl mor l
Example 81. In a fuel cell, methanol is used as fuel and
l oxygen gas is used as an oxidiser. The reaction is:
11ll7200K = - 32.93 kJ mor
3
2
!:J.S7200K = -2.96x 10- kJ mor
l '2
CH pH(I) + O2 (g) -----t CO 2 (g) + 2H 20(1)
CHEMICAL THERMODYNAMICS AND THERMOCHEMISTRY 465

Calculate standard Gibbs free energy change for the reaction The values of /),.S and t1G for conversion of ice into liquid
that can be converted into electrical work. Ifstandard emhalpy of water at O°C are:
combustion jar methanol is -726 kJ mol -I;. calculate the (a) -21.9 J K- 1 mol- I and 0 (b) 0.219 J K- 1 mol-I and 0
efficiency ofconversion of Gibbs energy into usiful work. (c) 21.9 J K-1 mol- l and 0 Cd) 0.021'::1 J K- 1 mo]-l and 0
[Ans. (e»)
/),.G; for CO 2 , H 20, CHPH, O 2 are -394.36; -237.13;
[Hint: t1G 0,
-166.27 and zero respectively.
;'. lili TIlS 0
Solution: The reaction for combustion of methanol is:
3 tJ.S == lili = 6000 = 21.9 J K- 1 mor i ]
CH 3 OH(l) + 02 (g ) ~ CO 2 (g) + 2H 2 0(l) T 273
2
44. For a reaction:
/),.o;eaclion = [/)"Gj CO 2 (g) + 2 /)"Gj H20(l)] I
Ag20(S)~2Ag(s)+ 2 O 2 (g)
[/)"{7[ CH3 0H(l)+ 3 /),.G;.Ol(g)]
2 MI,t1S and Tare 40.63 kJ mol-I; 108.8 J K- 1 mol-I and
= [-394.36 + 2 (-237.13)] [-166.27 + 0] 373.4 K respectively. Predict the feasibility of the reaction.
:::= -702.35 kJ mol- 1 (a) feasible (b) non-feasible
(c) remains at equilibrium (d) not predicted
Efticiency of conversion of Gibbs free energy into useful work [Ans. (c»)
x 100 [Hint: t1G IV{ - T IlS
== -~="-----
o = 40.63 x 1000 373.4 x 108.8 == 0]
/),.fl reaction
45. For the homogeneous reactions:
= -702.35 X 100:::= 96.7% xA + yB~IY +mZ
-726 t1H = - 30 kJ mol-I, /),.S = -100 J K-1 mol-I.
,UUSTtATfONS .Ofc'OB1ECTIVE QUlmONS . At what temperature the reaction is at equilibrium?
(PMT (Kerala) 2006)
40. The tree energy for a reaction having Mf == 31400 cal, (a) 50°C (b) 250°C (c) 100 K (d) 27°C
1 1 (e) 500 K
t1S = 32 cal K- mol- at 1000°C is: IJEE (Orissa) 2005)
[Ans. (d)]
(a) -9336 cal (b) -7386 cal tc) -1936 cal (d) +9336 cal
[Hint: t1G lili - T t1S
[Ans. (a»)
[Hint: t1G == IV{ - T 1lS= 31400 -1273 x 32 == 9336 cal] At equilibrium, t1G == 0
.. T == lili = -30 x 1000 = 300 K, i.e., 27° C]
41. F2C=CF-CF=CF2 ~ F2C--CF2 AS -100
I
CF 46. For the reaction CO(g) + .! O2(g ) ~ CO2(g 1 t1H and
2
For this reaction (ring closure), t1H = 49 kJ mol-I, are 't"283,:kJi,~d T$1-J;,K~\ tnoF!;:rtlspectively. It was
/),.S
t1S = -40.2 J K- I mol-I. Upto what temperature is the intended;td ~out this~reaW0n!atl'l'OO<i; 1500, 3000 and
forward reaction spontaneous? 3500 K. At which of these temperatures would this reaction
(a) 1492°C (b) 1219°C (c) 946°C (d) I080°C be thermodynamically spontaneous? (PMT (Kerala) 2006)
[Ans. (e)] (a) 1500 and 3500 K '. (b) 3000 and 3500 K
[Hint: For spontaneousproeess, (c) 1000, 1500 and 3000 K (d) 1500,3000 and 3500 K
(e) At all ofthe,:above temperatures
t1G = MI. -fIlS (should be negative)
[Ans. (e)]
T > lili [Hint: t1G = Mi - T IlS
IlS
At 1000 K, t1G =- 283 - (-1000 x 0.087) (Spontaneous)
T > -49 x 1000
=-196
-40.2
At 1500 K, t1G = - 283 (-1500 x 0.087) (Spontaneous)
T > 1219 K, i.e., 946°C] 152.5 kJ
42. For the equilibrium reaction, the value of Gibbs free energy At 3000 K, tJ.G 283 - (-3000 x 0.087) (Spontaneous)
change is: (DeE 2005) = -22 kJ
(a) > 0 (b) < 0 (c) 0 (d):1: 0
At 3500 K, t1G = - 283 (-3500xO.087) (Non~ontaneous)
[Ans. (e)]
== + 21.5 kJ]
LHint: tJ.G 0 at equilibrium]
47. For a reaction,
43. At O°C, ice and water are in equilibrium and t1H = 6kJ mol- I. MI = + 29kJ mol-I; M = - 35 JK- I mo(1
For this process:
at what temperature, the reaction will be spontaneous?
H20(S)~ H 2 0(l) (PMT (Haryana) 20071
466 G.R.B. PHYSICAL CHEMISTRY FOR COMPETITIONS

(a) 828.7°C C3HS (propane) 270 Si0 2 -805


(b) 828.7 K C4H iO (n~butane) 310 Pb02 -219
(c) Spontaneous at all temperature 310 ,..,166.2
C4H iO (iso-butane) CHpH
(d) Not possible
.[Ans. (d)]. CZH 2 (acetylene) 209.2 C2H5OH -174.8
[Hint: AG = All - T AS ~H4 (eJ;hylene) 219.4 ~I\; -124.5
When All = + ve, AS = - ve then IlG will be positive and the C31\; (propylene) 266.9 CH3COOH -392
reaction is non-spontaneous.]
C4HS (lcbutene) 307.4
7..28 STANDARD FREE ENERGY CHANGE Example 82. Will the reaction,
Just like enthalpy and internal energies, we cannot. determine l z (s) + HzS(g)------'? 2Hl(g) + S(s)
absolute value of Gibbs free energy. The standard free energy proceed spontaneously in the forward direction at 298 K,
change can be determined and itis defined as the free energy AGf Hl(g) = L8kJ mor l , AGfHzS(g) = 33.8kJmor l ?
changefor a PrQcess at 298 K and 1 aim pressure in which the Solution:
reactants in their stdndard state are converted to products in AGO ='2..G;(products)-XGj(reactants)
,
their standard state. It is denoted as AGO; it can be related to
standard enthalpy and entropy change in the following manner: [2GjHI(g)+ GjS(s)]-[lx G;1 2 (s)+ GjH 2 S(g)]
AGO = Mio'- T ASo =[2 x 1.8 +0] [0 + 33.8] =~ 30;2 kJ
Like that of MfO; 'Aoo can be calculated from the standard free -ve value shows that the process is spontaneous in forward
energies of formation of the products and the reactants. direction.
AGO = '2..G; (prodticts)- '2..Gj (reactants) Example 83. Compute the standard free energy of the

=
Sum of standard free 1
[Suniofstandard free
energies of formation - energies of formation
1 reaction at 27° C for the combustion of methane using the given
data:
[ of products . of reactants . CH4 (g) + ']()2 (g) ------'? CO2(g) + 2H20 (I)
Species CH 4 02 CO2 H 20
Let us consider a general reaction:
. . " n; A + n2B------'? n3L + n4M Mi; / (kJ mor1 ) 74.8 - 393.5 285.8
AGO =, '2..G;(products) - '2..G; (reactants) SO/(J K- 1 mor1 ) 186 205 214 70

=[n3G;(L)+ n4 Gj(M)] - [n\ G;(A)+ n2 Gf(B)] Solution: Mio::; Mi; (C0 2) + 2Mil (H;P) Mij (CH 4)
-

f)
The standard free energy of formation (A G may be defined as
the free energy change when 1mole ofa compound is formed from =
393.5 + 2 x (-285.8) - (.;..74.8)
890kJmol-1
its constituent elements in their standard state. The standardfree
energy offormation ofan element in its standard state is assumed
AS ° S (~02) +' 2S (~;P) - S (~H 4) - 2S t
0 2)

to be zero. 214 + 2x 70-186- 2x 205


242JK-1mor 1
Standard Gibbs Free Energy of
Formation (kJ mol'-· at 298 K). AGO =!JJ.IO - TASo
=- 890- 300x (-242 x 10-3 )
HCl . ,- ~'"
-95.27 C4Hg (iso-butene) 294
=-890+72.6 =-817.4kJmol- 1
H~O -228.6 BaO -528.4
H20 2 . -1033 BaC03 -1139 7.29 RELA"nONSHIP BETWEEN STANDARD
CO -137.3 BaS04 -1465 FREE ENERGY CHANGE (~GO) AND
CO 2 -394.4 CaO 604.2 EQUILIBRIUM CONSTANT
S02 -300.4 Ca(OHh -896.6 For any reversible reaction, A + B C + D, the free energy
S03 -370.4 CaC03 -1129 change and standard free energy change are related with each
N02 51.84 CUO -127 other by the following relation:
N20 104 -384.0 AG =AGO + RT loge Q
N~1
-i6.6 KCl 408.3 where, R is universal constant; T is temperature and Q is
NH3 , [CUD]
03 163.4 NH4Cl -203.0 reaction quotient Q = - - .
[A][B]
NO 86.69 AlZ03 -15.82.4
At equilibrium state, AG =.Oand Q
CH4 -50.79 F~03 -741.0
0= AGO + RT loge Keq
Czl\; (ethane) 229 ZnO -318.2 or AGO -RT loge Keq
CHEMICAL THERM,ODYNAMICS AND THERMOCHEMISTRY 467
or I1G o - 2.303RT log K eq minimum in the curve lies clos~r to the reactants, the reaction
This equation helps in the c&lculation of K eq if I1G o of the favours reactants (K «< 1). The reaction in this case will be
reaction is known and vice-versa. The value of K eq can give an farthest from completion. (See. Fig. 7.23)
idea about the extent of chemical reaction before the equilibrium :;, Example 84. What is the equilibrium constant Kc for the
is attained. .following reaction at 400K?
Alternatively, mOCl(li) mO(g) + Cl2 (g)
K == e-!:J.GofRT MiO = 77.2kl and MO::: 122J K- l at 400K.
Equilibrium constant of unity implies that standard free Solution: According to Gibbs-Helmholtz equation,
energy change 1100 is zero and that the positive values of 1100 I1G o MfO-TMo.
implies the equilibrium constant to be less than unit.
::: 77.2- 400 x 122x 10-3
28.4 kJ =·28.4 x 103 J
K We know that, I1G o = - 2.303RT log K c
o 3
log K :::: -I1G
1 ..
c 2.303RT
:::: 28.4 X 10
2.303 x 8.314 x 400
= 3.7081
K = 1 ~---------------'--""!-_ _ __
Kc antilog (-3.7081):::: 1.958 x 10-4
_Example 85. For the equilibrium.
-ve o +ve PCI5 (g) (g)+CI2 (g)at 25°C
~Go_
Kc 1.8 x 10-7
Fig. 7.21 Variation of equilibrium constant against Calculate I1G O for the reaction (R = 8.314J K- l mor l ).

standard free energy change Solution: We know that,


I1G o :::: - 2.303RT log Kc
If we draw a graph of free energy against extent of reaction,
we get U-shaped graph Fig. 7.22. = - 2.303 x 8.314 x 298 log (1.8 x 10-7 )
= 38484 J mol- 1 = 38.484 kJ mol- l

(Reactant) R Example 86. The equilibrium constant at 25°C for the


process:
1 Co 3+ (aq.) + 6NH3 (aq.) [Co(NH3 )6]3+ (aq.)
is2x107 .
P (Product) Calculate the value of I1G O at 25°C, (R :::: 8.314 J K -lmor l )
..~ In which direction, the reaction is spontaneous when the
AG ~O;:::;:~~--t- Equilibrium reactants and products are in standard state?
... r I
1 (minimum of the curve) Solution: We know that, I1G ° =- 2.303R T log Kc
Extent of reaction - Given1 Kc =2x10 7 ,T 298K,R.:;::8.314JK- l mol- l
Thus, from above equation,
Fig. 7.22 Plot of free energy against extent of reaction
I1G 0 = - 2.303 x 8.314 x 298 log 2 x 10 7
If the minimum in the curve lies closer to products, it means = -12023.4 J:::: -12.023 kJ
that reaction is in favour of productS (K »> 1). The reaction in Example 87. The eqUilibrium constant for the reaction.
this case will be closer to completion. On the other hand, if the COz (g) + Hz CO(g) + H zO(g ) at 298 K
is 73. Calculate the value of the standard free energy change
(Reactant) (R = 8.314J K- l mor l ).
R P
Solution: We know that, I1G o 2.303RTlog,Kc '. \;
t Given, Kc ::::73,R l
8.314JK- mol- ,T=298KI

:.From above equation,


I1G 0 = - 2.303 x :8.314 x 29810g 73
I Equilibrium
I
I
. ' = 10.632kJ
I
Extent of reaction - Extent of reaction - Example 88. For the water gas reaction,
FiS{7.2S' P~~diction ofexf,~nt of reaction by the plot of free C(s)+HzO(g) CO(g)+Hz(g)
energy against extent of reaction
468 G.R.B. PHYSICAL CHEMISTRY FOR COMPETITIONS

the standard Gibbs free energy of reaction (at 1000K ) is [Hint: 13.Go = - 2.303RT log Kr
- 8.1 kJlmol. Calculate its equilibrium constant.
=- 2.303 x 8.314 x 29810g (4.42 x 104 )
Solution: We know that,
= -26.5 kJ mol- i ]
-llGo \
K = antilog (
2.303RT
j ... (i) 49. What is the sign of 13.0° and the value of K for an
Given that, llGo=-8.lkJ/mol electrochemical cell for which Ec~1l = 0.80volt?
R = 8.314 X 10-3 kJ K~l mol- 1 llGo K
T=looOK (a) >I
Substituting these values in eq. (i), we get (b) + >1

K = antilog
" -(" -81)
.
[ 2.303 x 8.314 x 10-3
1 (c) +
(d)
<1
<1
X 1000 [Ans. (a)]
[Hint: 13.GO nFE"; 13.Go =- 2.303RT log K]
.,K =2.65
Example 89. Calculate llG ° for the conversion of oxygen 50. The free energy of formation of NO is 78 kJ mol- 1 at the
to ozone, (3/2)0 2 (g) ~ 0 3 (g) at 298 K, if K p for this temperature of an automobile engine (1000 K). What is the
conversion is 2.47 x 10-29 • • equilibrium constant for this reaction at 1000 K?
Solution: We know that, ..! N 2(g)+ I 02(g)~NO(g)
llG 0 = - 2.303RT log K p
2 2
(a) 8.4 x 10- 5
(b) 7. I X 10-9
= 2.303 x 8.314 x 298 log 2.47 x 10-29
(c) 4.2 X 10- 10 (d) 1.7 X 10- 19
::::; 163228 J 1 mol
[Ans. (a)]
" . = 163.228kJI mol [Hint: K = antilog -13.GO J
Example 90. Acetic acid CH 3COOH can form adimer [ 2.303RT
(CH 3COOH}z in the gas phase. The dimer is held together by
two H-bonds with a total strength of66.5 kJ per mol ofdimer anti'1og [ -78xlOOO ]
0- lh 2.303 x 8.314 x 1000
~O .... H-O,
"
= 8.4 x 10-5 ]
CH3-C~ /-CH3
"O-H.... O " 51. Equilibrium constant for the reaction:
0+ 0-
H2 (g) + 12 (g)~ 2H1(g)
If at 25° C, the equilibrium constant for the dimerization is
1.3 x 10 3 . Calculate MO for the reaction: is K c 50 at 25°C.

2CH3COOH(g)~(CH3COOHh(g)
The standard Gibbs free energy change for the reaction will
be:
Solution: llGo =- 2.303 RT log K" (a) - 6.964 kJ (b) - 9.694 kJ
(c) ~ 4.964 kJ (d) - 6.496 kJ
= 2.303 x 8.314 x 29810g (1.3 x 103 ) [Ans. (b)]

= -17767.688J = 17.767kJ [Hint: 13.G " = - 2.303RT log Kc


= - 2.303 x 8.314 x 298 logSO
= - 9694 J =- 9.694 kJ]
-17.767=-66.5 298xMo 52. Standard Gibbs free energy change AGO for a reaction is
MO =- 66.5 + 17.767 0.163kJ
zero. The value of the equilibrium constant will be:
" 298 (a) 10 (b) 1 (c) 100 (d) 00
[Ans. (b)]
[Hint: 13.Go =- 2.303RT log k
WhenK = 1, 13.Go = 0]
48. What is 13.Go for this reaction?
53. The standard free energy change of a reaction is
1 3 AGO = - liS kJ at 298 K. Calculate the equilibrium constant
- N2 (g ) + - Hz (g)~ NH3 (g ); K p = 4.42 x 104 at 25° C
2 2 Kp in 10gKp. (R = 8.314 Jr l mol-I)
(a) - 26.5 kJ mol- 1 (b) -11.5 kJ mol- 1 (a)20.16 (b) 2.303 (c) 2.016 (d) 13.83
(c) 2.2 kJ mol- 1 (d) - 0.97 kJ mol-I (VITEEE 2008)
[Ans. (a)] [Ans. (a)]
CHEMICAL'J:I.IERMODYNAMICS AND THERMOCHEMISTRY 469

[Hint: log K = - AGo AG and emf of the cell (E) are related by the following
p 2.303RT relation:
- (-115 x 1000) AG -nFE
2.303 x 8.314 x 298 where, F =faraday = 96500coulomb
20.16] E = emf of the cell

7.30 PHYSICAL SIGNIFICANCE OF GIBBS n = Number of moles of electrons involved in


balanced electrochemical reaction
FREE ENERGY CHANGE (Free Energy
and Useful Work) If the reactants and products are in their standard states, then
AGO = -nFEo
According to first law of thennodynamics,
.. , (i) when E ° = standard emf of the cell.
AE=q-w
Example 91. The emf ofthe cell reaction,
(-ve sign as the work is done by system)
Zn(s) + Cu 2+(aq.)~ Zn 2+(aq.) + Cu(s)
Here, q is heat absorbed by the system and w is the work done by
the system while AE is the change in internal energy. The work is 1.1 V. Calculate free energy change for the reaction. Ifenthalpy
(w)actually includes two types of work, i.e., wexpansion (or PAy) of the reaction is 216.7 kJ mol -I, calculate the entropy change
and w non.expansion (useful work). The two types of works can be for the reaction.
understood by taking an example of the electrolysis of water to
fonn H2 (g) and 02 (g ) leading to an increase in the volume of Solution: -AGO = n x F x EO = 2 x 96500 x 1.1 = 212.3 kJ
the system. The work non-expansion (non-mechanical) is done to
cause the decomposition of water while the work expansion
AGO = - 212.3 kJ mol-I
(mechanical) is due to expansion in volume of the system. Thus, AGO = Ml" - T ASo
the electrical work is called non-pressure-volume work or AS o = Ml" - AGO = - 216.7 - (- 212.3)
non-expansion work or non-mechanical work. The T 298
non-expansion work is also known as useful work. Thus,
=-0.01476kJK- 1 mol-I
AE =q - wexpansion - W non-expansion ... (ii)
= q - P AV - wnon-expansion (.: wexpansion = P AV) =-14.76JK- 1 mol-I
or q =AE + PAV + wnon-expansion ... (iii) E1'ample 92. Calculate equilibrium constant for the
following reaction:
We know that,
Cu(s) + 2Ag+ (aq. )r:=:Cu 2+(aq.) +'2Ag(s)
AE+PAV All
At 25°C, E':elt == 0.47 volt, R = 8.314 J K -I mol-I, F 96500
or q = All + wnon-expansion ... (iv) coulomb.
According to second law of thennodynamics, for a reversible Solution: Let us apply Nernst equation at eqq.ilibrium
change taking place at constant temperature, T
E:ell = 0.0591 log Kc
AS= n
T
0.47= 0.0591 log Kc
or qrev, T AS ... (v) 2
Substituting in equation (iv)
K c = anti'1og [0.47 x 2]
T AS = All + W non -expansion ... (vi) 0.0591
or All - T AS =- W non -expansion =8.5x 1015,
Or AG -- - W non -expansion (.: All - T L.lJ.)
A C' =AG) ; , , .>~.;} . ' ·:;."'< .. ~'-t ~. .' •

or -AG = W non -expansion work


W useful ,.. (vii) 1M" ABSOLUTE ENTROPIES ANDTH1RO
Thus, the decrease in Gibbs free energy is a measure ofusefol "LAW OF THERMODYNAMICS -:.;':,
work or non-expansion work done by the system. The greater the
Third law of Thennodynamics was given by T. W. Richards,
free energy change, the greater is the amount of work that can be
Walter Nernst and Max Planck Some statements for third law Of
obtained from the process. This relation is useful in assessing the
thennodynamics are given below:
electrical work that may be produced by electrochemical cells
and fuel cells. Statement-l. "Every substance has a finite positive entropy.
but at the absolute zero temperature the entropy may become zero
Free Energy Change and Electrical and does so become in the case ofperfectly crystalline solids,"
Work Done in a Cell Beeause
Free energy change in electrochemical cells is related to the Statement-2. "Entropy of a solid or liquid approaches zero
electrical work done in the cell. at the absolute zero of temperature. "
470 G.R.B. PHYSICAL CHEMISTRY FOR COMPETITIONS

It can be mathematically expressed as CuS04· 5H20 300.4 Hg 76.0


lim S ~O C I2H22o.11 (sucrose) 360.8 H 2O 69.9
T-40K
CaO 39.8 C 2H sOH 160.7
Because CaC0 3 (calcite) 92.9 ~H6. 173.3
. Statement-3. "It is impossible by (my method, no matter H2 130.7 CHlCOOH 159.8
how idealised it is, to reduce the temperature ofany system to the
absolute zero in a finite number of operations. " Tephigraph
Graph of entropy of a substance against temperature is called
Application of Third Law of Thermodynamics
tephigraph. In the following graph variation of entropy with
The third law of thermodynamics has been useful in temperature is represented:
calculating the absolute entropies of solids, liquids and gases at
different temperatures. Moreover, this law is also useful to
calculate entropy changes of a chemical or physical prqcess. Let
us calculate absolute entropy. . .
dS=dQ
T Entropy
.. dT
dS=C p -
T
t
On integrating the above equation .between temperature limits of
o and T K, we get
T dT
ST-SO=r Cp -
Jo ·T.
or S T - So = f: C pd In T - - Temperature in K
Fig. 7.24
From third law of thermodynamics, So = 0

.. ST = f: C p d In T
.From the graph it is clear that entropy of a substance increases
with increase in temperature; there is sudden change in entropy at
the stage of phase transformation.
When entropy ofone mole ofa substance is expressed at 298K
and 1 aim pressure, it is called standard entropy of that Example 93. Calculate /).G °for the folloWing reaction:
substance. It is denoted by S 0. The standard entropy change M ° 1
for a chemical reaction can be calculated as CO(g) + (- ) 02 (g) ~ CO 2 (g); MfO = - 282.84 kJ
2
MO = LS~produc~) - ~S~reactants) Given,

[Sum of the standard] [sum of the standard


= entropies of products - ~tropies of reactants
] S~02 = 213.8J K

S~2 = 205.0J K
-I

-I
mo/- I , S~o(g) = 197.9J K
mol -I.
-1 mol-I,

e. g. , in the reaction:
C 2H6 (g ) ~ H2 (g) + C 2H4 (g)
Solution: /).SO = ~S ~products) - LS ~reactants)

° -
= [Se02] ° + -1·So° ]
/).SO = ~S~products) - ~S~reactants) [Seo
2 2
= [S~2 + S~H4]"- [S~2H6] = 213.8- [197.9+.! 205]
2
I
Standard molar entropies in JIK mol at 298 K = - 86.6JK-

Solids Entropy SolIds Entropy According to Gibbs-Helmholtz equation,


/)'Go=Mr-T MO
C (graphite) 5.7 N2 191.6 = - 282.84 - 298 x (- 86.6 X 10-3 )
C(diamond) 2.4 °2 205.1
= - 282.84 + 25.807
Fe 27.3 CO 2 213.7 = - 257.033 kJ
Pb 64.8 NO. 2 240.1 . Example 94. For the reaction;
Cu 33.1 N2o.4 304.3 A(s)~B(s)+ C(s)
Al 96.2 NH3 192.3
calculate the entropy change at 298 K and 1 aim if absolute
Fezo. 3 CH4 . 186.2
87.4 entropies (in JK- I mor l ) are:
CHEMICAL THE:RMODYNAMICS AND THERMOCHEMISTRY 471

A = 130,B 203,C= 152 volume V3 (point C). In this operation, no heat is absorbed and
o 0 thus, the work, wz, is done by the system at the expense of
Solution: .'ES (products) - ~~ (reactants) internal energy, i. e., the temperature of the system falls from
temperature T2 to temperature TI .
[S~ +S~]-[S~],
11E=-w2 =-CV (T2 -Td ... (ii)
=[203 + 152] [130]
=225JK- I Third operation-Isothermal reversible
compression
7.32 CONVERSION OF HEAT INTO WORK- The gas at point C is subjected to reversible isothermal
THE CAR NOT CYCLE compression at temperature TI •The path followed is CD when the
volume decreases from V3 to V4 (point D). In this operation work
Carnot, a French engineer, in 1824, employed merely theoretical is done on the system and heat ql is given out by the system to
and an imaginary reversible cycle known as Carnot cycle to surroundings.
demonstrate the maximum convertibility of heat into work.
The system consists of one ~ole of an ideal gas enclosed in a .... (iii)
cylinder fitted with a piston which is subjected to a series of four
successive operations. The four operations are:
(i) Isothermal reversible expansion Fourth operation-Adiabatic reversible· .
compreSSion
(ii) Adiabatic reversible expansion
(iii) Isothermal reversible compression Finally, the gas at point D is subjected to reversible aqiabatic
compression through the path DA at temperature TI . The volume
(iv) Adiabatic reversible compression
changes from V4 to VI , i. e. , the original volume is restored. The
The four operations have been shoWn in Fig. 7.25.
temperature increases from TI to T2 (original temperature). The
(Pl, V1) work done increases the internal energy of the system as q ::: Q .
---- W4 Cv(T2 -T1 ) ... (iv)
The net heat absorbed, q, by the ideal gas in the cycle is given
by
V V
., q = q2 + (-ql) RT2 loge - 2 + R1I loge - 4
VI V3
V V3
=RT2 loge - 2 RTI loge- ... (v)
·VI · V4
According to the expression governing adiabatic changes,

T2 =(V3 1Y I . (For adiabatic expansion)


11 V2 )
Volume - -
Fig. 7.25 1I=(!l)Y
T2 V
I (For adiabatic compression:)
4
First operation-Isothermal reversible expansion
The gas is allowed to expand reversibly and isothermally at V34 V
or' -=-
the temperature T2 through AB path. The volume VI (point A) V2 VI
increases to volume V2 (point B). In this operation, let the heat V V3
absorbed by the system be q2 (+ve) and the work done by the or -2= -
system on the surroundings be WI (-ve). VI V4
=
Since, the expansion is isothermal, I1E O. The heat absorbed, Substituting the value of V3 in eq. (v),
q 2' should be equal to the work done by the system on the V4
surroundings, i. e. ,
q = q2 - ql = RT2 log e
... (i) VI
. . V2
Second operation-Adiabatic reversible
. = R(T2 - T, ) log e v:-
I
... (vi)

expansion
Similarly, net work done by the gas is given by
The gas is now allowed to expand reversibly and adiabatically
through BC path. The volume increases from V2 (point B) to
G.R.B. PHYSICAL CHEMISTRY FOR COMPETITIONS

'., Example 96. Heat supplied ~o u Camut engine is 453.6


kcal. How much usefol work can be done by the engine which
works between 10° C and 100° C?

Solution: T2 = 100 + 273 ;:::: 373 K; Tl 10+ 273 = 283 K;

q2 = 453.6 x 4.184 = 1897.86 kJ


We know that,

W=q2 .-=--..:..
T2
=R(T2 -11 ) log e V. ... (vii)
=1897.86 x (373 - 283)
I
373
Thus, q == w. For cyclic process, the essential condition is that
net work done is equal to heat absorbed. This condition is 1897.86 x 90 457.92 kJ
satisfied in a Camot cycle. 373

Calculation of thermodynamic efficiency of 'l~f·(if;'Qi_J.:iQUilij"S,[ . :1


Carnot engine
54. For which of the following reactions, the entropy change will
Total work done from eq. (vii),
be positive?' .[JEE (WB) 2008}
" v: (a) Hz(g)+ 12 (g)~ 2HI(g)
w= R(T2 -11 ) loge 2-
VI (b) HCI (g) + NH3 (g)...-- NH4Cl (s)
(c)NH4N03 (s) NzO(g)+ 2H20(g)
and the heat absorbed at temperature Tz from eq. (i),
(d) MgO(s) + H2(g)~ Mg(s)+ Hz0(l)
z V [Ans. (c)]
RTz loge- [Hint: ~ng == 3 0 = 3,
VI
Since ~ng >0, there will be increase in entropy change.]
Dividing both equations, 55. If an endothermic reaction occurs spontaneously at constant
temperature T and pressure P, then which of the following is
~ = Tz -11 Thermodynamic efficiency true ? (VITEEE 2008)
q2 T2 (a) AG > 0 (b) Ml < 0 (c) L\S > 0 (d) L\S < 0
[Ans. (c)]
Thus, the larger the temperature difference between high and
[Hint: AS =ASsystem + ASsurroundings
low temperature reservoirs, the more the heat converted into
work by the heat engine. AS > 0 for spontaneous process.]
56. Which is the correct expression that relates changes of
Since, T2 - TI < 1, it follows that w < q 2' This means that only entropy with the change of pressure for an ideal gas at
, T2
constant temperature, among the following?
a part of heat absorbed by the system at the higher temperature is (PET (MP) 2008)
transformed into work. The rest of the heat is given out to p
surroundings. The efficiency of the heat engine is always less (a) ~S = nRT In -1. (b)L\S=T(P2 11)
than 1. This has led to the following enunciation of the second
11
law of thermodynamics. (c) L\S =: nR lnr 11 J (d) I'1S = 2.303 nRT In (~)
It is impossible to convert heat into work without ' \P2
compensation. [ADS. (c)]
Example 95. Calculate the maximum efficiency ~f an [Hint: From first law, I'1U = q W
engine operating between 100° C and 25° C. o=q - W (for isothermal process)

Solution: Efficiency
T2 11 q= W =PdV

T2 dS =!1. = PdV
T T
Tz 100+273= 373K dV
dS=nR-
11 = 25+ 273 298K V
.
EffilCiency =
373 - 298 75
=- 0.20 20% On integration, ~S =nR In ( ~~ )
373 373

I'1S = nR In (~ } Since PI VI =FF2 according to Boyle's law.]


CHEMICAL THERMODYNAMICS AND THERMOCHEMISTRY 473--

MISCELLANEOUS NUMERICAL EXAMPLES~ -'-'-'-'/


Example 1. Bond dissociation enthalpies of H 2 (g) and Mil -Mil =!::'C
p
N z (g) are 436.0 kI marl and 941.8 kI mor l and enthalpy of Tz -TI
formation of NH) (g) is 46 kI mor I. What is enthalpy of
MI 2 - (-11.02) = (8.38 _ ..!. x 6.96 ~ x 6.89) X 10-3
atomization of NH 3 (g)? What is th.e average bond enthalpy of 7TI-~8 2 2
N-Hbond?
Solution: N2 (g) + 3H z (g) ~ 2NH3 (g) ; MI 2 == - 13.6 kcal morl
Example 4. The heat of combustion of glycogen is about
MI "" - 2x 46kJ/mol
476 kllmol of carbon. Assume that average rate of heat loss by
MI I:(BEh - I:(BE)p an adult male is 150 watt. If we were to assume that all the heat
comes from oxidation of glycogen, how many units qf glycogen
(941.8 + 3 x 436) - (6x)== - 2 x 46 (1 mole carbon per unit) must be oxidised per day to provide for
(Here, x == BE of N - H bonds) this heat loss?
Solution: Total energy required in theday
x = 390.3 kJ mol- I
= 150x 24 x 60x 60 kJ (1 watt =Jsec-1 )
NH) ~N+3(H) 1000
Heat of atomization 3 x 390.3 == 1170.9 kJ mol- I = 12960kJ

-Example 2. Ethanol was oxidised to acetic acid in a · 0f


Umts lrequIre
g ycogen ' d 12960 27 .22 umts
.
catalyst chamber at 18° C. Calculate the rate of removal qf heat 476
to maintain the reaction chamber at 18° C with the feed rate of30 Example 5. At 25°C, the following heat offormations are
kg per hour ethanol along with excess oxygen to the system at given:
18° C, given that a 42 mole per cent yield based on ethanol is Compound S02 (g)
obtained. Given that, -296.81
l
MI~ kllmol
MlfHiJ(I) "" - 68.4 keal mor
For the reactions at 25° C,
MlfC2H 1)H(I) == - 66kcal mor l 2HzS(g) + Fe(s)~FeS2(s)+ 2Hz(g);Mlo == -137 kllmol
l
MlrcH3COOH(I) = -118keal mor 3
H2S(g)+-02(g)~H20(l)+S02(g); Mlo =-562k1lmol
2
Solution: C 2H sOH+02 ~CH3COOH+H20
Calculate heat offormation ofH 2 S (g ) and FeS 2 (s) at 25° C.
MI=I:(Hf)p I:(Hf)R =(-118-68.4)-(- 66) 120.4 kcal
Solution: Heat of formation of H2 S = x kJ I mol
Yield is 42%. Thus, energy produced per hour will be
Heat of formation of FeS2 = y kJ I mol
120.4 x 42 x 30000 = 32979.13 kcal hour-I
lOOx 46 2H 2S(g) + Fe(s) FeS2 (s)+ 2H2 (g); Mlo = -137kJ/mol
-137= y 2x ... (i)
Example 3. The standard heat of formation listed for
gaseous NH 3 is 11.02 kca/lmol at 298 K. Given that at 298 K. 3
H2 S(g) + - O 2 (g) ~ H 2 0(l) + S02 (g );Mlo == - 562 kJI mol
the constant pressure heat capacities qf gaseous N 2 , H 2 and 2
NH 3 are respectiveZv 6.96, 6.89, 8.38 callmol. Determine
-562 285.83 - 296.81- x ... (ii)
MI~98 K and MI 773 K for the reaction, 1
From eq. (ii), x -20.64kJmol-
1 3
N z (g) + - Hz (g) ~ NH, (g ) From eq. (i), we get
2 2 -
I 3 y= 2x 137 2(- 20.64)-137= -178.28kJ mol- J
Solution: N2 (g)+- H2(g)~ NH 3 (g)
2 2 MI f H 28 == 20.64 kJI mol
Ml 298K I:(Hf)p -I:(Hj)R =(-11.02 0) MI f FeS2 178 kJ/mol
1
11.02 kcal mol-
I

474 G.R.B. PHYSICAL CHEMISTRY FOR COMPETITIONS

EJample 6. The "heat oftotal cracking" ofhydrocarbons Solution:


MI TC is defined as MI at 298.15 K and 10 1.325 kPa for the H Ii H H
process below: II I I
-~) H 2 (g)---t'nCH 4(g)
4C(s) + 5H2 + S---tH-C-C-S-C-C-H
CnHm + ( 2n
I I I I
H H H H
Given that, MITe is 65.2 kl for C 2 H 6 and - 87.4.kl for
C 3H g, calculate MI for H H H H
CH4 (g)+ C 3 Hg (g)---t 2C 2 H 6 (g) II I I
4C(s) + 5H2 + 2S ---t H-C-C-S-S-C-C-H
Solution: MI TC of CH4 0
I I I I
CH 4 (g) + C3 Hg (g) ---t 2C 2 H 6 (g) H H H H
MI = 2M1TdC2H6)-:- MlTC<~3H8) MI = ~(BEh - ~(BE)p
= 2(- 65.2)- (- 87.4)= - 43kJ (i) 147.2= Heat of atomization of 4C, IOH, IS
'l£-iample 7. A constant pressure calorimeter consists ofan -BE of 1O(C- H), 2(C- S), 2(C~ C)
insulated beaker of mass 92 g made up of glass with heat (ii) - 201.9 == Heat of atomization of 4C, 10H, 28
capacity 0.75 J K- 1 g-l. The beaker contains 100 mL of 1 M -BE of IO(C- H), 2(C~ S), 2(C-C),(S- S)
HCI at 22.6°C to which 100 mL of1M NaOH at 23.4°C is aaded. Subtracting (i) from (ii),
The final temperature after the reaction is complete at 29.3 0 C. - 201.9+ 147.2= Heat of atomization of IS - BEof (S-8)
What is MI per mole for this neutralization reaction? Assume
that the heat capacities ofall solutions are equal to that of same ::::: 222.8kJ BEof(S-S)
volumes of water. . BE of (S-S) = 277.5 kJ
Solution: Initial average temperature of the acid and base ;.'" Example 10. A natural gas may be assumed to be a
= 22.6+ 23.4 23.00 C mixture of methane and ethane only. On complete combustion of
2 10 litre of gas at STp, the heat evolved was 474.6 kl. Assuming
1
MlcombCH4(g) 894klmol- andMlcombC2H6=-1500kJ.
Rise in temperature = (29.3 23.0) = 6.3° C
Calculate the percentage composition of the mixture by volume.
Total heat produced =(92x 0.75+ 200 x 4.184)x 6.3
Solution:
(905.8) x 6.3 :: 5706.54 1
x litre ---t CH 4 ; mole of CH4 = x 122.4
Enthalpy of neutralization 5706.54 x 1000 x 1
(10 x) litre ---t C2 H 6 ; mole ofC 2 H 6 == (10 - x)1 22.4
100
x (lO-x)
57065.4 1 = - 57 kJ Heat evolved = - - x 894 + x 1500
22.4 22.4
EJample8. C 2H 4 +C/2 ---tC2H 4C1 2
474.6 x x 894 + (10- x) x 1500
MI = 270.6 kl mol -I K -I ; 6.S = 139 JK-1 22.4 22.4
(i) Is the reaction favoured by entropy, enthalpy both or
x == 0.745, % CH 4 =: 74.5%
none?
(it) Find 6.G if T =300K. [eDSE (Mains) 2005] Example 11. From the data at 25° C:
Solution: (i) Since, MI =: - ve, exothermic process and is
Fe203 (s)+ 3C(graphite) ---t 2Fe(s) + 3CO(g);
will
favoured, i. e. , it be spontaneous.
Mlo= 492.6kllmol
(ii) 6.G == MI - T6.S
= 270.6xlOOO-300x(-139) FeO(s) + C(graphite) ---tFe(s)+ CO(g); Mlo = 155.8kllmol
= 2289001 C(graphite) + 02 (g) ---t CO 2(g); MlO = - 393.51 kllmol
-228.9kJ . 1
CO(g) + '2°2 (g) ---t CO 2 (g); Mlo =: 282.98 kllmol
Example 9. Find bond enthalpy of S - S bond from the
following data: . Calculate standard heat offormation ofFeO(s) and Fe203 (s).
C2HS -S -C2HS; MIl =-147.2kl mor l Solution:
Fe(s) + CO(g)---t FeO(s) + C(graphite);
C 2H S-S S-C 2 H S; MIl =-201.9klmor l MlO 158.88 kJ/mol
l
S(g); MIl = 222.8 kl mor
CHEMICAL THERMODYNAMICS AND THERMOCHEMISTRY 475~

C(graphite) + O2(g) ~ CO 2 (g);


or Q = W= 2.303 nRTlog (;:)
AHO 393.5 kJlmol
1
C02(g)~ CO(g)+"2 02 (g); = - 2303 x Ix 8314 x 273 log (_1 )
. Ql
AHo = 282.98kJ/mol 5227.169J = 5.227kJ
I .. .
On adding Fe(s) + - O 2(g) ~ FeO(s); . AH:::: -.2663 kJ/mol Example 15. A sample of argon at 1 atm pressure and
2· . . .. ' ...; .
. 27° C expands reversibly and adiabatically from 1.25 dm 3 to 2.50
Similarly we may calCulate heat of formation ofFe203' dm 3. Calculate the enthalpy change in this process. C v.m. for
Jt~ample 12. Calculate, the AH at 85° C for the reaction: argon is 12.48 J K -I mol -I. (liT 2000)
Fe203(s)+ 3H2(g)~2Fe(s)+ 3H 20(l) Solution: Number of moles of argon present in the sample
The data are: AH~9S = 33.29kllmoland = PV = 1.25 x 1 = 0.05075
RT 0.0821 x 300
Substance Fez03(s) Fe(s) H 20(I) H2(g)
For adiabatic expansion,
C; JIK. mol
T (V )r-I .
103.8 25.1 753 28.8
Solution: Fe203(s)+3H2 ~2Fe+3H20 -L=...l.
T2 VI
I1C p =(Cp)p - (Cp)R
= (2 x 25.1 + 3 x 753) - (103.8 + 3 x 28.8) or 300=(2.50)1.66-1
T2 1.25
= 276.1-190.2= 85.9J
or T2 = 188.55K
I1H T2 AH TI =fj,C p C p =Cv +R
T2 1\
= 12.48+ 8314
AHsss - (-33.29) 85.9x 10-3
20.794 J K -I mol-I
358- 298
AH n x C p x fj,T
AH3S8 =-28.14kJ/mol
= 0.05075 x 20.794 x (300 - 188.55)
Example 13. The standard heats offormation at 298 K for
CCI 4 (g),·H 2 0(g), CO 2(g) and HCI(g) are 25.5,-57.8, =117.6J
94.1 and - 22.1 kcal mol -I respectively, calculate AH~98K for Example 16. Show that the reaction,
the reaction: CO(g)+G)02(g)~C02(g)
CCl4 (g) + 2H20(g ) ~ CO 2 (g) + 4HCI(g)
at 300 K is spontaneous and exothermic, when the standard
[BCECE (Mains) 2005]
entropy change is 0.094 kl mol -I K- I . The standard Gibb ~
Solution:
free energies of formation for CO 2 and CO are - 394.4 and
AH reaction = 1: Heat of formation of products
- 137.2 kl mol -I respectively. (lIT 2000)
. -1: Heat of formation of reactants
Solution: The given reaction is,
= [AHf(C0 2) + 4 AHf (HCI)] - [AHf (CCI4)
= [- 94.1+ 4 x (- 22.1)]- [- 25.5+ 2x (- 57.8)]
+ 2AHf (HP)]
CO(g) + G)02 (g)~ CO 2 (g)

= - 41.4 kcal
fj,G (for reaction) =
o
G~02 - G~o G)G~2
Example 14. Calculate Q, W, I1E and AH for the
isothermal reversible expansion of 1 mole ofan ideal gas from an 394.4 -(- 137.2) 0
initial pressure of 1.0 bar to a final pressure of 0.1 bar at a
= 257.2 kJ mol-I
constant temperature of273 K. (liT 2000)
Solution: In isothermal process as temperature remains I1G = f1HO - TI1S o
o

constant both I1E and AH are zero. 257.2 = f1HO - 298 x (0.094 )
11E=0; AH 0
or f1HO = 288.2 kJ
Applying first law of thermodynamics,
I1G ° is ve, hence the process is spontaneous, and AH ° is
I1E=W+Q
also -ve, hence the process is also exothermic.
O=W+Q Example 17. Diborane is a potential rocket foel which
undergoes combustion according to the reaction,
476 G.R.B. PHYSICAL CHEMISTRY FOR COMPETITIONS

B zH 6 (g)+ 302 (g)----tBP3 (s)+ 3H20(g) 2x 50 100 0


From the following data, calculate the enthalpy change for the We know that, .AG = AGO - 2.303 RT log Q
combustion of diborane: = 0 - 2.303 x 8.314 x 29810g 10
2B(s) + (~)02 (g) ----t B 20 3 (s); (All = 1273 kJ mor = -5705J
l
) ••• (i)
Since, AG is negative hence reaction will be spontaneous in
H 2 (g)+O)02(g)----tH 20(l); (All: 286kJ mol-' ) ... (ii) forward direction.
Example 20. When I-pentyne (A) is treated with 4N
H 20(l)----tHP(g);(All 44kJmol- l ) ••• (iii) alcoholic KOH at 175°C, it is slowly converted into an
equilibrium mixture of 1.3 % of I-pentyne (A). 95.2% 2-pentyne
2B(s) + 3H2(g)----t B2H6 (g); (All = 36kJ mol-I ) ... (iv)
(B) and 3.5% of 1,2-pentadiene (C). The equilibrium was
(lIT 2000) maintained at 175°C. Calculate AGO for the following equilibria:
Solution: The required equation,
B A; AG~ =?
B2H6 (g) + 302 (g) ----t B20 3 (s) + 3H2 0(g)
can be obtained from B~C; AG; =?
eq. (i) + 3eq. (ii) + 3eq. (iii) - eq. (iv) From the calculated value of AG~ and AG; indicate the order of
: - 1273"": 858 + 132 - 36 stability of (A), (B) and (C). Write a reasonable reaction
. mechanism showing all intermediates leading to (A), (B) and
: 2035 kJ mol- 1 (C). (lIT 2001)
i. e., Enthalpy of combustion of diborane is -2035 kJ mol-I. Solution:Pentyne-1 Pentyne-2 + I, 2-pentadiene
Example 18. An insulated container contains 1mole of a (A) (8) (C)
liquid, molar volume 100 mL at 1 bar. When liquid is steeply 1.3 95.2 3.5
passed to 100 bar, volume decreases to 99 mL. Find l!Jl and AU [B][C] = 95.2x 3.5 256.31
for the process. rlIT 2004 (Memory based)]
[A] 1.3
Solution: From first law of thermodynamics:
AU=q+W B~A

AU=q+PAU 0+~100~9 100» K =[A]= [C] =~=0.013


:: 100barmL I [B] Keq 256.31
All=AU+(PzVz -P1V,)
= 100+(100x 99-lx 100)
AG~ = - 2.303RT log KI
:: 9900 bar mL = - 2.303 x 8.314 x 448 log 0.013
Example 19. In the following equilibrium: = 16178J= 16.178kJ

N204(g)~2N02(g) for B~C

when 5 mole ofeach are taken and temperature is kept at 298 K, K = [C] = Keq [A] = 256.31 x 13 = a037
the total pressure was found to be 20 bar. 2 [B] [8]2 (95.2)2
Given: AGf NP4 = 100kJ
AG; = - 2.303RT log K 2
AGf NOz =50 kJ = 2.303 x 8.314 x 448 log 0.037
(i) Find AG Qfthe reaction at 298 K.
= 12282J = 12.282kJ
(ii) Find the direction of the reaction.
lIlT 2004 (Memory based)] Stability will lie in the order
Solution: The reaction is: B>C>A
N 2 0 4 (g)~ 2N02 (g) ',Example 21. Two moles of a perfect gas undergo the
Since, number of moles of both N Z0 4 and NO z are same following process:
hence their partial pressure will also be same. (a) A reversible isobaric expansionfrom (latm 20 L)to (latm
20 40L).
PN:P4 = PN0 2 = 2: =lObar
(b) A reversible isochoric change ofstate from (1 atm 40 L) to
(0.5atm 40L).
(c) A reversible isothermal compression from (0.5atm 40 L )to
(latm 20L).
(i) Sketch with labels each of the process on the same P-V
diagram.
CHEMICAL THERMODYNAMICS AND THERMOCHEMISTRY 477

(ii) Calculate the total work (W ) and the total heat change =- 1x 20 x 101.3 + 0
( q ) involved in the above process.
(iii) What will be the value of AU,!J.H and AS for the overall
+ 2.303 x 2x 8.314 x T log (~~) ... (1)
process? (lIT 2002)
PV = nRT (At A)
Solution:
Ix 20= 2x 0.0821 x T
T= 20 =121.8K
2xO.0821
i!!! 1 atm
AB Fromeq. (1),

J 0.5 atm
:::::::::::::rS;lI
,
I
I
,
I
I
I
Total work = 2026 + 2.303 x 2 x 8.314 x 121.8 log 2
= -622.06J
W q -622.06J
I
,
I
I In cyclic process:
, I
AU =O,!J.H =oand AS = 0
20L 40L
Volume ---to-

AB ---t Isobaric process


BC ---t Isochoric process
CA ---t bothennal compression
Total work WAB + WBC + WCA
=-P x AV + 0+ 2.303 nRT log (V2)
. VI

';-. ...SUMMARY
,., ,.. AND IMPORTANT POINTS TO REMEMBER
'~l )-. .....,,Ie ....
1. Thermodynamics: It is the branch of science which boiling points, etc. (ii) Extensive properties are those whose
deals with all changes in energy or transfer of energy that magnitude depends upon the quantity of matter present 111 .be
accompany physical and chemical processes. It is not concerned system such as volume, total energy, enthalpy, entropy, etc.
with the total energy of body but only with energy changes taking (d) State variables: The fundamental properties which
place. The laws of thermodynamics apply only to matter in bulk, determine the state of the system are termed state variables. The
i.e., to macroscopic system· and not to individual atoms or change in state property depends only upon the initial and final
molecules. states of the system, i.e., do not depend on the path followed.
2. Terms used in thermodynamics: Pressure, volume, temperature, internal energy, enthalpy, entropy,
(a) System, surroundings and boundary: A system is force, energy and number of moles are the state variables.
defmed as a specified part of the universe which is under (e) Thermodynamic equilibrium: A system in which the
experimental investigation and the rest of the universe which can fundamental properties do not undergo any change with time is
interact with the system is surroundings. Anything which said to be in thermodynamic equilibrium. Actually the system
separates system and surroundings is called boundary. It may be should be in thermal equilibrium, mechanical equilibrium and
real or imaginary, conductor or non-conductor. chemical equilibrium.
(b) Types of system: There are three types of system: (1) Thermodynamic processes: When the system changes
(i) Isolated system is one which has no interaction with its from one state to another, the operation is called a process. The
surroundings. The boundary is sealed and insulated. Neither various types of processes are:
matter nor energy can be exchanged with surroundings. (0 Isothermal: Temperature remains fixed, i.e., dT 0.
(ii) Closed system is one which can exchange energy but not This is achieved by placing the system in a thermostat.
matter with surroundings. (iii) Open system is one which can (ii) Adiabatic: When no exchange of heat occurs between
exchange matter as well as energy with surroundings. A system is the system and surroundings, i.e., the system is thermally isolated
said to be homogeneous if it is rrutde of one-phase only. A system (dq = 0).
is said to be heterogeneous when it consists of two or more (iii) Isobaric: Pressure remains constant throughout the
phases. change, i.e., dP O.
(c) Thermodynamic properties: These are of two types:
(iv) Isochoric: Volume remains constant throughout the
(i) Intensive properties are'those which do not depend upon the
change. i.e., dV =O.
quantity of matter present in the system such as pressure,
temperature, specific he.~:surface tension, viscosity, melting and
.,-:'l
478 G.R.B. PHYSICAL CHEMISTRY FOR COMPETITIONS

(v) Cyclic: When a system undergoes a number of different When heat energy ( q ) is given to a system and work ( w) is also
processes and finally returns to initial state, dE = 0 and dH 0 done on it, its energy increases from E I to E 2 .
(vi) .Reversible: A process which occurs infinitesimally E2 =EI +q+w or E2 -El =q+w or M:::q+w
slowly and at every small change it is virtually in a state of
In case heat energy (q) is given to a system and .work (w) is
eqUilibrium. It takes infinite time.
done by the system then
(vii) Irreversible: When the process goes from initial to
final state in single step in fmite time. All natural processes are M = q + (-w) = q w
irreversible. 6. Enthalpy or heat content: .The' quantity E + PV is
3. Nature of work and heat: Work is a mode of transfer known as entluilpy or heat .content. Jt is denoted by H. It
of energy to or from a system with reference to surroundings. If represents the total energy stored i~ a system.
an object is displaced through a distance dx against a constant M = q -P(V2 ViJ or E2 - EI =-q - PV2 + PVI
force F , then the amount of work which has to be done is equal to
F x dx. The mechanical work or pressure volume work is equal to or (E2+P2V2)-(EI+PIVI)=q or H 2 -HI :::q
P ext (V2 ~ VI ) or P x dV. The work done on the system increases or !lH =q or M + P IlV !lH qp
its energy while work done by the system decreases its energy. when V2 =VI' i.e., Il V::: 0, M ::: qv
On this basis, the work done has the sign convention. qv +llnRT qp in the case of gases
Work done by the system = - w
where, Iln ::: total number of moles of products total number
Work done on thesystem=+ w
The unit of work is erg. It is very small. The bigger unit is . of moles of reactants
joule or kilojoule. . 7. Heat capacity: . It is the quantity of heat required to
7
1 joule = 10 ergs; 1kJ (kilojoule) = 1000 J .raise the temperature of the system by one degree.
Heat is the quantity of energy which flows between system dq
Heat capacity ::: -
and surroundings on account of temperature' difference. It is equal dT
to m x s x At where m is the mass, s is the specific heat and Ilt is
the temperature difference. The sign convention for heat is: In the case of gases;
Heat gained by system = + q
Heat lost by system = -' q
C v (at constant volUll)e) :::( ~~) V
The unit of heat is calorie (cal). Since, heat and work are
.;!::elated the SI unit of heat is joule. ::: Molar heat capacity at constant volume
1 joule = 0.2390cal .
and C p (at constant pressure) =
(aH)
- ..
1calorie = 4.184 J . .. . aT p
'L I kcal =4.184 kJ
::: Molar heat capacity at.constant pressure
1litre-atm = 101.3 J =·1.013 x 109 erg = 24.206cal
C p -Cv:::R
4. Internal energy: Sum of all forms of energy that a
8. Isothermal expansion of an ideal gas:. The expansion
system possesses is termed internal energy. It is denoted by E.lt is
in an ideal gas at constant temperature can be done reversibly or
an extensive property. It is also a state property.
irreversibly. .
E ::: E translational +. E rotational + E vibrational + E bonding
w Work done by gas in reversible expansion
+ E electronic + ...
. V . P
The internal energy of a particular system is a defInite quantity ::: - 2.303nRT log ...1.. 2.303nRT log _I
but it is impossible to determine its exact value. It is a state VI . P2
function. In thermodynamics, one is concerned only with energy
q =Heat abS()rbed from surroundings
change which occurs when the system moves from one state to . '. . V
ancitherJe., M ::: Ej - Ei. M is +ve if E f > E; andnegativeif =- w 2.303nRT
.
log ...1..
. VI
E f < E i' M does not depend on the path by which final state is
achieved. For a cyclic process, MO. W = Work done by gas in irreversible expansion
5. First law of thermodynamics: Energy can neither be
created nor destroyed but it can be converted from one form to = - Pext (V2 - VI )
another. Maximum work i~ done in reversible isothermal expansion. In
isothermal process, M =,.oand IlH= O. . '. '.. . .
.Or Adiabatic expansion of an ideal' gas: q in the process is
The energy of the Universe is constant. zero, hence M = w. Work is done by the gas during expansion at
Or the expense of internal energy. In expansion, M decreases while
Total energy of an isolated system remains constant, though it in.compression M increases. .
may change from one form to another.
M = w = C v x IlT
I

CHEMICAL THERMODYNAMICS AND THERMOCHEMISTRY 479

The value of !:!.T depends upon the process whether. carried 13. Heat offormation or enthalpy offormation: It is the
reversibly or irreversibly. The following relationships are amount of heat evolved or absorbed when one gram mole of
followed by an ideal gas under adiabatic conditions: substance is directly obtained from its constituent elements. It is
represented as !:!.H f.
PV Y = constant If all the substances of the chemical reaction are in their
standard states (i.e., at 25° Cor 298 K and one atmospheric
pressure), the heat of reaction or fonnation is called standard heat
of reaction or fonnation respectively. These are denoted as Mfo
or Mt;- respectively. The enthalpies of free elements at standard
9. Joule-Thomson effect: . The phenomenon of producing conditions are taken arbitrary as zero.
lowering of temperature when a gas is made to expand The compounds which have positive enthalpies of fonnation
adiabatically from a region of high pressure into a region of low are called endothermic compounds (less stable) and which have
pressure, is known as Joule-Thomson effect. This effect is negative enthalpies of fonnation are called exothermic
observed below the inversion temperature ofa gas. The inversion compounds (stable).
temperature ofa gas depends upon the van der Waals' constants .14. Heat of combustion or enthalpy of combustion: It is
of a gas. It is expressed in tenns of vap der Waals' constants 'a' the amount of heat evolved when one mole of the substance is
and 'b' as: completely oxidised. It has a number of applications:
2a (i) Detennination of calorifIc values of foods and fuels.
r. =Rb
I
- (ii) Enthalpy of fonnation can be determined by application
of Hess's law.
10. Thermochemistry: It is concerned with heat changes (iii) Structural problems can be evaluated.
accompanying physical and chemical transfonnations. It is also 15. Enthalpy of hydrogenation: The enthalpy change
tenned as chemical energetics. It is based on the fIrst law of associated when one mole of an unsaturated organic compound is
thennodynamics. If E J and E2 represent total energies associated fully hydrogenated.
with reactants and products respectively, three cases may arise: 16. Enthalpy of solution: The amount of heat evolved or
Case I: E J = E 2 . Neither heat is evolved nor absorbed. Such absorbed when one mole of the substance is dissolved in excess
reactions are very rare. of water so that further dilution does. not bring any heat change.
Case II: E J > E 2. The difference (E J - E2 ) of energy will be 17. Enthalpy or heat of neutralisation: The amotmtof
evolved, i.e., heat is evolved. heat evolved when one gram equivalent of an acid is neUli'ah",.::d
Case III: E J < E 2 . The difference (E2 - E J )ofenergywill be by one gram equivalent of a base in dilute solution. Heat of
neutralisation of a strong acid against· a strong· base is always
absorbed, i.e., heat is absorbed.
constant, i.e., 13.7 kcal or 57.1 kJ. It is due to common reaction
11. Exothermic reactions: Reactions which o~cur with
between H + and OH - ions. The heat offonnation of water from
evolution of heat
these ions is 13.7 kcal.
H productS < H reactants' i. e. , !:!.H = - ve
H+ +OW ~H20; !:!.H = - 13.7 kcal
Endothermic reactions: Reactions which occur with
absorption of heat In the case of weak acids and weak bases,the heat of
neutralisation is always less than 13.7 kcal because some ofthe
H products > H reactants' i.e., !:!.H = + ve evolved heat energy is utili~ed in bringing about complete
Sign conventions q !:!.E !:!.H ionisation of acid or base in solution.
18. Enthalpy of fusion: Heat absorbed in converting one
Exothennic (-) (-) (-) mole of solid into liquid at its melting point.
Endothennic (+) (+) (+) 19. Enthalpy of vaporisation: Heat absorbed in
12. Heat of reaction or enthalpy of reaction: The converting one mole of a liquid into its vapours at its boiling
amount of heat evolved or absorbed when quantities of the point.
substances indicated by chemical equation have completely Enthalpy of sublimation: Heat absorbed in converting one
reacted, it is represented as !:!.H. mole of solid directly into its vapour.
Enthalpy of reaction = L H products - L H reactants

20. Laws of thermochemistry:


The factors which affect the magnitude of heat ofreaction are : (i) Lavoisier and Laplace law: The heat which is required
(i) Physical state of reactants and products, (ii) Allotropic to break a compound into its elements is equal to the heat evolved
fonns of the elements (iii) Reaction carried at constant pressure during its fonnation from its elements.
or constant volume (!:!.H = !:!.E + !:!.nRTHiv) Enthalpy of solution
(v) Temperature at which the reaction is carried out. !:!.Hdecomposition = - !:!.Hf
480 G.R.B. PHYSICAL CHEMISTRY FOR COMPETITIONS

(ii) Hess's law (the lawof constant heat summation): If a value of entropy cannot be determined. The change in entropy is
chemical reaction can be made to take place in a number of ways denoted by flS.
in one or in several steps, the total change is always the same, i.e.,
heat change depends upon the initial and final stages only and IlS == q reversible
independent of intermediate steps, The thermochemical T
equations can be added, subtracted or multiplied by a number to where, q is the heat supplied at temperature T.
obtain a desired equation. flS + indicates increase in randonmess whereas, t:.S ==
21. Bond energy: The energy required to break one mole indicates the decrease in randonmess',
of bond of a particular type is termed bond dissociation energy. It
depends upon the nature of bond and also the molecule in which flS S final - S initial'
the bond is present. The bond energies can be used for 24. Free energy: Free energy change, t:.G, was introduced
determining heats of reactions. '
by Gibbs. The Gibbs-Helmholtz equation is:
[Heat of reaction"" Sum of bond energies of reactants
Sum of bond energies of products] flG == IlH - T IlS or G==H-TS
The bond energies can also be used for determining resonance
energy. and flG"" G products - G reactants
Resonance energy Observed heat of formation where, G is known as free energy. This is the maximum energy
- Calculated heat of formation available to a system during a process which can be converted
22. Experimental determination of heat of into useful work.
reaction: The apparatus used is called calorimeter. Two of the Negative value of !J.G· indicates that the proces~ is
common types of calorimeters are: (i) Water calorimeter (ii) spontaneous. Zero value means that there is a state of
Bomb-calorimeter. The principle of, measurement is that heat equilibrium.
given out is equal to heat taken, i.e., 25. Second law of thermodynamics: It is impossible to
construct a machine that is able to convey heat by a cyclic process
Q""(W+m)xsx(T2 -Td
from a colder to a hotter body unless work is done on the machine
where, W == Water equivalent of the calorimeter, m mass of the by some outside agency.
liquid, s == its sp. heat, T2 =fmal temperature and Tl == initial Or
temperature. Work can always be converted into heat but the conversion of
23. Spontaneous process: A process which proceeds of heat into wor)c does not take place under all conditions.
its own accord without any outside help is termed spontaneous 26. Carnot· cycle: It demonstrates the maximum
process. All natural processes' proceed spontaneously and are convertibility of heat into work in a theoretical and an imaginary
thermodynamically irreversible. cycle.
Driving force: It is the force which makes the process to W
occur by itself. It is the resultant of two basic tendencies: Thermodynamic efficiency ==--=-_.:0..
(i) Tendency to attain a state of minimum energy.
(ii) Tendency to attain a state of maximum randonmess. The efficiency of a heat engine is always le:;>s than 1.
The disorder 9r randonmess in a system is measured in terms 27. Third law of thermodynamics: The entropy of all
of a thermodynamic property known as entropy. The absolute pure crystals is zero at the absolute zero of temperature.
CHEMICAL THERMODYNAMICS AND THERMOCHEMISTRY 481

~l!tilINifj
l. Matrix-Matching Problems (For lIT Aspirants); [E] Match the reactions in Column-I with relations of
[A] Match the List-I with List-II: Column -II:
List-I List-II Column-I Column-II
(a) (P) Spontaneous (Reaction) (Relation)
flSsy&em > 0
(Isolated system) (a) H2(g) + CI 2(g) ~ 2HCl(g) (P) MI == ~U + RT
(b) ~G<O (q) Non-spontaneous
(b) N 2(g)+ 02(g)~ 2NO(g) (q) MI= ~U
photochemical reaction
(c) (r) Equilibrium (c) Hz(g)+ I2(g)~ 2HI(g) (r) MI == ~U 2RT
flSTotal == 0
(d) (~G)TP > 0 (s) Non-spontaneous (d) N2 (g ) + 3Hz(g ) ~ 2NH3 (g) (s) Forward shift by
increasing pressure
[B] Match the Column-I with Column-II:
Column-I Column-II [F] Match the processes of Column-I with entropy or,~J.lthalpy
= 2.303nRT log (~)
changes in Column-II:' .
(a) Isothermal process (P) W
(reversible) Column-I Column-II
(b) Adiabatic process (q) PV Y =constant (a) Nz(g) + 02(g) ~ 2NO(g) (P) ~S 0
(b) 2KI(aq.) + HgIz(aq.)~
(c) W = nR (Tz 1\) (r) W == 2.303nRT log (V~ \J K 2 [HgI4](aq. )
(q) ~ < 0
r-I
(c) PCI 3 (g)+ CI2(g)~ PCldg) (r) MI > 0
(d) Irreversible i~otherrnal (s) W == - Pext (V2 - Vi )
process (d) NH3(g)+HCI(g)~NH4Cl(s) (s) MI<O
[C] Match the physical properties ill,tList-I with their relations [0] Match the List-f\lvith List-II: .
in List-II:
List-I List-II
List-I List-II (Reaction) .' (Process)
(Quantity) (Relation)
I .
(a) ~G (P) -nFE or - nFE O (a) C(s)+ '2 ~(g)~ CO(g) (P) Combustion

(b) ~Go (q) MI-TflS . I .


(b) CO(g) + - 02(g) ~ CO2(g) (q) Neutralization
2
(c) Wor Wmax (r) -RT loge K
(c) NaOH(aq.) + HCl(aq.) ~ (r) Process of
(d) ~SO (s) 2.303nR log 10 (~ )
NaCl(aq.)+ H 2 0 formation

[D] Match the Column-I with Column-II: (d) Hz(g) + I 02(g) ~ H20(I) (s) Reaction of apollo
Column-I Column-II 2 fuel cell
(Gas) (Thermodynamic property)
[H] Match the relations of Column-I with their names in
(a) °2 (p) Y = 1.4 Column-II:
Column-I Column-II
7
(b) N2 (q) Cp =-R
2 (a) Mlz MIl =~Cp (P) Trouton equation
(c) CO2 ,CH4 (r) 23 R -1\
6 (b) =~Cy (q) Effect of temperature
(d) I mol 0z + 2 mol 0 3 (s) Y = 1.33 T2 -1\ on the heat of
reaction
Here: C p = Heat capacity at constant pressure
y CplCy (c) MI fusion _ ~ (r) Kirchhoff's equation
T - fusion
mp

(d) lim S~O (s) Third law of


T~OK
thermodynamics
482 G.R.B. PHYSICAL CHEMISTRY FOR COMPETITIONS

[I] Match the Column-I with Column-II: [J] Match the Column-I with Column-II :
Column-I Column-IJ Column-I Column-II
(a) Amount of heat required (P) Specific heat x molar
(a) Mf = + ve,I1S + ve (P) Spontaneous at all
to raise the temperature mass
temperature
of I mol substance by
laC (b) Mf = - ve, I1S = + ve (q) Non-spontaneous at all
temperature
Mf I1U
(b) - o r - (q) Heat capacity C,Cp or (c) Mf + ve, I1S =- ve (r) Non-spontaneous at high
I1T I1T Cv temperature
(c) Heat evolved in the (r) Electron gain enthalpy (d) Mf = ve, I1S - ve (s) Spontaneous at high
combustion of I g of a temperature
substance
(d) Heat evolved when an (s) Calorific value
extra electron is added to
valence shell of an
isolated gaseous atom
. . --:' .
[,~l,tr-?;Cf..·'\',c';i>h;fd-~:}~;t{ii~i~____________ ------------------------i
1. [A] (a-p); (b-p, q); (c-r); (d-s) [G] (a-p, r); (b-p); (c-q); (d-p, r, s)
[B] (a-p, r);(b-q); (c-q); (d-s) [H] (a-q, r); (b-q, r); (c-p); (d-s)
[C] (a-p, q); (b-p, r); (c-p); Cd-s) [1] (a-p, q); (b-p, q); (c-s); (d-r)
[D] (a-p, q); (b-p, q); (c-s); (d-r) . [J] (a-s); (b-p); (c-q); (d-r)
[E] (a-q); (b-q); (c-p); (d-r, s)
[F] (a-p, r); (b-q, s); (c-q, r); (d-q, s)

•. RACTICE PROBLEMS.
1. Write 'yes' if heat, work or matter are able to cross the from the state A ~ B (ii) Work done in taking the gas from
boundary of the corresponding system and 'no' if passage is B ~ C (iii) Work done in a complete cycle. (1 litre == 10-3 mC)
forbidden: [Ans. (i) -60 XI02 J (ii) zero (iii) 36 x 102 J, i.e" network is
done by the gas]
Close Isolated Adiabatic
Property
system system system 4. A sample of a gas contracts 200 cm 3 by an average of 0.5
atmosphere while 8.5 J heat flows out into the surroundings.
Heat YesfNo YesfNo YesfNo YesfNo What is the change in energy of the system?
Work YesfNo YesfNo YesfNo YesfNo (1 litre-atm 101.3 1)
[Ans. Energy of the system increases by 1.63 J]
Matter YesfNo YesfNo YesfNo YesfNo
5. Calculate the pressure-volume work done by the system when
2. Which of the following are state functions? the gas expands from 1.0 litre to 2.0 litre against a constant
(i) Q (ii) W (iii) Q + W (iv) Q - W (v) Qv(vi) Qp external pressure of 10 atmospheres. Express the answer in
calorie and joule.
(vii) fL (viii) fL (ix) Mf (x) E + PV (xi) E
PV [Ans. -10 litre-atm, -242.2 cal, -1013.28 J]
W T T .
[Ans.. (iv), (v), (vi), (viii), (ix), (x).] 6. A sample of a gas in a cylinder contracts by 7.5 litre at a
3. In the adjoining diagram, the P -V graph of an ideal gas is constant pressure of 5.0 atmosphere. How much work is done
shown, Find out from the graph (i) Work done in taking the gas on the gas by the surroundings?
[Ans. 37.5litre-atm or 3801.75 J]
i
c:J 15
f=
"'§ 12

ic:
<fl
0
9
6
DB
.... ----
iD C:
7. A sample of a gas expands from 200 cm3 to 500 cm3 against an
average pressure of 750 torr while 1.5 J h~at flows into the
system. What is the change in energy of the system?
(1 litre-atm == 10 1.3 J)
[Ans. Energy of the system is decreased by 28.5 J;
tlE 28.5 J 1
..- 3 I I

it I
I ,
I 8. Calculate the work done when 65.38 g of zinc dissolves in
0 hydrochloric acid in an open beaker at 300 K. (At. mass of .
2 3 4 5 6 7
Zn 65.38)
V (litre) -
CHEMICAL THERMODYNAMICS AND THERMOCHEMISTRY 483
[Ans. - 2494.2 J, i.e., work is done by the system] 19. Calculate M and Mwhen lOdm 3 of helium atNTPis heated
9. 6 moles of an ideal gas expand isothermally and reversibly in a cylinder to 100° C, assuming that the gas behaves ideally.
from a volume of 1 dm 3 to a volume oflOdm3 at 27°C. What (C v = 3/ 2R)
is the maximum work done? Express your answer in joule. [Ans. till = 556.74 J
[Ans. - 34464.8 J, work is done by the system] /lli = 927.9 J]
10. 1 mqle of an ideal gas at 25° C is allow\!d to expand reversibly [Hint: till = n· Cv . I'l.T and /lli = n· C p . I'l.T
at constant temperature from a volume of 10 litre to 20 litre.
Calculate the work done by the gas in joule and calorie.
=~X~X8.314 x 100 =~ x~ x 8.314 x 100
22.4 2 22.4 2
[Ans. - 1717.46 J or -4 n cal, i.e., work is done by the gas] = 556.74 J = 927.9 J]
11. Find the work done when 1. mole of the gas is expanded 20. For the conversion of 1 mole of S02 (g) into S03 (g) the
reversibly and isothermally from 5 atm to 1 atm at 25° C. enthalpy of reaction at constant volume, M, at 298 K is
[Ans. - 3984 J] -97.027 kJ. Calculate the enthalpy of reaction, /j.Jf, at constant
[Hint: Use w = - 2.303 nRT log PI ] pressure.
P2 [Ans. /lli = - 98.267 kJ]
12. How much energy is absorbed by 10 moles of an ideal gas if it [Hint: . Use the reaction,
expands from an initial pressure of 8 atmosphere to 4 1 3 1
S02(g) + - o/g) ~ S03(g), I'l.n = 1- - = - -]
atmosphere at a constant temperature of 27° C? 2 2 2
(R = 8.3lJ mol- I K-!) 21. The heat liberated on complete combustion of7.8 g benzene is
[Ans. 1.728 x 104 J] 327 kJ. This heat has been measured at constant volume and at
[Hint: In isothermal process, till = 0, 27 ° C. Calculate the heat of combustion ot benzene at
so q (heat absorbed) = - w. Thus, apply the equation constant pressure. (R = 8.3 JK-1 mol-I) (lIT 1996)
q = 2.303nRT log...l. ]
P, [Ans. -3273.7 kJ mor l ]
P2 22. ,1 mole of naphthalene (ClOHg) was burnt in oxygen gas at
13. A given mass of a gas at 0° C is compressed reversibly and 25 ° C at· constant volume. The heat evolved was found to be
adiabatically to a pressure 20 times the initial value. Calculate 5138.8 kJ. Calculate the heat of reaction at constant pressure.
(R = 8.3 JK- l mol-I)
the final temperature of the gas. [~~ = 142]
[Ans. 5l43.8kJ]
[Ans. T2 = 662.2 K] [Hint: Water is present in liquid state at 25° C and naphthalene
in solid state.]
[Hint: Apply the formula [~J y =[ ~J I - y] 23. The enthalpy of formation of methane at constant pressure and
300 K is -75.83 kJ. What will be the heat of formation at
14. 3 moles of hydrogen are compressed isothermally and constant volume? (R = 8.3 J K -I mol -I)
reversibly from 60 dm 3 to 20 dm 3 and 8.22 kJ of work is done [Ans. -73.34 kJJ
on it. Assuming ideal behaviour, calculate. the temperature of 24. The heat change for the reaction,
the gas.
N 2(g) + 3H2(g) -----7 2NH3I(g)
[Ans. 300 K]
is - 92.2 kJ. Calculate the heat of formation of ammonia.
15. To what pressure must a certain ideal gas(y = 1.4) at 373 K
[Ans. - 46.1 kJ mol-I]
and 1 atmospheric pressure be compressed adiabatically in
25. /j.Jf0 for the reaction,
order to raise its temperature to 773 K?
[Ans. 7.89 atm] 4S( s) + 60 2(g ) -----7 4S0 3(g )
16. 1 mole of an ideal gas (C v = 12.55 J K-! moe!) at 300 K is is -1583.2 kJ. Calculate /j.Jf; of sulphur trioxide.
compressed adiabatically and reversibly to one-fourth of its [Ans. - 395.8 kJ]
original volume. What is the final temperature of the gas? 26. Calculate the heat change in the reaction,
[Ans. 752 K] 4NH3 (g ) + 302 (g) -----7 2N2 (g) + 6H20( I)
17. Calculate q, w, M and M for the reversible isothermal
at 298 K given that heats of formation at 298 K for NH3 (g )
expansion of 1 mole of ideal gas at 27° C from a volume of 10 and H 2 0(I) are - 46.0 and- 28{).0 kJ mol- I respectively.
dm 3 to a volume of 20 dm 3 .
[Ans. /lli 0 =: -1532 kJ] ''',
[Ans. q= -W= 1729 J] 27. Calculate the heat of combustion of 1 mole of C2H 4 (g) to
18. Calculate the internal energy change for the process in which
form CO2(g) and HzO(g) at 398 K and 1 atmosphere, given
1.0 kcal of heat is added to 1.2 litre of O2 gas in a cylinder at
that the heats of formation of CO 2(g ), H 20(g ) and CZH 4 (g)
constant pressure of 1.0 atm and the volume changes to 1.5 litre.
are - 94.1, - 57.8 and + 12.5 kcal mol-! respectively.
[Ans. 0.993 kcal]
[Ans. - 316.3 kcal]
484 G.R.B. PHYSICAL CHEMISTRY FOR COMPETITIONS

28. The heats of combustion of CH4 (g ) and ~H6 (g ) are -890.3 36. The standard enthalpy of combustion at 25° C of hydrogen,
and - 1560 kJ mol- 1 respectively. Which has higher calorific cyclohexene (C6HlO ) and cyclohexane (C6H12 ) are - 241,
value? - 3800 and - 3920 kJ mol-' respectively. Calculate the heat of
[Ans. Methane has higher calorific value.] hydrogenation of cyclohexene. [ISM (Dhanbad) 19921
29. The heat of combustion of butane is 2880 kJ mol- t . What is [Ans. -121 kJ mol-I]
the heat liberated by burning 1 kg of butane in excess of [Hint:
oxygen supply? 1
H2(g) + '2 °2(g) ---7 HzO(g); ... (i)
[Ans. 49655 kJ]
30. The heat of formation of CH4 (g ), ~H6 (g ) and C4 HlO (g ) are (M! =-241kJmor-')
- 74.8, - 84.7 and - 126.1 kJ mol-I respectively. Arrange them C6H 10 (g) + -17 02(g) ---7 6C02 (g ) + 5HzO(g); ...(11
.. )

in order of their efficiency as fuel per gram. Heats of 2 (M! 3800 kJ mort)
formation of CO2(g ) and H2 0(l) are - 393.5 and - 285.8 kJ C6H I2 (g) + 90 2(g) ---7 6C02(g) + 6H 20(g); ... (iii)
1
morl respectively. (Mf = 3920 kJ mor )
[Ans. CH 4 > C2~ > C4H IO ] Adding eqs. (i) and (ii) and subtracting eq. (iii),
[Hint: First determine heat of combustion in each case and then ~HIO(S) + H2 (g) ---7 C6Hds);
find the calorific value.] M! == - 241- 3800 - (-3920) = -121 kJ mor' ]
31. The heat of combustion of carbon to CO2(g ) is 37. From the following data of heats of combustion, find the heat
- 393.5 kJ mol-'. Calculate the heat released upon formation offormation ofCH3 0H(l):
3
of 35.2 g of CO 2(g) from carbon and oxygen gas. CH30H(I) + 02(g) ~ CO2 (g)+ 2H20(I);
[Ans. - 315 kJ] 2
AH -726kJ
32. Calculate LVI;ofC6 H12 0 6 (S) from the following data: . C(s) + O2 (g ) ~ CO2 (g); AH = - 394 kJ
I
LVIcomb of C6 H I2 0 6(s) == - 2816 kJ mOrl , LVI; of CO2(g) . H2 (g)+ - 02(g)~ H2 0(l); AH = 286kJ
2
'= - 393.5kJ mor' and LVI; (Hpj - 285.9 kJ mol-I. {BIT (Ranchi) 19911
[Ans. 1260 kJ mOrl] [Ans.. :"'240kJmor']
33. Calculate the amount of heat released when: 38. Calculatp the heat;offQrmation of methane, given that
(i) 100 mL of 0.2 MHCl solution is mixed with 50 mL of 0.2 . heat of formation of water - 286 kJ mol-I
MKOH. heat of c6'inbustion of methane == 890 kJ morl
(ii) 200 mL of 0.1 MH zS04 is mixed with 200 mL of 0.2 M
KOH solution.
.
\
hea~ of
'.-
combustion of carbon - 393.5 k1 morl
([SM (DhanbacO ! 9931
[Ans. (i) 0.57 kJ (ii) 2.18 kJ]
[Ans. 75.5 kJmorl]
34. When 100 mL each of HCI and NaOH solutions are mixed,
5.71 kJ of heat was evolved. What is the molarity of two 39. Calculate the standard heat of formation of ~OH8
solutions? The heat of neutralisation ofHCI is 51.1 kJ. (naphthalene) if standard heat of combustion of naphthalene is
[Ans. 1M] 1231.0 kcal at 298 K and standard heat of formation of
CO2 (g) and H20( l) are - 94.0 kcal and - 68.4 kcal
35. Determine the enthalpy of the reaction,
respectively.
C3 Hg(g) + H2(g)~ ~H6(g)+ CH4 (g)
[Ans. 17.4kcal]
at 25° C, using the given heat of combustion values under
40. The heat of combustion of liquid ethanol is 327.0 kcal.
standard conditions.
Calculate the heat offormation of ethanol, given that the heats
. Compound H 2 (g) CH4 (g) ~H6(g) C(graphite)
offormation of CO2 (g ) and H2 0(l) are -94.0 kcal and 68.4
!1l-F' (kJ mol-I) --285.8 -890.0 -1560.0 -393.5 kcal respectively.
The standard heat offormationofC3 Hg (g )is-103.8 kJ mol-I. . [Ans. 66.2 kcal]
nn 1992) 41~ .C;aLculate heat of formation of cane sugar from following data:
'.' ,",'
[Ans. 55.7 kJ] ,,~: I
I H2 (g) + - O2 (g) ~ H2 0(g); LVI = 68.4 kcal
[Hint: H2(g) + - 02(g) H 20; (LVI - 285.8 kJ) ...(i) 2
2
... (ii) C(g) + O2 (g ) ~ CO 2(g); LVI:= - 94.4 kcal
(Mf 890.0 kJ)
~2H22011 (s) + 1202 (g) ~ I2C02(g) + IIHz0(l);
7
C2~(g) + - 0z(g) ---7 2C0 2(g) + 3H zO; ... (iii)
LVI - 1350.0 kcal
2 .
(Mf == 156u.0 kJ) [Ans. 535.2 kcal] .
C + O2 (g) ---7 CO 2 (g); (LVI - 393.5 kJ) ... (iv) 42. The heats of formation of C6~(l), H2 0(l) and CO2 (g) are
3C+ 4H2(g)---7C 3H g (g); (M! =-103.8kJ) ... (v) 11.70, - 68.4 and 94.0 kcal respectively. Calculate the heat
MUltiplying eq. (i) by' 5 and eq. (iv) by 3 and adding both the of combustion of benzene (I) .
.equations and subtracting eqs. (ii), (iii) and (v).]
.. [Ans. -780.9 kcal]
CHEMICAL THERMODYNAMICS AND THERMOCHEMISTRY 485

[Hint: c;H6(I) + 1; Gig) ~ 6C0 2 (g) + 3HzO(I); tili = ? 51. Use the bond energy data and calculate the enthalpy change
for:
!:JH = 6 x tilif (C0 2) + 3 x tilif (H2 0 ) - tilif (C6H 6)] H
43. Calculate the heat of hydrogenation ofc;H2 to C2H4 . I
2C(g) + 2H(g) + 2Cl(g)~ H- C-CI
H 2 (g) + ..!. O2(g) ~ H 20(l); AH = - 68.32 kcal . I
2 Cl
The bond energies of C-H and C-Cl are 413 and 328 kJ
mol-I respectively.
AH = - 310.61 kcal [Ans. -1482kJ mor l ]
C;~ (g) + 302 (g ) ~ 2CO z(g) + 2H2 0(l); 52. Calculate the heat of formation of ammonia from the
following data:
AH = - 337.32 kcal
Nz(g)+ 3H2(g)~ 2NH3 (g)
[Ans. - 41.61 kcal]
44. Calculate the heat of hydrogenation,
==
The bond energies of N N, H - H and N - H bonds are
226, 104 and 93 kcal respectively.
c;H4 (g) + H2(g) ~ c;H6(g) [Ans. -10 kcal]
given that, the heat of combustion of ethylene, hydrogen and 53. Use the following bond dissociation energies to compute the
ethane are - 337.0, - 68.4 and - 373.0kcal respectively. C-H bond energy in methane:
[Ans. -324 kcal] (i) CH4(g)~ CH (g) + H(g); 3 AH = 475 kJ mor l
45. Ifthe heat offormation of A1 2 0 3 (s)and Cr203(s)~ 1596 kJ (ii) CH3 (g ) ~ CH z(g) + H(g ); AH = 470 kJ mol-I
and 1134 kJ (both exothermic) respectively. Calculate AH of
(iii) CH2(g) ~ CH(g) + H(g ); AH = 416 kJ mol-I
the thermite reaction.
(iv) CH(g) ~ C(g) + H(g); AH = 335 kJ mol-I
Cr203(s) + 2A1(s) ~ 2Cr(s) + A1 2 0 3(s)
[Ans. 424 kJ mol-I]
[Ans. - 462 kJ] o .
46. Calculate the enthalpy of transition of rhombic sulphur to 54. Calculate AHf for chloride ion from the following data:
monoclinic sulphur from the following data:
S(rhombic) + 02 (g ) ~ S02 (g); AH = - 297.5 kJ
'21 H2 (g) + '21 Cl 2 (g ) ~ HC1(g ); AH = - 92.4kJ morl
S(monocliriic) + O 2(g ) ~ SOZ (g ); AH = - 299.9 kJ HCI(g) + nH20 ~ H +(aq.) + cr (aq.);
[Ans. + 2.4 kJ] AH = - 74.8 kJ mol-I
47. Calculate AH forthe reaction, AH C _ -I
f (H + aq.) - 0.0 kJ mol (III 19'1L)
H 2 (g)+ Yz02(g)~ H 20(g)
given that bond energies of H-H and = ° °
bond and O-H
bond are 433 kJ mol-I, 492 kJ mol- I and 464 kJ mol-I.
[Hint: Required equation is
YzCI 2(g) + nHzO ~ CI+ (aq.); tili =?
[Ans. - 249 kJ] Adding both equations.
48. Using the bond enthalpy data, calculate AH of the following Yz H 2(g) + Yz Clz(g) + nHzO ~ H+ (aq.) + C1-(aq.);
reaction: tili =-167.2
2Cl 2 (g) + 2HzO(g ) ~ 4HCI(g) + 0z (g) Yz H 2(g) ~ H+ (aq.); tili =0
given that, bond energies ofCl-Cl; H-Cl, O-H and 0=0
are 242.8, 431.8, 464 and 442 kJ morl respectively.
So, Yz CI 2 (g) + nHzO ~ Cqaq.); tili =-167.2 kJ mol-I]
[Ans. 172.4 kJ mol-I] 55. The heat of ionisation of formic acid is 1.5 kJ / mol. 9.2 g
formic acid on reaction with 7 g ainmonium hydroxide gives
49. Calculate the enthalpy of the reaction,
10.8 kJ of heat. Calculate the heat of ionisation of ammonium
Sn02(S)+ 2H z(g) ~ Sn(s) + 2H20(l) hydroxide. (1 cal = 4.2 J)
[Ans. 2.04 kJ / mol]
given that, enthalpy of formation of Sn02(S) and H 20(l) are
580.7kJ and - 285.8 kJ respectively. 56. Assuming that 50% of the heat of useful, how many kg of
water at 15 0 C can be heated to 95.00 C by burning 200 litre of
[Ans. +9.1 kJ]
methane measured at NTP? The heat of combustion of
50. Calculate the enthalpy change for the reaction,
methane is 211 kcal mol-I.
H 2 (g)+ Clz(g)~ 2HCl(g) [Ans. 11.76 kg]
57. The standard potential for the reaction,
given that, bond energies of Cl-Cl and H-Cl are
436, 243 and 432 kJ mol-I. Ag +(aq.)+ Fe2+(aq.) ~ Fe 3+ (aq.) + Ag(s)
[Ans. -185 kJ mol-I] is 0.028 V What is the standard free energy change for this
reaction?
[Ans. 2.702 kJ mol-I]
486 G.R.B. PHYSICAL CHEMISTRY FOR COMPETITIONS

58. Calculate the theoretical maximum efficiency of a heat engine 67. An engine operating between 150°C and 25°C takes 500 J
operating between 373 K and 173 K. heat from a high temperature reservoir. Assuming there is no
. [Ans. 0.$36 or 53.6%] frictional loss, calculate the work that can be done by the
59. The standard free energy of formation in the gaseous state of engine.
methanol, dimethyl ether and water are - 38.7, - 27.3 and [Ans. 148 J]
-54.6 kcal respectively. Is the transformation of methanol to 68. At 300 K, the standard enthalpies of formation of
dimethyl ether and water in gaseous state possible? C6 H sCOOH(s), COz(g ) and Hz0(l) are -408, -393 and -286
2CH~OH CH3 0CH3 + H 2 0 kJ mol- 1 respectively. Calculate the heat of combustion of
[Hint! t:..G'" for the transformation benzoic acid at (i) constant pressure and (ii) constant volume.
1 I
= L t:..G(;roductS) L t:..G(:eactants) (R =8.3IJmol- K- ) (lIT 1995)
= ve (the transformation is possible)] [Ans. (i)!:..H =- 3201 kJ mol-I;
(ii) t:..E = - 3199.75 kJ mol-I]
60. Ethanol boils at 78.4° C and standard enthalpy of vaporization
of ethanol is 42.4 kJ morl. Calculate the entropy of 69. For the reaction,
vaporization of ethanol. Brz (I) + CI 2 (g) ----+ 2BrCI(g)
[Ans. 120.9 J K- l mol-I] Mf 29.37 kJ mor 1 and t:..S = 104 J K-1 mol-I. Find the
6t. The following data is known about the melting ofKCl: temperature above which the reaction would become
Mf 7.25 kJ mOrl and t:..S = + 0.007 J K- 1 mor 1 spontaneous.
[Ans. Above 282.4 K]
Calculate its melting point.
70. For the synthesis of ammonia,
[Ans. 1035.7 K]
N z(g ) + 3Hz(g) ----+ 2NH3 (g)
[Hint: At melting point t:..G =0] Calculate. MfO ,t:..S ° and t:.Go at 300 K using the following
62. For the reaction,
data:
Ag20~ 2Ag(s) + ..!. 02(g) Species H2 NH3
2
Calculate the temperature at which free energy change is zero. Mf; I (kJ mol-I) o 46.2
At a temperature lower than this, predict whether the forward SO/(JK- 1 mol-I) 130.6 192.5
or the reverse reaction will be favoured. Give reason.
(t:.H = + 30.56 kJ and t:..S = + 0.066 kJ K- 1 mol-I at one
[Ans. !:..H0 = - 92.4 kJ mol-I, t:..so = :"198.3 J K- I mol- I
t:..Go = 32.91 kJ mol-I]
atmosphere)
71. How much heat is required to change 15.6 g of benzene from
[Hint: Calculate T by applying tfie formula liquid into vapour at its boiling point of 80°C? Entropy of
t:..G = !:..H T t:..S and t:..G = O. vaporization of benzene is 87 J K- I morl.
Then find the value of t:..G at lowertemperature than T. The value of
t:..G com"s positive; hence, the reverse reaction will be favoured. At [Ans. 6142.2 J]
temperature higher than T, the reaction is spontaneous.] 72. Calculate the standard free energy change for the combustion
63. Calculate the boiling point of the liquid if its entropy of of glucose at 298 K, using the given data.
vaporization is 110 J K- 1 mol- 1 and the enthalpy of C6 H 12 0 6 + 60 2 ----+ 6C02 + 6H 20

vaporization is 40.85 kJ mor 1• Mfo = 2820 kJ morl , t:.so = 210 J K -J mOrl .

[Ans. 371.36 K] [Ans. t:..Go - 2882.58 kJ mor l ]


64. Using So values, calculate the entropy of the reaction, 73. The specific heat at constant volume for a gas is 0.075 cal/g
1 and at constant pressure it is 0.125 caVg. Calculate:
S02(g) + 2 °2(g) ----+ S03(g) (i) the molecular weight of gas,
(ii) atomicity of gas.
The So values for S02' 0z and S03 are 248.5, 205.0 and 256.2
JK- 1 mOrl. [Hint: (i) C p - Cv = ~ where, M = molecular weight of gas
M
[Ans. = -94.2 J K- I moll]
[Hint: t:..s:eaction = L S;roduct, - L S:eact.ntsl 0.125 - 0.G75 = 1.987
M
65. Calculate the entropy change for the following reaction:
M =39.74 ",40
CaC03 (s ) ----+ CaO(s ) + COz (g )
So 92.9 39.7 213.6 J K- 1 mol-I C
(ii) -.f. = Y
Cv
[Ans. +l .•,i 4 J ;..•. I "101- 1]
66. Compute tne va :: .of t:..S at 298 K for the reaction, 0.125
1.66
H2 (g ) + ;0 O 2 (g) ----+ HzO(g ) 0.G75
given that, t:.G = 228.6 kJ and Mf 241.8 kJ. .. 1.66 value ofy shows that the gas is monoatomic.]
[Ans. 44.3 J K- l]
CHEMICAL THERMODYNAMICS AND THERMOCHEMISTRY 487

74. The polymerisation of ethylene to linear polyethylene is 78. For the reaction,
represented by the reaction, 4C (graphite) + 5H2 (g) nC4HIO (g );
nCH 2 CH2 CH2 - CHdii !::..H0 124.73 kJ mol- 1
where, n has a large integral valu~. Given that the average lJ.So - 365.8 J K- 1 mol-I
enthalpies of bond dissociation for C = C and C-C at 298 K
are +590 and + 331 kJ mol-I respectively, calculate the 4C (graphite) + 5H2 (g ) iso-C4 H IO (g)
enthalpy of polymerisation per mole of ethylene at 298 K.
!::..H 0
131.6 kJ mol~l
(lIT 1994)
[Hint: nCH 2 CH 2 CH 2 CHdii t:..s o 381.079 J K- 1 mol-I
There are equal number of C-H bonds on both sides but on
reactant side there are nC C bonds and on product side Indicate whether normal butane can be spontaneously
(2n + l)C-'-Cbonds. converted to iso-butane or not.
Enthalpy of polymerisation [ATls, Yes, t:..Go == - 2.32 kJ]
(2n + 1)6H(c_C) [Hint: For nC4H IO ,
590n (2n + 1)(331) t:..Go = 6H o - T6S o
590n 662n [2n + .I-t 2n as n is very large] 124.73 298(-365.8 x 10-3 ) = 15.72 kJ
72n kJ For iso-butane (iso-C 4H IO ),
Enthalpy of polymerisation per mole 6GO 131.6 298(-:- 381.079 x 10-3 )
6H 72n _ 72 kJ mor l ] 18.04 kJ
n n
For conversion of nC4H 10 iso-C4H 10 ,
73. An athlete is given 100 g of glucose (C6H I2 0 6 ) of energy t:..Go = 18.04 (-15.72) = 2.32 kJ
equivalent to 1560 kJ. He utilises 50% of this gained energ-f in Negative value shows that the process is spontaneous.]
the event. In order to avoid storage of energy in the body,
'79. The temperature of a bomb calorimeter was found to rise by
calculate the mass of water he would need to perspire. The 1.617 K, when a current of 3.20 amp was passed for 27 sec
enthalpy of evaporation of water is 44 kJ mol-I. from a 12 V source. Calculate the calorimeter constant.
[Hint: Unused energy 1560 =780 kJ [Hint: Energy absorbed by the calorimeter
2 \ = I x t x V 3.2 x 27 x 12 = 1036.8 J
Mass of water needed for perspiration = 18 x 780 = 318.96 g] Calorimeter constant (ms) can be calculated as,
44 q ms!::"t
76. Calculate the entropy of ideal mixing when 2 moles of N z, 3 1036.8 = ms X 1.617
moles of H2 and 2 moles of NH3 are mixed at constant ms 641.187 kJ]
temperature, assuming no chemical reaction is occurring.
80. 1 mole of an ideal gas is allowed to expand isothermally at
[Ans. 62.80 J K- 1] 27 ° C until its volume is tripled. Calculate t:..S sys and t:..S univ
[Hint: Use the relation, under the following conditions:
6S R :E nj loge Xi (a) the expansion is carried out reversibly.
when ni =no. of moles of component (b) the expansion is a free expansion.
x; = mole fraction ofthe component] [A'TIs. (a) IJ.Ssys 9.135 J K- i rnol- 1
n. Calculate free energy change for the reaction: 6S surr = 9.135 J K- 1 rnor 1
H2(g)+ CI 2(g)-t 2H-Cl(g)
t:..Suniv 0
by using the bond energy and entropy data.
Bond energies Cl-CI and H-CI bonds are 435 kJ (b)6Suniv =6Ssys=9.135JK- 1 mor l ]
mol-I, 240 kJ mol-I and 430 kJ mol- 1 respectively. Standard [Hint: (a) In isothermal reversible process:
entropies of Hz, Cl2 and HCl are 130.59,222.95 and 186.68
J K- 1 mol-I respectively. . 'T
[ATls. 190.9 kJ] q W = 2.303RT log V2
[Hint: 6Go can be calculated by using: VI
6Go 6Ho TIJ.S° 2.303 x 8.314 x 300 log 3 = 2740.6 J mol-I
6H o :E (BE\.eactants - :E (BE)products
= 2740.6 = 9.135 J K- 1 rnol- 1
=435 + 240 - 2 x 430 -185 kJ T 300
t:..so == :E S;roducls :E S ;eactants t:..S surr = - t:..Ssystem = - 9.135 J K- 1 mol-I
2 x 186.68 130.59 - 222.95
!::,.Suniv = 0
19.82 J K- 1 19.82 X 10-3 kJ K- 1 (b) In case offree expansion:
t:..Go 6Ho Tt:..S o
185 298 x 19.82 x 10- 3 = -190.9 kJ]
!::"S r
2.303nR log V2 J == 9.135 J K- 1 mol-I
\V1
488 G.R.B. PHYSICAL CHEMISTRY FOR COMPETITIONS

~Suniv = ~Ssys = 9.135 J K- 1 mol-1


qBC == W := 2303 nRT log ( ;: J
Msurr =0]
81. One kilogram water at 0° C is brought into eontact with a heat 4
== 2.303 x 2 x 2 x 600 log -
reservoir at 100" C. Find: 2
(a) change in entropy when temperature reaches to 100" C. = 1.663 X 103 cal
(b) what is the change in entropy of reservoir? C ~ D:
(Isochoric process)
(c) change in entropy of universe. 3
(d) the nature of process. qCD =nCv ~T =2x - x 2 (-300)==-ISoocal
2
[Ans. (a) 312 cal K- 1, (b) -26S.1 cal K- 1, (c) 43.9 cal K- 1 D ~A: (Isothermal process)
(d) spontaneous]
VA
[Hint: (a) ~S = 2.303nC p log G~) qDA == 2.303nRTA log-
VD
1
2.303 x 2 x 2 x 300 log
:= 2.303 x 1000 x 18 log (373) 312 cal K- 1 4
18 273 = -1.663 X 103 cal
~Q
(b) M T := - Total heat change = 3000 + 1.663 x 103 - ISoo - 1.663 X 103
reservoir
= 1200 cal
~Q = ms I1t =1000 x I x 100 := 105 cal Work done:= -1200 cal]
-105 I 83. The reaction, SOCl 2 + H 20 ----? S02 + 2HCI is endothermic
M:= - == -268.1 cal K-
373 by 49.4 kJ and exoergonic by 50.8 kJ. What is the factor that
(c) ~Sunive"", = 312 - 26S.1 == 43.9 cal KI makes the reaction to be spontaneous? Calculate the entropy
change at 298 K.
(d) ~S > 0, the process will be spontaneous.]
[Ans. M°:= 0.3362 kJ K- I ]
82. A monoatomic ideal gas of two moles is taken through a cyclic [Hint: Exoergonic means ~Go =- ve.
process starting from A as shown in figure. The volwne ratios ~Go == 50.S kJ,
. V V
=
are .JL 2 and JL =4. If the temperature TA at A is 27"C. Mfo =49.4 kJ
VA VA Substitute these values in ~G" == Mfo - TM" to calculate entropy.
Calculate: M O := ~Ho ~G" == 49.4 - (-50.S)
T 298

VD ---------H = 0.3362 kJ K- 1 ]
84. What amoimt of ice will remain when 52 g ice is added to

!
100 g of water at 40" C?

VB
VA
--------1;:71"!
--------A:
Specific heat of water is I callg and latent heat of fusion of ice
is 80 caUg.
[Ans. 2 g ice]
I I [Hint: At the stage of thermal equilibrium at 0° C,
I I
I I
Heat lost by water Heat absorbed by ice to melt.
TA TB ms~T =mL
Temp_ looxlx40=mx80
m==50g
(a) the temperature of gas at B. Remaining ice = 52 - 50 =2 g]
(b) heat absorbed or evolved in each process. 85. Calculate the Mi~ of C6HI2 0 6(s) from the following data:
(c) total work done in cyclic process. MiCOnID [C6HI2 0 6 (s)]:= - 2816 kJ I mol
[Hint: A ~ B: (It is isobaric process)
Mi~ of CO2 (g) == - 393.5 kJ I mol
VA = VB Mi~of H 20(1) == - 285.9 HI mollBCECE 2006]
TA TB
V [Ans. -1260 kJ/mol]
TB == -I!.. X TA == 2 x 300:= 600 K
VA 86. A liquid freezes into solid (Mi := - 500 J mol-I ) at 100 K and
·5 I atm. Find the values of:
qAB = nCp ~T =2x- R ~T
2 (i) Gibbs free energy change
. 5 (ii) entropy change. rCBSE (Mains) 20061
== 2 x- x 2 x 300 = 3000 cal
2 [Hint: Liquid ~ Solid
r (Isothermal process) (i) ~G =0, at equilibrium
~U==O (11") ~
A" -
_ Mf _ -500 _ 5 J K- 1
------ mo1-1]
T 100 .

. CHEMICAL THERMODYNAMICS AND THERMOCHEMISTRY 489~~

=~ OBJECTIVE UESTIONS
13. A thermodynamic quantity is that:
Set-1: Questions with single correct answer
(a) which is used in thermochemistry
1. Thermodynamics is concerned with: (b) which obeys all laws of thermodynamics
(a) total energy ofa system (c) quantity whose value depends only upon the state of the
(b) energy changes in a system system
(c) rate ofa chemical change (d) quantity which is. used in measuring thermal change
(d) mass changes in nuclear reactions 14. Thermodynamic equilibrium involves: [CET (Ph.) 19911
2. An isolated system is that system in which: (a) chemical equilibrium (b) thermal equilibrium
[PET (MP) 19931 (c) mechanical equilibrium (d) all the three
(a) there is no exchange of energy with the surroundings 15. For the reaction of one mole zinc dust with one mole sulphuric
(b) there is exchange· of mass and energy with the acid in a bomb calorimeter, L\U and w correspond to:
surroundings (AIIl\IS 200S)
(c) there is no exchange of energy and mass With the (a) L\U < 0, w = 0 (b) L\U < 0, w < 0
surroundings (c) L\U > 0, W = 0 (d) L\U> 0, W > 0
(d)· there is exchange of mass with the surroundings [Hint: In bomb calorimeter, W = 0, L\U = q( -ve)]
3. Identify the intensive property from the following: 16. A system is provided with 50 joules of heat and the work done
(a) volume (b) mass on the system is 10 joules. What is the change in internal
(c) enthalpy (d) temperature energy of the system in joules? rEAMCET (Engg.) 2()10]
4. Which one of the following is an extensive property? (a) 60 (b) 40 (c) 50 (d) 10
(Corned (Karnataka) 2008] 17. During an isothermal expansion o(an ideal gas, its:
(a) Enthalpy (b) Concentration (CBSE 1991)
(c) Density (d) Viscosity (a) internal energy increases
5. For an adiabatic process, which of the following relations is (b) enthalpy decreases
correct? (CPMT 1990) (c) enthalpy remains unaffected
(a) I1E == 0 (b) P L\V == 0 (c) q == 0 (d) q == + w (d) enthalpy reduces to zero
6. Which one is true from the following for isobaric process? 18. The work done in ergs for a reversible expansion of one mole
. fCET (Gujar~t) 2008] of an ideal gas from a volume of 10 litre to 20 litre at 25" Cis:
(a) ~:: 0 (b) L\q = 0 ICMC (Vellore) 19911
(c) MI== 0 (d) I1E = 0 (a) 2.303 x 8.31 x 107 x 298 log 2
7.· Foran ideal gas, the value of (dE)
dV T
is:
[PET (MP) 19931
(b) 2.303 x 0.0821 x 298 log 2
(c) 2.303 x 0.0821 x 29~ log 0.5
(a) positive (b) zero (d) 2.303 x 2 x 298 log 2
(c) negative (d) interchangeable 19. When an ideal gas is compressed adiabatically and reversibly,
8. A process, in which pressure remains constant, is called: the final temperature is:
(a) isochoric process (b) isothermal process (a) higher than the initial temperature
(c) adiabatic process (d) isobaric process (b) lower than the initial temperature
9. A process in which volume remains constant, is called: (c) the same as initial temperature
(a) isoehoric process (b) isothermal process (d) dependent upon the rate of compression

r-1
(c) adiabatic process (d) isobaric process 20. Adiabatic reversible expansion of a gas is represented by:

(~r == (~ (~J (~r-Y


10. For a cyclic process, the condition is:
(a) I1E == 0 (b) Ml == 0 (a) (b)

11.
(c) I1E > Oand MI> 0

(a) Heat
(c) Internal energy
(d) both I1E == oand MI = 0
Which one of the following is a state property?
(b) Work
(d) Potential energy
(c) (~ r=( ~r I (d) all are correct

21. Total energy change for a reversible isothermal cycle is:


12. Internal energy of a system of molecules is determined by taking
into consideration its: (a) always positive (b) zero
(a) kinetic energy (c) always negative (d) always 100 kJ per degree
(b) vibrational energy 22. "Heat energy cannot be completely transformed into work
(c) rotational energy without producing some change somewhere", is the statement
(d) all kinds of energies present in the molecules of:
490 G.R.B. PHYSICAL CHEMISTRY FOR COMPETITIONS

(a) Hess's law 33. Under which of the following conditions is the relation,
(b) first law of thennodynamics M! = IJ.E + P I1V valid for a closed system?
(c) Kirchhoff's law (a) Constant pressure
(d) second law ofthennodynamics (b) Constant temperature
23. The heat content of the system is called: (c) Constant temperature and pressure
(a) internal energy (b) enthalpy (d) Constant temperature, pressure and composition
(c) free energy (d) entropy 34. Which of the following is an endothennic reaction?
24. If one mole of ammonia and one mole of hydrogen chloride (a) 2H2 (g)+ 02(g)~ 2H2 0(l)
are mixed in a closed container to fonn ammonium chloride (b) N 2 (g)+ 02(g)~ 2NO(g)
gas, then: (KCET 2008) (c) NaOH(aq.)+ HCl(aq.)~ NaCl(aq.)+ H 2 0(l)
(a) M! < I1U (d) C2H sOH(aq.)+ 302(g)~ 2C0 2 (g)+ 3H2 0(l)
(b) M! > I1U
(c) I1H =IJ.U 35. Which ofthe following reactions is endothennic?'
(d) there is no relationship between M! and tiU (a) CaC03(s)~ CaO(s)+ CO2 (g)
25. An exothennic reaction is one in which the reacting (b) Fe(s) + S(s)~ FeS(s)
substances: (c) NaOH(aq.) + HCl(aq.) ~ NaCI(aq.) + H 20(l)
(a) have same energy as products (d) CH4 (g) + 202 (g) ~ CO2 (g)+ 2H2 0(I)
(b) have less energy than the products 36. The fonnation of water from H2 (g) and O2 (g) is an
(c) have more energy than the products exqthennic process because:
(d) are at higher temperature than the products
(a) the chemical energy of H2 (g ) and O2 (g ) is more than that
26. In endothennic reactions, the reactants: of water
(a) have more energy than products
(b) the chemical energy of H2 (g) and O2(g ) is less than that
(b) have as much energy as the products
of water
(c) are at lower temperature than products
(e) the temperature ofH2 (g) and 02(g) is higher than that of
Cd) have less energy than the products
water
27. IdentifY the reaction in which the heat liberated corresponds to
the heat of fonnation (M!): (EAMCET 2006) (d) the temperature of H2 (g) and 02 (g) is lower than that of
water
(a) C (diamond) + 02(g) -7 cO 2(g) + heat
(b) 2H2 (g ) + 02 (g) -7 2H2 0(g) + heat
37. Which one of the following reactions is an exothennic
reaction? .
(c) C (diamond) + 2H 2 (g) -7 CH 4 (g) + heat
(a) CaC03(s) ~ CaO(s) + CO2(g)
(d) S (rhombic) + O2(g ) -7 S02 (g) + heat
28. In an exothennic reaction, M! is: (b) N 2 (g) + 02(g) ~ 2NO(g)
(a) positive (b) negative tc) 2HgO(s)~ 2Hg(s)+ 02(g)
(c) zero (d) may be positive or negative \d) C(s) + 02(g) ~ CO2 (g)
29. Evaporation of water is: 38. Which one of the following is not applicable for a
(a) a process in which neither heat is evolved nor absorbed thermochemical equation?
(b) a process accompanied by chemical reaction (a) It tells about physical state of reactants and products
(c) an exothennic change (b) It tells whether the reaction is spontaneous
(d) an endothennic change (c) It tells whether the reaction is exothermic or endothennic
30. An endothermic reaction is one in which: (d) It tells about the allotropic fonn (if any) of the reactants
(a) heat is converted into electricity "9. The enthalpies of all elements in their standard state at 25° C
(b) heat is absorbed and one atmospheric pressure are:
(c) heat is evolved (a) same (b) always positive
(d) heat changes to mechanical work
(c) always negative (d) zero
31. If total enthalpy of reactants and products is H Rand H p
respectively, then for exothennic reaction: 4U. The heat of reaction does not depend upon:
(a) HR Hp (b) HR < Hp (a) physical state of the reactants and products
(c) HR >Hp (d) HR Hp 0 (b) whether the reaction has been carried at constant pressure
32. Calculate the work done when I mol of an ideal gas is or constant volume
compressed reversibly from 1 bar to 4 bar at a constant (c) the manner by which the reaction has been carried
temperature of 300 K : (DPMT 2009) (d) the temperature at which the reaction has been carried
(a) 4.01 kJ (b) -8.02 kJ 41. The value of M! I1U for the following reaction at 27°C will
(c) 18.02 kJ (d) -14.01 kJ be:

[Hint: w 2.3~3 nRT 109( ~ ) 2NH3(g)~ N 2 (g) + 3H2 (g)


. [AMU (Medical) 2(07)
4 (a) 8.314 x 273 x (-2) (b) 8.314 x 300 x (-2)
2.303 x 1x 8.314 x 3001og- (c) 8.314 x 273 x 2 (d) 8.314 x 300x 2
1
= 3458.3 J '" 3.458 kJ ]
I
CHEMICAL THERMODYNAMICS AND THERMOCHEMISTRY 491

42. Since, the enthalpy of the elements in their standard states is which one of the following statements is correct?
taken to be zero, the heat of formation (!J.H f) of compounds: (a) x - y> 0 (b) x y < 0
(a) is always negative (b) is always positive (c) x y = 0 (d) None of these
(c) is zero (d) may be positive or negative 53. According to the equation,
43. The difference between heats of reaction at constant pressure 15
C6 H6 (I) + - 02 (g ) ------t 6C0 2(g ) + 3H2 0(1);
and constant volume for the reaction, 2
2C6 H6 (I) + 1502(g ) ------t 12C02(g) + 6H 20(l) at 25" C in !J.H - 3264.4 kJ mol- i
kJ is: (liT 1991)
the energy evolved when 7.8 g benzene is burnt in air will be:
(a) +7.43 (b) +3.72 (c) -7.43 (d) -3.72
IPET (MP) 1990)
44. For a gaseous reaction, A (g)+ 3B (g) -,0 3C (g) + 3D (g 1
(a) 163.22 kJ (b) 32.64 kJ (c) 3.264 kJ (d) 326.4 kJ
IJ..E is 17 kcal at 27°C. Assuming R 2cal K- I mol-I, the
54. !J.Hf (x), !J.Hf (yl' !J.Hf (Rl and !J.Hf (S) denote the enthalpies of
value of 6H for the above reaction is:
formation of x, y, R and S respectively. The enthalpy of the
(a) 15.8 kcal (b) J8.2 kcal
reaction, x + y ------t R + S is given by:
(c) 20.0 kcal (d) 16.4 kcal
(a) !J.Hf (xl + !J.Hf (Yl
45. Which of the following statements is correct for the reaction,
(b) !J.Hf(R) + !J.Hf(S)
CO(g ) + -1 O 2 (g ) ------t CO 2( ' temperature and
g) at constant (c) !J.Hf(x) + !J.Hf(y) - !J.Hf(R) AHf(S)
2 '
(d) AHr (R) + !J.Hf (S) - !J.Hf (x) !J..Hf (y)
pressure?
55. The enthalpy change for the reaction,
(a) !J.H = IJ..E (b) !J.H < IJ..E
CzH4 (g) + 302 (g) ------t 2C02 (g ) + 2H20(g ) is called:
(c) !J.H> IJ..E (d) None ofthese
(a) enthalpy offormation (b) enthalpy of combustion
46. For the reaction, Ag 2 0(s) ------t 2Ag(s) + I 02(g ) which one (c) enthalpy of vaporisation (d) enthalpy of sublimation
2
56. The heat of combustion of methane is -880 kJ mor l . If 3.2 g
of the following is true? of methane is burnt ........... of heat is evolved.
(a) !J.H = IJ..E (b) !J.H < IJ..E (a) 88 kJ (b) 264 kJ (c) 176 kJ (d) 440 kJ
(c) !J.H > IJ..E (d) AH I IJ..E 57. The enthalpy change for the r('-action,
2
2C(graphite} + 3H 2 (g} C2~(g}
47. Thelmochemistry is the study of relationship between heat is called:
energy and:
(a) enthalpy offormation (b) enthalpy of combustion
(a) chemical energy (b) activation energy
( c) enthalpy of hydrogenation (d) enthalpy of vaporisation
(c) friction energy (d) none of these
58. The enthalpy of formation of water from hydrogen and
48. Assume each reaction is carried out in an open container. For oxygen is - 286.0 kJ mol-I. The enthalpy ofdecomposition of
which reaction will!J.H = AU? reBSE (Med.) 2006) water into hydrogen and oxygen is ...... kJ mol-I:
(a) H2 (g) + Br2 (g) ------t 2HBr(g) ('1) -286.0 (b) -143.0 (c) 286.0 (d) 143.0
(b) C( s) + 2H20(g) ------t 2H2 (g ) + CO2 (g ) 59. The heat of combustion of C(graphite) is -393.5 kJ mol-I. The
+
(c) PCIs(g) ------t PCI 3 (g) CI 2 (g) heat of formation of CO2 from graphite is ..... kJ mor l :
(d) 2CO(g) + 0z (g ) ------t 2COz (g ) (a) 393.5 (b) -393.5 (c) -787.0 (d) 787.0
49. The enthalpy change in the reaction, 2CO + 02 ------t 2C02 is 60. The enthalpies of formation of organic compounds are
termed as: conveniently determined from their:
(a) enthalpy of reaction (b) enthalpy of fusion (a) boiling points
(c) enthalpy offormation (d) enthalpy of combustion (b) melting points
50. Reaction, H2 (g ) + 12 (g ) ------t 2HI; !J.H = 12.40 kcal ( c) enthalpies of neutralization
According to this, heat of formation of HI will be: (d) enthalpies of combustion
/pET (MP) 1991)1 61. !J.H combustion of a compound is always:
(a) 12.40 kca! (b) -12.4 kcal (a) positive (b) negative (c) zero (d) uncertain
(c) - 6.20 kcal (d) 6.20 kcal 61. The apparatus used for measuring the heat changes of a
S1. For the reactions, reaction is called:
(i) H2 (g)+ Clz(g) == 2HCl(g) + x kJ (a) a thermometer (b) a colorimeter
(ii) H2 (g) + Clz(g) = 2HCI(l) + y kJ (c) a calorimeter (d) none of these
which one of the foUowing statements is correct? 63. The heat of neutralization of any strong acid and strong base is
IPET (MP) 2007) always constant and AH = 57.3 kJ. This is because:
(a) x>y (b) x<y (c) x-y=O (d) x=y (Corned (Karnataka) 2008)
52. For the reactions, (a) both the acid and base undergo complete ionization.
(i) Hz (g) + Clz(g) = 2HCI(g) + x kJ (b) during neutralization, salt and water are formed.
(ii) 2HCI(g) == H2(g) + CI 2 (g) y kJ (c) I mole of water is formed from Wand OH- ions.
(d) the reaction is exothermic.
492 G.R.B. PHYSICAL CHEMISTRY FOR COMPETITIONS

64. The enthalpy of neutralization of oxalic acid by a strong base 1


CO + 2:°2 ~ CO2; Ml= - 283.2 kJ
is -25.4 k cal mol-I. The enthalpy of neutralization of strong
acid and strong base is -13.7 kcal eq -I. The enthalpy of the heat of reaction of C(graphile) + O2 ~ CO2 is:
dissociation of oxalic acid is: (DPMT 2009) (a) +393.7 kJ (b) -393.7 kJ (c) -172.7 kJ (d) +172.7 kJ
(a) 1 kcal moC I (b) 2 kcal mol-I 74. If
I
H2 + -02 ~ H2 0; Ml = - 68.09 kcal
(c) 18.55 kcal mol-1 (d) 11.7 kcal moC I 2
[Hint : Oxalic acid is dibasic acid hence expected heat of 1
K + H2 0 + water ~ KOH(aq.) + - H2 ; Ml == - 48.0 kcal
neutralization will be (2 x -13.7 kcal). 2
The dissociation energy of oxalic acid 2 x 13.7 - 25.4 KOH + water ~ KOH( aq. ); Ml = 14.0 kcal
= 2 kcal morl] the heat of formation of KOH is:
65. The amount of heat liberated when one mole ofNH40Hreacts (a) -68.39+48 14.0 (b) -68.39 48.0+14.0
with one mole ofHCI is: (IIT 1990) (c) + 68.39 48.0 + 14.0 (d) + 68.39 + 48.0 14.0
(a) 13.7 kcal (b) more than 13.7 kcal 75. The enthalpies of combustion of C(graphile) and C(diamond) are
(c) less than 13.67 kcal (d) cannot be predicted -393.5 and -395.4 kJ/mol respectively. The enthalpy of
66. Heat of neutralisation for the reaction,
conversion of C(grapbite) to C(diarrond) in kJ/mol is:
NaOH + HCl ~ NaCI + H2 0
I (a) -1.9 (b) -788.9
is 57.1 kJ moC . The heat released when 0.25 mole ofNaOH is
(c) 1.9 (d) 788.9
titrated against 0.25 mole ofHCl will be:
76. The heat of combustion of yellow phosphorus arid red
(a) 22.5 kJ (b) 57.1 kJ
phosphorus are -9.91 kJ and -8.78 kJ respectively. The heat of
(c) 28.6 kJ (d) 14.3 kJ
transition of yellow phosphorus to red phosphorus is:
67. If It + OH - = H2 0 + 13.7 kcal, then heat of complete (Bihar; 1992i
neutralisation of one gram mole of H2 S04 with strong base
will be: [Pl\1T (l\IP) 1990] (a) -18.69 kJ (b) +1.13 kJ
(c) +18.69 kJ (d) -1.13 kJ
(a) 13.7 kcal (b) 27.4 kcal
77. What will be the heat of formation of methane, if the heat of
(c) 6.85 kcal (d) 3.425 kcal
combustion of carbon is '- x' kJ, heat of formation of water is
68. In which of the following neutralization reactions, the heat of
'- y' kJ and heat of combustion of methane is '- z' kJ?
neutralization will be highest?
(A OW; 20(18)
(a) HC! and NaOH (b) CH 3 COOHand NaOH
(a) (-x- y+ z)kJ (b)(- z -x+ 2y)kJ
(c) CH3COOHandNH40H (d) HClandNH4 0H
(c)(-x 2y-z)kJ (d)( - x - 2y + z) kJ
69. "The enthalpy of formation of a compound is equal in
78. Given,
magnitude but of opposite sign to the enthalpy of
C(s)+ 02(g)~ CO 2 (g) + 94.2 kcal
decomposition of that compound under the same conditions."
1
This law was presented by: H2 (g) + 2: °2 (g ) ~ Hzo(l) + 68.3 kcal
(a) Hess (b) Le Ohatelier
(c) Kirchhoff (d) Lavoisier and Laplace CH4 (g)+ 202(g)~ COz(g) + 2H2°(l) + 210.8 kcal
70. "The change of enthalpy of a chemical reaction is the same The heat of formation of methane in kcal will be:
whether the reaction takes place ,in c;me step or in several WET (i\H'j 1'f90j
steps". This law was presented by: (a) 45.9 (b) 47.8 (c) 20.0 (d) 47.3
(a) Hess (b) Le Chatelier 79. On combustion carbon forms two oxides CO and CO2 , heat of
(c) van't Hoff (d) Kirchhoff formation of CO 2 is -94.3 kcal and that of CO is -26.0 kcal.
71. The enthalpy change in a reaction does not depend on the: Heat of combustion of carbon is: iEA!iKn Z(} Illj
(a) conditions under which the reaction is carried out (a) -26.0 kcal (b) - 68.3 kcal
(b) initial and final enthalpies of the reactants and products (c) -94.3 kcal (d) -120.3 kcal
(c) state of reactants and products 80. The heat of combustion of ethanol determined in a bomb
(d) number of intermediate steps to convert reactants to calorimeter is -670.48 kcal mol-I at 298 K. What is Mat
products 298 K for the reaction?
72. Hess's law deals with: (a) -760 kcal morl (b) - 670.48 kcal mol- l
(a) changes in heat of reaction (c) +760 kcal mol- t (d) + 670.48 kcal mort
(b) rate of reaction 81. 1 calorie is equivalent to:
(c) equilibrium constant (a) 4.184 J (b) 41.84 J (c) 418.4 J (d) 0.4184 J
(d) influence of pressure on volume of a gas 82. For an endothermic reaction, where Ml represents the
73. From the thermochemical reactions, enthalpy of reaction, the minimum value for the energy of
1 activation will be: rUT IIf~E; (t'l;-', 1'';~31
C(grapbite) + 2:°2~ CO; Ml = 110.5 kJ
(a) less than Ml (b) zero
(c) equalto Ml (d) more than Ml
CHEMICAL THERMODYNAMICS AND THERMOCHEMISTRY 493--

83. If S+ 02 ~ S02; Ml= 298.2kJ 91. The bond dissociation energy of C-H in CH4 from the
I equation
,S02 + - 02 -----t S03; Llli = -98.7 kl
2 C(g)+ 4H(g)~ CH4 (g); Ml 397.8kcal
S03 + H20 -----t HZS0 4; Llli = -130.2 kl is:
I (a) + 99.45 kcal (b) -99.45 kcal
HZ + - 02 -----t HzO; Llli = - 227.3 kl
2 (c) + 397.8 kcal (d) + 198.9 kcal
the heat offonnation of HZS04 will be: 92. The dissociation energy ofCH4 is 400 kcal morl and that of
(a) -754.4 kJ (b) + 320.5 kJ(c) 650.3 kJ (d) - 433.7 kJ ethane is 670 kcal mori. The C-C bond energy is:
84. Which of the following units represents the largest amount of (a) 270kcal (b) 70kcal (c) 200kcal (d) 240kcal
energy? 93. Heat of reaction at constant volume is measured in the
(a) Electron-volt (b) Erg apparatus:
(c) Joule (d) Calorie (a) bomb calorimeter (b) calorimeter
85. IfMl~ for H20 2(l) and Hz0(l)are-188 kJ mol- I and-286kJ (c) pyknometer (d) pyrometer
mor l , what will be the enthalpy change of the reaction 94. When 10 mL of a strong acid is added to 10 mL of an alkali,
2HzOz(/) ~ 2Hz°(l) + 02(g)? [PMT (lVIP) 19921 the temperature rises by 5° C. If 100 mL of the same acid is
mixed with 100 mL of the same base, the temperature rise
(a) 146 kJ mol- l (b) -196 kJ mol- i
would be:
(c) - 494 kJ mol- l (d) -98 kJ mol- l (a) 5°C (b) 50°C
86. The bond dissociation energies for C1 2, 12 and ICI are 242.3, (c) 20°C (d) cannot be predicted
151 and 211.3 kJ/mol respectively. The enthalpy of
95. Energy required to dissociate 4 g of gaseous hydrogen into
sublimation of iodine is 62.8 kJ I mol. What is the standard
free gaseous atoms is 208 kcal at '25 0 C. The bond energy of
enthalpy offormation ofICI(g)?
H-H bond will be:
(a) -211.3kJ/mol (b) 14.6kJ/mol
(a) 1.04 cal (b) 10.4 kcal
(c) 16.8kJ/mol (d) 33.5 kJ/mol
(c) 104 kcal (d) 1040 kcal
[Hint: CI 2(g) -----t 2C1(g); Llli I = 242.3 kl/mal
96. The heat of neutralisation of strong base and strong acid is
12(g) -----t 2I(g); Llli 2 = lSI kl/mal 57.0 kJ. The heat released when 0.5 mole ofHN0 3 solution is
ICl(g) -----t l(g) + Cl(g); Llli3 = 211.3 kl/mal added to 0.20 mole ofNaOH solution is: (KCET 1993)
12(s) -----t 12(g); Llli4 = 62.8 kl/mal (a) 57.0 kJ (b) 28.5 kJ
Required equation: (c) 11.40 kJ (d) 34.9 kJ
1 1 97. The enthalpy of combustion of cyclohexane, cyc10hexene and
Zlz(s)+ZClz(g) -----tICI(g); Llli=?
H2 are respectively -3920, -3800 and -241 kJ morl. The heat
Llli=62.8+151+242.3 211.3 of hydrogenation of cyclohexene is: (AHMS 2007)
2 (a) -121 kJ mol-I (b) 121 kJ morl
= 16.75 kl I mal] (c) -242 kJ morl (d) 242 kJ morl
87. Standard heat of formation for CCI 4, H20, CO2 and HCI at
98. If C(s) + 0z(g)~ CO2 (g); Ml = r
298K are - 25.5, - 57.8, 94.1 and 22.1 kJl mol respectively.
For the reaction, and CO(g) + I O2(g ) ~ CO2 (g); Ml = s
CCl4 + 2H20 ~ CO2 + 4HCI 2
what will be Ml? (SCRA 2007) then the heat of formation of CO is:
(a)36.4kJ (b)20.7kJ (c)-20.7kJ (d) 4l4kJ (a) r+ S (b) r s (c) S - r (d) r x S
88. Heat of combustion of CH4 , Cz~, CzH4 and CzHz are -212.8, 99. Given that,heat of neutralisation of strong acid and strong
-373.0, -337.0 and -310.5 kcal respectively at the same base is 57.1 kJ. Calculate the heat produced when 0.25 mole of
temperature. The best fuel among these gases is: HCl is neutralised with 0.25 mole of NaOH in aqueous
(a) C~ (b) CzH6 solution: (CPMT 1990)
(c) CzH4 (d) CzH z (a) 22.5 kJ (b) 57 kJ
89. Given, C(s) + 02 (g) ~ CO2 (g); Ml := - 395 kJ (c) 14.275 kJ (d) 28.55 kJ
S(S) + 0z(g)~ SOz(g); Ml:= -295 kJ 100. Which one of the following values of MIo represents that the
product is least stable?
CS z(I) + 302 (g ) ~ CO2(g ) + 2S0z(g); Ml =- 1110 kJ
(a) 94.0 kcal mol- 1 (b) -231.6 kcal mol- l
The heat offormation ofCS 2 (l ) is: IeEE (Bihar) 1992J
(c) +21.4 kcal morl (d) + 64.8 kcal morl
(a) 250 kJ (b) 62.5 kJ (c) 31.25 kJ (d) 125;':
90. The heats of combustion of rhombic and monoclinic sulphur 101. The value of MlO-H is 109 kcal mol-I. The formation of one
are -70960 and -71030 calorie respectively. What will be the mole of water in gaseous state from H(g) and O(g) is
heat of conversion of rhombic sulphur to monoclinic sulphur? accompanied by: (eBSE 1990)
(a) -70960 cal (b) -71030 cal" '," (a) 218 kcal (b) -109 kcal
(c) 70 cal (d) -70 cllI (c) -218 kcal (d) unpredictable
494 G.R.B. PHYSICALCHEMISTRY FOR COMPETITIONS

102. Heat of neutralisation of a strong dibasic acid in dilute solution (a) 750 K (b) 1000 K
by NaOH is nearly: (c) 1250 K (d) 500 K
(a) -27.4 kcalleq (b) -13.7 kcaiJeq [Hint: IJ.S L S Products - L S Reactants
(c) +13.7 kcalleq (d) -13.7 kcallmol
103. The enthalpy changes at 298 K in successive breaking of =50-[~X 60+~ X 40J
O-H bonds of water are
H2 0(g) ---7 H(g) + OH(g ); MJ:::: 498 kJ mort = 50- (30+ 60) = -40 J r i mol- t
At equilibrium, IJ.G = MJ - TIJ.S = 0
and OH(g) ---7 H(g) + O(g ); MJ 428 kJ mol- 1
i.e., MJ TIJ.S
The bond enthalpy of O-H bond is:
T = MJ = -30x 1000 = 750 K]
(a) 498 kJ mol- 1 (b) 428 kJ mol- t
IJ.S -40
(c) 70 kJ mort (d) 463 kJ mol- 1 113. 'At absolute zero the entropy of a perfect crystal is zero.' This
104. MJ and till for the reaction, S(s) + ~ O2 (g ) ---7 SO} (g) statement corresponds to which law of thermodynamics?
are related as: (a) First law (b) Second law
(a) MJ till - 0.5RT (b) MJ till 1.5RT (c) Third law (d) None of these
(c) MJ till+RT (d) MJ=till+1.5RT 114. Unit of entropy is: (PMT (Punjab) 2007]
105. A spontaneous change is one in which the system suffers: (a) J K- 1 morl (b) J mol- 1
(VITEEE 2008)
(c) rl K- 1 morl (d) JKmol- 1
(a) an increase in internal energy -
(b) lowering in entropy 115. Given that MJf(H) = 218 kJ/mol, express the H-H bond
( c) lowering in free energy energy in kcallmol: (EAMCET 2009)
. (d) no energy change (a) 52.15 (b) 911
106. The free energy change for a reversible reaction at equilibrium (c) 109 (d) 5.2153
is: 116. For which reaction from the following, IJ. S will be maximum?
(a) zero (b) positive (a) Ca(s)+~02(g)---7CaO(s)
(c) negative (d) none of these (b) CaC03 (s) ---7 CaO(s) + CO2 (g)
107. In the evaporation of water, the entropy: (c) C(s) + 02(g) ---7 CO2 (g)
(a) decreases (b) increases (d) N2 (g) + 02(g) ---7 2NO(g)
(c) does not change 117. A particular reaction at 27°C for which MJ > oand IJ.S > 0 is
(d) sometimes increases, sometimes decreases found to be non-spontaneous. The reaction may proceed
108. In which of the following change entropy decreases? spontaneously if:
(a) Crystallisation of sucrose from solution (a) the temperature is decreased
(b) Dissolving sucrose in water (b) the temperature is kept constant
(c) Melting ofice (c) the temperature is increased
(d) Vaporisation of camphor (d) it is carried in open vessel at 27° C
109. For the precipitation reaction of Ag + ions with NaCI which of 118. The least random state of water system is:
the following statements is true? (a) ice '(b) liquid water
(a) MJ is zero for the reaction (c) steam (d) randomness is same in all
(b) IJ.G is zero for the reaction 119. Although the dissolution of ammonium chloride in water is an
( c) IJ.G is negative for the reaction endothermi<; reaction, even then it is spontaneous because:
(d) IJ.G should be equal to MJ (a) MJ is positive, IJ.S is -ve
110. If enthalpy of vaporisation of water is 186.5 kJ/mol, the (b) MJ is +ve, IJ.S is zero
entropy of its vaporisation will be: (c) MJ is positive, T IJ.S < MJ
(a) 0.5 (b) 1.0 (c) 1.5 (d) 2.0 (d) MJis +ve, IJ.S is positive and MJ < T IJ.S
111. A reaction is non-spontaneous when: [AMU (Medical) 2006] 120. For an exothermic reaction to be spontaneous:
(a) Mi is +ve, IJ.S is -ve (a) temperature must be high
(b) both MJ and IJ.S are -ve (b) temperature must be zero .
(c) MJ is -ve and IJ.S is +ve (c) temperature may have any magnitude
(d) none of the above (d) temperature must be low
112. Standard entropies of X 2' Y2 and XY3 are 60, 40 and 50 121. . For the reversible process, the value of IJ.S is given by the
JK-1mor1 respectively. For the reaction: expression:
~X2+2Y2
2 . 2
~ XY3 ; MJ -30kJ
(a) T (b) T-qrev
to be at equilibrium, the temperature should be : (c) qrev X T (d) qrev - T
[CBSE (PMT)2010]
I

CHEMICAL THERMODYNAMICS AND THERMOCHEMISTRY 495


122. In an electrochemical cell, if EO is the emf of the cell involving 131. The enthalpy change of a reaction does not depend on:
n mole of electrons, then I:!.GO is: (AIIMS 1997)
(a) h.Go = nFEo (b) I:!.Go = nFEo (a) state of reactants and products
(c) EO=nFh.Go (d) h.Go = nF!EO (b) nature of reactants and products
123. The correct relatior. between equilibrium constant (K), (c) different intermediate reactions
standard free energy (I:!.G ° ) and temperature (T) is: (d) initial and final enthalpy change of reaction
(CET (J&K) 2006] 132. Which plot represents an exothermic reaction?
(a) I:!.Go RT In K (b) K = e-llGo/2.303RT
p

L--
= lO- llG '/2.303 RT
(c) I:!.Go RT log K (d) K
t R",
1/
(aIH~
1
(e) I:!.Go=RlnK
(b) H R
124. The value of entropy in the universe is:
(a) constant
(c) increasing
(b) decreasing
(d) zero --
Reaction coordinate Reaction coordinate

(Clll~
125. Which of the following thermodynamic relation is correct?

(a) dG =VdP SdT


(JEE (WB) 2010]
(b) dU = PdV + TdS (dl t I R P
(c) dH=-VdP+TdS (d) dG=VdP+SdT H~
[Hint: dG dH - TdS - SdT ---+- ---+-
+
(G=H-TS) Reaction coordinate Reaction coordinate
3 .
dH dU +PdV +VdP 133. 8 + -2 O?- -----7 80, + 2x kcal;
~

(H =U +PV) 1
S02 + "2 O 2 -----7 S03 + Y kcal
and dU TdS -PdV
dG (TdS - PdV)+ PdV + VdP TdS SdT The heat offormation of80 2 is:
dG VdP SdT] (AIIMS 1997; PET (Kerala) 2008]
126. If enthalpies of formation for ~H4 (g ), CO2 (g) and H2 0(l) (a) y 2x (b) 2x+ y (c) x+ y (d) 2x! y
at 25°C and 1 atm pressure be 52, -394 and -286 kJ mol- 1 134. Given,
respectively, enthalpy of combustion of ~H4 (g) will ge: NH3 (g ) + 3CI 2 (g ) ~ NCl 3 (g ) + 3HCI(g );
(CBSE 1995) N 2 (g)+ 3H2(g)~ 2NH3 (g);
1
(a) +141.2 kJ mol- (b) +1412 kJ rhol- 1 H2 (g) + Cl z (g ) ~ 2HCI(g ); !:!.H3
1
(c) -141.2 kJ mol- (d) -1412 kJ mol- 1 The heat of formation of Nel 3 (g ) in terms of !:!.HI' !:!.H2 and
127. Identify the correct statement regarding entropy: (CBSE 1998) lli3 is: (EAMCET 1998)
(a) at absolute zero, the entropy of a perfectly crystalline 1 3
( a) !:!.Hf - !:!.HI + - !:!.H2 - !:!.H3
substance is + ve. 2 2
(b) at absolute zero, the entropy of a perfectly crystalline 1 3
substance is zero. (b) !:!.H f = !:!.HI +"2 !:!.H2 - 2 !:!.H3
(c) at O°C the entropy of a perfectly crystalline substance is 1 3
taken to be zero. ( c) !:!.Hf !:!.HI - - !:!.H2 - - !:!.H3
(d) at absolute zero of temperature the entropy of all 2 2
crystalline substances is taken to be zero. (d) none of the above
128. The enthalpy and entropy change for a chemical reaction are 135. The word 'standard' in standard molar enthalpy change
- 2.5 x 10-3 cal and 7.4 cal deg- 1 respectively. Predict implies:
(a) temperature 298 K
whether the nature of reaction at 298 K is: (AFMC 1998)
(a) spontaneous (b) reversible (b) pressure 1 attn
(c) irreversible (d) non-spontaneous (c) temperature 298 K and pressure 1 atm
129. One mole of an ideal gas at 300 K is expanded isothermally (d) all temperatures and all pressures
from an initial volume of 1 litre to 10 litre. The M for this 136. The heat of formation (!:!.H~ ) of H2 0 (I) is equal to:
process is: (R 2 cal K- 1 mol-I) (CBSE 1998) (a) zero
(a) 163.7 cal (b) 1381.1 cal (b) molar heat of combustion of Hz (I)
(c) 9 litre-attn (d) zero (c) molar heat of combustion of Hz (g)
130. The latent heat of vaporisation of a liquid at 500 K and I atm (d) sum of heat offormation ofH2 0(g ) and 0z(g)
pressure is 10 kcaVmoL What will be the change in internal 137. An example of extensive property is:
energy (M ) of3 moles ofliquid at the same temperature? (a) temperature (b) internal energy
(CBSE 1998) (c) viscosity (d) molar heat capacity
(a) 13.0 kcal (b) -13.0 kcal 138. For a diatomic molecule AB, the electronegativity difference
(c) 27.0 kcal (d) -27.0 kcal between A and B == 0.2028.fi. [where, I:!. bond energy of
I

496 G.R.B. PHYSICAL CHEMISTRY FOR COMPETITIONS

AB - ~eometric mean of the bond energies of A2 and B 2]. The (a) 29.8 kJ (b) -11.8 kJ
electronegativities of fluorine and chlorine are 4.0 and 3.0 (c) -20.6 kJ (d) -29.4 kJ
. I
respectively and the bond energies are ofF-F: 38 kcal mo.- 147. For the reaction,
and ofCI-Cl: 58 kcal mo.-I. The bond energy ofCI-F is: A(g) + 2B(g) ~ 2C(g) + 3D(g)
(a) ~ 71 kcal/mol (b) - 61 kcal/mol the value of MI at 27°C is 19.0 kcal. The value of AE for the
(c) - 48 kcal/mol (d) -75 kcallmol reaction would be: (Given, R 2.0cal K- 1 mo.-I): (lIT 1998)
139. The value of MIo for the reaction Cu + (g) + 1- (g) ~ Cul(g) (a) 20.8 kcal (b) 19.8 kcal (c) 18.8 kcal (d) 17.8kcal
is - 446 kJ mol-I. If the ionisation energy of Cu(g) is 745 kJ 148. In thermodynamics, a process is called reversible when:
mol-I and the electron affmityof I(g) is -295 kJ mo.-I, then [liT (Screening) 2000, 01 J
the value of MIo for the formation of one mole of CuI(g ) from
(a) surroundings and system change into each other
Cu(g) and I(g ) is:
(b) there is no boundary between system and surroundings
(a) 446 kJ (b) 450 kJ (c) 594 kJ (d) 4 kJ
(c) the surroundings are always in equilibrium with the system
140. If the enthalpy change for the reaction,
(d) the system changes into the surroundings spontaneously
CH4 (g ) + CI 2(g ) ~ CH3Cl{g ) + HC1{g ); l:J{ = - 25 kcal,
149. The heat liberated when 1.89 g of benzoic acid is burnt in a
Bond energy of C-H is 20 kcal mol-1 greater than the bond
bomb calorimeter at 25°C increases the temperature of 18.94
energy of C-Cl and bond energies of H-H and H-Cl are
kg of water by 0.632°C. If the specific 'heat of water at25°e is
same in magnitude, then for the reaction Hz (g) + Cl z (g ) ~
0.998 calg-Jdeg- I ,the val~$'ofheat of combustion ofbenzoic
2HC1(g 1l:J{ ?
acid is: ' '(AFMC 2010)
(a) -22.5 keallmol (b) -20.5 kcallmol
(a) 881.1 kcal (b) 771.4 kcal
(c) -32.5kcal/mol (d) -12.5kcal/mol
(c) 981.1 kcal (d) 871.2 kcal
141. The standard heat of formation of sodium ions in aqueous
[Hint: Heat liberated by 1.89 g of benzoic acid,
solution from the following data:
q = msllT
Heat of formation ofNaOH{aq.) at 25° C =- 470.7 kJ
= 18940x 0.998 x 0.632
Heat of formation ofOIr(aq.) at 25°C = - 228.8 kJ
11946.14 cal
is: Heat liberated by the combustion of
(a) -251.9kJ (b) 241.9kJ (c) -24L9kJ(d) 251.9kJ 1 mol benzoic acid, i.e.,
142. AB, A z and .B z are diatomic molecules. If the bond enthalpies 122 g benzoic acid
of A z , AB and B2 are in the ratio 1: 1: 0.5 and the enthalpy of
formation of AB from A2 and B2 is -100 kJ mol-I, what is the = 11946.14 x 122 == 771126.5 cal
189
bond enthalpy of A2 ?
771-.12 kcal mol-I]
(a) 400 kJ mol-I (b) 200 kJ mol-1
150. One mole of a non-ideal gas undergoes a change state (2 attn,
(c) 100 kJ mo.-I (d) 300 kJ mol-I
3 L, 95 K) to (4 atm, 5 L, 245 K) with a change of internal
143. The lattice energy of solid NaCl is 180 kcal per mol. The energy, IlU = 30 L atm. The change in enthalpy (MI) of the
dissolution of the solid in water in the form of ions is process in L atm.: lilT (8) 20021
endothermic to the extent of 1 kcal per mol. If the solvation
(a) 40.0 (b) 42.3
energies ofNa + and Cl- ions are in the ratio 6 : 5, what is the
(c) 44.0
enthalpy of hydration of sodium ion? (d) not defined, because pressure is not constant
(a) - 85.6kcallmol (b) - 97.5 keallmol 151. Which of the following reactions defines MI~ ,?
(c) 82.6kcal/mol (d) +100kcallmol lilT (8) 20031
144. Which one of the following statements is false? (a) C(diarnond) + 02 (g )~ COz (g )
[liT (Screening) 2000, 01 J 1 1
(a) Work is a state function '2
(b) H2(g)+ 2 F2(g)~ HF{g)
(b) Temperature is a state function (c) Nz{g) + 3H2 (g) ~ 2NH3(g)
(c) Work appears at the boundary of the system
(d) Change in the state is completely defined when the initial (d) CO(g) +..!. 02{g) ~ CO2(g)
2
and final states are specified 152. If the enthalpies offormation of Ai 20 3 and Cr203 are -1596 kJ
145. AGO for the reaction x + y ~ z is -;- 4.606 kcal. The value and -1.134 kJ respectively, then the value of l:J{ for the
of equilibrium constant of the reaction at 227° Cis: reaction;
(lIT 1999) 2Ai + Cr203 ~ 2Cr + Ai 20 3 is: (AIlMS 2004)
(a) 100 (b) 10 (c) 2 (d) 0.01 (a) -462kJ (b) -1365kJ
(R == 2.0eal K- I mol-I) (c) -2530kJ (d) +2530kJ
[Hint: AG ° = - RT x 2.303 x log K ] 153. Which of the following is true for spontaneous process?
146. The enthalpy of solution of BaCl2 (s) and BaC1 2 . 2HzCXs) are [PMT (Himachal) 20061
-20.6 and 8.8 kJ mol-I ,respectively. The enthalpy change for (a) AG> 0 (b) t.G < (l (c) 6.G = 0 (d) 6.G T t.S
the reaction is: (liT 1998)
CHEMICAL THERMODYNAMICS AND THERMOCHEMISTRY 497

154. Considering entropy(s) as a thennodynamic parameter, the (a) 100 (b) 50


criterion for the spontaneity of any process is: (c) zero (d) different for different substances
(CBSE (PMT) 20041 164. Which of the following expressions is correct?
(a) M system + t1S surroundings > 0 [JEE (Orissa) 20051
(b) t1S system - t1S surroundings > 0 (a) t1Go == nFEO (b) t1GO = + nFE 0
(C) Msystem> Oonly (c) t1G 0 . 2.303RTnFE~ell (d) t1G 0 = - nF log Kc
(d) MSUI'!oundings> Oonly 165. Consider the reaction, N z(g ) + 3H2 (g ) ----j 2NH3 (g ) ;
155. An ideal gas expands in volume from 1x 10-3 m3 to
carried out at constant temperature and pressure. If !lH and
l x 10-2 m3 at 300 K against a constant pressure of
t1U are enthalpy change and internal energy change
. 1x lOs Nm-2 . The work done is: (AIEEE 2(04)
respectively, which of the following expressions is true?
(a) '-900J (b) -900kJ (AIEEE 20(5)
(c) 270kJ (d) +900 kJ (a)!lH 0 (b) Ml == t1U
156. What is the value of internal energy change (t1U)at 2rc ofa (c)Ml< t1U (d)!lH> t1U
gaseous reaction 2A2 (g) + 5Bz(g)~ 2A2BS (g) (whose
[Hint: Ml=t1U+b.nRT,t1n=2-4 2
heat change at constant pressure is -50700 J)?
R == 8.314 JK- 1 morl (SCRA 2(09) .. Ml <t1U]
(a) -50700J (b) -63171 J 166. The absolute enthalpy of neutralisation of the reaction,
(c) -38229 J (d) +38229 J MgO(s) + 2HCl(aq.) ----7 MgCI2(aq.) + H 20(/)
[Hint: !lH == t1U + t1ngRT will be: ICBSE-PMT (Pre) 20051
(a) -57.33 kJ ~01-1
-50700==t1U + (-5)x 8.314 x 300
(b) greater than -57.33 kJ mol- 1
t1U = -38229J]
(c) less than -57.33 kJ mol- 1
157. Two mole of an ideal gas is expanded isothennally and
reversibly from 1 litre to 10 litre at 300 K. The enthalpy (d) 57.33 kJ mol- 1
change (in kJ) for the process is: (liT (S) 20041 [Hint: Since, MgO is an oxide of a weak base, hence its
(a) 11.4kJ (b) -11.4kJ (c) OkJ (d) 4.8kJ neutralisation will evolve the heat less t!J:cin 57.33 kJ mol-I]
158. The enthalpy of vaporization of a liqu~ is 30 kJ mor 1 and . 167. The entropy values (in J K- 1 mol-I) ofH 2 (g) = 130.6, C12 (g)
entropy of vaporization is 75 J mol- 1 K- . The boiling point of = 223 and HCl(g) 186.7 at 298 K and I atm pressure are
the liquid at I atm is: lIlT (S) 2004J given. Then entropy change for the reaction,
(a) 250K (b)400K (c)450K (d)600K H 2 (g) + Cl2 (g) ----7 2HC1(g)
159. The sublimation energy of 12 (s) is 57.3 kJ/mol and the is: !BHU (Pre) 20051
enthalpy of fusion is 15.5 kJ/mol. The enthalpy of (a) + 540.3 (b) + 727.3 (c) 166.9 . (d) + 19.8
vaporisation of 12 is: (DCE 20(5) 168. A mixture of2 mole ofCO(g ) and one mole of O2 , in a closed
(a) 41.8 kJ/mol (b)- 41.8 kJ/mol vessel, is ignited to convert the carbon monoxide to carbon
(c) 72.8 kJ/mol . (d) - 72.8 kJ/mol dioxide. If !lH and t1U are enthalpy and internal energy
[ Hint: !lHsublimation == !lH fusion + Mf vaporisation] change, then: (KCET 20(5)
(a)!lH > t1U
160. !lH and M for a reaction are +30.558 kJ mol- 1 and 0.066 kJ (b)!lH < t1U
K-1 mol- 1 at 1 atm pressure. The temperature at which free (c)!lH t1U
energy change is zero and the nature of the reaction below (d) the relationship depends on the capacity of the vessel
this temperature is: [PET (Kerala) 20051
1:69. Consider the following reactions at 1000°C,
(a) 483 K, spontaneous (b) 443 K, non-spontaneous
(c) 443 K, spontaneous (d) 463 K, non-spontaneous A. Zn(s) + 1. 02(g) ----7 ZnO(s); t1GO = - 360 kJ mol- 1
2
(e) 463 K, spontaneous · 1 · '
161. What would be the amount of heat released when an aqueous B. C (graphite) +"2 02(g) ----7 CO(g); t1GO = -460 kJ/mol
solution,containirig 0.5 mole of HN0 3 is mixed with 0.3 mole
. ofOH- (enthalpy of neutralisation is -57.1 kJ)? Choose the correct statement at 100OOC:
[PET (Kerala) 2005] IPMT (Kerala) 20061
(a)28.5kJ (b) 17.1 kJ (c)45.7kJ (d)1.7kJ (a) zinc can be oxidised by carbon monoxide.
(e) 2.85 kJ . (b) ZnO can be reduced by graphite.
[Hint: 0.3 mole OH ion will be completely neutralised,. (c) both (a) arrd(b)are true.
.. AH == - 57.1 x 0.3 = -17.13 kJ] (d) both (a)and (b) are false.
162. A process in which the system does not exchange heat with the . (e). catbon monoxide can be reduced by zinc.
surroundings is known as: teET (J&K) 2005] 170. Which. one of the following equations does notcorrectfy
(a) isothennal (b) isobaric (c) isochoric (d) adiabatic represent the first law of thennodynamics for the given
163. The entropy of a crystalline substance at. absolute .zero on the . process? rPET (Kerala) 20061
basis of third law ofthennodynamics should be taken.as:
· , C E T (J&K) 20B51
498 G.R.B. PHYSICAL CHEMISTRY FOR COMPETITIONS

(a) Isothermal process q=- w 160.2 JK- I mo1- 1 respectively at 298 K and 1 bar. Assuming
(b) Cyclic process . q=- w that, /:,.H0 and /:,.S ° do not change with temperature;
(crI~ochoric proces's', /:,.U = q temperature above which conversion of limestone to lime will
(d) . Ad,iabatic process /:,. U = - w be spontaneous is: . (AIEEE2007)
(eY
'Expansion of gasit:tto vacuum /:,.U = q (a) 1118 K (b) 1008 K (c) 1200 K (d) 845 K
[mnti In adiabatic process q=0
178. Assuming that, water vapour is an ideal gas, the internal energy
:. Froni/:,.U=q-w' change (/:,. U) when 1 mp1 of water is vaporised at 1 bar pressure
Work is done by the system, and 100°C, (given: molar enthalpy of vaporisation of water at 1
. '. /:,.U = -w ' . for adiabatiC process] bar and 373K = 41 kJ mo1- 1 and R = 8.3 J K- I mol-I) will be:
171. The,enthalpy and e~tr:opy changes for the reaction:
(AIEEE 2007)
.' , Bril)+Clig)~2BrCl(g) 1
(a) 41.00kJmol- (b) 4.l00kJmol- 1
are 30. kJ mol-land 105 J K- I mol- I respectively. The
temperature at which the reaction will be in equilibrium is: (c) 3.7904 kJmo1- 1 (d) 37.904 kJmol- 1
'. [CBSE(Med.) 2006] ,[Hint: I1U = Ml - /:"~RT
(a) 285.1 K (b) 273 K (c) 450 K (d) 300 K = 41000 - 1 x 8.314 x 373
172. (/:,.H - /:,.U) for the formation of carbon monoxide (CO) from = 37898.878 J mo1- 1
its .element at 298K is:(R = 8.314 J K- 1 mol-I) = 37.9 kJ mol-I]
(AIEEE 2006) 179. For the process,
(a) "":1238,78 Jmol- I (b) 1238.78 J mol- I H20(l) (1 bar, 373K) ~ H20(g)(1 bar, 373K).
! (c) -2477.57 J mol-I. (d) 2477.57 J mol- I The correct set of thermodynamic parameters is: (lIT 2007)
,' 1· 1 1
[Hink C(s) + - 02(g)~ CO(g) I1ng .= 1- - =- (a) /:,.G=O,/:,.S=+ve (b) /:,.G=O,/:,.S=-ve
. '." 2" ,,' 2 2 (c) /:,.G=+ve,/:"S=O (d) /:,.G=-ve,/:,.S=+ve
JiH - /:,.U = I1nRT 180. For the reaction,
." .' 1 , " ,
= - x 8.314 x 298 = 1238.786 J mol-I] 2H2 (g) + 02 (g) ~ 2H20(g ); /:,.H0 = - 573.2 kJ
2. ' ,
The heat of decomposition of water per mol is:
173. For a phase change: (MHT-CET 2007)
. H 2 0(l) ~ H 2 0(s) (a) 286.6 kJ (b) 573.2 kJ (c) - 28.66 kJ (d) zero
O°C, 1 bar (AnMS 2006) 181. The free energy change /:"G = 0, when: [CET (J&K) 2007]
(a)/:"G = 0 (b) /:,.S = 0
(a) the reactants are completely consumed
(c) /:,.H = 0 (d) /:,.U = 0
(b) a catalyst is added
174., We can drive any thermodynamically forbidden reaction in the
(c) the system is at equilibrium
desired direction bycol,lpling with: . [PET (Kerala) ,2006]
. (a) highly exothermic reaction (d) the reactants are initially mixed
(b) highly endothermic reaction 182. In a closed container, a liquid is. stirred with a paddle to
increase the temperature, which of the following is true?
(c) .h,ighly exergonic reaction
[PMt (Pb.) 2007]
(d) "highly endergonic reaction
(e) reaction with large positive /:,.S values
*
(a) /:,.U = /:,.W 0, q = 0 (b) /:,.U = W = 0, q 0 *
.175•. Given that; bond energies of H - Hand Cl - Cl are
(c) /:,.U = 0, W = q 0 *
(d) W = 0, /:,.U = q 0 *
183. For the gas phase reaction,
.A30kJ/mol and 240 kJ/mol respectively. /:,.HI for HCI is PCls (g)'~ PCl 3 (g ) + Cl 2(g )
- 90 kJ/mo!. Bond enthalpy ofHCl is: ICBSE (Med.) 2007]
Which of the following conditions are correct?
('1),380 kJ mol- I (b) 425 kJ mol- I [CBSE-PMT (Pre) 2008]
(c) 245kJmol- 1 (d) 290kJmol- 1 (a) /:,.H < 0, /:,.S < 0 (b) /:,.H > 0 and /:,.S < 0
d~. The ~in6uitt .of heat rel!!ased, when 20mL 0[0.5 M NaOH is (c) /:,.H = 0, /:,.S < 0 (d) /:,.H > oand /:,.S > 0
mixed with 100 mLaf 0.1 M HCl;is x kJ. The heat of 184. Which of the following is correct? :[BHU (Screening) 2008]
neutralization (in kJJrK;r~l) is:
(~) -100x (b) - 50x
.. ~ f(M:alns))~OD'7]
(c) + 100i Cd) + 50x ' (a)Cv = (~~]p (b)C
p=(~~]v
, MV 0.5x 20
[Hmt~ Number of moles ofNaOH = - - =
' . 1000 1000
= 0.01 (c) Cp - Cv = R (d) (au] -a
" .' MV 0.1 x 100 V av:.t 2

Number of moles ofHCl = - - , = = 0.01 (Internal pressure in van dar Waals' equation)
" 1000 1000
. -x lti. Which of the following represents total kinetic energy of one
Heat of'neUtralization = - = - 1OOx ] mole of gas?I[HEE«(Orissap,n08]
. . ", 0.01
lifil.. LIl,J4~i~qJ1version of limestone to lime, (a).!RT (b)~RT (c)(Cp-Cv)RT (d)~RT
, ., CaC03 (s) ~ CaO(s) + CO2(g) 2 2 3
the \,alues of /:,.H0 and !:,.So are +179.1kJmol- 1 and
CHEMICAL THERMODYNAMICS AND THERMOCHEMISTRY +--499
186. Standard entropy of X 2' Y2 and XY3 are 60, 40 and 50 JK - I
I "
(a) Na(s)+ H20(l) - - - t NaOH(aq)+-H2 (g)1'
1
mol- respectively. For the reaction: """ 2
I 3 (b) Ag+(aq)+Cnaq) AgCI(s)
- X 2 +- Y2 -----? XY3 , Ml 30 kJ,
2 2 I .
(c) H2(g)+2:02(g)---t H20(l)
to be at equilibrium, the temperature will be: (AIEEE 2008)
[Cu(NH3 )4]t~.)
(a) lOOOK (b) 1250K (c) 500K (d) 750K 2
(d) Cu + (aq)+ 4NH3 (g)
[Hint: .!.X2 + ~12 --7 XJ3 191. The species which by defmition has zero standard molar
2 2 enthalpy of formation at 298 K is: (UT 2010)
MO Reaction =MO XlS3 -.!.2 MO X 2 - ~2 MO l'1 (a) Br2 (g) (b) Clz(g)
(c) H 2 0(g) (d) CH4 (g)
= 50 - .!. X 60 - ~ X 40 = - 40 JK- 1 mol- 1 192. The standard enthalpy of formation' of NHj is -46 kJ morl. If
2 2
AG = I:l1f - T M = 0 at equilibrium. the enthalpy offormation ofH2 from its atoms is -436 kJmorl
.. T = I:l1f = - 30 X 1000 750 K ] and that ofN2 is -712 kJ mor l , the average bond enthalpy of
M 40 N-H bond in NH3 is: (AIEEE WI:O)
187. 4.48 L of an ideal gas at S.T.P requires 12 calories to raise its (a) + 1056 kJ mor l " (b) -1l02 kJ mor l
temperature by 15Co at constant volume. The C p of the gas is : (c) -:964 kJ mol- 1 (d) +352 kJ mol- 1
[PMT (Kerala) 2009]
(a) 3 cal
(e) 9 cal
(b) 4 cal (c) 7 cal (d) 6 cal [Hint: ~ (N N)+ ~ H) H~N<:
[Hint.. Cv- - 4.48 -
- AU , n - - 0.2 I:l1f -46 kJ mol-1
nAT 2.24 BE (N N) =-712 kJ mol- 1
12 H) -436 kJ mol-1
= - - = 4 cal
0.2 x 15 Let RE. H) x kJ mol- I
C p = C v + R = 4 + 2 = 6 cal] I:l1f L(B. E. )Rcactants - L(B. E. )Products
[~(712)+~(436)J-3X
188. On the basis offollowing thermochemical data :
-46
[A JGo H+ (aq) = 0]
."
HzO(l)---t W(aq)+ Of1(aq); Ml = 57.32kJ x 352 kJ mol-I]
1 193. A 1 g sample of substance A at 100°C is added to 100 mL of
H2(g)+2: 02 (g)---t H2°(l) Ml -286.2kJ H20 at 25°C. Using separate 100 mL portion of H20, the
procedure is repeated with substance B and then with substance
The value of enthalpy of formation of OH- ion at 25°C is: C. How will the final temperatures of the water compare?
(AIEEE 2009) Substance $pecific heat
(a) -22.88 kJ (b) -228.88 kJ A 0.6 J g-I 0 C- 1
(c) +228.88 kJ (d) -343.52 kJ
B O.4J 0C- I
[Hint: The equation for the formation ofOf1(aq) is:
I
C 0.2 J 0C- (DUMET 2010)
H2(g)+.!.02(g) ~ H+ (aq) + Of1(aq)
2 (a) Tc > TB > TA (b) > TA > Tc
It is obtained by adding the two given equations : (c) > TB > (d) TA TB Tc
I:l1f = 57.32 + (-286.2) = -228.88 kJ] [Hint: q msl:J.T
189. The values of Ml and AS for the reaction, 1
sOC
C(graphite) + CO2 (g)---t 2CO(g) are 170 kJ and 170 AT
JK- 1 respectively. This reaction will be spontaneous at : Higher is the temperature of given solution, lesser is the
temperature so higher is the specific h~t. Order of
[CRSE (PMT) Pre 2009]
specific heat is A > B > C. Hence order of temperature will be :
(a) 510K (b) 710K (c) 910K (d) IIIOK '" "-~'i
TA >TB ]
[Hint: AG = I:l1f - TM, AG should be negative for spontaneous
process. 194. For vaporisation of water at 1 atmospheric pressure, the values
of Ml and I:J.S are 40.63 kJ mol-I and 108.8 J r1mor l
I:l1f -TM < 0
respectively. The temperature when Gibbs free energy change
I:l1f <TM
(I:J.G) for transformation will be zero, is:rCBSEi(pMl'~ 2010]
I:l1f T> 170x 1000
or T>- (a) 273.4 K (b) 393.4 K
M' 170
(c) 373.4 K (d) 293.4 K
T > 1000K]
[Hint: AG I:l1f - TM
190. In which reaction, there will be increase in entropy?
(DCE2009)
o 40.63xlOOO-TxlO8.8
T 373.4 K]
500 G.R.B. PHYSICAL CHEMISTRY FOR COMPET1TIONS

Set·2: The questions given below may have more (a) YzMf;(N2) - Mf;(NaN 3)
than one correct answers
(b) - Mf; (NaN 3)
I. Which is intensive property?
(a) Mass (b) Mass/volume (c) Mf;(NaN 3) -[Mf;(Na) + Yz
Mf;(N z)]
(c) Volume (d) Volume/mass
2. Which is an irreversible process?
(d) Mf; (Na) + Yz Mf; (Nz) Mf; (NaN3 )
(a) Mixing of two gases by diffusion 10. The lattice energy of KCI is 202 kcal/mol. When KCI is
(b) Evaporation of water at 373 K and 1 attn pressure dissolved in water 2 kcal/mol is absorbed. If the sol energies of
K+ and Cl- are in the ratio 2 : 3; then Mfhydration ofK+ is:
(c) Dissolution ofNaCI in water
(d) All of the' above (a) -80 kllmol (b) -120 kllmol
3. One mole of anhydrous MgCl2 dissolves in water and liberates (c) -150 kllmol (d) 133.3 kllmol
25 cal/mol of heat. Mf hydration of MgCl2 = - 30 cal/mol. Heat 11 •. Which is a correct relationship?
of dissolution of MgCl2 . H 20 is:
(a) +5 cal/mol (b) -5 cal/mol
(a) [dH] _[dE] = (+ve) '.
dT p dT T
(c) 55 cal/mol (d) -55 cal/mol
4. Following enthalpy changes are given: (b) [dE] 0 (for ideal gas)
a-D g\ucose(s) ----7 a-D g\ucose(aq.); Mi = 10.72 kJ dV T
~ -D glucose(s) ----7 J3 -D glucose(aq.); Mi = 4.68 kJ
a - D glucose(s) ----7 J3 - D glucose(aq.); Mi 1.16 kJ (c) [dV] = nR (for ideal gas)
Calculate the enthalpy change in, dT p P
a-D glucose(s)----7J3-D glucose(s) (d) All of the above
(a) 14.24 kl (b) 16.56 kl (c) -7.2 kl (d) 4.88 kl 12. The standard Gibbs free energy 6.G ° is related to eqUilibrium
5. If x and yare arbitrary extensi.ve variables, then: constant K pas:
(a) (x + y) is an extensive variable (a) Kp RT log 6.G o (b) Kp = [el RT ]I'.G
(b) x/ y is an intensive variable (c) Kp = -6.GoIRT (d) Kp = e-l'.GolRT
(c) dx/ dy is an intensive variable
13. For the two equations given below:
(d) both (b) and (c)
H 2(g) + .x02(g) ----7 H 20(l) + Xl kJ
6. If x and yare arbitrary intensive variables, then:
(a) xy is an intensive variable
.x
Hz(g) + 02(g) ----7 H 20(g) + x 2 kJ
Select the correct answer:
(b) xl y is an intensive variable
(a) Xl > X2 (b) x 2 > Xl
(c) (x + y) is an extensive property
(c) Xl =x2 (d) Xl + X z o
(d) dx/ dy is an intenSive property
1 14. M = 0, for which process?
7. H 2(g) + "2 02(g) H 20(g); Mi = x (a) Cyclic process (b) Isothermal expansion
1 (c) Isochoric process (d) Adiabatic process
H 2(g) + 2 °2(g) ----7 HP(I); Mi = Y 15. For a reaction to be spontaneous in neither direction, which of
Heat of vaporization of water is: the following is/are correct regarding the closed system?
(a) x+ y (b) x- Y (c) y-x (d) (x+ y) IBHU (Mains) 2007]
8. Which is correct about 6.G? (1) (6.G)r.p = 0 (2) (6.G)T,P < 0
(a) 6.G = Mf - T 6.S (b) At equilibrium, 6.G 0 = 0 (3) (6.S)u.v 0 (4) (M)u,v >0
(c) At eq. 6.G RT logK (d) 6.G 6.G ° + RT log K Codes:
9. Dissociation of sodium azide is given by, (a) 1,2 and 3 are correct (b) 1 and 2 are correct
NaN 3 ----7 Na + 312N2; (c) 2 and 4 are correct (d) 1 and 3 are correct
Mi for this is:
CHEMICAL THERMODYNAMICS AND THERMOCHEMISTRY 501

Assertion-Reason TYPE QUESTIONS


Set-1 Set-2
The questions given below consist of statements' Assertion' (A) and
The questions given below consist of an 'Assertion' (A) and the
'Reason' (R).
'Reason' (R). Use the following keys to choose the appropriate
answer: (a) If both (A) and (R) are correct and (R) is the correct
reason for (A).
(a) If both (A) and (R) are correct and (R) is the correct
explanation for (A). (b) Ifboth (A) and (R) are correct but (R) is not the correct
explanation for (A).
(b) Ifboth (A) and (R) are correct but (R) is not the correct
explanation for (A). (c). If (A) is true but (R) is false.
(c) If (A) is correct but (R) is incorrect. (d) Ifboth (A) and (R) are false.
1. (A) Enthalpy of graphite is lower than that of diamond.
(d) If (A) is incorrect but (R) is correct.
(R) Entropy of graphite is lower than that of diamond.
1. (A) The endothermic reactions are favoured at lower
temperature and the exothermic reactions are favoured at (AUMS1994)
higher temperature. 2. (A) The enthalpy of formation of gaseous oxygen molecules at
(R) When a system in equilibrium is disturbed by changing the 298 K and under a pressure of one atrn is zero.
temperature, it will tend to adjust itself so as to overcome (R) The entropy of formation of gaseous oxygen molecules
the effect of the change. under the same condition is zero. (AUMS 1996)
2. (A)Cp Cv Rforanidealgas. 3. (A) Heat of neutralization for both HN03 and HCl with NaOH
is 53.7 kJ per mol.
(R) [iJE ] = 0 for an ideal gas. (R) NaOH is a strong electrolyte/base. (AllMS 1997)
av T
4. (A) Decrease in free energy causes spontaneous reaction.
3. (A) When hydrogen gas at high pressure and room (R) Spontaneous reactions are invariably exothermic.
temperature expands adiabatically into a region of low (AIlMS 1997)
pressure, there is a decrease in temperature. 5. (A) Many endothermic reactions that are not spontaneous at
(R) Hydrogen gas at room temperature is above its inversion room temperature become spontaneous at high
temperature. temperature.
4. (A) The thermodynamic factor which determines the (R) Entropy of the system increases with increase in
spontanei ty of a process is the free energy. For a process to temperature. (AUMS 1998)
be spontaneous the free energy must be -ve. 6. (A) The enthalpy offormation ofH2 0(l) is greater than that of
(R) The change in free energy is related to the change in H 2 0(g).
enthalpy and change in entropy. The change in entropy for (R) Enthalpy change is negative for the condensation reaction
a process must always be positive if it is spontaneous.
H 20(g) ----t HP(I)
5. (A) The Joules-Thomson coefficient for an ideal gas is zero.
7. (A) For a particular reaction, heat of combustion at constant
(R) There are no intermolecular attractive forces in an ideal pressure (qp) is always greater than that at constant
gas. volume (qv ).
6. (A) As temperature increases, heat of reaction also increases (R) Combustion reactions are invariably accomplished by
for exothermic as well as endothermic reactions. increase in number of moles. (AUMS 1998)
(R) Mireaction varies according to the relation: 8. (A) The enthalpy of both graphite and diamond is taken to be
--"-----'- =AC P zero, being elementary substances.
T2 -T1 (R) The enthalpy of formation of an elementary substance in
7. (A) All exothermic reactions are spontaneous at room any state is taken as zero.
temperature. 9. (A) Heat of neutralization of perchloric acid, HCI0 4, with
(R) AG ve for above reactions and for spontaneous NaOH is same as that of HCl with NaOH.
reactions AG = - ve. (R) Both HCI and HCI0 4 are strong acids.
8. (A) Efficiency of a reversible engine is 100% (maximum) 10. (A) Heat of ionization of water is equal to the heat of
when the temperature of sink is - 273°C. neutralization of a strong acid with a strong base.
. T2 -1j
(R) Efficiency of engme, II == - - - . (R) Water ionizes to a very small extent while W ions from
T2 acid combine very rapidly with OIr from base to form H2 0.
502 G.RB. PHYSICAL CHEMISTRY FOR COMPETITIONS
" ',.,

11. (A) Enthalpy offormation ofHCI is equal to the bond energy 17. (A) There is noreaction known for which I1G is positive, yet it
ofHCI. .. is spontaneous.
(R) Enthalpy of formation and bond energy both involve (R) For photochemical reaction, I1G is negative.
formation of one mole of HCI from the elements. 18. (A) A reaction which is spontaneous and accompanied by
ll. (A) Pressure, volume and temperature are all extensive decrease of randomness must be exothermic.
properties. (R) All exothermic reactions are accompanied by decrease of
(R) Extensive properttes depend upon the amount and nature randomness.
of the substance. 19. (A) Molar entropy of vaporization of water is different from
13. (A}When a gas at high pressure expands against vacuum, the ethanol.
work done is maximum. (R) Water is more polar than methanol.
(R) Work done in expansion depends upon the pressure inside 20. (A) Heat of neutralization for both HN0 3 and HCI with NaOH
the gas and increase in volume. is 53.7 kJ/mol.
14. (A) When a real gas is allowed to expand adiabatically (R) NaOH is a strong electrolytelbase.
through a fine hole from a region of high pressure to a 21. (A) In the following reaction:
region of low pressure, the temperature of the gas falls. C(s) + 02(g) ~ CO 2 (g); Ml = I1U - RT
(R) Work is done at the cost of internal energy ofthe gas. (R) MI is related to I1U by the equation,
15. (A) Internal energy change in a cyclic process is zero. Ml = au I1ngRT
(R) Internal energy is a state function. 22. (A) Enthalpy of graphite is lower than that of diamond.
16:.- (A) An exothermic process, non-spontaneous at high (R) Entropy of graphite is greater than that of diamond.
temperature, may become spontaneous at low
23. (A) For a reaction
tempemture.
2NH3(g)~ N 2 (g)+ 3H2 (g); MI > AE
(R) With decrease in temperature, randomness (entropy)
(R) Enthalpy change is always greater than internal energy
decrease.
change. (ADMS 2008)
I
CHEMICAL THERMODYNAMICS AND THERMOCHEMiSTRY

Set-1 . ;;':r';':.·~';;',.~,·.
"

1. (b) 2. (c) 3. (d) . 4. (a) 5. (c) ~. (a) ",.7. (b) ,:,Idd}


"."1~t,·,

9. (a) 10.. (d) ,1. (c) 12. (d) 13. (c) 14. (d) . ; 15,. (a) . i6.,;(a)
:,,,'oc.

17. (c) 18. (a) 19. (a) 20.. (a) 21. (b) 22. (d) 23. (b) 'l'4,;r(a)
25. (c) 26. (d) 27. (d) 28. (b) 29. (d) 3~. (b) 31: (e) '3~!/(a .
""'V' )
"J.';:' "

33. (c) 34. (b) 35. (a) 36. (a) 37. (d) . ~8. (b) 39. (d) .4Q. (c)
; ." "
~
: ..-- '

41. (d) 42. (d) 43. (c) 44. (b) 45. (b) 46. (c) 47. (a) . '•.:.(a)
·'i,f! .. ·:,
49. (a) 50.. (d) 51. (b) 52. (c) 53. (d) 54. (d) 55. (b) 5(;'''(c )
'1 t·
57. (a) 58. (c) 59. (b) 60.; (d) 61. (b) 62. (c) 63. (a) . _~~.(b)
65. (c) 66. (d) 67. (b) 63. (a) 69. (d) 70.. (a). 71. (d) 7~da)
73. (b) 74. (b) ·75. (c) 76. (d) 77. (d) 78. (c) 79. (c) 80. (b)
81. (a) 82. (c) 83. (a) 84. (d) 85. (b) 86. (c) 87. (d) 8ft. (a)
89. (d) 90.. (c) JJ1. (a) "92. (b) 93•. (a) 94. (a)
j:;'- :.. ',' ':
95.
f
(c)
j' , ~ . 2~~M'"
99. :·:t~f(d) .. ~~~/\~)
1~'

97. (a) !}8. (b) (c) 10.0.. (d) Un~(c) to':!. (b) '","

10.5..
':','!~,~
(c) 1~6. (a) W7. (b) 10.8. (a) ..,9.(c) 1!~•.. <a) .J,!.(a) . )
'.
t'i
t':

~
"·"'1."0;" ,

~13. (c) 114.


:';,'-
(a) 115. (c) 11~. (b) 117. (c) l1~.(a) "~~Jd), ' m
11:
;~:''(d) ~ftt; (b)
,;';
111. (a) 122. (b) 123. (d) 124~ (c) 12S.(a) ~J

119. (d) 130. (c) 13.1. (c) 132. (a) 133. (a) ,<l.~;-.(a) '(b) ) ,-- ~
'<' ',: ,'. ',' '/.,1 ~
"~~')
~
137. (b) 138. (a) 139. (d) 14~. (a) 141. (c) '142; (a) 0'»
~
"':'

'fn~'(b)
"

145. (a) 146. (a) 147. (d) 148. (e) Hf:,;t(b) 'l~i"(c) 'lf~L(a)

I
. "" "";.i',
153. (b) 154. (a) 155. (a) 156. (e) l~f(b) 15& (b) 15': (a) l~~\;i4).
161. (b) 161. (d) 163. (e) 164. (a) 16i(c) 166. (e) 107. (d) 168. (0)---
II
III
169. (b) 170. (d) 171. (a) 1'n~" .(b) 173. (a) 174. (e) 17~. (b) 179~(a) t~
~j
..
177. (a) 178. (d) 179. (a) l$Oj(a) IS1> (c) tS2~ (a) UU. (d)
.,
iiU:\c)

I
t~.~-(b) i~~~;(~l)
, (" 1-",

185. (b) 1~. (d) ~87~ (d) 189. (d) 190. (a) ~9t. (b)
193. (e) 194. (c)
~. ,. , ...
Set-2
1. (b, d) 2. (a, e) J. (a) 4~ (d) 5. (a, b, c) 6. (a, b, d) 7. (b) 8. (a, d)
9. (b) 10.. (a) n. (d) 12. (d) . 13. (a) 14• (a, b) 15. (d)
f'
.

. Set-1
1. (d) 2. (b) 3. (d) '4. (e) 5. (a) 6. (d) 7. (a) 8. (a)

,Set-2
1. (b) 2. (e) 3. (b) 4. (e) 5. (b) 6. (a) 7. (d) . II. (d)
9. (a) to. (b) . 11. (d) 12. (d) 13. (d) 14. (a) 15. (a) te. (b)
17. (d) 18. (e) 19. (b) 20. (a) 21. (d) 22. (b) 23. (e)
, .-

504 G.A. B. PHYSICAL CHEMISTRY FOR COMPETITIONS

lIT ASPIRANTS 1 ,.j


Tbe foQo'ft'ing questions contain siagle correct option : [Hint: MI =:= MI fusion of two moles + ms t:.T
1. In which of the following pairs, both properties are intensive? ' = 6.01 x 2 +36 x 4.18 x 25 x 10-3
(a) Pressure, temperature (b) Density, volume '" 16 kJ]
(c) Temperature, density (d) Pressure, volume 8. Alarge positive value of t:.Go corresponds to which of these?
1. Although the dissolution of ammonium chloride (NH 4 CI) in
(a) Small positive K (b) Small negative K
water is an endothermic reaction, even then it is spontaneous (c) Large positive K (d) Large negative K
because: 9. Consider the values for MIO (in kJ mol-I) and for MO (in J
(a) t:.S - ve (b) t:.s zero mol-I K- 1) given for four different reactions. For which
(c) T M. < MI (d) t:.S := + ve and MI < T M reaction will t:.Go increase the most (becoming more positive)
3. The S-S bond energy is if MI'f(Et - S E t ) -147kJ/mol; . when the temperature is increased from ooe
to 25°C?
MI'f(E t ~S S- E t ) -202 kJ/mol and MI'fS(g):= + 223 (a) MIo 50, MO == 50 (b) MIo 90, MO := 20
kJ/mol: (c) MIo = 20, MO - 50 (d) MIo = - 90, MO =- 20
(a) 168 kJ (b) 126 kJ (c) 278 kJ (d) 572 kJ [Hint: Use the relation, t:.GO == MIO - T MO]
[Rial:
CzH s - S- C2HS + S(s) ---7 CzHs - S- S- CzH s 10. Fe203(S)+ "23 C(s)~ 32 CO 2 (g) + 2Fe(s);
o ' °
Mf reaction = ~f (products) - "'£tJff (reactants) . '- MIo = + 234.1 kJ
== (-202) - (-147) - 55 kJ C(s) + 0z(g) ---7 CO 2(g); MIo == 393.5 kJ
MIreaction L (BE)reactonts:'" l:(BE)prodUCts Use these equations and MI ° values to calculate MI ° for this
-55 = Heat of sublimation or enthalpy of atomisation reaction:
of sulphur BE(S- S) 4Fe(s) + 30 2(g) ---7 2FeP3(s)
-55 =223 BE(S- S) (a) -1648.7 kJ (b) -1255.3 kJ
BE(S- S) == 223 + 55 278 kJ (c) -1021.2 kJ (d) -129.4 kJ

°
(Common bonds of reactants and products are rejected.)]
4. Standard enthalpies of formation of 3, CO2 , NH3 and HI are [Iliac 2Fe(s) + 3 CO2 (g) ---7 FeP3(s) +
2. 2
~C(s) ... (i)

142.2, -393.2, 46.2 and +25.9 kJ mol- l respectively. The (t:.W = - 234.1kJ)
order of their increasing stabilities will be: C(s) + 0z(g) ---7 COz(g) (AW:= - 393.5 kJ) ... (ii)
(a) 03'COZ ' NH3 ,HI (b) CO2 , NH3 ,HI'03 MUltiplying eq. (i) by 2 and eq. (ii) by 3 and on adding both
(c) 03' HI, NH 3 , CO2 (d) NH 3 , HI, CO2 , 0 3 equations, we get:
[llial: (i) Exothermic compounds are more stable than 4Fe(s) + 30 2 (g) ---7 2FeP3(s);
endothermic compounds. MIo = (-234.1 x 2) + (-3 x 393.5)
(ii) Greater is the amount of heat evolved in the formation of a = 1648.7 kJ]
compound, more will be its stability.] 11. Consider this equation and the associated value for MI 0:
S. Combustion of octane takes place in an automobile engine. 2H2 (g) + 2C1 2 (g) ---7 4HCI(g); mo = -92.3 kJ
The homogeneous equation of combustion is:
Which statement about this information is incorrect? .
CSHlS(g) + ¥02(g) ---7 8C02 (g} + 9H 20(g) (a) If the equation is reversed, the MIO value equals + 92.3 kJ
The signs of MI, t:.S and t:.G for the reaction will be: (b) The four HC] bonds are stronger than four bonds in H2 and
(a) +ve, -ve, +ve (b) -:-ve; +ve, -ve Cl 2
(c) -ve, +ve, +ve (d) +ve, +ve, -ve (c) TheMIO value will be -92.3 kJ if HC! is produced as a
6. Which among the following represents the reaction of liquid
formation of the product? (d) 23.1 kJ of heat will be evolved when 1 mole of HCI(g ) is
(a) C(diamond) + 02(g)~ CO2 (g) produced
(b) S(monoclinic) + 02(g) ~ S02(g) 11. If the internal energy of an ideal ~as decreases by the same
(c) 2N2 (g)+ 02(g)~ 2N20(g) amount as the work done by the system, the process is:
(d) None of the above (a) cyclic (b) isothermal (c) adiabatic (d) isolated
7. How much energy must be supplied to change 36 g of ice at 13. The enthalpy of neutralization of a strong acid by a strong base
0° C to water at room temperature 25°C? is -57.32 kJ mol-I. The enthalpy of formation of water is
Data for water -285.84 kJ mol-I. The enthalpy of formation of hydroxyl
MI~on 6.01 kJ/mol ion is:
C p liquid 4.18 J K- l g-: (a) +228.52 kJ mol-I (b) -114.26 kJ mol-I
(c) -228.52 kJ mol-I (d) +114.2 kJ mo]-l
(a) 12 kJ (b) 16 kJ (c) 19 kJ (d) 22 kJ
CHEMICAL THERMODYNAMICS AND THERMOCHEMISTRY 505

[Hint: The process of neutralization is: 2NO~)+Oz~) ~~~-----------


H+ (aq.) + OW(aq.) H20(l); 1lH0 = - 57.32 kJ mol-I MfO = -112 kJ
IlH reaction' =L Heat of formation of products
L Heat of formation of reactants
= 1lH'H 0. -[IlH" + 1lH" ) 2N02~)
f 2 (1) fH+{aq.) fOW(aq,)
N2~)+ 202~) !J.H 0 = 180 kJ
-57.32 = 285.84 - (0 + x) ~------------------
x -285.84 + 57.32 Which of the following statements pertaining to the fonnation
of NO and N0 2 are correct?
=- 228.52 kJ)
14. For which process will Mfo and t:!.Go be expected to be most 1. Thestandard heat offonnation ofNOz is 68 kJ mol-I.
similar? 2. N02 is fonned faster than NO at higher temperature.
(a) 2Al(s) + Fez03(s)~ 2Fe(s) + AI 20 3 (s) 3. The oxidation reaction or nitrogen to N0 2 is endothennic.
(b) 2Na(s) + 2H z0(l) ~ 2NaOH(aq.) + Hz (g) 4. These two reactions often take place in troposphere and
(c) 2N02(g)~ N Z0 4 (g) causes green-house effeCt.
(d) 2H 2(g) + 02(g) ~ 2H 2 0(g) 5. These two reactions often take place and are responsible for
city smog.
[Hint: (a) In this reaction, I3.S o "" 0
Wl~2 OOI~3 ~1~4 ~3~4
t:!.GO = MfO - Tt:!.So
(e) 3 and 5
t:!.GO", MfO]
20. The enthalpy changes for two reactions are given by the
15. For a particular reaction, Mfo == - 38.3 kJ and t:!.SO -113 equations:
J K- I mol-I. This reaction is: 1
2Cr(s) + 12 Oz~) ----4 Cr203(s); IlH 1130 kJ
(a) spontaneous at all temperatures
(b) non-spontaneous at all temperatures 1
C(s) + 2 02~) ----4 CO~); IlH -110 kJ
(c) spontaneous at temperatures below 66°C
(d) spontaneous at temperatures above 66°C What is the enthalpy change, in kJ, for the reaction?
[Hint: t:!.G IlH - T t:!.S 3C(s) + Cr203(s) ----4 2Cr(s) + 3CO~)
(a) -1460 kJ (b) -800 kJ (c) +800 kJ (d) + 1020 kJ
For spontaneous process, t:!.G < O.
(e) +1460 kJ
.. IlH-T/JiS<O
[Hint: The given equations can be written as:
Mf<T 3 .
/J.S Cr203(s) ----4 2Cr(s) + - 02~); IlH = + 1130 kJ
2,
-38.3 x 1000 < T
3
-113 3C(s) + 2 02~) ----4 3CO~); !J.H 330 kJ
i.e., T > 338.93 K, i.e., 66° C]
Adding the above equations, we get:
16. Which halogen in its standard state has the greatest absolute Cr20 3(s) + 3C(s) ----4 2Cr(s) + 3CO~); t:!.H = + 800 kJ)
entropy per mole? '
21. The enthalpy change at 298 K for decomposition is given in
(a) F2(g) (bfCI 2Cg) (c) Br2(l) (d) 12 (s)
the following two steps:
17. For which of these processes is the value of /J.S negative? l
Step I: H20(g) ~ H(g) + OH(g); Mf == 498 kJ mor
I. Sugar is dissolved in water.
II. Steam condenses on a surface. Step II: OH(g) ~ H(g) + O(g); Mf == 428 kJ mOrl
IILCaC0 3 is decomposed into CaO and CO 2 , The bond enthalpy of the O-H bond is:
(a) I only (b) II only (a) 498 kJ mol- l (b) 463 kJ mo\-l
(c) I and III only (d) II and III only (c) 428 kJ mo\-I (d) 70 kJ mo\-I
. . 18. When solid NH4 N03 is dissolved in water at 25°C, the 22. Consider the following two reactions:
temperature of the solution decreases. What is true about the (i) Propene + H2 ~ Propane; Mfl
signs of Mf and /J.S for this process? (ii) Cyclopropane + H2 ~ Propane; Mf2

Mf t:!.S Then, Mfz - Mfl will be:


(a) 0 (b) 2BEc--.c BEc=c
(a) +
(c) BEc=c (d) 2BEc=c BEc-c
(b) [Hint: CH 3 CH=CH 2 + H2 ----4 CH 3-CH z -CH3;
(c) + + Mil = (BEc=c + BE H _ H) (2BEc_H + BEc-d
. CH
(d) + / 2~ + H2 ----4 CH3 -- CH2 CH3 ;
19. The diagram below shows the heat of reaction between N2 , 02' CH 2---CHz
NOandN02 : !J.H2 = (BEc_c + BE H _ H) - (2 X BEe_H)
IlH 2 - Mf I = 2BEc_c -:- BE c = c ]
506 G. R.B. PHYSICAL CHEMISTRY FOR COMPETITIONS

23. Under which circumstances would the free energy change for
a reaction be relatively temperature independent?
C. Mfo enthalpy change of a (iii) R T 2 rd In K J
reaction in standard state \ dT p
(a) 1::Jf0 is negative
(b) 1::Jf0 is positive D. Standard free energy change (iv) _ {d t1G} .
(c) MO has a large positive value (t1GO) dT p

(d) MO has a small magnitude Select the correct answer:


24. Use the free energy change, t1GO = + 63.3 kJ, for this reaction, Codes: A B C D
to calculate the K sp of Ag 2 C0 3(s) in water at 25°C: (a) (i) (ii) (iii) (iv)
(b) (ii) (iv) (iii) (i)
Ag2C03(s) ~ Ag+ (aq.) + CO~- (aq.)
(c) (iv) (ii) (iii) (i)
(a) 3.2 x 10-26 (b) 8 x 10- 12 (c) 2.9 X 10-3 (d) 7.9 x 10-2
(d) (i) (ii) (iv) (iii)
25. Which statement(s) is/are true?
1. So values for all elements in their states are positive. 32. An ideal gas is allowed to expand under adiabatic conditions.
The zero value is of:
2. So values for all aqueous ions are positive.
.(a) t1T (b) t1S
3. MO values for all spontaneous reactions are positive.
(c) t1G Cd) none of these
(a) 1 only (b) I and 2 only
33. Match the physical changes in List-I with their relations given
(c) 2 and 3 only (d) All of these
in List-II:
26. The enthalpy ofa reaction does not depend upon:
List-I List-D
(a) the intermediate reaction steps
(b) the temperature of initial and final state of the reaction A. t1G (i) t1U + Pt1V
(c) the physical states of reactants and products B. I::Jf (ii) -nFE
(d) use of different reactants for the formation of the same C. t1So (iii) -RT loge K
product v\
D. t1Go
27. When a solution of500 mL of2 MKOH is added to 500 mL of
2 M HCI, then the rise in temperature 1] is noted. When the
(iv) nR loge
r....l.j
\JIj
same experiment is repeated by mixing 250 mL of each Select the correct answer from the given codes:
solution, the rise in temperature T2 is noted: Codes: A B C D
(a) 1] T2 (b)1] = 2T2 (c) T2 = 21] (d) 1] 4T2 (a) (ii) (i) (iv) (iii)
28. The heat of combustion of solid benzoic acid at constant
(b) (i) (ii) (iii) (iv)
volume is -321.30 kJ at 27°e. The heat of combustion at
constant pressure is: (c) (iv) (iii) (ii) (i)
(a) 321.30 - 300R (b) -321.30 + 300R (d) (i) (ii) (iv) (iii)
(c) -321.30 150R (d) -321.30 + 900R
34. Given the following data:
[Hint:
15
C6H sCOOH(s)+ - 02(g)~ 7C02(g) + 3Hz0(l), Sa~ ~..,.)"~~~~'~:~i;~~
2 FeO(s) -266.3 57.49 -245.12
15
t1n=7_ =-.! C (Graphite) 0 5.74 0
2 2
Fe(s) 0 27.28 0
Mf =: t1U + t1nRT
CO(g) -110.5 197.6 -137.15
321.30 -.! x R x 300 =- 321.30 150R]
2 Determine at what temperature the following reaction is
29. A B; 40 kJ mol- I
t1U spontaneous?
If the system goes from A to B by a reversible path and returns FeO(s) + C (Graphite) ----" Fe(s) + CO(g)
to state A by an irreversible path, what would be the net (a) 298 K
change in internal energy? (b) 668 K
(a) More than 40 kJ (b) Zero (c) 966 K
(c) Less than 40 kJ (d) 40 kJ (d) t1G o is +ve, hence the reaction will never be spontaneous.
30. For the process, NH3(g) + HCI(g) ~ NH4 CI(s): [Hint: Mf reaction "L.{Mf; Fe(s) + Mf; CO(g)}
(a) I::Jf + ve, t1S = + ve (b) I::Jf = ve, t1S + ve
(c) I::Jf = + ve, t1U = - ve (d) I::Jf = ve, t1S ve {Mf; FeG(s) + Mf; C(graphite)}
31. Match the thermodynamic properties (List-I) with their (0 - 110.5) - (-266.3 + 0) = 155.8 kJ mo[-I
relation (List-II):
• List-I List-II t1Sreaction "L.[S;e(s) + S~o(g)] - [S;eG(s) + S~(graphite)]
A. Free energy change (t1GO) (i) RT loge K (27.28 + 197.6) - (57.49 + 5.74)
B. Entropy change MO (ii) -nFE 161.65 JK- I mor l
I
CHEMICAL THERMODYNAMICS AND THERMOCHEMISTRY 507

. W 155.8 x 1000 39. For the reaction shown, which is closest to the value of W?
For spontaneous reactIOn, T > - ; T > ----
!1S 161.65 2Cr 3+(aq.) + 3Ni(s) ----7 2Cr(s) + 3Ni 2+(aq.)
T >966K]
35. Which of the following equations haslhave enthalpy changes
W; (kJ mol-I)

equal to W comb C? Cr 3+ (aq.) -143


I. C( s) + O2(g ) ~ CO 2(g ) Ni 2+ (aq.) - 54
I .
II. C(s) + - 02(g) ~ CO(g) (a) 124 kJ (b) 89 kJ
2 (c) -89 kJ (d) -124 kJ
1
III. CO(g) + - O 2(g ) ~ CO 2(g ) 40. An ice cube at O.OO°C is placed in 200 is of distilled water at
2
25° C. The final temperature after the ice is completely melted
(a) I and II (b) I, II and III is 5°C. What is the mass ofthe ice cube?
(c) I and III (d) I only
(w fus =340Jg-l , Cp =418Jg-
.
1
°C-1)
36. The enthalpy change of which reaction corresponds to W~
(a) 23.6 g (b) 46.3 g
for Na 2C0 3 (s) at 298 K?
(c) 50.0 g (d) 800 g
3
(a) 2Na(s)+ C(s) + - 02(g) ~ Na 2C0 3 (s) 41. Which reaction occurs with the greatest increase in entropy?
2 (a) 2H2 0(l) ~ 2H2 (g) + 02 (g)
(b) Na 20(s)+ CO2(g) ~ Na 2C0 3 (s) (b) 2NO(g) ~ N 2(g) + 02(g)
(c) 2Na+ (aq.) + CO~- (aq.)~ Na2C0:i(s) (c) C(s) + 02(g) ~ CO 2 (g)
(d) 2Na + (aq. )+20Ir (aq. )+C02 (aq. )-7Na 2C0 3 (s)+ H 20 (d) Br2 (g ) + CI 2(g ) ~ 2BrCI(g)
42. The bond dissociation energies for single covalent bonds
37. Enthalpy is equal to: (VITEEE 2007)
formed between ·carbon and A, B, C, D and E atoms are:
(a) T 2 [ d(G/T) ] (b) _ T 2 [ d(G/T) ] Bond Bond energy (kcal mol-I)
dT p dT p (i) C-A 240
(ii) C-B· 382
(c) T 2 [ d(G/T) ] (d) _T2 [ d(G/T)]
dT v . dT v (iii) C-D 276
(iv) C-E 486
[Hint: G=H-TS .,.(i)
This indicates that the smallest atom is:
G =U + PV -TS
(a) A (b) B ( c ) C (d) E
!1G =!1U + P!1V + VM - ns - S!1T [Hint: C----'-E bond has highest bond energy; it means that the
From the first and second laws,
covalent bond C-E will be strongest. Smaller is the size of atom,
ns =!1U + P!1V stronger is the covalent bond.]
!1G = VM - S!1T
43. An ideal gas is taken around the cycle ABCA as:
At constant pressure, M = 0
!1G =-S
!1T
From eqs. (i) and (ii),
... (ii)

14:: B
---------A
G =H +T

OR
!1G
!1T

G =H + T ( dG J
Il..
GC 1
I
I
I
I
1
1
I
I
I
aT p

-;z =- :2 + t (~~ J p
(a) 12~fJ (b) 6~fJ (c) 3~fJ . (d) ~fJ
= [ a(G/T) ]
[Hint: Work done in the cyclic process
aT p
= Area bounded (ABCA)
H = _T2 [ a(G/T)] ] I
=-x AC xAB
aT p 2
38. When a bomb calorimeter is used to determine the heat of 1
= "2 x 2 VI X 3PI = 3PI Vd
reaction, which property of the system under investigation is
most likely to remain constant? 44. One gram mole of graphite and diamond were burnt to form
(a) Number of molecules (b) Pressure CO 2 gas.
(c) Temperature (d) Volume C(graphite) + O2(g) ~ CO2(g); wo = - 399.5 kJ
C(diamond) + 02(g) ----7 CO 2(g); wo = - 395.4 kJ
I
508 G.R.B. PHYSICAL CHEMISTRY FOR COMPETITIONS

(a) graphite is more stable than diamond ~ x (1500 1000) x (250 - 150)
(b) diamond is more stable than graphite ~----~~-------------------------xl00
(c) graphite has greater affinity with oxygen x (1500-1000) x (250 -150) + (1500-1000) x (150 - 0)
2
(d) diamond has greater affinity with oxygen
[Hint: Thermal stability of one isotope is directly proportional .!. x 500 x 100
to the heat of combustion.] = I 2 x 100
45. Which among the following is not an exact differential? - x 500 x 100 + 500 x 150
2
.(a) Q (dQ heat absorbed)
500 x 50x 100 = 25]
(b) U (dU change in internal energy) 500 x 50 + 500 x 150
(c) S (dS entropy change)
49. In Haber's process of ammonia manufacture:
(d) G (dG = Gibbs free energy change)
N 2 (g) + 3H 2 (g) ----4 2NH 3(g);
[Hint: Heat' Q' is a path dependent function, hence its exact
differentiation is not possible; however, internal energy, entropy lili;soc - 922 kJ
and Gibbs free energy are state functions, hence can be Molecules H 2 (g) NH3(g)
differentiated exactly.] Cp J K- I mol-I 28.8 35.1
46. A gas expands adiabatically at constant pressure such that:
If C p is independent of temperature, then reaction at 100c C as
1
T=- compared to that of25°C will be:
.JV (a) more endothermic (b) less endothermic
The value ofy, i.e., (C p I Cv ) of the gas will be: (c) more exothermic (d) less exothermic
(a) 1.30 . (b) 1.50 (c) 1.70 (d) 2
[Hint: Use: lili2 lili I == t.cp ]
I . T2 T1
[Hint: T oc-
.Jv SO. Consider the following statements:
TV 1/2 = constant I. Change in enthalpy is always smaller than change in
For adiabatic process, TV Iff TV Y- 1 == constant internal energy.
1 3 II. The variation in enthalpy of a reaction with temperature is
y-l=2,y==2"] given by Kirchhoff's equation.
III. The entropy change in reversible adiabatic process is equal
47. 2Zn + 0z ----42ZnO; t.GO = 616 J ... (i)
to zero.
2Zn + 8 ----42Zn8; t.Go = 293 J ... (ii) Select the correct answer:
2S+ 202 ----4 2802(g); t.Go = -408 J ... (iii) (a) I and II (b) II and III
t.Go for the following reaction: ( c) III and I (d) all are correct
2ZnS + 302 ----4 2ZriO + 2802 51. In CzH4 , energy offormation of(C C)and (C-C) are -145
would be: kJ/mol and 80 kJ/mol respectively. What is the enthalpy
(a)-731J (b)-l317J (c)+731J (d) +1317 J change when ethylene polymerises to form polythene?
[Hint: t.GO == 28.G~no + 2t.G;02 -2t.G~ns [JEE (Orissa) 2006]
(a) +650 kJ/mol (b) +65kJ/mol
[-616-408] [-293]
(c) 650 kJ mol- 1 (d) -65 kJ mol- 1
-1024 + 293
=-731J] [Hint: Po\ymerisation of ethene may be given as:
48. The efficiency of the reversible cycle shown in the figure will nCH 2 ----4 -+
CH 2 - CHzin
be: lili ~(BE)reaetants ~ (BE)producls
(a) 33.33% (b) 56% (c) 66% (d) 25% = (+145) (+80) = + 65 kJ/mo!]
[Note: C-H bonds are common in both reactants and prodUcts.]

t 250 ---------~
52. If 150 kJ of energy is needed for muscular work to walk a
distance of I km, then how much of glucose one has to consume
to walk a distance of 5 km, provided only 30% of energy is

Temp. (K) 150 ---------W


A: :cI
I
I
I
available for muscular work. The enthalpy of combustion of
glucose is 3000 kJ mol-I:
(a) 75 g (b) 30 g (c) 180 g
IPMT (Kerala) 2007]
(d) 150 g
I I
I I (e) 45 g
1000 1500 53. The value oflog 10 K for a reaction A ~ B is:
Entropy JI\1 __ (Given, t.rH~9gK - 54.07 kJ morl ,
t.rS~98K == 10JK- mor 1
and R l
= 8.314 JK- I mol-I;
. Area of closed
[Hint: EffiCiency of cycle = x 100 2.303 x 8.314 x 298 = 5705) (lIT 2007)
Area under the curve
(a) 5 (b) 10 (c) 95 (d) 100
I
CHEMICAL THERMODYNAMICS AND THERMOCHEMISTRY 509

[Hint: /j,G ° = Mlo - T/'o"S° Following questions may have more than one correct options:
= - 54.07 x 1000 - 298 x 10 1. Which of the following are correct about irreversible
= -57050 J isothermal expansion of ideal gas?
/j,Go = -2.303 RT loglOK (a) W = - q (b) /j,U = 0
p,
- 57050 = - 5705 logtoK (c) I1T =0 (d) W =- nRT In --.l..
P2
loglO K = 10]
54. The lattice enthalpy and hydration enthalpy of four 2. The work done during adiabatic expansion or compression of
compounds are given below : an ideal gas is given by :

Lattice enthalpy Hydration enthalpy (a) n C v I1T (b)~(T -T,)


(y_l) 2 1
Compound
(kJ/mol) (kJ/mol)
(e)-nRPext [T2~ -~Pi] (d)-2.303RTlog V2
P +780 -920 p[P2 Vi
Q +1012 - 812 3. For an ideal gas [CC pm
= yJ; of molar mass M, its specific heat
Vm
R + 828 - 878
capacity at constant volume is :
S + 632 -600
(a) yR (b) y (c) M (d) YRM
The pair of compounds which is soluble in water is : (y-I)M M(y-I) R(y-I) y-l
[PET (Kerala) 2008]
[Hint: CPm = y, CPm - C Vm = R
WP~Q ~Q~R ~R~S ~Q~S C Vm
(e) P andR R Ry
C Vm =--andCpni = -
[Hint: For solubility of ionic compounds, hydration energy must y-I y-I
be greater than lattice energy.]
-y_=C Vm xM
y-I
C x]
Vm M (y -I)

[,,f.~(HtfIA)
• Single correct option
1. (c) 2. (d) 3. (c) 4. (c) 5. (b) 6. (d) 7: (b) 8. (a)
9. (c) 10. (a) il. (c) 12. (c) 13. (c) 14. (a) 15. (d) 16. (b)
17. (b) 18. (c) 19. (e) 20. (c) 21. (b) 22. (b) 23. (d) 24. (b)
25. (a) 26. (a) 27. (a) 28. (c) 29. (b) 30. (d) 31. (b) 32. (d)
33. (a) 34. (c) 35. (d) 36. (a) 37. (b) 38. (d) 39. (a) 40. (b)
41. (a) 42. (d) 43. (c) 44. (a) 45. (a) 46. (b) 47. (a) 48. (d)
49. (d) 50. (b) 51. (b) 52. (d) 53. (b) 54. (e)
• One or more tllan ·GRe correct ,options
1. (a,b,e,d) 2. (a,b,e) 3. (b)

510 G.R.B. PHYSICAL CHEMISTRY FOR COMPETITIONS

Integer Answer TYPE QUESTIONS


9. Gas (Ax) has the ratio of specific heat, equal to 1.66. The
This section contains 11 questions. The answer to each value of x will be :
of the questions is a single digit integer, ranging from 10. For a liquid the vapour pressure is given by :
oto 9. Ifthe correct answers to question numbers X, Y, log 10 P
-400 + 10
Z and W (say) are 6, 0, 9 and 2 respectively, then the T
correct darkening of bubbles will look like the given Vapour pressure of the liquid is I if mm Hg. The value ofx will
figure: be:' .
11. One' mole of an ideal gas is taken from a to b along two paths
denoted by the solid and the dashed lines as shown in the
1. A bubble of8 moles of helium is submerged at certain depth in graph below. If the work done along the solid line path is Ws
water. The temperature of water increases by 30°C. How much and that along the dotted line path is Wd' then the integer
heat is added approximately to helium (in kJ) during closest to the ratio Wd / Ws is:
expansion?
2. For a Iiq~id, enthalpy of fusion is 1.435 kcal mol- 1 and molar 4.5
entropy change is 5.26 cal mol-IK~I. The freezing point of 4.0 a -----~-----,
I
I
liquid in ce\cius will be : 3.5 I
I

3. For the reaction, Ag20(S)~ 2Ag(s)+ .:!. O 2(g )


I
3.0 I
1
~ 1
2
E 2.5 't'1
W,6S and Tare 40.63 kJ mol-I, 108.8 JK-1 morl and :§.
2.0
1
I.
a. 1
373.4 K respectively. Free energy change I1G of the reaction 1.5
1
1
1
will be: I

1.0
4. Standard Gibbs Free energy change I1Go for a reaction is zero.
0.5 -------~-----------
The value of equilibrium constant of the reaction will be: . b
., 5. 6Go for the reaction x + y ~ z is 4.606 kcaL The value of 0.0 i .1

equilibrium constant of the reaction at 227°C is (x x 102 ). The 0.0 0.5 1.0 1.5 2.0 2.5 3.0 3.5 4.0 '4.5' 5.0 5.5 6.0
value of' x' is : V (lit.)
6. 4.48 L of an ideal gas at STP requires 12 cal to raise the
temperature by 15°C at constant volume. The Cp of the gas is (liT 2010)
........... cal. [Hint: wd = work done along dotted line
7.
'LP6V
4 x L5 + I x 1+ 25 x ~ 8.65 L atm
3
i
p
Ws :;:: It is reversible isothermal process

I == 2303nRT IOg[ ~~ J
= 2.303 x (PV) log ( ~: J
V-
= 2.303 x 2 log 5.5 = 4.79 L atm
In the present graph, the area of circle A and B are 25 unit and 0.5
20 unit respectively. Work done will be ........... unit. Wd = 8.65 "" 2 ]
'8•. For the reaction, Ws 4.79.
N2 (g ) + 3H2 (g) ----7 2NH3 (g )
Heat of reaction at constant volume exceeds the heat of
reaction at constant pressure by the value of xRT. The value of
x is:

ll. (3) j. (0) j .. (0) .5. (I) 6~ (6) 1. (5) 8. (2)


':9... (1) :t.tl. (9) 1t. (2)
CHEMICAL THERMODYNAMICS AND THERMOCHEMISTRY 511

~NStOt4 ''IVlrIC''1 '.,II:~~rllllll.t


',:";;;',"",, ,.,h""',,''''''~ ~~~~~;;;Z;;:~-'i
4. What value of flT should be used fQr the calorimetry
• Passage 1 experiment that gives the following graphical results?
Chemical rea,etions are invariably associated with the transfer of
energy either in the form oj heat or light. In the laboratory, heat
changes in physical and chemical processes are measured with an
instrument called calorimeter. Heat change in the process is
calculated as:
q ms flT; s = Specific heat
cflT; c == Heat capacity
25°C --
Heat of reaction at constant volume is measured using bomb
calorimeter.
qv flU, Internal energy change lime (min) __

Heat of reaction at constant pressure is measured using simple or (a) lOoC (b) 25°C (c) 20°C (d) 35°C
water calorimeter.' " 5. The enthalpy of fusion of ice is 6.02 kJ mol-I. The heat
qp=M capacity of water is 4.18 J g-l C-!. What is the smallest
number of ice cubes at DoC, each containing one mole of
qp :::: qv + P flV water, that are needed to cool 500 g ofliquid water from 20 0 e
M flU + flnRT to DOC?
The amount of energy released during a chemical change (a) I (b) 7 (c) 14 (d) 125
depends ',011 the physical state of reactants and products, the [Hint: Heat released to cool 500 g water from 20 0 e to O°C,
condition ofpressure, temperature and volume at which the reaction q=msflT
is carried out. The variation of heat ofreaction with temperature and 500 x 4.18 x 20 = 41800 J = 41.8 kJ
pressure is given by Kirchhoff's equation: Number of moles of water (ice) that will melt to absorb 41,8 kJ
_-"-_--'- == flC p flU 2 - flU! ==,flC =41.8", 7
v
T2 11 T2 -11 6.02
(At constant pressure) (At constant volume) :. Number of cubes of ice that will'melt = 7]
Answer the following questions: 6. The enthalpy change (M) for the reaction,
1. Match the List-I with List-II and select the answer from the N2 (g) + 3H2 (g) ---7 2NH3(g)
given codes: is -92.38 kJ at 298 K. The internal energy change flU at 298 K
List-I List-II is: ' ( A l I M S 2006)
(a) -92.38 kJ (b) -87.42 kJ (c) -97.34 kJ (d) -89.9 kJ
A. C(s)+ 02(g) CO2 (g) 1. M = flU + RT
[Hint: 2- 4 - 2
B. N 2 (g)+3H z(g) 2NH3(g) 2. M=flU
C. NH4HS(s) NH3 (g ) + H 2 S(g) 3. M = flU - 2RT 6H = flU + AngRT
flU 6H - flng RT
D. PCI 5 (g) PCI 3(g) + CI 2 (g) 4, M = flU + 2RT
92.38 x 1000 - (-2) x 8.314 x 298
E. 2S02 (g)+ 02(g) 2S03 (g) 5. M = flU - RT
87424 J = - 87.424 kJ]
Codes: ABC D E 7: The specific heat of 12 in vapour and solid state are 0.031 and
(a) I 2 3 4 5 0.055 caVg respectively, The heat of sublimation of iodine at
(b) 5 2 3 4 I e
200 0 is 6.096 kcal mol-I. The heat of sublinlation offodine
(c) I 3 4 2 5 at 250°C will be:
(d) 2 3 ;4 5 (a) 3.8 kcal mol- l (b) 4,8 kcal mol- 1
2. The heat capacity of a bomb calorimeter is 500JIK. When (c) 2.28 kcal mol-I (d) 5.8 kcal mol-I
0.1 g of methane was burnt in this calorimeter, the temperature [Hint: 12 (s) ~ 12 (g)
rose by 2°e. The value of flU per mole will be: Aep (0.031- 0.055) x 10-3 x 254
(a) +1 kJ, (b)-l kJ (c)+l60kJ (d)-160kJ
3. For which reaction will M = flU? Assume each reaction is -'-"'----" =flCp
Tz Tl
carried out in an open container:
(a) 2CO(g) + 02(g) 2C02 (g) --,--",--_6,_09_6 = (0,031 _ 0.055) x 10-3 x 254
50
(b) H2 (g) + Br2(g) 2HBr(g)
6H z = 5.79 kcal mol-!]
(c) C(s) + 2H zO(g) 2H 2 (g) + CO2 (g) ,
(d) PCIs(g) PCI 3(g) + CI 2 (g)

••
512 G.R.B. PHYSICAL CHEMISTRY FOR COMPETITIONS

• Passage 2 •.p.~~ge3. .. ... . .


" 'iWe chahge' In Gibbs free jen~rgy (t.G) of tlie sy;tem alone
I' :

In a chemical reaction, the bonds of reactants are decomp~;ed


and new bonds of products are formed. The amount of energy provides a criterion for the spontaneity of a proeess at constant
required to break a particular bond in a gaseous molecule under temperature and pressure. A change in the free energy of a system at
standard conditions homolytically is called the standard bond constant temperature and pressure will be:
dissociation enthalpy ofthat bond (AH~ _ B ).
t.GSYSlem =: AH,ystem - T D.S system
At constant temperature and pressure:
Bond energies can be used to obtain an approximate value for a
t.Gsystem < 0 (spontaneous)
reaction enthalpy of a gas phase reaction if the appropriate
enthalpies offormation are not available. t. Gsystem. =: 0 ( equilibrium)
o
AHreaetion LBond energy ofbonds, broken in the reactants t.Gsystem > 0 (non -spontaneous)
- L Bond energy of bonds, formed in the products . Free energy is related to the equilibrium constant, as:
Bond energy depends on three factors: t.Go 2.303RT loglo Ke
(a) Greater is the bond length, lesser is the bond energy. Answer the fonowing questions:
(b) Bond energy increases with the bond. multiplicity. 1. The free energy for a reaction having AH = 31400 cal,
(c) l!o,nd energy increases with the electronegativity difforence 1
D.S 32 cal K-1mol- at 1000°C is: [JEE (Orissa) 2005J
between the bonding atoms.
(a)-9336 cal (b)-7386 cal (c)-1936cal (d)+9336cal·
Answer the following questions:
1.· Arrange N-H, O--H and F-H bonds in the decreasing order 2. For a spontaneous .reaction t.G, equilibrium 'K' and E~u will
of bond energy: be respectively: (AIEEE 2005)
(a) F-H > O-H > N-H (b) O-H > N-H > F~H (a) -ve, > I, +ve (b) +ve, > I, -ve
(c) N-H > O-H > F-H (d) F-H> 1\-H > O~H (c) -ve, < I, -ve (d) -ve, > I, -ve
2. X 2 represents halogen molecule. Bond energy of different 3. For a system in equilibrium, I1G = 0, under conditions of
halogen molecules wiIllie in following sequences: constant.. ..... : (KCET 2005)
(a) F2 >C1 2 > Br2 > 12 (b) Cl 2 > Br2 > F2 > 12 (a) temperature and pressure (b) temperature and volume
(c) pressure and volume . (d) energy and volume
(c) 12 >C1 2 >Brz >12 (d)Br2 >F2 >12 >Cl z
[Hint: Bond energy of F2 is surprisingly low due to strong 4. If both AH and D.S are negative, the reaction will be
spontaneous:
repulsion between the lone pairs of two fluorine atoms.]
(a) at high temperature (b) at low temperature
3. Which among the following sequences is correct about the (c) at all temperatures (d) at absolute zero
bond energy of C-C, C=C and C=-=C bonds?
5. A reaction has positive values. of Mi and t.S. From this you
(a)C C>C C>C-C{b)C=-= C<C = C<C-C can deduce that the reaction:
(c)C = C>C=-= C>C-C{d)C =-=C>C-C>C = C (a) must be spontaneous at any temperature
4. InCij4 molecule, which Clfthe following statements is correct (b) cannot be spontaneous at any temperature
about the C-H bond energy? (c) will be spontaneous only at low temperature
(a) All C-H bonds of methane have same energy (d) will be spontaneous only at high temperature
(b) Average of all G-H bond energies is considered 6. For a reaction to be spontaneous at all temperatures:
(c) Fourth C-H bond requires highest energy to break (a) t.G - ve, AH + ve and D.S + ve
(d) None ofthe above . (b) t.G + ve, AH - ve and t.s + ve
5. Use the bond energies to estimate AH for this reaction: (c) t.G - ve, AH ve and D.S -" ve
Hz(g) + 02(g) ---t H Z02(g) (d) t.G - ve, AH - ve and D.S + ve
3 moles of CO2 gas expands isothermally against external
Bond Bond energy pressure of 1 bar. Volume increases from 10 L to 30 L respectively.
H-H 436 kJmol- 1 The system is in thermal contact of surroundings at temperature
15°C. Entropy change in Isothermal process is:
0-0 142 kJ mol- 1
0=0 499 kJ mol-I D.S.=: 2.303
.
nR log Vi.
(V2)
H-O 1
460 kJmol-
Answer the following questions:
(a)-127kJ (b)-209kJ (c)-484kJ (d)-841kJ
7. If CO 2 behaves like an ideal gas, ihen,entropy change of
6. The heat of formation of NO from its elements is +90 kJ mol-I.
What is the approximate bond dissociation energy of the bond system (D.S system) will be:
in NO? (a) +27.4 J K-1 (b) 9.1jK- 1
BE NaN = 941kJ mol- l BEo=o = 499 kJ mor l (c) -27.4J K- 1 (d) -9.\ J K- 1
(a) 630 kJ mol- 1 (b) 720 kJ mol- 1 8. Select the correct relation:
(c) 760 kJ mol- 1 (d) 810 kJ mol- 1 (a) t.Ssystem > 0, D.Ssurr. = 0 (b) t.S surr.< O,t.Ssystem > 0
(c) t.s system 0, D.S SWT. 0 (d) D.S SWT. > 0, t.S system < 0
CHEMICAL THERMODYNAMICS AND THERMOCHEMISTRY 513

• Passage 4 4. The process which occurs in going from, B ~ Cis:


Consider the following energy level diagram:, (a) isothermal (b) adiabatic (c) isobaric (d) isochoric
5. The pressUres at A and B in the atmosphere are respectively:
6C(s) + 6H2 (g) + 902 (g)
o x
(a) 0.821 and 1.642 (b) 1.642 and 0.821
(c)l and 2 (d) 0.082 and 0.164
, C(yH120 6 + 602(g)
• Passage 6
The thermodynamic property that measures the extent of
Energy Y Z molecular disorder is called entropy. The direction ofa spontaneous
process for which the energy is constant is always the one that
i 6C02(g) + 6H20(1) increases the molecular disorder. Entropy change of phase
transformation can be calculated using Trouton ~ formula

( a;J.
\ '

Answer the following qnestions on the basis of the given IlS = In the reversible adiabatic process, however, ilS will
diagram:
1. The heat of formation of glucose is: be zero. The rise in temperature in isobaric and isoc/Joric conditions
(a)-x (b)-y (c)x-y (d)-x+z is found to increase the randomness or entropy ofthe ,frstem.
2. In the given diagram z refers to: IlS = 2.303C log (1j / T2 )
(a) 6 x Ml fco2 (b) ANf~H IP6 ' C=Cp orCv
(c) AlI'::ombustion ~H rA f H P(I)
(d) AN':.orrIJustion C{s) +l:Jf Answer the following questions:
1. The entropy change in an adiabatic process is:
3. The quantity y is equal to:
(a) zero
(a) ANcorrIJustionC{s) + ANcombustion H 2(g) (b) always positive
(b) x + z (c) always negative
(c) x - z (d) sometimes positive and sometimes negative
(d) AN/ CO2 + AliHp , 2. If, water in an insulated vessel at-lOOC, suddenly freezes, the
4. Select the incorrect statement(s): entropy change of the system will be: I

(a) combustion of glucose is exothermic process


(a) +10 J K- 1 mol-I .
(b) standard state of glucose is C6HI2 0 6 (s)
(b) -lOJ K- 1 mol-I
(c) heat of fonnation of glucose:: Heat of combustion of
glucose (c) zero
(d)x-i- y= z (d) equal to that of surroundings
3. The melting point of a solid is 300 K and its latent heat of

,'
• Passage 5 fullion is 600 cal mol-I. The entropy change for the fusion of 1
mole of the solid (in cal K -I) at the same temperature would
be:
20 L '--:-----:--A ~' (a) 200 (b) 2 (c) 0.2 (d) 20
i
~'
~
,

10 L --------- V
c:
I
I
I
,
,
' I

!:
I
I
I
I
I
4: For which of the following cases IlS ==

(a) A process for which ACp = Obllt ACI' == 0


(b) An adiabatic process
(c) An isobaric or isothermal process.
AN

?

(d) An isothermal reversible' phase transition process


200K 400K 5. When 1 mol of an ideal gas is· compressed to half of its
Temperature - volume, its temperature becomes double; then the change in
entropy (1lS ) would be: •
Graph for one mole gas
(a)Cv In 2 (b)C p In 2
Ans.wer the following questions based on the above diagram: (c)Cv Rln2 (d)(Cv R)ln2xCp
1. Process, A ~ B represents:
(a) isobaric (b) isochoric (c) isothermal (d) adiabatic [Hint: 6.S == 2.303C p log 1o (~J
2. The pressure at Cis:
(a) 3.284 atm (b) 1.642 atm (c) 0.0821 atm(d) 0.821 atm
3. Work one in the process C ~ A is:
C 1n(Vl)=C
p
V
10.,(,_1
p
1/2
)
2
(a) zero (b) 8.21 L atm =C p 1o.,2]
(c) 16.2 L atm (d) unpredictable
514 G.R.B. PHYSICAL CHEMISTRY FOR COMPETITIONS

• Passage 7 The net work done during the com~ .. cycle is given by the
area:
The pressure-volume behaviour of various thermodynamic
(a) cycle ACBDA
processes is shown in graphs~'
(b) AA1B1BDA
(c) AA2B2B
(d) half of area bounded by curve
r------- Isobaric 3. P - V plots for two gases during adiabatic processes are given
1 in the given figure:
Isothermal

Isochoric
Adiabatic
1
Volume -
Work is the mode oftransference ofenergy. If the system involves
gaseous substance and there is difference ofpressure between system
and surroundings, such a work is referred to as pressure-volume
work (wpv = - Pext b.V). It has been observed that reversible work Volume -
done by the system is the maximum obtainable work. Plot A and plot B should correspond to:
wrev > wirr (a) He and 02 (b)HeandAr (c) O2 and He (d)02andF2
[Hint: Slope of the adiabatic curve oc y
The works of isothermal and adiabatic processes are different
from each other. Slope of B > Slope of A
He (y =:: 1.66); O 2 (y =:: 1.44)
W isotherm,,1 reversible - = 2.303nRT logw (~)
"2.303nRT loglO (V2) - Thus, correct answer will be (c).]
, ,~ ~ 4. The q value and work done in isothennal reversible expansion
W"diabatic reversible =Cv (11 - T2 ) of one mole of an ideal gas from initial pressure of 1 bar to
final pressure of 0.1 bar at constant temperature 273 K are:
Answer the following questions:
(a) 5.22 kJ, -5.22 kJ (b) -5.22 kJ, 5.22 kJ
1. If WI' W2' W3 and w4 are work done in isothermal, adiabatic,
(c) 5.22 kJ, 5.22 kJ (d) -5.22 kJ, -5.22 kJ
isobaric and isochoric reversible processes, then the correct
sequence (for expansion) would be: [Hint: w:: 2.303nRT log (~)
(a) WI > W2 > w3 > w 4 (b) w3 > w2> WI > w 4
(c) w3 > w2 > W4 > WI (d) w3 > WI > w2 > W4 = - 2.303 x I x 8.314 x 273 log (~)
,[Hint: W = Work done = Area undercurve.] OJ
2. A thennodynamic system goes in acyclic process as = 5.227 kJ
represented in the following P-V diagf~_ q =- w =+ 5.227 kJ for isothermal process]
5. Calculate work done when 1 mole of an ideal gas is expanded
reversibly from 20 L to 40 L at a constant temperature of
300K.
(a)7.78kJ (b)-1.73kJ (c) 11.73kJ (d)-4.78kJ

.
CHEMICAL THERMODYNAMICS AND THERMOCHEMISTRY 515

Passage 1. 1. (d) 2. (d) "'3. (b) 4. (c) 5. (b) " 6. (b) 7. (d)
Passage2~ 1. (a) 2~ (b) 3. (a) 4. (b) 5. (a) 6. (a)
-.- .......
Passage 3. 1. (a) 2. (a) 3. (a) 4. (c) 5. (d) 6. (d) .. 7. (a) 8.' (b)
Passage 4. 1. (a) 2~ ,(c) 3. (b) 4. (c, d)
PassageS. 1. (b) 2~ (b) . 3. (b) 4. (c) 5. (a)
Passage 6. 1. (a) 2. (c) 3. (b) 4. (d) 5~ (b)
Passage 7. 1. (d) 2. (a) ,3•. (c) 4. (a) 5. (b)

~.
'SEtFAssESSMENT
~, '\.,. ':','. ,,,
.' ', C~
(:p .'
~ ..
ASSIGNMENT NO.7

5. For a process to be in equilibrium, it is necessary that:


(a) aSsyaem = IlS surr, (b) llS.ystem -assurr. =
Straight Objective Type Questions ,. ( c) as system = 0 (d) as surr, 0
This section contains 10 :multipie choice questions. Each 6. Predict the sign of IlS for each of the following processes,
question has 4 choices (il), (b), (c) and (d), ,()ut of w~ich only which occur at constant temperature:
one is correct.' '. , .. <,.,' ;. '
I. The volume of2 moI?f0 2 (g) increases from 44L to 54L
1. Reaction of silica with mineral ~cids' may be given as: II. The pressure of 2 mol of 02 (g) increases from I' atm to
Si02 + 4HF Sif4 + 2H26; . Ml:i; -10:17 kcal 1.2atm.
'Si0 2 + 4HCI ~ SiCl 4 +2HiO; Mf = + 36.7 kcal I n
which among the foHowing is correct? (a)~S == - ve IlS ve =-
(a) HF and HCI both will react with silica (b) IlS = - ve as + ve =
(b) Only HF will react with silica (c) IlS = +ve IlS -ve =
, (c) Only HCI will react with silica (d) IlS = + ve ' as ::; + ve
(d) Neither HI" nor HClwill react with silica 7. Which of the tollowing statements must be true for the entropy
[Hint: Exothermic reactions are spontaneous] of a pure solid to be zero?
2. In Mayer's relation, I. The temperature must be zero kelvin
Cp -C~ R II. The solid must be perfectly crystalline;,
'R' stands for: III. The solid must be an element
(a) translational kinetic energy of 1 mol gas IV. The solid must be ionic
(b) rotational kinetic energy of 1 mol gas (a) I (b) I and II
(c) vibrational.kipet:i~ energy-ofl mol gas (c) I, II andllI 'i, (d) All'arecorrect
(d) work done to increase the teolperature of J mol gas by 8. Which of the following statements is correct?
one degree (a) Slope of adiabatic. P -Y curve is smaller than that in
[Hint: '" PV (ForfmolgaS) ,.. (i) isothermal one
P(V + ,aV) f.?(T tJ) ... (ii) (b)Slope of adiabatic P - V curve will be sanle as that in
From eqs. (i)al1d(ii) isothermal one '
paV,=.R (c) Slope of adiabatic P - V curve will be larger than in.
, isothermal one '
i.e., W =R (work d()ne)}
(d) S~ope of adiabatic P - V curve will b~zero
3.· FOl: an ideal gas, the.Joule-Thomson coefficient is:'
, (a) z e r o ( b } positive
(c) negative (d) depends on atomicity of gas 9. (iJB)
iJp T
'for an ideal gas is equal to: '
" ..
4. Entrofl.Y change in reversible adiabatic process is:
(a) infinite (b) zero
(a) zero (b) aVRT (c) P~V (d) nRaT
P T
(c) equal to CvaT (d) equal to nR In (V21
VlJ
516 G.R.B. PHYSICAL CHEMISTRY FOR COMPETITIONS

10. A refrigerator is used to remove heat from enclosure at 0° C at


. '.' the TateOC 60()watf. I1tliesuUoundings temperature,is' 30° C, ..
"'SECTION-m
calculate tlle power needed: . . ....... . .' . .. . Assertion-Reason Type Question~
(a}:303-watt' ... . (b) llOOOwatt This section contains 5 questions. Each question contains
(c) 65.9 watt. (d) 110 watt . Statement-l (Assertion) and Statement-2 (Reason): Each
question has following 4 choices (a), (b), (c) and (d), out of
which only one is correct. - ."
SEc·nON-1i (a) Statement-l is true; statement-2 is true; statement·2 is a
Multiple Answers Type Objective Questions correct explanation for statement-I. . .
11. flU will be zero for which processes? (b)Statement-1 is true; statement-2 is true; statement~2 is not a
correct explanation for statement-I. .
(a) Cyclic process (b) Isothermalexpansion
(c) lsochoric process (d) Adiabatic process (c) Statement-l is true; statement-2 is false.
12. If x and yare two intensive variables then: (d) Statement-l is false; statement-2 is true.
19. Statement-I: Most of the combustion reactions' are
(a) xy is an intensive variable
exothermic.
(b) ~ is an intensive variable Beeause
y
1~H~~.>f~:.;#;:1i ~':':;;.':~i.~ Sfjt~: Products are more stable than reactants in
(c) (x + .y )is an intensive variable ;; ~.!.. ,.""~~~.;.,...,,,,,,~:..... :;,,,,,,... ,~. e,{otlternu~process.

(d) dx is an extensive pro~rty 20. Statement-I: There is no exchange in internal energy in a


dy ... cyclic process.
13. Which of the following expressions is/are correct for an Beeause
adiabatic process? Statement-2: In, II "p;¥~~. sRr~cr.ss, the system returns to
original state in a h'liiiibet.6f-steps-l .
(a) T2 =( f\)Y I 21. Statement-I: The value of enthalpy of neutralization of weak
1j V2 acid and strong base is always numerically l.essthan 57.1 kJ.
(c) P2V2Y = ~v. Y Betause
Statement-2: All the OH- ions furnished by I gram
14. Seleet the state functions among the following:
equivalent of strong base are not completely neutralized.
(a) temperature i' "(b) entropy
22. statement-I: Heat of solution is positive when
(c) work (d) enthalpy
Na ~04·1 OH20 is dissolved in water put it is negative when
15. Select the correet expressions among the following: anhydrous CUS04 is dissolved in water.
(a) flG - Mf :::: (.aflG) (b) flG- Mf =[a(flG)] Beeause
T aT p T aT v Statement-2: MoiarmassesofNa2S04.10H20andCuS04 are
(c) flS : : (aEeel!) (d) (aT) =_(aH) . different.
nF . aT p ap H . ap rlCp 23. Statement-I: The extensive pro~rty ofa single pure
substance dependS upon the numb~ of moles of the substance
16. Which of the following are correct for an ideal gas? .
present.
(a) (au)
av r =0 .
(b) (aH)
ap r =0
Beeause
Statement-2: Any extensive property expressed ~r mole
(c) (aT) =0 (d) (dP) =0
becomes' intensive.
ap H aT v
:~:~:S(i;:)c~ap~: eq[~a~o~ m]a
y SECTION-IV
17. be given as:
Matrix-Matching Type Questions'
~ 2.303R 11 1; This section contains 3 questions. Each question contains
(b) dP
= flS statement given in two columns which have to be matched.
c
Statements (a, b, and d) in Column-I have to be matchedwitb
dT flV
dP q statements (p, q, r and s) in Column-II. The answers to these
(c) dT= TflV . questions have to be appropriately bubbled as illustrated in the
following examples: .
. (d) dP flV = If the correct matches are (a-p,s); (b-q,r); (c-p,q) and (d-s);
dT flS then correct bubbled 4 x 4 ~atrix should be as follows:
18. Which of the following is/are not state function?
(a) q (b) q - w
(c)!. (d) q + w
w
CHEMICAL THERMODYNAMICS AN~"tfi'~RMOCHEMISTRY
:;, ~. ~ -'" ..
I 517
p q r s 25. Match the Li:t-I wi~
List-ll and select the correct, answer

a
e®0 e ,from the given codes:
List-I List-II
b

d
®e 0
ee0 0
®®0
•e (Thermodynamic properties)
(I)~G

(II) Afr

(III)M
(N)AG o
'
(Expression) .
(A)- RT loge K
(B)RT 2 (d In

(C)nFE
. (D)_[dAG] .
dT. /"
K) .

oT /'
List-I List-II Codes:
(a) I-C,II-B, III-D, IV-A
(a) (dT) (P) V
dP H, (b) I-B, II-C, III-D, IV-A
(c) I-A, II-B, III-C, IV-D
(b) (dG) (q) T (d) 1-0, II-A, III-B, IV-C
dP T 26. Match the List-I with List-II:
(c) (dH)
. " , ' ......,">.
-. > . ".-'
(r)-S 'Ust~1I
dS /' List-I
(a) Perfectly crystaliine solid (P) AU =()
(d) (dG) . I(S)~jT
dT /' (b) Reversible reaction at (q) T = constant
equilibrium
( c) Isothermal process (r) Lim S --+ ()
T~OK

(d) (dG)
dP = -S
, . . (s) Muniv~ = 0

. ,., "......
[~ ,.---~----------I
~, ., ... ·.:'i"~'''''';''·----~-----------------------.......
1. (b) 2. (d) 3. (a) 4. (b) 5. (b) 6. (c) 7. (b) 8. (c)
9. (a) 10. (c) 11. (a, b) 12. (a, b, d) 13. (a, c) 14. (a, b, d) 15. (a, c, d) 16. (a, b, c)
17. (a,b, c) 18. (a, b, c) 19. (a) 20. (a) 21. (c) 22. (c) 23. (b)
24. (a-s) (b-p) (c-q) (d~r) 25. (a) 26. (a-r) (b-s) (c-p, q) (d-q)

/
INTRODUCTION (iii) Neutralisation of an acid with a base when their aqueous
solutions are mixed.
Thermodynamics deals with the feasibility of a chemical change.
The fre~ energy change, aG, of a reaction helps us to understand HCI + NaOH ----*NaCI + H 20
Acid Base Salt
whether the reaction will occur or not. Even though there may be (b) Very slow reactions: There are certain reactions which
decrease in free energy but reactants do not always form the are extremely slow. They may take months together to· show any
products instantaneously and actual rate of the reaction may vary measurable change at room temperature. It is also difficult to
from extremely slow to very fast. Thermodynamics is concerned study the kinetics of such reactions.
only with initial and final states of reacting systems but offers no Some examples are:
explanation about the various stages through which the reactants (i) Reaction between hydrogen and oxygen at room
pass to reach the final state. This leads to following questions temperature.
concerning chemical changes. Room temperature
(i) How fast do the chemical reactions go? 2H2 + O 2 ) 2H 20 (Very slow reaction)
(ii) How can the speed of the reaction change? (ii) Reaction of atmospheric H 2S on basic lead acetate.
(iii) What intermediate steps, the reactants follow to reach the Atmospheric
final state of products, i.e., the mechanism of the reaction. White basic lead acetate paint -----» Blackening
The branch of physical chemistry which deals with the rate at H2 S
which the chemical reactions occur, the mechanism by which the of paint occurs very slowly
chemical reactions take place and the influence of various factors (due to formation of PbS.)
such as concentration, temperature, pressure, catalyst, etc., on the (iii) Reaction between carbon and oxygen.
reaction rates, is called the chemical kinetics.
Room temperature
Different ,chemical reactions occur at different rates. On the C+02 ) CO 2
basis of rates, the chemical reactions are broadly divided into
three categories: Carbon and oxygen are thermodynamically less stable than
(8) Very fast or instantaneous reactions: These reactions CO 2 at 298 K, yet coke does not spontaneously catch fire in air
are so fast that they occur as soon as the reactants' are brought and remains unreacted even for years.
together. Generally, these reactions involve ionic species and thus (iv) Reaction between carbon monoxide and hydrogen.
known as ionic reactions. These reactions take about 10- 14 to Room temperature
10-16 seconds for completion. It is almost impossible to CO+ 2H2 ----~) CH)OH
determine the rates of these reactions. Some such examples are:
The reaction is thermodynamically feasible at 298 K but in
(i) Precipitation of AgCI when solutions of silver nitrate and
actual practice the reaction proceeds infinitesimally slowly;
sodium chloride are mixed.
(v) Rusting of iron occurs very slowly.
AgN0 3 + NaCl ----* AgCI + NaN0 3 (c) Moderate reactions: Between the above two extremes,
(ii) Precipitation ofBaS04 when solutions of barium chloride there are a number of reactions which take place at moderate and
and sulphuric acid are mixed. . measurable rates at room temperature and it is these reactions
which are studied in chemical kinetics. Mostly these reactions are
BaC1 2 + H 2S04 ----*BaS04 +2HCl
,- .,;

CHEMICAL KINETICS 519

molecular in natl:lre. Some. common examples of s4<:h type ,are destination. The odometer reading at destination is y miles. The·
given below: rate of his travel can be calculated as: ~~
(i) Decomposition of hydrogen peroxide.
2H2 O 2 ---t 2H20 + O 2
Rate =d( distance) =---""-'------
distance (j) - distance (in)
d(time) time(j) - time(in)
(ii) Decomposition of nitrogen pentoxide.
=
y-x 'I hr-I
y-x =--mtes .
2N 20 S ---t 2N 20 4 + O 2 11.0- 9.0 2
(iii) Hydrolysis of an ester. . The above example indicates that the car hns been driven with
CH 3 COOC 2Hs + NaOH ---t CH3 COONa + C 2HsOH unifonn rate but actually it has been driven sometimes faster and
Ester Sodium acetate sometimes slower depending upon the conditions of the road.
(iv) Inversion of cane sugar in aqueous solution. Thus, the overall rate is an average rate and the rate at which the
car was moving at any instant, i. e., instantaneous rate was
C 12 H22 0 11 + H 20---t C 6 H I2 0 6 + C 6 H I2 0 6 changeable.
Cane sugar Glucose Fructose
Tbe rate measured over a long time interval is called
(v) Reaction between nitrogen dioxide and carbon monoxide. average rate and tbe rate measured for an infinitesimally
N0 2 + CO ---t NO + CO2 small time interval is called instantaneous rate.
(vi) Reaction between ferric chloride and stannous chloride. In a chemical change, reactants and products are involved. As
the chemical reaction proceeds, the concentration of the reactants·
2FeCI 3 (aq.) + SnCI 2 (aq.) ---t 2FeCl 2 (aq.) + SnCI 4 (aq.) decreases, i. e., reactants are consumed and the concentration of
(vii) Decolourisation of acidified potassium pennanganate products increases, i. e., products are produced. Tbe rate of
with sodium oxalate. reaction (average rate) is defined as tbe cbange .of
(viii) Reaction between nitric oxide and chlorine. concentration of anyone of its reactants (or products) per
NO+ Cl 2 ---tNOC1 2 unit time.
Average rate of reaction,
The chemical reactions can be slowed down or speeded up by
Change of concentration of one of the reactants or products
changing conditions under which they occur. For example, very ~v =-~~--------------~---
slow reaction, CO + 2H2 ---t CH 3 0H, can be speeded up by Time taken for the change
maintaining temperature around 400°C, pressure about 300 Consider a common hypothetical reaction,
atmospheres and using a catalyst containing ZnO and Cr2 3 , The ° A~B
decay of food articles can be slowed down by preserving them in
The average rate of reaction may be expressed either in tenns'
refrigerators. There are two principal reasons for studying
of decrease in concentration of A (reactants) or in tenus of
chemical kinetics. .
increase in concentration of B (product).
(i) To predict tbe rate of a particular reaction under
· Decrease in concentration of A
specified conditions: The conditions can be adjusted to make Average rate 0 f reactIOn =------,-.- - - - - - -
the reaction to go at a desired rate, either rapidly or slowly or Time taken
moderately. The field of chemical kinetics is useful in industry as · concentration of B
the conditions for maximum yields of industrial products can be or Average rate 0 f reactIOn =Increase inTime taken .
.
ascertained.
(ii) To predict the mechanism of tbe reaction: The The concentrations of reactants or products are expressed as
intelligent guess regarding various elementary processes number of moles per litre. The symbol for the molar
responsible for the formation of products can be made which concentration consists of the fonnula of the substance enclosed in
should be consistent with experimental data, bracket. The symbol [A] stands for the molar concentration of A.
The symbol d[A], therefore, stands for the change in molar
8.2 RATE OF REACTION (Average and concentration of A.
The rate of decrease or increase in the concentration of
Instantaneous Rate)
reactants or products may also be expressed in terms of change in
We are all familiar with processes in which something changes their concentration during the time interval, M, as:
with time. Rate is usually expressed as the ratio of the amount of
change in some quantity to the time required to produce that Average rate of reaction = _ d[A]
M
change.
Change in some quantity t:.X or ' Average rate of reaction = + d[ B]
Rate = =- M
Time taken for the change !1t
(A negative sign placed before a reaction rate symbol signifies
The tenn t:.X means X final - X initial and !1t is the amount of a decrease in concentration of tile reactant with increase of time
time elapsed. For example, a car driver starts his journey at 9.0 and a positive sign before the rate symbol signifies that the
a.m. with odometer reading x miles. At 11.0 a.m. he reaches his concentration of the product increases with increase in time. The
520 I, G.R.B. PHYSICAL CHEMISTRY FOR GpMPETITIONS
"'" '

_---[8]
" :.::::::_SOME SOLVED EXAMPLES\a:::,:::,
Example 1. Ina reaction, the concentration of a reactant
(A) changes from 0.200 mol litre-I to 0.150 molUtre- 1 in 10
i minutes. What is the average rate of reaction during this
interval?
Solution: A[A] = [A]flnal- [A]initial
=[0.150- 0.200] = ":'O~050mollitre-1
At = 10 minutes
,[A]
, 'f " . -A[A] -[- 0.050]
Average rate 0 reaction ='-'- =~--...::
, At 10
Time - = 0.050 0.005 mol litre -\ min-I
"
,,
\1"1.J~J'" n"
',Fi'g: 8:1
~: ~·<;·.r,·
' 10
concentration"change may. be PQsitive or negative but therate,of Expression of Rate
reaction is .f!.l~ays positive. The minus sign ,is, always written Consider the following reaction between CO and N02 :
whenrequi~jbut the plus sign is usually not mentioned.)
Figure 8.1 shows thech~l1ge of concentrations of reactant (A) , CO+N02 ~C02 +NO
and product (B -) ~s the chemiclll reaction A -4 B progresses with The, equation shows that when one mole of CO reacts with one
time, Let [A]II aQd [A]IZ ,be the, concentrations of A at. time mole of N0 2 • one mole each of CO2 and NO are formed; The
I, and t2,re~ectively. " ' ',", ',,' average rate of reaction can, beexpres~d either by decrease of
concentration of anyone ofthe reactants (CO or N02) or by the
A[A] = [ALz ,- [A]II increase in concentration of anyone of the products (C02 or
,and ,At == t2 :- I, , NO);
, A[A] [A112 - [A]II Thus _ AlCO] = _ A[N02']=A[C02 ] = A[NO] ,
r. =- - - = - ~'---'=-----.!.. , At At At At,
av At ' t 2 -t,
However, for the reaction,
Over the same time interval t, to 12 , the concentration of B 2H202'~ 2H 2 0 + O 2
changes from[B]11 to[BJI2' it is observed that when 2 nlOles of H 2 0 2 decompose, one,
, A[B] [R]IZ - [B]II' mole of O 2 is formed in the same time interval. The rate of
r. = - - =--=---..;... increase in the concentration of O 2 , therefore, is half that of the
av At "t2 - I, ,"
disappearance of the ,concentration of H2 O2 in the same time
Since, 'one mole of A produces one mole of B, the rate of in~erval; ,
decrease in concentration of A should be equal to the rate of
increase in concentration of B. Thus, '
ray = r;v ,In general, for a reaction,
A[A] A[B]
---=-- n,A +n2B ~m!C+m2D
At At
the rate expression may be expressed as:
Reaction rate has' the units of concentration or 'molarity
1 A[A] 1 MB} 1 A[e] I' &[D]
divided by time. Therefore; the units ohate of reaction may be - --=---- ----=---
expressed as: ' nl At nz At m, At mz At
rinlellitre sec (mol L-, s-')
Thus, for the reaction,
imlellitre min (rilOl L-' min:-') H2 +I z ~ 2HI
or molellitre hour(mol L -I, h -I,) the rate may be expressed as:
or molellitre day (Illol L-I d -I) A[H 2 ] , A[1 2 ] 1
---=---==-
or molellitre year (mol L-' y-') At At 2At

Note: In case of gaseous reaction, pressure may be taken in place of concentration. Thus, rate will have the unit of attn sec-lor attn min -I,
PV =nRT ,or P =[fJRT or P=CRT '

.1[P] = .1[Cl. RT or Rate in [a~sec]= Rate in [;';'olarity/sec] xRT


.1t .11
I
CHEMICAL KINETICS. 521

Similarly, for the decomposition of N 20 S in CCl 4 medium, Experimental Determination of Instantaneous Rate of
the rate may be expressed as: Reaction
2N 2 0 S ~ 4N0 2 + O 2 \ In order to determine changes in concentration of reactants or
1 A[N20 s ] l' A[N0 2 ] A[02 1 products, it is customary to take small portions of the reaction
-- =- =-- mixture at suitable intervals of time and freeze them rapidly to
2 At 4 At At
about O°C as to stop the reaction. The concentration is then
Example.:!. Decomposition of N 20S is expressed by the measured with the help of a suitable method. In several cases,
equation, concentration changes are measured by observing changes in
N 20 S ~2N02 + X02 certain physical properties which are proportional to it such as
q during a certain time interval, the rate ofdecomposition of optical densities, electrical conductivity, optical rotation, etc. A
N 2 0 5 is 1.8 X 10-3 mol litre-I min-I, what will be the rates of curve is plotted between concentration and time. A tangent is
formation ofN02 and02 during the same inierval? . drawn to the curve at the point corresponding to time interval' t'.
Solution:. The rate expression for the decomposition ofN 2Os is: The slope of this tangent gives the instantaneous rate of reaction.
This is shown in Fig. 8.2 (a). .'
_ A[N 20sl =1 A[N02 ] = 2. A[02l Instantaneous rate of reaction =Slope of curve
At At 2 At
Intercept along ordinate Ax
So, A[N0 2 l = 2A[N 20s1 =2x1.8 x 10-3 = Intercept along abscissa =-
At
At At
= 3.6 X 10-3 mol litre -I min-I Since, the concentrations of the reactants keep on decreasing
with time, the r.ate of reaction correspondingly decreases with
and A[02l =.!. A[N 20 s ] =.!. x 1.8 X 10- 3 time. Thus, the rate of reaction will depend on the stage
At 2 At 2
considered during progress of the reaction. The rate of reaction is
=0.9 x 10-3 mol litre -I
min-\
maximum at the . initial stage and decreases with time.
'. A[NO]
(Rate is always positive and hence - 2 . s is taken Theoretically, infinite time wpuld be required for a reaction to
At complete. But the reaction rate becomes so slow after a certain
positive.) time that for all practical purposes, the reaction can be considered
to be completed. It is evident from Fig. 8.2 (b) that the rate of
Instantaneous Rate reaction is varying from'moment to moment.
In chemical kinetics, the rate at any particular instant, i. e. ,
instantaneous rate rather than the average rate over a time interval
has much more practical application and importance. It is defined
as the rate of cbange of concentration of anyone of the
reactants or products over a very small interval of time.
If we take infinitesimal small interval of time dt, it may be
assumed that the rate is uniform through this interval; then if dx is
amount of substance A transformed to B during this interval, the
rate of reaction at that instant is given as _ dx .
dt
[In differential calculus, when At becomes very small and
-t- Time -
Fig.8.2(a) Determination
Time --.-+

Fig.8.2(b) Variation of rate


A[A] . of rate of reaction
approaches zero, the ratio may be replaced by the
At Reaction Life Time
. . d[A].
denvatlve, - - , I. e. , It is defined as the time .taken by a reaction to proceed to 98%
dt of completion. The shorter the life time, the faster the reaction.
r, = lim _ A[A] =:... d[A]] Reaction life times are used to compare the various reactions.
mst At ~O At dt Reactions are also compared with half life periods. The half life
On the other hand, if the rate of reaction is expressed in terms period is defined as the time during which the concentration of a
of the concentration of any of the products which goes on reactant is reduced to one-half of its initial value or the time in
dx which half ofthe reaction is completed. It is generally denoted by
increasing, then rate of reaction at particular instant t l / 2 • The shorter the half life period, the faster is the reaction.
dt
In general, for a reaction, Example 3. For each of the following reactions express
nlA +n2B~mlC+mzD the given rate of change of concentration of the reactant or
The instantaneous rate may be expressed as~ product in terms of the rate of change of concentration of other
reactants or products in that reaction:
1 d [A] 1 d [B] 1 d[C] I d[D]
'inst ----=- - - : - - - . ,= --
n, dt n2 dt· ml dt m2 dt
522 G.R.B. PHYSICAL CHEMISTRY FOR COMPETITIONS

, 1 £[1-:- 0.002
(a) HzO z + 2H + + 31 ------" 13 +. 2HP; _ d[l ] = ? Rate of appearance ofIz =- = --
dt ., 2 At 2
d[H+] =0.001 mol L- I min-I
---=?
dt
='1 x 10-3 mol L -I min-I
(b) 1611+ +2MnO:; +101 ------"2Mn 2+ +8H20+5/ 2 ;
_ d[MnO:; ] _? -'Example 5. The reaction between CrzO~- and HN0 2 in an
dt -. acidic medium is:
, . d[NH] Crp~- + 5H+ + 3HN02 -; 2Cr 3+ + 3N0'3 + 4H2 0
(c) 4NH3 + 50z ------,,4N02 + 6Hp; - 3 =?
, dt The rate of disappearance of Cr20~- is found to be
Solution: We have, 2.4 x 10-4 mol rl s-I during a measured time interval. Find the
(a) HzOz + 2H+ + 3r ------" 1:3 + 2H zO rate of disappearance of HN02 and the rate of appearance of
Cr 3+ during this time interval.
The equality in this case is:
Solution: The equality in this case is:
d[H z 02] 1 d[ H+ ] 1 d[r d[I:3 ] I d[H20]
- A[Cr20~-] =.- I A[HN0 2 ] = 1 A[Cr +]
3
-' =-- =- =--
dt 2 dt 3 dt dt 2 dt
At 3 At 2 At
So,- d[r] =_3 d[H20 2] = _ ~ d[ H+] = 3 d[I:3] = ~ d[H20] It is given that,
dt dt 2dt dt 2 dt
~ A[Cr20~-] := 2.4 X 10-4 mol L- I S-1

°
and _ d[H+] = _ 2 d[H2 2 ] = _ ~ d[ r ] = 2 d[ 1'3 ] = d[H20] 'At
dt dt 3 dt dt dt So, - A[HN0 2 ] = 3 x 2.4 X 10-4 = 7.2 X 10-4 mol L- 1 S- I
(b) 16H+ +2Mn04 +IOr -;2Mn2+ +8H 20+5I 2 At
3
The equality in this case is: and A[Cr +] = 2 x 2.4 X 10-4 =4.8 X 10-4 mol L-1 S-I
1 d[W] I d[Mn04 ] 1 d[r] At
----=-- =---
16 dt 2 dt 10 dt fUUSTfATrON$ OF OBJECTIVE QUEmONS"
1 d[Mn 2+ 1 d[H 20] I d[Iz]
=- - =---
2 dt 8 dt 5 dt 1. For the reaction, 2N20 S ~ 4NOz + Oz rate of reaction is:
So, _ d[MnO;j] = _ 1 d[H+ I d[1 (AIIMS 2006)
=-- 1d
dt 8 dt 5 dt (a) - - [N20 S ] (b) 2 ~ [N 20s]
2 dt dt
d[Mn 2+] 1 d[H20] = 2 d[1 2 ]
= -"'-~--=-
dt 4 dt 5dt
. (c) 1 ~ [NOz]
4 dt
(d) 4 ! [N0z]

(c) 4NH3 +502 ------,,4N0 2 +6H 20 [Ans. (c)]


[Hint: For the reaction:
The equality in this case is:
I d[NH 3 ] =_ 1 _d[_0_2] =1.
d[N0 2 ] =~ . . .,;d[:.. . H.::,.20.. ;;,] 2NzOs ~4N02 + 0z
4 dt 5dt 4 dt 6dt _.!.. d[N2 0 S ] = +
1 d[NO z] = d[ 0 2J = Rate]
i
So, _ d[NH 3 ] =_ _d[_O_z] = d[N0 2 ] ~ . . . ;d[:.. . H.::,.z°.. ;;,] 2 ~ 4 ~ ~

dt 5dt dt 3 dt 2. Observe the following reaction:


'>Example 4. The following reaction was carried out in A(g) + 3B(g) ~ 'lC(g)
water:
CIZ + 21 - ------" 12 + 'lCr
The rate of this reaction { -dd/ l} is 3 X 10-3 mol litre- 1

The initial concentration of 1 was 0.25 mol L- I and the min-I. What is the value of -d[B] in mol litre-I min-I?
dt
concentration after I ominutes 0.23 mol rl. Calcuiate the rate of
(EAMCET 2006)
disappearance of1 - and appearance ofI z .
, (a) 3 x 10-3 (b) 9 x 10-3 (c) 10-3 (d) 1.5 x 10-3
Solution: A[r] =[r ]finaJ - [r ]initisl
[Ans. (b)]
0.23 - 0.25 = - 0.02 mol L- 1 -d[A] 1 d[B]
[.Hint: --=
At = 10- 0= 10 min dt 3 dt
0002'moI L. . I nun
A[r =- (- 0.02) =. ·-1 -d[B] = 3 r-d[A]} 3 x 3 X 10-3
At 10 dt 1 dt
= 9 x 10-3 mol litre- 1 ~in-l]
CHEMICAL KINETICS 523
3. For the reaction, 2A + B ----+ 3C + D, which of the Active mass =molar concentration of the substance '
following does not express the reaction rate? =number of gram moles of the substance
[CBSE (Med.) 2006)
volume in litres
(a) -d[C] (b) -d[B] (c) d[D] (d) -d[A]
3 dt dt dt 2 dt w/M n
=--=-
[Ans. (a)] V V
[Hint! ...,.!. d[A] = -d[D]: +.!. d[C]:;: +d[D] =Rate] where, w = mass of substance and •M' is the molecular mass in
2 dt dt 3 dt dt gram, • n' is the number of gram moles and V is volume in litre.
4. Consider the, reaction, N2(g) + 3H2(g) ----+ 2NH3(g). The .',~ .. _~~ample 6. 4 g of hydrogen and 128 g of hydrogen iodide
are present in a 2 litre flask. What are their active masses?
· "1 ".' h"Ip between'
d[NH
. .3 ] an d - -
d[H 2 ]·,
equaIIty

Fe atIons
••• '- IS:

[CBSE (Med.) 2006)


Solution: Mass of hydrogen = 4 g
Mol. mass of hydrogen = 2
(a)d[NHd = _ ~ d[H2] (b) + d[NH3 ] =_ ~ d[H2 ] Volume of the flask =2 litre
dt 3
dt 'dt 3 dt
(c) + d{NH3] = _ ~d[H2] , (d) d[NH3 ] = d[H 2 ] ,
Active mass of hydrogen =_4_ =I mol L- 1
2x2
dt 2 dt dt dt
[Ans. (b)] Mass of HI =128 g
[Hint! _.!. d[H 2 ]=+ 1 d[NH 3 ]] Mol. mass of HI=128
3 dt 2 dt Volume of the flask =2 litre
5. For' a reaction ~ A ----+ 2B rate of disappearance of A is
Active mass of hydrogen iodide = ~ =0.5 mol L-
1
2
128x2
related to the rate of appearance of B by the expression:
(AIEEE 2008)
(a) -d[A]=4 d[B] =
(b) - d [A] ~ d [B] 8.4' RATE CONSTANT
dt dt dt 2, dt Consider a simple reaction A ~ B.
.!.
(c) - d [A] = d [B] (d) d = + d [B] If CA is the molar concentration or active mass of A at a
dt 4 dt dt dt particular instant, then
[Ans. (c)] . . . dx ex: C or-=kC
dx
[Hint! The reaction may be given as A A
A~4D
dt dt
-d[A] , 1 dID] . where, k is a proportionality constant, called velocity constant
- - = + - - - Rate of reactIOn.] or rate constant or specific reaction rate.
dt 4 dt
6. In a reaction of acidified hydrogen peroxide with potassium At a fixed temperature, if CA = I, then
iodide, the concentration of iodine formed rises from 0 to Rate dx = k ... (i)
10- 5 mol dm -3 in 10 seconds. What is the rate of reaction? dt
[PET (Raj.) 2008) Let us consider a general reaction.
(a) 10- 6 mol dm- 3 S -1 (b) 10 6 mol dm- 3S-1 aA + bB ---:-7 Product
(c) 10- 5 mol dm- 3 S -1 (d) 10 4 mol dm- 3S-1
[ADs. (a)] Rate (:)ex:[A]a[B]b
[Hint: The reaciion is
2r + H20 2 + 2H+ ~ 12 + 2H20 Rate k [A]a [B]b ... (ii)

d[12] 10-
Rate=+--=-'=
5
10- 6 mo I d m- When [A] =[B] =1mol/litre, then
dt ' 10 Rate =k
8~9; LAW OF MASS ACTION Rate of a reaction at unit concentration of reactants is
(Guldberg and Waage, 1864) called rate constant.
This law relates rate of reaction with active mass or molar The value of rate constant depends on:
concentration of reactants. (i) Nature of reactant
"At a given temperature, the rate of a reaction at a (ii) Temperature
particular instant is proportional to the product of the active (iii) Catalyst
masses of the reactants at that instant raised to powers which
are numericaUy equal to the numbers of their respective Unit of Rate Constant
molecul~ in the stoichiometric equation describing the Rate constant has different units for reactions of different
reactio,,:? order. General rule for rate of reaction may be given as:
524 G.R.B. PHYSICAL CHEMISTRY FOR COMPETITIONS

In the graph (Fig. 83), 'E' corresponds to minimum or


Unit of rate consu.nt = [
.
I .
unit of concentration
J nix time- I threshold energy for effective collision in a hypothetical reaction.
There is an energy barrier for each reaction. The reacting
species must be provided with sufficient energy as to cross the
=[ I ] n I X sec- I energy barrier. .
mol/litre
The minimum amount of energy required by reactant
=[litre] n i x sec-I molecules to participate in a reaction is caUed activation
energy.
mol
Activation energy = threshold energy - average kinetic
where, n = order of reaCtion. . . energy of reacting molecules
Threshold energy ::: initiar potential energy of reactant
S.
No.
Rate of reaction ReactIoD rate conStant molecules + activation energy.
A collision between high energy molecules overcomes the
1. It is the speed with which reac- It is proportionality constant. forces of repulsion and brings the formation of an .unstable
tants are converted into prod-
molecule cluster, called the activated complex. The life span of
ucts.
an activated complex is very small. Thus, the activated complex
2. It is measured as the rate of de- It is equal to the rate of reaction breaks either into reactants again or new substances, i. e. ,
crease of concentration of reac- when the concentration of each products. The activation energy (Ea ) depends upon the nature of
tants or the rate of increase of of the reactants is unity.
chemical bonds undergoing rupture and is independent of
concentration of products with
time.
enthalpies of reactants and products. The energy changes during
exothermic and endothermic reactions versus the progress of the
3. i It dependsupon the initial con- It is independent of the initial reaction are shown in Fig. 8.4.
centration of reactants. concentration of the reactants.
It has a constant value at fixed
Activated Activated
temperature. complex complex

8.5 COLLISION THEORY OF REACTION


RATE (Arrhenius Theory of Reaction
Rate)
(I) A chemical reaction takes· place due tei collision among
reactant molecules. The number of collisions taking place per
second per unit volume of the. reaction .mixture lS known as
collision frequency (Z). Tile vall,l~ of Collision frequency is very
high, of the order of 1025 to 1028 in case of binary collisions.
(2) Every collision does not bring a chemical change. The
Progress of the reaction - - Progress of the reaction -'--+
collisions that actually produce the products are effective
(Exothermic) (Endothermic)
co16sions. The effective collisions which bring chemical change
are few in comparison to the· total number of collisions. The Fig. 8.4 Activation energy of exothermic
collisions that do not form a product are ineffective elastic and endothermic reaction
collisions, i. e., molecules just collide and disperse in different
Thus, every chemical reaction whether exothermic or
directions with different velocities. For a collision to be effective,
endothermic has an energy barrier which has to be overcome
the following two barriers are to be cleared:
before reactants can be transformed into products. If the reactant
Energy Barrier molecules have sufficient energy, they can reach the peak of the
The minimum amount of energy which the colliding energy barrier after collision and then they can go to the right side
molecules must possess as to make the chemical reaction to of the slope and consequently change into products. If the
occur, iskno~ as threshold energy. activation energy for a reaction is low, the fraction of effective
collisions will be large and the reaction will be fast. On the other
t hand, if the activation energy is high, then fraction of effective
collisions will be small and the reaction will be slow. When
Fraction of molecules
capable of bringing temperature is increased, the number of active molecules
effective collisions increases, i. e. , the number of effective collisions will increase
and the rate of reaction will increase.
Activation energyEa E(aclivated ~~mplex)' E(ground s~te)
an ";'activation energy of forward reaction .. .
- activation energy of backward reaction.
Energy-
Fig. 8.3
CHEMICAL KINETICS 525

Orientation Barrier reaction is maximum because in powdered state, surface· area is


Energy alone ~oes not determine the effectiveness of the .maximum:
collision. The reacting molecules must collide in proper manner (c) Chemical nature of reactants: Consider the following
if the reaction is to occur. This has been shown in Fig. 8.5. two reactions:
Rate of reaction is directly proportional to the number of 2NO(g) + 02(g) ---7 2N0 2 (g) ... (i)
effective collfsions. . . CH4 (g) + 202 (g) ---7 CO 2 (g) + 2H2 0 ... (ii)
Rate =- dx =collision frequency x fraction of The first reaction is faster than the second because in the first
reaction only N==O bond is to be broken whereas in the second
dt effective collisions
reaction four (C~H) bonds are to be broken.
=Zxj
Similarly, consider another example of two similar reactions:
2NO(g)+ °2 (g) ---7 2N0 2 (g) ... (iii)
I 2CO(g) + °2 (g) ---7 2C02(g) .., (iv)
at) NO bond is weaker than CO bond, hence broken easily. Thus,
reaction (iii) is faster than (iv).

- 83-8- (ii) Concentration of reactants


Let us consider the reaction:

-- A +B---7C+D;
Rate == k [A][B]
Rate of the above reaction
i
A A decreases with the passage of time ~
A---A A r---------1A
I
I
I
I
because the concentration of a:
I I
I
I
I
I
reactants A and B will decrease as
6---6 6 '----------'6 time passes on.
6 6 Time-
Products Let us consider the following
Reactants reversible reaction: Fig.8.S
.. / A+B~C+D

-Ll
. . . . '. ' Rate offorward reaction =k f [A][B]

fT . Rate of backward reaction = kb[C][D]


Rate of forward reaction decreases and that of backward
reaction increases with passage of time. At equilibrium both the

- rates become equal.

8
f
No product
formation i Equilibrium

Fig. 8.5 Orientation of collisions


Factors Affecting Rate of Reaction
(i) Nature of the reactants Time-
(a) Physical state of reactants: This has considerable Fig. 8:1
effect over rate of reaction.
Gaseous state> Liquid state> Solid state (iii) Effect of temperature
The rate of reaction increases considerably with an increase in
Decreasing rate of reaction temperature. The rates of many reactions are approximately
Similarly, in a heterogeneous system, collision is not so doubled or tripled for every looe rise in temperature. The
effective as in homogeneous system. Thus, reactions in liquid temperature coefficient of a chemical reaction is defined as the
phase or solution phase will be faster in comparison to ratio of the specific reaction rates of a reaction at two
heterogeneous conditions when same concentrations of the temperatures differing by lOoe.
reactants are taken. . k t + to
(b) Physical size ofthe reactants: . Among the solids, rate /.! =Temperature coeffiCient == - -
kt
increases with decrease in particle size. In powdered state rate of
526 G.R.B. PHYSICAL CHEMISTRY FOR COMPETITIONS

Let temperature coefficient of a reaction be 'J1' when Logarithmic Arrhenius equation is:
temperature is raised from Tj to T2 ; then the ratio of rate constants
or rate may be calculated as: log 10 k log 10 A [..!.]
2.303R T
= (J..t.)(T2 TI)1I0=J1 AT11O Y=C+M X
kTI It is the equation of straight line with negative slope. On
plotting log 10 k against [1/ T ] we get a straight line as shown in
log 10 l'_k1'_2] Fig. 8.8. The graph gives two kinetic parameters.
kTI The slope gives activation energy and intercept gives
frequency factor.
=antilog [ ~~ log 10 J1 ] The dependence of rate constant on temperature for two
kTI reactions is shown in the Fig. 8.9.
Its value lies generally between 2 and 3.
When the temperature is increased, heat energy is supplied
which increases the kinetic energy of the reacting molecules.
This will increase the number of collisions and ultimately the rate
of reaction will be enhanced. Arrhenius suggested an equation
-. i
. 10910 k
which describes k as a function of temperature, i. e. ,
k =Ae-EaIRT
®
1._
T
l Fig. 8.9

In the reaction (2), the modulus of slope is greater than that of


reaction (1), hence reaction (2) has higher activation energy. The
reaction (2) will be more sensitive to temperature change.
1/T- Reactions on the basis of influence of temperature are
I classified into five types. These are shown in the graphs.

log(~)
rI
t).
1ii
a:
.45"C
1._ T- Temperature - - Temperature - - Temperature __
T (Normal reactions) (Explosive reactions) (Biological reactions)
Fig. 8.8 (a) (b) (c)

where, k -7 rate constant


A -7 a constant (frequency factor) (For 2NO + O2 -? 2N02)
E a -7 energy of activation t The only reaction with
-ve temp .. coefficient
At two temperatures Tj and T2 , taking log of Arrhenius
equation, we get
... (i)
Temperature __ TeITIperature - -
. ;;. (ii) (Oxidation of carbon)
and log e k2 = log A -
(d), (e)
Subtracting eq. (ii) from eq. (i) and converting the log to .the
Fig. 8.10
base 10, we get .
log 10 k =- ---
-
j Ea [T2
---T j ]
or log 10 . [T2 - ~ ] (iv) Presence of a positive catalyst
The function· of
a positive catalyst is to lower down the
k2 2.303R ~ T2 kj 2.303R ~ T2
"activation energy. The" greater the decrease in the· activation
This equation can be used for calculation of energy of
eHergy caused by the catalyst, higher will be the reaction rate. In
activation.
CHEMICAL KINETICS 527

the presence of a catalyst, the reaction follows a path of lower


k p = antilog [ l:!.E ]
activation energy. Under tbis condition, a large number of ka 2.303RT
reacting molecules are able to cross over the energy barrier and
thus the rll-te of reaction increases. Fig. 8.11· shows how the :::::::_SOME SOLVED EXAMPLES\ I::: :::
activation energy is lowered in presence of a catalyst. . Example 7. The rate of a reaction triples when
temperature changes from 20°C to 50°C. Calculate energy of
activation for the reaction. (R = 8.314JK -I mol-I)
Solution: The Arrhenius equation is,

1
/
-f----;:~;?--',~\
E; \
loglo ~: =R x 2.303 [T~I~; ]

I
Reaction \
; path with \
Given:~=3;R=8.314JK-l mol- I ;TI =20+273=293K

'------:~·l~;;~~~ , -
kl
Reacta;;';- and T2 = 50+ 273 = 323 K
reaction
The effect of a ________ _______________
'I ___ Substituting the given values in the Arrhenius equation,
catalyst is to lower Products
the energy of activation 10 3- Eo [323- 293]
glO - 8.314 x 2.303 323 x 293
Collision number -
2.303 x 8.314 x 323 x 293 x 0.477
Fig. 8.11 A catalyst changes the Ea =~-------------------
30
reaction path (Positive catalyst)
:::: 28811.8J mor l

(v) Presence of negative catalyst ::::: 28.8118 kJ mor]


A negative catalyst increases the activation energy of reaction Example 8. In Arrhenius equation for a certain reaction,
by forming a new intermediate of high energy, i. e. , by changing the value ofA and Eo (activation energy) are 4 x 1013 sec -] and
the reaction mechanism.
98.6kJ mol-I respectively. At what temperature, the reaction
Due to increased activation energy, some active molecules
become inactive, therefore, rate of reaction decreases. will have specific rate constant 1.1 x 10-3 sec -I ?
Solution: According to Arrhenius equation,

I. ncreased I ---- -------------/-', I \


k = Ae-Ea IRT
activation
energy 1 / 1t!~-_ _

\
\
\
Catalyst
present or
E
_a
RT
.
logee
\
\
;., \
Catalyst
or 2.303 log 10 k = 2.303 log 10 A - RT
~
Q) \ absent
c: \
Q) \ . 98.6x 103
iii \ or 2.303 log (1.1 x 10-3 ) = 2.303 log (4 X 1013 ) - - - -
:e
Q)
\
\
\
8.314 x T
o
0..
\
\ , T= 98.6x 10
3
K
~
8.314 x 2.303 x 16.56
Products
= 31O.96K
Collision number -
Example 9. The rate constant is given by Arrhenius
Fig.8.12 Negative catalyst equation,
\

Let' p' denote presence of.catalyst and 'a' denote absence of k Ae-EaIRT

catalyst. Calculate the ratio of the catalysed and uncatalysed rate


k = Ae-EpIRT
p ... (i) constants at 25°C if
the energy of activation of a catalysed
reaction is 162 kJ and for the uncatalysed reaction the value is
ka Ae-EaIRT ... (ii) 350kJ.
Solution: Let kca and k un be the rate constants for catalysed
Dividing eq. (i) by eq. (ii), we get
and uncatalysed reactions.
kp . =e(Ea-Ep)/RT =eI!.EIRT 162x 103
2.303 log 10 kca = 2.303 log 10 A - ... (i)
ka RT
I
528 G.R.9. PHYSICAL CHEMISTRY FOR COMPETITIONS

350x 103 or log


A
. -'
75782.3
, .,' :::: 9.7246
and 2.303 log 10 k un :::: 2.303 log 10 A - - - - ... (ii) 5
, RT , 9.5x,10- 2.303 x 8.314 x 407
Subtracting eq. (ii) fromeq.'(i) , A 5.04 x lOS S-I
k 103
10glO ~:::: (350-162) Example 12. The energy ofactivation for a reaction is 100
k un 2.303RT kJ mol -I . Presence ofa catalyst lowers the energy ofactivation
3 by 75%. What will be effect on rate of reaction at 20°C, other
188 x 10 :: 32.95
x8.314 x 298 things being equal?
Solution: The Arrhenius equation is,
kca =8.88 x 1032 k= Ae-EaIRT
k un
In absence of catalyst, k, :::: Ae-'OOIRT
Example 10. Calculate the rate constant of a reaction at
293 K when the energy ofactivation is 103 kJ mol -I and the rate In presence of catalyst, k2 Ae-25IRT
7
constant at 273K is 7.87 x 10-
~
s-I.
So, k2 e 75(RT or 2.303 log k2 ::::
(R 8.314 x 10- kJ mor' K -, )
3
~ ~ RT
Solution: The Arrhenius equation is, 75
or 2.303 log =----:---
[T2 ]1]
3
k( 8.314 X 10- x 293
10glO k2 ::::
kl 2.303R TI T2 or log k2 75
7 k, 8.314 x 10-3 x 293 x 2.303
Given: kl = 7.87x 10- s-I;E a :::: 103 kJ mol-I;
R=8.314xlO- kJmol- K- 1 ; 3 1 or k2 2.34 x 1013

1j :::: 273 K andT2 = 293 K As the things being equal in presence or absence of a catalyst,
Substituting the values in Arrhenius equation, k2 must be = rate in,presence of catalyst.

I k2 = 103 x 20 k, rate in absence of .catalyst


oglo 7.87 x 10-7 2.303 x 8.314 x x 293x273 = k2 == 2.34 X 1013
i.e., r2
fj kl '
:::: 1.345
: 2 :::: 1.74 x 10-5 s-I

Exa.uple 11. "i' 407 K, the rate constant of a chemical 7. For a reaction Ea= 0 and k:::: 3.2 X 104 s-I at 300 K.The
reaction is 9.5 x 10-5 s -I and at 420 K, the rate constant is value of k at 310 K would be:
1.9 x 10-4 S -I. Calculate the frequ~ncy factor of the reaction. (a) 6.4 x 104 s-I (b) 3.2 X 104 S-I

Solution: The Arrhenius equation is, (c)3.2 X 108 s-, (d) 3.2 x 105 s-I
[Ans. (b)]
k2
k; : : 2.303 x R 1j T2
log 10
[T2 -T1 ] [Hint: k =Ae-Eo IRT
WhenEa =0, k = A = constant
Given: k, =9.5xlO-5 s-';k2 =1.9 x 10-4 s-I; k310 = k300 = 3.2 X 104 S-I]
8. For a gaseous reaction, following data is given:
R:::: 8.314 Jmorl K- 1 ;
. A ~B,kl :::: 1015 e-2OOOIT
T( ::::407KandT2 = 420K
C
Substituting the values in Arrhenius equation Jb.e tempel1lture at which k, :::: k2 is:
log L9x 10-4 :::: [420- 407] . (a) lOOQ K (b) 2000 K (c) 868.82 K (d) 434.2 K
, 10 9.5 X 10-5 2.303 x 8.314 420 x 407 . [Ans. (d)]
[Hint: When k, =k2'
Ea :::: 75782.3 J J;DOr( 1Ols e-2ooolT =1014 e-IOOOIT
E 10 = e1OOO/ T
Applying now log k, :::: log A _ ,a
2.303R1j
2303 log 10 = 1000
I 9 5 10-5 I A 75782.3 T
og . x :::: og - 2.303 x 8.314 x 407 T =434.2 K]
CHEMICAL KINETICS 529

9. For N 2 (g) + 3H2(g) ~ 2NH3(g) + 22 kcal, Ea for the


(c) IOglO .!. = ~ [~-~]
reaction is 70 kcal. Hence, the activation energy for 2 2.303 T2 11
2NH3(g) ~ N2(g) + 3H2(g) is:
(a) 92 kcal (b) 70 kcal (c) 48 kcal (d) 22 keal (d)IOglo2=~[-.!.
2.303R 11
~]
T2
[Ans. (a)]
[Hint: Ml=Ef Eb [Ans. (d) ]

-22 = 70 - E" [Hint: We know, log (k2) =.~ [~-~]


k . 2.303 R T T
t t z
Eb = 92 kcal]
When reaction rate b~o~es double then k2 will be equal to 2.
10. On introducing a catalyst at 500 K, the rate of a first order kt
reaction increases by 1.718 times. The activation energy in the
presence of a catalyst is 6.05 kJ mol-I. The slope of the plot of
In k (sec-I) against liT in the absence of catalyst is:
(a) +1 (b) -1 (c) +1000 (d) -1000
I~'.~ MOLECULARITY OF REACTION
[Ans. (d)]
[Hint: Rate in presence of catalyst = A'1
Rate in absence of eatalyst
ntl og l +t:.E ]
2.303RT
In general, molecularity of simple reactions is equal to tbe sum
of tbe number of molecules of reactants involved in tbe
E-E balanced stoicbiOJ,netric equation.
1.718 =Antilog ap
2.303 x 8.314 x 500 OR
Ea - Ep = 2.25 kJ
Tbe molecularity of a reaction is tbe number of reactant
Ea = Ep + 2.25 = 6.05 + 2.25 = 8.30 kJ mort molecules taking part in a single step of tbe reaction.
= 8.3 kJ mort
e.g., PCl5 ---t PCl 3 + Cl 2 (Unimolecular)
lnk=lnA- x 2HI ---t H2 + 12 (Bimolecular)
R T
Slope = -8.3 x 1000 = -1000] 2S0 2 + 02 ---t 2S0 3 (Trimolecular)
R 8.3
NO + 0 3 ---t N02 + O 2 (Bimolecular)
11. For which of the following reactions k310 I k300 would be
maximum? 2CO + 02 ---t 2C02 (TriInolecular)
(a) A +B~C; Ea = 50kJ 2FeCl 3 + SnCl 2 ---t SnCl 4 + 2FeC1 2(Trimolecular)
(b)X+Y~Z; Ea= 40kJ Tbe minimum number of reacting particles (molecules,
(c)P+Q~R; Ea = 60kJ atoms or ions)tbat come togetber or collide in a rate
.., (d)E+F~G; Ei> = lOOkJ determining step to form product or products is called tbe
[Ans. (d)] molecularity of a reaction.
[Hint: Increase in rate constant is maximum for the react.ion
For example, decomposition of H 20 2 . takes place in the
having maximum activation energy.]
following two steps:
12. The activation energy of exothermic reaction A ~ B is
80 kJ mol-I. The heat of reaction is 200 kJ mol-I. The H 20 2 ~H20+ 1 02 . (overall reaction)
2
activation energy for the reaction B ~ A (in kJ/mol) will
Step 1: H 20 2 ---t H 20+ [0] (slow)
be: [PET (Kerala) 2007]
(a) 80 (b) 120 (c) 40 (d) 200 Step 2: [0] + [0] ---t 02 (fast)
(e) 280 The slowest step is rate-determining; Thus, from step 1,
[Ans. (e)] reaction appears'to be unimolecular.
[Hint: Ml =Ef - Eb Note: (i) Molecularity is a theOretical concept.
(ii) Molecularity cannot be zero, -ve, fractional, iIlfInite and
imaginary. .
-200= 80 Eb
(iii) Molecularity cannot be greater than three because more
Eb = 280 kJ/mol] than three molecules may not mutually collide with each other.
13. Which equation is true to calculate the energy of activation, if There are some chemical reactions whose molecularity
the rate of reaction is doubled by increasing temperature from appears to be more than three from stoicbiometric equations, e. g. ,
11 K to T2 K? (CET (Gujarat) 2008] in

(a) loglO (~) = 2.303 R [;. - ;J 4HBr + 02 ~ 2H 20 + 2Br2


2MnO:;: +16H+ +5C20~- ---t2Mn 2+ + 10 CO 2 +8H20

(b) log 10 ( ~: ) = 2.;3 R [ ;2 ;. ] In the ftrst reaction, molecularity seems to be '5' and in the
second reaction molecularity seems to be '23'. Such reactions
530 G .RB. PHYSICAL CHEMISTRY FOR COMPETITIONS

involve two or more steps; each step has its own molecularity not Step 1: H Z02 -----7 H 20+ [0] (slow)
greater than three, e. g. , in first reaction,
HBr + 02 -----7 HOOBr Step 2: [0] + [0] -----7 0z (fast)
HOOBr + HBr -----7 2HOBr dx
Actual rate - = k[H 2 0 2 ]
[HOBr + HBr -----7 H 20 +Br2 ] x 2 dt
4HBr + 0z -----7 2H 20 + Br2 Thus, order of reaction is unity..
Molecularity of each of the above steps is 2. (ii) 2NO z + F2 -----72N02F
(a) Reaction between Br- and 8 2 0 2 in acidic Rate law from law of mass action:
medium: The overall reaction is: dx 2
- - k [N02 ] [Fz]
2Br- + H 20 2 + 2H + -----7 Brz + 2H 20 dt
The proposed mechanism is: Experimentally observed rate law:
Br- + H 20 2 + H + -----7 HOBr + H 20 (slow)
- dx = k [N0 2 ][F2 ]
HOBr + H + + Br- -----7 Br2 + H 20 (fast) dt
Rate =k [Br- ][H20 2 ][H+] Slowest step is N0 2 + F2 -----7 N0 2F + [F]
The reaction is trimolecular. Thus, order of reaction 1+ 1= 2
(b) Reaction between N0 2 andFz : The overall reaction is: (iii) CH 3 CH0-----7CH 4 + CO
2N0 2 + F2 -----72N0 2F The rate equation derived from experimental data is found to be
The proposed mechanism is:
_ dx = k[CH 3 CHO]1.5
NO z + F2 -----7 N0 2F + F (slow) dt
N0 2 + F -----7 N0 2F (fast)
The order of reaction is 1.5.
=
Rate k [N0 2 ] [F2 ]
Some typical linear plots for the reactions of different
The reaction is bimolecular. orders:

irs;' V. ipe,
(c) Decomposition of 8 2 0 2 : The overall reaction is: (a) Plots of rate vs concentrations [Rate k(conc. t]
°
2H 2 0z -----7 2Hz + 0z

~~d.'
The proposed mechanism is:
H Z 02 -----7 H 20+
H 20z + 0-----7 HzO+ 02
° (slow)
(fast) . :$.
t1 d ord. er
ga _g> "[Ai [A)2
Rate =k [H 2 02 ]
The reaction is unimolecular. U t- t- t- t-
(b) Plots from integrated rate equations:
8i7 ORDER OF REACTION
Let us consider a general reaction:
ml A + mzB + m3C -----7 Product
i Zero order
!l/e, !L/' !V e
,
Let active moles of 'A', 'B' and 'e' be 'a', '[3' and 'y'
respectively. Then, rate of reaction may be given as:
Rate = k[A](l[B]f3 [C]Y
*
a:
Conc.-
iii
a: ..
Conc.-
iii
a:
(Conci-
~
(Conc.)3_

Sum of powers of concentration terms involved in rate law From the study of the kinetics of many simple reactions, it is
expression is called order of reaction. observed that for a large number of reactions, the molecularity
a + ~ + y == order and order are the same. Some examples are given below to justifY
When a + ~ + Y mJ + m2 + m3 , then this point.
Order of reaction = molecularity of reaction. L Dissociation ofN 20 s ,
Order is an experimentally determined quantity. It may be equal I
N 2 0 S -----7 N 20 4 + - 02
to zero, positive, negative, fractional and greater than three. 2
Infinite and imaginary values are not possible. Order = 1, Molecularity = 1
Examples:
(i) 2H 20 2 -----7 2H zO + 02
2. Dissociation of H2 ° 2,
1
dx . z H 20 2 -----7 H 20+ - 02
-=k[HzO z ] 2
dt Order 1, Molecularity = 1
(observed from law of mass action)
CHEMICAL KINETICS 531

3. Dissociation of HI, Solution: From an examination of above data, it is clear that


2HI~H2 +12 .when the concentration of B2 is doubled, the rate is doubled.
Hence, the order of reaction with respect to B2 is one.
Order = 2, Molecularity = 2
Further, when concentration of A is doubled, the rate remains
4. Formation ofN0 2, unaltered. So, order of reaction with respect to A is zero.
2NO + O 2 ~2N02 The probable rate law for the reaction will be
Order = 3, Molecularity = 3 - dx = k[B 2 ][A]o k[B 2 ]
dt
8.8 PSEUDO-ORDER REACTION Alternatively, Rate:::: k[B 2 rx
Reactions whose actual order is different from that expected 1.6 x 10-4 = k[0.5]a.
using rate law expression are called pseudo-order reactions,
e.g., 3.2x1O-4 :::= k[l]a.
(i) RCI + H 2 0 ~ ROH + HCI On dividing we get, 0: 1
Expected rate law: .. Rate k[A]o[B 2 ]1:::= k[B 2 ]
Rate = k [RCI][H 2 0] Expected order =I + 1 =2 'J~;:,: ;lExample 14. For the reaction,
Actual rate law: A+2B X
the following data were obtained:
Rate = k' [ReI]; Actual order =1
Expt. lllitial cancentmlions (mo~' L'-I) lllitial reaction rates
Water is taken in excess; therefore, its concentration may be
taken constant. The reaction is, therefore, pseudo first order. No. (mol 1: min-l)
._ ..
[A] [B] '
Similarly, the acid catalysed hydrolysis of ester, viz., .•.. "

.
~
~

RCOOR' + H20~RCOOH + R'OH 1. 1.0 1.0 0.15


follow first order kinetics: 2. 2.0 1.0 0.30
Rate =k [RCOOR ' ] 3. 3.0 1.0 0.45
4. 1.0 2.0 0.15
It is also a pseudo fitst order reaction.
The main differences between molecularity and order of 5. 1.0 3.0 0'.15
reaction are given below: Write down the rate law for the reaction.
Solution: . Let the rate law be
Molecularity . Order of reaction .
_ dx :::= k[AY[BY
1. It is the total number of reacting It is the sum of powers of molar dt
species (molecules, atoms or concentrations ofthe reacting
ions) which bring the chemical. species in the rate equation of By keeping the concentration of B constant in experiments (1),
change. the reaction. (2) and (3) and increasing the conc,entration uniformly, the rate
also increases uniformly. Thus,
2. It is always a whole nUItlber. It may be a whole number, zero,
fractional, positive or negative. Rate <:>::[A], i.e., x=1
3. It is a theoretical concept. It is experimentally determined.
By keeping the concentnition of A constant in experiments
(1), (4) and (5)and increasing the concentration of B, the rate
4. It is meaningful only for simple It is meant for the reaction and remains the same.
reactions or individual stepsof a not for its individual steps.
complex reaction. It is meaning- Hence, yO,
less for overall complex reaction. dx
The rate law is -==k[A]
Example 13. The experimental data for the reaction, dt

2A +B2 ~2AB Alternative method:


is From expt. (1), k[1.0Y [l.oy == 0.15 ... (i)

[A] [B z] Rllte From expt. (2), k[2.0Y[l.Oy := 0.30 ... (ii)


Expt. No. (moIL-i) (moLL-I) (molL- 1 s·l) Dividing eq. (ii) by eq. (i),
------_...
1. 0.50 0.50 1.6 x 10-4 [2.0Y _ 0.30 2
2. 0.50. 1.00 3.2 x 10-4 [1.0Y 0.15
--_ 3. . 1.00 1.00 3.2 x 10-4
S~ x=1
Write !ne most probable equation for the rate of reaction
giving reason for your answer. f' • T 11)() I \ From expt.(I), k[1.0Y [l.oy = 0.15 ; ... (i)
532 G.R.B. PHYSICAL CHEMISTRY FOR COMPETITIONS

From expt. (4), k[1.0Y[2.0Y == 0.15 ... (iii) k = 1.2 X 10-4 = 1.2 X 102 mol-2 L2 s-1
Dividmg eq. (iii) by eq. (i), [0.010]3

[2.0y == 1 Example 16. For the hypothetical reaction


[l.OY 2A + B ....., Products

So, y==O the followi:!g data are obtained:

dx Initial cone, of Initial cone. of


Initiail'ate
Hence, the rate law is - - = k[A] Expt. No. (A) (B)
dt l l ",01 C l s-1
(mol L- ) (moIC )

Example 15. For the reaction, 1. '0.10 0.20 3 x 102


. 2NO + Clz ~ 2NOCI 2. 0.30 0.40 3.6 x 103
at 300 K, following data are obtained: 3.- 0.30 0.80 . 1.44 x 104
IniliaI Concentration 4. 0.10 0.40
Expt. No,' ]nitillll'ate 5. 0.20 0.60
[NO] [Cl:il" '6. 0.30 1.20
1. 0.010 0.010 1.2 X 10-4 Find out how the rate of the reaction depends upon the
concentration ofA andB andfill in the blanks.
2. 0.010 0.020
Solution: ' From expt. (2) and (3), it is clear that when
3. 0.020 0.020 concentration of A is kept constant and that of B is doubled, the
rate increases four times. This shows that the reaction is of
Write rate law for the reaction. What is the order of the second order with respect to B.
reaction? Also calculate the specific rate constant. Similarly, from expt. (1) and (2), it is observed that when
Solution: Let the rate law for the reaction be concentration of A is increased three times and that of B two
times, the rate becomes twelve times. Hence, the reaction is first
Rate = k[NOY[ClzY
order with respect to A.
From expt.(I), 1.2 x 10-4 = k[0.0IOy[0.010Y ... (i) Thus, the rate law for the reaction is:
From expt. (2), 2.4 X 10-4 k[O.OIOY [0.020Y ... (ii) Rate k[A][B]z

Dividing eq. (ii) by eq. (i), FiQ in the blanks:' Substituting the values of expt. (1) in the
rate eqiIation,
2.4 X 10-4 =[0.020Y
3 x IO z = k[0.10][0.20]2
1.2 x [0.0 lOy
2
or 2 (2)Y or k= 3 X 10 7.5 X 104 L2 mol-z S":I
. [0.1O][0.20f
y=1
Expt. (4): Rate k[O.lO][0.40]2
, FrOtilexpt. (2), 2.4 x 10-4 = k[O.OIOY [0.020Y ... (ii) 4
= 7.5 X 10 x O.lOx 0.40 x 0.40
From expt. (3),9.6 x 10-4 = k[0.020Y [0.020Y ... (iii)
= 1.2 x 103 mol L -1 s-I
Dividing eq. (iii) by eq. (ii),
Expt. (5): Rate = k[0.20][0.60]2
9.6x 10-4 _ [0.020Y
2.4 x 10-4 - [O.OlOy =7.5 X 104 x 0:20 x 0.60 x 0.60
or 4=2X = 5.4 x 10 3 mol L- 1 S-l

x=2 Expt. (6): Rate =k[0.30][1.20]2


Order of reaction = x + y = 2 + 1= 3 = 7.5 X 104 x 0.30 x 1.20 x 1.20
Rate law for the reaction is = 3.24 x 104 mol L-'-I S-l

Rate = k[NO]z [CI 2 ] Example 17. The table given below gives kinetic data for
the following reaction at 298 K:
Considering eq. (i) again,
1.2 x 10-4 =k[0.01O]2 [0.010] ocr +! -.....,0/ - +CI
CHEMICAL KINETICS 533

[OCr] [r] [OH'"] 10- 4 xd[IQ-]/dt Example 18. The rate law for the reaction,
Expt.
No. moldm-J mol d",-J r:.ol dnt-J moldm~3s-1 2ClzO -----? 2Clz + O2
1. 0.0017 0.0017 1.0 1.75 at 200°C is founa to be: rate =k[ ClzO] 2 •
2. 0.0034 0.0017 1.0 3.50 (a) How would the rate change if [Cl 20] is reduced to
0.0017 0.0034 1.0 3.50 one-third of its original value?
3.
0.5
(b) How should the [ClzO] be changed in order to double the
4. 0.0017 0.0017 3.50
.' rate? -
What is the rate law and what is the value of rate constant? (c) How would the rate change if[CI 20] is raised to threefold
=
Solution: Let the rate law be k[OCl- U- Y[OH-]Z r of its original value?
From expt. (1), 1.75 x 10-4 k[0.0017Y[0.0017]Y [l.O]Z ... (i) Solution: (a) Rate equation for the reaction,
Fromexpt. (2), 3.50 x 10-4 = k[0.OO34l" [O.0017]Y [l.O]Z ... (ii)

ri
r= k[CI 2 0]2

Dividing eq. (ii) by eq. (i),


Let the new r~te be r; so, r = k[ CI~O = r
3.50 X 10-4 [0.0034Y
----:- = ::..---~
(b) In order to have the rate = 2r, let the concentration of
1.75 X 10-4 [O.0017Y
Cl 2 0bex.
x
or 2= 2 So, 2r= 10: 2 ... (i)
or x = I, i. e., first order w.r. t. OCI- We know that, r= k[C1 2 0f ... (ii)
From expt. (1),1.75 x 10-4 = k[O.0017Y [0.0017]Y [1.oy ...(i) Dividing eq. (i) byeq. (ii),

From expt. (3), 2r


r
3.50 x 10-4 = k[O.OOl7]X [0.0034]Y [1.0Y ... (iii)
x2
Dividing eq. (iii) by eq. (i), or 2=--':'~-
4 [CI 2 0]2
3.50x 10- [0.0034Y
- - - - ="'----'-- or x 2
=2[Cl 2 0]2
1.75 x [O.OOl7]Y
or 2= 2 Y or x=.J2[C1 2 0]

or y = 1, i. e., first order w. r. t. r (c) New rate = k[3CI 2 0]2 = 9k[CI 20]2 = 9r
From expt. (1),1.75 x 10-4 = k[O.OOI7]x [O.OOI7]Y [1.0]Z ... (i) i. e. , nine times of the original rate.
t~~IExalllple 19. For a reaction in which A and B form C. the
From expt. (4), following data were obtained from three experiments:
3.50 x 10-4 = k[O.OO I 7]x [O.OOI7]Y [0.5]Z ... (iv)
Dividing eq. (i) byeq. (iv), &pi.
.No.
/1fiIitd COliC. (M r')
1'",.
(_1:"8-1)
1.75 x 10-4 [1.oy [A] [B]
=--
3.50 x [O.5]z
1. 0.03 0.03 0.3 x 10-4
or 2. 0.06 0.06 1.2 x 10-;4
3. 0.06 0.09 2.7x 10-4
or
z = -I, i.e., order w.r.t. OIr is-I. What is the rate equation ofthe reaction and what is the value of .
rate constant? . .
k[OCr ][1-]
Rate 1aw Solution: Let the rate equation be k[AY [BY.
[OH-]
From expt. (1), 0.3 x 10-4 = k[0.03Y[0.03]y ... (i)
From expt. (I) k =1.75 x 10- 4 [Oli- ] =1.75 X 10-4 x 1.0 From expt. (2), 1.2 x 10-4 = k[0.06Y [0.06]Y ... (ii)
[0Cl- ][1-] 0.0017 x 0.0017
=6O.55s- 1 1.2 x 10-4 = [0.06Y [0.06Y
0.3 x 10-4 [0.03Y[0.03]Y
... (iii)

534 G.R.B. PHYSICAL CHEMISTRY FOR COMPETITIONS

Similarly, from expt. (1) and expt. (3), increased by 1.5 times. The order of reaction with respect to A
x is:
2 x 3Y = 9 .. , (iv)
(a)1 (b) l.5 (c) 2 (d) -1
Solving eqs. (iii) and (iv), [Ans. (b)]
x =.0, y= 2 [Hint: Rate = k [A]n ... (i)

Rate equation, Rate = k[B]2 1.837 x Rate = k [l.5A r ... (ii)

Considering eq. (i) again, Dividing eq. (ii) by eq. (i),


-4 1.837 = l.5 n
k = 0.3 x 10 = 3.33 X 10-2 mol-I L s-I
[0.03]2 n=312 (solving by logarithmic method)]
18. The rate of the reaction,
3A + 2B ~ Products
is given by the rate expression:
14. For the decomposition of HI at 1000 K (2HI ~ H2 + 12),
Rate = k [A][B]2
the following data were obtained:
Rate ofdecOlDpoSition.' '",
If A is taken in excess, the order of the reaction would be:
of HI (IDol L -1 S-I)" 'C' , ' • (a) 3 (b) 2 (c) 1 (d) 5
[Ans. (b)]
0.1 2.75x 10-8 [Hint: When A is taken in excess, its concentration will become
0.2 11 x 10-8 constant; the rate law may, therefore, be given as:
0.3 24.75 x 10-8 Rate = k'[Br Order = 2]
The order of reaction is: 19. For a reaction A +B ~ C + D, if the concentration of A is
(a) 1 (b) 2 (c}O ' ' (d) l.5 doubled without altering the concentration of B, the rate gets
[ADs. (b)] doubled. If the. concentration of B is increased by nine times
[Hint: Rate = k [HW without altering the concentration of A, the rate gets tripled.
The order of the reaction is: (KCET 2006)
11 x 10-8 = k [0.2r ... (i) (a) 2 (b) 1 (c) 312 (d) 4/3
[Ans. (c)]
2.75x ur lr
';;;' k [0.1]" ... (ii)
[Hint: Rate=k[A]"[B]~ ... (i)
Dividing eq. (i) by eq~ (ii),
2 x rate = k[2A]"[B]P ... (ii)
4 = 2n , n = 2]
3 x rate = k[A ]"[9B]P ... (iii)
15. Consider a reaction, A ~ B + C.1f the initial concentration
From eqs. (i) and (ii),
a =1
of A was reduced from 2 M to 1 M in 1 hour and from 1 M to
From eqs. (i) and (iii),
!3 = 1/2
0.25 Min 2 hours, the order of the reaction is:
,(a)1
[Ans. (a)]
(b) ° (c) 2 (d) 3
1 3
Order=a+!3=I+-=- ]
2 2
20. Inversion of cane sugar in dilute acid is: [CET (J&K) 201)71
[Hint: Half life of the given reaction is independent of initial (a) bimolecular reaction
concentration, hence, it is a first order reaction.
(b) pseudo-unimolecular reaction
1 (c) unimolecular reaction
t I/2 °c----;;-::)]
a (d) trimolecular reaction
16. The rate of a gaseous reaction is given by the expression [Ans. (b)]
H+
k [Af[Bf The volume of the reaction vessel is suddenly
[Hint: C 12H 22 0 11 C6H I20 6 + C6H I2 0 6
+ H20 -----7
reduced to one half of the initial volume. The reaction rate
relative to the original rate will be: Rate = k [C'2H220U J[H 201
(a) 1124 (b) 1132 (c) 32 (d) 24 When water is in excess, its concentration will be constant.
[ADs. (c)] .. Rate = k' [C12 H 22 0,,1
[Hint: ... (i) The reaction is, therefore, pseudo first order or pseudo
unimolecular reaction.] .
When volume is halved, the concentration win become double.
21. The brominatlon of acetone that occurs in acid solution is
.. Rate = k [2A]2[2B]3 represented by
= 32k [A]2[B]3 CH3COCH3 (aq.) + Br2 (aq.) ~ CH 3COCH2Br (aq.)
= 32 x Original rate from (i)) + W(aq.)+ Br- (aq.)
17. For a chemical reaction, A ~ B, the rate of reaction These kinetic data were obtained for given reaction
increases by a factor of 1.837 when the concentration of A is concentrations:
CHEMICAL KINETICS 535
Initial concentrations (M) Initial rate of 12~21 (fast) ... (i)
disappearance of

=
ICHf:0CH3) (Br2) (8+) H2 +1 H21 (fast) ... (ii)
Br2,Ms-1
5 H2I+I~2HI (slow) ... (iii)
0.30 0.05 0.05 5.7 x 10-
0.30 0.10 0.05 5.7 x 10- 5 Overall reaction: H2 + 12 ~
0.30 0.10 0.10 1.2 x 10- 4 • 2. The reaction of NO and Br2 is known-to be of second
0.40 0.05 0.20 3.1 x 10- 4 order in NO and first order in Br2'
ICBSE (PMT) 20081
2NO + Br2 ~ 2NOBr
Based on these data, rate equation is : . Rate k [NO]2 [Br2 ]
(a) Rate k[CH3 COCH3 ][Br2 ][Wf The likelihood of three molecules (2 molecules of NO and 1
molecule of Br2) colliding simultaneously is far less than the
(b) Rate k[CH 3 COCH3 ][Br2 ][W]
likelihood that two molecules will collide.
(c) Rate k[CH 3COCH 3 ][W] The mechanism is believed to be as under:
(d) Rate = k[CH 3 COCH 3 ] [Br2 ]
[Ans. (c)] (fast) ... (i)

[Hint: Rate = k[CH 3COCH 3 rx [Br ]11 [H+]"


2 NOBr2 + NO ~ 2NOBr
k"
(slow) ... (ii)
5.7 x 10- 5 = k[0.30]£X [0.05]11 [0.05P ...(i)
5
Overall reaction: 2NO + Br2 2NOBr
5.7xl0- =k[0.30]£X [0.10]11 [0.05]" ...(ii)
The rate determining step involves I molecule of NO and I
1.2 x 10- 4 =k[0.30]£X [0.10]11 [0.10]" ... (iii) molecule of NOBr2 . Thus, the expected rate expression should be .
3.1 x 10- 4 = k [0.40]£x [0.05]11 [0.20]" ... (iv) =
Rate k" [NOBr2 ] [NO]
However, NOBr2 is a reaction intermediate and its
Dividing eq. (i) by eq. (ii), concentration at the beginning of second step cannot be directly

;[~J.
measured.
1 i.e, 13 = 0 Concentration ofNOBr2 will be equal to
[NOBr2] == k' [NO][Br2]

ur
Dividing eq. (ii) byeq. (iii),
If this is substituted in the above equation, we get
~ i.e, y=1 Rate = k' k" [NO] 2 [Br2 ]
= k[NO] 2 [Br2 ]
Dividing eq. (i) by eq. (iv),

S.7x 10-
5=[~J£X X[.!Jl 3. Depletion of ozone takes place in the following steps:
Overall reaction:
3.1 x 4 4
a~l
Step 1: (fast)
Thus, rate law will be
(Equilibrium constant Kc )
rate k [CH3COCH 3]1 [H+ ]1)

k
8.9 REACTION MECHANISM Step 2: 03 +[O]~ 202 (slow)
Knowledge about involved steps in a reaction, and to determine Rate =k[03 ][0] ... (i)
which step is slowest or rate determining, is called mechanism. Nascent oxygen [0] is not present in overall reaction; thus it
The intelligent guess depending on the observed rates of should be i1iminated. .
reactions about the series of steps (known as elementary
processes) leading to the formation of products is called the K c == ][0]; [0] = Kc [0 3 ] ... (ii)
[0 3 ] [0 2 ]
reaction mechanism.
For illustration a few examples of reaction mechanisms are From eqs. (i) and (ii),
given below: Rate k[O]. K [0 3 ]
1. The reaction between H2 and 12 to form hydrogen iodide 3 c [0 2 ]
was originally postulated as a simple one step reaction.
k Kc [03]2[02r1
H2 +12 =2HI =K[03]2[02r 1
Rate = k[H 2 ][1 2 ]
K=kxK c
But, the formation of HI has been explained on the basis of the
following mechanism: Order =2 - 1= I
536 G.R.B. PHYSICAL CHEMISTRY FORCOMPEnTIONS

Iii REACTIONS OF VARIOUS ORDERS (b) The time required for the reaction to be cOnlplete, i. e. ,
time at which [A] is zero.
(i) Zero order reactions
[Alo Initial concentration
A reaction is said to be of zero order if its rate is independent t Completion ~ -k- = . Rate constant
of the concentration of the reactants, i. e., the rate is proportional
to the zeroth power of the concentration of the reactants. (c) T~e units of k ~e mol L- I time-I.
For the reaction (ii) First order reactions
A ----? Products
A reaction is said to be of first order if its rate is determined by
to be 'of zero order;
the change of one concentration term only:
_ dx k[A]o =k Consider the reaction,
, dt
A ----? Products
Some photochemical reactions and a few heterogeneous Let a be the concentration of A at the start and after time t, the
reactions are zero order reactions. Such reactions are not concentration becomes (a - x), i. e. ,x has been changed into",
common. products. The rate of reaction after time 't' is given by the .
Examples: expression
1. Photochemical reaction between hydrogen and chlorine: dx
hv -=k(a-x)
H2 (g ) + Cl 2 (g) ----? 2HCI(g ) .. . dt ...• , .' .
(/);",', ·,.t·" '
This photochemical' reaction' is zero order reaction. The or --'-"=ldt
(a-x)
reaction is studied by placing H2 and Cl 2 gases over water. The
rate of reaction is studied by noting the rate at which water rises upon integration of above equation,
in the vessel due to dissolution ofHCI formed. The rate of rise of
water is the same as the rate of disappearance of Hz and C1 2 , i.e.,
I dx =kldt
(a-x)
the concentration of the gases per unit volume in the gaseous or -loge (a x)=kt+c
phase will not change with time, although the quantities will where c is integration constant.
change. When t = 0, x =0,
2. Decomposition ofN 20 on hot platinum surface: c= loge a
1 Putting the value of' c',
NzO----?N z +-Oz
2 -loge (a x)=kt-Ioge a
Rate oc [NzO]o = k[NzO]o ::;! k or log e a - log e (a - x) = kt
d[N 2 0] ::;!k a
or or loge - - = k t
dt (a-x)
3. Decomposition of NH3 in presence of molybdenum or
or
2.303 I a
k = - - oglo--
tungsten is a zero order reaction.
I (a - x)
;
2NH3 N z +3H 2
This is known its· the kinetic equation for a reaction of the first
The surface of the catalyst is almost completely covered by order. The following two important conclusions are drawn from
NH3 molecules. The adsorption of gas on the surface cannot this equation: .
change by increasing the pressure or concentration ofNH3. Thus, (a) A change in concentration unit "ill not change the
the concentration of gas phase remains constant although the numerical value of k. Let the new unit be n times the frrst one.
product. -is formed. Therefore, this reaction shows zero order
2.303 na
kinetics. So, k = - - loglo - - -
Other examples of zero order are: I n(a x)
4. Decomposition of HI on the gold surface. 2.303 a
or k=--loglO--
S. Iodation of acetone in presence of H + ions, I (a - x)
H+
CH3 COCH3 + I z ----? ICH2COCH3 + HI Thus, for first order reactions, ai1Y quantity which is
proportional to concentration can be used in place of
The rate equation of this reaction does not include [12] concentration for evaluation of' k'.
factor, i. e. ,
(h) The time taken for the completion of same fraction of
- dx =k[CH3COCH 3 ][H + 1 change is independent of initial concentration. For example,
dt for half change,
Characteristics of zero order reaction x = O.Sa and t =11/2
(a) The concentration of reactant decreases linearly with time. 2.303 I a 2.303 I 2
So, k= oglo = - - oglO
[All::;! [A]o - kt. 1112 O.5a 11/2
CHEMICAL KINETICS 537

0.693 First order growth kinetics


=--
tl/2 It is used for population growth and bacteria multiplication,
e.g.,
O~69T
or Time Population
k o a
Thus, tliZ is independent of initial concentration' a'. dt (a+x)
This time 't' in which the initial concentration becomes half is Growth rate is directly proportional to present population.
termed as half life period. Half life period of a first order dx
reaction is independent of the initial concentration of the reactant. -oc(a+x)
dt
Since, the velocity constant is independent of concentration
=k(a+x)
and depends inversely on the. time, the unit of k will be time -I,
i. e., sec-lor min -lor hour-I. The equation of the first order can ~ kdl ... (i)
(a -f x)
also be written in the following form when initial concentration is
not known: . It isa differential equation of first order and first degree in
variable separable form. It may be solved on integration.
·2.303 I (a-xI)
k=
(l z -t l )
oglO --'---..:...:..
(a-xz) J ~=kJ
(a+x)
dl+c
(~ - XI ) is the concentration after time II and (a - x2 ) the log e (a + x) = kt + c ... (ii)
concentration after time t z when t 2 > II . Here, c =integration constant
When the log of the concentration of the reactant at various
intervals of time is plotted against the time intervals, a straight
At 1=0, x=o
log e a =k x 0 + c
line is obtained (Fig. 8.13). The slope of this line gives the value
c= loge a .. , (iii)
2.303Ik, from which k can be evaluated.
Substituting the value of' c' in eq. (ii), we get
loge (a+x)=kt+loge a

1
I:
kt -log - -
a
e(a+x)
.2
N
1::
k == - 2.303 log 10
t
(_a_)
a+x
~
I:
o
u This is the kinetics for first order growth kinetics.
Note: . (1) If volwnes of reagents are given in volumetric analysis
.3'----------
lime-
then we use the following equation to determine rate constant:
2.303 I V~ - Vo
k - - oglO---
t V.. - VI
Fig. 8.13
where, Vo =:: volume used at zero time,
Examples of first order reactions Vt :; volume used at time' t' ,
I. Decomposition ofH z0 2 in aqueous solution. V.. = volume used at infinite time
Case I: When Vo is not given, we use
HZ02 ~H20+.!.02 2.303)' V~
2 k : ; - - oglO--
t V.. - VI
2. Hydrolysis of methyl acetate in presence of mineral acids.
Acid Case II: When V.. is not given, then
CH 3COOCH3 +H20~CH3COOH+CH30H
3. Inversion of cane sugar in presence of mineral acids.
r
Vo
k -- 2.303 IoglO - )
t \ VI
Acid (2) If information is given in terms of angle of rotation of
C I2 H 22 0 11 +HZO~C6HIZ06 +C6H12 0 6 optically active compounds, measured by polarimeter with
respect to time, then
4. Decomposition of ammonium nitrite in aqueous solution.
NH4 NO z ~ N z + 2H zO - 2.303
k-
t
--S1
- - Iog10 { "... - ro
r,., - 'i
5. Hydrolysis of diazo derivatives.
where, ro :; angle of rotation at zero time,
C 6 HsN=NCI + H 20 ~ C 6 Hs0H + N2 + HCI 'i = angle of rotation at time' t' ,
Note: In case of gases, pressure can be used in place of concentration. r~ = angle of rotation at infinite time
538 G.R.B. PHYSICAL CHEMISTRY FOR COMPETITIONS

Case I: If ro is not given, then Characteristics of the second order reactions

k == 2.303 )OglO{ ~r_ ~}. (i) The value of k (velocity constant) depends on the unit of
conc~ntrlition.The unit:6fk is expressed as (IIlolllitre) -I ~ time~1
t r=-li
or litre mol-I time-I.
Case II: If r_ is not given, then
(ii) Half life period (t 1/2) == ~ . 0.5a = 1
- - IoglO fro}
- 2.303
k- - . k a x O.5a ka
t li
Thus, half life is inversely proportional to initial concentration.
(3) If pressure is given in gaseous reactions, then we use the
(iii) Second order reaction conforms to the first' order when
following kinetic equation:
one of the reactants is present in large excess.
k- - - 1oglO - -
- 2.303 {po} 111..1,' k 2.303 I b(a - x)'f b th
t Po-x i:U'..mg, o g l O ' 1 a »> en
tea-b) a(b-x)
where, Po = pressure of reactant at initial stage,
(a -x)= a and (a - b)"" a
( Po x) = pressure of such a reactant at 't' time.
Values of 'Po' and 'x' can be calculated using the following 2.3031 ba
Hence, k - - oglo---
examples: ta a(b -x)
A(g) ----t B(g) + C(g) + D(g)
At t = 0 Po 000 or - - Ioglo--
.ka =k ' =2.303 b
Pressure afteitime 't' (Po'-': x) x x x t (b - x)
Pressure after a long 0 Po Po Po (since, 'a' being very large, may be treated as constant after the
time or infinite time change). Thus, the reaction follows first order kinetics with
Case I: If total pressure of reaction mixture is given in place of . respect to the reactant taken relatively in small amount.
pressure of reactant, then
Examples of second order reactions
PI == (Po - x + x + x + x)
where, PI == pressure of vessel at time' t ' . 1. Hydrolysis of ester by an alkali (saponification).
Case II: Ifpressure of vessel after a long time or infinite time is CH 3 COOC 2Hs + NaOH ~ CH3 COONa + C 2HsOH
given, then 2. The decomposition of N02 into NO and O 2 ,
P- == Po + Po + Po 2N0 2 ~2NO+02
(iii) Second order reactions 3. Conversion of ozone into oxygen at 100°C.
A reaction is said to be of second order if its reaction rate is 203 ~302
determined by th~ variation of two concentration terms. 4. Thermal decomposition of chlorine monoxide.
The kinetics oNecond order reactions are given as follows:
2C120~ 2CI 2 + O?
(a) When concentrations of both reactants are equal or two
molecules of the same reactant are involved in the change, i. e., (iv) Third order reactions
A + B ~ Products A reaction is· said to be of third order if its rate is determined
or 2A ~ Products. by the variation of three concentration terms.
dx = k(a _ x)2 , When the concentration of all the three reactants is same or
dt three molecules of the same reactant are involved, the rate
On solving this equation, expression is given as:
1 x
k=-·--- 3A ~ Products
t a(a x)
A +B+C~Products
where, a initial concentration lof the reactant or reactants and
x =concentration of the reactant changed in time t. dx = k(a _x)3
(b) When the initial concentrations of the two reactants are dt
different, i. e. , On solving this equation,
A + B ~ Products
k=~. x(2a x)
Initial conc. a b
t 2a 2 (a _x)2
dx k(a -'x)(b - x)
dt
Characteristics of third order reactions
2.303 I b(a x)
k == og 10 --'---'- 1. Halflife period
tea-b) a(b-x)
1 0.5a(2a - O.5a) =: 1
=-.-,
0.5a x 1.5a
(a - x) and (b x) are the concentrations of A and B after time
interval, t. k 2a 2 (0.5ci)2 k x 0.5a x 0.5a
I
CHEMICAL KINETICS 539
3 Let after a definite interval x mol/litre of Band y mol/litre of C
::=--
2 are formed.
2a k
Thus, half life is inversely proportional to the square of initial x !f.L
concentration. y k'],
2. The change in the unit of concentration changes the
i. e., d [B]/d [C]:;::~
numerical value of k. dt dt k2
3. The unit of k is expressed as (molllitre)-2 time-lor
Variation of concentration A, Band C with time may be
litre 2 mol-2 time-I. graphically represented as,

Examples of .third order reactions


I. Reaction between nitric oxide and oxygen. B

2.
2NO + O 2 ~ 2NO z

Reaction between nitric oxide and chlorine.


i
Cone.
C

2NO+CI 2 ~2NOCI
A
3. Reduction of FeCI 3 by SnCI 2 .
Time-
2FeCI 3 + SnCl 2 ~ SnCl 4 + 2FeCl 2 Fig. 8.14

Expressions for rate constant for reactions of different Examples:


orders: OH
Reaetlon

A ~ Products
Order Rate laweqn. Expreulon for rate eon.t.

0 Rate =k k= ![[A]o-[A]]
OH kl ONO,
~I (Main) .

k =2.303 10LA]o (i)


o~o, OH

0"
A ~ Products Rate = k[A]
t g [A]

=H[~] -[1]J
. k2
2A ~ Products 2 Rate = k[ A ] 2 k
(Side)

A+ B~ _ k =2.303· 10 [Blo[A] I
Products 2 Rate -.k[A][B] t([Alo-[B]o) g [A]o[B]

3A ~ Products

Parallel or Competing reaction


The reaction in which a substance reacts or decomposes in Consecutive reaction
more than one way are called parallel or side reactions,
kl
This reaction is defined as that reaction which proceeds from

AC: reactants to final products through one or more intermediate


stages. The ovemll reaction is a result of s~veral successive or
consecutive steps.
A~B~Candsoon

-d [A] :;:: (k l + k2 )[A]::= kay [A] Examples: (i) Decomposition of ethylene oxide:
dt kl
(CH 2 hO~ CH3CHO
kl ::= fractional yield of B x kay k2
k2 :;:: fractional yield of C x kay CH3CHO~ CO+ CH 4
If kl »k2 then, (ii) The pyrolysis of acetone:
A~Bmain and
(CH3)2CO~CH4 +CH 2 =C=0
A ~ C is side reaction Ketene
540 G.R.B. PHYSICAL CHEMISTRY FOR COMPETITIONS

1 First possibility :
CH 2 =C=O--7 C 2 H4 +CO
2 Step 1 A ----"7 A' ----"7 B
Step 2 B ----"7B' ----"7C

1
Potential
Energy

Tlme-
Fig. 8.15 c
Variation of concentration of various substances during the
progress of reaction (A ~ 8 ~ C ) Reaction coordinate -

''I kz
Fig. 8.17
A~B~C
Activation energy of step 1 is greater than that of step 2 thus
Let initially (t =0), [B] =0 and kl < k2 then maximum
step 1 will be slow and rate determining.
concentration of[B] may be calculated as,
Second possibility :
k ] kZlkJ kz
[B]max [A]O-1 s*
[ k2
where, [A]o =initial concentration of A

as,
Time in which B attains maximum concentration may be given 1
potential
Energy
t =k22.303 Iog - (k2)
max - k J kJ
c
Concentration of[B] after time 'f' may be calculated as,
Reaction coordinate -
[B]t k\ [A]o [e-kJt e-k2t]
Fig. 8.18
k2 k\
Activation energy of step 2 is greater than that of step 1 hence
Activation energy diagram of a composite reaction involves step 2 will be slow and rate determining
more than one peaks and number of peaks indicate the number of
different types of activated complexes involved. The numher of Reversible Reactions
valleys in the activation energy diagram indicates the number of The reactions in which the products of chemical change react
different types of reactive intermediates involved. together to form the original reactants, are called reversible
reactions. These are also called opposing or counter reactions .
.Let us consider a reversible reaction in which both forward
* and backward reactions are of fIrst order.
* ~ Activated complex kJ

1
Energy
I ~Intermediate
AH =+ve, endothermic
reaction
A~B
k2

Initial state (I = 0) a 0
Conc. at time (t) a-x x
Equilibrium conc. (a - Xe )xe
Reaction coordinate -

Fig. 8.16 Activation energy diagram .


-t- oglo {Xe}
. 2.303 1
kJ + k2 = --
Xe -x
of a two steps composite reaction
Energy Diagram of two Steps Reaction eMl METHODS FOR DETERMINATION OF
Let us consider a reaction of the type : ORDER OF A REACTION
A. Step I ) B Step 2 ) C (Exothermic) The important methods used are the following:
These are two possibilities in this reaction
I

CHEMICAL KINETICS 541


1. Method of integration (Hit and trial method) Taking logarithms on both sides,
The most simple method is the one in which the quantities log 10 (t 112)' - log 10 (t 112 h = (n - l)[log 10 a2 - log load
a, x and t are determined and substituted in the kinetic equations log 10 (t 112)' - log 10 (t 112 h
of various orders. The equation which gives the most constant (n 1)= - - - - - - - -
value for the specific rate constant (k) for a series of time log 10 a2 -logIO al
intervals is the one corresponding to the order of reaction. If all loglo (t1l2)1 -loglo (l1I2h
the reactants are at the same molar concentrations, the kinetic or n 1+ ... (iv)
log 10 a2 - log 10 a l
equations are:

lV' lV'
Plots of half lives vs. concentration (t 112 ex: a 1- n) :
2.303
k=- - Iog 10 - -a . , fi d .
- ,~or lrst or er reactions;

k=
t

l[_I__ l]
(a - x)

for second order reactions;


l Zemo~, ll'~o~,
t (a-x) a '
Conc.-+, Conc.-+ 1/a-+ 1/a2 -+

k =I [ I _
2t (a-x)2 a
~] , for third order reactions. This relation can be used to determine order of reaction 'n'.
4. van't Hoff differential method
As we know that, the rate of a reaction varies as the nth power
2. Graphical method of the concentration of the reactant where' n' is the order of the
reaction. Thus, for two different initial concentrations C 1 and C 2'
A graphical method based on the respective rate laws can also
equations can be written in the form
be used.
If the plot of log (a - x) versus 't' is a straight line, the _ dC, =ken and _ dC 2 =kCn
dt I dt 2
reaction follows first order.
Taking logarithms,
If the plot of _1_ versus' t ' is a straight line, the reaction
d~l ) =log 10 k +n loglo CI
(a x)
loglo (- ... (i)
follows second order.
If the plot of I versus' t ' is a straight line, the reaction
(a-x)2 and ... (ii)
follows third order.
In general, for a reaction of nth \)rder, a graph Subtracting eq. (ii) from eq. (i),

versus 't ' must be a straight line.


3. Half life method
A general expression for the half life, (11/2 ), is given by
1
t 112 oc---;;-:t or , ... (iii)
a 10gIO C1 -loglO C2
where, 'n' is the order of the reaction.
Starting with two different initial concentrations al and a2 for dCI
- - - and - -dC- 2
are determlOe
. d from concentration
. vs.
. time
the same reaction, the half lives are (t112)1 and (11/2 h dt dt
respectively are determined. As we know that, graphs and the value of' n' can be determined.
1
(t 112)1 oc ---;;-:t ... (i) :: ::::I_SOME SOLVED EXAMPLES\I::::::
a1
1 . I£xample 20. Rate of a reaction A + B ~ product, is
and ( tll2h oc _11-1
_ ... (ii) given as a function ofdifferent initial concentrations ofA and B.
a2
[A] [B] Initial rate
Dividing eq. (i) by eq. (ii), (mol L- 1) (moIL-I) (moIL -1 min-I)
,,-1
(t1l2 )1 a2
... (iii) 0.01 0,0} 0.005
(t1/2 h = ( ~)
0.02 0.01 0.010
0.01 0.02 0.005
I
542 G.R.B. PHYSICAL CHEMISTRY FOR COMPETITIONS

Determine the order ofthe reaction with respect to A and w. r. t. -3 - 2.303 I 0.062
6 .2 x 10 - - - oglO
B. What is the half life ofA in the reaction? 100 PI
Solution: Let the rate of reaction be
6.2 x 10-3 x 100 . ,
R~te = k[AY[B]Y or 2.303 = log 10 0.062 - log 10 (PI)
From the data given, it is clear that by doubling the
concentration of A, the rate also becomes double when B is kept or 0.2692 = log 10 0.062 - log 10 (PI)
constant. Thus, the rate is directly proportional to concentration
ofA. or log 10 (~ ) =log 10 0.062 0.2692
Rate oc: [A], i.e., x 1 = (2.7924 - 0.2692)
Or the order of reaction w.r.t. A is 1.
When the concentration of A is kept constant and the PI =0.033 atmosphere
concentration of B is doubled, the rate does not change, i. e. , Pressure after 100 sec = 0.033 atm
y = 0; or the order of reaction w.r.t. B is zero.
dx Example 23. The half life of a first order reaction is 60
Thus, reaction rate, - ~ = k[ A] min. How long will it take to consume 90% of the reactant?
dt
Solution: For the first order reaction
Again k-..:. 0.005
- - -- 0 .5 mIn
. -I
0.01 k = 0.693 0.693 == 11.55 x 10-3 min- I
Half life of A := 0.693 0.693 = 1.386 min tl/2 60
k 0.5 Applying first order kinetic equation,
. Example 21. Thermal decomposition of a compound is of 2.303 a
t =--log 10 - -
the first order. If 50% of a sample of the compound is k (a-x)
decomposed in 120 minutes, how long will it take for 90% of the
compound to decompose? Given: a= 100,x== 90, i.e., (a-x)=(100-90) 10
Solution: Half life of reaction = 120 min Hence, t 2.303. log 10
We know that, 11.55 X 10-3 10

= 199 min
k -- 0.693 -_ -
0.693
-- :... 5. 77 x 10-3 mIn
·-1
tl/2 120 . Example 24. A first order reaction has a rate constant of
15 x 10-3 S -I. How long willS.O g ofthis reactant take to reduce
Applying first order reaction equation, to 3.0 g?
2.303 a Solution: Applying first order kinetic equation,
t = log 10
k a-x 2.303 a
1
k oglo(a_x)
Ifa=100, x 90or(a-x)=IO,
Given: k::::: 15x 10-3 sec- I ,a =-5 g,(a -x)::::: 3 g
S0, 2.303 . Iog 10 10 2.303 :::::
399'
mIn
5.77 x 10-3 5.77 x 10-3 2.303 5
So, t= 3 log 10 :::: 34.07 sec
15x 10-: 3
Example 22. The decomposition ofCl20 7 at 400K in the
gas phase to Cl 2 and 02 is a first order reaction. . Example 25. Catalytic decomposition of nitrous oxide by
(i) After 55 seconds at 400K, the pressure ofCl20 7 falls from gold at 900°C at an initial pressure of 200 mm was 50% in 53
0.062 to 0.044 atm. Calculate the rate constant. minutes and 73% in 100 minutes.
(ii) Calculate the pressure of Cl 20 7 after 100 seconds of (a) What is the order of reaction?
decomposition at this temperature. (b) How much it will decompose in 100 minutes at the same
Solution: (i) As pressure oc concentration, temperature but at an initial pressure of600mm? (lIT 1990)
. Solution: (a) Using first order kinetic equation and
- 2.303 1 Pi (initial pressure) substituting given values,
k - - - og 10 --'----=-----'----
PI (pressure after time t) In first case: k = 2.303 10 200 = 0.0131 min- I
53 glO 200-100
- -
- - . IoglO -
2.303 0.062
-- - 62
. x 10-3 s-1
55 0.044 In second case: k = 2.303 10 200 = 0.0131 min -I
100 glO 200-146
(ii) Again applying the first order kinetic equation,
As the values of k come out to be the same in both cases, the
2.303' Pi (initial pressure)
k= log 10 -.- - - - - - - - reaction is of first order.
t Pt {pressure after time t)
CHEMICAL KINETICS 543

(b) As in the first order reaction, the time required for the Example 28. In Arrhenius equation for a certain reaction,
completion of same fraction is independent of initial the values ofA andE a (activation energy) are 4 x 1013 sec- 1 and
concentration; the percentage decomposition in 100 minutes
98.6kJ mol- 1 respectively. If the reaction is of first order, at
when the initial pressure is 600 rum will also be 73%.
what temperature will its halflife period be 10minutes?
. Example 26. A substance reacts according to the first
order rate law and the specific reaction rate for the reaction is (liT 1990)
Solution: According to Arrhenius equation,
I x 10-2 S-I. Ifthe initial concentration is 1.0 M
k= Ae-Ea IRT
( a) What is the initial rate?
(b) What is the reaction rate after I minute? E
or loge k == loge A _ _
a
Solution: (a) Initial rate of a first order reaction kC RT
=IxI0-2 X 1.0= Ix 10-2 molL-1 S-I
. . Ea
or 2.303 log 10 k = 2.303 log 10 A - RT
(b) Concentration after 60 seconds is calculated by applying
.
first order kinetic equation, For a fiIrst ord er reactIOn t 112 = 0.693
k
k 2.303 1
60 1oglo (I-x) So, _,'
_,'1.
It --
0.693
sec
-I (
t 112 10min 600 sec)
.600
or 1x 10-2 = 2.303 log (1- x)] 1.1 x 10-3 sec-I
60 3
60x 10-2 Hence, Iog (l.lx.'.:10-3 ) == 1og (4 x 1013 ) - _ _
98_.6_x__
10__
or - - - = -log (I-x)= 0.2605 2.303 x 8.314 x T
2.303
T= 31O.95K
log (1- x) = - 0.2605 Example 29. A second order reaction, in which both the
= 1.7395 reactants have same concentration, is 20 % completed in 500
seconds. How much time it will take for 60% completion?
(1- x) antilog of (1.7395) = 0.5489 mol L- I
Solution: The second order equation when both the
Rate of reaction after 1 minute = k x C reactants have same concentration is

=I X 10-2 x 0.5489 k~. x


t a(a - x)
= 5.489 x 10-3 mol L-:I S-I
If a = 100, x = 20, t == 500 seconds.
Example 27. Afirst order reaction is 50% completed in 30 So, k==_I_ x 20
minutes at 27°C and in IOminutes at 47°C. Calculate the reaction 500 lOOx (100 20)
rate constant at 27°C and the energy ofactivation ofthe reaction When a 100, x == 60, t == ?
in kJ mol-I.
1 60
. 0.693 t==-·---
Solution: For first order reaction k = - - k 100 x 40
tl/2
Substituting the value of k,
k 270C 0.693
=- - == 0.0231 mIn
. -I
30 500x 100 x 80 60
t= x---
20 l00x40
k 470C =0.693
- - = 00693
. . -I
mm
10 or t == 3000 seconds
Now applying the following equation:
1 kl r
T2 Ii
og 10 k2 = 2.303 x R . \ T2 . Tl
J.
Example 30. A first order reaction is 20% complete in 10
minutes. Calculate the time taken for the reaction to go to 80%
completion.
or I 0.0231 (320 - 300 \j Solution: Applying first order equation,
og 10 0.0693 2.303 x 8.314' 320 X 300
k - 2.303 10 100
20 - t· g 10 (100- 20)
or -loglO 0.3333 = x~-
19.1471 96000
- 2.303 1 100 _ 0 0223 . -I
Ea = - 19.1471 x 96000 x log 0.3333 -W oglo 80-' mm
20
91906x (- 0.4772)
= 43857 J mol-I = 43.857kJ mol-I
544 G.R.B. PHYSICAL CHEMISTRY FOR COMPETITIONS

Again applying first order equation, . Example 33. The gas phase decomposition of dimethyl
2.303 100 ether follows first order kinetics.
t==-k- 1oglO (l00-80)
CH30CH3 (g) -----t CH4 (g ) + H 2 (g ) + CO(g)

== 2.303 I 100 The reaction is carried out in a constant volume container at


0.0223 og 10 20 500 0 e and has a half life of14.5 minutes. Initially only dimethyl
ether is present at a pressure of 0.40 atm. What is the total
= 72.18 min pressure after 12 minutes? Assume ideal gas behaviour.
Example 31. The decomposition of dinitrogen pentoxide (lIT 1993)
(N 20 5 )follows first order rate law. Calculate the rate constant Solution: k 0.693 = 0.693 = 0.047793 min- I
,f; om the given data: tU2 14.5

-
t 800 sec [N 20 S ] = 1.45mo/L-I =[Ad Let the pressure of dimethyl ether after 12 minutes be p atm.
Applying first order equation,
t=1600sec [N 20 S ] = 0.88 mol L- l =[A 2 J 2.303
k=--loglo
t P
Solution: Applying the formula,
log 10 0.4 0.047'93 x 12 0.2490
'-
k-
2.303 I [Ad P 2.~03
oglo--
(t2 tl ) [A 2 ]
or 0.4 =1.7743
= 2.303 I 1.45 P
P = ~ = 0.2254 atm
(1600 800) og 10 0.88
or
1.7743
= 2.303 x 0.2169= 6.24 x 10-4 sec-I
800 Decrease in pressure, x = 0.4 - 0.2254 == 0.1746 atm
CH 3 0CH 3 (g) -----t CH 4 (g)+ H2(g)+ CO(g)
Example 32. The decomposition ofN 20 5 according to the
Po x x x x
equation, Total pressure = Po + 2x
N 20S (g) -----t 4N0 2 (g) + O 2 (g)
= 0.4 + 2x 0.1746
is a first order reaction. After 30 minutes from the start of the
=0.7492atm
decomposition in a closed vessel, the total pressure developed is
found to be 284.4 mm Hg and on completion, the total pressure is , . Example 34. The half life offirst order decomposition of
584.5 mm Hg. Calculate the rate constant of the reaction. nitramide is 2.1 hours at 15°C.
(lIT 1991)
Solution:
If6.2g ofNH2N02 is allowed to decompose calculate (i) time
On decomposition of2 moles ofN 2 0 s , 4 moles ofN0 2 and 1 taken for NH2 N0 2 to decompose 99% and (U) the volume ofdry
mole of O 2 are produced. Thus, the total pressure after N 20produced at this point, measured at STP. (lIT 1994)
completion corresponds to 5 moles and initial pressure to 2 . (.)
SoI ution. 1 k - 0.693
------- .
- 0.693 - 0 33 hr- I
moles. . t 112 2.1
Initial pressure ofN2 0 s , Po = ~ x 584.5= 233.8mmHg Applying kinetic equation of first order reaction,
5 2.303 a
k==--, log 10 - -
After 30 minutes, the total pressure = 284.5 mm Hg t (a-x)
2N 2 0 S -----t 4N02 + 02 2.303 I 100
or
PO-2P 4p P t = 0.33 oglO (100- 99)
or Po + 3p 284.5 =l3.96hrs
or 3p= 284.5 233.8= 50.7mmHg (ii) No. of moles ofNH 2N0 2 decomposed

or p= 50.7 16.9mmHg = 0.99 x 6.2


3 62
Pressure ofN 2 0 s after 30 minutes = 233.8 - (2 x 16.9) =0.099

/ =200mmHg No. of moles of N2 0formed == 0.099

- 2.303 I
k --30 233.8 - 5 2 10-3 ·-1
oglO - - - . x mm
Volume of N2 ° at ~TP =O.099'x 22400 mL
200.0 = 2217.6mL
I
CHEMICAL KINETICS 545

Example 35. From the following data for the reaction


A '
or -= 4.29 x 10 9
between A and B: k
or A = 4.29 X 109 x 2.67 X 108
Initial rate (mol L- I S-I)
Expt. [A] [B] =1.145 X 1018
No. (moILI) (molL-I)
'-
...;!.
,c. 300K 320K Example 36. At a certain temperature, the half change
period for the catalytic decomposition ofammonia were found as
(1) 2.5 x 10-4 3.0 X 10-5 5.0 X 10-4 2.0 X 10-3
follows: '
(2) 5.0 x 10-4 6.0 x 10-5 4.0 X 10-3 Pressure (Pascals): 6667 13333 26666

1.0 x 10-3 Half life period in hours: 3.52 1.92 1.0


(3) 6.0 X 10-5 1.6 X 1{;-2
Calculate the order ofreaction,
Calculate the following:
(z) The order of the reaction with respect to A and with Solution:
'('~::::;l~
) ( )n-l where, n is order of reaction
respect to B, , (tll2 h a1
(il) The rate constant at 300K,
(iii) The energy ofactivation and , From the given data,
(iv) The pre-exponentialfactor. (liT 1994) 3.52 __ (13333Jn-l
(a oc initial pressure) ,
Solution: (i) Let the rate law be: 1.92 6667
Rate = k[AY [BJY (2t- 1
From expt. (1), 5.0 x 10-4 =k[2.5 x 10-4 Y [3.0 x 10-5 JY ... 0)
log ::~~ (n-I)log2
From expt. (2),4.0 x 10-:3 k[ 5.0 x 10-4 n 6.0 x 10- Y 5
... (ii)
, -3 = 0.3010 x (n -I)
D IVl 1, 4,.0 x 10
11 by eq. (.)
· 'd'mg eq. '(") 2x 2Y
=.::::; 8 0.2632 = 0.3010 x (n I)
, 5.0 X 10-4
n 1.87 z 2
From expt. (3),1.6 x 10-2 ::::; k[l.O X10-3 Y [6.0 X 10-5 Y :.. (iii) Similar calculations are made between fITst and third'
2 observations. n comes equal to 1.908 (z 2) .
Dividing eq. (iii) by eq. (ii), 1.6 x 10- ::::; 2X = 4
Thus, the reaction is ,of second order.
4.0x 10-3
" Example 37. On heating, arsine (AsH} ) decomposes as:
or x=2 and y=1 2AsH3(g)~2As(s)+ 3H2 (g)

Hence, order w.r.t. A is 2nd and order w.r. t. B is 1st. The total pressure measured at constant temperature and
constant volume varies with time as follows:
(ii) Rate = k· [A]2[B]
t (min)
pi mm Hg 760
° 5
836
7.5
866.4
10
896.8
From expt. (I ) , 5 x 10-4 = k[2.5 x 10-4]2 [3.0 X 10-5 ]
Calculate therate constant assuming the reaction to follow the
. 5x10-4 , firs/order rate law.
or k= =2.67x10 8 L2 mol-2 S-1
[2.5 x 10-4]2 [3.0 X 10-5 ] Solution: For fITst order, the,rate equation is
Initially after time t
(iii) From Arrhenius equation,
2.303 1 [A]o 2.303,1
I 2.0x 10-3 , 20
k - - og--= og
t [A] t P
ogIO 5.0x 10-4 ::::; 2303 x 8.314 x 300x 320
given, Po == 760rrmi Hg.
2.3,03x8.314x300x320 I 4 The decomposition reaction is:
- - - . , . - - - - - - - - x og 10
20,
2AsH3 (g) --? As(s) + 3H2 (g)
=55.333 k1 mor l . Po o 0
3x
" PO-2x
(iv) Applying log 10k = log 10 A - --=-- Total pressure, PI Po - 2x+ 3x= Po +X
, , 2.303RT
,A ' 55.333 X=PI-PO
10glO ,- '" :::: 9.633
k' 2303 x 8.314 x 300 PAsH] (Po - 2x)= Po - 2Pt + 2po= 3po - 2pt
I
546 GR.B. PHYSICAL CHEMISTRY FOR COMPETITIONS

After 5 minutes, PAsH) (3 x 760) (2 x 836) titrated with a standard alkali s.olution. The following results
were obtained:
==608mmHg
Time (min) 0 75 119 183 00

k == 2.303 log \0 760 0.0446 min- I Alkali used (mL) 19.24 24.20 26.60 29.32 42.03,
5 608 Show that the reaction follows first order kinetics.
After 7.5 minutes,PAsH) (3 x 760) - (2 x 866.4) Solution: In case the hydrolysis follows first order kinetics, then
2.303 2.303 I V= -
== 547.2 mm Hg k == 1oglO - - oglo - - " -
t [A] t V"" - VI
k 2.303 10 . 760 == 0.0438 min -I
7.5 glo 547.2 When, t == 75 min, V"" = 42.03, Vo == 19.24, VI 24.20
After)O minutes, PAsH 3 (3 x 760)(2 x 896.8) k 2.303 (42.03 - 19.24) = 2.303 10 22.79
10
486.4mmHg 75 glO (42.03-24.~o') 75 glO 17.83

2.303 == 0.00327 min -I


k - - 1oglo -760
,- -- 0.0446 mm
'. -I
'10 486.4 t 119min, V<x> = 42.03, Vo 19.24, VI == 26.60
" Example 38. Cane sugar is gradually converted into , 2.303 1 (42.03 - 19.24) 2.303 1 ,22.79
. dextrose and laevulose by dilute acid. The rate of inversion is k 119 oglO (42.03 .... 26.60)= 119 ogiO 15.43
observed by measuring the polarisation angle, at various times,
0.00327 min- I
when the following results are obtained:
Since, the values of k are constant, hence, it follows first order
Time (min) 0 10 20 30 40 100 00
kinetics.
Angle 32.4 28.8 25.5 22.4 19.6 -6.1 -14.1 , Example 40. The gaseous decomposition of ozone
Show that the reaction is offirst order. Calculate the value oft, 20 3 ~302
when the solution is optically inactive. d[03] ]2,
Solution: In case, the inversion of cane sugar is a first order obeys the rate law r ---==~..::..;;;.-
dt [0 2 ]
change, then
2.3031 2.303 1 Show that the following mechanism is consistent with the
k == oglo - - oglo , above rate law: '
t [A] t 'i r~ Keq \
0 3 ~ O2 + 0 (fast)
When, t 10, ro = 32.4, 'i == 18.8, ,r"" == -14.1
kJ
k - 2.303 I 32.4 - (-14.1) 0+0 3 ~202 (slow)
-ill oglO 28.8- (-14.1) Solution: From the slow rate detennining step
2.3C3 1 46.5 0008 . -1 r==_d[03]=k d
10 ogiO 42.9 ==. mm dt ,¥,

From the fast reaction,


When, t = 20, ro 32.4, 'i 25.5, r"" == 14.1
k 2.303
1
32.4 - (-14.1) 2.303 i46.5
20 oglO 25.5 (-14.1) 20 og 39.6
or
== 0.008 min -I
Substituting the value of [0] in the above expression
When, t 30, ro 32.4, 'i 22.4, r= == -14.1 , d[03] kIKeq[03]2 k[03]2
r==--- = =---
k _ 2.303 _32_.4_..::-(-_14_'-:;,.1) 2.303 I 46.5 dt [0 2 ] [02]
10
- 30 glO 22.4 (-14.1) 30 ogiO 36.5
Example 41. For the formation of phosgene from CO(g)
I and chlorine,
== 0.008 min-
CO(g) + C!2 (g ) ~ COCl2 (g )
Thus, the reaction is of first order as the value of k is constant. the experimentally determined rate equation is,
The solution will be optically inactive when half of the cane
d[COCI2 1== k[CO][CI ]312
sugar is inverted. • dt 2
3
,t1l2 == 0.693 0.67 == 86.6min Is the following mechanism consistent with the rate equation?
k 0.008
(i) Cl2 2CI (fast)
Ex.mple 39. 1 mL of methyl acetate was added to a flask
containing 20 mL of NI20 HCl maintained at 25 0 C. 2 mL of the, (ii) CI + CO ~ COCI (fast)
reaction mixture were withdrawn at different mtenJals and (iii) COC! + Cl 2 ~ COCl2 + Cl (slow)
CHEMICAL KINETICS 547

Solution: Multiplying equation (ii) by 2 and adding (i), we 10 (4.5 x lO- J_'.
IO
Eo 1 I]
get: g 1.3 X 10- 11 - 2.303 .~ 8.314 x 10-3 543 623
Cl 2 + 2CO~ 2COCI
Ea =120 kJ]
K = [COClf ~26. The reaction of0 3 with chlorine atom is given as:
[Cl z][CO]2 0 3 (g) + CI(g) - - t 02(g) + CIO(g);
[COCI] = (K)1I2 [CI 2 ]V2 [CO] ... (i) " kl = 5.2x 10 9 L mol-I sec- I
Slowest step is rate determining, hence, C10(g) + O(g) ~ CI(g) + 02(g);
Rate k[COCl][C1 2 ] ... (ii) k2 ='2:6x 1010 Lmol- I sec- 1
From eqs. (i) and (ii), we.get Which of these values is closest to the rate constant of the
Rate = kK 112 [e1 2 ]lI2 [C1 2 ][CO] overall reaction?
Rate = k' [CI 2 ]3J2[CO] 03(g) + O(g)--t 20 2 (g)
., (a) 5.2 x 109 • (b) 2.6 x 1010
Thus, rate law is in accordance with the mechanism.
10
(c) 3.lx 10 (d) 1.4 x 1020
[Ans. (a)]
. if

22. A first order reaction has half life of 14.5' hrs .. What [Hint: Lowest value of k shows that the step is rate detennining.j
percentage of the reactant will remain after 24 hrs? 27. If a first order reaction takes 32 minutes for 75% completion,
(a) 18.3% (b)31.8% (c) 45.5% (d) 68.2% then time required for 500/0' completion is:
. [Ans. (b)] [A'VIU (Medic.al)
(a) 32 min (b) 16 min (c) 8 min (d) 4 min
[Hint: k = 2.303 log [_a_) [Ans. (b)]

0.693
t

2.303 I
a

100
x
. [Hint: k = 2.303 log
t a
(_a_) x
og
14.5 24 (a-x)
When, a = 100, x = 75, t = 32
On solving, (a":'x) 31.8%]
k - 2.303 I . (100.) -00433 ·-1
23. Half life of a first order reaction is 10 min. What % of reaction - 32 ogiO 2s -. mm
will be completed in 100 min?
(a) 25% (b) 99.9% (c)75% (d) 80% TIme lor 01/0 compIetlOn
· "50 ' (t1l2 ) = -
0.693 0.693 . ]
- = - - '" 16mm
[Aus. (b)] . k 0.0433
2
28. Rate constant of areaction is l751itre- mol- 2 sec-I. What is
[Hint:
0.693
- .-
2.303 I
- - oglO
[a
--I1 the order of reaction? reET (,~'; , ' , ' ; ' . [
tl/2 t a - x) (a) First (b) Second (c) Third (d) Zero
[Ans. (c)]
0.693 "" 2.303 10 (~)
10 100 g 100.,.. x . . [litre] n-I _
[Hmt: Dmt of k - x sec I ... (i)
x = 99.9%]
mol

24. A certain zero order reaction has k = 0.025 M s-I for the Given unit of k = litre2 mol- 2 sec- I ... (ii)
disappearance of A. What will be the concentration of A after Comparing theI1llit of litre,
15 seconds if the initial concentration is 0.5 M? I 2 n
(a)0.5M (b)0.32M (c)0.12M (d)0.06M
n = 3]
[Ans. (c)]
29. The half life of a fIrst order reaction having rate constant
[Hint: x = kt = 0.025 x 15 0.375 M 5 I
k = 1.7x 10- sec- is: (.J!PYIER 2006)
Remaining conc. = .0.5 - 0.375 0.125 M] (a) 12.1 hrs (b) 9.7 hrs (c) 11.3 hrs (d) 1.8 hrs
[Ai1S~ (c)]
25. A first order reaction:
2N zO(g) - - t 2N 2(g) + 02(g) . 0.693 0.693 4
[H mt:. tli2 = = 5 = 0764 sec =11.3 hrs]
has a rate constant of 1.3 x 10- 11 8- 1 at 270"C and . k 1.7 x 10-
4.5 x 10- 10 at 350°C. What is the activation energy for this 30. At500 K, the halflife period oJ a gaseous reaction at aninitial
reaction? pressure of80 kPais 350 sec. When the pressure is 40 kPa, the
(a) 15kJ (b) 30 kJ (c) 68 kJ (d) 120 kJ half life period is 175 sec; the order of the reaction is:
[Ans. (d)] !PET (Kerala) 201)71
(a) zero (b) one (c) two {J) three
[Hint: 'log (k 2 J =~ 1 1] (e) half
k1 2.303R TI T2 [Ans. (a)]
548 G.R~B. PHYSICAL CHEMISTRY FOR COMPETITIONS

[Hint:
(tli2)l _(Pz)n-l [Hint: k = 2.303 log a
t a-x
(tli2h Pi
2.303 1 100
350 f 40\"-1 70 og 100- 90
175 lso; . = 0.0329 min- ]
i

2=(~r-'1 . 32. The half life period of a first order reaction is 1 min 40
seconds. Calculate its rate constant. 2008)
n 1=-1 (a) 6.93 x 10- 3 min-I (b) 6.93 x 10- sec-I
n = 0 (zero order reaction)] (c) 6.93 x 10- 3 sec (d) 6.93 x 103 sec
31. 90010 of the first order reaction is corp.pleted in 70 minutes. The [Ans. (b)]
velocity constant of the reaction is: [Corned (Karnataka) ~{)i)81
(a)0.0329 (b) 0.329 (c) 3.29 (d) 0.0293 [Hint: k = 0.693 0.693 = 6.93 xlO- 3 sec-I]
t1l2 100
. [Ans. (a)]

MISCELLANEOUS
' .
NUMJ~CAL EXAMPLES~ H~
. ~

Example 1. Ammonia and oxygen react at .higher la) What is the energy ofactivation ofr rea~tiqn?
temperature as, (b) What is the value of kat 670 K? ~.
4NH3 (g) + 502 (g) -----7 4NO(g) + 6H 20(g) Solution: (a) We know that,
in an experiment, the. concentration of NO increases by E
log 10 k=log 10 A - - - - (i)
1.08x 10-2 moilitre- I in 3 seconds. Calculate:
1 ...
. . 2.303RT
(i) rate ofreaction, (tl) rate ofdis.appearance ofammonia,
loglO k (s-I )= 14.34 ~ 1.25 x 10
4

(iii) rate offormation ofwater. ... (ii)


T
Solution:
Comparing eq. (i) and eq. (ii), we get
(i) Rate = _ .!.d[NH3 ] = 1 d[021=.!. d[NO] =.!. d[H 2 0]
_E_ 1.25 x 10" .
. 4 dt 5 dt 4 dt 6 dt .
2
2.303R .
. R ate = -1 x. 1.08 X 10- = 9 x 10- 4 mo11'Itre -I sec-1 E = 1.25 X 10 4 X 2.303 x 8.314 x 10-3
. 4 . 3 E = 239.339kJ/mol
(ii) ...: d [NH 3 ] ;" 4 x rate 4 x 9 X 10-4 (b) Substituting the value ofT, i.e., at 670 K, in equation (ii),
dt we get,
= 36 x 10-4 mol litre-1 sec-I 4
logk(s-I) 14.34_1.25X10 ='-4.3167
(iii) d[H 2 0] =6 x rate = 6 x 9 X 10-4 670 .
dt k::::: 4.82 X 1O-s s-I
== 54 X 10-4 mol litre -I sec- 1
. Example 4. A drug becomes ineffective· after 30%
Example 2. Decomposition of NzOs (g) into NOz(g) and decomposition. The original concentration ·of a sample was
02 (g)' is a first order reaction. If initial concentration of 5 mg/mL, which becomes 4.2 mg/mL during 20 months.
NzOs(g), i.e., [N2 0S]o is 0.03 mol litre-I, what will be'it$ Assuming the decomposition offirst order, calculate the expiry
concentration after 30 minutes? Rate constant of the reaction is time of the drug in months..What is the halflife of the product?
1.35 x 10-4 S-I.
.SOlutl·on.; 2.303 1oglo .(.-a-) =2.303
k ==--. 5 '1
- - IoglO ( -I
t .. a-'x 20 4.2;
Solution: k == 2.303 log 10 [Ao]
t [A] = 0.00872 month-I
2.303 1 . 0.03 Expiry time 't' may be calcUlated as:
1.35 X 10-4 2.3031' a
30x 60 oglO [A] k = - - oglO--·
t a-x
[A] = 0.0235 mol litre-I 0.00872 = 2.303 log 10 100
f
t· 70
Example 3. Rate. constant. lor the decomposition of
ethylene oxide into eH4 and CO may be described by the t::::: 40.9 "" 41 month
4
. Iog k( s -1) ·1434 1.25 x 10 0.693
equatlOll, . - - -- -. tl/2 = ::::: 79.4 month
T 0.00872
CHEMICAL KINETICS 549

Example 5: Two reactions of the same order have equal SolutiQIl: We know that,
exponential factors but their activation energies differ by 24.9
kJ I mol. Calculate the ratio between the rate constants of these k --'- 2.303 Ioglo -a-
t a-x
l' J
reactions at 27 0 C. (R = 8.31J K -I mol-I) (Dhanbad1990)
Solution: We know that, At 298 K, x 10, a = 100,
E 2.303 100
loglO k - 10gIO A k298 = - - log 10 - .. ' (i)
2.303RT tl 90

So, loglo k2 = log 0 A - At 308 K, a = 100, x = 25, (a - x)= 75


1 2.303RT
1 A E\ - - IoglO (100)
2.303 - ... (ii)
and log lo kl = oglO - 2.303RT t2 75
t 2 ,dividingeq.(ii) by eq.(i)
1 (k J (EI -E
2
ogJO k: = 2.303RT
2)
k308' :::: 2.73
k298
J
1
og 10
( k2
lk:
24.9 x 1000
= 2.303 x 8.314 x 300 log
k298
_E_ ( ~ -
l
2.303R TI ' T2
l,J
k2 =2.199x 104 10 2.73 = E. (_1___1_)
kl
g 2.303 x 8.314 308 298
Example 6. Pseudo first order rate for the reaction.
E = 76.622kJ/mol
A+B~P
Similarly, we can solve for k318 which is equal to
when studied in 0.1 M ofB is given by 9.22 x 10-4 S-I.
d[A] = k[A] Example 8. The rate constant ofa reaction is 1.5 x 107 s-I
dt at 50°C and 4.5x10 7 S-I at 100°C. Calculate the Arrhenius
where, k =1.85 x 104 sec -I . Calculate the value of second order parameter A and Ea' ' (HT 1995}
rate constant. . ,k2 ( 1 1
Solution: Solution: log 10 - = -'-
k, 2.303R TI T2
_ d[A] = k[A]
dt
4.5 X 10
7
: (1 1), ,
log 10 1.5 X 107 = 2.303 x 8.314 323 - 373
d[A] = 1'.85 x 104 x [A] ... (i)'
dt Ea = 2.2x 104 J/mo1
Assuming the reaction to be of second order, We,know that,
_ d[A] = k'[A][B] log 10 k = log 10 A - -......=..-
dt 2.303RT
2.2x 104
_ d[A]=k'[A][O.1] ... (ii) loglO(1.5X10 7 ),log lo A
dt 2.303 x ,8.3l'4 x 323
Dividing eq. (i) by eq. (ii), we get . A:::: 5.42 X 10'0 ;-,
Example 9. In hydrogenation reaction at 25° C, it is
4 (fbserved that hydrogen gas pressure falls from 2atm to 1.2 atm in
1= 1.85 X 10
k' [0.1] 50 min. Calculpte the rate of; reaction in ma/arityper sec.
(R = 0.0821litre- atm degree- 1 mol-' )
k' ::;: 1.85 xl0 5 litre mo1- 1 sec- I
,
dP 2-1.2
,
Solution: Rate:::: - -'-.-
Example 7. The time required for 10% completion of a dt 50x 60
first order reaction at 298 K is equal to that required for its 25%
comple.tion at 308 K. Ifthe pre-exponentialjactor for the reaction = 2.666 X 10-4 atm, s::'
is 3.56 x 109 sec -I , calculate its rate constant at 318 K and also'
PV nRT
the energy ofactivation. (Ii T 1997)
P "(n'l RT
sec::;:v)se~
550 G.R.B. PHYSICAL CHEMISTRY FOR COMPETITIONS

Time (minute) 0 15 30 45 60
. Number of bacteria 50· 100 200 400 800
. 4 Show that the 'rate ofproduction of bacteria is offirst order. How
. lanty
Rate lmo ' I sec) :::: -
2.666
-- x-
10-- many bacteria will be there after 3 hours?
0.0821x 298 Solution: For bacterial growth,
:::: 1.09 X 10-5 mollitre- I
Example 10. A drop ofsolution (volume 0.05 mL) contains.
k =_ 2.30310glo
t
(_a_\j
\a+x
3x 10-6 mole H + ions. If the rate of disappearance o/the H +
io~ is I x 107 mol litre -I sec -I ,how long would it take for H + At 15 minutes: k= - 2.303 10 (~\J
15 glO 100
ions in the drop to disappear?
Solution: 0.0462 min- I
. mole 2.303 I ( 50 ']
Concentration of drop = x 1000 At 30 minutes: k=
. volume in mL 30 og1o 200)
3x10-6 . _ :::: 0.0462 min -I
= x 1000 = 0.06 mol htre 1
0.05
. cone. change
Rate of;"hsappearance = --.---"'---
At45 minutes: k - 2.30310glo (~) = 0.0462 min- 1
45 400 .
.. tIme
/ Ix 10 7 = 0.06 Same values of rate constants show that the process corresponds
time to first order.
Time 6 x 10-9 sec Let there be' n' bacteria after 3 hrs.

Example 11. A constant temperature and volume X k - - I oglo. ( -a-


2.303 J
decomposes as, t a+x
'2X(g)~ 3Y(g)+ 2Z(g) 2.303 I
O.0462 -- _ - 50
- ogIO-
Px is the partial pressure ofX 180 n
Observation No. 7ime(min) Px(inmmHg) n = 2.04 x 105
800 Example 13. A viral preparation was inactivated in a
2 chemical bath. The inactivation process was found to be offirst
3 order in virus concentration, and at the beginning of the
experiment 2.0% of the virus was found to be inactivated per
(i) What is the order of the reaction with respect to X?
minute. Evaluate' k'for the inactivation process.
(ii) Find the rate constant.
Solution: a 100, a x 98, t = 60 sec
(iii) Find the time for 75% completion of the reaction.
(iv) Find the total pressure when pressure of X is 700 mm
Hg. (111' 2005)
k -- 2.303 I oglo If-a-
t a x
J
Solution: (i) Data shows that half life of the reaction is
constant, i. e. , 100 min; hence, it is a first order reaction. 2.303
:::: --Iog If 100) = 3.3 x 10-4 .sec -I
1o -
60 \ 98
(11.. ) k 0.693 - 0.693 -
---
- 6. 93 x10-
m 3
m·-1
tll2 100 Example 14. Trans-l,2-dideuterocyclopropane (A) under-
goes a first order decomposition. The observed rate constant at a
(iii) t 75% , i. e. , t 3/4 = 2 x t 112 2 r: 100 = 200 min certain temperature, measured in terms of disappearance of' A'
(iv) 2X(g)~3Y(g)+2Z(g) was 1.52 x 10-4 sec- l Analysis of products showed that the
t =0 800 0 0 reaction followed two parallel paths, one leading to
dt 800 - 2X 3X 2X dideuteropropane (B) and the other to cis-l,2-dideutero-
cyclopropane (C). (B) was found to constitute 11.2% of the
Total pressure = 800 - 2X + 3X + 2X = 800 + 3X ... (i)
reaction product, independently of extent of reaction. What is the
800 2X = 700 order of reaction for each path and what is the value of the rate
X=50 constant for the formation of each of the products?
Total pressure = 800 + 3 x 50:::: 950 rnrn Hg B 11.2%
'Example 12. Suppose 50 bacteria are placed in a flask Solution: A<
containing nutrients for the bacteria so that they can multiply_ C 88.8%
A study at 35° C gave the following results:
CHEMICAL KINETICS 551

In case of parallel path reaction, Example 17. In hypothetical reaction A2 + B2 ---'?2AB.


kB = kA X fractional yield of B follows the mechanism as given below:
J.52x 10-4 x 0.112= 1.7 x 10-5 sec-I <fast reaction)
kc = kA x fractional yield of C (slow reaction)
= 1.52 x 10-4 x 0.888= 1.35 x 10-4 sec-I (fast reaction)
The reaction will be fIrst order for each individual path. Give the rate law and order ofreaction.
Example 15. In milk, at 37°C, lactobacillus acidophilus Solution: Slowest step is rate determining.
has a generation time of about 75 minutes. Calculate the
population relative to the initial value at 30,60, 75, 90 and 150 Rate::::: k[A][B2] ... (i)
minutes.
Here, [A] should be eleminated.
Solution: For growth kinetics,
k = - 2.303 io glo
t
(_a_)
a+x
K:;;;;
C [A z ]
[A]2
=-
[A z ]
[A]= K~/2[A2]1I2
k-- - 2.3031
t
oglO (No]
-
N From eq. (i), Rate = kK~/2 [A 2f2 [B 2 ]
=K[A 2 f 2
[B 2 ]; [K=k·K c ]
Generation time 75 minutes; a = 1, a + x 2
Order =1 + 1/ 2 =3 / 2
k 2.303
-
75
-
- 1oglO (1
-
1+ 1
-0. 00924 mmJ-
. -I
-. Example 18. Calculate order of reaction from the
following data:
After 30 minutes, 2NH 3 ---'? N2 + 3H2 (reaction)
- -2.3031
k- - - oglo - (No \j Pressure (mmHg) 50 100 200
t . N Halflives (min) 3.52 1.82 0.93

0.00924 = - 2.303 log 10 (No] Solution: We know that,


30 N n = 1 + IOglO (t I/2 )1 loglo (t1l2 h
N log 10 pz loglO PI
1.319", 1.32
No = 1+ log 10 3.52 -log 10 1.82
Similarly, we may calculate after 60, 75, 90 and 150 minutes. log 10 100 - log 10 50
Example 16. Rate law for ozone layer depletion is, 1.95", 2
]2 Similarly, we may calculate for other set of conditions .
dt [0 2 ] .' . Example 19. The chemical reaction between mercuric
chloride and potassium oxalate proceeds as under:
Give the probable mechanism ofreaction?
Solution: 0 3 ~ O2 + 0 (fast reaction)
(equilibrium constant K c) the mass of Hg 2Cl2 precipitated from different solutions in a
given time, at 100°C was as follows:
(slow reaction, rate constant k) ~~~"~-h.~-;-'-_~_M' ________ ,-,:,"'___ '__ .M~_"'ri;:' __ ~-:~_'-''' ____ • ..- ......

Rate:;;;; k[03 ][0] . ... (i) JltP1. XIJIljr:'!fM·:fft~


!mtI! J.~1 ll!M!l J;) . (1fI.~~M.t#.~) .~"!'~ ..
K :;;;; [°2][0] or [0] K [0 3 ] . ~-~------~.~-.-:--'--------
• ____ ~ _ _ _ _ _ r _

c [0 3 ] c [° ]
2
, (1) 0.0836 0.404 65 0,0068
putting the value in eq. (i) (2) 0.0836 0.202 120 0.0031
(3) 0.0418 00404 60 0.0032
From eq. (i), From these data calculate order ofthe reaction.
Rate k·[O]. K [03J k. K . [03 ]2 =K. [0 3 ]2 Solution: Rate::= k[HgC1 2 r'[K 2 C Z0 4 )r:;
3 c [° 2 ] c [° 2 ] [Oz]
0.0068 = k[0.0836]U[00404]Il; ... (i)
Here, 65 .
552 G.R.B. PHYSICAL CHEMiSTRY FOR COMPETITIONS

0.0031 = k[0.0836]<l [0.202]13 . ... (ii) Similarly, for 2nd reaction it becomes 3 times,
120 . . ' :. At 75°e
0',0 0032 = k[0.0418]<l[OA04]13 ••• (111
... ) rilte of reaction for 2nd (3)5 R2 '
6 rate of reaction for 1st = (2)5 Rl = 7.5937 (": Rl = R 2 )
Dividing eq. (i) by eq. (ii), we get 4 = 213 ; ~ =2
Example 22. The reaction, A + OH - ----t Products, obeys
Dividing eq. (i) by eq. (iii), we get
rate law expression as:
Thus, overall order =ex + ~ =3 -d[A] = k[A][OH -]
Example 20. The rate constant of the reaction A ----t B is dt
k = 0.5; the initial concentration of A bei/i;j, 1 mol/litre.
If initial concentrations of[A] and [OH -] are 0.002 Mand
Calculate the degree ofconversion ofsubstance 'A' within 1 hour
if the reaction is of zeroth, first and second order. How does the 0.3 M respectively and ifittakes 30 sec for 1%A to react at 25°C,
degree ofconversion depend on the order of reaction? calculate the rate constant for the reaction.
Solution: Zero order: ,

x=kt '
. Solution:
A + ----tProducts
t= 0 0.002
x = 0.5 x 1 = 0.5
t = 30 [0.002 _ 0.002 x 1] [0.3 _ 0.002 x 1]
2.303 a 100 . 100
k=--loglo - -
t a-x
· k 2.303 1 b(a-x)
k = 0.5; t = Ihr; a = 1M U smg = oglO ,
t(a-b) a(b-x)
0.5 = 2.303 10 _I_
I glO (1- x) 0.3 [0.002.!. 0.002 x 1]
k= 2.303 10' 100
x= 0.39
1 x 30x(0.002-0.3) glO O.002[0.3_0.002Xl].
Second order: k=:....--- 100
t a(a - x)
. 1 x k= 1.12x 10-3 litre mol-l S-l
0.5=-·--
1 1(1- x)
'E'timple 23. A hydrogenation reaction is carried out at
x = 0.33 500 K. If the same reaction is carried out in presence ofa catalyst
, Example 21. Two first order reactions proceed at 25° Cat at the same rate, the temperature required is 400 K. Calculate the
the same rate. The temperature coefficient of the rate of the first activation energy of the reaction if the catalyst lowers the
reaction is 2 and that ofsecond reaction is 3. Find the ratio ofthe activation barrier by 20 kJ mol -I. (liT 2000)
rates of these reactions at 75° C. Solution: Arrhenius equation may be given as,
Solution: For fIrst order reaction, 1j = k[A]1 k = Ae-EaIRT

•• 1j /1-2 == kl / k2 = temperature coeffIcient Let k500 and k 400 be the rate constants at temperatures 500 K
and 400 K (in presence of catalyst) respectively. E 500 and E 400 be
Let the rate of reaction for 1st at 25°e be Rl and the rate of the activation energi~s at temperatures 500 K and 400 K
reaction for lInd at 25°e be R 2 . respectively..
Also, Rl =·R 2 k 500 -- Ae- E500 IR x 500 ... (i)
Rates of reaction
k 400 -_Ae-E400IRx400 ... (ii)
.. At 25°e Rl R2
35°e 2Rl 3R 2 Given, k500 = k 400 (same rates in presence and absence -of a
45°e (2)2 Rl (3)2 R 2 . catalyst).
55°e (2)3 Rl (3)3 R2 On comparing eq. (i) with eq. (ii),
E500 _ E 400
65°e (2)4 Rl (3)4 R2
R x 500 R x 400
75°e (2)5 Rl (3)5 R2
E500 =' E 40'o "
or
:. Temperature coefficient for 1st reaction = k35 = R35 = 2 5 4
k25 'R25 E 400
or E500 E:O-- X 5 ... (iii)
i. e. , for each 10 0 e rise in temperature, rate becomes 2 times. 4
Given, E500 = E 400 + 20
CHEMICAL KINETICS 553
Substituting in eq. (iii), Solution: Let order with respect to A and B are a and fl
E 400 + ?-O= E 400 >< 1.25 respectively. Rate law expression may be given as,
Rate = k[A]ct [B] ~
or E 400 = 20 = SOkJ mol~1 ,i~:,f;' 0.05 = k[O.l]ct [0.2] ~ ... (i)
025 J ," ."',", ,""
..~ ," ,< ,;~~.
k[O.2]U [0.2] ~
• •
0.10 ... (ii)
So, Esoo = SO + 20 = 100 kJ mol/-I' ,
0.05 k[O.lt [0.1] ~ ... (iii)
. Example 24. Some PH3 (g) is introduced into a fask at r
600°C containing an inert gas, PH 3 proceeds to decompose into Dividing eq. (i) by eq. (ii), we get
P4 (g) and H2 (g) and the reaction goes to completion. Total
pressure is given below as a function of time. Find the order of ~ [~J u, i. e. , a::::: 1
the reaction and calculate the rate constant,
Time (sec) 0 60 120 00 Divi~ing eq. (i) by eq. (iii), we get
Pressure (mm Hg) 262.40 272.90 275.51 276.40
1 =[2] ~ , i. e. , f) =0
(Ill' 2001)
Rate=k[A]1 [B]o
Solution: 4PH 3 (g) + Inert gas ~ P4 (g) + 6H 2 (g) + Inert gas
Substituting the value of a and f) in eq. (i), we get
tOP P; 0 0 a
t=t (P x) P; x/4 6x/4 P; 0.05 k [0.1]1 [0.2]0
t = 00 OP; P/4 6P/4 P;
k = 0.5 sec-I
At t = 0, 262.40 = P + Pi ... (i)
t 60 sec 272.90 = PPH3(left) + PP4 (g) + PH2 (g) + Pi Example 26. A + 2B ~ 3C + 2D
The rate of disappearance of B is I x 10-2 mol litre -\ sec -I .
t 120 sec 275.51 ::::: PPH 3 (left) + PP4(g) + PH2 (g) + Pi
, What will be the (i) rate of the reaction and (if) rate ofchange in
t = 00 276.40 = PP4(g) + PH2 (g) + Pi concentration ofA and C? [CSSE (Mains) 200::;;1
P 6P 1 d[B]
276.40 = - + - + P Solution: (i) Rate = - -
4 4 I
2 dt
276.40 x 4 ::::: 7P + 4Pi ... (ii)
=- ~ [-Ix 10-2 ]
Att = 0, 262.40 P + Pi ... (iii) 2
On solving eqs. (ii) and (iii), we get 0.5 x 10-2 mol litre -I sec-\

P 1S.66 mm, P j 243.74 mm


::::: d[A]
, x 6x , (ii) - - - 1 [d[B]] == 0.5 x 10-2 mol litre-I sec- 1
At 60°C: 272.90::::: (P - x) + PI' + +- dt 2 dt . '
, 4 4 i
",:,~ I d[C] I d[B]
~:')t272.90= IS.66- x + 243.74 + 7x 3 dt 2 dt
4
d[C]
x= 14mm ~ x (-Ix 10-2 )
dt 2
k (a )
2.303 Iog - - =2.303
- - Io g -
IS.66
--- '. , ,= 1.5 X 10-2 mollitre -1 sec- I
t a-x 60 IS.66-14 i ' ~'.~~ ,
For the reaction, A + B ~ Products, rate
Exa.;npl~ ·1,1.
k 2.32 x 10-2 sec is given ~s, ra;~== k[A][B]2.
How wiltthe rate change ifvolume of the reaction mixture is
Similarly, at 120 sec k = 2.30 x 10-2 sec- 1 (do yourself)
reduced to li3rd of its originaivolume? (BCECE 2(06)
Since, .: values of' k' are same, hence the reaction belongs to fIrst
Solution: When volume' is reduced to 1/3rd, then
order. ,concentration will increase three times. ".
Example 25. For the given reaction, A + B ~ Products .. Ii = k[A)[B]2 ... (i)
following data were obtained: ' ,
r2 k[3A][3B]2 ... (ii)
From eqs. (i) and (ii), .
1. OJM 0.2M 0.05
2. O.2M O.2M 0.10 Ii
-::::;:-
1
r2 27
(i) Write the rate law expression. r2 = 271i
(ii) Find the rate constant. lIlT 2004 (Memory based)) .. Rate will increase 27 times.
554 G.R.~. PHYSICAL CHEMISTRY FOR COMPETITIONS

4
. Example 28. For first order reaction, if rate constant at
17°Cis2.8xlO- 5 s-l andat27°Cis2.8xlO- 4 s-I.
1og
2.8><; 10- Ea [1
- I J.
2.8 X 10- 5 2.303 X 8.3 290 300
(i) Write the equation for calculation ofactivation energy. • •
. E = 2.303 X 8.3 X 290x 300
(ii) Calculate the activation energy. a 10
1
(R= 8.3 JmoZ- k-I) IAIPMT (Mains) 20081 =: 166299.6 J mol- 1
Solution. We know, 166.3 kJ mol- l
1og l(k;z ) = 2.303R [1Tl - T1]
k Ea
z

SUMMARY AND IMPORTANT POINTS TO REMEMBER


'-" ""'r, I'" 1:-.. ..,'-",. I ...

1. Chemical.kinetics: It is the branch of chemistry which masses of the reactants at that instant raised to powers which are
deals with the rates of reactions, the factors which influence it numerically equal to the numbers of their respective molecules in
and the mechanism by which the reactions occur. the stoichiometric'equation describing the reaction. Consider the
2. Rate of reaction: It is the speed with which the reactants following general reaction,
are converted into products. It may be expressed in terms of rate
ml Al + mz A 2 + m3 A 3 + ..: ~ Products
of disappearance of any of the reactants or rate of appearance of
any of the products. Rate of reaction ~ [AI ]m! [A 2 ]m2 [.43 ]m3 •••
The rate measured over a long time interval is called the
average rate. 4. Rate const~nt: It is equal to rate of reaction when
concentration of each reactant is unity, i. e. ,one m~-le litre -:-1 .It is
Decrease in conc. of reactant
Average rate of reaction = a constant for a particular reaction at a given temperature and also
Time taken known as. specific t:ate constant or specific reaction rate.
Consider a general reaction,
Increase in conc. of product
+---------=---- aA + bB + ... ~ Products .
Time t.aken

A -ve sign signifies a decrease in concentration of the reactant dx = k[At [B]b ...
dt
and +ve sign indicates ail increase in concentration of the
product. If CA=CB 1 or [A]=[B]
In general, for a reaction,
Then, dx k
nlA + nzB +n 3C + ... -->m1X + mzY + m3 Z +", dt
I 6.A 1 tlB· I AC
5. Collision theory of reaction rate: A chemical reaction
=--
n l At nz At n3 At occurs when the existing bonds of the reactants are broken down
AY I AZ
and new bonds come int0 existence in the molecules of products.
1 AX
-- The basic requirement for a reaction to occur is that there should
m1 At mz At m3 At be physical contact between reacting species, i. e., they must
Average rate is not very useful as it does not remain constant collide with one another. Effective collisions which bring
throughout its life span. It is more appropriate to express chemical change are fewinspite of large number of actual
instantaneous rate. collisions. For a collision to be effective, the following two
conditions must be satisfied:
Rate of reaction with reference to a particular instant of time is
called instantaneous rate of reaction. It is defined as the rate of (i) Reacting species must possess· adequate energy to
change of concentration of anyone of the reactant or product overcome the energy barrier.
species over a small interval of time. (ii) Reacting molecules must be properly oriented at the time
of collision.
. dx dx
R ate 0 f reactIOn =- - or + - The minimum amount of energy which the colliding particles
dt dt
must possess as to make the chemical reaction to occur is called
1
Units ofrate of reaction are conc. time- ,e.g.,mol L-1s- 1 or threshold energy. The excess energy (over and above the average
mol L- 1 hr- 1 , etc. energy of the reactllg species) required by the reacting species to
undergo chemical reaction is called activation energy (Ea ).
3. Law of mass action: It was presented by Guldberg and
W~ge in 1864. At a given temperature, the rate of a reaction at it Activation energy = Threshold energy Average kinetic
particular instant 'is proportional to the product of the active energy of the reacting molecules
CHEMICAL KINETICS 555

Every chemical reaction whether exothermic or endothermic involved in the balanced equation. There are number of reactions
has an energy barrier" which has to be overcome before the which are complex in nature and occur in several steps; the
reactants can be transformed into products. The reacting molecularity of "such reactions is determined by the slowest step.
molecules must have "Sufficient energy as to cross' this energy Thus, molecularity depends upon the mechanism of the reaction.
barrier. The effective collisions increase with increase of (ii) Order of reaction: Rate equation or rate law is an
temperature, i. e., rate of reaction increases with increase of experimentally determined mathematical expression relating the
temperature. molar concentrations of the reactants to the actual reaction rate.
6. Factors influencing the reaction rate: The chief factors Consider a general reaction,
which influence the rates of reactions are:
0) Nature of reactants: The reactants with weak bonds nl A + n2B + n3C + ... -7 Products
react quickly whereas the reactions involving reactants with The rate law or rate equation is"
strong bonds are slow. The molecular reactions are generaily
slow while ionic reactions are fast.
(ii) Concentration of reactants: The rate of reaction
increases with increase in concentration of reactants as the
Order of reaction is the sum of the exponents (powers) to
number of collisions increases.
which molar concentration terms are raised in the rate law.
(iii) Catalyst: Positive catalysts increase the reaction rate
Order of reaction = (x + y + z + ... )
by providing an alternative path involving lesser amount of
activation energy. The value of (x + y + z + ... ) usually ranges between 0 and 2
and even 3. It may have positive, negative, zero and fractional
(iv) Temperature: The rate of reaction increases with an
values. It is termed overall order. The reactions are classified as
increase in temperature. The number of effective collisions
zero, first," second, third order according as (x + y + z + ... ) is
" increases with increase" of temperature. The rates of many
zero, one, two or three respectively. The order can be expressed
reactions are approximately doubled or tripled for every
either in terms ofthe order oftM specific reactant or in terms of
10 0 C rise in temperature. The temperature coefficient is defined overall order of the reaction.
as the ratio of specific reaction rates of a reaction at two
The reactions which obey a first order rate equation, although
temperatures differing by 100 C.
they are not uriimolecular, are said to be pseudo first order.
k/ +10 '.
Temp. coefficient - - (value bes generally 8. Reactions of various orders: In such reactions, the
kt concentrations of various reactants except one remain practically
between 2 and 3)
constant during the course of reaction.
Arrhenius suggested an equation which describes k as a (i) Zero order reactions:. A reaction in which the rate is
function of temperature, i. e. , independent of the concentration of the reactant molecules, i. e. ,
k Ae-EaIRT the rate is proportional to the zeroth power of the concentration of
the reactants. .
where, k = rate constant, A = a frequency factor (constant), dx
E a energy of activation, R = gas constant and T = temperature. " k[Alo = k
dt
At two temp~ratur? ~ T~ a (1
and
J 1 J i. e., the reaction velocity remains constant throughOut the
progresS of the reaction. The concentration decreases linearly
agio lk; = 2.303R lTI - T2 with time. "
7~ Molecularity and order of reaction: [AL =[A]o kt
(i) Molecularity of ~action: The minimum number of [A]o ,Initial concentration
reacting particles (molecules, atoms or ions) that come together t completion = -k- Rate constant
or collide in a rate determining step to form product or products is
called the molecularity of the reaction. The "reactions are termed The units of k are mol L-I time -I.
as unimolecular, bimolecular or trimolecular, etc., depending (ii) First order reactions: The rate depends upon the
upon the number of reacting particles as 1,2,3, etc:, in the rate concentration of one reactant only.
determining step. Simultaneous collisiolls involving more than
"three molecules are very rare and hence reactions with dx = k[A]
molecularity more than three are also rare. dt .

NH4 N0 2 -7N2 +2H 20 (Unimolecular) 2.303 I a 2.303 I [A]o


k = - - oglO - - - = - - oglO--
t (a-x) t [AL
NO+0 3 -7N0 2 +02 (Bimolecular)
time, a = initial concentration and (a - x) =concentration at
2FeCl 3 + SnCl 2 -72FeC1 2 + SnC1 4 (Trimolecular) time t.
In simple reactions involving one step only, the molecularity is A change in concentration unit does not affect the numerical
"equal to the sum of number of molecules of the reactants value of k. Thus, for first order reactions, any quantity which is
556 G·8.B. PHYSICAL CHEMISTRY FOR COMPETITIONS

proportional to concentration can be used in place of concentrations. . The experiments. are repeated with other
concentration in the integrated equation. reactants in a similar way. Likewise, the orders w.r.t. all reactants
The time taken for the completion of same fraction of change are determined. The overall order is the sum of the orders of all
is independent of initial' concentration. When x =. 0.5 a and the reactants.
t t 1l2 , (ii) Integration methOd: The quantities 'a,x and t are
k :;:: 2.303 log to ~ == 2.303 log to 2 = 0.693 detwmined and. substituted in the kinetic equations of various
t 112 O.5a t 112 t 112 orders. The equation which gives the most consistent values for
the specific rate constant (k) for a series of time intervals is the
0 ..693 one corresponding to order of reaction.
k 2.303 a
k == - - log 10 - - for first order reactions
Thus, t 112 is independent of initial concentration. t (a - x)
The equation. of the first order can also be written in the
following form when initial concentration is not known. k 1 [
t a(a-x)
a] for second order reactions
2.303' I (a xI) 2.303 I' [A]II
k == oglo = ogiO - -
, (t2 - t I ) (a- x 2') (t2 - t,). [A]t2 Rate =k for zero order reactions

The unit of k is time -I . (iii) Graphical method: If the plot oflog (a x) versus 't'
(iii) Second order reactions: Reaction rate depends upon is a straight line, the reaction follows first order.
the concentration of two reactants. If a plot of _1_ versus 't' is a straight line, the reaction·
.(a-x)
dx = k[A][B] = k[A]2 follows second order.
dt
2.303 1 b(a x) If a plot of 1 versus' t' is a straight line, the reaction
k= og to --'-----'- (a x)2
tea-b) a(b-x)
follows third order.
[(a - x) and (b - x) are the concentrations of A and B after time
interval t.] . (iv) Half life method: Starting with two different
concentrations al and a2 for the same reaction, the half lives
When concentrations of both the reactants are same,
(tIl2)1 and (til 2 )2 are determined.
k~. x
t a(a - x)
, (x is the concentration changed in time interval t)
Half life period, t 112 2. , i. e. ,inversely proportional'to initial n is the order of reaction:
lea
concentration. The units of k are litre mol- l time-I. (v) van't Hoff differential method: For two different
(iv) Third order reactions: Expression for third order initial concentrations C I and C 2 equations can be written in the
reactions of the type form
3A ~ Products _dCI == kCf and- dC 2 == kCi
. dt dt
dx = k[A]3
dt Taking logarithms of two equations and then subtracting .
x(2a - x)
k z
t 2a Z (a-x)2 log ( -. dt
dC I ) - log (dC
- dt )
The units of kareL2 mol-2 time-I. n:;:: --'-------~---
9. Methods for the determination of order of a reaction:
(i) Initial rate method: A number of experiments are 10. Mechanism: An intelligent guess depending upon the
carried out by varying concentration of a reactant w.r.t. which the observed rates of reactions about the series of steps (known as
order is to be determined. keeping the concentrations of all other elementary processes) leading to the formation of products is
reactants constant. The initial rate of the reaction at each . called the reaction mechanism. The overall reaction is the
concentration is determined by concentration-time curves. The summation of all elementary steps: The slowest step is taken as
order of the reactant is then c,alculated from the rates lit various the rate determining step.
CHEMICAL KINETICS 557

1. Matrix Matching Problems: [E] Match the reactions of List-I with the increase in rate when
. (According to the new pattern of lIT Screening) concentr~tion is raised two times in List-II:
[AJ Match the reactions of LisH with their orders in List-II: List~I List-II
List-I List-II (Reactions) (Increase in rate when
H+ cone. of reactant is
(a) C12 H22 0 11 + H20 ~ (P) Pseudo first order doubled)
C6 H12 0 6 + C6H l2 0 6
. HOH
(a) CH3CHO ---7 CH4 + CO (P) 2 times
(b) CH3COOC2H s 1 (q) Zero order
H+ orOH- 1- (b) SOzClz ---7 S02 + Cl 2 (q) 21.5 times
(c) 2H20 2 2H20 + 02 (r) 2 times in acid medium
CH3COOH + c;HsOH
hv HOH
(c) H2 + Cl 2 ---7 2HCI (r) Second order (d) CH3C00c;Hs ~ (s) 22 times in basic
(d) CH3CI + OH- ---7 (s) First order CH3COOH + C2HsOH medium
CH30H+Cr
[F] Match the List-I with Liwst-II and List-III:
[B) Match the reactions in Colump.-I with the units of their rate
constant in Column-II: List-I List-II List-UI
Column-I Column-II (a) x is plotted aga- (P) Slope= k!2.303 (u) Intercept = (,
(Reactions) (Unit of k) inst 't' in zero
order reaction
(a) SOzCl z ---7,S02 + Cl 2 (P) sec-~,
(b) log (a - x lis (q) Slope 2k (v) Intercept
(b) CH3COOCzHs + NaoH~ (q) min':" l plotted against 11 a
't' in first order
CH3COONa + CZH 50H reaction
(e) 2H20z ---7 2H20+ 02 (r) Lmol- I min- 1 , (c) (a-xr l is (r) Slope = - k (w)Intercept
(d) HzOz + 21 + 2W (s) Lmol- 1 s-I plotted against (when log is 11 a 2
, t' in second natural)
2H2 0+ 12
order reaction
(d) (a xi is (s) Slope k (x) Intercept
[C] Match the kinetic equations of Column-I with the units of
plotted against = loglo a
their rate ,constant in Column-II:
't' in second
Column-I Column-II
, order reaction
(Kinetic equations) (Units of rate constant)
where, a = Initial concentration of reactant
(a) x kt (P) sec':"!
x Extent of reaction in time 't' sec
(b) k = 2.303 logw r_a_) [G] Match the List-I with List-II:

l[t 1 \~]-x List-I List-II


(c) k - ---- (r) mol (a) Molecularity (P)O
t (a-x), a
(b) Order (q) 112
(d) k = 2.303 log [b(a X)] (s) atm- I sec-I (c) Temperature coefficient (r) Between 2 to 3
tea-b) a(b-x)
(d) Activation energy for (s) 2
[D] Match the half lives in Column-.! with the orders in participation of all reactant
Column-II: molecules
Column-I Cohtmn-U [H] Match the Column-I withColwim-II:
(Ualflife) (Qnler) 0lIwnn-1 Column-II
(P) First order '(Rate Jaw) (Order)
(a) t1l2 = constant
(a) Rate k x Intensity of light(P) Second order
~) tl/2 oc: a (q) Pseudo first order
, 1 (0) Rate k[A]l[B]1 (q) Zero order
(c) t1l2 ex.- (r) Second order
a I (c) Rate k[A]3/2[B]1I2 (r) First order when A is
(d) tl/Z IX p- (s) Zero order
excess
where, a Initial concentration of the reactant (8) Second order whenB is
p = Initial pressure of the reactant excess
558 G.R.S. PHYSICAL CHEMISTRY FOR COMPETITIONS

[I] Match the Column-l with Column-II: . Here, tl12 half life
Column-I Column-II t314= 3/4th life
0.693 k = rate constant
(a) tll2 =-- (P) Zero order [K] Match the Column-1 with Column-II:
k
a (q) First order Column-I Column-U
(b) t1l2 = 2k
(a) First order reaction (p)Co O.lM; tl/2 25min
1 (r) Average life
(c) 'C Co = 0.05 M; = 12.5 min
tl/2
k
(d) (8) 75% completion (b) Second order reaction (q)Co = O.IM; til2 = 12 min
t3/4 = 2t1l2
Co = 0.05 M ; tl/2 = 24 min
(t) 25% completion
(c) Zero order reaction (r) Radioactive decay
[J] Match the Column-l with Column-II:
Column-I Column-II (d) tl12 varies inversely to (s) Co 0.1 M; t1l2 8 min
the concentration of the Co = 0.05 M; t1l2 8 min
3 reactants·
(a) t3/4 2 X'I·"
,- (p) 99% completion

(b) t314 = 2 X tl/2 (q) Zero order


(c) 6.909/ k (r) First order
(d) 2.303/ k (s) 90% completion

1. [A] (a-p), (b-r, s), (c-q), (d-r) . [G] (a-s), (b-p, q, s), (c-r), (d--p)
[B] (a-p, q), (b-r, s), (c-p, q), (d-r, s) [H] (a-q), (b-p), (c-p), (d-r, s)
[C] (a-r), (b-p), (c-q, s), (d-q, s) [1] (a-q), (b-p), (c-q, r), (d-q, s)
[D] (a-p, q), (b-s), (c-r, s), (d-r, s) [J] (a-q), (b-r), (c-p, r), (d-r, s)
[E] (a-q),(b-p), (c-p), (d-~ s) [K] s), (b-q), (c-p), (d-q)
[F] (a-s-u), (b-p, r-x), (c-s-v), (d-q-w)

• CTICE PROBLEMS •
1. For the reaction A --t B, it was found that the concentration of d[NOCI]
(iii) 2NO +. Cl z --t 2NOCI; ?
B increased by 0.3 mol L- 1 in 2 hours. What is t:p.e average rate
of reaction? (iv) CO + N02 --t CO2 + NO;
[Ans. 0.15 mol L- 1 hr- I ]
2. It;l the reaction X --t Y, the initial concentration of X is 2.5 [Ans. (i) d[H z ) = - 3 . = + ~ -7'----"-':
1
dt dt 2 dt
moL and its concentration after 3 hours is 0.7 mol L- .
(ii) d[02l =_ 1 =..!. d[H20);
What is the average rate of the reaction? dt 2 . dt 2 dt
[*ns. 6.0 x 10-1 mol L-1hr- 1) (iii)d[NOCI) d[NO] = 2.
3. For the reaction 2A + B --t C, the rate offormation ofC is 0.25 dt dt dt
mol L- 1 hr-:-I. What is the rate of disappearance of A and B? _ d[COl d[e0 2 ] = d[NO] ]
(iv) -
[Ans. 0.5 mol L- 1 hr- I ; 0.25 mol L- I hr- 1] dt dt dt dt
4. For each of the following reactions express the given rate of 5. When annnonia is treated with O 2 at elevated temperatures,
2
change of the concentration of the reactant or product in terms the rate of disappearance of annnonia is found to be 3.5 x 10-
of the rate of change of concentration of the other reactants or 3
mol dm- S-1 during a measured time interval. Calculate the
products in that reaction. rate of appearance of nitric oxide and water.
_ d[H2]
(i) N2 + 3H2 --t 2NH3 ; ? [Ans. = 3.5x 10-2 rool dm-3 S-I;
dt dt
_ d[Oz]
(ii) Hz + ~ O 2 --t H2 O; ? Ll[H 20) = 5.3 x 10-2 mol dm-3
dt dt .
CHEMICAL KINETICS 559
[Hint: Consider the following equation: 15, The reaction,
4NH3 + 502 -7 4NO + 6H 20
2N20 S -7 2N 20 4 + O2
The equality in this case is
1 ~[NH3] = 1 ~[NO] =.!. ~[H20] ] occurs in carbon tetrachloride. The rate constant is
4 dt 4dt 6dt 2.35 x 10-4 sec-I at 20 0 C and 9.15 x 10-4 sec": J at 30°C.
6. In a reaction, N 20 S -7 2N0 2 + Yz 2 , ° Calculate the activation energy of the reaction.
the rate of disappearance of N 20 S is 6.5 X 10-3 mol L- 1 S-I. [Ans. 100,343 kJ mol-I]
Compute the rates offormation ofN0 2 and 02' 16. For the inversion of cane sugar,
[Ans. d[N°2] = 1.25 x 10-2 mol S-I. C12 H22 0 11 + H2 0 -7 C6 HI2 0 6 + C6 H12 0 6 ,
dt '
the rate constant is 2.12 x 10"""" L mol-I sec-I at 27° C. The
d[°2] 3.125 x 10-3 mol L- 1
dt activation energy of the reaction is 1.07 x lOs J mol-I. What is
7. For the reaction, the rate constant of the reaction at 37° C?
203 .....:.-.: 302 [Ans. 8.46 L mol- J sec-I]

- ~[03] was found to be 4.0 x 10"""" atrus-I. 17. From the data given below for the reaction 2A + B -7 3C at
dt 298 K, fInd out the rate law expression for the reaction.
Determine the value of ~[02 ] in atm s -I during this period of ~

/It Expt. [A) .(B) . Rate of


time. No. (moIL1 ) (lOol!;l) formation of C
.....
[Ans. 6.0xlO- 4 atms-1 ]
----.~.

1. 0.10 '0.10 4.0x 10-4


8. The following reaction was carried out at 44" C :
2. 0.30 0.30 1.2 x 10-3
N 20 S -7 2N0 2 + Yz 02
The concentration ofN02 is 6.0 X 10-3 M after 10 minutes of 3. 0.10 0.30 4.0 x 10-4
the start of the reaction. Calculate the rate of production of 4. 0.20 0.40 8.0x 10-4
N01 over the fIrst ten minutes of the reaction.
[Ans. Rate = k[ A]]
[Ans. 6.0 x 10"""" mol L- 1 min -1]
9. The rate of a particular reaction doubles when temperature 18. What is the rate law expression forthe reaction,
changes from 27" C to 37" C. Calculate the energy of A +B -7C?
activation for such reaction. (R = 8.314 J K- I mor l ) The following data were collected from the experiments:
[Ans. 53.59 kJ morl] lllitiail A) Initial fBJ
Expt. Rate of .
10. The rate' const~t of a reaction is 2 x 10-2 S-I at 300 K and No. (mol );"1) (mol );"1) formation of C
.. -.
8 x 10-2 S-I at 340 K. Calculate the energy of activation of the ---~--

l. 0.10 0.10 4.0 x 10-4


reaction.
[Ans. 29.39 kJ mol-I] 2. 0:20 0.20 3.2 x 10-3
U. For achemicalreaction the energy of activation is 85 kJ mOrl. 3. 0.10 0.20 1.6 x 10-3
If the frequency factoris 4.0 x 109 L mol- I S-I, what is the rate --
[Ans. Rate = k[A HBr]
constant at 400 K?
[ADS. k = 3.19 X 10-2 L moriS-I] 19. The reaction,
12. The energy of activation of a reaction is 140 kJ morl. If its
rate constant at 400 K is 2.0 x 10-6 S-I, what is the value at
2N20 S -7 2N02 + ° 2,

was studied and the following data were collected:


500K?
[Ans. 9.06 x 10-3 s-I] Rate of disappearance of
13. '. Calculate the ratio of the catalysed and uncatalysed rate Expt. (N1Osl
N10 S
No. . (mol Ll)
constant at 20° C if the energy of activation of a catalysed (mol L-1 miD -1)
rea~tion is 20 kJ morl and for the uncatalysed reaction is
75 kJmorl. 1. 1.13 X 10-2 34 X 10-5

fAns', 6.4 x 109 ] 2, 0.84 X 10-2 25 X 10-5


14, The decv,nposition of methyl iodide, 3. 0.62 X 10-2 18 X 10-5
LC.l!~I(g) -7 C2~ (g) + 12 (g) Determine (i) order the reaction (ii) the rate law and (iii) rate
at 273 0 C has a rate ccn,;t"f!t of 2.418 x 10-5 s -I. If activation constant for the reaction.
energy for the reaction is + 179.9 kJ mor l , what is the value of [Hint:' Let the rate law for the reaction be
collision factor' A' at 273 ° C? Rate=k[Nz05Y
[Ans. 3.9 x 10 12 S-I] 34 x 10- 5 = k[L13 X 1O-2y
560 G.R.B. PHYSICAL CHEMISTRY FOR COMPETITIONS

25 X 10-5 = k[0.84 X 1O-2 y [A]mol 0.1 0.1 0.2


2
34 X 10-5 [1.13 x' 1O- ]X [B] mol L- 1 0.2 0.4 0.2
or
25x [0.84xI0- 2 y 16x 104 8 x 104
1.36 = [1.345y [Ans. Order of the reaction = 3., k = 1.0 X 107 L2 mor2
x 1 25. The data given in the following table pertain to the reaction,
Order of the reaction is I. Rate law k[NzOs]
5 2A +B -7C.
. k= Rate = 34 x 10- =' 30.08 X 10-3 min-I]
[N 20 5 ] 1.13 x 10-2 Determine the form of the rate equation and the value of the
20. For the reaction, rate constant.
2NO + 2Hz -7 N z + 2H20, Initial concentration
the following kinetic data were obtained: Expt. (mol Ll) Initial rate
------ No. --------------'--.~~~~,.;ol L~~:t::l)'.
Expt. [NO] Rate of reaction
(mol L-1 ) (mol r l 5-1) IA] . ~:~);'.:7~:' .~.: .
No.
1. 1 x 10-'3
1. 0.12 0.12 0.25 x 10-4
0.24 l.0 x 10-4
2. 1~ 10-3 2 X 10-3
2. 0.12
3. 0.24 0.24 2.0x 10-4 3. ·1 X 10-3 3 X 10-3 2lx 10-6

Detenrune the rate law and the rate constant. 84 x 10-6


[Ans.· Rate k[H2 ] [NOY; k = 1.45 X 10-2 L2 mor2 [Ans. Rate ~ k[A ]2[B]; k = 7 x 103 1
21. Fill in the blanks in the foll.owing table which treats reaction of
26. The following initial rate data were obtained for the reaction: .
a compound A with a compound B, that is of the first order
with respect..!? A and zero ord~!..with respect to B: 2NO(g) + Br2 (g ) -7 2NpBr(g)
Expt. (A] (B). Initial rate
. No. . (mol L-1 ) (mol Lt) (mol Ll 5-1) Rnn (NO]/M (Dr-1 11M . RateiMmm-I
1. 1.0 1.0 . .L30x 10-3
1. 0.1 0.1 2.0 x W- 2
2. 0.2 4.0·x.1O-2 2. 2.0 1.0 5.20 x.1O- 3

3. 0.4 0.4 3. 4.0 '2.0 4.16x 10-2


4. 0.2 . 2.0x 10-2 Determine the reaction rate law and the value of the rate
[Ans. (2) [A] =0.2 mol constant. (Dhanbad 1992)
dx . . .
(3) Rate = 8.0 'x 10-2 mol L- 1 [Ans. - = k[NO]2[Br21; K 1.30 x 10- mol-2 s-I]
dt
3
e
I
(4)[A]= 0.1 mol L- ]
27. The data below are for the reaction of NO and C1 z to form
22. The reaction 2NO + O2 -7 2NOz, follows the rate law
NOCI at 295 K.
== k[N0]2[02]. What is the order of the reaction? If
k 2.0 x 10-6 mol-2 L2 8-1 , what is the rate of the r~action Expt. rC1 1 ) (NO) Initial rate
when [NO] 0.04 mol and(Oz] 0.2 mol L- I ? . No. . (moIL1 ) (mol Ll) (mol L-l S-:I) .

[Ans. Order of the reaction = 3; 1. 0.05 0.05 l.Ox 10-3


Rate = 6.4 x 10-10 molL- i S-l]
2. 0.15 0.05 3.0 x 10-3
. 23. From the fflllowing data of initial concentrations and rates,
calculate the order of reaction aA -7 Products; and its 'rate 3. 0.05 0.15 9.0 X 10-3 .
--~.-~---~----'"---"'.- ..--~--
constant.
[A] mol L-1 . 0.1 0.2 0.4 (a) What is the order w.r.t. NO and Cl 2 in the reaction?
1 5 5 (b) Write the rate expression.
Rate mol 1: S-I 9 X 1'B-5 36 X 10- 144 X 10-
(c) Calculate the rate constant.
[Ans. Order of reaction = 2; (d)'Detennine the reaction rate when concentrations of Cl 2
Rate constant == 9.0 x 10-3 morl L S-I] and NO are 0.2 M and 0.4 M respectively. (liT 1993)
24. For a reaction aA + bB -7 mM + nN, the ra~e of reaction is [Ans. (a) Orderw.r.t. NO = 2; order w.r.t. Cl2 = 1
given as k[Af[BY. (b) Rate expression,r k[NO]2[C1 2 ]
Calculate the order ofthe reaction and the rate constant from e
(c) k 8.0 mor2 S-I
the given initial concentrations and the corresponding rates.
(d) Rate;:: 0.256 mol L-1 s-l]
CHEMICAL KINETICS 561

28. In the hydrolysis of propyl acetate in presence of dilute HCI in would have changed into the product? What is the half life
aqueous solution, the following data were recorded: period of this reaction?
Time from start (in minute) 60 350 [AilS. tIl2::: 128.3 hours; 5.27%]'
% of ester decomposed 18.17 69.12 39. A first order reaction is 20% completed in 10 minutes.
Calculate the rate constant of decomposition and time in Calculate (i) the specific rate constant of the reaction and (ii)
which half of the ester was decomposed. the time taken for the reaction to go to 75% completion.
[Ans. 3.34 x 10-3 min -1; 207.6 min] [Ans. k 2.232 x 10-2 min -\ t =62.12 min]
. 29. In a first order reaction the concentration of the reactant is 40. The rate constant of a reaction with respect to reactant A is 6
reduced to one-fourth of its initial value in 50 seconds. min-I. If we stali with [A] 0.8moIL- I , when would [A]
Calculate the rate constant of the reaction.
reach the value of 0.08 mol L- I? '
[Ans. 2.77 x 10-2 S-I]
[Ans. 0.3838 min]
30. For a first order reaction, the rate constant is 0.1 S-I. How
41. What will be the initial rate of a reaction if the rate constant is
much time will it take to reduce the concentration from initial
10- 3 min -\ and the concentration is 0.2 11101 dm- 3 ? How much
value of 0.6 mol L-I to 0.06 mol C l ?
of the reactant will be converted into the product in 200
[Ans. 23.0 second] minutes?
31. A substance decomposes following first order reaction. If the [Ans. Initial rate 2 x 10-4 mol S-I; 0.036 mol]
half life period of the reaction is 35 minutes, what is the rate
constant of this reaction? . 42. A substance A decomposes by the first order reaction. Startinr
initially with [A] = 2.00 M, after 200 minutes [A] 0.25/,
[Ans. 1.98 x 10-2 min -lj
For this reaction what is tll2 and k?
. 32. For a certain first order reaction, it takes 5 minutes for the [Ans. tl12 66.66min,k 0.01039 min-I]
initial concentration of 0.6 mol L- I to become 0.2 mol L-1 .
43. The specific rate constant for the concentration of H2 and I z to
What is the rate constant for this reaction? [log 3 0.4771].
fonn HI is 0.0234 mol-I LSI at 400" Cand 750 mol-I L at
[Ans. 0.2197 min-I]
0
500 C. Calculate the activation energy of the reaction.
33. Find the two-thirds life (t213 )of a first order reaction in which
!eL
k = 5.48 X 10- 1 sec-I (log 3 = 0.4771; log 2 0.3010).
[Ans. 107.9 kcal]
[Ans. 2.005 s] 44. For a certain reaction it takes 10 minutes for the initial
[Hint: tiJ3:::
2.303
loglo
a] concentration of 2.0 mol to become 1.0 mol L-1 and
k a Jia another 10 minutes to become 0.5 mol L- I . Calculate the rate
34. A first order reaction has a specific rate of 10-3 sec-!. How constant of the reaction.
much time will it take from 109 of the reactant to reduce to [Ans. 0.0693 min -I]
7.5 g (log 2 = 0.3010; log 4 =0.6020 and log 6 0.7782)?
45. The catalytic decomposition of HzOz was studied by titrating
[Ans. 287.7 s]
35. In a reaction 5 g ethyl acetate is hydrolysed per litre in it at different intervals with KMn04 and the following data
presence of dilute HCI in 300 min. If the reaction is of first were obtained:
order and initial concentration is 22 g per litre, calculate the t (seconds) 0 600 1200
rate constant for the reaction. V ofKMn04 (rnL) 22.8 13.8 8.3
[Ans. k::: 8.59 X 10-4 min- 1 ] Calculate the velocity constant for the reaction assuming it to
..be a first order reaction.
36. talculate the half life of the reaction A -7 B, when the initial
[Ans. 8.4 x 10-3 S-1]
concentration of A is 0.01 mol C 1 and initial rate is 0.00352
[Hint: Volume of KMnO 4 used oc concentration of H 20 2
mol L-1 min -1. The reaction is of the first order.
2.303 Vo
[Ans. 1.969rnin] Hence, k = - - loglo-]
t VI
. d:x'
[Hint: Evaluate k from ~ = k[ A] ] 46. The half life of a first order reaction is 10 seconds. Calculate
dt
the time for completion of 99.9% of the reaction.
37. In a reaction, A -7 B + C, the following data were obtained:
[Ans. 99.7 sec"" 100 sec]
t in seconds 0 900 1800 47. A first order reaction is 40% complete after 8 min. How long
Concentration of A 50.8 19.7 7.62 will it'take before it is 90% complete? What is the value of the
Prove that it is a first order reaction. rate constant?
[Hint: Calculate the value of k in both cases by using first order [Ans. k 0.06386 min -I; t 36.06 min]
equation.]
48. The reaction S02Clz -7 S02 + C1 2 , is a first order gas
,38. The first order reaction has k = 1.5 x 10-6 per second at
reaction with k 2.2 x 10-5 s-I at 573 K. What per cent of
200 0 C. If the reaction is allowed to run for 10 hours at the
SOzCl z is decomposed on heating at 573 K for 90 min?
same temperature, what percentage of the initial concentration
[Ans. 11.2]
562 G.R.B. PHYSICAL CHEMISTRY FOR COMPETITIONS

49. The decomposition of AB(g) ~ A (g) + B(g), is first order when tIS, k==0.0198
4 when t 20, k = 0.0196]
reaction with a rate constant k 4 x 10- at 318 K. If AB
has 26664.5 Pa pressure at the initial stage, what will be the 60. Optical rotation of sucrose in' 1 N HCI at various times was
partial pressure of AB after half an hour? found as shown below:
[Ans. 12981.7 Pa] Time (sec) 0 7.18 18.0 27.05 00

50. In the first order reaction A (g ) ~ B (g) + C(g), the initial Rotation (deg) + 24.09 + 21.7 + 17.7 +15.0 -10.74
Show that the inversion of sucrose is a first order reaction.
pressure of A is 11200 Pa and the total pressure at the end of
16 minutes is 14667 Pa. Calculate the half life period of [Hint: Initial concentration, [A 1 == '0 - r;"
reaction. At time t, [AL='i-I~
[Ans. 30 min] k == 11.8 X 10-3 S-I]
51. It was found that a solution of cane sugar was hydrolysed to
61. The rate law for the decomposition of gaseous N2 0 S
the extent of 25% in 1 hr. Calculate the time taken for the
sugar to be 50% hydrolysed assuming that the reaction is of N 20 S ~ 2N0 2 + 0 2 h
first order. is observed to be:

52.
[Ans. 2.408 hours]
A reaction is 20% complete in 20 minutes. How long will it
- dlN 2
dt
°sJ = k[N
2
°]
S

take for 80% completion if the order of reaction is (a) zero and A reaction mechanism which has been suggested to .be
(b) one? consistent with this is:
[Ans. (a) 80 minutes '(b) 144.27 minutes] K
N2 0 S ~ NOz + NO} (fast)
53. A second order reaction where a b is 20% completed in 500
kl
seconds. How long will the reaction take to be 60% complete?
NO z + NO} ~ N02 + NO + 02 (slow)
[Hint: Apply
k2
k == x 20 _1_ mol-I L S-l NO + NO) ~ 2N02 (fast)
ta(ti''-x) 500[100x(lOO 20)] 200000
Show that the mechanism is consistent with the observed law.
60
t = = - - - - 60 x 200000 3000 seconds] [Hint: r k l [NO z ][N0 3 ],
k x 100 x 40 100 x 40
K [NO z][NO)] or [NO, ] == --"--=-"-"
54. At a certain temperature, the half life periods of a reaction at
(Nps] 0 [N0 2 ]
different initial pressures were as follows:
p (kPa) 100 66.67 33.33 r kIK[NzOs]=k[NzOs]]
t (min) 105 235 950 62. Write the overall reaction and the rate law equation from the
Find out the order of the reaction. following elementary steps:
[Ans. 3] NO + Brz ~ NOBrz
55. The time for half change for a gaseous reaction was measured
NOBr2 + NO ~ 2NOBr
for various initial pressures and the following data were
obtained: [Hint: Adding both the steps, we get overall real'; ,'1,
p (kPa) 26.6 40 53.3 2NO + Brz ~ 2NOBr.
t (min) 150 99.8 75.3 The second step is slow and, therefore, a rate determining step.
Find out th.: order of the reaction. r k'[NOBrz ][NO]
[Ans. 2]
K or [NOBr2] = K[NO][Br2]
56. Show that for a first order reaction the time required for 99% [NO][Brz]
completion is twice the time required for the completion of
90% of the reaction. r k' K[NOY[Brz] ]
57. Show that for a first order reaction the time required to 63. The reaction, 2NO + 0z ~ 2N02 proceeds in two steps. If
complete 99.9% of the reaction is 10 times that required for one elemental), reaction is NO + O 2 ~ N0 3, write the second
half the reaction. elementary reaction. Write the rate law of the reaction.
58. Show that for a first order reaction, the time required for a . [Ans. NO + N0 3 ~ 2NO z, rate = k[NO] 2 [° 2 ] ]
50% reaction is half that for 75% reaction.' . 64. At 3800 C, the halflife period for the first order decomposition
59. 1
The decomposition of an aqueous solution of ammonium ofH2 0 z is 360 min. The energy of activation ofthe reaction is
nitrite was stl.1cied. The .volume of nitrogen gas collected at
200 kJ mol-I. Calculate the time required for 75%
different inteI;Vals of time was as follows:
Time (minutts) 10 IS 20 00
decomposition at 450 0 C. (HI' ] 995)
Vol. ofN2 (mL) 6.25 9.00 11.40 35.05 [Ans. 20.39 min]
From the above data prove that the reaction is of the first order. 65. Calculate the average rate of decomposition of NzOs by the
[Hint: [A]o == volume of nitrogen at 00 == 35.05 reaction,
2N 2 0 S (g ) ~ 4NOz (g ) + O 2 (g ).
[A],=V~ V,!
. During the time interval-from t = 600 s to t = 1200 s using the
when t == 10, k = 2.303 log 35.05 0.0196 following data:
10 (35.05 6.25)
CHEMICAL KINE1'ICS; II 563

Time (NzOsl 11. Let us consider the following mechanism:


2
600 s 1.24 x 10- M CH3 CN + H + -;:::::::": CH3 CNH + (fast)
1200 s 0.93 X 10-2 M CH3CNH + + H 20 -'7 Product (slow)
What would be the rate law?
[Ans. 5.166 x 10-6 M
[Am~ Rate =:: k[CH3CN][HzO][H'"] ]
66. Consider the diagram for reaction:
U The thermal isomerization of cyclopropane occurs according
A2 + B} -;:::::::": 2AB to the equation:
(a) Calculate the t:Jl of the reaction.
CHz CH2 ---t CH 3
(b) Calculate the energy of activation for forward and ~~
backward reaction. CH2
(c) The dotted line curve is the path in the presence of a
The specific reaction rate constant for this reaction at 499 0 C
catalyst. What is the energy of activation for the
was 5.95 x 10-4 sec -I. What is the half life of the reaction?
forward and backward reactions in the presence of the
catalyst? What fraction of the cyclopropane will remain after I hour at
499.5° C?
[Alts:. 1165 s; 11.7%]
t 140 1.3. A substance' A' was found to undergo two parallel first order
reactions,
130
B
A ( , i.e.,A -'7B and A -'7C,
C
~ 110
<D with rate constants 1.26 x 10--4 S -I and 3.8 x 10- 5 s -: respecc
ill 100
tively. What were 'the percentage distributions of Band C?
[Ans.. 76.83% ofB and 23.17% ofC ]
Progress of reaction -
14. Two reactions proceed at 25° C at the same rate; the
What is the lowering in activation energy In the
temperature co,efficient of the rate of the first reaction is 2 and
presence of the catalyst?
that of the sf'!cond reaction is 2.5. Find the ratio of rates of
(d) Will the catalyst change the extent of the reaction? these reactions at 95° C.
[Ans. (a) I:lH 10 kJ I mol; [Ans. 5 times]
(b) Ef = 30 kJ / mol; Eb = 40 kJ / mol; 7S. The popUlation of India in the year 1988 was 800 million.
(c) Ef = 10 kJ / mol; Eb == 20 kJ / mol; 20 kJ / mol; Growth rate is 25 per thousand per year. What will be the
(d) no] population in the year 2000 if the growth follows first order
67. Temperature coefficient, !.I. k35"C of a reaction is kinetics?
k25"C [Ans. 1075 million]
1.82. Calculate the energy of activation in calories. (R 1.987 16. Half lives against initial pressure are given below. Calculate
cal degree -1 mol-I ). the order of the reaction.
[Ans. 10.924 kcallmol] p (rom) 750 500 250
t (min) 105 235 950
68. For first order gaseous reaction, log k when plotted against 1 Find out the order of the reaction.
T
[AM.. 3]
gives a straight line with a slope of -8000. Calculate the
activation energy of the reaction. 11. For a given reaction at temperature T, the velocity constant k,
is expressed as:
[Ans. 36.6 kcal]
k = Ae-Z7000 efT (k' Boltzmam1 constant)
69. The rate at 27° C of a chemical reaction increases 1000 times
when a suitable catalyst is introduced. Calculate the change in Given, R = 2 cal K-I morl. Calculate the value of energy of
the energy ~f activation that the catalyst has brought in. activation. Comment on the results.
[ADS. 4118 cal] [Ans. E 18 x 10-20 cal]
70. Dehydration of tertiary butyl alcohol follows a first order [Rin.~ Compare the equation with Arrhenius equation
reaction, k ... (i)
k Ae-27DOOk'IT ... (ii)
CHz(g) + HzO(g)
Eo == 27000k'
the rate constant at 300° C is 2.27 x 10-8 S -I. Calculate the rate
RT T
constant at 4000 C if the energy of activation for the reaction is
Ea == 27000Rk'
58 kcal.
[Ans. 4.392 x 10-5 27000 R x !!:.- = 27000 X (2)2
N 6.023 X 1023
,
..

G. A.B. PHYSICAL CHEMISTRY FOR COMPETITIONS

= 18 X 1O-z0 cal [Hint: Activation energy of forward reaction = 0.12 - 0.02


. ·'Activation energy is low, therefore reaction will be fast.] =0.10 kcal
78: If the activation energy of a reaction is 80.9 kJ mol-I, calculate Fraction of molecules which are active
the fraction of molecules at 400°C which have enough energy or which crossover the energy barrier (~) =
to react to form the products. .
[Ans. 5.257 x 10-7 ] kl E
·.··[Hint: Fraction of total molecules which arc active at a loge (-:4) RT
·temperattire is given as,
k antilog [ -E ]
A 2.303 RT

= antI.1 og [-O.lOX 1000 ]


k
log A = 2.303 RT 2.303 x 2 x 325

!!... = antilog ( -Ea )


= antilog [-0.06680] 0.8574
A l2.303RT . . Percentage of reactant molecules crossing over the barrier
Substituting the values of E and T we can solve the problem.] 0.8574 x 100 = 85.74] .
1f£ The following data were obtained for a given reaction at 83. In a milk at 37°C Lactobacillus acidophillus has a generation
,300;._ .. . time of about 75 minutes. Calculate the population relative to
Reaction .E~ergyof activation (kJ/mol) the initial value at 30,60, 75, 90 and 150 minutes.
[Ans. tmin 30 60 75 90
1. Uncatalysed 76
N I No 1.32 1.74 2 2.30]
2. Catalysed 57 [Hint: In the case of population growth we have to use growth
Calculate by what factor the rate of catalysed reaction is . kinetics.
increased?
[AJis. 2000]
-k 2.303 log
t
(_a_i
la+x;
[Hint: Use:
k
.J!.. == antilog
ka
f M
l---
2.303RT
)
a = initial population
x = growth in population in time' t'
, 'k p == 'rate constant in presence of Catalyst Here, No = a; N = a + x]
ka == rate constant in absence of catalyst 84. Two reactions of same order have equal pre-expbnential
M = 76 57 = 19 kJ / mol] factors but their activation energies differ by 41.9 J/mo\.
The Arrhenius equations, for cis-trans isomerization of Calculate the ratios between rate constants of these reactions
but-2-ene (CHJ - CH=CH- CHJ ) and but-2-ene, I-nitrile at 600 K.
(CHJ CH=CH-CN)are, [Ans. 0.002]
k(S-I) == IOns exp (-263.5 kJ mor l 1RT) [Hint: Use the relation,
E
k -I (s-I ) lOll exp (-214.5 kJ mol-II RT) loglO k = 10giO A ... (i)
2.303RT
Calculate the temperature at which k k -I.
... (ii)
[Ans. 914 K] 2.303 x 8.314 x 600
St. The half life for a reaction between fixed concentration of ... (iii)
reactants varies with temperature as follows: loglO k2 = loglO A - 2.303 x 8.314 x 600
tOC 520 533 555 574 El - E2 = 41.951 mol. ... (iv)
'.'
.t1/2 sec 1288 813 562 477 Subtract eq. (iii) from eq. (ii) to determine the ratio.]
Calculate the activation energy of this reaction. 85. A decomposition reaction has following mechanism
} . [Ans. 86.47 kJ mol-I] 2Nz0 5 ---t 4N02 + 02 (overall)

•[Hint: Use: log lk~


(k ) E
2.303R
(I T 1 -'
N 20 5 ----" N02 + N03 (fast decomposition)
N01 + N03~ NO + NOz + 02 (slow)
NO + N03 ---t 2NOz (fast)
k z = 0.693/ t2 Determine rate law, is the mechanism corresponds to first
k j = 0.693/ tl order reaction.
. (tl)"f
log - E (1-TI - -T1\J ] [Ans. -d[N 2 S 1=
dt
° k j + k3
[N20 S ]]
t2 ' 2.303R z
86. Rate constant of a reaction changes by 2% by O.loC rise in
.. 82•. What percentage of reactant molecules will crossover the temperature at 25°C. The standard heat of reaction is 121.6 kJ
: 'energy barrier at 325 K? Heat of reaction is 0.12 kcal and mol-I. Calculate Ea of reverse reaction .
. activation energy of backward reaction is 0.02 kcal.
[Ans. 24.7 kJ/mol]
CHEMICAL KINETICS .565 .
.~

r\kk21 f 1 1 J [Ans. Rate = [NH~ ][OCN~


[Hint: log
j ) 2.303R lT j - T2 k- I
88. For the reaction N2 (g) + 3R 2 (g) ----1 2NH3 (g), under
102 E ( 1 1 ') certain conditions of pressure and temperature of the reactants,
log 100 = 2.303 x 8.314 298 - 298. tJ the rate of fonnation of NH3 is 0.00 I kg hr~l. Calculate the
E = 1.463 X 105 J I mol 146.3 kJ I mol rate of reaction for Nl and H 1 .
Ml=Ef [Ans. 1.76 x 1O~4 kg hr ~jl
121.6 = 146.3 Eb _d[N:d Id[R2l=+.!.
[Hint:
Eb = 24.7 kJ I moll dt 3 dt 2 dt
87. Urea (NH2 )2CO can be prepared by heating ammonium
; .
= O.OOlkg hrl 0.001 x 1000 mol hr-I" ,
cyanate NH4 0CN. dt 17
j
NH4 0CN ----1 (NH2 hCO 0.058 molhr-
The reaction may occur by the following mechanism:
-d[N,l
-~-=
I x.
0058 mo Ihr··· j
dt 2
NH4+ + OCN .,,-- NH3 + HOCN
k I = 0.029 mol hr-!
(fast, equilibrium)
k, = 0,029 x 28 x 10-3 kg hr~!
NH3 +HOCN - --+ (NH2 )2 CO (slow) = 8.12 X 1O~4 kg hr~!
What is rate law predicted by this mechanism?
- -2-l
' ') ar IY ,d[R
SIml
dt

_£OBJE~IIVE·QUESTIONS
Set-1: Questions with single correct answer (c)
2d[N0 2 ] =
dt dt dt
1. Chemical kinetics, a branch of physical chemistry, deals with: d[NO]
(d) - ---C'--"-" = -.- =
(a) heat changes in a reaction dt dt dt
(b) physical changes in a reaction 6. The rate of reaction that does not involve gases, is not
(c) tates of reactions dependent on:
(d) structure of molecules (a) pressure (b) temperature
2. The rate at which a substance reaGts depends upon its: (c) concentration (d) catalyst
(a) atomic mass (b) equivalent mass 7. For the reaction, A + B 2C + D, which one is, the
(c) molecular mass (d) active mass incorrect statement?
3. The rate of a chemical reaction: (a) Rate of disappearance of A Rate of disappearance.af B
(a) increases as the reaction proceeds (b) Rate of disappearance of A Rate of appearance of D
(b) decreases as the reaction proceeds (c) Rate of disappearance of B 2 x rate of ap}".'arance of C
(c) may increase or decrease during the reaction , I
(d) remains constant as the reaction proceeds (d) Rate of disappearance ofB = - x rate of appearance of C
. '2
4. For the reaction, N2 + 3H2 2NH3 , rate is expressed as: 8. In a reaction, 2Al + B2 2A 2 B, the reactant A will
d[H2] 3 d[NH3] disappear at:
(a)-3 dt dt 2 dt
(a) half the rate that B will decrease
(b) _ d[N2] I d[H 2 ] 1 d[NH3J (b) the same rate that B will decrease
dt 3 --;Jt = '2-;Ii
(c) double the rate that A2B will fonn
d[N2] ___ 3d[H 2 ] 2d[NH 3 ]
(c) (d) twice the rate that B will decrease
dt dt dt
(d) _ d[N2] d[H2] 9. In the reaction 3A ~ 2B, rate of reaction + is equal to:
dt .
dt dt dt
[JEE (WO) 10081
5. For the re!lction, 2N0 2 ----1 2NO + O2 , rate is expressed as:
(a) .!. d[A] (b) 2 d[A] (c) + 2 (d) _ ~ d[A];
(a) _ 1 d[N0 2 ] = 1 d[NO] = d[02] 3 dt 3 dt dt 2 dt"
, 2 dt 2 dt dt 10. For the reaction, 2NO ----1 N2 + O 2 , the expression
(b)_l _.!. =d[02]
_ .!. d[NO] represents:
2 dt 2 dt dt 2 dt
566 II
(a) the rate offormation of NO !t. In general, with every 10°C rise in temperature, the rate of
(b) the average rate (Jf the reaction reaction becomes approximately ........... .
(c) the instantaneous rate of the reaction (a) ten times (b) double
(d) all of the above (c) half (d) one tenth
itt. The term dx in the rate expression refers to the: 11. Activation energy of the reaction is:
dt (a) the energy released during the reaction
(a) instant.aneous rate of reaction (b) the energy evolved when activated complex is form(~d
(b) average rate of reaction (c) minimum amount of energy needed to overcome the
(c) in<::rease in the concentration of reactants potential barrier
(d) '.;oncentration of reactants (d) the energy needed to form one mole of the product
tf.2. According to collision theory of reaction rates: n. The temperature coefficient of most of the reactions lies
'. (a) every collision between reactant molecules leads to a between:
chemical reaction (a) 2 and 3 (b) I and 2
(b) rate of reaction is proportional to the velocity of the molecules (c) 2 and 4 (d) 3 and 4
(c) rate of reaction is proportional to the average energy of the According to Arrhenius equation, rate constant of a chemical
molecules reaction is equal to:
(d) rate of reaction is proportional to the number of collisions IRT (b) Ae E" IRT
(a)
per second (c) Ae
RT1E
" (d) Ae- NT / Ea
iJ'S. According to collision theory: .14. The correct expression for Arrhenhr - ,;t,r,' .ne
(a) all collisions are sufficiently violent effect of temperature on the rate constant is (T2 > 1\ ):
(b) all collisions are responsible for product fonnation !1'CE :::f'~H'
(c) all collisions are effective
(d) only a fraction of collisions are effective which have
enough energy to form products
a 10 k2 - [ 1\T2
() glo k) -2.303R T2 -1)
l~
n.. Which of the following has been used to explain the subject of
(b)
R
chemical kinetics?
(a) Collision theory of bimolecular reactions
(b) The activated complex theory (c) loglo k T.T
(c) Arrhenius equation ) 1 2

(d) All of the above [T2 - 1) ]


(d) 10 k2 _
lfS. The activation energy for a chemical reaction depends upon: g)O k) - 2.303R 7 1\
2
(a) temperature
(b) nature ofreacting species
a In order to react, a molecule, at the time of collision, must
possess a certain amount of energy known as:
(c) concentration of the reacting species (a) free energy (b) kinetic energy
(d) collision frequency (c) threshold energy (d) internal energy
1.1.16. In a reaction, the threshold energy is equal to: l6. The activation energy of a reaction is zero. The rate constant
(a) activation energy + normal energy of reactants of the reaction: ri ~ 9~;
(b) activation energy - normal energy of reactants (a) increases with increase of temperature
(c) normal energy of reactants activation energy (b) decreases with decrease of temperature
(d) average kinetic energy of molecules of reactants (c) decreases with increase of temperature
11~. The influence of temperature on the rate of reaction is (d) is nearly independent of temperature
determined by: 7lJ1.. The activation energy of a reaction can be lowered by:
(a) Nernst's equation (b) Gibbs-Helmholtz equation (a) using a positive catalyst
(c) Arrhenius equation (d) van't Hoff equation (b) increasing temperature
1l~. According to the collision theory of reaction rates, the rate of
(c) decreasing temperature
reaction increases with temperature due to:
(d) increasing concentration of the reactants.
(a) greater number of collisions
A The activation energy for a hypothetical reaction A ~ Product
(b) higher velocity of reacting molecules
is 12.49 kcallmol. If temperature is raised from 295 to 305, the rate
(c) greater number of molecules having the activation energy
of reaction increases by:
(d) decrease in the activation energy
(a) 60% (b) 100%
l~. The temperature coefficient of a reaction is:
(c) 50% (d) 20%
(a) ratio of rate constants at two temperatures differing by 1°C
::29. The plot between concentration versus time for a zero order
(b) ratio of rate constants at temperature 35°C and 25°C reaction is represented by: ,.\,\H li.;x;
(c) ratio of rate constants at temperature 30°C and 25°C
(d) specific reaction rate at 25°
CHEMICAL KINETICS 567

(a) first order in A (b) . second order in B


j j (c) third order overall (d) all are COlTect
38. For the reaction A ---'t B, the rate of reaction is quardrupled
(a) C (b) InC
when the concentration of A is doubled; the rate of the
I I r;
reaction is r k[ A the value of n is:
(a) I (b) zero (c) 3 (d) 2
-t- -t-
39. Select the rate law that corresponds to the data shown for the
following reaction: A + B ---'t·e (CB"E 1994)

j j Expt. No. [AJo [Blo Initial rate


(e) C (d) C 1. 0.012 0.035 0.10
I I 2. 0.024 0.070 0.80
3. 0.024 0.035 0.10
-t- -t- 4. 0.012 0.80
30. The rate of reaction increases with rise in temperature because (a) rate k[B]} (b) rate k[ B]4
of: [CoOled (Karnataka) 20081
(c) rate k[A][B]3 (d) rate k[A]2[Bf
(a) increase in the number of activated molecules
(b) increase in the activation energy 40. For a chemical reaction, A E, it is found that rate of
(c) decrease in the activation energy reaction is doubled when the concentration of A is increased
(d) increase in the number of molecular collisions four times. The order ofthe reaction is:
31. Which of the following is a first order reaction? (a) I (b) 2
(a) 2HI ---'t + 12 (b) 2N02 ---'t 2NO + O2 (c) 1/2 (d) zero
(c) 2NO+ O 2 ---'t 2N0 2 (d) NH4NO, ---'t N, + 2H,O 41. The rate of reaction, A + B ---'t Products, is given by the
32. Which one of the following is not a first o;der reaction? - equation. r k[A J[B]. If B is taken in large excess. the order
H+ of reaction would be: (BIT 1992)
(a) CH}COOCH} + H2 0 ---'t CH}COOH + CH)OH (a) 2 (b) 1
(b) CH)COOC2 Hs + NaOH ---'t CH)COONa + C2H sOH (c) zero (d) unpredictable
(c) 2H 2 0 2 ---'t 2H 20 + O 2 42. The rate of reaction between A and B increases by a factor of
(d) 2NzOs ---'t 4N02 + 02 100, when the concentration of A is increased 10 folds. The
order of reaction with respect to A is:
33. Rate expression of a chemical change is dx = k[Af[B][Ct (a) 10 (b) 2 (c) I (d) 20
dt
The order of reaction is: 43. The rate law for the reaction, .
(a) 3 (b) 2 (c) I (d) zero RCI + NaOH(aq.) ~ ROH + NaCI
34. For a reaction, 2A + B ---'t C + D, the concentration of B is is given by, Rate k[RCI]. The rate of reaction will be:
(a) unaffected by increasing temperature of the reaction
kept constant and that of A is tripled the new rate of reaction
will be: (b) doubled on doubling the concentration ofNaOH
(a) nine times (b) three times (c) halved on reducing the concentration ofNaOH to one half
(d) halved on reducing the concentration of RCI to one half
(c) same (d) zero ".
35. The rate ofthe reaction, A + B + C ---'t Products, is given by
44. A zero order reaction is one:
(a) in which reactants do not react
_ d[A] = k[A ]1/2[B f3[c ]1/4 (b) in which one of the reactants is in large eXces~
dt (c) whose rate is unifonn and not affected by time
The order of the reaction is: (d) whose rate increases with time
(a) 112 (b) l3112 45. The rates of a certain reaction at different time intervals are as
(c) I (d) 2 follows:
36. Which of the following rate laws has an overall order of 0.5 Tiine (second) 0 10 20
for the reaction involving substances x, yand z? Rate 1.8x 1O~2 1.82x 10~2 l.79x 1O~2 mol L- 1
(a) Rate = ~[Cx][Cy][Cz]
(b) Rate k[Cx t 5 [C ]O.5[C ]05
v z
The reaction is of:
(a) zero order (b) first order
(c) Rate = k[Cxt5[Cyrl[Cz]O (c) second order (d) third order
46. F or which of the following, the units of rate and rate constant
(d) Rate==k[Cx][Cyr2[Cz]o of the reaction are identical? ;'
37. If the rate of reaction between A and B is given by rate (a) First order reaction (b) Zero order reaction
== k[A][B]2, then the reaction is: (e) Second order reaction (d) Fractional order reaction
568 G.R.B. PHYSICAL CHEMISTRY FOR COMPETITIONS

47. The unit of rate constant for a zero order reaction is: 59. The following data are for the decomposition of ammonium
(a) mol time-I (b) Ltime- I nitrite in aqueous solution:
1
(c) mol L- time- 1
(d) L mol-I time- 1 Vol. ofN2 incm3 6.25 9.0 11.42 13.65 35.02
48. Fora zero order reaction: IEAMeE! ;(90) Time (min) 10 15 20 25 00

1 1
(a) t1/2 0<: a (b) t1/2 oc;; (e) tll2 cc a 2
(d) tl/2 a2cc The order of reaction is:
Ca) 3 (b) 2 (c) (d) zero
.49. The rate of the first order reaction, A Products, is
[Hint: a 35.02, (a - x) = 35.02 VI
4
7.5 x 10- mol , when the concenlla,~,)fl of A is 0.2 mol = 2.303 10 35.02
Apply, k
L- 1 , The rate constant of the reaction is: t g (35.02 Vt)
(a) 2.5 x 10- 5 S-1 (b) 8.0 x 10~ S-1
60. In the first order reaction, the concentration of reactant
(c) 6.0x 10-4 S-I (d) 3.75 x 107'3 decreases from 1.0 M to 0.25 M in 20 minutes. The value of
specific rate is:
50. If the concentration is expressed in mole per litre and time in
second, the units of rate constant for a first order reaction is: (a) 69.32' . (b) 6.932 (c) 0.6932 (d) 0.06932
(a) mol L-1 (b) 6L The half life period of a first order reaction is 10 minutes. The
time required for the concentration of the reactant to change .'
(c) mol (d) mol-I from 0.08 M to 0.02 M is: (1fT i99i))
51. 'Which one of the following formula represents a first order
(a) 10 min (b) 20 min (c) 30 min (d) 40 min
reaction?
62. The rate constant for the hydrolysis reaction of an ester by
- - Iog
(a) ' k --2.303 (b) k = 1. x dilute acid is 0.6931 x 10-3 S-I. The time required to change
t [A]o t a(a x)
the concentration of ester from 0.04 M to 0.01 Mis:
(c) k = 2.303 log _a_ (d) k
2.303 I
- - og-
Ii
(a) 6931 sec (b) 4000 sec
t a- x t . x
(c) 2000 sec (d) 1000 sec
52. 75% of a first order reaction was completed in 32 minute;
63. The rate of a gaseous reaction is given by the expression
when was 50% of the reaction completed?
k[A][B]. Ifthe volume of reaction vessel is suddenly reduced
(a) 4 min (b) 8 min (c) 24 min Cd) 16 min to one-fourth of the initial volume, the reaction rate relative to
53. The first order rate constant for the decomposition ofN2 0 s is the original rate will be: (lIT 1992)
6.2 X 10-4 The half life period for this decomposition is:
• (a) 1/16 (b) 1/8 (c) 8 (d) 16
(;\iL'iR 19q i) 64. The velocity of a reaction is doubled for every 10°C rise in
(a) 1117.7 s (b) 1117 s (c) 223.4 s (d) 160.9 s temperature. If the temperature is raised to 50°C, the reaction
54. The half life period of a first order reaction is: velocity increases by about:
Ca) 0.693 (b) 0.693 (c) 0.69~ (d) 0.693 (a) 12 times (b) 16 times (c) 32 times (d) 50 times
a k t ak
65. If the rate expression for a reaction is dx = k[A]1I2[B]3i2, the
55. The specific rate constant of a first order reaction depends on: dt
Ca) concentration ofthe reactants overall order of the reaction is: [CEE (Bihar) 19921
. (b) concentration of the products (a) 2 (b) 112 (c) 3/2 (d) 1
(c) time 66. The rates of a certain reaction at different times are as follows:
(d) temperature Time (second) 0 10 20 30
56. For a given reaction of first order, it takes 20 minutes for the Rate 2.8 x ro- 2 2.78 x 10-2 2.81 x 10-22.79 x 10-2
concentration to· drop from 1.0 M to 0.6 M. The time required (mol L- 1
for the concentration to droP. from 0.6 M to 0.36 M will be:
(a) more than 20 minutes (b) less than 20 minutes The reaction is of:
(c) equal to 20 minutes (d) infinity (a) third order (b) second order
57. For the first order reaction A ------* Product, the half life is 200 (c) first order (d) zero order
. seconds. The rate constant of the reaction is: 67. The second order rate constant is usually expressed as:
(a) 3.46 x 10-2 S-I (b) 3.46 x 10-3 s-! (a) mol (b) mor l L- I S-I

(c) 3.46 x 10~ S-I (d) 3.46 X 10- 5 S-l (c) mol- 1 Ls- 1 (d) mol Ls- I

58. The following data were obtained for the thermal 68. The hydrolysis of ethyl acetate is a reaction of:
H+
decomposition of nitrous oxide on finely divided gold:
CH3 COOC2Hs + H 20 ------* CH3 COOH + C 2H sOH
t (minutes) 0 20 40 60 80
(a) zero order (b) first order
Concentrati{)n of N 20 (mol LI) 0.10 0.08 0.06 0.04 0.02
(c) second order (d) third order
The order of reaction is:
(a) zero (b) I (c) 2 (d) 3
CHEMICAL KINETICS 569

69. The inversion of cane sugar is represented by, [Hint: r k'[03][0] k'kllf~:i[03] k[Od[Ozr 1 ]

78. Time required to decompose half of the substance for nth


It is a reaction of: tpMT (MP) 19931 order reaction is inverse1y proportional to:
(a) second order (b) unimolecular (a)'a"+ I (b) a"-I
(c) pseudo-unimolecular . (d) zero order (c) an 2 (d) an
70. In a second order reaction, the time needed for the initial 79. What is the order of the reaction which obeys the expression
concentration of the reactant to reduce to halfthat value is: 1
(a) independent of the initial concentration 11/2 =-?
ka
(b) proportional to the initial concentration (a) Zero (b) Third
(c) inversely proportional to initial concentration
. (c) First . (d) Second
(d) proportional to the square of the initial concentration
80. ·The order of reaction can be deduced from:
71. Which of the following statements regarding molecularity of
(a) experiment \ (b) chemical equation
the reaction is wrong?
(c) rate constant ' (d) thermochemical equations
(a) It may be either whole number or fractional
81. The order of a reaction may be determined by: .
(b) It is calculated from the reaction mechanism
(a) differential method (b) initial rfte method
( c) It depends on the rate determining step
(c) graphical method (d) all of these .
(d) It is number of molecules of reactants taking part in a
82. A graph between time (t ) and the substance. consumed at any
single step chemical reaction
72. For the reaction, H2 (g ) + Br2 (g) ---j 2HBr(g), the reaction time is found to be a straight line passing through the origin.
rate is: This indicates that the reaction is of:
t
rate == k[H 2 ] [Br2 2
(a) second order
(c) zero order
(b) first order.
(d) fractional order
Which one of the following statements is true for this 83. A plot of log (a - x) against time' t' is a straight line. This
reaction? indicates that the reaction is of:
(a) The reaction is of second order (a) zero order (b) first order
(b) The molecularity of the reaction is 3/2 (c) second order (d) third order
(c) The unit of k is sec- 1 1
84. For the reaction N 20S(g)--t 2N02 (g) + "2 O2 (g1 the
(d) The molecularity of the reaction is two
73. Molecularity of the reaction: value of nite of disappearance ofN20 s is given as 6.25 xl 0-3
(a) is always a whole number mol . The rate of formation of N0 2 and O 2 is given
(b) is never a whole number
respectively a s : · ' [eBSE (PMT) 2ihOi
(c) can have a fractional value (a) 6.25 x 10-3 mol CiS-I and 6.25 x JO-3 mol L-1s- 1
(d) can be zero
(b) L25xlO-2 moIL- 1s- 1 and 3.125 x 10-3 mol L-1s-1
74. For a single step reaction, A + 2B Products, the
(c) 6.25 x 10-:3 mol L-1s- 1 aIid3.125x 10-3 mol .
molecuiarity is: (d) 1.25 x 10-2 mol L-1s- 1 and 6.25>< 10- 3 mol L-1s- 1
(a). zero (b) three (c) two (d) one
[Hint: -d[N z0 5 1 +! d[NP5] +2d[ 0 2]
75. The hypothetical reaction, A z + B2 ----72AB, follows the dt 2dt dt
following mechanism: 0
'd[N2 5] == 2{-d[NP51} = 2x6.25xlO-3
A2 A+A (fast) dt . . dt
A + B2 ~ AB + B (slow)
== L25 x 1O-2mol L-1g- 1
A + B ~ AB (fast)
The order of the overall reaction is: IPMT (Pb.) 19911 d[02] ==!{ -d[N 20s1} =! x 6.25 X 10-3
dt 2 dt 2
(a) zero (b) 1 (c) 2 (d) 3/2
76. The molecularity of a reaction is: == 3.125 x 1O-3 mol L-1g- 1 ]
(a) same as its order 85. The ratio of the time required for ~th of the reaction of ·first
(b) different from order order to complete to that required for half ofthe reaction:
(c) may be same or different as compared to order (a) 4: 3 (b) 3: 2
(d) always zero (c) 2: 1 (d) 1 : 2
77. The chemical reaction, 20 3 ----7 30 2 proceeds as follows: 86. In pseudo-unimolecular reactions:
0 3 ~ O2 + 0 (fast) . (a) one ofthe reactants is.present in large excess
0+ 0 3 ~ 20 2 (slow) (b) both the reactants have same concentration
The rate law expression should be: (c) both the reactants.are present in low concentration
(d) one of the reactants is less reactive
(a) r k[03]2 (b) r== k[03f[Ozr I
(c) r k[03][02] (d) r [0 3 ][02]2
570 G.R.B. PHYSICAL CHEMISTRY FOR COMPETITIONS

87. The reaction, (a) rate k[NOf[Cl z ] (b) rate = k[NO] [CI 2 ] 2
2N 20 5(g) 4NO z(g) + °2(g), (c) rate = k[NOCl z] . (d) rate = k[NOCI] 2
is first order with respect to N 2 0 5 . Which of the following
graph, would yield a straight line? [Hint:
(a) log (PN 20 S ) versus time ~ith -ve slope rate = k' [NO)[NOCl z ] k' K[NO][NO][Clzl k[NO:rrCI 2 ]l

(b) (PN 20 S rl versus time 94. The rate of a reaction can be increased in general by all the
following factors except:
(c) (PN 20 S ) versus time (a) by increasing the temperature
(d) log (PNzOs ) versus time with +ve slope (b) using a suitable catalyst
88. Consider the following statements: (c) by increasing the concentration of reactants
(i) increase in concentration of reactant increases the rate of a (d) by an increase in activation energy
zero order reaction
95. How will the rate r k[80 2 ][Oz] of reaction,
(ii) rate constant k is equal to collisionfrequency , A' if Eo 0
(iii) rate constant k is equal to collision frequency A if Ea 00 2S0 2 + 02 -----72S0 3 ,
(iv) log e k vs T is a straight line . change if the volume of the reaction vessel is tripled?
(v) loge k vs II T is a straight line (a) It will be '!rd of its initial value
. 3
Correct statement are: [PMT (Kcrala) 2010]
(a) (1) and (iv) (b) (ii) and (v) . (b) It will be 3 times of its initial value
(c) (iii) and (iv) (d) (ii) and (iii) (c) Jt will be 9 times of its initial value
(e) (i) and (v) (d) It will be .!th of its initial value
1 . . 9
89. Ifwe plot a graph between log k and T by Arrhenius equation, .. 96. Radioactive disintegration is an example of:
the slope is: IEAMCET (Engg.) 20101 (a) zero order reaction (b) first order reaction
(c) second order reaction (d) third order reaction
(a) -EaIR (c) (d) Ea·
2.303R + 2.303R 97. In .the formation of sulphur trioxide by the contact process,
90. The rate of the reaction, . 280 2 + O 2 2803' the rate of reaction was measured as
CC1 3CHO + NO CHC1 3 + NO + CO, d[02] 3.0x 10-4 mol L- I S·-I.
dt
is given by the equation, rate = k[CCI 3 CHO][NO]. If
The rate of reaction expressed in terms of 803 will be:
concentration is expressed in moll litre, the units of k are:
. [PMT (MP) 19931 (a) 3.0 x 10-4 mol (b) 6.0 x 10-4 mol S-I

(a) mol-2 L2 S-I (b) mol S-I (c) 1.5 X 10-4 mol L- I S-I (d) 4.5 x 10-4 mol L- I S-I
(c) L mo.-I S-I (d) S-I 98. The rate constant for a first order reaction is 6.909 min -I.
91. In a reaction, 24 + B ---; A2B, the reactant A will disappear Therefore, the time required in minutes for the participation of
75% of the initial reactant is: (PET (J&K) 2009)
at: !PMT (MP) 19931
2 2 3 3
(a) half the rate that B will decrease W-~2 ~-~4 W-~2 ~ ~4
(b) twice the rate that B will decrease 3 . 3 2 2
(c) the same rate that B will decrease 99. For the reaction, .
(d) the same rate that A2B will form A + 2B -----7C + D, d[A] k[A][BJ2
92. The rate of the reaction, 2NO + 02 ---; 2N0 2 , at 25°C is dt
0.028 mol L- I s -l. The experimental rate is given by . the expression for d[B] will be:
dt
r= k[NO]2[OZ]
If the initial concentrations of the reactants are O2 = 0.040 . (a) 2k[A ][Bf (b) .! k[A ][B]2
2
mol and NO = 0.0 I mol , the rate constant of the
(c) k[A][Bf (d) k[A][BI2f
reaction is:
100. Which of the following statements is not correct for the
(a) 7.0 x 10-2 L morl
reaction,
(c) 7.0 X 102 LZ mor2 S-I (d) 7.0 x 103 L2 mol-z s-l 4A + B 2C + 2D?
[Hint: 0.028 k[0.01Y[0.040]] (a) The rate of appearance of C is one-half the rate of
93. For the reaction, disappearance of B
2NO + Cl 2 2NOCI , (b) The rate of disappearance of B is one-fourth ofthe rate of
disappearance of A
the following mechanism has been proposed.
(c) The rate of formation of D is one-half of the rate of
NO + Cl z NOCl 2 (fast)
consumption of A
NOCl z + NO ---; 2NOCI (slow)
(d) The rates offormation ofC and D are equal
The rate law for the reaction is:
CHEMICAL KINETICS '1 571

101. The overall rate of a reaction is governed by: (b) d[Br2J _~ d[Br J
(a)
(a) the rate of the fastest intermediate step dt 5 dt dt 5 dt
(b) the sum total of the rates of all the intermediate steps
5 d[Br-]
(c) the average of the rates of all the intermediate steps (c) (d) :::::+----
(d) the rate of the slowest intermediate step
dt 3 dt dt 3 dt
102. The rate constant of a reaction does not depend upon: Ill. Given that k is the rate constant for some order of any reaction
(a) temperature at temp. T, then the value of lim log k, (where, A is the
Arrhenius constant) is: T -> =
(b) activation energy
(c) catalyst (a) A /2.303 (b) A (c) 2.303A (d) togA
(d) concentration of reactants and products 112. The curve depicts the change in
concentration of species X and
103. The half-life period of a first order chemical reaction is 6.93 . Y as a function of time. The ,g X
minutes. The time required for the completion of 99% of the
point of intersection of the two
chemical reaction will be: (log 2::::: 0.301) (AIEEE 2(09) ~
curves represents: 'E
(a) 230.3 minutes (b) 23.03 minutes
(c) 46.06 minutes (d) 460.6 minutes
(a) 1112 ~
104. For the chemical reaction x ~ y, it is found that the rate of
(b) t3/4 . 8 "--'--------
(c) t 2l3 lime-
reaction increases 2.25 times when the concentration of x is (d) data insufficient to predict
increased 1.5 times. What is the order ofreaction? 113. In gaseous reactions, important for the understanding of the-
(a) One (b) Two (c) Three (d) Zero upper atmosphere H2 0 and 0 react bimolecularly to form two
[Hint: r = k[x t and 2.25r = k[1.5x r OH radicals. Mf for this reaction is 72 kJ at 500 K and Ea is
So, 2.25 = (1.5t or a = 2] 77 kJ mol-I; then for the bimolecular recombination of two
105. The halflife period of a reaction is constant for: OH radicals to form H2 0 and 0 is:
(a) zero order (b) first order (a) 3 kJ mor l (b) 4 kJ mol-I
(c) second order (d) none ofthese (c)'5kJrnol- 1
(d) 7kJmol- 1
106. From different sets of data of tl12 at different initial
114. From the following data, the activation energy for the reaction
,?oncentrations say 'a' for a given reaction, the [11/2 X a] is (cal/mol) is:
found to be constant. The order of reaction is:
(a) zero (b) I (c) 2 (d) 3
107. Diazonium salt decomposes as
C6Hs~C1- ~ C6HsCI + N 2·
At O°C, the evolution ofN2 becomes two times faster when the
initial concentration of the salt is doubled. Therefore, it is:
(\IL\R i9 4 -i} (a)4xl04 (b) 2x 104
(a) a first order' reaction
(c) 8x 104 (d) 3 \10
4
(b) a second order reaction
(c) independent of the initial concentration of the salt 115. The hydrolysis of an ester was carried out separately with-
(d) a zero order reaction 0.05 NHCl and 0.05 NH2 S0 4 . Which of the following will be
108. For the reaction, N2(g)+3H2(g)~2NH3(g), under true?
certain conditions of temperature and pressure of the reactants, (a) k HC1 >kH2S04 (b) kH2S04 >kHCI
'. (c) KH 2S0 4 = 2kHC1 (d) kH2S04 = k HCI
the rate offormation of ammonia is 0.001 kg hr- I . The rate of
consumption of hydrogen under the same conditions is ...... kg 116. For an endothermic reaction, where, Mf represents the
hr- I . enth'ilJpy of the reaction in kJ/mol, the minimum value for the
energy of activation will be:
(a) 0.001 (b) 0.003 (c) 0.002 (d) 0.0015
(a) less than Mf (b) zero
109. The rate constant, the activation energy and the Arrhenius (c) more than Mf (d) equal to Mf
parameter of a chemical reaction at 25°C are 3.0 x 10-4
117. In the following first order competing reactions;
104.4 kJ mor l and 6.0 x 1014 S-I respectively. The value of . A + Reagent Product
rate constant at T ~ 00 is: (iIT 1996) B + Reagent Product
(a) 2.0xlOI8 s- J (b) 6.0 x 1014 S-I . The ratio of kl / k2 if onfy 50% of B will have been reacted
(c) infinity (d) 3.6xW30s-1 when 94% of A has been reacted is:
110. In the reaction (a) 4.06 (b) 3.06
(c) 2.06 (d) 0.06
BrO'3(aq.}+ 5Br-(aq.)+ 6ft' ~3Br2(l}+ 3H2 0(l}
118•. Two reactions A Products and B ~ Products, have rate
The rate of appearance of bromine (Br2 ) is related to rate of
constants k,4 and kB at temperature T and activation energies
disappearance of bromide ions as following:
EA and EB respectively. If kA > kB and EA < EB and
!CBSE. (Pi\1T) 200(11
assuming that A for both the reactions is same, then:
572 G.R.B. PHYSICAL CHEMISTRY FOR COMPETITIONS

(a) at higher temperatures k A will. be greater than kB (a) 2kl k2 4k3 (b) kl = k2 = k3
(b) at lower temperatures k A and kB will be close to each (c) 2kl 4k2 k3 (d) none of these
other in magnitude 127. The rate constant is numerically the same for three reactions of
(c) as temperature rises, k A and kB will be close to each other first, second and third order respectively. Which one is true for
in magnitude the rates of the three reactions if the concentration of the
(d) at lower temperature kB > kA reactant is than 1 M?
119. A substance undergoes first order decomposition. The (a) l'j 12 r3 (b) l'j > 12 > '3
decomposition follows two parallel first order reactions as: (c) l'j < 12 < 13 (d) All of these

A< kl
klB 128. In the Q.No. 127, if the concentration of the reactant is less
than 1 M, then:
= 1.26 X 10-4 S-l and k2 = 3.8 X 10-5 S-l
(a) l'j = 12 13 (b) l'j > 12 > 13
k2 C (c) l'j < 12 < lj (d) all of these
The percentage distributions of Band Care: 129. In the Q.No. 127, if the concentration of the reactant is 1 M,
(a) 80%B and 20%C (b) 76.83% B and 23.17% C then:
(c) 90% Band 10% C (d) 60% B and 40% C (a) l'j 12 13 (b) l'j >·rz > 13
120. Inversion of a sugar follows first order rate equation which can (c) 11 < r2 < 13 (d) all of these
be followed by noting the change in rotation of the plane of 130. For a first order reaction, A - - - t Product, the rate of reaction
polarisation of light in a polarimeter. If 1:", li and '0 are the at[A] O.2mol L- 1 is l.Ox 10-2 mol min-I. The halflife
fotations at t = 00, t = t and t = 0, then first order reaction can period for the reaction is: (liT 1999)
be written as: (a) 832s (b) 440s (cl416s. (d) 14 s
(a) k = ~ loge li - r= (b) k = ~ loge 10 - I';,.,
131. The rate constant of a first order reaction, A - - - t Products, is
t '0-1';,., t li-ro 60x 10-4 min -I. Its rate at [A] 0.01 mol would be:.
(lIT 1997)
(c) k ~ loge r~ - '0 (d) k = ~ loge 1:" -li (a) 60x 10-6 mol (b) 36x 10-4 mol L- 1 min-I
t r~-li t r~-rO
f. (c) 60 x 10-2 mol 36 x 10- 1 mol L:"! min-I
121. Rate ofa chemical reaction can be kept constant by:
132. For.a first order reaction, the half life is independent of: .
(BHU 1993)
(a) stirring the compounds (CBSE 1999)
(a) initial concentration
(b) keeping the temperature constant
(b) cube root of initial concentration
(c) both (a) and (b)
(c) first power of final concentration
. (d) none of the above (d) square root of final concentration
122. The number of molecules of the reactants taking part ·in a 133. Activation energy of a chemical reaction can be determined
single step of the reaction tells about: (AMU 1992) by: (CBSE 1998)
(a) molecularity of the reaction (a) changing concentration of reactants
(b) mechanism of the reaction (b) evaluating rate constant at standard temperature
(c) order of reaction (c) evaluating rate constants at two different temperatures
Cd) all of the above (d) evaluating velocities of reaction at two different tempera-
123. Number of moles of a substance present in 1 litre volume is tures
known as: [PMT (MP) 1993) 134. The experimenta~ d~ta for the reaction 2A + B2 ~ 2AB, is:
(a) activity (b) molar concentration
(c) active mass Cd) none of these
EIpt.No. (A) •.ad "("'S-I)
124. The inversion of cane sugar into glucose and fructose is: 1. 0.50 0.50 1.6 x 10-4
Ca) I order (b) II order (c) III order (d) zero order 2. 0.50 1.00 3.2 X 10-4
125. The unit of rate constant obeying the rate expression
3. 1.0 l.00 3.2 X 10-4
r k[AnBf!3 is:
(a) mol-213 litre 2f 3t irne:"I (b) moI 2!3litre-2f3 tirne- I The tate equation for the above data is: (CBSE 1997)
(c) mol-513
litre Sl3
time-I (a) rate = k[B 2 f (b) r.ate k[B2J
(d) none of these
126. For the reaction, (c) rate k[Af[Bf (d) rate k[Af[B]
N 20 S ---? 2N0 2 + 11202, 135. For the reaction A ~ B, the rate law is: rate k[A]. Whichof
- d[N 20 S ] ='k [N
dt 1 2
°]
5
the following statements is incorrect? IPMT (Ph.) 19981
(a) The reaction follows first order kinetics
d[N0 2 ]=k[NO ] (b) The of the reaction depends on initial concentration
dt 2 2 5
(c) k is constant for the reaction at a constant temperature
d[02] ~ k [N 0 ] (d) The rate law provides a simple way of predicting the con-
dt 3 2 S
centration of reactants at any time after the start of the re-
The relation in betWeen ki , k2 and k3 is: action
CHEMICAL KINETICS 573

136. Cyclopropane rearranges to fonn propene: (a) frequency factor (b) activation factor
!J,.---'t CH 3 CH CH 2 (c) pre-exponential factor (d) Boltzmann factor
] 43. Unit of frequency factor (A ) is:
This follows first order kinetics. The rate constant is
2.714 X lO-3 sec-I. The initial concentration of cyclopropane is (a) moles/lit (b) moles/lit/sec
0.29 M. What will be, the concentration of cyclopropane after (c) depends upon order (d) no unit
100 sec? IJEE (Orissa) 2009] 144. For the reaction A + B C + D, the variation of the
concentration of the products is given by the curve:
(a) 0.035 M (b) 0.22 M
(c) 0.145 M (d) 0.0018 M t
c::
137. The rate constantfor the reaction, o
1 ~
N20S(g)~ 2N0 2 (g)+ 2 02 (g), 'E
(l)
o w
c::
is 2.3 x 10-2 sec-I. Which equation given below describes the o
Q
""'-----~-""-
change of[N 2 0 s ] with time [N 2 0 s 1o and [N 20 S ] t correspond
Time-
to concentration ofN20 s initially and time 't'?(AIlMS 2004)
(a) [N 20 S ]o =[N2 0 s 1 it (a) x (b) y (c) z , (d) 111

145. Which graph shows zero activation energy?


(b) loge [N2 0 S ]0 = kt
(a)l~ (b)l~
[N2 0 S ],
(c) ioglQ [N20S]/ = log 10 [N 20sJo kt
(d) [N20S]/ = [N2 0S]O + kt Reaction- Reaction-
138. The reaction, X ~ Y (Product) follows first order kinetics.

(c)l~
, In 40 minutes, the concentration of X changes from 0.1 M to
0.025 M, then the rate of reaction when concentration of X. is
0.0 I M is: lilT (S) 20041
(a) 1.73xlO-4 M/min (b) 3.47xlO-s M/rnin Reaction-
4 5
(c) 3.47 x 10- M/min (d) 1.73 x 10- M/rnin ]46. EThreshold can never be:
139. In a first order reaction, the concentration of the reactant, (a) > ER (b) > Ep
decreases from 0.8 M to 0.4 Min 15 minutes. The time taken
(c) < Ep (d) > ER as well as >
for the concentration to change from 0.1 M to 0.025 Mis:
147. A reaction takes place in three steps; the rate constants are

(a) 30 min (b) 15 min (c) 7.5 min (d) 60 min k], k2 and k3. The overall rate constant k =: k]k3 . If energies
k2
140. A substance undergoes first order decomposition. The
of activation are 40, 30 and 20 kJ, the overall energy of
decomposition follows two parallel first order reactions as;
activation is:
A<klB k] = 1.26 X 10-4 sec-I (a) 10 (b) 15 (c) 30 (d) 60
148. For hypothetical reaction A ~ B takes place according to
k C k2 3.8 X 10-5 sec-] k2
2
A C, A +C~D(slow)
The percentage distribution of Band Care:
Rate. law will be:
IPMT (Kerala) 2004)
(a) k 2 [A][C] (b) k]k2[A] (c) k,k2[A]2 (d) klk2[AJ[C]
(a) 75% Band 25% C (b) 80% Band 20% C
149. If concentration of reactant is 'increased by .om'; then k
(c) 60% Band 40% C (d) 90% B and 10% C becomes:
(e) 76.83%Band23.l7%C (a) ek!m, (b) k (c) k/ m (d) mk

]41. For a reaction 2NH3 N2 + 3H2 , it is observed that 150. aA + bB ~ P; k[At . [Bt If conc. of A is
dx/ dt
doubled, rate is doubled. If B is doubled, rate becomes four
.:...d( NH3) = k (NH ), d(N 2 ) k (NH ), d(H 2 ) =: k (NH ).
dt I 3 dt 2 3 dt 3 3 times. Which is correct?
What is the relation between k] , k2 and k3 ? (a) -d[A] =: (b) -d[A]::=_-"---,,
dt dt dt dt
~"~'~l '" k2 =: k3 (b) 3k! 6k2 =: 2k3
(c) 2k! =: l,k =: 6k3 (d) 6k1 3k2 =: 2k3 (c) -2d[A] (d) None of these
dt dt
142. In Arrhenius equdti<)n k =: i RT, factor e - Ea ! RT is known

as:
574 G.R.B. PHYSICAL CHEMISTRY FOR COMPETITIONS

151. A drop of solution (volume 0.05 mL) contains 3 x 10-6 mole of 158. ForX--7Y;k 10IOe-500IT,andforW--7Z;k 1012 e-IOOO!T
7 1
W. If the rate constant of disappearance ofH+ is 10 mol litre- at what temperature 'T' both reactions will have same value of
sec-I, how long would it take for W in the drop tel disappear? k?
(a) 6 x 10-8 sec (b) 6 x 10-9 sec (a) 500 K (b) 1500 K
(c) 6 X 10- 10 sec (d) 6 x 10-12 sec (c) 4.606 K (d) 2.303 K
500 5000
152. For the reaction, Hz + l z 2Hl. The rate law expression 159. For a reversible reaction where the forward reaction is
exothermic, which of the following statements is correct?
is: {JEE (WB) 20101
(a) - -1 -
d[RI]]
- :::: kl [H2 ][I2 ] (a) The backward reaction has higher activation energy than
[ 2 dt the forward reaction
(b). The backward and the forward processes have same
(b) [_ I = --'-''---''-_
activation energy
2 dt . k2 [H2 ][1 2 ] /
(c) The backward reaction has lower activation energy
(c) [- ~ d[RI]] k1 [H2 1[12 ] - k2 [RIf (d) No activation energy is required at all since energy is
2 dt '
liberated in the process
(d) [- ~2 d[RI]]
dt
= kl k2 [H, 1fT,]
..
160. For the reaction, A + 3B ---7 2C + D
which one of the following is not correct?
153: The chemical reaction, 203 ,~ 302 proceeds as; !EA;\,ICET (;vIed.) 2010j
0 3 ~ 02 + ° (fast), 0+ 0 3 --72°2 (slow) (a) Rate of disappearance of A = Rate of formation of D
The rate law expression will be:
(a) rate:::: k[O][03] (b) rate = k[03f (b) Rate of formation of C :::: ~ x Rate of disappearance of B
. 3,
1
, (c) rate:::: k[03f[02r (d) can't be determined
(c) Rate offormation of D = ~ x Rate of disappearance of B
154. For the reaction NH~ + OCN ~ NH2CONH Z' the 3
probable mechanism is, (d) Rate of disappearance of A :::: 2 x Rate of formation of C
NH:
+ OCN ~ NH 4 0CN (fast) 161. 2A ~ B + C, would be a zero order reaction when:
and NH40CN ~ NH1CONH2 (slow) !ellSE (PMT) 20021
The rate law will be: (a) the rate of reaction is proportional to square of concentra-
(a) rate:::: k[NH 2CONH2 ] (b) rate:::: k[NH4t[OCNr tion of' A'
(c) rate k[NH4 OCN] (d) none of these , (b) the rate of reaction is same at any concentration of 'A'
(c) the rate remains unchanged at any concentration of 'B' and
. 155. For a 1st order' decomposition,
'c'
kn
An~A
~'l I (d) the rate of reaction doubles if concentration of 'B' is
increased to double
k2
A2 162. Units of rate constants for first and zero order reactions in
overall k will be given by: , terms of molarity M unit are respectively: (AIEEE Z002}
(a) k kl + k2 + ... + kn (b) k= (k1 + k2 + .... + kll)ln Ca) sec-I, M ~C-l (b) sec-I, M
(c) k kl X k2 x ... X kll (d) none of these (c) M sec-I ,sec-I (d) M,sec- I
156. For a reaction A + 3B --7 P, Rate:::: , the expression for 163. Following is the graph between log and log a (a = initial
tll2
dt
the'rate of reaction in terms of change in the concentration of concentration) for a given reaction at 27°e. Hence, order is:
B, -d[B] will be:
dt I
(a) k[A]2[B]
(c) 3k[Af[B]
. (b) k[A f[3B]
(d) (~)k[A]2[B]
i
log tl/2
45°
157. Which ofthe following statements is correct?
. 'T-
(a) Rate of reaction oc
Eo loga--
(b) At lower temp., increase in temp. causes more change in
the value of k (a) 0 (b) 1 (c) 2 Cd) 3
(c) Both (a) and (b) are correct 164. Following is the graph between (a - xrl and time t for
(d) None is correct second order reaction. 8 tan-I (1/ 2);OA l
2 L mor , hence
rate at the start of reaction will be:
CHEMICAL KINETICS 575

(a) I x 10-4 mol L-1s- 1 (b) 3 x 10-4 mol L-1s- 1


(c) 4 X 10-4 mol L-1s- 1 (d) 6 x 10-4 mol L-1s- 1
168. Temperature dependent equation can be written as:
[JEE (Orissa) 2005J
(a) In k = In A - eEalRT (b) In k = In A + eEalRT
(c) In k = In A - eRTIEa (d) all of these
(a) 1.25 Lmol- min- I I
(b) 0.5 mol L- 1 min- I 169. If the rate of reaction A -----t B doubles on increasing the

(d) 1.25 mol L- 1 min- I concentration of A by 4 times, the order of reaction is:
(c) 0.125 mol L- 1 min- I
fCET (J&K) 2005]
165. Graph between concentration of the product' x' and time' t' for
(a) 2 (b) 1 (c) 112 (d) 4
A -7 B is given ahead:
170. For the reaction: 2N 20 5 (g) -----t 4N0 2(g) + O 2(g) if the
concentration of N0 2 increases by 5.2 x 10-3 M in 100 sec,

i then the rate of reaction is:


(a) 1.3 x 10-5 M S-I
WET (Kcrala) 20051
(b) 5 x 10-4 M S-I
(c) 7.6 X 10-4 M S-I (d) 2 x 10-3 M S-I

t- (e) 2.5 x 10-5 M S-I

171. A first order reaction is 10% complete in 20 min. The time


The graph between - d[A] and time wil1 be of the type: taken for 19% completion is: fPET (Kerala) 2005]
dt (a) 30 min (b) 40'min (c) 50 min (d)38 min
(e) 45 min

(a)
d[A)
---at
(b)
d[A)
---at
/ 172. An endothermic reaction with high activation energy for the
forward reaction is given by the diagram: (AIIMS 20(5)

t- t-
t
Potential
energy ~p t
Potential
energy
1\p
(c) (d)
_d[A)~
dt
Reaction -
coordinate
(a)
Reaction - +
coordinate
(b)

P pP
I
t- t-

166. Consider the chemical reaction,


N 2 (g) + 3H 2(g) ----7 2NH3(g)
The rate of reaction can be expressed in terms of time
derivative of concentration of N2 (g ), H2 (g) or NH3 (g ) .
Identify the correct relationship amongst the rate expressions:
(I1T2002)
t
Potential
energy
1 Reaction . -
coordinate
t
Potential
energy

Reaction - +
coordinate
(c) (d)
(a) rate = _ d[N2] =_~ d[H2] = + ~ d[ NH3]
dt 3 dt
dt 2
(b) rate';'; - d[N2] = _ 3 d[H2] = + 2 d[ NH 3] 173. For reaction aA -----t xP, when [A] =; 2.2 m-M, the rate was
dt dt dt found to be 2.4 m M S-I. On reducing concentration of A to
(c) rate = d[N21 = ~ d[H2] = ~ d[ NH 3] half, the rate changes to 0.6 m M s -I. The order of reaction
dt 3 dt 2 dt with respect to A is: (AJIMS 20(5)
(d) rate = _ d[N2] =_ d[H2] = d[ NH 3] (a) 1.5 (b) 2.0 (c) 2.5 (d) 3.0
dt dt dt [Hint: Rate = k [A]a
167. For the reaction, 2.4 = k [2.2 t ... (i)
Nz(g) + 3H2(g)-----72NH 3(g), if 0.6 = k [l.it ... (ii)
.:~~ffi, 1= 2 x 10-4 mol L-1s -I, the value "f -d[H2] would be: Dividing eq. (i) by eq. (ii), a =2 :. order = 2]
~ ~ )
rCBSE (PMT) 2009J 174. According to the law of mass action, rate of a chemical
reaction is proportional to: (AFMC 2005)
576 r A.B. PHYSICAL CHEMISTRY FOR COMPETITIONS

(a) concentration of reactants 181. The rate constant of a first order reaction at 27°C is 10-3 min -I.
(b) molar concentration of reactants The temperature coefficient of this reaction is 2. What is the rate
(c) concentration of products constant (in min -I) at 17°C for this reaction?
(d) molar concentration of products ;~T 2006\
(a) 10-3 (b) 5 x 10-4 (c) 2 X 10- 3 (d) 10-2 '
175. Consider the endothermic reaction X ~ Y with the activation
energies Eb and for backward and forward reactions, [Hint:
respectively. In general: (AIEEE lOGS)
(a) < Ef
(Il )10110 2
(b)Eb > E f
kTl
(c)Eb::: Ef
10-3
(d) there is no definite relation between Er and Eb ky; = - 0.5 X 10-3
1 2
176. Which one of the following statements for order of reaction is = 5 X 10-4 min-I]
not correct? lIlT (S) 2{)05]
182. In a chemical reaction, two reactants take part. The rate of
(a) Order can be determined experimentally
reaction is directly proportional to the concentration of one of
(b) Order of reaction is equal to sum of powers of them and inversely proportional to the concentration of the
concentration terms in differeIl.tial rate law "- other. The order of the reaction is: . [PMT (Raj.) 2G06j
(c) It is not affected by the stoichiometric coefficient-or-the (a) zero (b) 1 (c) 2 ' (d) 4 '
reactants
183. According to Arrhenius equation, the rate constant (k ) is
(d) Order cannot be fractional
related to temperature (T ) as:
177. The rate constant of a reaction is found to be 3 x 10-3 mol L- 1
min -1. The order of the reaction is: leET P&K) 20061 (a) In
( --.£)
k\ = (IIJ
- -
(a) zero (b) I (c) 2 (d) 1.5
k[ R, 11 12,
[Hint: Unit of rate 'and rate constants are same for zero order (b) --.£ =- rllJ
lnl\ikJ --
reaction.] kJ R ,,11 T2
178. In the first order reaction, the concentration of the reactants is
reduced to 25% in one hour. The half life period ofthe reaction (c) In [ k2
kl
J= R '( 11I + T2I J
is: jUGET (Manipal) 20051

:~ J= ~ +~ J
(a) 2 hrs (b) 4 hrs (c) 112 hr (d) 114 hr
[Hint: k = 2.303 log
t a
(_a_Jx
(d) In ( R (

184. Consider a reaction aG + bH Products. When


k 2.303 10 (1001 2 x 0.693
1 g 25) concentration of both the reactants G and H are doubled, the
0.693 0.693 rate increases by eight times. However, when the
.!. hr] concentration of G is doubled keeping the concentration of H
k 2
2 x 0.693
fixed, the rate is doubled. The overall order of reaction is:
179. A substance reacts with initial concentration of a mol dm-] (HT 200'7)
according to zero order kinetics. The time it takes for the
(a) 0 (b) I (c) 2 (d) 3
completion of the reaction is: (k rate constant)
[Hint: Order with respect to 'G' will two and with respect to
CET (Karnataka) 20091
. , H' the order will be one.
(a) k (b) a (c) a (d) 2k Rate = k [G]2 [H] 1
a' 2k k a
When concentration of both G and H are doubled, the rate will
(e) ka
increase eight times.]
[Hint: x=k xt
185. The temperature dependence of rate constant (k) of a
x
chemical reaction is written in terms of Arrhenius equation
k
. Time for completion of reaction (when x a), i.e., t = *] k = Ae- Ea I RT. Activation energy (Ea ) of the reaction can be
calculated by plotting:
I
.\is.. :. n
I
180. Rate of a reaction can be expressed by Arrhenius equation as, (a) log k vs T(b) log k vs (c) k vs T (d) k vs--
'k =A e -EIRT . I n thoIS equatIOn,
. E represents: ",\1Ftl': 20(6)
' T log T

(a) the energy above which all the colliding molecules will 186. Consider the reaction, 2A + B Products. When
react concentration of B alone was doubled, the half life did not
(b) the energy below which colliding molecules will not react change. When the concentration of A alone was doubled, the
(c) the total energy of the reacting molecules at a temperature T rate increased by two times. The unit of rate constant for this
reaction is:
(d) the fraction of molecules with energy greater than the
activation energy of the reaction (a) S-I (b) L mol-1s- 1 (c) unit less (d) mol
CHEMICAL KINETICS 577
-I
[Hint: Concentration change in 'B' does not change half life, it sec
0.5 0.5 mol- 1 dm 3 ]
means the reaction is fIrst order with respect to B. When ko mol dm- 3 sec-I
concentration of only' A' is doubled, the rate of reaction becomes
193~ The decomposition of HI on the surface of gold is :
double, thus order with respect to A will also be one.
Overall order of reaction 2 [Corned (Ktaroataka) 2008]
unit of rate constant'" L mol-Is- I ] (a) Pseudofrrst order (b) zero order
(c) fIrst order (d) second order
187. In a fIrst order reaction A --,--------7 B, if k is rate constant and
194. Consider following two reactions '
initial concentration of the reactant A is 0.5 M then halflife is:
[CBSE (Med) 2007] A ----7 Product, = k) [ A ]0
2 log 2 dt .
(a) (b) - - (c) In 2 (d) 0.693
k k.JUs k O.5k B ----7 Product, - 4[B] = k [B]
dt 2. 0
188. For the fIrst order reaction, halflife is 14 sec, the time required kr and k2 are expressed in terms of molarity (mol L- 1) and
for the initial conceritration to reduce to 118 of its value is: time (S-I) as: (BHU (Mains) 20081
[CET (J&K) 20071 (a)s-I,Ms-1L- 1 (b)M ,Ms- 1 •
(a) (14)3 sec (b) 28 sec (c) 42 sec (d) (14lsec
'(c)s-I,M-Is- 1 (d)M,
189. 75% of a fIrst order reaction was completed in 32 min, when
195. If a plot of log 10 C versus t gives a straight line for a given
was 50% of the reaction completed? (BHU (Mains) 20071
(a) 24 min (b) 16 min (c) 8 min (d) 48 min reaction, then the re~ction is : (VITEEE 2008f
(a) zero order (b) fIrst order (c) second oder (d) third order
190. For a zero order reaction,
A P [Hint:
t,X is: (k is rate constant) [BHU (Mains) 2007J
(a) (b) In 2 1 d In 2
2k k (C)k[A]o () [Alok
191. The rate constants kJ and k2 for two different reactions are
1016 e-20001T
, an d I v
hiS - I O O O I T ·
e "
, respectIvely. The temperature
at which kl = k2 is : ICBSE (PMT) 2008J
(a) 2000 K

[Hint: 10 15
(b) 1000 K

e-lOOOIT
2.303
=1016e-2000IT
(c) 1000 K (d) 2000 K
2.303 -
In first order reaction, 10glOC when plotted against time 't' then
we get straight line, Slope of the line (2.~03) gives the value of
e-2000IT 10 15

, ,e- 1000lT 10 16
e- I000/T = 10- I rate constant.]
196. For a zero order reaction the plot of concentration of reactant
loge e- )OOO/T = loge 10- 1
versus time is: (intercept refers to concentration axis)
1000 . _I fPET (Kerala) 20081
T = 2.303 10glO 10
(a) linear with +ve slope ,and zero intercept
2.303 (b) linear with -ve slope and zero intercept
T = 1000 K] (c) linear with -ve slope and non zero intercept
2.303 (d) linear with +ve slope and non zero intercept
192. Under the same reaction conditions, initial' concentration of (e) a curve asymptotic to concentration axis
1.386 mol dm - 3 of a substance becomes half in 40 seconds [Hint: '
and 20 seconds through fIrst order and zero order kinetics

respectiVely. Ratio ( :~) of the rate constant for fIrs.t o~der (kl ) . c:
o
i
and zero order (ko) of the reactiOn is :
(a) 0.5 mol-I dm3
(c) 1.5 mol dm-
'
[H mt : tl/2 =
.
3

0.693
kl
(b) I mol dm-3
(d) 2 mol-I dm 3

tl12 =
' 2ko
(liT 2008)

I L_'_,_ _ _ _ _ _.:....._

Tlme_
Linear with negative slope and non zero intercept.]
40 = 0.693 20 = 1.386 = 0.693
197. Tso of flist order reaction is 10 min. Starting with 10 mol
kl ' 2ko ko
rate· after 20 IIiin is: (AIIMS 2008)
20 0.693/ ko = ~ (a) 0.0693 mol L- I min-!
40 0.693/ kl ko (b) 0.0693 x 2.5 mol L- I min-I
578 G.R.B. PHYSICAL CHEMISTRY FOR COMPETITIONS •
(c) 0.0693 x 5 mol L- 1 min-I The pre exponential factor A and the activation energy Ea
(d) 0.0693 x 10moi L- 1 min-I respectively, are : (lIT 2009)
6
(a) Ix 10 s-I and 9.2 kJ mol- 1
[Hint: Remaining concentration of reactant after 20 min
= .!. x 10 = 2.5 mol L- I (b) 6 S-1 and 16.6 kJ mol- 1
4 (c) 1x 106 S-l and 16.6 kJ morl
Rate = k x [Reactant]
0.693 (d) IxIQ6 s-I and 38.3 kJ morl
= -- x [Reactant]
t1l2 [Hint: logk == 10glOA -~ xl
'2.303 T
== 0.693 x 2.5 = 0.0693 x 2.5 mol L-1 min-I] Comparing this equation with the given equation we get,
lO
198. For the decomposition of AB at 600 K, the following data A == l(f S-I, Ea == 38.3 kJ mor l ]
were obtained 201. The time for half-life period of a certain reaction,
Rate of deComposition of AB in A -} Product is I hour. When the initial concentration of the
mol dm-35-1 reactant' A' is 2 mol L- 1• How much time does it take for its
concentration to come from 0.50 to 0.25'moIL- 1 ifit is a zero
0.20 2.75 x 10-8 order reaction? (AIEEE 2010)
0040 llx 10-8 (a) 0.25 h (b) 1 h (c) 4 h (d) 0.5 h
0.60 24.75 x 10-8
The order of the decomposition of AB is :
[Hint: !l = [~
t2
fl
all
[eET (Karnataka) 2009)
(a) 0 (b) I (c) 2 (d) 1.5
..!. = (0.5Y-1
t2 \. 2 /
199. For a reaction between A and B, the initial rate of reaction is
measured for various initial concentrations of A and B. The, t2 =0.25 h]
data provided are: (DeE 2009) 202. Consider the reaction:
"I ~ ." IaitiaI ractioa CI 2 {aq)+ H 2 S(aq) - - } S(s)+ 2W (aq)+ 2cr (aq)
EIp.No. (A) (B)
rate (mol L-ls-I) The rate equation for this reaction is :
Rate = k [CI 2 ] [H2 S]
1. 0.2 M O.3M· 5 x 10-5
Which of these mechanisms is/are consistent with this rate
2. 0.2M 0.1 M 5 x 10-5 equation?
3. OAM 0.05M 7.5 x 10-5 A. Cl 2 + H2S ~ W + cr + CI+ + HS- (slow)
The overall order ofthe reaction is: (DeE 2009) CI+ + HS- ~ W + CI- + S (fast)
(a) one (1) (b) two (2) B. H2S~ U" +HS- (fast equilibrium)
(c) two and a half (2.5) (d) between 1 and 2 Cl z +HS- ~ 2CI- +W + S, (slow)
200. For a first order reaction A ----7 P, the temperature (T)
(AIEEE 2010)
dependent rate con",ant (k )was found to follow the equation: (a) neither A nor B . (b) Aonly
log k = -(2000)-1 + 6.0 (c) B only (d) both A and B
T
CHEMICAL KINETICS 579

Assertion-Reason TYPE QUESTIONS


rhe questions given below consist" of two statements each printed as
'Assertion' (A) and 'Reason' (R). While answering these questions 7. (A) k IRT, the Arrhenius equation represents the de-
'ou are required to choose anyone .of the following four options: pendance of rate constant with temperature. .
(a) If both (A) and (R) are correct and (R) is the correct (R) Plot of log k against 11 T is linear. and the activation en-
explanation for (A). ergy can be calculated with this plot.
(b) Ifboth (A) and (R) are correct but (R) is not the correct 8. (A) If the activation energy of a reaction is zero, temperature
explanation for (A). will have no effect on the rate constant.
(c) If (A) is correct but (R) is incorrect. (R) Lower the activation energy, faster is the reaction.
(d) If (A) is incorrect but (R) is correct. 9. (A) Order with respect to any reactant or product can be zero,
positive, negative and fractional.
1. (A) The rate of reaction sometimes does not depend on con- (R) Rate cannot decrease with increase in concentration of a
centration. reactant or product.
(R) The order of reaction can be negative. 10. (A) Formation of HI is a bimolecular reaction.
2. (A) The rate of reaction increases generally by 2 to 3 times for (R) Two molecules of reactants are involved in this reaction.
every 10°C rise in temperature.
11. (A) The order ofthe reaction,
(R) Increase in temperature increases the collision frequency.
CH 3COOC 2Hs + H20 ~ CH 3COOH + C2HsOH
a
3. (A) Hydrolysis of ethyl acetate in presence of acid is reaction
is 2.
of first order whereas in presence of alkali, it is a reaction
(R) The molecularity of this reaction is 2.
of second order.
12. (A) For: aA + bB ~ Product. The order of the reaction is
(R) Acid acts as catalyst only whereas alkali acts as one of the
equal to (a + b).
reactant.
4. (A) The molecuhirity of the reaction, (R) Rate of reaction = k [At[Bt
Hz + Br2 ~ 2HBr is 2. 13. (A) The hydrolysis of methyl acetate by dil HCI is a pseudo
(R) The order of this reaction is 3/2. first order reaction.
5. (A) Positive catalysts lower the activation energy of the reac- (R) HCI acts as a catalyst for the hydrolysis. (AIIMS 2007)
tion whereas heat of reaction remains same. 14. (A) The order of a reaction can have fractional value.
(R) Heat of reaction is equal to the difference between activa- (R) The order of a reaction cannot be written from balanced
tion energies for forward and the backward reactions. equation of a reaction. (AIIMS 2008)
6. (A) Positive catalysts increase the rate of reaction.
(R) Catalysts decrease the value of 6.Go.
I •
'It"

580 I G.R.B. PHYSICAL CHEMISTRY FOR COMPETITIONS

[rl.'~ .. QUEStiONS
: OBJEOTIVE ' ~ " ,. _. ~., 0. ";",.. ' : ::,.: ";

1~ (e) 2. (d) 3. (b) 4. (b) 5. (a) 6. (a) 7. (e) 8. (d)


. 9. (b) 10. (e) 11. (a) 12. (d) 13. (d) 14. (d) 15. (b) 16. (a)
17. (e). 18. (e) 19. (b) 20. (b) 21. (e) 22. (a) 23. (a) 24. (d)
25~ (e) 26. (d) 27. (a) 28. (b) 29. (d) 30. (d) 31. (d) 32. (b)
33. (a) 34. (;1) 35. (b) 36. (e) 37. (d) 38. (d) 39. (a) 40. (e)
4L (b) 42. (b) 43. (d) 44. (e) 45. (a) 46. (b) 47. (e) 48. (a)
49. (d) 50. (b) 51. (e) 52. (d) 53. (a) 54. (b) 55. (d) 56. (e)
57. (b) 58. (a) 59. (e) 60. (d) 61. (b) 62. (e) 63. (d) 64. (e)
65. (a) 66. (d) 67. (e) 68. (b) 69. (e) 70. (e) 71. (a) 72. (d)
73. (a) 74; (b) 75. (d) 76. (e) 77. (b) 78. (b) 79. (d) 80. (a)
81. (d) 82. (c) 83. (b) 84. (b) 85. (e) 86. (a) 87. (a) 88. (b)
89. (e) 90. (e) ?L (b) 92. (d) 93. (a) 94.' (d) 95. (d) 96. (b)
97. (b) 98. (a) . 99. (a) 100. (a) 101. (d) 102. (d) 103. (e) 104. (b)
105. (b) 106. (e) 107. (a) 108. (d) 109. (b) 110. (b) 111. (d) 112. (a)
113. (c) - 114. (a) 115; (a) 116. (e) 117. (a) 118. (a, e) 119. (b) 120. (b)
. 121. (b) 122. (a) 123. (e) 124. (a) 125. (a) 126. (a) 127. (e). 128. (b)
129. (a) 130., (a) 131. (a) 132. (a) 133. (e) 134. (b) 135. (b) 136. (b)
137. (b) 138. (e) 139. (a) 140. (e) 141. (e) '142. (d) 143. (b)' 144. (b)
145. (e) 146. (e) 147. (e) 148. (e) ,149. (b) 150. (c) 151. (b) 152. (e),
153. (e) 154. (b) 155. (a) 156. (e) 157. (e) 158. (e) 159. (a) 160. (d)
161. (b) '162. (a) 163. (a) 164. (e) 165. (e) 166. (a) 167. (b) 168. (a)
169. (e) 170., (a) . '171. (b) 172. (e) 173. (b) 174. (b) 175. (a) 176. (d)
177. (a) 178. (e) 179. (e) 180. (b) 18t. (b) ]82. (a) 183. (a) 184. (d)
185. (b) 186. (b) . 187. (e) 188. (e) 189. (b) . 190. (a) 191. (b) 192. (a)
193. (b) 194. (d) 195. (b) 196. (c) 197. (b) 198. (e) 199. (d) 200. (d)
201. (a) 202. (b)

[~
.~ : 'ASSi:RTION-REASON TyPE QUESTIONS
.'

; ,
I
1. (by 2. (b) 3. (a) 4~ (b) 5. (b) 6. (c) 7. (a) 8. (b)
9. (e) 10. (a) 11. (d) ]2. (b) 13. (b) 14. (b)
I
CHEMICAL KINETICS 581

,BRAiN STORMING PROBLEMS


lOBlECTIVEQUESTIONS l/or ( IIT'~S9IRANrS'
1. \\rylich of the following is correct? [Hint: For zero order reaction,
(a) Molecularity of areaction can be fractional x =kt
(b) Zero order reaction never stops
(c) A first order reaction must be homogeneous
.. '2= k xtIl2,1.e.,tI12=2k
a . a
... (iJ

(d) The frequency factor 'A" in Arrhenius equation For first order reaction,
(k Ae - Ea I RT ) increases with increase in temperature 2
,.. (ii)
2. The rate constant of the reaction, k
2H20 2 (aq.)-..? 2H 2°(l) + 0l(g), From eqs. (i) and (ii), a
is 3 x 10-3 min-I, ' 2k
a log" 4 M]
.At what concentration ofH20 2, the rate ofthe reaction will be
6. Which of the following is pseudo-unimolecular reaction?
2 x 10-4 M S-I?
(a) 2HzOl 2HzO + 0z
(a) 6.67 x 10-3 M (b) 2M '(b) C6HSN2Cl + HOH ~ C6 HsOH + N2 + HCI '
(c)4M (d) 0.08 M (c) CH3 C00CzHs + NaOH ~ CH 3COONa + CzHsOH,
[Hint: Rate k [H20 2 ]l (d) 203 ~ 302
-4 3 X10-3 7. The order and molecularity of the chain reaction,
2 x 10 = 60 X [HP2]
2HCI (g),
[H 20 l ]= 4 M] are:
3. The mechanism of the reaction,
2NO(g)+ 2H2(g)~ Nz(g)+ 2HzO(g),
(a)2,0 (b) 0, 2 (c)l,l
8. A reactant (A ) forms two products:
(d) 3, °
is:
kI
A ~ B Activation energy E a 1
fust k2
Step 2:, H 2 0 2 + Hz 2H zO
A ~ C Activation energy Ea2
(a) rate = k [N01l[Hlf
(b) rate = k [H 2 0 Z ][H l 1' If Eaz 2Eal , then kl and k z will be related as:
(c) on doubling the concentration of Hl , keeping the (a)kl kleEajlRT (b)kz=kleEazIRT
concentration of NO constant, the rate will become double
'(c)kl = Ak2eEaliRT (d)k =2kleEazlRT
(d) if the initial concentration of Hz and NO is Co and after 1

, time' t' the concentration of N z is x, then [Hint: k2 = Ae-Eazl RT


Rate k(C o - 2xy - A -EflJIRT
k 1- e
[Hint: Slowest step is rate determining
,,; e(-Eaz +EflJl/RT
:. Rate k[NO]2[H 2 ]
kl
:. On doubling the concentration of H2 without changing the
concentration of NO, the rate will become double.] kl
4. ' The rate law for a reaction between A and B is given by:
k 2 -- k IeEflJIRT]
Rate = k [AnBt
9. Collision theory is satisfactory for:
On doubling the, concentration' of A and halving the
, (a) first order reactions (b) second order reactions
concent:fa,tio~~of D, the ratio of ,new, rate to the earlier rate of
(c) bimolecular reactions (d) zeroth order reactions
the reactiop will be a s : ' "
(a)n m (b) 21l.-m , (t}i /(n4:lir) (d)2m-n 1(;;' Initial concentration of reactant for nth order reaction is 'a'.
. Which' of the following r~lations is correct abaH': tl/2 of the
k ,
5. The reaction A ~ Product, is zero order while the reaction reaction?
k
(a) In tl/2 = In (constant) - (n - I) loge a
B ~ Product, is first order reaction, For what initial (b) In tl/Z = In n + In (constant)-:-In a
concentration of A, are the half lives of the two reactions (c) t1/2 In n = In (constant) + In ao
equal? (d) In tl/2 n.ln ao '
(a)(loge4)M (b)2M (c)2Iog2M (d}In2M
582 G.R.B. PHYSICAL CHEMISTRY FOR COMPETITIONS

[Hint:
1 2.4 X 10-5 3 X 10-5 [NP5]
t 112 0<: d'-l
[NzOs]=0.8M]
1 15. Consider the following statements:
t 112 = k --;;-::t
a 1. The rate of reaction is always proportional to the
In t1l2 = In k - (n - 1) loge a] concentrations of reactants.
11. Half life of a reaction becomes half when initial concentrations 2. The order of an elementary chemical reaction step can be
of reactants are made double. The order of the reaction will be: determined by examining its stoichiometry.
(a) I (b) 2 (c) 0 (d) 3 3. The first order reactions follow an exponential time
1 , course.
[Hint: t1l2 O<:--;;-::t
a Of these statements:
1 (a) 1,2 and 3 are correct (b) land 2 are correct
1112 =
,a (c) 2 and 3 are correct (d) 1 and 3 are correct
where, ~ = order of reaction for second order reaction.] [Hint: Statement (1) cannotbe correct because the rate of zero
12. Arrhenius equation is: order reactions does not depend on concentration of reactant.]
k = Ae-E1RT 16. Two different first order reactions have rate constants kj and k2
Which of the following graphs represents the variation of rate at 11 (kt > kz ). If temperature is increased from 11 to Tz, then
constant k against temperature T ? (lIT 2010) new constants become k3 and k4 respectively. Which among
the fQllowing relations is correct?

!I~
(a) kt > k2 = k3 k4 (b) kl < k3 and k2 < k4
(c) kl k3 k4 ' (d) kl > k, > k, '> k4
i
k
17. In the reaction, A + B ~ C + 1), the rate l~) when
. dt
.... ~ J ,

plotted against time 't' gives a straight line parallel to time


T- T- axis. The order and rate of reaction will be:
(a) (b) (a) I, k + 1 (b) 0, k (c) 1, k + I (d) k, k + 1
18. In the Haber's process of ammonia manufacture,
N 2 (g) + 3Hz(g) ~ 2NH3(g)

i
k
t
k
the rate of appearance of NH3 is:
d[
NH
3] 2 x 10-4 mol L- 1 sec- 1
dt
The rates of the reaction expressed in termS ofN2 and H2 will
T- T ,,--. be:
(c) (d) Rates ill terms of HI , ~intennsofNI
13. If a homogeneous catalytic reaction follows three alternative
(mol L -I see-I) (mel L -1 see~I)
paths A, B and C, then which of the following indicates the 4
(a) 3 x 10- 2 X 10-4
relative ease with which the reaction moves?
(b) 3 x 10-4 I X 10-4

A
(c) 1 X 10-4 3 X 10-4
(d) 2 x 10-4 2 X 10-4

= -d[N21 = _.!. d[N 2 ]


t
Potential
[Hint: Rate
dt ~t.. . ....' ,)
"""'; .. .,dt, .
energy
dt dt
3
R x 2 X 10-4 = 3 X 10-4 mol L- 1 sec- 1
2
p 1 1 -4
=-x2xlO
Reaction coordinate - dt 2 dt 2
I x 10-4 mol L-1 sec-I]
(a) A > B > C (b) C > B > A (c) A > C > B (d) A B C
14. The rate constant for the reaction, 19., A reaction A B, involves following mechanism:
kl
2N2 0 5 ~ 4N02 + O 2
Step 1: A B (fast)
is 3 X l()-s sec-I. If the rate is 2.4 X 10-5 mol litre -I sec -1,
k2
then the concentration ofN2 0 s in mol litre- 1 will be: Step 2: B C (slow)
(a) 1.4 (b) 2.1 (c) 1.27 (d) 0.8 k3
~int: Rate = k [N20S] Step 3: C D (fast)
CHEMICAL KINETICS. 583·
The rate law of the reaction may be given as: From eq. (i), Rate = kK[NO z ][03][02r 1
(a) rate k1[A] (b) rate = k2[B] :. Both mechanisms show that reaction is first order with respect
(c) rate = k3[C] (d) rate klk2k3[B][C] to N0 2 and 03']
20.
24. Use the experimental data in the table to detennine the rate law
Concentration in mol L- 1 for this reaction:
Half life in sec 30 A+B~AB
The order of reaction i·s: These data were obtained when the reaction was studied:
(a) second (b) first (c) zero (d) fractional

21. The halflife of second order reaction is:


[A) [B] I 8(ABlmol L -I sec.,.l
M
(a) inversely proportional to the square of the initial
0.1 M 0.1 M 2 X 10-4
concentration of the reactants
(b) inversely proportional to the initial concentration of 0.2M O.IM .2 X 10-4
reactants O.3M O.3M 1.8 X 10-3
(c) proportional to the initial concentration of reactants
(d) independent ofthe initial concentration ofreactants What is the rate equation for the reaction?
I wIlere, a = IDltia
... \ cone., n = order (a) Rate = k [A][B]· (b) Rate =k [Af
[II int: t1l2 "" -;t'T;
(c) Rate = k [B] (d)R~te= k [B]2
1 .
tl12 (second order) 0<. ] 25. In which of the following reactions, the increase in the rate of
a reaction will be maximum?
22. What names apply to chemical species corresponding .to Ea Temperature rise
locations 1 and 2 on this reaction coordinate diagram?
(a) 40 kJ/mol 200- 210 K
(b) 90 kl/mol 300-320 K
(c) 80 kl/mol 300-310 K
(d) All will have same rate
26: Which of the reactions represented in these diagrams will
show the greatest increase in rate for a given increase in
. temperature?

Location 1
(a) Activated complex
Reaction progress -

Location 2
Activated complex
t
>-
0.
(ij
..c:.
'E
w
Ap
t
>-
0.
(ij
..c:.
'E
w
f\-p
Reaction (I) - Reaction (11)-
(b) Reaction intennediate Activated complex
(c) Activated complex Intermediate (a) Reaction I forward . (b) Reaction I reversed
(c) Reaction II forward (d) Reaction II reversed
(d) Reaction intennediate Intennediate
23. Consider this reaction: 27. Which function of [X ], plotted against time, will give a
2N0 2(g)+ 03(g)~ NzOs(g)+ 02(g) straight line for a second order reaction?
The reaction of nitrogen dioxide and ozone represented is first X ~Product
order in N0 2 (g ) and in 0 3(g ). Which of these possible (a) [X] (b)[xf (c) In [X] (d)_l_
reaction mechanisms is consistent with the rate law? [X]
. Mecl1anism I: N0 2 (g ) + 0 3 (g ) ~ N03(g) + O 2(g) (slow) 28. Decomposition ofH 2 0 2 is a first ordeueaction. ~ 16 volume
N03(g)+NOz(g)~N20SCg) (fast) solution ofH2 0 2 ofhalflife 30 min is present at start. When
Mechanism II: 03(g) 02(g)+[0] (fast) will the solution become one volume?
NOz(g) + [0] ~ N03 (slow) (a) After 120 min (b) After 90 min
N03 (g) + N02 (g ) ~ NzOs (fast) (c) After 60 min (d) After 150 min
(a) I only (b) II only 29. What is the activation energy for the reverse of this reaction?
(c) Both land II (d) Neither I nor II N 20 4 (g) ~ 2N0 2 (g)
°
fHint: Mechanism I, Rate = k[NO z ][ 2 ], Slow step is rate Data for the given reaction is: t:.H + 54 kJ and
kJ:
= + 57.2
determining
(a) -54 kl (b) +3.2 kJ (c) +60.2 kJ (d) + 111.2 kJ
Mechanism II, Rate = k [NOz][O] ... (i)
30. The reaction between chlorofonn, CHCl 3 (g) and chlorine
K [0 2 ][0], [0] = K [0 3 ]
[0 3 ] [0 2 ] Cl 2 (g ) to fann CCl 4 (g) and HCl(g) is believed to occur by
this series of steps:
584 G.R.B. PHYSICAL CHEMISTRY FOR COMPETITIONS

Step 1: Cl 2 (g ) ~CI(g ) + Cl(g) o 0.


I H+ II
Step 2: CHCl 3 (g) + Cl(g) ~ Cel 3 (g) + HCl(g) CH3 - c- CH3 + 12'~ CH3 - c- CH21 + HI
Step 3: CCl 3 (g ) + Cl(g ) ~ CCl 4 (g) These data were obtained~en the reaction was studied:
If this reaction is first order in CHCl 3 and half order in e1 2 , :;'.P' ,
which ,statement about the relative rates of steps 1,2 and 3 is 0 ' Relative
correct? II
[CD) -C~CH31,M
[I:zl,~ [Jr I,M, rate
. ,.(JJ. ,
(a) Step 1 is the slowest .. ,
(b) Steps 1 and 2 must both be slow 0.010 0.010 .J).OlO 1
-"
(c) Step 2 must be slower than step 1 0.02Q 0.010 )0.010 2
(d) st~p 3 must be the slowest "
0,020. ' 0.020 0.010 2
31. In the reaction, ·L
3BrO- ~ BrOi + 2Br- (aqueous alkaline medium at 80"C) ,Q\020 0.010 0.020 4
the value of the rate constant in the rate law in terms of Wb,at is the rate equation for the reaction?

-! [BrO~] is 0.056 L mol- 1 • What will be the rate


' 0

(a) Rate = k[CH3


II
c-.cH3 ] [12 ]
constant when the r~te law is stated in terms of:!..- [BrOi] ? o
, dt , ~
(a)18.7 x 10-3 L mol-I S-I (b) 37.4 x 10-3 L mol-I S-I (b) Rate =k[CH3 - c-cH3 f
1
(c) 0.0187 L mol- S-I (d) 18.7 x 10- 2
L mor l
S-l '" .'
o·11
. ,
[Hint .!. d[BrO-] = d[BrO; ] + 1 d[Br-] (e) Rate = k [CH:~ --,:c- CH3 ][Iz][H"]
3 dt dt 2dt
" " b
, , : II ,
Required rate constant = .!. x 0.056 L mol- 1 S-1 (d) Rate = k [CH3 ~ c- CH3 ](W]
3
= 0.0187 L morl 5- 1 ] . 36. Arrhenius equation k Ae-Eal RT
32. The dependence of the rate constant for a reaction on If the activation energy of the reaction is found to be equal to
temperature is given by the equation k Ae -En I RT • RT,then:
(a) the rate of reaction does not depend upon initial
Under what conditions is the rate constant k the smallest?
concentration
(a) High T and large Ea (b) High T and small Ea
,(b) the rate constant becomes. about 37% of the Arrhenius
(c) Low T and large Ea (d) Low T and smailEp constant A
33. The activation energy of a certain reaction is 87 kJ :;nol-l. (c) the rate constant becomes equaJ'to 73% of the Arrhenius
What is the ratio of the rate constants for this reaction when constant A
the temperature is decreased from 37°C to 15°C? (d) the rate of the reaction becomes infj.nite or zero
(a) 511 (b) 8.311 (c) 13/1 (d) 24/1
37. At 25°C, the values of rate constant, activation- energy and
34. Consider this reaction, Arrhenius constant ofareaction are 3 x 10-4 sec-I, 129 kJ/mol
2H2 (g) + 2NO(g) ~ N2 (g) + 2H 20(g) and 2 x 1015 sec -I respectively.
The rate law for this reaction is: The value ohate constant as T ~ 00 is:
Rate = k [H 2 ][NO]2 (a) zero (b)2xlOI5 (c)3xlO-4 (d)6xlOll
Under what conditions could these steps represent' the 38. The observed rate of a chemical reaction is substantially lower
mechanism? than the collision frequency. One or more of the following
Step 1: 2NO ~ N 20 2 statements islare true to account for this fact.
A. the reactants do not have the required energy.
Step 2: N 20 1 + H2 ~ N 20 + H 20 ,B. ,the partuers do not collide in the proper orientation
Step 3:N20 + H2 ~ N2 + H20 C. collision complex exists for a very short time.
(a) These steps cannot be, the mechanism under any D. collision frequency over estimates the number of effective
circumstances ,,' collisions' (ISAT 2010)
'(b) These steps could be the mechanism if step 1 is the slow (a) A, B andC (b) A, B andD
step (c)B,CandD (d) A,C andD
(c) These steps could be the mechanism if step 2 is the slow 39. the reaction between NO and Cl 2 takes place in the following
step 'two steps:
(d) These steps could be the mechanism if step 3 is the slow
step
35. Propanone reacts with iodine in acid medii.Jm according to the
following equati(jn:
CHEMICAL KINETICS 585

respectively40, 50 and 60 kJ mol-] ,then the overall energyof


II. NOCl 2 + NO ·2NOCI
slow . activation in kJ mol~ 1 is: [PMT(Kerala) 2008)
. The rate la~ of overall reaction, (a) 30 (b) 40 . (c) 60 (d) 50
2NO + Cl z ---7 2NOCI (e) 150
can be given by:
[Hint:
Ca) rate k [NO]2[CI 2] (b) rate k [NO][Cl z]
(c) rate = k [NOCl][NO] (d) rate = k [NO] [Cl zf . Ae-E[IRTxA -E2 IRT
Ae- EIRT = e
40. Which of the following reactions will have fractional order for
A2 or B z?
(a) A2 A + A (fast) (b) ~ C (slow)
E=E 1 + E2 E3
A + B2 ~ AB + B (slow) C + B2 D (fast)
40 +50 60=30kJmo[-I]
A + B~· AB (fast) D + A2 ~ Products
(c) Bz ~ B + B (fast) (d) All have fractional order 45. For an exothermic reaction, following two steps are involved_
Az + B AB + A (slow) Step 1. A + B ---4 I (slow)
AB ---4 Products (fast) Step 2. 1---4 AB (fast)
41. In which of the following, Eo for backward reaction is greater Which of the . following graphs correctly represent this.
than E a for forward reaction? reaction?
Eo = 50 kcal

i~··
(a) A ) B; Ml = -lOkcal
Ea 50 kcal
(b) A ) B-, Ml= + tOkcal
~>. I
Ea 60 kcal
(c) A ) B-, Ml +20kcal ~~
!l..cn (. +B . . AB
(d) All of the above
[Hint: . Ml == (Ea)J (Ea}b Reaction _
Reactio.n _
(Ea)b (Ea}f - Ml = 50 + 10 == 60 kcal] coordinate coordinate
(a) (b)
42. For n th order reaction ~ dependson(n =/:. 1):
t3/4
( a) initial concentration only
(b) 'n' only AS
(c) initial concentration and 'n' both
(d) sometimes 'n' and sometimes initial concentration
[Hint: Time for fractional change oc }_ I ] Reaction _ . Reaction_
a coordinate coordinate
43. For a second order reaction, 2A Products, a plot of log (c) (d)

tl12 vs log a (where, a is initial concentration) will give an 46. A reaction takes place in three .steps with indIvidual rate
intercept equal to which one of the following? (SeRA 2007) constant and activation energy,

. (a) i (b) log C~) Step 1


Rate constant
. kl
Activation energy
E01 = 180kJ/moi
t
. (c)! log G) (d) log k log t1/2
Step 2
Step 3
Ea2
Eill =
80kJ/moi
50 kJ/mol
[Hint: Kinetic equation for
second order reaction is: Iog a - overaII rate constant, k kklk32 )2/3
(

k l[ 1 1]
t (a x) a
···~~~~""""""~·\)verall activation energy of the reaction wHrbe :
(a) 140 kJ/mol (b) 150 kJ/mol
a (c) BOkJ/mol (d) 120kJ/moi
when x =2,t= tll2
1 1
- .
A. - EUj IRT A - E021 RT
[Hint: Ae-EaIRT = .. e .. e
1
213

[ - E031RT .
tll2 = k x~ . Ae .
== [Ae(- EUj- Ea} + Eo)}1 RT l213 .
log t1/2 = -log k log a]
44. ·For a reaction taking place in three steps, the rate constants are 2
Ea "3 [Eal + Ea2 - E. 3 ]
k], k2 and k3- The overall rate constant k = If the energy
k3 ="32 [180 + 80 50] '" 140 kJ/inol]
of activation values for the first, second and third stages are
586 G.R.B. PHYSICAL CHEMISTRY FOR COMPETITIONS

[;4~' I
1. (d) 2. (c) 3. (c) 4.. (b) 5. (a) 6. (b) 7. (b) 8. (a)
9. (c) 10. (a) 11. (b) 12. (e) 13. (b) 14. (d) 15. (c) 16. (b)
. 17. (b) 18. (b) 19. (b) 20. (b) 21. (b) 22. (b) 23. (e) 24. (d)
25. (b) 26. (b) 27. (d) 28. (a) 29. (b) 30. (e) 31. (e) 32. (c)
33. (c) 34. (c) 35. (d) 36. (b) .37. (b) 38. (a) 39. (a) 40. (a, e)
41. (a) 42. (c) 43. (d) 44. (a) 45. (a) 46. (a)

Integer Answer TYPE QUESTIONS


5. Ozone depletion takes place as :
203(g)~ 302(g)'
This section contains 10 questions. The answer to k
Step 1 : 03(g)~ 02(g)+O(g) (fast)
. .each of the questions is a single digit integer, ranging
from 0 to 9. If the correct answers to question numbers Step2: 03(g)+[O]~2(j2(g) (slow)
. X, Y, Z and W (say) are 6, 0, 9 and 2 respectively, then order ofthe reaction will be :
the correct darkening of bubbles will look like the 6. If the t1l2 for a first order reaction is 0.4 min, the time of or
given figure : 99.9% completion ofthe reaction is ............... min.
7. Consider following parallel first order reactions

1. In the reaction, A ~ B when the initial concentration of


reactant is halved, the half-life increases by a factor of eight,
what will be the order of the reaction?
2. Rate of a chemical reaction increases by 1024 times by 100°C
rise in temperature; the temperature coefficient of the reaction
A
< B (t1/2 = 4 brs) .

C (t 1/2
.
= 12 brs)
The half-life for the decay of A is .................hrs.
8. The half-life period of a first order reaction is I hr. What is the
time in hour taken for 87.5% completion of the reaction?
will be:·'
9. The half-life of a reaction is doubled' when the initial
3.. The rate of a reaction at 10 sec intervals are as follows: .
concentration is doubled. The order of reaction is :
Rate . 10. For the reaction A2 + 2B ~ 2AB, the following data were
Time(~) . (mol L -1 sec-I) observed: .
o 4.8x 10-2.
Exp.No. IAll- (BI
Rate
(mol C-1 S"':I)
10. 4.79 x 10-2
l. 0.1 . 0.01 1.5 x 10-3
20 . 4.78 x 10-2
30 4.81 x 10-2
2. . 0.1 0.04 6.0 x 10-3
3. 0.2 0.01 3.0 x 10-3
What will be the order of the reaction?
4. How many times of the half-life will require to complete 75% The overall order of the reaction will be :
of a reaction of first order?

[·J14~.--------------1f.
1. (4) 2. (2) 3. (0) 4. (2) 5. (1)' 6. (4) 7. (3) . 8. (3)
9. (0) 10•. (2)
CHEMICAL KINETICS .

=re·. LINKED COMPREHENSION r,vPE QUESTIONS • ~.


5. The rate constant of a certain reaction is given by
• Passage 1 k::: Ae-EaIRT (where A = Arrhenius constant). Which factor
A collision between reactant molecules must occur with a certain should be lowered so that the rate ofre~tion may increase?
minimum energy b40re it.is effective in yielding product molecules. (a)T (b)Z (c) A . (d)Ea
This minimum energy is called activation energy Ea' Larger is the
6. The activation energies for forward and backward reactions in
value of activation energy, smaller is the value of rate constant.
a chemiCal reaction are 30.5 and 45.4 kJ mol-I respectively.
Larger is the value of activation energy, greater is the effect of
The reaction is:
temperature rise on rate constant k. .
(a) exothermic
(b) endothermic
(c) neither exothermic nor endothermic
(d) independent of temperature
t
• Passage 2
the energy profile diagram for the reaction:
CO(g) +N02 (g) ~ CO2 (g) + NO(g)
is given below:
p

Collision number ..........

E / = Activation energy offorward reaction


Eb = Activation enerw of backward reaction r
Energy
6.H=E/ Eb
= Threshold energy
Answer the following questions: p
l. If a reaction, A + B ---* C, is exothermic to the extent of 30
kJ/mol and the forward reaction has an activation energy of ~eaction path - -
249 kJ/mol, the activation energy for reverse reaction in
kJ/mol is: Answer the following questions:
(a) 324 (b) 279 (c) 40 (d) 100 1. The activation energy of the forward reaction is:
2. For the following reaction at a particular temperature, (a) x (b) y (c) x + y (d) x - y
according to the equations, 2. The activation energy of the backward reaction is:
2NzOs ---* 4N02 + °z
(a) x (b)y (c)x+y . (d)x-':'y
) 3. The heat of the reaction is:
2NO z + "2 02 ~ N 2 0 S
(a)x' (b)y . (c)x+y (d)x-y
the activation energies are E) and E2 respectively; Then: . 4. The threshold energy of the reaction is:
(a) E) > E2 (b) EI < E2 (c) E) = 2£2 (d) ~EJEi = 1 (a)x+y-z (b),x-y+z (c)x+y+z (d)x-y-z
3. In a hypothetical reaction, A -> Y, the activation energies for
• Passage 3
the. forward and backward reactions~are 15 and 9 kJ morl
respectively. The potential energy of A is 10 kJ morl. Which Population growth of humans and bacteria follows first order
of the following is wrong? growth kinetics. Suppose 50 bacteria are placed in a flask containing
(a) Threshold energy of the reaction is 25 kJ nutrients for the bacteria so that they can multiply. A study at 35"C
gave the following results :
(b) The potential energy of B is 16 kJ
Time (minutes)' 0 15 30 45 60
(c) Heat of reaction is 6 kJ
Number ofbacterilf 100 200 400 800 1600
(d) The reaction is exothermic
4. For two reactions, activation energies are E a 1 and E 02'. rate Answer the following questions:
r
1. . The rate constant for the first order growth of bacteria can be
constants are kl and k2 at the same temperature. If kl > kz, calculated using:
then:
(a)Eul >E~2 (b)Eul =Ea2 (c)Eal <Ea2 (d)Ea[ ?Ea2 ta) k --2.303
- - 1ogJO II -a-)
t a-x
(b) k = _ 2.303 log
. t
(_a_) -
a+x·
588,1 G.RB. PHYS16AL CHEMISTRY FOR COMPETITIONS

0.693 x 3. The half life for a zer~order reaction equals to:


(c) k (d) k 2
t .
2. Unit of rate constant for first order growth is:
(a) 1 !.- (b) a (c) 2k (d).!!...
2 a2 2k a 2k
(a) min-I (b) min2 (c) min-3 (d) unitless 4. For a reaction:
3. The rate constant for the reaction is: r + ocr ~ 10- + Cl-
(a) 0.0462 m i n - : ( b ) 0.462 min-I in an aqueous medium, the rate of the reaction is given by
(c) 4.62 min --;1 (d) 46.2 min-I
d[IO-] = k [1 -] [OCI -]
4. The rate of gro\.\fth initially is:
dt [OIr]
( a) 4.62 bacteria per min (b) 23.1 bacteria per min
(c) 23.1 bacteria per sec (d) 0.231 bacteria per sec The overall order of the reaction is:
5. At what time, there will be 6400 bacteria in the flask?' (a) -1 (b) 1 (c) zero (d) 2
(a) 150 min (b)' 90 min (c) i60min (d) 120 m\n 5. In a chemical reaction A ~ B, it is found that the rate ofthe
reaction doubles when the concentration of A is increased four
• Passage 4 times. The order of the reaction with respect to A is:
(a) 0 (b) 112 (c) I (d) 2
Order of reaction is an experimentally determined quantity. It
may be zero, positive, negative and fractional. The kinetic equation
of nth order rl!laction is: • Passage 5
kxt=
(n
I [ 1
I) (a-xr I
anI I] Consider the reaction represented by the ,equation:
CH3Cl(g) + HP(g) ~ CHPH(g) + HCI(g)
Half life of nth order reaction depends on initial concentration
according to the following relation: These kine.tic data were 'obtained for the given reaction
I concentrations:
tl12 <X--;:I
a Initial teD~ (M) Initial rate of diappearaJKe of
Unit ofthe rate constant varies with the order but general relation ·~i'
(H:PI .CR"CJM.-1
for unit of nth order reaction is: . .:lCBPI
n I' Oi2 0.2 1
. " I . _I
Umtofk= - -
[ cone. ]
x time 0.4 0.2' 2
0.4 0.4 8
The differential rate law for nth order reaction may be given as: Answer the following questions based on these data:
d:x =k[Ar' 1. The rate law for the reaction will be:
dt (a) r. k [CH 3CI][H2 0] (b) r = k [CH 3CI]2[H2 0]
where, A denotes the reactant.
(e) r= k [CH 3Cl][H 20f (d) r k [CH3Clf[H20t'
Answer the following questions:
1. Theunit of rate and rate constant are same for a: 2. Order with respect to [CH 3 Cl] will be:
(a) zero order reaction (b) first order reaction WO ~1 W2 003
(c) second order reaction (d) half order reaction 3.. Overall order of the reaction will be:
2. The rate constant for zeFO order reaction is: WO 001 W2 003
(a) k = Co (b) k = Co - Ct 4. Unit of rate constant will be:
U t (a) sec-I (b) litre 2 mol-2 sec-I
(c)k In Co - CI (d)k=Co (c) litre mol-I (d) mol litre- 1 sec- 1
2t C( 5. If H 20 is taken in large excess, the order of the reaction will
where Co and'C I are concentrations of reactants at respective be:
times. (all (b) 0 , (c) 3 (d) 2
, ,

[/I~"--------------t
P~J. 1. (d) 2. (a) 3. (d) 4. (c) 5. (d) 6. (a)
P.....,..2. 1. (a) 2. (c) 3. (b) 4. (c)
~3. 1. (b) 2. (a) 3. (a) 4. (a) 5. (b)
·P.....,..4. 1. (a) 2. (h) . 3. (d) 4. (b) 5. (b)
P~5. 1. (c) 2. (b) 3. (d) , 4:. (b) 5. (a)
CHEMICAL KINETICS 589

c:> SELF ASSESSMENT c:>


ASSIGNMENT NO.8

if the rate offorrnation ofN2 is 0.7 MIs, determine the rate at


SECTION-I which NH3 is consumed:
Straight Objective Type Questions . (a) 1.4 mol L-ls- 1 (b) 0.7 mol L-1S 1
This section contains 10 multiple choice questions. Each
(d) none of these
question has 4 choices (a), (b), (c) and (d), out of which only.
one is correct. 8. Consider the reaction:
1. What is the order of reaction,. 2H2 (g) + 2NO(g) N 2 (g)+ 2H20(g)
A2 + B2 ----'? 2AB ? The rate law for this reaction is:
Mechanism: Rate = k[H2 1[Nof
A2~A+A (fast) Under what conditions· could these steps represent the
A + B2 ----'? AB' + B (slow) mechanism?
A +B ----'? AB (fast) Step 1: 2NO(g )~ N Z0 2 (g)
002 ~l ~X OOX Step 2: N 2 0 2 + Hz ----'? N 20 + H 20
2. For a gaseous reaction, A (g ) ----'? Product, which one of the Step 3: N 20 + H2 ----'? H20 +N2.
ell"
10 owmg IS . among -~.
correct reIatlOn , - an d~?. (a) These steps can never satisfy the rate law
dt dt dt (b) Step I should be the slowest step
(-dP =:
Rate 0 f reactIOn de: : Rate 0 f reactIOn
' .III atm sec'-1 ; - . .m (c) Step 2 should be the slowest step
dt dt (d) Step 3 should be the slowest step
molarity sec-1; dn =: Rate of reaction in mol sec_1) 9. Which of tJie following is not for zero order reaction?
dt .
de dn dP de Idn dP
(a)-==-=:-- (b)-- --.:....=:--
dt dt . dt dt V dt dt
(c) de
dt
V dn
RT dt
dP
dt
(d) None ofthese i
x
3. The rate law for a reaction between the substances A and B is
given by the, rate k[AJ"[Bt. On doubling. the a ---'-+ .t ---'-+
concentration of A and halving the concentration of B, the (a) (b)
ratio of the new rate to the earlier rate of the reaction will be
as:

i
~.
I
(a) 2m + n (b)m + n (c) n - m (d)i,,-m) .

4. A substance having initial concentration' a' reacts according to


dx
1
(a-x)
dt
zero order kinetics. What will be the time for the reaction to go
to completion?
(a-x) --'--+- t ---'-+
(a)~ (b)k a
(c) 2k
(d) 2k
(c) (d)
k a a
5. The rate constant for the reaction,
2N20S{g) ----'? 4N02 (g) + °2(g) 10. I. :: 15 kJ mol-I; I1H 70'kJ mol- 1
is 3 X lO-s sec-I. If the rate of reaction is 11.. Ea 30 kJ mol-!; Mf = -15 kJ mol-!
5 I
2.4 x 10- rnollitre- sec-I, then the concentration ofN2 0 5 in III. Ea = 600 mol-I; I1H + 20 kJ mol-I
mol litre-I is: If above reactions are at same frequency factor then fastest and
slowest reactions are:
(a) 1.4 (b) 1.2 (c) 0.04 (d) 0.8
(a) III is fastest, II is slowest
6. Half life of a reaction is inversely proportional to cube of
(b) I is fastest, III is slowest·
initial concentration. The order of reaction is: .
.(c) II is fastest, III is slowest
004 ~3 ~6 M2
(d) III is fastQt.I i.s slowest
7. The oxidation of ammonia takes place as,
~NH3 (g ) + 30 2 (g) ----'? 2N2 (g) + 6H2 0(g )
590 G.R.B. PHYSICAL CHEMISTRY FOR COMPETITIONS

(Reason). Each question has following 4 choices (a), (b), (c)


SECTION-II and (d), out of which only one is correct.
Multiple Answers Type Objective Questions
11. Which of the following graphs are correct? (a) Statement-I is true; Statement-2 is true; Statement-2 is a
correct explanation for statement-I.
(b) Statement-l is true; Statement-2 is true; Statement-2 is not

1
Rate
a correct explanation for Statement-I.
(c) Statement-l is true; Statement-2 is false.
(d) Statement-l is false; Statement-2 is true.
16. Statement-I: If temperature does not affect the rate of
a- reaction, Ea = o.
(a) . (b) Because
Statement-2: Lesser is the activation energy, slower is the
rate.

1 1
.. tl/2
17. Statement-I: In the reaction:
N0 2 (g) + CO(g) ~ CO2 (g) + NO(g)
Rate k[N02f
tl/2
Th~ rate of reaction does not depend on the· concentration of
CO. \
11<1 - 1/a-
Because
(c) (d)
Statement-2: Carbon monoxide is involved in fast step.
Here 'a' denotes initial concentration of reactants. 18. Statement-I: The reciprocal of time in which 66% of the
12. Arrhenius equation may be given as: reactant is converted to product is equal to the rate constant of
first order reaction.
(a) In A = ~ . (b) log A = log k + Ea
k RT 2.303RT Because
Statement-2: The rate constant for first order reaction
(c) log [-Ea ] k (d) d In k depends on initial concentration of reactants.
RT a dt RT
13. WhiCh of the following is/are correct for second order reaction? .sEcTION-IV
(a) till is inversely proportional to initial concentration Matrix-Matching Type Questions
(b) k = ~ [_1__ .!.] Thi!> section contains 3 questions. Each question contains
t (a-x) a statement given in two columns which have to be matched.
Statements (a, b, c and d) in Column-I have to be matched with
(c) k = 2.303 log (-.a_J statements (p, q,rand s) in Column-II. The answers to these
t a-x
questions have to be appropriately bubbled as illustrated in the
(d) k = 1 [ 1· _~]
2
following examples:
2t (a - xi a If the correct matches are (a-p,s); (b-q,r); (c-p,q); and (d-s),
14. For a 1st order reaction: then correct bubbled 4 x 4 matrix should be as follows:
A ~ B, with initial concentration = a p q r s
. k .
. (a) t!12 =
a
0.693
(b) t3/4 = 2tl/2
a8@00
(c) tl/2
k
(d) tl12 k x 0.693
15. Select the correct statement for Arrhenius equation
b0_e0
. k = Ae-EaIRT
(a) 'A' maybe termed as the rate constant at very high
c8_0
temperature .
(b) , A' may be termed as the rate constant at zero activation
d®@0
. energy
(c) Ea is the activation energy of reaction 19. Match the Column-I with Column-II:
(d) k is rate of reaction at zero concentration Column-I Column-II
(a) Rate of reaction . (P) May be fractional
(b) Rate constant (q) Whole number
Assertion~Reason Type Questions (c) Order ofreactiOil (r) Independent of temperature
This section contains 3 questions. Each question of this (d) Molecularity of reaction (8) Increase with temperature
section contains Statement-l (Assertion) and Statement-2
CHEMICAL KINETICS I. 591
20. Match the Co1umn-I with Column-II: 21. Match the Co1umn-I with Column-II:
ColuMn-) ColuMn-II CoIUMn-) ColuMn-II
(Order of reaction) (Grapb) (a) First order (P) Unit of k sec-I

(L~
(b) Pseudo first order (q) Unit ofk = dm6 mor2s-1
(P) (c) Third order (r) t112 oc: 11 a 2
(a) First order
1
(d) Second order (s) tl12 ex:-
t- a

(b) Second order (q)


l~ t 112 -
i .
(c) Third order (r)
I~
~
t-
.

(d) Zero order -(s)


!~ t1/2 -

[A~· 11

1. (c) 2. (b) 3. (d) 4. (a) 5. (d) 6. (a) 7. (a) 8. (c)


9. (c) 10. (b) n. (b,c) 12. (a,b) 13. (a,b) 14. (b,c) 15. (a;b,c) 16. (c)
17. (a) 18. (e) 19. (a-s) (b-s) (c-p,r) (d-q,r) 20. (a.:r) (b-p) (c-q) (d-s) 21. (a-p) (b-p) (c-q,r) (d-s)
CHEMICAL EaUILIBRIUM

9.1 INTRODUCTION I. (a) Thermal decomposition of potassIum chlorate,


Mn02
It is a well established fact that many reactions do not go to 2KCI0 3 ----j 2KCI + 302
Heat
completion. They proceed to some extent leaving considerable
amounts of unreacted reactants, i.e., the resulting mixture (b) Decomposition ofammonium nitrite,
contains both reactants and products. When such a stage is NfJ 4N0 2 ~N2 + 2H 20
reached in a course of reaction that no further reaction is 2. Precipitation reactions,
apparent, it is said in chemical language that the reaction has
(a) AgN0 3 + NaCI ~ AgCl + NaN0 3 .
attained the state of equilibrium where the composition of the
system becomes fixed.·At equilibrium state both the forward and • (b) Pb(N0 3)2 + 2KI ~ PbI 2 + 2KN0 3
. backward reactions move with equal speeds, i.e., the rate of. 3. Neutralisation reactions,
disappearance of reactants is exactly equal to rate of the H 2S0 4 + 2NaOH~Na2S04+2H20
ilppearance of reactants from the products. The study of chemical Strong acid . Strong base
equilibrium helps in the elucidation of the optimum conditions 4. Redox and com?ustion reactio'ns,
for the greater yields of the products in the case of those reactions
(a) SnCI 2 +2FeC13 ~ SnCI 4 + 2FeCI 2
which attain equilibrium in the course of reaction. .
Chemical reactions can be classified as irreversible and. ( b ) · 2Mg + 02 ~ 2MgO
reversible reactions. (c)
Irreversible reactions: The chemical reactions which Reversible reactions: The chemical reactions which take
proceed in such a way that reactants are completely converted place in both directions under similar conditions are called
into products, i. e. , the reactions which move in one direction, i.e., .reversible reactions. In such reactions, the products also react
forward direction only are ·called irreversible reactions. In such with each other and produce reactants again. For example, when
reactions, products do not react together to produce react.ants hydrogen is passed over heated magnetic oxide, metallic iron and
again, i. e. , the reaction does not move in backward direction. For water are produced.
example, when solutions containing equimolar concentrations of
Fe304 + 4H2 ----j 3Fe + 4H 20
barium chloride and sodium sulphate are mixed, reaction occurs
and practically whole of barium is precipitated as barium On the other hand, if steam is passed over powdered iron at the
sulphate. same temperature, magnetic oxide of iron and hydrogen are
formed.
BaCI 2 + Na 2S04 ~BaS04 + 2NaCI
ppt. 3Fe + 4H 20----jFe.30 4 + 4H2
The reaction between BaS04 and NaCl, i. e., backward If the reaction is carried out in a closed vessel, it is found that
reaction, is not possible under the experimental c.onditions. In in no case the reaction proceeds to completion. It is, thus, clear
such reactions the arrow (--+) is placed between reactants and that either of the two reactions can take place independently if
products which indicates the direction of the chemical change. steam or hydrogen is, allowed to escape from system by carrying
Some more examples of irreversible reactions are given below: out the reaction in open vessel but if a closed vessel is used and.
I
CHEMICAL EQUILIBRIUM 593
nothing 'is allowed to escape both ,forward and backward At equilibrium siate,
reactions can take place in the vessel. ' Rate of forward reaction = Rate of backward reaction.
Reactions which thus proceed in botb the directions and
It can be shown graphically as in Fig. 9.1.
do 'not rea~h to completion are known as reversible reactions. '
The reaction proceeding from left to right ,is conventionally Thus, chemical eqUilibrium in a reversible reaction is the
called the forward re_ction and the opposite one proceeding state at which both forward and backward reactions or two
from right to left is called the reverse or backward reaction. In opposing reactions occur at the same speed.
such reactions the arrow (-t ) or sign of equality (=) is replaced This state will continue indefinitely iUhe conditions such as
by two half arrows (, ') pointing the reaction in both the temperature and concentration are not changed. At the state of
directions. This sign « ') represents the reversibility oLthe equilibrium, the concentrations of reactan.ts and products in the
reaction. reaction mixture attain a constant value and the concentrations do
not change with time.
3Fe+4H20~Fe304 +4H2 When hydrogen gas and iodine vapours are heated in a closed
Some examples of reversible reactions are given below: vessel at a constant temperature 717 K, the two react to form
hydrogen iodide. In the initial stage, the colour of the reaction
CaC0 3 ~ CaO + CO 2
mixture is deep violet due to the presence of large amounts of
CH3COOH+C2H50H~CH3COOC2H5 +H 20 iodine. But as the reaction progresses, the intensity of the colour
decreases as more and more iodine is converted into hydroge'"
2Hl H2+I2
iodide. After sOl]letime, the intensity of the colour of the reaction-
PCIs ~ PCl 3 + Cl 2 mixture becomes constant. The constancy of intensity of colour
N2 + 3H 2 ~ 2NH3 indIcates that concentration of both reactants and products have
become constant and state of equilibrium has been attained.
2S02 + Oz 2S03 . The stage of the reversible reaction at which the
N2 + O 2 ---->. 2NO concentrations of the reactants and products do not change
with time is called the equilibrium state.
NH4HS~NH3 + H2 S
Or
CO 2 + Hz ..------. CO + H2 0 The state in which the measurable properties of the system
(such as pr~ssure, density, colour or concentration) do not
9.2 STATE OF CHEMICAL EQUILIBRIUM undergo any further noticeable change with time under given
The most important characteristic property of a reversible set of conditions is said to be a state of equilibrium.
reaction is that it always attains a state of chemical equilibrium. The equilibrium state is dynamic and not static in nature. The
Consider a general reversible reaction in a closed vessel. reaction does not stop but both the opposing reactions are going
A+B~C+D
on continuously with same speeds. At this stage, the number of
moles of substances produced per second in the forward reaction '
In the initial state only A and B are present both react with each is equal to the number of moles of substances which disappear
other, i. e., rate of forward reaction is maximum as only the per second in the backward reaction.
concentrations of A and B are involved (at the beginning, the
concentrations of products C and D are nil). As soon as the Characteristics of Equilibrium State
products C and D are produced, the backWard reaction starts (i) Equilibrium state can only, be achieved if a reversible
functioning. By the expiry of time, the rate of forward reaction reaction is carried ouLin closed space.
decreases as the concentrations of A and B decrease while the rate No product can leave nor any reactant from outside can enter
of backward reaction increases as the concentrations of products the space. If the system is not closed, some of the. products may .
C and D increase. Ultimately, a stage comes when the rate of escape and thus, the backward reaction will not occur.
forward reaction becomes equal to rate of backward reaction. . (ii) Chemical equilibrium, at a given temperature, is
This state is called the equilibrium state. characterised by constancy of certain properties such as pressure,
concentration, density or colour.
(iii) Chemical equilibrium can be attained from either side,
i. e. , from the side o~ reactants or products.
1 ,Equilibrium state
2m~H2 +12
or
~ila.{d
'Q'?>c
!
d'- ..i...p(\ ,
I
I At equilibrium, each reactant and each product has a fixed
!be 'qp.v' , : TIme at which system '
concentration and this is independent ofthe fact whether,we start
~ <;' !has reached equilibrium
the reaction with the reactants or with the products .
. TIme-

Fig. 9.1
5941 G.R.B. PHYSICAL CHEMISTRY FOR COMPETITIONS

This reaction can be graphically represented as, 9.3 THE LAW OF CHEMICAL EQUILIBRIUM
(Applicati9n of Law of Mass Action)
i
c
Equilibrium
region Consider a reversible homogeneous reaction which has attained
equilibrium state at a particular temperature.
o
----~----------H2
~C
A+B::;:::==:'=C+D
Ql
--+--------12 Let the active masses of A,B, C and D be [A], [B], [C] and [D]

u
U
C
o --11-------
1
HI respectively at equilibrium.
According to law of mass action,
1
1
'1
1
I' Rate of forward reaction oc [A][B]
---Time-- or Rate of forward reaction kf [A][B]
Fig. 9.2 (where, kf is the velocity constant for forward reaction).
Similarly, '
(iv) Equilibrium state can be attained in a lesser time by
Rate of backward reaction ex: [C][D]
the use of a positive catalyst, i. e., the relative concentrations
,of reactants and products' remain the same irrespective of or Rate of backward reaction kb[C][D]
the presence or absence of a catalyst. Thus,' a catalyst does (where, kb is the velocity constant for backward rel;lction).
not change the equilibrium state but it helps in attaining it rapidly. At equilibrium,
(v) It is dYnamic' in natunoj, i. e. , b()th-the reaction~ move with
same speed. However,the r~action seems to have come to stand Rate of forward reaction = Rate of backward reaction,
still because the concentrations of reactants and products do nQt
'change. ;
i. e., kf [A][B] = kb[C][D]
Reversible chemical reactions are classified into two types kf [C][D]
or
heterogeneous and homogeneous reactions. kb [A][B]
, Heterogeneous reactions: The reversible reaction in which
Since, kf and kb are both constants, the ratio kff kb is also a new
more than one-phase is present.
constant say K c' i. e. , \
" CaC0 3 (s) ::;:::==:'= CaO(s) + CO 2(g) [C][D]
Kc ... (i)
MgC0 3 (s)::;:::==:'= MgO(s) + cO 2 (g) [A][B] "
, 2Naz 0 2(s) + 2H 2 0(1) 4NaOH(l) +O 2 (g) Kc is known as equilibrium constant. Kc has a definite value
,3Fe(s) + 4f120(l) ::;:::==:'=Fe304(S) + 4H 2(g) for every chemical reaction at a given temperature regardless of
, "
the concentration of the reactants.
Homogeneous reactions: The reversible reaction in which
Considering a more general homogeneous reversible reaction
only one-phase is present, i. e., all the reactants and products are
at a particular temperature which is under chemical equilibrium,
in the same physical state.
mlA I +m2 A 2 +~A3 + ... ::;:::==:'=nIBl +n2 B 2 +n3 B 3 + ...
H2 (g) +I2(g) 2H1(g)
Rate of forward reaction k[ [AI ]ml [A2 ]m2 (A3 ]m3 •.•
CH 3 COOH(l) + C 2Hs OH(1) ----" CH 3 eOOC 2Hs (l) + H 2°(l)
Homogeneous reversible reactions are further classified into
Rate of backward reaction = kb[Bl tl {B 2] "2 [B 3 t 3 •••
three types: At equilibrium,
First type: When there is no change in number of kf [AI ]ml [A2 ]m2 [A3 ]"'3 ... = kb[BI ]nl [B2 t2 [B3 r 3 ...
molecules. Examples are:
kf [Bltl[B2t2[B3t3 ... =K
or ... (ii)
(iv H2 +12 ::;:::==:'=2H1 kb [Ad m1 [A2]~[A3]"'3 .:. c
(ii) , 2NO::;:::==:'=N2 +02 The equilibrium constant, at a given temperature,is the
ratio of the rate co.nstants of forward and backward
(iii) CH3COOH + C2HsOH::;:::==:'=CH3COOC2Hs + H 2 0
rea~tions.
Second, type: When there is .an increase in number of Or
molecules.
The equilibrium constant may be defined as the ratio.
PCl 5 ::;:::==:'=PC1h +C1 2 between the product of the molar concentrations of the
, 2NH3 ::;:::==:'=N 2 +3H 2 products to that of the product of the moiar concentratio.ns of
Third type: When there is a decrease In number of the reactants with each concentration term raised to a power
molecules. . equal to stoichiometric coeffiCient in the balanced chemical
equation.
N2 t 3H 2 2NH3
2S0 2 + O 2 ::;:::==:'= 2S0 3
I
CHEMICAL EQUILIBRIUM I 595
The value of equilibrium constant is independent of the '(b) Consider the following chemical equation,
following factors: Nz(g)+ 0z(g)+ Cil (g) 2NOCI(g)
1; Initial concentrations of the reactants involved.
, [NOCl] 2
2. The presence of a catalyst.
3. The direction from which the equilibrium has been
Kc =[N2 ][02 ][Cl z ] ... (i)
attained. Suppose the a\love equation is split into two as:
4. The presence of inert materials. [NO]z,
The value of equilibriUJ11 constant depends on the following N 2 (g)+ 02(g) 2NO(g); Kl = ..• (ii)
factors: ' [N2)[OZ]
1. The mode of representation of the reaction:
Conventionally, the concentrations of the products are put in the 2NO(g) +Cl z(g ) ~ 2NOCl(g); K
2
=' [NOG]2 ... (iii)
[No]2 [Cl z]
numerator and the concentrations oftbe reactants in denominator
in the equilibrium law equation. Consider the reversible reaction, Combining above two equations,
2
A+B C+D K xK - [NOf X [NOCI]z == [NOCI] =K
The equilibrium constant for the reaction, I 2 - [NzUOz) [NOf[Cl z] [N2][02][Cl z ] c

K = [C][D] ... (i) Thus, when an equation (having equilibrium constant, K c) ';"
c [AUB] written in two steps (having equilibrium constant k 1 and K2 )j--
Now, if the products are ,made reactants, i. e., the reaction is then K c :: K I X K 2: '
reversed, 3. U~e of partial pressures instead of concentrations:
C+D A+B When the reactants and products are in gaseous state, the partial
pressures can be used instead of concentrations at a definite
The equilibrium constant for the reaction is
temperature, as the partial pressure of a substance is proportional.
K' = [AUB] ... (ii) to its concentration in the gas phase .
C [CUD] Consider a general reversible homogeneous gaseous reaction
The equilibrium constant, K;, is actually the reciprocal of m1A 1 +miA2 +m3 A ).+... nlB I +n2 BZ +n3 B 3 + ...
K c ' i.e., Let the partial pressures of various reactants and products be
K,=_l_ pAl' pAz, pA3' ... , and pBI' pB 2, pB 3 , ••• respectively at
C Kc equilibrium. The equilibrium constant for the reaction,
2. Stoichiometric representation of the chemical Kp = (pBI tl(pB2)n2(p B 3)n3 ... ...(i)
equation:, (a) When a reversible reaction can be written with
( pAl )ml ( pA z )m2 (p A 3 )m3 .. .
the help of two or more stoichiometric equations, the value of . ' ' . ' .

equilibrium constant will be numerically different in these cases. The value of K c for the above reaction can be given as:
For example, the dissociation ofNOz can be represented as: n1
K= [Btl [B2r2[B3t3 ...
. .. (ii)
2NO z .,..:......: N z + 20z ... (i) c [Adm! [A z ]in2[A 3)1113 ...
I '
or NOz - Nz + 0z ... (ii)
, ,2, F or an ideal gas, pV=nRT
. . ' [Nz][Oz]z 'n
For equatIOn (I), the value of Kc = z' , ... (iii) or, p=-,RT,
ENOz ] V
[N ]1I2[0 ] = Active mass x RT
For equation (ii), the value of K; = z z ... (iv)
[NO z ] (wijere, n = number of moles and V =volume in litres. Thus,
Thus, the two constants are related to each other as: -Vn =moIar concentration
.'
. or actIVe
. mass) .
K;=.[K;c, ... (v)
Substituting the values of partial pressures in eq. (i),
In general, when a balanced' equation, having equilibrium
constant K e' is multiplied by a certain value n, the equilibnum K r
=. [BI tl (RT )nl . [B z )"2 '(R T t2 . [B3 3(RT )n3 .. .
constant for the new equation wjll be equal to (Ke t. P [Ad"!} (RT )m! . [A 2 ]"'2 (RT )m2 .[A3 ]m3 (RT) m3 .. .
A +B C +D, Kc = [C][D]
[A][B] HBtlnl(B'2rZ[B3)"3 ... }(RJ )nl +"2+ n3+'"
or Kp
{[A; )m} [A2 ]m2 [A3 )m3 .. .}(RT )mJ +mz +m3 +.. ,
nA+nB~nC+nD, it' :: [CY[Dt =(K )"
C [At[Bt c,
or K = K . (RT )Ln
p C (RT)Lnz
596 G.R.B. PHYSiCAL CHEMISTRY FOR COMPETITIONS

Kc (RT )EJI- Em or log K z > log K J


:::Kc(RT ),'111 ... (iii) or K 2 >K J
The value of equilibrium constant is higher at higher
An ::: total number of molecules of gaseous products
temperature in the case of endothermic reactions.
total number of molecules of gaseous reactants.
Third case: When /ili - ve, i. e., heat is evolved, the
Three cases may arise:
°, reaction is exothermic. The temperature Tz is higher than Tl .
First case: When, An
Kp :::Kc (RT)o Kc Thus, (~
T
-
z
J..'j
TJ
is negative.

Second case: When, An == + ve or An > 0, So, log K 2 - log K J ::: - ve


or log K 1 > log K 2
Kp >Kc
or K J >K2
Third case: When, An -ve or An < 0, The value of equilibrium constant is lower at higher
Kp <Kc temperature in the case of exothermic reactions.
4. Temperature: Accbrding to Arrhenius equation,
k==Ae- EJRT ••• (i)
where, k :::rateconstant, E::: activation energy, R :::gas
constant, T ::: absolute temperature and e == exponential constant.

log
k2
k; ::: E [I IJ
2.303R T2 - T
J
... (ii)

T-- T--
when, T2 > TJ (a) Endothermic reaction (b) Exothermic reaction

(l 1j
for forw!'-d reaction,
kh
log -;;;: == - 2.303R T2 - T
[1 IJ J
... (iii)
(Plots of log K versus T)

Fig. 9.3

for backward reaction, Units of Equilibrium Constant

[:~) ~[~_J..] The units of equilibrium constant vary in case of different


... (iv)
log 2.303R T2 Tl reactions. 'K' has no units for a reaction in which total number of
moles of reactants and products are the same. For example, for
Subtracting eq. (iv) from eq. (iii), we get dissociation of nitric oxide, K c has no units.

log
[ khlk~ 'j
kfl/kb l
- (E f Eb
2.303R
)[ 1 I]
---
T2 Tl
2NO~Nz +02

K 21 /ili [1 1]
or
log ( K: ; - 2.303R T2 - Tj
... (v)
K c will have units for a reaction in which the total number of
where, /ili is the heat of reaction at constant volume and
moles of reactants and products are different. For example, for
. K 1 and K 2 are the equilibrium constants of a reaction at
decomposition of PCI s , the K c has mol/litre units.
temperatures Tj and T2 (Tz > Tj ).
The effect of temperature can be studied in the following three PCl 3 + Cl z
cal"es: [PC1 3 ][C1 2 ]
First case: /ili == 0, i. e. ,neither heat is evolved, nor absorbed.
So, 1
or
log K 2 - log K I :::
log K 2 log K I
° In the formation of ammonia,
[PCl s ]

or Kz KJ N z + 3H 2 ---->. 2NH3
. Thus, equilibrium constant remains the same at all
-temperatUres. K = [NH 3 ]z
Second case: When, /ili = -+ veL e., heat is absorbed, the c [N 2 ][H 2 ]3
reaction is endothermic. The temperZture T2 is higher than TJ • K c has litre 2 mol-i units.
I
Thus,
( -T2 -I ' liS. negative.
. In general; unit of K c [M]Ll.n
Tj;
So, log K 2 log K j = + ve
I
CHEMICAL EQUILIBRIUM 597

where, M . mol litre -I and!1n number of gaseous moles of and for equation, ~N2 + 7i H2~ NH 3 ,
product or products number of gaseous moles of reactant or K' [NH 3 ]
reactants. ... (ii)
c [N2 ]112 [H2 ]312
Note: (i) The above relation can be used in homogeneous liquid system
also Squaring equation (ii),
!:>.n number of moles of product or products
number of moles of reactant or reactants
(K; )2 = [NH 3]2 3 ... (iii)
(ii) Similarly, the unit of Kp [atmjlln
[N 2 ][H 2 )
where, !:>.n number of gaseous moles of product or products Equations (i) and (iii) are same
- number of gaseous moles of reactant or reactants. Thus, Kc = (K~)2

9.4 REACTION QUOTIENT OR MASS or JK: =K;


ACTION RATIO Example 2: The equilibrium constant for the reaction,
Let us consider a reaction: N2 + 202 ~ 2N0 2
A+B~C+D at a particular temperature is 100. Write down the equilibrium
Q= [C ][D] law equations for the following reactions and determine the
values of equilibrium constants:
[A][B]
Q is denoted as Qc or Qp depending upon whether the 2NOz ~N2 + 202 " ••• (i)
concentration is taken in terms of moles per litre or partial N0 2 ~ ~N2 +02 ... (ii)
pressures respectively. With the help of mass action ratio we cruy ,
determine whether the reaction is at equilibrium or not. . . Solution: The eqUilibrium COBstant for the reaction,
(I) When, Qc K c or Qp K p. then the reversible reac:tion is N2 + 20 2 ~ 2N0 2
. at equilibrium, i. e., the rate of forward and backward reaction
becomes equal. Kc = "[N0 2 ]2 2 = 100 ,.. (a)
(II) When, Qc < K c or Qp < K p then the reaction is not at [N2 )[°2 ]
equilibrium. The reaction will be fast in forward direction, i. e. , The equilibrium constant equation for reaction (i),
reaction has a tendency to form product/products.
Rate of forward reaction> Rate of backward reaction. K"I = [N 2][°2 f ... (b)
(III) When, Qc > K c or Qp > K p then again the reaction is [N0 2 ]2
not at equilibrium. The reaction will be fast in backward This equation is reciprocal of eq. (a)
direction, i. e. , have a tendency to form reactant/reactants. So, = _1_ = 1x 10-2
K I = _1_
Rate of forward reaction < Rate of backward reaction. Kc 100
The equilibrium constant equation for reaction (ii),
0=0 K 0==
]112
I ·1 I K = ... (c)
(Reactant only) (Product only) 2
[N0 2 1
Reaction goes towards Comparing eqs. (b) and (c),
right. forming more products
K2 =.j""K; ~1O-2 10- 1 0.1
" Example 3. The value of K c for the reaction,
:: ::::I_SOME SOLVED EXAMPLES\I:::::: N 2 (g)+3H 2 (g) 2NH3(g)
is 0.50 at 400"C. Find the value of K p at 400"Cwhen concfmtra-
Example 1. For the reactions, tions are expressed in mal litre -I and pressure in atmosphere.
N2 +3H2~2NH3 Solution: Applying the relationship,
Kp = K c (RT)An
and 7iN2+7iH2~NH3
write down the expression for equilibrium constant K c and K ~. Kc = 0.50, R = 0.082litre-atm deg -\ mol-I
How is K c related to K ~ ? T(400+273)=673K,. lln (2 4)= 2
Solution: For equation N2 + 3H2~ 2NH 3 ,
Kp = 0.5(0.082 X 673)-2
[NH 3 ]2
Kc ... (i) = 0.5(55.1 85r2 1.64 x 10-4
[N 2 ][H2 ]3
598 G.R.B. PHYSICAL CHEMISTRY FOR COMPETITIONS

Example 4'. Determine K c for the reaction,. Solution: Using the relation,
1 1" l'
-N2(g)+ -0 2(g) + - Br2(g) NOBr(g)
log KP2 =~[T2 Tl]
2 2 2, KPI 2303R T21i
Given, 0"

from the following data at 298 K;


Kpl 1.6 x 10-4 ; All = - 25.14 kca:l;
·The equilibrium constants for the following reactions,
R=2xlO-3 kcaldeg- 1 mol- I
2NO(g ) ~ N 2 (g) + O2 (g)
• 1 11 = 400+ 273 = 673 K, T2 = 500+ 273= 773 K
and NO(g) + - Br2 (g) NOBr(g)
2 log KP2 - 25.14
= -------::- [773
-- - 673.]
-
30 (l:6 x 10-4 ) 2.303 x 2 x 10-3 773 x 673
are 2.4 x 10 and 1.4 respectively.
3
Solution: The net reaction is, 10 . K = 10 (1.6 X 10-4 ) _ 25.14 X 10 x 100
g P2 0 g 2.303 x 2 x 773 x 673
~N2(g)+ ~02(g) t ~Br2(g) NOBr(g) = 3.7960 - 1.049 = - 4.8450
KP2 1.429 x 10-5 atm-2
oK = "" [NOBr].
o C(net) [N2 ]1/2 [02 ]112 [Br2 ]1/2 Example 6.' The equilibrium constant for the reaction,
H2{g)+ S(s)~H2S(g)
Considering the given equations:
2NO(g) ~ N:z(g) + 02(g); equilibrium constant = 2.4 x 10 30 is 18.5 at 925 K and 9.25 at 1000 K respectively. Calculate the
enthalpy ofthe reaction.
, N 2(g) + 02(g)"':""""'>' 2NO(g); equilibrium constant = I 0 Solution: Using the relation,
2.4 X 103
o •

I K2 All [T2
"K; = 2.303R 11 T2
-lIj
.!.N2(g) + I °2 (g) ~ NO(g); .
og
2 2 ' 9.25 All 75
Io g - = x----
Eq~i1ibrium constant
18.5 2.303 x 8.314 925 x 1000
[ 1 30 )112
• 00 2.4 x 10 0.301 = All x 75
2.303 x 8.314 x 925 x 1000
= 0.6455 X 10- 15 or . . All:::: 71080.57 J mol- 1

o [NO] . = K:' =0.6455 X 1O- IS ... (i) OF O'BJEClIVEQUESTIONS


(N2 ]1/2[Od /2 c.
00 • 1 '1. For the hypothetical reactions, the equilibrium constant (K)
NO(g) +-Br2 (g) ~NOBr(g) values are given:
o 2 A~B;Kl =2;B~C;K2=4;C~D;K3=3
[NOBr] =K"= 1.4 The equilibrium constant (K ) for the reaction A ~ Dis:
,C • ... (ii)
(PMT (Kerala) 20051
[NO] [Br2 o

(a)48 . " . (b) 6 (c) 2.7 (d)12 0

Multiplying both equations (e) 24


[Ans. (e)]
~NO] x (NOB~] K~ x K~' 0.6455 x 10- 15 x 1.4 LHint: The reaction A D is obtained by adding the three
[N 2 ]12[0 2 ]1/2 [NOl[Brd /2 giv~ reactions.

or . [NOBr] == K = 0.9037 X'1O- 15


. [N ]1I2[021lf2 [Br2 ]1/2 c(ne!)
2. 'If, in the reaction N20 4(g) ~ 2NOz(g), x is that part of
2
N 20 4 which dissociates, then the number of molecules at
= 9.037 X 10-16 equilibrium will be: (KCET 2005)
(a) I (b) 3
Example 5. . The equilibrium' constant,· K P' for the
reaction (c)(l+x) . (d) (1+ x)2
2NH3 (g) is 1.6 x 10-4 atm -2 at 400°C. [Ans. (c)]
[Hint:
What will be the equilibrium constant at 500°C if heat of 1=0
o reaction in this temperature range is - 25.14 kcal?
; .
Total number of molecules at equilibrium = 1 x + 2x = (I + x)]
I
CHEMICAL EQUILIBRIUM 599

3. A schematic plot of In K eq. vs (ii) CH 4 (g) + H20(g) ~


CO (g) + 3H 2(g);

inverse of temperature for a


reaction is shown in the figure.
t&
(iii) CH 4 (g) + 2Hp(g) ~ COz (g) + 4Hz(g); K3
Which of the following relations is correct ?(AmI':E Z008)
The reaction must be: -::l
Cl (a)K3K; = K~ (b)K].fi[; K3
(AIEEE 2005) g
(a) exothermic (C)K2 K 3 = Kl
(b) endothermic 1/T- [Ans. (d)]
(c) one with negligible enthalpy change [Hint: Reaction (iii) is obtained by adding (i) and (ii) hence
(d) highly spontaneous at ordinary temperature K3 =K 1 xK2]
[Ans. (a)]
[Hint: Ke = Ae-t:JfoJ RT 9.5 ACTIVATION ENERGIES FOR
Ali ° FORWARD AND BACKWARD
logK =logA - - - -
eq" 2.303 RT REACTIONS
Y=C+MX
In a reversible reaction, the molec1,1les of the reactants in the
Slope of the line will be positive, when, Ali 0 -ve, i.e., the
forward reaction and the molecules of the products in the
reaction is exothermic.]
backward reaction fonow the same path" and foim the salne~
4. Given:
activated complex. However, th~ activation energies of both .
Nz(g) + 3Hz(g) ~ 2NH 3 (g); Kl
forward and backward reactions are different. Fig. 9.4 (a) and (b)
Nz(g) + 02(g) ~ 2NO(g); K2
show the activation energies, E a (f) and E a (b) respectively, for
H2 (g) + ~ 0i(g) ~ H2 0(g); K} the forward and backward reactions for exothennic and
2 endothennic reactions. Mathematically, E a (f) and (b ) are
The equilibrium constant for related to overall energy change, M, in the process as:
2NH3(g) + %0z(g) ~ 2NO(g) +3H zO(g) M = E a (f) - Ea (b) =I:!H (At constant volume)

will be: [CBSE (!Vied.) 20071


" 2 3
(e) K]K} (d)
K2 K]
"-
[Ans. (d)]
,t t
[NH ]" [NOl 2 ~
[Hint: K = 3 . K - .K Q)
C
I [N z ][H2 ]3' 2-[N2 ][02]' 3 W
Ep Er
The equilibrium constant for
Products Reactants
5' "
2NH3(g) + :2 °2(g) ~ 2NO(g) + 3HzO(g)
Progress of reaction - " Progress of reaction -:-
will be: If"
Fig. 9.4 (a)" Activation Fig. 9.4 (b) ,Activation
[NO] 2 [H 20] 3 = K"K3
2 X 3] energies for the forward and energies for th$ forward
[NH 3 f[02]5/2 Kl backward" reactions where and backward reactions
5. What is the equation for the equilibrium constant (K c) for the forward reaction is where forward reaction
exothermic is endothermic
following reaction? (EAMCET 2U06}
liTK 2 For exothermic reaction,
A(g) + -B(g)~ - C(g) " Ea(f)<Ea(b)
233
[A ]112[B]1/3 [C]3IZ Thus, M is negative.
(a) Kc (b) Kc = --=--~ tin the other hand, for the endothennic reaction,
[C]3/2
E a (f) > E a (b)
[Cf/3 Thus, M is positive
(c) K = -:-;-;;----;--;:;- Heat of reaction I1H = Activation energy of forward reaction
c [A]IIZ[B]1I3
" -Activation energy of backward reaction. \
(Ans. (c)] " Example 7. " For the dissociation ofgaseous HL the energy~
[Hint: Coefficient goes to the power of respective active mass. is
of activation 44.3 kcal.! Calculate the energy of activation for
[C ]213 the reVerse reaction. Given, I:!H for the formation ofl '7wle of HI
K - ]
c - [A]1/2[Bf3 from H2 and 12 is -=-1.35kcal.
6. For the following three reactions (i), (ii) and (iii), equilibrium Solution:
constants are given: 2HI(g) ~H2(g) +I 2 (g); Ea = 44.3kcal
(i) CO(g) + H20(g)~ CO 2(g)+ H 2 (g); Kl .. ",
I
600 G.R.B. PHYSICAL CHEMISTRY FOR COMPETITIONS

1 1 .----"
- H 2(g) + -I2(g) ~ HI(g); AH = 1.35kcal
2 2
2HI(g) ~ H 2(g) + I 2(g); AH =2x 1.35 = 2.70kcal
. AH E a (f) - E a (b) .
2.70 44.3-Ea(b)
Ea (b)= (44.3 - 2.70)
or = 41.60kcal
Exam}1le 8. For a reaction X --? Y, heat of reaction is
+ 83.68 kJ, energy of reactant X is 167.36 kJ and energy of
activation is209.20kJ. Calculate (i) threshold energyJii) energy
._- 1
T .

ofproduct Yand (iiI) energy ofactivation for the reverse reaction Fig. 9.5
(Y --?X). (i) When, AGO = 0, then K c 1.
Solution: (i) Given, Ea (f) = 209.20 kJ (ii) When,AGo > 0, i.e.,+ve, thenK c < 1. In this case reverse
AE + 83..68kJ reaction is feasible, i.e., less concentration of products at
Energy of teactant; X 167.36 kJ equilibrium state ..
Threshold energy = Energy of reactant X + Activation energy (iii) When, AGO < 0 • i.e., - \Ie, then K c > 1. In thiscase~-~
for forward reaction forward reaction is feasible showing thereby a large
167.36 + 209.20 = 376.56 kJ concentrations of products till the equilibrium is reached.
(ii) Energy of reaction, AE = Energy of product Energy of Example 9. For the reaction,
reactant 2NOCI(g ----" 2NO(g ) + Cl2 (g ), ....,;;
Energy of product = 83.68 + 167.36= 251.04 kJ
(iii) Activation energy for backward reaction calculate the standard equilibrium constant at 298K. Given that
the values of AHO and AS ° of the reaction at 298 K are 77.2 kJ
= Threshold energy - Energy of product mol -\ and 122JK -1 morl.
= 376.56 - 251.04 = 125.52 kJ
Solution: Using the relation,
AGO = mo - TAS °
9.6 STANDARD FREE ENERGY CHANGE = 77200 - 298 x 122
OF A REACTION AND ITS = 40844 'J mo1-1
EaUILIBRIUM CONSTANT
... Let the equilibrium constant be K~. We know that,
Let AGO· be the difference in free energy of the reaction
·when all the reactants and· products are in the standard state AGO = - 2.303RTlog K~
· (1 atmospheric pressure and 298 K) and Kc or Kp be the or log K~ = _ AGO = 40844
thermodynamic equilibrium constant of the reaction. Both are 2.303 x 8.314 x 298 2.303 x 8.314 x 298
related to each other at temperature T by the following relation:
. AGO =-2.303RT log Kc = 7.158
and AGO - 2.303 RTlog Kp. (in case of ideal gases) KOc 6.95 x 10-8
This equation represents one of the most important results of
Example 10. AGO for 1 N z (g) + ~ H 2 (g) ---"'. NH 3 (g)
· thermodynamics and relates to the equilibrium constant of a 2 2
reaction tp a thermochemical property. It is sometimes easier to is -16.5.kJ mol-I. Find out Kp for the reaction at 25°C. Also
calculate the free energy in a reaction rathet than: to measure the
equilibrium constant. report Kp and AGO for N 2 (g)+3H z (g) 2NH3(g) at
Standard free energy change can be thermodynamically 25°C.
calculated as: Solution:
AGO == AHo - TAS ° AGO (- 16.5 X 10 3 )
Here, AHO =standard enthalpy change, log Kp = - 2.303RT =- 2.303 x 8.314 x 298 2.8917
AS ° standard entropy change. Kp = 779.41
-RTloge K p =IJJ:lO-TASO . Kp for reaction N 2 (g)+3H 2 (g) 2NH3(g) is equal to
. ASO IJJ:lO
log K = - - - (779.41): 6.07x10s
e p R RT
.~\G = - 2.303 x 8.314 x 298 log 6.07 x lOs joule
10 K _ ASo IJJ:lO
gJO p - 2.303R 2.303RT == - 32.998 kJ mol-I
CHEMICAL EQUILIBRIUM 601

9.7 EQUILIBRIUM CONSTANT Thus,


EXPRESSIONS FOR SOME This also follows from the relationship, K p Kc(RT )'l.n.
REACTIONS
Iln = 0, So K p =K c '
Case I: Homogeneous gaseous equilibria when, Iln O. = i. e., for all gaseous reactions of first type, K p and K care,
Synthesis of hydrogen" iodide: . The fonnation of hydrogen
identical. Both K p and K c have no units.
iodide from hydrogen and iodine is represented by the ,equation:
. 4x 2
H2 (g) + 12 (g)...-- 2HI(g) (a) Effect of pressure: The equation, Kc =- - - - ,
(a-x)(b-x)
Let a start be made with' a' moles of hydrogen and' b' moles of
does not include the volume tenn; hence K c is independent of
I z. These are heated in a sealed bulb having a volume V litre by
pressure. Thus, change of pressure will not alter the final state
keeping the bulb'in a thennostat till equilibrium is established. If
at equilibrium, x moles of each of Hz and 12 have reacted, 2x of equilibrium.
moles of HI will be fonned. , (b) Effect of adding substances (Reactants or products):
Thus, the active masses of various reactants and products On adding H2 to the equilibrium mixture, the value of
present at equilibrium are: denominator of equation Kc = 4x 2 / [(a - x)(b - x)] will
increase. To maintain the constant value of K c' the value of
V numerator must also increase. This can happen f[more ofHli~_
[1 ]';' (b-x) molL- 1 fonned, i. e. ,hydrogen combines with iodine to fonn more of HI.
2 Thus, the reaction moves' in a direction in which the added
V
hydrogen is used up. A similar effect is observed when iodine is
2x mol L-1
[HI] added to the equilibrium mixture.
V
The reverse effect is observed when HI is added to ..the
Applying law of mass action,
equilibrium mixture, i. e. , HI decomposes into Hz and 12 ,
'(2X 12 (c) Effect of temperature: The fonnation of HI from
V ) 4x 2 H2 and 12 is an exothermic reaction, i. e., by increasing
Ko [H,][l,] (a~xl(b V xl (a x)(b-x) temperature, the value of equilibriJam constant, K c' decreases.
Thus, the yield of HI decreases, i. e., high temperature is not
favourable for greater yield of HI.
When, a = b = 1, x becomes degree of fonnation of HI and
2 (d) Effect of adding an inert gas: Addition of an inert gas
" K = 4x to equilibrium mixture wij? make no effect on equilibrium state as
',,; c (1 x)2 in the first type of reactions, volume and pressure changes have
The equilibrium constant, K p' can also be calculated no effect on equilibrium state.
considering partial pressures of reactants and products at Case II: Homogeneous gaseous reaction when, Iln > O.
equilibrium. Dissociation of PCl s : The dissociation of PCI 5 takes place
H 2 (g) + I 2 (g) 2HI(g) according to the equation:
Initial no. of moles
No. of moles at equilibrium.
a
(a-x) (b
b
x) °
2x
PCIs(g) ~ PCI 3 (g) '+ CI 2(g)
Let a moles of PCl 5 be taken in a closed vessel of volume V
Total number of moles at equilibrium litre. It is heated and by the time equilibrium is established, x
(a - x) + (b x) + 2x (a + b) moles are dissociated into PCl 3 and C1 2 . One molecule of PCIs
on dissociation gives one molecule ofPC1 3 and one molecule of
Let the total pressure of the system at equilibrium be P C1 2 . Thus, x moles ofPCl 5 will give x moles of PC1 3 and x moles
atmosphere. of C1 2 •
(a-x) At equilibrium,
Partial pressure ofH 2 , PH 2 = (a~b) .p
. (a - x) x x
[PCl s ] = - - ,[PCI 3 ]= ,[CI 2 ]=-
(b - x) V V V
Partial pressure ofl 2 , P12 -- (a + b) .
P
Applying law of mass action,
x x
Partial pressure of HI, PHI - -2x
_.p [PCI 3 ][CI 2 ]
-x
(a+ b)
[PCl s ] (a-x) (a-x)V
V
When, a = 1, x becomes degree of dissociation and
(a - x)(b - x)
K ~--­ (units = mol )
c (l-x)V
602 . G.R.B. PHYSICAL CHEMISTRY FOR COMPETITIONS

Calculation of K p: Let the total pressure at equilibrium be If a = land b 3, then,


P. Total number of moles at equilibrium 4x2V Z
=(a x)+x+x=(a+x)

PPC1s
a-
( a+x
xJ .P,' PPC13 (
x ) 'P; PCl
z
__x_.p
The effect of various factors at equilibrium is discussed here.
a+x (a +x)
(a) Effect of pressure: By increasing pressure; the volume
term decreases. The value of x must increase to maintain the
constant value of K c : In other words, the formation of ammonia
(a - x )(a+ x) , increases with increase of pressure.
(b) Effect of concentration: By increasing the
(units = atm) concentration of either nitrogen or hydrogen in the reaction
(a) Effect of pressure: In this case, K c == x 2 I (a - x) V. If mixture, the rate of forward reaction increases, i. e., the fonnation
pressure is increased, the value of' V' decreases, i. e. , the value of of annnonia increases.
denominator decreases. To maintain constant value of K c , x must (c) Effect of temperature: The synthesis of annnonia is an
also decrease. In other words, the dissociation ofPCl s decreases. exothennic reaction. The value of equilibrium constant d~creases ..
Thus, the increase of pressure would suppress the dissociation of with increase of temperature. Thus, high tempeniture is _~=,~_~~. ~.. _
PCIs· f!lvourable for the synthesis of ammonia: .
(b) Effect of concentration: If PCls is added to the Calculation of K p : Let the total pressure at equilibrium be P.
equilibrium mixture, the rate of forward reaction increases, i. e. ,
dissociation of PC Is increases. In case PCl 3 or Cl 2 is added, the Total number of moles at equilibrium
rate of backward reaction increases. This shall decrease the =(a-x)+(b-3x)+2x' (a+b-2x)
dissociation of PCIs . 2x
P NH 3 - xp· --'----"--'- x p.
(c) Effect of temperature: The dissociation of PCIs is an - (a + b - 2x) , (a+b-2x) ,
endothermic reaction. Thus, the value of equilibrium constant
increases with increase of .temperature. It is thus concluded that (b-3x) P
PH = X
dissociation increases with the increase of temperature. 2 (a + b 2x)
(d) Effect of adding an inert gas: When an inert gas is 2
added at constant volume the equilibrium state is not disturbed. 2x P
In case an inert gl\,s is added at constant pressure, the volume [ (a+ b- 2x) ]
increases, To maintain the constant value of K c , x must also 3
(a-x) P (b-)x) p
increase. Thus, the addition of an inert gas at constant pressure
increases the degree of dissociation of PCl s . [
(a+b-2x) J[(a+b-2x). ]
Case III: Homogeneous gaseous reaction when, t,n < O. 4x 2 (a + b 2x)2
Synthesis of ammonia: The formation .of ammonia from
(a - x)(b - 3x)3 p2
nitrogen and hydrogen is represented by the equation:
N 2 (g) +3H 2 (g) ~ 2NH3(g) Case IV: Homogeneous liquid system: Formation of
Let a start be made by mixing' a' moles ofN z and' b' moles of ethyl acetate:
hydrogen in a closed vessel of V litre capacity. By the time The reaction between .alcohol and acid to form ester is an
equilibrium is established 'x' moles of nitrogen have combined example ofhompgeneous equilibrium in liquid system.
with 3xmoles of hydrogen and produced 2xmoles of ammonia. CH3 COOH(l) + C 2 HsOH(l) ~CH3COOC2Hs(l) + H 2 0(l)
'l'b'
At eqUll num, [N ] (a-x) Let a start be made by mixing 'a' moles of an acid and 'b' moles
2
V of an alcohol. By the time equilibrium is established, x moles of .
[H J=(b-3x) ester and 'x' moles of water are formed, i. e., x moles of acid and x
2
V moles of alcohol have been consumed. Let V be the total volume.
2x At equilibrium, the active masses of reactants and products are:
[NH 3 J=--v (a-x) (b-x)
[CH 3 COOH]=,[C zHsOH1== ,
Applying law of mass action, V V
K == [NH 3]2 x x
[CH 3 COOC z Hs ] -and [H 2 0]=-
c [N2 ][H Z ]3 V· V

_ (~r Applying law of mass action,

K == [ester][water]
x x
VV
x-

.(a;X)(b~3Xr
(a - x)(b - 3x)3 c [acid] [a1cohol]
CHEMICAL EQUILIBRIUM 603

:: ::::I_SOME SOLVED EXAMPLES\I::::::


(a - x)(b - x)
Example 11. For the following reaction,
x2
If a = b = 1, then K c = " (K c has no units) H 2 (g)+ I2(g)~ 2HI(g),
. (1- x)2 '
If initially 25 mL ofH 2 and 20 mL ofI 2 are present in a container
This does not involve the volume tenn. Thus, K c is not and at equilibrium 30 mL of HI is formed then calculate
affected by the change of volume. equilibrium constant. [AIPMT (Mains) 2009J
Case V: Heterogeneous equilibria: . Solution: The given reaction is:
Law of mass action can also be applied to the study of H 2(g) +I2(g) ~2HI(g),
equilibria in which the substances are not in the same phase. to 25 mL 20 mL 0
Considering the decomposition of solid NaHC0 3 to produce t eq, 25-x 20-x 2x
solid Na 2C0 3, gaseous CO 2 and H 20. Give, 2x = 30mL :. x = 15mL
2NaHC0 3(s) ~ Na 2C0 3(s) + CO 2(g) + H 20(g) Equilibrium constant may be calculated as,
Applying the law of mass action, K = [HI]2 = [30]2 = 900 = 60
K' = [Na 2C0 3][C0 2 ][H20] c [H 2 ][1 2 ] [10][5] 15
c [NaHC0 3 ]2 ----Example 12. In the reaction,
It involves two pure solids, N a 2 C0 3 and NaHC0 3. It is H2 (g)+ 12 (g)~ 2HI(g)
customary not to include the concentrations of pure solids in the amounts of H 2 ,12 and HI are 0.2 g, 9.2525 g and 44.8 g
equilibrium expressions. respectively at equilibrium at a certain temperature. Calculate
the equilibrium constant of the reaction.
-<!' K;[NaHC0 3]2 = [C0 ][H 0] Solution: H 2(g) +1 2(g) ~2HI(g),
2 2
[Na 2 C03]
Applying law of mass action,
or Kc = [C0 2][H 20] K = [Hl]2
In tenns of partial pressures we have K p = Pco 2 X PH 2 0' c [H2 HI 2 ]
Similarly, in reactions in which a reactant or product occurs as a Let the total volume be V litre, then
pure liquid phase, the concentration of that substance in the pure [H ] = 0.2 =~ . [I ] = 9.2525 = 0.0364
liquid is also constant. As a result, the concentrations of pure 2 2V V' 2 254V V
solid and pure liquid phases do not appear in the equilibrium
44.8
[HI] = - 0.35
- = - - mo
1L- 1 respectIve
. 1
y
constant expression.
128V V
Many other examples of heterogeneous equilibria are
available. Such as:
1. Decomposition of CaC0 3 ' (~v·35)2
= 0.35 x 0.35 = 33.65
, CaC0 3(s) ~ CaO(s) + CO 2(g)
So, 'K c =
0.1 0.0364 0.1 x 0.0364
-x--
Kc =[C0 2 ] or Kp = Pco 2 V V
When CaC0 3 is heated in a closed vessel at a definite Example 13. 0.5 mole of hydrogen and 0.5 mole of iodine
temperature, the' pressure or concentration of CO 2 produced react in a 10 litre evacuated vessel at 448°C; hydrogen iodide is
becomes constant irrespective of the amount of CaC0 3 taken. formed. The eqUilibrium constant, K c for the reaction is 50.
2. Reaction of steam on heated iron, (a) Calculate the 1,lumber of moles of iodine which remain
~Fe(s) + 4H 20(g) ~ Fe304(s) + 4H 2(g) unreacted at equilibrium .
(b) What is the value of K p?
. K = [H 2 ]4 orK'= PH 2 =4fj(
c 4 P V"' p Solution: (a) H2(g) + 12(g) ~ 2HI(g)
[H 2 0] PH 2 0
Initial moles 0.5 0.5 0
3. Reaction of steam on heated carbon,. No.ofmolesatequilibrium (0.5-x) (0.5-x) 2x
C(s) + H 20(g) ~ CO (g) + H2(g) 2
K = 4x = 50
K = [CO][H2] or K = Pco PH 2
X c (0.5-x)(0.5-x)
c [H 2 0]' P PH 0
2 2x =Fo= 7.07
4. Dissociation of ammonium carbamate, (0.5 - x)
NH 2COONH 4(s) ~ 2NH 3(g) + CO 2(g) 2x = 0.5 x 7.07 - 7.07x
Kc = [NH 3 ]2[C0 2 ]; Kp = [PNH 3]2 X[ PC0 2] or 2x + 7.07x = 0.5 x 7.07
x= 0.5x 7.07 = 039
, . mo e
1
9.07
604 G.R.B. PHYSICAL CHEMISTRY FOR COMPETITIONS

(b) No. of moles OfI2 at equilibrium == (0.5 . 0.39) 0.11 mole Example 16. The equilibrium constant of the reaction,
Kp ==Kc(RT ),"n A 2 (g)+B 2 (g) 2AB(g)
As An 0 , K p == K c 50 at 100°C is 50. If a one litrejlask containing one mole of A2 is
Example 14. 25 mL of H 2 and 18 mL of I 2 vapours were connected to a two litre jlask containing two moles of B 2 • how
heated in a sealed glass tube at 465°C and at equilibrium 30.8 mL many moles ofAB will be formed at 373 K?
of HI was formed. Calculate the percentage degree of Solution: . A2(g)+ B2(g) 2AB(g)
dissociation ofHI at 465°C. Initial no. of moles I 2 o
Solution: Since, the number of moles in a gas under the No. of moles at equilibrium ~ 1 x) (2 x) 2x
same conditions of temperature and pressure are proportional to (Total volume ~ 3 litre)
volumes (Avogadro's Law), the volumes in mL of gases may be (I-x) (2-x) 2x
used instead of concentrations in the case of reversible reactions Active masses
3 3 3
in which there is no change in the number of molecules of Applying law of mass action,
reactants and products.
Given, 2x 30.8mL [ABf ( 3 )2
2x

Kc == -,-----'-,,---:;.,-----,-
So, x== 15.4 mL
Vol. of H2 at equilibrium =(25.;.;; 15.4)= 9.6 mL
z
[A ][B 21 C3 ~)(~; x) . (l-x)(2-x)
VoL OfI2 at equilibrium (I8 -15.4) 2.6 mL 4x 2
But, ----=50
[HI] 2 (30.8)Z (1 x)(2-x)
Kc 38.0 or 4x 2 = (2 - 3x + x 2 )50
[H2 ][I z ] 9.6 x 2.6
or 2x 2 = (2 3x + XZ )25
Consider the equation, 2
or 23x
2HI(g) ~ H 2 (g) + 12 (g)
Let the degree of dissociation be x 75±
2 x= - - ' - - - - - - - -
K' == _ _
x __ 2x 2.3
c 4(1- x = 2.317 or 0.934
The value of x cannot be more than 1, i. e., greater than the
x
or number of moles of A2 and hence x = 0.943.
38 6.1644 2(1 x)
No. of moles of AB = 2x (2 x 0.934)
or x = 0.245, i.e.• 24.5% dissociated
1.868
Example 15. 3.2 mole of HI were heated in a sealed bulb
at 444°C till the equilibrium state was reached. Its degree of Example 17. The vapour density of N Z0 4 at a certain
dissociation was found to be 20%. Calculate the number of temperature is 30. CalCJ,llate the percentage dissociation of
moles of hydrogen iodide, hydrogen and iodine present at the N 204 at this temperature.
equilibrium point and determine the equilibrium constant. Solution: N 2 0 4 (g) 2NO z(g)
Solution: The dissociation of HI is represented by the equation,'
Mol. mass of N20~ == (28 + 64) =92
2Hl(g) H 2 (g) +I2(g)
(I-x) x/2 x/2 Vapour density, D = 92 = 46
Degree of dissociation; x == 0.20 and initial concentration of 2
HI, a == 3.2 mole. Let the degree of dissociation be x
At equilibrium, Given, d = 30
No. of moles of HI a(l-x) 3.2xO.8 2.56 . Applying the relationship,
a·x
No. of moles of H2 3.2x 0.1 0.32 x= D- d = (46- 30) = 16 0.533
2
d 30 30
a·x
No. of moles of 12 == - == 3.2 x 0.1 == 0.32
. 2 Degree of dissociation 53.3%
2
K = x . == 0.2xO.2 =00156 Example 18. 3 g mole of phosphorus pentachloride is
c 2 .
4(I-x) 4x 0.8x 0.8 heated in a flask of 4 litre volume. At equilibrium it dissociates to
K = [Hz ][1 2 ] == 0.32 x 0.32 give 40% ofphosphorus trichloride and chlorine. Calculate the
Also, equilibrium constant. . -.
c [HI]2 2.56x 2.56
0.0156
CHEMICAL EQUILIBRIUM 605

Solution: Degree of dissociation =: 0.4 ~xample 20. 1.0 mole of nitrogen and 3.0 moles of PCl 5
PCl 5 (g) ~ PCI 3 (g) + Cl 2 (g) are placed in 100 litre vessel heated to 227°C. The equilibrium
a(I x) ax ax (at equilibrium) pressure is 2.05 atm. Assuming ideal behaviour, calculate the
degree of dissociation for PCl s and K p for the reaction,
Given, a =: 3, x =: 0.4 and V = 4.
PCl s (g ) ~ PCl 3 (g) + Cl2 (g)
So, at equilibrium,
Solution: PCls (g ) ~ PC1 3 (g) + Ci l (g)
[PCI
5
] = 3(1 0.4) = 3 x 0.6 mol L-1 Initial 3 0 0
4 4 At equilibrium 3(l-x) 3x 3x
[PCI ] =: 3 x 0.4 mol L-1 (x = degree of dissociation)
3
4
Total moles = 3(1 x) + 3x + 3x = 3(1 + x)
[CI 2 ] = 3 x 0.4 mol L-1
.4 1 mole of nitrogen is present, hence actual total number of
Applying law of mass action, moles at equilibrium 3(1 + x) + 1
According to gas equation,
. K = [PCI 3 ][Cl z] 3 x 0.4 x 3 x 0.4 = 0.2 mol L-1
c [PCIs] 4x 3x 0.6 PV=nRT
Given, P =: 2.05 atm, V = IOOlitres, R = 0.082
Example 19. N 204 is 25% dissociated at 37°C and one
atmospheric pressure. Calculate (i) K p and (ii) the percentage and T = (273 + 227) 500 K
dissociation at 0.1 atmosphere and 37°C. n 2.05x 100 5
So,
Solutibn: (i) N Z0 4 (g) ..--2NOZ(g) 0.082x 500
Initial 1 0 or 3(l+x)+ 1= 5
At equilibrium (I-x) 2x 3x= 1 or X= 0.333
Total moles = (1 x) + 2x (l + x) Thus, 3(1 + x) + 1=: 3 + 3 x 0.333 + 1 4 + 0:999

-_2x . p At equilibrium, p = 3(1- x) x 2.05


PCl s (3x+4)
(1+ x)
3x
Given, x =: 0.25 and P=:I atm PPCJ 3 =: PCI
z := X 2.05
(3x + 4)

PN20 = [1- 0.25J x 1= 0.6 atm 3x 3x


- - - x 2.05
4 1+ 0.25

2X 0.25'jx 1 O.4atm
( 1+ 0.25
Z
9x x 2.05 3 X (0.333)2 x 2.05
0.4 x 0.4
0.267 atm 3(3x+ 4)(l-x) (4 + 0.999)(0.667)
0.6
= 0.204 atm
(ii) Let the degree of dissociation of N Z0 4 at 0.1 atm be 'a.',
. then, Example21. For a gas reaction, 3Hz (g).+N 2 (g)
2NH 3 (g), th~ partial pressures of H 2 and N 2 are 0.4 and 0.8
PN 0 = (~J x 0.1 and PNO =: 2a. x 0.1 atmosphere, respectively. The total pressure of the entire system
2 4 1+ a. 2 (1 + a. )
is 2.8 atmosphere.. What will be the value of K p' if all the
concentrations are given in atmospheres?
2a.J2 X (0.1)2 Solution: i'~1 (g) + 3Hz (g) ~ 2NH3 (g)
( 1+ a. 4a. 2 x 0.1 0.4a. 2
Kp Partial pressures 0.8 0.4 [2.8 - (0.8 + 0.4) 1.. 6]

( I +a.
~J' x 0.1 (1- a. )(1 + a.) (1- a. 2) at equilibrium
Applying law of mass action,
1
0.267 0.4a. or 0.267 =: 0.667a. 2 K = (PNH3 )2 1.6 x 1.6
or p 3
(1-0.. 2 ) .. ( PN 2)(PH 2 ) 0.8 x 0.4 x 0.4 x 0.4
a. = 0.632 50atm-2
Hence, dissociation ofN z0 4 63.2%
'606 I " G.R.B. PHYSICAL CHEMISTRY FOR COMPETITIONS

Example 22. One mole of nitrogen and three moles of


hydrogen are mtied in a 4 litre container. If 0.25 per cent of If the observed pressure at equilibrium is 1.12 atm at 380 K, what
nitrogen is converted to ammonia by the following reaction: , is the equilibrium constant K p ofthe reaction?
N 2 (g ) + 3H 2 (g) 2NH3 (g)
calculate the equilibrium constant (K c ) in coy/centration units.
Solution: NH 4 HS(s) ~NH3(g) + H2 S(g)
PNH3 ";,, PH2S == ~ x 1.12
What will be the value ofK for the following equilibrium ? .
2
=0.56 atm
1 3
2 N 2 (g) + 2 H 2 (g) NH 3 (g)
K p = PNH3 X PH2S 0.56 x 0.56 == OJ 136
Solution: N 2 (g) + 3H 2 (g)
---'>.
2NH:i(g) Example 25. A vessel at 1000 K contains carbon dioxide
with a pressure of 0.5 atm. Some of the carbon dioxide is
At equilibrium (1 x) (3 3x) 2x(x == 0.0025) converted to carbon monoxide on addition ofgraphite. Calculate
(1- 0.0025) (0.0050) the value ofK p iftota/pressure at equilibrium is 0.8 atm.
Active masses
4 4 4 (IIT 1993)
Applying law of mass action,
Solution: cO 2 (g) + C(s) 2CO(g)
Initial OSatm Oatm
At equilibrium (05 -x )atm 2x atm
At equilibrium, the total pressure is 0.8 atm.
Ptola! Peo z + Peo
0.8 x) + 2x:::: 0.5 + x
(0.5
or OJ atm X"-"

Applying law of mass action,


K for the reaction,
~N2(g)+ 3 H2 (g)---'>. NH3 (g) K :::: (Peo)2 (2 x 03)2 = 036 18atm
2 2 , p p e02 02 02
is equal to .,JK:;.
K Example 26. A sample ofCaC03 (s) is introduced into a
sealed container of volume 0.654 litre and heated to 1000 K until
== 3.86 X 10-3 litre mol- l equilibrium is reached. The equilibrium constantfor t~e reaction,
Example 23. In an experiment one mole ofacetic acid and CaC0 3 (s)
one mole of alcohol were allowed to react until equilibrium was' 2
established. The equilibrium mixture was found to contain 2/3 is 3.9 X 10- atm at this temperature. Calculate the mass ofCaO
mole ofester. Calculate the equilibrium constant of tbe reaction. present at equilibrium.
Solution': Solution: CaC0 3 (s) CaO(s) + CO 2 (g)

CH 3COOH(l) + C 2 H50H(l) CR 3COOC 2 H5(l) + H 2 °(l) Kp Peo2


Initial no.
1. 0 O. Let the number of moles of CO 2 be.formed n
of moles
xV 3.9 x 10-2 x 0.654
No. of n=-,- - ' = - - -
moles at
,equilibrium \
r ~}
1
(1 ~) 2
3
2
3
RT 0.082 x 1000
3.llx 10-4 mol
-Active 1 2 2 The amount of CaO(s) formed will also be 3.11 x 10-4 mol
-
masses 3V 3V 3V 3V . 4
Hence, mass of CaO formed 3.11 x 10- x 56:::: 0.0174 g
V is the total volume in litres.
Example 27. The decomposition ofammonium carbamate
Applying law ofinass action,
at 3<fC is represented as:
[ester] [water]
Kc NH 2COONH4 (s) ~ 2NH3 (g) + CO2 (g)
[acidHakohol] The equilibrium constant K p is 2.9 X 10-5 attn 3 . .
2
x
2 What is the total pressure of gases in equilibrium with
3V 3V NH 2COONH4 (s) at 30°C?
4 Solution: NH 2 COONH 4(s) 2NH3(g) + CO 2(g)
1 1
x
3V 3V At equilibrium (1- x) 2x x
Kp (PNH 3 i X Peo2
Example 24. Ammonium hydrogen sulphide dissociates
according to the equation: 2.9 x 10-5 :::: (2x)2 X X =4x 3
CHEMICAL EQUILIBRIUM 607

x [2.9XIO-
4
S
f
/
3
0.0194atm
8.
Since, t!.n 0, hence, Kp == Kc]
A (g ) + 3B (g ) ~ 4C (g 1 initial concentration of A is equal
to that of B. The equilibrium concentrations of A and Care
Total pressure 2x + x 3x 3 x 0.0194 0.0582 atm equaLKc of the reaction will be:
Example 28. In an experiment starting with 1mole ofethyl (a) 0.08 (b) 0.8 (c) 8 (d) 80
alcohol, 1mole of acetic acid and 1 mole of water at IOCPC, the (e) 1/8
equilibrium mixture on analysis shows that 54.3% of the acid is [Ans. (e)]
esterified. Calculate the equilibrium constant of this reaction. [Hint: A(g) + 3B(g) ~ 4C(g)
Solution: t 0 a a 0
CH 3 COOH(l) + C 2 Hs OH(l) ~ CH 3 COOC 2 H5 (l) + H 2 0(l) teq. a x a 3x 4x
Initial 1 1 0 1 a - x == 4x given
At Ix I-x x l+x a 5x
equilibrium
1 0.543 K == [C]4 [4X]4 256 = 8]
1- 0.543 0.543 1 +0.543
c, [A][B]3 [4x][2x]3 32
Given, x 0.543.
Applying law of mass action, 9. For a hypothetical reaction:
[ester] [water] 0.543 x 1.543 = 4.0 4A(g) + 5B(g)...- 4P(g) + 6Q(g)
Kc The equilibrium constant K c has units:
[acid][alcohol] 0.4571< 0.457
(a) mol L -1 (b) mol- 1 L
Example 29. The equilibrium constant for the reaction,
(c) (molL -1)-2 (d) unitless
CH3 COOH(l) + C2H50H(l)~CH3COOC2Hs (l) + Hp(l)' [Ans. (a)]
is 4. What will be the composition of the equilibrium mixture
[Hint: Unit of Kc (mol L-1 )illl
when one mole ofacetic acid is taken along with 4 moles ofethyl
alcohol? = (mol L- 1 i = mol L- 1 ]
Solution: . 10. Starting with 1 mol of 02; 2 mol of S02' the equilibrium for
CH 3COOH(l) + C 2 lL;OH(l) ~ CH 3COOC2 lL;(I) + H 2 °(l) the formation of 803 (g) was established at a certain
Initial 1 4 0 0 temperature. If V is the volume ,of the vessel and 2x is the
number of moles of S03 present, the equilibrium constant will
At equilibrium 1- x 4 x x x
be:
[esterJ[water] x2 2 2
Kc 4 (a) x V (b) 4x
[acid] [akohol] (l-x)(4-x) (1- (2-x)(l-x)
x
2
=4(1- x)( 4 x) 2
(c) -"--.:...... (d) x
4[45x+x2] 2V (2-x)(1-x)
2
= 16 2Ox+ 4x [Ans. (a)]
or 3x 2 20x + 16 = 0 [Hint: 2S0 2 (g) + 02(g) ~ 2S0 3(g)
20±
to '2 1 0
x =-----''---.-- =- - - -
6 6
20± 14.42
'I eq, [2/X] [l;X] [~]
x= 0.93 or 5.7366 2
The value 5.7366 is not possible, he~ce x = 0.93 ,K = [S0 3 f = /V == xV ]
Thus, the composition of mixture at equilibrium is •. c [SOd[Oz] [2 V2x J [1/ J (1 x)3
[CH 3 COOH] = (1 0.93) = 0.07 mole
[C 2 Hs OH] = (4 - 0,93),= 3.07 mole 11. For the reaction, H 2 (g) + 12(g) " . ' 2ID(g), K =47.6, if the
[CH3 COOC 2Rs] 0.93 .mole = initial number of moles of each reactant and product is 1 mole,
[H 2 0] =0.93 mole then at equilibrium: [JEE (Orissa) 20061
(a) [1 2 ]=[H2 J;[I2 ]>[ID] (b) [1 2 ] < [H2 ];[I 2 ] [ID]
~'OF08JECTIVEQUE$TIONS (c) [1 2 ] [H2 ];[I 2] < [ID] (d) [1 2 ] > [Hz];[I21 =[ID]
[Ans. (e)]
7. For the reaction, N 2(g) + 02(g) ~ 2NO(g), the value of
Kc at ;:;JO°C is O.L When the equilibrium concentrations of
both the ,_:!c~'ints is 0.5 mol, what is the value of Kp at the
same temperature'? . (KCET 200S) [H 2 ] '" [1 2 ] will be same at equilibrium, since. it is same at initial
(a) 0.5 (b),O.1 (c) 0.01 Cd) 0.Q25 . state.
.[Ans. (b)] [HI] 2
Kc
[Hint: Kp = KARTi'"
608 G ..R,B. PHYSICAL CHEMISTRY FOR COMPETITIONS

Kc [1 2 ] 2 =[Hl1 2
47.6 [12 ] 2 [Ill] 2
.. [12] < [Ill]]
12. In a closed vessel of volume V, a mol of nitrogen and b mol of
oxygen are made to react to give nitric. oxide, according to the
reaction:
2x
N 2 (g) + O 2 (g) ~ 2NO(g) I-x
If at equilibrium, 2xmol of NO are obtained, then: x 0.33
.[BV (Pune) 2006J
2 % ofH2 reacted = 33 ]
4x I
(a)K - x-
c - (a - x)(b x) V
4x2 . 9.8 LE CHATELIER'S PRINCIPLE
-----xV
(a -x)(b-x) There are three main factors which can change the state of
X
2
. equilibrium in a reversible system. These are concentration,
(c) Kc = xV pressure and temperature. Le Chatelier, a French chemist,
(a-x)(b-x)
. presented a quaJitative principle known as Le Chatelier's

(d) Kc ~ (a _:~: x)
principle which can . describe theeffect..... ofchangein---
concentration, pressure and temperature on any reversible system
[Ans. (d)] whether physical or chemical.
[Hint: N 2 (g) + 02(g)~ 2NO(g) According to this principle, if a system at equilibrium is
t= 0 abO subjected to a 'change of concentration, pressure or
b-xi 2x temperature, the equilibrium shifts in the direction that tends
V ( V .J V to undo the effect of the change.
Or
4x 2 If a system at equilibrium is subjected to a change of any
----] . one of the factors su,-:h as concentration, pressure or
(a - x)(b -x)
temperature, the system adjusts itself in such a way as to
aunul the effect of that change.
13. Equivalent amounts of H2 and 12 are heated in a closed till
(a) Change in concentration: If an additional amount of
equilibrium is obtained. If 80% of the hydrogen, can be any reactant or product is added to the system, the stress is
converted to HI, the K c at this temperature is: (V IT E E.E 2Oil7) relieved as the reaction that consumes the added substance occurs
(a) 64 (b) 16 (c) 0.25 (d) 4 more rapidly than its reverse reaction, i. e. , if any of the reactants
[Ans. (a)] .
is added, the rate of forward reaction increases and if any of the
[Hint: H 2 (g) + I 2 (g) ~ 21ll(g) products is added, the rate of backward reaction increases.
t= 0 110
In general, in. a chemical equilibrium, increasing the
1 - 0.8 1 - 0.8 2 x 0.8
. teq. concentrations of the reactants results in shifting the equilibrium
V VV
in favour of products while increasing concentrations of the
[
1.6 J2 products results in shifting the equilibrium in favour of reactants.
K = [Illf = V = 64] (b) Change in pressure: If a system in equilibrium consists
c[H 2][I2] 0.2 x 0.2 of gases, then the concentrations of all the components can be
V V altered by changing the pressure. When the pressure on the
14. Equimolar concentration of Hz and 12 are heated. to system is increased, the volume decreases proportionately. The
equilibrium in a 2 litre flask. At equilibrium, the forward and t;)ta\ 11lJ.mber of moles per unit volume will now be more thari
backward rate constants are found to be equal. What beforelAccording to Le Chatelier's principle, the equilibrium
percentage of initial concentration of Hz has reacted at will shift in the direction in which there is decrease in number of
. equilibrium? WMT C';;:n!ia) ZUOl1j moles, i. e., towards the direction in which there is decrease in
(a) 33% (b) 66% volume.
(c) 50% (d) 40% In general, an increase in pressure applied to a system at
(e) 20% equilibrium, favours the reaction in the direction that
[Ans. (a)] produces smaller number of moles of gases and a decrease in
[Hint: H 2 (g) + 12 (g) ~ 2Hl(g) pressure favours the opposite reaction. If there is no change in
to 0 number of moles of gases in a reaction, a pressure change does
"l-x 1 x 2x not affect the eqUilibrium. .
teq. -
2 2 2 (c) Change in temperature: A chemical reaction
(reversible) involves two opposing reactions, forward and
CHEMICAL EQUILIBRIUM 609
backward. If one reaction is endothermic, the other will be absorption of heat. The formation of HI is an exothermic
exothermic in nature. When heat energy is added by raising reaction. Thus, the backward reaction moves faster when
temperature, the system can relieve itself from the stress if the temperature is increaSed.
reaction which, absorbs heat moves faster, i. e., endothermic In short, we can say that the favourable conditions for greater
reaction is always favoured with increase of temperature. yield ofE1 are: .
Conclusions: (i) High concentrations ofH2 and 12 and
.. '
(ii) Low tempenltur~ .
(i) Increase in concentration of any substance favours the
(b) formation of qitric oxide:
reaction in which it is used up.
(ii) High pressure is favourable for the reaction in which there
N 2(g) + O 2 (g) , ;. 2NO(g) - 43200 cal
is decrease in volume. Effect of concentration:' ' When concentrationofN2 or 02
(iii) A rise in temperature favours the endothermic reaction.
is increasetl, the system moves in a direction in which N 2 or 0 3 is·,"
used up, i. e., rate of forward reaction increases thereby
(d) Role of catalyst: Positive catalyst increases the rate of
increasing the concentration of NO·. . . ',.
both forward and backward reactions equally; the equilibrium
will be attained in less time, i. e. , same amount of pre juct will be Effect of pressure: In the formation: of nitric oxide, the
formed in less time. number of moles remains the sam\l, i. e. , no c:hange in volume
On the other hand, negative catalyst lowers the rate of both occurs. Consequently, the equilibrium state is not affected by any
forward and backward reactions equally. Same amount of change in pressure.
product will be formed in more time. Effect of temperature: The formation of NO is endother-
Catalyst does not affect equilibrium cOnstant and heat of reaction. mic in nature. If the temperature is raised, the equilibrium shifts
in the direction in which heat is absorbed. The concentration of
Summary of LeChatelier's Principle NO will, therefore, be higher at higher temperature.
Effect of inert gas addition Thus, favourable conditions;!or. gre,ater yield of nitric oxide are:
r'-,-~·",·''-:----'''''''----'------cI----'''''''-- (i) High concentrations ofN2 -an.d O2 and
'COndition . Effect
! ,"
<,,'
JLi'v-o~'V'=Coristant
(ii) High temperature
II tJ.n= 0, +- veor -ve No effect (c) Dissociation of pels:
I tJ.V '* 0, V '* Constant
1 tJ.n = 0 No effect
PCls (g) ~ PCl 3 (g) + 'cl 2 (g) -15000 cal
i tJ.V '* 0, V '* Constant 'I tJ.n> 0 Forward shift
I tJ. V '* 0, V '* Constant
r ...._«_'"
,tJ.n < 0 Backward shift Effect of concentration: When 'concentration of PCls is
increased at equilibrium, the rate of forward reaction increases as
where, An number of gaseous moles of product - number of
to decrease the added concentration. Thus, more ofPCl 3 and Cl 2
gaseous moles of reactant are formed., ,','
Effect of temperato,re and pressure Effecf,<:cof ,.pressure: The volume increases in the
Effect of Air, Side with " Effed of " dissochition of PCls . When preS!,lure is lrn;.reased, the system
~!'Iature of reaction' increased fewer mole of increase in P moves in the directiOn in which 'there is decrease in volume.
"' temperature .gas. Thus, ·high pressure does not favour disso()iation ofPC Is.
1. Exothennic backward shift 0; neither no shift Effect of temperature: The dissociation of PCIs is an
" " 2. 'Exothermic bac:cwardshift -ve; right forward shift , endothermic reaction. Thus, increase of temperature favours the
~::. ' 3: Et;ldothe~c forward shift -ve; right forward shift dissociation.
4. Endothermic forward shift +ve; left backward shift , Thus, favourable conditions for dissociation of PCIs are:
5. Exothermic backward shift +ve; left backward shift
(i) Higl, concentration of PCl s ,
6. Exothermic backward shift ..eve; right forward shift
7, Endothennic forward shift +ve; left backward shift
(ii) Low pressure and
8. Endothermic forward shift +ve; left backward shift (iii) High temperature.
(d) Formation of ammonia:
Application of Le Chatelier's Principle N2 (g) + ?H2 (g) 2NH3 (g); IlH =- 22.4 kcall mol
(a) Formation ofOI:
(i) At high pressure reaction will shift in forward direction
H 2 (g) +I2(g) ~ 2HI(g) +3000 cal
to form more product.
Effect ofconcentriltion: When concentration ofH2 orI2 is
(ii) When concentration of N2 and H2 is raised or
increas~d at eq~iiibriu:m,;,the ~ystem moves in a direction which concentration " of NH3 is lowered, then again the
decreases the, concentration, i. e., the rate of forward reaction equilibrium shifts in forward direction to form more
increases thereby increasing the'concentration of81. ammonia.
Effect of pressure: As there is no;'Change in the number or If concentration of only one reactant is raised, then again
moles in the reaction, the equilim.1um state remains unaffected by the equilibrium shifts in forward direction till the other
change of pressure. reactant is available for reaction.
Effect of temperature: By increasing temperature, the
equilibrium state shifts towards the reaction which -moves with
I
610 R.B. PHYSICAL CHEMISTRY FOR COMPETITIONS

(iii) The reaction shifts in forward direction at low temperature. [Hint: Reaction is endothennic and t:.n> 0, hence, the
But at very low temperature the rate of reaction becomes fonnation of product will be favoured by increasing the
very low; thus moderate temperature is favourable for temperature and decreasing the pressure.]
this reaction. 16. The exothermic fonnation of ClF 3 is .represented by the
equation:
9.9 APPLICATION OF LE CHATELIER'S CI 2 (g) + 3F2(g) 2ClF)(g); I1H = - 329 kJ
Which of the following will increase the quantity ofClF 3 in an
PRINCIPLE TO PHYSICAL EQUILIBRIA
equilibrium mixture ofCl z, F2 and ClF)? 1,\1) '::. :;::d:,,
(i) Solid Liquid ~ Gas (a) Increasing the temperature
(b) Removing Cl 2
When temperature is raised, the above equilibrium shifts in
(c) Increasing the volume of the container
forward direction.
(d) Adding F2
(ii) Effect of pressure on boiling point: When pressure is [AIlS. (d)]
raised, condensation of vapour takes place. Thus, vapour [Hint: Equilibrium will shift in forward direction by increasing
pressure is lowered. Now more heat is required to equate vapour he concentration of reactant.]
pressure with atmospheric pressur:e as a resuit of which boiling
17. Which among the following reactions will be favouredat low
point increases. .
pressure?
(iii) Effect of pressut:'e on melting point: There are two ta) N 2 (g)+
. types of solids: b) H 2 (g) +
(a) Solids whose volume decreases' on melting, e. g., ice, tc) PCls(g)
diamond, carborundum, magnesium nitride and quartz. (d) Nig) + 3H2 (g)
Solid (higher volume) ~ Liquid (lower volume) [Ans. (c)]
[Hint: On lowering the pressure, equilibrium favours the
The process of melting is facilitated at high pressure, thus direction of higher volume.]
melting point is lowered. 18. At constant pressure, the addition of argon to:
(b) Solids whose volume increases on melting, e.g., Fe, Cu, N2(g) + 3H2 (g) ~ 2NH3 (g)
Ag, Au, etc. will:
Solid (lower volume) ~ Liquid (higher volume) (a) shift the equilibrium in forward direction
(b) shift the equilibriumin backward direction
In this case the process of melting become difficult at high
pressure; thus melting point becomes high. (c) not affect the equilibrium
(d) stop the reaction
(c) Solubility of substances: When solid substances are
[Ans. (b)]
dissolved in water, either heat is evolved (exothermic) or heat is
[Hint: If volume is not constant, the addition of inert gas
absorbed (endothermic).
favours the direction where volume is increasing.]
KCl + aq. ~ KCl(aq.) - heat 19. The equilibrium reaction that is not influenced by volume
In such cases, solubility increases with increase in change at constant temperature is: ICE'! (.l&~~) 2llU(d
temperature. Consider the case of KOH; when this is dissolved, (a) H2 (g) + 12 (g) 2HI(g)
heat is evolved. (b) N2(g)+3Hz(g) 2NH)(g)
(c) N204(g)~ 2N02 (g)
KOH+ aq. ---'" KOH(aq.) + heal
(d) 2NO(g) + 02(g) 2N0 2 (g)
In such cases, solubility decreases with increase In [Ans. (a)]
temperature. [Hint: When, t:.n 0, there is no effect of pressure and volume
(d) Solubility of gases in liquids: When a gas dissolyes in change on the equilibrium.]
liquid, there is decrease in volume. Thus, increase of pressure 20. N2 (g) + 3H z(g) 2NH3 (g ) + Heat
will favour the dissolution of gas in liquid. What is the effect of the increase of temperature on the
equilibrium of the reaction ? ~ j,( 'V! 2lj(17~)
fUl1mATfONS OF OIJECTIVE QUESTIONS
(a) Equilibrium is unaltered
15. Consider the following reversible reaction at equilibrium: (b) Reaction rate does not change
2B2 0(g) 2H 2(g) + 02(g); I1H = + 24.7 kJ (c) Equilibrium is shifted to the left
which one of the following changes in conditions will lead to (d) Equilibrium is shifted to the right
maximum decomposition ofH2 0(g)? [Ans.(c)] .
(a) Increasing both temperature and pressure [Hint: The given reaction is exothemlic, therefore, the equilibrium
(b) Decreasing temperature a~d increasing pressure will shift in backward direction by the increase of temperature.]
(c) Increasing temperature and decreasing pressure
(d) Increasing temperature at constant pressure
(e) Increasing pressure at constant temperature
[Ans. (c)]
CHEMICAL EQUILIBRIUM 611

9.10 CALCULATION OF DEGREE OF Calculation of Degree of Reaction using


DISSOCIATION FROM DENSITY Pressure-Temperature Determination
MEASUREMENTS Let us consider the following gaseous reaction:
Degree of dissociation is defined as the fraction of one molecule PCIs(g) PCI 3 (g) + CI 2 (g)
dissociated. It is denoted by x or a.. Its value is always less than 1. Initial pressure and temperature are P]and 7i. At i;:quilibrium,
When the value becomes equal to 1,it is said that the substance is pressure and temperature are and Tz respectively. The volume
completely dissociated~ of vessel is constant, equal to 'V'.
The degree of dissociation in the case of second type of
PCIs(g) ~PCI3(g) + Clz(g) 11 V T P
reactions (reversible) is determined by measuring density of
reaction mixture at equilibrium. t 0 a· 0 0 a V T] PI
Consider the general reversible reaction, t eq . a ax ax ax a+ax V Tz
A ~nB Initial state PI V == aRT, ... (i)
Initial no. of moles 1 0 Equilibrium state P2 V:=; (a + ax)RT2 ... (ii)
No. of moles at equilibrium(l x) nx Dividing equation (i) by eq. (ii), we get
Total number of moles at equilibrium = (hi)+ nx Tj
= 1+ (n 1)x.·
P2 (1+ x)TZ
Let the initial volume be V litre. Thus, the volume at equilibrium
=[I+(n-l)x]V.. (l+x)==
Let 'd' be the observed· vapour density at a particular T2 P1
temperature when degree of dissociation is 'x' and D be the . T1P 2 - T2P1
vapour density when there is no dissociation. Again, . X == --'--'.,""",,-----='-'-
.. I· . T2 Pl
Doc ... (i)
v· QUESTIONS
and d ex }.
... (ii)
[1+(n-l)x]V 21. At equilibrium:
N2 0 4 (g) ~ 2N0 2(g)
Dividing eq. (i) by eq. (ii),
the observed molecular weight of N 2 0 4 is 80 g mol- l at
D 350 K. The percentage dissociation of NZ0 4(g) at 350 K is:
l+(n-l)x
d
D (a) 10% (b) 15% (c) 20% (d) 18%
or --l=(n l)x
d (e) 13%
[Ans. (b)]
D-d
or ==(n-l)x [Hint: Degree of dissociation may be calculated as,
d M 111
D-d X= .. n = 2 (number of gas moles .
or x ... (iii) (n - I)m produced by 1 mol reactant)
(n l)d 92- 80
(M 92, m =: 80)
M rn (2 - 1)80
x:=; .
(n -l)rn 12
0.15
80
where, M = initial molecular mass
Percentage dissoeiation 0.15 100 = 15]
rn = molecular mass at equilibrium. 22. The vapour density of PCls is 1·~)4.25 but when heated to
In the dissociation of PCIs, NH4 CI and N 2 0 4 , the value of
230°C, its vapour density is reduced to 62. The degree of
n = f. dissociation of PC Is at this temperature will be:
PCls ~ PCl 3 + Cl 2 (a) 6.8% (b) 68% . (c)46% (d) 64%
[Ans .. (b)] .
NH.}CI~NH3 +HCI
[Hint: The reaction is:
N 2 0 4 ..,-2N0 2
'>.. PCls(g) PCl 3(g) + CI 2 (g)
D d D-d n 2
So, x ---
(2 1) d d D - d 104.25 62
u=--=----
(n - I)d (2 - 1) x 62
0.68
Percentage dissociation 0.68 x 100 =: 68% 1
612 G.R.B. PHYSICAL CHEMISTRY FOR COMPETITIONS

MISCELLANEOUS NUMERICAL EXAMPLES4///';"


Example 1. At a certain temperature, equilibrium Example 3. At 700 K, CO 2 and H 2 react to form co and
constant (K c ) is 16for the reaction: H 2 O. For this process K c is O.II.ifa mixture of0.45 mole ofCO 2
S02(g)+ N02(g)~ S03 (g)+ NO(g) and 0.45 mole ofH 2 is heated to 700 K.
if we take· one mole of each of the four gases in one litre (i) Filld out the amount of each gas at equilibrium state.
container, what would be the equilibrium concentration of NO (if) After equilibrium is reached another 0.34 mole of CO 2
andN0 2 ? and 0.34 mole of H 2 are added to the reaction mixture. Find the
composition of the mixture at the new equilibrium state.
Solution:
Solution: (i) CO 2 (g) + Hz (g) ..-- CO(g) + H20(g)
Initial concentration rial no. of moles 0.45 0.45 0 0
Equilibrium concentration 1 x 1 x 1+ x I+x . of moles at 0.45 x 0.45 x x x
Applying law of mass action, equilibrium
K = UNO]. 16 Applying law of mass action,
c [ S0 2 ][N0 2 ] 2
K = [CO][H 2 0] = xxx = x =
l+x =::4 or 1+,\:=4 ··4x c ~Tr02T[H2J (0.45 x)(0.45-x) (0:45 xl " ..•..,.-------~-
I-x
x
or 5x 3, i.e.,x= 3 0.6 So, 0.33
5 (0.45 x)
Cone. ofN0 2 at equilibrium (1- 0.6) 0.4 mole "
Cone ..of NO at equilibrium = (I + 0.6) =1.6 mole x = 0.11
Example 2. At 817° C, K p for the reaction between At equilibrium, [C0 2 ] = [H 2 ] 0.34 mole
CO 2 (g ) and excess ofhot graphite (s) is 10 atm. [CO] = [H 2 0] = 0.11 mole
(a) What are the equilibrium concentrations of the gases at
. (ii) CO 2 + Hz ~ CO+ H 2 0
817°C and a total pressure of5 atm? Initial moles· 0.34 + 0.34 0.34 +0.34 O.ll 0.11
(b) At what total pressure does the gas contain 5% CO 2 by =0.68 =0.68
volume? tilT 20(0) At equilibrium 0.68 - y 0.68-y 0.11+y O.ll+y
Solution: (a) CO 2 (g) + C(s) ~ 2CO(g); K p 10 atm
K = [CO][H 2 01 (0.11 + y)(0.11 + y) = 0.11
Given: . Pco(g) + Pco 2 (g) = 5 atm
c [C0 2 ][H 2 ] (0.68 y)(0.68 - y)
Let Pco(g) = x atm
So, Peo (g) = (5 x)atm or O.ll + Y = 0.33
2 2
K _ [Peo] x
2 0.68- y
p - Peal (5-x) or 0.11+ y=0.33xO.68 0.33y
or y 0.086
x2
or 10=-- At equilibrium, [CO] = [H 2 0] 0.11 + 0.086 0.196 mole
(5 x) [C0 2 ] = [H 2 ] 0.68 - 0.086 = 0.594 mole
2
or x + 10x- 50 0 . Example 4. A mixture of S03' S02 and 02 gases is
On solving, we get . maintained in a 10 litre flask at a temperature at which the
x= 3.66 equilibrium constant for the reaction is 100 : .
Peo(g) = 3.66 atm
or Mole fraction CO at equilibrium 73.2% (by volume) (i) If the number a/moles of S02 and,S03 in the flask are
Peo 2(g) = 1.34 atm equal, how many moles of02 are present?
or Mole fraction CO 2 at equilibrium = 26.8% (by volume) (ii) if the number of moles of S03 in the flask is twice the
(b) Let the total pressure be P atm number ofmoles 0fS02' how many moles ofoxygen are present?
Given, % CO 2 (by volume) = 5 and % CO (by volume) = 95 Solution: (i) 2S02 (g) + 02(g) ~2S03 (g)
5 ! 95 At equilibrium, let the number of moles of each of S02 an'd
Peo = - x P 0.05Pand Peo = - x P 0.95P S03 be nl and of oxygen n2 , i. e.,
2 100 100
K 10= [Pea]2 0.95P x 0.95P = 18.05P [S02] "" !!.!.
10
p Peo2 0.05P
or P = 0.554 atm
CHEMICAL EQUILIBRIUM 613

(ii) Let the degree of dissociation be a when pressure is 2


[S03] ==
]0 atmospheres.
Applying law of mass action, At equilibrium
(1-0. ) (I-a)
PPCl s :::: (1 + a) . P ---x 2
K, [SO,!," (-\':if· "100 (1+0.)

[S 02]2[02] (~Y(~) _a_. p = a x2


(1+0.) (1+0.)
llO) 10
a a
or n2 0.1 mole PCI == ·P=--x2
2 (1+0.) (1+0.)
Oxygen = 0.1 mole
(ii) Let the number of moles of S02 be == n l . _a_x2x_a_x2
(l + a) (1 + a)
So, number of moles of S03 = 2nl
Let the number of moles of oxygen be n2 (l-a)x2
(1+0.)
2
0.
or ------<2- = 0.89
(I-a )
or 0. 2 == 0.89 0.890. 2

.n2 0.4 mole or 1.890. 2 0.89

"' Oxyg~n 0.4 mole 0.89

Example 5. At 250°C and 1 atmospheric pressure, the 1.89


vapour density ofpels is 57.9. Calculate (i) K p for the reaction, 0.==0.686
PCIs (g)..,.--- PCl3 (g) + el 2 (g), . Thus,PCl s is68.6% dissociated .
at 250°C, (it) the percentage dissociation when pressure is doubled. . ~anipJe 6,. For the reaction,
. CO(g) + 2H2(g) CH30H(g)
Solution: (i) Mol. mass ofPC1 s 208.5
hydrogen gas is introduced into a five litre flask at 327°C,
containing 0.2 mole of co(g) and a catalyst. until the pressure is
Vapour density, D == 208.5:::: 104.25
2 4.92 atmosphere. At this point 0.1 mole ofCH 30H(g) is formed.
Calculate the equilibrium constants K p and (ln 1'1"':;
Observed vapour de~sity. d:::: 57.9 Solution: Let the number of moles of hydrogenintrodl.fCed
'" D-d 10425-57.9 be mmoles. .
Degree 0 fd IssoclatlOn, a :::: - - :::: - - - - - Total moles of CO and hydrogen:::: 0.2 + m
d 57.9
=0.80 Applying, PV = nRT
PCls(g) ~ PCI 3(g) + CI 2 (g} P == 4.92atm, V == 5 litre, R·= 0.082,
At equilibrium 1 a a a
(1- 0.80) 0.80 0.80 T == (273 + 327) 600 K
Total number of moles :::: (1 + a) (l + 0.80):::: 1.80
4.92x 5 0.082x 600 x (0.2+ m)
Partial pressure of PCls == 0.2 XII
. 1.80 9 or 0.2 + m = 4.92 x 5
0.082 x 600
Partial pressure ofPCl 3 :::: 0.80 xl:::: ~
1.80 9 or m= 0.3 mole
CO(g) +
2H 2 (g) ~ CHpH(g)
Partia~ pressure of CI 2 (No. of moles
0.2-x 0.3 - 2x x
or 0.2-0.1 0.3' 0.2 0.1 at equilibrium)
4/9x4/9 16
So, = 1.78 or 0.1 0.1 0.1
1/9 9
0.1 0.1. 0.1
- (Active masses)
5 5 5
614 ' I ...•. R.B. PHYSICAL CHEMISTRY FOR COMPETITIONS

Applying law of mass action, or 5x 2 =4


0.1 2
or x -
_ _-"--_ _ = 2500 mol ~2 L 2
IS
0~1 x (O~lr Hence, number of moles of A formed = 2 times initial

We know that, Kp K c (RT)/:'n,l1n 2 moles of AB taken


Example 9. 0.15 mole of co taken ina 2.5 litre flask is
or Kp = 2500(0.082 x 600)""2 maintained at 705 K along with a catalyst so that the following
reaction can take place:
or K = 2500 = 1.0327 atm- 2
p 49.2x 49.2 COI,g) + 2H2(g) CHPH(g)
Hydrogen is in(roduced until the total pressure ofthe system is
Example 7.. When sulphur in the form of S 8 is heated at
8.5 atm at equilibrium and 0.08 mole of methanol is formed.
900 K the initial pressure of 1 atm falls by 29% at equilibrium.
Calculate (i) K p and K c and (ii) the final pressure if the same
This is because olcol1versiol1 ofsome S 8 to S 2' Find the value of
amount of co and H 2 as before are used, but with no catalyst so
equilibrium constant for this reaction. that the reaction does not take place. : II r t \)1) ,; ,
Solution: Sg(g) 4S 2 (g) Solution: (i) Let the total number of moles of gases be n aL_
equilibrium.
Initial mole 1 o Applying equation, PV nRT
At equilibrium 1 x 4x
Given, P = 8.5 atm; V 2.5 L;
(1- 0.29) (4x 0.29)
R = 0.0821 atm L mol-I K- 1 and T = 705 K
= 0.71 1.16
Applying law of mass action, n= 8.5 x 2.5 0>367 mol
0.0821 x 705 '
K =[ ]4 (1.16)4 = 2.5582 atm 3
P [PSg] 0.71 CO(g) + 2H 2 (g) ---'" CH 3 0H(g)
Initial mole 0.15 (nH 2 )0 0
Example 8. For the equilibrium,
At equilibrium 0.15 x [(nH 2 )0 - 2x] x = 0.08
AB(g)--" A(g)+B(g),
K p is equal tofour times the total pressure. Calculate the number Number of moles of CO at equilibrium
of moles of A f o r m e d . · (0.15 0.08) = 0.07 mole
Solution~ Let the total equilibrium pressure be = P atm Number of moles ofH2 at equilibrium
Given, . K I' = 4P total moles moles of CO - moles of CH 3 OH
Let the start be made with 1 mole of AB(g ) and the degree of
= (0.367 0.07 0.08)
dissociation be x.
AB(g) A(i) + B(g) = 0.2 17 mole
At equilibrium 1. x x x Applying law of mass action,
0.08
Total moles at equilibrium = 1- x + x + x I + x
. x [CH 3 0H] _ 2.5 151.6mol-2 L2
Thus, Partial pressure of A = - - . P [CO] [H 2 ]2 - 0.07 X l( 0.217)2
l+x
2.5 2.5
. x
PB = Partial pressure of B = - - . P
l+x Now, K p = K c (RT )/:,"151.6 x (0.0821 x 705)-2

I-x
_.p 0.045atm-2
P AB Partial pressure of AB
l+x (ii) SinGe, [(I1 H7 )0 2x] 0.217
Applying the law of mass action, or - (nH 2 )0=0.217+2xO.08
= 0.377 mole
(/leoh 0.15
Total moles (flO) 0.377 + 0.15 0.527
170RT 0.527 x 0.0821 x 705
Rence, Po =-V-= 2.5
2 == 12.20atm
So, 4P -x_ . p
I x
2 Example 10. The equilibrium constant for the reaction,
2 2
or 4 4x =x CO(g) + H 20(g)---" CO 2 (g) + H2 (g),
I
CHEMICAL EQUILIBRIUM 615

at 986°C is 0.63. A mixture oflOlnole olwater vapour and 3.0 POl 2 + 0.0115 = 2.0115 atm
mole of co is allowed to come to equilibrium. The equilibrium
pressure is 2.0 atm.
PS03 =1 2 x 0.0115 = 0.977 atm
(i) How many moles of H 2 are present al eqllilibrium? Example 12. What is the concentration of co in
(ii) Calculate the partial pressures of gases ill equilibrium equilibrium at 25°C in a sample of a gas originally containing
mixture. : l i ~ ']'::; l.OOmoi of CO 2 ? For the dissociation olC02 at 25°C,
Solution: (i) In the given reaction, fj,n 0, hence, K C' K p Kc 2.96 X 10-92 .
CO(g) + H 1 0(g) ~ CO 2 (g) + H 2 (g) Solution: 2C0 2 (g) ~ 2CO(g) + 02 (g)
At equilibrium (3 x) (1 x) x x Atequilibrium (\ 2x) (2x) (x)
Applying law of mass action, Applying law of mass action,
__x_x_x__ = 0.63 2
K:: :1[COI x x (2x-f~ =? 96 x 10-92
(3 x)(I- x) c _.
. [C0 2 (1- 2yt
x2 0.63(3 4x + x 1 )
It can be assumed that 1 2x '" 1.0 as K c is very small.
2
0.37x + 2.52x - l.89 = 0 So, 4x 3 = 2.96 x 10-92
or + 6.81x 5.1 0
or X l.95xlO- 31 mol L- 1 .

x = .-6.81±
.- - - - = - - -x6.81-4x(-5.
- - - - - - - - ' - ' ' - - - = 0.68 mole Hz [CO] = 2x 2 x 1.95 X 10 31
2
3.90x 10-31 mol L- 1
(ii) Partial pressure of CO 2 = partial pressure of H2
Example 13. Ammonia is heated at 15 atmfrom 27°Cjo
= mole fraetion x total pressure
347°C assuming volume conslallt. Tile new pressure becomes 50
0.68 x 2 =03
=-- . 4 atm atm at equilibrium. Calculate % ol NH 3 actually decomposed.
4
Partial pressure of CO(g) = (3 0.68) x 2 = 1.16 atm Solution: Pressure of NH 3 at 27°C == IS atm
4 Pressure of NH3 at 347 0 C P atm
Partial pressure of H" O(g) = (1- 0.68) x 2 = 0.16 atm P 15
- 4
300 620
Example 11. The equilibrium constant K p for the 31 atm P
reaction, Let a moles of ammonia be present. Total pressure at
2S0 2 (g ) + 02 (g) 2S0 3 (g) equilibrium = 50 atm
is 900atm- 1 at 800K. A mixture containing S03 and 02 having 2NH 3 (g) ~ Nz(g) + 3H 2 (g)
initial partial pressure of I and 2 atm respectively is heated at At equilibrium (a-2x) x 3x
constant volume to equilibrate. Calculate the partial pressure of Total moles a - lx + x + 3x = a + 2x
each gas at 800K. .
Initial number of moles Initial pressure
Solution: The system in the initial stage does not contain
S02' S03 wiIl, thus, decompose to fonn S02 and 02 until Moles at equilibrium Equilibrium pressure
equilibrium is reached. The partial pressure of S03 will decrease. a
Let the decrease in partial pressure be lx. .
(a +
2S0 2 (g) + 02(g) ----'- 2S0 3 (g) 19
At equilibrium (2x) (2 + x) (1- 2x) x -a
62
Applying law of mass action, 19
(l_2X)2 Amount of ammonia deeomposed = 2x 2 x 19 a -a
K p =--,--- 62 31
(2xt(2+x) . 19x a
% of ammo 111 a decomposed == x 100
900 -'-1_---.:._ 31 x a
8x 2 = 61.3
or . 1- 2x = 84.85 Example 14. Calculate the per cent dissociation of
x H 2 S(g )ifO.l mole ofH 2 S is kept il1 OAlitre vessel atlOOOKfor
or x==O.01l5atm the reaction,
Thus, the partial pressures at equilibrium are: 2H2S(g)~ 2H2(g)+ S2(g)
Pso} 2 x 0.0115 == 0.023 atm The value of K c is 1.0 X 10-6 . .
616 G.R.B. PHYSICAL CHEMISTRY FOR COMPETITIONS

. Solution: 2H2 S(g) Solution: ·2AB 2 (g)


At equilibrium (OJ. x) x x!2 At equilibrium. (1- x) x x!2
(0.1 x) x Total moles afequilibrium == 1- x + x + x / 2
Molarconc.
0.4 0.4 0.8 2+x
=--
2
_2(1 x) p. 2x ~.p.
p AB2 - (2 + x)' ,p AB -_.p; PB2
(2+ x) (2+ x) ,

. .
2
. . -[
2x ] 2( x \\
-p --p
K =(PAB) (PB 2)= (2+x) 2+xJ_.
or p 2 . 2
0.8(0.1 ( PAB 2) (2(1 X)pJ
. l(2+x)
asx is very small; 0.1- x -? 0.1

1.0 X 10-6 (2 + x)(l-


As x is very small, (2+ x) -?2and(i- x) -? 1.
x3 8 X 10-9
x 3p
So, Kp
or x=2x 10-3 2 ..-.
3 3 2Kp
or x
.' So, R~r cent dissociation 3x 10- X 100 2.0 p
0.1
Example 15. The vapour density (hydrogen = 1) of a or x (~pr3
mixture containing N0 2 and N 204 is 38.3 at 26.7° C. Calculate
the number of moles o.f NO z in 100 grams of the mixture.
Examplel7. At 25°C and one atmospheric pressure, the

Solution: N 2 0 4 (g) ~ 2N0 2 (g)


(i\I U\ R 19')J)
partial pressures in an equilibrium mixture ofN 2 4 and NO z are
0.7 and 0.3 atmosphere, respectively. Calculate the partial
°
pressures ofthese gases when they are in equilibrium at 25°C and
At equilibrium (1x) 2x
at a total pressure oflOatmospheres. CIILNR 19%)
" ' ; : 1 " ' ) := -
x (d. egree OJ.,l;!lssoclatlon D- d- Solution: N2 04 (g) ~2N02(g)
.' •.... . .. (n - 1)d
At equilibrium 0,7 0.3 atm
· d"- '3' 8' 3' '~'" Mol. mass of 92 := 46 n 2
G.Jlven, - ., 2 2'
K = ( PN02 )2 0.3 x 0.3 = 0.1285 atm
So, x 46- 38.3 = 0.2 p PN 0
2 4
0.7
38.3
Let the degree of dissociation of N 2 0 4 be x when total
At equilibrium, amount ofN.i04 = 1 0.2:= 0;8 mol pressure is 10 at:l10sphere.
and amount of N0 2 . 2 x 0.2 0.4 mol
N 2 0 4 (g) --->. 2N0 2 (g)
Mass of the mixture = Q.8 x'9~ :+'0.4 x 46
At equilibrium (l-x) 2x
73.6 + 18.4 := 92.0 g
Total number of moles 1- x + 2x = 1+ x
Since, 92 gram of the mixture contains = 0.4 mol N0 2 (l x) 2x
PN 0 = - - x 10; PNO =: - - )« 10.
So, 2 4 (1 + x) 2 .(1 + x)
100 gram of the mixture contains = 0.4 x 100 = 0.43 mol NO z
92 (~)2 X2 10
ll+x 40x 2
Example 16. At temperature T, the compound AB2 (g)
K p := 0.1285:= ( J
dissociates according to the reaction, ~ xlO
l+x

with a degree of dissociation, x, which is small compared with Since, x is very small, (1- x 2 )-? 1
unity. Deduce the expression for x in terms of the equilibrium . ,,-'. x2 0.1285
So,
constant, K p and the total pressure, P mT 199-l) 40
or x= 0.0566
CHEMICAL EQUILIBRIUM 617

= (1- x) x 10= 1- 0.0566 x 10= 0.9436x 10 = 8 93 atm 87


Ps0 3 = 100 x 1 = 0.87 atm
PN 20 4(1 + x) 1 + 0.0566 1.0566'
2x 2 x 0.0566 0.1132 8.67 . .
PNO = - - x 10= . x 10= - - x 10= 1.07 atm Ps0 2 = 100 xl = 0.0867 atm
2 (l + x) . 1 + 0.0566 1.0566
. 4.33
Example 18. At 450°C, the equilibrium constant, K p ,for P0 2 = --- x 1 = 0.0433 atm
100
the reaction,
N 2(g)+ 3H2(g)~2NH3(g) K = Ps0 3 = 0.87
was found to be 16 x 10-5 at a pressure of200 atm. IfN 2 and H 2 p x ( )112 (0.0867) X (0.0433)112
Ps0 2 '/ P0 2
are taken in 1: 3 ratio what is % of NH 3 formed at this
temperature? 0.87
= ------~---

Solution: N 2(g) + 3H 2(g) ~ 2NH3(g) (0.0867) x 0.208 "


At equilibrium (1-x) (3-3x) 2x = 48.24 atm- 1I2

Total number of moles = 1- x + 3 - 3x + 2x = 4 - 2x Example 20. N 204 dissociates as,


N204(g)~2N02(g)
_ (I-x) p. = (3-3x) p. ... = 2x P
at 55°C and one atmosphere % decomposition ofN 204 is50.3()I10~
PN z - (4-2x) , PH 2 (4-2x) , PNH 3 (4-2x)··

(4~~XP r At what pressure and same temperature, the equilibrium mixture


has the ratio ofN 204 : N0 2 as 1: 8?
Solution:

(1- (3 - Case I: N204(g)~2N02(g)


- - ' x- p.) - -3xp )3 At equilibrium (I- x) 2x
4 - 2x 4 - 2x . . _ (1- x) x p . ' 2x
PN 2 04 . - (1+x) ,PN0 2 = (1+x) xP
4x 2 (4 - 2x)2
, =------~--~---

(~,pr
'.

(1- x) x 27x (1- x)3 p2


4x 2 P
2
=~x-
1.6 x 10-5
27
x (2_x)2
(1- x)4 X (200)2 Kp = [:::p] (I-x')

1.6 X 10-5 x 27 X (200)2 Given, x'= 0.503 and P=1 .


or
16 K p = 1.3548 atm ,.
16 x 10-6 x 27 X (200)2 Case II: N 20 4 ~2N02
't;
16 (1- x) 2x

x(2-x)=iMxlO-:-3 xm=1.039 (I-x) 1


or Given, ----
(L';;;xi . 2x 8
or x;:: 0.30 x= 0.8
Moles of ammonia: foimed = 2 x 0.30 = 0.60 Let the new pressure be P atm.
Total moles at equilibrium = (4 - 2x)= (4 - 2x 0.30)= 3.40 2
K = 4x P = 4 x 0.8 x 0.8 x P = 1.3548
% of NH3 at equilibrium = 0.60 x 100 = 17.64 P (1.:.... x 2 ) (1 + 0.8 )(1- 0.8)
. 3.40
p = 0.19 atm
Jl:xampl~ 19. A mixture of S02 and 02 at 1 atmosphere in
Example 21. At 627°C and one atmosphere S03 is
the ratio of 2: 1 is passed through a catalyst at 1170°C for
partially dissociated into S02 and 02 by the reaction,
attainment of equilibrium. The exit gas is found to contain 87%
S03 by volume. Calculate Kpfor the reaction, S03 (g)~S02 (g)+ 1/ 2°2 (g).
. The density of the equilibrium mixiure is 0.925 g L- 1 • What is
S02 (g)+ .!.02 (g)~S03 (g) the degree ofdissociation?
2
Solution: Let the molecular mass of the mixture at
Solutio .. : The volume of S02 and 02 at equilibrium
, equilibrium be M mix'
= (100 - 87) = 13 mL
. 2 Applying the re1ati r m,
Volume of SOz = - x 13 = 8.67 mL
3 ;,..; . = dRT =. 0.925 x0.0821 x 900
1 mix P 1
Volume of oxygen = - x 13 = 4.33 mL
3 =,68.348
618 C; R.B, PHYSICAL CHEMISTRY FOR COMPETITIONS

Molecular mass of S03 == 80


Vapour density of S03 ,D 80 40
2
68.348 == 34.174
112 [ 3a pJ312
Vapour density of mixture, d
2 2(l+a)
=------='=-
Let the degree of dissociation be x.
D- d 40- 34.174 5.826x 2 I-a pJ
x==--- 0.34 l+a
(n - l)d
(~- I)X 34.174 34.

or x 34% dissociated
Solving fora. w~ get a = [1 + 3.J3
4 Kp
Lj- [/2

i. e., S03 is 34% dissociated. K c can be calculated by using K p = Kc (RT l"


Example 22. Density of eqllilibrium mixture ()f N 20 4 and I

Kp 78.1; T 673K; Lln == I


N0 2 at I atm and 384 K is 1.84g dm -3 Calculate the
equilibrium constant of the reaction. Example 24. The equilibrium constant K p for the
reaction,
N~O~~2N(j2 N2 + 3H 2 ~2NH3
Solution: We know that, is 1.64 x 10-4 at 400°C and 0.144 x 10-4 at 500°C. Calculate the
Pm==dRT mean heat offormatiQn ofl moleofNH3!rom ~ts elements in this
temperature range.
I x m == 1.84 x 0.0821 x 384
m=29x 2 Solution: K,- = -l-
We know that, log -
. . KI
l i- - -
2.303R Tl
[1
Vapour density (d)at equilibrium == 29
Initial vapour density == M 12 = 921 2 == 46
loa 0.144 =' lli (_1_
'" 1.64 2.303 x 1.987 x 10-3 673
x = D - d == 46 29 0.586 lli 25.14 kcal for 2 mole
(n -I) d 29
12.57kcal mol-I
N 2 0 4 ~2N02
o I 0 Example 25. When limestone is heated, qUicklime is
1- x 2x (Total moles = I + x) formed according to the equation.
I-x ' 2x
PN204 = -1- x P; PNO; I xP CaC0 3 (s) ----" CaO(s) + CO 2 (g)
+x '+x
2
The experiment was carried out in the temperature range
K = 4x P = 4 X (0.586)2 xl 2.09atm 800- 900°C. Equilibrium constant K p follows the relation,
PI:":? 1- (0.586i log Kp = 7.282- 8500/T
Example 23. For the reaction, where. T is temperature in Kelvin. At lvhat temperature the
decomposition will gi\'e CO 2 (g) at 1 atm?
NH3(g)~~ N 2 (g)+ 3 (g)
,2 2 Solution: K p = PCOl =1

-
show that degree of dissociation of NH 3 is given
I"
CIS:
10gKI' =7.282-
8500
r
a= 1+
[ j
3.J3L-
4 Kp log 1= 7.282 _ 8500
T
where, ' p' is equilibrium pressure. (f K p of the above reaction is
78.1 atm at 400°C, calculate K (" T == 8500 = 1167.26 K
7.282
Solution:
= 894.26°C
NH3 (g) ~ ~ N2 + ~ H2 (g) Total moles
Example 26. Equilibrium constant for the reaction of
t 0 0 0 iodine with propane according to the following equation was
1- a a/2 3a/2 l+a determined.
te'l'
Some results obtained at 545 K were as given ahead:
Pi U+:)p ( a
l2(1+a)P
J (P; ~ partial
pressure)
I
CHEMICAL EQUILIBRIUM I 619

Initial Pr.essur.es (m1l1 Hg) Equilihr.ium Pr.essur.es (mm) i Solution:


2NH3(g) ~ Nz(g) + 3H z(g) n P V T
HI_...._--- -- .
e3HsI t=O a 0 0 a 20 V 400
23.9 505.8 0 1.80 1.80 teq. a - aa au 12 3aa 12 a + aa 40 V 600
2.27 nRT axR x 400
V ... (1)
Calculate equilibrium constant according to the following P 20
equation, nRT a(l + a ) x R x 600
V ... (2)
P , 45
400x 45
Solution: 1st experiment: On dividing, we get I
I2 (g) + C3~(g) C3HsI(g) + HI(g) (1+ a )20 x 600
t=:O 23.9 505.8 0 0
0.5
teq (23.9 1.8) (505.8 - 1.8) 1.80 1.80
Percentage dissociation ofNH 3 =50
Kp = PC3H51 X PHl:= L8x 1.8 '= 2 9 X 10-4
.
Pl2 X PC3H6 22.1 x 504 Example 29. When 3.06 g of solid NH 4HS is introduced
into a two litre evacuated flask at 27°e, 30% of the solid
Similarly, solve for second experiment. decomposes into gaseous ammonia and hydrogensulphide._ . .
Example 27. At 817°C K p for the reactionbetHleen CO 2 (i) Calculate K c and K p for the reaction at 27°e. (ii) What
and excess hot graphite to form 2CO(g) is 10. would happen to the equilibrium' when more solid NH 4HS is
(I) What is the analysis (mole fraction) of the gases at introduced into the flask? {; n ; 'e;,(;\;'
equilibrium at 817°C and a total pressure of4 atm? What Solution: NH 4 HS(s) ~ NH3 (g) + H2 S(g)
is the partial pressure ofCO 2 at equilibrium?
(it) At what total pressure will the gas mixture have 6% CO 2 Moles ofNH 4 HS =3.06 = 0.06
by volume? . 51
Degree of dissociation ,= 0.3
Solution: (i) CO 2 (g) + C(s) 2CO(g)
At equilibrium, .
t·= 0 0
[NH 3 (g)]
0,3x 0.06 0.3 ~ 0.06
1- a 2a 2 2

Peq. (~Jp / 2a J K =[NH ( )][H S( )]= 0.3 x 0.06x 0.3x 0.06


c. 3 g . '2 g . 2x 2
1+ a ll+ a P
;, 8.1x 10-5
2 2
4a p 4a 4
Kp ""'--2 ;10 .Nowapplying,·
2
I a 1 a Kp K c (RT)~" = 8.1 x 10-5 X (0.Q82 X 300)2
On solving, we get a = 0.62 =0.049
I-a 1- 0 62
XC02 - - := . = 0.2345 = 23.45% (by volume) Since, NH 4 HS is solid, so it causes no change in equilibrium.
l+a 1 + 0.62 Example 30. At 540 K , 0.10 mole of PCl 5 are heated in a
xco = 0.7655= 76.55% (by volume) 8 litre flask. The pressure of equilibrium mixture is found to be
Peo2 = (0.2345 x 4.0) atm = 0.938 atm 1.0 at/no Calculate,K p and K c for the reaction. i '}'.;;}

(ii) Let the-total pressure be P atm. Solution: PCl 5 ~PCI3 + Cl 2


At equilibrium (mole) (O.l-x) x x
'K :=10= [Peo]2 . 0.94xPx0.94xP
p . Total number of moles, n = (0.1 ~ x) + x + x (0.1 + x)
Peo2 0.06 x P
PV=nRT
P 0.68 atm Ix 8= (O.l+x)x 0.082 x 540
Example 28. Ammonia under a pressure of 20 atm at or x = 0.08
127°C is heated to 327°C in a closed vessel. Under these 2
K = [PCI 3 ][CI 2 ] = x
conditions NH 3 is partially decomposed to N 2 and H 1 according
C • [PCIs] (0.1 x)x 8
to the equation: .
2NH 3 (g)--'-" N 2 (.g) + 3H2 (g) 0.08 x 0.08 4 x 10-2 mol L- 1
After decomposition at constant volume in a vessel the (0.1 - 0.08)8
pressure increases to 45 atm, What is the percentage ofammonia Kp =Kc(RT)~1I =Kc RT(~n=+I)
dissociated?' .
4 x 10-2 x 0.082 x 540= 1.77 atm
620 G.. R.B. PHYSICAL CHEMISTRY FOR COMPETITIONS

Example 31. At 817°C, K p for the reaction between Solution: 2NO(g)+ Clz(g) ~ 2NOCI(g)
CO 2 (g )and excess hot graphite( s) is 10 atm. t= 0 0.02 o o
(a) What are the equilibrium concentrations of gases at 0.02x 99 0.02 x 1
p
817°C and a total pressure of 5 atm? 100 100
(b) At what total pressure, the gas contains 5% CO 2 by
volume? (liT 2000)
(0.02 X 0.01)2 = 10-3
Solution: (a) CO 2 (g)+ C(s)~2CO(g) (0.99 X 0,02)2 x P
Atequilibrium (S-x)atm xatm . p = 0.102 atm
2
K = [CO]2 x
PV= nRT
p [C0 2 ] (5 x)
2 0.102 x V =n x R x T ... (i)
. 10= x
5-x 1x V = 0.2 x R x T ... (ii)
x 2 = 50-lOx From eqs. (i) and (ii), n 0.0204 (no. ofmoiesofCl 2 at
equilibrium)
+ lOx- 50= 0
Pressure of Cl z involved in reactioll
-lOi ~100+200 . """1 "
x= . = 3.66 = - x pressure of NOCI
2 2
i.e., Peo = 3.66atm; Peo2 1.34 attn 1 0.02
=- x 0.0001 atm
Concentration of CO = _---.,.3_.6_6_ _ 0.041 mol litre -I 2 100
RT 0,0821 x 1090 PV nRT
Concentration ofC0 2 = 1.34 =0.015 mol litre- I 0,0001 x V n x RT ... (iii)
. RT 0.0821 x 1090
From eqs. (ii) and (iii), n 2 x 10-5 (moles of Cl 2 involved
(b) CO 2 (g) + C( s) ~ 2CO (g )
in reaction)
I'" 0 0 0
5
I a' 20. Initial moles ofC1 2 taken 0.0204 + 2 x 10-
1"'1"
1- a. 20. = 0.02042 .
Pi -P P
1+ a. 1 + a.
Example 33. In the following equilibrium,

[l:a pr N 20 4 (g) ---->. 2N0 2 (g)


when 5 moles of each is taken and the temperature is kept at

. [I-a
--p1
... (i)
Kp = 298 K, the total pressure was found to be 20 bar.
l+a Given: I:lG°
f Ni0 4 100 kJ, I:lGf°N02 = 50 kJ
5 I-a
- (a) Find I:lG of the reaction at 298 K.
100 l+a (b) Find the .direction of the reaction. (111' lO1l4)
a 0.95
') 0 4 X (0.95)2 Solution: Reaction Quotient [ PN0 2 f -= 100 __ 10
.. From eq. (1, 1 xP PN 2 0 4 10
. 1- (0.95)2
0 0 0
P = 0.27atm I:lG r~action 21:l G f N02 -I:lG f N2 0 4

Jj:xample 32. ThevalueofK p is Ix 10-3 atm- 1 at 25°Cfor = 2x 50 100 0


the "reaction 2NO(g) + Cl 2 (g) ~ 2NOC1(g). A flask' We know that, I:lG = I:lGo 2.303 RT log Qp
contains NO at 0.02 atm and at 25°C. Calculate the mole ofCl2
that must be added ifl % of the NO is to be converted to NOCl at
o 2.303 x 8.314 x 298 log 10
equilibrium. The volume of the flask is such that 0.2 mole of gas - 5705.8 J = - 5.705 k:J
produces 1atm pressure at 2SOC. (Ignore probable association of Negative value sho;ws that reaction will be in forward
NO to N P2)' (UT 2001) direction ..
CHEMICAL EQUILIBRIUM 621

•••.SUMMARY AND IMPORTANT POINTS


-~ ".r&.~' " TO REMEMBER
~~

1. Irreversible 'and reversible reactions: Chemical


reactions can be classified as irreversible and reversible or ....:.::...~....::....~....::.......::..::..._~: k f ""K
... }
kb C
reactions. The reactions which move in one direction only are
called irreversible reactions. In these reactions products do not The equilibrium constant, K c' at a given temperature, is the
react to produce original reactants. In such reactions an arrow ratio of rate constants of forward and backward ,reactions. It is
(-7) is placed between reactants and products. The chemical also defined as the ratio between the molar concentrations of the
reactions which take place in both directions' under similar products to the molar concentrations of the reactants with each
conditions are called reversible reactions. In such reactions concentration term raised to the power equal to stoichiometric
products also react with each other to produce reactants again. coefficient in the balanced chemical equation.
The sign ( -c-- ) is placed between reactants and products. The value of equilibrium constant is independent of the
2. Chemical equilibrium: Chemical equ~librium iS,the following factors; ,
most important chara:ct~ristic property of reversible reactions. It ,> (i) Initial concentration of reactants,
is, the state-at which both forward and backward reactions occur ,(ii)Ihedir.ectionJ'rom which equilibrium has been attaine.d~_
at the same sPeed. (iii)The presence of a catalyst.
At equilibrium state, , (iv) The Pl'eSence of inert materials.
Rate of forward reaction = Rateof,backward reaction The value of eqUilibrium constant depends upon the following
At equilibrium state, the concentrations of tlie reactants and factors:
products do not change with time. The following are the (i) The mode of reprt'sentation oft be r0action: Consider
characteristics of the equilibrium state: the reversible reaction,
(i) It can be achieved only if the reversible rea,ction is carried A+B.~C+D
out in a closed space. K : [C][D]
(ii) It is characterised by constancy of certain properties such c [A][B]
as concentration, pressure, density, colour, etc. '
The products are made the reactants; i. 'e., the reaction is
(iii) It can be attained from either side ofthe reaction. reversed:
(iv) It can be attained in lesser time by use of a catalyst.
C+D~A+~B·
(v) It is dynamic in nature, i. e., reaction does not stop, but
both the forward and backward reactions move with the same K' : [A][B]
speed. " [C][D]
(vi) Change of pressure, concentration or temperature favours
one of the reaction and thus shifts the equilibrium point in one So, K; = :'
c
direction.
(ii) Stoichiometric representation of a chemical equation:
Reversible chemical reactions are classified into 1;wo types:
The value Of equilibrium constant will be numerically different if
(i) Heterogeneous reactions: The reversible reactions in the reaction can be written with. the help of two or more
which more than one-phase is present. stoichiometric equations.
(ii) Homogeneous reactions: The reversible reactions in
which only one-phase is present. These are further classified into
three types: '
(a) When there is no change in the number of molecules,
i.e.;An O. or N0 2 (g) (1I2)N2 (g) + 02 (g);
(b) When there is an increase in the number of molecules,
"i.e.,An +ve.' ' Thus, 'K':
c Vfj(
1 \'c

(c) When there is a decrease in the number of molecules, (iii) Use of partial pressures: When the reactants and
i.e.,An ve. products are in gaseous state, the partial pressures can be used
3. Law of chemical equilibrium (Application of law of instead of concentrations at a definite temperature.
mass action): Consider the general homogeneous reversible ml Al + m2 A 2 + m3 A 3 + ... ~ nlB I + n2 B 2 + n3 B 3 + ...
reaction in which equilibrium has been attained at a certain ( )nl ( (
temperature. K : ----'----=----=----
p (
PAl
)"'1 ( PA2 )m2 ( PA3')m3
mlA I +m2~2 +m3 A 3 + ... ~nIBI +n2 B Z +n3 B 3 + ...
Kp : K c (RT l',tJ
Rate of forward reaction = Rate of backward reaction
where, An total number of molecules of products - total
t
k/ [AI ]"'1 [A2 ]'112 [A3 ]'113 ... : kb [HI tl [B2 ]"2 [B 3 3 ... , number of molecules of reactants.
I
622 .1 c:; A.B. PHYSICAL CHEMISTRY
., FOR COMPETITIONS

When, An = 0, K p = K c; x x
-x- 2'
An = + ve, Kp >Kc; "----"-:..::-...::..:::. =L L =__x __
and An - ve, K p < K c; (a-x) (a-x)V
(iv) Temperature: The value of equilibrium constant V
changes with temperature. The values of equilibrium constants at

wo diffcren:~~:~~7::S;:e reb:?: lt7r ~;,t:o;,


[~,p)x(_x .pl
" a+Xr~:J+X
\a+x
) (a+x)(a x)

. When, All 0, i. e. , heat of reaction at constant


voluI?e is zero, (e)
Initially a b o
When, +
All == 'ye, i. e. , endothermic reaction, At equilibrium (a x) (b- 3x) 2x
K2 >K\
and when, All = - ve, i. e. , exothermic reaction Active masses
K2 <K\
Units of equilibrium constant: Partial pressures .. are
measured in terms of atmospheres. Therefore, units of K p will be
(atm)JlJI. Since, concentrations are measured in terms of moles per
litre, the llruts of K c are (mol ) 1::.11 •
K p and K c will be pure numbers when, An = O.
4. Equilibrium expressions for some reactions:
(a) 2HI(g)
" [(a+~X_2x)pr
Initially a b o (a-x)
[ (a+b-2xt
P
] _ ]3
At equilibrium (a x) (b x) (2x) (a+b-2x)P
K = [HIf =_-'---"--_ 4x2(a+b-2x)2
,c [H 2 ][I 2 ] (a-x)(b x) (a -x)(b -x)
(a -x)(b - 3x)3 p2
5. Activation energies for the forward and backward
reactions: Both forward and backward reactions follow the
(a - x)(b- x)
same path and from the same activated complex. However, the
activation energies of both reactions are different.
" AE Ea (f)-Ea (b)
So, Kc
(b) 2NO(g) For· exothermic reaction, Ea (f)<Ea (b},i.e., AE ve
Initially a o o For endothermic reactiori, E a (f) > E a (b), i. e. , AE + ve.
At equilibrium (a x) xl2 xl2 Threshold energy Energy of reactants + Activation energy
K =[N2 ][02] xl2xxl2 x2 of forward reaction
c [NO]2 (a x)2 4(a - xi Threshold energy Energy of products + Activation energy
of backward reaction
(c) 6. ' Standard free energy change of a reaction and its
CH 3 COOH(l) +C 2H s OH(l) equilibrium constant are related to each other at temperature
Initi:'llly a b o o T by the following relation:
Atequil. (a - x) (b-x) x x AGO = - 2.303RT log K p
AGO = Allo - TASO
(a x)(b - x) When, AGO ve, the value of equilibrium constant will be
large positive quantity and when AGO is positive, the value of K
(d) PCIs (g) -----'>. PCl 3 (g)+ Cl 2 (g)
is less than I, i. e., low concentration of products at equilibrium
Initially a 0 0 state.
At equilibrium (a x) x x 7. Degree of dissociation from density measurements:
Active masses
x x Degree of d~ssoci~tion in the case of reversible reactions in which
V V V there is increase iIi the number of molecules can he determined by
CHEMICAL EQUILIBRIUM 623

measuring density of the reaction mixture at equilibrium. Let d be Applications of Le Chatelier's Principle
the obserVed density at a particular temperature when degree of (i) Ice water system (melting of ice):
dissociation is x and D be the vapour density when there is no
dissociation. Ice + Heat ~ Water

D-d It is an endothermic process and there is decrease in volume.


x= ; where, n is the number of molecules of products. Thus, the favourable conditions for melting of ice are:
(n - l)d
(a) High temperature and (b) High pressure.
8. Heterogeneous equilibria: Law of mass action can (ii) Solubility of gases in liquids: When a gas dissolves in
also be applied to the heterogeneous system. In such systems the a liquid, there is decrease in volume. Thus, increase in pressure
concentrations of pure solids and liquids are not considered. in will favour the dissolution of a gas in liquid.
equilibrium expressions .. (iii) Formation of nitric oxide:
C(s) + H 20(g) ~ CO(g) + H 2(g) N2 (g) + 02 (g) ~ 2NO(g) - 43200 calorie
K = Pea x PH 2 or K = [CO][H2] In this chemical reaction, there is no change in the number of
P PH 0 . c [H 2 0] molecules and heat is absorbed (endothermic). Thus, favourable
2
conditions for greater yields of NO are:
i. e. ,concentration of C(s) is not taken into account. (a) High concentrations ofN 2 and 02'
9. Le.Chatelier's principle:. It is a qualitative .principle. (b) -Highteniperature ami
which can describe the effect of change in concentration,
(c) No effect of pressure and catalyst.
pressure and temperature on the reversible system whether
(iv) N 2 (g) + 3H 2 (g) ~ 2NH3 (g) + 22400 calorie: The
physical or chemical. It is· stated as "If the system at equilibrium
is subjected to a change of anyone of the factors such as reaction involves decrease in number of molecules and evolution
concentration, temperature or pressure, the system adjusts itself of heat (exothermic). The favourable conditions are:
in such a way as to annul the effect of that chang~" The (a) High concentrations ofN2 and H 2,
following conclusions have been derived from this prinCiple: (b) High pressure and
(i) Increase in concentration of any substance favours the (c) Low temperature.
reaction in which it is used up. To speed up the rate of reaction at low temperatun:;, a suita:ble
(ii) High pressure is favourable for the reaction in which there catalyst is always employed.
is decrease in volume.
(iii) A rise in temperature favours the endothermic reaction.
624 G.A.B. PHYSICAL CHEMISTRY FOR COMPETITIONS

'f:r0_~M%1'[I]W•.
1. Match List I (Equations) with List II (Type of Processes) and (r) atm
(C)Hz(g)+lz(g) 2HI(g)
select the correct option:
List~1 List-II (d)Nz(g)+ 3H2(g)~ 2NH)(g) (s)(atm)-z
(Equations) (Type of Processes) [C] Match the reactions of the Column-I with the factors in
(a)Kp>Q (i) Non-spontaneous Column-II:
(b)6.Go< RTlogeQ (ii) Equilibrium Column-I Column-II
(c)Kp=Q (iii) Spontaneous and (a) N2(g} + 3H2(g) ..-- 2NH3(g) (p) Forward shift by
endothennic (Exothermic) rise in pressure
. (d)T > 6.H' (iv) Spontaneous (b)2S02(g) + °2(g) 2S03 (g) (q) Unaffected by
!lS (Exothennic) change in pressure
(A) a = (i); b (ii); c (iii)~ d (iv)
(B) a = (iii); b= (iv); c= (ii); d (i) (c) N 2 (g) + 02(g) 2NO(g) (r) Forward shift by
(Endothennic) rise in temperature
(C) a = (iv); b = (i); c (ii); d (iii)
(D) a = (ii); b (i); c (iv); d=(iii) teBS.,.; (PMT) 2010] (d)PCI 3 (g) + Clz(g) pels (g ) (S)F orward shift· by'
[Hint: When Kp > Q; rate offorward reaction> rate of backward (Endothermic) lowering the
reaction, i.e., process is spontaneous. temperature
When 6.Go < RT loge Q, 6.Go is positive, thus reaction is non [D] Match the reactions of the Column-I with the relations in
spontaneous. Column-lI:
When Kp = Q, reaction is at equilibrium. Column-I - Column-II
When TM > 6.H, 6.G will be negative only when 6.H = +ve, thus
is spontaneous and endothermic.] (a)H2(g)'+ I2(g).--" 2m(g) (P)K p = Kc(RT)
2. Matrix-Matching Problems (For lIT Aspirants): (b)N2(g) + 3H2 (g) 2NH3(g) (q)K p = Kc(RT)2
[A] M!)tch the Column-I with Column-II: .
(c)PCl5(g)~ PCI 3 (g)+ CI 2 (g) (r)K p K c (RT)-2
Column-I
,- Column-II
(d)NH4HS(s) . NH 3 (g)+HzS(g)(s)K p =K c
2HI(g) (P) Unaffected by
. inert gas addition [E] Match the List-I with List-II:·
CaO( s) + CO2 (g) (q) Forward shift by
L.ist-I List-If
rise in pressure (a)Q= K (P) Reaction is nearer to completion
and backward
shift by inert gas (b)Q < K (q) Reaction is not at equilibrium
addition (c)Q > K (r) Reaction is fast in forward direction
2NH3 (g) (r) Unaffected by (d)K >>> I (s) Reaction at equilibrium
increase ir.
pressure [FlMatch the List-I with List-fL
PC1 3(g) + C1 2 {g) (s) Backward shiftb,; .List-I List-U
rise in pres~ur" •. (Reaction) (KpIK,,,]
and forward !Shift
by inert gas
(P)(RT r2
addition (q)(RT )0
[B] Match the reactions in the C.Qlumn-I with the units of (r) (RT i/2
equilibrium constant in Coluinn~II: ,-
(s) (RT rl/2
Column-I Column-II .
(a) CaC03 (s) CaO(s) + COz(g) (P)mol L
(b)PCls(g) PCI J (g) + Clz(g) (q) Unitiess
I
CHEMICAL EQUILIBRIUM 625

1. (C)(a = iv);(b = i);(c = ii);(d iii) [D] (a-s); (b-r); (c-p); (d:"'q)
2. [A] (a-p, r); (b-p, r); (c-q); (d-s) [E] (a-s); (b-q, r); (c--q);(d-p)
[B] (a-p, r); (b-p,r); (c-q); (d-s) [F] (a-p); (b-q); (c-s); (d-r)
[C] (a-p, s); (b-p, s); (c-q, r); (d-p, r)

-~:::"-*:.:-':. -;
~,'"~-~~PRACTICE PROBLEMS. ..~ "·~·,.·,;·e . '.' -1~o:~j:'~T .~~;r ';~:':

1. Kc for the reaction, NH3(g) ~ 1I2N2 (g) + 3/2 H 2 (g), at [Ans. (a) 23.2 (b) 11.45 kPa (c) 0.0337 kPa]
298 K is 5.2 x 10-5 . What is the value of K c at 298 K for the 8. From the following data at 1000 K:
reaetion, COCI 2 (g) CO(g) + CI 2 (g); K J = 0.329
N2 (g) + 3H2 (g) 2NH3(g)?
2CO(g) + 02(g) 2C02 (g); K2 := 2.24 X 1022
[Hint: K~ for reaction 1/2Nz(g)+3/2H2(g)~NH3 is~, Calculate the equilibrium constant for the following reaction:
Kc
. 1 2COCI 2(g) + 2C0:i(g) + C1 2 (g)
I.e., 5
5.2 x 10- [Ans. 2.43 x 102l]
K e forthereaction,Nz(g)+ 3Hz(g) ~ 2NH3(g), will be= (K~)2 9. For the reaction, S02(g) + Y;02(g).,-- S03\g), Kp is

=[ 1 .]23.7XI08]
5.2 x 10-)
32 (atm)-lIzat 800 K and Mf for the reaction is -187.9 kJ
mol-I. Calculate its value at 900 K, if it is assumed that Mf
2. The value of K p for the reaction, remains constant over this range of temperature.
[Ans. 1.387 atm- 112 ]
2H zO(g) + 2CI 2(g) ~ 4HCl(g) + 02(g),
is 0.035 atm at 400°C, when the partial press\lfes are expressed 10. The equilibrium constants for the reaction,
in atmosphere. Calculate the value of K c for the same reaetion. N2 (g) + 0z(g) 2NO(g), at 1727°C and 2227 C are Q

[Ans. 6.342 x 10-4 mol L- J ] 4.08 x 10-4 and 3.6 x 10-3 respectively. Calculate the enthalpy
3. For the reaction,
change for the reaction. (Given, R := 1.987 caL)
2S0 3 (g) ~ 2S02 (g) + °2(g),
[Ans. 43273 cal)
the equilibrium constant Kc = 2.52 X 10- 2 at 27°C; calculate
11. Calculate the equilibrium constant (Kp) for the reaction,
K p' (Given, R = 0.082 litre-atm deg- 1 mol-I)
C(s) + CO 2 (g) 2CO(g), at 1300 K from the following
[Ans. 61.99 x 10-2] data: C(s) + 2H 2 0(g) ~ CO2 (g) + 2H2 (g);
4. If Kp for the reaction, N2 (g) + 3H2 (g) ~ 2NH3 (g), is Kp (1300 K) 3.9 atm
1.64 x 10-4 atm -2 at 400°C. What will be the equilibrium H 2 (g) + COz(g) CO(g) + H 2 0(g);
constant at 500°C, if heat of reaetion in this temperature range Kp (1300K) 0.7 atm
is 105185.8joule? [Ans. 1.91 atm]
[Ans. 1.44 x 10-5 atm] [CO]2 K [CO]2[H 20)2 x [Hd[C0 2 1
[Hint:
5. For the reaction, NZ0 4 \g) 2NOz(g), K p = 0.157 atm at [C0 2 l p(net) [H 2 12[C0 2 l" [H 2 0f
300 K. Calculate the value of Kc for the same reaction at the
=(0.7f x3.9]
same temperature.
[Ans. 6.38 x 10-3 mol L- J ] 12. The standard equilibrium constant for the reaction,
Yz
6. For the reaction, S02(g) + 02(g) ...--c- S03(g), the value PCIs(g) ~ PC1 3 (g) + C1 2 (g),
of Kc is 1.7 X 10 12 at 300 K. Calculate the equilibrium is 1.8 x 10-7 at 298 K. Calculate its I1Go value.
constants for the following reaetions at 300 K: [Ans. 38.49 kJ mol- 1]
2S02 (g) + 02(g) 2S03 (g)
13. For the equilibrium, Ag 2C03(s) ~ AgzO(s) + CO2 (g),
and S03(g) S02(g) + lf20 2 (g)
equilibrium constants are 3.98 X 10-4 and 1. 41 x 10-2
[Ans. 2.89 x 1024 ,5.88 x 10-13]
respectively at 350 K and 400 K. Calculate the standard
7. Determine Kp for the following reactions: enthalpy of decomposition.
(a) CO(g) + HzO(g) COz(g) + H 2(g); [AilS. 83.06 kJ/mol]
. Kc 23.2 at 600 K 14. The equilibrium constant for the reaction,
(b) N2 0 4 (g) ~ 2N02(g); 4.62 x 10-3 M at 298 K Nz(g) + 3H2 (g) ~ 2NH3 (g),
(c) 2S02 (g) + 02(g) ~ 2S0 3(g);
Kc == 2.8 X 102 M -1 at 1000 K
626 G.A.B. PHYSICAL CHEMISTRY FOR COMPETITIONS

at 715 K is 6.0 X 1O~2. If in a particular reaction, there are 0.25 23. In the dissociation of HI, it is found that 20% of the acid is
1 1
mol L- ofH2 and 0.06 mol L- ofNH 3 present, calculate the dissociated when equilibriilmis reached, Calculate the value
of K p for the equilibrium,
concentration ofN 2 at equilibrium.
2HI(g) ~ H2(g) + I 2(g)
[Ans. [N 2 ] =3.84 mol L- 1]
[Ans. Kp == 1.56 x 10- 2]
15. One mole of H20 and one mole of CO were heated in a 10 litre
24. The equilibrium constant of the reaction,
closed vessel at 1260 K. At equilibrium, 40% of water was
found to react in the equation, A +B~C+D
H20(g) + CO(g) ~ H2(g) + CO 2(g) is unity. What per cent of A will be transformed ifthree moles
of A are mixed with 5 moles of B?
Calculate the equilibrium constant of the reaction.
[Ans. 0.44] . [Ans. 62.5%]
[Hint: H 2 0Cg) + CO(g) ~ C0 2 Cg) + H 2 Cg) [Hint: At equilibrium,
(1- 0.4) (1- 0.4) 0.4 0.4 A + B ~ C+D
(3 - x) (5 - x) x x
(at equilibrium)]
x
16. Two moles of PCI s were introduced in a 2 litre flask and Find the value of x. % of A transformed == .::... x 100]
3
heated at 600 K to attain equilibrium. PCl, was found to be 25. 2 moles of A and 3 moles of B are mixed and the reaction is
40% dissociated into PCI 3 and C1 2. Calculate the value of KG'
carried at 400°C according to the equation,
[Ans. KG = 0.267 mole L- 1]
A +B~2C
17. 0.1 mole of PC 15 is heated in a litre vessel at533 K. Determine
the concentration of various species present at equilibrium; if the equilibrium constant of the reaction' is 4. Find the
the equilibrium constant for the dissociation of PCIsat 533 K concentration of C at equilibrium.
is 0.414. [Ans. 2.4 mole]
[Ans. [PCIs ] = 0.0531 mol L- 1; [PCI 3 ] = 0.0469 mol L- 1; 26. A combination of hydrogen and iodine is carried out by
heating 60 mL of hydrogen and 42 mL of iodine in a closed
[CI 2 ] = 0.0469 mol L- 1 ]
vessel. At equilibrium 28 mLaf hydtoiodic acid is present in
18. At 1000 K, the equilibrium constant, Kc, for the reaction, the vessel. Calculate the degree of dissociation of HI.
CO(g) + Clig) ~ COCI 2 (g)
[Hint: H 2 Cg) + I2Cg) ~ 2HICg)
is equal to 3.04. If 1 mole of CO and 1 mole of CI 2 are At equilibrium (60 - 14) (42 -14) 28 mL
introduced into a 1 litre box at 1000 K, what will be the final
concentration of COCI 2 at equilibrium? K = 28 x 28 ==~
[Ans. [COCI 2] = 0.568 mol L- 1 ] c 46 x 28 23
19. Given, that KG = 13.7 at 546 K for 2HI(g) ~ H 2 (g) + Il(g)
1- x x/2 .1'/2 (.\ is ·he degree of 'iation)
PCIs(g) ~ PCI 3 (g) + CI 2(g), calculate what pressure will
develop in a 10 litre box at equilibrium at 546 K when 1.00 K' = .. /2 xx/2 =_ "

mole of PC Is is injected into the empty box? C (I-xi . ,-x, .4

[Ans. P = 8.93 atm] x 23


or -- = 1.2817
[Hint: First detennine degree of dissociation, then evaluate the 2(1- x 14
total pressure by applying, P =!!.. RT where, 11 = I + ex.] x = 0.72]
V
20. In the following reaction, 27. One mole ofH 2, 2 moles of 12 and 3 moles of HI are injected
2HI(g) ~ H2(g) + I2(g) in a one litre flask. What will be the concentration of H 2, 12
the amounts ofH 2, 12 and HI are 7.8 g, 203.2 g and 1638.4 g and HI at equilibrium when K c is 45.9?
respectively at equilibrium at a certain temperature. Calculate [Ans. [H 2 ] = 0.316 mol L- 1 , [10] = 1.316 mol L- 1
the equilibrium constant of the reaction. [HI] = 4.368 mol L- 1 ] - '

[Ans. 0.019] [Hint: H 2 Cg) + I2 Cg) ~ 2HICg)


21. Concentrations of two reactants A and Bare 0.8 ~ol L- 1 each. At equilibrium (1 - x) C2 - x) (3 + 2x)

On mixing the two, the reaction sets in at a slow fate to form C K _ (3 + 2xi _
c - - 45.9,
andD. (I-x)(2 -x)
A+B~C+D
At equilibrium, concentration ofC was found to be 0.60 mol On solving, x = 0.684 mole]
L- 1. Calculate the· equilibrium constan!. 28. Four moles of hydrogen iodide were taken in a 10 litre flask
kept at 800 K. When equilibrium was attained, the mixture
[Ans. KG =9]
was found to contain 0.42 mole of iodine. Calculate the
22. IS moles of hydrogen rea~ting with 5.2 moles of iodine form
equilibrium constant for dissociation of HI.
109 mole of hydrogen.! iodide. Calculate the equilibrium
[Ans. Kc = 1.76 x 10-2]
constant of the reaction.
29. The equilibrium constant, Kc, for the reaction,
[Ans. Kc = 50]
H2(g) + 12(g) ~ 2HI(g)
is 56.8 at 800 K. When the mixture was analysed, it was found
CHEMICAL EQUILIBRIUM 627

to contain 0.316 moUL HI at 800 K. What are the 36. At 3000 K, chlorine gas dissociates into chlorine atoms in an
concentrations of H2 and 12 at equilibrium? (Assume that equilibrium reaction for which K = 0.37. What is the
initial concentrations ofH2 and 12 were the same.) concentration of chlorine atoms in a vessel that originally
[ADs. [H 2 ] = [12 ] = 0.0419 mol L- 1 ] contained 1.0 mol L -I of molecular chlorine?
30. At same temperature and under a pressure of 4 atm, PCI s is [ADS. [el] = 0.54 mol L- 1 ]
10% dissociation. Calculate the pressure at which PCI s will be 37. The equilibrium constant K p for the reaction,
20% dissociated, temperature remaining same. (lIT 1996)
[Ails. 0.96 atm]
N 2(g) + 02(g) ~ 2NO(g)
31. The reaction of the formation of phosgene gas from CO and at 1500°C is 120. IfN z and 02 at an initial pressure of 0.25 atm
CI 2 is as follows: each are maintained at 1500°C till equilibrium is established,
CO(g) + CI 2(g) ~ COCI 2(g) calculate the partial pressure.. of N 2, 02 and NO in the
In an experiment, starting with equimolecular CO and CI 2 in a equilibrium mixture. '
250 mL flask, the equilibrium mixture on analysis is found to [ADS. PN z = POz = 0.04; PNo = 0.42 atm]
contain 0.05 mole CO, 0.05 mole Cl z and 0.15 mole COCI 2.
38. Find K p for the following reaction,
Calculate the equilibrium constant of the reaction.
[ADs. Kc = 15 mol- 1 L] S02CI2(g) ~ S02(g) + CI 2(g)
32. At 35°C and I atmospheric pressure, N20 4 is 27.2% If sulphuryl chloride decomposes to the extent of 91.2% at
dissoCiated into N02. What is the value of Kp under these 102°C and total pressure 1. atmosphere.
conditions? [ADS. Kp = 4.94 atm]
[ADs. Kp = 0.3195 atm]
39. If carbon dioxide is 2% dissociated at 1800°C and I
33. Nitrogen and hydrogen are added to a 5 litre flask under
atmospheric pressure,
pressure. The flask was sealed and heated. The equilibrium
mixture contained 19.0 gofammonia, 0.16 g of hydrogen and 2C02(g) ~ 2CO(g) + 02(g)
304 g of nitrogen. Calculate the equilibrium constant, Kc, of Calcul~e K p for the reaction.
the reaction: [ADS. Kp = 4.12 X 10-6 atm]
N 2(g) + 3H2(g) ~ 2NH3(g) 40. At a certain temperature, Kc for the reaction,
[ADs. Kc = 5.02 X 105 mol-2L 2]
34. An equilibrium mixture at 300 K contains N20 4 and N02 at POC1 3(g) ~
POCI(g) + CI 2(g)
0.28 and 1.1 atm pressures respectively. If the volume of the is 0.30. If 0.6 mole ofPOCl 3 is placed in a closed vessel of3.0
container is doubled, calculate the new equilibrium pressure of litre capacity at this temperature, what percentage of it will be
the two gases. (lIT 199'1) dissociated when equilibrium is established?
[HiDt: N 2 0 4 (g) ~ 2N0 2 (g) [ADS. 68.5%]
Kp = %oj = (1.1i = 4 .32 atm 41. A reaction carried out by I mole of N2 and 3 moles of Hz
%2 04) 0.28 shows at equilibrium the mole fraction of NH3 as 0:012 at
500°C and total pressure 10 atmosphere; calculate K p' Also
When the volume is doubled, the pressure becomes half and the
report the pressure at which mole percentage of NH3 in
system again adjusts to achieve equilibrium.
equilibrium mixture increased to 1004.
N 2 0 4 (g) ~ 2N0 2 (g)
[ADS. Kp = 1.431 X 10-5 atm- 2 ; P = 105.41 atm]
0.28
( -2- - x
J(1.1
-+ 2 x J 42. The degree of dissociation of N 20 4 into N0 2 at one
2 atmosphere and 40°C is 0.310. Calculate its K p at 40°C. Also
report degree of dissociation at 10 atmospheric pressure at the
(0.14 - x) (0.55 + 2x)
2 same temperature.
K = (0.55 + 2x) = 4.32 [ADS. Kp = 0.425 atm; degree of dissociation = 0.1025]
p (0.14-x)
43. The vapour density of PCIs at 200°C and 252°C are 70.2 and
x = 0.045 57.2 respectively at one atmosphere. Calculate its value of
PN 20 = (0.14 - 0.045) = 0.095 atm dissociation constant at these temperatures.
4
PN0 = (0.55 + 2 x 0.045) = 0.64 atm]
2 [ADS. Kp = 0.307, 1.19]
35. At 21.5°C and a total pressure of 0.0787 atm, N20 4 is 48.3% 44. Some solid NH4 HS is placed in a flask containing 0.5 atm of
dissociated into N02. At what total pressure will the per cent NH 3. What would be the pressures of NH3 and H 2S when
dissociation be 1O.0%? . equilibrium is reached?
[ADS. 2.37 atm]
[HiDt: First calculate the value of Kp ~nd then evaluate the
_ N.JI4 HS(s) ~ NH3 (g) + H2 S(g); K p = 0.11

pressure by applying, [AilS. NH3 =0.83 atm; H2 S =0.33 atm] ("'lL~R 1994)
[Hint:
' ( 0.2 J2 p2
K = 1.1 = 0 0959] At equilibrium x+ 0.5 x
p (0.9J p . Kp = (x + 0.5)x = 0.11 J
1.1
628 G.R.s. PHYSICAL CHEMISTRY FOR COMPETITIONS

45. Sn02(s) + 2H2(g) .,.-- 2HZO(steamj + Sn(s) The value of x cannot be more than 0.2 as Brz is a limiting
For the above reaction find K P' if at 900 K, the equilibrium reactant. Thus, when reaction is complete,
mixture contains 45% Hz by volume. H2 =(0.6 - 0.2) = 0.4 mol; Brz 0; HBr 0.4 mol
[Ans. Kp 1.5] At this point some HBr will dissociate.
46. The equilibrium constant for the reaction is 9AO at 900°e. 2HBr(g) H 2 (g) + Br2(g)
At equilibrium (OA - 2x) (OA + x) (x)
S2(g) + C(s) ~ CS 2 (g)
Calculate the pressure of two gases at equilibrium, when 1.42
atm ofS2 and excess ofC(s) come to equilibrium.
[A,ns. == 1.28 atm; PS2 '" 0.14 atm] o.4x
or 2 x 1O-IO(since, x is very small)
47. For the'reaction, 0.4
. CaC0 3 (s) ~ CaO(s) + CO;(g)
or
K P 1.16atm at SOO°C. If20.0 g ofCaC03 were kept in a 10
litre container and heated up to 800°C, what percentage of [Br2 ] = 2 x 10- 10 mql; [H 2 ]= 0.4 mol; [HBr] = 0.4 mol]
CaC03 would remain unreacted at equilibrium? 52. One mole of H 2 , two moles OfI2 and three moles of HI are
[Ans. 34%] injected in a 1 litre flask. What will be the concentration of
[Hint: K p Pco 2 = 1.16 atm H 2 ,I 2 and HI at equilibrium at 490°C? The equilibrium
constant for the reaction at 490°C is 45.9. .
n= xV Ll6x 10 == O.132inol
RT 0.0821 x IOn [Ans. [Hz] = 0.316; [Iz ] l.316; [ID] =4.36 mol]
.. I 20.0 I
I mba amount of CaC0 3 =-- == 0.2 mo 53. Solid ammonium carbamate dissociates to give ammonia and
100 carbon dioxide as follows:
Unreacted CaC0 3 = 0.2 - 0.132 NH2 COONH4 (s) 2NH 3 (g) + CO 2(g)
= 0.068 mo\ At equilibrium, ammonia is added such that partial pressure of
0.068 NH3 now equals to the original total pressure. Calculate the
% unreacted CaCO, = - - x 100 34]
. 0.2 ratio of total pressure now to the original total pressure.
48. In the reaction, C(s) + CO 2 (g) ~ 2CO(g), the equilibrium [Ans. 3li7]
pressure is 12 atm. If 50% of CO2 reacts, calculate K p- 54. Two solid compounds A and C dissociate into gaseous product
[Ans. Kp 16 atm] at temperature T as follows:
49. A rnixture of 0.373 atm of NO(g) and 0.310 atm of (i) A(s)~B(g)+D(g)
(ii) C(s)~E(g)+D(g)
CI 2(g) is prepared at 500°e. The reaction,
2NO(g) + Cl 2 (g) ~ 2NOCI, occurs. The total pressure at At 20°C, pressure over excess solid A is 50 atm ' that over
excess solid Cis 68 atm. Find th", tote' nreSSUfC ~ gas""
equilibrium is 0.544 atm. Determine Kp of the reaction.
over the solid mixture.
[Ans. Kp 50.08 atm- I ]
[Ans. 84.38 atm]
50. For the reaction, 55. Variation of equilibriu' onstant K with temperature l' is
3A(g)+B(g)~ 2C(g) given by van't Hoff eqL. In,
at a given temperature, Kc is 9.0. What must be the volume of 1:Jf0
a flask if the mixture of 2.0 moles each of A, Band C is log K log A - -'----
2.303RT
obtained at equilibrium?
[Ans. V 6 litre 1 A graph between log K and T -1 was a straight line as shown
51. At 700 K hydrogen and bromine react to form hydrogen in the figure and having e
== tan -1 (0.5) and OP = 10.
bromide. The value of equilibrium constant for this reaction is . Calculate:
5 X 108 . Calculate the amount of Hz. Br2 and HBr 'at (a) 1:Jf0 (standard heat of reaction) when T == 298K,
equilibrium if a mixture of 0.6 mole of H2 and 0.2 mole of (b) A (pre-exponential factor),
bromine is heated to 700 K. (TIT 1995) p
[Hint: H2 (g) + Br2(g) ~ 2HBr(g)
Initial 0.6 0.2
Since, equilibrium constant is very high the reaction will be
complete and bromine is cO;Isumed.
Thus, 0.2 mole of Br2 and 0.2 mole of H2 will be consumed to
produce 0.4 mole ofHBr. This can be ¢alculated by applying law
of mass action. . o T-1
4x" ( c) Equilibrium constant K at 298 K,
Kor------
(0.6 - x)(0.2 x) (d) K at 798 K, if I1Ho is independent oftemperature.
x = 0.6 or 0.2 [Ans. (a) 9.574 J mol-I; (b) A = lOlO; (e) 9.96 x 109;
(d) 9.98 x 109 ]
CHEMICAL EQUILIBRIUM 629
/:;}f0
[Hint: (a) log K. log A - 2.303RT teq [O.1l~-2X] [0.1l~8-X] 2,X
It is equation of straight line of the type y = c + mx 2x 0.0824
/:;}f0
[NO] 0.112 - 2x
Slope'm' tan e=
2.303R 0.112 - 0.0824 = 0.0296 M
/:;}f0
0.5=---- [Br2 ] =0.1138 - 0.0824 '" 0.0726 M
2.303 x 8.314 2
!1H" = 9.574 J mor J [NOBd ~~=106.74]
(b) Intercept' c' = log A 10 (NO]2[BrzJ [0.0296]2[0.0726]

A = 10JO 57. Solid ammonium carbamate is put into a closed container and
allowed to come to equilibrium with the gaseous products at
(c) logK=IO 9.574 35°C, the total pressure is found to be 0.30 atmospheres. What
2.303 x 8.314 x 298 is the value of K p for tbis reaction at 35°C?
K = 9.96 X 109 [Ans. 4 x 10-3]

(d) IOgl~~J 2.~R (;J -;J 58. K p for the equilibrium of,
FeO(s) + CO(g) Fe(s) + COi(g)
atlOOO°CisOA03, if CO(g),at a pressure·of·l.Oatm arid-
9.574 .excess ofFeO(s) are placed in a container at 1000°C. What are
log-~"---;;-
9.96 x 2.303 x 8.314 the pressures of CO(g ) and CO2 (g) when the equilibrium is
attained?
On solving, K z =9.98xI09 ]
fAns. Pco 0.713 atm; Pco 2 = 0.287 atm]
56. When 0.1 12 mole of NO and lS.22g of bromine are placed in
59. A mixture of 1.0 mole qf sulphur and 0.2 mole of hydrogen is
a 1.00 L reaction vessel and sealed, the mixture is heated to
heated at 90°C in a one litre flask. The equilibrium constant
350 K and the following equilibrium is established:
for the formation of hydrogen sulphide,
2NO(g) + Br2(g) 2NOBr(g)
HzCg) + S(s) ~ H2S(g) is 6.8 x 10-2 . Calculate the partial
If the equilibrium concentration of nitrosyl bromide is 0.OS24
pressure ofH2 S at equilibrium.
11, what is Kc?
fAns. PH 2s 0.397 atm]
fAns. 106.74]
[Hint: [Hint: First determine the initial pressure of Hz by applying
18.22
2NO(g) + Brz(g) 2NOBr(g); nBr = - - 0.1138 n
2 160 Po = . RT = 5.953 atm]
t 0 0.1'2 0.1138 o V

ESTIONS
(b) The reactants are present in the initial stage but after that
8et-1: Questions with single correct answer the reactants and products are always present in the
1. A reversible reaction is one which: reaction mixture
(a) proceeds in one direction (c) At equilibrium only products are present
(b) proceeds in both directions (d) When the reaction is carried out in closed space, it attains
(c) proceeds spontaneously equilibrium state after suitable time
(d) all the statements are wrong 5. The law of mass action was proposed by:
2. An example of reversible reaction is: (a) Guldberg and Waage (b) Le Chatelier and Braun
(a) Pb(NO)h + 2NaI = Pbl z + 2NaNO) (c) Kossel and Lewis (d) van'tHoff
(b) AgNO) + HCl AgCI +. RNO) 6. The rate at which a substance reacts, depends on its:
(c) 2Na + 2H2 0 = 2NaOH + H2 (a) activemass (b) molecular mass
(d) KN03 + NaCI = KCl + NaN03 (c) equivalent mass (d) total volume
3. Which one of the following is not a reversible reaction? 7. Active mass is defined as:
(a) 2HI(g) = H2(g) + Iz(g) (a) number of g equivalent per unit volume
(b) PC!s(g) = PCI 3 (g) + CI 2 (g) . (b) number of g mol per litre
(c). 2KCI03(s) = 2KCl(s) + 302 (g) (c) amount of substance in gram per unit volume
(d) CaC03(s) = CaO(s) + CO2 (g) (d) number of g mole in 100 litre
4. ·Which one is not correct for a reversible reaction? 8. 8.50 g ofNH3 is present in 250 mL volume. Its active mass is:
(a) The reaction is never completed (a) LOM L- 1 (b) 0.5M (c) I.5M (d) 2.0ML- 1
630 G.R.B. PHYS1CAL CHEMISTRY FOR COMPETITIONS

9. Theqry of 'active mass' indicates that the rate of a chemical . are K 1 and K 2 respectively. The relationship between
'reaction is directly proportional to the: IPET (MP) 1990] K, and K2 is:
(a) equilibrium constant (b) properties of reactants (a) K, =K 2
(c) volume of apparatus (d) concentration of reactants (c) K,2 =K 2
10. The state of equilibrium refers to: jPMT \ Ph.} 19931
(a) state of rest (b) dynamic state 19. The equilibrium constant for the given reaction is 100.
(c) stationary state (d) state of inertness N 2 (g) + 202 (g) ~ 2N0 2(g)
.n.. In a reversible chemical reaction equilibrium is said to have What is the equilibrium constant for the reaction given below:
been established when the: 1
N0 2 (g) 2 N2 (g)+02(g)
(a) concentrations of reactants and products are equal
(b) opposing reactions cease (EAMCET 2009)
(c) speeds of opposing reactions become equal (a) 10 (b) 1
(d) temperatures of opposing reactions are equal (c) 0.1 (d) 0.01
12. A chemical reaction, A B, is said to be in equilibrium 20. For the reaction, 2N0 2(g) 2NO(g) + 02(g),
6
when: Kc 1.8 x 10- at 185°C. The value of Kc at 185°C for the
(a) rate of forward reaction is equal to rate of backward reaction;
reaction NO(g) + !.02(g),.....:.....::. N0 2(g) is: (MLNR 1993)
(b) conversion of A to B is only 50% complete (a) 0.9 x 106 2 (b) 1.95 x 10-3
... (c) complete conversion of A to 'Bhastakenplace
(d)' only 25% conversion of A to B has taken place (c) 1.95 X 103 Cd) 7.5 x 102
13. The reaction between barium chloride and sodium sulphate goes 21. The equilibrium constant for the synthesis of HI at 490°C is
to completion because: 50.0. The value of K for the dissociation of HI will be:
(a) barium sulphate is almost insoluble (liT 1990)
(b) the solubilityofbarium~hloride decreases (a) 0.02 (b) 50.0 (c) 0.50 (d) 0:20
(c) lattice energy of barium sulphate is very high 22. Consider the following cquilibrium,
(d) the reactIon is irreversible in nature
1
14. What is the equilibrium constant expression for the reaction, S02(g) + 202(g) S03 (g);
P4(S) + 502(g)~ P40 JO (s)?
K2
(.Illmia \Jillia Islamin Engg. Ent. 2007; l'PSEE 2(07) 2S03(g)~ 2S0 2 (g)+ 02(g)
(a) Kc [P4 0 IO ] (b) K = [P4 0 IO ]
What is the relation betwecn K\ and K 2? [PET (MP) 1993]
[P4 ] [02 ]5 c 5 [P4 ] [02]
1 1 1
(a)K\ = (b)K] ~(c)K] K2
(c) Kc = [02 ]5 (d) Kc = 1 K2 'ljK2
[°2
23. ·For the reaction AB(g)~ A(g)+ B(g1 AB is 33%,
15. Equilibrium constant for the reaction,
.clissoGiated ata,;tptal pressun: of?' Theryfor~, P is related to
H 2 (g) + I 2(g) ~ 2Hl(g), is correctly given by the
expression:
j( if bX·0-qe:~Qtt~~foI19wjngoptio.n>· .... 'JAl.\1iU (Med.) 2010]
(a) P = K p (b) P == 3K p
(c) P 4Kp (d) P 8K p
[Hint: '. AB(g) A(g) +B(g)

16. Equilibrium constant for the reaction,


2NO(g) + CI 2 (g) ~ 2NOq(g), is correctly given by the
expressIOn:
a K _ [NOClf (b) K = [2NOCl]
() - [NO]2[CI ] [2NO] [CI 2 ]
2
[NOf + [CI 2 ] (d) K [NOf[CI 2 ] !!..X!!..
(c) K
[NOCI] [NOCI]2 Kp ~=!!..
!!.. 8
17. For the system, 3A + 2B .,-- C, the expression for 2
equilibrium constant is: IJEE (\VB) 20071 P = 8Kp]
(a) [A]3[B]2 (b). [e] (c) =--.::-=--.c=-- (d) 24. For a system, A + 2B ~ e, the equilibrium concentrations
[e] [A]3[B]2 [e] [A][B] are [A] = 0.06, [B] = 0.12 and ] = 0.216. The Kc for the
reaction is:
18. The equilibrium constants of the reactions,
(a) 125 (b) 415
1
S02(g) + -02(g).,-- S03(g) (e) 4 x 10-3 (d) 250
2
and 2S02 (g) + 02(g) ~ 2S03(g)
CHEMICAL EOUILIBRIUM 631

25. The equilibrium concentrations of x, yand yX2 are 4, 2 and 2 [Hint: Fe3- + SeN ~ FeSCN:+
. respectively for the equilibrium 2x + y ~ YXz, The value 3.1 3,2 0
of equilibrium constant, K (' is: (EAMCET 1990) OJ 0.2 3
(a) 0,625 (b) 6,25 (c) 0.0625 (d) 62.5
'26. 4 mole of A are mixed with 4 mole of B when 2 mole of Care 2
fOllned at equilibrium, according to the reaction,
K = [FeSCN + J
A+B C+D c [Fe3+ ][SCN]
the equilibrium constant is: . (CPMT 1992) 3
150]
(a) ..fi (b) 2 (c) I (d) 4 0.1 x 0.2
27. If the equilibrium constant of the reaction 2HI(g) ~ H 2 (g) 36. K p and K ~ are the equilibrium constants of the two reactions,
+ 12 (g). is 0,25, the equilibrium constant of given below:
H2 (g) + 12 (g) ~ 2HI(g), under similar conditions, will be: ~N?(g)+~H?(g)~
2 - 2-
NH3(g)
(a) 4.0 (b) 3.0 (c) 2.0 (d) 1.0
28. HI was heated in a sealed tube at 400°C till the equilibrium N2 (g) + 3H2 (g ) ~ 2NH3 (g )
was reached. HI was found to be 22% decomposed. The Therefore, K p and K~ are related by \AMU (PMT) 2009)
equilibrium constant for dissociation, is: (a) == K~2 (b) K p .JK;;
. (a) 1.99 (b) 0,0199 (c) 0.0796 Cd) 0.282
29. For a reversible reaction if the concentrations ohhereactants
are doubled at a definite temperature, then equilibrium
constant will: (CPMT 1990; MLNR 1992) 37. In which of the following equilibrium K c and K p are not
(a) also be doubled (b) be halved equal? rCBSE (PMT) 20)01
(c) become one-fourth (d) remain the same (a) 2NO(g) ~ N1 (g) + 02(g)
. 30. In a reversible gaseous system, molar concentrations (active (b) S02(g)+N02(g)~ S03(g) + NO(g)
. masses) of reactants and products are proportional to: (c) H 2 (g)+I 2 (g) 2HI(g)
(a) partial pressure (d) 2C(s) + 02(g) ~ 2C02 (g)
(b) total pressure 38. .select the COll'ect statement from the following:
i( a) equilibrium constant changes with addition of a catalyst
(c) amounts' of reactants and products
(b) catalyst increases the rate of forward reaction
(d) none of the above
(c) the ratio of mixture at equilibrium does not change by
31. At 3000 K, the equilibrium partial pressure of CO 2 , CO and
catalyst
O2 are 0.6, 0.4 and 0.2 atmospheres respectively. K p for the
(d) catalysts are active only in solution
reaction,
39. In which of the following, the reaction proceeds towards
2C0 2 (g ) 2CO(g ) + O2 (g ) is : completion? (MLNR 1990)
IBHU (Mains) 20101
(a) K=! (b) K=10-2
(a) 0.088 (b) 0.0533 (c) 0.133 (d) 0.177
32. Equilibrium constant depends on: (c) K = 10 (d) K 103
(a) the actual quantities of reactants and products 40. For the following reaction at 250°C, the value of K c is 26, then
(b) the presence of a catalyst the value of K p at the same temperature will be:
(c) temperature PCI 3(g) + CI 2 (g) ..-- PCI 5 (g) (MLNR 1990; CBSE 1993)
. (d) the presence of inert material . (a) 0.57 (b) 0.61 (c) 0,83 (d) 0.91
33. For the reaction, H2 (g) + 12(g) ..-- 2HI(g), the equilibrium 41. In the reaction, PCI5 (g) ~ PC1 3 (g) + CI 2 (g), the amounts
constant, K P' changes with: of PC Is, PCl 3 and Cl 2 are 2 mole each at equilibrium and the
(a) total pressure 'tota.l pressure is 3 atmospheres. The equilibrium constant, K p'
(b) catalyst is: (liT 1991)
(c) the amounts ofH 2 and 12 taken (a) 1 atm (b) 2 atm (c) 3 atm (d) 1.5 atm
(d) temperature
42. In which of the following K p is less than K c?
34. 2HI(g ) ~ Hz (g ) + 12 (g)
(a) N2 0 4 (g) --->. 2N0 2 (g)
The equilibrium constant of the above reaction is 6.4 at 300 K.
If 0.25 mole each of H2 and 12 are added to the system, the :(b) 2HI(g) ~ H 2(g) + 12(g)
equilibrium constant will be: (CET Karnataka 2009) (c) 2S0 2 (g) + 02(g) ~ 2S0 3 (g)
(a) 0.8 (b) 3.2 (c) 1.6 (d) 6.4 (d) PC15 (g) --->. PCl 3(g) + CI 2 (g)
35. 3.1 mol of FeCI 3 and 3.2 mol of NH4 SCN are added to one
43. For a reversible reaction, the rate constants for the forward and
litre of water: At equilibrium, 3.0 mol of FeSCN2+ an" formed.
backward reactions are 2.3'8 x 10-4 ,tr,;j 8.15 x 10-5
The equilibrium constant Kc of the reaction:
respectively. The equilibrium constant for the reaction is:
Fe3+ + SCN- ~ FeSCN2+
(a) 0.342 (b) 2.92
will be : IBHU (Screening) 20101
(c) 0.292 (d) 3.42
(a) 6.66x 10-3 (b) 0.30
44. The equilibrium constant, in a reversible reaction at a given
(c) 3,30 (d) 150 temperature:
I
632 G.R.B. PHYSICAL CHEMISTRY FOR COMPETITIONS

(a)
depends on the initial concentration of the reactants 51. For which of the following reactions does the equilibrium
depends on the concentration of products at equilibrium
(b) constant depend on the units of concentration?
it is not characteristic of the reaction
(c) 1 I
(a) NO(g) - - ' > . 2 N2 (g) + 2" 0 2(g)
(d)
does not depend on initial concentrations
45. LO g mole of ethyl alcohol and 1.0 g mole of acetic acid are (b) C 2Hs0H(l) + CH 3 COOH(l) ~ CH 3 COOC 2 Hs(l)
mixed. At equilibrium 0.666 g mole ofthe ester is present. The + H2°(l)
value of equilibrium constant is: [PET (MP) 1993) (c) 2HI(g) ~ Hz{g) + I 2 (g)
(a) ~ (b) ~ (c) 2 (d) 4 (d) COCl 1 (g) ~ CO(g) + ClzCg)
4 2 52. The units of K p in the following reaction are:
46. The equilibrium constant of the reaction, N2 (g) + 3H2 (g) 2NH3 (g)
2
. CH 3 COOH(l) + CJIs0H(l) ;;--" CH 3COOC 2Hs(l) + H 20(l) (a) atrn (b) atm- (c) atm2 (d) atm- 1
is 4. If one mole of each of acetic acid and ethyl alcohol are 53. The equilibrium of formation of phosgene is represented as:
heated in presence of a little concentrated H 2S04 , at CO(g) + CI 2 (g) ~ COCI 2 (g)
equilibrium the amount of ester present is: [PET (MP) 1992] The reaction is carried out in a 500 mL flask. At equilibrium
Ca) I mole (b) 2 mole 0.3 mole of phosgene, 0.1 mole of CO and 0.1 mole ofCl 2 are
2 I present. The equilibrium constant of the reaction is:
(c) mole (d) mole
3 2 (a) 10 (b) 15 (c).5 (d) 3
47. It different quantities of ethanol and acetic acid are used in the 54. In an equilibrium reaction, for which 6.G" = 0, the equilibrium
following reversible reaction, constant K should be equal to:
CH 3 COOH(l) + C2HsOH(l)~CH3COOC2Hs(l) + H 20(l)
(a) zero (b) ~O (c) 1 (d) 2
the equilibrium constant will have values which will be?
55. The equilibrium constant K p for the homogeneous gaseous
(a) Different in all cases
reaction is 10-3 . The standard Gibbs free energy change t:.Go
(b) Same in all cases for the reaction at 27°C (using R = 2 cal K- 1 mol-I) is:
(c) Higher in cases where higher concentration of ethanol is (a) zero (b) - 1.8 kcal
used
(c) - 4.154 kcal (d) + 4.154 kcal
(d) Higher in cases where higher concentration of acetic acid
56. At 500 K, the equilibrium constant for reaction
is used
cis-C 2 H zCl 2 trans-C 2 H 2Cl 2 is 0.6. At the same
48. The reaction, temperature, the equilibrium constant for the reaction
2A(g)+B(g)~ 3C(g)+D(g)
trans-C zH2Cl 2 ~ cis-C 2 H2 CI 2 , will be:
is begun with concentration of A and B both at initial value of
(a) 1.67 (b) 0.6 (c) 1.76 (d) 1.64
1M. When equilibrium is reached, the concentration of D is
. 57. Which one of the following oxides is most stable? The
measured and found to be 0.25 M. The value for the
equilibrium constants are given at the same temperature:
equilibrium constant for this reaction is given by the
expression: tCBSE (PMT) 2010] (a) 2N 2 0 5 (g) 2N2 (g) + 50 2(g); K 1.2 x 1034
(b) 2N,O(g) 2N 2 (g) + 02(g); K 3.5 x 1035
(a) [(0.75)\0.25)] + [(l.OOi (LOO)] - ~
(c) 2NO(g) ~ N2 (g) + 02(g); K = 2.2 x 10
(b) [(0.75)3 (0.25)] [(0.50i (0.75)] (d) 2N0 2 (g) - - ' > . N2 (g) + 20 2 (g); K = 6.71 X 1016
(c) [(0.75)3 (0.25)] + [(0.50)2 (0.25)] 58. In a chemical equilibrium, K c = K p when:
ICEE (Bihar) 19921
(d) [(0.75)3 (0.25)]':;' [(O.75i (0.25)] (a) the number of molecules entering into a reaction is more
[Hint: 2A(g) + B(g) ~ 3C(g) + D(g) than the number of molecules produced
o o (b) the number of molecules entering into the reaction is equal
0.50 0.75 0.75 0.25 to the number of molecules produced
(c) the number of molecules entering into the reaction is less
[C ]3[D J = (0.75ho.25) ]
K to the number of molecules produced
[Af[B] (0.50)2(0.75)
(d) none ofthe above
49. The decomposition of N2 0 4 to N02 is carried at 280 K in 59. In a general reaction, A + B ~ AB, which value of
chloroform. When equilibrium has been established, 0.2 mole equilibrium constant most favours the production Of AB?
of N2 0 4 and 2 x 10-3 mole of N02 are present in 2 litre
(a) 9.0 x 10- 3 (b) 3.5 x 10-3
solution. The equilibrium constant for the reaction,
7
N 2 0 4 (g) ;;--" 2N02 (g), is: ec) 4.0 x 10- (d) 4.0 x 10-12
(a) 1 x 10-2 (b) 2 x 10-3 60. During thermal dissociation of a gas, the vapour density:
(c) Ix 10- 5
(d) 2 x 10- 5 Ca) remains the same
(b) increases
50. The unit of equilibrium constant, K c' for the reaction (c) decreases
A +B ~ C would be:
(d) increases in some cases and decreases in others
(a) mol- 1 L (b) mol L- 1 (c) mol L (d) no unit
CHEMICAL EQUILIBRIUM 633
61. The vapour density of fully dissociated NH4 Cl would be: (a) First law of thermodynamics
(a) less than half of the vapour density of pure NH4 Cl (b) Le Chatelier's principle
(b) double of the vapour density of pure NH4 CI (c) Hess's law
(c) half of the vapour density of pure NH4 CI (d) Ostwald's law
(d) one-third of the vapour density of pure NH4 Cl 70. Le Chatelier's principle is applicable to:
62. In the dissociation of 2HI ~ Hz + 12 , the degree of (a) only homogeneous chemical reversible reactions
dissociation will be affected by: (b) 'only heterogeneous chemical reversible reactions
(a) increase of temperature (b) addition of an inert gas (c) only physical equilibria
(c) addition of H2 and 12 (d) increase of pressure (d) all systems, chemical or physical, in equilibrium
63. In lime kilns, the following reversible reaction, 71. In the melting of ice, which one of the conditions will be more
favourable?
CaC0 3 (s) ~ CaO(s) + cO 2 (g) (a) High temperature andhigh pressure
proceeds to completion because of: (CPMT 1990) (b) Low temperature and low pressure
(a) high temperature (c) Low temperature and high pressure
(b) CO2 escapes (d) High temperature and low pressure
(c) low temperature 72. Solubility of a gas in liquid increases on:
(d) molecular mass ofCaO is less than that ofCaC0 3 (a) additionofacatalyst (b) of pressure
64. The equilibrium constant for ~he reaction, (c) increasing of pressure (d) increasing oftemperature--
CaC0 3 (s) ~ CaO(s) + CO 2 (g), is: 73. When KOH is dissolved in water, heat is evolved. If the
temperature is raised, the solubility ofKOH:
1
(a) increases (b) decreases
[C0 2 ]
(c) remains the same (d) cannot be predicted
74. The yield of product in the reaction,
A2(g)+2B(g) C(g)+QkJ
65. For the reaction, C(s) + cO 2 (g) ~ 2CO(g), the partial would be higher at:
pressure of CO2 and CO are 2.0 and 4.0 atm, respectively, at (a) low temperature and high pressure
equilibrium. The K p of the reaction is: (lIT 1992) (b) high temperature and high pressure
(a) 0.5 (b) 4.0 (c) low temperature and low pressure
(c) 32.0 (d) 8.0 (d) high temperature and low pressure
66. Iron fillings and water were placed in a 5 litre vessel and 75. Manufacture of ammonia from the elements is represented by,
sealed. The tank was heated to 1000°C. Upon analysis the tank • N 2 (g) + 3H2 (g) -----' 2NH3(g) + 22.4 kcal
was found to contain 1.1 g of hydrogen and 42.5 g of water The maximum yield of ammonia will be obtained when the
vapour. If the reaction in the tank is represented by, process is made to take place:
3Fe(s) + 4H 2 0(g) ~ Fe 3 0 4 (s) + 4H2 (g) (a) at low pressure and high temperature
the value of equilibrium constant, K c' is: (b) at low pressure and low temperature
(a) 30 (b) 0.03 (c) at high pressure and high temperature
(c) 3 (d) 0.003 (d) at high pressure and low. temperature
67. Ammonium hydrogen sulphide is contained in a closed 76. In the reaction, 2S0 2(g) + 02(g) 2S0 3 (g) + X cal,
vessel at 313 K when total pressure at equilibrium is found most favourable conditions of temperature and pressure for
to be 0.8 atm. The value of K p for the reaction, greater yield of S03 are:
NH4 HS(s) NH3 (g) + H 2 S(g) is: (a) low temperature and low pressure
(a) 0.16 (b) 1.6 (c) 0.016 (d) 16 (b) high temperature and low pressure
68. Variation of K with temperature as given by van't Hoff (c) high temperature and high pressure

:u~::n;:n b, :~:'[;,
(d) low temperature and high pressure
:,1 77. What is the direction of a reversible reaction when one of the
products of the reaction is removed?

(b) log
K1
till
2.303R T2
[I 7JI] (a) The reaction moves towards right hand side
(b) The reaction moves towards left hand side
(c) The reaction moves equally on both the sides

(c) log K1 2.:R [;2 ;] (cD The reaction stops


78 .. In the reaction, Nz(g) + 3H2 (g) 2NH3 (g), I1H = - 93.6
(d) none of the above kJ, the yield of ammonia does not increase when:
69. Wh~-!! any system in equilibrium is subjected to a change in (a) pressure is increased
pre~re, conccntration or temperature, the equilibrium is (b) pressure is decreased
shifted in the direction which tends to undo the effect of the (c) temperature is lowered
change. This statement is known as: (d) volume of the reaction vessel is decreased
634 G.R.B. PHYSICAL CHEMISTRY FOR COMPETITIONS

79. A cylinder provided with a pistOFl has some PCl s which is in (a) amount of solid to decrease
equilibrium with PCi} and C1 2. The system is compressed with (b) amount of liquid to decrease
the help of piston. Indicate the correct statement: (c) temperature to rise
(a) some more PCi s wil1 decompose (d) temperature to fal1
(b) the system remains unaffected 88. For the reaction:
(c) PCl} and Cl 2 wil1 combine to form PCl s 2A(g)+B(g)~ 3C(g)+D(g)
(d) explosion occurs two moles each of A and B were taken into a flask. The
80. In the manufacture of nitric oxide, the forward reaction is fol1owing must always be true when the system attained
favoured by: equilibrium:
(a) high pressure (b) low pressure (a) [A] = [B] (b) [A] < [B]
(c) high temperature (d) low temperature (c) [B] = [C] (d) [A] > [B]
81. The reaction, 89. In a vessel containing S03' S02 and 02 at equilibrium, some
C 2H 4 (g) + H 2(g) ~ C 2H 6 (g); I'ili = - 32.7 kcal helium gas is introduced so that total pressure increases while
is carried out in a vessel. The equilibrium concentration of temperature and volume remain the -same. According to
C 2H4 can be increased by: Le Chatelier's principle, the dissociation of SO}:
(a) increasing the temperature (MLNR 1991)
(b) decreasing the pressure (a) increases (b) decreases
(c) removing some hydrogen; (c) remains unaltered - Cd) changes unpredictably ----~
(d) al1 of these 90. The equilibrium, S02CI2(g) ~ S02(g) + CI 2(g) is
82. In an exothermic reaction, a 10°C rise in temperature will: attained at 25°C in a closed container and an inert gas, helium,
[PMT (Bangalore) 1993] is introduced. Which of the following statements is correct?
(a) decrease the value of equilibrium constant (a) Concentrations of S02Ci2' S02 and Ci 2 do not change
(b) double the value of equilibrium constant (b) More Cl 2 is formed
(c) not produce any change in equilibrium 'constant (c) Concentration of S02 is reduced
(d) produce some increase in equilibrium constant (d) More S02Ci2 is fonned
83. If K J! for a reaction, 91. The vapour density of undecomposed N20 4 is 46. When
A (g ) + 2B (g ) ~ 3C(g) + D (g) heated," vapour density decreases to 24.5 due to its dissociation
to N02. The % dissociation ofN 20 4 is:
is 0.05 atm at 1000 K, its K c in terms of R wil1 be:
'(a) 40 (b) 57
(a) 20000 R (b) 0.02 R (c) 5 X lO-s R (d) 5 x lO-s (c) 67 (d) 87
R
84. Consider'the reaction, CaCO}(s) ~ CaO(s) + CO 2 (g) . 92. K/Kc for the reaction, CO(g) + ~02(g) ~ CO2(g) is:
in closed container at equilibrium. What would be the effect of
addition of CaCO} on the equilibrium concentration of CO 2? (a) RT (b) _1_ (c).fiT (d) 1
(AIIMS 1991) JRT
(a) Increases 93. For the reactions,
(b) Decreases A ~ B, Kc = 1; B ~ C, Kc = 3; C ~ D, K c = 5
(c) Remains unaffected K c for the reaction A ~ Dis:
(d) Data is not sufficient to predict it (a) 15 (b) 5 (c) 3 (d) 1
85. XY2 dissociates as: . ruint:[B]=l' ~=3' [D]=5
XY2(g)~ XY(g)+ Yeg) LU. [A ] ' [ B] , [C] ,
Initial pressure of XY2 is 600 mm Hg. The total pressure at
M~ltiplying all the three, [D] = 1 x 3 x 5]
equilibrium is 800 mm Hg. Assuming volume of system to . [A]
remain constant, the value of K pis: (BHU 1992) 94. The equilibrium constant of the reaction,
(a) 5.0 (b) 100 (c) 200 (d) 400 H 2 (g) + I 2(g) ~ 2H1(g) is 50. If the volume of the
[Hint: XJ2(g) ~ XY(g) + Y(g) container is reduced to one half of its original value, the
600 - x x x equilibrium constant wil1 be:
600 + x = 800 or x = 200 ] (a) 25 (b) 50
86. Which of the fol1owing reactions wil1 be favoured at low (c) 75 (d) 100
pressure? 95. C(s) + H 2 0(g) ~ CO (g) + H2 (g)
(a) H2 (g) + 12(g) ~ 2H1(g) The above equilibrium when subjected to pressure:
(b) N 2(g) + 3H 2(g) ~ 2NH}(g) [PMT (Raj.) 1997.)
(a) remains unaffected
(c) PCis(g) ~ PCi 3 (g) + Ci 2(g)
(b) proceeds in the backward direction
(d) N 2(g) + 02(g) ~ 2NO(g)
(c) proceeds in the forward direction
87. According to Le Chatelier's principle, adding heat to a solid
and liquid in equilibrium will cause the: (MLNR 1990) (d) none of the above
CHEMICAL EQUILIBRIUM 635

96. Reaction, A + B -----7 C + D + 38 kcal has activation energy (b) solid and liquids react slowly
20 kcal. Activation energy for the reaction, (c) solid and liquids at equilibrium do not interact with
C + D -----7 A + B is: [PMT (Pb.) 1993] gaseous phase
(a) 20 kcal (b) -20 kcal (d) the molecules of solids and liquids cannot migrate to the
(c) 18 kcal (d) 58 kcal gaseous phase
97. The equilibrium constant for the reaction, 105. For an equilibrium reaction involving gases, the forward
CaS04 ·5H2 0(s) ~ CaS0 4 ·3H2 0(s) + 2H 2 0(g), reaction is first order while the reverse reaction is second
is equal to: order. The units of K p for forward equilibrium is:
(a) [CaS04 ·3H2 0][H2 0f (b) [CaS04 ·3H2 0] (a) atm (b) atm 2 (c) atm- I (d) atm- 2
[CaS04 ·5H2 0] [CaS04 ·5H2 0] 106. At temperature T, a compound ABo (g) dissociates according·
t9 the reaction: •
(c) [H 20]2 (d) [H 2 0]
2AB2(g)~ 2AB(g)+B2(g)
9S. One mole ofN2 0 4(g) at 300 K is kept in a closed container
with a degree of dissociation 'x' which is small as compared to
under one atmosphere. It is heated to 600 K when 20% by
unity. The expression for K p' in terms of 'x' and total pressure
mass of N 2 0 4(g) decomposes to N02 (g). The resultant
'P'is
pressure is: 3 2
(a) 1.2 atm (b) 2.4 atm (c) 2.0 atm (d) 1.0 atm (a) Px (b) Px
99. The equilibrium constant for the reaction" 2 3
3 2
N 2 (g) + 02(g) ~2NO(g) is 4.0 x 10-4 at 2000 K. I~the (c) Px (d) Px
presence of a catalyst the equilibrium is attained ten times , 3 2
faster. Therefore, the equilibrium constant in presence of the 107. 40% mixture of 0.2 mole of N2 and 0.6 mole of H2 react to
catalyst at 2000 K is: (MLNR 1994) give NH3 according to the equation:
(a) 4 x 10-4 • N 2 (g) + 3H2 (g) ~ 2NH3(g)
(b) 4Cl x 10-4 at constant temperature and pressure. Then the ratio of the
(c) 4 X 10-2 final volume to the initial volume of gases is:
(d) difficult to compute without more data (a) 4: 5 (b) 5 : 4
100. The equilibrium constant for a reaction, A + B ~ C + D'is (c) 7: 10 ' (d) 8 : 5
I x 10-2 at 298 K and is 2 at 273 K. The chemical process lOS. Two systems,
resulting in the formation of C and Dis:
PCIs(g) ~ PCI 3 (g) + CI 2 (g)
(a) exothenuic
and COCI 2 (g) ~ CO(g) + CI 2 (g)
(b) endothermic
(c) unpredictable are simultaneously in equilibrium in a vessel at constant
(d) there is no relationship between Ml and K volume. If some CO(g) is introduced in the vessel at constant
101. The equilibrium constant for the reaction, A + B ~ C + D volume, then at new equilibrium, the concentration of:
is 2.85 at room 'temperature and 1.4 x 10-2 at 698 K. This (a) PCls is greater (b) PC1 3 remains unchanged
shows that forward reaction is: (c) PCls is less (d) CI 2 is greater ,
(a) exothermic 109. For the reaction, [Ag(CN}zr ~ Ag+ + 2CN-, the
(b) endothenn1c equilibrium constant Kc at 25°C is 4 x 10- 19 ; then Ag+
(c) unpredictable concentration in a solution which has 0.1 M KCN and 0.03 M
AgN0 3 is:'
(d) there is no relationship between I'1H and K
102. If Er and Er are the activation energies of forward and (a) 7.5x lOIS (b) 7.5 x 1O- lg
backward reactions and the reaction is known to be (d) 7.5 x 10- 19
exothermic, then:
110. When NaNO} is heated in a closed vessel, oxygen is liberated
(a) Er > Er and NaN0 2 is left behind. At equilibrium:
(b) E f < Er (a) addition of NaN0 2 favours reverse reaction
(c) E f = Er (b) addition of NaN0 2 favours forward reaction
(d) no relation can be given between Er and Er
(c) increasing temperature favours forward reaction
103. K p for a reaction at 25°C is 10 atm. The activation energy for
(d) decreasing pressure favours reverse reaction
forward and reverse reactions are 12 and 20 kJ/mol
111. For the reaction,
respectively. The K c for the reaction at 40°C will be:
CuS04 ·5H2 0(s) ~ CUS04 ·3H2 0(s) + 2H 2 0(g)
(a) 4.33 x 10- 1 M (b) 3.33 x 10-2 M
which one is the correct representation?
(c) 3.33 10- 1 M (d) 4.33 x 10-2 M
(~) K p = [ PH 2 0f
X
(b) Kc = [H 20]2
104. Concentration of pure solid and liquid is not included in the
expression of equilibrium constant because: (c) K p = K c CRT)2 (d) All of these
(a) solid and liquid concentrations are independent of their 112. Which one is the correct representation for,
quantities 2S0 2 (g) + 02(g) ~ 2S0 3(g)?
636 G.R.B. PHYSICAL CHEMISTRY FOR COMPETITIONS

121. If340g of a mixture of N2 and H2 in the correct ratio gave a


.20% yield ofNH 3, the mass produced would be:
(ii},-16 g (b) 17 g (c) 20 g (d) 68 g
122. If the pressure ofN 2 I H2 mixture in a closed apparatus is 100
atm and 20% of the mixture then reacts, the pressure at the
2 same temperature would be:
(c) K = [ns0 3l x[ p ] -1 (a) 100 (b) 90 (c) 8S (d) 80
p [nso)2[n01] Total mole 123. The equilibrium constants for the reaction, Br2 ~ 2Br, at
(d) All of the above SOO.K and 700 K are I x 10- 10 and I x IO- s respectively. The
reaction is:
113. For the reaction,
(a) endothermic (b) exothermic
PCIs(g) ~ PCI3(g) + CI 2(g) (lIT ]99])
The forward reaction at constant temperature is favoured by: (c) fast (d) slow
(a) introducing inert gas at constantvoluI1)e 124. If the concentration ofOH- ions in the reaction:.
(b) introducing chlorine gas at constant volume Fe(OHh(s)~ Fe 3+(aq.) + 30Ir(aq.)
is decreased by y;; times, then equilibrium concentration of
(c) introducing an inert gas at constant pressure ,.
Fe 3+ will increase by: [CBSE.(PMT) 2008)
(d) none of the above
(a) 64 times (b) 4 times (c) 8 times (d) 16 times
114. In a flask colourless N20 4 is in equilibrium with browri
coloured N0 2. At equilibrium, when the flask is heated to
125. K; for A + B ~ C + Dis' 10 at 2SoC. If a container
contains I, 2, 3 and 4 mol per litre of A, B, C and D
100°C the brown colour deepens and on cooling, the brown
respectively at 2SoC, the reaction shall:
colour became less·coloured. The change in enthalpy tJf for
the system is: (KCET ]993) (a) proceed from left to right (b) proceed from right to left
(a) negative (b) positive (c) zero (d) not defined (c) be at equilibrium (d) none of these
115. Le Chatelier's principle is not applicable to: 126. In the preparation of CaO from CaC0 3 using the equilibrium,
(a) Fe(s) + S(s) ~ FeS(s) CaC0 3(s) ~ CaO(s) + COi(g)
(b) H2(g) + I 2(g) ~ 2Hl(g) K p is expressed as:
(c) N2(g) + 3H2(g) ~ 2NH 3(g) log K p = 7.282 _ 8S00
(d) N2(g) + 02(g) ~ 2NO(g) T
116. Densities of diamond and graphite are 3.S and 2.3 g/mL For complete decomposition of CaC03 the temperature in
respectively. Increase in pressure on the equilibrium celsius to be used is:
Cdiamond ~ Cgraphile:
(a) 1167 (b) 894
(a) favours backward reaction (c) 8S00 (d) 8S0 , ........ -
(b) favo'lrs forward reaction 127. To the system,"'~
(c) has no effect LaCI 3(s) + H20(g) ~ LaCIO(s) + 2HCI(g) - hea~
(d) increases the reaction rate already at equilibrium, more water vapour is added without
117. For tht: reaction, N2 + 3H 2 ~ 2NH3 in a vessel, after the altering T or V of the system. When equilibrium is
addition of equal number of moles of N2 and 'H 2 equilibrium re-established, the pressure of water vapour is doubled. The
state is achieved. Which of the following is correct? pressure of Hel present in the system increases by a factor of:
(a) [H2l = [N 2l (b) [H2l < [N 2l (a) 2 . (b) il2 (c) 3 (d) 4
(c) [H2] > [N 2l (d) [H2l> [NH3l
128. For the chemical reaction~
118. If pressure is applied to the equilibrium of solid ~. liquid,
. 3X(g)+ Y(g)~ X 3 y(g)
the melting point of the solid:
the amount of X 3Y at equilibrium is affected by: (lIT] 999)
(a) will not change
(a) temperature and pressure
(b) may increase or decrease depending upon its nature
(b) temperature only
(c) will always increase
(c) pressure only
(d) will always decrease
(d) temperature, pressure and catalyst
119. If concentrations of N2, H2 and NH3 are I, 2 and' 3
respectively, their concentration at equilibrium will be: 129. In a SOO mL capacity vessel CO and CI? are mixed to form
COCI 2. At equilibrium, it contains 0.2 ~ole of COCI 2 and
N2 + 3H2 ~ 2NH3
0.1 mole of each of CO and C1 2. The equilibrium constant K
(a) (1- x) (2- 3x)
for reaction, CO + CI 2 ~ COCI 2 is:
2x
(eBSE 1998)
(b) (1- xl3) (2 - x) 2xl3
(a) S (b) 10 (c) IS (d) 20
(c) (I-x) (2-x) (3+x)
130. The partial pressures ofCH 30H, CO and H2 in the equilibrium
(d) (I-x) (2-3x) (3+2x)
mixture for the reaction,
120. A reaction attains equilibrium, when the free energy change is: CO + 2H2 ~ CH 30H
(a) zero (b) positive and large
at 427°C are 2.0, 1.0 and 0.1 atm respectively. The value of K
(c) negative and small (d) negative and large for the decomposition ofCH 30H into CO and H2 is: P

(lIT 1999)
I CHEMICAL EQUILIBRIUM 63~

(a) 'Ix 102 atm (b) 2x 1()2 atm- I dM.IS +ve ..lor t E (0 , ) where, M is average molar
( c)
'dt
(c) 50 atm 2 (d) 5 x 10-3 atm 2
mass of the reaction mixture
131. 8 mole of a gas AB3 are introduced into a 1.0 dm 3 vessel. It
(d) total gaseous weight at equilibrium is less than 68 g
dissociates as:
136. A vessel (volume = 2 L) contains 60 g of water gas. When
2AB3(g)~ A2(g)+ 3B 2 (g)
steam is passed through the vessel, the reaction,
At equilibrium, 2 mole of A2 are found to be present. The
CO + H20~ COz + Hz occurs and equilibrium is attained:
equilibrium constant of the reaction is: (lIT 1997)
2
Ca) 2 mol L- 2 2
(b) 3 mol L- z (a) n(H z ) < I (at equilibrium)
n(CO)
(c) 27 mol 2 L -2 (d) 36 moe L-2
(b) n(H2 ) + n(CO) = 4 (at any instant)
132. At constant temperature, the equilibrium constant (K p ) for the dAf f .
decomposition reaction, (c) - dt > 0 (where, M is average molar mass 0 gas mixture
N204~2N02 before the attainment of equilibrium)
is expressed by:
(d) dM < 0
dt .
137. A vessel contains CO 2 and CO with pressures 2 atm and 3 atm
Where, P pressure, x = extent of decomposition. Which one respectively at 27°C. At a temperature of 2727°C, the reaction
of the following statements is true? (lIT (Screening) 20001 2CO(g) COz(g) + C(s) occurs and equilibrium -is~~
(a) K p increases with increase of P attained. If the eqUilibrium pressure is 45 atm, then:
(b) K p increases with increase of x (a) K p = 2/9 atm- 1
(c) K p increases with decrease of x
(b) Pea: Pea) = 4: 5 at equilibrium
(d) K p remains constant with change in P and x (c) K p 16 atm- I
133. 0'" ;der the reactions,
(d) % dissociation of CO = 0.333
N02(g)~ ~ N 2(g) + 02(g) KI 13.8. A vessel (volume 8.2 L) contains Hz (g) at 2 atm pressure.
2
, When H 2 S(g) at a pressure of 4 atm is introduced into the
N 20 4 (g) 2N0 2 (g) K2
vessel, the reaction, 8HzS(g) ~ 8Hz(g) +. S8 (s ) occurs at a
Give the equilibrium constant for the fonnation ofNz0 4 from
°
N z and 2 : . (DeE 2006)
temperature of 2000 K, It is found that:

I
(a)-x
1 1
(b)-+-(c)
I
(d)-
K2
[
n (Hz) J [n(H 2 S)]
Kj2
K2 2K) K2 K] n(H 2 S) at equilibrium n(H2) at 1=0
1 Ca) maximum weight of solid fonned is 32 g
[Hint: N0 2 (g) 2 N 2 (g) + 02(g) K j
(b) maximum weight of solid fonned is 0.32 g
.. N 2 (g) + 20 2 (g) ~ 2N0 2 (g) K = ~2 ••. 0) (c) Kp =Kc RT
I (d) Kc = 256
139. N2 (g) and H 2 (g) are taken in a vessel in inass ratio of 7 : 1.
The only reaction N2 + 2H2 --->. N2H4 (g) occurs. Pressure
.. 2N02(g)~N204(g) ... (ii)
due to N zH 4 at equilibrium is 0.2 times of total pressure' P'.
Adding eqs. (i) and (ii), Then at equilibrium:
N 2(g) + 202(g)~N204(g) (a) partial pressure ofN2 2P 115
(b) partial pressure of Hz 8PI25
134; Phosphorous pentachloride dissociates as follows: (c) 2PNz PH z
PCI5(g)~ PCI 3 (g) + CI2(g). (d) PN) =
If total pressure at equilibrium is P and the degree of 140. In the equilibrium, S02CI2 ~ S02 + CI 2 at 2000 K and 10
dissociation of PC Is is x, the partial pressure ofPCl 3 will be: atrn pressure, % Clz % S02 = 40 by volume. Then:
(AIEEE 2006)
(a) Kp = 2 atrn
(a) x \IP (b)
+1) \I-x
r~\Jp ec) (lx~]p
I)
(d)
I
x , n(S02 Cl ?)
(b) - = -1 at eqUl'l'b .
1 num
n(S02) 4
135. At t = 0 a vessel (volume 1 litre) contains 1 mole N z, 3 mole (c) Kp = 8atm
Hz and 2 mole NH3 . The value of K c for N z + 3H2 ~ 2NH) (d) n(SOCI 2 ) n(SOz)=n(CI 2 )
is 17.5 LZ mol-2 • Then: ,
141. A 20 litre box contains and O? at equilibrium at 202 K.
tal LUI,,1 ~aseous weight at equilibrium is more than 68 g 14
Kp = 2x 10 for 203 302' ~Assume that Po 2 » Po)
(b) total r.··m.:'er of moles (gaseous) at equilibrium are more and total pressure is 8 atm; the partial pressure of 0 3 IS:
than 6 moles (a) exactly 1.6 x 10-6 atm
(b) s~fficiently less than 1.6 x 10-6 atm
638 G R.B. PHYSICAL CHEMISTRY FOR COMPETITIONS

(c) slightly more than 1.6 x 10-6 atm


(d) very slightly less than l.6 x 10-6 atm
142. For. the reactions, A B,B ~ C and C ~ D,
equilibrium constants are K l' K 2 and K 3 respectively. What is
(b) NaCI
i
log S
(c) sucrose
. the value of equilibrium constant for A ,---" D ?
(a) KI + K2 + K3 (b) KI x K2 X K3
(d) CaO
(c) K]K l I3 (d) None of these 1/T - -

143. AO~UJ A2 + UJ 02;K = 5x 10 5


°II
149. CH 3 - C- CH 3 (g) ~ CH 3 CH 3 (g) + CO(g)
BO (~J B2 + r~I 02; K = 1.1 Ox 10
12 Initial pressure ofCH 3 COCH3 is IOOmm. When equilibrium
l2 \2; is achieved, the mole fraction of CO(g ) is 113 hence, K pis:

CO~U)C2 +UJ02; 8 (a) 100 mm (b) 50 mm (c) 25 mm. (d) 150 mm


K 2.3x 101 150. In which of the following equilibrium, change in the volume
of the system does not alter the number of moles?
DO 1 Dz +(~J 02; K := 1.4 x 10
21
(AIEEE 2002)
(a) N 2 (g) + 02(g) ~ 2NO(g)
Which oxide ismost s!able? (b) PCls(g)..-- PC1 3 (g)+C1 2 (g)
(a) AO (b) BO (c) CO (d) DO (c) N2 (g) + 3H2 (g) 2NH3(g)
144. Which graph will show equilibrium condition? (d) S02CI2(g) ..-- S02(g) + CI 2(g)
151. What are tlte most favourable conditions for the reaction;
i~_ S02(g) +.!.2 02(g)
. ~ S03(g);
.....Mfo .= - ve
(b)~L: to occur? [DPMT 2002; JEE (Orissa) :'\oc"'1
Time---- (a) low temp. and high press. (b) low temp. and low press.
(c) high temp. and low press. (d) high temp. and high press.
152. Consider the following reactions in which all the reactants and
(d) None of these products are in gaseous state
2PQ ~ P2 + KI = 2.5x 105
Time-
PQ+~R2 ~ PQR K2 = 5x 10- 3
145. For the reaction, N2 + O2 ~ 2NO equilibrium constant 2
K c := 2. Degrees of dissociation of N 2 and 02 are: The value of K 3 for the equilibrium:
( ) I _1_ (b) _1_ I -I p + I +-..--I R .---->. PQR IS:
.
a 1+ 'I-fi I - f i ' 1+ 2 2 2 2 [PET (Kerala) 20101
1 2 2 (a) 2.5xlO~3 (b) 25xlO3
(c) both are ----z::' (d) -- (c) 10xlO- 5
(d) 5x103
l+v2 'I-fi 3
(e) 5x 1.0-
146. A ~ 2B,Kp; C~D+E ,Kp" If degrees of
[Hint:
dissociation of A and C are same and K p 2K p', tlten the
ratio. of total pressure pi p' ? I.
PQ+-R2 PQR
(a) 1/2 (b) 113 (c) 114 (d) 2 2
147. Equilibrium constants for four different reactions are given as: 1
K j = 106 , K2 10-4, K3 := 10, K4 1. Which reaction will K3 --rXK2
KI'~
take maximum time to attain equilibrium?
5 x 10- 3
(a) KI 106 (b) K2 = 10-4
(2.5 x
(c) K3 = 10 (d) K4 = 1
= lX'10~5l
148. Solubility of a solute in water is dependent on temperature as
given by, 153. Consider the following equilibrium in a closed container,
S = Ae- Mf IRT where, Mi = heat of solution N2 0 4 {g) ~ 2N02 (g)
At a fixed temperature, the volume of the reaction container is
Solute + H2 0(l) Solution; Mi ± x
halved. For this change, which of the following statements
For a gioven solution, variation of log S with temperature is holds true regarding the equilibrium constant (K p ) and degree
shown graphically. Hence, solute is: of dissociation (ex )? (liT 2002)
(a) neither K p nor 'ex' changes
(b) both K p and 'ex' change
CHEMICAL EQUILIBRIUM I· 639

(c) K p changes but '0:' does not 162. Partial pressure of 02 in the reaction,
(d) K p does not change but '0:' changes 2Ag 2 0(s) ~ 4Ag(s) + 02(g)
154. For the reaction, PCI 3(g) + CI 2 (g) PCls(g), the position is: (DCE 2005)
of equilibrium can be shifted to the right by: (a) K p (b)jK; (c) (d) 2K p
[PET (MP) 2004]
163. Two moles of PCls are heated in a closed vessel of 2 litre
(a) doubling the volume
capacity. When the equilibrium is attained 40% of it has been
(b) increasing the temperature 3
found to be dissociated. What is the value of Kc in mol/dm ?
(c) addition of equimolar quantities ofPCl 3 and PCIs
[PET (KeraJa) 201151
(d) addition of Cl z at constant volume
(a) 0.532 (b) 0.266 (c)0.133 (d) 0.174
155. The most favourable condition for th~ manufacture ofNH3 is:
(e) 0.25
[CECE (SibaI') 2004]
[Hint: PCls(g) ---'- PCI 3 (g) + CI 2 (g)
(a) high temperature and high pressure 1=0 2 0 0
(b) low temperature and low pressure 2 - 08 0.8 0.8
- 2
- 2 2
(c) high temperature and low pressure
(d) low temperature and high pressure
156. The chemical equilibrium of a reversible reaction is not K =[PCI 3 ][C1 2 ] 0.64 = 0.266]
influenced by: (KCET 2004) c [PCls] 2.4
(a) catalyst (b) pressw'e 2
(c) temperature (d) concentration _..... ,.... 164. For the reaction:
157. Which of the following change will shift the' r~'i2tion"in' 2NOzeg) 2NO(g) + 02(g)
forward direction? . Kc = 1.8 X 10-6 at 184°C
R = 0,0831kJ K- 1 mol- 1
Iz(g) ~ 2I(g); ARo + 150 kJ (AIlMS 2004)
(a) Increase in total pressure when, K p and K c are compared at 184°C it is found that:
(b) Increase in temperature (AIEEE 2005)
(c) Increase in concentration of 1 (a) K p. is greater than K c
(d) Decrease in concentration of 12 (b) K p is less than K c
K (c) K p = Kc
158. For the reaction, CO(g) + CI 2 (g) COCI 7 (g) , the ---E.. is (d) whether K p is greater than, ,less than or equal to K c
• Kc
depends upon the total gas pressure
equal to: (AIEEE 2004)
[Hint: Kp = Kc (RTl"
(a) II RT (b) RT (c) ill (d) 1
Lln=3-2=1
159. The equilibriwn constant for the reaction,
Kp = Kc (RT)l
Nz(g) + 0z(g) 2NO(g)
at temperature T is 4 x 10-4 . .. Kp >Kcl
The value of K c for the reaction, 165. NH 4 COONH,(s) ---->. 2NH 3(g) + CO,(g). If equilibrium
pressure is 3.~tm for the abo~e reactio~ K p will be:
NO(g) ~ .!. N2 (g) + I 02(g) (DPMT 2005)
2 2
at the same temperature is: (a) 4 (b) 27 (c) 4/27 (d) 1/27
(AlEEE 2004)
(a) 2.5 x 102 (b) 50 (c) 4 x 10-4 Cd) 0.02 [Hint: PNH 3 : Pe02 = 2 : I

160. 2 moles of N2 are mixed with 6 moles of H2 in a closed vessel .. PNHJ = 2 atm: PC02 1 atm
of one litre capacity. If 50% N2 is converted into NH3 at Kp = [ PNH) ]2[pC02 ] = 22 X 1 = 4]
equilibrium, the value of K c for the reaction, 1.66. A + B ~ C + D. If initially the concentration of A and B
N 2 (g) + 3Hz{g) 2NH3(g) are both equal but at equilibrium, concentration of D will be
IPMT (Kerala) 2004] twice of that of A, then what will be the equilibrium constant
(a) 4127 (b) 27/4 of the reaction? [BHU (Pre.) 2005; JIPMER 20061
4 9
(c) 1127 (d) 27 (a) 9 (I'» 4 (c) 9I (d) 4
(e) 9
161. For the reaction, H2(g) + CO?(g) ..,-- CO(g) + H,O(g), if [Hint: A + B ~C+D
'/ = 0 a a () ()
the initial concentration of [H 2] [C0 2 ] and x mol/litre of H2 t eq . a x a x x x
is consumed at equilibrium, the correct expression of K pis: .
IJEE (Orissa) 2005] x 2(a x)
? J
;c 3x 2a
(a) ---;;:- (d)~
(I 1- x· xxx
K =xxx=4]
(a x)(a x) ~ x::
2 2
640 G.R.B. PHYSICAL CHEMISTRY FOR COMPETITIONS

167. 15 moles ofH? and 5.2 moles ofI? are mixed and allowed to 171. If the equilibrium constant for the reaction,
attain equilibri~m at 500°e. At eq~ilibrium, the concentration N2(g)+3H2(g)~ 2NH3(g)
of HI is found to be 10 moles. The equilibrium constant for the at 750 K is 49, then the equilibrium constant for the reaction,
formation of HI is: (KCET 2005)
(a) 50 (b) 15 (c) 100 (d) 25 NH3(g) ~ ~ N2(g) + ~ H1(g)
[Hint: H2 + I z ~2HI at the same temperature is: [PMT (Kerala) 2006)
1=0 15 5.2 o
t eq . 15 - 5 5.2 - 5 10 (a) 1149 (b) 49 (c) 7 (d) 49 2
(e) 117
K = [HIf =~-=50] 172. The equilibrium constant for the reaction,
c [H z ][I2] lOxO.2
168. For the reaction: 2NOCl(g) ~ 2NO(g) + CI 2(g), Kc at
S03(J!)~ SO?(g)
,~ - + ~2O;(g)
-
427°C is 3 x 106 L mol-I.. The value of K p is nearly: isKc =4.9x 10-2.
. (AIIMS 2005)
The value of K c for the reaction,
(a) 7.5 x 10-5 (b) 2.5 x 10- 5 (c) 2.5 X 10-4 (d) 1.75 x 10-4
2S0 2(g ) + O 2 (g ) ~ 2S03(g )
169. For the chemical equilibrium, will be: (AIEEE 2006)
CaC03(s) ~ CaO(s) + CO 2(g) (a) 416 (b) 2.40 x 10- 3
!JJi~ can be detelmined from which one ofthe following plots?
(c) 9.8 X 10-2 (d) 4.9 x 10-2
(AIIMS 2005)
[Hint: S03(g) ~ S02(g) + ~ 02(g); Kc = 4.9 10-2

r
X

t0'
u
c..
Q)
OJ
.Q

1/T-
/ T-
2S02(g) + °2(g) ~ 2S0 (g);
3 Kc = [ 4.9

173. For a reaction, H2(g)+I2(g)~ 2HI(g) at 721 K, the


~ 10-
= 416.5 '" 416]

value of equilibrium constant is 50. If 0.5 mol each of H2 and


2

(a) (b) I? is added to the system, the value of equilibrium constant


~i1l be: [VMMC (Safdarjung) 2006]
(a) 40 (b) 60 (c) 50 (d) 30

0'
u
c..
OJ
OJ
.Q

10gT-
/ 1/T-
[Hint: . Equilibrium constant does not change on adding the
reactant.]
174. In the reaction, H2 (g) + 12 (g) ~ 2HI(g), in a 2 litre flask
0.4 mole of each Hz and 12 are taken. At equilibrium 0.5 mole
of HI are formed. What will be the value of equilibrium
constant Kc? [PMT (Raj.) 2006]
(c) (d)
(a) 20.2 (b) 25.4 (c) 0.284 (d) ILl
[Hint: CaC0 3(s) ~ CaO(s) + CO 2(g)
175. For the reaction, Hi (g) + 12 (g) ~ 2HI(g), the equilibrium
Kp = Peol constant K p changes with: (VITEEE 2007)
According to Anhenius equation: (a) total pressure (b) catalyst
(c) the amount ofH 2 and 12 (d) temperature
K = Ae-MJ,"IRT
176. Given the equilibrium system,
!JJi" NH4Cl(s)~ NH;t(aq.) + Cl-(aq.)
10gKp=logA- ,.
2.303RT . (!JJi 0 = + 3.5 kcall mol)
!JJi" 1 what change will shift the equilibrium to the right?
loa P, = log A - --'_. - ... (i) (VITEEE 2007)
'" CO2 2.303R T
(a) Decreasing the temperature
Y=C + MX
(b) Increasing the temperature
Graph (a) represents (i) and its slope will be used to determine the (c) Dissolving NaCI crystals in equilibrium mixture
heat of the reaction.] (d) Dissolving NH4 NO, crystals in the equilibrium mixture
170. In gaseous reversible reaction, 177. The equilibrium constant (K p) for the decomposition of
N2(g)+ 02(g)~ 2NO(g)+ Heat gaseous H20 :
If pressure is increased, then the equilibrium constant would
be: [UGET (Manipal, Medical) 2006) H 20(g ) ~ H1 (g ) + ~ 0 1(g) is related to degree of
(a) unchanged dissociation (CI. ) at a total pressure P is given by:
(b) increased [PMT (Kerala) 20071
(c) decreased
(d) sometimes increased, sometimes decreased
CHEMICAL EQUILIBRIUM 641

< a 3 p 1I2 (2 +x)z2, (I-x)';" 1


(a) K =-----;-:-;:- (b) Kp 3
. p 0+0.)(2+0. KPx === :. x
a 312p2 P 2
(c) K = -----;-:-;:- (d) Kp 181. In which of the following reactions, the concentration of the
p (1- a )(2+ a (1- a )(2 + a F2 product is higher than the concentration of reactant at
2
a p equilibrium? (K = equilibrium constant) (AIL'lS 2008)
(e) K p = ~
(a)A~B; K 0.001 (b)M~N; K 10
, -;2
(c) X ~ Y; K "" 0.005 (d) R ~ P; K 0.01
[Hint: H 2 0(g)
182. 1.6 mole of PC15 (g) is placed in 4 dm 3 closed vessel. When the
t=O o o temperature is raised to 500 K, it decomposes and at
equilibrium 1.2 mole ofPCI 5 (g) remains. What is the K c value
for the decomposition of PCIs(g) to (g) and CI 2(g) at
500K? O\:<>falai ::;(!:q
(a) 0.013 (b) 0.050 (c) 0.033 (d) 0.067 .'
(e) 0.045
183. Consider the following statements regarding chemical
<

equilibrium:
1. For the gaseous reaction, the equilibrium can be establis!! :i
in open vessel.
2. The state of equilibrium is dynamic in nature.
3. If temperature is kept constant, the colour of the reacting
system changes with time.
(1 0.)(2 +
Which ofthe statements given above is/are correct?
178. The equilibrium constant for the reaction, ' 20(P9)
2N02(g)~ 2NO(g)+ 02(g)is2x 10-6 at 185"C (a) 1 < (b) 2 only
Then. the equilibrium constant for the reaction,
(c) 3 only (d) 2 and 3 only
4NO(g) + 202 (g ) ~ 4NO z(g) at the same temperature
]84. The dissociation constants for acetic acid and HCN at 25°C
would be: [PET (Kerala) 2007)
are 1. 5 x 10- 5 and 4.5 x 10- 10 respectively. The equilibrium
(a) 2.5 x 10-5 (b) 4 x lO- IZ (c) 2.5 X 1011 (d) 2 x 106 constant for the equilibrium:
(e) 5x lOS eN" +CH3COOH~ HCN + CH 3 COO-
179. 1 mole ofH2 and 2 mole oflz are taken initially in a 2 L vessel. would be : [eESE (PMT) 2(09)
4
The number of moles Hz at equilibrium is 0.2. Then, the (a) 3x 10 (b) 3x 105
number of moles OfI2 and HI at equilibrium are: (c) 3xlO-5 (d) 3xlO-4
[PMT (Raj.) 2007] [Hint: Given
(a) 1.2, 1.6 (b) l.8, l.0 (c) 0.4,2.4 (d) 0.8,2.0 (i) CH3COOH~CH3COO- +W KI === 1.5x 10-5
180. The dissociation equilibrium of a gas AB z can be represented
(ii)< HCN~ W +CW K2 =4.5xlO- 10
as,
1 1
2AB 2 (g) 2AB(g)+ Bz(g) (iii) W + Cw ~ HCN
The degree of dissociation is x and is small as compared to 1.
The. expression relating the degree of dissociation (x) with Adding (i) and (iii) we get
equilibrium constant K p and total pressure Pis: <
CH3COOH + CW~ CH3COO- + HCN
(a)(2Kplp)1I2 (b)K/P (c)2K/P' (d)(2K/P),13<
K1. 5 x 10-5 X --:----;-;::-
rCBSE (PMT) 20081 4.Sx
(Hint: 2AB2 (g) ~ 2AB(g) + B2 (g) = 3x 104]
to 2 0 0 185. One mole of N 2 0 4 is heated in a flask with a volume of 0.1
teq < 2x x Total moles 2+x . dm3 . At equilibrium, 1.708 mole ofNOz and 0.146 mole of
Partial 2x x NZ0 4 were found 134°C. The equilibrium constant will be:
pressure xP xP xP
2+x 2+x 2+x 20(l9)
(a) 250 mol dril-3 (b) 300 moldm-3
(c) 200 mol dm- 3 (d) 230 mol dm-3
186. What is the effect of a tenfold increase in pressure on K p in the
reaction?
N 2 (g)+ 3H2 (g) ~2NH3(g)
At equilibrium:
4 (2 + x)(l- x)2
I
642 I G.R.B. PHYSICAL CHEMISTRY FOR COMPETITIONS

(a) a tenfold increase (b) a tenfold decrease 10. Which of the following will not affect the value of equilibrium
(c) no change Cd) equal to Kc constant of a reaction?
(a) Change in the concentration ofthe reactants
Set-2: The Questions given below may have more (b) Change in temperature
than one correct answers (c) Change in pressure
(d) Addition of catalyst
1. Unit of equilibrium constant is: 11. Which ofthe following statements is/are wrong?
(a) (moIlL)1 -" (b) (moIlL)8n (a) At equilibrium, concentrations of reactants and products
(c) (atm}811 .(d) all of these become constant because the reaction stops
2. Which is/are correct? (b) Addition of catalyst speeds up the forward reaction more
(a) 2.303 log K - 6.1[O/RT + !:J.so/R than the backward reaction
(c) Equilibrium constant of an exothermic reaction decreases
(b) !:J.GO = - 2.303RT log K
with increase of temperature
(c) 2.303 log K = -!:J.H°/RT 2 + t:J,S°/R (d) K p is always greater than Kc
(d) 2.303 log K = (1/ RT )(!:J.H + !:J.SO)
0 12. For the gas phase reaction,
3. For the reaction, CaC03 (s) -7 CaO(s) + COz(g), which is C zH4 + Hz C 2 H6 ;!:J.H° = 136.8 kJ mol-I
correct representation? carried out in a vessel, the equilibrium concentration of C 2H 4
(a) K p =(Pco2 ) (b) Kp=KcCRT) can be increased by: ISHU ~i\'h.:.'·' 2008l
(c) Kc = (C02 )/1 (d) All of these (a) increasing the temperature

4. Nz +0 ~ 2NO, K,{i) N2 + u·) O


2 2 ~ NO, K2 ; (b) decreasing the pressure
(c) removing some Hz

2NO---'" N2 + 02' K 3; NO~ G) (±) °


Nl + 2, K4
(d) adding some C 2 H6
13. When NaN03 is heated in a closed vessel oxygen is liberated
and NaN02 is left behind. At equilibrium, which are not
...: Correct relation(s) between K j , K 2 , K 3 and K4 is/are:
correct?
(a) K, x K3 =1 (b) fK: x K4 = 1 (a) Addition of NaN0 2 favours reverse reaction
(c) .[K; X K z = I (d) none of these (b) Addition of NaN03 favours forward reaction
5. For the reaction, N2 0 4 ~ 2N02 , if degree of dissociation (c) Increasing temperature favours forward reaction
of N2 0 4 are 25%, 50%, 75% and 100%, the gradation of (d) Increasing pressure favours reverse reaction
observed vapour densities is: 14. An increase in temperature increases which of the following?
(a) d t > d 2 > d 3 > d4 (b) d 4 > d 3 > d 2 > d l I. The rate constant of a reaction
(c) d, = d 2 = d 3 = d4 (d) none of these 2. The ionic product of water
3. The equilibrium constant of an exothermic reaction
6. The equation, a = D d is correctly matched for: Select the correct answer using the code given below:
(n - l)d
(SCRA 2007)
(a) A~nBI2+nC/3 (b) A nB/3+(2n/3)C (a) 1 and 2 only (b) 1 and 3 only
(c)A-7(n/2)B+(nI4)C (d) A~(nI2)B+C (c) 2 and 3 only (d) 1,2 and 3 only
7. For a reaction, nA A,l' degree of dissociation when A 15. In the following question, more than one answers are correct.
trimerises is: Select the correct answer and mark it according to the codes:
(a)2(d~D) (b)~(d~D}C)~(d d D }d)2(D dd) Codes;
(a) 1,2 and 3 are correct
IBHU (Mains) 20HI1
(b) 1 and 2 are correct
8. For the reaction, ( c) 2 and 4 are correct (d) 1 and 3 are correct
PCIs(g) Water gas, an industrial fuel, consisting CO and H2 in
the forward reaction at constant temperature is favoured by: equimolar amounts is obtained by passing steam over red-hot.
(a) introducing an inert gas at constant volume carbon in accordance with the" reaction:
(b) introducing chlorine gas at constant volume C(s)+ H20(g)~ CO(g)+ H2(g);
(c) introducing an inert gas at constant pressure !:J.H +l30.5kJ
(d) increasing the volume of the container The yield of water gas can be increased by
(e) introducing PCIs at constant/volume (1) -7 Reducing the total pre sure of the system
9. Which of the following will favour the formation of NH3 by (2) -7 Increasing pressure of steam
Haber's process? (3) -7 Raising the temperature
(a) Increase oftemperature (b) Increase of pressure (4) -7 Introducing hot carbon
(c) Addition of catalyst (d) Addition of promoter
CHEMICAL EaUIUBRIUM

Assertion-Reason TYPE QUESTIONS


The questions given below consist of two statenients each printooas (R) Addition of inert gas to the equilibrium mixture at
'Assertion' (A) and 'Reason' (R). While answering these questions constant volume, does not alter the equilibrium.
you are required to choose anyone of the following'rour: 6. (A) At equilibrium flG ::; 0
(a) If both (A) and (R) are correct and (R) is the correct (R) flGo= -RTlogeKc at equilibrium.
explanation for (A). 7. (A) At equilibrium flG::; 0
(b) Ifboth (A) and (R) are correct and (R) is not the correct (R) The Gibbs free energy of reactants and products decreases
explanation for (A). and become equal at equilibrium.
(c) If (A) is correct but (R) is incorrect. 8. (A) Equilibrium constant of a reaction changes on changing
(d) If (A) is incorrect but (R) is correct. the stoichiometric coeffidents of the reaction. '
1. (A) K p can be equal to or less than or even greater than the (R) The reactions: N2 (g) + 3H2(g ) 2NH3 (g )
valueofK c ' and.!. N2 (g) + ~ Hz (g ) ~ NH3 (g) have same equili-
(R)K p -:: Kc (RT)!!.n 2 2
Relation between K p and K c depends on the change in the brium constant.
number of moles of gaseous reactants and productl!. 9. (A) The Qc for the reaction:
2. (A) For N 2(g) + 3Hig) ~ 2NH3(g), the equilibrium
2802 (.g )+ O2 (g ) ,...--2:. 2S03 (g) is Qc = [SO] ]2 _
constant is 'K'then for.!. Ni(g) + ~H2(g) ...----"NH3(g) [S02] [Oz.;
2 . 2
(R) When Qc -< Kc, the reaction is not at equilibrium at --l it
the equilibrium constantwill be JK,
(R) If concentrations are changed to half the equilibrium . in backward direction.
will be fast .
10. (A)The equilibrium of[PCls(g)~ PCI 3 (g)+ Clz(g)] is
constant will be halved.
not affected by changing the volume of container. '
3. (A) A catalyst does not influence the values of equilibrium
'(R)K ::;[PCI 3 ][CI2 ]
constant. .
c [PCl s]
(R) Catalysts influence the rate of both forward and backward
reactions equally. Equilibrium constant K c does not depend on the volume
4. (A) The active mass of pure solid and pure liquid is taken of vessel.
unity. 11. (A) The reaction: N2 (g)+ 3H2 (g) ~ 2NH3 (g) will shift
(R) The active mass of pure solids and liquids depends on in forward direction by increase in pressure.
density and molecular mass. The density and molecular (R) On increasing the pressure, the equilibrium will shift in
mass of pure liquids and solids are constant. forward direction.
5. (A)FQrPCIs(g) ~ PCI 3(g) + CI 2 (g). If more Cl 2 is added
the equilibrium will shift in backward direction hence,
equilibrium constant will decrease.
644 I G.R.B. PHYSICAL CHEMISTRY FOR COMPETITIONS

"",:- - .

[t!~ .,::'~oeJEcTlvE
~.
~:'-~;r .. 'f

.. - -. > QUESTIONS" ~-;-~.-_:: :;:-'::~-i~.,·-·-L:.- -'-~--\,,- '.'-u.-_~L;-'<,r~:;r>;'--·_.

• Set-1
1. (b) 2. (d) 3. (e) 4. (e) 5. (a) 6. (a) 7. (b) 8. (d)
9. (d) 10. (b) 11. (e) 12. (a) 13. (d) 14. (d) 15. ,(b) 16. (a)
17. (b) 18. (e) 19. (e) 20. (d) 21. (a) 22. (b) 23. '(d) 24. (d)
25. (e) 26. (e) 27. (a) 28. (b) 29. (d) 30. (a) 31. (a) 32. (e)
33. (d) 34. (d) 35. (d) 36. (b) 37. (d) 38. (e) 39. (d) 40. (b)
41. (a) 42. (e) 43. (b) 44. (d) 45. (d) 46. (e) 47. (b) 48. (b)
49. (e) 50. (a) 51. (d) 52. (b) 53. (b) 54. (e) 55. (d) 56. (a)
57. (d) 58. (b) 59. (a) 60. (e) 61. (e) 62. (a) 63. (b) 64. (b)
65. (d)· 66., (d) 67. (a) . 68. (e) 69. (b) 70. (d) 71. (a) 72. (e)
73. (b) 74. (a) 75. (d) 76. (d) 77. (a) 78. (b) 79. (e) 80. (e)
81. Cd), '82. (a) 83. (d) 84. (e) 85., (b) 86. (e) 87.-(a) 88. (b)
89. (e) 90:;(a) 91. (d) 92. (b) 93. (a) 94. (b) 95. (b) . 96. (d)
97. (e) 98~ '(b) . 99. (a) 100. (a) 101. (a) 102. (b) 103. (e) 104. (a)
105. (a) 106. (a) 107. (a) 108. (e) 109. (b) no. (e) 111. (d) 112. (d)
113. (e) 114. (b) 115. (a) 116. (a) 117. (b) 118. (b) 119. (d) 120. (a)
121. (d) 122. (b) 123. (a) 124. (a) 125. (a) 126. (b) 127. (b) 128. (a)
129. (b) 130. (d) 131. (e) 132. (d) 133. (a) 134. (a) 135. (e) 136. (b)
137. (b) 138. (d) 139. (e) 140. (e) 141. (d) ·142. (b) 143. (d) 144. (e)
145. (e) 146. (a) 147. (b) 148. (d) 149. (b) 150. (a) 151. (a) 152. (e)
153; (d) 154. (d) 155. (d) 156. (a) 157. (b) 158. (a) 159. (b) 160. (a)
161. (a) 162. (a) 163. (b) 164. (a) 165. (a) .166. (d) 167. (a) 168. (d)
169-. (a) 170. (a) 171. (e) 172. (a) 173. (e) 174. (d) 175. (d) 176. (b)
177. (d) 178. (e) 179. (a) 180. (d) 181. (b) 182. (e) 183. (d) 184. (a)
185. (e) 186. (e)

• Set-2 2. (a, b) 3. (d) 6. (b) .


1. (b, e) --" ~ 4. (a, b, e) 5. (a) 7. (b) 8. (e, d, e)
9.. (b, e, d) 10. (a, b, d) 11. (a, b, d) . 12. (a, b, e, d) , 13. '(a, b, d) 14. (a) 15. (a)
• p.-', '.', ':-,"-'" -,- ,. ,-,-- ,- .,', - , .

['Jt~:::~ERTI()~..~SON·~P.Q~~--------------t

1. (a) 2. (e) 3. (a) 4. (a) • 5. (e) 7. (a) 8. ,(e)


, 9. (e) 10. (d) U. (a)
I
CHEMICAL EQUILIBRIUM 645

l OBJECTIVE QUESTIONS ) for (lIT ASPIRANTS I


1. Ammonium carbamate when heated to 20(J'C gives a mixture
of NH3 and CO2 vapour with a density of 13. What is the
degree of dissociation of ammonium carbamate?
[eEE (Kerafa) 20041
-
(c) concentration of ammonia does not change with pressure
(d) K p changes significantly with pressure
6. The equilibrium constant for the decomposition of water,
[HzO(g) ~ Hz(g) + °z(g)]
I
"2
3
(a) - (b) ..! (c) 2 (d) 1
. 2 2 is given by:
(e) 5
2
o 3 1/2
II (c) K = 0: P
[Hint: NH z C-ONH 4 ~ 2NH3(g) + COz(g)
.J2'
. Molecular mass'
Initial vapour denSIty D <:;: < . 2 7. On heating a mixture of S020Z and CO, two equilibria are
simultaneously established:
== 79 = 39 SOzCI 2 (g) ~ SOz(g) + Clz<g)
2 CO(g) + CI 2 (g) ~ COCI 2 (g)
D -d 39-13 26 On adding more S02 at equilibrium what will happen?
0:=---= =
(n I)d (3 -1) xI3 26 (a) Amount of CO will decrease
here, n == number of moles of product formed by dissociation of 1 (b) Amount ofS02Cl z and COCl z will increase
mole reactant.] (c) Amount of CO will remain unaffected
2. In the heterogeneous equilibrium: (d) Amount of S02Cl2 and CO will increase
CaC03(s) ~ CaO(s) + COz(g) 8. Consider the reactions,
what would be the effect of addition of CaC03 on the (i) PCIs(g) ~ PCI 3(g) + CI 2 (g)
equilibrium concentration of COz ? (ii) NZ0 4 (g) ~ 2N02(g)
(a) Increases (b) Unpredictable The addition of an inert gas at constant volume:
(c) Decreases (d) Remains unaffected (a) will increase the dissociation ofPCl 5 as well as N Z0 4
[Hint: Gaseous components are not present on both sides, (b) will reduce the dissociation ofPCl s as well as N Z0 4
hence, equilibrium will not be affected on addition ofCaC0 3 (c) will increase the dissociation of PCl 5 and step up the
Kp :;: Peo2 = constant (at constant.temperature)] formation ofNO z
3. For the reaction, r:h04(g) ~ 2NOz(g); if percentage (d) will not disturb the equilibrium ofthe reactions
dissociation of N2 0 4 are 25%, 50%, 75% and <I 00%, then the Mint: At constant volume, inert gas will not affect any of the
sequence of observed vapour densities will be: eqUilibrium. ]
W~>~>~>~ ~~>~>~>~ 9, Densities of diamond and graphite are 3.5 and 2.3 g/mL
=
(c) d1 d z "" d 3 d 4 =
(d) (d! d 2 ) > (d 3 == d 4 ) respectively. Increase of pressure on the equilibrit;m
Mint: Lesser is the percentage dissociation; greater is the C (diamond) ~ C (graphite)
vapour density.] (a) favours backward reaction(b) favours forward reaction
4. . In a system: (c) has no effect (d) increases the reaction rate
A(s) 2B(g)+3C(g) 10. A react\on at eqUilibrium involving 2 moles each of
if the concentration ofC at equilibrium is increased by a factor PCIs, PCl 3 and Cl 2 is maintained at 250°C and a total pressure
of2, it will cause the equilibrium concentration of B to change of 3 atm. The value of K pis:
to: (a) 2 (b) 3 (c) 4 (d) I
(a) two times the original value
Mint: PCls ~ PCl 3 + Cl z
(b) qne half of its original value At equilibrium 2 2 2
(c) 2.J2 times the original value
2 2
X 3,PCl 2 := - X 3
(d) I times the original value 6 6
2.J2 .
K = fJpc13XPCl 2 =1 XII]
5. For the decomposition ofNH3 (g) in a sealed tube: P PPCl s I
. 2NH3(g) N 2 (g) + 3Hz(g)
11. In case of gaseous homogeneous reaction, the active mass of
(a) K p does not change with pressure the reactant is given by the expression:
(b) concentration of Hz (g) is less than that of nitrogen
I
646 G.R.B. PHYSICAL CHEMISTRY FOR COMPETITIONS

(a) RT (c) PV K
(b) RT (d) n RT K =--p-
P RT V c (RT)6n
[Hint: PV nRT
17. Match the List-I (equilibria) with List-II (conditions) and
n P . select the correct answer using the options given below~ _ _
v RT
List-I List-II
l P
:. Active mass (mol L- )
P Hz(g) + Iz(g) 2HI(g) 1. High temperature
RT ]
12. In the dissociation ofI2 (g) at 100'0' Kin i:1 ::::-"tdiner of 1 litre: Q. 2S0 z(g) + 0z(g) ~ 2S03 (g) 2. Low temperature
12(g)~ 2I-(g); Kc:::: 10'-6 R. 2NH3(g) ~ N2 (g) + 3H2(g) 3. High pressure
Select the correct relation: 4. Low pressure
(a)[I 2 ]>W] (b)[I21<W] (c)[Iz]=[r] (d)[I 2 ] ~W] 5. Independent of
pressure
[Hint: P Q R
(a) 1,3 2,3 2,4
(b) 2,3 1,4 1,3
10'-6 -2~3 .
(c) 1,5 2,4
I :. / [I2 ]>[r]]"
(d) 2,4 1,5 1,3

13. Acetic acid undergoes dimerisation, when dissolved in 18. At constant pressure, the presence of inert gases:
benzene (a) reduces the dissociation ofPC15
" .-::r0... H~ (b) increases the dissociation of PCJ s .
2CH3-COOH...-- CH 3 - C~ /./C - CH3 (c) does not affect the degree ofdissociation of PC Is
OH ... O~ (d) steps up the formation of PC Is
Molecular mass of acetic acid is found 120. Which among the [Hint: PCls(g) PCI 3(g) + CI 2 (g)
following relation is correct? On adding inert gas, the equilibrium will shift towards higher

(D dJ (D-dJ volume direction.]


(a) a = 2 - d - (b) a :: 2 -;;- 19. Which of the following expressions is correct?

d-D)
(c) a =2 (-
2d (a)Kp=Kc(~:J6n (b)Kp=Kx(~)M
d-
D-d
(d)a ::::--

p (P )-lln+l
where, d = Observed vapour density \M 1

D :::: Theoretical vapour density


(c)Kp ::::Kn ( Lnj (d)Kp =Kc Ln
[Hint: For association of molecule 20. The most stable oxide of nitrogen will be:
a=---
d D d-D (a) 2NO(g) ...-- Nz(g) + 0z(g); K ::: 2.2 X 1030
del II n) d(l- 1/ 2) (b) 2N2 0(g) ~ 2N z(g) + Gig); K = 3.5 X 1033
(c) 2N20 5 (g) ~ 2Nz<g) + 50 z(g); K :: 1.2 X 10'24
(d) 2N02(g) ~ N2 (g) + 20z(g); K :::: 6.7 X 1016

14. The equilibrium: [Hint: Stability constant 1


P4 (g) + 6a z(g) 4PCI 3(g) K
is attained by mixing equal moles of. P4 and C12 in an :. Smaller is the value of equilibrium constant greater is the
evacuated vesseL Then at equilibrium: stability of oxide.]
(aHa2l'> [PCl3] (b)[CI 2] > [1>4] ,ll. Equilibrium constant for two complexes are:
(C)[P4] > [Cl z ] (d)[PCI 3] > [P4] A: K 4 [Fe(CN)6] 2.6 x 10'37 (for dissociation)
15. In the reaction: A + B ...--.2C + D. The initial concentration
of A and B are 1 M each. The value of K c is 10'8. What is the B: K.3[Fe(CN)6] 1.9 x 101 7 (for dissociation)
equilibrium concentration of A? .
(a) A and B are equally stable(b) A is more stable than B .
(a) 2 x 10-4 M (b) 2 X 104 M (~) 0'.0'0'5 M (d) 0'.0025 M (c) B is more stable than A (d) the predictable stability
16. For the reaction:
A (g) + 2B(g 3C(g) + D(g );K p "" 0'.0'5 atrn at 1000 K [Hint: Stability oc I ]
Equilibrium constant for dissociation
The value of K c is represented by: .
S 22. At the equilibrium of the reaction, N Z0 4 (g) ~ 2N0 2(g),
(a)5xlO-4 R (b)5XIO-4 (c)5xIO'-5 R (d)5XIO- the observed molar mass ofN20 4 is 77.70' g. The percentage
R R dissociation ofN20 4 is:
[Hint: Kp =Kc(RT )6n (a) 28.4 (b) 46.7 (c) 22.4 (d) 18.4
CHEMICAL EQUILIBRIUM

[Hint:
Mo
(j;=._--
M
(n 1)M (b)J>oM
92 77.7 3D-M
0.184
(2-J)x77.7 (c) PoM (d) 2PoD
2D M
% Dissociation:::: 18.4]
28. S03 (g ) is heated in a closed vessel. An equilibrium:
23. Consider the following statements:
2S03(g) ~ 2S02(g) + °z(g)
In the Haber method of synthesis of ammonia
is established. The vapour density of the mixture, in which
l. Increase of pressure favours the formation ofNH3 S03 is 50% dissociated, is:
2. Decrease of pressure produces more NH3 (a) 26.5 (b) 35 (c) 42 (d) 55
3. Increase of temperature dissociates NH3 D d
4. Addition of inert gas favours the formation ofNH3 [Hint: a :::: - - -
(n l)d
Which of the statements given above are correct?
(SeRA 2009) 0.5 == 40-d
(a) 1 and 3 (b) 2and4 (c) 1 ~nd4 (d) 2and3 (2 -l)d
24. For the decomposition of PCls (g) in a <;losed vessel, the d 26.5]
degree of dissociation is (j; at totai pressure P. .
PCls(g) ~PC13(g) + Ch(g); Kp 29. When In K is plotted against ~ using the yan't Hoff equationE
Which among the following relations is correct?
a straight line is expected with a slope equal to:
f K IK +P
(a) a :::: ~Kp: P (b)a ~~ (a)AHoIRT. (b)-:-AHoIR (c)!1HoIR (d)RIAHo
30. AB z(g) dissociates as,
I AB2 (g )~ AB(g) + B(g)
(c)a::::~Kp+p (d)a= I
vKp+P The initial pressure of AB2 is 600 mm Hg and total pressure is
800 mm Hg. The equilibrium constant for the reaction will be:
25. In the decomposition reaction of ammonia: (a) 500 (b) 100 (c) 200 (d) 400
2NH 3(g) ~ Nz(g) + 3H2(g) 31. For the gaseous reaction,
2 moles of NH3 are introduced in the vessel of 1 litre. At
C2 H4 + H2 C 2 H6 ,
equilibrium, I mole NH3 was left, the value of K c will be:
the equilibrium constant has the units:
(a) 0.75 (b) 0.70 (c) 1.75 (d) 1.70
(a) mol 2 dm-J (b) drn 3 mol- I (c) dm-3 mol- I (d) mol dm- 3
[Hint: 2NH3(g) ~ N 2(g) + 3H 2(g)
32. The equilibrium constant for the reaction:
2 o
.112 31 :; H 2(g) + Br2(g) ~ 2HBr(g)
is 67.8 at 300 K. The equilibrium constant for the dissociation
ofHBr is:
(a) 0.0147 (b) 67.8 (c) 33.90 (d) 8.349

[Hint: Kdissociation =:= K


formation
1.685 1.7]
e 33. A large positive value of !1Go corresponds to which of these?
(a) Small positive K (b) Small negative](
26. By which of the following relations, the equilibrium constant
varies with temperature? (c) Large positive K (d) Large negative K
[Hint: AGO - 2.303RT log Ke]
(a) InK2 -lnKI ::::
z
T d
R
J TI
(~J
T 34. What is !1Go for the following reaction?
4.42 X 104 at 25°C
AHo J
T2 J
(b) lnK2 -lnKl == IIT
z
d (ll "21 N 2(g) + 32 Hl(g) ..--
---' NH3(g); Kp
R IITI
(a) 26.5 kJ mol- 1 (b) -11.5kJ mol- 1
(c) lnKl lnKI - ~o fT~2 d (fJ (c) - 2.2 kJ mol- I (d) 0.97 kJ mol- I
35. For the following reaction: K :::: 1.7 x 107 at 25°C
(d) lnK z -lnKI _ AHo rllTI d(~\j Ag+(aq.)+ 2NH3(aq.)~ [Ag(NH 3 >zt
R JlIT2 T
what is the value of !1GO in kJ?
27. For dissociation of a gas NzOs as: (a)-41.2 (b)-17.9 (c) + 17.9 (d;+ 41.2
N 2 0 5 (g) ~ 2N02(g) + 1 02(g); 36. If EOce11 for a given reaction is negative, which gives the correct
. 2 .

If 'D' is the vapour density of equilibrium mixture and Po is relationships for the values of !1Go and K eq?
the initial pressure of N 2 0 S (g), then its equilibrium pressure (a) !1Go > 0, Keq. < 1 (b) AGo> 0, Keq. > I
must not be (M is molecular mass ofN 20 s ): (c) AG~ < 0, Keq >1 (d) !1Go < 0, Keq < I
648 G. R. B. PHYSTGAL CHEMISTRY FOR C()MPETITIONS

37. The free energy of formation of NO is 78 kJ mol-I at the 44. Which of the following do not change the value of K for a
temperature of an automobile engine (1000 K). What is the reaction?
equilibrium constant for this reaction at 1000 K? (a) Addition of catalyst
1 I (b) Increase in temperature
2 N 2(g)+ 2 02(g)~ NO(g).
(c) Increase in pressure
, (a) 8.4 x 10-5 (b) 7.1 x 10-9 (c) 4.2 X 10- 10 (d) 1.7 x 10- 19 (d) Removal of one of the products
38. 2S0z(g) + 02(g)~ 2S0 3 (g) . 45. For which ofthe following reactions at equilibrium at constant
temperature doubling the volume will cause a shift to the
Given,that the equilibrium constant fv! c"e rew::tion above has
right?
a value of 278 at a particular temperature, what is the value of
(a) NZ0 4 (g) ~ 2N0 2(g)
equilibrium constant for the following reaction at same
temperature? (b) CaC03 (s) ~ CaO(s) + CO 2 (g)
(c) 2CO(g) + 0z(g) ~ 2CO~(g)
S03(g)~ S02(g)+ I02(g) Cd) Nz(g) + 02(g)~ 2NO(g)
. 2
46. In the reaction,
(a) 1.3 x 10-5 (b) 1.8 x 10-3 (c) 3.6 X 10-3 (d) 6 x 10-2
N Z0 4 (g) 2N02 (g) if D and d are the vapour
39. In which of the following reactions, the increase in volume of densities at iRitial stage and at eqUilibrium then what will be
the container will favour the formation of products?
(a) C(s) + H 2 0(g) CO(g) + H 2 (g) the value ofD at point A in the following graph?
.d .'
. (b) H2(g) + 12UD 2ID(g)
'. (c) 4~3(g) + 502 (g) ~ 4NO(g) + 6H20(l) i
(d) 302(g) ~ 203 (g)
40. Consider the reaction, whose K c =: 33,
S02(g) + N02(g)·~ S03(g) + NO(g)
If 0.1 mol each of SOz and NO z are placed in 1 L container,
what is the concentration of SOz at equilibrium?
(a) 0.003 M (b) O.oI5 M (c) 0.085 M (d) 0.097 M
(f-
o
41. Reaction K
(a) 0 (b) 1.5 (c) I (d) 0.5
~ N2 (g) + 02(g)~ N02 (g)
2 . D d
[Hint: \1.=--.-
2N02.(g)~ N Z04(g) (n -l)d

Using above equations, Wlite down expression for.Kof the For the given reaction, n = 2 and \I. = 0 at A
following reaction: .' '. D-d
--=0
. I . '. (2 -l)d
Nz0 4(g) . . .+
N 2 (g) ' -02(g)
2 . D
K2 'd=lJ
(b)_2 47. A schelTIatic plot ofln Keq. versus inverse of temperature for a
K) K 2K 1 . . reaction is shown below: .
42. Cu 2+ ions react with Fe 2+ ions accordill.gto the following
. reaction:
eu 2+ + 2fe 2 +
At equilibrium, the concentration pf Cu 2+ ions is not changed
by the addition of:
i
In Keq
(a)Cu 2+ (b) Fe 2 + (c)Cu (d) Fe 3 +
43. Consider this equilibrium, for which Mf < 0,
HgO(s) + 4r(aq.) + H2 0(l) ~ HgI~- + 20ll 1
Which changes will increase the equilibrium concentration of "T-
HgI~- ? The reaction must be:
1. Increasing the mass of HgO( s) present." (a) exothermic
II. Increasing [r]. (b) endothermic
III. Adding I MHCl. (c) one with negligible enthalpy change
(a) I only (b) II only (d) highly spontaneous at ordinary temperature
(c) II and III only (d) I, II and III
CHEMICAL EQUILIBRIUM 649
din 1'3H 0 50. The equilibrium constants K PI and K P2 for the reactions
[Hint: We ~ow that, dT RT 2
X ~ 2Y and Z ~ P + Q; respectively are in the ratio of
1 : 9. If the degree of dissociation of X and Z be equal then the
In Keq, ratio of total pressures at these equilibria is: . (AIEEE 2008)
(a) I : 9 (b) I : 36
The plot has positive slope, hence I1H 0 will be negative,]
(c) 1 : I (d) 1 : 3
48. The equilibrium: [Hint: X 2Y Z -:;;=:::: P + Q
NH4 HS(s) ~ NH3 (g) + H2 S(g), 0 o 0
to
is followed to set-up at 1270 C in a closed vessel. The total 1- a 2a I-a a a
teq.
pressure at equilibrium was 20 atm. The K c for the reaction is:
(a) 0.092 M2 (b) 0.085 M2 [~p,r
I+a
(c) 3.045 M2, (d) none of these

[Hint: pNH 3 = pH2 S =


. 20
2. atm [~p]
1+ a I

Kp pNH3 x pH 2 S = 100 atm 2 4


Kp = Kc (RT )Al1 1 1-
2 2
100 =Kc x (0,(:)821 x 400)2 .4a .pI/ · a p2
Kc = 0.092 M2] Kp2 l-a 2
~
49. 20% ofN20 4 molecules are dissociated in a sample of gas at 1 = 4Pt
27 0 C and 760 torr. Mixture has the density at equilibrium 9 ~
equal to: PI = 1 ]
(a) 1.48 giL (b) 1.84 giL P2 36
(c) 2.25 giL (d) 3.12 giL
51. . Which of the following plots is correct about endothermic
[Hint: The reaction is: reaction?
N 2 0 4 (g) ~ 2N0 2 (g) f

n = 2 and a =-
100
20,
0.2 i
In Kp
i
In Kp
D
d
46, initial vapour density
vapour density at equilibrium
I I
-·11T -+ 11T -+
D d
a= (a) (b)
(n l)d
46-d
0.2

d
(2 -l)d
38.3
i
In Kp
i.
In Kp
Molar mass equilibrium = 2 x 38.3 = 76.6 I I
Pm = dRT Here, d = density of gas mixture 11T --Ii- 11T -+
d(mix) = Pm = lx76.6 =3.12 IL] (c) (d)
RT 0.0821 x 300 g

1. (d) 2. (d) 3. (a) 4. (d) 5. (a) 6. (b) 7. (d) 8.- (d)


9. (a) 10. (d) 11. (b) 12. (a) 13. (e) 14. (e) 15. (b) 16. (d)
17. (c) 18. (b) 19. (b) 20. (d) 21. (e) 22. {d) 23. (a) 24. (a)
25. (d) 2.6. (c) 27. (a) 28. (a) 29. (b~ 30. (b) 31. (b) 32. (a)
33. (a) 34. (a) 35. (a) 36. (a) 37. (a) 38. (d) 39. (a) 40. (b)
41. (d) 42. (e) 43. (e) 44. (a, e, d) 45. (a, b) 46. (e) 47. (a) 48. (a)
49. (d) SO. (b) 51. (b)
650 .1 G.R.B. PHYSICAL CHEMISTRY FOR COMPETITIONS

Integer Answer TYPE QUESTIONS


6. A reaction at equilibrium involving 2 mol each of PCls , PCl 3 '
This section contains 10 questions. The answer to Cl 2 is maintained at 250°C and total pressure of 3 atm. The
each of the questions is a single digit integer, ranging value of K p is :
from 0 to 9. If the correct answers to question numbers. 7. For the reaction:
X, Y, Z and W (say) are 6, 0, 9 and 2 respectively, then A(g)+ B(g )~C(g )+D(g)
the correct darkening of bubbles will look like the The ,initial concentrations of A and B are equaL The
given figure ; equilibrium concentration of C is two times the equilibrium
concentration of A. The value of equilibrium constant is :
8. N2 +02 ----" 2NO; KI
1. For the reaction:
AB(g)~ A(g)+ B(g) ..!.N z +..!.02~NO; K2
2 2
AB'is 33% dissociated at a total pressure of p. Then, will
2NO~N2+02;K3
be equalto: : . . . Kp
2. If the reaction A B, has AG' == 0, then its' equilibrium NO~..!.Nz +..!.02;K 4
2 2
constant will be equal to :
3. Consider the following reaction KI XK 3=X; FtxK 4 =y; ..[K;XK2=Z
A+B~E .K c =6 What will be the value of xyz?
2B+C~2D Kc==4 9. 4.5 moles each ofH2 and Iz are heated in a sealed 10 L vessel.
What will be the equilibrium constant (K c) for the following At equilibrium, 3 mole of HI were found. The equilibrium
reaction? constant for:
A+D~E+C H 2(g)+ I2(g)~ 2HI(g)is :
4. For the equilibrium ofthe reaction: 10. SOzCI 2 and Cl 2 are introduced into a 3 L vessel. Partial
NH4CI(s)~ NH3(g)+HCI(g) Kp = 8latm2 pressure of S02CI2 and Cl z at equilibrium'are 1 atrn and 2 atm
respectively. The value of K p for the following reaction is 10.
Total pressure at equilibrium will be x times the pressure of S02CI2(g)~ S02(g)+C1 2(g)
NH 3 • The total pressure in atm at equilibrium would be :
The value of x will be ;
5. Mixing 4 moles of A with 4 moles of B forms 2 moles of Cat
equilibrium, according to the following reaction:
A(g)+B(g) ~ C(g)+D(g) .
The value of equilibrium constant is :

1. (8) 2. (1) 3. (3) 4. (2) 5. (1) 6. (1) 7. (4) 8. (1)



9. (I) 10. (8) •
CHEMICAL EQUILIBRIUM 651

, • LINKED COMPREHENSION TYPE QUESTIONS • ,. .


temperature pressure relationship .for its % yield. Then for
• Passage 1
temperature 1j , Tz and T3 the correct option is:
Phosphorous penta chloride when heated in a sealed tube at
700 K, it undergoes decomposition as,
100 T3
PCI5(g)~ PCI3 (g)+CI2 (g); Kp 38atm
vapour density of the mixture is 74.25.
1
"'C
80
60
T2
(jj
Answer the f!lllowing questions: 's:. 40 T1
-;R
1. The reaction is:
° 20
(a) endothermic
(b) exothennic
100 200 300 400
(c) may be endothennic or exothennic
Pressure (atm) -
• (d) unpredictable
2. Percentage dissociation of PCIs maybe giyenas:
(a)4.04 (b) 40.4 (c) 44.0 (d) 0.404 (a)T3>T2>1j (b)1j>T2>T3
(c) 1j = T2 T3 (d) nothing could be predicted
3. Equilibrium constant K c for the reaction will be:
[Hint: The % yield will decrease with rise in temperature, since
(a)0.66M (b)0.56M (c)0.46M (d)0.36M
it is exothermic.]
4. If pressure is increased then the equilibrium will:
3. 500°C is considered optimum temperature for Haber's process
(a) be unaffected
because:
(b) shift in backward direction
(a) catalyst has maximum activity at this temperature
(c) shift in forward direction (b) energy required is easily obtained at this temperature
(d) cannot be predicted (c) yield is maximum at this temperature
5. When inert gas is added to the given reyersible process then (d) rate is fast enough while the yield is also appreciable at
the equilibrium will: this temperature
(a) be unaffected
4. If K p for the reaction is 1.44 x 10-5 , then the value of K p for
(b) shift in backward direction the decomposition of NH 3 ,
(c) shift in forward direction . 2NH3(g) ---->. N2 (g) + 3H2 (g)
(d) cannot be predicted will be:
(a) ~'-1.-44-X-.l-0--:-5 (b) (1.44 X 10- 5 )2
• Passage 2
(c)~'-~ (d) 2.88 x 10-5
In Haber s process, (he ammonia is manufactured according to 1.44 x 10-5
the follOWing reaction: 5. 30 litre Hz (g) and 30 litry were taken for the reaction in
Haber's process which yields only 50% of the expected
ammonia due to r.eversibility of the reaction. What will be the
The pressure inside the chamber is maintained at 200 atm and composition of reaction mixture under the given condition?
temperature at 500°C Generally, this reaction is carried out in . NH3 N2 H2
presence of Fe catalyst. (a) 20L 20L 20L
Answer the following questions: (b) IOL 25L 15L
1. If K p for the giYen reaction is 1.44 x 10-5, then the value of K c (c) 20L lOL 30L
will be:
5 (d) 20L 25 L 15 L
(a) 1.44 x 10- mol L- 1
(0.082 x 500r2
5
• Passage 3
(b) 1.44 x 10- mol L- 1
When all the coefficients in a balanced chemical equation are
(8.314x
multiplied by a constant factor 1, the equilibrium constant
5
(c) 1.44 x 10- mol (originally K ) becomes KJ. Similarly, when balanced equations are
{0.082x added together, the equilibrium constant for the combined process is
5 equal to the product of the equilibrium constants for each ·step.
(d) 1.44 x 10- mol L- 1
Equilibrium constant of the reversed reaction is numerically equal
(0.082 x
to the reciprocal of the equilibrium constant ofthe original equation.
2. The preparatiQn of ammonia by Haber's process is an
exothennic reaction. If the preparation follows the following Unit of K p = (atmi!>n; Unit ofKc = [mol ]'~n
652 G.R.B .. PHYSICAL CHEMISTRY FOR COMPETITIONS

Answer the following questions: when Q > K, reaction will favour backward direction and· whel1
1. Consider the reactions: Q < K, it will favour forward direction,
(i) CO(g) + H20(g) ~ COz(g) + H2 (g);K] Answer the following questions:
(ii) CH4 (g) + HzO(g) ~ CO(g) + 3H2(g);K 2 1. The reaction quotientQ for:
(iii) CH4 (g) + 2H2 0(g) ~ COt(g) + 4H2(g); K 3 N 2(g) + 3H2(g) ~ 2NH)(g)
Which of the following is correct? , , b Q = [ NH3]2 ) . Th·e reactIOn
, '11·proceed .,In
IS given y WI
(a)K:j~=K]IK2 (b)K) =K(IK; [Nz][H2 ] .

(c)K)=K]K2 . (d)K 3 =K]..{K; backward direction, when:


2. The equilibrium constants for the following reactions at 1400 (a)Q=Kc (b)Q<Kc (c)Q>Kc (d)Q==O
Kare given: 2. For the reaction:
. 2H20(g)~ 2H2(g) + 02(g); K] ::: 2.1x 10-13 2A +B ~ 3C at 298 K,K c == 49
A. 3L vessel contains 2, 1 and 3 moles of A, B and C
2C{Jz(g) 2CO(g) + 02(g);K 2 = 1.4 x lO-]2
respectively. The reaction at the same temperature:
Then, the equilibrium constant K for the reaction, (a) must proceed in forward direction
. H2(g)'+ C0:2(g) ~CO(g)+ H20(g) (b) must proceed in backward direction
is: (c) must be in equilibrium.
(a) 2.04 (b)20.5 (c)~ ..6· (d) 8.4 (d) cannot be predicted
3. Given: 2NO(g) + 02(g).--------" 2N02(g);K1 . 3. Ina reaction mixture containing H 2 , N 2 and NH3 at partial
2N02 (g) ~ NZ0 4(g); K z .pressure of 2 attn:, 1 atm and 3 atm respectively, the value of
2NO(g) + 0z(g) NZ0 4(g); K3 K p at 725 K is 4.28 X lO-5 atm-2. In which direction the net
Which of the following relations is·correct? reaction will go?
(a)K) KIIK2 (b)K3=KIXK2 N 2 (g) + 3H2 (g) 2NH 3(g)
(a) Forward .
(d)K3 == ~ (b) Backward
VK 2
(c) No net reaction
4. H3P04 is a tribasic acid, it undergoes ionization as:
(d) Direction.cannot be predicted
H 3P04 ~ If" + H2 P04; K]
4. In the following reaction:
HZP04 H+ + HPO!-; K2 2S02(g) + 02(g) ..-- 2S0 3(g)
HPO~- ~If" +PO~-;K3 the equilibrium is not attained. The rate of forward reaction is
greater than that of backward reaction. Thus, which of the
Then, equilibrium constant for the following reaction will be:
following is the correct relation between K p and Qp?
H3 P04 ~ 3If" + PO~-
(a)Kp=Qp (b)Qp>Kp (c)Qp<Kp (d)Kp==Qp=l
(a) K]K2 K 3 (b) K]K2 . (c) K]K) (d) K]K Z 5. In the reaction:
K3 K2 . K2 . PCI5 (g) ~ P0 3 (g) + CI 2(g)
3
5. Consider the two reactions: . a graph is plotted to show the variation of rate of forward and
XeF6 (g) + H20(g) ~ XeOF4 (g) + 2HF(g); K] backward reactions against time:
xe04 (g) + XeF6 (g) ~ XeOF4(g) + X~3F2(g); K2 Which of following is correct?
Then, the equilibrium constant for the following reaction will
be:
Xe04 (g) + 2HF(g)
is given by:
(a)K]IKi
1 2
Sas
II:
• Passage4
Mass action ratio or reaction quotient Q for a reaction can be
lime-
calculated using the law of IWlSS action,
A(g) + B(g).--------" C(g)+ D(g) Q>K Q==K Q<K
. [C][D] (a) 3 2 1
Q::: [A][B] (b) 1 2 3
(c) 2 3 1
The value of Q decides whether the reaction is at equilibrium or (d) 2 3
. not,
At equilibrium, Q ::: K
For non-equilibrium process,Q -:f. K
CHEMICAL E:OUILIBRIUM 653

• PassageS Graph 3
In a reversible chemical reaction, the rate of f01ward reaction
decreases and that, of backward reaction increases with the passage
'0 '0
of time; at equilibrium the rate offorward and backward reactions >CI)
become same. ~~t 'I ,'5j~~"8
~o
C]):::l

Let us consider the formation of S03 in the following reversible 1a l


CD
CDC]) C])~

reaction: C]) ,~ . a) 0..


....
.' 2S03 (g)
It
x u..

Following graphs are plottedfor this reaction:


Reaction progress
Graph 1
Answer the following questions:
. ,,, 1. In the graph (1), A, Band C respectively are:
, , (a) S03' S02 and 02 (b) S03' O2 and S02
~~----~----~:A
, (c) S02, 02 and S03 (d) 02' S02 and S03
,, 2. In the graph (1), the equilibrium state is reached at:
g -+-------16 I
(a) tl (b) t2 (c) t3 (d) t4
.1 ..
3. The graph (2) tells us th~t:
8 --~------------~:c
, (a) the reaction is irreversible
, I
(b) the reaction is reversible
, I
(c) the reaction is exothermic
ta 4 (d) the reaction is endothermic
lime- 4. The graph (2),tells us that:
Graph 2 (a) equilibrium is never achievable
(b) equilibrium is achieved after the concentrations of
reactants and products become equal
(c) equilibrium is achieved before the concentrations of
reactants and products become equal
- - - Reactants (d) none of the above
<.i 5. From the graph (3), it can be interpreted that:
r:: _ - - - - Products
8 (a) equilibrium is achieved at X
(b) reaction is nearer to the completion
(c) ~G::= Oat X
lime- (d) all of the above

"

Passage 1. •• (a) 2. (b) 3. (a) 4. (b) 5. (c)


. Passage%. 1. (d) 2. (b) 3. (d) 4. (c) 5. (b)
Passage 3. 1. (c) 2. '(d) 3. (b) 4. (a) 5. (d)
Passage 4. 1. (c) 2. (a), 3. (b) 4. (c) 5. (a)
PassageS. 1. (a) 2. (c) 3. (b) 4. (c) 5. (d)
654 G.R.S. PHYSICAL CHEMISTRY FOR COMPETITIONS

~ SELF AsSESSMENT~'
ASSIGNMENT NO.9

found to be equal to the equilibrium concentration of B. What


SECTION-I is the concentration of D at equilibrium:
Straigbt Objective Type Questions x 'I x
(a)- (b)3x (c)x-- (d) x
This section contains 10 mUltiple choice questions. Each 2 2 2
question has 4 choices (a), (b), (c) and (d), out of which only
. one is correct. SECTION-II
1. By applying the law of mass action, the equilibrium constant Multiple Answers Type Objective Questions
K for the reaction: 8. Equilibrium constant does not depend on:
HA + H20~ HjO+ A- [CET(J&K) 20071 (a) catalyst (b) temperature
(a)K [HA][H20] . (b)K = [HjO][A-] (c) pressure (d) inert gas
[H30+][A -] [HA][H20] 9. Which of the following reactions have K p < Kc?
(a)H2(g)+I2(g)~ 2HI(g) ._
(c)K= [HjO][H20] (d) K [-HA ][A-]
(b) CO(g) + CI2(g)~ COCI 2(g)
[A-][HA] [H2 0][H30+]
(c) 2BrCI(g)~ Br2 (g) + CI 2(g)
2. For the reaction, (d) N2 (g) + 3H2 (g) ~ 2NH3(g)
N2 (g)+3Hz(g) 2NH3(g) + Heat 10. Which of the following reactions are not affected by inert gas
When temperature increases: [PMT (Ha.-yana) 20071 addition?
(a) formation ofNH3 increases (a)N2(g)+ 02(g)~ 2NO(g)
(b) formation ofNH3 decreases (b) 2S02(g)+ 02(g)~ 2S03 (g)
(c) concentration ofN2 decreases (In a rigid container of constant volume)
(d) concentration ofH2 decreases (c) PCI3(g) + CI 2 (g) ~ PCl 5 (g)
3. For the reaction 1 g mole of CaC03 is enclosed. in 5 L (d) CO(g) + 2Hz(g) ~ CH30H(g)
container 11. Which of the following are not affected by pressure change?
CaC03 (s) CaO(s) + CO2(g) K p =116 atm at
(a) CaC03(s)~ CaO(s) + CO2 (g)
1073 K then percentage dissociation of CaC03 is:
(b)2NaN°2(s)+ 02(g)~ 2NaN03 (s)
(a) 65% (b) 100% (c) 6.5% (d) zero
(c) NH2COONH4(S)~ 2NH3(g) + COz(g)
" 4. The activation energy for forward and backward reactions are
50 kJ / mol and 40 kJ I mol respectively. If K, and K2 are the (d) C(s) + H20(g)~ CO(g) + H2(g)
equilibrium constants of reaction at temperature 1\ and Tz 12. The reaction,
respectively and T2 > 1\ then: 2S0z(g) + 02 (g ) ~ 2S03(g ) (Exothermic)
will shift in forward direction by:
(a) K, < K2 (b) KI = K2 (c) KI > K2 (d) K2 K?
(a) adding S03 at constant volume
[Hint: MI = Ef - Eb 50, - 40 10 kJ I mol. In endothermic (b) increasing volume of container
reactions, equilibrium constant increases with 'increase in
(c) adding S02 at constant volume
temperature. ]
(d) adding inert gas at constant volume
5. Given, X(g)~ nY(g).
If degree of dissociation is a, then Kc of the reaction in a SEcnON-IU
vessel of 1 litre is: '
Assertion-Reason Type Questions
(a) nan (b) na (c) (nar (d) (nat This section contains 5 questions. Each question contains
I a I-a l+a I-a Statement-I (Assertion) and Statement-2 (Reason). :pach
6. Consider the following equation in a closed container, question has following 4 choices (a), (b), (c) and (d), out of
N20 4(g) 2N02 (g) which only one is correct.
at a fIxed temperature, the volume of the reaction container is (a) Statement-l is true; statement-2 is true; statement-2 is a
halved, for this change, which of the following statement correct explanation for statement-I.
holds true regarding the K p and degree of dissociation 'a ': (b) Statement-l is true; statement-2 is true; statement-2 is not
(a) neither K p nora changes a correct explanation for statement-I.
(b) both K p and a change (c) Statement-I is true; statement-2 is false.
(c) K p changes but a does not change (d) Statement-I is false; statement-2 is true.
(d) K p does not change but a changes 13. Statement-I: The value of equilibrium constant depends on
7. The reaction, A (g ) + B (g ) ~ C(g) + D(g) is studied in a the stoichiometry of the equation.
vessel of one litre at 25(f' C. The initial concentration of A was Because
3x and that of B was x. At equilibrium, concentration of C was
CHEMICAL EQUILIBRIUM 655

Statement-2: The value of equilibrium constant does not 18. Match the List-I with List-II:
change when the equation is multiplied or divided by a List-I List-II
number. . Ir--K- -
14. Statement-I: K p :::: Kc for all reactions. (a) A(g) B(g)+C(g) (p)a::::.1 p

Because VKp + 4P
Statement-2: K p and Kc are interrelated by the equation
Kp (RT)D.n.
(b)A2(g)~ 2A(g) (q)a Jp ~~p
15. Statement-I: Catalyst does not change the position of
Mre.ctant
equilibrium.
Because l+a
Statement-2: Catalyst only changes the equilibrium time. (d)N204(g)~ 2N02 (g) (s) Forward shift on
16. Statement-I: Free energy of both reactants and products are decreasing the
minimum at equilibrium. . pressure
Because 19. Match the List-I with List-II:
Statement-2: The free energy of reactants and products List-I List-II
decreases with passage of time and become equal at (Mathematical process) (New equilibrium constant)
equilibrium. (a) Divided by n (P)K n
17. Statement-I: Addition of inert gases at equilibrium will
(b) Multiplied by n (q)~
support the dissociation of PCls at constant temperature. K
Because (c) Reaction is reversed (r)Kl xK 2 xK 3 x ... xKn
Statement-2: The addition of inert gas at constant volume will
not affect the equilibrium.
(d) n-equations are added (s) (K i 'n
20. Match the List-I with List-II for the following reaction:
SECTION-IV NH4HS(s) NH3(g) + H 2S(g)
Matrix-Matching Type Questions List-I List-II
This section contains 4 qJlestions. Each question contains
(a) Partial pressure of (P) PNH3 XPH2S
statement given in two columns which have to be matched.
NH4 HS(s)
Statements (a, b, c and d) in List-I have to be matched with
statements (p, q, rand s) in List-II. The answers to these (b)Kp (q) PNH3 + PH 2 S
questions have to be appropriately bubbled as illustrated in the (c) Total pressure at (r) ( PNH 3 x PH2) (RT r2
following examples: equilibrium
If the correct matches are (a-p,s); (b-q,r); (c-p,q) and (d-s),
(d) Kc (s) Zero
then correct bubbled 4 x 4 matrix should be as follows:
p q r s
a
G®0 0
b
®.e e G\
c
G e0 (0
d
®® 0 0

[ ,4~.-----------------I
1. (b) 2. (b) 3. (c) 4. (a) 5. (d) 6. (d) 7. (a) 8. (a, b, d)
9. (b. d) 10. (a, b) 11. (a, b, c) 12. (a, b) 13. (c) 14. (d) 15. (a) 16. (a)
17. (b) 18. (a-q,r,s) (b-p,q,s) (c-q, r,s) (d-p,s) 19. (a-s) (b-p) (c-q) (d-r)
20. (a-s) (b-p) (c-q) (d-r)
IONIC EQUILIBRIUM

Solutions (aqueous)
10.1 INTRODUCTION
The compounds which give ions either in molten state or in
I
solution are called electrolytes. In the solid state they are bad ! .1
Non-conducting (solutions of
Conducting (solutions of
conductors, but become good conductors either in the molten non-electrolytes such as urea,
electrolytes, i. e., acids, bases
state or in solution. and salts) glucose. sugar. glycerine, etc.)
There are two types of electrolytes:
(1) Strong electrolytes: These electrolytes are almost
completely ionised when dissolved in a polar medium like water.
r
Strongly conducting (solns. of
1
Weakly conducting (solutions
In solution they are excellent conductors, e.g., HN0 3 , HCI, strong electrolytes which of weak electrolytes which
KOH, NaOH, etc. Their degree of ionization is high and ionise almost completely in water slightly ionise in water such as
approaches unity. such as HCl, H2 S0 4 , HN0 3, CH 3COOH, NH4 0H, H 3P0 4 ,
NH4 Cl, NaCl, NaOH, KOH, etc.) HCN, etc.)
(2) Weak electrolytes: These are not completely ionized
when dissolved in a polar solvent and they behave as poor However, an electrolyte may behave as a strong one in aqueous
conductors of electricity, e.g., CH 3 COOH, H 3P0 4 , H 3B0 3 , solution, but it may behave as a weak one in another solvent. For
NH4 OR, etc. Equilibrium between ions and unionised molecules example, sodium chloride behaves as a strong electrolyte and
is established in solution; e.g., acetic acid as a weak electrolyte when dissolved ~n water but their
con9.ucting abilities are comparable in liquid ammonia solvent.
CH 3 COOH ~ ~H3 COO- + H+
The above equilibrium is termed as ionic equilibrium. Degree of 10.2 OSTWALD'S .DILUTION LAW
ioni~ation of weak electrolytes is much less than unity. According to Arrhenius theory of electrolytic dissociation, the
Degree of ionization 'a.' may be defined as a fraction of total molecules of an e'lectrolyte in solution are constantly splitting up
number of molecules of an electrolyte which dissociate into ions. into ions and the ions are constantly reuniting to form. unionized
% ionization molecules. Therefore, a dynamic equilibrium exists between ions
100 and unionized molecules of the electrolyte in solution. It was
Numberof molecules dissociated as ions pointed out by Ostwald that like chemical equilibrium, law of
Total number of molecules of electrolyte dissolved . mass action can be applied to such systems also.
Consider a binary electrolyte AB which dissociates into A +
The following classification of electrolytes is based on their
behaviour in a particular solvent, i. e., water. and B - ions and the equilibrium state is represented by the
equation:
IONIC EQUILIBRIUM 657

AB ~A+ +B- Solution:


Initially t = 0 COO
At equilibrium C(l- 0:) Ca Ca Applying Ostwald's dilution law for a weak acid,
So, dissociation constant may be given as: a ~KaV
[A + ][B-] Ca x Ca
K
[AB] C(1-a) Ka =4.0x10- 1O , v=_l_=lOOlitre
0.01
Ca 2
... (i) a = ~4 x 1O- 1? x 102 = 2 X 10-4
(1-a)
Concentration of hydrogen ions '
For very weak electrolytes,
4
a <~<I,(1-a)'" 1 _ a _ 2xlO- =2x10-6 molL-I
K =Ca 2 V 100
or concentration of hydrogen ions
a=.JKIC '" (ii)
== .JCK ~r-0-.0-1-x-4-X-I-0--I'-'--O = 2 X 10-6 mol L- 1
Concentration of any ion = Ca = .JCK.
From equation (ii), it is clear that degree of ionization ;::tExalllpl~3~ The concentration oj H+ ions in O,lO'M
increases on dilution. solution of a weak acid is 1.0 x 10-5 mol L- I . Calculate the
Thus, degree of dissociation of a weak electrolyte is dissociation constant of the acid.
proportional to the square root of dilution.
Limitations. of Ostwald's dilution law, The law holds Solution: HA
good only for weak electrolytes and fails completely in the case Initial concentration 0.1 o .0
of strong electrolytes. The value of' ~a' is determined by Equilibrium concentration
conductivity measurements by applying the formula A I A~. The (mol L- 1) 0,1- 1.0 X 10-5 1.0 X 10-5 1.0 X 10-5
value of 'a' determined at various dilutions of an electrolyte
[HA] can be taken as 0.1 Mas 1.0 x 10-5 is very small.
when substituted in eq. (i) gives a constant value of K only in the
,case of weak electrolytes ,likeCH3COOa.;;NH40H, 'etc. The Applying law of mass action,
cause
'. ,
of failure of Ostwald's ~r ' '• . lawin
1
the case of strong'
'.,.
[H:t][A 1.0 x 10-5 1.0 X 10-5
electroJytes is due to the foIl, actors:', .; Ka = -~--'-----'-
. [HA] 0.10
(i) The 'aw is based on'
th,e'~,fa.ctthat only' a portion of the
electrolyte i!\\, dissociated into 'ions at ordinary dilution and ==lx10-9
completely at ''in.finite dilution. Strong electrolytes are almost
~xam.ple 4. What will be the dissociation constant ofO.1N
completely ionisooatall dilutions' and A / A"" does not give
aqueous ammonia solution in terms of degree of dissociation
accurate value of'a'.' ,
'a'? What will be the, value if the concentration is O.OlN?
(ii) When concentration of the ions is very high, the presence
of charges on the ions appreciably affects the equilibrium. Hence, Solutlon:NH40H~NH~ +OW
law of mass action in its simple form cannot be strictly applied in At equilibrium (1- a) a a
the case of strong electrolytes. Since, the solution is 0.1 N,
::::::I_soME SOLVED EXAMPLES~ I::::: : .' V = 1 = 10 litre
0.1
;Example 1. A 0.01 M solution ofacetic acid is 5% ionised
at 25° C. Calculate its dissociation constant. [NH 'OH] (1- a ) [NH + ] a and [OR~ ] ~ ~
4 10'. 4 . 10 ~'" . ,.' 10
Solution: According to Ostwald's dilution law,
a2 Applying law of mass action,
K=--- a a
a (1-a)V + -x-
[NH4 ][OH- ] :::: 10 10
0.05, V = _1_ = 100 litre [NH 40H] 1 a 10(1-a)
0.01
10
Hence; K = 0.05 x 0.05 = 2.63 xlO-5 For 0.01 N solution, K b remains the same at the same
a (1- 0.05)100 . temperature but degree of dissociation value becomes different
Example 2. Calculate the H+ ion concentration of a . Example 5. A 0.0128N solution of acetic acid has A = 14
0.01 N weak monobasic acid. the value of dissociation constant mho equiv- 1 and A"" = 391 mho eq-l at 25°C Calculate the
is 4.0 x 10-10 . dissociation constant of the acid,

658 G.R.B. PHYSICAL CHEMISTRY FOR COMPETITIONS

Solution: Degree of dissociation, In presence of strong electrolyte, the total CN- concentration
A 14 ~ comes from KCN which undergoes complete dissociation. It is
0 , = - = - = 3.58 x 10
further assumed that dissociation ofHCN is very-very sman and
A"" 391
the concentration of HCN can be taken as the concentration of
[1- a ---71 as a is very small] undissociated HCN.
Now applying Ostwald's dilution law,
Thus, [HCN] = 0.2 M and [CN- ] 1M
2
K _a__
a .(l-a)V Putting these values in the expression
10
[H+] = K b [HCN] = 4 x 10- x 0.2 = 8 x 10- 11 mol [;1
a = 3.58 x 10-2 and V = _1_ litre
[CN-] 1
0.0128
2 2 Note: When KeN is not present, the [H+ ] concentration is equal to
So, Ka =3.58xl0- x 3.58 x 10- x 0.0128 1.64xlQ-5
..[CK, i.e., ~O.2 x 4 X 10- 10 = 8.94 X 10-6 mol L- 1• This shows
that concentration of H+ ions falls considerably when KeN is
added to HeN solution.
Let AB be the weak electrolyte. Considering its dissociation, ~Jample 7. Determine the concentration of hydroxyl ions
AB A+ +B- in 0.4 M NH 4 OH solution having (i) no ammonilim c,hlorideqnd
. and applying law of mass action, we have (ii) 5.35 g ofN}/4Clin a Utreof the solution. Ionisation constant
ofNH4 0H is 1.8 x 10-5 .
[A+][B-]
K==-'---=-..... Solution: (i) Let 'a' be the degree of dissociation of
[AB]
NH4 OH in absence ofNH4 Cl.
The equilibrium constant, K, has a definite value 11t any given
temperature. If now another electrolyte furnishing the A + and B- a =J?. .
ions be added to the above solution, it will increase the So, [OW] = Co, == ~K bC == ~1.8 X 10-5 x 0.4
concentration of either A + ions or B - ions (whichever has been
2.68 x 10-3 mol L-1
added) and in order that K may remain constant, the
concentration of AB must increase, i.e., the equilibrium will shift (ii) In presence ofNH4C~
to the left hand side. + 5.35
[NH4]=-=0.IM and [NH 40H]=004M
In other words, the degree of dissociation of an electrolyte 53.5 . '
(weak) is suppressed by the addition of another electrolyte 5
So, [OW] K b [NH40H] == 1.8x 10- x 004
(strong) containing a common ion. This is termed as common
[NH~] 0.1
ion effect. Acetic acid is a weak electrolyte and its.ionisation is
suppressed in presence of a strong acid (H+ ion as common ion) =7.2xlO- 5 molL-I
or a strong salt like sodium acetate (acetate ion as common ion). if&W:~~:xample 8. When 0.100 mole of ammonia, NH 3 , is
Similarly, the addition of NH4 CI or NaOH to NH40H solution dissolved in sufficient water to make 1.0 L of solution, the
will suppress the dissociation of NH4 0H due to common ion solution is found to have a hydroxide ion concentration of
either NH~ or OH . 1.34 x 10-3 M. Calculate K b for ammonia.
As a result of common ion effect, the concentration of the Solution:
ion not in common in two electrolytes, is decreased. The use of
this phenomenon is made in qualitative analysis to adjust NH3' + H20~ NH!
At equilibrium (0.100 - 1.34 x 10-3 ) M 1.34 x 10-3 M
concentration of S2- ions in second group and OH- ion
concentration in third group of analysis. + OW
=O.09866M 1.34 x 10-3 M

K _ [NH~ ][OW]
3 3
1.34 X 10- x 1.34 X 10- = 1.8199x 10-5
!;:J{(;~.~xample 6. The ionisation constant of HCN is 4 x 10- 10 .
b - [ NH 3] 0.09866
Calculate the concentration ofhydrogen ions in 0.2 M solution of
HCN containing Imol rl ofKCN ;{Ci(cExainple 9. : K a for HA is 4.9 x 10-8 . After making the
Solution: The dissociation ofHCN is represented as necessary approximation, calculate for its decimolar solution
(a) % dissociation
, HCN~Hi +CW
(b) H+ ion concentration"
Applying law of mass action, Solution: (a) For a weak electrolyte,
[H+][CW]
~Ka ~4.9X 10-
8
K ,=----- or [H+]= Ka [HCN] a = 7x 10-4
a [HCN] [CN-] C 0.1
IONIC EaUIUBRIUM I 659

% dissociation 100 x a == 100 x 7 X 10-4 Dividing eq. (ii) by eq. (i),


7x 10-2 a l ~0,066
0.0145 == 0,02 1.8166
(b) HA or a = 0,0145x 1.8166= 0.0263
l
C0-~ ~ ~ _ _mp)«: 12. A solution contains 0.1 M H 2 S and 0.3 M
[H+ ] == C x a == 0.1 x 7 x 10--4 == 7 X 10-5 mol HCI. Calculate the concentration of S 2- and HS ions iil
13
solution. Given Kal and Ka2 for H 2 S are 10-7 and 1.3 x 10-
• •a.mple 10. Nicotinic acid (Ka = 1.4 x 10-5 ) is
represented by the formula HNiC. Calculate its per cent respectively. (lIT 1992)
dissociation in a solution which contains 0.10 mole of nicotinic Solution: H2 S ~ H+ + HS-
acid per 2 litre ofsolution. (lIT 1993)
[H+ ][HS-]
Solution: Initial concentration of the nicotinic acid K = ". (i)
al [H 2 S]
== 0.10 0.05 mol L· I
2 Further HS- H+ + S2-
[H+][S2- ]
K ==---=-..:.
Equilibrium cone. (0.05 - x) x x a2 [HS-]
As x is very small, (0.05 x) can be taken as 0.05 ;
I
K == [H+ ][NiC- ] _ x x X
a
[HNiC] 0.05 I·'

2 Due:to conunon ion, the ionisation of H 2S is suppressed and


or x = (0.05) x (1.4 x 10-5 )
the [H+] in solution is due to the presence of 0.3 MHCl.
3
or X == 0.83 X 10- mol FI
3
[S2- ] [H 2 S] = 1.0 X 10-7 X 1.3 X 10- 13 x (0.1)
d'IssocmtlOn
01
70
. . == 0.83 X 10- x 100 1.66 (0.3)2
0.05 20
1.44 x 10- M
Alternative method: Let a be the degree of dissoci{ltion
Putting the value of [S2- ] in eq. (ii),
HNiC H+ + NiC
0.3 x 1.44 x 10-20
At equilibrium 0,05 (1- a) 0.05a 0.05a 1.3 X 10-13
K = 0.05a x 0.05a [HS- ]
a 0.05 (1 a)
or
0.3 x 1.44 x10-20 3.3 X 10-8 M
As a is very small, (1 a) ----? I. 1.3 X 10-13
So, 1.4 X 10- 5 == 0.05a 2
~p)e13. What is the H+ ion concentration of a
5
or a 1.4 x 10- == 1.67 x 10-2 solution known to contain O.lg mole ofCH3COONH 4 in one litre
0.05 ofO.1MCH 3COOH? Assume effective ionisation of ammonium
acetate is 80%.K a for acetic acid is 1.8 x 10-5 .
Per cent dissociation 100 x a = 100 x 1.67 X 10-2
= 1.67 Solution: CH 3 COOH ~ CH 3 COO- + H+
(0.1 x) x x
~~DlPle 11. At 30°C, the degree of dissociation of
0.066M HA is 0.0145. What would be the degree ofdissociation K == [CH 3 COO- ][H+]
ofO.02M solution of the acid at the same temperature? a [CH 3 COOH]
Solution: Let the ionisation constant of the acid be K a • The solution also contains CH 3 COONH4 which is 80%
Degree of dissociation at O.066M concentration = 0.0145. dissociated, i. e., a == 0.8. Thus, the acetate concentration
provided by O.lM CH 3 COONH 4 :;:: 0.1 x 0.8= 0.08M
Applying a = ~? Total [CH3COO~]=(0.08+x)M

0.0145 == ~ 0.066
Ka ... (i) So,
(0.08+
, x)x
(OJ-x)
Let the degree of dissociation of the acid at 0.02 M
concentration be a I . As xis very small, (0.1 x) ----? 0.1 and neglecting x 2 ,
fK: , .. (ii) K == O.08x
V0.02 a 0,1
660 G.R. B. PHYSICAL CHEMISTRY FOR CoMPETITIONS

or 1.8 X 10-5 x 0.1 =0.08x [Ans. (d)]

or X [H+ ] = 1.8 X 10- x 0.1


5
= 2.25 x 10-5 mol L- I [Hint: a=={f or Ka==ca2=O.lx(0~~~lr
0.08
lxlO- 11 ]
_In,,ple 14. Ostwald obtained the following table by 2. The bydrogen ion concentration in'mol/dm 3 in 0.2 M solution
determining the equivalent conductivities containing one gram
of we~ acid, HA (K a 2 X 10-5 ) is close to:
equivalent of acetic acid in V litre. If the equivalent conductivity
of acetic acid at infinite dilution be 364, show that the results are (a) 2 X 10-5 ,{b)2xlO-4 (c)2xlO-J (d)2xl(J2
in agreement with Ostwald slaw. [Ans. (t)]
Volume in lilre (V) Equivalent conductivity [Hint: [H30+] = ~CKa == ~0.2 x 2 x 10-5 == 2 X 10-3 ]
8' 4.34 3.. Autoprotolysis constant ofNH3 is:
64 12.09
(a) [Nn:][NH3J (b)[NH.2 ][NH3]
128 16.99
(c) [NH~][NH2] (d) ~]
Solution: Degree of dissociation, a = A v [NH.2 ]
A=
[Ans. (c)]
(i) a l = 4.34 = 0.01 19 [Hint: Autoprotolysis ofNHJ takesplace as:
364 .
2NH3 ~ NH! + NHZ"
(ii) a 2
= 12.09 =0.0332 Autoprotolysis constant == [NH! ][NHi ] J
364
4. The degree of ionization of an acid HA is 0.00001 at 0.1 M
(iii) 16.99 = 0.0467 concentration. Its dissociation constant will be:
364
(a) 10-9 (b) 10-8 (c) 10-11 (d) 10-7
Now applying Ostwald's dilution law, [An~ (c)]
2
( i) K =~=0.0119xO.0119 1.77 x 10-5 [Hint: For monoprotic acid HA:
a VI 8 Ka == Ca2 == O.lx (10- 5 )2 ~ 10- 11 ]
2 5. The first and second disso<;iation constants of an acid, H2 A,
(ii) K = a 2 = 0.0332 X 0.0332 1.72 x 10-5
a V2 64 are 1.0 x 10-5 and 5.0 x 10-10 respectively. The overall
dissociation constant of the acid will be: . (AI:EEE 2007)
(iii) Ka = a ~ = 0.0467 x 0.0467 = 1.70 X 10-5 (a) 0.2 x 105 , (b) 5.0x 10-
5
V3 (c) 5.0 X 10- 15 .' (d) 5.0 x 1(j5
The values of K a are nearly the same which show the validity [Ans. (c)]
of Ostwald's dilution law. [Hint: Ka Kal XK a2
~~ple 15. Calculate the dissociation constant of = 1 x 10-5 X 5 X 10-10
H 2C 2 0;. (oxalic acid)i/0.02 M solution is 2% dissociated. ,
5 x 10- 15 ]
\,
Solution: H2C20~ 2H+ + C20~- " .
C (1- a) 2Ca Ca
Iifj SOLUBILITY PRODUCT
K
, [Hf
= f[c 2 0 4-]2
If to a given amount of solvent at a particular temperature, a
a [H 2 C2 0 4 ],
solute is added gradually in. increasing amounts,' a stage is
(2Ca)2 x (Ca) , reached when some of the solute remains undissolved, no matter
C(1-a) how long we wait or how vigorously we stir. The solution is then
said to be saturated. A solution which remaius in coutact with
[I a ~ las a is small] undissolved solute is said to be saturated. At saturated stage,
the quantity of the solute dissolved is always constant for the
given amount of a particular solvent at a definite temperature.
4C 2a 3
In case, the solute is an electrolyte, its ionisation occurs in
=4 x 0.02 x 0.02 x (0.02)3 solution and degree of dissociation depends on the concentration
= 1.28 X 10-8 of dissolved electrolyte at a particular temperature. Thus, in a
saturated solution of an electrolyte two equilibria exist and can be
represented as:
1. A monoprotic acid in 0.1 M solution ionizes to 0.001 %. Its AB.,-- AB
Solid Unionised
ionisation constant is: (Dissolved)
(fl) I x 10- 3 (b) 1 x 10-b (c) I X 10-8 (d) 1 x 10- 11 Applying the law of mass action to the ionic equilibrium,
IONIC EaUILIBRIUM 1- 6:-'
[A+][B-] (iv) Electrolyte of the type AlB): Its ionisation is
"------"-''--....:. = K
[AB] represented as:
----->. 2A 3+ + 3B 2-
A 2 B 3...----.--
Since, the solution is saturated, the concentration of unionised
molecules of the electr<;>lyte is constant at a particular Thus, Ks =[A H ]2[B 2 -]3
temperature, i. e. , [AB] = K' = constant.
As 2S 3 ~2Asr +3S 2-; Ks =[ASH]2[S2-]3
Hence, [A + ][B-] = K[AB] = KK' =Ks (constant)
Sb 2S 3 ~2SbH +3S 2-; Ks =[Sb H ]2[S2-]3
K s is termed as the solubility product. It is defined as the
product of the concentration of ions in a saturated solution of (v) Electrolyte of the type AB): Its ionisation IS
an electrolyte at a given temperature. represented as:
Consider, in general, the electrolyte of the type AxB y which is AB3 ~A3+ + 3B-'
dissociated as: Thus, Ks = [A 3+ ][B-]3
Ax By ~xAY+ + yB X -
Fe(OH)3 ~FeH +30W; Ks =[Fe H ][OW]3
Applying law of mass action, AlI3 ~~3++3r; Ks =[Al 3+][r]3
[AY+nBx-y Solubility product of a weak electrolyte: Let degree_oJ
-=------='--=---=-- = K
[Ax B Y] ionisation of weak electrolyte A mE n be 'a'.

When the solution is saturated, AmBn ~mAn+ +nB m-


[AxBy]=K' (constant) t =0 S o o
or [AY+nBx-y =K[AxBy]=KK'=K, (constant) teq S -Sa mSa , nSa
Ksp = [An+ r[B m- t
Thus, solubility product is defined as the product of
concentrations of the ions raised to a power equal to the = [mSa]m[nSaf .
number of times the ions occur in the equation representing Ksp = mmnn (Sa)m+n
the dissociation 'of the electrolyte at a given temperature
when the solution is saturated. Criteria of precipitation of an electrolyte: A very useful
[Note: Solubility product is not.the ionic product under all conditions conclusion is derived from the solubility product concept. No
but only when the solution is saturated.] precipitation of the electrolyte occurs if the ionic product is less
than the solubility product, i. e. , the solution has not reached the
Different Expressions for Solubility Products saturation stage.
. (i) Electrolyte of the typeAB: Its ionisation is represented Case I: When Kip < K sp' then solution is unsaturated in
as:" ...
which more solute can be dissolved.
AB~A+ +Ir Case .II: When Kip = K sp' then solution is saturated in
Thus, which no more solute can be dissolved.
Case m: When Kip > K sp' then solution is supersaturated
AgCI~Ag+ +CI-; Ks =[Ag+][CI-]
and precipitation takes place.
BaS04 ~Ba2+ + SO~-; Ks = [Ba 2+ ][SO~-l When the ionic product exceeds the solubility product, the
(ii) Electrolyte of the typeAB l : Its ionisation IS equilibrium shifts towards left hand side, i. e" iI1creasing the
represented as: concentration of undissociated molecules of the- electrolyte. As
the solvent can hold a fixed amount of electrolyte at a defiriite
AB2 ~A2+ +2B-
temperature, the excess of the electrolyte is thrown out from the
Thus, K s = [A 2+ ][B-]2 solution as precipitate. . .
. Thus, for the precipitation of an electrolyte, it is necessary
PbCl 2 ~ Pb 2+ + 2CI-; Ks = [Pb 2+ ][Cl-]2 that the ionic product must exceed its solubility product. For
CaF2 ~Ca2+ +2F-; Ks = [Ca2+][F-]2 example, if equal volumes of 0.02 M AgN0 3 solution and 0.02 M
K 2Cr0 4 solution are mixed, the precipitation of Ag2Cr04
(iii) Electrolyte of the type AlB: Its ionisation is occurs as the ionic product exceeds the solubility. product of
represented as: Ag 2Cr0 4 which is 2x 10- 12 . '

A2B~2A+ +B 2- In the resulting solution,


Thus, Ks =[A+]2[B2:-] [Ag +] = 0.02 = O.oI = 1 x 10-2 M
2 .
Ag2Cr04 ~2Ag+ +CrO~-; Ks =[Ag+]2[CrO~-]
H2S~2H+ +S2-; Ks =[H+]2[S2-]
and [CrQ~- ] = 0.02 = O.oI = 1 x 10-2 M
2
662 I G.R.B. PHvsIcAL CHEMIsTRy FOR COMPEmlONS

Ionic product of Ag 2 Cr0 4 [Ag + ]2[CrO~-] Let S mol litre -I be the solubility of AB 3'
= (Ix 10-2 )z (1 X 10-2 )' Ks =[A 3+][B-]3 SX(3S)3 ==27S 4
= 1 X 10-6 or S::::~KsI27
6 IZ
I X 10- is higher than 2 x 1O- and thus precipitation of The presence of common ion affects the solubility of a salt.
Ag 2CrO 4 occurs. Let AB be a sparingly soluble salt in solution and A' B be added to
it. Let Sand S' be the solubilities of the salt AB before and after
AeIaIionship ......... su....-, and SaIubIiI.J IIIocIua addition ofthe electrolyte A' B. Let c be the concentration of A' B.
Salts like Ag~ BaS04 , PbS0 4 , PbI z, etc., are ordinarily Before addition of A' B, K s == S 2 ... (i)
. considered insoluble but they do possess some solubility. These
are sparingly soluble electrolytes. A saturated solution of After addition of A' B, the concentration of A + and B - ions
sparingly soluble electrolyte contains a very small amount of the become S' and (S' + c),respectively.
dissolved electrolyte. It is assumed that whole of the dissolved So, Ks :::: S' (S' + c) ... (ii)
electrolyte is present in the form of ions, i. e., it is completely
dissociated. Equating eqs. (i) and (ii),
The equilibrium for a saturated solution of any sparingly S2 =S'(S' +c)
soluble salt may be expressed as: Caladatioa of rwnNing coneeatntion after precip~_ ...
.....: Sometimes an ion remains after precipitation if it isitf~
A B--:-"xA Y+ + yB X-
x y-V-
excess. Remaining concentration can be determined, e.g.,
Thus, solubility product, K s :::: [A Y+ Y [B x- Y
+ Ksp [AB]
(i) [A ]Ieft == [B-]
Let'S' mol litre -I be the solubility of the salt; then
AXBy ~xAY+ + yB X-
") [C 2+] _ K sp [Ca(OHh]
xS yS (11 a left -. - - - ' - - - -
[OH-]2,
So, Ks [xSnysy
";xx, yY(Sr+Y
(i) I: Ilftle aIts; Examples: AgCI, AgI, BaSO4' PbSO 4'
Percentage precipitation of an ion
A+ +B- Initial conc. - Left conc.] x 100
8 8
[ Initial conc. ,
Let solubility of AB be S mol litre -I .
So, Ks [A+][B-] SxS S2 SinUIaneaus SaIubIiI.J
or S=Ji(' , Solubility of two electrolytes having common ion; when they
(1)1:2 .. 2:t lftIe salls:: Examples: Ag 2 C0 3 , are dissolved in the same solution, is called simultaneous
solubility, e.g.,
Ag 2 CrO 4' PbCI 2 , CaF2 , etc.
(i) Solubility of AgBr and AgSCN, when dissolved together.
(ii) Solubility ofCaF2 and SrF2 , when dissolved together.
(iii) Solubility ofMgF2 and CaF2, when dissolved together.
Let solubility of AB2 be S mol litre -I. Calculation of simultaneous solubility is divided into two
SO, Ks =[A 2+][B-f :=SX(2S)2 ::::4S 3 cases:
or S VK s l4 c.e I: When the two electrolytes are almost equally strong
(having close solubility product), e.g.,
AgBr(Ksp 5xlO- 13 ); AgSCN(Ksp 10- 12 )
Let S be the solubility of A'2B. (See ........ 26.)
Ks [A+]2[B2-] Here, charge balancing concept is apr-lied.
:::: (2S)2 (S)== 4S 3 Charge of Ag + = Charge of Br - + Charge of SCN-
or S =VKJ4 [Ag+]= [Br-] + [SCW]

(iii) t:llftle~ Examples: AlI 3 , Fe(OH)3, Cr(OHh, (a+b)= a b


Al(OH)3' etc. c.e U: When solubility products of two electrolytes are
~tIcdI~AB3 A 3+ + 3B- not close, i. e., they are not equally strong, e.g.,
8 38
I
IONIC EQUILIBRIUM 663
CaF2 (Ksp = 3.4 x 10-11 ); SrF2 (Ksp = 2.9x 10-9 ) (ii) LetS'bethesolubilityofBaS04 in 0.1 MBaC1 2 solution.
Total Ba 2+ ions concentration = (S' + c) mol L-I
Most of fluoride ions come of stronger electrolyte.
and S.O~- ions concentration =S' mol L- I
',' ,.:'::::a.SOME SoLVED ExAMPLES' I:::;;~JJ' So, Ks =(S' + c)S' ;;;,(S' + O.1)S'
9
t;~T>EX~mple 16. The solubility product of silver chloride is or 1.5 x 10- (S' + 0.1) S'
1.5625 x 10- 10 at 25°C. Find its solubility in g r l . or (S,)2 + 0.1 S' =1.5 X 10-9
Solution: Let the solubility of AgCl be S mol litre-I . Neglecting (S' )2,
AgCl Ag+ + CI- 0.1 S' 1.5 x 10-9
s s
or S' = 1.5 X 10-8 mol L- I
Hence, S 2 = 1.5625 X 10- 10
,~gfExample 20. The solubility of Mg( OHh in pure water is
or 9.57 x 10-3 g L- I • Calculate its solubility in g L- 1 in 0.02 M
Molecular mass of AgCl (l08 + 35.5) = 143.5 Mg(N0 3 )2 so!ution.
So, Solubility in g litre -I = Mol. mass x S Solution: Solubility of Mg(OH}z in pure water
=143.5 x 1.25 x 10-5 = 9.57 x 10-3 g L- I
3
1.79 x 10-3 g L- I = 9.57 X 10- mol L-I
Mol. mass
;rU\~Example 17. The solubility of PbS04 in water is 0.038
g r l at 25°C. Calculate its solubility product at the same
9.57 x 10-3
= 1.65 X 10-4 mol L- I
temperature. 58
Solution: Solubility (S)ofPbS0 4 in mol L- I , Further, Mg(OH}z ..-- Mg2+ + 20W
s 2S
0.038 = 0.038 = 1.254 x 10-4 Ks =[Mg 2+ ][Ow]2 =S x (2S)2 =4S 3 =4 x (1.65 X 10-4 )3
MoL mass of PbS0 4 303
The equilibrium is = 17.9685 X 10- 12

PbS0 4 Pb 2+ + SO~- Let S' be solubility of Mg(OH}z in presence of Mg(N0 3 }z


S S [Mg2+] (S' + c)= (S' + 0.02)
So, Ks =[Pb 2 + ][SO~-]= S x S =S2
[OH'] 2S'
or K s =1.254 x 10-4 x 1.254 X 10-4 = 1.573 X 10-8
So, Ks = (S' + 0.02)(2S,)2
~~.~;.Example 18. The concentration of Ag + ion in a saturated
17.9685 X 10- 12 4(S' )2 (S' + 0.02)
solution of Ag2Cr04 at 20°C is 1.5 X 10-4 mol r l . Determine
12
the solubility product of Ag 2CrO4 at 20°C. 17.9685 x 10- = (S' i + 0.02(S,)2
Solution: The equilibrium is: 4
Ag 2 CrO 4 2Ag + + CrO~- [neglecting (S' )3]

On the basis of this equation, the concentration ofCrO~- ions 4.4921 X 10- 12 0.02(S,)2
will be half of the concentration of Ag + ions. or (s')2 4.4921 x 10- 12
Thus, [Ag +] =:: 1.5 X 10- M and [CrO~-] = 0.75 X 10-4 M
4 0.02
S' 14.9868 X 10-6 mol L- 1 .
Ks [Ag + ]2[CrO~-] =(1.5 x 10-4 i (0.75 X 10-4) or

1.6875 X 10- 12 Solubility of Mg(OHh in g litre -I = S' x M

iU'Jtxample 19. The solubility product of BaSO4 is = 14.9868 X 10-6 x 58


1.5 X 10-9 . Find out the solubility in (i) pure water and (ii) O.1M 8.69 x 10-4 g L"':'
BaCl2 solution. Example 21. The solubility product of lead iodide is
Solution: The equilibrium is: 1.4 x 10-8 . Calculate its molar solubility in 0.1 M K.l solution.
(i) BaSO 4 . . - - Ba 2+ + SO~-
Solution: Let the solubility of PbI 2 be S. Then,
Let S be the solubility in mol litre -I; then
PbI 2 Pb 2+ + 21-
K" [Ba 2+ ][SO~-] = S 2
s s 2S
or 1.5 X 10-9 S2 Potassium 'iodide is a strong electrolyte and is completely
So, S = 3.87 X 10-5 mol L- I ionised. It shall provide r
ion concentration = 0.1 M.
664 G.R.B. PHYSICAL CHEMISTRY FOR COMPETITIONS

~Pb2+] S Whole of the concentration of NH; ions is provided by


[r] (2S + O.1)M NH4 Cl.

K sp = [Pb 2+][r]2
[NH: ] K NH40H X [NH 4 0H]
[OH-]
=S X (2S + 0.1)2
1.8 x 10,-5 x 0.05 = 0.067 M
=S X(4S2 +O.OI+O.4S) l.34x 10-5
4S 3 + O.OIS + 0.4S 2 i. e., [NH4 Cl] = 0.067 M
. Neglecting S 3 and S 2 , Example 24. The solubility product of Ag 2C 20 4 at 25°C
1.4 X 1O~8 =O.OIS is 1.29 x 10- 11 mol3 C 3. A solution of K 2C 20 4 containing
8
0.1520 mole in 500 mL of water is shaken with excess of Ag 2 C03
or S = 1.4 X 10- 1.4 X 10-6 mol L- 1 till the following equilibrium is reached.
0.01 Ag 2C0 3 +K 2C 20 4 ~Ag2C204 +K 2C03
Example 22. The solubility product of lead, bromide is At equilibrium, the solution contains 0.0358· mole of K 2C03'
8 x 10-5 . If the salt is 80% dissociated in saturated solution, find Assuming the degree of dissociation of K 2C204 and K 2C03 to
be equal, calculate the solubility product of Ag 2 C03 • (lIT 1991)
the solubility ofthe salt. Solution: . .,., ,.
Solution: Let S be the solubility of the salt.
Ag 2C0 3 + K 2C20 4 ~Ag2Ci04 + K 2 C0 3
Degree of dissociation of the salt = 0.8. Initial 0.1520 mole 0
PbBr2 ~ Pb 2+ + 2Br- At equilibrium (0.1520 0.0358)
0,1162 mole
0.0358 mole
. 0.8S 2 x 0,.8S Concentration
2 x 0,1.162 2 x 0.0358

K sp = [Pb 2+][Br - ]2 =0,2324 M 0.0716 M

= (0.8S) x (1.6S)2

= 2.048S 3
5
or S3 = 8x 10- = 3.906x 10-5
2.048
S = V3.906 X 10-5 = 3.39, X 10-2 mol L- 1
Mol. mass ofPbBr2 = 367
Solubility of PbBr2 ' 3.39 x 10-2 x 367 = 12.44 g L-1
Example 23. A solution has 0.05 M. Mg2+ and 0.05 M
NH 3' Calculate the concentration of NH 4 CI required to prevent
the formation of Mg( OHh in this solution. K sp of Mg( OH)z
= 9.0 X 10-12 and ionisation constant ofNH3 =1.8 X 10-5 .
(lIT 1993)
Solution: The maximum concentration of [OIr] ion~ that
will precipitate Mg(OHh is calculated by applying the equation
K sp = [Mg 2+ ][OH- f
12
[OIr]2= Ksp
9.0xlO- =1.8xl0-10
[Mg 2+] 0.05 .
or 5
[OH-] = 1.34 X 10- M
NH3 is present in solution in the form ofNH4 OH Solution: No. of moles of Ag + li120mL
NH3 +H20~NH40H.--"'NH: +OH- 20 x 0.001= 2x 10-5
0.05 0.05 1000
The ionisation of NH 40H is suppressed by the addition of No. of moles crO~- in one litre = 0.002 = 2 x 10-3
NH4 CI (Strong electrolyte).
After mixing, total vs:llume of the solution
= 1000 + 20 =1020 rnL =1.02 L
I
IONIC EaUILIBRIUM I 665
5
[Ag +] = 2 X 10- = 1.96 x 10-5 8 =0.0223 mol L- 1
1.02 After mixing the two solutions, the total volume becomes
3 1 litre.
[cr02- ] 2 x 10- = 1.96 X 10-3
4 1.02 [Ca 2+] = 0.0223 x 500 = 0.01115 mol L- 1
Ionic product =[Ag+]2[CrO~-] 1000
5 3
= (1.96 x 10- )2 (1.96 X 10- ) [OH- ] 0.0223 x 2 x 500 + 0.4 x 500 = 0.2223 mol L-1
= 7.53 X 10- 13 1000 1000
[From Ca(OHh] (From NaOH)
There will be no precipitation of Ag 2crO4 as ionic product is
less than solubility product. Under thl? high concentration of OH- ions, some Ca(OHh
. tiample 26. Calculate simultaneous solubility of AgCN8 will be precipitated.
and AgBr in a solution of water. K sp of AgCN8 1 x 10- 12 , [Ca 2+heft[OH-]2 =Ksp
Ksp of AgBr= 5x 10- 13 •
[Ca 2+] = 4.42 x 10- 5 =8.94 X 10- 4 mol L-1
,Solution: Let the solubility of AgCNS and AgBr in water be left (0.2223)2
a and b respectively. .
Moles of Ca(OHh precipitated = Moles of Ca 2+ precipitated
AgCNS·~ Ag + + CNS-
a a =[Ca 2+ ]initial - [Ca 2+ ] left
AgBr .----. Ag + + Br- = 0.01115 8.94 x 10-4
b b
[Ag+]=a+b,[CNS-]=a and [Br-] b 111.5 X 10-4 8.94 x 10-4

KspAgCNS =[Ag+ ] [CNS- ] = a(a + b) =102.56 X 10-4 M = 102.56 X 10-4 x 74 g


= 7589.44 X 10-4 g =758.944 mg
IxlO- 12 =a(a + b) ... (i)

Ksp AgBr =[Ag+][Br- J= b(a + b)

5 X 10-13 =b(a + b) ... (ii) 6. The solubility product of sparingly soluble salt Ag 2 Cr04 is
4 X10- 12 • The molar solubility of the salt is:
Dividing
, eq. (i) by (ii),
(a) I x 10-4 mol L- 1 (b) 2 x 1O~6 mol L- 1
Ix 10-12 a
13
=- (c) I X 10-5 mol L- 1 (d) 2 x 10- 12 mol L- 1
5 x 10- b
[Ans. (a)]
[Hint: Ksp = [Ag+ ]2[CrO~-]
2=~
. b or a=2b
IfsOlubility of Ag2Cr04 is'S', then [Ag+ ] 2S and[CrO~-] S.
Putting the value of a in eq. (i), K sp 4S 3
=
6h 2 I X 10- 12 ·4 X 10-12 =·4S 3
b 2 = ~ X 10- 12 S =I X 10-4 M]
6
7. The solubility ofPbS04 at 25°C is 1.1 x 10-4 moUL. Then its .
. b 4.08 X 10-7 mol L-1
Ksp is:
a = 2 x 4.08 X 10-7 (a) 1.21 x 10-8 (b) 12.1 X 10-6
- =8.l6x 10-7 molL-1 (c) 121 X 10~1l (d) 1.21 x 10-10
Exalllple 27. The K sp ofCa(OHh is 4.42 KIO- 5 at 25° C. [Ans. (a)]
[Hint: PbS0 4 is a binary electrolyte, hence its Ksp can be
500 mL of saturated solution ofCa(OH)2 is mixed with equal calculated as:
volume of 0.4 M NaOH. How much Ca(OHh in mg is
precipitated? . (lIT 1992)
. Ksp =S2 = [1.1 X 10-4]2 =1.21 X 10-8 ]
Solution: Let 8 be the solubility of Ca(OH)2 in saturated 8. The solubility ofCaF2 (K sp 3.4 x 10- 11 lin 0.1 M solution of
solution. NaP would be:
Ca(OHh Ca 2+ + 20Ir (a) 3.4 x 10-12 M (b) 3.4 X 10-10 M
8 28
(c) 3.4 X 10-9 M (d) 3;4·x 10-13 M
Ksp Ca(OHh =[Ca 2+ ][OH-]2 [Ans. (c)]
4.42 x 10-5 = S x 48 2 =48 3 [Hint: CaF2 :r== Ca 2+ + 2F-
x 2x
666 .1 G.R.B. PHYSICAL CHEMISTRY FOR COMPETITIONS

NaF~ Na+ + F- [Hint: Sb 2S3 ~ 2Sb3+ + 3S2-


0.1 .0.1 8 2S 38

[Ca +] = x,
2
[F-] = [2x + O.l] '" 0.1 M Ksp [Sb3+ ]2[S2-]3 :::: [2S ]2[3S]3 = 108 S5
Ksp = [Ca 2+ ][F-]2 = 108 X (10-5 )5 :::: 108 x 10-25 ]
3.4 X 10- 11 =x(O.l)2
Applications of Solubility Product
x:::: 3.4 x 10-9 M] (i) Purification of common salt
9. The solubility product of Agel is 1.8 x 10- 10 . Precipitation of Natural common salt consists of many insoluble and soluble
AgCl will occur only when equal volumes of which of the impurities. Saturated solution of common salt is prepared and
following solutions are mixed? insoluble impurities are filtered off. Hydrogen chloride gas (HCl)
(a) 10-4 M Ag+ and 10-4 MCl- is circulated through the saturated solution. HCl and NaCI
dissociate into their respective ions as:
(b) 10-7 M Ag+ and 10-7 MC1-
NaCI ~Na + + Cl-
(c) 10-5 M Ag+ and 10-5 MCI
HCl~H+ +CI-
(d) 10- 10 M Ag+ and 10-10 MCl-
The concentration of CI- ions increases considerably in·
[Ans. (a)]
solution due to ionisation of HC\; Hence,theionic product-
[Hint: After mixing, [Na + ][CI-] exceeds the solubility product of sodium chloride
[ Ag+ ] '" ~ X 10-4 = 5 X 10-5 M and, therefore, pure sodium chloride precipitates out from
2 solution.
[CI-] 1 x 10-4 = 5 X 10-5 M (ii) Salting out of soap
2
Soap is a sodium salt of higher fatty acids. From the solution,
Kip = [Ag+ ][Cl-] (5 x 1O-5i = 2.5 x 10-9
soap is precipitated by the addition of concentrated solution of
Since, ionic product is greater than solubility product, hence sodium chloride. Soap and sodium chloride are present in the
precipitation will take place.] form of ions.
10. If x mol L -I is the solubility ofKAl(S04 b then Ksp is equal C nH211 + 1 COONa ~ C nH 2n +! COO- + Na +
Soap
to:
(a) x 3 NaCI~Na+ +cr-
[Ans. (b)] Thus, the concentration ofNa + ions increases considerably on
[Hint: KAl(S04h ~ K+ + A1 3+ + 2S0~- addition of NaCl solution. Hence, the ionic product
x x 2x [CnHZn + 1COO- ] [Na +] exceeds the solubility product of soap
Ksp '" [K+ ][A1 + ][SO~-]2
3
and, therefore, soap precipitates out from the solution.
:::: X X X X (2x)z :: 4x4] (iii) Manufacture of sodium bicarbonate (baking soda)
11. For which of the following sparingly soluble salt, the In Solvay's soda process, CO 2 gas is passed through
solubility (S) and solubility product (Ksp) are related by the ammonical brine to precipitate out NaHC0 3.
expression: NH40H+C02 ~NH4HC03
NH 4HC0 3 + NaCl ~ NaHC0 3 + NH4 CI
IPET (Kerala) 2006]
NaHC0 3 is precipitated first because of its lower solubility
product as compared to those ofNH 4Cl, NH 4HC0 3 and NaCI.
(a) BaS04
(e) CuS Thus, baking soda (NaHC0 3 ) can be quantitatively estimated.
[Ans. (c)] (Iv) Application of solubility product in quantitative
analysis
[Hint: Hg2Cl z ~ H~+ + 2CI-
8 8 2S 1. Estimation of barium as barium sulphate: H2 S04 as
precipitating agent is added to the aqueous solution ofBaCl 2.
BaCl 2 + H2 S04 ~ BaS0 4 + 2HCI
Precipitation of BaSO 4 takes place when its ionic product
exceeds solubility product. H2 SO 4 is added in slight excess to
ensure complete precipitation. Large excess of H 2 S0 4 is harmful
12. The solubility of Sb 2S3 in water is 1 x 10-5 mollL at 298 K. for complex formation.
What will be its solubility product? IPMT (Raj.) 2006] 2. Estimation of silver as silver chloride: NaCI solution
(a) 108 x 10-25 (b) 1.0 x 10-25 is added to the silver nitrate solution; slight excess of NaCI is
(c) 144 X 10-25 (d) 126 x 10-24 added to ensure complete precipitation.
[Ans. (a)] AgN03 + NaCI ~ AgCI + NaN0 3
I
IONIC EQUILIBRIUM 667

Again, precipitation of AgCI takes place when ionic product (vi) PreCipitation of III group hydroxides
of AgCI exceeds its solubility product. When NH 40H is added in presence of NH4 Cl then
3. ' In a similar manner, we estimate lead as lead chromate, precipitation of III group hydroxides takes place, i.e., Al(OH)3 ,
calcium as calcium oxalate, etc. Fe(OHh and Cr(OHh are precipitated. Solubility product of III
group hydroxides is less than those of higher group hydroxides.
(v) Precipitation of the sulphides of group II and IV NH 4Cl ~ NH4+ + Cl-
Hydrogen sulphide is a weak electrolyte and is used for the NH40H~NH4+ + OW
precipitation of various sulphides of group II and IV in
qualitative analysis. . NH4+ ion furnished by NH 4CI lowers the ionisation of
NH 40H and hence the concentration of hydroxide ion OH- . At
It ionises to a small extent in water:
low concentration of hydroxide ion only III group hydroxides
H2S~2H + + S2-
precipitate.
Applying law of mass action,
[H+ ]2[S2-] Fractional Precipitation
K It is a technique of separating two or more ions from a
[H 2 S]
solution by adding a reagent that precipitates first one ion and
The concentration ofS 2- ions can be decreased by increasing then the second.
concentration of H+ ions and it can be increased by decreasing
Let us suppose 0.1 MBa2 + and OJ MSr 2+ in aqueous"<
concentration of H+ ions. In group II, lower concentration of
solution. K ZCr0 4 is added as precipitating agent. K sp ofBaCr04
sulphide ions is required as the solubility products of the
sulphides of group II are low while higher concentration of is 1.2 x 10- 10 andKsp ofSrCr04 is 3.5x 10-5 •
sulphide ions is required in group IV as the solubility products of [CrO~- ] concentration required to precipitate BaCrO 4
the sulphides of group IV are high. The values of solubility 10
products of various sulphides are given below: . =.Ksp 1.2 x 10- =1.2xlO-9

:';"c"liY l\f.~~.~ui~bid~,'{. ,S~dubilityproduct' [Ba 2+] 0.1


BaCr04 will precipitate first because it requires low
Bi 2 S 3 1.6 x 10-72
concentration of CrO~- ions. On addition of chromate ions,
HgS 4 X 10-54
BaCr04 starts precipitating when chromate ion concentration
Group II CuS 1x 10-44 reaches 1.2 x 10-9 M. When CrO~- ion concentration reaches up
PbS 5 x 10-29 to 3.5 X 10-4 M, then SrCr04 also starts precipitating.
CdS 1.4 X 10,,28
Remaining concentration' of Ba 2+ when SrCr04 starts
CoS 3 X 10-26 precipitation
1.4 X 10-24 KspBaCr0 4 1.2 X 10- 10
ilioop N 1 NiS
ZnS l.Ox 10-22
=-~-=--
[CrO~- ] 3.5 X 10-4
3.4 X 10-7 M

MnS 1.4 X 10- 15 . . .


7
% remammg concentration = 3.4 X 10- x 100
The concentration of S2- ions in group II is lowered by 0.1
maintaining acidic medium. In the presence of HCI, the ionisation 0.00034%
ofH 2 S is suppressed due to common ion effect. The concentration
Stability Constant
is so adjusted that only ionic products 'of the sulphides of group II
exceed their solubility products and, therefore, get precipitated. Let us consider dissociation of the ion FeBr+.
However, CdS has somewhat higher value. For its precipitation, FeBr+ Fe 2+ + Br-
dilution of the solution is done which increases ionisation of Hz S .. Dissociation constant for above equilibria may be given as:
and thereby increasing concentration ofS 2- ions. . . [Fe2+ ] [Br'- ]
In group IV, higher concentration of S2- ions is needed. This Kd
[FeBr+ ]
is done by changing the medium from acidic to alkaline.
Ammonium hydroxide is added, the OH- ions furnished by Reciprocal of dissociation constant is called stability constant.
NH 40H remove H+ ions from solution in the form of water K = [FeBr+]
molecules as, s [Fe 2+][Br-]
H+ +OW ~HzO Let us consider the formation of complex K 2 Cd(CN)4'
More of the ionisation of Hz S occurs and, thus, concentration Complex ion is Cd(CN)i- where oxidation state of central metal
of S2- ions increases. It becomes so high that ionic products of Cd 2+ is (+2). Complexing process proceeds in four steps as:
the sulphides of group IV exceed their solubility products and
they get precipitated. . Cd 2+ + CN- ~ CdCN+ ; K _ [CdCN+ ]
1- [Cd 2+ ] [CN- ]
668 G. A.B. PHYSICAL CHEMISTRY FOR COMPETITIONS

K2 = [Cd(CNh] (1) Arrhenius concept


[CdCN+ ][CN-] According to Arrhenius concept, aU substances which give
[Cd(CN)3 ] H+ ions when dissolved in water are called acids while those
Cd(CNh +Gr Cd(CN)3; K3 which iouise in water to furnish OH- ions are called bases.
[Cd(CNh ][CW]
HA ~H++ A- (Acid)
K _ [Cd(CN)~- ] BOH~ B+ + OH-
Cd(CN)3 +CW ~Cd(CN)~-; (Base)
4 - [Cd(CN)3 ] [CN- ] Thus, HCI is an acid because it gives H+ ions in water.
Overall reaction may be given as: Similarly, NaOH is a base as it yields OH- ions in water.
HCI H+ + CI-
Cd 2+ + 4CN- [Cd(CN)~- ]; NaOH Na + + OH-
Some acids and bases ionise almost completely in solutions
Here, KS' =KjK2K3K4 and are called strong acids and bases. Others are dissociated to a
Significance of stability constant: Greater will be the value limited extent in solutions and are termed weak acids and bases.
of stability constant more stable will be the complex. HCI, HN0 3 , H 2S04 , HCI0 4 , etc., are examples of strong acids
Note: (a) If on addition of a conunon ion in a salt solution (sparingly and NaOH, KOH, (CH 3 )4 NOH are strong bases. Every hydrogen
soluble), fonnation of complex ion takes place, then ionisation
compound cannot be regarded as an acid, e.g., CH 4 is not an
.. increases, i.~., equilibrium shifts towards right hand direction to
maintain the value of K sp constant. It means, addition of acid. Similarly, CH 30H,C2H 5 0H,etc., have OR groupsbllttbey
conunon ion in the case of complex fonnation increases the are not bases.
solubility of the sparingly soluble salt which is against the Actually free H+ ions do not exist in water. They combine with
concept of conunon ion effect. solvent molecules, i. e. , have strong tendency to get hydrated.
(b) When we add an electrolyte to another electrolyte solution HX + H 20 H30+ + X-
having no conunon ion, then ionisation of the latter increases. (Hydronium ion)
(c) For a given electrolyte solubility product is always constant at a
The proton in aqueous solution is generally represented as
particular temperature.
H+ (aq. ). It is now known that almost all the ions are hydrated to
Solubility of Metal Hydroxides in Acid Medium more or less extent and it is customary to put (aq. ) after each ion.
H + ion furnished by the medium affects the solubility of The oxides of many non-metals react with water to form adds
metal hydroxide, ~ay M(OH)2' because of neutralization ofOH- and are called acidic oxides or acid anhydrides.
ion by H + ion. CO 2 + H 20----7H 2C0 3 ~2H+ (aq.)+ CO~~ (~q;)
M(0H)2 ~M2+ +20W N20 S +H20----72HN03~2H+ (aq.)+ 2N03 (aq:)
KSf! of M(0H)2 :=[M2+][OW]2 Many oxides of metals dissolve in water to form hydroxides.
K Such oxides are termed basic oxides.
[M2+] Sf! ... (i)
[OW]2 Na20+H20~2NaOH~2Na+(aq.)+20W(aq.)
][OH-] = Kw 10-14 The substance like NH3 and N2H4 act as bases as they react
with water to produce OH- ions.
[OH-f .. ·Oi) NH3 + H 20----7 NH40H~NH1 (aq.)+ OH-(aq.)
The reaction between an acid and a base· is, termed
From eqs. (i) and (ii), we have neutralisation. According to Arrhenius concept, the
K [H+]2 .neutralisation in aqueous solution involves the reaction between
[M 2+ ] = -----'-Sf!_ _
10-28 Wand OH- ions or hydronium and OH- ions. This can be
represented as
1~~5 ACIDS AND BAS~S .
\ ,The earliest criteria for the characterisation of acids and bases were Limitations: (i) For the!!.cidic or basic properties, the'
the experimentally observed properties of aqueous solutions. Ail presence of water is absolutely necessary. Dry HCI shall not act
,acid* Was defined as a substance whose water solution tastes sour, as an acid. HCI is regarded as an add only when dissolved in
turns blue litmus red, neutralises bases and so on. A substance was water and not in any other solvent.
a base if its aqueous solution tasted bitter, turns red litmus blue, (ii) The concept does not explain acidic and basic character of
neutralises acids and so on. Faraday termed acids, bases and salts substances in non-aqueous solvents.
as electrolytes and Liebig proposed that acids are compounds (iii) The neutralisation process is limited to those reactions
containing hydrogen that can be replaced by metals., ' which can occur in aqueous solutions only, although reactions
Different concepts have been put forth by different involving salt formation do occur in the ·absence of solvent.
investigators to characterise adds and bases but the following are (iv) It cannot explain the acidic character of certain salts such
the three important modern concepts of acids and bases: ~ AlCl 3 in aqueous solution.

. * Acid is a Latin word-aciduS,meaning sour~ ,


IONIC EQUILIBRIUM 669
(v) An artificial explanation is required to explain the basic nature NH~ +H20~H30+ + NH3
ofNH3 and metallic oxides and acidic nature of non-metal oxides. Thus, every acid has its conjugate base and every base has its
(2) Bronsted-Lowry concept-The proton-donor· conjugate acid. It is further observed that strong acids have weak
acceptor concept " conjugate bases while weak acids have strong conjugate bases.
In 1923, Bronsted and Lowry independently proposed a HCI CI- CH 3 COOH CH 3COO-
Strong acid Weak base Weak acid Strong base
broader concept of acids and bases. According to
Bronsted-Lowry concept an acid is a substance (molecule or There are certain molecules which have dual character of an
ion) that can donate proton, i.e., a hydrogen ion, H+ , to some acid and a base. These are called ampbiprotic or amphoteric.
Examples areNH3 ,H20, CH 3COOH, etc.
other substance and a base is a substance tbat can accept a
proton from an acid. More simply, an acid is a proton-donor -H+ -H+
H20~ OH- NH3 NH-
(proto genic) and a base is a proton acceptor (protophilic). 2
Acid +H+ Base Acid +H+ Base
Consider the reaction,
+H+ +H+
HCI+H20~H30+ +Cl-
H 20.,....,. H3 0 + NH3 ------"'NH
...--- 4
+
In this reaction, HCI acts as an acid because it donates a proton Base -H+ Acid Base -H+ Acid
to the water molecule. Water, on the other hand, behaves as a base The strength of an acid depends upon its tendency to lose its
by accepting a proton from the acid. proton and the strength of the base depends uponjts tenden~J'Jo~:
The dissolution of ammonia"in water may be represented as gain the proton.
NH3 +H20~NH~ +OH-
Acid-base chart containing some common
In this reaction, Hz 0 acts as an acid as it donates a proton to conjugate acid-base pairs
NH3 molecule and NH3 molecule behaves as a base as it accepts
a proton. "
When an acid loses a proton, the residual' part of it has a HClO4 (Perchloric ClO 4 (Perchlorate
tendency to regain a proton. Therefore, it behaves as a base. acid) ion)
Acid H+ + Base H2 SO4 (Sulphuric HS04 (Hydrogen
The acid and base which differ by a proton are known to form acid) sulphate ion)
a conjugate pair. Consider the following reaction: HCI (Hydrogen cr (Chloride ion)
CH3COOH+H20~H30+ +CH 3COO- chloride)
It involves two conjugate pairs. The acid-base pairs are: HN03 . (Nitric acid) NO; (Nitrate ion)
HP+ (Hydronium
..c:
Oil
H2O (Water)
i=
CH3 COOH ...---CH3COO- and HzO
+H+ Base -H+
ion) ~ ~
'"
Acid Base '"
"0 .~
Such pairs of substances which can be formed from one HS04' (Hydrogen '1~3 SO~- (Sulphate ion) '"
~

another by loss or gain of a proton' are known as conjugate


sulphate ion) ....0 ,J:J

... ~
acid-base pairs. . , H~04 (Ortho- <1l H2 PO; (Dihydrogen ...
<1l
phosphoric 'E0 phosphate ion) 'E0
If in the above reaction, the acid CH 3COOH is labelled acid 1 acid) OIl OIl
and its conjugate base, CH3COO- as basel' H 2 0 is labelled as
CH3COOH(Acetic acid) = CH3COO-(Acetate ion)
';jl
~
=
';jl
basez and its conjugate acid H30+ as acid z, the reaction can be "~
~ RCO; (Hydrogen
written as:
H2 C0 3

H2 S
(Carbonic
acid)
(Hydrogen
- HS-
carbonate ion)
(Hydrosulphide
-g

sulphide) ion) .
Thus, any acid-base reaction involves two conjugate pairs,
i. e. , when an acid reacts with a base, another acid and base are NH! (Ammonium N1J3 (Ammonia)
ion)
formed. Some more examples are given below:
HCN (Hydrogen CN- (Cyanide ion)
Acid I + Base 2 ~ Acid 2 + Basel
cyanide)
HZO+NH 3 NH; + OW
C6HsOH (phenol) C6HsO- (Phenoxide ion)
HCN + H 2 0 H30+ + ~
H2 O (Water) OH- (Hydroxide ion)
HF+CH3COOH~CH3COOH; +F-
C2 HsOH (Ethyl alcohol) CzHsO- (Ethoxide ion)
CH 3COOH + NH3 ~NH; + CH3 COO-
NH3 (Ammonia) NH2" (Amide ion)
NH3 + NH3 ~NH; + NH2
CH4 (Methane) CHj (Methide ion)
H zO+H 20 H30+ +OH-
HC03' + H20~H30+ + CO~-
I
670 G.R.B. PHYSICAL CHEMISTRY FOR COMPETITIONS

In acid-base strength series, all acids above H 30+ in aqueous explained by the fact that bond energies decrease. (H-F = 135
solution fall to the strength ofH3 0+. Similarly, the basic strength kcal/mol, HCl = 103, HBr 88 and HI = 71 kcal/mol)
(c) Oxyacids: (i) The acidic character of oxyacids of the
of bases below OH- fall to the strength of OH- in aqueous
same element which is in different oxidation states increases with
solution. This is known as levelling effect.
increase in oxidation number.
The strength of an acid also depends upon the solvent. The +1 +3 +5 +7
acids HCl0 4 , H 2S0 4 , HCI and HN0 3 which have nearly the HCIO< HCI0 2 < HCI0 3 <HCI0 4
same strength in water will be in the order of HCI0 4 > H2 SO 4 +4 +6 +3 +5
> HCI > HN0 3 in acetic acid, since, the proton accepting H 2S03 <H 2S04; HN0 2 <HN0 3
tendency of acetic acid is much weaker than water. So, the real
'Strength of acids can be judged by solvents. On the basis of But this rule fails in oxyacids of phosphorus.
:,roton interaction, solvents can be classified into four types: H 3P0 2 > H 3P0 3 > H 3P0 4
(i) Protophilic solvents: Solvents which have greater
tendency to accept protons, i. e. , water, alcohol, liquid ammonia, (ii) The acidic character of the oxyacids of different elements
which are in the same oxidation state decreases as the atomic
etc.
number increases. This is due to increase in size and decrease in
(ii) Protogenic solvents: Solvents which have the tendency
electronegativity.
to produce protons, i.e., water, liquid hydrogen chloride, glacial
acetic acid, etc. HCI0 4 > HBr0 4 >HI0 4
(iii) Amphiprotic solvents: Solvents which act both as
H2 S03 > H2 Se03
. protophilic or protogenic, e.g., water, ammonia, ethyl alcohol,
etc. Limitations: There are number of acid-base reactions in
(iv) Aprotic solvents: Solvents which neither donate nor which no proton transfer takes place, e.g.,
accept protons, e.g., benzene, carbon tetrachloride, carbon S02 + S02 ~ S02+ + SO~-
disulphide, etc. Acid I Base2 Acid 2 Basel
HCI acts as acid in H 20, stronger acid in NH 3 , weak acid in Thus, the protonic definition cannot be used to explain the
CH 3 COOH,neutral in C6H6 and a weak base in HF. reactions occurring in non-protonic solvents such as COCI 2 ,
HCI + HF --t H 2CI+ + F- S02,N204,etc.
Base Acid Acid·· Bilse
(3) Lewis concept
Periodic Variations of Acidic and Basic Properti,~' This concept was proposed by GN. Lewis, in 1939.
(a) Hydracids ofthe elements-of the same perio\l: Consider According to this concept, a base is defined as a substance
n
the hydracids of the elements of period, viz., CH4 , NH3 , H2
and HF. These hydrides become increasingly acidic as we move
° which can furnish a pair of electrons to form a coordinate
bond whereas an acid is a substance which can accept a pair
from CH4 to HF. CH 4 has negligible acidic properties while HF of electrons. The acid is also known as electron acceptor or
is a fairly stronger acid. The increase in acidic nature is due to the electrophile while the base is electron donor or nucleophile.
fact that the stability of their conjugate bases increases in the A simple example of an acid-base is the reaction of a proton
order
CH) <NH z <OH- <F-
with hydroxyl ion.
H++:O:Ir
.. H:O:H
The increase in acidic properties is supported by. the •• ••
Acid Base
successive increase in the dissociation constant.
Some other examples are:
CH 4 (=1O-58 )<NH3 (=10-35 )<H20 1O-14)<HF 10-4)
H3N: + BF3 H3N -t BF3
(b) Hydracids of the elements of same group: (i) Hydrides Base Acid

of V group elements (NH3' PH 3 , AsH 3 , SbH 3 and BiH3 ) show H+ + : NH3 =[Hf-NHd+
basic character which decreases due to increase in size and Acid Base
decrease in electronegativity from N to Bi. There is a decrease in
BF3 + [Fr =[F -t BF3 t
electron density in sp 3-hybrid orbital and thus electron donor Acid Base
capacity decreases.
Lewis concept is more general than the Bronsted Lowry
(ii) Hydracids of VI group elements (H 20, H 2S, H 2Se and concept.
H2 Te)act as weak acids. The strength increases in the order
According to Lewis concept, the following species can act as
H z O<H2S <H2Se <Hz Te Lewis acids:
The increasing acidic character reflects decreasing trend in the (i) Molecules in which the central atom bas incomplete
electron donor capacity of OH- , HS- , HSe- or HTe - ions .. octet: All compounds having central atom with less than 8
(iii) Hydracids of VII group elements (HF, HCI, HBr and HI) electrons are Lewis acids, e. g., BF3 , BCI 3 , AICI 3 , MgC12'
show acidic nature which increases from HF to HI. This is BeC1 2 , etc.
IONIC EQUILIBRIUM 671

(ii) Simple cations: All cations are expected to act as Lewis infinite dilution the dissociation of an acid is nearly complete and
acids since they are deficient in electrons. However, cations such all acids are equally strong at infinite dilution.
as Na + , K +, Ca 2+, etc., have a very little tendency to accept The concentration of H+ ions at all other dilutions of
electrons, while the cations like H+, Ag + , etc., have greater equimolar solutions of the acids may not be equal and depends on
tendency to accept electrons and, therefore, act as Lewis acids. their degree of dissociation. Thus, to measure the relative
strength of the two acids, the measurements of hydrogen ion
(iii) Molecules in which the central atom has empty
concentration, i. e. , degree of dissociation is made of equinormal
d-orbitals: The central atom of the halides such as SiX 4 ,GeX 4,
solutions of the two acids. Various methods are used for this
TiCI 4 , SnX 4, PX 3' PFs , SF4 , SeF4 , TeCI 4 , etc., have vacant purpose. Some are described below.
d-orbitals. These can, therefore, accept an electron pair and act as
(i) The conductivity method: The degree of dissociation
Lewis acids.
(iv) Molecules having a multiple bond between atoms of of a weak acid is equal to conductivity ratio A . Thus, the
dissimilar electronegativity: Typical examples of molecules A"" .
falling in this class of Lewis acids are CO 2 , S02 and S03' Under degrees of dissociation a 1. and a 2 for two equinorrnal acids are
the influence of attacking Lewis base, one 1t-electron pair will be given by:
shifted towards the more negative atom. For acid I,
OH
I and For acid II, a2 =
o LewisC= 0+ OH- ~ - 0 - C=O or HCO:3
A ""2
acid Lewis base
At infinite dilution, all weak electrolytes have almost the same
The following species can act as Lewis bases: value of A",,; hence,
(i) Neutral species having at least one lone pair of
electrons: For example, ammonia, amines, alcohols, etc., act as A""l = A""2

..
NH2 , R
..
Lewis bases because they contain a pair of electrons.

O-H
Strength of acid I =_a_
1 = =
1000 x sp. condo acid I
C
Strength ofacid II a 2 A2 1000 x sp. condo acid II
(ii) Negatively charged species or anions: For example, C
chloride, cyanide, hydroxide ions, etc., act as Lewis bases. Sp. condo acid I
CN-, CI-, OW Sp. condo acid II
It may be noted that all Bronsted bases are also Lewis bases
The relative strength of two acids is, thus, equal to the ratio of
but all Bronsted acids are not Lewis acids.
their equivalent conductance or specific conductance of
Limitations: Since, the strength of the Lewis acids and bases equinormal solutions which can be determined experimentally.
is found to depend on the type of reaction, it is not possible to
(ii) Comparing dissociation constants: Let K I and K 2 be
arrange them in any order of their relative strength.
The choice of which definition of acids and bases one wishes the dissociation constants of two acids and let a 1 and a 2 be their
to use in a particular instance depends largely on the sort of degree of dissociation in equinormal solutions.
chemistry that is studied. But Arrhenius concept is perfectly
satisfactory and simplest for dealing with reactions in aqueous
Applying Ostwald's dilution law, a I = f%. and a 2 = ~~
solutions. It explains satisfactorily the strength of acids and bases
Thus,.
Strength of acid I _ a I _ ~1
in aqueous solutions, neutralisation, salt hydrolysis, etc.
Strength of acid II a 2 K2
-jQ"~RELATIVE STRENGTH OF ACIDS Dissociation constants of some weak acids are given in the
tahle:
AND BASES
According to Arrhenius concept, an acid is a substance which Acid-ionization constants at 25 DC
furnishes H+ ions when· dissolved in water. All the acid
properties on an acid are due to H+ ions present in the solution.
Su-.stallql .aa
Fo......
Acetic acid HC2HP2
The extent to which an acid property is given by an acid is
a measure of its strength. The strength of the acid solution Benzoic acid HC 7H s0 2 6.3xlO-s
does not depend on its concentration but on the number of Boric acid H 3B03 5.9 x 10- 10
H+ ions present. The concentration of H+ ions depends on the
Carbonic acid H2C0 3 4.3 x 10-7
ionisation of an acid in solution. On dilution, the ionisation
increases and more of H+ ions come to solution with the result
HCOi 4.8 x 10-11
that the strength ofthe acid increases. Thus, str~ngth ofthe acid Cyanic acid HOCN 3.5 x 10-4
increases on dilution while its concentration decreases. At
I
::,...

672 I G.R.B. PHVSICAL CHEMISTRV FOR COMPETITIONS

Substance (iv) HCIO) > HBr0 3 > HI0 3


Formic acid 1.7 X lO-4 (v) CCI 3COOH>CHCI 2COOH>CH 2 CICOOH
>CH)COOH
Hydrocyanic acid HCN 4.9 X 10-10
(vi) HCOOH>CH 3COOH>C 2H s COOH .
Hydrofluoric acid HF 6.8 X 10-4 The relative strengths of some of the bases are as follows:
Hydrogen sulphate ion HSO; 1.1 X lO-2 (i) KOH > NaOH > Ca(OH)2 > NH 4 0H
(ii) (CH3 )2 NH > CH 3NH2 > (CH3 h N > NH3
Hydrogen sulphide H2S 8.9 X 10-8
HS- (iii) (C2HS )2NH>C2HsNH2 >NH3 > (C 2HS )3N
1.2 X 10-13
(iv). NaOH > NH3 > H 20
Hypochlorous acid HelO 3.5 X 10-8 (v) NH3 > NH2 . NH2 > NH 20H
;~itrous acid HN02 4.5 X 10-4 (vi) NH3 >CsHsN>C6HSNH2
Oxalic acid H2~04 5.6x 10-2
HC20; 5.1 X 10-5
13. Which one is correct statement? (nCE 2007)
Phosphoric acid H 3P0 4 6.9 X 10-3 (a) Basicity ofH)P04 and H 3P0 3 are 3 and 3 respectively
H 2PO; 6.2 X 10-8 (b) Acidity ofH3P04 and H3P03 are 3 and 3 respective~~~~
·HPO~- 4.8 X 10-13 (c) Acidity ofH3P0 4 and H3P03 are 3 and 2 respectively
Phosphorous acid H2PH0 3 1.6 X 10-2 (d) Basicity ofH3 P04 and H3P03 are 3 and 2 respectively
HPHO; [Ans. (d)]
7.0 X 10-7
Propionic acid
Pyruvic acid
Sulphurous acid
HC3Hs0 2
HC3H 30 3
H2S03
1.3 X 10-5
1.4 X 10-4
1.3 X 10-2
[HIn" [ HO- L J~ OH 3H' + Pol- (B";,ity
HSO-
. 3 6.3 X 10-8
(iii) Thomson thermal method: In this method, heat of
neutralisation of two acids is first determined separately with
NaOH. Let it be 'x' and 'y' calorie. The one gram equivalent of
rHo-I-OH] ~ 211' + 0 1-0- (BoOoity 2)]

each of the two acids is rnbc.ed and one gram equivalent ofNaOH
is added. Let the heat evolved in this case be 'z' calorie~ The two 14; In the reaction, .
acids will neutralise a fraction of the base proportional to their H~O; + po~- ~ HPO~- + ~o~-
relative strength.· Suppose n .gram equivalent Of NaOH is the Bronsted base are: (PET (Raj.) 2004]
neutralised by acid I and the rest (1- n ) by acid II. (a) P~- ,~O~- (b) PO!- ,~-
Total heat evolved, z::;: nx + (1- n)y (c) H~O;, HPO~- (d) H~O;, ~O~-
:::,nx+y-ny
[Ans. (a)]
or z-y=n(x-y)
(z y) [Hint: Acid H+ = Conjugate base]
or n=---
(x- y) 15. COnjugate base of HCO; is: . [PET (Raj.) 2005]
(a) CO2 (b) H 2C03 (c) H20 (d) coi-
Strength of acid I n (x":,,y) (z-y) [Ans. (d)]
So, ----==::..---- = - - ::;: ::;: - - - [Hint: Acid R"'; Conjugate base, HeO; - H~ COi-]
Strength of acid II (1- n) 1- (z - y) (x - z)
16; Four species are listed below: .
(x- y)
Relative strength of bases:. A base is a substance which
(i) HCO; (ii) H3°
+
(iii) HSO; (iv) HS03F
Which one of the following is the correct sequence of their
gives OH ions when dissolved in water. The strength of the acid strength? (AIEEE 2008)
base depends on OH - iori concentration. The above methods can (a) (iii) < (i) < (iv) < (ii) (b) (iv) < (ii) < (iii) < (i)
be used for measuring relative strengths of bases also. In the (c) (ii) < (iii) <0) < (iv) (d) (i) «iii) < (ii) «iv)
Thomson thermal method, the two bases and their mixtures will [Ans. (d)]
be neutralised by strong acid, say HCt.
The relative strengths of some of the acids are as follows: [Hint:
+ .
(i) HCI0 4 >HBr>HCl>HN03 > H 2S04 >H30+ >·H 2 S0 3 HS0 3F is super acid and it is most acidic. H3 0 is more acidic than
> H2 C0 3 > CH 3 COOH anions because . anions (HCO;, HSO;jJ do not release H+ ion
(ii) HCI04 > HCI0 3 > HCI02 > HClO easily. HSO~ is more acidic than HCO; becauSe sulphur is more
(iii) H~ > HBr > HCl > HF . e1ectronegativethan carbon.]
IONIC EOOIUBRIUM I 673

17. Identify Bronsted-Lowry acids in the reaction given: (iii) A weak acid plus a strong base gives a basic solution as
[AI(H20)6]3+ + HCOi ~ [AI (H 2 0)s (011 )]2+ + H2C03 the weak acid is not completely ionised. The reactnm does not go
(A) (B) (C) (D) to completion and there is an excess of hydroxyl ions in solution.
The correct answer is: (EAMCET (Med.) 2008) CH 3COOH+Na+ +OH- H 20+CH3COO- +Na+
(a) A, C (b) B, D (c) A, D (d) B, C
[ADs. (e)] (iv) A weak acid plus a weak base gives an acidic or a basic or
[Ilint : [Al(HPh;]3+ and H2CO) are proton donor or Bronsted a neutral solution depending on the relative strength of acid and
acids] base. In case, both have equal strength, the resulting solution is
neutral in nature.
18. Conjugate base ofH2PO:;- is: [Corned (Karnataka) 2008J
(a) HPO~- (b) PO~- (c) H3P04 (d) HPO:;- CH3COOH+NH40H~H20+NH; +CH 3 COO-

[Ans. (a)] SaIts: Salts are regarded as compounds l1!-ade up of positive


[Hint: Acid - H+ "" Conjugate base and negative ions. The positive part comes from a base while
negative part from an acid. Salts are ionic compounds. Salts may
H2PO:; H+ =HPO~-] taste salty, sour, bitter, astringent or sweet or .tasteless. Solutions
of salts may be acidic, basic or neutral. Fused salts and aqueous
~fOi~ ACID-BASE NEUTRALISATION-SALTS solutions of salts conduct electricity and undergo electrolysis~
When aqueous solutions of hydrochloric acid and sodium The properties of salts in aqueous solutions are the properties of
hydroxide are mixed in the proper proportion, a reaction takes ions. The salts are generally crystalline solids.
place to form sodium chloride and water. The salts are classified into the following classes:
HCI( aq. ) + NaOH( aq. ) NaCI( aq. ) + Hp(l) (I) Simple salts: The salt formed by the neutralisation
Sodium chloride process, i. e., interaction between acid and base, is termed as
Such a reaction is termed neutralisation because both acidic simple salt. These are of three types:
(H + ) and basic (OH-) properties are eliminated during the (8) Normal salts: The salts formed by the loss of all
reaction. The hydrogen ion, which is responsible for the acidic possible protons (replaceable hydrogen atoms as H +) are called
properties, has reacted with the hydroxyl ion which is responsible normal salts. Such a salt does not contain either a replaceable
for the basic properties, producing neutral water. The Na + and hydrogen or a hydroxyl group.
CI- ions have undergone no chemical change and appear in the Examples are: NaCl, NaN0 3 , K 2 S0 4 , Ca3 (P0 4 h,
form of crystalline sodium chloride upon evaporation of the N a 3 B0 3 ,N a 2HP0 3 (one H atom is not replaceable as H3 P0 3 is
solution. Sodium chloride is an example of the class of a dibasic acid), NaH2P92 (both H atoms are not replaceable as
compounds called salts. . H3 P0 2 is a monobasic acid), etc.
(b) Acid salts: Salts formed by incomplete neutralisation of
H+ (aq.)+ CI- (aq.)+ Na + (aq.)+ OH- (aq.) poly-basic acids are called acid salts. Such salts still contain one
H2 °(l) + Na + (aq.)+ CI- (aq.) or more replaceable hydrogen atoms. These salts when
neutralised by bases form normal salts.
or H+ (aq.)+ 011 (aq.)~ H 20(l)
Examples are: NaHC0 3 , NaHS0 4 , NaH 2 P0 4 , Na2HP04'
Thus, the neutralisation of a base with an acid involves the etc.
interaction between OH- and H + ions. (c) Basic salts: Salts formed by incomplete neutralisation
Or of polyacidic bases are called basic salts. Such salts still contain
one or more hydroxyl groups. These salts when neutralised by
The reaction between an acid and a base to form salt and
acids form normal salts.
water is termed neutralisation.
Examples are: Zn(OH)Cl, Mg(OH)CI, Fe(0H)2 Cl,
The process of neutralisation does not produce the resulting
Bi(0H)2 Cl, etc.
solution always neutral; no doubt it involves the interaction ofH+
(ii) Double salts: The addition compounds formed by the
and OH- ions. The nature of the resulting solution depends on
combination of two simple salts are termed double salts. Such
. the particular acid and a particular base involved in the reaction.
salts are stable in solid state only.
The following examples illustrate this point when equivalent
Examples are: Ferrous ammonium sulphate, FeS04'
amounts of acids and bases are reacted in aqueous solution.
(i) A strong acid plus a strong base gives a neutral solution (NH4 h S04 ·6H2 0, Potash alum, K 2S0 4 ·Al2(S04 h .24H 20
and other alums.
because both are completely ionised and the reaction goes to
completion. . Properties: (a) When dissolved in water, it furnishes all the
H+ +CI- +Na+ +OH- H 20+Na+ +CI- ions present in the simple salts from which it has been constituted.
(b) The solution of double salt shows the properties of the
(ii) A strong acid plus a weak base gives an acidic solution as simple salts from which it has been constituted.
the weak base is not completely ionised. The reaction does not go (iU) Complex salts: These are formed by combination of .
to completion and there is an excess of hydrogen ions in solution. simple salts or molecular compounds. These are stable in solid
H+ +CI- +NH40H~H20+NH; +CI- state as well as in solutions.
674 G.R.B. PHYSICAL CHEMISTRY FOR COMPETITIONS

10.8 IONIC PRODUCT OF WATER


Pure water is a very weak electrolyte and ionises according to the
Cos0 4 + 6NH3 -----7 CO(NH3 )6 S04 equation,
Simple salt Molecular Complex salt
·compound H2 0..,-- H+ + OH-
Properties: (a) On dissolving in water, it furnishes a
complex ion. Applying law of mass action at equilibrium, the value of
dissociation constant, K comes to
K 4Fe(CN)6 ~4K+ + [Fe(CN)6]4-
Complex ion [H+][OW]
K
Cu(NH3 )4 S04 ~ [CU(NH 3 )4 f+ + SO!:- [HzO]
Complexion
or [H+][OW]=K[H 2 0]
(b) The properties of the solution are different from the
properties of the substances from which it has been constituted. Since, dissociation takes place to a very small extent, the
(iv) Mixed salts: The salt which furnishes more than one concentration of undissociated water molecules, [Hz 0], may be
cation or more than one anion when dissolved in water is called a regarded as constant. Thus, the product K [H 2 0] gives another
mixed salt. constant which is designated as K w • SO,
Examples ;:rre: [H+][QW] =Kw
The constant, K w , is termed as ionic product of water.
ca(CI;
CI The product of concentrations of n+ and on- ions in
water at a particular temperature is known as ionic product
Acidic, Basic and Amphoteric oxides of water. The value of K w increases with the increase of·
(i) Nein-metal oxides are acidic, they dissolve in water to Torm temperature, i. e. ,the concentration ofH+ and OH- ions increases
acid. These oxides form salt with bases. with increase in temperature.
e.g., SOZ,S03,P401O,COz,NOz,Nz05 }N al'd Temperature (oq Value ofK w
·O B 0· . on-met oX! es O.l1x 10-14
S1 z, 2 3 o
Some Transition metal oxides are also acidic. 10 0.31 x 10- 14
25 1.00 x 10-14
e.g., Cr03' Mo0 3 , W0 3 ,Mn 207 .
100 7,50x 10- 14
CO z ·+ H 2 0-----7 H ZC0 3 (Carbonic acid)
The value of K w at 25°C is 1x 10- 14 . Since, pure water is
2NO z + Hz 0-----7 HNO z + HN0 3 (Nitrous and nitric acid)
neutral in nature, H + ion concentration must be equal to OH
PzOs + 3H z0-----7 2H 3 P04 ion concentration.
(Phosphoric acid)
[H+]=[OIr]=x
NaOH + S03 -----7 NaHS04
Salt or [H+ ] [OH- ] = x 2 = 1X 10-14
2KOH + COz -----7 K Z C0 3 + H 2 0 or x= Ix 10-7 M
Salt. .

(ii) Usually, oxides of highly electropositive metals are basic. or [H+] = [OW ] = I X 10-7 mol litre-1
These oxides dissolve in water to form base and they form salt This shows that at 25° C, in 1 litre only 10-7 mole of water
with acids.
is in ionic form out of a total of approximately 55.5 moles.
e.g., Na 2 0, KzO, MgO, CaO, SC 20 3 , Ti02,Zr02
When an acid or a base is added to water, the ionic
Na20+ H z 0-----7 2NaOH concentration product, [H +] [OH- ], remains constant, i. e. , equal
(Sodium hydroxide, a base)

CaO + 2HCl -----7 CaCl 2 + H2


Salt
° to K w but concentrations of H + and OH- ions do not remain
equal. The addition of acid increases the hydrogen ion
concentration while that of hydroxyl ion concentration
Mg + H 2 S04 -----7 MgS0 4 + H 2 0
decreases, i. e. ,
Salt
(Acidic solution)
(iii) Oxides of metalloids and less electropositive metals are
amphoteric. These oxides form salt with both acids and bases. Similarly, when a base is added, the OH ion concentration
e.g., BeO, Al 20 3 , GeO, SnO, Sb z 0 3 ,PbO increases while H + ion concentration decreases,
ZnO, CrZ 0 3 i. e. , [OW] > [H+ ]; (Alkaline or basic solution)

Al 20 3 + 6HCI -----7 2AlC1 3 + 3H 2


Al 2 0 3 +2NaOH-----72NaAl0 z +HzO
° In neutral solution,
In acidic solution,
[H+] = [OH- ] 1 x 10-7 M
[H+] > [OU- ]
IONIC EQUILIBRIUM 675
or [H+]>lx 10-7 M
or
and [OW]<lxlO-7 M
In alkaline solution, [OW] > [H+] pH of a solution is, thus; defined as the negative logarithm of
the coIicentration (in mol per litre) of hydrogen ions which it
or [On-]>lxlO-7 M contains or pH of the solution is the logarithm of the reciprocal
and [H+ ] < 1X 10-7 M of H + ion concentration.
Just as pH indicates the hydrogen ion concentration, the pOH
Thus, if the hydrogen ion concentration is more than
represents the hydroxyl ion concentration, i. e.,
I x 10-7 M, the solution will be acidic in nature and if less than
pOH = -log [OH- ]
I x 10-7 M, the solution will be alkaline.
[H+] = 10-0 10- 1 10-2 10-3 10-4 10-5 10-6 (Acidic) Considering the relationship,
[H+ ] = 10-7 (Neutral) [H+][On-]=K w =Ix 10- 14
[H+] 10-14 10-:-13 1O~12 10-11 10- 10 10-9 10-8 (Alkaline) Taking log on both sides, we have
We shall have the following table if OH - ion cOncentration is log [H+] + log [OH ] log Kw = log (1 x 10- 14 )
taken into account: .
[On-] = 10-14 10:-13 10- 12 10- 11 10- 10 10-9 10-8 (Acidic) or -log [H+] -log [OH . . ] =-Iog Kw-Iog (Ix 10- 14 )
[OH- ] = 10-7 (Neutral) or pift- pOH=pK:---14
[OH-] = 10-0 10- 1 10-2 10-3 10-4 10-5 10-6 (Alkaline) i. e., sum of pH and pOH is equal to 14 in any aqueous solution at
It is, thus, concluded that every aqueous solution, whether 25° C. The above discussion can be summarised in the following
acidic, neutral or alkaline contains both H + and OH ions. The marmer:
product of their concentrations is always constant, equal to
1 x 10-14 at' 25° C. If one increases, the other decreases
accordingly so that the product remains 1 x 10- 14 at 25° C Acidic solution
If [H+] 2
10- M, then [On-]= 10- 12
M; the product, Neutral solution 7 7
Basic solution > 10-7 >7 <7
[H+][on-] 10-2 x 10-12 = 10- 14 ; the solution is acidic.
'--:-07~~:'~'::''":~~:-- ·S,"';:"~~':-'~,~~::~~~:~'~::~,:~:-·~·~r:r:·'7'f2f"::f-~·~T::f:~~~ n;-!.~{; ~~~-.-",":7';"
If [H+] 10-10 M, then [On-] 10- 4 M; the product, : [1l+'j'[()H-r ' .' pH' pOH Nature oholutlou
10 14
[H+ ] [OH- ] = 10- X 10-4 10- ; the solution is alkaline. - - - - - -...
10° 10- 14 0 14 Strongly acidic
10-2 10-12
11.1 HYDROGEN ION CONCENTRATION-
10-5 10-9
2
5
12
9
Acidic
Weakly acidic
pH SCALE
10-7 10-7 7 7 Neutral
It is clear from the above discussion that nature of the solution 10-9 10-5 9 5 Weakly basic
(acidic, alkaline or neutral) can be represented in terms of either
10- 11 10-3 11 3 Basic
hydrogen ion concentration or hydroxyl ion concentration but it
is convenient to express acidity or alkalinity of a solution by 10-14 100 14 0 Strongly basic
referring to the concentration of hydrogen ions only. Since, H + The following table shows the pH range for a few common
ion concentration can vary within a wide range from 1 mol per substances:
litre to about 10 x 10- 14 mol per litre, a logarithmic notation has
been devised by Sorensen, in 1909, to simplify the expression of ~!I~~~:~~nf~:,J?H~auge.· s'pl)s~al)~,e r. pBrange
these quantities. The notation used is termed as the pH scale. Gastric 1.0 3.0 Milk (cow) 6.3 -6.6
The hydrogen ion concentrations are expressed in terms of the contents
numerical value of negative power to which 10 must be raised. Soft drinks 2.0 - 4.0 Saliva (human) 6.5 -7.~
This numerical value of negative power was termed as pH, i. e. , Lemons 2.2 - 2.4 Blood plasma (human) 7.3 7.5
Vmegar 2.4 ~ 3.4 Milk of magnesia 10.5
[H+] 1O-pH
Apples 2.9 - 3.3 Seawater 8.5
or log [H+ ] log 1O-pH = - pH log 10 pH lJrine 4.8 8.4
(human)
or pH=-log [n+]
. 'I", r" : - - ,,~~, -.' .- " _ . ,

*pK value: p stands fot negative logarithm. Just as H+ and OH - ion cQucentrations range over many negative poWers of.1 0, iUs convenient to
express them as pH or pOH, the dissociation constant (K ) values also range over many negative powers of 10 and it is. convenient to write them as pK.
Thus; pK is the negative logarithm of dissociation constant. .
pKa =-logKa and pKb =-logKb
Weak acids l]ave higher pKa values. Similarly, weak bases have higher pKb values.
676 G.R.B. PHYSICAL CHEMISTRY FOR COMPETITIONS

Any method which can measure the concentration of H+ ions [H+] [A-]+[B-]+[OW]
or OH- ions in a solution can serve for finding pH value. = Ka[HA] + Kb[HB] + Kw
Limitations of pH Scale: (i) pH values of the solutions do [H+ ] [H+ ] [H+ ]
not give us immediate idea of the relative strengths of the
solutions. A solution of pH = I has. a hydrogen ion concentration [W] =.JKa [HA]+ Kb[HBJ+Kw
100 times that of a solution of pH 3 (not three times). A =~CKo +CK b +Kw
4 x 10-5 N HCI is twice concentrated. of a 2 x 10-5 N HCI
(vi) Let us consider mixture of two weak bases AOHandBOH
solution, but the pH values of these solutions are 4.40 and 4.70 with dissociation constant K 1 ,K 2 and concentration C1 , C 2
(not double). respectively
(ii) pH value of zero is obtained in I N solution of strong acid.
In case the concentration is 2 N, 3 N, 10 N, etc. The respective
pH values will be negative.
(iii) A solution of an acid having very low concentration, say ~t~: pH OF WEAK ACIDS AND BASES
10-8 N, cannot have pH 8, as shown by pH formula, but the
actual pH value will be less than 7. Weak acids and bases are not completely ionised; an equilibrium
[Note: (i) Nonnality of strong acid = [H30+ ] is found to have been established between ions and unionised
molecules. Let us consider a weak acid of basicity' n' ..
Normality of strong base = [OH- ]
AHn A n- + nH+
pH - log.[N ] for strong acids 1=0 COO
leq C(l a) Ca nCa
pOH = -log [N] for strong bases
(ii) Sometimes, pH of acid comes more than 7 and that of base [H+] =nCo.; pH=-logIO [nCo.] ... (i)
comes less than 7. It shows that the solution is very dilute; in For monobasic acid, n:= I
such cases, H + or OH - contribution from water is also pH= -log}O [Cql ... ,(ii)
considered, e.g., in 10-8 N Hel,
Dissociation constant of acid K a may be calculated as:
[H + homl = [10~ kid + [l0-71watcr [An-][H+]n [Ca][nCa]1I
Ko
= 11 X 10- 8 M :: 1.1 X 10-7 M [AHn] C(l-a)-
(iii) pH of mixture: Let one litre of an acidic solution of pH 2 be a[nCa]n
mixed with two litre of other acidic solution of pH 3. The
=~--=- [for weak acids, a «1
(I-a)
resultant pH of the mixture can be evaluated in the following
way: :.(1-«)= 1] .
:=a[nCat
~j~';":'$~1 ';: .Sampi.~'
pH::2 pH=3
nCK 0 = nCar nCa]n =[nCo. ](n + 1)
[nCal = [nCKa ]1/(n+ 1)
[W J= 10-2 M [H+] 10-3 M
[H+ ] == [nCK a ]"(n + I)
V = 1 litre v = 2 litre
1
MtVI + M 2V2 = MR(VJ + V2 ) pH=---loglO (nCKa) ... (iii)
(n + 1)
10:"2 xl + 10-3 X 2 = M R (1 + 2)
For monobasic acid, n =1
12 X 10-3 _ M
3 - R pH= -loglO ~CKa ... (iv)
4 X 10-3 = MR (Here, MR Resultant molarity)
Since, Ka a[nCat
3
pH = log (4 x 10- )
K
(iv) Total concentration of [H+ ] or [Hj 0] in a mixture of weak acid _0 =(nCat
a
and a strong acid
= C 2 + ~C; + 4KaCJ K ]"n =[H+]
2
[nCo.] =[ -:
where, C I is the concentration of weak acid (in mol litre -I) having
dissociation constantKa and C 2 is the concentration of strong acid.
... (v)
(v) Let us consider mixture of two weak acids HA and HB.
HA~H++A
HB~H++B­
For n = 1, ... (vi)
On applying charge balance
I
IONIC EQUILIBRIUM 677

Ba(OHh ~ Ba 2+ + 20H-
One molecule on dissociation furnishes 20H- ions.
~::;V{~~!'Jnp.e28. The hydrogen ion concentration ofa solution So, [OIr] 2 x 10-3 M
is 0.001 M. What will be the hydroxyl ion concentration of pOH = -log [OH- ]
solution?
= -log (2 x 10-3 ) =2.7
Solution: We know that, [H+ ][OH- ] = 1.0 x 10-14
We know that, pH + pOH 14
Given that, [H+ ] = 0.001 M 10-3 M So, pH = (14- 2.7) = 11.3
14 14
So, [OIr] = LOx 10- Ix 10- =10- 11 M '?l~~~E:lample 31. Find the pH ofa o.002N acetic acid solution,
[H+] 10-3 ifi! is 2.3% ionised at a given dilution.
JLt':j;J.a~J1le 29. What is the pH of the following solutions? Solution: Degree of dissociation, a. = 2.3 =~.023
3 100
(a) 10- M HCI (b) O.OOOIM NaOH (c) O.OOOIM H 2 S04
Concentration of acetic acid, C = 0.002M
Solution: HCI is a strong electrolyte and is completely The equilibrium is,
ionised.
CH3COOH~CH3COO- + H+
HCI~H+ +C1- C(1-a) Ca Ca
So, [H+] = 10-3 M So, [H+]=Ca. =0.002 x 0.023
pH =-log [H+] log (10-3 )=3 = 4.6x 10-5 M
(b) NaOH is a strong electrolyte and is completely ionised. pH =-log [H+]
NaOH~Na+ +OH- =-log (4.6 x 10-5 ) = 4.3372
So, [OH+ ] = 0.000 1M = 10-4 M 't~~fe~~ain'Ple32. Calculate the pH value of a solution
obtained by mixing 50 mL of 0.2 N HCl with 50 mL of O.1N __
pOH - log (10-4 ) = 4
NaOH
As pH+pOH=14 Solution: Number of milli-equivalents of the acid
So,_ pH+4 14 orpH=1O 50x 0.2 10
Alternative method: [OH-] = 10- 4 M Number of milli-equivalents of the base
=50x 0.1=5
We know that, [H + ][OIr ] = 1.0 xlO- 14 Number of milli-equivalents of the acid left after the addition
14 of base
So, [H+ ] 1.0 X 10- = 10-10 M
10-4 = (10- 5)= 5
Total volume of the solution 50 + 50 = 100 mL
pH= log [H+] log(10-lo)=10
Thus, 5 milli-equivalents of the acid are present in 100 mL of
(c) H2 S04 is a strong electrolyte and is ionised completely. solution.
H 2 S04 2H+ + SO~- or 50 milli-equivalents of the acid are present in one litre of
solution.
One molecule of H 2S04 furnishes 2H + ions.
or 0.05 equivalents of the acid are present in one litre of
So, [H+] 2x1Q- 4 M solution.
. pH = - log [H+ ] The acid is monobasic and completely ionised in solution.
-log (2 x 10-4)= 3.70 0.05 N HCI 0.05 M HCI
So, [H+]=0.05M
!)ff~~li~.mP:i~30. Calculate the pH of the following solutions
assuming complete dissociation: pH =-Iog [H+] =-log 5 x 10-2 [log 5.0+ log 10-2 ]

(a) 0.365 g rl HCI solution = [0.70- 2] 1.3


l;:\\~ample 33. What will be the pH ofa solution obtained by
(b) O.OOIM Ba(OHh solution. . mixing 800mL of 0.05 N sodium hydroxide and 200mL of 0.1 N
Solution: (a) Mole. mass ofHCI 36.5 HCI, assuming the complete ionisation of the acid and the base?
Concentration of HCI = 0.365 = 1.0 x 10-2 mol L-1 Solution: Number of milli-equivalents ofNaOH
36.5 = 800 x 0.05
40
HCI is a strong electrolyte and is completely ionised.
Number of milli -equivalents of HCI = 200 x 0.1 =20
So, [H+] = I X 10-2 mol L- 1
Number ofmilli-equivalents ofNaOH left after the addition of
pH = -log [H + ] = -log (1 x 10-2 ) = 2 HCI
(b) Ba(OHh is a strong electrolyte and is completely ionised (40 20)= 20
678 ('RB. PHYSICAL CHEMISTRY FOR COMPETITIONS

Total volume (200 + 800) mL = lOOO mL = llitre = 3.69 X 10- 10


20 mill i-equivalents or 0.02 equivalents of NaOH are present ';~;:~~Iij;ple 37. . Calculate the pH of thefollowing solutions:
in one litre, i. e. , (i) 1.0 x lO-8 M HCI, (ii) 1.0 x 10-8 M NaOH
0.02 NNaOH = 0.02 MNaOH (Mono-acidic) and the base is
completely ionised. Solution: (i) The neutral water has[H+] I x lO-7 M
8
So, [OH- ] = 0.02 M By adding 1.0 x 10- M HCl, a concentration of 1.0 x 10-8 M
or [OH-] 2x lO-2 M H + ions has increased in solution.
pOH log (2 x lO-2 ) = 1.7 Thus, total [H+] (Ix 10-7 + lx)0:-8)M
We know that, pH + pOH=14 (lxlO-7 +0.lxlO-7 )M
So, pH=(l4 1.7)=12.3
=1.1 x lO-7 M
':\i· . '¢~~u,~ple 34.
What is the hydrogen ion concentration of a
solution (i)whose pH is 12, (ii) whose pH is 5.6? pH =-Iog (1.1 x 10-7 ) [log l.1+ log 10-7 ]
Solution: . [0.0414 - 7.0]:::: 6.9586
(i) pH = -, log [H+ ] (i) The neutral water has [OW] 1 X 10-7 M
or log [H+] =-pH= 12 By adding 1.0 x 10-8 M NaOH, a concentration of
"[H+]=lO-12 M 1.0 x 10-8 M OW ions hasincreaied ins()l'ution.

(ii) pH';'; -log [H+ ] Thus, total [OW] = (I X 10-7 + 1.0 X 10-8 )M
Ux 10-7 M
or log [H+] 5.6
pOH=-log 1.1 x 10-7
or
= 6.9586
~";:·~;.~..mp.e 35. How many moles of calcium hydroxide must pH=(14 pOH)=(l4 6.9586) 7.0414
be dissolved to produce 250 mL Of an aqueous solution of

Вам также может понравиться